Sie sind auf Seite 1von 920

LABOR REVIEW DIGEST

Atty. Joyrich Golangco

RA 8042 - MIGRANT WORKERS ACT

1. People v. Melissa Chua


G.R. No. 184058 Date: March 10, 2010
Ponente: Carpio Morales, J.

Article 38. Illegal Recruitment

DOCTRINE/S:

Thus for illegal recruitment in large scale to prosper, the prosecution has to prove three essential elements, to wit:
1. Accused undertook a recruitment activity under Article 13(b) or any prohibited practice under Article 34 of the
Labor Code; 2. The accused did not have the license or the authority to lawfully engage in the recruitment and
placement of workers; and 3. The accused committed such illegal activity against three or more persons
individually or as a group.

Intent is immaterial in illegal recruitment in large scale, violation of which is malum prohibitum.

FACTS:
Appellant Melissa Chua together with one Josie Campos, who remained at large, were charged with Illegal
Recruitment (Large Scale) and Estafa.

PLAINTIFF:
According to the testimony of Marilyn Macaranas, Erik de Guia Tan and Harry James King, the three
complainants, in separate occasions, they paid appellant placement fees upon the latter’s assurance that they
could work in Taiwan as factory workers. However, appellant welched on her promise to deploy them and failed to
refund their money despite demand. They later learned that appellant was not a licensed recruiter and that Golden
Gates’ license had already expired.

DEFENDANT:
Appellant denied the charges. Claiming having worked as a temporary cashier at the office of Golden Gate owned
by one Marilyn Calueng, she maintained that Golden Gate was a licensed recruitment agency and that Josie, who
is her godmother, was an agent. She also claimed that she turned over the money to the documentation officer
who in turn remitted the money to Marilyn Calueng whose present whereabouts she did not know.

RTC RULING:
The Manila RTC convicted appellant of Illegal Recruitment (Large Scale) and three counts of Estafa.

CA RULING:
The CA affirmed the trial court’s decision holding that appellants defense that, as temporary cashier of Golden
Gate, she received the money which was ultimately remitted to Marilyn Calueng is immaterial, she having failed to
prove the existence of an employment relationship between her and Marilyn, as well as the legitimacy of the
operations of Golden Gate and the extent of her involvement therein.

ISSUE:

Whether Appellant Chua committed illegal recruitment in large scale.

HELD:
Angeles| Bajana | Balladares | Brillantes | Briones | Cabansag | Callanta | Chua | David|

De Leon | Gomez | Lopez | Macalino | Nostratis | Padilla | Reynon | Santos | Tan |Velasco
4E / 4F - 2018-2019
Page 1 of 920
LABOR REVIEW DIGEST
Atty. Joyrich Golangco

Yes. Any recruitment activities to be undertaken by non-licensee or non-holder of contracts, or as in the present
case, an agency with an expired license, shall be deemed illegal and punishable under Article 39 of the Labor
Code of the Philippines. And illegal recruitment is deemed committed in large scale if committed against three or
more persons individually or as a group.

For illegal recruitment in large scale to prosper, the prosecution has to prove three essential elements, to wit: (1)
the accused undertook a recruitment activity under Article 13(b) or any prohibited practice under Article 34 of the
Labor Code; (2) the accused did not have the license or the authority to lawfully engage in the recruitment and
placement of workers; and (3) the accused committed such illegal activity against three or more persons
individually or as a group.

In the present case, Golden Gates’ license expired and it was delisted from the roster of licensed agencies. Even
if appellant was a mere temporary cashier, that did not make her any less an employee to be held liable for illegal
recruitment as principal by direct participation, together with the employer, as it was shown that she actively and
consciously participated in the recruitment process

Other Notes:
Assuming arguendo that appellant was unaware of the illegal nature of the recruitment business of Golden Gate,
that does not free her of liability either. Illegal Recruitment in Large Scale penalized under Republic Act No. 8042,
or The Migrant Workers and Overseas Filipinos Act of 1995, is a special law, a violation of which is malum
prohibitum, not malum in se. Intent is thus immaterial.

2. People v. Rodolfo Gallo y Gadot


G.R. No. 187730 Date: June 29, 2010
Ponente: Velasco, Jr., J.

Article 38. ILLEGAL RECRUITMENT

DOCTRINE:

To commit syndicated illegal recruitment, three elements must be established: (1) the offender undertakes either
any activity within the meaning of recruitment and placement defined under Article 13(b), or any of the prohibited
practices enumerated under Art. 34 of the Labor Code; (2) he has no valid license or authority required by law to
enable one to lawfully engage in recruitment and placement of workers; and (3) the illegal recruitment is
committed by a group of three (3) or more persons conspiring or confederating with one another.

FACTS:

PLAINTIFF:
Edgardo Dela Caza was introduced to accused Rodolfo Gallo y Gadot, Fides Pacardo, Pilar Manta, Mardeolyn
Martir, Lulu Mendanes, Yeo Sin Ung and another Korean national at the office of MPM International Recruitment
and Promotion Agency (MPM Agency). Dela Caza was told that Mardeolyn was the President of MPM Agency,
while Nelmar Martir was one of the incorporators. Also, that Marcelino Martir, Norman Martir, Nelson Martir and
Ma. Cecilia Ramos were its board members. Mendanes acted as the cashier and accountant, while Pacardo
acted as the agencys employee who was in charge of the records of the applicants. Manta, on the other hand,

Angeles| Bajana | Balladares | Brillantes | Briones | Cabansag | Callanta | Chua | David|

De Leon | Gomez | Lopez | Macalino | Nostratis | Padilla | Reynon | Santos | Tan |Velasco
4E / 4F - 2018-2019
Page 2 of 920
LABOR REVIEW DIGEST
Atty. Joyrich Golangco

was also an employee who was tasked to deliver documents to the Korean embassy. Gallo then introduced
himself as a relative of Mardeolyn and informed Dela Caza that the agency was able to send many workers
abroad.

Dela Caza was convinced and later paid MPM Agency through the accused Gallo. Two weeks later, he
discovered that the office had moved to a new location and was renamed to New Filipino Manpower Development
& Services, Inc.

After two (2) more months of waiting in vain to be deployed, Dela Caza and the other applicants decided to take
action. The first attempt was unsuccessful because the agency again moved to another place. However, they
were able to locate the new address. The agency explained that it had to move in order to separate those who are
applying as entertainers from those applying as factory workers. Accused-appellant Gallo, together with Pacardo
and Manta, were then arrested.

DEFENDANT:
Accused-appellant Gallo denied having any part in the recruitment of Dela Caza. In fact, he testified that he also
applied with MPM Agency for deployment to Korea as a factory worker. According to him, he gave his application
directly with Mardeolyn because she was his town mate and he was allowed to pay only Ten Thousand Pesos
(PhP 10,000) as processing fee. Further, in order to facilitate the processing of his papers, he agreed to perform
some tasks for the agency, such as taking photographs of the visa and passport of applicants, running errands
and performing such other tasks assigned to him, without salary except for some allowance. He said that he only
saw Dela Caza one or twice at the agencys office when he applied for work abroad. Lastly, that he was also
promised deployment abroad but it never materialized.

RTC RULING:
The RTC convicted the accused of syndicated illegal recruitment and estafa.

CA RULING:
The CA affirmed the RTC decision with modification to the penalty for estafa.

ISSUE/S:
1. Whether the accused-appellant is guilty of illegal recruitment committed by a syndicate.
2. Whether the accussed-appellant is guilty of estafa.

HELD:

1. YES. To commit syndicated illegal recruitment, three elements must be established: (1) the offender
undertakes either any activity within the meaning of recruitment and placement defined under Article
13(b), or any of the prohibited practices enumerated under Art. 34 of the Labor Code; (2) he has no valid
license or authority required by law to enable one to lawfully engage in recruitment and placement of
workers; and (3) the illegal recruitment is committed by a group of three (3) or more persons conspiring or
confederating with one another. When illegal recruitment is committed by a syndicate or in large scale, i.e., if it
is committed against three (3) or more persons individually or as a group, it is considered an offense involving
economic sabotage.

Under Art. 13(b) of the Labor Code, recruitment and placement refers to any act of canvassing, enlisting,
contracting, transporting, utilizing, hiring or procuring workers, and includes referrals, contract services, promising
or advertising for employment, locally or abroad, whether for profit or not.

Angeles| Bajana | Balladares | Brillantes | Briones | Cabansag | Callanta | Chua | David|

De Leon | Gomez | Lopez | Macalino | Nostratis | Padilla | Reynon | Santos | Tan |Velasco
4E / 4F - 2018-2019
Page 3 of 920
LABOR REVIEW DIGEST
Atty. Joyrich Golangco

Here, the evidence readily reveals that MPM Agency was never licensed by the POEA to recruit workers for
overseas employment. Even with a license, however, illegal recruitment could still be committed under Section 6
of Republic Act No. 8042 (R.A. 8042), otherwise known as the Migrants and Overseas Filipinos Act of 1995.

In the instant case, accused-appellant committed the acts enumerated in Sec. 6 of R.A. 8042. Testimonial
evidence shows that, in consideration of a promise of foreign employment, hereceived the amount of Php
45,000.00 from Dela Caza. When accused-appellant made misrepresentations concerning the agency’s
purported power and authority to recruit for overseas employment, and in the process, collected money in
the guise of placement fees, the former clearly committed acts constitutive of illegal recruitment.

The Court likewise finds the existence of a conspiracy between the accused-appellant and the other persons in
the agency who are currently at large, resulting in the commission of the crime of syndicated illegal recruitment.

In this case, it cannot be denied that the accused-appellent together with Mardeolyn and the rest of the officers
and employees of MPM Agency participated in a network of deception. Verily, the active involvement of each in
the recruitment scam was directed at one single purpose to divest complainants with their money on the pretext of
guaranteed employment abroad. The prosecution evidence shows that complainants were briefed by Mardeolyn
about the processing of their papers for a possible job opportunity in Korea, as well as their possible salary.
Likewise, Yeo Sin Ung, a Korean national, gave a briefing about the business and what to expect from the
company. Then, here comes accused-appellant who introduced himself as Mardeolyns relative and specifically
told Dela Caza of the fact that the agency was able to send many workers abroad. Dela Caza was even showed
several workers visas who were already allegedly deployed abroad. Later on, accused-appellant signed and
issued an official receipt acknowledging the down payment of Dela Caza. Without a doubt, the nature and extent
of the actions of accused-appellant, as well as with the other persons in MPM Agency clearly show unity of action
towards a common undertaking. Hence, conspiracy is evidently present.

Actual or direct proof of previous agreement to commit a crime is not essential to establish conspiracy as
it may be deduced from the mode and manner in which the offense was perpetrated.

Positive identification where categorical and consistent and not attended by any showing of ill motive on
the part of the eyewitnesses on the matter prevails over alibi and denial.

2. YES. All the elements of Estafa are present in the instant case: the accused-appellant, together with the other
accused at large, deceived the complainants into believing that the agency had the power and capability to send
them abroad for employment; that there were available jobs for them in Korea as factory workers; that by reason
or on the strength of such assurance, the complainants parted with their money in payment of the placement fees;
that after receiving the money, accused-appellant and his co-accused went into hiding by changing their office
locations without informing complainants; and that complainants were never deployed abroad. As all these
representations of the accused-appellant proved false, paragraph 2(a), Article 315 of the Revised Penal Code is
thus applicable.

Other Notes:
In establishing conspiracy, it is not essential that there be actual proof that all the conspirators took a direct part in
every act. It is sufficient that they acted in concert pursuant to the same objective.

Accused-appellants denial of the crimes charged crumbles in the face of the positive identification made by Dela
Caza and his co-complainants as one of the perpetrators of the crimes charged.

3. Claudio Yap v. Thenamaris Ships Management and Intermare Maritime Agencies, Inc.

Angeles| Bajana | Balladares | Brillantes | Briones | Cabansag | Callanta | Chua | David|

De Leon | Gomez | Lopez | Macalino | Nostratis | Padilla | Reynon | Santos | Tan |Velasco
4E / 4F - 2018-2019
Page 4 of 920
LABOR REVIEW DIGEST
Atty. Joyrich Golangco

G.R. No. 179532 Date: May 30, 2011


Ponente: Nachura, J.

Section 10 of RA 8042

DOCTRINE:
The clause or for three months for every year of the unexpired term, whichever is less provided in the
5th paragraph of Section 10 of R.A. No. 8042 is unconstitutional for being violative of the rights of Overseas
Filipino Workers (OFWs) to equal protection of the laws.

FACTS:
PLAINTIFF:
Claudio Yap was employed on August 14, 2001 as electrician of the vessel M/T SEASCOUT by the Intermare
Maritime Agencies, Inc., in behalf of its principal, Vulture Shipping Limited for a duration of 12 months. However,
on or about November 8, 2001, before the expiration of the contract, the vessel was sold. Yap received his
seniority bonus, vacation bonus, extra bonus along with the scrapping bonus. However, with respect to the
payment of his wage, he refused to accept the payment of one-month basic wage. He insisted that he was entitled
to the payment of the unexpired portion of his contract since he was illegally dismissed from employment. He
alleged that he opted for immediate transfer but none was made.

Petitioner filed a complaint for Illegal Dismissal with Damages and Attorneys Fees before the Labor Arbiter (LA).
He claimed that he was entitled to the salaries corresponding to the unexpired portion of his contract.
Subsequently, he filed an amended complaint, impleading Captain Francisco Adviento of respondents Intermare
Maritime Agencies, Inc. (Intermare) and Thenamaris Ships Management (respondents), together with C.J.
Martionos, Interseas Trading and Financing Corporation, and Vulture Shipping Limited/Stejo Shipping Limited.

DEFENDANT:
Respondents for their part, contended that Yap was not illegally dismissed. They alleged that following the sale of
the M/T SEASCOUT, Yap signed off from the vessel on 10 November 2001 and was paid his wages
corresponding to the months he worked or until 10 November 2001 plus his seniority bonus, vacation bonus and
extra bonus. They further alleged that Yap’s employment contract was validly terminated due to the sale of the
vessel and no arrangement was made for Yaps transfer to Thenamaris other vessels.

Labor Arbiter /RTC RULING / NLRC RULING:


The LA rendered a decision in favor of petitioner, finding the latter to have been constructively and illegally
dismissed by respondents. Moreover, the LA found that respondents acted in bad faith when they assured
petitioner of re-embarkation and required him to produce an electrician certificate during the period of his contract,
but actually he was not able to board one despite of respondents’ numerous vessels. Petitioner made several
follow-ups for his re-embarkation but respondents failed to heed his plea; thus, petitioner was forced to litigate in
order to vindicate his rights. Lastly, the LA opined that since the unexpired portion of petitioner’s contract was less
than one year, petitioner was entitled to his salaries for the unexpired portion of his contract for a period of nine
months.
The NLRC affirmed the LA’s findings that petitioner was indeed constructively and illegally dismissed; that
respondents’ bad faith was evident on their wilful failure to transfer petitioner to another vessel; and that the award
of attorneys fees was warranted. However, the NLRC held that instead of an award of salaries corresponding to
nine months, petitioner was only entitled to salaries for three months as provided under Section 10 of Republic Act
(R.A.) No. 8042, as enunciated in the ruling in Marsaman Manning Agency, Inc. v. National Labor Relations
Commission.

Angeles| Bajana | Balladares | Brillantes | Briones | Cabansag | Callanta | Chua | David|

De Leon | Gomez | Lopez | Macalino | Nostratis | Padilla | Reynon | Santos | Tan |Velasco
4E / 4F - 2018-2019
Page 5 of 920
LABOR REVIEW DIGEST
Atty. Joyrich Golangco

Upon the petitioner’s Motion for Partial Consideration, he NLRC reversed its earlier Decision, holding that there
can be no choice to grant only three (3) months salary for every year of the unexpired term because there is no
full year of unexpired term which this can be applied

CA RULING:
The CA affirmed the findings and ruling of the LA and the NLRC that petitioner was constructively and illegally
dismissed. However, the CA ruled that the NLRC erred in sustaining the LA’s interpretation of Section 10 of R.A.
No. 8042. In this regard, the CA relied on the clause or for three months for every year of the unexpired term,
whichever is less provided in the 5th paragraph of Section 10 of R.A. No. 8042

ISSUE/S:
Whether or not Section 10 of R.A. No. 8042, to the extent that it affords an illegally dismissed migrant worker the
lesser benefit of salaries for the unexpired portion of his employment contract or for three (3) months for
every year of the unexpired term, whichever is less is constitutional.

HELD:

NO. In the meantime, while this case was pending, the Court declared as unconstitutional the clause or for
three months for every year of the unexpired term, whichever is less provided in the 5th paragraph of
Section 10 of R.A. No. 8042 in the case of Serrano v. Gallant Maritime Services, Inc. on March 24, 2009 for
being violative of the rights of Overseas Filipino Workers (OFWs) to equal protection of the laws.

The Court concludes that the subject clause contains a suspect classification in that, in the computation
of the monetary benefits of fixed-term employees who are illegally discharged, it imposes a 3-month cap
on the claim of OFWs with an unexpired portion of one year or more in their contracts, but none on the
claims of other OFWs or local workers with fixed-term employment. The subject clause singles out one
classification of OFWs and burdens it with a peculiar disadvantage.

Moreover, this Court held therein that the subject clause does not state or imply any definitive governmental
purpose; hence, the same violates not just therein petitioners right to equal protection, but also his right to
substantive due process under Section 1, Article III of the Constitution. Consequently, petitioner therein was
accorded his salaries for the entire unexpired period of nine months and 23 days of his employment contract,
pursuant to law and jurisprudence prior to the enactment of R.A. No. 8042.
Thus, the present case should not be different from Serrano.

Other Notes:
As a general rule, an unconstitutional act is not a law; it confers no rights; it imposes no duties; it affords no
protection; it creates no office; it is inoperative as if it has not been passed at all. The doctrine of operative fact, as
an exception to the general rule, only applies as a matter of equity and fair play. It nullifies the effects of an
unconstitutional law by recognizing that the existence of a statute prior to a determination of unconstitutionality is
an operative fact and may have consequences which cannot always be ignored. The past cannot always be
erased by a new judicial declaration. The doctrine is applicable when a declaration of unconstitutionality will
impose an undue burden on those who have relied on the invalid law.

Angeles| Bajana | Balladares | Brillantes | Briones | Cabansag | Callanta | Chua | David|

De Leon | Gomez | Lopez | Macalino | Nostratis | Padilla | Reynon | Santos | Tan |Velasco
4E / 4F - 2018-2019
Page 6 of 920
LABOR REVIEW DIGEST
Atty. Joyrich Golangco

4. People v. Hon. Domingo Panis, Presiding Judge of the Court of First Instance of Zambales & Olongapo
City, Branch III and Serapion Abug
G.R. Nos. L-58674-77 Date: July 11, 1990
Ponente: Cruz, J.

Article 13(b) of PD 442

DOCTRINE/S:
The number of persons dealt with is NOT an essential ingredient of the act of recruitment and placement of
workers. The words “shall be deemed” merely creates that presumption.

FACTS:
Four informations were filed alleging that Serapio Abug, private respondent, without first securing a license from
the Ministry of Labor as a holder of authority to operate a fee-charging employment agency, did then and there
wilfully, unlawfully and criminally operate a private fee charging employment agency by charging fees and
expenses and promising employment in Saudi Arabia to four separate individuals, in violation of Article 16 in
relation to Article 39 of the Labor Code.

Abug filed a motion to quash on the ground that the informations did not charge an offense because he was
accused of illegally recruiting only one person in each of the four informations. Under the proviso in Article 13(b),
he claimed, there would be illegal recruitment only "whenever two or more persons are in any manner promised or
offered any employment for a fee.”

PLAINTIFF:
The posture of the petitioner is that the private respondent is being prosecuted under Article 39 in relation to
Article 16 of the Labor Code; hence, Article 13(b) is not applicable. However, as the first two cited articles penalize
acts of recruitment and placement without proper authority, which is the charge embodied in the informations,
application of the definition of recruitment and placement in Article 13(b) is unavoidable.
On the other hand, the petitioner argues that the requirement of two or more persons is imposed only where the
recruitment and placement consists of an offer or promise of employment to such persons and always in
consideration of a fee. The other acts mentioned in the body of the article may involve even only one person and
are not necessarily for profit.

DEFENDANT:
The view of the private respondents is that to constitute recruitment and placement, all the acts mentioned in this
article should involve dealings with two or more persons as an indispensable requirement.

RTC RULING:
Denied at first, the motion was reconsidered and finally granted by the trial court.

ISSUE:
Whose interpretation of Article 13(b) of PD 442 is correct?

RATIO:

Angeles| Bajana | Balladares | Brillantes | Briones | Cabansag | Callanta | Chua | David|

De Leon | Gomez | Lopez | Macalino | Nostratis | Padilla | Reynon | Santos | Tan |Velasco
4E / 4F - 2018-2019
Page 7 of 920
LABOR REVIEW DIGEST
Atty. Joyrich Golangco

Neither interpretation is acceptable. We fail to see why the proviso should speak only of an offer or promise of
employment if the purpose was to apply the requirement of two or more persons to all the acts mentioned in the
basic rule. For its part, the petitioner does not explain why dealings with two or more persons are needed where
the recruitment and placement consists of an offer or promise of employment but not when it is done through
"canvassing, enlisting, contracting, transporting, utilizing, hiring or procuring of workers.

The proviso was intended neither to impose a condition on the basic rule nor to provide an exception thereto but
merely to create a presumption. The presumption is that the individual or entity is engaged in recruitment and
placement whenever he or it is dealing with two or more persons to whom, in consideration of a fee, an offer or
promise of employment is made in the course of the "canvassing, enlisting, contracting, transporting, utilizing,
hiring or procuring workers. "

The number of persons dealt with is not an essential ingredient of the act of recruitment and placement of
workers. Any of the acts mentioned in the basic rule in Article 13(b) will constitute recruitment and placement even
if only one prospective worker is involved. The proviso merely lays down a rule of evidence that where a fee is
collected in consideration of a promise or offer of employment to two or more prospective workers, the individual
or entity dealing with them shall be deemed to be engaged in the act of recruitment and placement. The words
"shall be deemed" create that presumption.

5. Trans Action Overseas v. DOLE Secretary. et. al


G.R. No. 109583 September 5, 1997
ROMERO, J.:

Article 35. Suspension and/or Cancellation of License or Authority

DOCTRINE:
The power to suspend or cancel any license or authority to recruit employees for overseas employment is
concurrently vested with the POEA and the Secretary of Labor.

FACTS:
Trans Action Overseas Corp. , a private fee-charging employment agency, scoured Iloilo City for possible recruits
for alleged job vacancies in Hongkong. Private respondents sought employment as domestic helpers through
petitioners employees. The applicants paid placement fees ranging from P1,000.00 to P14,000.00, but petitioner
failed to deploy them. Their demands for refund proved unavailing; thus, they were constrained to institute
complaints against petitioner for violation of Articles 32 and 34(a) of the Labor Code.

Petitioner denied having received the amounts allegedly collected from respondents and that their employees only
duty was to pre-screen and interview applicants and they were not authorized to collect fees from applicants.

LA RULING:
Labor Undersecretary decided in favor of the private respondents and rendered liable the agency for twenty eight
(28) counts of violation of Article 32 and five (5) counts of Article 34 (a) with a corresponding suspension in the
aggregate period of sixty six (66) months. Further, the license of the agency was cancelled on the ground that any
suspension amounting to a period of 12 months merits the imposition of the penalty of cancellation.

Angeles| Bajana | Balladares | Brillantes | Briones | Cabansag | Callanta | Chua | David|

De Leon | Gomez | Lopez | Macalino | Nostratis | Padilla | Reynon | Santos | Tan |Velasco
4E / 4F - 2018-2019
Page 8 of 920
LABOR REVIEW DIGEST
Atty. Joyrich Golangco

PETITIONER:
Petitioner contended that Secretary Confesor acted with grave abuse of discretion in rendering the assailed
orders on alternative grounds, viz.:
(1) it is the Philippine Overseas Employment Administration (POEA) which has the exclusive and original
jurisdiction to hear and decide illegal recruitment cases, including the authority to cancel recruitment licenses, or
(2) the cancellation order based on the 1987 POEA Schedule of Penalties is not valid for non-compliance with the
Revised Administrative Code of 1987 regarding its registration with the U.P. Law Center.

ISSUES:
1. Whether or not the Secretary of Labor and Employment has jurisdiction to cancel or revoke the license of a
private fee-charging employment agency.
2. Whether or not the 1987 POEA Schedule of Penalties is valid

HELD:
1. Yes, the Secretary of Labor and Employment has jurisdiction to cancel or revoke the license of a private fee-
charging employment agency as provided in Article 35 of the Labor Code. Article 35 provides that the Minister of
Labor shall have the power to suspend or cancel any license or authority to recruit employees for overseas
employment for violation of rules and regulations issued by the Ministry of Labor, the Overseas Employment
Development Board, and the National Seamen Board, or for violation of the provisions of this and other applicable
laws, General Orders and Letters of Instructions. We rule that the power to suspend or cancel any license or
authority to recruit employees for overseas employment is concurrently vested with the POEA and the
Secretary of Labor.

2. Yes, the 1987 POEA Schedule of Penalties is valid on the ground that it was issued pursuant to Article 34 of
the Labor Code. The questioned schedule of penalties contains only a listing of offenses. It does not prescribe
additional rules and regulations governing overseas employment but only detailed the administrative sanctions
imposable by this Office for some enumerated prohibited acts. Further, the license of the respondent agency was
cancelled on the authority of Article 35 of the Labor Code, as amended, and not pursuant to the 1987 POEA
Revised Rules on Schedule of Penalties.

6. Republic v Principalia Management and Personnel Consultants Inc.


G.R. No. 167639 April 19, 2006
YNARES-SANTIAGO, J.:

Section 5, Rule V, Part VI of the 2002 POEA Rules and Regulation

DOCTRINE:
Until such time that the appeal (as to the Order of Suspension) is resolved with finality by the DOLE, Principalia
has a clear and convincing right to operate as a recruitment agency.

FACTS:
This case stemmed from two separate complaints filed before POEA against Principalia. The complaints
essentially alleged the failure of Principalia to deploy the applicants for employment abroad despite paying a fee
required by Principalia.

The first complaint was filed by Ruth Yasmin Concha where she alleged that after paying P20,000.00 fee required
by Principalia which was not properly receipted, Principalia failed to deploy her for employment abroad as
caregiver or physical therapist. The Adjudication Office of the POEA found Principalia liable for violations of the
2002 POEA Rules and Regulations.
Angeles| Bajana | Balladares | Brillantes | Briones | Cabansag | Callanta | Chua | David|

De Leon | Gomez | Lopez | Macalino | Nostratis | Padilla | Reynon | Santos | Tan |Velasco
4E / 4F - 2018-2019
Page 9 of 920
LABOR REVIEW DIGEST
Atty. Joyrich Golangco

The second complaint was filed by Rafael E. Baldoza. He alleged that Principalia assured him of employment in
Doha, Qatar as a machine operator with a monthly salary of $450.00. After paying P20,000.00 as placement fee,
he departed but when he arrived, he was made to work as welder. An alternative position as helper was offered to
him, which he refused. Thus, he was repatriated.

Baldoza and Principalia entered into a compromise agreement with quitclaim and release whereby the latter
agreed to redeploy Baldoza for employment abroad. Principalia, however, failed to deploy Baldoza as agreed
hence, in an Order dated April 29, 2004, the POEA suspended Principalia’s documentary processing. Principalia
moved for reconsideration which the POEA granted on June 25, 2004. The latter lifted its order suspending the
documentary processing by Principalia after noting that it exerted efforts to obtain overseas employment for
Baldoza within the period stipulated in the settlement agreement but due to Baldoza’s lack of qualification, his
application was declined by its foreign principal.

DEFENDANT:
Before the promulgation of POEA’s order lifting the suspension Principalia filed a complaint against the
Administrator of POEA and the POEA Conciliator before the RTC for Annulment of Order for Suspension of
Documentation Processing with Damages and Application for Issuance of a Temporary Restraining Order
and/or Writ of Preliminary Injunction. Principalia claimed that the suspension of its documentary processing
would ruin its reputation and goodwill and would cause the loss of its applicants, employers and principals. Thus,
a writ of preliminary injunction and a writ of mandatory injunction must be issued to prevent serious and
irreparable damage to it.

RTC RULING:

The RTC granted a 72-hour restraining order enjoining the defendants from imposing the suspension orders
before the matter can be heard in full. However, since the issue has become moot because POEA had already
released the renewal of license of Principalia, the RTC issued the Writ of Preliminary Prohibitory Injunction prayed
for by the plaintiff, upon posting of a bond in the amount of Php 500,000.00, stressing that the Order of
Suspension dated March 15, 2004 is still pending appeal before the Office of the Secretary of Labor and
Employment, and that the said Order dated March 15, 2004 does not categorically state that the suspension of
Plaintiff’s License is immediately executory contrary to the contention of the defendants.

PETITIONER:
Counsel for POEA argued that the basis for the immediate implementation thereof is Section 5, Rule V, Part VI
of the 2002 POEA Rules and Regulation, which provides that the decision of the Administration shall be
stayed during the pendency of the appeal; Provided that where the penalty imposed carried the maximum
penalty of twelve (12) months suspension o[r] cancellation of license, the decision shall be immediately
executory despite pendency of the appeal.

POEA avers that the trial court gravely abused its discretion in granting the writ of preliminary prohibitory
injunction when the requirements to issue the same have not been met. It asserts that Principalia had no clear
and convincing right to the relief demanded as it had no proof of irreparable damage as required under the Rules
of Court.

CA RULING:
POEA filed a petition for certiorari before the CA but the case was dismissed outright for failure to attach copies of
all relevant pleadings and transcripts of the hearings and denied the motion for reconsideration.

ISSUES:
Angeles| Bajana | Balladares | Brillantes | Briones | Cabansag | Callanta | Chua | David|

De Leon | Gomez | Lopez | Macalino | Nostratis | Padilla | Reynon | Santos | Tan |Velasco
4E / 4F - 2018-2019
Page 10 of 920
LABOR REVIEW DIGEST
Atty. Joyrich Golangco

1. Whether or not a Writ of Preliminary Prohibitory Injunction will lie against the immediate implementation of the
Order of Suspension of License issued by POEA Administrator.
2. Whether or not the CA erred in dismissing the petition for certiorari based on technical grounds.

HELD:
1. Yes, the writ of preliminary prohibitory injunction will lie against the order of suspension on the ground that
POEA would have no authority to exercise its regulatory functions over Principalia because the matter had
already been brought to the jurisdiction of the DOLE. Principalia has been granted the license to recruit and
process documents for Filipinos interested to work abroad. Thus, POEAs action of suspending Principalia’s
license before final adjudication by the DOLE would be premature and would amount to a violation of the
latter’s right to recruit and deploy workers.

For all intents and purposes, POEA can determine whether the licensee has complied with the
requirements. In this instance, the trial court observed that the Order of Suspension dated March 15, 2004 was
pending appeal with the DOLE Secretary. Thus, until such time that the appeal is resolved with finality by the
DOLE, Principalia has a clear and convincing right to operate as a recruitment agency.

Furthermore, irreparable damage was duly proven by Principalia. Suspension of its license is not easily
quantifiable nor is it susceptible to simple mathematical computation, as alleged by POEA.If the injunctive writ was
not granted, Principalia would have been labeled as an untrustworthy recruitment agency before there could be
any final adjudication of its case by the DOLE. It would have lost both its employer-clients and its prospective
Filipino-applicants. Loss of the former due to a tarnished reputation is not quantifiable.

2. No, the CA did not err in dismissing the petition for certiorari based on technical grounds. POEA failed to
comply with Section 3, Rule 46 and Section 1, Rule 65 of the Rules of Court. Further, POEA still failed to attach a
copy of the Memorandum which the CA deemed essential in its determination of the propriety of the trial courts
issuance of the writ of preliminary prohibitory injunction. We cannot fault the CA for dismissing the petition
because the POEA did not demonstrate willingness to comply with the requirements set by the rules and to submit
the necessary documents which the Court of Appeals need to have a proper perspective of the case.

7. Datuman v. First Cosmopolitan Manpower


G.R. No. 156029 November 14, 2008
LEONARDO-DE CASTRO, J.:

SECTION 10, RA 8042

DOCTRINE:
The signing of the "substitute" contracts with the foreign employer/principal before the expiration of the POEA-
approved contract and any continuation of petitioner's employment beyond the original one-year term, against the
will of petitioner, are continuing breaches of the original POEA-approved contract.

FACTS:
First Cosmopolitan Manpower recruited Datuman as a saleslady in Bahrain with a monthly salary of US$370 for a
period of 1 year. On April 17, 1989, petitioner was deployed to Bahrain after paying the required placement fee.
However, her employer took her passport when she arrived there; and instead of working as a saleslady, she was
forced to work as a domestic helper with a salary of Forty Bahrain Dinar (BD40.00), equivalent only to One
Hundred US Dollars (US$100.00). On September 1, 1989, her employer compelled her to sign another contract,
transferring her to another employer as housemaid with a salary of BD40.00 for the duration of two (2) years.

Angeles| Bajana | Balladares | Brillantes | Briones | Cabansag | Callanta | Chua | David|

De Leon | Gomez | Lopez | Macalino | Nostratis | Padilla | Reynon | Santos | Tan |Velasco
4E / 4F - 2018-2019
Page 11 of 920
LABOR REVIEW DIGEST
Atty. Joyrich Golangco

PETITIONER:
In May 1995, petitioner filed a complaint before the POEA Adjudication Office against respondent for
underpayment and nonpayment of salary, vacation leave pay and refund of her plane fare. While the case was
pending, she filed the instant case before the NLRC for underpayment of salary for a period of one year and six
months, nonpayment of vacation pay and reimbursement of return airfare.

RESPONDENT:
In its Position Paper, respondent countered that petitioner actually agreed to work in Bahrain as a housemaid for
one (1) year because it was the only position available then. However, since such position was not yet allowed by
the POEA at that time, they mutually agreed to submit the contract to the POEA indicating petitioners position as
saleslady. Also, respondent raised the defense of prescription of cause of action since the claim was filed beyond
the three (3)-year period from the time the right accrued, reckoned from either 1990 or 1991.

LA RULING:
The Labor Arbiter ruled that respondent violated the terms of employment contract and ordered it to pay the
amount of US$4,050.00 representing the salary differentials for 15 months and the refund of her plane ticket.

NLRC RULING:
On appeal, the NLRC affirmed with modification the decision of the LA by reducing the amount to US$2,970
representing the salary differentials for 11 months.
CA RULING:
Upon a petition for certiorari before the CA, the CA reversed the NLRC and LA on the ground that the agency was
liable only with the first contract.

ISSUES:
1. Whether or not the agency was liable only with the first contract.
2. Whether or not the claims for underpaid salaries have prescribed.

HELD:
1. No, the agent was not liable only with the first contract because respondent is jointly and solidarily liable with
the latter’s principal employer abroad for her (petitioners) money claims. The signing of the substitute contracts
with the foreign employer/principal before the expiration of the POEA-approved contract and any continuation of
petitioners employment beyond the original one-year term, against the will of petitioner, are continuing breaches
of the original POEA-approved contract. Republic Act No. 8042 explicitly prohibits the substitution or alteration to
the prejudice of the worker of employment contracts already approved and verified by the Department of Labor
and Employment (DOLE) from the time of actual signing thereof by the parties up to and including the period of
the expiration of the same without the approval of the DOLE. Hence, in the present case, the diminution in the
salary of petitioner from US$370.00 to US$100 (BD 40.00) per month is void for violating the POEA-approved
contract which set the minimum standards, terms, and conditions of her employment. Consequently, the solidary
liability of respondent with petitioners foreign employer for petitioners money claims continues although she was
forced to sign another contract in Bahrain.

2. No, the claims for underpaid salaries have not prescribed. Article 291 of the Labor Code provides that all
money claims arising from employer-employee relations accruing during the effectivity of this Code shall be filed
within three years from the time that cause of action accrued; otherwise, they shall be forever barred. The right to
claim unpaid salaries (or in this case, unpaid salary differentials) accrue as they fall due. To determine for which
months petitioners right to claim salary differentials has not prescribed, we must count three years prior to the
filing of the complaint on May 31, 1995. Thus, only claims accruing prior to May 31, 1992 have prescribed when
the complaint was filed on May 31, 1995. Petitioner is entitled to her claims for salary differentials for the period
May 31, 1992 to April 1993, or approximately eleven (11) months.
Angeles| Bajana | Balladares | Brillantes | Briones | Cabansag | Callanta | Chua | David|

De Leon | Gomez | Lopez | Macalino | Nostratis | Padilla | Reynon | Santos | Tan |Velasco
4E / 4F - 2018-2019
Page 12 of 920
LABOR REVIEW DIGEST
Atty. Joyrich Golangco

8. Stolt-Nielsen Transportation Group v Medequillo


G.R. No. 177498 January 18, 2012
PEREZ, J.:

SECTION 10, RA 8042


Art. 2199, NCC

DOCTRINE: Even before the start of any employer-employee relationship, contemporaneous with the perfection
of the employment contract was the birth of certain rights and obligations, the breach of which may give rise to a
cause of action against the erring party.

FACTS:
Medequillo was hired by petitioners as Third Assistant Engineer for a period of 9 months in the first contract but
after 3 months Medequilllo was sent home and deployed to another vessel. Thereafter, they executed another
contract but petitioners failed to deploy him.

Medequillo filed a complaint before the Adjudication Office of the POEA against the petitioners for illegal dismissal
under a first contract and for failure to deploy under a second contract. Medequilla prayed for payment of
damages as well as attorney’s fees for his illegal dismissal and in view of the Petitioners’ bad faith in not
complying with the Second Contract. The case was transferred to the Labor Arbiter of the DOLE upon the
effectivity of the Migrant Workers and Overseas Filipinos Act of 1995.

Petitioners failed to submit their respective pleadings despite the opportunity given to them.

LA RULING:
The LA found the first contract entered into by and between the complainant and the respondents to have been
novated by the execution of the second contract, declared the petitioners guilty of constructively dismissing
Medequillo, and ordered to pay Medequillo $12,537.00.

NLRC RULING:
The NLRC affirmed with modification the decision of LA by deleting the amount of overtime pay in the total
amount.

CA RULING:
The petitioners filed a Petition for Certiorari before the Court of Appeals. The CA affirmed the decision of the labor
tribunal.

Petitioners argued that under the POEA Contract, actual deployment of the seafarer is a suspensive condition for
the commencement of the employment.

ISSUES:
1. Whether or not the petitioners are liable to Medequillo despite the absence of employer-employee relationship
between them.
2. In case of absence in the POEA Rules as to award of damages in favor of the employees, what law should
govern?

HELD:

Angeles| Bajana | Balladares | Brillantes | Briones | Cabansag | Callanta | Chua | David|

De Leon | Gomez | Lopez | Macalino | Nostratis | Padilla | Reynon | Santos | Tan |Velasco
4E / 4F - 2018-2019
Page 13 of 920
LABOR REVIEW DIGEST
Atty. Joyrich Golangco

1. Yes, the petitioners are liable to Medequillo despite absence of employer and employee relationship between
them because the perfected contract gives rise to obligations on the part of petitioners. Under the POEA Standard
Employment Contract, employment shall commence “upon the actual departure of the seafarer from the airport or
seaport in the port of hire.” Thus, even if by the standard contract employment commences only “upon actual
departure of the seafarer”, this does not mean that the seafarer has no remedy in case of non-deployment without
any valid reason.

2. The absence of the POEA Rules with regard to the payment of damages to the affected seafarer does not
mean that the seafarer is precluded from claiming the same. Section 10 of Republic Act No. 8042 (Migrant
Workers Act) provides for money claims by reason of a contract involving Filipino workers for overseas
deployment. Applying the rules on actual damages, Article 2199 of the New Civil Code provides that one is
entitled to an adequate compensation only for such pecuniary loss suffered by him as he has duly proved.
Petitioners are thus liable to pay Medequillo actual damages in the form of the loss of nine (9) months’ worth of
salary as provided in the contract. This is but proper because of the non-deployment of respondent without just
cause.

9. THE PEOPLE OF THE PHILIPPINES vs. CAROL M. DELA PIEDRA


G.R. No. 121777 Date: January 24, 2001
Ponente: KAPUNAN, J.

Articles 13 (b) and 34 of Labor Code

DOCTRINE: Illegal recruitment is committed when two elements concur. First, the offender has no valid license or
authority required by law to enable one to lawfully engage in recruitment and placement of workers. Second, he or
she undertakes either any activity within the meaning of "recruitment and placement" defined under Article 13 (b),
or any prohibited practices enumerated under Article 34 of the Labor Code.38 In case of illegal recruitment in
large scale, a third element is added: that the accused commits said acts against three or more persons,
individually or as a group.

FACTS:
Carol Dela Piedra (aka Carol Figueroa/Carol Llena) was charged with illegal recruitment in large scale before
Zamboanga City RTC. The information alleged that she, without having previously obtained a license or authority
to engage in recruitment and overseas placement of workers from POEA, offered and promised for a fee
employment as nurses in a certain hospital in Singapore to the 3 registered nurses (Maria Lourdes Gadrino,
Nancy Araneta, and Jennelyn Baez) who all advanced P2K for the promised employment that did not
materialized. Carol was arrested in a raid conducted by the POEA and the PNP-Criminal Investigation Service
(CIS). The raiding team utilized a certain Eileen, who pretended to be an applicant, the latter saw the 3 other
applicants arrived at the house to follow up the result of their applications and to give their advance payment to
certain Jasmine Alejandro, who handed their papers to them. In the course of their investigation, POEA revealed
that the Carol was not licensed or authorized to conduct recruitment.

Accused’s version: (frame-up)


 Carol as a tourist in Singapore, met in a Christmas party a certain Laleen Malicay who sought her help. Laleen
wanted Carol to verify from Jasmine Alejandro (Laleen’s cousin), if Laleen’s father received the P15K she sent
him.
 When Carol arrived at Jasmine’s house, the latter informed her that the money was received and told Carol to
come back before her flight to Singapore as she will be sending something for Laleen.
Angeles| Bajana | Balladares | Brillantes | Briones | Cabansag | Callanta | Chua | David|

De Leon | Gomez | Lopez | Macalino | Nostratis | Padilla | Reynon | Santos | Tan |Velasco
4E / 4F - 2018-2019
Page 14 of 920
LABOR REVIEW DIGEST
Atty. Joyrich Golangco

 At Jasmine’s house – Carol met a woman who asked her, “Are you Carol from Singapore?” and inquired if she
was recruiting. She replied in the negative, explaining she was just saying goodbye to Jasmine. The woman
further asked what the requirements if she were to go to Singapore, Carol replied that she would need a
passport.
 Minutes after, CIS and POEA arrived. They showed Carol some papers (filled up application forms) found in the
house. Carol alleged those papers were given by Laleen to Jasmine because Laleen wanted to go home but
could not find a replacement.

RTC RULING: GUILTY BEYOND REASONABLE DOUBT OF ILLEGAL RECRUITMENT COMMITTED IN A


LARGE SCALE W/ PENALTY OF LIFE IMPRISONMENT

DEFENDANT:
1. Sec. 13 (b) of the Labor Code, as amended, defining recruitment and placement is void for
vagueness and violates the due process clause, hence, unconstitutional. She invokes People vs.
Panis where SC allegedly criticized the definition of recruitment and placement, adding that if SC in the said
case had to speculate on the meaning of Article 13 (b), what more the ordinary citizen who does not possess
the necessary [legal] knowledge?
2. The acts that constitute recruitment and placement suffer from overbreadth because by merely
referring a person for employment, a person may be convicted of illegal recruitment.
Violation of the equal protection clause. Although the evidence shows that Jasmine Alejandro handed out
application forms and even received payment, the accused was the only one criminally charged. The prosecution
discriminated against her on grounds of regional origins. Accused is Cebuana while Alejandro is a Zamboanguea,
and the alleged crime took place in Zamboanga City.

ISSUE/S:
1. Whether Sec. 13 (b) of the Labor Code, as amended, unconstitutional.
2. Whether the accused guilty of illegal recruitment in large scale.
Whether the equal protection clause was violated.

HELD:
1. NO
a. Accused’s reliance on People vs. Panis is misplaced. The issue in Panis was whether, under the proviso of
Article 13 (b), the crime of illegal recruitment could be committed only whenever two or more persons are in
any manner promised or offered any employment for a fee. SC held in the negative, explaining that the
proviso was intended neither to impose a condition on the basic rule nor to provide an exception thereto but
merely to create a disputable presumption or prima facie evidence that where a fee is collected in
consideration of a promise or offer of employment even if only one prospective worker is involved, the
individual or entity dealing with them shall be deemed to be engaged in the act of recruitment and placement.
The SC, in Panis, merely bemoaned the lack of records that would help shed light on the meaning of the
proviso. Notwithstanding the absence of such records, the SC was able to arrive at a reasonable
interpretation of the proviso by applying principles in criminal law and drawing from the language and intent
of the law itself. Section 13 (b), therefore, is not a perfectly vague act whose obscurity is evident on its face. If
at all, the proviso therein is merely couched in imprecise language that was salvaged by proper construction.
It is not void for vagueness.
An act will be declared void and inoperative on the ground of vagueness and uncertainty, only upon a showing
that the defect is such that the courts are unable to determine, with any reasonable degree of certainty, what the
legislature intended. An Act will not be declared inoperative and ineffectual on the ground that it furnishes no
adequate means to secure the purpose for which it is passed, if men of common sense and reason can devise
and provide the means, and all the instrumentalities necessary for its execution are within the reach of those

Angeles| Bajana | Balladares | Brillantes | Briones | Cabansag | Callanta | Chua | David|

De Leon | Gomez | Lopez | Macalino | Nostratis | Padilla | Reynon | Santos | Tan |Velasco
4E / 4F - 2018-2019
Page 15 of 920
LABOR REVIEW DIGEST
Atty. Joyrich Golangco

entrusted therewith.

b. Accused misapprehends concept of overbreadth. That Section 13 (b) encompasses what accused apparently
considers as customary and harmless acts such as labor or employment referral (referring an applicant,
according to appellant, for employment to a prospective employer) does not render the law overbroad.
A statute may be said to be overbroad where it operates to inhibit the exercise of individual freedoms affirmatively
guaranteed by the Constitution, such as the freedom of speech or religion. A generally worded statute, when
construed to punish conduct which cannot be constitutionally punished is unconstitutionally vague to the extent
that it fails to give adequate warning of the boundary between the constitutionally permissible and the
constitutionally impermissible applications of the statute. In the present case, however, accused did not even
specify what constitutionally protected freedoms are embraced by the definition of recruitment and placement that
would render the same constitutionally overbroad.

2. NO, but she is guilty of 2 counts of simple illegal recruitment. Illegal recruitment is committed when 2
elements concur:
(1) The offender has no valid license or authority required by law to enable one to lawfully engage in recruitment
and placement of workers.
(2) He or she undertakes either any activity within the meaning of recruitment and placement defined under
Article 13 (b), or any prohibited practices enumerated under Article 34 of the Labor Code.
(3) The accused commits said acts against three or more persons, individually or as a group (only in
case of illegal recruitment in large scale).
In this case, the 1st and 2nd elements are present. (1st element) The certification of POEA states that
accused is not licensed or authorized to engage in recruitment and placement. (2nd element) Accused is presumed
engaged in recruitment and placement under Article 13 (b) of the Labor Code based on the testimonies (Araneta
and Modesto) coming from the 2 credible witnesses, meeting the standard of proof beyond reasonable doubt that
accused committed recruitment and placement.
However, a conviction for large scale illegal recruitment must be based on a finding in each case of illegal
recruitment of three or more persons whether individually or as a group. In this case, only 2 persons, Araneta
and Modesto, were proven to have been recruited by appellant. The 3rd person named in the complaint, Jennelyn
Baez, was not presented in court to testify. It is true that law does not require that at least three victims testify at
the trial; nevertheless, it is necessary that there is sufficient evidence proving that the offense was committed
against three or more persons. The affidavit of Baez was neither identified, nor its contents affirmed, hence, it is
hearsay and inadmissible.
Neither can accused be convicted for recruiting CIS agent Eileen or even the other persons present in her
briefing. The information does not include Eileen or the other persons present in her briefing but includes only
Araneta, Modesto and Baez. To convict her for the recruitment and placement of persons other than those alleged
would violate her right to be informed of the nature and cause of the accusation against her. Thus, the penalty of
life imprisonment imposed must be reduced to 2 counts of simple illegal recruitment.

3. NO.
The prosecution of one guilty person while others equally guilty are not prosecuted, however, is not, by itself, a
denial of the equal protection of the laws. Where the official action purports to be in conformity to the statutory
classification, an erroneous or mistaken performance of the statutory duty, although a violation of the statute, is
not without more a denial of the equal protection of the laws. But a discriminatory purpose is not presumed,
there must be a showing of clear and intentional discrimination.

10. ESTATE OF NELSON R. DULAY, represented by his wife MERRIDY JANE P. DULAY vs. ABOITIZ
JEBSEN MARITIME, INC. and GENERAL CHARTERERS, INC.

Angeles| Bajana | Balladares | Brillantes | Briones | Cabansag | Callanta | Chua | David|

De Leon | Gomez | Lopez | Macalino | Nostratis | Padilla | Reynon | Santos | Tan |Velasco
4E / 4F - 2018-2019
Page 16 of 920
LABOR REVIEW DIGEST
Atty. Joyrich Golangco

G.R. No. 172642 Date: June 13, 2012


Ponente: PERALTA, J.

- Rule 7, Sec. 7 of Omnibus IRRs of R.A. No. 8042, as amended by R.A. No. 10022
-
DOCTRINE:

DOCTRINE/S: Disputes involving claims of Filipino seafarers wherein the parties are covered by a CBA should be
submitted to the jurisdiction of a voluntary arbitrator or panel of arbitrators. Only in the absence of a CBA that
parties may opt to submit the dispute to either the NLRC or to voluntary arbitration.

FACTS:
Nelson R. Dulay was hired as a seaman by General Charterers Inc. (GCI), a subsidiary of Aboitiz, later he
became a bosun (officer on merchant ship in charge of the hull and related equipment) on a contractual basis. 25
days after the completion of his contract, Nelson died due to acute renal failure secondary to septicemia. He was
a bona fide member of GCI’s collective bargaining agent–Associated Marine Officers and Seamans Union of the
Philippines (AMOSUP). Nelsons widow, Merridy Jane, claimed for death benefits through the grievance procedure
of the CBA between AMOSUP and GCI, but the procedure was declared deadlocked the denial of her benefits.
Thus, she filed a complaint for death and medical benefits and damages against GCI with NLRC. GCI was
released form liability after paying Nelson’s relatives P20K pursuant to Article 20(A)2 of the CBA. Merridy Jane
averred that the P20K should be considered advance payment of the total claim of $90K.

LA RULING:
It took cognizance of the case under Article 217 (a), par.6 of the Labor Code and the existence of a
reasonable causal connection between the employer-employee relationship and the claim asserted. It ordered
GCI to pay the equivalent of $90K less P20K but ruled that the proximate cause of Nelsons death was not work-
related.

NLRC RULING:
Affirmed the LA decision as to the grant of death benefits under the CBA but reversed the ruling as to the
proximate cause of Nelsons death.

CA RULING:
While the suit filed by Merridy Jane is a money claim, the same basically involves the interpretation and
application of the provisions in the subject CBA. Thus, jurisdiction belongs to the voluntary arbitrator and not
the labor arbiter.

PETITIONER:
Section 10 of R.A. 8042 (Migrant Workers and Overseas Filipinos Act of 1995) applies in this case. It vests
jurisdiction on the appropriate branches of the NLRC to entertain disputes regarding the interpretation of a CBA
involving migrant or overseas Filipino workers. The Section 10 amended Article 217(c) of the Labor Code, which
confers jurisdiction upon voluntary arbitrators over interpretation or implementation of collective bargaining
agreements and interpretation or enforcement of company personnel policies.

RESPONDENTS:
1. NLRC had no jurisdiction over the action on account of the absence of employer-employee relationship
between GCI and Nelson at the time of his death. Nelson also had no claims against petitioners for sick leave
allowance/medical benefit by reason of the completion of his contract with GCI.

Angeles| Bajana | Balladares | Brillantes | Briones | Cabansag | Callanta | Chua | David|

De Leon | Gomez | Lopez | Macalino | Nostratis | Padilla | Reynon | Santos | Tan |Velasco
4E / 4F - 2018-2019
Page 17 of 920
LABOR REVIEW DIGEST
Atty. Joyrich Golangco

Article 217, par. (c) and Article 261 of the Labor Code apply in this case. Under these provisions, the
unresolved grievances arising from the interpretation and implementation of collective bargaining agreements
remains with voluntary arbitrators.

ISSUE/S:
Whether the Labor Arbiter has jurisdiction over the instant case.

HELD:

No.
It is true that R.A. 8042 is a special law governing overseas Filipino workers. However, a careful reading of this
special law would readily show that there is no specific provision thereunder which provides for jurisdiction over
disputes or unresolved grievances regarding the interpretation or implementation of a CBA. Section 10 of R.A.
8042 speaks, in general, of claims arising out of an employer-employee relationship or by virtue of any law or
contract involving Filipino workers for overseas deployment including claims for actual, moral, exemplary and
other forms of damages. On the other hand, Articles 217(c) and 261 of the Labor Code are very specific in
stating that voluntary arbitrators have jurisdiction over cases arising from the interpretation or implementation of
collective bargaining agreements. The instant case involves a situation where the special statute (R.A. 8042)
refers to a subject in general, which the general statute (Labor Code) treats in particular. In the present case, the
basic issue raised by Merridy Jane in her complaint filed with the NLRC is which provision of the subject CBA
applies insofar as death benefits due to the heirs of Nelson are concerned. The SC agrees with the CA in holding
that this issue clearly involves the interpretation or implementation of the said CBA. Thus, the specific or special
provisions of the Labor Code govern.

The SC also agrees with petitioner's contention that the CBA is the law or contract between the parties. Article
13.1 of the CBA between GCI and AMOSUP provides that: “The Company and the Union agree that in case of
dispute or conflict in the interpretation or application of any of the provisions of this Agreement, or enforcement of
Company policies, the same shall be settled through negotiation, conciliation or voluntary arbitration.” It is clear
that the parties, in the first place, really intended to bring to conciliation or voluntary arbitration any dispute or
conflict in the interpretation or application of the provisions of their CBA. It is settled that when the parties have
validly agreed on a procedure for resolving grievances and to submit a dispute to voluntary arbitration then that
procedure should be strictly observed.

The quoted provision of the CBA is in consonance with Rule 7, Section 7 of the present Omnibus Rules and
Regulations Implementing the Migrant Workers and Overseas Filipinos Act of 1995, as amended by R.A.
No. 10022, which states that: “For OFWs with collective bargaining agreements, the case shall be submitted for
voluntary arbitration in accordance with Articles 261 and 262 of the Labor Code.” Likewise, Section 29 of the
prevailing Standard Terms and Conditions Governing the Employment of Filipino Seafarers on Board
Ocean Going Vessels, promulgated by the POEA provides that: “In cases of claims and disputes arising from
this employment, the parties covered by a collective bargaining agreement shall submit the claim or dispute to the
original and exclusive jurisdiction of the voluntary arbitrator or panel of arbitrators.”

It is clear from the above interpretation of the DOLE, DFA and POEA that with respect to disputes involving
claims of Filipino seafarers wherein the parties are covered by a CBA, the dispute or claim should be
submitted to the jurisdiction of a voluntary arbitrator or panel of arbitrators. Only in the absence of a
collective bargaining agreement that parties may opt to submit the dispute to either the NLRC or to voluntary
arbitration.

Angeles| Bajana | Balladares | Brillantes | Briones | Cabansag | Callanta | Chua | David|

De Leon | Gomez | Lopez | Macalino | Nostratis | Padilla | Reynon | Santos | Tan |Velasco
4E / 4F - 2018-2019
Page 18 of 920
LABOR REVIEW DIGEST
Atty. Joyrich Golangco

11. PAUL V. SANTIAGO vs. CF SHARP CREW MANAGEMENT


G.R. No. 162419 Date: July 10, 2007
Ponente: TINGA, J.

SECTION 10, RA 8042

DOCTRINE/S: Even before the start of any employer-employee relationship, contemporaneous with the perfection
of the employment contract was the birth of certain rights and obligations, the breach of which may give rise to a
cause of action against the erring party. If the reverse had happened, that is the seafarer failed or refused to be
deployed as agreed upon, he would be liable for damages.

FACTS:

Paul V. Santiago was a seaman for Smith Bell Management, Inc. for 5 years. Later, he signed a new contract of
employment for 9 months with monthly salary of $515, overtime pay and other benefits. After the contract was
approved by the POEA, Paul was scheduled to leave the port of Manila for Canada on 1998 on board the MSV
Seaspread. A week before his departure, Capt. Pacifico Fernandez (CF Sharp’s Vice President) sent a facsimile
message to the captain of the ship that: “I received a phone call today from Paul’s wife asking me not to send Paul
anymore. Other unknown callers gave me feedbacks that if Paul is allowed to depart, he will jump ship in Canada
like his brother Christopher Santiago, who jumped ship in Japan last 1997. We do not want this to happen again
and have the vessel penalized.” Paul filed a complaint for illegal dismissal, damages, and attorney's fees against
respondent and its foreign principal, Cable and Wireless (Marine) Ltd.

LA RULING:
The employment contract remained valid but had not commenced since Paul was not deployed. However,
respondent violated the rules and regulations governing overseas employment when it did not deploy Paul,
causing him to suffer actual damages representing lost salary income for (9) months and fixed overtime fee, all
amounting to $7, 209.

NLRC RULING:
There is no employer-employee relationship between petitioner and respondent because under the Standard
Terms and Conditions Governing the Employment of Filipino Seafarers on Board Ocean Going Vessels (POEA
Standard Contract), the employment contract shall commence upon actual departure of the seafarer from the
airport or seaport at the point of hire and with a POEA-approved contract. In the absence of an employer-
employee relationship between the parties, the claims for illegal dismissal, actual damages, and attorneys’ fees
should be dismissed. Respondent’s decision not to deploy Paul to be a valid exercise of its management
prerogative.

CA RULING:
Petitioner is not entitled to actual damages because damages are not recoverable by a worker who was not
deployed by his agency within the period prescribed in the POEA Rules. It agreed with the NLRCs finding that
petitioner’s non-deployment was a valid exercise of respondents management prerogative, and added that since
petitioner had not departed from the Port of Manila, no employer-employee relationship between the parties
arose and any claim for damages against the so-called employer could have no leg to stand on.

PETITIONER:

Angeles| Bajana | Balladares | Brillantes | Briones | Cabansag | Callanta | Chua | David|

De Leon | Gomez | Lopez | Macalino | Nostratis | Padilla | Reynon | Santos | Tan |Velasco
4E / 4F - 2018-2019
Page 19 of 920
LABOR REVIEW DIGEST
Atty. Joyrich Golangco

3. Respondent violated the Migrant Workers Act and the POEA Rules when it failed to deploy him within (30)
calendar days without a valid reason. Since deployment is a condition to the consummation of the POEA
contract, such contract is deemed consummated. Hence, he being considered an employee for all intents and
purposes, the LA and/or the NLRC has jurisdiction to take cognizance of his claims. He should also be
awarded actual damages, consisting of the stipulated salary and fixed overtime pay.
4. He should be considered a regular employee, having worked for (5) years on the same vessel owned by the
same principal and manned by the same local agent. Respondents act of not deploying him was a scheme
designed to prevent him from attaining the status of a regular employee.
5. Respondent had no valid and sufficient cause to abandon the employment contract, as it merely relied upon
alleged phone calls from his wife and other unnamed callers in arriving at the conclusion that he would jump
ship like his brother. His wife had executed an affidavit denying having called respondent.

RESPONDENTS:
1. Labor Arbiter and/or the NLRC have no jurisdiction over the monetary claims much less award damages to
him. Paul’s employment did not commence because his deployment was withheld for a valid reason.
The controversy involves a breach of contractual obligations and as such is cognizable by civil courts.

ISSUE/S:
1. Whether the seafarer, who was prevented from leaving the port of Manila and refused deployment without
valid reason but whose POEA-approved employment contract provides that the employer-employee
relationship shall commence only upon the seafarers actual departure from the port in the point of hire, is
entitled to relief.
2. Whether the Labor Arbiter and/or the NLRC have jurisdiction despite the absence of an employer-employee
relationship between petitioner and respondent?
3. What are the reliefs that petitioner is entitled to receive?
4. Whether petitioner can be considered as a regular employee.

HELD:

1. YES. A distinction must be made between the perfection of the employment contract and the commencement
of the employer-employee relationship. The perfection of the contract in this case coincided with the date of
execution thereof, occurred when petitioner and respondent agreed on the object and the cause, as well as
the rest of the terms and conditions therein. The commencement of the employer-employee relationship
would have taken place had petitioner been actually deployed from the point of hire. Thus, even before the
start of any employer-employee relationship, contemporaneous with the perfection of the employment
contract was the birth of certain rights and obligations, the breach of which may give rise to a cause of action
against the erring party. Thus, if the reverse had happened, that is the seafarer failed or refused to be
deployed as agreed upon, he would be liable for damages.

While the POEA Standard Contract must be recognized and respected, neither the manning agent nor the
employer can simply prevent a seafarer from being deployed without a valid reason. Respondents act of
preventing petitioner from departing the port of Manila and boarding MSV Seaspread constitutes a breach
of contract, giving rise to petitioners cause of action. Respondent unilaterally and unreasonably reneged
on its obligation to deploy petitioner and must answer for the actual damages he suffered. The fact that the
POEA Rules are silent as to the payment of damages to the affected seafarer does not mean that the seafarer is
precluded from claiming the same. The sanctions provided for non-deployment do not end with the suspension or
cancellation of license or fine and the return of all documents at no cost to the worker. They do not preclude a
seafarer from instituting an action for damages against the employer or agency which has failed to deploy him.

2. YES. The POEA Rules only provide sanctions that the POEA can impose on erring agencies. It does not
Angeles| Bajana | Balladares | Brillantes | Briones | Cabansag | Callanta | Chua | David|

De Leon | Gomez | Lopez | Macalino | Nostratis | Padilla | Reynon | Santos | Tan |Velasco
4E / 4F - 2018-2019
Page 20 of 920
LABOR REVIEW DIGEST
Atty. Joyrich Golangco

provide for damages and money claims recoverable by aggrieved employees because it is not the POEA, but
the NLRC, which has jurisdiction over such matters. Despite the absence of an employer-employee
relationship between petitioner and respondent, the NLRC has jurisdiction over the complaint. The jurisdiction
of labor arbiters is not limited to claims arising from employer-employee relationships. Section 10 of R.A. No.
8042 provides that: “The LA’s of the NLRC shall have the original and exclusive jurisdiction to hear and
decide…the claims arising out of an employer-employee relationship or by virtue of any law or contract
involving Filipino workers for overseas deployment including claims for actual, moral, exemplary and
other forms of damages.” Since the present petition involves the employment contract entered into by
petitioner for overseas employment, his claims are cognizable by the labor arbiters of the NLRC.

3. PETITIONER IS ENTITLED TO ACTUAL DAMAGES AND ATTORNEYS FEES BUT NOT OVERTIME PAY
AND MORAL DAMAGES.
(a) ACTUAL DAMAGES - Article 2199 of the Civil Code provides that one is entitled to an adequate
compensation only for such pecuniary loss suffered by him as he has duly proved. Respondent is liable to pay
petitioner actual damages in the form of the loss of (9) months worth of salary as provided in the contract.
(b) OVERTIME PAY - Petitioner is not, however, entitled to overtime pay. Even though petitioner was
prevented without valid reason from rendering regular much less overtime service, the fact remains that there is
no certainty that petitioner will perform overtime work had he been allowed to board the vessel. In Stolt-Nielsen
Marine Services (Phils.), Inc. v. National Labor Relations Commission, “the rendition of overtime work and
the submission of sufficient proof that said work was actually performed are conditions to be satisfied before a
seaman could be entitled to overtime pay which should be computed on the basis of 30% of the basic monthly
salary. Realistically speaking, a seaman, by the very nature of his job, stays on board a ship or vessel beyond the
regular eight-hour work schedule. For the employer to give him overtime pay for the extra hours when he might be
sleeping or attending to his personal chores or even just lulling away his time would be extremely unfair and
unreasonable.”
(c) ATTORNEYS FEES - Petitioner is entitled to attorneys fees in the concept of damages and expenses of
litigation. Respondents basis for not deploying petitioner is unfounded and unreasonable. Time and again, this
Court has upheld management prerogatives so long as they are exercised in good faith for the advancement of
the employers interest and not for the purpose of defeating or circumventing the rights of the employees under
special laws or under valid agreements.
(d) MORAL DAMAGES - Moral damages cannot be awarded in this case. While respondents failure to deploy
petitioner seems baseless and unreasonable, such action is not tainted with bad faith, or done deliberately to
defeat petitioners rights, as to justify the award of moral damages. At most, respondent was being overzealous in
protecting its interest when it became too hasty in making its conclusion that petitioner will jump ship like his
brother.

4. NO
Respondents failure to deploy petitioner as an act designed to prevent the latter from attaining the status of a
regular employee. Even if petitioner was able to depart the port of Manila, he still cannot be considered a regular
employee, regardless of his previous contracts of employment with respondent. In Millares v. National Labor
Relations Commission, “seafarers are considered contractual employees and cannot be considered as regular
employees under the Labor Code. Their employment is governed by the contracts they sign every time they are
rehired and their employment is terminated when the contract expires. The exigencies of their work necessitates
that they be employed on a contractual basis.”

12. SAMEER OVERSEAS PLACEMENT AGENCY, INC. vs. JOY C. CABILES


G.R. No. 170139 Date: August 5, 2014

Angeles| Bajana | Balladares | Brillantes | Briones | Cabansag | Callanta | Chua | David|

De Leon | Gomez | Lopez | Macalino | Nostratis | Padilla | Reynon | Santos | Tan |Velasco
4E / 4F - 2018-2019
Page 21 of 920
LABOR REVIEW DIGEST
Atty. Joyrich Golangco

Ponente: LEONEN, J.

SECTION 10, RA 8042

DOCTRINE: The re-enactment of the last paragraph of Sec. 10 of R.A. No. 8042 did not erase its
unconstitutionality and a central bank circular on interest rates cannot repeal a positive provision R.A. No. 8042,
as regards interest rates on certain pecuniary awards granted to illegally dismissed OFWs.

FACTS:

After paying a P70K placement fee, Joy C. Cabiles was deployed by Sameer Overseas Placement Agency, Inc.
(SOPA) to work as a quality control for Taiwan Wacoal, Co. Ltd. under a 1-year employment contract with a
monthly salary of $15,360. SOPA claims that a certain Mr. Huwang from Wacoal informed Joy, without prior
notice, that she was terminated and that "she should immediately report to their office to get her salary and
passport." Joy claims that she was told that from June 26-July 14, 1997, she only earned a total of NT$9K
deducted by NT$3K plane ticket. Hence, Joy filed a complaint against SOPA and Wacoal with the NLRC claiming
that she was illegally dismissed, and asking the return of her placement fee, the withheld amount for repatriation
costs, payment of her salary for 23 months and moral and exemplary damages. SOPA alleged that respondent's
termination was due to her inefficiency, negligence in her duties, and her failure to comply with the work
requirements of her foreign employer. It added that it did not ask for a placement fee.

LA RULING:
Dismissed Joy’s complaint as it was based on mere allegations. There was no excess payment of placement fees,
based on the official receipt presented by petitioner.

NLRC RULING:
Declared that Joy was illegally dismissed. It reiterated the doctrine that the burden of proof to show that the
dismissal was based on a just or valid cause belongs to the employer. SAPO failed to prove that there were just
causes for termination, and the procedural due process was not observed in terminating respondent. NLRC did
not rule on the issue of reimbursement of placement fees for lack of jurisdiction but awarded only (3) months
worth of salary, the reimbursement of the amount withheld from her, and attorney’s fees.

CA RULING:
Affirmed the NLRC as to the finding of Joy’s illegal dismissal, entitlement to the equivalent of 3 months worth of
salary, reimbursement of withheld repatriation expense, and attorney’s fees.

PETITIONER:
1. There was just cause for termination because there was a finding of Wacoal that respondent was inefficient in
her work.
NOTE: (4) issue is an important issue in this case. Though the parties never raised this issue, the SC ordered
them and the OSG to comment. Both the parties argued that the clause was unconstitutional, the OSG contends
that the clause was valid and constitutional.

ISSUE/S:
1. Whether there was a just cause for respondent’s termination.
2. Whether petitioner failed to comply with the due process requirements.
3. What are the claims that respondent is entitled to receive?
4. Whether the clause "or for three (3) months for every year of the unexpired term, whichever is less" that was
declared by SC as unconstitutional in Serrano v. Gallant Maritime Services, Inc. but reinstated in RA No.
8042 by the amendment of RA No. 10022 in 2010 constitutional?
Angeles| Bajana | Balladares | Brillantes | Briones | Cabansag | Callanta | Chua | David|

De Leon | Gomez | Lopez | Macalino | Nostratis | Padilla | Reynon | Santos | Tan |Velasco
4E / 4F - 2018-2019
Page 22 of 920
LABOR REVIEW DIGEST
Atty. Joyrich Golangco

5. What are the applicable interest rates?


What are the liabilities of the local agency and the foreign employer?

HELD:
1. NO
Sameer Overseas Placement Agency failed to show that there was just cause for causing Joy’s dismissal and
Wacoal also failed to accord her due process of law. Indeed, management prerogative is recognized in law and in
our jurisprudence. This prerogative, however, should not be abused. It is "tempered with the employee’s right to
security of tenure." Workers are entitled to substantive and procedural due process before termination. They may
not be removed from employment without a valid or just cause as determined by law and without going through
the proper procedure.

Employees are not stripped of their security of tenure when they move to work in a different jurisdiction. With
respect to the rights of overseas Filipino workers, we follow the principle of lex loci contractus. In Triple Eight
Integrated Services, Inc. v. NLRC, SC noted that: “the rule that lex loci contractus (the law of the place where
the contract is made) governs in this jurisdiction. There is no question that the contract of employment in this case
was perfected here in the Philippines. Therefore, the Labor Code, its implementing rules and regulations, and
other laws affecting labor apply in this case. Furthermore, settled is the rule that the courts of the forum will not
enforce any foreign claim obnoxious to the forum’s public policy. Herein the Philippines, employment agreements
are more than contractual in nature. The public policy in Article XIII, Section 3 of the Constitution which
guarantees the special protection of workers should be borne in mind because to allow foreign employers to
determine for and by themselves whether an overseas contract worker may be dismissed would encourage illegal
or arbitrary pretermination of employment contracts.” By our laws, overseas Filipino workers (OFWs) may only be
terminated for a just or authorized cause under Article 282 of the Labor Code and after compliance with
procedural due process requirements.

4. YES
Respondent’s dismissal less than one year from hiring and her repatriation on the same day show that the
employers did not comply with the due process requirement. A valid dismissal requires both a valid cause and
adherence to the valid procedure of dismissal. The employer is required to give the charged employee at least two
written notices before termination. Aside from the notice requirement, the employee must also be given "an
opportunity to be heard."

3. RESPONDENT IS ENTITLED TO HER SALARY FOR THE UNEXPIRED PORTION OF THE


EMPLOYMENT CONTRACT, ATTORNEY’S FEES AND REIMBURSEMENT OF AMOUNTS WITHHELD
FROM HER SALARY.
Section 10 of RA No. 8042 states that: “overseas workers who were terminated without just, valid, or authorized
cause "shall be entitled to the full reimbursement of his placement fee with interest of twelve (12%) per annum,
plus his salaries for the unexpired portion of his employment contract or for three (3) months for every year of the
unexpired term, whichever is less." The liability of the principal/employer and the recruitment/placement agency
for any and all claims under this section shall be joint and several. Such provision shall be incorporated in the
contract for overseas employment and shall be a condition precedent for its approval. The performance bond to
be filed by the recruitment/placement agency, as provided by law, shall be answerable for all money claims or
damages that may be awarded to the workers. If the recruitment/placement agency is a juridical being, the
corporate officers and directors and partners as the case may be, shall themselves be jointly and solidarily liable
with the corporation or partnership for the aforesaid claims and damages. Such liabilities shall continue during the
entire period or duration of the employment contract and shall not be affected by any substitution, amendment or
modification made locally or in a foreign country of the said contract. Section 15 of RA No. 8042 states that:
"repatriation of the worker and the transport of his [or her] personal belongings shall be the primary responsibility
of the agency which recruited or deployed the worker overseas.” The Labor Code also entitles the employee to
Angeles| Bajana | Balladares | Brillantes | Briones | Cabansag | Callanta | Chua | David|

De Leon | Gomez | Lopez | Macalino | Nostratis | Padilla | Reynon | Santos | Tan |Velasco
4E / 4F - 2018-2019
Page 23 of 920
LABOR REVIEW DIGEST
Atty. Joyrich Golangco

10% of the amount of withheld wages as attorney’s fees when the withholding is unlawful.

In Serrano v. Gallant Maritime Services, Inc., SC ruled that: “the clause "or for three (3) months for every year
of the unexpired term, whichever is less" is unconstitutional for violating the equal protection clause and
substantive due process.” The said clause was reinstated in RA No. 8042 upon promulgation of RA No.
10022 in 2010. However, the reinstatement was not yet in effect at the time of respondent’s termination
from work in 1997. Hence, Republic Act No. 8042 before it was amended by Republic Act No. 10022
governs this case.

4. NO, JOY IS ENTITLED TO HER SALARY FOR THE UNEXPIRED PORTION OF HER CONTRACT, IN
ACCORDANCE WITH SECTION 10 OF REPUBLIC ACT NO. 8042.
The reinstated clause as provided in Republic Act. No. 10022 violates the constitutional rights to equal protection
and due process. Equal protection of the law is a guarantee that persons under like circumstances and falling
within the same class are treated alike, in terms of "privileges conferred and liabilities enforced." A law that does
not violate the equal protection clause prescribes a reasonable classification.
A reasonable classification -
(1) must rest on substantial distinctions;
(2) must be germane to the purposes of the law;
(3) must not be limited to existing conditions only; and
(4) must apply equally to all members of the same class.

The reinstated clause does not satisfy the requirement of reasonable classification. The clause distinguished:

a. Between fixed-period overseas workers and fixed-period local workers.


b. Between overseas workers with employment contracts of less than 1 year and overseas workers with
employment contracts of at least 1 year.
i. Distinction within the class with at least 1-year employment contracts – between those with at least a year
or more left in their contracts and those with less than a year left in their contracts when they were
illegally dismissed.
ii.
Overseas workers with employment contracts of:

less than 1 year at least 1 year

(a) those with at least a year or more left in their


contracts when they were illegally dismissed – the
money claims granted a cap equivalent to 3 months of
their salary for the unexpired portions of their contract
(COVERED BY REINSTATED CLAUSE)

(b) those with less than a year left in their contracts


when they were illegally dismissed - the money claims
granted the amount equivalent to the entire unexpired
portion of their employment contracts.

Angeles| Bajana | Balladares | Brillantes | Briones | Cabansag | Callanta | Chua | David|

De Leon | Gomez | Lopez | Macalino | Nostratis | Padilla | Reynon | Santos | Tan |Velasco
4E / 4F - 2018-2019
Page 24 of 920
LABOR REVIEW DIGEST
Atty. Joyrich Golangco

iii.
iii.
SC found that the subject clause creates a sub-layer of discrimination among OFWs whose contract periods are
for at least 1 year. Overseas workers regardless of their classifications are entitled to security of tenure, at
least for the period agreed upon in their contracts. This means that they cannot be dismissed before the
end of their contract terms without due process. If they were illegally dismissed, the workers’ right to
security of tenure is violated. The rights violated when a fixed-period local worker is illegally terminated are
neither greater than nor less than the rights violated when a fixed-period overseas worker is illegally terminated. It
is state policy to protect the rights of workers without qualification as to the place of employment. The same with is
true for the distinctions between overseas workers with an employment contract of less than one year and at least
one year.

The classifications are not relevant to the purpose of the law, which is to "establish a higher standard of protection
and promotion of the welfare of migrant workers, their families and overseas Filipinos in distress, and for other
purposes." Putting a cap on the money claims of certain overseas workers does not increase the standard of
protection afforded to them. On the other hand, foreign employers are more incentivizedby the reinstated clause
to enter into contracts of at least a year because it gives them more flexibility to violate our overseas workers’
rights. The reinstated clause violates due process rights. It is arbitrary as it deprives overseas workers of their
monetary claims without any discernable valid purpose.

5.
(a) REIMBURSEMENT OF PLACEMENT FEES – The Bangko Sentral ng Pilipinas Circular No. 799 of June 21,
2013 revised the interest rate for loan or forbearance from 12% to 6% in the absence of stipulation. However, it is
not applicable when there is a law that states otherwise.
Section 10 of Republic Act No. 8042 provides that unlawfully terminated overseas workers are entitled to the
reimbursement of his or her placement fee with an interest of 12% per annum. Such has the effect of removing
awards for reimbursement of placement fees from Circular No. 799’s coverage.
(b) AWARDS OF SALARY FOR THE UNEXPIRED PORTION OF THE EMPLOYMENT CONTRACT UNDER
REPUBLIC ACT NO. 8042 - These awards are covered by Circular No. 799 because the law does not provide for
a specific interest rate that should apply.

6. BOTH ARE JOINTLY AND SEVERALLY LIABLE FOR MONEY CLAIMS.


iv. Section 10 of the Migrant Workers and Overseas Filipinos Act of 1995 provides that: “the foreign
employer and the local employment agency are jointly and severally liable for money claims including
claims arising out of an employer-employee relationship and/or damages.” This section also provides that
the performance bond filed by the local agency shall be answerable for such money claims or damages if
they were awarded to the employee. Hence, in the case of overseas employment, either the local agency or
the foreign employer may be sued for all claims arising from the foreign employer’s labor law violations. The
Act ensures that overseas workers have recourse in law despite the circumstances of their employment.
However, it must be emphasized that the local agency that is held to answer for the overseas worker’s
money claims is not left without remedy. The law does not preclude it from going after the foreign employer
for reimbursement of whatever payment it has made to the employee to answer for the money claims
against the foreign employer.

13. INDUSTRIAL PERSONNEL & MANAGEMENT SERVICES, INC. (IPAMS), SNC LAVALIN ENGINEERS &
CONTRACTORS, INC. AND ANGELITO C. HERNANDEZ v. JOSE G. DE VERA AND ALBERTO B. ARRIOLA
G.R. No. 205703 Date: March 07, 2016

Angeles| Bajana | Balladares | Brillantes | Briones | Cabansag | Callanta | Chua | David|

De Leon | Gomez | Lopez | Macalino | Nostratis | Padilla | Reynon | Santos | Tan |Velasco
4E / 4F - 2018-2019
Page 25 of 920
LABOR REVIEW DIGEST
Atty. Joyrich Golangco

Ponente: MENDOZA, J.

Section 3, Art. XIII, 1987 Philippine Constitution

DOCTRINE: The general rule is that Philippine laws apply even to overseas employment contracts. As an
exception, the parties may agree that a foreign law shall govern the employment contract.

FACTS:
Petitioner Industrial Personnel & Management Services, Inc. (IPAMS) is a local placement agency duly organized
and existing under Philippine laws, with petitioner Angelito C. Hernandez as its president and managing director.
Petitioner SNC Lavalin Engineers & Contractors, Inc. (SNC-Lavalin) is the principal of IPAMS, a Canadian
company with business interests in several countries.

Respondent Alberto Arriola (Arriola) is a licensed general surgeon in the Philippines. Arriola was offered by SNC-
Lavalin, through its letter, dated May 1, 2008, the position of Safety Officer in its Ambatovy Project site in
Madagascar. It was for a period of nineteen (19) months starting from June 9, 2008 to December 31, 2009.

Arriola was then hired by SNC-Lavalin, through its local manning agency, IPAMS, and his overseas employment
contract was processed with the Philippine Overseas Employment Agency (POEA). According to Arriola, he
signed the contract of employment in the Philippines.

Arriola received a notice of pre-termination of employment, dated September 9, 2009, from SNC-Lavalin. It stated
that his employment would be pre-terminated effective September 11, 2009 due to diminishing workload in the
area of his expertise and the unavailability of alternative assignments. Aggrieved, Arriola filed a complaint against
the petitioners for illegal dismissal and non-payment of overtime pay, vacation leave and sick leave pay before the
Labor Arbiter (LA). He claimed that SNC-Lavalin still owed him unpaid salaries equivalent to the three-month
unexpired portion of his contract. Arriola also insisted that the petitioners must prove the applicability of
Canadian law before the same could be applied to his employment contract.

Petitioners claimed that SNC-Lavalin was greatly affected by the global financial crises during the latter part of
2008. The economy of Madagascar, where SNC-Lavalin had business sites, also slowed down.

In the present case, as all of Arriola's employment documents were processed in Canada, not to mention that
SNC-Lavalin's office was in Ontario, the principle of lex loci celebrationis was applicable. Thus, the petitioners
insisted that Canadian laws governed the contract.

The said foreign law did not require any ground for early termination of employment, and the only requirement
was the written notice of termination. Even assuming that Philippine laws should apply, Arriola would still be
validly dismissed because domestic law recognized retrenchment and redundancy as legal grounds for
termination.

LA RULING:
The LA ruled that the rights and obligations among and between the OFW, the local recruiter/agent, and the
foreign employer/principal were governed by the employment contract pursuant to the EDI-Staffbuilders case. LA
opined that there was no other conclusion but to uphold the validity of Arriola's dismissal based on Canadian law.

NLRC RULING:
NLRC reversed the LA decision and ruled that Arriola was illegally dismissed by the petitioners. Citing PNB v.
Cabansag, the NLRC stated that whether employed locally or overseas, all Filipino workers enjoyed the protective
mantle of Philippine labor and social legislation, contract stipulations to the contrary notwithstanding.
Angeles| Bajana | Balladares | Brillantes | Briones | Cabansag | Callanta | Chua | David|

De Leon | Gomez | Lopez | Macalino | Nostratis | Padilla | Reynon | Santos | Tan |Velasco
4E / 4F - 2018-2019
Page 26 of 920
LABOR REVIEW DIGEST
Atty. Joyrich Golangco

CA RULING:
CA affirmed that Arriola was illegally dismissed by the petitioners. The appellate court wrote that the ESA allowed
an employer to disregard the required notice of termination by simply giving the employee a severance pay. The
ESA could not be made to apply in this case for being contrary to our Constitution, specifically on the right of due
process. Thus, the CA opined that our labor laws should find application.

ISSUE/S:
Whether or not the foreign law should govern the present overseas employment contract.

HELD:
No.
The general rule is that Philippine laws apply even to overseas employment contracts. This rule is rooted
in the constitutional provision of Section 3, Article XIII that the State shall afford full protection to labor,
whether local or overseas. Hence, even if the OFW has his employment abroad, it does not strip him of his
rights to security of tenure, humane conditions of work and a living wage under our Constitution.
As an exception, the parties may agree that a foreign law shall govern the employment contract. A
synthesis of the existing laws and jurisprudence reveals that this exception is subject to the following
requisites:
 That it is expressly stipulated in the overseas employment contract that a specific foreign law shall
govern;
 That the foreign law invoked must be proven before the courts pursuant to the Philippine rules on
evidence;
 That the foreign law stipulated in the overseas employment contract must not be contrary to law,
morals, good customs, public order, or public policy of the Philippines; and
 That the overseas employment contract must be processed through the POEA.
The Court is of the view that these four (4) requisites must be complied with before the employer could
invoke the applicability of a foreign law to an overseas employment contract.

If the first requisite is absent, or that no foreign law was expressly stipulated in the employment contract which
was executed in the Philippines, then the domestic labor laws shall apply in accordance with the principle of lex
loci contractus.

If the second requisite is lacking, or that the foreign law was not proven pursuant to Sections 24 and 25 of Rule
132 of the Revised Rules of Court, then the international law doctrine of processual presumption operates. The
said doctrine declares that "[w]here a foreign law is not pleaded or, even if pleaded, is not proved, the
presumption is that foreign law is the same as ours."

If the third requisite is not met, or that the foreign law stipulated is contrary to law, morals, good customs, public
order or public policy, then Philippine laws govern.

Finally, if the fourth requisite is missing, or that the overseas employment contract was not processed through the
POEA, then Article 18 of the Labor Code is violated. Article 18 provides that no employer may hire a Filipino
worker for overseas employment except through the boards and entities authorized by the Secretary of Labor.

A judicious scrutiny of the records of the case demonstrates that the petitioners were able to observe the second
requisite, or that the foreign law must be proven before the court pursuant to the Philippine rules on evidence. The
petitioners were able to present the ESA, duly authenticated by the Canadian authorities and certified by the

Angeles| Bajana | Balladares | Brillantes | Briones | Cabansag | Callanta | Chua | David|

De Leon | Gomez | Lopez | Macalino | Nostratis | Padilla | Reynon | Santos | Tan |Velasco
4E / 4F - 2018-2019
Page 27 of 920
LABOR REVIEW DIGEST
Atty. Joyrich Golangco

Philippine Embassy, before the LA. The fourth requisite was also followed because Arriola's employment contract
was processed through the POEA.

Unfortunately for the petitioners, those were the only requisites that they complied with. As correctly held by the
CA, even though an authenticated copy of the ESA was submitted, it did not mean that said foreign law could be
automatically applied to this case. The petitioners miserably failed to adhere to the two other requisites, which
shall be discussed in seratim.

The petitioners failed to comply with the first requisite because no foreign law was expressly stipulated in the
overseas employment contract with Arriola. They failed to show on the face of the contract that a foreign law was
agreed upon by the parties. Rather, they simply asserted that the terms and conditions of Arriola's employment
were embodied in the Expatriate Policy, Ambatovy Project - Site, Long Term. A better rule would be to obligate
the foreign employer to expressly declare at the onset of the labor contract that a foreign law shall govern it. In
that manner, the OFW would be informed of the applicable law before signing the contract.

A perusal of the ESA will show that some of its provisions are contrary to the Constitution and the labor laws of
the Philippines.

First, the ESA does not require any ground for the early termination of employment.
At its own pleasure, the foreign employer is endowed with the absolute power to end the employment of an
employee even on the most whimsical grounds.

Second, the ESA allows the employer to dispense with the prior notice of termination to an employee. The
employee under the ESA could be immediately dismissed without giving him the opportunity to explain and defend
himself.

The provisions of the ESA are patently inconsistent with the right to security of tenure. Not only do these
provisions collide with the right to security of tenure, but they also deprive the employee of his constitutional right
to due process by denying him of any notice of termination and the opportunity to be heard.

In fine, as the petitioners failed to meet all the four (4) requisites on the applicability of a foreign law, then
the Philippine labor laws must govern the overseas employment contract of Arriola.

14. SUNACE INTERNATIONAL MANAGEMENT SERVICES, INC. v.


NATIONAL LABOR RELATIONS COMMISSION, Second Division; HON. ERNESTO S. DINOPOL, in his
capacity as Labor Arbiter, NLRC; NCR, Arbitration Branch, Quezon City and DIVINA A. MONTEHERMOZO,
G.R. No. 161757 Date: January 25, 2006
Ponente: CARPIO MORALES, J.

ARTICLE 1924, NCC

DOCTRINE: The theory of imputed knowledge ascribes the knowledge of the agent to the principal not the other
way around. Also, the agency between a foreign principal and its local recruitment agent is revoked if the foreign
principal directly manages the business (hiring of employee) entrusted to the local recruitment agent, dealing
directly with third persons.

FACTS:

Angeles| Bajana | Balladares | Brillantes | Briones | Cabansag | Callanta | Chua | David|

De Leon | Gomez | Lopez | Macalino | Nostratis | Padilla | Reynon | Santos | Tan |Velasco
4E / 4F - 2018-2019
Page 28 of 920
LABOR REVIEW DIGEST
Atty. Joyrich Golangco

Petitioner, Sunace International Management Services (Sunace), a corporation duly organized and existing under
the laws of the Philippines, deployed to Taiwan Divina A. Montehermozo (Divina) as a domestic helper under a
12-month contract effective February 1, 1997.[1] The deployment was with the assistance of a Taiwanese broker,
Edmund Wang, President of Jet Crown International Co., Ltd.

After her 12-month contract expired on February 1, 1998, Divina continued working for her Taiwanese employer,
Hang Rui Xiong, for two more years. Shortly after her return or on February 14, 2000, Divina filed a complaint
before the National Labor Relations Commission (NLRC) against Sunace, one Adelaide Perez, the Taiwanese
broker, and the employer-foreign principal alleging that she was jailed for three months and that she was
underpaid.

Reacting to Divinas Position Paper, Sunace filed on April 25, 2000 an . . . answer to complainants position paper
alleging that Divinas 2-year extension of her contract was without its knowledge and consent, hence, it had no
liability attaching to any claim arising therefrom, and Divina in fact executed a Waiver/Quitclaim and Release of
Responsibility and an Affidavit of Desistance.

LA RULING:
The Labor Arbiter, rejected Sunaces claim that the extension of Divinas contract for two more years was without
its knowledge and consent because as Annex shows, Sunace and Edmund Wang have not stopped
communicating with each other. What Sunace should have done was to write to POEA about the extension and its
objection thereto, copy furnished the complainant herself, her foreign employer, Hang Rui Xiong and the
Taiwanese broker, Edmund Wang.

And because it did not, it is presumed to have consented to the extension and should be liable for anything that
resulted thereform (sic).

NLRC RULING:
On appeal of Sunace, the NLRC, by Resolution of April 30, 2002, affirmed the Labor Arbiters decision.

CA RULING:
Via petition for certiorari, Sunace elevated the case to the Court of Appeals which dismissed it outright. It is
undisputed that petitioner was continually communicating with private respondents foreign employer (sic). As
agent of the foreign principal, petitioner cannot profess ignorance of such extension as obviously, the act of the
principal extending complainant (sic) employment contract necessarily bound it. Grave abuse of discretion is not
present in the case at bar.

ISSUE/S:
Whether or not the act of the foreigner-principal in renewing the contract of Divina is attributable to Sunace.

HELD:
No.
1. The finding of the Court of Appeals solely on the basis of the telefax message that Sunace continually
communicated with the foreign principal (sic) and therefore was aware of and had consented to the execution of
the extension of the contract is misplaced. The message does not provide evidence that Sunace was privy to the
new contract executed after the expiration on February 1, 1998 of the original contract. That Sunace and the
Taiwanese broker communicated regarding Divinas allegedly withheld savings does not necessarily mean that
Sunace ratified the extension of the contract.

Angeles| Bajana | Balladares | Brillantes | Briones | Cabansag | Callanta | Chua | David|

De Leon | Gomez | Lopez | Macalino | Nostratis | Padilla | Reynon | Santos | Tan |Velasco
4E / 4F - 2018-2019
Page 29 of 920
LABOR REVIEW DIGEST
Atty. Joyrich Golangco

2. The theory of imputed knowledge ascribes the knowledge of the agent, Sunace, to the principal,
employer Xiong, not the other way around. The knowledge of the principal-foreign employer cannot,
therefore, be imputed to its agent Sunace.

There being no substantial proof that Sunace knew of and consented to be bound under the 2-year employment
contract extension, it cannot be said to be privy thereto. As such, it and its owner cannot be held solidarily liable
for any of Divinas claims arising from the 2-year employment extension. As the New Civil Code provides,

“Contracts take effect only between the parties, their assigns, and heirs, except in case where the rights and
obligations arising from the contract are not transmissible by their nature, or by stipulation or by provision of law.”

3. Furthermore, as Sunace correctly points out, there was an implied revocation of its agency relationship
with its foreign principal when, after the termination of the original employment contract, the foreign
principal directly negotiated with Divina and entered into a new and separate employment contract in
Taiwan. Article 1924 of the New Civil Code reading

“The agency is revoked if the principal directly manages the business entrusted to the agent, dealing directly with
third persons.” thus applies.

In light of the foregoing discussions, consideration of the validity of the Waiver and Affidavit of Desistance which
Divina executed in favor of Sunace is rendered unnecessary.

15. MAERSK-FILIPINAS CREWING, INC., A.P. MOLLER SINGAPORE PTE. LIMITED, AND JESUS
AGBAYANI v. TORIBIO C. AVESTRUZ
G.R. No. 207010 Date: February 18, 2015
Ponente: PERLAS-BERNABE, J.

SECTION 17, POEA-SEC

DOCTRINE/S: It is only in the exceptional case of clear and existing danger to the safety of the crew or vessel
that the required notices are dispensed with; but just the same, a complete report should be sent to the manning
agency, supported by substantial evidence of the findings.

FACTS:
On April 28, 2011, petitioner Maersk-Filipinas Crewing, Inc. (Maersk), on behalf of its foreign principal, petitioner
A.P. Moller Singapore Pte. Ltd. (A.P. Moller), hired Avestruz as Chief Cook on board the vessel M/V Nedlloyd
Drake for a period of six (6) months.

Angeles| Bajana | Balladares | Brillantes | Briones | Cabansag | Callanta | Chua | David|

De Leon | Gomez | Lopez | Macalino | Nostratis | Padilla | Reynon | Santos | Tan |Velasco
4E / 4F - 2018-2019
Page 30 of 920
LABOR REVIEW DIGEST
Atty. Joyrich Golangco

On June 22, 2011, in the course of the weekly inspection of the vessel’s galley, Captain Charles C. Woodward
(Captain Woodward) noticed that the cover of the garbage bin in the kitchen near the washing area was oily.

As part of Avestruz’s job was to ensure the cleanliness of the galley, Captain Woodward called Avestruz and
asked him to stand near the garbage bin where the former took the latter’s right hand and swiped it on the oily
cover of the garbage bin, telling Avestruz to feel it.

Shocked, Avestruz remarked, “Sir if you are looking for [dirt], you can find it[;] the ship is big. Tell us if you want to
clean and we will clean it.” Captain Woodward replied by shoving Avestruz’s chest, to which the latter complained
and said, “Don’t touch me,” causing an argument to ensue between them.

Later that afternoon, Captain Woodward summoned and required Avestruz to state in writing what transpired in
the galley that morning. Avestruz complied and submitted his written statement10 on that same day. Captain
Woodward likewise asked Messman Jomilyn P. Kong (Kong) to submit his own written statement regarding the
incident, to which the latter immediately complied.

On the very same day, Captain Woodward informed Avestruz that he would be dismissed from service and be
disembarked in India. Subsequently, he filed a complaint13 for illegal dismissal, payment for the unexpired portion
of his contract, damages, and attorney’s fees.

He alleged that no investigation or hearing was conducted nor was he given the chance to defend himself before
he was dismissed, and that Captain Woodward failed to observe the provisions under Section 17 of the Philippine
Overseas Employment Administration (POEA) Standard Employment Contract (POEA-SEC) on disciplinary
procedures. Also, he averred that he was not given any notice stating the ground for his dismissal.
In their defense, Maersk, A.P. Moller, and Agbayani (petitioners) claimed that during his stint on the vessel,
Avestruz failed to attend to his tasks, specifically to maintain the cleanliness of the galley, which prompted Captain
Woodward to issue weekly reminders. Unfortunately, despite the reminders, Avestruz still failed to perform his
duties properly. On June 22, 2011, when again asked to comply with the aforesaid duty, Avestruz became angry
and snapped, retorting that he did not have time to do all the tasks required of him.

Petitioners maintained that Avestruz was dismissed for a just and valid cause and is, therefore, not entitled to
recover his salary for the unexpired portion of his contract. They likewise claimed that they were justified in
deducting his airfare from his salary, and that the latter was not entitled to moral and exemplary damages and
attorney’s fees. Hence, they prayed that the complaint be dismissed for lack of merit.

LA RULING:
The LA found that he failed to perform his duty of maintaining cleanliness in the galley, and that he also
repeatedly failed to obey the directives of his superior, which was tantamount to insubordination.

NLRC RULING:
NLRC sustained the validity of Avestruz’s dismissal but found that petitioners failed to observe the procedures laid
down in Section 17 of the POEA-SEC, which states:

SECTION 17. DISCIPLINARY PROCEDURES.

The Master shall comply with the following disciplinary procedures against an erring seafarer:
The Master shall furnish the seafarer with a written notice containing the following:

Grounds for the charges as listed in Section 33 of this Contract or analogous act constituting the same.

Angeles| Bajana | Balladares | Brillantes | Briones | Cabansag | Callanta | Chua | David|

De Leon | Gomez | Lopez | Macalino | Nostratis | Padilla | Reynon | Santos | Tan |Velasco
4E / 4F - 2018-2019
Page 31 of 920
LABOR REVIEW DIGEST
Atty. Joyrich Golangco

Date, time and place for a formal investigation of the charges against the seafarer concerned.

The Master or his authorized representative shall conduct the investigation or hearing, giving the seafarer the
opportunity to explain or defend himself against the charges. These procedures must be duly documented and
entered into the ship’s logbook.

If after the investigation or hearing, the Master is convinced that imposition of a penalty is justified, the Master
shall issue a written notice of penalty and the reasons for it to the seafarer, with copies furnished to the Philippine
agent.

Dismissal for just cause may be effected by the Master without furnishing the seafarer with a notice of dismissal if
there is a clear and existing danger to the safety of the crew or the vessel. The Master shall send a complete
report to the manning agency substantiated by witnesses, testimonies and any other documents in support
thereof. (Emphases supplied)

As the records are bereft of evidence showing compliance with the foregoing rules, the NLRC held petitioners
jointly and severally liable to pay Avestruz the amount of P30,000.00 by way of nominal damages.

CA RULING:
CA reversed and set aside the rulings of the NLRC and instead, found Avestruz to have been illegally dismissed.
It found that petitioners, as employers, were unable to discharge the burden of proof required of them to establish
that Avestruz was guilty of insubordination, which necessitates the occurrence of two (2) conditions as a just
cause for dismissal: (1) the employee’s assailed conduct must have been willful, that is, characterized by a
wrongful and perverse attitude; and (2) the order violated must have been reasonable, lawful, made known to the
employee, and must pertain to the duties which he had been engaged to discharge.

ISSUE/S:
Whether or not Avestruz was illegally dismissed.

HELD:
Yes. The Court finds that there was no just or valid cause for his dismissal; hence, he was illegally dismissed.
1. Insubordination, as a just cause for the dismissal of an employee, necessitates the concurrence of at least two
requisites: (1) the employee’s assailed conduct must have been willful, that is, characterized by a wrongful and
perverse attitude; and (2) the order violated must have been reasonable, lawful, made known to the employee,
and must pertain to the duties which he had been engaged to discharge.

In this case, the contents of Captain Woodward’s e-mails do not establish that Avestruz’s conduct had been
willful, or characterized by a wrongful and perverse attitude. The Court concurs with the CA’s observation that
Avestruz’s statement regarding the incident in the galley deserves more credence, being corroborated by Kong, a
messman who witnessed the same.

Conversely, apart from Captain Woodward’s e-mails, no other evidence was presented by the petitioners to
support their claims. While rules of evidence are not strictly observed in proceedings before administrative bodies,
petitioners should have offered additional proof to corroborate the statements described therein.

2. At the least, they could have offered in evidence entries in the ship’s official logbook showing the infractions or
acts of insubordination purportedly committed by Avestruz, the ship’s logbook being the official repository of the
day-to-day transactions and occurrences on board the vessel. Having failed to do so, their position that Avestruz
was lawfully dismissed cannot be sustained.

Angeles| Bajana | Balladares | Brillantes | Briones | Cabansag | Callanta | Chua | David|

De Leon | Gomez | Lopez | Macalino | Nostratis | Padilla | Reynon | Santos | Tan |Velasco
4E / 4F - 2018-2019
Page 32 of 920
LABOR REVIEW DIGEST
Atty. Joyrich Golangco

3. Similarly, the Court affirms the finding of the CA that Avestruz was not accorded procedural due process, there
being no compliance with the provisions of Section 17 of the POEA-SEC as above-cited, which requires the two-
notice rule.

In this case, there is dearth of evidence to show that Avestruz had been given a written notice of the charge
against him, or that he was given the opportunity to explain or defend himself. The statement given by Captain
Woodward requiring him to explain in writing the events that transpired at the galley in the morning of June 22,
2011 hardly qualifies as a written notice of the charge against him, nor was it an opportunity for Avestruz to
explain or defend himself. While Captain Woodward claimed in his e-mail that he conducted a disciplinary hearing
informing Avestruz of his inefficiency, no evidence was presented to support the same.

Neither was Avestruz given a written notice of penalty and the reasons for its imposition. Instead, Captain
Woodward verbally informed him that he was dismissed from service and would be disembarked from the vessel.
It bears stressing that only in the exceptional case of clear and existing danger to the safety of the crew or vessel
that the required notices may be dispensed with, and, once again, records are bereft of evidence showing that
such was the situation when Avestruz

16. ELIZABETH M. GAGUI v. SIMEON DEJERO AND TEODORO R. PERMEJO


G.R. No. 196036 Date: October 23, 2013
Ponente: SERENO, C.J.

Section 10, RA 8042

DOCTRINE/S:
The liability of corporate directors and officers is not automatic. To make them jointly and solidarily liable with
their company, there must be a finding that they were remiss in directing the affairs of that company.

FACTS:

On 14 December 1993, respondents Simeon Dejero and Teodoro Permejo filed separate Complaints5 for illegal
dismissal, nonpayment of salaries and overtime pay, refund of transportation expenses, damages, and attorney's
fees against PRO Agency Manila, Inc., and Abdul Rahman Al Mahwes.

The LA rendered a decision ordering respondents Pro Agency Manila, Inc., and Abdul Rahman Al Mahwes to
jointly and severally pay complainants.

When the writ was returned unsatisfied, an Alias Writ of Execution was issued, but was also returned unsatisfied.

On 30 October 2002, respondents filed a Motion to Implead Respondent Pro Agency Manila, Inc.'s Corporate
Officers and Directors as Judgment Debtors. It included petitioner as the Vice-President/Stockholder/Director of
PRO Agency, Manila, Inc.

Angeles| Bajana | Balladares | Brillantes | Briones | Cabansag | Callanta | Chua | David|

De Leon | Gomez | Lopez | Macalino | Nostratis | Padilla | Reynon | Santos | Tan |Velasco
4E / 4F - 2018-2019
Page 33 of 920
LABOR REVIEW DIGEST
Atty. Joyrich Golangco

On 10 June 2003, a 2nd Alias Writ of Execution was issued, which resulted in the garnishment of petitioner's bank
deposit in the amount of P85,430.48. Accordingly, the 3rd Alias Writ of Execution was issued on 6 June 2005,
resulting in the levying of two parcels of lot owned by petitioner located in San Fernando, Pampanga.

Petitioner alleged that apart from not being made aware that she was impleaded as one of the parties to the case,
the dispositive portion of the 7 May 1997 Decision (1997 Decision) did not hold her liable in any form whatsoever.
More importantly, impleading her for the purpose of execution was tantamount to modifying a decision that had
long become final and executory.

LA RULING:
The Executive Labor Arbiter isued an Order stating that under Section 10 of Republic Act No. 8042 (R.A. 8042) or
the Migrant Workers and Overseas Filipinos Act of 1995, corporate officers may be held jointly and severally liable
with the placement agency for the judgment award.

NLRC RULING:
The NLRC ruled that in so far as overseas migrant workers are concerned, it is R.A. 8042 itself that describes the
nature of the liability of the corporation and its officers and directors. x x x [I]t is not essential that the individual
officers and directors be impleaded as party respondents to the case instituted by the worker. A finding of liability
on the part of the corporation will necessarily mean the liability of the corporate officers or directors.

CA RULING:
The CA affirmed the NLRC in a Decision. The CA stated that there was no need for petitioner to be impleaded x x
x because by express provision of the law, she is made solidarily liable with PRO Agency Manila, Inc., for any and
all money claims filed by private respondents. The CA further said that this is not a case in which the liability of the
corporate officer must be established because an allegation of malice must be proven. The general rule is that
corporate officers, directors and stockholders are not liable, except when they are made liable for their corporate
act by a specific provision of law, such as R.A. 8042.

ISSUE/S:
Whether or not petitioner may be held jointly and severally liable with PRO Agency Manila, Inc. in accordance with
Section 10 of R.A. 8042, despite not having been impleaded in the Complaint and named in the Decision.

HELD:
No.
1. The pertinent portion of Section 10, R.A. 8042 reads as follows:
SEC. 10. MONEY CLAIMS. - Notwithstanding any provision of law to the contrary, the Labor Arbiters of the
National Labor Relations Commission (NLRC) shall have the original and exclusive jurisdiction to hear and
decide, within ninety (90) calendar days after filing of the complaint, the claims arising out of an employer-
employee relationship or by virtue of any law or contract involving Filipino workers for overseas deployment
including claims for actual, moral, exemplary and other forms of damages.

The liability of the principal/employer and the recruitment/placement agency for any and all claims under this
section shall be joint and several. This provision shall be incorporated in the contract for overseas employment
and shall be a condition precedent for its approval. The performance bond to be filed by the
recruitment/placement agency, as provided by law, shall be answerable for all money claims or damages that may
be awarded to the workers. If the recruitment/placement agency is a juridical being, the corporate officers and
directors and partners as the case may be, shall themselves be jointly and solidarily liable with the corporation or
partnership for the aforesaid claims and damages. (Emphasis supplied)

Angeles| Bajana | Balladares | Brillantes | Briones | Cabansag | Callanta | Chua | David|

De Leon | Gomez | Lopez | Macalino | Nostratis | Padilla | Reynon | Santos | Tan |Velasco
4E / 4F - 2018-2019
Page 34 of 920
LABOR REVIEW DIGEST
Atty. Joyrich Golangco

2. As held in the case of Sto. Tomas v. Salac, "But the Court has already held, pending adjudication of this case,
that the liability of corporate directors and officers is not automatic. To make them jointly and solidarily liable with
their company, there must be a finding that they were remiss in directing the affairs of that company, such as
sponsoring or tolerating the conduct of illegal activities..."

Hence, for petitioner to be found jointly and solidarily liable, there must be a separate finding that she was
remiss in directing the affairs of the agency, resulting in the illegal dismissal of respondents. Examination
of the records would reveal that there was no finding of neglect on the part of the petitioner in directing
the affairs of the agency. In fact, respondents made no mention of any instance when petitioner allegedly
failed to manage the agency in accordance with law, thereby contributing to their illegal dismissal.

3. Moreover, while labor laws should be construed liberally in favor of labor, we must be able to balance this with
the equally important right of petitioner to due process. Because the 1997 Decision of Labor Arbiter Ramos was
not appealed, it became final and executory and was therefore removed from his jurisdiction. Modifying the tenor
of the judgment via a motion impleading petitioner and filed only in 2002 runs contrary to settled jurisprudence,
rendering such action a nullity.

17. ASSOCIATION OF MEDICAL CLINICS FOR OVERSEAS WORKERS, INC., (AMCOW) vs. GCC
APPROVED MEDICAL CENTERS ASSOCIATION, INC (GAMCA) . AND CHRISTIAN CANGCO
GR No. G.R. No. 207132 Date: December 06, 2016
Ponente: Brion, J.

Section 16 of RA No. 10022, amending Section 23 of RA No. 8042

DOCTRINE: The prohibition against the referral decking system under Section 16, RA No. 10022, is a valid
exercise of police power.

FACTS:

The DOH issued Administrative Order No. 5, Series of 2001 (AO 5-01) which directed the decking or equal
distribution of migrant workers among the several clinics who are members of GAMCA. AO 5-01 was issued to
comply with the Gulf Cooperative Countries (GCC) States' requirement that only GCC-accredited medical
clinics/hospitals' examination results will be honored by the GCC States' respective embassies. It required an
OFW applicant to first go to a GCC APPROVED MEDICAL CENTERS ASSOCIATION (GAMCA Center) which, in
turn, will refer the applicant to a GAMCA clinic or hospital.

Thereafter, the DOH issued AO No. 106, Series of 2002 holding in abeyance the implementation of the referral
decking system. The DOH reiterated its directive suspending the referral decking system in AO No. 159, Series
of 2004.

The DOH issued AO No. 167, Series of 2004 repealing AO 5-01, reasoning that the referral decking system
did not guarantee the migrant workers' right to safe and quality health service.

In Department Memorandum No. 2008-0210, dated September 26, 2008, then DOH Secretary Francisco T.
Duque III expressed his concern about the continued implementation of the referral decking system despite the
DOH's prior suspension directives. The DOH directed the "OFW clinics, duly accredited/licensed by the DOH

Angeles| Bajana | Balladares | Brillantes | Briones | Cabansag | Callanta | Chua | David|

De Leon | Gomez | Lopez | Macalino | Nostratis | Padilla | Reynon | Santos | Tan |Velasco
4E / 4F - 2018-2019
Page 35 of 920
LABOR REVIEW DIGEST
Atty. Joyrich Golangco

and/or by the Philippine Health Insurance Corporation (PHILHEALTH) belonging to and identified with GAMCA x x
x to forthwith stop, terminate, withdraw or otherwise end the x x x 'referral decking system.

GAMCA questioned the DOH's Memorandum No. 2008-0210 before the Office of the President (OP). In a
decision dated January 14, 2010, the OP nullified Memorandum No. 2008-0210.

On March 8, 2010, RA No. 10022 lapsed into law without the President's signature. Section 16 of RA No. 10022
amended Section 23 of RA No. 8042, adding two new paragraphs - paragraphs (c) and (d). The pertinent portions
of the amendatory provisions read:

Section 16. Under Section 23 of Republic Act No. 8042, as amended, added new paragraphs (c) and (d) with their
corresponding subparagraphs to read as follows:

(c) Department of Health. - The DOH shall regulate the activities and operations of all clinics which conduct
medical, physical, optical, dental, psychological and other similar examinations, hereinafter referred to as health
examinations, on Filipino migrant workers as requirement for their overseas employment;
(c.3) No group or groups of medical clinics shall have a monopoly of exclusively conducting health examinations
on migrant workers for certain receiving countries;
(c.4) Every Filipino migrant worker shall have the freedom to choose any of the DOH-accredited or DOH-operated
clinics that will conduct his/her health examinations and that his or her rights as a patient are respected. The
decking practice, which requires an overseas Filipino worker to go first to an office for registration and then farmed
out to a medical clinic located elsewhere, shall not be allowed;

On August 13, 2010, the Implementing Rules and Regulations (IRR) of RA No. 8042, as amended by RA No.
10022, took effect. Pursuant to Section 16 of RA No. 10022, the DOH, through its August 23, 2010 letter-order,
directed GAMCA to cease and desist from implementing the referral decking system.

(case) GAMCA filed with the RTC of Pasig City a petition for certiorari and prohibition with prayer for a
writ of preliminary injunction and/or temporary restraining order (GAMCA's petition).

It assailed: (1) the DOH's August 23, 2010 letter-order on the ground of grave abuse of discretion; and (2)
paragraphs c.3 and c.4, Section 16 of RA No. 10022, as well as Section 1 (c) and (d), Rule XI of the IRR, as
unconstitutional.

AMCOW filed an urgent motion for leave to intervene and to file an oppositionin-intervention, attaching its
opposition-in-intervention to its motion.

The RTC issued a writ of preliminary injunction directing the DOH to cease and desist from implementing its
August 23, 2010 and November 2, 2010 orders. The RTC likewise issued an order denying the motion for
inhibition/disqualification filed by AMCOW.

RTC RULING:

In its August 10, 2012 decision, the RTC granted GAMCA's certiorari petition and declared null and void ab initio
the DOH CDO letters. It also issued a writ of prohibition directing "the DOH Secretary and all persons acting on
his behalf to cease and desist from implementing the assailed Orders against the GAMCA.

The RTC upheld the constitutionality of Section 16 of RA No. 10022, amending Section 23 of RA No. 8042,
but ruled that Section 16 of RA No. 10022 does not apply to GAMCA.

Angeles| Bajana | Balladares | Brillantes | Briones | Cabansag | Callanta | Chua | David|

De Leon | Gomez | Lopez | Macalino | Nostratis | Padilla | Reynon | Santos | Tan |Velasco
4E / 4F - 2018-2019
Page 36 of 920
LABOR REVIEW DIGEST
Atty. Joyrich Golangco

The RTC reasoned out that the prohibition against the referral decking system under Section 16 of RA No. 10022
must be interpreted as applying only to clinics that conduct health examination on migrant workers bound for
countries that do not require the referral decking system for the issuance of visas to job applicants.

It noted that the referral decking system is part of the application procedure in obtaining visas to enter the GCC
States, a procedure made in the exercise of the sovereign power of the GCC States to protect their nationals from
health hazards, and of their diplomatic power to regulate and screen entrants to their territories. Under the
principle of sovereign equality and independence of States, the Philippines cannot interfere with this system and,
in fact, must respect the visa-granting procedures of foreign states in the same way that they respect our
immigration procedures.

The DOH and AMCOW separately sought reconsideration of the RTC's August 10, 2012 decision, which motions
the RTC denied.23 The DOH and AMCOW separately filed the present Rule 45 petitions.

The Court also suspended the implementation of the permanent injunction issued by the RTC of Pasay City,
Branch 108 in its August 10, 2012 decision.

Petitioner AMCOW's contention:

AMCOW posits that GAMCA availed of an improper remedy, as certiorari and prohibition lie only against quasi-
judicial acts, and quasi-judicial and ministerial acts, respectively. Since the disputed cease and desist order is
neither, the RTC should have dismissed the petition outright for being an improper remedy.

Respondent GAMCA's contention:

The DOH CDO letters prohibiting GAMCA from implementing the referral decking system embodied under Section
16 of Republic Act No. 10022 violates Section 3, Article II of the 1987 Constitution for being an undue taking of
property.

ISSUE/S:
1. Does the RTC have jurisdiction to grant GAMCA’s petitions for certiorari and prohibition against the DOH CDO
letters? – No.
2. Did the RTC err in finding grave abuse of discretion on the part of the DOH’s issuance of the DOH CDO letters?
– Yes.
3. Is the prohibition against the referral decking system under Section 16, RA No. 10022 a valid exercise of police
power? – Yes.
4. Does the prohibition against the referral decking system against GAMCA violate the principle of sovereign
equality and independence? – No.

HELD:
1. The petitions for certiorari and prohibition against the DOH CDO letters fall within the jurisdiction of the Court of
Appeals. Since the CDO Letter was a quasi-judicial act, the manner by which GAMCA assailed it before the courts
of law had been erroneous; the RTC should not have entertained GAMCA's petition.

First, acts or omissions by quasi-judicial agencies, regardless of whether the remedy involves a Rule 43 appeal or
a Rule 65 petition for certiorari, is cognizable by the Court of Appeals.
Angeles| Bajana | Balladares | Brillantes | Briones | Cabansag | Callanta | Chua | David|

De Leon | Gomez | Lopez | Macalino | Nostratis | Padilla | Reynon | Santos | Tan |Velasco
4E / 4F - 2018-2019
Page 37 of 920
LABOR REVIEW DIGEST
Atty. Joyrich Golangco

Since the DOH is part of the Executive Department and has acted in its quasi-judicial capacity, the petition
challenging its CDO letter should have been filed before the Court of Appeals. The RTC thus did not have
jurisdiction over the subject matter of the petitions and erred in giving due course to the petition for certiorari and
prohibition against the DOH CDO letters. In procedural terms, petitions for certiorari and prohibition against a
government agency are remedies avaiJable to assail its quasi-judicial acts, and should thus have been filed
before the CA.

Second, the Regional Trial Court of Pasay City unduly disregarded the requirements that there be "no other plain,
speedy and adequate remedy at law" and the doctrine of exhaustion of administrative remedies, when it gave due
course to the certiorari and prohibition petition against the DOH's CDO.

2. Yes, the RTC of Pasay reversibly erred in law when it held that the DOH acted with grave abuse of discretion m
prohibiting GAMCA from implementing the referral decking system. In exempting GAMCA from the referral
decking system that RA No. 10022 prohibits, the RTC of Pasay City noted that the regulation per se was not
unconstitutional, but its application to GAMCA would violate the principle of sovereign equality and independence.
While we agree with the RTC's ultimate conclusion upholding the constitutionality of the prohibition against the
referral decking system under RA No. 10022, our agreement proceeds from another reason; we disagree that the
prohibition does not apply to GAMCA and with the consequent ruling nullifying the DOH's CDO Letter.

The DOH had sufficient basis when it determined that the referral decking system prohibited under RA No. 10022
was the same decking system practiced by GAMCA. To reiterate, the referral decking system was not something
new; it was an old system that GAMCA practiced and was known to all in its scope and operating details. That
GAMCA had previously questioned the DOH prohibition and had been given ample opportunity to be heard when
it filed an appeal before the OP, negate the conclusion that GAMCA had been aggrieved by precipitate and unfair
DOH action

3. Yes, the prohibition against the referral decking system under Section 16, RA No. 10022, is a valid exercise of
police power. The State's police power is vast and plenary and the operation of a business, especially one that is
imbued with public interest (such as healthcare services), falls within the scope of governmental exercise of police
power through regulation.

RA No. 10022 expressly reflects the declared State policies to "uphold the dignity of its citizens whether in the
country or overseas, in general, and Filipino migrant workers," and to "afford full protection to labor, local and
overseas, organized and unorganized, and promote full employment and equality of employment opportunities for
all. Towards this end, the State shall provide adequate and timely social, economic and legal services to Filipino
migrant workers." The prohibition against the referral decking system in Section 16 of RA No. 10022 is an
expression and implementation of these state policies.

The guarantee under Section 16 for OFWs to be given the option to choose a quality healthcare service provider
as expressed in Section 16 (c) of RA No. 10022 is guaranteed by the prohibition against the decking practice and
against monopoly practices in OFW health examinations.

Section 16 likewise requires employers to accept health examinations from any DOH-accredited health facility; a
refusal could lead to their temporary disqualification under pertinent rules to be formulated by the Philippine
Overseas Employment Authority (POEA).97 These rules are part of the larger legal framework to ensure the
Overseas Filipino Workers' (OFW) access to quality healthcare services, and to curb existing practices that limit
their choices to specific clinics and facilities.

Angeles| Bajana | Balladares | Brillantes | Briones | Cabansag | Callanta | Chua | David|

De Leon | Gomez | Lopez | Macalino | Nostratis | Padilla | Reynon | Santos | Tan |Velasco
4E / 4F - 2018-2019
Page 38 of 920
LABOR REVIEW DIGEST
Atty. Joyrich Golangco

Separately from the Section 16 prohibition against the referral decking system, RA No. 10022 also prohibits and
penalizes the imposition of a compulsory exclusive arrangement requiring OFWs to undergo health examinations
only from specifically designated medical clinics, institutions, entities or persons. Section 5, in relation to Section 6
of RA No. 10022, penalizes compulsory, exclusive arrangements98 by imprisonment and fine and by the
automatic revocation of the participating medical clinic's license.

While Section 16 of RA No. 10022 does not specifically define the consequences of violating the prohibition
against the referral decking system, Republic Act No. 4226 (Hospital Licensure Act), which governs the licensure
and regulation of hospitals and health facilities, authorizes the DOH to suspend, revoke, or refuse to renew the
license of hospitals and clinics violating the law.

These consequences cannot but apply to the violation of the prohibition against the referral decking system under
RA No. 10022. If, under the law, the DOH can suspend, revoke, or refuse to renew the license of these hospitals
upon the finding that they violated any provision of law (whether those found in RA No. 4226 or in RA No. 10022),
it follows- as a necessarily included lesser power - that the DOH can likewise order these clinics and their
association to cease and desist from practices that the law deems to be undesirable.

4. No, the prohibition against the referral decking system against GAMCA does not violate the principle of
sovereign equality and independence. The RTC's decision misapplied the principle of sovereign independence
and equality to the present case. While the principles of sovereign independence and equality have been
recognized in Philippine jurisprudence, our recognition of this principle does not extend to the exemption of States
and their affiliates from compliance with Philippine regulatory laws.

The principle of sovereign equality and independence of states does not exempt GAMCA from the referral decking
system prohibition under RA No. 10022.

In the present case, GAMCA has not adduced any evidence in the court below, nor has it presented any argument
before us showing that the principle of sovereign equality and independence has developed into an international
custom shielding state agents from compliance with another state's domestic laws. Under this situation, the Court
is in no position to determine whether the practice that GAMCA alleges has indeed crystallized into an
international custom.

GAMCA has never proven in this case, too, that the GCC has extended its sovereign immunity to GAMCA.
Sovereign immunity belongs to the State, and it must first be extended to its agents before the latter may be
considered to possess sovereign immunity.

We point out furthermore that the prohibition against the referral decking system applies to hospitals and clinics,
as well as to OFW employers, and does not seek to interfere with the GCC's visa requirement processes. RA
10022 prohibits hospitals and clinics in the Philippines from practicing the referral decking system, and employers
from requiring OFWs to procure their medical examinations from hospitals and clinics practicing the referral
decking system.

The regulation applies to Philippine hospitals and clinics, as well as to employers of OFWs. It does not apply to
the GCCs and their visa processes. That the regulation could affect the OFWs' compliance with the visa
requirements imposed by GCCs does not place it outside the regulatory powers of the Philippine government.

In the same manner, GCC states continue to possess the prerogative to apply their visa requirements to any
foreign national, including our OFWs, who seeks to enter their territory; they may refuse to grant them entry for
failure to comply with the referral decking system, or they may adjust to the prohibition against the referral decking

Angeles| Bajana | Balladares | Brillantes | Briones | Cabansag | Callanta | Chua | David|

De Leon | Gomez | Lopez | Macalino | Nostratis | Padilla | Reynon | Santos | Tan |Velasco
4E / 4F - 2018-2019
Page 39 of 920
LABOR REVIEW DIGEST
Atty. Joyrich Golangco

system that we have imposed. These prerogatives lie with the GCC member states and do not affect at all the
legality of the prohibition against the referral decking system.

Lastly, the effect of the prohibition against the referral decking system is beyond the authority of this Court to
consider. The wisdom of this prohibition has been decided by Congress, through the enactment of RA No. 10022.
Our role in this case is merely to determine whether our government has the authority to enact the law's
prohibition against the referral decking system, and whether this prohibition is being implemented legally. Beyond
these lies the realm of policy that, under our Constitution's separation of powers, this Court cannot cross.

Other Notes/ SC Pronouncements:


 The petitions for certiorari and prohibition against the DOH CDO letters were premature challenges -
they failed to comply with the requirement that there be "no other plain, speedy and adequate remedy"
and with the doctrine of exhaustion of administrative remedies.
_________________________________________________________________________
18. Jakerson Gargallo vs. DOHLE Seafront Crewing and Mayronilo Padiz
GR No. 215551 Date: August 17, 2016
Ponente: Perlas-Bernabe, J.

Section 10, RA 8042

DOCTRINE: Section 10 of RA 8042, as amended, expressly provides for joint and solidary liability of corporate
directors and officers with the recruitment/placement agency for all money claims or damages that may be
awarded to OFWs.

FACTS:
This is a consolidated MR of the petitioner and respondents of a SC decision (original case).
(Orig. Case)
Plaintiff/Petitioner:
Petitioner Gargallo filed a complaint for permanent total disability benefits against respondents before the NLRC.
He claimed that he had an accident while lifting heavy loads of lube oil drum which rendered him permanently
unfit to work. His permanent total unfitness to work was duly certified by his chosen physician whose certification,
petitioner claims must prevail over the palpably self-serving and biased assessment of the company-designated
physicians.

Defendant/Respondent:
Respondents, on the otherhand, claimed that the fit-to-work findings of the company-designated physicians must
prevail over that of petitioner's independent doctor, since they were the ones who continuously treated and
monitored petitioner's medical condition; and petitioner failed to comply with the conflict-resolution procedure
under the POEA Standard Employment Contract (POEA-SEC). Respondents further averred that the filing of the
disability claim was premature since petitioner was still undergoing medical treatment within the allowable 240-day
period at the time the complaint was filed.

Labor Arbiter / NLRC RULING: The LA and the NLRC gave more credence to the medical report of petitioner's
independent doctor and, thus, granted petitioner's disability claim, and ordered respondents to jointly and
severally pay petitioner his permanent total disability benefits, albeit at different amounts.

CA RULING: However, upon appeal to the CA, the CA reversed the NLRC decision and dismissed the petitioner's
complaint.

Angeles| Bajana | Balladares | Brillantes | Briones | Cabansag | Callanta | Chua | David|

De Leon | Gomez | Lopez | Macalino | Nostratis | Padilla | Reynon | Santos | Tan |Velasco
4E / 4F - 2018-2019
Page 40 of 920
LABOR REVIEW DIGEST
Atty. Joyrich Golangco

SC RULING:
Upon appeal to the SC, the Court affirmed the CA decision dismissing petitioner's claim for permanent total
disability benefits, and ordered respondents Dohle Seafront and Dohle Manning, jointly and severally, to pay
petitioner his income benefit for 194 days, plus 10% of the total amount of the income benefit as attorney's fees.
However, the Court found no basis to hold Padiz solidarity liable with the manning agency for payment of the
monetary awards to petitioner, absent any showing that acted beyond the scope of his authority or with malice.

(Consolidated Case:) ---> GR 215551

Petitioner's MR:
Petitioner argues that the lapse of the 120-day period from the onset of disability rendered him permanently and
totally disabled because the extension of the medical treatment was unjustified; and the resort to a third doctor is
a directory, not a mandatory requirement.

Respondents' MR:
Respondents contend that entitlement to income benefit is applicable only to land-based employees compulsorily
registered with the SSS. They also claim that if petitioner is entitled to income benefit, it is SSS which has the duty
to pay the same. Lastly, they dispute the grant of the award of attorney's fees.

ISSUE/S:
1. Was the CA correct in dismissing the case because of prematurity and failure to comply with the conflict-
resolution procedure under the POEA-SEC? - YES
2. Should Mr. Padiz be held jointly and severally liable with DOHLE? - YES
3. Is the entitlement to income benefit also applicable to seafarers? - YES
4. If the seafarers are entitle to income benefit, may the manning agency refuse to pay it because is it the duty of
SSS to pay the same? - NO
4. Should Attorney's fees be awarded? - NO

HELD:
1. YES, the CA correctly ruled that the filing of the complaint for permanent total disability benefits was
premature, and should have been dismissed for lack of cause of action, considering that at that time: (a)
petitioner was still under the medical treatment of the company-designated physicians within the allowable
240-day period; (b) the latter had not yet issued any assessment as to his fitness or disability; and (c)
petitioner had not yet secured any assessment from his chosen physician, whom he consulted only more
than 2 months thereafter.

Also, petitioner failed to comply with the prescribed procedure under Section 20 (A) (3) of the 2010 POEA-
SEC on the joint appointment by the parties of a third doctor, in case the seafarer's personal doctor
disagrees with the company-designated physician's fit-to-work assessment.

The seafarer's non-compliance with the mandated conflict-resolution procedure under the POEA-SEC and the
CBA militates against his claims, and results in the affirmance of the fit-to-work certification of the company-
designated physician.

The POEA-SEC and the CBA clearly provide that when a seafarer sustains a work-related illness or injury
while on board the vessel, his fitness or unfitness for work shall be determined by the company-
designated physician.

Angeles| Bajana | Balladares | Brillantes | Briones | Cabansag | Callanta | Chua | David|

De Leon | Gomez | Lopez | Macalino | Nostratis | Padilla | Reynon | Santos | Tan |Velasco
4E / 4F - 2018-2019
Page 41 of 920
LABOR REVIEW DIGEST
Atty. Joyrich Golangco

If the physician appointed by the seafarer disagrees with the company-designated physician's
assessment, the opinion of a third doctor may be agreed jointly between the employer and the seafarer to
be the decision final and binding on them.

Thus, while petitioner had the right to seek a second and even a third opinion, the final determination of whose
decision must prevail must be done in accordance with an agreed procedure. Unfortunately, the petitioner did not
avail of this procedure; hence, we have no option but to declare that the company-designated doctor's certification
is the final determination that must prevail.

2. YES. Section 10 of RA 8042, as amended, expressly provides for joint and solidary liability of corporate
directors and officers with the recruitment/placement agency for all money claims or damages that may be
awarded to OFWs.

While a corporate director, trustee, or officer who entered into contracts in behalf of the corporation generally,
cannot be held personally liable for the liabilities of the latter, in deference to the separate and distinct legal
personality of a corporation from the persons composing it, personal liability of such corporate director,
trustee, or officer, along (although not necessarily) with the corporation, may validly attach when he is
made by a specific provision of law personally answerable for his corporate action.
Applicable laws form part of, and are read into, contracts without need for any express reference thereto;
more so, when it pertains to a labor contract which is imbued with public interest. Each contract thus
contains not only what was explicitly stipulated therein, but also the statutory provisions that have any
bearing on the matter."

As applied herein, Section 10 of RA 8042, as amended, and the pertinent POEA Rules are deemed incorporated
in petitioner's employment contract with respondents. These provisions are in line with the State's policy of
affording protection to labor and alleviating the workers' plight, and are meant to assure OFWs immediate and
sufficient payment of what is due them. Thus, as the law provides, corporate directors and officers are
themselves solidarily liable with the recruitment/placement agency for all money claims or damages that
may be awarded to OFWs.

3. YES, considering that the 2010 POEA-SEC accords upon the manning agency/foreign principal the duty to
cover Filipino seafarers under the SSS and other social protection government agencies.

4. NO, because the income benefit arising from a covered employee's temporary total disability is to be advanced
by the employer, subject to reimbursement by the SSS upon compliance with the conditions set forth under
Section 1, 41 Rule X of the Rules Implementing Title II, Book IV of the Labor Code.

5. NO, in labor cases, the withholding of wages and benefits need not be coupled with malice or bad faith to
warrant the grant of attorney's fees since all that is required is that the refusal to pay was without justification,
thus, compelling the employee to litigate.

Since the complaint was filed: (a) when the petitioner was still under treatment; (b) prior to the assessment of the
company-designated physician within the allowable 240-day period; and (c) without complying with the prescribed
conflict-resolution procedure, therefore, there was no unlawful withholding of benefits, rendering the award of
attorney's fees to be improper.

19 Princess Talent Center Production, Inc. and/or Luchi Singh Moldes v. Desiree Masagca
G.R. No. 191310 April 11, 2018
Leonardo-De Castro, J.

Angeles| Bajana | Balladares | Brillantes | Briones | Cabansag | Callanta | Chua | David|

De Leon | Gomez | Lopez | Macalino | Nostratis | Padilla | Reynon | Santos | Tan |Velasco
4E / 4F - 2018-2019
Page 42 of 920
LABOR REVIEW DIGEST
Atty. Joyrich Golangco

SECTION 10, RA 8042

Doctrines:
Dismissal from employment has two facets: first, the legality of the act of dismissal, which constitutes substantive
due process; and, second, the legality of the manner of dismissal, which constitutes procedural due process. The
burden of proof rests upon the employer to show that the disciplinary action was made for lawful cause or that the
termination of employment was valid. When in doubt, the case should be resolved in favor of labor pursuant to the
social justice policy of our labor laws and the 1987 Constitution.

Second paragraph of Section 10 of RA 8042 is plain and clear, the joint and several liability of the
principal/employer, recruitment/placement agency, and the corporate officers of the latter, for the money claims
and damages of an overseas Filipino worker is absolute and without qualification.

Facts:

Respondent auditioned for a singing contest when a talent manager approached her to discuss her show
business potential. She then went to the office of petitioner PTCPI, a domestic corporation engaged in the
business of training and development of actors, singers, dancers, and musicians in the movie and entertainment
industry. Respondent met petitioner Moldes, president of PTCPI, who persuaded her to apply for a job as a
singer/entertainer in South Korea. An employment contract was executed between respondent and petitioner
PTCPI as the Philippine agent of Saem Entertainment Company, Ltd. (SAENCO), the Korean principal/promoter.

Respondent left for South Korea and worked there as a singer for nine months, until her repatriation to the
Philippines. Believing that the termination of her contract was unlawful and premature, respondent filed a
complaint against petitioners and SAENCO with the NLRC.

LA RULING:

LA dismissed respondent’s complaint for lack of merit noting that the POEA approved contract declares that the
duration of employment was for six months only. Respondent therefore was not illegally dismissed since she was
able to finish the duration of the contract as approved by the POEA. Respondent is likewise not entitled to
payment of the unexpired portion of the employment contract.

NLRC RULING:
The NLRC ruled in respondent’s favor. The vouchers that the respondent received covering nine months implied
that they consented to her working up to nine months despite having a visa good for only six months.

Acting on a Motion for Reconsideration, the NLRC reversed its decision and dismissed respondent’s appeal for
several fatal procedural defects. Nonetheless, NLRC ultimately found that respondent failed to present evidence
to prove that she had been illegally dismissed. Records show that the employment contract was for a period of six
months only.

CA RULING:
The CA took a liberal approach by excusing the technical lapses of respondent’s appeal before the NLRC for the
sake of substantial justice. It then held that the respondent was dismissed from employment without just cause
and without procedural due process, and that petitioners and SAENCO were solidarily liable to pay respondent
her unpaid salaries for one year.

Petitioners failed to discharge the burden of proving that respondent was terminated from employment for a just
and valid cause. Their claim that respondent was deported because her employment contract has already
Angeles| Bajana | Balladares | Brillantes | Briones | Cabansag | Callanta | Chua | David|

De Leon | Gomez | Lopez | Macalino | Nostratis | Padilla | Reynon | Santos | Tan |Velasco
4E / 4F - 2018-2019
Page 43 of 920
LABOR REVIEW DIGEST
Atty. Joyrich Golangco

expired, was without any basis. Before being deployed to South Korea, they made her believe that her contract
was for a period of one year. She then relied on such misrepresentation and thus continuously worked for more
than nine months.

Issue:
1. Was respondent illegally dismissed?
2. Is respondent entitled to monetary claims against petitioners and SAENCO?
3. Who may be liable for the monetary claims?

Ratio:

1. YES. The Constitutional guarantee of security of tenure extends to Filipino overseas contract workers (Sameer
Overseas Placement Agency, Inc. v. Cabiles). Since respondent’s Employment Contract was executed in the
Philippines, Philippine Constitution and Labor Laws governed her employment with petitioners and SAENCO. An
employee’s right to security of tenure, protected by the Constitution and statutes, means that no employee shall
be dismissed unless there are just or authorized causes and only after compliance with procedural and
substantive due process.

It is undisputed that when respondent was dismissed from employment and repatriated, her original six-month
Employment Contract had already expired. Although respondent’s employment was good for six months only, the
Court is convinced that it was extended under the same terms and conditions for another six months.

Respondent and petitioners submitted evidence establishing that respondent continued to work for SAENCO even
after the original six-month period expired. Even in the absence of a written contract, the six-month extension of
respondent’s employment is practically admitted by the petitioners. Petitioners presented nine vouchers to prove
that respondent received her salaries from SAENCO for nine months. Petitioners also did not deny that Moldes
confronted respondent in Korea about the latter’s outstanding loan. These revealed that petitioners were aware
that respondent was still working for SAENCO up to that time.

Hence, respondent had been working for SAENCO in South Korea, pursuant to her employment contract,
extended for another six-month period, when she was dismissed and repatriated to the Philippines. Respondent
was therefore illegally dismissed. (It is unnecessary to address that she had been misled into believing that her
contract and visa was good for one year).

Also, petitioners failed to afford respondent procedural due process. There had been no attempt at all by the
petitioners and/or SAENCO to submit proof of compliance with the requirements of notice and hearing prior to
respondent’s dismissal from employment.

The lack of a valid cause, together with the failure of SAENCO to comply with the twin-notice and hearing
requirements, underscored the illegality surrounding respondent’s dismissal.

2. YES. Respondent’s monetary claims against petitioners and SAENCO are governed by Section 10 of RA No.
8042 (Migrant Workers and Overseas Filipinos Act of 1995).

Respondent is entitled to an award of her salaries for the unexpired three months of her Extended Employment
Contract. Respondent also has the right to the reimbursement of her placement fee. The award of attorney’s fees
is likewise justified.

3. SAENCO, as principal/employer, is jointly and solidarily liable with petitioner PTCPI for respondent’s monetary
awards. Second paragraph of Section 10 of RA 8042 is plain and clear, the joint and several liability of the
Angeles| Bajana | Balladares | Brillantes | Briones | Cabansag | Callanta | Chua | David|

De Leon | Gomez | Lopez | Macalino | Nostratis | Padilla | Reynon | Santos | Tan |Velasco
4E / 4F - 2018-2019
Page 44 of 920
LABOR REVIEW DIGEST
Atty. Joyrich Golangco

principal/employer, recruitment/placement agency, and the corporate officers of the latter, for the money claims
and damages of an overseas Filipino worker is absolute and without qualification.

20. Eileen David vs. Glenda Marquez


GR No. 209859 Date: June 5, 2017
Ponente: Tijam, J.

Section 9, RA 8042

Doctrine: A criminal action arising from illegal recruitment may be filed in the place where the offended party
actually resides at the time of the commission of the offense.

FACTS:

Two separate informations (illegal recruitment and estafa) were filed against petitioner David based on the
complaint of respondent Marquez that petitioner approached her in Kidapawan City and represented that she
could recruit her to work abroad. Further, petitioner demanded payment of placement fees and other expenses
from the respondent for the processing of the latter's application, to which the respondent heeded. Respondent's
application was, however, denied and worse, the money that she put out therefor was never returned.

Petitioner filed a Motion to Quash the Information arguing that the City Prosecutor of Manila had no jurisdiction
over the case as the alleged crime was committed in Kidapawan City.

RTC Ruling:
The RTC Manila Branch 55 ruled that it has jurisdiction to take cognizance of the case, citing Section 9 of RA
8042. However, upon MR, the RTC reversed itself and held that it had no jurisdiction to try the cases since the
crimes of Illegal Recruitment and Estafa were not committed in its territory but in Kidapawan City.

CA Ruling:
Respondent filed a Petition for Certiorari before the CA. The CA ruled that the RTC has jurisdiction over the cases
of Illegal Recruitment and Estafa, citing Section 9 of RA 8042, which provides that a criminal action arising from
illegal recruitment may be filed in the place where the offended party actually resides at the time of the
commission of the offense. According to the CA, it was established that herein respondent was residing in
Sampaloc, Manila at the time of the commission of the crimes. Therefore, the 2 Informations herein were correctly
filed with the RTC of Manila. Petitioner’s MR was denied. Hence, this petition.

Appeal to the SC:

Petitioner's Contention:

Petitioner argues that the CA committed a grave abuse of discretion in declaring that the respondent had the legal
personality to assail the dismissal of the criminal cases as respondent is not the proper party to do so. Petitioner
argues that the OSG is the appellate counsel of the People of the Philippines in all criminal cases and as such,
the appeal in the criminal aspect should be taken solely by the State and the private complainant is limited only to
the appeal of the civil aspect.

Petitioner maintains that the RTC of Manila has no jurisdiction over the cases as the alleged acts constituting the
crimes charged were committed in Kidapawan City and not in Manila.
Angeles| Bajana | Balladares | Brillantes | Briones | Cabansag | Callanta | Chua | David|

De Leon | Gomez | Lopez | Macalino | Nostratis | Padilla | Reynon | Santos | Tan |Velasco
4E / 4F - 2018-2019
Page 45 of 920
LABOR REVIEW DIGEST
Atty. Joyrich Golangco

Respondent's Contention:

Respondent argues that the argument as regards her legal personality in filing the petition for certiorari before the
CA reveals that petitioner misunderstood the difference between an appeal and a special civil action for certiorari
under Rule 65 of the Rules of Court. In fact, respondent agrees with the petitioner that only the State, through the
OSG, may file an appeal in a criminal case. As an appeal is not available for a private complainant in a criminal
case, an independent action through a petition for certiorari under Rule 65, therefore, is available to the said
aggrieved party.

ISSUES:
1. Does the RTC of Manila have jurisdiction over the cases of Illegal Recruitment and Estafa? – YES
2. Does the respondent, on her own, have legal personality to file the petition for certiorari before the
CA? - YES

HELD:
1. Yes, the RTC of Manila has jurisdiction over the cases of Illegal Recruitment and Estafa. Venue in criminal
cases is an essential element of jurisdiction. It is a fundamental rule that for jurisdiction to be acquired by courts in
criminal cases, the offense should have been committed or any one of its essential ingredients took place within
the territorial jurisdiction of the court. Territorial jurisdiction in criminal cases is the territory where the court has
jurisdiction to take cognizance or to try the offense allegedly committed therein by the accused.
At the risk of being repetitive, Sec. 9 of RA 8042, however, fixed an alternative venue from that provided in
Section 15(a) of the Rules of Criminal Procedure, i.e., a criminal action arising from illegal recruitment may also be
filed where the offended party actually resides at the time of the commission of the offense and that the court
where the criminal action is first filed shall acquire jurisdiction to the exclusion of other courts.

The RTC of Manila committed grave abuse of discretion and in fact, a palpable error, in ordering the quashal of
the Informations. The express provision of the law is clear that the filing of criminal actions arising from illegal
recruitment before the R TC of the province or city where the offended party actually resides at the time of the
commission of the offense is allowed. It goes without saying that the dismissal of the case on a wrong ground,
indeed, deprived the prosecution, as well as the respondent as complainant, of their day in court.
The respondent resides in Manila; hence, the filing of the case before the RTC of Manila was proper. Thus, the
trial court should have taken cognizance of the case, and if it will eventually be shown during trial that the offense
was committed somewhere else, then the court should dismiss the action for want of jurisdiction.

2. Yes, the respondent has the legal personality to file a petition for certiorari under Rule 65. Generally, the
prosecution cannot appeal or bring error proceedings from a judgment rendered in favor of the defendant in a
criminal case due to the final and executory nature of a judgment of acquittal and the constitutional prohibition
against double jeopardy. Despite acquittal, however, the offended party or the accused may appeal, but only with
respect to the civil Aspect of the decision.

However, there have been occasions when this Court has allowed the offended party to pursue the criminal action
on his/her own behalf, as when there is a denial of due process as in this case. Indeed, the right of offended
parties to appeal or question an order of the trial court which deprives them of due process has always been
recognized, the only limitation being that they cannot appeal any adverse ruling if to do so would place the
accused in double jeopardy.

The Court upheld the CA's finding that double jeopardy does not exist in this case. Inasmuch as the dismissal of
the charges by the RTC was done without regard to due process of law, the same is null and void. It is as if there
was no acquittal or dismissal of the case at all, and the same cannot constitute a claim for double jeopardy.
Angeles| Bajana | Balladares | Brillantes | Briones | Cabansag | Callanta | Chua | David|

De Leon | Gomez | Lopez | Macalino | Nostratis | Padilla | Reynon | Santos | Tan |Velasco
4E / 4F - 2018-2019
Page 46 of 920
LABOR REVIEW DIGEST
Atty. Joyrich Golangco

21. Powerhouse Staffbuilders International Inc. vs. Rey et al


GR No. 190203 Date: November 7, 2016
Ponente: Jardeleza, J.

Section 10, RA 8042

Doctrine: The solidary liability of the principal and the recruitment agency to the employees shall not be affected
by any substitution, amendment or modification for the entire duration of the employment contract.

FACTS:
Powerhouse hired respondents as operators for its foreign principal, Catcher Technical Co. Ltd. (Catcher), based
in Taiwan, for the duration of two years commencing upon their arrival at the jobsite. Later on, Catcher informed
respondent employees that they would be reducing their working days due to low orders and financial difficulties.
The respondent employees were later repatriated to the Philippines before their contract ended.

Respondent employees filed separate complaints for illegal dismissal, refund of placement fees, moral and
exemplary damages, as well as attorney's fees, against Powerhouse and Catcher before the LA which were later
consolidated upon their motion.

Powerhouse maintained that respondent employees voluntarily gave up their jobs following their rejection of
Catcher's proposal to reduce their working days. It contended that before their repatriation, each of the
respondents accepted payments by way of settlement, with the assistance of Labor Attache Salud.

Powerhouse moved to implead JEJ International Manpower Services (JEJ) as respondent on account of the
alleged transfer to the latter of Catcher's accreditation.

JEJ submitted its position paper, arguing that the supposed transfer of accreditation to it did not affect the joint
and solidary liability of Powerhouse in favor of respondent employees. It averred that any contract between JEJ
and Powerhouse could not be enforced in the case as it involved no employer-employee relationship and is
therefore outside the jurisdiction of the labor arbiter

LA Ruling:

The LA ruled in favor of the respondents, finding the respondent employees' dismissal and/or pre-termination of
their employment contracts illegal. However, the LA ruled that in accordance with Section 10 of RA 8042,the
amount of wages the respondent employees are entitled to by reason of the illegal dismissal/pre-
termination of their employment contracts is equivalent to the unexpired term thereof or to three months
for every year of service whichever is less.

NLRC Ruling:

Upon appeal to the NLRC affirmed the LA’s decision with modification. The NLRC absolved JEJ from liability,
upon the NLRC's findings that it was not privy to the respondents' deployment. It also held Powerhouse jointly and
severally liable with William Go, Catcher, and Chen Wei to reimburse to respondents. Powerhouse filed a MR but
was denied.

CA RULING:

Angeles| Bajana | Balladares | Brillantes | Briones | Cabansag | Callanta | Chua | David|

De Leon | Gomez | Lopez | Macalino | Nostratis | Padilla | Reynon | Santos | Tan |Velasco
4E / 4F - 2018-2019
Page 47 of 920
LABOR REVIEW DIGEST
Atty. Joyrich Golangco

Powerhouse filed before the CA a Petition for Certiorari imputing grave abuse of discretion on the part of the
NLRC in declaring the repatriation of respondent employees as an act of illegal dismissal. However, the CA
dismissed said petition.

ISSUE/S:
1. Were respondent employees illegally dismissed? - Yes
2. Are respondent employees entitled to the payment of monetary claims?
3. Does the transfer of accreditation to another recruitment and placement agency relieve the original
recruitment agency from any liability?

HELD:
1. Yes. The onus of proving that an employee was not dismissed or, if dismissed, his dismissal was not illegal,
fully rests on the employer, and the failure to discharge the onus would mean that the dismissal was not justified
and was illegal. The burden of proving the allegations rests upon the party alleging and the proof must be clear,
positive, and convincing.

Here, there is no reason to overturn the factual findings of the Labor Arbiter, the NLRC and the CA, all of which
have unanimously declared that respondent employees were made to resign against their will after the foreign
principal, Catcher, stopped providing them food for their subsistence as early as March 2, 2001, when they were
informed that they would be repatriated, until they were repatriated.

The filing of complaints for illegal dismissal immediately after repatriation belies the claim that respondent
employees voluntarily chose to be separated and repatriated. Voluntary repatriation, much like resignation, is
inconsistent with the filing of the complaints.

2. Yes, respondent employees are entitled to the payment of monetary claims. The Court agrees that respondent
employees are entitled to money claims and full reimbursement of their respective placement fees. However, the
award of the three-month equivalent of respondent employees' salaries should be increased to the amount
equivalent to the unexpired term of the employment contract.

In Serrano, the Court declared unconstitutional the clause in Section 10 of R.A. No. 8042 limiting the wages that
could be recovered by an illegally dismissed overseas worker to three months. The Court held that the clause "or
for three (3) months for every year of the unexpired term, whichever is less" (subject clause) is both a
violation of the due process and equal protection clauses of the Constitution. In 2010, upon promulgation of
Republic Act No. 10022, the subject clause was reinstated. Presented with the unique situation that the law
passed incorporated the exact clause already declared unconstitutional, without any perceived substantial change
in the circumstances, in Sameer, we, once again, declared the reinstated clause unconstitutional, this time as
provided in Section 7 of R.A. No. 10022.

We likewise affirm the refund to the respondent employees of the unauthorized monthly deductions in the amount
of NT$10,000.00. Contrary to Powerhouse's contention that the claim for refund was based merely on allegations,
respondent employees were able to present proof before the NLRC in the form of the 2 passbooks given to each
of them by their foreign employer.

The burden of proving monetary claims rests on the employer. The reason for this rule is that the pertinent
personnel files, payrolls, records, remittances and other similar documents are not in the possession of the worker
but in the custody and absolute control of the employer. Thus, in failing to present evidence to prove that
Catcher, with whom it shares joint and several liability with under Section 10 of R.A. No. 8042, had paid all

Angeles| Bajana | Balladares | Brillantes | Briones | Cabansag | Callanta | Chua | David|

De Leon | Gomez | Lopez | Macalino | Nostratis | Padilla | Reynon | Santos | Tan |Velasco
4E / 4F - 2018-2019
Page 48 of 920
LABOR REVIEW DIGEST
Atty. Joyrich Golangco

the monetary claims of respondent employees, Powerhouse has, once again, failed to discharge the onus
probandi; thus, the LA and the NLRC properly awarded these claims to respondent employees.

Also, the respondent employees are likewise entitled to the payment of interest over their monetary claims.

3. Yes, Powerhouse is liable for the monetary claims. JEJ could not be held liable for the monetary claims of
respondent employees on account of the alleged transfer of accreditation to it.

The terms of Section 10 of R.A. No. 8042 clearly states the solidary liability of the principal and the recruitment
agency to the employees and this liability shall not be affected by any substitution, amendment or modification for
the entire duration of the employment contract.

In this case, even if there was transfer of accreditation by Catcher from Powerhouse to JEJ, Powerhouse's liability
to respondent employees remained intact because respondent employees are not privy to such contract, and in
their overseas employment contract approved by POEA, Powerhouse is the recruitment agency of Catcher. To
relieve Powerhouse from liability arising from the approved overseas employment contract is to change the
contract without the consent from the other contracting party, respondent employees in this case.

To rule otherwise and free Powerhouse of liability against respondent employees would go against the
rationale of R.A. No. 8042 to protect and safeguard the rights and interests of overseas Filipinos and
overseas Filipino workers, in particular, and run contrary to this law's intention to an additional layer of
protection to overseas workers. This ensures that overseas workers have recourse in law despite the
circumstances of their employment. By providing that the liability of the foreign employer may be "enforced to the
full extent" against the local agent, the overseas worker is assured of immediate and sufficient payment of what is
due them. Corollarily, the provision on joint and several liability in R.A. No. 8042 shifts the burden of going after
the foreign employer from the overseas worker to the local employment agency. However, the local agency that is
held to answer for the overseas worker's money claims is not left without remedy. The law does not preclude it
from going after the foreign employer for reimbursement of whatever payment it has made to the employee to
answer for the money claims against the foreign employer.

ART. 22-81, LABOR CODE

8.Century Canning Corp. vs CA and Gloria Palad


GR No.:152894 Date: August 17, 2007
Ponente: Carpio, J.

DOCTRINE: The TESDA’s approval of the employers apprenticeship program is required before the
employer is allowed to hire apprentices. No enterprise shall be allowed to hire apprentices unless its
apprenticeship program is registered and approved by TESDA.

FACTS:

On 15 July 1997, Century Canning Corporation (petitioner) hired Gloria C. Palad (Palad) as fish cleaner at
petitioners tuna and sardines factory.
Palad signed on 17 July 1997 an apprenticeship agreement with petitioner.
On 25 July 1997, petitioner submitted its apprenticeship program for approval to The TESDA.
On 26 September 1997, the TESDA approved petitioners apprenticeship program.

Angeles| Bajana | Balladares | Brillantes | Briones | Cabansag | Callanta | Chua | David|

De Leon | Gomez | Lopez | Macalino | Nostratis | Padilla | Reynon | Santos | Tan |Velasco
4E / 4F - 2018-2019
Page 49 of 920
LABOR REVIEW DIGEST
Atty. Joyrich Golangco

According to petitioner, a performance evaluation conducted on 15 November 1997, where petitioner gave Palad
a rating of “needs improvement” since she scored only 27.75% based on a 100% performance indicator,
numerous tardiness and absences.

Petitioner issued a termination notice dated 22 November 1997 to Palad effective 28 November.

Palad then filed a complaint for illegal dismissal, underpayment of wages, and non-payment of pro-rated
13th month pay for the year 1997.

LABOR ARBITER’S RULING:


The Labor Arbiter dismissed the complaint for lack of merit but ordered petitioner to pay Palad her last salary and
her pro-rated 13th month pay.

NLRC RULING:
On appeal, the NLRC affirmed with modification, the LA’s decision adding backwages for 2 months.

CA RULING:
The Court of Appeals held that the apprenticeship agreement which Palad signed was not valid and binding
because it was executed more than two months before the TESDA approved petitioners apprenticeship program
and that prior approval of the proposed apprenticeship program is a condition sine qua non before an
apprenticeship agreement can be validly entered into.

Palad was also Illegally Dismissed for failure to be properly apprised of the required standard of performance and
was not afforded due process because petitioner did not comply with the twin requirements of notice and hearing.

ISSUE/S:

1.Whether the CA erred in holding Pala was not an apprentice; and


2.Whether the CA erred in holding that Petitioner had not adequately proven the existence of a valid cause in
terminating Palad.

HELD:
1.The Labor Code defines an apprentice as a worker who is covered by a written apprenticeship agreement with
an employer.

Republic Act No. 7796 emphasizes TESDAs approval of the apprenticeship program as a pre-requisite
for the hiring of apprentices.

The TESDAs approval of the employers apprenticeship program is required before the employer is allowed to hire
apprentices. Prior approval from the TESDA is necessary to ensure that only employers in the highly technical
industries may employ apprentices and only in apprenticeable occupations. Thus, under RA 7796, employers can
only hire apprentices for apprenticeable occupations which must be officially endorsed by a tripartite body and
approved for apprenticeship by the TESDA. This is to ensure the protection of apprentices and to obviate possible
abuses by prospective employers who may want to take advantage of the lower wage rates for apprentices and
circumvent the right of the employees to be secure in their employment.

This is further emphasized by Department Order No. 68-04, which provides the guidelines in the implementation
of the Apprenticeship and Employment Program of the government, specifically states that no enterprise shall
be allowed to hire apprentices unless its apprenticeship program is registered and approved by TESDA.
Angeles| Bajana | Balladares | Brillantes | Briones | Cabansag | Callanta | Chua | David|

De Leon | Gomez | Lopez | Macalino | Nostratis | Padilla | Reynon | Santos | Tan |Velasco
4E / 4F - 2018-2019
Page 50 of 920
LABOR REVIEW DIGEST
Atty. Joyrich Golangco

Since Palad is not considered an apprentice because their apprenticeship agreement was enforced before the
TESDAs approval of petitioners apprenticeship program, Palad is deemed a regular employee performing the job
of a fish cleaner.

2.Petitioner failed to substantiate its claim that Palad was terminated for valid reasons. In fact, the NLRC found
that petitioner failed to prove the authenticity of the performance evaluation which petitioner claims to have
conducted on Palad, where Palad received a performance rating of only 27.75%. Petitioner merely relies on the
performance evaluation to prove Palads inefficiency. It was likewise not shown that petitioner ever apprised
Palad of the performance standards set by the company. When the alleged valid cause for the termination of
employment is not clearly proven, as in this case, the law considers the matter a case of illegal dismissal.

9.Bernardo vs NLRC
GR No.: 122917 Date: July 12, 1999

Ponente: Panganiban, J.

DOCTRINE: the Magna Carta for Disabled Persons mandates that a qualified disabled employee should be given
the same terms and conditions of employment as a qualified able-bodied person.

FACTS:

Complainants numbering 43 are deaf-mutes who were hired on various periods from 1988 to 1993 by respondent
Far East Bank and Trust Co. as Money Sorters and Counters through a uniformly worded agreement called
Employment Contract for Handicapped Workers.
Petitioners maintain that they should be considered regular employees, because their task as money sorters and
counters was necessary and desirable to the business of respondent bank. They further allege that their contracts
served merely to preclude the application of Article 280 and to bar them from becoming regular employees.
Private respondent, on the other hand, submits that petitioners were hired only as special workers and should not
in any way be considered as part of the regular complement of the Bank. Rather, they were special workers under
Article 80 of the Labor Code and were even hired temporarily under a special employment arrangement which
was a result of ”pakiusap”.

LABOR ARBITER & NLRC RULING:


The NLRC affirmed the LA ruling that Art. 280 is not controlling, giving due credence to the conclusion that
complainants were hired as an accommodation to a “pakiusap” and whose employments were covered by the
Special Employment Contracts with special provisions on duration of contract as specified under Art. 80. Hence,
as correctly held by the Labor Arbiter a quo, the terms of the contract shall be the law between the parties.
The NLRC also declared that the Magna Carta for Disabled Persons was not applicable, considering the
prevailing circumstances/milieu of the case.

ISSUE/S:
1.Whether the Commission committed grave abuse of discretion in holding that the petitioners - money sorters
and counters working in a bank - were not regular employees.
2.Whether the Commission committed grave abuse of discretion in not applying the provisions of the Magna Carta
for the Disabled (RA 7277), on proscription against discrimination against disabled persons.

Angeles| Bajana | Balladares | Brillantes | Briones | Cabansag | Callanta | Chua | David|

De Leon | Gomez | Lopez | Macalino | Nostratis | Padilla | Reynon | Santos | Tan |Velasco
4E / 4F - 2018-2019
Page 51 of 920
LABOR REVIEW DIGEST
Atty. Joyrich Golangco

HELD:
Respondent bank entered into the aforesaid contract with a total of 56 handicapped workers and renewed
the contracts of 37 of them. The renewal of the contracts of the handicapped workers and the hiring of others lead
to the conclusion that their tasks were beneficial and necessary to the bank. More important, these facts show that
they were qualified to perform the responsibilities of their positions. In other words, their disability did not render
them unqualified or unfit for the tasks assigned to them.
The Magna Carta for Disabled Persons mandates that a qualified disabled employee should be given the
same terms and conditions of employment as a qualified able-bodied person.
The fact that the employees were qualified disabled persons removes the employment contracts from the
ambit of Article 80 and since the Magna Carta accords them the rights of qualified able-bodied persons, they are
thus covered by Article 280 of the Labor Code, which provides:
ART. 280. Regular and Casual Employment. -- The provisions of written agreement to the contrary
notwithstanding and regardless of the oral agreement of the parties, an employment shall be deemed to be
regular where the employee has been engaged to perform activities which are usually necessary or
desirable in the usual business or trade of the employer, except where the employment has been fixed for a
specific project or undertaking the completion or termination of which has been determined at the time of the
engagement of the employee or where the work or services to be performed is seasonal in nature and the
employment is for the duration of the season.
An employment shall be deemed to be casual if it is not covered by the preceding paragraph: Provided, That, any
employee who has rendered at least one year of service, whether such service is continuous or broken, shall be
considered as regular employee with respect to the activity in which he is employed and his employment shall
continue while such activity exists.
As regular employees, the 27 petitioners whose contracts were renewed beyond the initial 6 months are entitled to
security of tenure; that is, their services may be terminated only for a just or authorized cause. Because
respondent failed to show such cause, the 27 are deemed illegally dismissed and therefore entitled to back wages
and reinstatement (or in this case separation pay since the position is no longer offered) without loss of seniority
rights and other privileges.

RECALL:
The test of whether an employee is regular was laid down in De Leon v. NLRC, in which this Court held:
The primary standard, therefore, of determining regular employment is the reasonable connection
between the particular activity performed by the employee in relation to the usual trade or business of the
employer. The test is whether the former is usually necessary or desirable in the usual business or trade of
the employer. The connection can be determined by considering the nature of the work performed and its
relation to the scheme of the particular business or trade in its entirety. Also if the employee has been
performing the job for at least one year, even if the performance is not continuous and merely intermittent,
the law deems repeated and continuing need for its performance as sufficient evidence of the necessity if not
indispensability of that activity to the business. Hence, the employment is considered regular, but only with
respect to such activity, and while such activity exists.
-----------------------------------------------------------------------------

Article 82 – 94

Employer-Employee Relationship

1. Francisco vs NLRC
G.R. No. 170087; August 31, 2006

PONENTE: Ynares-Santiago, J:

Angeles| Bajana | Balladares | Brillantes | Briones | Cabansag | Callanta | Chua | David|

De Leon | Gomez | Lopez | Macalino | Nostratis | Padilla | Reynon | Santos | Tan |Velasco
4E / 4F - 2018-2019
Page 52 of 920
LABOR REVIEW DIGEST
Atty. Joyrich Golangco

DOCTRINE: The better approach to determine the existence of a ER-EE relationship would be to adopt a two-
tiered test involving: (1) the putative employers power to control the employee with respect to the means and
methods by which the work is to be accomplished; and (2) the underlying economic realities of the activity or
relationship.

FACTS: Kasei Corporation, private respondent herein, hired petitioner Angelina Francisco during its incorporation
stage as an Accountant, Corporate Secretary and its Liaison Officer. A year after, petitioner was designated
Acting Manager, and performed her duty as such for five (5) years.

Thereafter, petitioner was replaced but was assured that she was still connected with Kasei Corporation as
Technical Assistant to the corporation’s Technical Consultant.

Thereafter, Kasei Corporation reduced her salary by P2,500.00 a month was not paid of her mid-year bonus.
When she claimed for such she was informed that she is no longer connected with the company.

Petitioner filed a case for constructive dismissal against respondent before the Labor Arbiter.

As a defense, Kasei Corporation averred that petitioner is not its employee. That she is technical consultant,
performing her work at her own discretion without control and supervision of Kasei Corporation. The company
never interfered with her work except that from time to time, the management would ask her opinion on matters
relating to her profession. Petitioner did not go through the usual procedure of selection of employees, and that
the money received by petitioner from the corporation was her professional fee.

LA and NLRC RULING:

LA found that petitioner was illegally dismissed ordering respondents to reinstate complainant to her former
position without loss of seniority rights and to pay her her money claims.

NLRC affirmed LA decision modifying only the amount that shall be paid to petitioner.

CA Ruling:

Court of Appeals reversed the NLRC decision and denied petitioners motion for reconsideration, hence, the
present recourse.

ISSUE/S:
(1) Whether there was an employer-employee relationship between petitioner and private respondent Kasei
Corporation; and if in the affirmative,

(2) Whether petitioner was illegally dismissed.

HELD:
(1) YES. There was an employer-employee relationship between petitioner and private respondent Kasei
Corporation

The better approach to determine the existence of a ER-EE relationship would be to adopt a two-tiered
test involving: (1) the putative employers power to control the employee with respect to the means and methods
by which the work is to be accomplished; and (2) the underlying economic realities of the activity or relationship.

Angeles| Bajana | Balladares | Brillantes | Briones | Cabansag | Callanta | Chua | David|

De Leon | Gomez | Lopez | Macalino | Nostratis | Padilla | Reynon | Santos | Tan |Velasco
4E / 4F - 2018-2019
Page 53 of 920
LABOR REVIEW DIGEST
Atty. Joyrich Golangco

The control test provides that there is an employer-employee relationship when the person for whom the
services are performed reserves the right to control not only the end achieved but also the manner and means
used to achieve that end.

In the economic activity test, existence of ER-EE relationship depends upon the ff: (1) the extent to which
the services performed are an integral part of the employers business; (2) the extent of the workers investment in
equipment and facilities; (3) the nature and degree of control exercised by the employer; (4) the workers
opportunity for profit and loss; (5) the amount of initiative, skill, judgment or foresight required for the success of
the claimed independent enterprise; (6) the permanency and duration of the relationship between the worker and
the employer; and (7) the degree of dependency of the worker upon the employer for his continued employment in
that line of business.

By applying the control test, there is no doubt that petitioner is an employee of Kasei Corporation
because she was under the direct control and supervision the corporations Head Technical Consultant.

Under the broader economic reality test, the petitioner is an employee of respondent corporation
because she had served the company for six years before her dismissal, receiving check vouchers indicating her
salaries/wages, benefits, 13th month pay, bonuses and allowances, as well as deductions and Social Security
contributions. Thus, petitioner is economically dependent on respondent corporation for her continued
employment in the latters line of business.

(2) YES. Petitioner was illegally dismissed

The corporation constructively dismissed petitioner when it reduced her salary. A diminution of pay is
prejudicial to the employee and amounts to constructive dismissal. Constructive dismissal is an involuntary
resignation resulting in cessation of work resorted to when continued employment becomes impossible,
unreasonable or unlikely; when there is a demotion in rank or a diminution in pay; or when a clear discrimination,
insensibility or disdain by an employer becomes unbearable to an employee.

Other Notes/ SC Pronouncements:

______________________________________________________________________

2. SONZA vs ABS-CBN BROADCASTING CORP.


GR No. 138051; June 10, 2004

PONENTE: Carpio, J.

DOCTRINE: elements of an employee-employer relationship are selection and engagement of the employee, the
payment of wages, the power of dismissal and the employer’s power to control the employee on the means and
methods by which the work is accomplished. The last element, the so-called "control test", is the most important

Angeles| Bajana | Balladares | Brillantes | Briones | Cabansag | Callanta | Chua | David|

De Leon | Gomez | Lopez | Macalino | Nostratis | Padilla | Reynon | Santos | Tan |Velasco
4E / 4F - 2018-2019
Page 54 of 920
LABOR REVIEW DIGEST
Atty. Joyrich Golangco

element. The greater the supervision and control the hirer exercises, the more likely the worker is deemed an
employee.

FACTS:
Respondent ABS-CBN Broadcasting Corporation (ABS-CBN), one of the biggest television and radio networks in
the country, and Jose Jay Sonza (Sonza),a known television and radio personality, through its managing
corporation Mel and Jay Management and Development Corporation (MJMDC), entered into an Agreement
(Agreement) in which the latter agrees to provide his services exclusively to ABS-CBN as talent for radio and
television.

Two years after, Sonza wrote a letter to ABS-CBN’s President informing the latter of his decision to rescind the
agreement and that he is waiving and renouncing recovery of the remaining stipulated amount in the agreement
but reserves the right to seek recovery of the other benefits.

Sonza filed a complaint against ABS-CBN before the Department of Labor and Employment, alleging that ABS-
CBN did not pay his salaries, separation pay, service incentive leave pay, 13 th month pay, signing bonus, travel
allowance and amounts due under the Employees Stock Option Plan (ESOP).

ABS-CBN filed a Motion to Dismiss on the ground that no employer-employee relationship existed between the
parties.

LA/ NLRC Ruling: LA, finding that there is no employer-employee relationship, dismissed the complaint for lack
of jurisdiction.

NLRC affirmed LA decision.

CA Ruling:
CA likewise affirmed NLRC and LA decision.

ISSUE:
Whether or not there is an employer-employee relationship between Sonza and ABS-CBN

HELD:
NO. There is no employer-employee relationship between Sonza and ABS-CBN.

a) As to Selection and Engagement of Employee


The specific selection and hiring of SONZA, because of his unique skills, talent and celebrity status not
possessed by ordinary employees, is a circumstance indicative, but not conclusive, of an independent contractual
relationship.

b) As to Payment of Wages
SC held that all the talent fees and benefits paid to SONZA were the result of negotiations that led to the
Agreement. Likewise, Sonza’s talent fees, amounting to P317,000 monthly in the second and third year, are so
huge and out of the ordinary that they indicate more an independent contractual relationship rather than an
employer-employee relationship. ABS-CBN agreed to pay SONZA such huge talent fees precisely because of
SONZAs unique skills, talent and celebrity status not possessed by ordinary employees. The power to bargain
talent fees way above the salary scales of ordinary employees is a circumstance indicative, but not conclusive, of
an independent contractual relationship.

Angeles| Bajana | Balladares | Brillantes | Briones | Cabansag | Callanta | Chua | David|

De Leon | Gomez | Lopez | Macalino | Nostratis | Padilla | Reynon | Santos | Tan |Velasco
4E / 4F - 2018-2019
Page 55 of 920
LABOR REVIEW DIGEST
Atty. Joyrich Golangco

c) As to Power of Dismissal
SONZA failed to show that ABS-CBN could terminate his services on grounds other than breach of contract,
such as retrenchment to prevent losses as provided under labor laws. During the life of the Agreement, even if it
suffered severe business losses, ABS-CBN could not retrench SONZA because ABS-CBN remained obligated to
pay SONZAs talent fees during the life of the Agreement. This circumstance indicates an independent contractual
relationship between SONZA and ABS-CBN.

d) Power of Control
The control test is the most important test our courts apply in distinguishing an employee from an
independent contractor. The greater the supervision and control the hirer exercises, the more likely the worker is
deemed an employee.
To perform his work, SONZA only needed his skills and talent. How SONZA delivered his lines, appeared on
television, and sounded on radio were outside ABS-CBNs control. SONZA did not have to render eight hours of
work per day. ABS-CBN could not dictate the contents of SONZAs script. However, the Agreement prohibited
SONZA from criticizing in his shows ABS-CBN or its interests. The clear implication is that SONZA had a free
hand on what to say or discuss in his shows provided he did not attack ABS-CBN or its interests.
ABS-CBN merely reserved the right to modify the program format and airtime schedule for more effective
programming. ABS-CBN did not exercise control over the means and methods of performance of SONZAs work.

Other Notes/ SC Pronouncements:

______________________________________________________________________

3. Javier vs. Fly Ace Corp.


G.R. No. 192558; February 15, 2012
Mendoza, J.

FACTS:
On May 23, 2008, Javier filed a complaint before the NLRC for underpayment of salaries and other labor standard
benefits. He alleged that he was an employee of Fly Ace since September 2007, performing various tasks at the
respondents warehouse such as cleaning and arranging the canned items before their delivery to certain
Angeles| Bajana | Balladares | Brillantes | Briones | Cabansag | Callanta | Chua | David|

De Leon | Gomez | Lopez | Macalino | Nostratis | Padilla | Reynon | Santos | Tan |Velasco
4E / 4F - 2018-2019
Page 56 of 920
LABOR REVIEW DIGEST
Atty. Joyrich Golangco

locations, except in instances when he would be ordered to accompany the companys delivery vehicles, as
pahinante; that he reported for work from Monday to Saturday from 7:00 oclock in the morning to 5:00 oclock in
the afternoon; that during his employment, he was not issued an identification card and payslips by the
company;that on May 6, 2008, he reported for work but he was no longer allowed to enter the company premises
by the security guard upon the instruction of Ruben Ong (Mr. Ong), his superior; that after several minutes of
begging to the guard to allow him to enter, he saw Ong whom he approached and asked why he was being barred
from entering the premises; that Ong replied by saying, Tanungin mo anak mo; that he then went home and
discussed the matter with his family; that he discovered that Ong had been courting his daughter Annalyn; that
Annalyn tried to talk to Ong and convince him to spare her father from trouble but he refused to accede; that
thereafter, Javier was terminated from his employment without notice; and that he was neither given the
opportunity to refute the cause/s of his dismissal from work.

For its part, Fly Ace averred that it was engaged in the business of importation and sales of groceries. Sometime
in December 2007, Javier was contracted by its employee, Mr. Ong, as extra helper on a pakyaw basis. Mr. Ong
contracted Javier roughly 5 to 6 times only in a month whenever the vehicle of its contracted hauler, Milmar
Hauling Services, was not available. On April 30, 2008, Fly Ace no longer needed the services of Javier. Denying
that he was their employee, Fly Ace insisted that there was no illegal dismissal. Fly Ace submitted a copy of its
agreement with Milmar Hauling Services and copies of acknowledgment receipts evidencing payment to Javier for
his contracted services bearing the words, daily manpower (pakyaw/piece rate pay) and the latters
signatures/initials.

LA dismissed the complaint for lack of merit on the ground that Javier failed to present proof that he was a regular
employee of Fly Ace. Complainant has no employee ID showing his employment with the Respondent nor any
document showing that he received the benefits accorded to regular employees of the Respondents. As to the
claim for underpayment of salaries, the payroll presented by the Respondents showing salaries of workers on
pakiao basis has evidentiary weight because although the signature of the complainant appearing thereon are not
uniform, they appeared to be his true signature.

On appeal with the NLRC, Javier was favored. It ruled that a pakyaw-basis arrangement did not preclude the
existence of employer-employee relationship. Payment by result is a method of compensation and does not define
the essence of the relation. It is a mere method of computing compensation, not a basis for determining the
existence or absence of an employer-employee relationship. Finding Javier to be a regular employee, the NLRC
ruled that he was entitled to a security of tenure.

CA annulled the NLRC findings that Javier was indeed a former employee of Fly Ace and reinstated the dismissal
of Javiers complaint. He contracted work outside the company premises; he was not required to observe definite
hours of work; he was not required to report daily; and he was free to accept other work elsewhere as there was
no exclusivity of his contracted service to the company, the same being co-terminous with the trip only. Since no
substantial evidence was presented to establish an employer-employee relationship, the case for illegal dismissal
could not prosper. The CA likewise added that Javiers failure to present salary vouchers, payslips, or other pieces
of evidence to bolster his contention, pointed to the inescapable conclusion that he was not an employee of Fly
Ace.

ISSUE: w/n Javier is deemed a regular employee

HELD:
NO. Whoever claims entitlement to the benefits provided by law should establish his or her right thereto. Sadly,
Javier failed to adduce substantial evidence as basis for the grant of relief.

Angeles| Bajana | Balladares | Brillantes | Briones | Cabansag | Callanta | Chua | David|

De Leon | Gomez | Lopez | Macalino | Nostratis | Padilla | Reynon | Santos | Tan |Velasco
4E / 4F - 2018-2019
Page 57 of 920
LABOR REVIEW DIGEST
Atty. Joyrich Golangco

In this case, the LA and the CA both concluded that Javier failed to establish his employment with Fly Ace. By way
of evidence on this point, all that Javier presented were his self-serving statements purportedly showing his
activities as an employee of Fly Ace. Clearly, Javier failed to pass the substantiality requirement to support his
claim.

The lone affidavit executed by one Bengie Valenzuela was unsuccessful in strengthening Javiers cause. In said
document, all Valenzuela attested to was that he would frequently see Javier at the workplace where the latter
was also hired as stevedore. Certainly, in gauging the evidence presented by Javier, the Court cannot ignore the
inescapable conclusion that his mere presence at the workplace falls short in proving employment therein. The
supporting affidavit could have, to an extent, bolstered Javiers claim of being tasked to clean grocery items when
there were no scheduled delivery trips, but no information was offered in this subject simply because the witness
had no personal knowledge of Javiers employment status in the company.

The Court is of the considerable view that on Javier lies the burden to pass the well-settled tests to determine the
existence of an employer-employee relationship, viz: (1) the selection and engagement of the employee; (2) the
payment of wages; (3) the power of dismissal; and (4) the power to control the employees conduct.

In this case, Javier was not able to persuade the Court that the above elements exist in his case. He could not
submit competent proof that Fly Ace engaged his services as a regular employee; that Fly Ace paid his wages as
an employee, or that Fly Ace could dictate what his conduct should be while at work.

Fly Ace does not dispute having contracted Javier and paid him on a per trip rate as a stevedore, albeit on a
pakyaw basis. The Court cannot fail to note that Fly Ace presented documentary proof that Javier was indeed paid
on a pakyaw basis per the acknowledgment receipts admitted as competent evidence by the LA. Unfortunately for
Javier, his mere denial of the signatures affixed therein cannot automatically sway us to ignore the documents
because forgery cannot be presumed and must be proved by clear, positive and convincing evidence and the
burden of proof lies on the party alleging forgery.

Other Notes/ SC Pronouncements:

______________________________________________________________________

4. San Miguel Corp. Employees Union vs. Bersamira


G.R. No. 87700 June 13, 1990
Melencio-Herrera, J.

FACTS:
Sometime in 1983 and 1984, SanMig entered into contracts for merchandising services with Lipercon and D'Rite.
These companies are independent contractors. In said contracts, it was expressly understood and agreed that the
workers employed by the contractors were to be paid by the latter and that none of them were to be deemed
employees or agents of SanMig. There was to be no employer-employee relation between the contractors and/or
its workers, on the one hand, and SanMig on the other.

Petitioner San Miguel Corporation Employees Union-PTWGO (the Union) is the duly authorized representative of
the monthly paid rank-and-file employees of SanMig with whom the latter executed a CBA. Section 1 of their CBA
specifically provides that "temporary, probationary, or contract employees and workers are excluded from the
bargaining unit and, therefore, outside the scope of this Agreement."

Angeles| Bajana | Balladares | Brillantes | Briones | Cabansag | Callanta | Chua | David|

De Leon | Gomez | Lopez | Macalino | Nostratis | Padilla | Reynon | Santos | Tan |Velasco
4E / 4F - 2018-2019
Page 58 of 920
LABOR REVIEW DIGEST
Atty. Joyrich Golangco

In a letter, the Union advised SanMig that some Lipercon and D'Rite workers had signed up for union membership
and sought the regularization of their employment with SMC. The Union alleged that this group of employees,
while appearing to be contractual workers supposedly independent contractors, have been continuously working
for SanMig for a period ranging from six (6) months to fifteen (15) years and that their work is neither casual nor
seasonal as they are performing work or activities necessary or desirable in the usual business or trade of
SanMig. Thus, it was contended that there exists a "labor-only" contracting situation. It was then demanded that
the employment status of these workers be regularized.

On the ground that it had failed to receive any favorable response from SanMig, the Union filed a notice of strike
for unfair labor practice, CBA violations, and union busting.

SMC filed a verified Complaint for Injunction and Damages before respondent Court to enjoin the Union from
representing and/or acting for and in behalf of the employees of LIPERCON and/or D'RITE for the purposes of
collective bargaining.

The Union filed a Motion to Dismiss SanMig's Complaint on the ground of lack of jurisdiction over the case/nature
of the action. The Court held that the absence of employer-employee relationship negates the existence of labor
dispute. Verily, this court has jurisdiction to take cognizance of plaintiff's grievance.

Petitioners take the position that 'it is beyond dispute that the controversy in the court a quo involves or arose out
of a labor dispute and is directly connected or interwoven with the cases pending with the NCMB-DOLE. On the
other hand, SanMig denies the existence of any employer-employee relationship and consequently of any labor
dispute between itself and the Union.

Anchored on grave abuse of discretion, petitioners are now before us seeking nullification of the challenged Writ.

ISSUE: whether or not respondent Court correctly assumed jurisdiction over the present controversy

HELD:
NO. While it is SanMig's submission that no employer-employee relationship exists between itself, on the one
hand, and the contractual workers of Lipercon and D'Rite on the other, a labor dispute can nevertheless exist
regardless of whether the disputants stand in the proximate relationship of employer and employee.

That a labor dispute, as defined by the law, does exist herein is evident. At bottom, what the Union seeks is to
regularize the status of the employees contracted by Lipercon and D'Rite in effect, that they be absorbed into the
working unit of SanMig. This matter definitely dwells on the working relationship between said employees vis-a-vis
SanMig. Terms, tenure and conditions of their employment and the arrangement of those terms are thus involved
bringing the matter within the purview of a labor dispute. Further, the Union also seeks to represent those
workers, who have signed up for Union membership, for the purpose of collective bargaining.

Whether or not the Union demands are valid; whether or not SanMig's contracts with Lipercon and D'Rite
constitute "labor-only" contracting and, therefore, a regular employer-employee relationship may, in fact, be said
to exist; whether or not the Union can lawfully represent the workers of Lipercon and D'Rite in their demands
against SanMig in the light of the existing CBA — those are issues the resolution of which call for the application
of labor laws, and SanMig's cause's of action in the Court below are inextricably linked with those issues.

Other Notes/ SC Pronouncements:

Angeles| Bajana | Balladares | Brillantes | Briones | Cabansag | Callanta | Chua | David|

De Leon | Gomez | Lopez | Macalino | Nostratis | Padilla | Reynon | Santos | Tan |Velasco
4E / 4F - 2018-2019
Page 59 of 920
LABOR REVIEW DIGEST
Atty. Joyrich Golangco

______________________________________________________________________

5. Locsin vs PLDT
GR No: 185251 Date: October 2, 2009
Ponente: Velasco, Jr., J

Doctrine: “..power of control has been explained as the "right to control not only the end to be achieved but also
the means to be used in reaching such end". With the conclusion that respondent directed petitioners to remain at
their posts and continue with their duties, it is clear that respondent exercised the power of control over them;
thus, the existence of an employer-employee relationship.”

FACTS:

On November 1, 1990, respondent Philippine Long Distance Telephone Company (PLDT) and the Security and
Safety Corporation of the Philippines (SSCP) entered into a Security Services Agreement (Agreement) whereby
SSCP would provide armed security guards to PLDT to be assigned to its various offices. Pursuant to such
agreement, petitioners Raul Locsin and Eddie Tomaquin, among other security guards, were posted at a PLDT
office.

On August 30, 2001, respondent issued a Letter dated August 30, 2001 terminating the Agreement effective
Angeles| Bajana | Balladares | Brillantes | Briones | Cabansag | Callanta | Chua | David|

De Leon | Gomez | Lopez | Macalino | Nostratis | Padilla | Reynon | Santos | Tan |Velasco
4E / 4F - 2018-2019
Page 60 of 920
LABOR REVIEW DIGEST
Atty. Joyrich Golangco

October 1, 2001. Despite the termination of the Agreement, however, petitioners continued to secure the
premises of their assigned office. They were allegedly directed to remain at their post by representatives of
respondent. In support of their contention, petitioners provided the Labor Arbiter with copies of petitioner Locsin’s
pay slips for the period of January to September 2002.

Then, on September 30, 2002, petitioners’ services were terminated. Thus, petitioners filed a complaint before the
Labor Arbiter for illegal dismissal and recovery of money claims such as overtime pay, holiday pay, premium pay
for holiday and rest day, service incentive leave pay, Emergency Cost of Living Allowance, and moral and
exemplary damages against PLDT.

LA/RTC/NLRC RULING:

The Labor Arbiter rendered a Decision finding PLDT liable for illegal dismissal. It was explained in the Decision
that petitioners were found to be employees of PLDT and not of SSCP. Such conclusion was arrived at with the
factual finding that petitioners continued to serve as guards of PLDT’s offices. As such employees, petitioners
were entitled to substantive and procedural due process before termination of employment. The Labor Arbiter held
that respondent failed to observe such due process requirements.

PLDT appealed the above Decision to the NLRC which rendered a Resolution affirming in toto the Arbiters
Decision.

Thus, PDLT filed a Motion for Reconsideration of the NLRCs Resolution which was also denied.

CA RULING:

CA rendered the assailed decision granting PLDTs petition and dismissing petitioners complaint.

The CA applied the four-fold test in order to determine the existence of an employer-employee relationship
between the parties but did not find such relationship. It determined that SSCP was not a labor-only contractor
and was an independent contractor having substantial capital to operate and conduct its own business. The CA
further bolstered its decision by citing the Agreement whereby it was stipulated that there shall be no employer-
employee relationship between the security guards and PLDT.

APPEAL TO THE SC:

Petitioner's Contention:

Petitioners who are under SSCP (even after the termination of the agreement) followed the orders of PLDT to
continue guarding its premises, filed a complaint before the Labor Arbiter for illegal dismissal and recovery of
money claims such as overtime pay, holiday pay, premium pay for holiday and rest day, service incentive leave
pay, Emergency Cost of Living Allowance, and moral and exemplary damages against PLDT.||

Respondent's Contention:

That there exist no ER-EE relationship since the agreement between SSCP and PLDT was terminated, that the
petitioners are employees of SSCP and not PLDT.

ISSUE/S:
Is there employer-employee relationship?
Angeles| Bajana | Balladares | Brillantes | Briones | Cabansag | Callanta | Chua | David|

De Leon | Gomez | Lopez | Macalino | Nostratis | Padilla | Reynon | Santos | Tan |Velasco
4E / 4F - 2018-2019
Page 61 of 920
LABOR REVIEW DIGEST
Atty. Joyrich Golangco

HELD:

Yes. From the foregoing circumstances, reason dictates that we conclude that petitioners remained at their post
under the instructions of PLDT. We can further conclude that PLDT dictated upon petitioners that the latter
perform their regular duties to secure the premises during operating hours. This, to our mind and under the
circumstances, is sufficient to establish the existence of an employer-employee relationship.

To reiterate, while PLDT and SSCP no longer had any legal relationship with the termination of the Agreement,
petitioners remained at their post securing the premises of PLDT while receiving their salaries, allegedly from
SSCP. Clearly, such a situation makes no sense, and the denials proffered by PLDT do not shed any light to the
situation. It is but reasonable to conclude that, with the behest and, presumably, directive of PLDT, petitioners
continued with their services. Evidently, such are indicia of control that respondent exercised over petitioners.

Evidently, PLDT having the power of control over petitioners must be considered as PLDT’s employer––from the
termination of the Agreement onwards––as this was the only time that any evidence of control was exhibited by
PLDT over petitioners and in light of our ruling in Abella. Thus, as aptly declared by the NLRC, petitioners were
entitled to the rights and benefits of employees of PLDT, including due process requirements in the termination of
their services.

Both the Labor Arbiter and NLRC found that PLDT did not observe such due process requirements. Having failed
to do so, PLDT is guilty of illegal dismissal.

Other Notes/ SC Pronouncements:


______________________________________________________________________

6. People’s Broadcasting Service (Bombo Radyo) vs Sec. of Labor


GR No: 179652 Date: October 6, 2012
Ponente: Velasco, Jr., J

Doctrine: the DOLE, in determining the existence of an employer-employee relationship, has a ready set of
guidelines to follow, the same guide the courts themselves use. The elements to determine the existence of an
employment relationship are:
(1) the selection and engagement of the employee;
(2) the payment of wages;
(3) the power of dismissal;
(4) the employer's power to control the employee's conduct.
The use of this test is not solely limited to the NLRC. The DOLE Secretary, or his or her representatives, can
utilize the same test, even in the course of inspection, making use of the same evidence that would have been
presented before the NLRC.

(this case primarily tackles Article 128)


Under Art. 128(b) of the Labor Code, as amended by RA 7730, the DOLE is fully empowered to make a
determination as to the existence of an employer-employee relationship in the exercise of its visitorial and
enforcement power, subject to judicial review, not review by the NLRC.

DOLE has the full power to determine the existence of an employer-employee relationship in cases brought to it
under Article 128 (b) of the Labor Code.This power is parallel and not subordinate to that of the NLRC.|

FACTS:
Angeles| Bajana | Balladares | Brillantes | Briones | Cabansag | Callanta | Chua | David|

De Leon | Gomez | Lopez | Macalino | Nostratis | Padilla | Reynon | Santos | Tan |Velasco
4E / 4F - 2018-2019
Page 62 of 920
LABOR REVIEW DIGEST
Atty. Joyrich Golangco

Private respondent Jandeleon Juezan filed a complaint against petitioner with the Department of Labor and
Employment (DOLE) Regional Office No. VII, Cebu City, for illegal deduction, non- payment of service incentive
leave, 13th month pay, premium pay for holiday and rest day and illegal diminution of benefits, delayed payment
of wages and non coverage of SSS, PAG-IBIG and Phil-health. (This is the only facts provided by this 2012
case, its background is in its 2009 case)

(background of 2009 case)


The case arose when the DOLE Regional Office No. VII conducted an inspection of Bombo Radyo's premises in
response to Juezan's money claims against the broadcasting company, resulting in an order for Bombo Radyo to
rectify/restitute the labor standards violations discovered during the inspection. Bombo Radyo failed to make any
rectification or restitution, prompting the DOLE to conduct a summary investigation. Bombo Radyo reiterated its
position, made during the inspection, that Juezan was not its employee. Both parties submitted evidence to
support their respective positions.
DOLE Director Rodolfo M. Sabulao found Juezan to be an employee of Bombo Radyo. Consequently, Director
Sabulao ordered Bombo Radyo to pay Juezan P203,726.30 representing his demanded money claims. Bombo
Radyo moved for reconsideration and submitted additional evidence, but Director Sabulao denied the motion.
Bombo Radyo then appealed to the DOLE Secretary, insisting that Juezan was not its employee as he was a
drama talent hired on a per drama basis. The Acting DOLE Secretary dismissed the appeal for non-perfection due
to Bombo Radyo's failure to put a cash or surety bond, as required by Article 128 (b) of the Labor Code.

Bombo Radyo went to the Court of Appeals (CA) through a petition for certiorari under Rule 65 of the Rules of
Court. The CA dismissed the petition for lack of merit. Bombo Radyo then sought relief from this Court, likewise
through a Rule 65 petition, contending that the CA committed grave abuse of discretion in dismissing the petition.
It justified its recourse to a petition for certiorari instead of a Rule 45 appeal by claiming that there was no appeal
or any plain and adequate remedy available to it in the ordinary course of law.
On May 8, 2009, the Court's Second Division rendered a Decision reversing the CA rulings and dismissing
Juezan's complaint. It reviewed the evidence and found that there was no employer-employee relationship
between Juezan and Bombo Radyo. The Court overruled the CA's recognition of the DOLE's power to
determine the existence of employer-employee relationship in a labor standards case under Article 128 (b)
of the Labor Code. It stressed that the power to determine the existence of employer-employee relationship is
primarily lodged with the National Labor Relations Commission (NLRC) based on the clause "in cases where the
relationship of employer-employee still exists" in Article 128 (b). cSCADE

LA/RTC/NLRC RULING:
The DOLE Regional Director found that private respondent was an employee of petitioner, and was entitled to his
money claims. The Acting DOLE Secretary dismissed petitioner’s appeal on technical grounds.

CA RULING:
The CA held that DOLE Secretary had jurisdiction over the matter, as the jurisdictional limitation imposed by Article 129 of
the Labor Code on the power of the DOLE Secretary un-der Art. 128(b) had been repealed by R.A. 7730.

APPEAL TO THE SC:


The Supreme Court reversed and set aside the decision of CA, and dismissed the complaint against petitioner.
The Court found that there was no employer-employee relationship between petitioner and private respondent. It was held
that while the DOLE may make a determination of the existence of an employer-employee relationship, this function could not be co-
extensive with the visitorial and enforcement power provided in Art. 128(b) of the Labor Code, as amended by RA 7730.
The NLRC is the primary agency in determining the existence of an employer-employee relationship.

Angeles| Bajana | Balladares | Brillantes | Briones | Cabansag | Callanta | Chua | David|

De Leon | Gomez | Lopez | Macalino | Nostratis | Padilla | Reynon | Santos | Tan |Velasco
4E / 4F - 2018-2019
Page 63 of 920
LABOR REVIEW DIGEST
Atty. Joyrich Golangco

The Public Attorney’s Office sought clarification as to when the visitorial and enforcement power of the DOLE
be not considered as co-extensive with the power to determine the existence of an employer-employee relationship. The
DOLE sought clarification as well, as to the extent of its visitorial and enforcement power under the Labor Code, as amended.

ISSUE/S: Whether DOLE can make a determination of the existence of employer-employee relationship.

HELD:
Yes. No limitation in the law was placed upon the power of the DOLE to determine the existence of an employer-
employee relationship. No procedure was laid down where the DOLE would only make a preliminary finding, that
the power was primarily held by the NLRC. The law did not say that the DOLE would first seek the NLRC’s
determination of the existence of an employer-employee relationship, or that should the existence of the
employer-employee relationship be disputed, the DOLE would refer the matter to the NLRC. The DOLE must
have the power to determine whether or not an employer-employee relationship exists, and from there to decide
whether or not to issue compliance orders in accordance with Art. 128(b) of the Labor Code, as amended by RA
7730.
The determination of the existence of an employer-employee relationship by the DOLE must be respected. The
expanded visitorial and enforcement power of the DOLE granted by RA 7730 would be rendered nugatory if the
alleged employer could, by the simple expedient of disputing the employer-employee relationship, force the
referral of the matter to the NLRC. If the DOLE makes a finding that there is an existing employer-employee
relationship, it takes cognizance of the matter, to the exclusion of the NLRC. The DOLE would have no jurisdiction
only if the employer-employee relationship has already been terminated, or it appears, upon review, that no
employer-employee relationship existed in the first place.
If a complaint is brought before the DOLE to give effect to the labor standards provisions of the Labor Code or
other labor legislation, and there is a finding by the DOLE that there is an existing employer-employee
relationship, the DOLE exercises jurisdiction to the exclusion of the NLRC. If the DOLE finds that there is no
employer-employee relationship, the jurisdiction is properly with the NLRC. If a complaint is filed with the DOLE,
and it is accompanied by a claim for reinstatement, the jurisdiction is properly with the Labor Arbiter, under Art.
217(3) of the Labor Code. If a complaint is filed with the NLRC, and there is still an existing employer-employee
relationship, the jurisdiction is properly with the DOLE. The findings of the DOLE, however, may still be
questioned through a petition for certiorari under Rule 65 of the Rules of Court.

7. Ymbong vs. ABSCBN


GR No: 184885 Date: March 7, 2012
Ponente: Villarama, Jr., J.

Doctrine:
An employee is deemed resigned when he ran for public office.

FACTS:
Petitioner Ernesto G. Ymbong started working for ABS-CBN Broadcasting Corporation (ABS-CBN) in 1993 at its
regional station in Cebu as a television talent, co-anchoring Hoy Gising and TV Patrol Cebu. His stint in ABS-CBN
later extended to radio when ABS-CBN Cebu launched its AM station DYAB in 1995 where he worked as drama
and voice talent, spinner, scriptwriter and public affairs program anchor. Like Ymbong, Leandro Patalinghug also
worked for ABS-CBN Cebu. Starting 1995, he worked as talent, director and scriptwriter for various radio
programs aired over DYAB.
On January 1, 1996, the ABS-CBN Head Office in Manila issued Policy No. HR-ER-016 or the Policy on
Employees Seeking Public Office. The pertinent portions read:
1. Any employee who intends to run for any public office position, must file his/her letter of
resignation, at least thirty (30) days prior to the official filing of the certificate of
candidacy either for national or local election.
Angeles| Bajana | Balladares | Brillantes | Briones | Cabansag | Callanta | Chua | David|

De Leon | Gomez | Lopez | Macalino | Nostratis | Padilla | Reynon | Santos | Tan |Velasco
4E / 4F - 2018-2019
Page 64 of 920
LABOR REVIEW DIGEST
Atty. Joyrich Golangco

xxxx
3. Further, any employee who intends to join a political group/party or even with no political
affiliation but who intends to openly and aggressively campaign for a candidate or
group of candidates (e.g. publicly speaking/endorsing candidate, recruiting campaign
workers, etc.) must file a request for leave of absence subject to managements
approval. For this particular reason, the employee should file the leave request at least
thirty (30) days prior to the start of the planned leave period.
Because of the impending May 1998 elections and based on his immediate recollection of the policy at that time,
Dante Luzon, Assistant Station Manager of DYAB issued the a memorandum stating that any employee/talent
who wants to run for any position in the coming election will have to file a leave of absence the moment he/she
files his/her certificate of candidacy, that the services rendered by the concerned employee/talent to will then be
temporarily suspended for the entire campaign/election period.
Luzon, however, admitted that upon double-checking of the exact text of the policy and subsequent confirmation
with the ABS-CBN Head Office, he saw that the policy actually required suspension for those who intend to
campaign for a political party or candidate and resignation for those who will actually run in the elections. After the
issuance of the March 25, 1998 Memorandum, Ymbong got in touch with Luzon. Luzon claims that Ymbong
approached him and told him that he would leave radio for a couple of months because he will campaign for the
administration ticket. It was only after the elections that they found out that Ymbong actually ran for public office
himself at the eleventh hour. Ymbong, on the other hand, claims that in accordance with the March 25, 1998
Memorandum, he informed Luzon through a letter that he would take a few months leave of absence from March
8, 1998 to May 18, 1998 since he was running for councilor of Lapu-Lapu City.
As regards Patalinghug, Patalinghug approached Luzon and advised him that he will run as councilor for
Naga, Cebu. According to Luzon, he clarified to Patalinghug that he will be considered resigned and not just on
leave once he files a certificate of candidacy. Thus, Patalinghug wrote Luzon a resignation letter.
Unfortunately, both Ymbong and Patalinghug lost in the May 1998 elections.
Later, Ymbong and Patalinghug both tried to come back to ABS-CBN Cebu. According to Luzon, he informed
them that they cannot work there anymore because of company policy. This was stressed even in subsequent
meetings and they were told that the company was not allowing any exceptions. ABS-CBN, however, agreed out
of pure liberality to give them a chance to wind up their participation in the radio drama, Nagbabagang Langit,
since it was rating well and to avoid an abrupt ending. The agreed winding-up, however, dragged on for so long
prompting Luzon to issue to Ymbong a memorandum stating that his services had already been terminated when
he ran for local government position last election.
Ymbong in contrast contended that after the expiration of his leave of absence, he reported back to work as a
regular talent and in fact continued to receive his salary. On September 14, 1998, he received a memorandum
stating that his services are being terminated immediately, much to his surprise. Thus, he filed an illegal
dismissal complaint against ABS-CBN, Luzon and DYAB Station Manager Veneranda Sy. He argued that the
ground cited by ABS-CBN for his dismissal was not among those enumerated in the Labor Code, as
amended. And even granting without admitting the existence of the company policy supposed to have been
violated, Ymbong averred that it was necessary that the company policy meet certain requirements before willful
disobedience of the policy may constitute a just cause for termination. Ymbong further argued that the company
policy violates his constitutional right to suffrage. Patalinghug likewise filed an illegal dismissal
complaint against ABS-CBN.
ABS-CBN prayed for the dismissal of the complaints arguing that there is no employer-employee relationship
between the company and Ymbong and Patalinghug. ABS-CBN contended that they are not employees but talents
as evidenced by their talent contracts. However, notwithstanding their status, ABS-CBN has a standing policy on
persons connected with the company whenever they will run for public office.

LA RULING:
The Labor Arbiter found that there exists an employer-employee relationship between ABS-CBN and Ymbong and
Patalinghug considering the stipulations in their appointment letters/talent contracts. The Labor Arbiter noted
Angeles| Bajana | Balladares | Brillantes | Briones | Cabansag | Callanta | Chua | David|

De Leon | Gomez | Lopez | Macalino | Nostratis | Padilla | Reynon | Santos | Tan |Velasco
4E / 4F - 2018-2019
Page 65 of 920
LABOR REVIEW DIGEST
Atty. Joyrich Golangco

particularly that the appointment letters/talent contracts imposed conditions in the performance of their work,
specifically on attendance and punctuality, which effectively placed them under the control of ABS-CBN. The
Labor Arbiter likewise ruled that although the subject company policy is reasonable and not contrary to law, the
same was not made known to Ymbong and Patalinghug and in fact was superseded by another one embodied in
the March 25, 1998 Memorandum issued by Luzon. Thus, there is no valid or authorized cause in terminating
Ymbong and Patalinghug from their employment.

NLRC RULING
As to the issue of whether they were illegally dismissed, the NLRC treated their cases differently. In the case of
Patalinghug, it found that he voluntarily resigned from employment on April 21, 1998 when he submitted his
resignation letter. The NLRC noted that although the tenor of the resignation letter is somewhat involuntary, he
knew that it is the policy of the company that every person connected therewith should resign from his
employment if he seeks an elected position in the government. As to Ymbong, however, the NLRC ruled
otherwise. It ruled that the March 25, 1998 Memorandum merely states that an employee who seeks any elected
position in the government will only merit the temporary suspension of his services. It held that under the principle
of social justice, the March 25, 1998 Memorandum shall prevail and ABS-CBN is estopped from enforcing the
September 14, 1998 memorandum issued to Ymbong stating that his services had been automatically terminated
when he ran for an elective position.

CA RULING:
CA rendered the assailed decision reversing and setting aside the March 8, 2004 Decision and June 21,
2004 Resolution of the NLRC. The CA declared Ymbong resigned from employment and not to have been illegally
dismissed. The award of full back wages in his favor was deleted accordingly.
The CA ruled that ABS-CBN is estopped from claiming that Ymbong was not its employee after applying the
provisions of Policy No. HR-ER-016 to him. It noted that said policy is entitled Policy on Employees Seeking
Public Office and the guidelines contained therein specifically pertain to employees and did not even mention
talents or independent contractors. It held that it is a complete turnaround on ABS-CBNs part to later argue that
Ymbong is only a radio talent or independent contractor and not its employee. By applying the subject company
policy on Ymbong, ABS-CBN had explicitly recognized him to be an employee and not merely an independent
contractor.
The CA likewise held that the subject company policy is the controlling guideline and therefore, Ymbong should be
considered resigned from ABS-CBN. While Luzon has policy-making power as assistant radio manager, he had
no authority to issue a memorandum that had the effect of repealing or superseding a subsisting policy. Contrary
to the findings of the Labor Arbiter, the subject company policy was effective at that time and continues to be valid
and subsisting up to the present. The CA cited Patalinghugs resignation letter to buttress this conclusion, noting
that Patalinghug openly admitted in his letter that his resignation was in line with the said company policy. Since
ABS-CBN applied Policy No. HR-ER-016 to Patalinghug, there is no reason not to apply the same regulation to
Ymbong who was on a similar situation as the former. Thus, the CA found that the NLRC overstepped its area of
discretion to a point of grave abuse in declaring Ymbong to have been illegally terminated. The CA concluded that
there is no illegal dismissal to speak of in the instant case as Ymbong is considered resigned when he ran for an
elective post pursuant to the subject company policy.

APPEAL TO THE SC:

Petitioner's Contention:
Ymbong contends that the ground relied upon by ABS-CBN is not among the just and authorized causes provided
in the Labor Code, as amended. And even assuming the subject company policy passes the test of validity under
the pretext of the right of the management to discipline and terminate its employees, the exercise of such right is
Angeles| Bajana | Balladares | Brillantes | Briones | Cabansag | Callanta | Chua | David|

De Leon | Gomez | Lopez | Macalino | Nostratis | Padilla | Reynon | Santos | Tan |Velasco
4E / 4F - 2018-2019
Page 66 of 920
LABOR REVIEW DIGEST
Atty. Joyrich Golangco

not without bounds. Ymbong avers that his automatic termination was a blatant disregard of his right to due
process. He was never asked to explain why he did not tender his resignation before he ran for public office as
mandated by the subject company policy.

Respondent's Contention:
ABS-CBN likewise opposes Ymbongs claim that he was terminated. ABS-CBN argues that on the contrary,
Ymbongs unilateral act of filing his certificate of candidacy is an overt act tantamount to voluntary resignation on
his part by virtue of the clear mandate found in Policy No. HR-ER-016. Ymbong, however, failed to file his
resignation and in fact misled his superiors by making them believe that he was going on leave to campaign for
the administration candidates but in fact, he actually ran for councilor. He also claims to have fully
apprised Luzon through a letter of his intention to run for public office, but he failed to adduce a copy of the same.

ISSUES:
(1) whether Policy No. HR-ER-016 is valid;
(2) whether the March 25, 1998 Memorandum issued by Luzon superseded Policy No. HR-ER-016
(3) whether Ymbong, by seeking an elective post, is deemed to have resigned and not dismissed by ABS-CBN.

HELD:
1. Policy No. HR-ER-016 is valid.
ABS-CBN, like Manila Broadcasting Company, had a valid justification for Policy No. HR-ER-016. Its rationale
is embodied in the policy itself, to wit:
Rationale:
ABS-CBN BROADCASTING CORPORATION strongly believes that it is to the best interest of
the company to continuously remain apolitical. While it encourages and supports its
employees to have greater political awareness and for them to exercise their right to
suffrage, the company, however, prefers to remain politically independent and
unattached to any political individual or entity.
Therefore, employees who [intend] to run for public office or accept political appointment
should resign from their positions, in order to protect the company from any public
misconceptions. To preserve its objectivity, neutrality and credibility, the company
reiterates the following policy guidelines for strict implementation.
We have consistently held that so long as a companys management prerogatives are exercised in good faith for
the advancement of the employers interest and not for the purpose of defeating or circumventing the rights of the
employees under special laws or under valid agreements, this Court will uphold them. In the instant case, ABS-
CBN validly justified the implementation of Policy No. HR-ER-016. It is well within its rights to ensure that it
maintains its objectivity and credibility and freeing itself from any appearance of impartiality so that the confidence
of the viewing and listening public in it will not be in any way eroded. Even as the law is solicitous of the welfare of
the employees, it must also protect the right of an employer to exercise what are clearly management
prerogatives. The free will of management to conduct its own business affairs to achieve its purpose cannot be
denied.
It is worth noting that such exercise of management prerogative has earned a stamp of approval from no less than
our Congress itself when on February 12, 2001, it enacted Republic Act No. 9006, otherwise known as the Fair
Election Act. Section 6.6 thereof reads:
6.6. Any mass media columnist, commentator, announcer, reporter, on-air correspondent
or personality who is a candidate for any elective public office or is a campaign volunteer
for or employed or retained in any capacity by any candidate or political party shall be
deemed resigned, if so required by their employer, or shall take a leave of absence from
his/her work as such during the campaign period: Provided, That any media practitioner who is
an official of a political party or a member of the campaign staff of a candidate or political party
shall not use his/her time or space to favor any candidate or political party.
Angeles| Bajana | Balladares | Brillantes | Briones | Cabansag | Callanta | Chua | David|

De Leon | Gomez | Lopez | Macalino | Nostratis | Padilla | Reynon | Santos | Tan |Velasco
4E / 4F - 2018-2019
Page 67 of 920
LABOR REVIEW DIGEST
Atty. Joyrich Golangco

2. Policy No. HR-ER-016 was not superseded by the March 25, 1998 Memorandum
The CA correctly ruled that though Luzon, as Assistant Station Manager for Radio of ABS-CBN, has policy-
making powers in relation to his principal task of administering the networks radio station in the Cebu region, the
exercise of such power should be in accord with the general rules and regulations imposed by the ABS-CBN
Head Office to its employees. Clearly, the March 25, 1998Memorandum issued by Luzon which only requires
employees to go on leave if they intend to run for any elective position is in absolute contradiction with Policy No.
HR-ER-016 issued by the ABS-CBN Head Office in Manila which requires the resignation, not only the filing of a
leave of absence, of any employee who intends to run for public office. Having been issued beyond the scope of
his authority, the March 25, 1998 Memorandum is therefore void and did not supersede Policy No. HR-ER-016.
Also worth noting is that Luzon in his Sworn Statement admitted the inaccuracy of his recollection of the
company policy when he issued the March 25, 1998 Memorandum and stated therein that upon double-checking
of the exact text of the policy statement and subsequent confirmation with the ABS-CBN Head Office in Manila, he
learned that the policy required resignation for those who will actually run in elections because the company
wanted to maintain its independence. Since the officer who himself issued the subject memorandum
acknowledged that it is not in harmony with the Policy issued by the upper management, there is no reason for it
to be a source of right for Ymbong.
3. As Policy No. HR-ER-016 is the subsisting company policy and not Luzons March 25, 1998 Memorandum,
Ymbong is deemed resigned when he ran for councilor.
We find no merit in Ymbongs argument that [his] automatic termination x x x was a blatant [disregard] of [his] right
to due process as he was never asked to explain why he did not tender his resignation before he ran for public
office as mandated by [the subject company policy]. Ymbongs overt act of running for councilor of Lapu-
Lapu City is tantamount to resignation on his part. He was separated from ABS-CBN not because he was
dismissed but because he resigned. Since there was no termination to speak of, the requirement of due process
in dismissal cases cannot be applied to Ymbong. Thus, ABS-CBN is not duty-bound to ask him to explain why he
did not tender his resignation before he ran for public office as mandated by the subject company policy.
In addition, we do not subscribe to Ymbongs claim that he was not in a position to know which of the two
issuances was correct. Ymbong most likely than not, is fully aware that the subsisting policy is Policy No. HR-ER-
016 and not the March 25, 1998 Memorandum and it was for this reason that, as stated by Luzon in his Sworn
Statement, he only told the latter that he will only campaign for the administration ticket and not actually run for an
elective post. Ymbong claims he had fully apprised Luzon by letter of his plan to run and even filed a leave of
absence but records are bereft of any proof of said claim. Ymbong claims that the letter stating his intention to go
on leave to run in the election is attached to his Position Paper as Annex A, a perusal of said pleading attached to
his petition before this Court, however, show that Annex A was not his letter to Luzon but the September 14, 1998
Memorandum informing Ymbong that his services had been automatically terminated when he ran for a local
government position.
Moreover, as pointed out by ABS-CBN, had Ymbong been truthful to his superiors, they would have been able to
clarify to him the prevailing company policy and inform him of the consequences of his decision in case he
decides to run, as Luzon did in Patalinghugs case.

______________________________________________________________________

8. PROFESSIONAL SERVICES, INC vs CA


GR No: 126297, 126467, 127590 Date: February 11, 2008
Ponente: Sandoval Gutierrez, J.

Doctrine:
An employer-employee relationship exists between hospitals and their attending and visiting physicians

FACTS:

Angeles| Bajana | Balladares | Brillantes | Briones | Cabansag | Callanta | Chua | David|

De Leon | Gomez | Lopez | Macalino | Nostratis | Padilla | Reynon | Santos | Tan |Velasco
4E / 4F - 2018-2019
Page 68 of 920
LABOR REVIEW DIGEST
Atty. Joyrich Golangco

Natividad Agana was admitted at the Medical City General Hospital (Medical City) because of difficulty of bowel
movement and bloody anal discharge. Dr. Ampil diagnosed her to be suffering from cancer of the sigmoid. Thus,
Dr. Ampil, assisted by the medical staff of Medical City, performed an anterior resection surgery upon her. During
the surgery, he found that the malignancy in her sigmoid area had spread to her left ovary, necessitating the
removal of certain portions of it. Thus, Dr. Ampil obtained the consent of Atty. Enrique Agana, Natividads
husband, to permit Dr. Juan Fuentes, to perform hysterectomy upon Natividad. Dr. Fuentes performed and
completed the hysterectomy. Afterwards, Dr. Ampil took over, completed the operation and closed the incision.
However, the operation appeared to be flawed. After a couple of days, Natividad complained of excruciating pain
in her anal region. She consulted both Dr. Ampil and Dr. Fuentes about it. They told her that the pain was the
natural consequence of the surgical operation performed upon her. Dr. Ampil recommended that Natividad consult
an oncologist to treat the cancerous nodes which were not removed during the operation. Natividad, accompanied
by her husband, went to the United States to seek further treatment. After four (4) months of consultations and
laboratory examinations, Natividad was told that she was free of cancer. Hence, she was advised to return to
the Philippines. Natividad flew back to the Philippines, still suffering from pains. Two (2) weeks thereafter, her
daughter found a piece of gauze protruding from her vagina. Dr. Ampil was immediately informed. He proceeded
to Natividads house where he managed to extract by hand a piece of gauze measuring 1.5 inches in width. Dr.
Ampil then assured Natividad that the pains would soon vanish. Despite Dr. Ampils assurance, the pains
intensified, prompting Natividad to seek treatment at the Polymedic General Hospital.While confined thereat, Dr.
Ramon Gutierrez detected the presence of a foreign object in her vagina -- a foul-smelling gauze measuring 1.5
inches in width. The gauze had badly infected her vaginal vault. A recto-vaginal fistula had formed in her
reproductive organ which forced stool to excrete through the vagina. Another surgical operation was needed to
remedy the situation. Thus, Natividad underwent another surgery.
Natividad and her husband filed with the Regional Trial Court, Branch 96, Quezon City a complaint for damages
against PSI (owner of Medical City), Dr. Ampil and Dr. Fuentes. Pending the outcome of the above case,
Natividad died. She was duly substituted by her above-named children (the Aganas).
RTC RULING:
in favor of spouses Agana finding PSI, Dr. Ampil and Dr. Fuentes jointly and severally liable

CA RULING:
affirmed the assailed judgment with modification in the sense that the complaint against Dr. Fuentes was
dismissed

APPEAL TO THE SC:

Petitioner's Contention:
PSI contends that the Court erred in finding it liable under Article 2180 of the Civil Code, there being no employer-
employee relationship between it and its consultant, Dr. Ampil. PSI stressed that the Courts Decision
in Ramos holding that an employer-employee relationship in effect exists between hospitals and their attending
and visiting physicians for the purpose of apportioning responsibility had been reversed in a subsequent
Resolution. Further, PSI argues that the doctrine of ostensible agency or agency by estoppel cannot apply
because spouses Agana failed to establish one requisite of the doctrine, i.e., that Natividad relied on the
representation of the hospital in engaging the services of Dr. Ampil. And lastly, PSI maintains that the doctrine of
corporate negligence is misplaced because the proximate cause of Natividads injury was Dr. Ampils negligence.

ISSUE/S:
Angeles| Bajana | Balladares | Brillantes | Briones | Cabansag | Callanta | Chua | David|

De Leon | Gomez | Lopez | Macalino | Nostratis | Padilla | Reynon | Santos | Tan |Velasco
4E / 4F - 2018-2019
Page 69 of 920
LABOR REVIEW DIGEST
Atty. Joyrich Golangco

Whether or not employer-employee relationship exists between Medical City and Dr. Ampil, thus making
the former liable for the latter’s negligence

HELD: YES
Employer-employee relationship in effect exists between the Medical City and Dr. Ampil. Consequently,
both are jointly and severally liable to the Aganas. This ruling proceeds from the following ratiocination in Ramos:
We now discuss the responsibility of the hospital in this particular incident. The
unique practice (among private hospitals) of filling up specialist staff with attending and visiting
consultants, who are allegedly not hospital employees, presents problems in apportioning
responsibility for negligence in medical malpractice cases. However, the difficulty is only
more apparent than real.
In the first place, hospitals exercise significant control in the hiring and firing of
consultants and in the conduct of their work within the hospital premises. Doctors who
apply for consultant slots, visiting or attending, are required to submit proof of completion of
residency, their educational qualifications; generally, evidence of accreditation by the
appropriate board (diplomate), evidence of fellowship in most cases, and references. These
requirements are carefully scrutinized by members of the hospital administration or by a review
committee set up by the hospital who either accept or reject the application. This is particularly
true with respondent hospital.
After a physician is accepted, either as a visiting or attending consultant, he is
normally required to attend clinico-pathological conferences, conduct bedside rounds
for clerks, interns and residents, moderate grand rounds and patient audits and perform
other tasks and responsibilities, for the privilege of being able to maintain a clinic in the
hospital, and/or for the privilege of admitting patients into the hospital. In addition to
these, the physicians performance as a specialist is generally evaluated by a peer
review committee on the basis of mortality and morbidity statistics, and feedback from
patients, nurses, interns and residents. A consultant remiss in his duties, or a
consultant who regularly falls short of the minimum standards acceptable to the
hospital or its peer review committee, is normally politely terminated.
In other words, private hospitals hire, fire and exercise real control over their
attending and visiting consultant staff. While consultants are not, technically employees, a
point which respondent hospital asserts in denying all responsibility for the patients
condition, the control exercised, the hiring, and the right to terminate consultants all
fulfill the important hallmarks of an employer-employee relationship, with the exception
of the payment of wages. In assessing whether such a relationship in fact exists, the
control test is determining. Accordingly, on the basis of the foregoing, we rule that for
the purpose of allocating responsibility in medical negligence cases, an employer-
employee relationship in effect exists between hospitals and their attending and visiting
physicians. This being the case, the question now arises as to whether or not respondent
hospital is solidarily liable with respondent doctors for petitioners condition.
The basis for holding an employer solidarily responsible for the negligence of its
employee is found in Article 2180 of the Civil Code which considers a person accountable not
only for his own acts but also for those of others based on the formers responsibility under a
relationship of partia ptetas.
Clearly, in Ramos, the Court considered the peculiar relationship between a hospital and its consultants
on the bases of certain factors. One such factor is the control test wherein the hospital exercises control in the
hiring and firing of consultants, like Dr. Ampil, and in the conduct of their work.
Other Notes/ SC Pronouncements:

Angeles| Bajana | Balladares | Brillantes | Briones | Cabansag | Callanta | Chua | David|

De Leon | Gomez | Lopez | Macalino | Nostratis | Padilla | Reynon | Santos | Tan |Velasco
4E / 4F - 2018-2019
Page 70 of 920
LABOR REVIEW DIGEST
Atty. Joyrich Golangco

Even assuming that Dr. Ampil is not an employee of Medical City, but an independent contractor, still the
said hospital is liable to the Aganas.
In Nograles, et al. v. Capitol Medical Center, et al., through Mr. Justice Antonio T. Carpio, the Court
held:
The question now is whether CMC is automatically exempt from liability considering
that Dr. Estrada is an independent contractor-physician.
In general, a hospital is not liable for the negligence of an independent contractor-
physician. There is, however, an exception to this principle. The hospital may be liable if the
physician is the ostensible agent of the hospital. (Jones v. Philpott, 702 F. Supp. 1210
[1988])This exception is also known as the doctrine of apparent authority. (Sometimes referred
to as the apparent or ostensible agency theory. [King v. Mitchell, 31 A.D.3rd 958, 819 N.Y. S.2d
169 (2006)].
xxx
The doctrine of apparent authority essentially involves two factors to determine the
liability of an independent contractor-physician.
The first factor focuses on the hospitals manifestations and is sometimes described as
an inquiry whether the hospital acted in a manner which would lead a reasonable person to
conclude that the individual who was alleged to be negligent was an employee or agent of the
hospital. (Diggs v. Novant Health, Inc., 628 S.E.2d 851 (2006) citing Hylton v. Koontz, 138 N.C.
App. 629 (2000). In this regard, the hospital need not make express representations to the
patient that the treating physician is an employee of the hospital; rather a representation
may be general and implied. (Id.)
The doctrine of apparent authority is a specie of the doctrine of estoppel. Article 1431
of the Civil Code provides that [t]hrough estoppel, an admission or representation is rendered
conclusive upon the person making it, and cannot be denied or disproved as against the person
relying thereon. Estoppel rests on this rule: Whether a party has, by his own declaration, act, or
omission, intentionally and deliberately led another to believe a particular thing true, and to act
upon such belief, he cannot, in any litigation arising out of such declaration, act or omission, be
permitted to falsify it. (De Castro v. Ginete, 137 Phil. 453 [1969], citing Sec. 3, par. A, Rule 131
of the Rules of Court. See also King v. Mitchell, 31 A.D.3rd 958, 819 N.Y.S.2d 169 [2006]).
xxx
The second factor focuses on the patients reliance. It is sometimes characterized as
an inquiry on whether the plaintiff acted in reliance upon the conduct of the hospital or its agent,
consistent with ordinary care and prudence. (Diggs v. Novant Health, Inc.)

PSI argues that the doctrine of apparent authority cannot apply to these cases because
spouses Agana failed to establish proof of their reliance on the representation of Medical City that Dr. Ampil is its
employee.
The argument lacks merit.
Atty. Agana categorically testified that one of the reasons why he chose Dr. Ampil was that he knew him
to be a staff member of Medical City, a prominent and known hospital.
Clearly, PSI is estopped from passing the blame solely to Dr. Ampil. Its act of displaying his name and
those of the other physicians in the public directory at the lobby of the hospital amounts to holding out to the
public that it offers quality medical service through the listed physicians. This justifies Atty. Aganas belief that Dr.
Ampil was a member of the hospitals staff. It must be stressed that under the doctrine of apparent authority,
the question in every case is whether the principal has by his voluntary act placed the agent in such a
situation that a person of ordinary prudence, conversant with business usages and the nature of the
particular business, is justified in presuming that such agent has authority to perform the particular act in
question. In these cases, the circumstances yield a positive answer to the question.
Angeles| Bajana | Balladares | Brillantes | Briones | Cabansag | Callanta | Chua | David|

De Leon | Gomez | Lopez | Macalino | Nostratis | Padilla | Reynon | Santos | Tan |Velasco
4E / 4F - 2018-2019
Page 71 of 920
LABOR REVIEW DIGEST
Atty. Joyrich Golangco

The challenged Decision also anchors its ruling on the doctrine of corporate responsibility. The duty
of providing quality medical service is no longer the sole prerogative and responsibility of the physician. This is
because the modern hospital now tends to organize a highly-professional medical staff whose competence and
performance need also to be monitored by the hospital commensurate with its inherent responsibility to provide
quality medical care. Such responsibility includes the proper supervision of the members of its medical
staff. Accordingly, the hospital has the duty to make a reasonable effort to monitor and oversee the
treatment prescribed and administered by the physicians practicing in its premises.
Unfortunately, PSI had been remiss in its duty. It did not conduct an immediate investigation on the
reported missing gauzes to the great prejudice and agony of its patient. Dr. Jocson, a member of PSIs medical
staff, who testified on whether the hospital conducted an investigation, was evasive. His testimony shows Dr.
Jocsons lack of concern for the patients. Such conduct is reflective of the hospitals manner of
supervision. Not only did PSI breach its duty to oversee or supervise all persons who practice medicine
within its walls, it also failed to take an active step in fixing the negligence committed. This renders PSI, not
only vicariously liable for the negligence of Dr. Ampil under Article 2180 of the Civil Code, but also directly
liable for its own negligence under Article 2176.

______________________________________________________________________

9. South East International Rattan, Inc. and/or Estanislao Agbay v. Jesus J. Coming
G.R. No. 186621
March 12, 2014
Villarama, J.

Doctrine:

To ascertain the existence of an employer-employee relationship jurisprudence has invariably adhered to the four-
fold test, to wit: (1) the selection and engagement of the employee; (2) the payment of wages; (3) the power of
dismissal; and (4) the power to control the employee’s conduct, or the so-called "control test."

FACTS:

Petitioner SEIRI is engaged in manufacturing and exporting furniture abroad.

On November 3, 2003, Respondent Coming filed a complaint for illegal dismissal, underpayment of wages, non-
payment of holiday pay, 13th month pay and service incentive leave pay, with prayer for reinstatement, back
wages, damages and attorney’s fees.

Respondent alleged that he was hired by petitioners as Sizing Machine Operator on March 17, 1984. Despite
being an employee for many years, respondent was dismissed on January 1, 2002 without lawful cause. He was
told that he will be terminated because the company is not doing well financially and that he would be called back
to work only if they need his services again. Respondent waited for almost a year but petitioners did not call him
back to work.

Petitioners denied having hired respondent asserting that SEIRI was incorporated only in 1986, and that
respondent actually worked for SEIRI’s furniture suppliers because when the company started in 1987 it was
engaged purely in buying and exporting furniture and its business operations were suspended from the last

Angeles| Bajana | Balladares | Brillantes | Briones | Cabansag | Callanta | Chua | David|

De Leon | Gomez | Lopez | Macalino | Nostratis | Padilla | Reynon | Santos | Tan |Velasco
4E / 4F - 2018-2019
Page 72 of 920
LABOR REVIEW DIGEST
Atty. Joyrich Golangco

quarter of 1989 to August 1992. They stressed that respondent was not included in the list of employees
submitted to the SSS.

Respondent also included an Affidavit executed by five former employees of herein petitioner which supports his
claim that he was an employee of the latter.

LA/RTC/NLRC RULING:
In his Decision dated April 30, 2004, Labor Arbiter Ernesto F. Carreon ruled that respondent is a regular employee
of SEIRI and that the termination of his employment was illegal.

On July 28, 2005, the NLRC’s Fourth Division rendered its Decision setting aside the decision of the Labor Arbiter.
The NLRC likewise denied respondent’s motion for reconsideration.

CA RULING:

By Decision dated February 21, 2008, the CA reversed the NLRC and ruled that there existed an employer-
employee relationship between petitioners and respondent who was dismissed without just and valid cause.

APPEAL TO THE SC:

Petitioner's Contention:

See Facts

Respondent's Contention:

See Facts

ISSUE/S:

1. Whether there exists an ER-EE relationship between the petitioners and respondents.
2. Whether petitioners are liable for illegal dismissal.

HELD:

1. Yes. To ascertain the existence of an employer-employee relationship jurisprudence has invariably adhered to
the four-fold test, to wit: (1) the selection and engagement of the employee; (2) the payment of wages; (3) the
power of dismissal; and (4) the power to control the employee’s conduct, or the so-called "control test."

In Tan v. Lagrama, the Court held that the fact that a worker was not reported as an employee to the SSS is not
conclusive proof of the absence of employer-employee relationship. Otherwise, an employer would be rewarded
for his failure or even neglect to perform his obligation.

Petitioners’ admission that the five affiants were their former employees is binding upon them. While they claim
that respondent was the employee of their suppliers Mayol and Apondar, they did not submit proof that the latter
were indeed independent contractors; clearly, petitioners failed to discharge their burden of proving their own
affirmative allegation. There is thus no showing that the five former employees of SEIRI were motivated by malice,
bad faith or any ill-motive in executing their affidavit supporting the claims of respondent.

Angeles| Bajana | Balladares | Brillantes | Briones | Cabansag | Callanta | Chua | David|

De Leon | Gomez | Lopez | Macalino | Nostratis | Padilla | Reynon | Santos | Tan |Velasco
4E / 4F - 2018-2019
Page 73 of 920
LABOR REVIEW DIGEST
Atty. Joyrich Golangco

2. As a regular employee, respondent enjoys the right to security of tenure under Article 279 of the Labor Code
and may only be dismissed for a just or authorized cause, otherwise the dismissal becomes illegal.

Respondent, whose employment was terminated without valid cause by petitioners, is entitled to reinstatement
without loss of seniority rights and other privileges and to his full back wages, inclusive of allowances and other
benefits or their monetary equivalent, computed from the time his compensation was withheld from him up to the
time of his actual reinstatement. Where reinstatement is no longer viable as an option, back wages shall be
computed from the time of the illegal termination up to the finality of the decision. Separation pay equivalent to
one month salary for every year of service should likewise be awarded as an alternative in case reinstatement in
not possible.

Other Notes/ SC Pronouncements:

______________________________________________________________________

10. Bernard A. Tenazas, et al. v. R. Villegas Taxi Transport and Romualdo Villegas
G.R. No. 192998
April 2, 2014
Reyes J.

Doctrine:
In Macasero v. Southern Industrial Gases Philippines, the Court reiterated, thus:
An illegally dismissed employee is entitled to two reliefs: backwages and reinstatement. The two reliefs provided
are separate and distinct. In instances where reinstatement is no longer feasible because of strained relations
between the employee and the employer, separation pay is granted. In effect, an illegally dismissed employee is
entitled to either reinstatement, if viable, or separation pay if reinstatement is no longer viable, and backwages.
The normal consequences of respondents’ illegal dismissal, then, are reinstatement without loss of seniority
rights, and payment of backwages computed from the time compensation was withheld up to the date of actual
reinstatement. Where reinstatement is no longer viable as an option, separation pay equivalent to one (1) month
salary for every year of service should be awarded as an alternative. The payment of separation pay is in addition
to payment of backwages.

FACTS:

Tenazas and Francisco filed a complaint for illegal dismissal against R. Villegas Taxi Transport and/or Romualdo
and Andy. A similar case was filed by Endraca against the same respondents. The two (2) cases were
subsequently consolidated.

Tenazas alleged that on July 1, 2007, the taxi unit assigned to him was sideswiped by another vehicle, causing a
dent on the left fender near the driver seat. Upon reporting the incident to the company, he was scolded by
respondents Romualdo and Andy and was told to leave the garage for he is already fired. He was even
threatened with physical harm should he ever be seen in the company’s premises again. Despite the warning,
Tenazas reported for work on the following day but was told that he can no longer drive any of the company’s
units as he is already fired.

Francisco, on the other hand, averred that his dismissal was brought about by the company’s unfounded
suspicion that he was organizing a labor union. He was instantaneously terminated, without the benefit of
procedural due process, on June 4, 2007.
Angeles| Bajana | Balladares | Brillantes | Briones | Cabansag | Callanta | Chua | David|

De Leon | Gomez | Lopez | Macalino | Nostratis | Padilla | Reynon | Santos | Tan |Velasco
4E / 4F - 2018-2019
Page 74 of 920
LABOR REVIEW DIGEST
Atty. Joyrich Golangco

Endraca related that before he was dismissed, he brought his taxi unit to an auto shop for an urgent repair. He
was charged the amount of ₱700.00 for the repair services and the replacement parts. As a result, he was not
able to meet his boundary for the day. Upon returning to the company garage and informing the management of
the incident, his driver’s license was confiscated and was told to settle the deficiency in his boundary first before
his license will be returned to him. He was no longer allowed to drive a taxi unit despite his persistent pleas.

Respondents admitted that Tenazas and Endraca were employees of the company, the former being a regular
driver and the latter a spare driver. The respondents, however, denied that Francisco was an employee of the
company.

LA/RTC/NLRC RULING:

The Labor Arbiter (LA) ruled that there was no illegal dismissal in the case at bar.

On June 23, 2009, the NLRC rendered a Decision holding that the evidence submitted by the petitioners sufficed
to establish the existence of employer-employee relationship and their illegal dismissal.

CA RULING:

The CA rendered a Decision, affirming with modification the Decision dated June 23, 2009 of the NLRC. The CA
agreed with the NLRC’s finding that Tenazas and Endraca were employees of the company, but ruled otherwise
in the case of Francisco for failing to establish his relationship with the company.

APPEAL TO THE SC:

Petitioner's Contention:
See Facts
Respondent's Contention:
See Facts

ISSUE/S:
1. Whether there is employer-employee relationship.
2. Whether there was an illegal dismissal.

HELD:
1. As Francisco was claiming to be an employee of the respondents, it is incumbent upon him to proffer evidence
to prove the existence of said relationship.

In this case, however, Francisco failed to present any proof substantial enough to establish his relationship with
the respondents. He failed to present documentary evidence like attendance logbook, payroll, SSS record or any
personnel file that could somehow depict his status as an employee. Anent his claim that he was not issued with
employment records, he could have, at least, produced his social security records which state his contributions,
name and address of his employer, as his co-petitioner Tenazas did. He could have also presented testimonial
evidence showing the respondents’ exercise of control over the means and methods by which he undertakes his
work. This is imperative in light of the respondents’ denial of his employment and the claim of another taxi
operator, Emmanuel Villegas, that he was his employer. Specifically, in his Affidavit, Emmanuel alleged that
Francisco was employed as a spare driver in his taxi garage from January 2006 to December 2006, a fact that the
latter failed to deny or question in any of the pleadings attached to the records of this case. The utter lack of
evidence is fatal to Francisco’s case especially in cases like his present predicament when the law has been very
Angeles| Bajana | Balladares | Brillantes | Briones | Cabansag | Callanta | Chua | David|

De Leon | Gomez | Lopez | Macalino | Nostratis | Padilla | Reynon | Santos | Tan |Velasco
4E / 4F - 2018-2019
Page 75 of 920
LABOR REVIEW DIGEST
Atty. Joyrich Golangco

lenient in not requiring any particular form of evidence or manner of proving the presence of employer-employee
relationship.

2. Yes. The CA’s order of reinstatement of Tenazas and Endraca, instead of the payment of separation pay, is
also well in accordance with prevailing jurisprudence

This doctrine of strained relations, however, should not be used recklessly or applied loosely nor be based on
impression alone. "It bears to stress that reinstatement is the rule and, for the exception of strained relations to
apply, it should be proved that it is likely that if reinstated, an atmosphere of antipathy and antagonism would be
generated as to adversely affect the efficiency and productivity of the employee concerned."

Other Notes/ SC Pronouncements:


_____________________________________________________________________
11. GREGORIO V. TONGKO vs. THE MANUFACTURERS LIFE
INSURANCE CO. (PHILS.), INC.
G.R. No. 167622; November 7, 2008
QUISUMBING, J.

DOCTRINE: The general law on agency, on the other hand, expressly allows the principal an element of control
over the agent in a manner consistent with an agency relationship. In this sense, these control measures cannot
be read as indicative of labor law control. Foremost among these are the directives that the principal may impose
on the agent to achieve the assigned tasks, to the extent that they do not involve the means and manner of
undertaking these tasks. The law likewise obligates the agent to render an account; in this sense, the principal
may impose on the agent specific instructions on how an account shall be made, particularly on the matter of
expenses and reimbursements. To these extents, control can be imposed through rules and regulations without
intruding into the labor law concept of control for purposes of employment.

FACTS:

Manufacturers Life Insurance Co. (Phils.), Inc. (Manulife) is a domestic corporation engaged in life
insurance business. Renato A. Vergel De Dios was, during the period material, its President and Chief Executive
Officer. Gregorio V. Tongko started his professional relationship with Manulife on July 1, 1977 by virtue of a
Career Agents Agreement (Agreement) he executed with Manulife. The agreement provided among others that
the Agent is an independent contractor and nothing contained therein shall be construed or interpreted as creating
an employer-employee relationship between the Company and the Agent. In 1983, Tongko was named as a Unit
Manager in Manulifes Sales Agency Organization. In 1990, he became a Branch Manager.

The problem started sometime in 2001, when Manulife instituted manpower development programs in
the regional sales management level. Relative thereto, De Dios addressed a letter dated November 6, 2001to
Tongko regarding an October 18, 2001 Metro North Sales Managers Meeting. In the letter, De Dios stated that
the changes can end at that point and they need not go any further and he would want everybody on board and
that any resistance or holding back by anyone will be dealt with accordingly. Subsequently, De Dios wrote
Tongko another letter dated December 18, 2001, terminating Tongkos services stating that all the efforts have
failed in helping him align his directions with Managements avowed agency growth policy. And on account
thereof, the Management is exercising its prerogative under Section 14 of his Agents Contract.

Therefrom, Tongko filed a Complaint dated November 25, 2002 with the NLRC against Manulife for
illegal dismissal. In the Complaint, Tongko, in a bid to establish an employer-employee relationship, alleged that
De Dios gave him specific directives on how to manage his area of responsibility in the latters letter
dated November 6, 2001. He further claimed that Manulife exercised control over him in that It was Manulife who
Angeles| Bajana | Balladares | Brillantes | Briones | Cabansag | Callanta | Chua | David|

De Leon | Gomez | Lopez | Macalino | Nostratis | Padilla | Reynon | Santos | Tan |Velasco
4E / 4F - 2018-2019
Page 76 of 920
LABOR REVIEW DIGEST
Atty. Joyrich Golangco

hired, promoted and gave various assignments to him. Manulife prescribed a Code of Conduct which would
govern in minute detail all aspects of the work to be undertaken by employees, including the sales process, the
underwriting process, signatures, handling of money, policyholder service, confidentiality, legal and regulatory
requirements and grounds for termination of employment.

Manulife then filed a Position Paper with Motion to Dismiss dated February 27, 2003, in which it alleged
that Tongko is not its employee, and that it did not exercise control over him. Thus, Manulife claimed that the
NLRC has no jurisdiction over the case.

LA RULING: Labor Arbiter Marita V. Padolina dismissed the complaint for lack of an employer-employee
relationship. Padolina found that applying the four-fold test in determining the existence of an employer-employee
relationship, none was found in the instant case.

NLRC RULING: The NLRCs First Division, while finding an employer-employee relationship between Manulife
and Tongko applying the four-fold test, held Manulife liable for illegal dismissal. It further stated that Manulife
exercised control over Tongko as evidenced by the letter dated November 6, 2001 of De Dios.

CA RULING: CA issued the assailed Decision dated March 29, 2005, finding the absence of an employer-
employee relationship between the parties and deeming the NLRC with no jurisdiction over the case. The CA
arrived at this conclusion while again applying the four-fold test. The CA found that Manulife did not exercise
control over Tongko that would render the latter an employee of Manulife.

ISSUE:
Whether or not there was an employer-employee relationship.
RULING:

YES. Manulife had the power of control over Tongko that would make him its employee. Several factors
contribute to this conclusion. Under the provision in the Agreeent, an agent of Manulife must comply with three (3)
requirements: (1) compliance with the regulations and requirements of the company; (2) maintenance of a level of
knowledge of the companys products that is satisfactory to the company; and (3) compliance with a quota of new
businesses.

Among the company regulations of Manulife are the different codes of conduct such as the Agent Code
of Conduct, Manulife Financial Code of Conduct, and Manulife Financial Code of Conduct Agreement, which
demonstrate the power of control exercised by the company over Tongko. The fact that Tongko was obliged to
obey and comply with the codes of conduct was not disowned by respondents.
Thus, with the company regulations and requirements alone, the fact that Tongko was an employee of
Manulife may already be established. Certainly, these requirements controlled the means and methods by which
Tongko was to achieve the companys goals.

More importantly, Manulifes evidence establishes the fact that Tongko was tasked to perform
administrative duties that establishes his employment with Manulife.

OTHER NOTES:

Manulife failed to overcome the burden of proof in proving that it legally dismissed Tongko. It must be
reiterated that Manulife even failed to identify the specific acts by which Tongkos employment was terminated
much less support the same with substantial evidence. To repeat, mere conjectures cannot work to deprive
employees of their means of livelihood. Thus, it must be concluded that Tongko was illegally dismissed.

Angeles| Bajana | Balladares | Brillantes | Briones | Cabansag | Callanta | Chua | David|

De Leon | Gomez | Lopez | Macalino | Nostratis | Padilla | Reynon | Santos | Tan |Velasco
4E / 4F - 2018-2019
Page 77 of 920
LABOR REVIEW DIGEST
Atty. Joyrich Golangco

_____________________________________________________________________

12. TELEVISION AND PRODUCTION EXPONENTS, INC. (TAPE) vs. ROBERTO C. SERVAA
G.R. No. 167648; January 28, 2008
QUISUMBING, J.,

DOCTRINE: The factors to be considered in determining the existence of employer-employee relationship,


namely: (a) the selection and engagement of the employee; (b) the payment of wages; (c) the power of dismissal;
and (d) the employer's power to control the employee with respect to the means and method by which the work is
to be accomplished. The most important factor involves the control test. Under the control test, there is an
employer-employee relationship when the person for whom the services are performed reserves the right to
control not only the end achieved but also the manner and means used to achieve that end.

FACTS:

TAPE is a domestic corporation engaged in the production of television programs, such as the long-
running variety program, Eat Bulaga!. Its president is Antonio P. Tuviera (Tuviera). Respondent Roberto C.
Servaa had served as a security guard for TAPE from March 1987 until he was terminated on 3 March 2000.

Respondent alleged that he was first connected with Agro-Commercial Security Agency but was later on
absorbed by TAPE as a regular company guard. He was detailed at Broadway Centrum in Quezon City where Eat
Bulaga! regularly staged its productions. On 2 March 2000, respondent received a memorandum informing him of
his impending dismissal on account of TAPEs decision to contract the services of a professional security
agency. At the time of his termination, respondent was receiving a monthly salary of P6,000.00.

Respondent filed a complaint for illegal dismissal and nonpayment of benefits against TAPE before the
NLRC. Respondent insisted that he was a regular employee having been engaged to perform an activity that is
necessary and desirable to TAPEs business for thirteen (13) years. TAPE for its part averred that respondent was
an independent contractor falling under the talent group category and was working under a special arrangement
which is recognized in the industry.

LA RULING: Labor Arbiter Daisy G. Cauton-Barcelona declared respondent to be a regular employee of


TAPE. The Labor Arbiter relied on the nature of the work of respondent, which is securing and maintaining order
in the studio, as necessary and desirable in the usual business activity of TAPE. The Labor Arbiter also ruled that
the termination was valid on the ground of redundancy, and ordered the payment of respondents separation pay
equivalent to one (1)-month pay for every year of service.

NLRC RULING: National Labor Relations Commission (NLRC) reversed the Labor Arbiter and considered
respondent a mere program employee.

CA RULING: Denied the Motion for Reconsideration filed by TAPE.

TAPE asseverates that the Court of Appeals erred in applying the four-fold test in determining the
existence of employer-employee relationship between it and respondent. With respect to the elements of
selection, wages and dismissal, TAPE proffers the following arguments: that it never hired respondent, instead it
was the latter who offered his services as a talent to TAPE; that the Memorandum dated 2 March 2000 served on
respondent was for the discontinuance of the contract for security services and not a termination letter; and that
the talent fees given to respondent were the pre-agreed consideration for the services rendered and should not be
construed as wages. Anent the element of control, TAPE insists that it had no control over respondent in that he
Angeles| Bajana | Balladares | Brillantes | Briones | Cabansag | Callanta | Chua | David|

De Leon | Gomez | Lopez | Macalino | Nostratis | Padilla | Reynon | Santos | Tan |Velasco
4E / 4F - 2018-2019
Page 78 of 920
LABOR REVIEW DIGEST
Atty. Joyrich Golangco

was free to employ means and methods by which he is to control and manage the live audiences, as well as the
safety of TAPEs stars and guests.

ISSUE:
Whether or not there exists an employer-employee relationship between TAPE and the respondent.

RULING:

YES. The Memorandum informing respondent of the discontinuance of his service proves that TAPE had
the power to dismiss respondent. Control is manifested in the bundy cards submitted by respondent in
evidence. He was required to report daily and observe definite work hours. Aside from possessing substantial
capital or investment, a legitimate job contractor or subcontractor carries on a distinct and independent business
and undertakes to perform the job, work or service on its own account and under its own responsibility according
to its own manner and method, and free from the control and direction of the principal in all matters connected
with the performance of the work except as to the results thereof. TAPE failed to establish that respondent is an
independent contractor. More importantly, respondent had been continuously under the employ of TAPE from
1995 until his termination in March 2000, or for a span of 5 years.Regardless of whether or not respondent had
been performing work that is necessary or desirable to the usual business of TAPE, respondent is still considered
a regular employee under Article 280 of the Labor Code

Other Notes/ SC Pronouncements:

_____________________________________________________________________

13. ENCYCLOPAEDIA BRITANNICA (PHILIPPINES), INC. v. NATIONAL LABOR RELATIONS COMMISSION,


HON. LABOR ARBITER TEODORICO L. ROGELIO and BENJAMIN LIMJOCO
Angeles| Bajana | Balladares | Brillantes | Briones | Cabansag | Callanta | Chua | David|

De Leon | Gomez | Lopez | Macalino | Nostratis | Padilla | Reynon | Santos | Tan |Velasco
4E / 4F - 2018-2019
Page 79 of 920
LABOR REVIEW DIGEST
Atty. Joyrich Golangco

GR No: 87098 Date: November 4, 1996


Ponente: Torres, Jr., J.

Doctrine: In determining the existence of an employer-employee relationship the following elements must be
present: 1) selection and engagement of the employee; 2) payment of wages; 3) power of dismissal; and 4) the
power to control the employee's conduct. Of the above, control of employee's conduct is commonly regarded as
the most crucial and determinative indicator of the presence or absence of an employer-employee relationship.
Under the control test, an employer-employee relationship exists where the person for whom the services are
performed reserves the right to control not only the end to be achieved, but also the manner and means to used in
reaching that end.

FACTS:
Private respondent Benjamin Limjoco was a Sales Division Manager of petitioner Encyclopaedia Britannica and
was in charge of selling petitioner's products through some sales representatives. As compensation, private
respondent received commissions from the products sold by his agents. He was also allowed to use petitioner's
name, goodwill and logo. It was, however, agreed upon that office expenses would be deducted from private
respondent's commissions.
On June 14, 1974, private respondent Limjoco resigned from office to pursue his private business. Then on
October 30, 1975, he filed a complaint against petitioner Encyclopaedia Britannica with the Department of Labor
and Employment, claiming for non-payment of separation pay and other benefits, and also illegal deduction from
his sales commissions.

Petitioner Encyclopaedia Britannica alleged that Limjoco was not its employee but an independent dealer
authorized to promote and sell its products and in return, received commissions therefrom. Limjoco did not have
any salary and his income from the petitioner company was dependent on the volume of sales accomplished. He
also had his own separate office, financed the business expenses, and maintained his own workforce. The
salaries of his secretary, utility man, and sales representatives were chargeable to his commissions. Thus,
petitioner argued that it had no control and supervision over the complainant as to the manner and means he
conducted his business operations. The latter did not even report to the office of the petitioner and did not observe
fixed office hours. Consequently, there was no employer-employee relationship.

Limjoco maintained otherwise. He alleged that he was hired by the petitioner in July 1970, was assigned in the
sales department, and was earning an average of P4,000.00 monthly as his sales commission. He was under the
supervision of the petitioner's officials who issued to him and his other personnel, memoranda, guidelines on
company policies, instructions and other orders. He was, however, dismissed by the petitioner when the Laurel-
Langley Agreement expired. As a result thereof, Limjoco asserts that in accordance with the established company
practice and the provisions of the collective bargaining agreement, he was entitled to termination pay equivalent to
one month salary, the unpaid benefits (Christmas bonus, midyear bonus, clothing allowance, vacation leave, and
sick leave), and the amounts illegally deducted from his commissions which were then used for the payments of
office supplies, office space, and overhead expenses.

LA RULING: The Labor Arbiter ruled that Limjoco was under the control of the petitioner since he was required to
make periodic reports of his sales activities to the company and all transactions were subject to the final approval
of the petitioner.

NLRC RULING: The NLRC found no evidence supporting the allegation that Limjoco was an independent
contractor or dealer. The petitioner dictated Limjoco how and where to sell its products. NLRC ruled that there
existed an employer-employee relationship and petitioner failed to disprove this finding.

PETITION TO THE SC:


Angeles| Bajana | Balladares | Brillantes | Briones | Cabansag | Callanta | Chua | David|

De Leon | Gomez | Lopez | Macalino | Nostratis | Padilla | Reynon | Santos | Tan |Velasco
4E / 4F - 2018-2019
Page 80 of 920
LABOR REVIEW DIGEST
Atty. Joyrich Golangco

ISSUE/S:
1. Whether or not there is an Employer-Employee Relationship between Encyclopaedia Britannica and Limjoco?

HELD:
1. None. In determining the existence of an employer-employee relationship the following elements must be
present: 1) selection and engagement of the employee; 2) payment of wages; 3) power of dismissal; and 4)
the power to control the employee's conduct. Of the above, control of employee's conduct is commonly
regarded as the most crucial and determinative indicator of the presence or absence of an employer-
employee relationship. Under the control test, an employer-employee relationship exists where the person for
whom the services are performed reserves the right to control not only the end to be achieved, but also the
manner and means to used in reaching that end. The fact that petitioner issued memoranda to private
respondents and to other division sales managers did not prove that petitioner had actual control over them.
The different memoranda were merely guidelines on company policies which the sales managers follow and
impose on their respective agents. It should be noted that in petitioner's business of selling encyclopedias
and books, the marketing of these products was done through dealership agreements. The sales operations
were primarily conducted by independent authorized agents who did not receive regular compensations but
only commissions based on the sales of the products. These independent agents hired their own sales
representatives, financed their own office expenses, and maintained their own staff. Thus, there was a need
for the petitioner to issue memoranda to private respondent so that the latter would be apprised of the
company policies and procedures. Nevertheless, private respondent Limjoco and the other agents were free
to conduct and promote their sales operations. Private respondent was merely an agent or an independent
dealer of the petitioner. He was free to conduct his work and he was free to engage in other means of
livelihood. At the time he was connected with the petitioner company, private respondent was also a director
and later the president of the Farmers' Rural Bank. Had he been an employee of the company, he could not
be employed elsewhere and he would be required to devote full time for petitioner. If private respondent was
indeed an employee, it was rather unusual for him to wait for more than a year from his separation from work
before he decided to file his claims

Other Notes/ SC Pronouncements:

_____________________________________________________________________

14. ATOK BIG WEDGE COMPANY, INC. vs. JESUS P. GISON


GR No: 169510 Date: August 8, 2011
Ponente: Peralta, J.

Doctrine: Article 280 of the Labor Code is not applicable in the case at bar. Indeed, the Court has ruled that said
provision is not the yardstick for determining the existence of an employment relationship because it merely
distinguishes between two kinds of employees, i.e., regular employees and casual employees, for purposes of
determining the right of an employee to certain benefits, to join or form a union, or to security of tenure; it does not
apply where the existence of an employment relationship is in dispute. It is, therefore, erroneous to rely on Article
280 in determining whether an employer-employee relationship exists between respondent and the petitioner

FACTS:

Angeles| Bajana | Balladares | Brillantes | Briones | Cabansag | Callanta | Chua | David|

De Leon | Gomez | Lopez | Macalino | Nostratis | Padilla | Reynon | Santos | Tan |Velasco
4E / 4F - 2018-2019
Page 81 of 920
LABOR REVIEW DIGEST
Atty. Joyrich Golangco

Respondent Jesus P. Gison was engaged as part-time consultant on retainer basis by petitioner Atok Big Wedge
Company, Inc. through its then Asst. Vice-President and Acting Resident Manager, Rutillo A. Torres. As a
consultant on retainer basis, respondent assisted petitioner's retained legal counsel with matters pertaining to the
prosecution of cases against illegal surface occupants within the area covered by the company's mineral claims.
Respondent was likewise tasked to perform liaison work with several government agencies, which he said was his
expertise.
Petitioner did not require respondent to report to its office on a regular basis, except when
occasionally requested by the management to discuss matters needing his expertise as a consultant. As payment
for his services, respondent received a retainer fee of P3,000.00 a month, which was delivered to him either at his
residence or in a local restaurant. The parties executed a retainer agreement, but such agreement was misplaced
and can no longer be found. The said arrangement continued for the next eleven years.
Sometime thereafter, since respondent was getting old, he requested that petitioner cause his
registration with the Social Security System (SSS), but petitioner did not accede to his request. He later reiterated
his request but it was ignored by respondent considering that he was only a retainer/consultant. Respondent filed
a Complaint with the SSS against petitioner for the latter's refusal to cause his registration with the SSS. On the
same date, Mario D. Cera, in his capacity as resident manager of petitioner, issued a Memorandum advising
respondent that within 30 days from receipt thereof, petitioner is terminating his retainer contract with the
company since his services are no longer necessary.
Respondent filed a Complaint for illegal dismissal, unfair labor practice, underpayment of wages, non-
payment of 13th month pay, vacation pay, and sick leave pay.

LA RULING: Found no employer-employee relationship between petitioner and respondent, the Labor Arbiter
dismissed the complaint for lack of merit.

NLRC RULING: Affirmed the decision of the Labor Arbiter.

CA RULING: In ruling in favor of the respondent, the CA cited Article 280 of the Labor Code, or the provision
which distinguishes between two kinds of employees, i.e., regular and casual employees. Applying the provision
to the respondent's case, he is deemed a regular employee of the petitioner after the lapse of one year from his
employment. Considering also that respondent had been performing services for the petitioner for eleven years,
respondent is entitled to the rights and privileges of a regular employee.

PETITION TO THE SC:


Petitioner’s Contention: Petitioner contends that where the existence of an employer-employee relationship is in
dispute, Article 280 of the Labor Code is inapplicable. The said article only set the distinction between a casual
employee from a regular employee for purposes of determining the rights of an employee to be entitled to certain
benefits.

ISSUE/S: Whether or not there is an Employer-Employee Relationship between Atok and Gison?

HELD:
None. To ascertain the existence of an employer-employee relationship jurisprudence has invariably adhered to
the four-fold test, to wit: (1) the selection and engagement of the employee; (2) the payment of wages; (3) the
power of dismissal; and (4) the power to control the employee's conduct, or the so-called "control test.” Of these
four, the last one is the most important. The so-called control test is commonly regarded as the most crucial and
determinative indicator of the presence or absence of an employer-employee relationship. Under the control test,
an employer-employee relationship exists where the person for whom the services are performed reserves the
right to control not only the end achieved, but also the manner and means to be used in reaching that end.

Angeles| Bajana | Balladares | Brillantes | Briones | Cabansag | Callanta | Chua | David|

De Leon | Gomez | Lopez | Macalino | Nostratis | Padilla | Reynon | Santos | Tan |Velasco
4E / 4F - 2018-2019
Page 82 of 920
LABOR REVIEW DIGEST
Atty. Joyrich Golangco

Applying the aforementioned test, an employer-employee relationship is apparently absent in the case
at bar. Among other things, respondent was not required to report everyday during regular office hours of
petitioner. Respondent's monthly retainer fees were paid to him either at his residence or a local restaurant. More
importantly, petitioner did not prescribe the manner in which respondent would accomplish any of the tasks in
which his expertise as a liaison officer was needed; respondent was left alone and given the freedom to
accomplish the tasks using his own means and method. Respondent was assigned tasks to perform, but
petitioner did not control the manner and methods by which respondent performed these tasks. Verily, the
absence of the element of control on the part of the petitioner engenders a conclusion that he is not an employee
of the petitioner.

Moreover, the absence of the parties' retainership agreement notwithstanding, respondent clearly
admitted that petitioner hired him in a limited capacity only and that there will be no employer-employee
relationship between them.
Respondent was well aware of the agreement that he was hired merely as a liaison or consultant of the petitioner
and he agreed to perform tasks for the petitioner on a temporary employment status only. However, respondent
anchors his claim that he became a regular employee of the petitioner based on his contention that the temporary
aspect of his job and its limited nature could not have lasted for eleven years unless some time during that period,
he became a regular employee of the petitioner by continually performing services for the company.
The premise that regular employees are those who perform activities which are desirable and
necessary for the business of the employer is not determinative in this case. In fact, any agreement may provide
that one party shall render services for and in behalf of another, no matter how necessary for the latter's business,
even without being hired as an employee. Hence, respondent's length of service and petitioner's repeated act of
assigning respondent some tasks to be performed did not result to respondent's entitlement to the rights and
privileges of a regular employee.

Article 280 of the Labor Code, in which the lower court used to buttress its findings that respondent
became a regular employee of the petitioner, is not applicable in the case at bar. Indeed, the Court has ruled that
said provision is not the yardstick for determining the existence of an employment relationship because it merely
distinguishes between two kinds of employees, i.e., regular employees and casual employees, for purposes of
determining the right of an employee to certain benefits, to join or form a union, or to security of tenure; it does not
apply where the existence of an employment relationship is in dispute. It is, therefore, erroneous on the part of the
Court of Appeals to rely on Article 280 in determining whether an employer-employee relationship exists between
respondent and the petitioner

Other Notes/ SC Pronouncements:


_____________________________________________________________________

15. DUMPIT-MURILLO vs. COURT OF APPEALS


G.R. No. 164652 June 8, 2007

Doctrine: The assertion that a talent contract exists does not necessarily prevent a regular employment status.

FACTS:

Associated Broadcasting Company (ABC) hired Thelma Dumpit-Murillo as a newscaster and co-anchor for
Balitang-Balita under a Talent Contract for a period of three-months. The said contract was renewed multiple
times. When the last contract expired, Dumpit-Murillo sent a letter to Jose Javire, Vice President for News and
Public Affairs of ABC, informing the latter of her interest in renewing her contract. She sent another letter stating
that she was not able to receive any reply from her previous letter. She also stated that she considered the failure
Angeles| Bajana | Balladares | Brillantes | Briones | Cabansag | Callanta | Chua | David|

De Leon | Gomez | Lopez | Macalino | Nostratis | Padilla | Reynon | Santos | Tan |Velasco
4E / 4F - 2018-2019
Page 83 of 920
LABOR REVIEW DIGEST
Atty. Joyrich Golangco

of a formal response on the part of the company as her constructive dismissal. She then sent a demand letter
requesting her reinstatement and payment of unpaid wages and other benefits. ABC replied that the checks for
her talent fees are being processed but claimed that the other claims hand no basis. Dumpit-Murillo filed a
complaint against ABC for illegal dismissal before the NLRC.

LA RULING:

The Labor Arbiter dismissed the Complaint.

NLRC RULING:

The NLRC held that an employer-employee relationship existed between Dumpit-Murillo and ABS; that the subject
talent contract was void; and that she was a regular employee illegally dismissed.

CA RULING:

The CA reversed the decision of the NLRC. It ruled that Dumpit-Murillo was a fixed-term employee and not a
regular employee and should not be allowed to renege from the stipulation she had voluntarily and knowingly
executed.

ISSUES:

1. Whether or not there is an employer-employee relationship between ABC and Dumpit-Murillo?


2. Whether or not Dumpit-Murillo is a regular employee?

RULING:

1. Yes. The practice of having fixed-term contracts in the industry does not automatically make all talent contracts
valid and compliant with labor law. The assertion that a talent contract exists does not necessarily prevent a
regular employment status. The duties of Dumpit-Murillo as enumerated in her employment contract indicate that
ABC had control over the work of petitioner. Aside from control, ABC also dictated the work assignments and
payment of her wages. ABC also had the power to dismiss.

2. Yes. Dumpit-Murillo’s work was necessary or desirable in the usual business or trade of the employer, which
includes its participation in the government’s news and public information dissemination. Her work was continuous
for a period of four years and her repeated engagement under contract of hire is indicative of the necessity and
desirability of her work in ABC’s business. There is no valid fixed-term employment between Dumpit-Murillo and
ABC. Fixed-term employment will not be considered valid where, from the circumstances, it is apparent that
periods have been imposed to preclude acquisition of tenurial security by the employee. It should satisfactorily
appear that the employer and the employee dealt with each other on more or less equal terms with no moral
dominance being exercised by the employer over the employee. Patently, Dumpit-Murillo occupied a position of
weakness vis-à-vis the employer. She was merely one of the numerous newscasters/broadcasters of ABC and
she was left with no choice but to affix her signature of conformity on each renewal of her contract or risk the loss
of her job.

Other Notes/ SC Pronouncements:

_____________________________________________________________________

Angeles| Bajana | Balladares | Brillantes | Briones | Cabansag | Callanta | Chua | David|

De Leon | Gomez | Lopez | Macalino | Nostratis | Padilla | Reynon | Santos | Tan |Velasco
4E / 4F - 2018-2019
Page 84 of 920
LABOR REVIEW DIGEST
Atty. Joyrich Golangco

16. BERNATE v. PBA


G.R. No. 192084 September 14, 2011

FACTS:

Complainants Jose Mel Bernarte and Renato Guevara were referees of the PBA. They claim that they had been
made to sign contracts on a year to year basis until 2003, when Bernarte was made to sign a one and a half
month contract for the period of July 1st to August 5th 2003. In January 2004, Bernarte Received a letter advising
him that his contract would not be renewed citing his unsatisfactory performance on and off the court. Bernarte
was shocked, and felt that the dismissal was caused by his refusal to fix a game upon order of Ernie De Leon.
Complainant Guevarra, a referee since 2001, was likewise no longer made to sign a contract beginning February
2004. Complainants aver that they were employees of the PBA and were illegally dismissed. Respondents PBA
aver that the complainants entered into contracts of retainer with the PBA which after the lapse of their respective
periods, were not renewed. Respondents argue that complainants were not illegally dismissed because they were
Angeles| Bajana | Balladares | Brillantes | Briones | Cabansag | Callanta | Chua | David|

De Leon | Gomez | Lopez | Macalino | Nostratis | Padilla | Reynon | Santos | Tan |Velasco
4E / 4F - 2018-2019
Page 85 of 920
LABOR REVIEW DIGEST
Atty. Joyrich Golangco

not employees of the PBA, that their respective contracts were simply not renewed, and that the PBA had the
prerogative of whether or not to renew their contracts.

LA RULING:

In her 31 March 2005 Decision, the Labor Arbiter declared petitioner an employee whose dismissal by
respondents was illegal. Accordingly, the Labor Arbiter ordered the reinstatement of petitioner and the payment of
backwages, moral and exemplary damages and attorney's fees.

NLRC RULING:

Affirmed the decision of the LA.

CA RULING:

The Court of Appeals found petitioner an independent contractor since respondents did not exercise any form of
control over the means and methods by which petitioner performed his work as a basketball referee. The Court of
Appeals held: While the NLRC agreed that the PBA has no control over the referees acts of blowing the whistle
and making calls during basketball games, it, nevertheless, theorized that the said acts refer to the means and
methods employed by the referees in officiating basketball games for the illogical reason that said acts refer only
to the referees skills. How could a skilled referee perform his job without blowing a whistle and making calls?
Worse, how can the PBA control the performance of work of a referee without controlling his acts of blowing the
whistle and making calls?

ISSUE:

Whether or not complainants are employees of the PBA.

RULING:

The Petition is bereft of merit. To determine the existence of an employer-employee relationship, case law has
consistently applied the four-fold test, to wit: (a) the selection and engagement of the employee; (b) the payment
of wages; (c) the power of dismissal; and (d) the employers’ power to control the employee on the means and
methods by which the work is accomplished. The so-called control test is the most important indicator of the
presence or absence of an employer-employee relationship. The contractual stipulations do not pertain to, much
less dictate, how and when petitioner will blow the whistle and make calls. On the contrary, they merely serve as
rules of conduct or guidelines in order to maintain the integrity of the professional basketball league. As correctly
observed by the Court of Appeals, how could a skilled referee perform his job without blowing a whistle and
making calls? x x x [H]ow can the PBA control the performance of work of a referee without controlling his acts of
blowing the whistle and making calls? We agree with respondents that once in the playing court, the referees
exercise their own independent judgment, based on the rules of the game, as to when and how a call or decision
is to be made. The referees decide whether an infraction was committed, and the PBA cannot overrule them once
the decision is made on the playing court. The referees are the only, absolute, and final authority on the playing
court. Respondents or any of the PBA officers cannot and do not determine which calls to make or not to make
and cannot control the referee when he blows the whistle because such authority exclusively belongs to the
referees. The very nature of petitioners job of officiating a professional basketball game undoubtedly calls for
freedom of control by respondents.

Angeles| Bajana | Balladares | Brillantes | Briones | Cabansag | Callanta | Chua | David|

De Leon | Gomez | Lopez | Macalino | Nostratis | Padilla | Reynon | Santos | Tan |Velasco
4E / 4F - 2018-2019
Page 86 of 920
LABOR REVIEW DIGEST
Atty. Joyrich Golangco

Further, unlike regular employees who ordinarily report for work eight hours per day for five days a week,
petitioner is required to report for work only when PBA games are scheduled or three times a week at two hours
per game. In addition, there are no deductions for contributions to the Social Security System, Philhealth or
PagIbig, which are the usual deductions from employees salaries. These undisputed circumstances buttress the
fact that petitioner is an independent contractor, and not an employee of respondents. In addition, the fact that
PBA repeatedly hired petitioner does not by itself prove that petitioner is an employee of the former. For a hired
party to be considered an employee, the hiring party must have control over the means and methods by which the
hired party is to perform his work, which is absent in this case. The continuous rehiring by PBA of petitioner simply
signifies the renewal of the contract between PBA and petitioner, and highlights the satisfactory services rendered
by petitioner warranting such contract renewal. Conversely, if PBA decides to discontinue petitioners services at
the end of the term fixed in the contract, whether for unsatisfactory services, or violation of the terms and
conditions of the contract, or for whatever other reason, the same merely results in the non-renewal of the
contract, as in the present case. The non-renewal of the contract between the parties does not constitute illegal
dismissal of petitioner by respondents.

Other Notes/ SC Pronouncements:

_____________________________________________________________________

17. Angel Jardin, et. al., vs. NLRC and Goodman Taxi
GR No: 119268 Date: February 23, 2000
Ponente: Quisumbimg, J.

Doctrine:

The relationship between jeepney owners/operators on one hand and jeepney drivers on the other under the
boundary system is that of employer-employee and not of lessor-lessee..

FACTS:

Petitioners were drivers of private respondent, Philjama International Inc., a domestic corporation engaged in
the operation of "Goodman Taxi." Petitioners used to drive private respondents taxicabs every other day on a
24-hour work schedule under the boundary system. Petitioners earned an average of P400 daily. Nevertheless,
Private respondent deducts P30 from each of them daily supposedly for the washing of the taxi units.

Petitioners decided to form a labor union to protect their rights and interests. Upon learning of their plan,
Goodman Taxi refused to let petitioners drive their taxicabs when they reported for work. Petitioners suspected
that they were singled out because they were leaders and active members of the proposed union.
Angeles| Bajana | Balladares | Brillantes | Briones | Cabansag | Callanta | Chua | David|

De Leon | Gomez | Lopez | Macalino | Nostratis | Padilla | Reynon | Santos | Tan |Velasco
4E / 4F - 2018-2019
Page 87 of 920
LABOR REVIEW DIGEST
Atty. Joyrich Golangco

The incident caused them to file with the labor arbiter a complaint against private respondent, Goodman Taxi for
unfair labor practice, illegal dismissal and illegal deduction of washing fees.|

LA RULING:

The labor arbiter dismissed the complaint for lack of merit.

NLRC RULING:

On appeal, the NLRC reversed and set aside the decision of the labor arbiter. The NLRC declared that petitioners
are employees of private respondent, and, as such, their dismissal must be for just cause and after due process.

Private respondent filed its first motion for reconsideration, which was denied. However upon filing a second
motion for reconsideration, the NLRC ruled that it lacks jurisdiction over the case as petitioners and private
respondent had no employer-employee relationship. It held that the relationship is leasehold, which is covered by
the Civil Code rather than the Labor Code.

Petitioners sought reconsideration of the NLRC's latest decision, which was denied.

APPEAL TO THE SC:

Hence, this petition special civil action for certiorari to annul the decision of the NLRC.DEASC

ISSUE/S:
1. Did the NLRC err in entertaining the 2nd Motion for Reconsideration of the private respondent? Yes
2. Is there an Employer-Employee relationship between the parties?
3.

HELD:
1. Yes. The second motion for reconsideration filed by private respondent is indubitably a prohibited
pleading which should have not been entertained at all. Public respondent cannot just disregard its own rules on
the pretext of "satisfying the ends of justice," especially when its disposition of a legal controversy ran afoul with a
clear and long standing jurisprudence in this jurisdiction as elucidated in the subsequent discussion. Clearly,
disregarding a settled legal doctrine enunciated by this Court is not a way of rectifying an error or mistake. In our
view, public respondent gravely abused its discretion in taking cognizance and granting private respondent's
second motion for reconsideration as it wrecks the orderly procedure in seeking reliefs in labor cases.|||

2. Yes, Under the boundary system the driver takes out his unit and pays the owner/operator a fee commonly
called 'boundary' for the use of the unit. In the determination the existence of employer-employee relationship, the
Supreme Court has applied the following four-fold test: '(1) the selection and engagement of the employee; (2) the
payment of wages; (3) the power of dismissal; and (4) the power of control the employees conduct.'

In a number of cases decided by this Court, it ruled that the relationship between jeepney owners/operators on
one hand and jeepney drivers on the other under the boundary system is that of employer-employee and not of
lessor-lessee. The Court explained that in the lease of chattels, the lessor loses complete control over the chattel
leased although the lessee cannot be reckless in the use thereof, otherwise he would be responsible for the
damages to the lessor.
Angeles| Bajana | Balladares | Brillantes | Briones | Cabansag | Callanta | Chua | David|

De Leon | Gomez | Lopez | Macalino | Nostratis | Padilla | Reynon | Santos | Tan |Velasco
4E / 4F - 2018-2019
Page 88 of 920
LABOR REVIEW DIGEST
Atty. Joyrich Golangco

In the case of jeepney owners/operators and jeepney drivers, the former exercise supervision and control over the
latter. The management of the business is in the owner's hands. The owner as holder of the certificate of public
convenience must see to it that the driver follows the route prescribed by the franchising authority and the rules
promulgated as regards its operation.

Now, the fact that the drivers do not receive fixed wages but get only that in excess of the so-called "boundary"
they pay to the owner/operator is not sufficient to withdraw the relationship between them from that of employer
and employee. The Court have applied by analogy the above-stated doctrine to the relationships between bus
owner/operator and bus conductor, auto-calesa owner/operator and driver, and recently between taxi
owners/operators and taxi drivers. Hence, petitioners are undoubtedly employees of private respondent because
as taxi drivers they perform activities which are usually necessary or desirable in the usual business or trade of
their employer.|||

Other Notes/ SC Pronouncements:

______________________________________________________________________

18. PSI vs CA
(same as number 8)
Other Notes/ SC Pronouncements:

_____________________________________________________________________
19. LOCSIN and TOMAQUIN v. PHILIPPINE LONG DISTANCE COMPANY
G.R. No. 185251 October 2, 2009

VELASCO, JR., J.

FACTS:
- November 1, 1990, respondent Philippine Long Distance Telephone Company (PLDT) and the Security
and Safety Corporation of the Philippines (SSCP) entered into a Security Services Agreement
(Agreement) whereby SSCP would provide armed security guards to PLDT to be assigned to its various
offices.
- Petitioners Raul Locsin and Eddie Tomaquin, among other security guards, were posted at a PLDT
office.
-
- On August 30, 2001, respondent issued a Letter dated August 30, 2001 terminating the Agreement
effective October 1, 2001.
- Despite the termination of the Agreement, however, petitioners continued to secure the premises of their
assigned office.
- They were allegedly directed to remain at their post by representatives of respondent.
-
- In support of their contention, petitioners provided the Labor Arbiter with copies of petitioner Locsins pay
slips for the period of January to September 2002.
- Then, on September 30, 2002, petitioners services were terminated.

Angeles| Bajana | Balladares | Brillantes | Briones | Cabansag | Callanta | Chua | David|

De Leon | Gomez | Lopez | Macalino | Nostratis | Padilla | Reynon | Santos | Tan |Velasco
4E / 4F - 2018-2019
Page 89 of 920
LABOR REVIEW DIGEST
Atty. Joyrich Golangco

- petitioners filed a complaint before the Labor Arbiter for illegal dismissal and recovery of money claims
such as overtime pay, holiday pay, premium pay for holiday and rest day, service incentive leave pay,
Emergency Cost of Living Allowance, and moral and exemplary damages against PLDT.

LA RULING:
- Labor Arbiter rendered a Decision finding PLDT liable for illegal dismissal.
o It was explained in the Decision that petitioners were found to be employees of PLDT and not of
SSCP
o As such employees, petitioners were entitled to substantive and procedural due process before
termination of employment.
o The Labor Arbiter held that respondent failed to observe such due process requirements.
o
NLRC RULING:
- PLDT appealed the above Decision to the NLRC which rendered a Resolution affirming in toto the
Arbiters Decision. MR likewise denied.
- PLDT filed a Petition for Certiorari with the CA asking for the nullification of the Resolution issued by the
NLRC as well as the Labor Arbiters Decision.
o The CA rendered the assailed decision granting PLDTs petition and dismissing petitioners
complaint.
o CA applied the four-fold test in order to determine the existence of an employer-employee
relationship between the parties but did not find such relationship.
o It determined that SSCP was not a labor-only contractor and was an independent contractor
having substantial capital to operate and conduct its own business.
o Anent the pay slips that were presented by petitioners, the CA noted that those were issued by
SSCP and not PLDT; hence, SSCP continued to pay the salaries of petitioners after the
Agreement.
-
-
- APPEAL TO THE SC:
- Petition for Review on Certiorari under Rule 45 seeks the reversal of the CA decision
-
ISSUE:
- whether petitioners became employees of respondent after the Agreement between SSCP and
respondent was terminated.

HELD:
 Petition should be Granted.
 SUPREME COURT relied on the Rule on Evidence on Disputable Presumptions
o Rule 131, Section 3(y) of the Rules of Court provides
o The following presumptions are satisfactory if uncontradicted, but may be contradicted and
overcome by other evidence:
 (y) That things have happened according to the ordinary course of nature and the
ordinary habits of life.

 In the ordinary course of things, responsible business owners or managers would not allow security
guards of an agency with whom the owners or managers have severed ties with to continue to stay within
the business premises.

Angeles| Bajana | Balladares | Brillantes | Briones | Cabansag | Callanta | Chua | David|

De Leon | Gomez | Lopez | Macalino | Nostratis | Padilla | Reynon | Santos | Tan |Velasco
4E / 4F - 2018-2019
Page 90 of 920
LABOR REVIEW DIGEST
Atty. Joyrich Golangco

 This is because upon the termination of the owners or managers agreement with the security agency, the
agency’s undertaking of liability for any damage that the security guard would cause has already been
terminated.

 Thus, in the event of an accident or otherwise damage caused by such security guards, it would be the
business owners and/or managers who would be liable and not the agency.

 The business owners or managers would, therefore, be opening themselves up to liability for acts of
security guards over whom the owners or managers allegedly have no control.

 At the very least, responsible business owners or managers would inquire or learn why such security
guards were remaining at their posts, and would have a clear understanding of the circumstances of the
guards stay.
 It is but logical that responsible business owners or managers would be aware of the situation in their
premises.

 We point out that with respondents hypothesis, it would seem that SSCP was paying petitioners salaries
while securing respondents premises despite the termination of their Agreement.

 Obviously, it would only be respondent that would benefit from such a situation. And it is seriously
doubtful that a security agency that was established for profit would allow its security guards to secure
respondents premises when the Agreement was already terminated.

 From the foregoing circumstances, reason dictates that we conclude that petitioners remained at their
post under the instructions of respondent.
 We can further conclude that respondent dictated upon petitioners that the latter perform their regular
duties to secure the premises during operating hours.
 This, to our mind and under the circumstances, is sufficient to establish the existence of an employer-
employee relationship.

 Certainly, the facts as narrated by petitioners are more believable than the irrational denials made by
respondent.

 To reiterate, while respondent and SSCP no longer had any legal relationship with the termination of the
Agreement, petitioners remained at their post securing the premises of respondent while
receiving their salaries, allegedly from SSCP.

 It is but reasonable to conclude that, with the behest and, presumably, directive of respondent,
petitioners continued with their services.
 Evidently, such are indicia of control that respondent exercised over petitioners.

 Such power of control has been explained as the right to control not only the end to be achieved
but also the means to be used in reaching such end.
 With the conclusion that respondent directed petitioners to remain at their posts and continue with their
duties, it is clear that respondent exercised the power of control over them; thus, the existence of an
employer-employee relationship.

Angeles| Bajana | Balladares | Brillantes | Briones | Cabansag | Callanta | Chua | David|

De Leon | Gomez | Lopez | Macalino | Nostratis | Padilla | Reynon | Santos | Tan |Velasco
4E / 4F - 2018-2019
Page 91 of 920
LABOR REVIEW DIGEST
Atty. Joyrich Golangco

 Jurisprudence is firmly settled that whenever the existence of an employment relationship is in dispute,
four elements constitute the reliable yardstick:
o (a) the selection and engagement of the employee;
o (b) the payment of wages;
o (c) the power of dismissal; and
o (d) the employers power to control the employees conduct.

 It is the so-called control test which constitutes the most important index of the existence of the
employer-employee relationship that is, whether the employer controls or has reserved the right to
control the employee not only as to the result of the work to be done but also as to the means and
methods by which the same is to be accomplished.

 Stated otherwise, an employer-employee relationship exists where the person for whom the
services are performed reserves the right to control not only the end to be achieved but also the
means to be used in reaching such end.

 Furthermore, Article 106 of the Labor Code contains a provision on contractors, to wit:
o Art. 106. Contractor or subcontractor. Whenever an employer enters into a contract with another
person for the performance of the formers work, the employees of the contractor and of the
latter’s subcontractor, if any, shall be paid in accordance with the provisions of this Code.
o In the event that the contractor or subcontractor fails to pay the wages of his employees in
accordance with this Code, the employer shall be jointly and severally liable with his contractor
or subcontractor to such employees to the extent of the work performed under the contract, in
the same manner and extent that he is liable to employees directly employed by him.
o The Secretary of Labor and Employment may, by appropriate regulations, restrict or
prohibit the contracting-out of labor to protect the rights of workers established under
this Code. In so prohibiting or restricting, he may make appropriate distinctions between
labor-only contracting and job contracting as well as differentiations within these types
of contracting and determine who among the parties involved shall be considered the
employer for purposes of this Code, to prevent any violation or circumvention of any
provision of this Code.
o There is labor-only contracting where the person supplying workers to an employer does not
have substantial capital or investment in the form of tools, equipment, machineries, work
premises, among others, and the workers recruited and placed by such person are performing
activities which are directly related to the principal business of such employer. In such cases,
the person or intermediary shall be considered merely as an agent of the employer who shall be
responsible to the workers in the same manner and extent as if the latter were directly employed
by him.
 The Secretary of Labor issued Department Order No. 18-2002, Series of 2002, implementing Art. 106 as
follows:
o Section 5. Prohibition against labor-only contracting. Labor-only contracting is hereby declared
prohibited. For this purpose, labor-only contracting shall refer to an arrangement where the
contractor or subcontractor merely recruits, supplies or places workers to perform a job, work or
service for a principal, and any of the following elements are present:
o (i) The contractor or subcontractor does not have substantial capital or investment which relates
to the job, work or service to be performed and the employees recruited, supplied or placed by
such contractor or subcontractor are performing activities which are directly related to the main
business of the principal; or
Angeles| Bajana | Balladares | Brillantes | Briones | Cabansag | Callanta | Chua | David|

De Leon | Gomez | Lopez | Macalino | Nostratis | Padilla | Reynon | Santos | Tan |Velasco
4E / 4F - 2018-2019
Page 92 of 920
LABOR REVIEW DIGEST
Atty. Joyrich Golangco

o (ii) the contractor does not exercise the right to control over the performance of the work
of the contractual employee.

 petitioners were entitled to the rights and benefits of employees of respondent, including due process
requirements in the termination of their services.
 SET ASIDE CA decision. REINSTATE NLRC

Other Notes/ SC Pronouncements:

_____________________________________________________________________

20. YMBONG v. ABS-CBN BROADCASTING CORPORATION, VENERANDA SY AND DANTE LUZON


G.R. No. 184885 March 7, 2012

VILLARAMA, JR., J.

FACTS:
- Ernesto G. Ymbong started working for ABS-CBN Broadcasting Corporation (ABS-CBN) in 1993 at its
regional station in Cebu as a television talent, co-anchoring Hoy Gising and TV Patrol Cebu.
- His stint in ABS-CBN later extended to radio when ABS-CBN Cebu launched its AM station DYAB in
1995 where he worked as drama and voice talent, spinner, scriptwriter and public affairs program anchor.
- Like Ymbong, Leandro Patalinghug also worked for ABS-CBN Cebu. Starting 1995, he worked as talent,
director and scriptwriter for various radio programs aired over DYAB.
- January 1, 1996, the ABS-CBN Head Office in Manila issued Policy No. HR-ER-016 or the Policy on
Employees Seeking Public Office.
o 1. Any employee who intends to run for any public office position, must file his/her letter of
resignation, at least thirty (30) days prior to the official filing of the certificate of candidacy
either for national or local election.
xxxx
3. Further, any employee who intends to join a political group/party or even with no
political affiliation but who intends to openly and aggressively campaign for a candidate or group
of candidates (e.g. publicly speaking/endorsing candidate, recruiting campaign workers, etc.)
must file a request for leave of absence subject to managements approval. For this particular

Angeles| Bajana | Balladares | Brillantes | Briones | Cabansag | Callanta | Chua | David|

De Leon | Gomez | Lopez | Macalino | Nostratis | Padilla | Reynon | Santos | Tan |Velasco
4E / 4F - 2018-2019
Page 93 of 920
LABOR REVIEW DIGEST
Atty. Joyrich Golangco

reason, the employee should file the leave request at least thirty (30) days prior to the start of
the planned leave period.
xxxx
- Dante Luzon, Assistant Station Manager of DYAB issued a memorandum:
o Please be informed that per company policy, any employee/talent who wants to run for any
position in the coming election will have to file a leave of absence the moment he/she
files his/her certificate of candidacy.
- After the issuance of the memorandum, Ymbong got in touch with Luzon.
o Luzon claims that Ymbong approached him and told him that he would leave radio for a couple
of months because he will campaign for the administration ticket.
- It was only after the elections that they found out that Ymbong actually ran for public office himself at the
eleventh hour.
o Ymbong, on the other hand, claims that in accordance with the March 25, 1998 Memorandum,
he informed Luzon through a letter that he would take a few months leave of absence from
March 8, 1998 to May 18, 1998 since he was running for councilor of Lapu-Lapu City.
- Patalinghug approached Luzon and advised him that he will run as councilor for Naga, Cebu. According
to Luzon, he clarified to Patalinghug that he will be considered resigned and not just on leave once he
files a certificate of candidacy.
- Unfortunately, both Ymbong and Patalinghug lost in the May 1998 elections.
- Later, Ymbong and Patalinghug both tried to come back to ABS-CBN Cebu.
- According to Luzon, he informed them that they cannot work there anymore because of company policy.
- ABS-CBN, however, agreed out of pure liberality to give them a chance to wind up their participation in
the radio drama, Nagbabagang Langit, since it was rating well and to avoid an abrupt ending.
- The agreed winding-up, however, dragged on for so long prompting Luzon to issue to Ymbong the a
memorandum stating that:
o It has been decided therefore that all your drama participation shall be terminated effective
immediately.
o However, your involvement as drama spinner/narrator of the drama NAGBA[BA]GANG LANGIT
continues until its writer/director Mr. Leandro Patalinghug wraps it up one week upon receipt of
a separate memo issued to him.
- Ymbong in contrast contended that after the expiration of his leave of absence, he reported back to work
as a regular talent and in fact continued to receive his salary.
- Ymbong filed an illegal dismissal complaint against ABS-CBN, Luzon and DYAB Station Manager
Veneranda Sy.
- And even granting without admitting the existence of the company policy supposed to have been
violated, Ymbong averred that it was necessary that the company policy meet certain requirements
before willful disobedience of the policy may constitute a just cause for termination.
- Ymbong further argued that the company policy violates his constitutional right to suffrage.
- Patalinghug likewise filed an illegal dismissal complaint against ABS-CBN.
- Labor Arbiter found that
o there exists an employer-employee relationship between ABS-CBN and Ymbong and
Patalinghug considering the stipulations in their appointment letters/talent contracts.
o noted particularly that the appointment letters/talent contracts imposed conditions in the
performance of their work, specifically on attendance and punctuality, which effectively placed
them under the control of ABS-CBN.
o likewise ruled that although the subject company policy is reasonable and not contrary to law,
the same was not made known to Ymbong and Patalinghug and in fact was superseded by
another one embodied in the March 25, 1998 Memorandum issued by Luzon.

Angeles| Bajana | Balladares | Brillantes | Briones | Cabansag | Callanta | Chua | David|

De Leon | Gomez | Lopez | Macalino | Nostratis | Padilla | Reynon | Santos | Tan |Velasco
4E / 4F - 2018-2019
Page 94 of 920
LABOR REVIEW DIGEST
Atty. Joyrich Golangco

- On appeal to the NLRC, ABS-CBN contended that the Labor Arbiter has no jurisdiction over the case
because there is no employer-employee relationship between the company and Ymbong and
Patalinghug, and that Sy and Luzon mistakenly assumed that Ymbong and Patalinghug could just file a
leave of absence since they are only talents and not employees.
- NLRC rendered a decision modifying the labor arbiters decision Ordering respondent ABS-CBN to
reinstate complainant Ernesto G. Ymbong.
- NLRC treated their cases differently:
o In the case of Patalinghug, it found that he voluntarily resigned from employment on April 21,
1998 when he submitted his resignation letter.
o noted that although the tenor of the resignation letter is somewhat involuntary, he knew that it is
the policy of the company that every person connected therewith should resign from his
employment if he seeks an elected position in the government.
o As to Ymbong, however, the NLRC ruled otherwise.
o It ruled that the March 25, 1998 Memorandum merely states that an employee who seeks any
elected position in the government will only merit the temporary suspension of his services.
o It held that under the principle of social justice, the March 25, 1998 Memorandum shall prevail
and ABS-CBN is estopped from enforcing the September 14, 1998 memorandum issued to
Ymbong stating that his services had been automatically terminated when he ran for an elective
position.
- CA rendered the assailed decision reversing and setting aside the March 8, 2004 Decision and June 21,
2004 Resolution of the NLRC. The CA declared Ymbong resigned from employment and not to have
been illegally dismissed.

ISSUE/S:
- whether Policy No. HR-ER-016 is valid.
HELD:
 Policy No. HR-ER-016 is valid.
 ABS-CBN, like Manila Broadcasting Company, also had a valid justification for Policy No. HR-ER-016. Its
rationale is embodied in the policy itself, to wit:
 Rationale:
o ABS-CBN BROADCASTING CORPORATION strongly believes that it is to the best interest of
the company to continuously remain apolitical. While it encourages and supports its
employees to have greater political awareness and for them to exercise their right to
suffrage, the company, however, prefers to remain politically independent and
unattached to any political individual or entity.
Therefore, employees who [intend] to run for public office or accept political
appointment should resign from their positions, in order to protect the company from
any public misconceptions. To preserve its objectivity, neutrality and credibility, the
company reiterates the following policy guidelines for strict implementation.
xxxx
 We have consistently held that so long as a companys management prerogatives are exercised in
good faith for the advancement of the employers interest and not for the purpose of defeating or
circumventing the rights of the employees under special laws or under valid agreements, this
Court will uphold them.
 In the instant case, ABS-CBN validly justified the implementation of Policy No. HR-ER-016.
 It is well within its rights to ensure that it maintains its objectivity and credibility and freeing itself
from any appearance of impartiality so that the confidence of the viewing and listening public in it
will not be in any way eroded.

Angeles| Bajana | Balladares | Brillantes | Briones | Cabansag | Callanta | Chua | David|

De Leon | Gomez | Lopez | Macalino | Nostratis | Padilla | Reynon | Santos | Tan |Velasco
4E / 4F - 2018-2019
Page 95 of 920
LABOR REVIEW DIGEST
Atty. Joyrich Golangco

 Even as the law is solicitous of the welfare of the employees, it must also protect the right of an
employer to exercise what are clearly management prerogatives.
 The free will of management to conduct its own business affairs to achieve its purpose cannot be denied.
 It is worth noting that such exercise of management prerogative has earned a stamp of approval
from no less than our Congress itself when on February 12, 2001, it enacted Republic Act No. 9006,
otherwise known as the Fair Election Act.
 Section 6.6 thereof reads:
o 6.6. Any mass media columnist, commentator, announcer, reporter, on-air correspondent
or personality who is a candidate for any elective public office or is a campaign volunteer
for or employed or retained in any capacity by any candidate or political party shall be
deemed resigned, if so required by their employer,
xxx
ISSUE:
- whether the March 25, 1998 Memorandum issued by Luzon superseded Policy No. HR-ER-016
HELD:
 CA correctly ruled that though Luzon, as Assistant Station Manager for Radio of ABS-CBN, has policy-
making powers in relation to his principal task of administering the networks radio station in the
Cebu region, the exercise of such power should be in accord with the general rules and
regulations imposed by the ABS-CBN Head Office to its employees
 1998 Memorandum issued by Luzon which only requires employees to go on leave if they intend to run
for any elective position is in absolute contradiction with Policy No. HR-ER-016 issued by the ABS-
CBN Head Office
 Having been issued beyond the scope of his authority, the March 25, 1998 Memorandum is
therefore void and did not supersede Policy No. HR-ER-016.

ISSUE:
- whether Ymbong, by seeking an elective post, is deemed to have resigned and not dismissed by ABS-
CBN
HELD:
 Ymbong is deemed resigned when he ran for councilor.
 As Policy No. HR-ER-016 is the subsisting company policy and not Luzons March 25, 1998
Memorandum, Ymbong is deemed resigned when he ran for councilor.
 Ymbongs overt act of running for councilor of Lapu-Lapu City is tantamount to resignation on his
part.
 He was separated from ABS-CBN not because he was dismissed but because he resigned.
 Since there was no termination to speak of, the requirement of due process in dismissal cases cannot be
applied to Ymbong.
 Thus, ABS-CBN is not duty-bound to ask him to explain why he did not tender his resignation before he
ran for public office as mandated by the subject company policy.
 Moreover, as pointed out by ABS-CBN, had Ymbong been truthful to his superiors, they would have been
able to clarify to him the prevailing company policy and inform him of the consequences of his decision in
case he decides to run, as Luzon did in Patalinghugs case.
PETITION IS DENIED.

Other Notes/ SC Pronouncements:

Angeles| Bajana | Balladares | Brillantes | Briones | Cabansag | Callanta | Chua | David|

De Leon | Gomez | Lopez | Macalino | Nostratis | Padilla | Reynon | Santos | Tan |Velasco
4E / 4F - 2018-2019
Page 96 of 920
LABOR REVIEW DIGEST
Atty. Joyrich Golangco

_____________________________________________________________________

21. Pedro Chavez v. NLRC, Supreme Packaging, and Alvin Lee(Plant Manager)
G.R. No. 146530; Jan. 17, 2005
Callejo, Sr., J.:

DOCTRINE: Compared to an employee, an independent contractor is one who carries on a distinct and
independent business and undertakes to perform the job, work, or service on its own account and under its own
responsibility according to its own manner and method, free from the control and direction of the principal in all
matters connected with the performance of the work except as to the results thereof.

FACTS:
Respondent company, Supreme Packaging, Inc., is in the business of manufacturing cartons and other
packaging materials for export and distribution. It engaged the services of the petitioner, Pedro Chavez, as truck
driver on October 25, 1984. Petitioner was tasked to deliver the respondent company's products from its factory in
Mariveles, Bataan, to its various customers, mostly in Metro Manila. For this purpose, he was “given” a truck to be
used in his services.

Most of the petitioner's delivery trips were made at nighttime, commencing at 6:00 p.m. from Mariveles, and
returning thereto in the afternoon two or three days after. The deliveries were made in accordance with the routing
slips issued by respondent company indicating the order, time and urgency of delivery. Initially, the petitioner was
paid the sum of P350.00 per trip. This was later adjusted to P480.00 per trip and, at the time of his alleged
dismissal, the petitioner was receiving P900.00 per trip.

Sometime in 1992, the petitioner expressed to respondent Alvin Lee, respondent company's plant manager, his
(the petitioner's) desire to avail himself of the benefits that the regular employees were receiving such as overtime
pay, nightshift differential pay, and 13th month pay, among others. Although he promised to extend these benefits
to the petitioner, respondent Lee failed to actually do so.

Petitioner filed a complaint for regularization with the Regional Arbitration Branch No. III of the NLRC in San
Fernando, Pampanga. Before the case could be heard, respondent company terminated the services of the
petitioner. petitioner filed an amended complaint against the respondents for illegal dismissal, unfair labor practice
and non-payment of overtime pay, nightshift differential pay, 13th month pay, among others. Respondents, for
their part, denied the existence of an employer-employee relationship between the respondent company and the
petitioner, and that their contract was appropriately denominated as a “Contract of Service”. Respondent insisted
that Chavez was had complete control over the means and methods by how the work was accomplished, and that
he was the one who provided the wages to his employees and exercised complete control over them. It also
Angeles| Bajana | Balladares | Brillantes | Briones | Cabansag | Callanta | Chua | David|

De Leon | Gomez | Lopez | Macalino | Nostratis | Padilla | Reynon | Santos | Tan |Velasco
4E / 4F - 2018-2019
Page 97 of 920
LABOR REVIEW DIGEST
Atty. Joyrich Golangco

alleged that the reason for Chavez’s dismissal was his negligence in caring for the truck furnished him, causing
the company to overhaul the said truck. As an added cause for his dismissal, Supreme Packaging alleged that he
had also already abandoned his work.

LA RULING: Ruled that Chavez was an employee, given that he was performing a service necessary and
desirable to the respondent’s business, and taking into consideration his uninterrupted service of 10 years. Given
that due process was not afforded him, Chavez was ordered reinstated. It ruled that Contracts of Service are not
contrary to laws, good morals, etc.

NLRC Ruling: Initially dismissed Supreme’s appeal and lamented Respondent’s “scheme” BUT did a complete
180 degree turn upon MR and reversed the decision of the LA, ruling that Petitioner had control over the means
and methods of his work, hired his own workers, and paid their wages.

CA Ruling: The CA initially reinstated the ruling of the LA, and affirmed the grounds upon which it stood. It also
did not give credence to the claim of abandonment because this ran contrary to the logic of applying for his
regularization. HOWEVER, AGAIN, it made a 180 and cited:

(1) the respondents had no say on how the goods were to be delivered to the customers;
(2) the petitioner had the right to employ workers who would be under his direct control; and
(3) the petitioner had no working time.

APPEAL TO THE SC:

Petitioner’s Contention: There is E-E Relationship


Respondent’s Contention: There is NO E-E Relationship

ISSUE: Whether there is an E-E Relationship.

RULING: NO. The court held in the affirmative, setting forth the 4-Fold Test: The elements to determine the
existence of an employment relationship are:

(1) the selection and engagement of the employee;


(2) the payment of wages;
(3) the power of dismissal; and
(4) the employer's power to control the employee's conduct.

The most important element is the employer's control of the employee's conduct, not only as to the result of the
work to be done, but also as to the means and methods to accomplish it. All the four elements are present in this
case.

Undeniably, it was the respondents who engaged the services of the petitioner without the intervention of a third
party.
As to wages, That the petitioner was paid on a per trip basis is not signi�cant. This is merely a method of
computing compensation and not a basis for determining the existence or absence of employer-employee
relationship. One may be paid on the basis of results or time expended on the work, and may or may not acquire
an employment status, depending on whether the elements of an employer-employee relationship are present or
not.

Third. The respondents' power to dismiss the petitioner was inherent in the fact that they engaged the services of
the petitioner as truck driver
Angeles| Bajana | Balladares | Brillantes | Briones | Cabansag | Callanta | Chua | David|

De Leon | Gomez | Lopez | Macalino | Nostratis | Padilla | Reynon | Santos | Tan |Velasco
4E / 4F - 2018-2019
Page 98 of 920
LABOR REVIEW DIGEST
Atty. Joyrich Golangco

Fourth. As earlier opined, of the four elements of the employer-employee relationship, the "control test" is the most
important. Compared to an employee, an independent contractor is one who carries on a distinct and independent
business and undertakes to perform the job, work, or service on its own account and under its own responsibility
according to its own manner and method, free from the control and direction of the principal in all matters
connected with the performance of the work except as to the results thereof. Hence, while an independent
contractor enjoys independence and freedom from the control and supervision of his principal, an employee is
subject to the employer's power to control the means and methods by which the employee's work is to be
performed and accomplished.
_____________________________________________________________________

22. Coca-Cola Bottlers Phils & Eric Montinola v. Dr. Dean Climaco
G.R. No. 146881; Feb. 5, 2007
Azcuna, J.:

DOCTRINE: The power to control the employee's conduct, or the so-called "control test," considered to be the
most important element in the Four-Fold Test.

FACTS:

Dr. Dean N. Climaco is a medical doctor who was hired by petitioner Coca-Cola Bottlers Phils., Inc. by virtue of
a Retainer Agreement, and as a consequence of which the former made a Comprehensive Medical Plan for the
employees of the latter.

The Retainer Agreement, which began on January 1, 1988, was renewed annually. The last one expired on
December 31, 1993. Despite the non-renewal of the Retainer Agreement, respondent continued to perform his
functions as company doctor to Coca- Cola until he received a letter 4 dated March 9, 1995 from petitioner
company concluding their retainership agreement effective 30 days from receipt thereof.

As early as September 1992, petitioner was already making inquiries regarding his status with petitioner company.
First, he wrote a letter addressed to Dr. Willie Sy, the Acting President and Chairperson of the Committee on
Membership, Philippine College of Occupational Medicine. In response, Dr. Sy wrote a letter to the Personnel
Officer of Coca-Cola Bottlers Phils., Bacolod City, stating that respondent should be considered as a regular part-
time physician, having served the company continuously for four (4) years. He likewise stated that respondent
must receive all the benefits and privileges of an employee under Article 157 (b) 6 of the Labor Code.

Petitioner company, however, did not take any action. Hence, respondent made another inquiry directed to the
Assistant Regional Director, Bacolod City District Office of the Department of Labor and Employment (DOLE),
who referred the inquiry to the Legal Service of the DOLE, Manila. In his letter dated May 18, 1993, Director
Dennis P. Ancheta, Legal Service, DOLE, stated that he believed that an employer-employee relationship existed
between petitioner and respondent based on the Retainer Agreement and the Comprehensive Medical Plan, and
the application of the "four-fold" test. An inquiry was likewise addressed to the Social Security System (SSS).
Thereafter, Mr. Romeo R. Tupas, OIC-FID of SSS-Bacolod City, wrote a letter to the Personnel Officer of Coca-
Cola Bottlers Phils., Inc. informing the latter that the legal staff of his office was of the opinion that the services of
respondent partake of the nature of work of a regular company doctor and that he was, therefore, subject to social
security coverage. Respondent inquired from the management of petitioner company whether it was agreeable to
recognizing him as a regular employee. The management refused to do so.

Angeles| Bajana | Balladares | Brillantes | Briones | Cabansag | Callanta | Chua | David|

De Leon | Gomez | Lopez | Macalino | Nostratis | Padilla | Reynon | Santos | Tan |Velasco
4E / 4F - 2018-2019
Page 99 of 920
LABOR REVIEW DIGEST
Atty. Joyrich Golangco

On February 24, 1994, respondent filed a Complaint 9 before the NLRC, Bacolod City, seeking recognition as a
regular employee of petitioner company and prayed for the payment of all bene�ts of a regular employee,
including 13th Month Pay, Cost of Living Allowance, Holiday Pay, Service Incentive Leave Pay, and Christmas
Bonus. A few weeks before the filing of the action, respondent received from the petitioner a letter that it was
terminating the Retainership Agreement, and thus another complaint was filed for Illegal Dismissal.

LA Ruling: Both Arbiters Pelaez and Rodriguez dismissed the actions filed by Dr. Climaco and ruled that he was
not an employee of Respondent. (No reason cited in case)

NLRC Ruling: Affirmed the LA (No reason cited ulit lol) and dismissed the cases for lack of merit.

CA Ruling: The CA overturned the rulings below, ratiocinating using the Control Test:

It was averred by Coca-Cola in its comment that they exercised no control over petitioner for the
reason that the latter was not directed as to the procedure and manner of performing his
assigned tasks. It went as far as saying that "petitioner was not told how to immunize, inject,
treat or diagnose the employees of the respondent (Rollo, page 228). We believe that if the
"control test" would be interpreted this strictly, it would result in an absurd and ridiculous
situation wherein we could declare that an entity exercises control over another's activities only
in instances where the latter is directed by the former on each and every stage of performance
of the particular activity. Anything less than that would be tantamount to no control at all.

As an addition, it noted that 6 years of unbroken service rendered by the Respondent and affirmed his status as a
regular employee based on Art. 240 of the Labor Code.

APPEAL TO THE SC:

Petitioner’s Contention: No control.

Respondent’s Contention: Yes control.

ISSUE: Whether Respondent is the Petitioner’s Employee by virtue of the Control the latter exercised over the
former.

RULING: NO. The Labor Arbiter and the NLRC correctly found that petitioner company lacked the power of
control over the performance by respondent of his duties. The Labor Arbiter reasoned that the Comprehensive
Medical Plan, which contains the respondent's objectives, duties and obligations, does not tell respondent "how to
conduct his physical examination, how to immunize, or how to diagnose and treat his patients, employees of
[petitioner] company, in each case." It also cited Neri v. NLRC where is was stated that guidelines were not for the
purpose of controlling how the work was done, but merely to ensure that the desired results were achieved. As to
the argument that respondent was on-call for emergencies, and that there was a schedule of work that he was
bound to follow, the Court found that the schedule of work and the requirement to be on call for emergency cases
do not amount to such control, but are necessary incidents to the Retainership Agreement.

Other Notes/ SC Pronouncements:

_____________________________________________________________________

Angeles| Bajana | Balladares | Brillantes | Briones | Cabansag | Callanta | Chua | David|

De Leon | Gomez | Lopez | Macalino | Nostratis | Padilla | Reynon | Santos | Tan |Velasco
4E / 4F - 2018-2019
Page 100 of 920
LABOR REVIEW DIGEST
Atty. Joyrich Golangco

23. Melencio Gabriel rep. by surviving spouse Flordeliza V. Gabriel v. Nelson Bilon, Angel Brazil and
Ernesto Pagaygay
GR No: 146989 Date: Feb 7, 2007
Ponente: Azcuna, J.

Doctrine:
In the case of jeepney owners/operators and jeepney drivers, the former exercises supervision
and control over the latter. The fact that the drivers do not receive fixed wages but get only that in excess of the
so-called boundary [that] they pay to the owner/operator is not sufficient to withdraw the relationship between
them from that of employer and employee. Thus, private respondents were employees because they had been
engaged to perform activities which were usually necessary or desirable in the usual business or trade of the
employer.

FACTS:
Melencio Gabriel was an owner-operator of a public transport business, Gabriel Jeepney, with a fleet of
54 jeepneys plying the Baclaran-Tondo-Divisoria route, Melencia died prior to the promulgation of the LA decision
so he was substituted by his surviving wife, Flordeiza. Respondents were allegedly regular drivers of Gabriel
Jeepney under boundary system of P400 per day, plying Baclaran to Divisoria via Tondo, and vice versa, since
December 1990, November 1984 and November 1991, respectively, up to April 30, 1995, driving five days a
week, with average daily earnings of P400.
Respondents filed a complaint for illegal dismissal and illegal deductions after they were required to pay
P55 (P20police protection, P20 washing, P10 deposit, P5 garage fees), and that on Apr 30, 1995 petitioner told
them not to drive anymore.
The case was first filed on Nov. 15, 1995 with the NLRC and raffled to LA Roberto I. Santos but on Sept.
18, 1996 the case was re-raffled to LA Ricardo C. Nora after LA Santos inhibited himself from handling the case
due to personal reasons.
Petitioner-appellant claims that he does not remember if the respondents were ever under his employ as
drivers of his passenger jeepneys. Certain, however, is the fact that neither the respondents nor other drivers who
worked for him were ever dismissed by him. As a matter of fact, some of his former drivers just stopped reporting
for work, either because they found some other employment or drove for other operators, and like the
respondents, the next time he heard from them was when they started fabricating unfounded complaints against
him.

LA/RTC/NLRC RULING:
LA Nora handed down his decision on Mar. 17, 1997 declaring that herein respondents were illegally
dismissed and ordered herein petitioner to pay P1,034,000 worth of backwages to respondents (321,000 to Bilon,
391,600 to Brazil, 321,200 to Pagaygay).
On Appeal to the NLRC, the decision of the LA was reversed finding that no er-ee relationship exists (Apr
18, 1998 decision). Respondents filed an MR by virtue thereof the NLRC rendered its 2nd decision, Oct 29, 1998,
dismissing the complaint and directing the complainants to pursue their claims against the proceeding for the
settlement of the estate.
Note: Melencio Gabriel died on April 4, 1997 without receiving a copy of the first LA decision. May issue pa dito as
to timeliness of the filing of the appeal, isama ko na lang as note sa ruling part since rem topic sya

CA RULING:

Angeles| Bajana | Balladares | Brillantes | Briones | Cabansag | Callanta | Chua | David|

De Leon | Gomez | Lopez | Macalino | Nostratis | Padilla | Reynon | Santos | Tan |Velasco
4E / 4F - 2018-2019
Page 101 of 920
LABOR REVIEW DIGEST
Atty. Joyrich Golangco

Aggrieved, respondents filed a petition for certiorari with the CA which granted their petition and reversed
the NLRC ruling and reinstating the LA Mar. 17, 1997 decision subject to the modification to immediately reinstate
herein respondents.
APPEAL TO THE SC:
Petitioner now files a petition for review on certiorari which contends that the CA erred in finding that the
petitioner’s appeal to the NLRC was filed out of time, and that it erred when it reversed the NLRC ruling which had
solid legal basis.

ISSUE/S:
Whether or not an employer-employee relationship exists

HELD:
YES. With regard to the substantive aspect of the case, the Court agrees with the CA that an employer-employee
relationship existed between petitioner and respondents. In Martinez v. National Labor Relations Commission,
citing National Labor Union v. Dinglasan, the Court ruled that:
[T]he relationship between jeepney owners/operators and jeepney drivers under the boundary system is that of
employer-employee and not of lessor-lessee because in the lease of chattels the lessor loses complete control
over the chattel leased although the lessee cannot be reckless in the use thereof, otherwise he would be
responsible for the damages to the lessor. In the case of jeepney owners/operators and jeepney drivers, the
former exercises supervision and control over the latter. The fact that the drivers do not receive fixed wages but
get only that in excess of the so-called boundary [that] they pay to the owner/operator is not sufficient to withdraw
the relationship between them from that of employer and employee. Thus, private respondents were employees
because they had been engaged to perform activities which were usually necessary or desirable in the usual
business or trade of the employer.

The same principle was reiterated in the case of Paguio Transport Corporation v. NLRC.
The Court also agrees with the labor arbiter and the CA that respondents were illegally dismissed by petitioner.
Respondents were not accorded due process. Moreover, petitioner failed to show that the cause for termination
falls under any of the grounds enumerated in Article 282 (then Article 283) of the Labor Code. Consequently,
respondents are entitled to reinstatement without loss of seniority rights and other privileges and to their full
backwages computed from the date of dismissal up to the time of their actual reinstatement in accordance with
Article 279 of the Labor Code.

Reinstatement is obtainable in this case because it has not been shown that there is an ensuing strained relations
between petitioner and respondents.
Other Notes/ SC Pronouncements:
As to the timeliness of the appeal in case you guys wanted to read it:
The Court considers the service of copy of the decision of the labor arbiter to have been validly made on May 28,
1997 when it was received through registered mail. As correctly pointed out by petitioners wife, service of a copy
of the decision could not have been validly effected on April 18, 1997 because petitioner passed away on April 4,
1997.
Section 4, Rule III of the New Rules of Procedure of the NLRC provides:
SEC. 4. Service of Notices and Resolutions. (a) Notices or summons and copies of orders, resolutions or
decisions shall be served on the parties to the case personally by the bailiff or authorized public officer within
three (3) days from receipt thereof or by registered mail; Provided, That where a party is represented by counsel
or authorized representative, service shall be made on such counsel or authorized representative; Provided
further, That in cases of decision and final awards, copies thereof shall be served on both parties and their
counsel .
Angeles| Bajana | Balladares | Brillantes | Briones | Cabansag | Callanta | Chua | David|

De Leon | Gomez | Lopez | Macalino | Nostratis | Padilla | Reynon | Santos | Tan |Velasco
4E / 4F - 2018-2019
Page 102 of 920
LABOR REVIEW DIGEST
Atty. Joyrich Golangco

For the purpose of computing the period of appeal, the same shall be counted from receipt of such decisions,
awards or orders by the counsel of record.
(b) The bailiff or officer personally serving the notice, order, resolution or decision shall submit his return within
two (2) days from date of service thereof, stating legibly in his return, his name, the names of the persons served
and the date of receipt which return shall be immediately attached and shall form part of the records of the case. If
no service was effected, the serving officer shall state the reason therefore in the return.

Section 6, Rule 13 of the Rules of Court which is suppletory to the NLRC Rules of Procedure states that: [s]ervice
of the papers may be made by delivering personally a copy to the party or his counsel, or by leaving it in his office
with his clerk or with a person having charge thereof. If no person is found in his office, or his office is not known,
or he has no office, then by leaving the copy, between the hours of eight in the morning and six in the evening, at
the partys or counsels residence, if known, with a person of sufficient age and discretion then residing therein.
The foregoing provisions contemplate a situation wherein the party to the action is alive upon the delivery of a
copy of the tribunals decision. In the present case, however, petitioner died before a copy of the labor arbiters
decision was served upon him. Hence, the above provisions do not apply. As aptly stated by the NLRC:
In the case at bar, respondent Melencio Gabriel was not represented by counsel during the pendency of the case.
A decision was rendered by the Labor Arbiter a quo on March 17, 1997 while Mr. Gabriel passed away on April 4,
1997, without having received a copy thereof during his lifetime. The decision was only served on April 18, 1997
when he was no longer around to receive the same. His surviving spouse and daughter cannot automatically
substitute themselves as party respondents. Thus, when the bailiff tendered a copy of the decision to them, they
were not in a position to receive them. The requirement of leaving a copy at the partys residence is not applicable
in the instant case because this presupposes that the party is still living and is not just available to receive the
decision.

The preceding considered, the decision of the Labor Arbiter has not become final because there was no proper
service of copy thereof to party respondent.[15]

Thus, the appeal filed on behalf of petitioner on June 5, 1997 after receipt of a copy of the decision via registered
mail on May 28, 1997 was within the ten-day reglementary period prescribed under Section 223 of the Labor
Code.

24. Felix vs Buenaseda


GR No: 109704 Date: January 17, 1995
Ponente: Kapunan, J.
Doctrine:
As respondent Civil Service Commission has correctly pointed out, the appointment was for a definite
and renewable period which, when it was not renewed, did not involve a dismissal but an expiration of the
petitioner's term.
FACTS:
Petitioner, Dr. Alfredo B. Felix, joined the National Center for Mental Health (then the National Mental
Hospital) after passing the Physician’s Licensure Examinations. Respondent, Dr. Brigida Buenaseda, was the
Director and Isabelo Bañenz Jr. is the Administrator of the National Center for Mental Health.
Petitioner was promoted as Senior Resident Physician until the Ministry of Health reorganized the NCMH
pursuant to E.O. 119. Under the reorganization, he was elevated to the position of Medical Specialist I (Temporary
Status) which was renewed the following year. The DOH issued D.O. 347 which required board certification as
prerequisite for renewal of specialist positions in various medical centers and it also extend appointments of
Medical Specialist positions in cases where the termination of medical specialist who failed to meet the
requirements for board certification.

Angeles| Bajana | Balladares | Brillantes | Briones | Cabansag | Callanta | Chua | David|

De Leon | Gomez | Lopez | Macalino | Nostratis | Padilla | Reynon | Santos | Tan |Velasco
4E / 4F - 2018-2019
Page 103 of 920
LABOR REVIEW DIGEST
Atty. Joyrich Golangco

On Aug. 20, 1991, after reviewing petitioner’s service record, non-renewal of petitioner’s Medical
Specialist I position was recommended. He was allowed to continue in the service even after being informed of
the termination of his appointment.
On Nov. 25, 1991, an emergency meeting of the Chiefs of Service was held, one of the matters they
discussed was petitioner’s case. The overwhelming consensus was for petitioner’s non-renewal. The matter was
thereafter referred to the Civil Service Commission, which on Feb 28, 1992 ruled that "the temporary appointment
(of petitioner) as Medical Specialist I can be terminated at any time . . ." and that "[a]ny renewal of such
appointment is within the discretion of the appointing authority." Consequently, in a memorandum dated March
25, 1992 petitioner was advised by hospital authorities to vacate his cottage since he was no longer entitled to
accommodation.
Refusing to comply with said memorandum petitioner filed a petition with the Merit System Protection
Board ((MSPB)) complaining about the alleged harassment by respondents and questioning the non-renewal of
his appointment.
MSPB/CSC RULING:
MSPB: Dismissed the petitioner for lack of merit. Complainant, has no basis in law, to assail the non-renewal of
his expired temporary appointment much less invoke the aid of this board for that purpose since this Board cannot
substitute its judgment to that of the appointing authority nor direct the latter to issue an appointment in the
complainant's favor.

Regarding the alleged Department Order secured by the complainant from the Department of Health (DOH), the
Board finds the same inconsequential. Said Department Order merely allowed the extension of tenure of Medical
Specialist I for a certain period but does not mandate the renewal of the expired appointment.
Civil Service Commission: DISMISSED the appeal and denied the subsequent Motion for Reconsideration of
petitioner
ISSUE/S:
1. Whether or not petitioner was illegally dismissed from his position

HELD:
1. No.
The patent absurdity of petitioner's posture is readily obvious. A residency or resident physician
position in a medical specialty is never a permanent one. Residency connotes training and temporary
status. It is the step taken by a physician right after post-graduate internship (and after hurdling the
Medical Licensure Examinations) prior to his recognition as a specialist or sub-specialist in a given
field.
Petitioner is guilty or estoppels and laches. Stringent standards and requirements for renewal of
specialist-rank positions or for promotion to the next post-graduate residency year are necessary because
lives are ultimately at stake.
Petitioner's insistence on being reverted back to the status quo prior to the reorganizations made
pursuant to Executive Order No. 119 would therefore be akin to a college student asking to be sent back to
high school and staying there.
It bears emphasis that at the time of petitioner’s promotion to the position of Medical Specialist I
(temporary) in August of 1988, no objection was raised by him about the change of position or the temporary
nature of the designation. The pretense of objecting to the promotion to specialist rank apparently came only
as an afterthought, three years later, following the non-renewal of his position by the Department of Health.
There are weighty reasons of public policy and convenience which demand that any claim to any position
in the civil service, permanent, temporary or otherwise, or any claim to a violation of the constitutional
provision on security of tenure be made within a reasonable period of time. An assurance of some degree
of stability in the civil service is necessary in order to avoid needless disruptions in the conduct of
public business. Delays in the statement of a right to any position are strongly discouraged. In the same
token, the failure to assert a claim or the voluntary acceptance of another position in government, obviously
Angeles| Bajana | Balladares | Brillantes | Briones | Cabansag | Callanta | Chua | David|

De Leon | Gomez | Lopez | Macalino | Nostratis | Padilla | Reynon | Santos | Tan |Velasco
4E / 4F - 2018-2019
Page 104 of 920
LABOR REVIEW DIGEST
Atty. Joyrich Golangco

without reservation, leads to a presumption that the civil servant has either given up his claim or has already
settled into the new position. This is the essence of laches which is the failure or neglect, for an unreasonable
and unexplained length of time to do that which, by exercising due diligence, could or should have been done
earlier; it is the negligence or omission to assert a right within a reasonable time, warranting a presumption
that the party entitled to assert it either has abandoned it or declined to assert it.
Finally, it is crystal clear, from the facts of the case at bench, that the petitioner accepted a
temporary appointment (Medical Specialist I). As respondent Civil Service Commission has correctly
pointed out, the appointment was for a definite and renewable period which, when it was not renewed,
did not involve a dismissal but an expiration of the petitioner's term.

Other Notes/ SC Pronouncements


__________________________________________________________________________
25. AUTO BUS TRANSPORT SYSTEMS, INC. vs. ANTONIO BAUTISTA
G.R. No. 156367; May 16, 2005
CHICO-NAZARIO, J.

DOCTRINE: Service Incentive Leave shall not apply to employees classified as field personnel. The phrase other
employees whose performance is unsupervised by the employer must not be understood as a separate
classification of employees to which service incentive leave shall not be granted. Rather, it serves as an
amplification of the interpretation of the definition of field personnel under the Labor Code as those whose actual
hours of work in the field cannot be determined with reasonable certainty.

FACTS:

Since 24 May 1995, respondent Antonio Bautista has been employed by petitioner Auto Bus Transport
Systems, Inc. (Autobus), as driver-conductor with travel routes Manila-Tuguegarao via Baguio, Baguio-
Tuguegarao via Manila and Manila-Tabuk via Baguio. Respondent was paid on commission basis, seven percent
(7%) of the total gross income per travel, on a twice a month basis.

On 03 January 2000, while respondent was driving Autobus No. 114 along Sta. Fe, Nueva Vizcaya, the bus
he was driving accidentally bumped the rear portion of Autobus No. 124, as the latter vehicle suddenly stopped at
a sharp curve without giving any warning.

Respondent averred that the accident happened because he was compelled by the management to go back
to Roxas, Isabela, although he had not slept for almost twenty-four (24) hours, as he had just arrived in Manila
from Roxas, Isabela. Respondent further alleged that he was not allowed to work until he fully paid the amount of
P75,551.50, representing thirty percent (30%) of the cost of repair of the damaged buses and that despite
respondents pleas for reconsideration, the same was ignored by management. After a month, management sent
him a letter of termination.

Respondent instituted a Complaint for Illegal Dismissal with Money Claims for nonpayment of 13th month pay
and service incentive leave pay against Autobus. Petitioner, on the other hand, maintained that respondents
employment was replete with offenses involving reckless imprudence, gross negligence, and dishonesty. To
support its claim, petitioner presented copies of letters, memos, irregularity reports, and warrants of arrest
pertaining to several incidents wherein respondent was involved.

LA RULING: Dismissed the complaint but ordered the petitioner to pay among others the respondent’s service
incentive leave pay for all the years he had been in service with the respondent, presently computed at
P13,788.05.

Angeles| Bajana | Balladares | Brillantes | Briones | Cabansag | Callanta | Chua | David|

De Leon | Gomez | Lopez | Macalino | Nostratis | Padilla | Reynon | Santos | Tan |Velasco
4E / 4F - 2018-2019
Page 105 of 920
LABOR REVIEW DIGEST
Atty. Joyrich Golangco

NLRC RULING: Maintained the award for service incentive leave.

CA RULING: Affirmed NLRC’s ruling.

ISSUE:
Whether or not the respondent is entitled to service incentive leave.

RULING:
YES. The grant of service incentive leave has been delimited by the Implementing Rules and
Regulations of the Labor Code to apply only to those employees not explicitly excluded by Section 1 of Rule V.
According to the Implementing Rules, Service Incentive Leave shall not apply to employees classified as field
personnel. The phrase other employees whose performance is unsupervised by the employer must not be
understood as a separate classification of employees to which service incentive leave shall not be granted.
Rather, it serves as an amplification of the interpretation of the definition of field personnel under the Labor Code
as those whose actual hours of work in the field cannot be determined with reasonable certainty.

The same is true with respect to the phrase those who are engaged on task or contract basis, purely
commission basis. Said phrase should be related with field personnel, applying the rule on ejusdem generis that
general and unlimited terms are restrained and limited by the particular terms that they follow. Hence, employees
engaged on task or contract basis or paid on purely commission basis are not automatically exempted from the
grant of service incentive leave, unless, they fall under the classification of field personnel.Therefore, petitioners
contention that respondent is not entitled to the grant of service incentive leave just because he was paid on
purely commission basis is misplaced. What must be ascertained in order to resolve the issue of propriety of the
grant of service incentive leave to respondent is whether or not he is a field personnel.

As observed by the Labor Arbiter and concurred in by the Court of Appeals:It is of judicial notice that
along the routes that are plied by these bus companies, there are its inspectors assigned at strategic places who
board the bus and inspect the passengers, the punched tickets, and the conductors reports. There is also the
mandatory once-a-week car barn or shop day, where the bus is regularly checked as to its mechanical, electrical,
and hydraulic aspects, whether or not there are problems thereon as reported by the driver and/or conductor.
They too, must be at specific place as [sic] specified time, as they generally observe prompt departure and arrival
from their point of origin to their point of destination. In each and every depot, there is always the Dispatcher
whose function is precisely to see to it that the bus and its crew leave the premises at specific times and arrive at
the estimated proper time. These, are present in the case at bar. The driver, the complainant herein, was
therefore under constant supervision while in the performance of this work. He cannot be considered a field
personnel. Therefore, as correctly concluded by the appellate court, respondent is not a field personnel but a
regular employee who performs tasks usually necessary and desirable to the usual trade of petitioners business.
Accordingly, respondent is entitled to the grant of service incentive leave.

Other Notes/ SC Pronouncements:

_____________________________________________________________________

Angeles| Bajana | Balladares | Brillantes | Briones | Cabansag | Callanta | Chua | David|

De Leon | Gomez | Lopez | Macalino | Nostratis | Padilla | Reynon | Santos | Tan |Velasco
4E / 4F - 2018-2019
Page 106 of 920
LABOR REVIEW DIGEST
Atty. Joyrich Golangco

26. David vs. Macasio


G.R. No. 195466; July 2, 2014
Brion, J.

FACTS: Macasio filed before the LA a complaint against petitioner Ariel David, doing business under the name
and style "Yiels Hog Dealer," for non-payment of SIL, holiday pay and 13th month pay.

Macasio alleged before the LA that he had been working as a butcher for David since January 6, 1995. Macasio
claimed that David exercised effective control and supervision over his work, pointing out that David: (1) set the
work day, reporting time and hogs to be chopped, as well as the manner by which he was to perform his work; (2)
daily paid his salary of ₱700; and (3) approved and disapproved his leaves.

In his defense, David alleged that he hired Macasio as a butcher or chopper on "pakyaw" or task basis who is,
therefore, not entitled to overtime pay, holiday pay and 13th month pay pursuant to the provisions of the IRR of
the Labor Code.

Refuting Macasio’s submissions, David claims that Macasio was not his employee as he hired the latter on
"pakyaw" or task basis. He also claimed that he issued the Certificate of Employment, upon Macasio’s request,
only for overseas employment purposes.

LA dismissed Macasio’s complaint for lack of merit. The LA gave credence to David’s claim that he engaged
Macasio on "pakyaw" or task basis. As Macasio was engaged on "pakyaw" or task basis, he is not entitled to
overtime, holiday, SIL and 13th month pay.

NLRC affirmed the LA ruling.

While the CA agreed with the LA and the NLRC that Macasio was a task basis employee, it nevertheless found
Macasio entitled to his monetary claims. The CA explained that as a task basis employee, Macasio is excluded
from the coverage of holiday, SIL and 13th month pay only if he is likewise a "field personnel."

ISSUE 1: whether or not an employer-employee relationship exists between Macasio and David
ISSUE 2: whether or not Macasio is entitled to his monetary claims

HELD 1: YES. At the outset, we reject this assertion of the petitioner. Engagement on "pakyaw" or task basis does
not characterize the relationship that may exist between the parties.

First, the LA and the NLRC denied Macasio’s claim not because of the absence of an employer-employee but
because of its finding that since Macasio is paid on pakyaw or task basis, then he is not entitled to SIL, holiday
and 13th month pay. In other words, aside from being factual in nature, the existence of an employer-employee
relationship is in fact a non-issue in this case.

Angeles| Bajana | Balladares | Brillantes | Briones | Cabansag | Callanta | Chua | David|

De Leon | Gomez | Lopez | Macalino | Nostratis | Padilla | Reynon | Santos | Tan |Velasco
4E / 4F - 2018-2019
Page 107 of 920
LABOR REVIEW DIGEST
Atty. Joyrich Golangco

At any rate, even if we indulge the petitioner, we find his claim that no employer-employee relationship exists
baseless. Employing the control test, we find that such a relationship exist in the present case.

To determine the existence of an employer-employee relationship, four elements generally need to be considered,
namely: (1) the selection and engagement of the employee; (2) the payment of wages; (3) the power of dismissal;
and (4) the power to control the employee’s conduct.
First, David engaged the services of Macasio, thus satisfying the element of selection and engagement of the
employee. Second, David paid Macasio’s wages. Third, David had been setting the day and time when Macasio
should report for work. This power to determine the work schedule obviously implies power of control. And fourth,
David had the right and power to control and supervise Macasio’s work as to the means and methods of
performing it.

HELD 2: YES for Holiday and SIL pays. Under labor code, the general rule is that holiday and SIL pay provisions
cover all employees. To be excluded from their coverage, an employee must be one of those that these provisions
expressly exempt, strictly in accordance with the exemption. Under the IRR, exemption from the coverage of
holiday and SIL pay refer to field personnel and other employees whose time and performance is unsupervised by
the employer including those who are engaged on task or contract basis. The payment of an employee on task or
pakyaw basis alone is insufficient to exclude one from the coverage of SIL and holiday pay. They are exempted
from the coverage of Title I (including the holiday and SIL pay) only if they qualify as "field personnel."

Macasio does not fall under the classification of "field personnel." Based on the definition of field personnel under
Article 82, we agree with the CA that Macasio does not fall under the definition of "field personnel." First, Macasio
regularly performed his duties at David’s principal place of business; second, his actual hours of work could be
determined with reasonable certainty; and, third, David supervised his time and performance of duties. Since
Macasio cannot be considered a "field personnel," then he is not exempted from the grant of holiday, SIL pay
even as he was engaged on "pakyaw" or task basis.

NO for 13th month pay. Note that unlike the IRR of the Labor Code on holiday and SIL pay, Section 3(e) of the
Rules and Regulations Implementing PD No. 851 exempts employees "paid on task basis" without any reference
to "field personnel." This could only mean that insofar as payment of the 13th month pay is concerned, the law did
not intend to qualify the exemption from its coverage with the requirement that the task worker be a "field
personnel" at the same time.

Other Notes/ SC Pronouncements:

_____________________________________________________________________

Angeles| Bajana | Balladares | Brillantes | Briones | Cabansag | Callanta | Chua | David|

De Leon | Gomez | Lopez | Macalino | Nostratis | Padilla | Reynon | Santos | Tan |Velasco
4E / 4F - 2018-2019
Page 108 of 920
LABOR REVIEW DIGEST
Atty. Joyrich Golangco

27. Begino vs ABS-CBN


GR No: 199166 Date: April 20, 2015
Ponente: Perez, J

Doctrine: To determine the existence of said relation, case law has consistently applied the four-fold test, to wit:

(a) the selection and engagement of the employee;


(b) the payment of wages;
(c) the power of dismissal; and
(d) the employer's power to control the employee on the means and methods by which the work is accomplished.

Of these criteria, the so-called "control test" is generally regarded as the most crucial and determinative indicator
of the presence or absence of an employer-employee relationship. Under this test, an employer-employee
relationship is said to exist where the person for whom the services are performed reserves the right to control
not only the end result but also the manner and means utilized to achieve the same.

FACTS:

Respondent ABS-CBN, through Respondent Villafuerte, engaged the services of Petitioners as cameramen,
editors or reporters for TV Broadcasting. Petitioners signed regularly renewed Talent Contracts (3 months - 1
year) and Project Assignment Forms which detailed the duration, budget and daily technical requirements of a
particular project. Petitioners were tasked with coverage of news items for subsequent daily airings in
Respondents’ TV Patrol Bicol Program.

The Talent Contract has an exclusivity clause and provides that nothing therein shall be deemed or construed to
establish an employer-employee relationship between the parties.

Petitioners filed against Respondents a complaint for regularization before the NLRC's Arbitration branch.

While the case was pending, Petitioners contracts were terminated, prompting the latter to file a second complaint
for illegal dismissal.

LA/RTC/NLRC RULING:

Arbitration Branch ruled that Petitioners were regular employees stating that the latter rendered services
necessary and related to ABS CBN’s business for more than a year as its regular employees, thus, ordering
Respondents to reinstate the Petitioners. The NLRC affirmed the ruling.

CA RULING:

No ER-EE relationship, CA overturned the decision

Petitioner's Contention:

Petitioners claimed that they worked under the direct control of Respondent Villafuerte - they were mandated to
wear company IDs, they were provided the necessary equipment, they were informed about the news to be
covered the following day, and they were bound by the company’s policy on attendance and punctuality.

Angeles| Bajana | Balladares | Brillantes | Briones | Cabansag | Callanta | Chua | David|

De Leon | Gomez | Lopez | Macalino | Nostratis | Padilla | Reynon | Santos | Tan |Velasco
4E / 4F - 2018-2019
Page 109 of 920
LABOR REVIEW DIGEST
Atty. Joyrich Golangco

Respondent's Contention:

Respondents countered that, pursuant to their Talent Contracts and Project Assignment Forms, Petitioners were
hired as talents to act as reporters, editors and/or cameramen. Respondents further claimed they never imposed
control as to how Petitioners discharged their duties. At most, they were briefed regarding the general
requirements of the project to be executed.

ISSUE/S:

W/N Petitioners are regular employees of Respondents.

HELD:

Yes. Of the criteria to determine whether there is an employer-employee relationship, the so-called "control test" is
generally regarded as the most crucial and determinative indicator of the said relationship.

Under this test, an employer-employee relationship is said to exist where the person for whom the services are
performed reserves the right to control not only the end result but also the manner and means utilized to achieve
the same.

Notwithstanding the nomenclature of their Talent Contracts and/or Project Assignment Forms and the terms and
condition embodied therein, petitioners are regular employees of ABS-CBN.

As cameramen, editors and reporters, it appears that Petitioners were subject to the control and supervision of
Respondents which provided them with the equipment essential for the discharge of their functions. The
exclusivity clause and prohibitions in their Talent Contract were likewise indicative of Respondents' control over
them, however obliquely worded.

Also, the presumption is that when the work done is an integral part of the regular business of the employer and
when the worker does not furnish an independent business or professional service, such work is a regular
employment of such employee and not an independent contractor.

Other Notes/ SC Pronouncements:

_____________________________________________________________________

22. Chevron Phils. vs Galit


GR No: 186114 Date: October 7, 2015
Ponente: Peralta, J.
Doctrine:
Under the control test, an employer-employee relationship exists where the person for whom the
services are performed reserves the right to control not only the end achieved, but also the
manner and means to be used in reaching that end.
Angeles| Bajana | Balladares | Brillantes | Briones | Cabansag | Callanta | Chua | David|

De Leon | Gomez | Lopez | Macalino | Nostratis | Padilla | Reynon | Santos | Tan |Velasco
4E / 4F - 2018-2019
Page 110 of 920
LABOR REVIEW DIGEST
Atty. Joyrich Golangco

FACTS:
Respondent, Vitaliano C. Galit, filed a complaint against Chevron (Phils) Inc., formerly Caltex Philippines
Inc., Sons Construction Corporation (SJS), and its president Reynaldo Salomon for alleged illegal dismissal. SJS,
in its position position paper, claims that it is a company engaged in the business of providing manpower to its
clients on a “per project/contract” basis.
Respondent alleged that he was a regular employee of Chevron since 1982, having been assigned in its
Pandacan depot as an “all around employee” and that he was directly under the control and supervision of
Chevron supervisors. On Jan. 15, 2005, he was verbally informed that his employment is terminated but was
promised that he will be reinstated soon. He followed up on his reinstatement but he never got his job back.
The complaint for illegal dismissal was filed with the NLRC, NCR, North Sector Branch in Quezon City.
In petitioner’s position paper with Motion to Dismiss, it contends that it entered into two (2) contracts for
janitorial services with SJS from May 1, 2001 to April 30, 2003 and from June 1, 2003 to June 1, 2004; under
these contracts, SJS undertook to "assign such number of its employees, upon prior agreement with [petitioner],
as would be sufficient to fully and effectively render the work and services undertaken" and to "supply the
equipment, tools and materials, which shall, by all means, be effective and efficient, at its own expense,
necessary for the performance" of janitorial services; Galit, who was employed by SJS, was assigned to
petitioner's Pandacan depot as a janitor; his wages and all employment benefits were paid by SJS; he was subject
to the supervision, discipline and control of SJS; on November 30, 2004, the extended contract between petitioner
and SJS expired; subsequently, a new contract for janitorial services was awarded by petitioner to another
independent contractor; petitioner was surprised that Galit filed an action impleading it.

LA/RTC/NLRC RULING:
The LA dismissed the complaint against Chevron for lack of jurisdiction and as against SJS and
Salomon for lack of merit but ordered SJS to pay Galit separation pay at the rate of half-month salary for every
year of service that Galit had with SJS. The Labor Arbiter held that SJS was a legitimate contractor and that it was
Galit’s employer.
On appeal to the NLRC, the decision was affirmed although the amount was modified to 1 month
salary for every year of service. A motion for consideration was filed but was denied by the NLRC.

CA RULING:
The CA reversed the decision of the NLRC finding that SJS was a labor-only contractor and that
petitioner was Galit’s actual employer and that the latter was unjustly dismissed. A motion for reconsideration was
filed by petitioner but was denied by the CA.

APPEAL TO THE SC:


A petition for review on Certiorari was filed by herein petitioner assailing the decision of the CA

Petitioner's Contention:
That the CA erred when it found that an employer-employee relationship exists between herein petitioner
and respondent and that SJS was a labor-only contractor.

Respondent's Contention:
That petitioner was his employer and that he was illegally dismissed.

ISSUE/S:
1. Whether a employer-employee relationship exists between Chevron and Galit
2. Whether or not SJS was a labor-only contractor

HELD:
Angeles| Bajana | Balladares | Brillantes | Briones | Cabansag | Callanta | Chua | David|

De Leon | Gomez | Lopez | Macalino | Nostratis | Padilla | Reynon | Santos | Tan |Velasco
4E / 4F - 2018-2019
Page 111 of 920
LABOR REVIEW DIGEST
Atty. Joyrich Golangco

1. None.
To ascertain the existence of an employer-employee relationship, jurisprudence has invariably
adhered to the four-fold test, to wit: (1) the selection and engagement of the employee; (2) the
payment of wages; (3) the power of dismissal; and (4) the power to control the employee's
conduct, or the so-called "control test." Of these four, the last one is the most important. The so-
called "control test" is commonly regarded as the most crucial and determinative indicator of the
presence or absence of an employer-employee relationship. Under the control test, an employer-
employee relationship exists where the person for whom the services are performed reserves the
right to control not only the end achieved, but also the manner and means to be used in reaching
that end.

4.1 In the fulfillment of its obligations to the COMPANY, the CONTRACTOR shall select and
hire its workers. The CONTRACTOR alone shall be responsible for the payment of their wages
and other employment benefits and likewise for the safeguarding of their health and safety in
accordance with existing laws and regulations. Likewise, the CONTRACTOR shall be
responsible for the discipline and/or dismissal of these workers.
4.2 The CONTRACTOR shall retain the right to control the manner and the means of performing
the work, with the COMPANY having the control or direction only as to the results to be
accomplished.
5.1 The CONTRACTOR shall maintain efficient and effective discipline over any and all
employees it may utilize in performing its obligations under this CONTRACT. . . .
6.1 The CONTRACTOR shall at its own expense maintain with a reputable insurance company,
acceptable to the COMPANY, a comprehensive liability insurance in the amount required by the
COMPANY to cover claims for bodily injury, death or property damage caused to any person or
persons by an act or omission of the CONTRACTOR or any of its employees, agents or
representatives.

The foregoing provisions of the Job Contract between petitioner and SJS demonstrate that the
latter possessed the following earmarks of an employer, to wit: (1) the power of selection and
engagement of employees, under Sections 4.1 and 6.1 (d); (2) the payment of wages, under Sections
4.1 and 6.1 (c); (3) the power to discipline and dismiss, under Section 4.1; and, (4) the power to control
the employee's conduct, under Sections 4.1, 4.2, and 5.1. As to SJS' power of selection and
engagement, Galit himself admitted in his own affidavit that it was SJS which assigned him to work at
Chevron's Pandacan depot. As such, there is no question that it was SJS which selected and engaged
Galit as its employee.

2. No, SJS is a legitimate contractor.


As to whether or not SJS is an independent contractor, jurisprudence has invariably ruled that
an independent contractor carries on an independent business and undertakes the contract work on his
own account, under his own responsibility, according to his own manner and method, and free from the
control and direction of his employer or principal in all matters connected with the performance of the
work except as to the results thereof. This embodies what has long been jurisprudentially recognized as
the control test, as discussed above. In the instant case, SJS presented evidence to show that it had an
independent business by paying business taxes and fees and that it was registered as an employer with
the Social Security System. Moreover, there was no evidence to show that SJS and its employees were
ever subject to the control of petitioner. On the contrary, as shown above, SJS possessed the right to
control its employees' manner and means of performing their work, including herein respondent Galit.

Other Notes/ SC Pronouncements:


Angeles| Bajana | Balladares | Brillantes | Briones | Cabansag | Callanta | Chua | David|

De Leon | Gomez | Lopez | Macalino | Nostratis | Padilla | Reynon | Santos | Tan |Velasco
4E / 4F - 2018-2019
Page 112 of 920
LABOR REVIEW DIGEST
Atty. Joyrich Golangco

______________________________________________________________________

23. Manila Memorial Park v Lluz


GR No: 208451 Date: February 3, 2016
Ponente:

Doctrine:
Labor-only contracting exists when the contractor or subcontractor merely recruits, supplies or
places workers to perform a job, work or service for a principal and any of the following elements are
present:
1) The contractor or subcontractor does not have substantial capital or investment which relates
to the job, work or service to be performed and the employees recruited, supplied or placed by
such contractor or subcontractor are performing activities which are directly related to the main
business of the principal; or
2) The contractor does not exercise the right to control the performance of the work of the
contractual employee.

FACTS:

On 23 February 2006, petitioner Manila Memorial Park Cemetery, Inc. (Manila Memorial) entered into a
Contract of Services with respondent Ward Trading and Services (Ward Trading). The Contract of Services
provided that Ward Trading, as an independent contractor, will render interment and exhumation services and
other related work to Manila Memorial in order to supplement operations at Manila Memorial Park, Parañaque
City.
Among those assigned by Ward Trading to perform services at the Manila Memorial Park were
respondents Ezard Lluz, Norman Corral, Erwin Fugaban, Valdimar Balisi, Emilio Fabon, John Mark Aplicador,
Michael Curioso, Junlin Espares, and Gavino Farinas (respondents). They worked six days a week for eight hours
daily and were paid P250 per day.
Respondents alleged that they asked Manila Memorial to consider them as regular workers within the
appropriate bargaining unit established in the collective bargaining agreement by Manila Memorial and its union,
the Manila Memorial Park Free Workers Union (MMP Union). Manila Memorial refused the request since
respondents were employed by Ward Trading, an independent labor contractor. Thereafter, respondents joined
the MMP Union. The MMP Union, on behalf of respondents, sought their regularization which Manila Memorial
again declined. Respondents then filed the complaint. Subsequently, respondents were dismissed by Manila
Memorial. Thus, respondents amended the complaint to include the prayer for their reinstatement and payment of
back wages.
Manila Memorial sought the dismissal of the complaint for lack of jurisdiction since there was no
employer-employee relationship. Manila Memorial argued that respondents were the employees of Ward Trading.

LA/RTC/NLRC RULING:
LA: Dismissed the petition for failure to prove the existence of employer-employee relationship.

NLRC: On Appeal to the NLRC, the decision of the LA was reversed and it found that Ward Trading was a labor-
only contractor and an agent of Manila Memorial. A Motion for Reconsideration was filed but was denied by the
NLRC.

CA RULING:

Angeles| Bajana | Balladares | Brillantes | Briones | Cabansag | Callanta | Chua | David|

De Leon | Gomez | Lopez | Macalino | Nostratis | Padilla | Reynon | Santos | Tan |Velasco
4E / 4F - 2018-2019
Page 113 of 920
LABOR REVIEW DIGEST
Atty. Joyrich Golangco

The CA AFFIRMED the ruling of the NLRC finding that there exists an employer-employee relationship
between Manila Memorial and respondents. A MR was filed but was denied by the CA.

APPEAL TO THE SC:


A petition for review on certiorari was filed based on the following issue

Petitioner’s Contention:
Manila Memorial contends that Ward Trading has total assets in excess of P1.4 million, according to
Ward Trading's financial statements for the year 2006, proving that it has sufficient capitalization to qualify as a
legitimate independent contractor. Manila Memorial insists that nowhere is it provided in the Contract of Services
that Manila Memorial controls the manner and means by which respondents accomplish the results of their work.
Manila Memorial states that the company only wants its contractors and the latter's employees to abide by
company rules and regulations.

Respondents’ Contention:
Respondents, on the other hand, assert that they are regular employees of Manila Memorial since Ward
Trading cannot qualify as an independent contractor but should be treated as a mere labor-only contractor.
Respondents state that (1) there is enough proof that Ward Trading does not have substantial capital, investment,
tools and the like; (2) the workers recruited and placed by the alleged contractors performed activities that were
related to Manila Memorial's business; and (3) Ward Trading does not exercise the right to control the
performance of the work of the contractual employees.

ISSUE/S:
1. Whether or not an employer-employee relationship exists between petitioner and respondents for
the latter to be entitled to their claim for wages and other benefits.

HELD:
1. Yes. The petition lacks merit
Labor-only contracting exists when the contractor or subcontractor merely recruits,
supplies or places workers to perform a job, work or service for a principal and any of the
following elements are present:

1) The contractor or subcontractor does not have substantial capital or investment which
relates to the job, work or service to be performed and the employees recruited, supplied
or placed by such contractor or subcontractor are performing activities which are
directly related to the main business of the principal; or

2) The contractor does not exercise the right to control the performance of the work of
the contractual employee.
In the present case, Manila Memorial entered into a Contract of Services with Ward Trading, a
single proprietorship owned by Emmanuel Mayor Ward with business address in Las Piñas City on 23
February 2006. In the Contract of Services, it was provided that Ward Trading, as the contractor, had
adequate workers and substantial capital or investment in the form of tools, equipment, machinery, work
premises and other materials which were necessary in the conduct of its business.
However, a closer look at the Contract of Services reveals that Ward Trading does not have
substantial capital or investment in the form of tools, equipment, machinery, work premises and other
materials since it is Manila Memorial which owns the equipment used in the performance of work needed
for interment and exhumation services.
Angeles| Bajana | Balladares | Brillantes | Briones | Cabansag | Callanta | Chua | David|

De Leon | Gomez | Lopez | Macalino | Nostratis | Padilla | Reynon | Santos | Tan |Velasco
4E / 4F - 2018-2019
Page 114 of 920
LABOR REVIEW DIGEST
Atty. Joyrich Golangco

The NLRC also found that Ward Trading's business documents fell short of sound business
practices For failing to register as a contractor, a presumption arises that one is engaged in labor-only
contracting unless the contractor overcomes the burden of proving that it has substantial capital,
investment, tools and the like.
In this case, however, Manila Memorial failed to adduce evidence to prove that Ward Trading
had any substantial capital, investment or assets to perform the work contracted for. Thus, the
presumption that Ward Trading is a labor-only contractor stands. Consequently, Manila Memorial is
deemed the employer of respondents. As regular employees of Manila Memorial, respondents are
entitled to their claims for wages and other benefits as awarded by the NLRC and affirmed by the CA.
Other Notes/ SC Pronouncements:
_____________________________________________________________________

24. Diamond Farms, Inc. v SPFL et al.


GR Nos.: 173254-55 & 173263 Date: January 13, 2016

Ponente: Jardeleza, J.

DOCTRINE: A finding that a contractor is a labor-only contractor is equivalent to a declaration that there is an
employer-employee relationship between the principal, and the workers of the labor-only contractor; the labor-only
contractor is deemed only as the agent of the principal.

FACTS:

DFI owns an 800-hectare banana plantation ("original plantation") in Alejal, Carmen, Davao. Pursuant to Republic
Act No. 6657 or the Comprehensive Agrarian Reform Law of 1988 ("CARL"), commercial farms shall be subject to
compulsory acquisition and distribution, thus the original plantation was covered by the law. DFI offered to give up
its rights and interest over the original plantation in favor of the government by way of a Voluntary Offer to Sell.
The DAR accepted DFI's offer to sell the original plantation. Out of the total 800 hectares, the DAR only approved
the disposition of 689.88 hectares.

The awarded plantation was turned over to qualified agrarian reform beneficiaries ("ARBs") under the CARL.
These ARBs are the same farmers who were working in the original plantation. They subsequently organized
themselves into a multi-purpose cooperative named "DARBMUPCO," which is one of the respondents in this
case.

DARBMUPCO entered into a Banana Production and Purchase Agreement ("BPPA") with DFI. Under the BPPA,
DARBMUPCO and its members as owners of the awarded plantation, agreed to grow and cultivate only high
grade quality exportable bananas to be sold exclusively to DPI. The BPPA is effective for 10 years.18

Hampered by lack of manpower to undertake the agricultural operation under the BPPA, DFI engaged the
services of the respondent-contractors, who in turn recruited the respondent-workers to assist DARBMUPCO in
meeting its production obligations under the BPPA,

Southern Philippines Federation of Labor ("SPFL")—a legitimate labor organization with a local chapter in the
awarded plantation filed a petition for certification election in the Office of the Med-Arbiter on behalf of some 400
workers (the respondent-workers in this petition) "jointly employed by DFI and DARBMUPCO" working in the
awarded plantation.
Angeles| Bajana | Balladares | Brillantes | Briones | Cabansag | Callanta | Chua | David|

De Leon | Gomez | Lopez | Macalino | Nostratis | Padilla | Reynon | Santos | Tan |Velasco
4E / 4F - 2018-2019
Page 115 of 920
LABOR REVIEW DIGEST
Atty. Joyrich Golangco

In another case, SPFL, together with more than 300 workers, filed a case for underpayment of wages, non-
payment of 13th month pay and service incentive leave pay and attorney's fees against DFI, DARBMUPCO and
the respondent-contractors before the National Labor Relations Commission ("NLRC").

DARBMUPCO and DFI denied that they are the employers of the respondent-workers. They claimed, instead, that
the respondent-workers are the employees of the respondent-contractors.

LABOR ARBITER & NLRC RULING:


The Labor Arbiter held that the respondent-contractors are "labor-only contractors." The LA gave credence to the
affidavits of the other contractors of DFI (who are not party-respondents in this petition) asserting that DFI
engaged their services, and supervised and paid their laborers. The affidavits also stated that the contractors had
no dealings with DARBMUPCO, except that their work is done in the awarded plantation.

The LA held that, under the law, DFI is deemed as the statutory employer of all the respondent-workers.The LA
dismissed the case against DARBMUPCO and the respondent-contractors.

the NLRC modified the Decision of the LA and declared that DARBMUPCO and DFI are the statutory employers
of the workers rendering services in the awarded plantation and the managed area, respectively. It adjudged DFI
and DARBMUPCO as solidarity liable with the respondent-contractors for the monetary claims of the workers, in
proportion to their net planted area

ISSUE/S:
1. Who among DFI, DARBMUPCO and the respondent-contractors is the employer of the respondent-workers?

HELD:
Petition is denied. Furthermore, the decision of the Court of Appeals declaring DFI to be the employer of
respondent-workers is affirmed.

This case involves job contracting, a labor arrangement expressly allowed by law. Contracting or subcontracting is
an arrangement whereby a principal (or employer) agrees to put out or farm out with a contractor or subcontractor
the performance or completion of a specific job, work or service within a definite or predetermined period,
regardless of whether such job, work or service is to be performed or completed within or outside the premises of
the principal.

The Omnibus Rules Implementing the Labor Code distinguishes between permissible job contracting (or
independent contractorship) and labor-only contracting. Job contracting is permissible under the Code if the
following conditions are met:

(1) The contractor carries on an independent business and undertakes the contract work on his own
account under his own responsibility according to his own manner and method, free from the control and
direction of his employer or principal in all matters connected with the performance of the work except as to
the results thereof; and
(2) The contractor has substantial capital or investment in the form of tools, equipment, machineries,
work premises, and other materials which are necessary in the conduct of his business.

In contrast, job contracting shall be deemed as labor-only contracting, an arrangement prohibited by law, if a
person who undertakes to supply workers to an employer:
(1) Does not have substantial capital or investment in the form of tools, equipment, machineries, work
premises and other materials; and
Angeles| Bajana | Balladares | Brillantes | Briones | Cabansag | Callanta | Chua | David|

De Leon | Gomez | Lopez | Macalino | Nostratis | Padilla | Reynon | Santos | Tan |Velasco
4E / 4F - 2018-2019
Page 116 of 920
LABOR REVIEW DIGEST
Atty. Joyrich Golangco

(2) The workers recruited and placed by such person are performing activities which are directly related
to the principal business or operations of the employer in which workers are habitually employed.

Based on the conditions for permissible job contracting, we rule that respondent-contractors are labor only
contractors.

There is no evidence showing that respondent-contractors are independent contractors. The respondent-
contractors, DFI, and DARBMUPCO did not offer any proof that respondent-contractors were not engaged in
labor-only contracting.

Herein respondents, Voltaire Lopez, Jr., et al., were commissioned and contracted by petitioner, Diamond Farms,
Inc. (DFI) to recruit farm workers, who are the complaining [respondent-workers] (as represented by Southern
Philippines Federation of Labor (SPFL) in this appeal by certiorari).

Farm tools, implements and equipment necessary to performance of such farm activities were supplied by
petitioner DFI. Herein respondents Voltaire Lopez, Jr. et. al. had no adequate capital to acquire or purchase such
tools, implements, equipment, etc.

Herein respondents Voltaire Lopez, Jr., et. al. as well as rcspondents-SPFL, et. al. were being directly supervised,
controlled and managed by petitioner DFI farm managers and supervisors, specifically on work assignments and
performance targets.

A finding that a contractor is a labor-only contractor is equivalent to a declaration that there is an employer-
employee relationship between the principal, and the workers of the labor-only contractor; the labor-only
contractor is deemed only as the agent of the principal. Thus, in this case, respondent-contractors are the labor-
only contractors and either DFI or DARBMUPCO is their principal.

We hold that DFI is the principal. The records show that it is DFI which hired the individual [respondent-
contractors] who in turn hired their own men to work in the 689.88 hectares land of DARBMUPCO as well as in
the managed area of the plantation.

DFI cannot argue that DARBMUPCO is the principal of the respondent-contractors because it (DARBMUPCO)
owns the awarded plantation where respondent-contractors and respondent-workers were working. That
DARBMUPCO owns the awarded plantation where the respondent-contractors and respondent-workers were
working is immaterial.DFI, as the principal, hired the respondent-contractors and the latter, in turn, engaged the
services of the respondent-workers.

Clearly, DFI is the true employer of the respondent-workers; respondent-contractors are only agents of DFI. Under
Article 106 of the Labor Code, DFI shall be solidarily liable with the respondent-contractors for the rightful claims
of the respondent-workers, to the same manner and extent, as if the latter are directly employed by DFI.
Other Notes/ SC Pronouncements:

_____________________________________________________________________

Angeles| Bajana | Balladares | Brillantes | Briones | Cabansag | Callanta | Chua | David|

De Leon | Gomez | Lopez | Macalino | Nostratis | Padilla | Reynon | Santos | Tan |Velasco
4E / 4F - 2018-2019
Page 117 of 920
LABOR REVIEW DIGEST
Atty. Joyrich Golangco

25. Sumifru Corp v. NAMASUFA


GR No: 202091 Date: June 7, 2017
Ponente: Caguioa, J.

Employer-Employee Relationship
Doctrine:
The factual findings by quasi-judicial agencies, such as the Department of Labor and Employment, when
supported by substantial evidence, are entitled to great respect in view of their expertise in their respective fields.
Judicial review of labor cases does not go so far as to evaluate the sufficiency of evidence on which the labor
official's findings rest. It is not our (the Supreme Court) function to assess and evaluate all over again the
evidence, testimonial and documentary, adduced by the parties to an appeal, particularly where the findings of
both the trial court (here, the DOLE Secretary) and the appellate court on the matter coincide, as in this case at
bar. The Rule limits that function of the Court to the review or revision of errors of law and not to a second
analysis of the evidence.

FACTS:

Petitioner Sumifru is the surviving corporation when it merged Fresh Banana Agricultural Corporation (FBAC) in
2008. FBAC is engaged in the buying, marketing, and exportation of Cavendish Bananas. On the other hand,
respondent NAGKAHIUSANG MAMUMUO SA SUYAPA FARM (NAMASUFA-NAFLU-KMU) is a labor
organization affiliated with the National Federation of Labor Unions and Kilusang Mayo Uno. On March 14, 2008,
respondent filed a petition for Certification Election with the DOLE to represent employees numbering around 140
of Packing Plant 90 of FBAC. However, FBAC filed an opposition to the petition, arguing that no employer-
employee relationship exists between it and respondent. It further alleged that the members of respondent
NAMASUFA are actually employees of A2Y Contracting Services (A2Y), a duly licensed independent contractor.

Respondent NAMASUFA, on the other hand, countered that its members were former Stanfilco workers before
FBAC took over the former’s operations. Their members were then required to join the Compostela Banana
Packing Plant Workers’ Cooperative (CBPPWC) before being hired and allowed to work at the Packing Plant of
FBAC, and that the members were working at Packing Plant 90 long before A2Y came.

During the pendency of the resolution on the petition for certification election, FBAC merged with Sumifru

DOLE MED-ARBITER RULING:


The DOLE Med-Arbiter granted the petition for certification election and ruled that an employer-employee
relationship existed between FBAC (Now Sumifru). Applying the four-fold test, it held:

“The "four-fold test" will show that respondent FBAC is the employer of petitioner's members. The elements to
determine the existence of an employment relationship are: (a) the selection and engagement of the employee;
(b) the payment of wages; (c) the power of dismissal; and (d) the employer's power to control the employee's
conduct. The most important element is the employer's control of the employee's conduct, not only as to the result
of the work to be done, but also as to the means and methods to accomplish it.

Angeles| Bajana | Balladares | Brillantes | Briones | Cabansag | Callanta | Chua | David|

De Leon | Gomez | Lopez | Macalino | Nostratis | Padilla | Reynon | Santos | Tan |Velasco
4E / 4F - 2018-2019
Page 118 of 920
LABOR REVIEW DIGEST
Atty. Joyrich Golangco

On the first factor, (selection and engagement of the employer), it is apparent that the staff of respondent FBAC
advised those who are interested to be hired in the Packing Plant to become members first of CBPPWC and get a
recommendation from it.

On the second factor (payment of wages), while the respondent tried to impress upon us that workers are paid by
A2Y Contracting Services, this at best is but an administrative arrangement. We agree with petitioner that the
payroll summary submitted does not contain the relevant information such as the employee's rate of pay,
deductions made and the amount actually paid to the employee.

On the third factor, (the power of dismissal), it is very clear that respondent FBAC is the authority that imposes
disciplinary measures against erring workers. This alone proves that it wields disciplinary authority over them.

Finally, on the fourth factor which is the control test, the fact that the respondent FBAC gives instructions to the
workers on how to go about their work is sufficient indication that it exercises control over their movements. The
workers are instructed as to what time they are supposed to report and what time they are supposed to return.
They were required to fill up monitoring sheets as they go about their jobs and even the materials which they used
in the packing plant were supplied by FBAC.
Viewed from the above circumstances, it is clear that respondent FBAC is the real employer of the workers of
Packing Plant 90. They are in truth and in fact the employees of the respondent and its attempt to seek refuge on
A2Y Contracting Services as the ostensible employer was nothing but an elaborate scheme to deprive them their
right to self-organization.

DOLE SECRETARY RULING:


Dismissed Sumifru’s appeal, affirmed Med-Arbiter’s decision. It held that Sumifru failed to proffer sufficient
evidence to show that CBPPWC is a duly-registered independent contractor, and that the latter engages in labor-
only contracting, making Sumifru the true employer of respondent NAMASUFA. In support of this, it held that the
evidence failed to show that CBPPWC possesses substantial capital in relation with the work or services that are
being performed by its members and that the employees placed by CBPPWC in Sumifru are performing activities
distinct and independent from that of the main business of Sumifru.

Further, to support the finding that Sumifru is the true employer of NAMASUFA, the DOLE secretary held:

“In this case, Sumifru's control over the subject employees is evident. The fact that the subject workers are
required by Sumifru to fill up monitoring sheets as they go about their jobs and the imposition of disciplinary
actions for non-compliance with the "No Helmet - No Entry and No ID - No Entry" policies prove that it is indeed
Sumifru, and not A2Y Contracting Services, that exercises control over the conduct of the subject workers.”

Petitioner Sumifru, appealed the DOLE secretary’s decision with the CA, citing grave abuse of discretion in ruling
that an employer-employee relationship exists between it and NAMASUFA.

CA RULING:

Affirmed DOLE Med-Arbiter and DOLE Secretary’s ruling, according respect to the findings of the two as both
have special knowledge and expertise over matters under their jurisdiction.

ISSUE:
Did the DOLE secretary committed grave abuse of discretion in ruling that an employer-employee relationship
exists between Sumifru and NAMASUFA?

Angeles| Bajana | Balladares | Brillantes | Briones | Cabansag | Callanta | Chua | David|

De Leon | Gomez | Lopez | Macalino | Nostratis | Padilla | Reynon | Santos | Tan |Velasco
4E / 4F - 2018-2019
Page 119 of 920
LABOR REVIEW DIGEST
Atty. Joyrich Golangco

HELD:
No. The Court pointed out that petitioner raised questions of fact in a petition for review on certiorari. It reiterated
that only questions of law may be raised in said petition. It then ruled upon that findings of fact of quasi-judicial
agencies such as the DOLE are afforded great respect when supported by substantial evidence and in the
absence of any showing of a whimsical or capricious exercise of judgment. Citing FFW v. Court of Appeals, it
held:

The factual findings by quasi-judicial agencies, such as the Department of Labor and Employment, when
supported by substantial evidence, are entitled to great respect in view of their expertise in their respective fields.
Judicial review of labor cases does not go so far as to evaluate the sufficiency of evidence on which the labor
official's findings rest. It is not our function to assess and evaluate all over again the evidence, testimonial and
documentary, adduced by the parties to an appeal, particularly where the findings of both the trial court (here, the
DOLE Secretary) and the appellate court on the matter coincide, as in this case at bar. The Rule limits that
function of the Court to the review or revision of errors of law and not to a second analysis of the evidence. Here,
petitioners would have us re-calibrate all over again the factual basis and the probative value of the pieces of
evidence submitted by the Company to the DOLE, contrary to the provisions of Rule 45. Thus, absent any
showing of whimsical or capricious exercise of judgment, and unless lack of any basis for the conclusions made
by the appellate court be amply demonstrated, we may not disturb such factual findings.

Other Notes/ SC Pronouncements:


SC decision only touched upon the binding effect of factual findings of the DOLE secretary, and only
mentioned that the decisions by the lower court (applying the four-fold test) indeed showed that an
employer-employee relationship exists. It didn’t even reiterate the earlier decisions, dwelling more on the
issue of whether the DOLE secretary committed grave abuse of discretion and whether its findings may
be reviewed upon by the Supreme Court. As such, substantial parts of the DOLE Med-Arbiter and DOLE
Secretary’s decisions were copied since those are the only decisions that are related to the topic of
employer-employee relationship.

_____________________________________________________________________

26. Wesleyan Univ. Philippines v. Maglaya


GR No: 212774 Date: January 23, 2017
Ponente: Peralta, J.

Employer-Employee Relationship
Doctrine:
A corporate officer's dismissal is always a corporate act, or an intra-corporate controversy which arises between a
stockholder and a corporation, and the nature is not altered by the reason or wisdom with which the Board of
Angeles| Bajana | Balladares | Brillantes | Briones | Cabansag | Callanta | Chua | David|

De Leon | Gomez | Lopez | Macalino | Nostratis | Padilla | Reynon | Santos | Tan |Velasco
4E / 4F - 2018-2019
Page 120 of 920
LABOR REVIEW DIGEST
Atty. Joyrich Golangco

Directors may have in taking such action. The issue of the alleged termination involving a corporate officer, not a
mere employee, is not a simple labor problem but a matter that comes within the area of corporate affairs and
management and is a corporate controversy in contemplation of the Corporation Code.

…the determination of the rights of a corporate officer dismissed from his employment, as well as the
corresponding liability of a corporation, if any, is an intra-corporate dispute subject to the jurisdiction of the regular
courts.

FACTS:
Petitioner Wesleyan University Philippines (WUP) is a non-stock, non-profit, non-sectarian educational corporation
duly organized and existing under the Philippine laws on April 28, 1948.

Respondent Atty. Guilleno T. Maglaya, Sr., was appointed as a corporate member of WUP on January 1, 2004,
and elected as a member of the Board of Trustees on January 9, 2004, both for a term of 5 years. Then on May 5,
2004, respondent was elected as president for a five-year term followed by his re-election as a trustee on May 25,
2007.

On a November 28, 2008 memorandum, the incumbent Bishops of the United Methodist
Church (Bishops) apprised all the corporate members of the expiration of their terms on December 31, 2008,
unless renewed by the former. The members including respondent sought the renewal of their term.

Dr. Dominador Cabasal, Chairman of the Board, however, informed the Bishops that some of the vacancies may
only be filled by the Bishops upon recommendation by the Board.

Respondent later on learned that the Bishops created an Ad Hoc committee for the turnover of the administration
of WUP and that incoming corporate members and trustees were already appointed.

On April 24, 2009, the Bishops, through a formal notice introduced the new corporate members, trustees, and
officers. The said notice also indicated that the new Board met, organized, and elected the new set of officers on
April 20, 2009. Manuel Palomo (Palomo), the new Chairman of the Board, then informed respondent Maglaya of
the termination of his services and authority as the President of the University on April 27, 2009.

Respondent subsequently filed a complaint for illegal dismissal with the Labor Arbiter. He alleged that he is an
employee of WUP, presenting the following evidence:
copies of his appointment as President, his Identification Card, the WUP Administration and Personnel Policy
Manual which specified the retirement of the university president, and the check disbursement in his favor
evidencing his salary

Petitioner WUP, on the other hand, alleged that respondent was a corporate officer, and hence, the matter is an
intra-corporate controversy not subject to the Labor Arbiter’s jurisdiction but rather the RTC’s.

LA RULING:
Labor Arbiter is without jurisdiction. It held that the action between employers and employees where the employer-
employee relationship is merely incidental is within the exclusive and original jurisdiction of the regular courts and
that since respondent was appointed as a president, he was a corporate officer and not an employee.

NLRC RULING:
Reversed LA ruling. It held that respondent’s cause of dismissal was not among the just causes provided Article
282 (Now 297) of the Labor Code hence the latter was illegally dismissed. In support of such conclusion, it pointed
Angeles| Bajana | Balladares | Brillantes | Briones | Cabansag | Callanta | Chua | David|

De Leon | Gomez | Lopez | Macalino | Nostratis | Padilla | Reynon | Santos | Tan |Velasco
4E / 4F - 2018-2019
Page 121 of 920
LABOR REVIEW DIGEST
Atty. Joyrich Golangco

out that contrary to the by-laws, respondent was not elected but rather appointed. Further, it ruled that although
the position of the President of the University is a corporate office, the manner of Maglaya's appointment, and his
duties, salaries, and allowances point to his being an employee and subordinate. The control test is the most
important indicator of the presence of employer-employee relationship. Such was present in the instant case as
Maglaya had the duty to report to the Board, and it was the Board which terminated or dismissed him even before
his term ends.

CA RULING:
Denied appeal since the decision and resolution of the NLRC already became final and executory on March 16,
2013.

ISSUES:
Is respondent a corporate officer?

HELD:
Yes. Respondent was a corporate officer whose office was created by virtue of WUP’s by-laws. Defining a
corporate officer, the Court stated:

"Corporate officers" in the context of Presidential Decree No. 902-A are those officers of the corporation who
are given that character by the Corporation Code or by the corporation's by-laws. There are three specific officers
whom a corporation must have under Section 25 of the Corporation Code. These are the president, secretary and
the treasurer. The number of officers is not limited to these three. A corporation may have such other officers as
may be provided for by its by-laws like, but not limited to, the vice-president, cashier, auditor or general manager.
The number of corporate officers is thus limited by law and by the corporation's by-laws.”

It further held:
The president, vice-president, secretary and treasurer are commonly regarded as the principal or executive
officers of a corporation, and they are usually designated as the officers of the corporation. However, other
officers are sometimes created by the charter or by-laws of a corporation, or the board of directors may be
empowered under the by-laws of a corporation to create additional offices as may be necessary. This Court
expounded that an "office" is created by the charter of the corporation and the officer is elected by the directors or
stockholders, while an "employee" usually occupies no office and generally is employed not by action of the
directors or stockholders but by the managing officer of the corporation who also determines the compensation to
be paid to such employee.

From the foregoing, that the creation of the position is under the corporation's charter or by-laws, and that
the election of the officer is by the directors or stockholders must concur in order for an individual to be considered
a corporate officer, as against an ordinary employee or officer. It is only when the officer claiming to have been
illegally dismissed is classified as such corporate officer that the issue is deemed an intra-corporate dispute which
falls within the jurisdiction of the trial courts.

It is apparent from the By-laws of WUP that the president was one of the officers of the corporation, and was an
honorary member of the Board. He was appointed by the Board and not by a managing officer of the corporation.
We held that one who is included in the by-laws of a corporation in its roster of corporate officers is an officer of
said corporation and not a mere employee.

The alleged "appointment" of Maglaya instead of "election" as provided by the by-laws neither convert the
president of university as a mere employee, nor amend its nature as a corporate officer. With the office specifically
mentioned in the by-laws, the NLRC erred in taking cognizance of the case, and in concluding that Maglaya was a
mere employee and subordinate official because of the manner of his appointment, his duties and responsibilities,
Angeles| Bajana | Balladares | Brillantes | Briones | Cabansag | Callanta | Chua | David|

De Leon | Gomez | Lopez | Macalino | Nostratis | Padilla | Reynon | Santos | Tan |Velasco
4E / 4F - 2018-2019
Page 122 of 920
LABOR REVIEW DIGEST
Atty. Joyrich Golangco

salaries and allowances, and considering the Identification Card, the Administration and Personnel Policy Manual
which specified the retirement of the university president, and the check disbursement as pieces of evidence
supporting such finding.
A corporate officer's dismissal is always a corporate act, or an intra-corporate controversy which arises between a
stockholder and a corporation, and the nature is not altered by the reason or wisdom with which the Board of
Directors may have in taking such action.59 The issue of the alleged termination involving a corporate officer, not a
mere employee, is not a simple labor problem but a matter that comes within the area of corporate affairs and
management and is a corporate controversy in contemplation of the Corporation Code.

To emphasize, the determination of the rights of a corporate officer dismissed from his employment, as well as the
corresponding liability of a corporation, if any, is an intra-corporate dispute subject to the jurisdiction of the regular
courts.

Other Notes/ SC Pronouncements:


Pertinent provisions of WUP’s by-laws:

ARTICLE VI. BOARD OF TRUSTEES

x x x x

Section 2. Membership (a) The Board of Trustees shall be composed of Ten (10) members of the corporation
from among themselves provided, that six (6) shall come from the Ministry and Laity of the United Methodist
[C]hurch in the Philippines, three (3) shall be non-Methodist, friends and sympathizers of the Wesleyan University-
Philippines and of the United Methodist Church, and one (1) representative of the Wesleyan Alumni Association,
as provided in section 1 (c), Article IV hereof, and (b) provided further that the incumbent area bishop and
the President of the Wesleyan University-Philippines shall be honorary members of the Board.

x x x x

ARTICLE VIII. OFFICERS

Section 1. Officers The officers of the Board of Trustees shall be:chanRoblesvirtualLawlibrary

(a) Chairman
(b) Vice-Chairman
(c) Secretary
(d) Treasurer

x x x x

Section 6. The President of Wesleyan University-Philippines � The President of the University, who must be an
active member of the United Methodist Church in the Philippines at the time of his election shall be in-charge of
and be responsible for the administration of the University and other institutions of learning that [m]ay hereafter be
established by the corporation, and
(a) May, with the Board of Trustees;chanrobleslaw

Angeles| Bajana | Balladares | Brillantes | Briones | Cabansag | Callanta | Chua | David|

De Leon | Gomez | Lopez | Macalino | Nostratis | Padilla | Reynon | Santos | Tan |Velasco
4E / 4F - 2018-2019
Page 123 of 920
LABOR REVIEW DIGEST
Atty. Joyrich Golangco

(1) Organize and/or reorganize the administrative set up of the Wesleyan University-Philippines to effect efficiency
and upgrade institutional administration and supervision;chanrobleslaw

(2) Employ, suspend, dismiss, transfer or replace personnel and prescribe and enforce rules and regulations for
their proper conduct in the discharge of their duties;chanrobleslaw

(3) Shall make reports during the different rumual conference of the United Methodist Church ru1d to such
agencies as may be deemed necessary on the operations of the university and related matters;chanrobleslaw

(4) Shall prescribe and enforce rules and regulations for the promotion and maintenance of discipline in the proper
conduct and discharge of the functions and duties of subordinate administrative officers, professors, teachers,
employees and students and other personnel.

(b) Shall make reports and recommendations to the Board of Trustees or to the Chairman of the Board of
Trustees on matters pertaining to the institution as he may find necessary;chanrobleslaw

(c) Shall countersign all checks drawn by the Treasurer from the depository of the University, and

(d) Shall exercise, perform and discharge all such other powers, functions and duties as are interest in the office
of the President.

_____________________________________________________________________

27. NESTLE PHILIPPINES, INC. (NPI) vs. Benny Puedan, Jr., et. al.
GR No: 220617 Date: Jnauary 30, 2017
Ponente: PERLAS-BERNABE, J p:|||
Doctrine: The imposition of minimum standards concerning sales, marketing, finance and operations is nothing
more than an exercise of sound business practice to increase sales and maximize profits for the benefit of both
Steelcase and its distributors. For as long as these requirements do not impinge on a distributor's independence,
then there is nothing wrong with placing reasonable expectations on them.
FACTS:
Respondents alleged that on various dates, Ocho de Septiembre, Inc. (ODSI) and NPI hired them to sell various
NPI products in the assigned covered area. After some time, respondents demanded that they be considered
regular employees of NPI, but they were directed to sign contracts of employment with ODSI instead. When
respondents refused to comply with such directives, NPI and ODSI terminated them from their position.
They then filed a complaint claiming that ODSI is a labor-only contractor and, thus, they should be deemed
regular employees of NPI; and there was no just or authorized cause for their dismissal.
ODSI averred that it is a company engaged in the business of buying, selling, distributing, and marketing of goods
and commodities of every kind and it enters into all kinds of contracts for the acquisition thereof. ODSI admitted
that on various dates, it hired respondents as its employees and assigned them to execute the Distributorship
Agreement it entered with NPI. ODSI also claimed that its business with NPI turned sour, which led to the closure
of its NPI unit, and that respondents were not dismissed but merely put in floating status.
LA RULING:

Angeles| Bajana | Balladares | Brillantes | Briones | Cabansag | Callanta | Chua | David|

De Leon | Gomez | Lopez | Macalino | Nostratis | Padilla | Reynon | Santos | Tan |Velasco
4E / 4F - 2018-2019
Page 124 of 920
LABOR REVIEW DIGEST
Atty. Joyrich Golangco

The LA dismissed the complaint for lack of merit. The LA found that: (a) respondents were unable to prove that
they were NPI employees; and (b) respondents were not illegally dismissed as ODSI had indeed closed down its
operations due to business losses. |||
NLRC RULING:
Upon appeal, the NLRC reversed and set aside the LA ruling. The NLRC found ODSI to be a labor-only contractor
of NPI, considering that:
(a) ODSI had no substantial capitalization or investment;
(b) respondents performed activities directly related to NPI's principal business; and
(c) the fact that respondents' employment depended on the continuous supply of NPI products shows that ODSI
had not been carrying an independent business according to its own manner and method. Consequently, the
NLRC deemed NPI to be respondents' true employer, and thus, ordered it jointly and severally liable with ODSI to
pay the monetary claims of respondents.|
Respondents moved for a partial reconsideration, arguing that since NPI was deemed to be their true employer,
NPI should reinstate them, or if not practicable, to pay them separation pay.|||
NPI also moved for reconsideration, contending that it was deprived of its right to participate in the proceedings
before the LA and the NLRC; andit had no employer-employee relationship with respondents as ODSI was never
its contractor, whether independent or labor-only.||| The NLRC denied both MRs.
CA RULING:
The CA affirmed the NLRC ruling. The CA opined that the following stipulations of the said Agreement evinces
that NPI had control over the business of ODSI, namely, that:
(a) NPI shall offer to ODSI suggestions and recommendations to improve sales and to further develop the
market;
(b) NPI prohibits ODSI from exporting its products (the No-Export provision);
(c) NPI provided standard requirements to ODSI for the warehousing and inventory management of the sold
goods; and
(d) prohibition imposed on ODSI to sell any other products that directly compete with those of NPI.
ISSUE/S:
Is ODSI a labor-only contractor of NPI, and consequently, NPI is respondents' true employer and, thus, deemed
jointly and severally liable with ODSI for respondents' monetary claims?
HELD:
No. a closer examination of the Distributorship Agreement reveals that the relationship of NPI and
ODSI is not that of a principal and a contractor (regardless of whether labor-only or independent), but that of
a seller and a buyer/re-seller. As stipulated in the Distributorship Agreement, NPI agreed to sell its products to
ODSI at discounted prices, which in turn will be re-sold to identified customers, ensuring in the process the
integrity and quality of the said products based on the standards agreed upon by the parties. As aptly
explained by NPI, the goods it manufactures are distributed to the market through various
distributors, e.g., ODSI, that in turn, re-sell the same to designated outlets through its own employees such as
the respondents. Therefore, the reselling activities allegedly performed by the respondents properly pertain to
ODSI, whose principal business consists of the "buying, selling, distributing, and marketing goods and
commodities of every kind" and "[entering] into all kinds of contracts for the acquisition of such goods [and
commodities]."
Thus, contrary to the CA's findings, the aforementioned stipulations in the Distributorship Agreement
hardly demonstrate control on the part of NPI over the means and methods by which ODSI performs its
business, nor were they intended to dictate how ODSI shall conduct its business as a distributor. Otherwise
stated, the stipulations in the Distributorship Agreement do not operate to control or fix the methodology on
how ODSI should do its business as a distributor of NPI products, but merely provide rules of conduct or
guidelines towards the achievement of a mutually desired result — which in this case is the sale of NPI
products to the end consumer.
Verily, it was only reasonable for NPI — it being a local arm of one of the largest manufacturers of
foods and grocery products worldwide — to require its distributors, such as ODSI, to meet various conditions
Angeles| Bajana | Balladares | Brillantes | Briones | Cabansag | Callanta | Chua | David|

De Leon | Gomez | Lopez | Macalino | Nostratis | Padilla | Reynon | Santos | Tan |Velasco
4E / 4F - 2018-2019
Page 125 of 920
LABOR REVIEW DIGEST
Atty. Joyrich Golangco

for the grant and continuation of a distributorship agreement for as long as these conditions do not control the
means and methods on how ODSI does its distributorship business, as shown in this case. This is to ensure
the integrity and quality of the products which will ultimately fall into the hands of the end consumer.
Thus, the foregoing circumstances show that ODSI was not a labor-only contractor of NPI; hence,
the latter cannot be deemed the true employer of respondents. As a consequence, NPI cannot be held jointly
and severally liable to ODSI's monetary obligations towards respondents.

Other Notes/ SC Pronouncements:


__________________________________________________________________

28. VALENZUELA vs ALEXANDRA MINING AND OIL VENTURES, INC.


GR No: 222419 Date: October 5, 2016
Ponente: Reyes, J.

Doctrine:
A company driver is an employee of the company

FACTS:
This case stemmed from a complaint for illegal dismissal, non-payment of backwages, overtime pay, separation
pay, moral and exemplary damages and attorney's fees filed by Ramil R. Valenzuela (Valenzuela) against
Alexandra Mining and Oil Ventures, Inc. (AMOVI) and its owner and president, Cesar E. Detera (Cesar)
(collectively, the respondents).

Valenzuela alleged that he was hired as a company driver of AMOVI, with an eight-hour work shift from 8:00 a.m.
to 5:00 p.m. and with a monthly salary of P12,000.00. After five years and five months of service, he was told that
he can no longer continue to work as there were no forthcoming funds to pay for his salary.

For their part, the respondents alleged that Valenzuela was actually hired as a family driver of the Deteras. They
alleged, however, that the P12,000.00 monthly salary of Valenzuela was charged to AMOVFs account for
convenience. They averred that on June 15, 2013, Valenzuela informed Cesar's wife, Annlynn, that he was going
home to his province to visit his parents. Annlynn granted him leave but when she asked him whether he can
return for work the following Monday, Valenzuela told her that he would give her a call. Come Monday, Valenzuela
did not show up for work and did not also call to inform the Deteras of the reason behind his absence. This
caused them inconvenience as their daughter's schooling has started and it was Valenzuela's responsibility to
bring her to and from school.

A week later, Valenzuela showed up at the Deteras' residence and informed them that he was resigning and
asked for his separation pay. To obviate further verbal altercation, Annlynn agreed but asked him to submit a

Angeles| Bajana | Balladares | Brillantes | Briones | Cabansag | Callanta | Chua | David|

De Leon | Gomez | Lopez | Macalino | Nostratis | Padilla | Reynon | Santos | Tan |Velasco
4E / 4F - 2018-2019
Page 126 of 920
LABOR REVIEW DIGEST
Atty. Joyrich Golangco

resignation letter. Ultimately, Annlynn told him to make up his mind but Valenzuela just walked out and never
returned.

In his Reply, Valenzuela emphasized that he did not just suffer to work for the company but also drove for the
members of the Detera family. He denied that he ever asked permission to visit his parents in Bicol, as the
Deteras knew that his parents had long been dead. Moreover, the remains of his deceased parents were buried in
Pateros. He alleged that he actually reported for work on June 17, 2013, but was prevented by Cesar who told
him that his service is no longer needed as there were no funds forthcoming to pay for his salary.

LA RULING: Valenzuela had been illegally dismissed.

NLRC RULING: Affirmed LA

CA RULING:
The CA held that since there was no clear evidence that Valenzuela was dismissed by the respondents and, on
the other hand, there was an equal lack of proof of abandonment of work on the part of Valenzuela. Following the
ruling of the Court in Exodus International Construction Corporation, et al. v. Biscocho, et at., the remedy was to
reinstate Valenzuela without backwages.

APPEAL TO THE SC:

ISSUE/S:
1. Whether or not Valenzuela was an employee of AMOVI
2. Whether or not Valenzuela was illegally dismissed

HELD:
1. YES.
The instant case does not share the same factual milieu with Exodus. It is noteworthy to emphasize that in all the
pleadings submitted by Cesar before the LA, NLRC and CA, he vigorously refuted the existence of an employer-
employee relationship between AMOVI and Valenzuela, and at the same time, presented himself as the real
employer of the latter. His argument was that Valenzuela was not a company driver but a family driver of the
Deteras.

The question regarding who may be deemed the real employer of Valenzuela had been unanimously resolved
and agreed by the LA, NLRC and the CA to be AMOVI. The labor tribunals and the CA were all in accord that
Valenzuela was an employee of AMOVI as evidenced by the identification card and payslips stating the
company as his employer. Moreover, the CA held that, utilizing the four-fold test of employer-employee
relationship, the result would show that Valenzuela was under the control of AMOVI. It ruled thus:

In determining the existence of an employer-employee relationship, jurisprudence spelled out the four-fold test, to
wit: (1) the selection and engagement of the employee; (2) the payment of wages; (3) the power of dismissal; and
(4) the employer's power to control the employee with respect to the means and methods by which the work is to
be accomplished,
x x x.

It was [AMOVI] which hired [Valenzuela] in January 2008, and which issued an identification card showing that
[Valenzuela] was an employee. [Valenzuela] was likewise included in the payroll of [AMOVI], although it was
claimed that it was merely "for convenience." We do not see what kind of convenience is afforded to [AMOVI].

The power to discipline and to dismiss is also present, and it was exercised by [Cesar] as President of [AMOVI]
Angeles| Bajana | Balladares | Brillantes | Briones | Cabansag | Callanta | Chua | David|

De Leon | Gomez | Lopez | Macalino | Nostratis | Padilla | Reynon | Santos | Tan |Velasco
4E / 4F - 2018-2019
Page 127 of 920
LABOR REVIEW DIGEST
Atty. Joyrich Golangco

which incidentally is a family corporation.

Finally, the control test is likewise satisfied. [Valenzuela] had no choice as to who his passengers would be. He
was a company driver who was required to render service to the President of the Corporation, including his
nuclear family. It was them who controlled and dictated the manner by which he performed his job.

2. YES
The CA, however, erred in holding that there was no evidence of dismissal as it is clear from Cesar's own
admission that Valenzuela was unceremoniously dismissed from service. In all his pleadings, while claiming to be
the real employer of Valenzuela, Cesar impliedly admitted dismissing him from employment by repeatedly
invoking Article 150 of the Labor Code to justify his action. The provision reads as follows:
chanRoblesvirtualLawlibrary
Art. 150. Service of termination notice. If the duration of the household service is not determined either in
stipulation or by nature of the service, the employer or the househelper may give notice to put an end to the
relationship five (5) days before the intended termination of the service.

On the basis of the foregoing provision, Cesar asseverated that as a family driver, Valenzuela's service may be
terminated at will by his employer. Thus, there is implied admission that he indeed terminated Valenzuela out of
his own volition, without sufficient ground and notice. Unfortunately for Cesar, the labor tribunals and the CA all
agreed that Valenzuela was a company employee and his admission on the fact of the latter's dismissal only
established that it was done without regard to substantive and procedural due process.

The Court elucidated on the requisites of a valid dismissal in Skippers United Pacific, Inc., et al. v. Doza, et
al, thus:
chanRoblesvirtualLawlibrary
For a worker's dismissal to be considered valid, it must comply with both procedural and substantive due process.
The legality of the manner of dismissal constitutes procedural due process, while the legality of the act of
dismissal constitutes substantive due process.

Procedural due process in dismissal cases consists of the twin requirements of notice and hearing. The employer
must furnish the employee with two written notices before the termination of employment can be effected: (1) the
first notice apprises the employee of the particular acts or omissions for which his dismissal is sought; and (2) the
second notice informs the employee of the employer's decision to dismiss him. Before the issuance of the second
notice, the requirement of a hearing must be complied with by giving the worker an opportunity to be heard. It is
not necessary that an actual hearing be conducted.

Substantive due process, on the other hand, requires that dismissal by the employer be made under a just or
authorized cause under Articles 282 to 284 of the Labor Code.

Evidently, the respondents raised no valid ground to justify Valenzuela's dismissal. As admitted by Cesar,
Valenzuela was terminated at will. This was corroborated by Valenzuela's claim that when he returned for work on
June 17, 2013, he was simply told that he can no longer continue to work as there were no funds forthcoming to
pay off his salary.

Further, the twin requirement of notice and hearing for a valid termination was not observed by the respondents.
Valenzuela was not at all informed of the ground of his dismissal and was deprived the opportunity to explain his
side. He was rashly dismissed from service without a valid ground and the required notices.

Other Notes/ SC Pronouncements:


Angeles| Bajana | Balladares | Brillantes | Briones | Cabansag | Callanta | Chua | David|

De Leon | Gomez | Lopez | Macalino | Nostratis | Padilla | Reynon | Santos | Tan |Velasco
4E / 4F - 2018-2019
Page 128 of 920
LABOR REVIEW DIGEST
Atty. Joyrich Golangco

Consistent with the finding that Valenzuela had been illegally dismissed, he is, therefore, entitled to reinstatement
and full backwages.

CA correctly ruled on the solidary liability of the respondents to pay the monetary awards due the dismissed
employee. As a rule, "[a] corporate officer is not personally liable for the money claims of discharged corporate
employees unless he acted with evident malice and bad faith in terminating their employment."37 Here, Cesar's
bad faith was manifested by his persistent assertion that Valenzuela was merely a family driver in order to justify
his unceremonious dismissal. He repeatedly insisted that as a family driver or member of the household service,
Valenzuela may be terminated at will, which was exactly what he did. He unreasonably sent Valenzuela home
when the latter reported for work, the latter unaware of what he had done to merit such an abrupt termination.
Cesar's admission on the reckless manner of Valenzuela's dismissal justifies holding him solidarity liable with
AMOVI.

29. HSY Marketing Ltd., Co. v. Virgilio O. Villastique


G.R. No. 219569
August 17, 2016
Perlas-Bernabe J.

Doctrine:

Service incentive leave is a right which accrues to every employee who has served 'within 12 months, whether
continuous or broken, reckoned from the date the employee started working, including authorized absences and
paid regular holidays unless the working days in the establishment as a matter of practice or policy, or that
provided in the employment contracts, is less than 12 months, in which case said period shall be considered as
one [(1)] year.' It is also commutable to its money equivalent if not used or exhausted at the end of the year. In
other words, an employee who has served for one (1) year is entitled to it. He may use it as leave days or he may
collect its monetary value.

FACTS:

On January 3, 2003, petitioner hired respondent as a field driver for Fabulous Jeans & Shirt & General
Merchandise. On January 10, 2011, respondent figured in an accident. Fabulous Jeans shouldered the
hospitalization and medical expenses of Dorataryo in the amount of P64,157.15, which respondent was asked to
reimburse, but to no avail. Respondent was allegedly required to sign a resignation letter, which he refused to do.
Convinced that he was already terminated on February 26, 2011, he filed a complaint for illegal dismissal with
money claims before the NLRC.

Petitioner, et al. contended that respondent had committed several violations in the course of his employment,
and had been found by his superior and fellow employees to be a negligent and reckless driver, which resulted in
the vehicular mishap involving Dorataryo. Respondent alledgedly went on absence without leave, presumably to
evade liability for his recklessness. Since respondent was the one who refused to report for work, he should be
considered as having voluntarily severed his own employment.

LA/RTC/NLRC RULING:

The LA dismissed the charge of illegal dismissal, finding no evidence to substantiate respondent's claim that he
was dismissed from his job on February 26, 2011. Nonetheless, the LA directed petitioner to pay the respondent
the amount of P86,580.00 as separation pay. The LA also ruled that respondent is entitled to service incentive
leave pay.

Angeles| Bajana | Balladares | Brillantes | Briones | Cabansag | Callanta | Chua | David|

De Leon | Gomez | Lopez | Macalino | Nostratis | Padilla | Reynon | Santos | Tan |Velasco
4E / 4F - 2018-2019
Page 129 of 920
LABOR REVIEW DIGEST
Atty. Joyrich Golangco

The NLRC affirmed the finding of the LA that there was no illegal dismissal to speak of.

CA RULING:

In a Decision dated October 29, 2014, the CA affirmed in toto the NLRC.

APPEAL TO THE SC:

Petitioner's Contention:

See Facts

Respondent's Contention:

See Facts

ISSUE/S:

1. WON an employment relationship existed between the parties in this case.


2. WON the award of separation pay is proper.
3. WON the award of service incentive leave pay is proper.

HELD:

1. Considering that the LA, the NLRC, and the CA consistently found petitioner liable as the employer of
respondent, the Court sees no compelling reason to depart from their judgment on this score.

In fact, it is even worth noting that respondent claimed in his Position Paper before the LA that he was hired by
petitioner and was required to report for work at its store in Cagayan de Oro City. This was confirmed by petitioner
in its own Position Paper, declaring respondent to be "a field driver for the Cagayan de Oro Branch of (petitioner)
HSY MARKETING LTD., CO., (NOVO JEANS & SHIRT)." Clearly, petitioner should be bound by such admission
and must not be allowed to continue to deny any employer-employee relationship with respondent.

2. The Court likewise upholds the unanimous conclusion of the lower tribunals that respondent had not been
dismissed at all. Other than the latter's unsubstantiated allegation of having been verbally terminated from his
work, no substantial evidence was presented to show that he was indeed dismissed or was prevented from
returning to his work. In the absence of any showing of an overt or positive act proving that petitioner had
dismissed respondent, the latter's claim of illegal dismissal cannot be sustained, as such supposition would be
self-serving, conjectural, and of no probative value.

Similarly, petitioner's claims of respondent's voluntary resignation and/or abandonment deserve scant
consideration, considering petitioner's failure to discharge the burden of proving the deliberate and unjustified
refusal of respondent to resume his employment without any intention of returning. It was incumbent upon
petitioner to ascertain respondent's interest or non-interest in the continuance of his employment, but to no avail.

Hence, since there is no dismissal or abandonment to speak of, the appropriate course of action is to reinstate the
employee without, however, the payment of backwages.

Properly speaking, liability for the payment of separation pay is but a legal consequence of illegal dismissal
where reinstatement is no longer viable or feasible. As a relief granted in lieu of reinstatement, it goes without
Angeles| Bajana | Balladares | Brillantes | Briones | Cabansag | Callanta | Chua | David|

De Leon | Gomez | Lopez | Macalino | Nostratis | Padilla | Reynon | Santos | Tan |Velasco
4E / 4F - 2018-2019
Page 130 of 920
LABOR REVIEW DIGEST
Atty. Joyrich Golangco

saying that an award of separation pay is inconsistent with a finding that there was no illegal dismissal. This is
because an employee who had not been dismissed, much less illegally dismissed, cannot be reinstated.
Moreover, as there is no reinstatement to speak of, respondent cannot invoke the doctrine of strained
relations to support his prayer for the award of separation pay.

3. Service incentive leave is a right which accrues to every employee who has served 'within 12 months, whether
continuous or broken, reckoned from the date the employee started working, including authorized absences and
paid regular holidays unless the working days in the establishment as a matter of practice or policy, or that
provided in the employment contracts, is less than 12 months, in which case said period shall be considered as
one [(1)] year.' It is also commutable to its money equivalent if not used or exhausted at the end of the year. In
other words, an employee who has served for one (1) year is entitled to it. He may use it as leave days or he may
collect its monetary value.

[R]espondent is not a field personnel as defined above because of the nature of his job as a company driver.
Expectedly, respondent is directed to deliver the goods at a specified time and place and he is not given the
discretion to solicit, select[,] and contact prospective clients. Respondent in his Position Paper claimed that he
was required to report for work from 8:00 a.m. to 8:00 p.m. at the company's store located at Velez-Gomez Street,
Cagayan de Oro City. Certainly then, respondent was under the control and supervision of petitioners.
Respondent, therefore, is a regular employee whose task is usually necessary and desirable to the usual trade
and business of the company. Thus, he is entitled to the benefits accorded to regular employees, including service
incentive leave pay.

Other Notes/ SC Pronouncements:

______________________________________________________________________

Article 95

1. AUTO BUS TRANSPORT SYSTEMS, INC. vs. ANTONIO BAUTISTA


G.R. No. 156367; May 16, 2005
CHICO-NAZARIO, J.

DOCTRINE: Service Incentive Leave shall not apply to employees classified as field personnel. The phrase other
employees whose performance is unsupervised by the employer must not be understood as a separate
classification of employees to which service incentive leave shall not be granted. Rather, it serves as an
amplification of the interpretation of the definition of field personnel under the Labor Code as those whose actual
hours of work in the field cannot be determined with reasonable certainty.

FACTS:

Since 24 May 1995, respondent Antonio Bautista has been employed by petitioner Auto Bus Transport
Systems, Inc. (Autobus), as driver-conductor with travel routes Manila-Tuguegarao via Baguio, Baguio-
Tuguegarao via Manila and Manila-Tabuk via Baguio. Respondent was paid on commission basis, seven percent
(7%) of the total gross income per travel, on a twice a month basis.

Angeles| Bajana | Balladares | Brillantes | Briones | Cabansag | Callanta | Chua | David|

De Leon | Gomez | Lopez | Macalino | Nostratis | Padilla | Reynon | Santos | Tan |Velasco
4E / 4F - 2018-2019
Page 131 of 920
LABOR REVIEW DIGEST
Atty. Joyrich Golangco

On 03 January 2000, while respondent was driving Autobus No. 114 along Sta. Fe, Nueva Vizcaya, the bus
he was driving accidentally bumped the rear portion of Autobus No. 124, as the latter vehicle suddenly stopped at
a sharp curve without giving any warning.
Respondent averred that the accident happened because he was compelled by the management to go back
to Roxas, Isabela, although he had not slept for almost twenty-four (24) hours, as he had just arrived in Manila
from Roxas, Isabela. Respondent further alleged that he was not allowed to work until he fully paid the amount of
P75,551.50, representing thirty percent (30%) of the cost of repair of the damaged buses and that despite
respondents pleas for reconsideration, the same was ignored by management. After a month, management sent
him a letter of termination.
Respondent instituted a Complaint for Illegal Dismissal with Money Claims for nonpayment of 13th month pay
and service incentive leave pay against Autobus. Petitioner, on the other hand, maintained that respondents
employment was replete with offenses involving reckless imprudence, gross negligence, and dishonesty. To
support its claim, petitioner presented copies of letters, memos, irregularity reports, and warrants of arrest
pertaining to several incidents wherein respondent was involved.
LA RULING: Dismissed the complaint but ordered the petitioner to pay among others the respondent’s service
incentive leave pay for all the years he had been in service with the respondent, presently computed at
P13,788.05.
NLRC RULING: Maintained the award for service incentive leave.
CA RULING: Affirmed NLRC’s ruling.
ISSUE:
Whether or not the respondent is entitled to service incentive leave.
RULING:
YES. The grant of service incentive leave has been delimited by the Implementing Rules and
Regulations of the Labor Code to apply only to those employees not explicitly excluded by Section 1 of Rule V.
According to the Implementing Rules, Service Incentive Leave shall not apply to employees classified as field
personnel. The phrase other employees whose performance is unsupervised by the employer must not be
understood as a separate classification of employees to which service incentive leave shall not be granted.
Rather, it serves as an amplification of the interpretation of the definition of field personnel under the Labor Code
as those whose actual hours of work in the field cannot be determined with reasonable certainty.

The same is true with respect to the phrase those who are engaged on task or contract basis, purely
commission basis. Said phrase should be related with field personnel, applying the rule on ejusdem generis that
general and unlimited terms are restrained and limited by the particular terms that they follow. Hence, employees
engaged on task or contract basis or paid on purely commission basis are not automatically exempted from the
grant of service incentive leave, unless, they fall under the classification of field personnel.Therefore, petitioners
contention that respondent is not entitled to the grant of service incentive leave just because he was paid on
purely commission basis is misplaced. What must be ascertained in order to resolve the issue of propriety of the
grant of service incentive leave to respondent is whether or not he is a field personnel.
As observed by the Labor Arbiter and concurred in by the Court of Appeals:It is of judicial notice that
along the routes that are plied by these bus companies, there are its inspectors assigned at strategic places who
board the bus and inspect the passengers, the punched tickets, and the conductors reports. There is also the
mandatory once-a-week car barn or shop day, where the bus is regularly checked as to its mechanical, electrical,
and hydraulic aspects, whether or not there are problems thereon as reported by the driver and/or conductor.
They too, must be at specific place as [sic] specified time, as they generally observe prompt departure and arrival
from their point of origin to their point of destination. In each and every depot, there is always the Dispatcher
whose function is precisely to see to it that the bus and its crew leave the premises at specific times and arrive at
the estimated proper time. These, are present in the case at bar. The driver, the complainant herein, was
therefore under constant supervision while in the performance of this work. He cannot be considered a field
personnel. Therefore, as correctly concluded by the appellate court, respondent is not a field personnel but a

Angeles| Bajana | Balladares | Brillantes | Briones | Cabansag | Callanta | Chua | David|

De Leon | Gomez | Lopez | Macalino | Nostratis | Padilla | Reynon | Santos | Tan |Velasco
4E / 4F - 2018-2019
Page 132 of 920
LABOR REVIEW DIGEST
Atty. Joyrich Golangco

regular employee who performs tasks usually necessary and desirable to the usual trade of petitioners business.
Accordingly, respondent is entitled to the grant of service incentive leave.
Other Notes/ SC Pronouncements:

______________________________________________________________________

Art. 97

1. Songco et.al. v. NLRC


GR No: L-50999 Date: March 23, 1990
Ponente: Medialdea, J.

Doctrine: Art. 97. In computing the basis for separation pay of a dismissed employee, allowances should be
included in the monthly salary.

FACTS:
Private respondent F.E. Zuellig (M), Inc., (hereinafter referred to as Zuellig) filed with the Department of Labor
(Regional Office No. 4) an application seeking clearance to terminate the services of petitioners Jose Songco,
Romeo Cipres, and Amancio Manuel (hereinafter referred to as petitioners) allegedly on the ground of
retrenchment due to financial losses. Initially, petitioners opposed the dismissal on the ground that they are
dismissed for being part of the union. However, at the last hearing of the case, petitioners agreed that the sole
issue to be resolved is the basis of the separation pay due to them.
Petitioners, who were in the sales force of Zuellig received monthly salaries of at least P40,000. In addition, they
received commissions for every sale they made.
Petitioners maintain that their earned sales commissions and allowances should be added together with their
salary to arrive at the basis for computing separation pay, citing Article 97(f) of the Labor Code. Zuellig on the
other hand argues that in the said article the term “wage”, “commission” is used only as one of the features or
designations attached to the word remuneration or earnings.

LA RULING: The basis of separation pay shall be equivalent to their one month salary (exclusive of commissions,
allowances, etc.) for every year in service that they have worked in with the company.

NLRC RULING: Appeal dismissed for lack of merit.

Angeles| Bajana | Balladares | Brillantes | Briones | Cabansag | Callanta | Chua | David|

De Leon | Gomez | Lopez | Macalino | Nostratis | Padilla | Reynon | Santos | Tan |Velasco
4E / 4F - 2018-2019
Page 133 of 920
LABOR REVIEW DIGEST
Atty. Joyrich Golangco

PETITION TO THE SC:


Petitioner's Contention: Petitioners' position was that in arriving at the correct and legal amount of separation
pay due them, whether under the Labor Code or the CBA, their basic salary, earned sales commissions and
allowances should be added together. They cited Article 97(f) of the Labor Code which includes commission as
part on one's salary

Respondent's Contention: Zuellig argues that if it were really the intention of the Labor Code as well as its
implementing rules to include commission in the computation of separation pay, it could have explicitly said so in
clear and unequivocal terms. Furthermore, in the definition of the term "wage", "commission" is used only as one
of the features or designations attached to the word remuneration or earnings.
ISSUE/S:
1. Whether the allowances should be included in the monthly salary of petitioners for the purpose of computation
of their separation pay
2. Whether the sales commissions should be included in the monthly salary of petitioners for the purpose of
computation of their separation pay

HELD:
1. Yes. In computing the basis for separation pay of a dismissed employee, allowances should be included
in the monthly salary. This has been settled in the case of Santos v. NLRC, et al. (GR No. 76721.
September 21, 1987) where the SC ruled that “in the computation of backwages and separation pay,
account must be taken not only of the basic salary but also of her transportation and emergency living
allowances.”

2. Yes. . In computation thereof, what should be taken into account is the average commissions earned
during their last year of employment. Article 97(f) by itself is explicit that commission is included in the
definition of the term “wage”. The law speaks in clear and categorical language, there is no room for
interpretation or construction. Said Article provides: “(f) ‘Wage’ paid to any employee shall mean the
remuneration or earnings, however designated, capable of being expressed in terms of money, whether
fixed or ascertained on a time, task, piece, or commission basis, or other method calculating the same,
which is payable by an employer to an employee under a written or unwritten contract of employment for
wok done or to be done, or for services rendered or to be rendered, and includes the fair and reasonable
value, as determined by the Secretary of Labor, of board, lodging, or other facilities customarily furnished
by the employer to the employee. ‘Fair and reasonable value’ shall not include any profit to the employer
or to any person affiliated with the employer.” Granting, in grantia argumenti, that the commissions were
in the form of incentives or encouragement, so that the petitioners would be inspired to put a little more
industry on the jobs particularly assigned to them, still these commissions are direct remunerations for
services rendered which contributed to the increase of income of Zuellig. Commission is the
recompense, compensation or reward of an agent, salesman, executor, trustee, receiver, factor, broker
or bailee, when the same is calculated as a percentage on the amount of his transactions or on the profit
to the Principal. The nature of the work of a salesman and the reason for such tyoe of remuneration for
services rendered demonstrate clearly that commissions are part of petitioners’ wage or salary.

Other Notes/ SC Pronouncements:

______________________________________________________________________

2. MILLARES ET AL V. NLRC
GR No. 122827 March 29, 1999

Angeles| Bajana | Balladares | Brillantes | Briones | Cabansag | Callanta | Chua | David|

De Leon | Gomez | Lopez | Macalino | Nostratis | Padilla | Reynon | Santos | Tan |Velasco
4E / 4F - 2018-2019
Page 134 of 920
LABOR REVIEW DIGEST
Atty. Joyrich Golangco

Doctrine: In determining whether a privilege is a facility, the criterion is not so much its kind but its purpose. The
Sec. of Labor may from time to time fix in appropriate issuances the “fair and reasonable value of board, lodging
and other facilities customarily furnished by an employer to his employees.

FACTS: Petitioners numbering 116 occupied positions of Technical Staff, Unit Manager, Section Manager,
Department Manager, Division Manager and Vice President in the mill site of PICOP in Bislig, Surigao del Sur.
Their services were terminated when the company undertook a retrenchment program. They received separation
pay at the rate of one (1) month basic salary for every year in service. They lodged a complaint for separation pay
differentials believing that the allowances they allegedly regularly received on a monthly basis during their
employment should have been included in the computation of their separation pay.

The allowances pertained to the following:

1. Staff/Manager’s Allowance – a. PICOP provides free housing facilities to supervisory and managerial
employees assigned in Bislig. The privilege includes free water and electric consumption. b. Owing to the
shortage of such facilities, PICOP was constrained to grant Staff allowance instead to those who live in rented
houses outside but near the vicinity of the mill site. The allowance ceases whenever a vacancy occurs in the
company’s housing facilities.

2. Transportation Allowance – Transportation allowance is granted to key officers and Managers assigned in the
mill site who use their own vehicles in the performance of their duties. It is a conditional grant such that when the
conditions no longer obtain, the privilege is discontinued.

3. Bislig Allowance – Given to Division Managers and corporate officers assigned in Bislig on account of the
hostile environment. But once the recipient is transferred elsewhere outside Bislig, the allowance ceases.
Petitioners maintain that the said allowances are included in the definition of “facilities” in Art. 97, par. (f), of the
Labor Code, being necessary and indispensable for their existence and subsistence.

Furthermore, they claim that their availment of the monetary equivalent of those “facilities” on a monthly basis was
characterized by permanency, regularity and customariness.

LA RULING:

Yes, the allowances are to be characterized as being received regularly and forming part of salary. It is also to be
considered as “facilities” under Art 97, par. (f) of the Labor Code for purposes of computing separation pay. The
LA cited Santos v NLRC and Soriano v NLRC that in the computation of separation pay account should be taken
not just of the basic salary but also of the regular allowances that the employee had been receiving.

NLRC RULING:

No, the NLRC reversed the Labor Arbiter. The NLRC found that petitioners’ allowances were contingency-based
and thus not included in their salaries.

ISSUES:

1. Whether the receipt of the above-mentioned allowances, on a monthly basis, ipso facto characterize it as
regular and forming part of salary; and Whether the above-mentioned can be considered as “facilitites” and
therefore included in the computation of separation pay as wage.

RULING:
Angeles| Bajana | Balladares | Brillantes | Briones | Cabansag | Callanta | Chua | David|

De Leon | Gomez | Lopez | Macalino | Nostratis | Padilla | Reynon | Santos | Tan |Velasco
4E / 4F - 2018-2019
Page 135 of 920
LABOR REVIEW DIGEST
Atty. Joyrich Golangco

The decision of the NLRC is affirmed. The allowances are not to be included in the computation of wage for
purposes of paying separation pay 1. The receipt of an allowance on a monthly basis does not ipso facto
characterize it as regular and forming part of salary because the nature of the grant is a factor worth considering.
The subject allowances were temporarily, not regularly, received by petitioners. Petitioners’ continuous enjoyment
of the disputed allowances was based on contingencies the occurrence of which wrote finis to such enjoyment.
For housing allowance, the same is discontinued once a vacancy occurs in the company-provided housing
accommodations.

Transportation allowance is given only to employees who have personal cars in the form of advances for actual
transportation expenses subject to liquidation. Bislig allowance is- once the officer is transferred outside Bislig, the
allowance stops. 2. The Staff/Managers allowance may fall under “lodging” but the transportation and Bislig
allowances are not embraced in “facilitites” on the main consideration that they are granted as well as the
Staff/Manager’s allowance for respondent PICOP’s benefit and convenience, i.e. to insure that petitioners render
quality performance. In determining whether a privilege is a facility, the criterion is not so much its kind but its
purpose.

Other Notes/ SC Pronouncements:


______________________________________________________________________

3. SLL INTERNATIONAL CABLES SPECIALIST and SONNY L. LAGON vs.


NATIONAL LABOR RELATIONS COMMISSION
G.R. No: 172161 Date: March 2, 2011
Ponente: MENDOZA, J.

DOCTRINE: The benefit or privilege given to the employee which constitutes an extra remuneration above and
over his basic or ordinary earning or wage is supplement; and when said benefit or privilege is part of the
laborers' basic wages, it is a facility.
Angeles| Bajana | Balladares | Brillantes | Briones | Cabansag | Callanta | Chua | David|

De Leon | Gomez | Lopez | Macalino | Nostratis | Padilla | Reynon | Santos | Tan |Velasco
4E / 4F - 2018-2019
Page 136 of 920
LABOR REVIEW DIGEST
Atty. Joyrich Golangco

FACTS:
On 1996, and January 1997, private respondents Roldan Lopez and Danilo Cañete and Edgardo Zuñiga were
hired by petitioner Lagon as apprentice or trainee cable/lineman. The three were paid the full minimum wage and
other benefits but since they were only trainees, they did not report for work regularly but came in as substitutes.
After their training, Zuñiga, Cañete and Lopez were engaged as project employees by the petitioners in their
Islacom project in Bohol. Private respondents started on March 15, 1997 until December 1997. Upon the
completion of their project, their employment was also terminated. Private respondents received the amount of
₱145.00, the minimum prescribed daily wage for Region VII. In July 1997, the amount of ₱145 was increased to
₱150.00 by the Regional Wage Board (RWB) and in October of the same year, the latter was increased to
₱155.00. In March 1998, Zuñiga and Cañete were engaged again by Lagon as project employees for its PLDT
Antipolo, Rizal project, which ended sometime in the late September 1998. As a consequence, Zuñiga and
Cañete’s employment was terminated. For this project, Zuñiga and Cañete received only the wage of ₱145.00
daily. The minimum prescribed wage for Rizal at that time was ₱160.00.
They re-applied in the Racitelcom project of Lagon in Bulacan. Zuñiga and Cañete were re-employed. Lopez was
also hired for the said specific project. For this, private respondents received the wage of ₱145.00. On May 21,
1999, private respondents for the 4th time worked with Lagon’s project in Camarin, Caloocan City with Furukawa
Corporation as the general contractor. Their contract would expire on February 28, 2000, the period of completion
of the project. From May 21, 1997-December 1999, private respondents received the wage of ₱145.00. At this
time, the minimum prescribed rate for Manila was ₱198.00. In January to February 28, the three received the
wage of ₱165.00. The existing rate at that time was ₱213.00.
Due to the delay in the delivery of imported materials from Furukawa Corporation, the Camarin project was not
completed on the scheduled date. Lagon was constrained to cut down the overtime work of its workers due to
economic problems including private respondents. Lagon refused when Private respondents requested that they
work overtime. Lagon told private respondents that if they insist, they would have to go home at their own
expense and that they would not be given anymore time nor allowed to stay in the quarters. Thus, private
respondents left heir work and went home to Cebu. Private respondents filed a complaint for illegal dismissal,
non-payment of wages, holiday pay, 13th month pay for 1997 and 1998 and service incentive leave pay as well as
damages and attorney’s fees against Lagon.
Petitioner’s contention:
Petitioners admit employment of private respondents but claimed that the latter were only project employees for
their services were merely engaged for a specific project or undertaking and the same were covered by contracts
duly signed by private respondents. Petitioners further alleged that the food allowance of ₱63.00 per day as well
as private respondents allowance for lodging house, transportation, electricity, water and snacks allowance should
be added to their basic pay. With these, petitioners claimed that private respondents received higher wage rate
than that prescribed in Rizal and Manila.
Lastly, petitioners alleged that since the workplaces of private respondents were all in Manila, the complaint
should be filed there. Thus, petitioners prayed for the dismissal of the complaint for lack of jurisdiction and utter
lack of merit.
LA RULING:
Labor Arbiter Reynoso Belarmino declared that his office had jurisdiction to hear and decide the complaint filed by
private respondents. Referring to Rule IV, Sec. 1 (a) of the NLRC Rules of Procedure prevailing at that time,the
LA ruled that it had jurisdiction because the "workplace," as defined in the said rule, included the place where the
employee was supposed to report back after a temporary detail, assignment or travel, which in this case was
Cebu.
The LA opined that private respondents were regular employees because they were repeatedly hired by
petitioners and they performed activities which were usual, necessary and desirable in the business or trade of the
employer. It found that private respondents were underpaid. It ruled that the free board and lodging, electricity,
water, and food enjoyed by them could not be included in the computation of their wages because these were
given without their written consent.
Angeles| Bajana | Balladares | Brillantes | Briones | Cabansag | Callanta | Chua | David|

De Leon | Gomez | Lopez | Macalino | Nostratis | Padilla | Reynon | Santos | Tan |Velasco
4E / 4F - 2018-2019
Page 137 of 920
LABOR REVIEW DIGEST
Atty. Joyrich Golangco

NLRC RULING: the NLRC affirmed the findings of the LA. In addition, the NLRC noted that not a single report of
project completion was filed with the nearest Public Employment Office as required
by the Department of Labor and Employment (DOLE) Department Order No. 19, Series of 1993. The NLRC later
denied the motion for reconsideration subsequently filed by petitioners.
CA RULING:
The matter was elevated to the CA on a petition for certiorari. It affirmed the findings that the private respondents
were regular employees. It considered the fact that they performed functions which were the regular and usual
business of petitioners. According to the CA, they were clearly members of a work pool from which petitioners
drew their project employees.
The CA also stated that the failure of petitioners to comply with the simple but compulsory requirement to submit a
report of termination to the nearest Public Employment Office every time private respondents’ employment was
terminated was proof that the latter were not project employees but regular employees. The CA also agreed with
the NLRC that there was no illegal dismissal. The CA opined that it was the petitioners’ prerogative to grant or
deny any request for overtime work and that the private respondents’ act of leaving the workplace after their
request was denied was an act of abandonment.
ISSUE: Whether or not the said value of facilities that Private Respondents received should be included in
the computation of “wages” received by them.
RULING:
No. As a general rule, on payment of wages, a party who alleges payment as a defense has the burden of proving
it. Petitioners, aside from bare allegations that private respondents received wages higher than the prescribed
minimum, failed to present any evidence, such as payroll or payslips, to support their defense of payment. Private
respondents, on the other hand, are entitled to be paid the minimum wage, whether they are regular or non-
regular employees.
Section 3, Rule VII of the Rules to Implement the Labor Code specifically enumerates those who are not covered
by the payment of minimum wage. Project employees are not among them.
Furthermore, Section 1 of DOLE Memorandum Circular No. 2 provides that an employer may provide subsidized
meals and snacks to his employees provided that the subsidy shall not be less that 30% of the fair and reasonable
value of such facilities. In such cases, the employer may deduct from the wages of the employees not more than
70% of the value of the meals and snacks enjoyed by the latter, provided that such deduction is with the written
authorization of the employees concerned.
Before the value of facilities can be deducted from the employees’ wages, the following requisites must all be
present:
1. Proof must be shown that such facilities are customarily furnished by the trade;
2. The provision of deductible facilities must be voluntarily accepted in writing by the employee.
3. Facilities must be charged at reasonable value.
Mere availment is not sufficient to allow deductions from employees’ wages.
These requirements, however, have not been met. SLL failed to present any company policy or guideline showing
that provisions for meals and lodging were part of the employee’s salaries. It also failed to provide proof of the
employees’ written authorization, much less show how they arrived at their valuations. It was also not clear
whether private respondents actually enjoyed said facilities.
The food and lodging, or the electricity and water allegedly consumed by private respondents in this case were
not facilities but supplements. In the case of Atok-Big Wedge Assn. v. Atok-Big Wedge Co. the two terms were
distinguished from one another in this wise:
"Supplements," therefore, constitute extra remuneration or special privileges or benefits given to or received by
the laborers over and above their ordinary earnings or wages. "Facilities," on the other hand, are items of expense
necessary for the laborer's and his family's existence and subsistence so that by express provision of law they
form part of the wage and when furnished by the employer are deductible therefrom, since if they are not so
furnished, the laborer would spend and pay for them just the same.

Angeles| Bajana | Balladares | Brillantes | Briones | Cabansag | Callanta | Chua | David|

De Leon | Gomez | Lopez | Macalino | Nostratis | Padilla | Reynon | Santos | Tan |Velasco
4E / 4F - 2018-2019
Page 138 of 920
LABOR REVIEW DIGEST
Atty. Joyrich Golangco

In short, the benefit or privilege given to the employee which constitutes an extra remuneration above and over his
basic or ordinary earning or wage is supplement; and when said benefit or privilege is part of the laborers' basic
wages, it is a facility.

Other Notes/ SC Pronouncements:


_____________________________________________________________________

Article 100

1. AMERICAN WIRE AND CABLE DAILY RATED EMPLOYEES UNION v. AMERICAN WIRE AND CABLE
CO., INC. and THE COURT OF APPEALS
G.R. No. 155059. April 29, 2005

CHICO-NAZARIO, J.

FACTS:
- American Wire and Cable Co., Inc., is a corporation engaged in the manufacture of wires and cables.
- There are two unions in this company:
o American Wire and Cable Monthly-Rated Employees Union (Monthly-Rated Union) and
o American Wire and Cable Daily-Rated Employees Union (Daily-Rated Union).
- 16 February 2001, an original action was filed before the NCMB of the Department of Labor and
Employment (DOLE) by the two unions for voluntary arbitration.
- They alleged that the private respondent, without valid cause, suddenly and unilaterally withdrew and
denied certain benefits and entitlements which they have long enjoyed. These are:
o a. Service Award;
o b. 35% premium pay of an employees basic pay for the work rendered during Holy Monday,
Holy Tuesday, Holy Wednesday, December 23, 26, 27, 28 and 29;
o c. Christmas Party; and
o d. Promotional Increase.
- A promotional increase was asked by the petitioner for 15 of its members who were given or assigned
new job classifications.
- According to petitioner, the new job classifications were in the nature of a promotion, necessitating the
grant of an increase in the salaries of the said 15 members.
- A Submission Agreement was filed by the parties before the Office for Voluntary Arbitration. Assigned as
Voluntary Arbitrator was Angel A. Ancheta
- a Decision was rendered by Voluntary Arbitrator Angel A. Ancheta in favor of the private respondent
declaring it not guilty of violating Article 100 of the Labor Code, or specifically for withdrawing the service
award, Christmas party and 35% premium for work rendered during Holy Week and Christmas season
and for not granting any promotional increase to the alleged fifteen (15) Daily-Rated Union Members in
the absence of a promotion
- MR was likewise denied.
- An appeal under Rule 43 of the 1997 Rules on Civil Procedure was made by the Daily-Rated Union
before the Court of Appeals
- Decision in favor of herein respondent company was promulgated by the Special Eighth Division of the
Court of Appeals

Angeles| Bajana | Balladares | Brillantes | Briones | Cabansag | Callanta | Chua | David|

De Leon | Gomez | Lopez | Macalino | Nostratis | Padilla | Reynon | Santos | Tan |Velasco
4E / 4F - 2018-2019
Page 139 of 920
LABOR REVIEW DIGEST
Atty. Joyrich Golangco

ISSUE:
- whether or not private respondent is guilty of violating Article 100 of the Labor Code, as amended, when
the benefits/entitlements given to the members of petitioner union were withdrawn.
- The petitioner submits that the withdrawal of the private respondent of the 35% premium pay for selected
days during the Holy Week and Christmas season, the holding of the Christmas Party and its incidental
benefits, and the giving of service awards violated Article 100 of the Labor Code.
- The grant of these benefits was a customary practice that can no longer be unilaterally withdrawn by
private respondent without the tacit consent of the petitioner.
- The benefits in question were given by the respondent to the petitioner consistently, deliberately, and
unconditionally since time immemorial.
- With respect to the fifteen (15) employees who are members of petitioner union that were given new job
classifications, it asserts that a promotional increase in their salaries was in order. Salary adjustment is a
must due to their promotion.

REM ISSUE:
- private respondent contends that this case should have been dismissed outright because of petitioners
error in the mode of appeal.
- According to it, the petitioner should have elevated the instant case to this Court through a petition for
review on certiorari under Rule 45, and not through a special civil action for certiorari under Rule 65, of
the 1997 Rules on Civil Procedure.
HELD:
 Assuming arguendo that the mode of appeal taken by the petitioner is improper, there is no question that
the Supreme Court has the discretion to dismiss it if it is defective. However, sound policy dictates that it
is far better to dispose the case on the merits, rather than on technicality.
 The Supreme Court may brush aside the procedural barrier and take cognizance of the petition as it
raises an issue of paramount importance. The Court shall resolve the solitary issue on the merits for
future guidance of the bench and bar

TRUE ISSUE HELD:


 Article 100 of the Labor Code provides:
o ART. 100. PROHIBITION AGAINST ELIMINATION OR DIMINUTION OF BENEFITS. Nothing
in this Book shall be construed to eliminate or in any way diminish supplements, or other
employee benefits being enjoyed at the time of promulgation of this Code.
 it is critical that a determination must be first made on whether the benefits/entitlements are in the
nature of a bonus or not, and assuming they are so, whether they are demandable and enforceable
obligations.
 In the case of Producers Bank of the Philippines v. NLRC we have characterized what a bonus is,
viz:
o A bonus is an amount granted and paid to an employee for his industry and loyalty which
contributed to the success of the employers business and made possible the realization
of profits.
o It is an act of generosity granted by an enlightened employer to spur the employee to
greater efforts for the success of the business and realization of bigger profits.
o The granting of a bonus is a management prerogative, something given in addition to
what is ordinarily received by or strictly due the recipient.
o Thus, a bonus is not a demandable and enforceable obligation, except when it is made part of
the wage, salary or compensation of the employee.

Angeles| Bajana | Balladares | Brillantes | Briones | Cabansag | Callanta | Chua | David|

De Leon | Gomez | Lopez | Macalino | Nostratis | Padilla | Reynon | Santos | Tan |Velasco
4E / 4F - 2018-2019
Page 140 of 920
LABOR REVIEW DIGEST
Atty. Joyrich Golangco

 it is obvious that the benefits/entitlements subjects of the instant case are all bonuses which were
given by the private respondent out of its generosity and munificence.
 The additional 35% premium pay for work done during selected days of the Holy Week and Christmas
season, the holding of Christmas parties with raffle, and the cash incentives given together with the
service awards are all in excess of what the law requires each employer to give its employees.
 Since they are above what is strictly due to the members of petitioner-union, the granting of the same
was a management prerogative, which, whenever management sees necessary, may be
withdrawn, unless they have been made a part of the wage or salary or compensation of the
employees.
 The consequential question therefore that needs to be settled is if the subject benefits/entitlements,
which are bonuses, are demandable or not. Stated another way, can these bonuses be considered
part of the wage or salary or compensation making them enforceable obligations?
 The Court does not believe so.
 For a bonus to be enforceable, it must
o have been promised by the employer and expressly agreed upon by the parties, or it
o must have had a fixed amount and had been a long and regular practice on the part of the
employer
 The benefits/entitlements in question were never subjects of any express agreement between the
parties. They were never incorporated in the Collective Bargaining Agreement (CBA).
 As observed by the Voluntary Arbitrator, the records reveal that these benefits/entitlements have not
been subjects of any express agreement between the union and the company, and have not yet been
incorporated in the CBA.
 In fact, the petitioner has not denied having made proposals with the private respondent for the
service award and the additional 35% premium pay to be made part of the CBA
 As correctly ruled by the CA:
o Notwithstanding that the subject 35% premium pay was deliberately given and the same was in
excess of that provided by the law, the same however did not ripen into a company practice
on account of the fact that it was only granted for two (2) years and with the express
reservation from respondent corporations owner that it cannot continue to rant the same
in view of the company’s current financial situation.
 To hold that an employer should be forced to distribute bonuses which it granted out of kindness
is to penalize him for his past generosity.
 Considering that the Union was unable to adduce proof that a promotion indeed occured with
respect to the 15 employees, the Daily Rated Unions claim for promotional increase likewise fails there
being no promotion established under the records at hand
 CA, VOLUNTARY ARBITRATOR AFFIRMED.

Other Notes/ SC Pronouncements:


_____________________________________________________________________

Angeles| Bajana | Balladares | Brillantes | Briones | Cabansag | Callanta | Chua | David|

De Leon | Gomez | Lopez | Macalino | Nostratis | Padilla | Reynon | Santos | Tan |Velasco
4E / 4F - 2018-2019
Page 141 of 920
LABOR REVIEW DIGEST
Atty. Joyrich Golangco

2. TSPIC Corporation v. TSPIC Employee’s Union (FFW)


G.R. No. 163419; Feb. 13, 2008
Velasco, Jr., J.:

DOCTRINE: There is diminution of benefits when it is shown that: (1) the grant or benefit is founded on a policy or
has ripened into a practice over a long period; (2) the practice is consistent and deliberate; (3) the practice is not
due to error in the construction or application of a doubtful or difficult question of law; and (4) the diminution or
discontinuance is done unilaterally by the employer.

FACTS:

TSPIC is engaged in the business of designing, manufacturing, and marketing integrated circuits to serve the
communication, automotive, data processing, and aerospace industries. Respondent TSPIC Employees Union
(FFW) (Union), on the other hand, is the registered bargaining agent of the rank-and-file employees of TSPIC.

In 1999, TSPIC and the Union entered into a Collective Bargaining Agreement (CBA) for the years 2000 to 2004.
The CBA included a provision on yearly salary increases starting January 2000 until January 2002. Among the
provisions is provided a “Crediting Provision” stating:

The wage salary increase of the first year of this Agreement shall be over and above the wage/salary
increase, including the wage distortion adjustment, granted by the COMPANY on November 1, 1999 as
per Wage Order No. NCR-07.

The wage/salary increases for the years 2001 and 2002 shall be deemed inclusive of the mandated
minimum wage increases under future Wage Orders, that may be issued after Wage Order No. NCR-07,
and shall be considered as correction of any wage distortion that may have been brought about by the
said future Wage Orders

Over the course of the years starting 2000, all rank and file employees received their 10% increase in accordance
with the CBA, and 17 employees received their regularization bonuses, as agreed in the CBA. A wage order was
released increasing the minimum wage from 223.50 to 250php. However, in 2001, 24 employees, herein
respondents, received notice that their salaries would be deducted in the course of 12 months because they had
been overpaid by reason of a computation error, and the deduction was on the basis of the Crediting Provision
which placed the CBA over the wage orders. The Union refused the contention of TSPIC arguing that the same
constituted a diminution of beneti and the case was referred to their agreed grievance machinery but to no avail.
The case was referred to voluntary arbitration.

LA’s Ruling: Ruled that the reduction constituted a diminution of benefits under Art. 100.

CA’s Ruling: Affirmed in toto the decision of the LA, stating that Petitioner failed to prove that there was indeed
an error in the computation of the payments.

Appeal to the SC:

Petitioner’s contention: That the courts a quo failed to consider the crediting provision which would be the
reason why they found a diminution of benefits.

Respondent’s contention: That to allow the same would constitute a diminution of benefits.
Angeles| Bajana | Balladares | Brillantes | Briones | Cabansag | Callanta | Chua | David|

De Leon | Gomez | Lopez | Macalino | Nostratis | Padilla | Reynon | Santos | Tan |Velasco
4E / 4F - 2018-2019
Page 142 of 920
LABOR REVIEW DIGEST
Atty. Joyrich Golangco

ISSUE: Whether or not there is a diminution of benefits.

RULING: NO. A CBA is law between the parties, and given the crediting provision which specifically provided the
crediting of the wage orders, the parties, or more particularly the employees are deemed to have waived any
wage order in favor of the benefits under the CBA. Given this, there could be no diminution of benefits in this case
given that respondents indeed have been receiving benefits to which they were not entitled.

Diminution of benefits is the unilateral withdrawal by the employer of benefits already enjoyed by the employees.
There is diminution of benefits when it is shown that: (1) the grant or benefit is founded on a policy or has ripened
into a practice over a long period; (2) the practice is consistent and deliberate; (3) the practice is not due to error
in the construction or application of a doubtful or dificult question of law; and (4) the diminution or discontinuance
is done unilaterally by the employer.

The benefits received only having been received by reason of error timely discovered, it is only proper that the
same be returned to TSPIC.

Other Notes/ SC Pronouncements:


_____________________________________________________________________

3. Lepanto Ceramics Inc. v Lepanto Ceramics Employees Association


GR No: 180866 Date: Mar. 2, 2010
Ponente: Perez, J.
Doctrine:
Given that the bonus in this case is integrated in the CBA, the same partakes the nature of a
demandable obligation.
It is a familiar and fundamental doctrine in labor law that the CBA is the law between the parties and they
are obliged to comply with its provisions. This principle stands strong and true in the case at bar.

FACTS:

Angeles| Bajana | Balladares | Brillantes | Briones | Cabansag | Callanta | Chua | David|

De Leon | Gomez | Lopez | Macalino | Nostratis | Padilla | Reynon | Santos | Tan |Velasco
4E / 4F - 2018-2019
Page 143 of 920
LABOR REVIEW DIGEST
Atty. Joyrich Golangco

Petitioner Lepanto Ceramics, Incorporated is a duly organized corporation existing and operating by virtue of
Philippine Laws. Its business is primarily to manufacture, make, buy and sell, on wholesale basis, among others,
tiles, marbles, mosaics and other similar products.
Respondent Lepanto Ceramics Employees Association (respondent Association) is a legitimate labor organization
duly registered with the Department of Labor and Employment. It is the sole and exclusive bargaining agent in the
establishment of petitioner.
In September 1999, petitioner and respondent Association entered into a Collective Bargaining Agreement (CBA)
which provides for, among others, the grant of a Christmas gift package/bonus to the members of the respondent
Association. The Christmas bonus was one of the enumerated existing benefit, practice of traditional rights which
shall remain in full force and effect.
In the succeeding years, 1999, 2000 and 2001, the bonus was not in cash. Instead, petitioner gave each of the
members of respondent Association Tile Redemption Certificates equivalent to P3,000.00. The bonus for the year
2002 is the root of the present dispute. Petitioner gave a year-end cash benefit of Six Hundred Pesos (P600.00)
and offered a cash advance to interested employees equivalent to one (1) month salary payable in one year. The
respondent Association objected to the P600.00 cash benefit and argued that this was in violation of the CBA it
executed with the petitioner.
The parties failed to amicably settle the dispute. The respondent Association filed a Notice of Strike with the
National Conciliation Mediation Board, Regional Branch No. IV, alleging the violation of the CBA. The case was
placed under preventive mediation. The efforts to conciliate failed. The case was then referred to the Voluntary
Arbitrator for resolution.

LA/RTC/NLRC/VA RULING:
The Voluntary Arbitrator rendered a Decision dated 2 June 2003, declaring that petitioner is bound to
grant each of its workers a Christmas bonus of P3,000.00 for the reason that the bonus was given prior to
the effectivity of the CBA between the parties and that the financial losses of the company is not a
sufficient reason to exempt it from granting the same. It stressed that the CBA is a binding contract and
constitutes the law between the parties. The Voluntary Arbitrator further expounded that since the employees had
already been given P600.00 cash bonus, the same should be deducted from the claimed amount of P3,000.00,
thus leaving a balance of P2,400.00.

CA RULING:
On appeal to the CA via a petition for certiorari under Rule 65 of the ROC, the CA affirmed in toto the
decision of the Voluntary Arbitrator.

APPEAL TO THE SC:


Petitioner now files a petition for review on certiorari under Rule 45

Petitioner's Contention:
The petitioner averred that the complaint for nonpayment of the 2002 Christmas bonus had no basis as the same
was not a demandable and enforceable obligation. It argued that the giving of extra compensation was based on
the companys available resources for a given year and the workers are not entitled to a bonus if the company
does not make profits. Petitioner adverted to the fact that it was debt-ridden having incurred net losses for the
years 2001 and 2002 totaling to P1.5 billion; and since 1999, when the CBA was signed, the companys
accumulated losses amounted to over P2.7 billion. Petitioner further argued that the grant of a one (1) month
salary cash advance was not meant to take the place of a bonus but was meant to show the companys sincere
desire to help its employees despite its precarious financial condition. Petitioner also averred that the CBA
provision on a Christmas gift/bonus refers to alternative benefits. Finally, petitioner emphasized that even if the
CBA contained an unconditional obligation to grant the bonus to the respondent Association, the present difficult
economic times had already legally released it therefrom pursuant to Article 1267 of the Civil Code.

Angeles| Bajana | Balladares | Brillantes | Briones | Cabansag | Callanta | Chua | David|

De Leon | Gomez | Lopez | Macalino | Nostratis | Padilla | Reynon | Santos | Tan |Velasco
4E / 4F - 2018-2019
Page 144 of 920
LABOR REVIEW DIGEST
Atty. Joyrich Golangco

Respondent's Contention:
Respondent Association insisted that it has been the traditional practice of the company to grant its members
Christmas bonuses during the end of the calendar year, each in the amount of P3,000.00 as an expression of
gratitude to the employees for their participation in the companys continued existence in the market. The bonus
was either in cash or in the form of company tiles. In 2002, in a speech during the Christmas celebration, one of
the companys top executives assured the employees of said bonus. However, the Human Resources
Development Manager informed them that the traditional bonus would not be given as the companys earnings
were intended for the payment of its bank loans. Respondent Association argued that this was in violation of their
CBA.

ISSUE/S:
1. Whether or not respondent is entitled to the Christmas bonus

HELD:
1. YES. We uphold the rulings of the voluntary arbitrator and of the Court of Appeals. Findings of labor
officials, who are deemed to have acquired expertise in matters within their respective jurisdictions, are
generally accorded not only respect but even finality, and bind us when supported by substantial
evidence. This is the rule particularly where the findings of both the arbitrator and the Court of Appeals
coincide.
As a general proposition, an arbitrator is confined to the interpretation and application of the CBA. He
does not sit to dispense his own brand of industrial justice: his award is legitimate only in so far as it draws its
essence from the CBA. That was done in this case.

Generally, a bonus is not a demandable and enforceable obligation. For a bonus to be


enforceable, it must have been promised by the employer and expressly agreed upon by the parties.
Given that the bonus in this case is integrated in the CBA, the same partakes the nature of a
demandable obligation. Verily, by virtue of its incorporation in the CBA, the Christmas bonus due to
respondent Association has become more than just an act of generosity on the part of the petitioner but a
contractual obligation it has undertaken.
It is a familiar and fundamental doctrine in labor law that the CBA is the law between the parties and they
are obliged to comply with its provisions. This principle stands strong and true in the case at bar.

All given, business losses are a feeble ground for petitioner to repudiate its obligation under the CBA.
The rule is settled that any benefit and supplement being enjoyed by the employees cannot be reduced,
diminished, discontinued or eliminated by the employer. The principle of non-diminution of benefits is founded
on the constitutional mandate to protect the rights of workers and to promote their welfare and to afford labor
full protection.
Hence, absent any proof that petitioners consent was vitiated by fraud, mistake or duress, it is presumed
that it entered into the CBA voluntarily and had full knowledge of the contents thereof and was aware of its
commitments under the contract.
Other Notes/ SC Pronouncements:
______________________________________________________________________

4. EASTERN TELECOMMUNICATIONS PHILIPPINES, INC., v EASTERN TELECOMS EMPLOYEES UNION,


GR. No.: 185665 Date: February 8, 2012

DOCTRINE:
WHEN BONUS IS DEMANDABLE:
GR: The grant of a bonus is basically a management prerogative which cannot be forced upon the employer.
XPNs:
Angeles| Bajana | Balladares | Brillantes | Briones | Cabansag | Callanta | Chua | David|

De Leon | Gomez | Lopez | Macalino | Nostratis | Padilla | Reynon | Santos | Tan |Velasco
4E / 4F - 2018-2019
Page 145 of 920
LABOR REVIEW DIGEST
Atty. Joyrich Golangco

1. A bonus becomes a demandable or enforceable obligation when it is


made part of the wage or salary or compensation of the employee.
TEST: If it is additional compensation which the employer promised and agreed to give without any
conditions imposed for its payment, such as success of business or greater production or output, then it is
part of the wage.But if it is paid only if profits are realized or if a certain level of productivity is
achieved, it cannot be considered part of the wage. Where it is not payable to all but only to some
employees and only when their labor becomes more efficient or more productive, it is only an inducement
for efficiency, a prize therefore, not a part of the wage. (Metro TransitOrganization, Inc. v. National Labor
Relations Commission)
2. A bonus may be granted on equitable consideration when the giving of such bonus has been the company’s
long and regular practice.
TEST: To be considered a "regular practice," however, the giving of the bonus should have been done
over a long period of time, and must be shown to have been consistentand deliberate. The test or rationale
of this rule on long practice requires an indubitable showing that the employer agreed to continue giving the
benefits knowing fully well thatsaid employees are not covered by the law requiring payment thereof.
(Philippine Appliance Corporation v. Court of Appeals)

FACTS:
Eastern Telecommunications Phils., Inc. (ETPI) is a corporation engaged in the business of providing
telecommunications facilities. Eastern Telecoms Employees Union (ETEU) is the certified exclusive bargaining
agent of the company’s rank and file employees. It has an existing CBA with the company to expire in the year
2004 with a Side Agreement signed on September 3, 2001.
In essence, the labor dispute was a spin-off of the company’s plan to defer payment of the 2003 14th, 15th
and 16th month bonuses sometime in April 2004. The company’s main ground in postponing the payment of
bonuses is due to allege continuing deterioration of company’s financial position which started in the year 2000.
However, ETPI while postponing payment of bonuses sometime in April 2004, such payment would also
be subject to availability of funds.
Invoking the Side Agreement of the existing CBA for the period 2001-2004 between ETPI and ETEU, the
union strongly opposed the deferment in payment of the bonuses by filing a preventive mediation complaint with
the NCMB.
Later, the company made a sudden turnaround in its position by declaring that they will no longer pay the
bonuses until the issue is resolved through compulsory arbitration.
Thus ETEU filed a Notice of Strike on the ground of unfair labor practice for failure of ETPI to pay the
bonuses in gross violation of the economic provision of the existing CBA.
ETPI insists that it is under no legal compulsion to pay 14th, 15th and 16th month bonuses for the year 2003
and 14th month bonus for the year 2004 contending that they are not part of the demandable wage or salary and
that their grant is conditional based on successful business performance and the availability of company profits
from which to source the same. To thwart ETEU’s monetary claims, it insists that the distribution of the subject
bonuses falls well within the company’s prerogative, being an act of pure gratuity and generosity on its part. Thus,
it can withhold the grant thereof especially since it is currently plagued with economic difficulties and financial
losses.
ETPI further avers that the act of giving the subject bonuses did not ripen into a company practice arguing
that it has always been a contingent one dependent on the realization of profits and, hence, the workers are not
entitled to bonuses if the company does not make profits for a given year. It asseverates that the 1998 and 2001
CBA Side Agreements did not contractually afford ETEU a vested property right to a perennial payment of the
bonuses. It opines that the bonus provision in the Side Agreement allows the giving of benefits only at the time of
its execution. For this reason, it cannot be said that the grant has ripened into a company practice.

ISSUE:

Angeles| Bajana | Balladares | Brillantes | Briones | Cabansag | Callanta | Chua | David|

De Leon | Gomez | Lopez | Macalino | Nostratis | Padilla | Reynon | Santos | Tan |Velasco
4E / 4F - 2018-2019
Page 146 of 920
LABOR REVIEW DIGEST
Atty. Joyrich Golangco

Is ETPI is liable to pay 14th, 15th and 16th month bonuses for the year 2003 and 14th month bonus for the year
2004 to the members of respondent union?

RULING:
Yes. From a legal point of view, a bonus is a gratuity or act of liberality of the giver which the recipient
has no right to demand as a matter of right. The grant of a bonus is basically a management prerogative which
cannot be forced upon the employer who may not be obliged to assume the onerous burden of granting bonuses
or other benefits aside from the employee’s basic salaries or wages.
A bonus, however, becomes a demandable or enforceable obligation when it is made part of the wage or
salary or compensation of the employee. Particularly instructive is the ruling of the Court in Metro Transit
Organization, Inc. v. NLRC, where it was written:
“Whether or not a bonus forms part of wages depends upon the circumstances and conditions for its
payment. If it is additional compensation which the employer promised and agreed to give without any
conditions imposed for its payment, such as success of business or greater production or output, then it
is part of the wage. But if it is paid only if profits are realized or if a certain level of productivity is achieved, it
cannot be considered part of the wage. Where it is not payable to all but only to some employees and only
when their labor becomes more efficient or more productive, it is only an inducement for efficiency, a prize
therefore, not a part of the wage.”
In the case at bench, it is indubitable that ETPI and ETEU agreed on the inclusion of a provision for the grant of
14th, 15th and 16th month bonuses in the 1998-2001 CBA Side Agreement, as well as in the 2001-2004 CBA
Side Agreement, which was signed on September 3, 2001. The provision, which was similarly worded, states:
“Employment-Related Bonuses
The Company confirms that the 14th, 15th and 16th month bonuses (other than the
13th month pay) are granted.”
A reading of the above provision reveals that the same provides for the giving of 14th, 15th and 16th month
bonuses without qualification. The wording of the provision does not allow any other interpretation. There were no
conditions specified in the CBA Side Agreements for the grant of the benefits contrary to the claim of ETPI that the
same is justified only when there are profits earned by the company. The said provision does not state that the
subject bonuses shall be made to depend on the ETPI’s financial standing or that their payment was contingent
upon the realization of profits. Neither does it state that if the company derives no profits, no bonuses are to be
given to the employees. In fine, the payment of these bonuses was not related to the profitability of business
operations.
The records are also bereft of any showing that the ETPI made it clear before or during the execution of the Side
Agreements that the bonuses shall be subject to any condition. Indeed, if ETPI and ETEU intended that the
subject bonuses would be dependent on the company earnings, such intention should have been expressly
declared in the Side Agreements or the bonus provision should have been deleted altogether. Verily, by virtue of
its incorporation in the CBA Side Agreements, the grant of 14th, 15th and 16th month bonuses has become more
than just an act of generosity on the part of ETPI but a contractual obligation it has undertaken. Moreover,
the continuous conferment of bonuses by ETPI to the union members from 1998 to 2002 by virtue of the Side
Agreements evidently negates its argument that the giving of the subject bonuses is a management prerogative.
Granting arguendo that the CBA Side Agreement does not contractually bind petitioner ETPI to give the subject
bonuses, nevertheless, the Court finds that its act of granting the same has become an established company
practice such that it has virtually become part of the employees’ salary or wage. A bonus may be granted on
equitable consideration when the giving of such bonus has been the company’s long and regular practice.
In Philippine Appliance Corporation v. CA, it was pronounced:
“To be considered a “regular practice,” however, the giving of the bonus should have been done over a
long period of time, and must be shown to have been consistent and deliberate. The test or rationale of this
rule on long practice requires an indubitable showing that the employer agreed to continue giving the
benefits knowing fully well that said employees are not covered by the law requiring payment thereof.”

Angeles| Bajana | Balladares | Brillantes | Briones | Cabansag | Callanta | Chua | David|

De Leon | Gomez | Lopez | Macalino | Nostratis | Padilla | Reynon | Santos | Tan |Velasco
4E / 4F - 2018-2019
Page 147 of 920
LABOR REVIEW DIGEST
Atty. Joyrich Golangco

The records show that ETPI, aside from complying with the regular 13th month bonus, has been further giving its
employees 14th month bonus every April as well as 15th and 16th month bonuses every December of the year,
without fail, from 1975 to 2002 or for 27 years whether it earned profits or not. The considerable length of time
ETPI has been giving the special grants to its employees indicates a unilateral and voluntary act on its part to
continue giving said benefits knowing that such act was not required by law. Accordingly, a company practice in
favor of the employees has been established and the payments made by ETPI pursuant thereto ripened into
benefits enjoyed by the employees.

Other Notes/ SC Pronouncements:


______________________________________________________________________

Article 106

1. GSIS vs NLRC
G.R. No. 180045; November 17, 2010

PONENTE: Nachura, J.:

DOCTRINE: In the event that the contractor or subcontractor fails to pay the wages of his employees in
accordance with the LC, the employer shall be jointly and severally liable with his contractor or subcontractor to
such employees to the extent of the work performed under the contract, in the same manner and extent that he is
liable to employees directly employed by him.

FACTS:
Respondents Dionisio Banlasan, et. al. were security guards under DNL Security Agency (DNL Security) assigned
to petitioner GSIS Tacloban City office.

In February 1993, DNL Security informed respondents that its service contract with petitioner was terminated.
Despite this they were instructed by DNL to continue reporting for work. Respondents worked until April 20, 1993,
but without receiving their wages; after which, they were terminated from employment.

Respondent filed a complaint with the NLRC against DNL Security and petitioner for illegal dismissal, separation
pay, salary differential, 13th month pay, and payment of unpaid salary.

LA/NLRC Ruling:
LA found that there was no illegal dismissal because the employment of security guards is dependent on the
service contract between security agency and its client. However, by reason of social juctice, LA ordered DNL and
GSIS to pay respondent separation pay, salary differential, and 13th month pay.

NLRC, dismissed DNL Securitys motion for reconsideration as it was not legally perfected, and petitioner’s
appeal, having been filed beyond the reglementary period.

CA Ruling:
CA affirmed NLRC ruling.

ISSUE:
Whether or not GSIS, not having an actual and direct ER-EE relationship with private respondents is
absolved from liability to the latter.

HELD:
Angeles| Bajana | Balladares | Brillantes | Briones | Cabansag | Callanta | Chua | David|

De Leon | Gomez | Lopez | Macalino | Nostratis | Padilla | Reynon | Santos | Tan |Velasco
4E / 4F - 2018-2019
Page 148 of 920
LABOR REVIEW DIGEST
Atty. Joyrich Golangco

NO. The fact that there is no actual and direct employer-employee relationship between petitioner and
respondents does not absolve the former from liability for the latters monetary claims. When petitioner
contracted DNL Security’s services, petitioner became an indirect employer of respondents, pursuant to Article
107 of the Labor Code.

After DNL Security failed to pay respondents the correct wages and other monetary benefits, petitioner, as
principal, became jointly and severally liable, as provided in Articles 106 and 109 of the Labor Code.

The contractor or subcontractor is made liable by virtue of his or her status as a direct employer, and the principal
as the indirect employer of the contractors employees. This liability facilitates, if not guarantees, payment of the
workers compensation, thus, giving the workers ample protection as mandated by the 1987 Constitution. This is
not unduly burdensome to the employer. Should the indirect employer be constrained to pay the workers, it can
recover whatever amount it had paid in accordance with the terms of the service contract between itself and the
contractor.

Petitioners liability covers the payment of respondents salary differential and 13th month pay during the time they
worked for petitioner. In addition, petitioner is solidarily liable with DNL Security for respondent’s unpaid
wages from February 1993 until April 20, 1993. While it is true that respondents continued working for petitioner
after the expiration of their contract, based on the instruction of DNL Security, petitioner did not object to such
assignment and allowed respondents to render service. Thus, petitioner impliedly approved the extension of
respondents services. Petitioner cannot be allowed to deny its obligation to respondents after it had benefited
from their services. Petitioners liability, however, cannot extend to the payment of separation pay.

Lastly, petitioner’s argument that the enforcement of the decision is impossible because its charter unequivocally
exempts it from execution is untenable.

The processual exemption of the GSIS funds and properties under Section 39 of the GSIS Charter should be read
consistently with its avowed principal purpose: to maintain actuarial solvency of the GSIS in the protection of
assets which are to be used to finance the retirement, disability and life insurance benefits of its members.

Furthermore, GSIS is granted the ancillary power to invest in business and other ventures for the benefit of the
employees, by using its excess funds for investment purposes. In the exercise of such function and power, the
GSIS is allowed to assume a character similar to a private corporation. Thus, it may sue and be sued, as also,
explicitly granted by its charter x x x.

*(The Court notes, however, while it agrees with the NLRC’s dismissal petitioners appeal for being filed
out of time, it nonetheless delved into the merits of the case)

Other Notes/ SC Pronouncements:


______________________________________________________________________

2. Aliviado vs. Procter & Gamble Phils., Inc.


G.R. No. 160506; June 6, 2011
Del Castillo, J.

FACTS:
Angeles| Bajana | Balladares | Brillantes | Briones | Cabansag | Callanta | Chua | David|

De Leon | Gomez | Lopez | Macalino | Nostratis | Padilla | Reynon | Santos | Tan |Velasco
4E / 4F - 2018-2019
Page 149 of 920
LABOR REVIEW DIGEST
Atty. Joyrich Golangco

On March 9, 2010, this Court rendered a Decision holding: (a) that Promm-Gem, Inc. (Promm-Gem) is a legitimate
independent contractor; (b) that Sales and Promotions Services (SAPS) is a labor-only contractor consequently its
employees are considered employees of Procter & Gamble Phils., Inc. (P&G). It was established that SAPS has
no substantial capitalization and it was performing merchandising and promotional activities which are directly
related to P&G's business.

ISSUE: whether or not P&G is liable for the illegal dismissal of petitioners

HELD: YES. Article 106 defines labor-only contracting, viz:

There is labor-only contracting where the person supplying workers to an employer does not have
substantial capital or investment in the form of tools, equipment, machineries, work premises, among
others, and the workers recruited and placed by such person are performing activities which are directly
related to the principal business of such employer. In such cases, the person or intermediary shall be
considered merely as an agent of the employer who shall be responsible to the workers in the same
manner and extent as if the latter were directly employed by him.

On the same vein, Rule VIII-A, Book III of the Omnibus Rules Implementing the Labor Code, as amended by
Department Order No. 18-02, pertinently provides:

Section 5. Prohibition against labor-only contracting. Labor only contracting is hereby declared
prohibited. For this purpose, labor-only contracting shall refer to an arrangement where the contractor or
subcontractor merely recruits, supplies or places workers to perform a job, work or service for a principal,
and ANY of the following elements are present:

i)The contractor or subcontractor does not have substantial capital or investment which relates to the job,
work or service to be performed and the employees recruited, supplied or placed by such contractor or
subcontractor are performing activities which are directly related to the main business of the principal;
OR

ii) [T]he contractor does not exercise the right to control over the performance of the work of the
contractual employee.

In our March 9, 2010 Decision, it was established that SAPS has no substantial capitalization and it was
performing merchandising and promotional activities which are directly related to P&G's business. Since SAPS
met one of the requirements, it was enough basis for us to hold that it is a labor-only contractor.

The records also show that Promm-Gem supplied its complainant-workers with the relevant materials, such as
markers, tapes, liners and cutters, necessary for them to perform their work. Promm-Gem also issued uniforms to
them. It is also relevant to mention that Promm-Gem already considered the complainants working under it as its
regular, not merely contractual or project, employees. This circumstance negates the existence of element (ii) as
stated in Section 5 of DOLE Department Order No. 18-02, which speaks of contractual employees. This,
furthermore, negates on the part of Promm-Gem bad faith and intent to circumvent labor laws which factors have
often been tipping points that lead the Court to strike down the employment practice or agreement concerned as
contrary to public policy, morals, good customs or public order.

Under the circumstances, Promm-Gem cannot be considered as a labor-only contractor. We find that it is a
legitimate independent contractor.

Angeles| Bajana | Balladares | Brillantes | Briones | Cabansag | Callanta | Chua | David|

De Leon | Gomez | Lopez | Macalino | Nostratis | Padilla | Reynon | Santos | Tan |Velasco
4E / 4F - 2018-2019
Page 150 of 920
LABOR REVIEW DIGEST
Atty. Joyrich Golangco

On the other hand, SAPS failed to show that its paid-in capital of P31,250.00 is sufficient for the period required
for it to generate [the] needed revenue to sustain its operations independently. Substantial capital refers to
capitalization used in the performance or completion of the job, work or service contracted out. In the present
case, SAPS failed to show substantial capital.

Other Notes/ SC Pronouncements:


______________________________________________________________________

3. Mandaue Galleon Trade vs Andales, et. al


GR No: 159668 Date: March 7, 2009
Ponente: Austria-Martinez, J

Doctrine: "labor-only" contracting exists when the following criteria are present:

(1) where the contractor or subcontractor supplying workers to an employer does not have substantial capital or
investment in the form of tools, equipment, machineries, work premises, among other things; and the workers
recruited and placed by the contractor or subcontractor are performing activities which are directly related to the
principal business of such employer; or

(2) where the contractor does not exercise the right to control the performance of the work of the contractual
employee.

FACTS:

Petitioners Mandaue Galleon Trade, Inc. (MGTI) and Gamallosons Traders, Inc.(GTI) are business entities
engaged in rattan furniture manufacturing for export.
Respondent Vicente Andales (Andales) filed a complaint with the Labor Arbiter against both petitioners for illegal
dismissal and non-payment of 13th month pay and service incentive leave pay.
His other co-workers numbering 260 filed a similar complaint against petitioner MGTI only.
The complainants alleged that MGTI hired them on various dates as weavers, grinders, sanders and finishers;
sometime in August 1998, workers in the Finishing Department were told that they would be transferred to a
contractor and they were given Visitor Identification Cards (IDs), while workers in the Weaving Department were
told to look for work elsewhere as the company had no work for them; sometime in September 1998, workers in
the Grinding Department were not allowed to enter the company premises, while workers in the Sanding
Department were told that they could no longer work since there was no work available; workers who were issued
IDs were allowed to go inside the premises; and they were dismissed without notice and just cause.

LA/RTC/NLRC RULING:
LA rendered a Decision holding that 183 complainants are regular piece-rate employees of MGTI since they were
made to perform functions which are necessary to MGTI's rattan furniture manufacturing business
The LA ordered petitioners to take back complainants and directed it to pay their 13th month pay.
NLRC rendered a Decision affirming the LA's finding of employer-employee relationship.

CA RULING:
CA rendered a Decision dismissing the petition and affirming the findings of the NLRC. It held that MGTI is liable
to the respondents because the alleged contractors are not independent contractors but labor-only contractors;

Angeles| Bajana | Balladares | Brillantes | Briones | Cabansag | Callanta | Chua | David|

De Leon | Gomez | Lopez | Macalino | Nostratis | Padilla | Reynon | Santos | Tan |Velasco
4E / 4F - 2018-2019
Page 151 of 920
LABOR REVIEW DIGEST
Atty. Joyrich Golangco

that respondents were constructively dismissed when they were unilaterally transferred to another contractor; and
that the allegation of retrenchment was not proven.

Petitioner's Contention:

They alleged that they are regular employees of MGTI because:


(a) they performed their work inside the company premises;
(b) they were issued uniforms by MGTI and were told to strictly follow company rules and regulations;
(c) they were under the supervision of MGTI's foremen, quality control personnel and checkers;
(d) MGTI supplied the materials, designs, tools and equipment in the production of furniture;
(e) MGTI conducts orientations on how the work was to be done and the safe and efficient use of tools and
equipment;
(f) MGTI issues memoranda regarding absences and waste of materials; and
(g) MGTI exercises the power to discipline them.

Respondent's Contention:

MGTI denied the existence of employer-employee relationship with complainants, claiming that they are workers
of independent contractors whose services were engaged temporarily and seasonally when the demands for its
products are high and could not be met by its regular workforce

ISSUE/S:

WON the piece-rate employees of GTI are to be considered as indirectly employed through labor-only contracting
by principal MGTI and are thus entitle to separation pay

HELD:

Article 106 of the Labor Code explains the relations which may arise between an employer, a contractor and the
contractor's employees thus:
“ART. 106. Contractor or subcontractor. Whenever an employer enters into a contract with another person for the
performance of the former's work, the employees of the contractor and of the latter's subcontractor, if any, shall be
paid in accordance with the provisions of this Code.
In the event that the contractor or subcontractor fails to pay the wages of his employees in accordance with this
Code, the employer shall be jointly and severally liable with his contractor or subcontractor to such employees to
the extent of the work performed under the contract, in the same manner and extent that he is liable to employees
directly employed by him.
The Secretary of Labor may, by appropriate regulations, restrict or prohibit the contracting out of labor to protect
the rights of workers established under this Code. In so prohibiting or restricting, he may make appropriate
distinctions between labor-only contracting and job... contracting as well as differentiations within these types of
contracting and determine who among the parties involved shall be considered the employer for purposes of this
Code, to prevent any violation or circumvention of any provision of this Code.”
The first two paragraphs of Article 106 set the general rule that a principal is permitted by law to engage the
services of a contractor for the performance of a particular job, but the principal, nevertheless, becomes solidarily
liable with the contractor for the wages of the contractor's employees. The third paragraph of Article 106, however,
empowers the Secretary of Labor to make distinctions between permissible job contracting and "labor-only"
contracting, which is a prohibited act further defined under the last paragraph. A finding that a contractor is a
"labor-only" contractor is equivalent to declaring that there is an employer-employee relationship between the
principal and the employees of the supposed contractor, and the "labor-only" contractor is considered as a mere
agent of the principal, the real employer.
Angeles| Bajana | Balladares | Brillantes | Briones | Cabansag | Callanta | Chua | David|

De Leon | Gomez | Lopez | Macalino | Nostratis | Padilla | Reynon | Santos | Tan |Velasco
4E / 4F - 2018-2019
Page 152 of 920
LABOR REVIEW DIGEST
Atty. Joyrich Golangco

Based on Article 106 of the Labor Code and Sections 5 and 7 of the Implementing Rules, "labor-only" contracting
exists when the following criteria are present: (1) where the contractor or subcontractor supplying workers to an
employer does not have substantial capital or investment in the form of tools, equipment, machineries, work
premises, among other things; and the workers recruited and placed by the contractor or subcontractor are
performing activities which are directly related to the principal business of such employer; or (2) where the
contractor does not exercise the right to control the performance of the work of the contractual employee.

In the present case, petitioners' claim that their contractors are independent contractors, and, therefore, this case
is one of permissible job contracting, is without basis.
First, respondents' work as weavers, grinders, sanders and finishers is directly related to MGTI's principal
business of rattan furniture manufacturing. Where the employees are tasked to undertake activities usually
desirable or necessary in the usual business of the employer, the contractor is considered as a "labor-only"
contractor and such employees are considered as regular employees of the employer.
Second, MGTI was unable to present any proof that its contractors had substantial capital. There was no evidence
pertaining to the contractors' capitalization; nor to their investment in tools, equipment or implements actually used
in the performance or completion of the... job, work, or service that they were contracted to render. The law casts
the burden on the contractor to prove that it has substantial capital, investment, tools, etc. Employees, on the
other hand, need not prove that the contractor does not have substantial capital investment, and tools to engage
in job-contracting.
Thus, the contractors are "labor-only" contractors since they do not have substantial capital or investment which
relates to the service performed and respondents performed activities which were directly related to MGTI's main
business. MGTI, the principal employer, is solidarily liable with the labor-only contractors, for the rightful claims of
the employees. Under this set-up, "labor-only" contractors are deemed agents of the principal, MGTI, and the law
makes the principal responsible to the employees of the "labor-only" contractor as if the principal itself directly
hired or employed the employees. In prohibiting "labor-only" contracting and creating an employer-employee
relationship between the principal and the supposed contractor's employees, the law intends to prevent employers
from circumventing labor laws intended to protect employees.

Other Notes/ SC Pronouncements:


______________________________________________________________________

4. SPIC N SPAN SERVICES CORP vs PAJE, ET. AL.


GR No: 174087 Date: August 25, 2010
Ponente: Brion, J.

Doctrine:
In permissible job contracting, the principal agrees to put out or farm out with a contractor or subcontractor the
performance or completion of a specific job, work or service within a definite or predetermined period, regardless
of whether such job, work or service is to be performed or completed within or outside the premises of the
principal. The test is whether the independent contractor has contracted to do the work according to his own
methods and without being subject to the principals control except only as to the results, he has substantial
capital, and he has assured the contractual employees entitlement to all labor and occupational safety and health
standards, free exercise of the right to self-organization, security of tenure, and social and welfare benefits.

FACTS:
Swift Foods, Inc. (Swift) is a subsidiary of RFM Corporation that manufactures and processes meat
products and other food products. SNSs business is to supply manpower services to its clients for a fee. Swift and
SNS have a contract to promote Swift products. Inocencio Fernandez, Edelisa F. David, Thelma Guardian, Juliet
C. Dingal, Fe S. Bernardo, Lolita Gomez, Myra Amante, Miriam S. Catacutan, Gloria O. Sumang, Gloria O. Paje,
Angeles| Bajana | Balladares | Brillantes | Briones | Cabansag | Callanta | Chua | David|

De Leon | Gomez | Lopez | Macalino | Nostratis | Padilla | Reynon | Santos | Tan |Velasco
4E / 4F - 2018-2019
Page 153 of 920
LABOR REVIEW DIGEST
Atty. Joyrich Golangco

and Estrella Zapata (complainants) worked as Deli/Promo Girls of Swift products in various supermarkets in
Tarlac and Pampanga. They were all dismissed from their employment on February 28, 1998.They filed two
complaints for illegal dismissal against SNS and Swift before the National Labor Relations Commission (NLRC)
Regional Arbitration Branch III, San Fernando, Pampanga, docketed as Case Nos. 03-9131-98 and 07-9295-
98. These cases were subsequently consolidated.

After two unsuccessful conciliation hearings, the Labor Arbiter ordered the parties to submit their position
papers. Swift filed its position paper; SNS did not. The complainants position papers were signed by Florencio P.
Peralta who was not a lawyer and who claimed to be the complainants representative, although he never showed
any proof of his authority to represent them.

In their position papers, the complainants alleged that they were employees of Swift and SNS, and their
services were terminated without cause and without due process. The termination came on the day they received
their notices; thus, they were denied the procedural due process requirements of notice and hearing prior to their
termination of employment. Swift, in its position paper, moved to dismiss the complaints on the ground that it
entered into an independent labor contract with SNS for the promotion of its products; it alleged that the
complainants were the employees of SNS, not of Swift.

LA RULING:
SNS to be the agent of Swift, and ordered SNS and Swift to jointly and severally pay Edelisa David P115,637.50
and Inocencio Fernandez P192,197.50, representing their retirement pay and service incentive leave pay. He
dismissed, without prejudice, the claims of the other complainants because they failed to verify their position
paper. He also denied all other claims for lack of factual basis.

NLRC RULING
It dismissed the complaint against Swift, and ordered SNS to pay Edelisa David a total of P256,620.13, and
Inocencio Fernandez a total of P280,912.63, representing backwages, separation pay, and service incentive leave
pay. It dismissed all other claims for lack of merit. Thereafter, Edelisa David and Inocencio Fernandez agreed to a
settlement, and their cases were thus closed.

CA RULING:
SNS was merely an agent of Swift; thus, the latter should not be exempt from liability. It ordered the remand of the
case to the Labor Arbiter for the computation of the respondents backwages, separation pay, and service
incentive leave pay.

APPEAL TO THE SC:

ISSUE/S:
Whether or not SNS is merely a labor-only contractor

HELD:
In order that a labor relationship can be categorized as legitimate/permissible job contracting or as
prohibited labor-only contracting, the totality of the facts and the surrounding circumstances of the relationship
ought to be considered. Every case is unique and has to be assessed on the basis of its facts and of the features
of the relationship in question. In permissible job contracting, the principal agrees to put out or farm out with a
contractor or subcontractor the performance or completion of a specific job, work or service within a definite or
predetermined period, regardless of whether such job, work or service is to be performed or completed within or
outside the premises of the principal. The test is whether the independent contractor has contracted to do the
work according to his own methods and without being subject to the principals control except only as to the
results, he has substantial capital, and he has assured the contractual employees entitlement to all labor and
Angeles| Bajana | Balladares | Brillantes | Briones | Cabansag | Callanta | Chua | David|

De Leon | Gomez | Lopez | Macalino | Nostratis | Padilla | Reynon | Santos | Tan |Velasco
4E / 4F - 2018-2019
Page 154 of 920
LABOR REVIEW DIGEST
Atty. Joyrich Golangco

occupational safety and health standards, free exercise of the right to self-organization, security of tenure, and
social and welfare benefits.

The CA found SNS to be Swifts agent, and explained its ruling as follows

To be legitimate, contracting or subcontracting must satisfy the following requirements:


1) The contractor or subcontractor carries on a distinct and independent business and
undertakes to perform the job, work or service on its own account and under its own
responsibility, according to its own manners and methods, and free from the control and
direction of the principal in all matters connected with the performance of the work except as to
the results thereof; 2) the contractor or subcontractor has substantial capital or investment; and
3) the agreement between the principal and contractor or subcontractor assures the contractual
employees entitlement to all labor and occupational safety and health standards, free exercise
of right to self-organization, security of tenure, and social and welfare benefit (Vinoya v. NLRC,
324 SCRA 469).

The parties failed to attach a copy of the agreement entered into between SNS and
Swift. Neither did they attach a copy of the financial statement of SNS. Thus, we are
constrained to rule on the issue involved on the basis of the findings of both the Labor Arbiter
and the NLRC.

The Labor Arbiter, in finding that SNS was merely a labor-only contractor, cited the
following reasons: First, the agreement between SNS and Swift shows that the latter exercised
control over the promo girls and/or merchandisers through the services of coordinators. Second,
it cannot be said that SNS has substantial capital. Third, the duties of the petitioners were
directly related, necessary and vital to the day-to-day operations of Swift. Lastly, the uniform
and identification cards used by the petitioners were subject to the approval of Swift.

The NLRC, on the other hand, in finding that SNS is an independent contractor gave
the following reasons: First, there is no evidence that Swift exercised the power of control over
the petitioners. Rather, it is SNS who exercised direct control and supervision over the nature
and performance of the works of herein petitioners. Second, by law, Swift and SNS have distinct
and separate juridical personality from each other.

The decision of the NLRC is bereft of explanation as to the existence of circumstances


that would make SNS an independent contractor as would exempt the principal from liabilities to
the employees.

Nowhere in the decision of both the Labor Arbiter and the NLRC shows that SNS had
full control of the means and methods of the performance of their work. Moreover, as found by
the Labor Arbiter, there was no evidence that SNS has substantial capital or investment.Lastly,
there was no finding by the Labor Arbiter nor the NLRC that the agreement between the
principal (Swift) and contractor (SNS) assures the contractual employees entitlement to all labor
and occupational safety and health standards, free exercise of right to self-organization, security
of tenure, and social and welfare benefit.

In view of the foregoing, we conclude that the requisites above-mentioned are not
obtaining in the present case. Hence, SNS is considered merely an agent of Swift which does
not exempt the latter from liability.

Angeles| Bajana | Balladares | Brillantes | Briones | Cabansag | Callanta | Chua | David|

De Leon | Gomez | Lopez | Macalino | Nostratis | Padilla | Reynon | Santos | Tan |Velasco
4E / 4F - 2018-2019
Page 155 of 920
LABOR REVIEW DIGEST
Atty. Joyrich Golangco

We note that the present decision does not affect the settlement entered into between
Edeliza David and Inocencio Fernandez, on the one hand and SNS, on the other. As held by the
NLRC, their complaints are considered closed and terminated.

WHEREFORE, premises considered, the instant petition is hereby GRANTED. The


Resolutions of the NLRC dated January 11, 2002 and December 23, 2003 are SET ASIDE in so
far as the dismissal of the petitioners case is concerned and in so far as Swift is found not liable
for the payment of the petitioners money claims.

The present case is hereby REMANDED to the Labor Arbiter for the computation of the
money claims of the petitioners, to wit: 1) Backwages; 2) Separation Pay; and 3) Service
Incentive Leave Pay.

The settlement of the claims of David and Fernandez is not affected by this decision.

We fully agree with this ruling. What we have before us, therefore, is a case of illegal dismissal
perpetrated by a principal and its illegal contractor-agent. Thus, we affirm the ruling of the CA with the
modification that the respondents are also entitled to nominal damages, for violation of their due process rights to
notice and hearing, pursuant to our ruling in Agabon v. NLRC. We peg this amount at P30,000.00 for each of the
respondents.

Other Notes/ SC Pronouncements:

______________________________________________________________________

Angeles| Bajana | Balladares | Brillantes | Briones | Cabansag | Callanta | Chua | David|

De Leon | Gomez | Lopez | Macalino | Nostratis | Padilla | Reynon | Santos | Tan |Velasco
4E / 4F - 2018-2019
Page 156 of 920
LABOR REVIEW DIGEST
Atty. Joyrich Golangco

Article 110

1. DBP v. NLRC
G.R. No. 108031
March 1, 1995
Bellosillo J.

Doctrine

The rationale is that to hold Art. 110 to be applicable also to extrajudicial proceedings would be putting the worker
in a better position than the State which could only assert its own prior preference in case of a judicial
proceeding. Art. 110, which was amended by R.A. 6715 effective 21 March 1989, now reads:
Art. 110. Worker preference in case of bankruptcy. — In the event of bankruptcy or liquidation of an employer's
business, his workers shall enjoy first preference as regards their unpaid wages and other monetary claims, any
provision of law to the contrary notwithstanding. Such unpaid wages and monetary claims shall be paid in full
before the claims of the Government and other creditors may be paid.
Obviously, the amendment expanded the concept of "worker preference" to cover not only unpaid wages but also
other monetary claims to which even claims of the Government must be deemed subordinate.

Facts

Private respondent Leonor A. Ang started employment as Executive Secretary with Tropical Philippines Wood
Industries, Inc. She was later promoted to the position of Personnel Officer.

Development Bank of the Philippines foreclosed TPWII plant facilities and equipment but the latter continued its
business operations. In January 1986 petitioner took possession of the foreclosed properties. From then on the
company ceased its operations. As a consequence, private respondent was verbally terminated from the service.

Private respondent filed with the Labor Arbiter a complaint for separation pay, 13th month pay, vacation and sick
leave pay, salaries and allowances against TPWII, its General Manager, and petitioner.

L.A. Ruling

After hearing the Labor Arbiter found TPWII primarily liable to private respondent but only for her separation pay
and vacation and sick leave pay because her claims for unpaid wages and 13th month pay were later paid after
the complaint was filed. DBP was held subsidiarily liable in the event the company failed to satisfy the judgment.

NLRC Ruling

The National Labor Relations Commission affirmed the ruling of the Labor Arbiter.

Petitioner’s Contention

Angeles| Bajana | Balladares | Brillantes | Briones | Cabansag | Callanta | Chua | David|

De Leon | Gomez | Lopez | Macalino | Nostratis | Padilla | Reynon | Santos | Tan |Velasco
4E / 4F - 2018-2019
Page 157 of 920
LABOR REVIEW DIGEST
Atty. Joyrich Golangco

Petitioner argues that the decision of public respondent runs counter to the consistent rulings of this Court in a
long line of cases emphasizing that the application of Art. 110 of the Labor Code is contingent upon the institution
of bankruptcy or judicial liquidation proceedings against the employer.

Respondent’s Contention

----

Issue

Is declaration of bankruptcy or judicial liquidation required before the worker's preference may be invoked under
Art. 110 of the Labor Code?

Held

 Art. 110 should not be treated apart from other laws but applied in conjunction with the pertinent
provisions of the Civil Code and the Insolvency Law to the extent that piece-meal distribution of the
assets of the debtor is avoided. Complementing Art. 110, Sec. 10, Rule VIII, Book III, of the Revised
Rules and Regulations Implementing the Labor Code.

 The rationale is that to hold Art. 110 to be applicable also to extrajudicial proceedings would be putting
the worker in a better position than the State which could only assert its own prior preference in case of a
judicial proceeding. Art. 110, which was amended by R.A. 6715 effective 21 March 1989, now reads:

Art. 110. Worker preference in case of bankruptcy. — In the event of bankruptcy or liquidation of an
employer's business, his workers shall enjoy first preference as regards their unpaid wages and other
monetary claims, any provision of law to the contrary notwithstanding. Such unpaid wages and monetary
claims shall be paid in full before the claims of the Government and other creditors may be paid.

Obviously, the amendment expanded the concept of "worker preference" to cover not only unpaid wages
but also other monetary claims to which even claims of the Government must be deemed subordinate.

In the present case, there is as yet no declaration of bankruptcy nor judicial liquidation of TPWII. Hence,
it would be premature to enforce the worker's preference.

Other Notes/ SC Pronouncements:


______________________________________________________________________

Angeles| Bajana | Balladares | Brillantes | Briones | Cabansag | Callanta | Chua | David|

De Leon | Gomez | Lopez | Macalino | Nostratis | Padilla | Reynon | Santos | Tan |Velasco
4E / 4F - 2018-2019
Page 158 of 920
LABOR REVIEW DIGEST
Atty. Joyrich Golangco

Article 111

1. HOEGH FLEET SERVICES PHILS., INC., and/or HOEGH FLEET SERVICES AS vs. BERNARDO M.
TURALLO
G.R. No. 230481; July 26, 2017
VELASCO, JR., J.

DOCTRINE: The extraordinary concept of attorney's fees is the one contemplated in Article 111 of the Labor
Code. This is awarded by the court to the successful party to be paid by the losing party as indemnity for damages
sustained by the former in prosecuting, through counsel, his cause in court.

FACTS:
On 9 November 2012, petitioners hired Turallo as a Messman on board vessel "Hoegh Tokyo" for nine
(9) months. The employment contract was signed on 27 December 2012. Turallo was found "fit for sea duty" in the
Pre-Employment Medical Examination (PEME). On 2 January 2013, Turallo boarded the vessel.
Sometime in September 2013 while on board the vessel, Turallo felt pain on the upper back of his body
and chest pain, which was reported to his superiors on 23 September 2013. Upon arrival in Manila, Turallo was
referred to the company-designated physician, who in turn referred him to an orthopedic surgeon and cardiologist.
He underwent medical and laboratory tests.
Despite Turallo' s continuous rehabilitation treatment, pain in his left shoulder persisted, hence, he
followed up his pending surgery therefor several times to no avail. This prompted Turallo to seek a second
opinion. On 13 May 2014, Turallo consulted with Dr. Manuel Fidel Magtira, a government physician of the Vizcarra
Diagnostic Center who, after x-ray of his left wrist and shoulder joints, found him to be "partially and permanently
disabled with separate impediments for the different affected parts of (his) body of Grade 8, Grade 10 and Grade
11, based on the POEA contract" but declared him as "permanently unfit in any capacity for further sea duties".
On 23 May and 2 June 2014, grievance proceedings were held between the parties at the AMOSUP,
where the petitioners offered the amount of Thirty Thousand Two Hundred Thirty One US Dollars (US$30,231. 00)
corresponding to .a Grade 8 disability compensation based on the maximum amount of Ninety Thousand US
Dollars (US$90,000.00). Turallo, however proposed the settlement .amount of Sixty Thousand US Dollars
(US$60,000.00). The parties failed to reach an agreement. Despite efforts to arrive at an agreement, the parties
failed to settle their differences, hence, they were directed to submit their pleadings and evidence for the
resolution of the issues before the panel of arbitrators.
PANEL DECISION: Ordered petitioners, jointly and severally, to pay complainant the following amounts:
1. Disability compensation in the amount of US$90,000.00, to be paid in the equivalent peso amount at the rate
prevailing at the time of payment.
2. Sickness Allowance in the amount of US$3,084.54 to be paid in its peso equivalent as in number l; and
3. Attorney's fees equivalent to ten percent (10%) of the total monetary award.
Finally, legal interests shall be imposed on the monetary awards herein granted at the rate of 6% per annum from
finality of this judgment until fully paid.
Hoegh Fleet argued that the Panel erred in ruling that Turallo is entitled to the award of attorney's fees
for being unwarranted as there was no showing of an unjustified act or evident bad faith on its part for denying
Turallo's claim.

CA RULING: CA did not dispute Turallo' s entitlement to attorney’s fees, it ruled that reducing the amount from
ten percent (10%) of the total monetary award to just One Thousand US Dollars (US$1,000.00) would be
reasonable.

Angeles| Bajana | Balladares | Brillantes | Briones | Cabansag | Callanta | Chua | David|

De Leon | Gomez | Lopez | Macalino | Nostratis | Padilla | Reynon | Santos | Tan |Velasco
4E / 4F - 2018-2019
Page 159 of 920
LABOR REVIEW DIGEST
Atty. Joyrich Golangco

ISSUE:
Whether or not Turallo is entitled to attorney’s fees.

RULING:
YES. The Court agrees with the CA that attorney's fees should be reduced, not to US$1,000.00,
however, but to five percent (5%) of the total monetary award.1âwphi1 Article 111 of the Labor Code indeed
provides that the culpable party may be assessed attorney's fees equivalent to 10 percent of the amount of wages
recovered. It also provides that it shall be unlawful for any person to demand or accept, in any judicial or
administrative proceedings for the recovery of wages, attorney's fees which exceed 10 percent of the amount of
wages recovered. Section 8, Rule VIII, Book III of the Implementing Rules of the Labor Code sustains the same
and states that attorney's fees shall not exceed 10 percent of the amount awarded. A closer reading of these
provisions, however, would lead us to the conclusion that the 10 percent only serves as the maximum of the
award that may be granted. Relevantly, We have ruled in the case of Taganas v. National Labor Relations
Commission that Article 111 does not even prevent the NLRC from fixing an amount lower than the ten percent
ceiling prescribed by the article when the circumstances warrant it. With that, the Court is not tied to award 10
percent attorney's fees to the winning party, as what Turallo wishes to imply.
In PCL Shipping Philippines, Inc. v. National Labor Relations Commissionthe Court discussed that there
are two commonly accepted concepts of attorney's fees, the so-called ordinary and extraordinary. In its ordinary
concept, an attorney's fee is the reasonable compensation paid to a lawyer by his client for the legal services he
has rendered to the latter. The basis of this compensation is the fact of his employment by and his agreement with
the client. In its extraordinary concept, attorney's fees are deemed indemnity for damages ordered by the court to
be paid by the losing party in a litigation. The instances where these may be awarded are those enumerated in
Article 2208 of the Civil Code, specifically par. 7 thereof which pertains to actions for recovery of wages, and is
payable not to the lawyer but to the client, unless they have agreed that the award shall pertain to the lawyer as
additional compensation or as part thereof. The extraordinary concept of attorney's fees is the one contemplated
in Article 111 of the Labor Code. This is awarded by the court to the successful party to be paid by the losing party
as indemnity for damages sustained by the former in prosecuting, through counsel, his cause in court.
Clearly, Turallo incurred legal expenses after he was forced to file an action to recover his disability
benefits. Considering that he was constrained to litigate with counsel in all the stages of this proceeding, and
keeping in mind the liberal and compassionate spirit of the Labor Code, where the employees' welfare is the
paramount consideration, this Court considers five percent (5%) of the total monetary award as more appropriate
and commensurate under the circumstances of this petition.
______________________________________________________________________

Article 113

1. SHS PERFORATED MATERIALS, INC., WINFRIED HARTMANNSHENN, and HINRICH JOHANN


SCHUMACHER v. MANUEL F. DIAZ
GR No: 185814 Date: October 13, 2010
Ponente: Mendoza, J.

Doctrine: Art. 113. Any withholding of an employee’a wages by an employer may only be allowed in the form of
wage deductions under the circumstances provided in Article 113 of the Labor Code

FACTS:
Petitioner SHS Perforated Materials, Inc. (SHS) is a start-up corporation organized and existing under
the laws of the Republic of the Philippines and registered with the Philippine Economic Zone Authority. Petitioner
Winfried Hartmannshenn (Hartmannshenn), a German national, is its president. Petitioner Hinrich Johann
Schumacher (Schumacher) is the Executive Vice-President of the European Chamber of Commerce of
Angeles| Bajana | Balladares | Brillantes | Briones | Cabansag | Callanta | Chua | David|

De Leon | Gomez | Lopez | Macalino | Nostratis | Padilla | Reynon | Santos | Tan |Velasco
4E / 4F - 2018-2019
Page 160 of 920
LABOR REVIEW DIGEST
Atty. Joyrich Golangco

the Philippines (ECCP)which is a separate entity from SHS. Both entities have an arrangement where ECCP
handles the payroll requirements of SHS to simplify business operations and minimize operational expenses.
Thus, the wages of SHS employees are paid out by ECCP, through its Accounting Services Department headed
by Juliet Taguiang (Taguiang).
Manuel F. Diaz (respondent) was hired by petitioner SHS as Manager for Business Development on
probationary status from July 18, 2005 to January 18, 2006, with a monthly salary of P100,000.00.
During respondents employment, Hartmannshenn was often abroad and, because of business
exigencies, his instructions to respondent were either sent by electronic mail or relayed through telephone or
mobile phone. When he would be in the Philippines, he and the respondent held meetings. As to respondents
work, there was no close supervision by him. During meetings with the respondent, Hartmannshenn expressed
his dissatisfaction over respondent’s poor performance. Respondent allegedly failed to make any concrete
business proposal or implement any specific measure to improve the productivity of the SHS office and plant or
deliver sales. Respondent also failed to communicate with Hartmannshenn upon the latter’s arrival in the
Philippines. On November 29, 2005, Hartmannshenn instructed Taguiang not to release respondent’s salary.
Later that afternoon, respondent called and inquired about his salary. Taguiang informed him that it was being
withheld and that he had to immediately communicate with Hartmannshenn. Again, respondent denied having
received such directive. The next day, respondent served on SHS a demand letter and resignation letter.
Respondent then filed a complaint for illegal dismissal and non-payment of his salaries
LA RULING: The LA found that respondent was constructively dismissed because the withholding of his salary
was contrary to Article 116 of the Labor Code as it was not one of the exceptions for allowable wage deduction by
the employer under Article 113 of the Labor Code. He had no other alternative but to resign because he could not
be expected to continue working for an employer who withheld wages without valid cause. The LA also held that
respondents probationary employment was deemed regularized because petitioners failed to conduct a prior
evaluation of his performance and to give notice two days prior to his termination as required by the Probationary
Contract of Employment and Article 281 of the Labor Code.

NLRC RULING: NLRC reversed the decision of the LA. The NLRC explained that the withholding of respondents
salary was a valid exercise of management prerogative. The act was deemed justified as it was reasonable to
demand an explanation for failure to report to work and to account for his work accomplishments. The NLRC held
that the respondent voluntarily resigned as evidenced by the language used in his resignation letter and demand
letters.

COURT OF APPEALS: Reversed NLRC decision, reinstated LA decision. Withholding respondents salary was not
a valid exercise of management prerogative as there is no such thing as a management prerogative to withhold
wages temporarily.

PETITION TO THE SC:


Petitioner's Contention: Petitioners contend that withholding respondent’s salary was justified because
respondent was absent and did not show up for work during that period. He also failed to account for his
whereabouts and work accomplishments during said period. When there is an issue as to whether an employee
has, in fact, worked and is entitled to his salary, it is within management prerogative to temporarily withhold an
employee’s salary/wages pending determination of whether or not such employee did indeed work.

ISSUE/S:
1. Whether or not respondent was constructively dismissed

HELD:
Yes. What made it impossible, unreasonable or unlikely for respondent to continue working for SHS was
the unlawful withholding of his salary. For said reason, he was forced to resign. Respondent was constructively
dismissed and, therefore, illegally dismissed. Although respondent was a probationary employee, he was still
Angeles| Bajana | Balladares | Brillantes | Briones | Cabansag | Callanta | Chua | David|

De Leon | Gomez | Lopez | Macalino | Nostratis | Padilla | Reynon | Santos | Tan |Velasco
4E / 4F - 2018-2019
Page 161 of 920
LABOR REVIEW DIGEST
Atty. Joyrich Golangco

entitled to security of tenure. Section 3 (2) Article 13 of the Constitution guarantees the right of all workers to
security of tenure. In using the expression all workers, the Constitution puts no distinction between a probationary
and a permanent or regular employee. This means that probationary employees cannot be dismissed except for
cause or for failure to qualify as regular employees.
Management prerogative refers to the right of an employer to regulate all aspects of employment, such
as the freedom to prescribe work assignments, working methods, processes to be followed, regulation regarding
transfer of employees, supervision of their work, lay-off and discipline, and dismissal and recall of work.] Although
management prerogative refers to the right to regulate all aspects of employment, it cannot be understood to
include the right to temporarily withhold salary/wages without the consent of the employee. To sanction such an
interpretation would be contrary to Article 116 of the Labor Code, which provides:

ART. 116. Withholding of wages and kickbacks prohibited. It shall be unlawful for any person,
directly or indirectly, to withhold any amount from the wages of a worker or induce him to give
up any part of his wages by force, stealth, intimidation, threat or by any other means whatsoever
without the workers consent.
Any withholding of an employees wages by an employer may only be allowed in the form of wage
deductions under the circumstances provided in Article 113 of the Labor Code, as set forth below:

ART. 113. Wage Deduction. No employer, in his own behalf or in behalf of any person, shall
make any deduction from the wages of his employees, except:

(a) In cases where the worker is insured with his consent by the employer, and
the deduction is to recompense the employer for the amount paid by him as
premium on the insurance;

(b) For union dues, in cases where the right of the worker or his union to check-off
has been recognized by the employer or authorized in writing by the individual
worker concerned; and

(c) In cases where the employer is authorized by law or regulations issued by the
Secretary of Labor.
As correctly pointed out by the LA, absent a showing that the withholding of complainants wages falls
under the exceptions provided in Article 113, the withholding thereof is thus unlawful. The Court finds petitioners
evidence insufficient to prove that respondent did not work from November 16 to November 30, 2005.

Respondents reinstatement, however, is no longer feasible as antagonism has caused a severe strain in
their working relationship. Under the doctrine of strained relations, the payment of separation pay is considered an
acceptable alternative to reinstatement when the latter option is no longer desirable or viable. Payment liberates
the employee from what could be a highly oppressive work environment, and at the same time releases the
employer from the obligation of keeping in its employ a worker it no longer trusts. Therefore, a more equitable
disposition would be an award of separation pay equivalent to at least one month pay, in addition to his full
backwages, allowances and other benefits.

Other Notes/ SC Pronouncements:


Petitioners Winfried Hartmannshenn and Hinrich Johann Schumacher are not solidarily liable with petitioner SHS
Perforated Materials, Inc.

______________________________________________________________________

Angeles| Bajana | Balladares | Brillantes | Briones | Cabansag | Callanta | Chua | David|

De Leon | Gomez | Lopez | Macalino | Nostratis | Padilla | Reynon | Santos | Tan |Velasco
4E / 4F - 2018-2019
Page 162 of 920
LABOR REVIEW DIGEST
Atty. Joyrich Golangco

Article 118

1. NORMAN PANALIGAN v. PHYVITA ENTERPRISES CORPORATION


GR No. 202086; 2017
FACTS: Petitioner Phyvita Enterprises Corporation is a domestic corporation organized and existing under the
Philippine laws engaged in the business of health club massages, parlor, spa and other related services under the
name and style of Starfleet Reflex. Private respondents, Panaligan et al. were the employees of Phyvita assigned
as Room boys at Starfleet.
Sometime, the Finance Assistant of Phyvita for Starfleet, Enriquez, discovered that the amount of One Hundred
Eighty Thousand Pesos (Php180,000.00) representing their sales for 22nd, 23rd and 24th of January 2005 was
missing including receipts, payrolls, credit card receipts and sales invoices.
She immediately reported the same to her immediate superior Jorge Rafols. As such, they searched for the
missing documents and cash. However, their search remained futile.
Jorge Rafols and Enriquez reported the incident to their Vice President for Operations Henry Ting.
As advised by Phyvita's Legal Officer Maria Joy Ting , they reported the alleged theft incident to the Parañaque
City Police Station to conduct an investigation. However, the Parañaque Police were not able to gather sufficient
information that would lead them as to who committed said theft. Being unsuccessful, the said police investigation
was merely entered into the police blotter.
In the interim, individual Office Memoranda were issued by Starfleet's Assistant Operations Manager Jerry Rafols
against petitioners directing them to explain in writing why no disciplinary action shall be imposed against them for
alleged violation of Class D1.14 of Starfleet's rules and regulation[s], particularly any act of dishonesty, whether
the company has incurred loss or not, more specifically their alleged involvement in a theft wherein important
documents and papers including cash were lost which happened last 25 January 2005 at [Phyvita]'s
establishment.
Acting on the said Office Memoranda, only Panaligan submitted his hand written explanation which merely stated
"wala ako kinalaman sa ibinibintang [sakin]."
Having failed to participate in the investigation proceedings conducted by Phyvita, Memoranda dated 26 May
2005 were issued against [Petitioners] informing them that they are terminated from their employment on the
ground that they violated the company's rules and regulation[s] by stealing company documents and cash. They
were also informed that such termination is without prejudice to the filing of criminal charges against them.
LA Jose G. De Vera declared in his Decision dated July 31, 2007 that PANALIGAN, et al., were legally terminated
from employment on the ground of loss of trust and confidence.
Upon appeal by PANALIGAN, et al., the aforementioned ruling was reversed and set aside by the NLRC in its
Decision dated June 9, 2009. The NLRC arrived at the conclusion that PANALIGAN, et al., were illegally
dismissed from employment. Thus, PHYVITA elevated this case to the Court of Appeals. The appellate court
reversed the NLRC issuances and reinstated.
ISSUE: Whether or not there exists just and valid cause for the termination of PANALIGAN, et al.'s, employment
by PHYVITA.
RULING: Misconduct is improper or wrong conduct; it is the transgression of some established and definite rule of
action, a forbidden act, a dereliction of duty, willful in character, and implies wrongful intent and not mere error in
judgment. The misconduct, to be serious within the meaning of the Labor Code, must be of such a grave and
aggravated character and not merely trivial or unimportant. Thus, for misconduct or improper behavior to be a just
cause for dismissal, (a) it must be serious; (b) it must relate to the performance of the employee's duties; and (c) it
must show that the employee has become unfit to continue working for the employer.
On the other hand, loss of trust and confidence, as a just cause for termination of employment, is premised on the
fact that an employee concerned holds a position where greater trust is placed by management and from whom

Angeles| Bajana | Balladares | Brillantes | Briones | Cabansag | Callanta | Chua | David|

De Leon | Gomez | Lopez | Macalino | Nostratis | Padilla | Reynon | Santos | Tan |Velasco
4E / 4F - 2018-2019
Page 163 of 920
LABOR REVIEW DIGEST
Atty. Joyrich Golangco

greater fidelity to duty is correspondingly expected. The betrayal of this trust is the essence of the offense for
which an employee is penalized.
In the case at bar, PHYVITA failed to adduce substantial evidence that would clearly demonstrate that
PANALIGAN, et al., have committed serious misconduct or have performed actions that would warrant the loss of
trust and confidence reposed upon them by their employer. Contrary to the findings of the Court of Appeals and
the Labor Arbiter, no substantial evidence supports the allegation of theft leveled by PHYVITA against
PANALIGAN, et al. - the said criminal act being the underlying reason for the dismissal of the latter from being
employees of the former.
The records of this case clearly indicate that no direct evidence was presented to link PANALIGAN, et al., to the
theft that they allegedly committed. In fact, the questioned payroll sheets that PANALIGAN, et al., attached to the
labor complaint they filed before the DOLE-NCR are the only concrete proof that PHYVITA used to support its
allegation. However, the said documents were not specifically enumerated as among the stolen items in the police
report of the alleged incident of theft, while a previous incident report merely stated that "several copies of payroll"
were taken. PHYVITA first claimed that these payroll sheets allegedly stolen from Enriquez's safekeeping were
the same ones in PANALIGAN, et al.'s, possession when its employee, Jesse Pangilinan, executed an affidavit to
that effect right after attending a preliminary hearing of the labor case initiated by PANALIGAN, et al. Pangilinan's
statement was supported by the joint affidavit made by Rommel Garcia and Jay R Kasing who were also in
PHYVITA's employ.The problem with Pangilinan's statement is that it is self-serving since it favors his employer
which is involved in a labor dispute with PANALIGAN, et al., and it does not show criminal liability since it only
establishes PANALIGAN, et al.'s, possession of the questioned payroll sheets but not the fact that they
themselves stole the same.Furthermore, Pangilinan's statement is inconsistent with the other facts on record.
According to Pangilinan's affidavit, he only knew that the questioned payroll sheets were in the possession of
PANALIGAN, et al., when they presented the same during the May 29, 2005 DOLE-NCR hearing.
The aforementioned date is crucial to this case because the month before, or on April 28, 2005, PANALIGAN, et
al., were preventively suspended from work by PHYVITA and given written notices to explain in writing within
twenty-four (24) hours why they should not face disciplinary sanction for their alleged involvement in the January
25, 2005 incident of theft. Due to their non-appearance at the scheduled in-house investigation and conference,
PANALIGAN, et al., were then served individual notices dated May 26, 2005, that they were terminated from
PHYVITA's employ for their alleged participation in the theft. Thereafter, sometime in June 2005, Garcia and
Kasing purportedly came forward and pointed to PANALIGAN, et al., as among the perpetrators of the alleged
theft. Considering the said chronology of events, there was no clear ground for PHYVITA to preventively suspend
and later terminate the services of PANALIGAN, et al., when the company's actions predated the bases for doing
so - the discovery of the questioned payroll sheets by Pangilinan allegedly on May 29, 2005 as stated in his
affidavit and the revelations of Garcia and Kasing allegedly made sometime in June 2005. Alternatively stated,
respondent company had charged and terminated PANALIGAN, et al., before it had even obtained its supposed
"proof' of their misdeed.

______________________________________________________________________

Angeles| Bajana | Balladares | Brillantes | Briones | Cabansag | Callanta | Chua | David|

De Leon | Gomez | Lopez | Macalino | Nostratis | Padilla | Reynon | Santos | Tan |Velasco
4E / 4F - 2018-2019
Page 164 of 920
LABOR REVIEW DIGEST
Atty. Joyrich Golangco

Article 124

1. P.I. MANUFACTURING, INCORPORATED vs. P.I. MANUFACTURING SUPERVISORS AND FOREMAN


ASSOCIATION and the NATIONAL LABOR UNION
GR No: 167217 Date: February 04, 2008
Ponente: Sandoval-Gutierrez, J.
Doctrine: Wage distortion means the disappearance or virtual disappearance of pay differentials between
lower and higher positions in an enterprise because of compliance with a wage order.
FACTS:
P.I. Manufacturing, Incorporated is a domestic corporation engaged in the manufacture and sale of household
appliances. On the other hand, respondent P.I. Manufacturing Supervisors and Foremen Association
(PIMASUFA) is an organization of petitioner’s supervisors and foremen, joined in this case by its federation, the
National Labor Union (NLU).
The President signed into law Republic Act (R.A.) No. 66402 providing, among others, an increase in the
statutory minimum wage and salary rates of employees and workers in the private sector. Section 2 provides:
SEC. 2. The statutory minimum wage rates of workers and employees in the private sector, whether
agricultural or non-agricultural, shall be increased by ten pesos (P10.00) per day, except non-agricultural
workers and employees outside Metro Manila who shall receive an increase of eleven pesos (P11.00)
per day: Provided, That those already receiving above the minimum wage up to one hundred
pesos (P100.00) shall receive an increase of ten pesos (P10.00) per day.
Petitioner and Respondent PIMASUFA entered into a new Collective Bargaining Agreement (1987 CBA) whereby
the supervisors were granted an increase of P625.00 per month and the foremen, P475.00 per month. The
increases were made retroactive to May 12, 1987, or prior to the passage of R.A. No. 6640, and every year
thereafter until July 26, 1989.

On January 26, 1989, respondents PIMASUFA and NLU filed a complaint with the Arbitration Branch of the
National Labor Relations Commission (NLRC), charging petitioner with violation of R.A. No. 6640.
Petitioner's Contention:
Angeles| Bajana | Balladares | Brillantes | Briones | Cabansag | Callanta | Chua | David|

De Leon | Gomez | Lopez | Macalino | Nostratis | Padilla | Reynon | Santos | Tan |Velasco
4E / 4F - 2018-2019
Page 165 of 920
LABOR REVIEW DIGEST
Atty. Joyrich Golangco

Petitioner asseverates that the wage distortion issue is already barred by Sec. 2 Article IV of the Contract
denominated as "The Company and Supervisors and Foremen Contract" dated December 18, 1987 declaring that
it "absolves, quit claims and releases the COMPANY for any monetary claim they have, if any there might
be or there might have been previous to the signing of this agreement." Petitioner interprets this as absolving
it from any wage distortion brought about by the implementation of the new minimum wage law.

LA RULING:
Petitioner was ordered to give the members of respondent PIMASUFA wage increases equivalent to 13.5% of
their basic pay they were receiving prior to December 14, 1987.
NLRC RULING:
The NLRC affirmed the LA’s ruling on appeal.
CA RULING:

The CA affirmed the NLRC with modification by raising the 13.5% wage increase to 18.5%. It further ruled that the
increase resulting from any wage distortion caused by the implementation of Republic Act 6640 is not waivable.
Thus, notwithstanding the stipulation provided under Section 2 of the Company and Supervisors and Foremen
Contract, we find the members of private respondent union entitled to the increase of their basic pay due to wage
distortion by reason of the implementation of RA 6640. It also stated that the increase of 13.5% in the supervisors
and foremen’s basic salary must further be increased to 18.5% in order to correct the wage distortion brought
about by the implementation of RA 6640.
ISSUE:
1. Whether the implementation of R.A. No. 6640 resulted in a wage distortion and whether such distortion
was cured or remedied by the 1987 CBA.

HELD:
YES. Wage distortion means the disappearance or virtual disappearance of pay differentials between lower
and higher positions in an enterprise because of compliance with a wage order. There was indeed a wage
distortion due to the implementation of R.A. No. 6640.

Despite the wage distortion, the same were cured or remedied when respondent PIMASUFA entered into the
1987 CBA with petitioner after the effectivity of R.A. No. 6640. The 1987 CBA increased the monthly salaries of
the supervisors by P625.00 and the foremen, by P475.00, effective May 12, 1987. The 1987 CBA wage
increases almost doubled that of the P10.00 increase under R.A. No. 6640.
The P625.00/month means P24.03 increase per day for the supervisors, while
the P475.00/month means P18.26 increase per day for the foremen. These increases were to be observed every
year, starting May 12, 1987 until July 26, 1989.
A CBA constitutes the law between the parties when freely and voluntarily entered into. Here, it has not been
shown that respondent PIMASUFA was coerced or forced by petitioner to sign the 1987 CBA. All of its thirteen
(13) officers signed the CBA with the assistance of respondent NLU. They signed it fully aware of the passage of
R.A. No. 6640.

______________________________________________________________________

2. BANKARD EMPLOYEES UNION-WORKERS ALLIANCE TRADE UNIONS v. NLRC, BANKARD, INC.


G.R. No. 140689. February 17, 2004

CARPIO MORALES, J.

FACTS:
Angeles| Bajana | Balladares | Brillantes | Briones | Cabansag | Callanta | Chua | David|

De Leon | Gomez | Lopez | Macalino | Nostratis | Padilla | Reynon | Santos | Tan |Velasco
4E / 4F - 2018-2019
Page 166 of 920
LABOR REVIEW DIGEST
Atty. Joyrich Golangco

- Bankard, Inc. classifies its employees by levels, to wit: Level I, Level II, Level III, Level IV, and Level V.
- The New Salary Scale increased the hiring rates of new employees, to wit: Levels I and V by P1,000.00,
and Levels II, III and IV by P900.00.

- Bankards move drew the Bankard Employees Union-WATU (petitioner), the duly certified exclusive
bargaining agent of the regular rank and file employees of Bankard, to press for the increase in the salary
of its old, regular employees.
- Bankard took the position, however, that there was no obligation on the part of the management to grant
to all its employees the same increase in an across-the-board manner.

- As the continued request of petitioner for increase in the wages and salaries of Bankards regular
employees remained unheeded, it filed a Notice of Strike on August 26, 1993 on the ground of
discrimination and other acts of Unfair Labor Practice (ULP).

- A director of the National Conciliation and Mediation Board treated the Notice of Strike as a Preventive
Mediation Case based on a finding that the issues therein were not strikeable.

- Petitioner filed another Notice of Strike on October 8, 1993 on the grounds of refusal to bargain,
discrimination, and other acts of ULP - union busting.
- The strike was averted, however, when the dispute was certified by the Secretary of Labor and
Employment for compulsory arbitration.

- Second Division of the NLRC, finding no wage distortion, dismissed the case for lack of merit.

- Petitioner thereupon filed a petition for certiorari before this Court, docketed as G.R. 121970.

- In accordance with its ruling in St. Martin Funeral Homes v. NLRC, the petition was referred to the Court
of Appeals which, by October 28, 1999, denied the same for lack of merit.

ISSUE:
- whether the unilateral adoption by an employer of an upgraded salary scale that increased the hiring
rates of new employees without increasing the salary rates of old employees resulted in wage distortion
within the contemplation of Article 124 of the Labor Code.

HELD:
 Upon the enactment of R.A. No. 6727 WAGE RATIONALIZATION ACT, amending, among others,
Article 124 of the Labor Code on June 9, 1989, the term wage distortion was explicitly defined as:
o ... a situation where an increase in prescribed wage rates results in the elimination or
severe contraction of intentional quantitative differences in wage or salary rates between
and among employee groups in an establishment as to effectively obliterate the
distinctions embodied in such wage structure based on skills, length of service, or other
logical bases of differentiation.

 Prubankers Association v. Prudential Bank and Trust Company laid down the four elements of wage
distortion, to wit:

o (1.) An existing hierarchy of positions with corresponding salary rates;


Angeles| Bajana | Balladares | Brillantes | Briones | Cabansag | Callanta | Chua | David|

De Leon | Gomez | Lopez | Macalino | Nostratis | Padilla | Reynon | Santos | Tan |Velasco
4E / 4F - 2018-2019
Page 167 of 920
LABOR REVIEW DIGEST
Atty. Joyrich Golangco

o (2) A significant change in the salary rate of a lower pay class without a concomitant increase in
the salary rate of a higher one;
o (3) The elimination of the distinction between the two levels; and
o (4) The existence of the distortion in the same region of the country.

 Normally, a company has a wage structure or method of determining the wages of its employees. In a
problem dealing with wage distortion, the basic assumption is that there exists a grouping or
classification of employees that establishes distinctions among them on some relevant or
legitimate bases.

 Involved in the classification of employees are various factors such as the degrees of responsibility, the
skills and knowledge required, the complexity of the job, or other logical basis of differentiation. The
differing wage rate for each of the existing classes of employees reflects this classification.

 Petitioner maintains that for purposes of wage distortion, the classification is not one based on levels or
ranks but on two groups of employees, the newly hired and the old, in each and every level, and not
between and among the different levels or ranks in the salary structure.

 Public respondent National Labor Relations Commission (NLRC) refutes petitioners position

o To determine the existence of wage distortion, the historical classification of the employees
prior to the wage increase must be established. Likewise, it must be shown that as between
the different classification of employees, there exists a historical gap or difference.
o The classification preferred by petitioner is belied by the wage structure of private
respondent as shown in the new salary scale it adopted on May 28, 1993, retroactive to
April 1, 1993, which provides
o employees of private respondent have been historically classified into levels, i.e. I to V, and
not on the basis of their length of service. Put differently, the entry of new employees to the
company ipso facto place[s] them under any of the levels mentioned in the new salary scale
which private respondent adopted retroactive [to] April 1, 1993. Petitioner cannot make a
contrary classification of private respondents employees without encroaching upon recognized
management prerogative of formulating a wage structure, in this case, one based on level.

 The issue of whether wage distortion exists being a question of fact that is within the jurisdiction of
quasi-judicial tribunals, and it being a basic rule that findings of facts of quasi-judicial agencies, like
the NLRC, are generally accorded not only respect but at times even finality if they are supported
by substantial evidence, as are the findings in the case at bar, they must be respected. For these
agencies have acquired expertise, their jurisdiction being confined to specific matters.

 As National Federation of Labor v. NLRC, et al. teaches, the formulation of a wage structure through
the classification of employees is a matter of management judgment and discretion.

o [W]hether or not a new additional scheme of classification of employees for compensation


purposes should be established by the Company (and the legitimacy or viability of the bases of
distinction there embodied) is properly a matter of management judgment and discretion,

Angeles| Bajana | Balladares | Brillantes | Briones | Cabansag | Callanta | Chua | David|

De Leon | Gomez | Lopez | Macalino | Nostratis | Padilla | Reynon | Santos | Tan |Velasco
4E / 4F - 2018-2019
Page 168 of 920
LABOR REVIEW DIGEST
Atty. Joyrich Golangco

and ultimately, perhaps, a subject matter for bargaining negotiations between employer
and employees. It is assuredly something that falls outside the concept of wage distortion.

 Apart from the findings of fact of the NLRC and the Court of Appeals that some of the elements of wage
distortion are absent, petitioner cannot legally obligate Bankard to correct the alleged wage distortion as
the increase in the wages and salaries of the newly-hired was not due to a prescribed law or wage order.

 The wordings of Article 124 are clear. If it was the intention of the legislators to cover all kinds of wage
adjustments, then the language of the law should have been broad, not restrictive as it is currently
phrased:

o Article 124. Standards/Criteria for Minimum Wage Fixing.


 xxx
Where the application of any prescribed wage increase by virtue of a law or Wage
Order issued by any Regional Board results in distortions of the wage structure
within an establishment, the employer and the union shall negotiate to correct the
distortions. Any dispute arising from the wage distortions shall be resolved through the
grievance procedure under their collective bargaining agreement and, if it remains
unresolved, through voluntary arbitration.

 Article 124 is entitled Standards/Criteria for Minimum Wage Fixing.


 It is found in CHAPTER V on WAGE STUDIES, WAGE AGREEMENTS AND WAGE DETERMINATION
which principally deals with the fixing of minimum wage. Article 124 should thus be construed and
correlated in relation to minimum wage fixing, the intention of the law being that in the event of an
increase in minimum wage, the distinctions embodied in the wage structure based on skills, length of
service, or other logical bases of differentiation will be preserved.

 If the compulsory mandate under Article 124 to correct wage distortion is applied to voluntary and
unilateral increases by the employer in fixing hiring rates which is inherently a business judgment
prerogative, then the hands of the employer would be completely tied even in cases where an
increase in wages of a particular group is justified due to a re-evaluation of the high productivity
of a particular group, or as in the present case, the need to increase the competitiveness of
Bankards hiring rate.

 An employer would be discouraged from adjusting the salary rates of a particular group of employees for
fear that it would result to a demand by all employees for a similar increase, especially if the financial
conditions of the business cannot address an across-the-board increase.

 Bankards right to increase its hiring rate, to establish minimum salaries for specific jobs, and to adjust the
rates of employees affected thereby is embodied under Section 2, Article V (Salary and Cost of Living
Allowance) of the parties Collective Bargaining Agreement (CBA), to wit:

o Section 2. Any salary increase granted under this Article shall be without prejudice to the right of
the Company to establish such minimum salaries as it may hereafter find appropriate for
specific jobs, and to adjust the rates of the employees thereby affected to such minimum
salaries thus established.[15] (Italics and underscoring supplied)

Angeles| Bajana | Balladares | Brillantes | Briones | Cabansag | Callanta | Chua | David|

De Leon | Gomez | Lopez | Macalino | Nostratis | Padilla | Reynon | Santos | Tan |Velasco
4E / 4F - 2018-2019
Page 169 of 920
LABOR REVIEW DIGEST
Atty. Joyrich Golangco

 This CBA provision, which is based on legitimate business-judgment prerogatives of the employer, is a
valid and legally enforceable source of rights between the parties.

 In fine, absent any indication that the voluntary increase of salary rates by an employer was done
arbitrarily and illegally for the purpose of circumventing the laws or was devoid of any legitimate purpose
other than to discriminate against the regular employees, this Court will not step in to interfere with this
management prerogative. Employees are of course not precluded from negotiating with its employer and
lobby for wage increases through appropriate channels, such as through a CBA.

 PETITION DENIED.

______________________________________________________________________

13th Month Pay

1. Central Azucarera de Tarlac v. Central Azucarera de Tarlac Labor Union-NLU


G.R. No. 188949; Jul. 26, 2010
Nachura,J.:

DOCTRINE: Salary- related benefits should be included as part of the basic salary in the computation of the 13th-
month pay if, by individual or collective agreement, company practice or policy, the same are treated as part of the
basic salary of the employees. Article 100 of the Labor Code, otherwise known as the Non-Diminution Rule,
mandates that benefits given to employees cannot be taken back or reduced unilaterally by the employer because
the bene�t has become part of the employment contract, written or unwritten.

FACTS:

Petitioner is a domestic corporation engaged in the business of sugar manufacturing, while respondent is a
legitimate labor organization which serves as the exclusive bargaining representative of petitioner's rank-and-file
employees. The controversy stems from the interpretation of the term "basic pay," essential in the computation of
the 13th-month pay. In compliance with Presidential Decree (P.D.) No. 851, petitioner granted its employees the
mandatory thirteenth (13th)-month pay since 1975. The formula used by petitioner in computing the 13th-month
pay was: Total Basic Annual Salary divided by twelve (12). Included in petitioner's computation of the Total Basic
Annual Salary were the following: basic monthly salary; first eight (8) hours overtime pay on Sunday and
legal/special holiday; night premium pay; and vacation and sick leaves for each year. Throughout the years,
petitioner used this computation until 2006.
Angeles| Bajana | Balladares | Brillantes | Briones | Cabansag | Callanta | Chua | David|

De Leon | Gomez | Lopez | Macalino | Nostratis | Padilla | Reynon | Santos | Tan |Velasco
4E / 4F - 2018-2019
Page 170 of 920
LABOR REVIEW DIGEST
Atty. Joyrich Golangco

On November 6, 2004, respondent staged a strike. During the pendency of the strike, petitioner declared a
temporary cessation of operations. In December 2005, all the striking union members were allowed to return to
work. Subsequently, petitioner declared another temporary cessation of operations for the months of April and
May 2006. The suspension of operation was lifted on June 2006, but the rank-and-file employees were allowed to
report for work on a fifteen (15) day-per-month rotation basis that lasted until September 2006. In December 2006,
petitioner gave the employees their 13th-month pay based on the employee's total earnings during the year
divided by 12.

Respondent objected to this computation. It averred that petitioner did not adhere to the usual computation of the
13th-month pay. It claimed that the divisor should have been eight (8) instead of 12, because the employees
worked for only 8 months in 2006. It likewise asserted that petitioner did not observe the company practice of
giving its employees the guaranteed amount equivalent to their one month pay, in instances where the computed
13th-month pay was less than their basic monthly pay.

The parties attempted to reconcile thru the grievance procedure set forth in their CBA, but the same proved
fruitless. Petitioner argued that the change in computation is a move to rectify an error in the previous formula,
and that the computation in the first place should only include the basic pay and not the other salary-related
benefits. Preventive Mediation was also resorted to, but the same proved equally as fruitless, resulting in the filing
of a money claim before the regional arbitration branch NLRC on the ground of diminution of benefits.

LA Decision: Dismissed based on the right of the employer to rectify errors of computation.

NLRC Decision: Overturned the Labor Arbiter, ruling that the inclusion of the salary-related benefits in the
computation of the 13th month pay had already ripened into company practice. MR Denied.

CA Decision: Affirmed the NLRC.

Appeal to the SC:

Petitioner’s Contention: It should be allowed to correct an error of computation.

Respondent’s Contention: The inclusion of salary-related benefits has already become company policy.

ISSUE: Whether the salary-related benefits should be included.

RULING: YES. Under the Revised Guidelines for the implementation of the 13th Month Pay Law, it was
specifically stated that the minimum 13th-month pay required by law shall not be less than one-twelfth (1/12) of
the total basic salary earned by an employee within a calendar year.

Furthermore, the term "basic salary" of an employee for the purpose of computing the 13th-month pay was
interpreted to include all remuneration or earnings paid by the employer for services rendered, but does not
include allowances and monetary benefits which are not integrated as part of the regular or basic salary, such as
the cash equivalent of unused vacation and sick leave credits, overtime, premium, night differential and holiday
pay, and cost-of-living allowances. However, these salary- related benefits should be included as part of the
basic salary in the computation of the 13th-month pay if, by individual or collective agreement, company
practice or policy, the same are treated as part of the basic salary of the employees.

Angeles| Bajana | Balladares | Brillantes | Briones | Cabansag | Callanta | Chua | David|

De Leon | Gomez | Lopez | Macalino | Nostratis | Padilla | Reynon | Santos | Tan |Velasco
4E / 4F - 2018-2019
Page 171 of 920
LABOR REVIEW DIGEST
Atty. Joyrich Golangco

Article 100 of the Labor Code, otherwise known as the Non-Diminution Rule, mandates that bene�ts given to
employees cannot be taken back or reduced unilaterally by the employer because the benefit has become part of
the employment contract, written or unwritten. The rule against diminution of benefits applies if it is shown that the
grant of the benefit is based on an express policy or has ripened into a practice over a long period of time and that
the practice is consistent and deliberate.

Clearly petitioner has been in the practice of the same for the past 29, almost 30 years. It cannot now allege that it
was in error in the computation of the 13th month pay, specially only after the souring of relations between
employer and employee.

______________________________________________________________________

Article 128
1. Peoples Broadcasting Services (BOMBO RADYO PHILS. INC.) vs Secretary of DOLE
GR No: 179652 Date: March 6, 2012
Ponente:

Doctrine:
To recapitulate, if a complaint is brought before the DOLE to give effect to the labor standards provisions of the
Labor Code or other labor legislation, and there is a finding by the DOLE that there is an existing employer-
employee relationship, the DOLE exercises jurisdiction to the exclusion of the NLRC. If the DOLE finds that there
is no employer-employee relationship, the jurisdiction is properly with the NLRC. If a complaint is filed with the
DOLE, and it is accompanied by a claim for reinstatement, the jurisdiction is properly with the Labor Arbiter, under
Art. 217(3) of the Labor Code, which provides that the Labor Arbiter has original and exclusive jurisdiction over
those cases involving wages, rates of pay, hours of work, and other terms and conditions of employment, if
accompanied by a claim for reinstatement. If a complaint is filed with the NLRC, and there is still an existing
employer-employee relationship, the jurisdiction is properly with the DOLE.

FACTS:
Private respondent Jandeleon Juezan filed a complaint against petitioner with the Department of Labor
and Employment (DOLE) Regional Office No. VII, Cebu City, for illegal deduction, nonpayment of service
incentive leave, 13th month pay, premium pay for holiday and rest day and illegal diminution of benefits, delayed
payment of wages and noncoverage of SSS, PAG-IBIG and Philhealth. After the conduct of summary
investigations, and after the parties submitted their position papers, the DOLE Regional Director found that private
respondent was an employee of petitioner, and was entitled to his money claims. Petitioner sought
reconsideration of the Directors Order, but failed. The Acting DOLE Secretary dismissed petitioners appeal on the
ground that petitioner submitted a Deed of Assignment of Bank Deposit instead of posting a cash or surety bond.

CA RULING:

Angeles| Bajana | Balladares | Brillantes | Briones | Cabansag | Callanta | Chua | David|

De Leon | Gomez | Lopez | Macalino | Nostratis | Padilla | Reynon | Santos | Tan |Velasco
4E / 4F - 2018-2019
Page 172 of 920
LABOR REVIEW DIGEST
Atty. Joyrich Golangco

When the matter was brought before the CA, where petitioner claimed that it had been denied due
process, it was held that petitioner was accorded due process as it had been given the opportunity to be heard,
and that the DOLE Secretary had jurisdiction over the matter, as the jurisdictional limitation imposed by Article 129
of the Labor Code on the power of the DOLE Secretary under Art. 128(b) of the Code had been repealed by
Republic Act No. (RA) 7730.

APPEAL TO THE SC (Initial Decision of the SC):

The Court found that there was no employer-employee relationship between petitioner and private
respondent. It was held that while the DOLE may make a determination of the existence of an employer-employee
relationship, this function could not be co-extensive with the visitorial and enforcement power provided in Art.
128(b) of the Labor Code, as amended by RA 7730. The National Labor Relations Commission (NLRC) was held
to be the primary agency in determining the existence of an employer-employee relationship. This was the
interpretation of the Court of the clause in cases where the relationship of employer-employee still exists in Art.
128(b).

From this Decision, the Public Attorneys Office (PAO) filed a Motion for Clarification of Decision (with
Leave of Court). The PAO sought to clarify as to when the visitorial and enforcement power of the DOLE be not
considered as co-extensive with the power to determine the existence of an employer-employee relationship. In its
Comment, the DOLE sought clarification as well, as to the extent of its visitorial and enforcement power under the
Labor Code, as amended.

ISSUE/S: May the DOLE make a determination of whether or not an employer-employee relationship
exists, and if so, to what extent

HELD:
Yes. No limitation in the law was placed upon the power of the DOLE to determine the existence of an
employer-employee relationship. No procedure was laid down where the DOLE would only make a preliminary
finding, that the power was primarily held by the NLRC. The law did not say that the DOLE would first seek the
NLRCs determination of the existence of an employer-employee relationship, or that should the existence of the
employer-employee relationship be disputed, the DOLE would refer the matter to the NLRC. The DOLE must
have the power to determine whether or not an employer-employee relationship exists, and from there to decide
whether or not to issue compliance orders in accordance with Art. 128(b) of the Labor Code, as amended by RA
7730.

The DOLE, in determining the existence of an employer-employee relationship, has a ready set of
guidelines to follow, the same guide the courts themselves use. The elements to determine the existence of an
employment relationship are: (1) the selection and engagement of the employee; (2) the payment of wages; (3)
the power of dismissal; (4) the employers power to control the employees conduct. The use of this test is not
solely limited to the NLRC. The DOLE Secretary, or his or her representatives, can utilize the same test, even in
the course of inspection, making use of the same evidence that would have been presented before the NLRC.

This is not to say that the determination by the DOLE is beyond question or review. Suffice it to say,
there are judicial remedies such as a petition for certiorari under Rule 65 that may be availed of, should a party
wish to dispute the findings of the DOLE.

It must also be remembered that the power of the DOLE to determine the existence of an employer-
employee relationship need not necessarily result in an affirmative finding. The DOLE may well make the
determination that no employer-employee relationship exists, thus divesting itself of jurisdiction over the case. It

Angeles| Bajana | Balladares | Brillantes | Briones | Cabansag | Callanta | Chua | David|

De Leon | Gomez | Lopez | Macalino | Nostratis | Padilla | Reynon | Santos | Tan |Velasco
4E / 4F - 2018-2019
Page 173 of 920
LABOR REVIEW DIGEST
Atty. Joyrich Golangco

must not be precluded from being able to reach its own conclusions, not by the parties, and certainly not by this
Court.

To recapitulate, if a complaint is brought before the DOLE to give effect to the labor standards provisions
of the Labor Code or other labor legislation, and there is a finding by the DOLE that there is an existing employer-
employee relationship, the DOLE exercises jurisdiction to the exclusion of the NLRC. If the DOLE finds that there
is no employer-employee relationship, the jurisdiction is properly with the NLRC. If a complaint is filed with the
DOLE, and it is accompanied by a claim for reinstatement, the jurisdiction is properly with the Labor Arbiter, under
Art. 217(3) of the Labor Code, which provides that the Labor Arbiter has original and exclusive jurisdiction over
those cases involving wages, rates of pay, hours of work, and other terms and conditions of employment, if
accompanied by a claim for reinstatement. If a complaint is filed with the NLRC, and there is still an existing
employer-employee relationship, the jurisdiction is properly with the DOLE. The findings of the DOLE, however,
may still be questioned through a petition for certiorari under Rule 65 of the Rules of Court.

In the present case, the finding of the DOLE Regional Director that there was an employer-employee
relationship has been subjected to review by this Court, with the finding being that there was no employer-
employee relationship between petitioner and private respondent, based on the evidence presented. Private
respondent presented self-serving allegations as well as self-defeating evidence. The findings of the Regional
Director were not based on substantial evidence, and private respondent failed to prove the existence of an
employer-employee relationship. The DOLE had no jurisdiction over the case, as there was no employer-
employee relationship present. Thus, the dismissal of the complaint against petitioner is proper.

______________________________________________________________________

2. Yanson v Secretary of Labor


GR No:159026 Date: February 11, 2008
Ponente: AUSTRIA-MARTINEZ, J

Doctrine: (main doctrine under the article)


The posting of the proper amount of the appeal bond under Article 128 (b) is mandatory for the perfection of an
appeal from a monetary award in labor standard cases.

FACTS:

Mardy Cabigo and 40 other workers (private respondents) filed with the Department of Labor and Employment-
Bacolod District Office (DOLE Bacolod) a request for payroll inspection of Hacienda Valentin Balabag owned by
Alberta Yanson (petitioner). DOLE Bacolod conducted an inspection of petitioner's establishment on May 27,
1998, and issued a Notice of Inspection Report, finding
petitioner liable for violations of labor standard laws.

LA/RTC/NLRC RULING:
DOLE Bacolod scheduled a summary investigation and issued, by registered mail, notices of hearing as well as a
subpoena duces tecum to the parties. Petitioner did not appear in any of the scheduled hearings or present any
pleading or document. A compliance order directing petitioner to pay, within five (5) days, P9,084.00 to each of
the 41 respondents or a total of P372,444.00 remained not satisfied so a writ of execution was issued.

Petitioner filed with DOLE Bacolod a Double Verified Special Appearance to Oppose "Writ of Execution" For Being
a Blatant and Dangerous Violation of Due Process, claiming that she did not receive any form of communication,
or participate in any proceeding relative to the subject matter of the writ of execution which DOLE Bacolod denied.
Angeles| Bajana | Balladares | Brillantes | Briones | Cabansag | Callanta | Chua | David|

De Leon | Gomez | Lopez | Macalino | Nostratis | Padilla | Reynon | Santos | Tan |Velasco
4E / 4F - 2018-2019
Page 174 of 920
LABOR REVIEW DIGEST
Atty. Joyrich Golangco

Petitioner filed with public respondent (Secretary of Labor) a Verified Appeal and Supplement to the Verified
Appeal, posting therewith an appeal bond of P1,000.00 in money order and attaching thereto a Motion to be
Allowed to Post Minimal Bond with Motion for Reduction of Bond.

CA RULING:
Petitioner filed a Petition for Certiorari which was denied due course and dismissed by the CA.

APPEAL TO THE SC:

Petitioner's Contention:
Petitioner contends that the CA and public respondent denied her the right to appeal when they rejected her
P1,000.00-appeal bond. She insists that her appeal bond cannot be based on the monetary award of P372,444.00
granted by DOLE Bacolod in its August 14, 1998 Order which, having been rendered without prior notice to her,
was a patent nullity and completely without effect. She argues that her appeal bond should instead be based on
her capacity to pay; otherwise, her right to free access to the courts as guaranteed under Article III, Section 2 of
the Constitution would be set to naught merely because of her diminished financial capacity.

ISSUE/S:
1. Whether or not there is a perfected appeal
2. Whether or not the appeal bond can be reduced

HELD:
1. No, Article 128 (b) of the Labor Code clearly provides that the appeal bond must be "in the amount equivalent
to the monetary award in the order appealed from." In this case, the petitioner only posted P1,000 appeal bond in
contrast with the monetary award of P372,444.00. The posting of the proper amount of the appeal bond under
Article 128 (b) is mandatory for the perfection of an appeal from a monetary award in labor standard cases.

2. No, Article 128 (b) deliberately employed the word "only" in reference to the requirements for perfection of an
appeal in labor standards cases. "Only" commands a restrictive application, giving no room for modification of said
requirements. The reduction of bond in the NLRC is expressly authorized under the Rules implementing Article
223 and no similar authority is given the DOLE Secretary in Department Order No. 18-02 (Implementing Rules),
Series of 2002, amending Department Order No. 7-A, Series of 1995, implementing Article 128 (b). Thus, under
the foregoing Implementing Rules, it is plain that public respondent has no authority to accept an appeal under a
reduced bond.

Other Notes/ SC Pronouncements:


______________________________________________________________________

Angeles| Bajana | Balladares | Brillantes | Briones | Cabansag | Callanta | Chua | David|

De Leon | Gomez | Lopez | Macalino | Nostratis | Padilla | Reynon | Santos | Tan |Velasco
4E / 4F - 2018-2019
Page 175 of 920
LABOR REVIEW DIGEST
Atty. Joyrich Golangco

3. NESTOR J. BALLADARES, et al. vs. PEAK VENTURES CORPORATION/ EL TIGRE SECURITY AND
INVESTIGATION AGENCY and YANGCO MARKET OWNERS ASSOCIATION/LAO TI SIOK BEE (YMOAA)
GR No: 161794 Date: June 16, 2009
Ponente: NACHURA, J.:

DOCTRINE:
R.A. No. 7730 in amending Article 128 (b) of the Labor Code abandoned the Servando ruling. The DOLE is now
empowered to hear and decide, in a summary proceeding, any matter involving the recovery of any amount of
wages and other monetary claims arising out of employer-employee relations at the time of the inspection, even if
the amount of the money claim exceeds P5,000, and provided that the case does not fall under the exception
clause.

FACTS:
Petitioners Balladares, et al. were 11 security guards employed by respondent Peak Ventures and assigned to
the premises of respondent YMOAA. They filed a complaint for underpayment of wages and other benefits
with the DOLE. In order to verify the allegations in the complaint, DOLE conducted an inspection using the
visitorial and enforcement powers of the Secretary of Labor and Employment, which yielded proof of
violations of labor standards.

DOLE REGIONAL DIRECTOR RULING: in favor of petitioners and ruled that the contractor was jointly and
severally liable with the principal, pursuant to the law and jurisprudence on the matter. Peak Ventures Corp.
and/or YMOAA are jointly ordered to pay the TOTAL AMOUNT of P1,106,298.07 to all the petitioners.

DOLE SECRETARY RULING: Regional Director committed serious errors in awarding the amount as it was
quite excessive.

CA RULING: in favor of respondent and ruled that Secretary of DOLE acted without, or in excess of, jurisdiction
or with grave abuse of discretion because:
(a) DOLE Regional Director had no jurisdiction to hear and decide the case - because the claims of each of
the petitioners exceeded P5,000
(b) Servandos, Inc. v. Secretary of Labor – Under Article 129 of the Labor Code, the power to hear and
decide claims of employees arising from employer-employee relations exceeding P5,000 for each employee
belonged to the Labor Arbiter.

Petitioner's Contention:
CA erred in applying Article 129 instead of Article 128 of the Labor Code; and erred in applying the Servandos,
Inc. v. Secretary of Labor, which had long been abandoned.

Respondent’s Contention:
(1) CA did not err in applying Article 129 and Article 217 of the Labor Code providing for the jurisdiction of the
Labor Arbiter because the instant case arose from a complaint for recovery of wages, simple money claims and
other benefits, and the claims exceeded P5,000.
(2) The inspection conducted by the DOLE thru the visitorial and enforcement powers of the Secretary of Labor
and Employment did not convert the case to the one falling under Article 128.
Angeles| Bajana | Balladares | Brillantes | Briones | Cabansag | Callanta | Chua | David|

De Leon | Gomez | Lopez | Macalino | Nostratis | Padilla | Reynon | Santos | Tan |Velasco
4E / 4F - 2018-2019
Page 176 of 920
LABOR REVIEW DIGEST
Atty. Joyrich Golangco

ISSUE/S:
Whether the DOLE under Article 128 (b) of the Labor Code has jurisdiction over the petitioners’ complaint
regardless of the monetary value of the claims involved therein. YES.

HELD:
By the nature of the complaint and from the result of the inspection, the authority of the DOLE, under
Article 128, came into play regardless of the monetary value of the claims involved. The extent of this
authority and the powers flowing therefrom are defined and set forth in Article 128 of the Labor Code, as
amended by R.A. No. 7730.

Reliance on the Servando ruling is no longer tenable in view of the enactment of R.A. No. 7730, amending
Article 128 (b) of the Labor Code. The Secretary of Labor or his duly authorized representatives (DOLE
Regional director) is now empowered to hear and decide, in a summary proceeding, any matter involving
the recovery of any amount of wages and other monetary claims arising out of employer-employee
relations at the time of the inspection, even if the amount of the money claim exceeds P5,000.

However, if the labor standards case is covered by the exception clause in Article 128 (b) of the Labor Code,
then the Regional Director will have to endorse the case to the appropriate Arbitration Branch of the NLRC.

“ART. 128 (b) - Notwithstanding the provisions of Articles 129 and 217 of this Code to the contrary, and in cases
where the relationship of employer-employee still exists, the Secretary of Labor and Employment or his duly
authorized representatives shall have the power to issue compliance orders to give effect to the labor standards
provisions of this Code and other labor legislation based on the findings of labor employment and enforcement
officers or industrial safety engineers made in the course of inspection. The Secretary or his duly authorized
representatives shall issue writs of execution to the appropriate authority for the enforcement of their orders,
except in cases where the employer contests the finding of the labor employment and enforcement officer
and raises issues supported by documentary proofs which were not considered in the course of
inspection.”

Elements to divest the Regional Director or his representatives of jurisdiction:


(1) that the employer contests the findings of the labor regulations officer and raises issues thereon;
(2) that in order to resolve such issues, there is a need to examine evidentiary matters; and
(3) that such matters are not verifiable in the normal course of inspection.
The rules also provide that the employer shall raise such objections during the hearing of the case or at any
time after receipt of the notice of inspection results.

APPLICATION - In this case, the Regional Director validly assumed jurisdiction over the money claims even if the
claims exceeded P5,000 because such jurisdiction was exercised in accordance with Article 128(b) of the Labor
Code and the case does not fall under the exception clause. Also respondent did not contest the findings of the
labor regulations officer. During the hearing, it never denied that petitioners were not paid correct wages and
benefits, and such fact was even admitted in its petition filed before the CA.
Other Notes/ SC Pronouncements:
__________________________________________________________________________

4. ALLIED INVESTIGATION BUREAU, INC., petitioner, vs. HON. SECRETARY OF LABOR & EMPLOYMENT,
acting through Undersecretary CRESENCIANO B. TRAJANO, respondents

Angeles| Bajana | Balladares | Brillantes | Briones | Cabansag | Callanta | Chua | David|

De Leon | Gomez | Lopez | Macalino | Nostratis | Padilla | Reynon | Santos | Tan |Velasco
4E / 4F - 2018-2019
Page 177 of 920
LABOR REVIEW DIGEST
Atty. Joyrich Golangco

G.R. No.: 122006 Date: November 24, 1999

Ponente: KAPUNAN, J.

Doctrine: Article 128 of the Labor Code explicitly excludes from its coverage Articles 129 and 217 of the Labor
Code by the phrase (N)otwithstanding the provisions of Articles 129 and 217 of this Code to the contrary x x x
thereby retaining and further strengthening the power of the Secretary of Labor or his duly authorized
representatives to issue compliance orders to give effect to the labor standards provisions of said Code and other
labor legislation based on the findings of labor employment and enforcement officers or industrial safety engineers
made in the course of inspection.
Clearly, as the duly authorized representative of respondent Secretary of Labor, and in the lawful exercise of the
Secretarys visitorial and enforcement powers under Article 128 of the Labor Code, respondent Regional Director
had jurisdiction to issue his impugned Order.

FACTS:
 Petitioner Allied Investigation Bureau, Inc. is a security agency. On January 11, 1994, it entered into
a security contract with Novelty Philippines, Inc. (NPI, for brevity) whereby it obligated itself to provide
security services to the latter.
 On January 17, 1995, private respondents Melvin T. Pelayo and Samuel Sucanel, two of the
security guards assigned by petitioner to NPI, filed a complaint with the Office of respondent Regional
Director Romeo A. Young charging petitioner with non-compliance with Wage Order No. NCR-03,[2] which
increased the minimum daily pay of workers by P17.00, or from P118.00 to P135.00 effective December
16, 1993; and further, by P10.00, or from P135.00 to P145.00 daily beginning April 1, 1994. Private
respondents, likewise, sought the recovery of wage differentials

 The Office of Regional Director Young conducted inspection visits at petitioner’s establishment.

FINDINGS AS A RESULT OF INSPECTION CONDUCTED:


- Non-implementation under W.O. # NCR-03 from Dec. 16, 1993 to Dec. 15, 1994 to security guards assigned at
Novelty Phils., Inc. However, their prime client has been granted an exemption by the Wage Board under W.O. #
NCR-03 with Case No. NCRO-W.O. # 3-E (9) dated June 7, 1994.Please see attached xerox copy.

- Non-remittance of SSS Premiums


- Excessive deduction or Bayanihan System (P20.00) every pay day instead of P5.00 only.
INSTRUCTIONS TO EMPLOYER:
You are required to effect restitution and/or correction of the foregoing at the company or plant level within five (5)
calendar days hereof.
Any question on the above findings should be submitted to this office within five (5) working days from notice
hereof, otherwise order of compliance shall be issued.
 Said report was explained to and received by petitioners Human Resources Director, Eufracio G.
Quiambao III on February 14, 1995.
 Thereafter, in order to facilitate amicable settlement between the parties, a series of conferences and
hearings were scheduled by the Office of the Regional Director.
 However, despite due notice, petitioner failed to appear in any of said hearings.
 On May 9, 1995, respondent Regional Director issued an Order, the dispositive portion of which
reads:
WHEREFORE, premises considered and considering further that the above computed wage differentials form part
of the legal remunerations of the complainants, respondent ALLIED INVESTIGATION BUREAU, INC., is hereby
Angeles| Bajana | Balladares | Brillantes | Briones | Cabansag | Callanta | Chua | David|

De Leon | Gomez | Lopez | Macalino | Nostratis | Padilla | Reynon | Santos | Tan |Velasco
4E / 4F - 2018-2019
Page 178 of 920
LABOR REVIEW DIGEST
Atty. Joyrich Golangco

ordered to pay to the ninety-two employees the total amount of EIGHT HUNDRED SEVEN THOUSAND FIVE
HUNDRED SEVENTY PESOS AND THIRTY-SIX CENTAVOS (P807.570.36) to be distributed to the individual
employees in accordance with the schedule mentioned above, within ten (10) days from receipt
hereof. Otherwise, Writ of execution shall issue to enforce this Order.

 Petitioner appealed the above Order to respondent and Employment, without however, posting a cash or
surety bond equivalent to the monetary award in the said Order appealed from.

SECRETARY OF LABOR RULING:


On September 19, 1995, the Secretary of Labor, thru Undersecretary Cresenciano B. Trajano issued an
Order dismissing petitioners appeal for failure to perfect said appeal.
Hence, the instant petition for certiorari with prayer for the issuance of a temporary restraining order/writ of
preliminary injunction

Petitioner's Contention:
Petitioner argues that the power to adjudicate money claims belongs to the Labor Arbiter who has exclusive
jurisdiction over employees claims where the aggregate amount of the claims of each employee
exceeds P5,000.00.

Petitioners cites Articles 129 and 217 of the Labor Code of the Philippines which provide, respectively, that the
power of the Regional Director to adjudicate employees money claims is subject to the condition that the
aggregate money claims of each employee does not exceed P5,000.00;[11] and, that the Labor Arbiter has
jurisdiction over all other claims arising from employer-employee relations, including those of persons in domestic
or household service, involving an amount exceeding five thousand pesos (P5,000.00), whether or not
accompanied with a claim for reinstatement.

Petitioner further contends that since the Order appealed from is void and without legal effect, said Order never
assumed finality and, therefore, it was improper for the respondent Secretary of Labor to outrightly dismiss the
appeal on the ground that petitioner failed to post a cash/surety bond.

ISSUE:
Whether or not respondent Secretary of Labor & Employment, acting through Undersecretary Cresenciano B.
Trajano, acted with grave abuse of discretion in dismissing herein petitioners appeal attacking the jurisdiction of
respondent Regional Director in adjudicating subject money claims of private respondents.

HELD:
 No. Petitioners arguments are untenable.
 While it is true that under Articles 129 and 217 of the Labor Code, the Labor Arbiter has jurisdiction to
hear and decide cases where the aggregate money claims of each employee exceeds P5,000.00, said
provisions of law do not contemplate nor cover the visitorial and enforcement powers of the Secretary of
Labor or his duly authorized representatives.
 Rather, said powers are defined and set forth in Article 128 of the Labor Code (as amended by R.A. No.
7730) thus:
Art. 128. Visitorial and enforcement power. -
(a) The Secretary of Labor or his duly authorized representatives, including labor regulation officers, shall have
access to employers records and premises at any time of the day or night whenever work is being undertaken
therein, and the right to copy therefrom, to question any employee and investigate any fact, condition or matter
which may be necessary to determine violations or which may aid in the enforcement of this Code and of any
labor law, wage order or rules and regulations issued pursuant thereto.
Angeles| Bajana | Balladares | Brillantes | Briones | Cabansag | Callanta | Chua | David|

De Leon | Gomez | Lopez | Macalino | Nostratis | Padilla | Reynon | Santos | Tan |Velasco
4E / 4F - 2018-2019
Page 179 of 920
LABOR REVIEW DIGEST
Atty. Joyrich Golangco

(b) Notwithstanding the provisions of Articles 129 and 217 of this Code to the contrary, and in cases where the
relationship of employer-employee exists, the Secretary of Labor and Employment or his duly authorized
representatives shall have the power to issue compliance orders to give effect to the labor standards provisions of
this Code and other labor legislation based on the findings of labor employment and enforcement officers
or industrial safety engineers made in the course of inspection. The Secretary or his duly authorized
representatives shall issue writs of execution to the appropriate authority for the enforcement of their orders,
except in cases where the employer contests the findings of the labor employment and enforcement officer and
raises issues supported by documentary proofs which were not considered in the course of inspection.

An order issued by the duly authorized representatives of the Secretary of Labor and Employment under this
article may be appealed to the latter. In case said order involves a monetary award, an appeal by the employer
may be perfected only upon the posting of a cash or surety bond issued by a reputable bonding company duly
accredited by the Secretary of Labor and Employment in the amount equivalent to the monetary award in the
order appealed from. (Underscoring supplied)
xxx
 The aforequoted provision explicitly excludes from its coverage Articles 129 and 217 of the Labor Code
by the phrase (N)otwithstanding the provisions of Articles 129 and 217 of this Code to the contrary x x x
thereby retaining and further strengthening the power of the Secretary of Labor or his duly authorized
representatives to issue compliance orders to give effect to the labor standards provisions of said Code
and other labor legislation based on the findings of labor employment and enforcement officers or
industrial safety engineers made in the course of inspection.
 In the case at bar, the Office of respondent Regional Director conducted inspection visits at petitioner’s
establishment on February 9 and 14, 1995 in accordance with the above-mentioned provision of law. In
the course of said inspection, several violations of the labor standard provisions of the Labor Code were
discovered and reported by Senior Labor Enforcement Officer Eduvigis A. Acero in his Notice of
Inspection Results. It was on the bases of the aforesaid findings (which petitioner did not contest), that
respondent Regional Director issued the assailed Order for petitioner to pay private respondents the
respective wage differentials due them.

 Clearly, as the duly authorized representative of respondent Secretary of Labor, and in the lawful
exercise of the Secretarys visitorial and enforcement powers under Article 128 of the Labor Code,
respondent Regional Director had jurisdiction to issue his impugned Order.

Other Notes/ SC Pronouncements:


______________________________________________________________________

Angeles| Bajana | Balladares | Brillantes | Briones | Cabansag | Callanta | Chua | David|

De Leon | Gomez | Lopez | Macalino | Nostratis | Padilla | Reynon | Santos | Tan |Velasco
4E / 4F - 2018-2019
Page 180 of 920
LABOR REVIEW DIGEST
Atty. Joyrich Golangco

5. Urbanes, Jr. doing business under Catalina Security Agency vs. DOLE Secretary and SSS
GR No. 122791 Date: February 19, 2003
Ponente: Carpio-Morales, J.

Doctrine: It is well settled in law and jurisprudence that where no employer-employee relationship exists between
the parties and no issue is involved which may be resolved by reference to the Labor Code, other labor statutes or
any collective bargaining agreement, it is the Regional Trial Court that has jurisdiction.

FACTS:

Urbanes, doing business under Catalina Security Agency (CSA), and SSS entered into an agreeement where
CSA will provide security services to SSS. During the effectivity of the agreement, Wage Order No. NCR-03 Was
was issued which provided for the increase of the salary of security service providers, among others. Urbanes
sent a letter to SSS requesting the adjustment of their contract rate by virtue of said wage order. SSS remained
unheeded thus, Urbanes pulled out his agency's services to SSS.

Urbanes then filed a complaint with the DOLE-NCR against SSS seeking implementation of WO No. NCR-03.
SSS moved for the dismissal claiming that Urbanes is not the real-party-in-interest. It argued that if any, its
obligation will be to the security guards.

Urbanes argued that the security guards have no legal basis to file a complaint against SSS because of lack of
contractual privity.

DOLE-NCR RULING:
The Regional Director (RD) of the DOLE-NCR found for Urbanes and ordered SSS to pay the wage differentials.
SSS filed a MR for the computation to be revised which the RD granted.

SSS then filed an appeal with the DOLE Secretary claiming that:
1. The RD had no juridiction over the case at bar
2. The RD erred in finding that Urbanes id a real party in interest
3. The RD erred in adopting complainants' computation for wage adjustment.

DOLE SEC RULING:


Angeles| Bajana | Balladares | Brillantes | Briones | Cabansag | Callanta | Chua | David|

De Leon | Gomez | Lopez | Macalino | Nostratis | Padilla | Reynon | Santos | Tan |Velasco
4E / 4F - 2018-2019
Page 181 of 920
LABOR REVIEW DIGEST
Atty. Joyrich Golangco

The DOLE Secretary set aside the DOLE RD order and remanded the case for proper computation of wage
adjustment. The DOLE Secretary also held Petitioner CSA jointly and severally liable for wage differentials, the
amount of which should be paid directly to the security guards.

APPEAL TO THE SC:


Urbanes filed a MR which was denied, hence, this petition for certiorari. He claimed that the DOLE Sec committed
grave abuse of discretion when he acted on the appeal of SSS.

Petitioner's Contention:
Urbaned claimed that the DOLE Sec does not have jurisdiction to review appeals from the decisions of RDs in
complaints filed under Art. 129 of the LC. He claimed that the appeal should have been to the NLRC.

Respondent's Contention:
SSS, on the otherhand, claims that it is Art. 128 and not Art. 129 that is applicable and claimed that the order
issued by the RD may be appealed to the DOLE Secretary.

ISSUE:
1. Where should the appeal from the order of the DOLE Regional Director directing respondent to
comply with the wage order be filed?

HELD:
1. Both the contentions of the parties are wrong.

It is the RTC which has jurisdiction over the subject matter of the present case. It is well settled in law and
jurisprudence that where no employer-employee relationship exists between the parties and no issue is involved
which may be resolved by reference to the Labor Code, other labor statutes or any collective bargaining
agreement, it is the Regional Trial Court that has jurisdiction. In its complaint, private respondent is not seeking
any relief under the Labor Code but seeks payment of a sum of money. The action is within the realm of civil law
hence jurisdiction over the case belongs to the regular courts. While the resolution of the issue involves the
application of labor laws, reference to the labor code was only for the determination of the solidary liability of the
petitioner to the respondent where no employer-employee relation exists.

In the case at bar, even if petitioner filed the complaint on his and also on behalf of the security guards, the relief
sought has to do with the enforcement of the contract between him and the SSS which was deemed amended
by virtue of Wage Order No. NCR-03. The controversy subject of the case at bar is thus a civil dispute, the proper
forum for the resolution of which is the civil courts.

Other Notes/ SC Pronouncements:

But even assuming arguendo that petitioners complaint were filed with the proper forum, for lack of cause of
action it must be dismissed.

The liability of the SSS to reimburse petitioner arises only if and when petitioner pays his employee-security
guards the increases mandated by Wage Order No. NCR-03 which is not shown in the records.

Petition is hereby DISMISSED, and petitioners complaint before the RD is dismissed for lack of jurisdiction and
cause of action.

Other Notes/ SC Pronouncements:


Angeles| Bajana | Balladares | Brillantes | Briones | Cabansag | Callanta | Chua | David|

De Leon | Gomez | Lopez | Macalino | Nostratis | Padilla | Reynon | Santos | Tan |Velasco
4E / 4F - 2018-2019
Page 182 of 920
LABOR REVIEW DIGEST
Atty. Joyrich Golangco

______________________________________________________________________

Article 134

1. Zialcita v PAL
GR. No.: RO4-3-3398-76 Date: February 20, 1977
Ponente: Decided by the Office of the President

DOCTRINE:
Article 136 is not intended to apply only to women employed in ordinary occupations, or it should have
categorically expressed so. The sweeping intendment of the law, be it on special or ordinary occupations, is
reflected in the whole text and supported by Article 135 that speaks of non-discrimination on the employment of
women.

FACTS:
Zialcita is a stewardess of PAL. She was fired from work because she got married. PAL argued and cited its
policy that stewardesses must be single. The policy also states that subsequent marriage of a stewardess shall
automatically terminate employment.

Zialcita anchored on Article 136 of the Labor Code. PAL sought refuge from Article 132.

[RECALL:]

Article 132 provides, "Facilities for women. The Secretary of Labor and Employment shall establish
standards that will ensure the safety and health of women employees. In appropriate cases, he shall, by
regulations, require any employer to: To determine appropriate minimum age and other standards for retirement
or termination in special occupations such as those of flight attendants and the like."

Article 136 provides, "Stipulation against marriage. It shall be unlawful for an employer to require as a
condition of employment or continuation of employment that a woman employee shall not get married, or to
stipulate expressly or tacitly that upon getting married, a woman employee shall be deemed resigned or
separated, or to actually dismiss, discharge, discriminate or otherwise prejudice a woman employee merely by
reason of her marriage."

ISSUE:
Was Zialcita’s termination proper?

RULING:
Angeles| Bajana | Balladares | Brillantes | Briones | Cabansag | Callanta | Chua | David|

De Leon | Gomez | Lopez | Macalino | Nostratis | Padilla | Reynon | Santos | Tan |Velasco
4E / 4F - 2018-2019
Page 183 of 920
LABOR REVIEW DIGEST
Atty. Joyrich Golangco

No. The termination was improper. During the time Zialcita was terminated, no regulation had yet been
issued by the Secretary of Labor to implement Article 132.

Even assuming that the Secretary of Labor had already issued such a regulation and to the effect that
stewardesses should remain single, such would be in violation of Article 136 of the Labor Code.

Article 136's protection of women is broader and more powerful than the regulation provided under Article 132.

Other Notes/ SC Pronouncements:


______________________________________________________________________

Angeles| Bajana | Balladares | Brillantes | Briones | Cabansag | Callanta | Chua | David|

De Leon | Gomez | Lopez | Macalino | Nostratis | Padilla | Reynon | Santos | Tan |Velasco
4E / 4F - 2018-2019
Page 184 of 920
LABOR REVIEW DIGEST
Atty. Joyrich Golangco

2. STAR PAPER CORPORATION vs SIMBOL


G.R. No. 164774; April 12, 2006

PONENTE: Puno, J.,

DOCTRINE: In the Philippine jurisdiction, we employ the standard of reasonableness of the company policy which
is parallel to the bona fide occupational qualification requirement. This requirement of reasonableness must
be clearly established to uphold the questioned employment policy. The employer has the burden to prove the
existence of a reasonable business necessity.

FACTS:
Petitioner Star Paper Corporation is a corporation engaged in trading principally of paper products. Respondnets
Ronaldo D. Simbol (Simbol), Wilfreda N. Comia (Comia) and Lorna E. Estrella (Estrella) were all regular
employees of the company.

Respondents Simbol and Comia both married a co-employee in the petitioner company, and alleged that they
were compelled to resign from the company pursuant to a company policy that states “in case of two of our
employees (both singles [sic], one male and another female) developed a friendly relationship during the course of
their employment and then decided to get married, one of them should resign to preserve the policy stated
above.”

As to respondent Estrella, she alleged that she had a relationship with co-worker Zulga who misrepresented
himself as a married but separated man. After he got her pregnant, she discovered that he was not separated.
Thus, she severed her relationship with him to avoid dismissal due to the company policy. However, she was still
dismissed on the ground of immoral conduct.

Respondents filed a complaint for unfair labor practice, constructive dismissal, separation pay and attorneys fees.
They averred that the aforementioned company policy is illegal and contravenes Article 134 (formerly 136) of the
Labor Code.

Angeles| Bajana | Balladares | Brillantes | Briones | Cabansag | Callanta | Chua | David|

De Leon | Gomez | Lopez | Macalino | Nostratis | Padilla | Reynon | Santos | Tan |Velasco
4E / 4F - 2018-2019
Page 185 of 920
LABOR REVIEW DIGEST
Atty. Joyrich Golangco

Petitioners allege that its policy may appear to be contrary to Article 134 of the Labor Code but it assumes a new
meaning if read together with the first paragraph of the rule. The rule does not require the woman employee to
resign. The employee spouses have the right to choose who between them should resign. Further, they are free
to marry persons other than co-employees. Hence, it is not the marital status of the employee, per se, that is
being discriminated. It is only intended to carry out its no-employment-for-relatives-within-the-third-degree-policy
which is within the ambit of the prerogatives of management.

LA/NLRC Ruling:
LA dismissed the complaint. LA opines that the company policy is a proper exercise management prerogative.

NLRC affirmed LA decision.

CA Ruling:
Court of Appeals reversed the NLRC decision and declared that the dismissal of respondents illegal and ordering
private respondents to reinstate petitioners to their former positions without loss of seniority rights with full
backwages from the time of their dismissal until actual reinstatement; and attorneys fees

ISSUE:
Whether or not the company policy is a proper exercise of management prerogative.

HELD:
NO. The questioned government policy is not a proper exercise of management prerogative and is in violation to
Article 134 of the Labor Code. In the Philippine jurisdiction, we employ the standard of reasonableness of the
company policy which is parallel to the bona fide occupational qualification requirement. This requirement of
reasonableness must be clearly established to uphold the questioned employment policy. The employer has the
burden to prove the existence of a reasonable business necessity.

The Court did not find a reasonable business necessity in the case at bar.

Petitioners failed to show how the marriage of Simbol, then a Sheeting Machine Operator, to Alma Dayrit, then an
employee of the Repacking Section, could be detrimental to its business operations. Neither did petitioners
explain how this detriment will happen in the case of Wilfreda Comia, then a Production Helper in the Selecting
Department, who married Howard Comia, then a helper in the cutter-machine. The policy is premised on the mere
fear that employees married to each other will be less efficient. If we uphold the questioned rule without valid
justification, the employer can create policies based on an unproven presumption of a perceived danger at the
expense of an employees right to security of tenure.

The failure of petitioners to prove a legitimate business concern in imposing the questioned policy cannot
prejudice the employees right to be free from arbitrary discrimination based upon stereotypes of married persons
working together in one company.
For failure of petitioners to present undisputed proof of a reasonable business necessity, we rule that the
questioned policy is an invalid exercise of management prerogative.

*(Example of reasonableness of the company policy which is parallel to the bona fide occupational qualification
requirement - Policy of a pharmaceutical company prohibiting its employees from marrying employees of any
competitor company. Company has a right to guard its trade secrets, manufacturing formulas, marketing
strategies and other confidential programs and information from competitors. (Duncan Association of Detailman-
PTGWO and Pedro Tecson v. Glaxo Wellcome Philippines, Inc))

Angeles| Bajana | Balladares | Brillantes | Briones | Cabansag | Callanta | Chua | David|

De Leon | Gomez | Lopez | Macalino | Nostratis | Padilla | Reynon | Santos | Tan |Velasco
4E / 4F - 2018-2019
Page 186 of 920
LABOR REVIEW DIGEST
Atty. Joyrich Golangco

Other Notes/ SC Pronouncements:


______________________________________________________________________

Sexual Harassment

1. Domingo vs. Rayala


G.R. No. 155831; February 18, 2008
Nachura, J.

FACTS: Ma. Lourdes Domingo, then Stenographic Reporter III at the NLRC, filed a Complaint for sexual
harassment against NLRC Chairman Rogelio Rayala before the DOLE. To support the Complaint, Domingo
executed an Affidavit narrating the incidences of sexual harassment complained of, thus:
Sa ibang mga pagkakataon nilalapitan na ako ni Chairman at hahawakan ang aking balikat sabay pisil sa
mga ito habang ako ay nagta-type at habang nagbibigay siya ng diktasyon.

Noong Oktubre 29, 1998, ako ay pumasok sa kwarto ni Chairman Rayala. Ito ay sa kadahilanang ang fax
machine ay nasa loob ng kaniyang kwarto. Nang mabigyan ko na ng fax tone yung kausap ko, pagharap
ko sa kanan ay nakaharang sa dadaanan ko si Chairman Rayala. Tinitingnan ako sa mata at ang titig
niya ay umuusad mula ulo hanggang dibdib tapos ay ngumiti na may mahalay na pakahulugan.

Domingo filed for leave of absence and asked to be immediately transferred. Thereafter, she filed the Complaint
for sexual harassment on the basis of Administrative Order No. 250, the Rules and Regulations Implementing RA
7877 in the DOLE.

Upon receipt of the Complaint, the DOLE Secretary referred the Complaint to the OP, Rayala being a presidential
appointee. The OP found Rayala guilty of the offense charged.

Rayala argues that for sexual harassment to exist under RA 7877, there must be: (a) demand, request, or
requirement of a sexual favor; (b) the same is made a pre-condition to hiring, re-employment, or continued
employment; or (c) the denial thereof results in discrimination against the employee.

Rayala asserts that Domingo has failed to allege and establish any sexual favor, demand, or request from
petitioner in exchange for her continued employment or for her promotion. According to Rayala, it was merely
Domingos perception of malice in his alleged acts a product of her own imagination

ISSUE: whether or not Rayala may be held guilty for sexual harassment

Angeles| Bajana | Balladares | Brillantes | Briones | Cabansag | Callanta | Chua | David|

De Leon | Gomez | Lopez | Macalino | Nostratis | Padilla | Reynon | Santos | Tan |Velasco
4E / 4F - 2018-2019
Page 187 of 920
LABOR REVIEW DIGEST
Atty. Joyrich Golangco

HELD: YES. The law penalizing sexual harassment in our jurisdiction is RA 7877. Section 3 thereof defines work-
related sexual harassment in this wise:

Sec. 3. Work, Education or Training-related Sexual Harassment Defined. Work, education or training-related
sexual harassment is committed by an employer, manager, supervisor, agent of the employer, teacher, instructor,
professor, coach, trainor, or any other person who, having authority, influence or moral ascendancy over another
in a work or training or education environment, demands, requests or otherwise requires any sexual favor from the
other, regardless of whether the demand, request or requirement for submission is accepted by the object of said
Act.

(a) In a work-related or employment environment, sexual harassment is committed when:

(1) The sexual favor is made as a condition in the hiring or in the employment, re-employment or continued
employment of said individual, or in granting said individual favorable compensation, terms, conditions,
promotions, or privileges; or the refusal to grant the sexual favor results in limiting, segregating or classifying the
employee which in a way would discriminate, deprive or diminish employment opportunities or otherwise
adversely affect said employee;

(2) The above acts would impair the employees rights or privileges under existing labor laws; or

(3) The above acts would result in an intimidating, hostile, or offensive environment for the employee.

It is not necessary that the demand, request or requirement of a sexual favor be articulated in a categorical oral or
written statement. It may be discerned, with equal certitude, from the acts of the offender. Holding and squeezing
Domingos shoulders, running his fingers across her neck and tickling her ear, having inappropriate conversations
with her, giving her money allegedly for school expenses with a promise of future privileges, and making
statements with unmistakable sexual overtones all these acts of Rayala resound with deafening clarity the
unspoken request for a sexual favor.

Likewise, contrary to Rayalas claim, it is not essential that the demand, request or requirement be made as a
condition for continued employment or for promotion to a higher position. It is enough that the respondents acts
result in creating an intimidating, hostile or offensive environment for the employee. That the acts of Rayala
generated an intimidating and hostile environment for Domingo is clearly shown by the common factual finding of
the Investigating Committee, the OP and the CA that Domingo reported the matter to an officemate and, after the
last incident, filed for a leave of absence and requested transfer to another unit.

Other Notes/ SC Pronouncements:


______________________________________________________________________

Angeles| Bajana | Balladares | Brillantes | Briones | Cabansag | Callanta | Chua | David|

De Leon | Gomez | Lopez | Macalino | Nostratis | Padilla | Reynon | Santos | Tan |Velasco
4E / 4F - 2018-2019
Page 188 of 920
LABOR REVIEW DIGEST
Atty. Joyrich Golangco

2. Philippine Aeolus Automotive United Corp. vs NLRC


GR No: 124617 Date: April 28, 2000
Ponente: Bellosillo, J

Doctrine: The gravamen of the offense in sexual harassment is not the violation of the employee's sexuality but
the abuse of power by the employer.

FACTS:

Petitioner Philippine Aeolus Automotive United Corporation (PAAUC) is a corporation duly organized and existing
under Philippine laws, petitioner, Francis Chua is its President while private respondent Rosalinda C. Cortez was
a company nurse of Petitioner Corporation until her termination.

A memorandum was issued by the personnel manager of Company asking her to explain why no action should be
taken against her for (1) throwing a stapler at plant manager William Chua; (2) for losing the amount of P1,488
entrusted to her, (3) for asking a co-employee to punch in her time card one morning when she was not there.

She was then placed on preventive suspension. Another memorandum was sent to her asking her to explain why
she failed to process the ATM applications of her co-employees. She submitted a written explanation as to the
loss of the P1,488 and the punching in of her time card.

A third memorandum was sent to her informing her of her termination from service for gross and habitual neglect
of duties, serious misconduct, and fraud or willful breach of trust.

PAAUC dismissed Private Respondent from service on the ground of serious misconduct, gross habitual neglect
and fraud or willful breach of trust. Among the acts she allegedly committed is throwing a stapler at Plant Manager
William Chua, her superior and uttering invectives against him.

She filed with the Labor Arbiter a complaint for illegal dismissal, non-payment of annual service incentive leave,
13th month pay and damages against PAAUC and its President Francis Chua. She claimed as a defense for the
offense charged against her that William Chua manifested a special liking for her. She claimed that William Chua
would oftentimes invite her for a date, make sexual advances – touching her hands, putting his arms around her
shoulders, running his finger on her arms and telling her she looked beautiful. The special treatment and sexual
advances continued during her employment for 4 years but she never reciprocated his flirtations, until finally, she
noticed that his attitude toward her changed. He made her understand that if she would not give in to his sexual
advances he would cause her termination from the service; and he made good his threat when he started
harassing her.

LA NLRC RULING:
Angeles| Bajana | Balladares | Brillantes | Briones | Cabansag | Callanta | Chua | David|

De Leon | Gomez | Lopez | Macalino | Nostratis | Padilla | Reynon | Santos | Tan |Velasco
4E / 4F - 2018-2019
Page 189 of 920
LABOR REVIEW DIGEST
Atty. Joyrich Golangco

The Labor Arbiter rendered a decision holding the termination of Cortez as valid and legal, at the same time
dismissing her claim for damages for lack of merit.
NLRC RULING:
The NLRC disbelieved the explanation proffered by private respondent on the ground she never filed a complaint
against William Chua for more than 4 years.

ISSUE/S:

Whether or not William Chua committed acts of sexual harassment against Cortez?

HELD:

Yes. The gravamen of the offense in sexual harassment is not the violation of the employee's sexuality but the
abuse of power by the employer. Any employee, male or female, may rightfully cry "foul" provided the claim is well
substantiated. Strictly speaking, there is no time period within which he or she is expected to complain through the
proper channels. The time to do so may vary depending upon the needs, circumstances, and more importantly,
the emotional threshold of the employee.

Private respondent admittedly allowed four (4) years to pass before finally coming out with her employer's sexual
impositions. Not many women, especially in this country, are made of the stuff that can endure the agony and
trauma of a public, even corporate, scandal. If petitioner corporation had not issued the third memorandum that
terminated the services of private respondent, we could only speculate how much longer she would keep her
silence. Moreover, few persons are privileged indeed to transfer from one employer to another. The dearth of
quality employment has become a daily "monster" roaming the streets that one may not be expected to give up
one's employment easily but to hang on to it, so to speak, by all tolerable means. Perhaps, to private respondent's
mind, for as long as she could outwit her employer's ploys she would continue on her job and consider them as
mere occupational hazards. This uneasiness in her place of work thrived in an atmosphere of tolerance for four (4)
years, and one could only imagine the prevailing anxiety and resentment, if not bitterness, that beset her all that
time. But William Chua faced reality soon enough. Since he had no place in private respondent's heart, so must
she have no place in his office. So, he provoked her, harassed her, and finally dislodged her; and for finally
venting her pent-up anger for years, he "found" the perfect reason to terminate her.

----------------------------------------

ART. 139

1. APEX MINING COMPANY, INC. vs NLRC


GR No: 94951 Date: April 22, 1991
Angeles| Bajana | Balladares | Brillantes | Briones | Cabansag | Callanta | Chua | David|

De Leon | Gomez | Lopez | Macalino | Nostratis | Padilla | Reynon | Santos | Tan |Velasco
4E / 4F - 2018-2019
Page 190 of 920
LABOR REVIEW DIGEST
Atty. Joyrich Golangco

Ponente: Gancayco, J.

Doctrine:
The mere fact that the househelper or domestic servant is working within the premises of the business of the
employer and in relation to or in connection with its business, as in its staffhouses for its guest or even for its
officers and employees, warrants the conclusion that such househelper or domestic servant is and should be
considered as a regular employee of the employer and not as a mere family househelper or domestic servant.

FACTS:
Private respondent Sinclita Candida was employed by petitioner Apex Mining Company, Inc. on May 18, 1973
to perform laundry services at its staff house located at Masara, Maco, Davao del Norte. In the beginning, she
was paid on a piece rate basis. However, on January 17, 1982, she was paid on a monthly basis at P250.00 a
month which was ultimately increased to P575.00 a month. On December 18, 1987, while she was attending to
her assigned task and she was hanging her laundry, she accidentally slipped and hit her back on a stone. She
reported the accident to her immediate supervisor Mila de la Rosa and to the personnel officer, Florendo D. Asirit.
As a result of the accident she was not able to continue with her work. She was permitted to go on leave for
medication. De la Rosa offered her the amount of P 2,000.00 which was eventually increased to P5,000.00 to
persuade her to quit her job, but she refused the offer and preferred to return to work. Petitioner did not allow her
to return to work and dismissed her on February 4, 1988. On March 11, 1988, private respondent filed a request
for assistance with the Department of Labor and Employment.

LA: in favor of respondent, ordering petitioner to pay private respondent

NLRC: Affirmed LA decision

APPEAL TO THE SC:

Petitioner's Contention:
Petitioner contends that it is only when the househelper or domestic servant is assigned to certain aspects of the
business of the employer that such househelper or domestic servant may be considered as such as employee.

ISSUE/S:
Whether private respondent should be treated as a mere househelper or domestic servant and not as a regular
employee of petitioner

HELD:
Private respondent is a regular employee of the petitioner.
Under Rule XIII, Section l(b), Book 3 of the Labor Code, as amended, the terms "househelper" or "domestic
servant" are defined as follows:
The term "househelper" as used herein is synonymous to the term "domestic servant" and shall refer to
any person, whether male or female, who renders services in and about the employer's home and which
services are usually necessary or desirable for the maintenance and enjoyment thereof, and ministers
exclusively to the personal comfort and enjoyment of the employer's family.3
The foregoing definition clearly contemplates such househelper or domestic servant who is employed in the
employer's home to minister exclusively to the personal comfort and enjoyment of the employer's family. Such
definition covers family drivers, domestic servants, laundry women, yayas, gardeners, houseboys and other
similar househelps.
The definition cannot be interpreted to include househelp or laundrywomen working in staffhouses of a company,
like petitioner who attends to the needs of the company's guest and other persons availing of said facilities. By the
Angeles| Bajana | Balladares | Brillantes | Briones | Cabansag | Callanta | Chua | David|

De Leon | Gomez | Lopez | Macalino | Nostratis | Padilla | Reynon | Santos | Tan |Velasco
4E / 4F - 2018-2019
Page 191 of 920
LABOR REVIEW DIGEST
Atty. Joyrich Golangco

same token, it cannot be considered to extend to then driver, houseboy, or gardener exclusively working in the
company, the staffhouses and its premises. They may not be considered as within the meaning of a "househelper"
or "domestic servant" as above-defined by law.
The criteria is the personal comfort and enjoyment of the family of the employer in the home of said employer.
While it may be true that the nature of the work of a househelper, domestic servant or laundrywoman in a home or
in a company staffhouse may be similar in nature, the difference in their circumstances is that in the former
instance they are actually serving the family while in the latter case, whether it is a corporation or a single
proprietorship engaged in business or industry or any other agricultural or similar pursuit, service is being
rendered in the staffhouses or within the premises of the business of the employer. In such instance, they are
employees of the company or employer in the business concerned entitled to the privileges of a regular employee.
The mere fact that the househelper or domestic servant is working within the premises of the business of the
employer and in relation to or in connection with its business, as in its staffhouses for its guest or even for its
officers and employees, warrants the conclusion that such househelper or domestic servant is and should be
considered as a regular employee of the employer and not as a mere family househelper or domestic servant as
contemplated in Rule XIII, Section l(b), Book 3 of the Labor Code, as amended.

Other Notes/ SC Pronouncements:

__________________________________________________________________________

Angeles| Bajana | Balladares | Brillantes | Briones | Cabansag | Callanta | Chua | David|

De Leon | Gomez | Lopez | Macalino | Nostratis | Padilla | Reynon | Santos | Tan |Velasco
4E / 4F - 2018-2019
Page 192 of 920
LABOR REVIEW DIGEST
Atty. Joyrich Golangco

BOOK IV

1. GSIS v. CA
G.R. No. 124208
January 28, 2008
Azcuna J.

Doctrine:

Art. 167 (l), Chapter 1, Title II, Book Four of the Labor Code of the Philippines, defines sickness as "any illness
definitely accepted as an occupational disease listed by the [Employees’ Compensation Commission], or any
illness caused by employment, subject to proof that the risk of contracting the same is increased by working
conditions." The same provision empowers ECC to determine and approve occupational diseases and work-
related illnesses that may be considered compensable based on peculiar hazards of employment.

Under Sec. 1 (b), Rule III of the Amended Rules on Employees’ Compensation, "[f]or the sickness and the
resulting disability or death to be compensable, the sickness must be the result of an occupational disease listed
under Annex ‘A’ of these Rules with the conditions set therein satisfied; otherwise, proof must be shown that the
risk of contracting the disease is increased by the working conditions."

FACTS:

In 1974, Abraham Cate joined the Philippine Navy. He was later promoted to Master Sergeant of the Philippine
Constabulary. On January 2, 1991, he was absorbed in the PNP with the rank of Senior Police Officer IV (SPO4).

In 1993, Abraham complained of a mass on his left cheek. He was admitted at the PGH payward, and on October
28, 1993, he underwent "Total Maxillectomy with Orbital Exenteration," which operation removed the mass on his
left cheek. In April 1994, another biopsy revealed the recurrence of the ailment. Post-operative course was
uneventful and he underwent radiotherapy.

On December 1, 1994, Abraham was compulsorily retired from the PNP.

On December 20, 1994, Abraham filed a claim for income benefits with the GSIS under P.D. No. 626, as
amended. The GSIS denied the claim on the ground that Osteosarcoma is not considered an occupational
disease under P.D. No. 626, and there is no showing that his duties as SPO4 in the Armed Forces of the
Philippines had increased the risk of contracting said ailment.

On May 2, 1995, Abraham died at the age of 45. The heirs of Abraham appealed the decision of GSIS to the
ECC. The ECC affirmed the decision of GSIS and dismissed the case for lack of merit.

The heirs of Abraham filed a petition for review of the decision of ECC with the CA.

CA RULING:

The CA reversed and set aside the decision of ECC declaring that the ailment of the late Abraham Cate is
compensable under PD 626, as amended. The GSIS and ECC separately filed a petition for review on certiorari of
the decision of the CA.

APPEAL TO THE SC:


Angeles| Bajana | Balladares | Brillantes | Briones | Cabansag | Callanta | Chua | David|

De Leon | Gomez | Lopez | Macalino | Nostratis | Padilla | Reynon | Santos | Tan |Velasco
4E / 4F - 2018-2019
Page 193 of 920
LABOR REVIEW DIGEST
Atty. Joyrich Golangco

Petitioner's Contention:

Petitioners aver that the applicable law in Abraham’s case is the Amended Rules on Employees’ Compensation
which is explained in Tanedo v. ECC, thus:

Awards of compensation benefits for death or disability can now no longer be made to rest on presumption, but on
a showing that the causative disease is among those listed by the ECC, or on substantial evidence that the risk of
contracting said disease is increased by the employee’s working conditions.

Petitioners argue Abraham’s ailment had no causal connection with his employment and working conditions. Nor
can it be said that the nature of his work had increased the risk of contracting his ailment. The illness is not
prevalent in the Philippine Navy or the PNP. Even under the less stringent evidentiary norm of substantial
evidence obtaining in employees’ compensation proceedings, private respondents failed to adduce such relevant
evidence as a reasonable mind might accept as adequate to support their claim.

Respondent's Contention:

Private respondents tried to establish a preliminary link between the illness and the employment of Abraham by
speculating that since Abraham did some dirty jobs during his stint as a rifleman in the Philippine Navy, he was
exposed to some elements like virus which could have contributed more or less to the development of his
ailment.

ISSUE/S:

1. Whether or not the CA erred in ruling that the ailment of the late Abraham is compensable under the present
law on employees’ compensation.

HELD:

1. In this case, Osteosarcoma is not listed as an occupational disease in the Amended Rules on Employees’
Compensation. Hence, it is supposed to be upon the claimant or private respondents to prove by substantial
evidence that the risk of contracting Osteosarcoma was increased by the working conditions of the late Abraham.
Substantial evidence means such relevant evidence as a reasonable mind might accept as adequate to support a
conclusion.11The records show that Abraham failed to present evidence to establish that the development of his
ailment was traceable to his working conditions in the Philippine Navy, the now defunct Philippine Constabulary
and the PNP. Further, private respondents’ allegation in their petition for review with the CA that Abraham, as a
rifleman in the Philippine Navy, may have been exposed to elements like a virus which could have contributed to
his ailment does not satisfy the requirement of substantial evidence. The rule is that awards of compensation
cannot rest on speculations and presumptions as the claimant must prove a positive thing. The application of the
rules would mean that absent any proof that the risk of contracting the ailment was increased by the working
conditions of the late Abraham, private respondents would not be entitled to compensation.

Considering, however, that it is practically undisputed that under the present state of science, the proof referred by
the law to be presented by the deceased private respondent claimant was unavailable and impossible to comply
with, the condition must be deemed as not imposed.

As earlier noted, however, in the specific case of respondent, the requirement is impossible to comply with, given
the present state of scientific knowledge. The obligation to present such as an impossible evidence must,

Angeles| Bajana | Balladares | Brillantes | Briones | Cabansag | Callanta | Chua | David|

De Leon | Gomez | Lopez | Macalino | Nostratis | Padilla | Reynon | Santos | Tan |Velasco
4E / 4F - 2018-2019
Page 194 of 920
LABOR REVIEW DIGEST
Atty. Joyrich Golangco

therefore, be deemed void. Respondent, therefore, is entitled to compensation, consistent with the social
legislation’s intended beneficial purpose.

Other Notes/ SC Pronouncements:

________________________________________________________________________

Angeles| Bajana | Balladares | Brillantes | Briones | Cabansag | Callanta | Chua | David|

De Leon | Gomez | Lopez | Macalino | Nostratis | Padilla | Reynon | Santos | Tan |Velasco
4E / 4F - 2018-2019
Page 195 of 920
LABOR REVIEW DIGEST
Atty. Joyrich Golangco

2. DOMINGA A. SALMONE vs. EMPLOYEES' COMPENSATION COMMISSION and SOCIAL SECURITY


SYSTEM
G.R. No. 142392; September 26, 2000
PARDO, J.

DOCTRINE: For a sickness and the resulting disability or death to be compensable, the said sickness must be an
occupational disease listed under Annex "A" of said Rules, otherwise, the claimant or employee concerned must
prove that the risk of contracting the disease is increased by the working condition.

FACTS:

Sometime in 1982, the Petitioner was employed as sewer by the Paul Geneve Entertainment
Corporation, a corporation engaged in the business of sewing costumes, gowns and casual and formal
dresses. She was later promoted as the officer-in-charge and the over-all custodian in the Sewing Department,
more particularly the procurement of all the materials needed by the Sewing Department as well as insuring the
quality of the products from the sewing department.
Sometime in the early part of 1996, Petitioner started to feel chest pains. In April, 1996, she filed a leave
of absence from work as the chest pains became unbearable. Per results of Petitioner's Medical examination
conducted by Dr. Claudio Saratan, Jr., petitioner was found suffering from Atherosclerotic heart disease, Atrial
Fibrillation, Cardiac Arrhythmia. Upon recommendation of her doctor, Petitioner resigned from her work hoping
that with a much-needed complete rest, she will be cured.
Petitioner later filed a disability claim with the SSS from the Employees' Compensation Fund, under
Presidential Decree No. 626, as amended.
SSS RULING: SSS denied Petitioner's claim.

CA RULING: Dismissed the petition, ruling that petitioner's illness was not compensable because petitioner failed
to adduce substantial evidence proving any of the conditions of compensability.

Petitioner insists in her Petition at bench that the nature of her employment and the working conditions in
her place of employment exacerbated the risks of contracting Atherosclerotic Heart disease, Atrial Fibrillation,
Cardiac arrhythmia. Hence, the Public Respondent committed a reversible error in finding and declaring that
Petitioner did not contract the disease that bedeviled her due to her work and working conditions and that
Petitioner's nature of employment did not predispose her to contract the disease and, hence, the Petitioner was
not entitled to her claim.

ISSUE:
Whether or not petitioner's illness is compensable as work-related.

RULING:

YES. Petitioner has shown by uncontroverted evidence that in the course of her employment, due to
work related stress, she suffered from severe chest pains which caused her to take a rest, per physician's advice,
and ultimately to resign from her employment. She was diagnosed as suffering from "atherosclerotic heart
disease, atrial fibrillation, cardiac arrhythmia" which, as heretofore stated, is included within the term
cardiovascular diseases.
Indisputably, cardiovascular diseases, which, as herein above-stated include atherosclerotic heart disease,
atrial fibrillation, cardiac arrhythmia, are listed as compensable occupational diseases in the Rules of the
Employees' Compensation Commission, hence, no further proof of casual relation between the disease and
claimant's work is necessary.

Angeles| Bajana | Balladares | Brillantes | Briones | Cabansag | Callanta | Chua | David|

De Leon | Gomez | Lopez | Macalino | Nostratis | Padilla | Reynon | Santos | Tan |Velasco
4E / 4F - 2018-2019
Page 196 of 920
LABOR REVIEW DIGEST
Atty. Joyrich Golangco

Angeles| Bajana | Balladares | Brillantes | Briones | Cabansag | Callanta | Chua | David|

De Leon | Gomez | Lopez | Macalino | Nostratis | Padilla | Reynon | Santos | Tan |Velasco
4E / 4F - 2018-2019
Page 197 of 920
LABOR REVIEW DIGEST
Atty. Joyrich Golangco

3. HEIRS OF DEAUNA v. FIL STAR MARITIME CORPORATION


GR No: 191563 Date: June 10, 2012
Ponente: Reyes, J.

Doctrine: While it is true that Article 22.1 of the IBF/AMOSUP/IMMAJ CBA considers a seafarer as terminated
when he signs off from the vessel due to sickness, the foregoing is subject to the provisions of Article 29. Under
Article 29, a seafarer remains under the respondents' employ as long as the former is still entitled to medical
assistance and sick pay, and provided that the death which eventually occurs is directly attributable to the
sickness which caused the seafarer's employment to be terminated.

FACTS:
Respondent Fil-Star Maritime Corporation (Fil-Star) is a local manning agency, with respondent Captain
Victor S. Millalos (Capt. Millalos) as its general manager. Respondent Grandslam Enterprise Corporation
(Grandslam) is among Fil-Star's foreign principals. Grandslam owns and manages the vessel M/V Sanko Stream
(Sanko) which Edwin boarded on August 1, 2004 for a nine-month engagement as Chief Engineer. As such, he
was responsible for the operations and maintenance of the entire vessel's engineering equipment. He also
determined the requirements for fuel, lube oil and other consumables necessary for a voyage, conducted
inventory of spare parts, prepared the engine room for inspection by marine and safety authorities, and took
charge of the engine room during maneuvering and emergency situations.
Prior to Edwin's deployment, he underwent the customary Pre-employment Medical Examination (PEME) and was
found as fit to work as was repeatedly the case in the past 30 years since his first deployment by Fil-Star in 1975.
He suffered from abdominal pains and was found to have kidney stones for which he was given
medication. Edwin was then repatriated. Respondents claimed that Edwin requested for an early termination while
petitioners averred that Edwin was repatriated due to the latter's body weakness and head heaviness. Edwin was
discovered to have Glioblastoma WHO Grade 4 (GBM) . It was then noted that Edwin could have acquired the
cancer as a result of radiation or vinyl products, or had worked in the vicinity of power lines.
The respondents claimed that out of compassion and intent to avoid legal battles, they extended to
Edwin an allowance of US$6,033.36. They also offered the payment of US$60,000.00 disability benefits despite
having no obligation to do so on their part as GBM can only be considered as work-related if a person who suffers
therefrom had exposures to radiation or vinyl products, or had worked in the vicinity of power lines. The
respondents claimed that Edwin did not have such exposure while under their employ.
Two demand letters seeking disability benefits were thereafter sent by the petitioners to the
respondents. The first, which was received by the respondents on November 21, 2005, sought the payment
of US$125,000.00 as allegedly provided under the International Bargaining Forum/Associated Marine Officers and
Seamens Union of the Philippines/International Mariners Management Association of Japan Collective Bargaining
Agreement (IBF/AMOSUP/IMMAJ CBA). The second letter, dated December 8, 2005, reiterated the petitioners'
claims for disability benefits. The respondents replied that they had already aptly dealt with the illness under the
respective employment agreement. Not long after, the petitioners again wrote the respondents informing the latter
that Edwin's condition was already critical. Hence, the possibility that the claims for disability benefits would be
converted to death benefits arose. The respondents denied the petitioners' demand.
In December 2005, a complaint for disability benefits, medical and transportation reimbursements, moral
and exemplary damages and attorney's fees were filed before the National Labor Relations Commission
(NLRC). Edwin died on April 13, 2006 during the pendency of the proceedings.

LA RULING: After finding that there was an arbitration clause in the IBF/AMOSUP/IMMAJ CBA, the Labor Arbiter
(LA) rendered a decision referring the complaint to voluntary arbitration. The case was thereafter docketed with
the National Conciliation and Mediation Board (NCMB) as AC 94-NCMB-NCR-39-01-13-07. Voluntary Arbitrator
Rene Ofreneo (VA Ofreneo), invoking the provisions of the Philippine Overseas Employment Administration

Angeles| Bajana | Balladares | Brillantes | Briones | Cabansag | Callanta | Chua | David|

De Leon | Gomez | Lopez | Macalino | Nostratis | Padilla | Reynon | Santos | Tan |Velasco
4E / 4F - 2018-2019
Page 198 of 920
LABOR REVIEW DIGEST
Atty. Joyrich Golangco

Standard Employment Contract (POEA SEC) and the IBF/AMOSUP/IMMAJ CBA, awarded death benefits to the
petitioners.

COURT OF APPEALS: The Court of Appeals reversed the decision of VA Ofreneo.

PETITION TO THE SC:


Petitioner's Contention: The petitioners emphasize that under the IBF/AMOSUP/IMMAJ CBA, a seafarer's death
is compensable regardless of its cause and its non work-relatedness as long as it occurs during the term of the
latter's employment. The only exception to compensability is when death is due to willful acts. In Edwin's case, he
had been under the respondents' employment for the past 30 years. Prior to boarding Sanko, he passed the
PEME but was thereafter medically-repatriated as stated in Dr. Cruz's report. He died of GBM, the origin of which
is unknown. Hence, it can be presumed that GBM had been contracted during his employment with the
respondents.

Respondent’s Contention: That Edwin's illness was not work-related and his death occurred not during the term
of his employment. Thus, the petitioners are not entitled to the payment of any benefits. The mere circumstance
that the manifestations of an illness appeared while the seafarer is on-board does not necessarily render it as
work-related. In the POEA SEC, the words during the term of contract refer to the time when death occurs while
work-related refers to the cause of death. The two requisites must both be proven especially in view of the Court's
declaration in Rivera v. Wallem Maritime Services, Inc., that in the absence of substantial evidence, working
conditions cannot be presumed to have increased the risk of contracting the disease. Petitioners' bare allegation,
that GBM was work-related as can be inevitably concluded from Edwin's lengthy and repeated employment with
the respondents, deserves no probative value unless corroborated by substantial evidence. Dr. Cruz, who had
attended to Edwin's medical needs for more than three months, opined that GBM was not work-related as the
latter, in the course of his employment with the respondents, was never exposed to factors which would have
increased the risk of contracting the illness.

ISSUE/S:
1. Whether or not within the purview of the IBF/AMOSUP/IMMAJ CBA, Edwin's death on April 13, 2006, or more
than a year from his repatriation, can be considered as one occurring while he was still in the employment of the
respondents.

HELD:
YES. Edwin's death can be considered can be considered as one occuring while he was still in the employment of
respondents. Under the IBF/AMOSUP/IMMAJ CBA provisions, Edwin's death a little more than a year from his
repatriation can still be considered as one occurring while he was still under the respondents' employ.
On August 22, 2005, or more or less 130 days from Edwin's arrival in the Philippines, the company-
designated physician, Dr. Cruz, indicated in a medical report addressed to Capt. Millalos that Edwin's repatriation
was due to body weakness and head heaviness since October 2004. Dr. Cruz also stated that upon Edwin's
arrival at the airport on April 3, 2005, the latter was noted to be drowsy and disoriented. Dr. Cruz diagnosed Edwin
to be suffering from GBM and submitted the monthly expenses for the latter's chemotherapy to Capt.
Millalos. Edwin was advised to come back on September 23, 2005. Edwin eventually died of GBM on April 13,
2006.
The Court noted that body weakness, head heaviness, drowsiness and disorientedness are among the
symptoms associated with GBM. Dr. Cruz indicated that these symptoms were exhibited by Edwin since October
2004 while he was still on board Sanko and were notable even when the latter was repatriated on April 3, 2005.
Nonetheless, the symptoms previously referred to were the cause of Edwin's repatriation more or less than a
month before his contract was about to expire. On May 4, 2005 or about a month after repatriation, Dr. Mercado
found that Edwin was afflicted with GBM and that the tumor had been progressively growing for months. Further,
the medical report, dated August 22, 2005, addressed to Capt. Millalos, submitting to him the monthly expenses
Angeles| Bajana | Balladares | Brillantes | Briones | Cabansag | Callanta | Chua | David|

De Leon | Gomez | Lopez | Macalino | Nostratis | Padilla | Reynon | Santos | Tan |Velasco
4E / 4F - 2018-2019
Page 199 of 920
LABOR REVIEW DIGEST
Atty. Joyrich Golangco

for Edwin's chemotherapy and advising the latter to come back on September 23, 2005, was an implied admission
on the part of Dr. Cruz that medical assistance and sick pay should indeed be extended to Edwin even beyond the
130-day period prescribed by Articles 25 and 26 of the IBF/AMOSUP/IMMAJ CBA.

At the time of Edwin's death on April 13, 2006 due to GBM, he was still in the employment of the
respondents. Under Article 29, a seafarer remains under the respondents' employ as long as the former is still
entitled to medical assistance and sick pay, and provided that the death which eventually occurs is directly
attributable to the sickness which caused the seafarer's employment to be terminated. As discussed above, the
company-designated physician, Dr. Cruz, in effect admitted that Edwin was repatriated due to symptoms which a
person suffering from GBM normally exhibits. Further, he recommended to Capt. Millalos Edwin's entitlement to
medical assistance and sick pay for a period beyond 130 days from repatriation. Edwin subsequently died of
GBM, the symptoms of which were the cause of his earlier repatriation. Hence, since Edwin's death is reasonably
connected to the cause of his repatriation, within the purview of the IBF/AMOSUP/IMMAJ CBA, he indubitably
died while under the respondents' employ, thus, entitling the petitioners to death benefits as provided for in
Appendix 3 of the said CBA.

Angeles| Bajana | Balladares | Brillantes | Briones | Cabansag | Callanta | Chua | David|

De Leon | Gomez | Lopez | Macalino | Nostratis | Padilla | Reynon | Santos | Tan |Velasco
4E / 4F - 2018-2019
Page 200 of 920
LABOR REVIEW DIGEST
Atty. Joyrich Golangco

4. DEBAUDIN v. SSS
G.R. No. 148308 September 21, 2007

DOCTRINE: An employee is entitled to compensation benefits if the sickness is a result of an occupational


disease listed under the Rules on Employees' Compensation; or in case of any other illness, if it is caused by
employment, subject to proof that the risk of contracting the same is increased by the working conditions. This is
as it should be because for an illness to be compensable, it must be (1) directly caused by such employment; (2)
aggravated by the employment; or (3) the result of the nature of such employment. Jurisprudence provides that to
establish compensability of a non-occupational disease, reasonable proof of work-connection and not direct
causal relation is required.

FACTS: Petitioner, Roberto Debaudin, is a seaman by profession as a utility staff who performed cleaning
chemical-spilloil on deck, slat dislodging, and spraying naphtha chemical and washing dirt and rusts inside the
tank. 18 years after, Debaudin sought medical assistance after he experienced episodes of bilateral blurring of
vision and was later diagnosed of chronic open angle glaucoma. On account of his ailment, petitioner filed before
the SSS a claim for compensation benefits under P.D. No. 626 claiming that the strenuous tasks required
climbing, bending over and running for so many times – acts which a medical book considered as contributory
factors that would increase intraocular pressure which causes glaucoma. He also adds that he was also subjected
to emotional strains of going through the perils of the sea and homesickness for being away from his family during
the entire duration of the contracts. He, thus, alleges that his employment as a seaman contributed, even in a
small degree, to the development of his ailment. His claim, however, was denied by SSS on the ground that there
is no causal relationship between the illness and his job as a seaman.

EMPLOYEES COMPENSTAION COMMISSION (EEC): EEC denied Debaudin’s motion for reconsideration
holding that Debaudin’s Chronic Open Angle Glaucoma is not an occupational disease under the law. Thus, he is
required to show by substantial evidence that the nature of his job as a Seaman had increased the risk of
contracting the disease. However, appellant failed to discharge the burden of proof required by the law.

CA RULING: The CA dismissed the case on the ground that petitioner failed to adduce substantial evidence
supporting the conclusion that the working conditions as a seaman increased the risk of contracting his chronic
open angle glaucoma.

ISSUE: Whether or not the work of Debaudin as a seaman contributed even in a small degree in or had increased
the risk of contracting his chronic open angle glaucoma.

RULING: NO. In the present case, petitioner’s chronic open angle glaucoma is not listed as an occupational
disease; hence, he has the burden of proving by substantial evidence, or such relevant evidence which a
reasonable mind might accept as adequate to justify a conclusion, that the nature of his employment or working
conditions increased the risk of contracting the ailment or that its progression or aggravation was brought about
thereby. It is enough that the hypothesis on which the workmen's claim is based is probable. Probability, not the
ultimate degree of certainty, is the test of proof in compensation proceedings since in carrying out and interpreting
the provisions of the Labor Code and its implementing rules and regulations the primordial and paramount
consideration is the employees' welfare. Other than positing petitioner’s allegations, petitioner presented no
competent medical history, records or physician’s report to objectively substantiate the claim that there is a
reasonable nexus between his work and his ailment. Without saying more, his bare allegations do not ipso facto
make his illness compensable. Awards of compensation cannot rest on speculations or presumptions. The
claimant must present concrete evidence to prove a positive proposition.

Angeles| Bajana | Balladares | Brillantes | Briones | Cabansag | Callanta | Chua | David|

De Leon | Gomez | Lopez | Macalino | Nostratis | Padilla | Reynon | Santos | Tan |Velasco
4E / 4F - 2018-2019
Page 201 of 920
LABOR REVIEW DIGEST
Atty. Joyrich Golangco

5. PABLO AUSTRIA vs. CA


GR No: 146636 Date: August 12, 2002
Ponente: Puno, J.

Doctrine: A disability is total and permanent if as a result of the injury or sickness, the employee is unable to
perform any gainful occupation for a continuous period exceeding 120 days; and a disability is partial and
permanent if as a result of the injury or sickness, the employee suffers a permanent partial loss of the use of any
part of his body.

FACTS:
Petitioner Pablo A. Austria was employed as bag piler at Central Azucarera de Tarlac from June 1, 1977 to
July 20, 1997. As bag piler, his duties were to:
(1) carry and pile sacks of refined sugar;
(2) relocate and move stock piles for shifting or return to the refinery;
(3) assist the production checker in random weighing of production;
(4) clean the warehouse, piling conveyor and its surroundings;
(5) assist in the repair and maintenance work during off-season; and
(6) do other related work assigned to him from time to time by his superior.
In 1994, Austria began to feel severe back pain. He underwent an MRI which revealed a small disc protrusion at
L4 and L5 level. He also underwent Laminectomy on March 17, 1995 at the Ramos General Hospital in Tarlac,
Tarlac. Due to his osteoarthritis, Austria filed with the SSS a claim for compensation benefits under PD 626 as
amended. The claim was granted and petitioner was awarded permanent partial disability benefits for eight (8)
months starting September 1, 1995, a second release for seven (7) months starting May 10, 1996, and a third
release for fifteen (15) months starting April 1, 1997. Petitioner thereafter requested the SSS for conversion of his
permanent partial disability benefit to permanent total disability benefit which the SSS denied since there was no
progression of his illness.

Austria appealed to the Employee’s Compensation Commission. The latter affirmed the decision of SSS. It ruled
that Austria has already availed of the maximum benefits to which he is entitled.

He appealed the ECC decision to the CA via a Petition for Certiorari. The CA denied Austria’s appeal. It held that
the law does not allow the conversion of permanent partial to permanent total.

ISSUE: Whether the conversion of permanent partial disability to permanent total disabiliy is allowed.

HELD:

PD 626 as amended provides three types of disability benefits to qualified employees: (1) temporary total
disability, (2) permanent total disability, and (3) permanent partial disability. In the case at bar, petitioner was
granted by the SSS, as affirmed by the ECC, permanent partial disability benefit, but he seeks to avail of
permanent total disability benefit. Under Section 2 Rule VII of the Amended Rules on Employees Compensation,
a disability is total and permanent if as a result of the injury or sickness, the employee is unable to perform any
gainful occupation for a continuous period exceeding 120 days; and a disability is partial and permanent if as a
result of the injury or sickness, the employee suffers a permanent partial loss of the use of any part of his body.

The Court previously ruled in Vicente vs. ECC: the test of whether or not an employee suffers from
permanent total disability is a showing of the capacity of the employee to continue performing his work
notwithstanding the disability he incurred. Thus, if by reason of the injury or sickness he sustained, the
employee is unable to perform his customary job for more than 120 days and he does not come within the
Angeles| Bajana | Balladares | Brillantes | Briones | Cabansag | Callanta | Chua | David|

De Leon | Gomez | Lopez | Macalino | Nostratis | Padilla | Reynon | Santos | Tan |Velasco
4E / 4F - 2018-2019
Page 202 of 920
LABOR REVIEW DIGEST
Atty. Joyrich Golangco

coverage of Rule X of the Amended Rules on Employees Compensability (which, in more detailed manner,
describes what constitutes temporary total disability), then the said employee undoubtedly suffers from permanent
total disability regardless of whether or not he loses the use of any part of his body.

Disability is intimately related to ones earning capacity. It should be understood less on its medical significance
but more on the loss of earning capacity. petitioner entitled to permanent total disability benefit under the law.
Petitioner has been employed as bag piler for twenty (20) years at the Central Azucarera de Tarlac. His duties
require him to carry heavy loads of refined sugar and to perform other manual work. Since his work obviously
taxes so much on his back, his illness which affects his lumbar spine renders him incapable of doing his usual
work as bag piler. Hence, his disability to perform his regular duties may be considered total and permanent.

Angeles| Bajana | Balladares | Brillantes | Briones | Cabansag | Callanta | Chua | David|

De Leon | Gomez | Lopez | Macalino | Nostratis | Padilla | Reynon | Santos | Tan |Velasco
4E / 4F - 2018-2019
Page 203 of 920
LABOR REVIEW DIGEST
Atty. Joyrich Golangco

6. ALEXANDER B. GATUS v. SSS


G.R. No. 174725 January 26, 2011

LEONARDO-DE CASTRO, J.

Facts:
- Petitioner Alexander B. Gatus worked at the Central Azucarera de Tarlac beginning on January 1, 1972.
- He was a covered member of the SSS (SS No. 02-0055015-6).
- He optionally retired from Central Azucarera de Tarlac upon reaching 30 years of service on January 31,
2002, at the age of 62 years.
- By the time of his retirement, he held the position of Tender assigned at the Distillery Cooling Tower.
- In the course of his employment in Central Azucarera de Tarlac, he was certified fit to work on October
21, 1975 and was accordingly promoted to a year-round regular employment.
- He suffered chest pains and was confined at the Central Luzon Doctors Hospital in Tarlac City on August
12, 1995.
- Upon discharge on August 17, 1995, he was diagnosed to be suffering from Coronary Artery Disease
(CAD): Triple Vessel and Unstable Angina. His medical records showed him to be hypertensive for 10
years and a smoker.
- On account of his CAD, he was given by the SSS the following EC/SSS Permanent Partial Disability
(PPD) benefits:
o (a) 8 monthly pensions effective September 1, 1994 and
o (b) 4 monthly pensions effective January 3, 1997.
o He became an SSS retirement pensioner on February 1, 2002.
- Sometime in 2003, an SSS audit revealed the need to recover the EC benefits already paid to him on the
ground that his CAD, being attributed to his chronic smoking, was not work-related.
- Convinced that he was entitled to the benefits, he assailed the decision but the SSS maintained its
position. The SSS also denied his motion for reconsideration.
- He elevated the matter to the ECC, which denied his appeal ruling that CAD was a cardiovascular
disease listed as an occupational disease under Annex A of the Implementing Rules on Employees
Compensation it was incumbent upon him to prove that the nature of his previous employment and the
conditions prevailing therein had increased the risk of contracting his CAD; and that he had failed to
prove this requisite
- Court of Appeals held that petitioner is not entitled to compensation benefits under Presidential Decree
No. 626, as amended, affirming the Decision of the Employees Compensation Commission

ISSUE:
- whether the Court of Appeals committed grave abuse of discretion in affirming the finding of the ECC that
petitioners ailment is not compensable under Presidential Decree No. 626, as amended.

HELD:
 The grounds for compensability are set forth in Section 1, Rule III of the Amended Rules on Employees
Compensation (the Amended Rules), the pertinent portion of which states:
RULE III
Compensability
Sec. 1. Grounds x x x

(b) For the sickness and the resulting disability or death to be compensable, the sickness must
be the result of an occupational disease listed under Annex A of these Rules with the conditions set

Angeles| Bajana | Balladares | Brillantes | Briones | Cabansag | Callanta | Chua | David|

De Leon | Gomez | Lopez | Macalino | Nostratis | Padilla | Reynon | Santos | Tan |Velasco
4E / 4F - 2018-2019
Page 204 of 920
LABOR REVIEW DIGEST
Atty. Joyrich Golangco

therein satisfied; otherwise, proof must be shown that the risk of contracting the disease is increased by
the working conditions.
 Further, under Annex A of the Amended Rules,
o For an occupational disease and the resulting disability or death to be compensable, all of the
following conditions must be satisfied:
1. The employee's work must involve the risks described herein;
2. The disease was contracted as a result of the employee's exposure to the described
risks;
3. The disease was contracted within a period of exposure and under such other
factors necessary to contract it;
4. There was no notorious negligence on the part of the employee.
 Cardiovascular diseases are considered as occupational when contracted under any of the following
conditions:
(a) If the heart disease was known to have been present during employment there
must be proof that an acute exacerbation clearly precipitated by the unusual strain by reason of
the nature of his work.
(b) The strain of work that brings about an acute attack must be of sufficient severity
and must be followed within twenty-four (24) hours by the clinical signs of a cardiac insult to
constitute causal relationship.
(c) If a person who was apparently asymptomatic before subjecting himself to strain at
work showed signs and symptoms of cardiac injury during the performance of his work and such
symptoms and signs persisted, it is reasonable to claim a causal relationship
 As found by the Court of Appeals, petitioner failed to submit substantial evidence that might have
shown that he was entitled to the benefits he had applied for. We thus affirm in toto the findings and
conclusions of the Court of Appeals in the questioned Decision and quote with approval the following
pronouncements of the appellate court
 The degree of proof required under P.D. 626 is merely substantial evidence, which means such
relevant evidence as a reasonable mind might accept as adequate to support a conclusion.
 Accordingly, the claimant must show, at least by substantial evidence, that the development of the
disease was brought about largely by the conditions present in the nature of the job. What the law
requires is a reasonable work connection, not a direct causal relation.
 Gatus was diagnosed to have suffered from CAD; Triple Vessel and Unstable Angina, diseases or
conditions falling under the category of Cardiovascular Diseases which are not considered
occupational diseases under the Amended Rules on Employees Compensation.
 His disease not being listed as an occupational disease, he was expected to show that the illness or the
fatal disease was caused by his employment and the risk of contracting the disease was increased or
aggravated by the working conditions.
 His proof would constitute a reasonable basis for arriving at a conclusion that the conditions of his
employment had caused the disease or that such working conditions had aggravated the risk of
contracting the illness or the fatal disease.
 Under ECC Resolution No. 432 dated July 20, 1977, cardiovascular disease is deemed
compensable under any of the following conditions, viz:
(a) If the heart disease was known to have been present during employment, there must be
proof that an acute exacerbation was clearly precipitated by the unusual strain by reasons of the
nature of his work.
(b) The strain of work that brings about an acute attack must be of sufficient severity and
must be followed within 28 hours of the clinical signs of cardiac insult to constitute causal
relationship.
xxxx
Angeles| Bajana | Balladares | Brillantes | Briones | Cabansag | Callanta | Chua | David|

De Leon | Gomez | Lopez | Macalino | Nostratis | Padilla | Reynon | Santos | Tan |Velasco
4E / 4F - 2018-2019
Page 205 of 920
LABOR REVIEW DIGEST
Atty. Joyrich Golangco

 Gatus did not discharge the burden of proof imposed under the Labor Code to show that his ailment
was work-related. While he might have been exposed to various smoke emissions at work for 30 years,
he did not submit satisfactory evidence proving that the exposure had contributed to the
development of his disease or had increased the risk of contracting the illness.
 Neither did he show that the disease had progressed due to conditions in his job as a factory
worker. In fact, he did not present any physicians report in order to substantiate his allegation that
the working conditions had increased the risk of acquiring the cardiovascular disease.
 Verily, his mere contention of exposure to various smoke emissions in the working environment for a
period of time does not ipso facto make the resulting disability compensable.
 Awards of compensation cannot rest on speculations or presumptions, for the claimant must prove
a positive proposition.
 As pronounced in Sante v. Employees Compensation Commission:
x x x What kind and quantum of evidence would constitute an adequate basis for a
reasonable man (not necessarily a medical scientist) to reach one or the other conclusion, can
obviously be determined only on a case-to-case basis. That evidence must, however, be real
and substantial, and not merely apparent; for the duty to prove work-causation or work-
aggravation imposed by existing law is real not merely apparent
Moreover, he failed to show the presence of any of the conditions imposed for cardio-
vascular diseases by Sec. 18. Hence, the affirmance of the SSS decision was properly made.
The petitioners plight might call for sympathy, particularly in the light of his 30 years of
service to the company, but his petition cannot be granted on that basis alone. The policy of
extending the applicability of P.D. 626 as many qualified employees as possible should be
balanced by the equally vital interest of denying undeserving claims for compensation.
 In fine, Gatus was not qualified for the disability benefits under the employees compensation law.
 The questioned Decision deemed as established fact that petitioner is a cigarette smoker; but petitioner
vehemently denies this, saying there is no competent evidence to prove he had that habit.
 What petitioner would like this Court to do is to pass upon a question of fact, which the ECC, the SSS,
and the Court of Appeals have used to deny his claim for compensation.
 This is not allowed under Section 1 of Rule 45, which states that "[t]he petition shall raise only
questions of law which must be distinctly set forth."
 Hence, questions of fact may not be taken up in a petition for review on certiorari such as this case now
before us.
 The matter of petitioners cigarette smoking, established by two competent government agencies and the
appellate court, is thus a matter that cannot be questioned before us via petition for review.
 PETITION DENIED.

Angeles| Bajana | Balladares | Brillantes | Briones | Cabansag | Callanta | Chua | David|

De Leon | Gomez | Lopez | Macalino | Nostratis | Padilla | Reynon | Santos | Tan |Velasco
4E / 4F - 2018-2019
Page 206 of 920
LABOR REVIEW DIGEST
Atty. Joyrich Golangco

7. Republic of the Philippines represented by the Employee’s Compensation Commission v. Pedro


Mariano
G.R. No. 139455; Mar. 28, 2003

DOCTRINE: What kind and quantum of evidence would constitute an adequate basis for a reasonable man (not
necessarily a medical scientist) to reach one or the other conclusion, can obviously be determined only on a case-
to-case basis.

FACTS:

ECC’s Ruling:

Pedro Mariano was an employee of the LGP Printing Press whose service was abruptly ended when he could no
longer perform any work due to a heart ailment. He filed a claim for employee's compensation benefit with the
Social Security System (SSS) for his hypertension and "Incomplete Right Bundle Branch Block.", but the latter
denied the same on the ground that there was no causal connection between his ailment and the various jobs he
performed, including that of a machine operator, paper cutter, monotype composer, film developer, and supervisor
of the printing press. It was also at this point that it was revealed that Mariano was suffering from Parkinson’s
disease. This was corroborated by the findings of Dr. Rogelio Mariano. The Executive Director of the ECC denied
Mariano’s claim finding that there was no causal relation between his occupation and his ailments.

CA’s Ruling: The CA reversed the ruling of the ECC. The Court of Appeals found that the nature of petitioner's
work at LGP resulted in his exposure to various toxic chemicals, which is a possible cause of Parkinson's
Disease. As to his hypertension, the appellate court ruled that the respondent's duties as machine operator and
paper cutter involved physical pressure and restlessness, since he was required to meet urgent deadlines for rush
print orders. This in turn caused respondent to suffer from stress and anxiety. In sum, the appellate court held that
respondent had substantially established the connection between the cause of his ailments and the nature of his
work.

Appeal to the SC:

Petitioner’s Contention: There is no causal connection between Mariano’s ailments and his occupation.

Respondent’s Contention: That the causal connection had already been adequately proven.

ISSUE: Whether there is a causal connection between Mariano’s illness and his occupation.

RULING: YES. At the onset, Workmen’s Compensation claims are governed by the laws in force at the time the
claimant contacted the illness. In this case, the applicable rule is Section 1 (b), 13 Rule III, of the Rules
Implementing P.D. No. 626. Under said Rule, for the sickness to be compensable, the same must be an
"occupational disease" included in the list provided, with the conditions set therein satisfied; otherwise, the
claimant must show proof that the risk of contracting it is increased by the working conditions.

What kind and quantum of evidence would constitute an adequate basis for a reasonable man (not necessarily a
medical scientist) to reach one or the other conclusion, can obviously be determined only on a case-to-case basis.
For reasons herein elaborated, the Court agreed with the appellate court. The ECC itself admitted that the toxic
chemicals to which the claimant was exposed could have caused the Parkinson’s disease, and that the nature of
his work could have caused his ailments. Regardless of the rulings on both ailments, hypertension certainly was
a compensable disease.

Angeles| Bajana | Balladares | Brillantes | Briones | Cabansag | Callanta | Chua | David|

De Leon | Gomez | Lopez | Macalino | Nostratis | Padilla | Reynon | Santos | Tan |Velasco
4E / 4F - 2018-2019
Page 207 of 920
LABOR REVIEW DIGEST
Atty. Joyrich Golangco

Angeles| Bajana | Balladares | Brillantes | Briones | Cabansag | Callanta | Chua | David|

De Leon | Gomez | Lopez | Macalino | Nostratis | Padilla | Reynon | Santos | Tan |Velasco
4E / 4F - 2018-2019
Page 208 of 920
LABOR REVIEW DIGEST
Atty. Joyrich Golangco

8. Magsaysay Maritime Corporation vs Oberto Lobusta


GR No: 177578 Date: January 25, 2012
Ponente: Villarama Jr., J.

Doctrine:
Disability should not be understood more on its medical significance but on the loss of earning
capacity. Permanent total disability means disablement of an employee to earn wages in the same kind of
work, or work of similar nature that [he] was trained for or accustomed to perform, or any kind of work
which a person of [his] mentality and attainment could do. It does not mean absolute helplessness.

FACTS:

Petitioner Magsaysay Maritime Corporation is a domestic corporation and the local manning agent of the
vessel MV “Fossanger” and of petitioner Wastfel-Larsen Management A/S.
Respondent Oberto S. Lobusta is a seaman who has worked for Magsaysay Maritime Corporation since
1994.4 In March 1998, he was hired again as Able Seaman by Magsaysay Maritime Corporation in behalf of its
principal Wastfel-Larsen Management A/S.
Lobusta boarded MV “Fossanger” on March 16, 1998.6 After two months, he complained of breathing
difficulty and back pain. On May 12, 1998, while the vessel was in Singapore, Lobusta was admitted at
Gleneagles Maritime Medical Center and was diagnosed to be suffering from severe acute bronchial asthma with
secondary infection and lumbosacral muscle strain. Dr. C K Lee certified that Lobusta was fit for discharge on May
21, 1998, for repatriation for further treatment.
Upon repatriation, Lobusta was referred to Metropolitan Hospital. On February 19, 1999, Dr. Lim reported
that Lobusta has been diagnosed to have a moderate obstructive pulmonary disease which tends to be a chronic
problem, such that Lobusta needs to be on medications indefinitely. Dr. Lim also stated that Lobusta has probably
reached his maximum medical care.
Petitioners “then faced the need for confirmation and grading by a second opinion” and “it took the
parties time to agree on a common doctor, until they agreed on Dr. Camilo Roa.”15 Dr. Roa’s clinical summary
states that Lobusta's latest follow-up check-up was on December 16, 1999; that Lobusta is not physically fit to
resume his normal work as a seaman due to the persistence of his symptoms; that his asthma will remain
chronically active and will be marked by intermittent exacerbations; and that he needs multiple controller
medications for his asthma.
As the parties failed to reach a settlement as to the amount to which Lobusta is entitled, Lobusta filed on
October 2, 2000, a complaint for disability/medical benefits against petitioners before the National Labor Relations
Commission (NLRC).
Sometime in October 2000, Magsaysay Maritime Corporation suggested that Lobusta be examined by
another company-designated doctor for an independent medical examination. The parties agreed on an
independent medical examination by Dr. Annette M. David, whose findings it was agreed upon, would be
considered final.

LA/RTC/NLRC RULING:
The Labor Arbiter ruled that Lobusta suffered illness during the term of his contract. Hence, petitioners
are liable to pay Lobusta his medical allowance for 120 days or a total of US$2,060. The Labor Arbiter held that
provisions of the Labor Code, as amended, on permanent total disability do not apply to overseas seafarers.
Hence, he awarded Lobusta US$20,154 instead of US$60,000, the maximum rate for permanent and total
disability under Section 30 and 30-A of the 1996 POEA Standard Employment Contract. The Labor Arbiter also
awarded attorney’s fees equivalent to 5% of the total award since Lobusta was assisted by counsel.

Lobusta appealed. The NLRC dismissed his appeal and affirmed the Labor Arbiter’s decision. The NLRC
ruled that Lobusta’s condition may only be considered permanent partial disability. While Dr. David suggested that
Angeles| Bajana | Balladares | Brillantes | Briones | Cabansag | Callanta | Chua | David|

De Leon | Gomez | Lopez | Macalino | Nostratis | Padilla | Reynon | Santos | Tan |Velasco
4E / 4F - 2018-2019
Page 209 of 920
LABOR REVIEW DIGEST
Atty. Joyrich Golangco

Lobusta’s prospects as seafarer may have been restricted by his bronchial asthma, Dr. David also stated that the
degree of impairment is mild. Said qualification puts Lobusta's medical condition outside the definition of total
permanent disability, said the NLRC.22 Later, the NLRC also denied Lobusta’s motion for reconsideration.

CA RULING:
Lobusta brought the case to the CA under Rule 65. The CA faulted the NLRC for “plucking only particular
phrases” from Dr. David’s report and said that the NLRC cannot wantonly disregard the full import of said report.
The CA ruled that Lobusta's disability brought about by his bronchial asthma is permanent and total as he had
been unable to work since May 14, 1998 up to the present or for more than 120 days, and because Dr. David
found him not fit to return to work as an able seaman.

APPEAL TO THE SC:

Petitioner's Contention:
Petitioners argue that the CA erred in applying the provisions of the Labor Code instead of the provisions
of the POEA contract in determining Lobusta’s disability, and in ruling that the mere lapse of 120 days entitles
Lobusta to total and permanent disability benefits. The CA allegedly erred also in holding them liable for attorney’s
fees, despite the absence of legal and factual bases.

ISSUE/S:
Whether or not the disability of Lobusta is one of temporary or permanent total disability

HELD:
Permanent total disability.
Petitioners are mistaken that it is only the POEA Standard Employment Contract that must be considered
in determining Lobusta's disability. In Palisoc v. Easways Marine, Inc.,25 we said that whether the Labor Code’s
provision on permanent total disability applies to seafarers is already a settled matter. In Palisoc, we cited the
earlier case of Remigio v. National Labor Relations Commission26 where we said (1) that the standard
employment contract for seafarers was formulated by the POEA pursuant to its mandate under Executive Order
No. 24727 “to secure the best terms and conditions of employment of Filipino contract workers and ensure
compliance therewith,” and “to promote and protect the well-being of Filipino workers overseas”; (2) that Section
29 of the 1996 POEA Standard Employment Contract itself provides that all rights and obligations of the parties to
the contract, including the annexes thereof, shall be governed by the laws of the Republic of the Philippines,
international conventions, treaties and covenants where the Philippines is a signatory; and (3) that even without
this provision, a contract of labor is so impressed with public interest that the Civil Code expressly subjects it to
the special laws on labor unions, collective bargaining, strikes and lockouts, closed shop, wages, working
conditions, hours of labor and similar subjects.

The Court affirmed the award of disability benefits to the seaman, citing ECC v. Sanico, GSIS v. CA, and
Bejerano v. ECC that “disability should not be understood more on its medical significance but on the loss of
earning capacity. Permanent total disability means disablement of an employee to earn wages in the same kind of
work, or work of similar nature that [he] was trained for or accustomed to perform, or any kind of work which a
person of [his] mentality and attainment could do. It does not mean absolute helplessness.” It likewise cited
Bejerano v. ECC, that in a disability compensation, it is not the injury which is compensated, but rather it is the
incapacity to work resulting in the impairment of one’s earning capacity.

Upon repatriation, Lobusta was first examined by the Pulmonologist and Orthopedic Surgeon on May 22,
1998. The maximum 240-day (8-month) medical-treatment period expired, but no declaration was made that
Angeles| Bajana | Balladares | Brillantes | Briones | Cabansag | Callanta | Chua | David|

De Leon | Gomez | Lopez | Macalino | Nostratis | Padilla | Reynon | Santos | Tan |Velasco
4E / 4F - 2018-2019
Page 210 of 920
LABOR REVIEW DIGEST
Atty. Joyrich Golangco

Lobusta is fit to work. Nor was there a declaration of the existence of Lobusta’s permanent disability. On February
16, 1999, Lobusta was still prescribed medications for his lumbosacral pain and was advised to return for
reevaluation. May 22, 1998 to February 16, 1999 is 264 days or 6 days short of 9 months.

On Lobusta’s other ailment, Dr. Roa’s clinical summary also shows that as of December 16, 1999,
Lobusta was still unfit to resume his normal work as a seaman due to the persistence of his symptoms. But
neither did Dr. Roa declare the existence of Lobusta’s permanent disability. Again, the maximum 240-day medical
treatment period had already expired. May 22, 1998 to December 16, 1999 is 19 months or 570 days. In Remigio,
unfitness to work for 11-13 months was considered permanent total disability. So it must be in this case. And Dr.
David’s much later report that Lobusta “ought not to be considered fit to return to work as an Able
Seaman” validates that his disability is permanent and total as provided under the POEA Standard
Employment Contract and the Labor Code, as amended.

In fact, the CA has found that Lobusta was not able to work again as a seaman and that his
disability is permanent “as he has been unable to work since 14 May 1998 to the present or for more than
120 days.” This period is more than eight years, counted until the CA decided the case in August 2006. On the
CA ruling that Lobusta’s disability is permanent since he was unable to work “for more than 120 days,” we have
clarified in Vergara that this “temporary total disability period may be extended up to a maximum of 240 days.”

Other Notes/ SC Pronouncements:

__________________________________________________________________________

Angeles| Bajana | Balladares | Brillantes | Briones | Cabansag | Callanta | Chua | David|

De Leon | Gomez | Lopez | Macalino | Nostratis | Padilla | Reynon | Santos | Tan |Velasco
4E / 4F - 2018-2019
Page 211 of 920
LABOR REVIEW DIGEST
Atty. Joyrich Golangco

9. Social Security Commission v Azote


GR No: 209741 Date: April 15, 2015
Ponente: Mendoza, J.

Doctrine:
The law in force at the time of Edgardo's death was RA No. 8282, the amendatory law of RA No. 1161 or the
"Social Security Law." It is a tax-exempt social security service designed to promote social justice and provide
meaningful protection to members and their beneficiaries against the hazards of disability, sickness, maternity, old
age, death, and other contingencies resulting in loss of income or financial burden.

Applying Section 8 (e) and (k) of R. A. No. 8282, it is clear that only the legal spouse of the deceased-member is
qualified to be the beneficiary of the latter's SS benefits.

FACTS:

Respondent Edna and Edgardo Azote, a member of the SSS, were married in civil rites. Edgardo submitted Form
E-4 twice to the SSS in which he designated Edna and their six children as beneficiaries.

Edgardo passed away. Shortly thereafter, Edna filed her claim for death benefits with the SSS as the wife of a
deceased-member. It appeared, however, from the records that Edgardo had earlier submitted another Form E-4
with a different set of beneficiaries, namely: Rosemarie Azote, as his spouse; and Elmer Azote, as dependent.
Consequently, Edna's claim was denied

Edna filed a petition with the Social Security Commission (SSC) to claim the death benefits, lump sum and
monthly pension of Edgardo. She insisted that she was the legitimate wife of Edgardo. In its answer,
the SSS averred that there was a conflicting information in the forms submitted by the deceased.

SSC Ruling:

The SSC dismissed Edna's petition for lack of merit. Citing Section 24 (c) of the SS Law, it explained that although
Edgardo filed the Form E-4 designating Edna and their six children as beneficiaries, he did not revoke the
designation of Rosemarie as his wife-beneficiary, and Rosemarie was still presumed to be his legal wife.

The SSC further wrote that the NSO records revealed that the marriage of Edgardo to Rosemarie was registered
earlier. Consequently, it opined that Edgardo's marriage to Edna was not valid as there was no showing that his
first marriage had been annulled or dissolved.

CA RULING:

The CA reversed and set aside the resolution of the SSC. It held that the SSC could not make a determination of
the validity or invalidity of the marriage of Edna to Edgardo considering that no contest came from either
Rosemarie or Elmer.

The CA explained that Edna had established her right to the benefits by substantial evidence, namely, her
marriage certificate and the baptismal certificates of her children. It ruled that the deliberate change of his wife-
beneficiary, was clearly his voluntary act manifesting his intention to revoke his former declaration. The form
designating Edna as his wife superseded his former declaration.

APPEAL TO THE SC:

Angeles| Bajana | Balladares | Brillantes | Briones | Cabansag | Callanta | Chua | David|

De Leon | Gomez | Lopez | Macalino | Nostratis | Padilla | Reynon | Santos | Tan |Velasco
4E / 4F - 2018-2019
Page 212 of 920
LABOR REVIEW DIGEST
Atty. Joyrich Golangco

Petition for review on certiorari under Rule 45 assailing the CA decision allowing respondent Edna to claim the
death benefits of her late husband.

SSC Contention:

The SSC argues that the findings of fact of the CA were not supported by the records. It submits that under
Section 5 of the SS Law, it is called upon to determine the rightful beneficiary in the performance of its quasi-
judicial function of adjudicating SS benefits. It cited a number of cases, where the SSC had passed upon the
validity of marriages for the purpose of determining who were entitled to SS benefits.

The SSC contends that Edna was not the legitimate spouse as the CA failed to consider the NSO certification
showing that Edgardo was previously married to Rosemarie. The death certificate of Rosemarie proved that she
was alive at the time Edna and Edgardo were married, and, therefore, there existed a legal impediment to his
second marriage, rendering it void.

The SSC claims that the right to designate a beneficiary is subject to the SS Law. The designation of a wife-
beneficiary merely creates a disputable presumption that they are legally married and may be overthrown by
evidence to the contrary. Edna's designation became invalid with the determination of the subsistence of a
previous marriage. The SSC posits that even though Edgardo revoked and superseded his earlier designation of
Rosemarie as beneficiary, his designation of Edna was still not valid considering that only a legitimate spouse
could qualify as a primary beneficiary.

ISSUE:
Whether Edna can claim the death benefits of Edgardo.

HELD:

NO. Section 8 (e) and (k) of RA No. 8282, the amendatory law of RA No. 1161 or the Social Security Law
expressly provides that only the legal spouse of the deceased-member is qualified to be the beneficiary of
the latter’s SS benefits.

In this case, there is a concrete proof that Edgardo contracted an earlier marriage with another individual as
evidenced by their marriage contract. Edgardo even acknowledged his married status when he filled out the 1982
Form E-4 designating Rosemarie as his spouse.

It is undisputed that the second marriage of Edgardo with Edna was celebrated at the time when the Family Code
was already in force. Using the parameters outlined in Article 41 of the Family Code, Edna, without doubt, failed to
adduce evidence to prove that the earlier marriage of Edgardo was either annulled or dissolved or whether there
was a declaration of Rosemarie's presumptive death before her marriage to Edgardo. What is apparent is that
Edna was the second wife of Edgardo. Considering that Edna was not able to show that she was the legal spouse
of a deceased-member, she would not qualify under the law to be the beneficiary of the death benefits of
Edgardo.

It is of no moment that the first wife, Rosemarie, did not participate or oppose Edna's claim. Rosemarie's non-
participation or her subsequent death did not cure or legitimize the status of Edna.

Leonen, Dissenting:

The latest Form E-4 (1994) submitted by the deceased to the Social Security System prior to his death designated
Edna as his wife-beneficiary. The 1994 Form E-4 should supersede the earlier one. As correctly ruled by the
Angeles| Bajana | Balladares | Brillantes | Briones | Cabansag | Callanta | Chua | David|

De Leon | Gomez | Lopez | Macalino | Nostratis | Padilla | Reynon | Santos | Tan |Velasco
4E / 4F - 2018-2019
Page 213 of 920
LABOR REVIEW DIGEST
Atty. Joyrich Golangco

Court of Appeals, the 1994 Form E-4 designating Edna as his wife manifested the deceased's intention to revoke
his formal declaration in the 1982 Form E-4.

This conclusion is consistent with Section 24 (c) of Republic Act No. 8282, which states that "records and reports
duly accomplished and submitted to the Social Security System by the employer or the member . . . [are]
presumed correct as to the data and other matters stated therein . . . [and will be] made the basis for the
adjudication of the claim" unless corrected before the right to the benefit being claimed accrued. There is nothing
in Republic Act No. 8282 expressly prohibiting the change of beneficiary. On the contrary, Section 24 (c), by
implication, acknowledges a member's right to change beneficiaries.

Edna established her right to the benefits through substantial evidence. She presented her marriage certificate
and the baptismal certificates of her children. Being public documents, these constitute prima facie proof of their
contents, and, therefore, her claim to death benefits as legal wife and dependent of Edgardo should have been
approved.

There was yet no attack on the validity of the deceased's marriage to Edna. No adjudicatory process was pending.
Certainly, the Social Security Commission was not invoked as the forum to test the validity of her marriage. The
validity of that marriage passed unchallenged. No right was asserted by the proper real party in interest under the
superceded forms submitted by the claimant. In these circumstances, the presumption in favor of the validity of
the second marriage must prevail, and sound reason requires that it be not lightly impugned and discredited by
the alleged prior marriage stated in the 1982 Form E-4.

Angeles| Bajana | Balladares | Brillantes | Briones | Cabansag | Callanta | Chua | David|

De Leon | Gomez | Lopez | Macalino | Nostratis | Padilla | Reynon | Santos | Tan |Velasco
4E / 4F - 2018-2019
Page 214 of 920
LABOR REVIEW DIGEST
Atty. Joyrich Golangco

10. Marlow Navigation Phil. v Heirs of Ganal


GR No: 220168 Date: June 7, 2017
Ponente: PERALTA, J.:

Doctrine: (main doctrine under the article)


The death of a seafarer by reason of any work-related injury or illness during the term of his employment
is compensable unless the injury or illness is caused by his willful or criminal act or intentional breach of
his duties.

FACTS:
Ricardo Ganal (Ganal) was employed by the petitioners as an oiler aboard a vessel. Around 7 o'clock in the
evening of April 15, 2012, a party was organized for the crewmen while the ship was anchored at Chittagong,
Bangladesh. At 12 midnight, Ganal joined the party. The ship captain noticed that Ganal was already drunk so he
directed him to return to his cabin and take a rest. Ganal ignored the ship captain's order. Thus, a ship officer, a
security watchman and a member of the crew were summoned to escort Ganal to his cabin. The crew members
attempted to accompany him back to his cabin but he refused. They then tried to restrain him but he resisted and,
when he found the chance to escape, he ran towards the ship's railings and, without hesitation, jumped overboard
and straight into the sea. Ganal was later found dead and floating in the water.
Subsequently, Ganal's wife, Gemma Boragay (Boragay), for herself and in behalf of their minor children, filed a
claim for death benefits with petitioners, but the latter denied the claim.
Thus, on October 29, 2012, Boragay, filed with the NLRC a complaint for recovery of death and other benefits,
unpaid salaries for the remaining period of Ganal's contract, as well as moral and exemplary damages.

LA/RTC/NLRC RULING:
LA dismissed the complaint for lack of merit. The LA held that respondents' allegations are self-serving and
hearsay; they failed to present evidence to substantiate their allegations; on the other hand, petitioners were able
to present documentary evidence, consisting of affidavits of Ganal's fellow crew members who have direct and
actual knowledge of what occurred on board and who attested to the fact that Ganal willfully jumped overboard.
Nonetheless, the LA ordered herein petitioners to pay respondents the amount of US$5,000.00 as financial
assistance.

Upon appeal, the NLRC issued a Resolution denying respondents' appeal and affirming the Decision of the LA.
The NLRC ruled that petitioners have duly proven that Ganal's death is not compensable as it was the result of
the deliberate and willful act of Ganal and, thus, is directly attributable to him.

CA RULING:
The CA reversed the decision of the NLRC and held that Ganal jumped into the sea while he was overcome by
alcohol and completely intoxicated and deprived of his consciousness and mental faculties to comprehend the
consequence of his own actions and keep in mind his own personal safety.

APPEAL TO THE SC:

Petitioner's Contention:
Petitioner contended that respondents are not entitled to death and other benefits, as well as damages, they are
claiming by reason of the demise of their predecessor-in-interest during the effectivity of his contract of
employment, because his death is directly attributable to him and was a result of his willful act.

Respondent's Contention:
Respondent argued that Ganal was no longer in control of his actions because of excessive alcohol intake.

Angeles| Bajana | Balladares | Brillantes | Briones | Cabansag | Callanta | Chua | David|

De Leon | Gomez | Lopez | Macalino | Nostratis | Padilla | Reynon | Santos | Tan |Velasco
4E / 4F - 2018-2019
Page 215 of 920
LABOR REVIEW DIGEST
Atty. Joyrich Golangco

ISSUE/S:
1. Whether or not the act of Ganal was willful?
2. Whether or not the heirs of Ganal are entitled to death benefits under the POEA-Standard Employment contract

HELD:
1. Yes, Ganal's act of intentionally jumping overboard, while in a state of intoxication, could be considered as a
deliberate and willful act on his own life which is directly attributable to him. There was no competent proof to
show that Ganal's state of intoxication during the said incident actually deprived him of his consciousness and
mental faculties which would have enabled him to comprehend the consequences of his actions and keep in mind
his personal safety.

2.. No, the heirs of Ganal are not entitled to death benefits under the POEA-Standard Employment contract on the
ground that the death did not arise from any work-related injury or illness during the term of his employment.
Section 20(D) of the same Standard Terms and Conditions states that: no compensation and benefits shall be
payable in respect of any injury, incapacity, disability or death of the seafarer resulting from his willful or criminal
act or intentional breach of his duties, provided however, that the employer can prove that such injury, incapacity,
disability or death is directly attributable to the seafarer.
In the present case, it may be conceded that the death of Ganal took place in the course of his employment, in
that it happened at the time and at the place where he was working. However, the accident which produced this
tragic result did not arise out of such employment.

Other Notes/ SC Pronouncements:


The words "arising out of' refer to the origin or cause of the accident and are descriptive of its character, while the
words "in the course of' refer to the time, place, and circumstances under which the accident takes place. By the
use of these words, it was not the intention of the legislature to make the employer an insurer against all
accidental injuries which might happen to an employee while in the course of the employment, but only for such
injuries arising from or growing out of the risks peculiar to the nature of work in the scope of the workmen's
employment or incidental to such
employment, and accidents in which it is possible to trace the injury to some risk or hazard to which the employee
is exposed in a special degree by reason of such employment.

__________________________________________________________________________

Angeles| Bajana | Balladares | Brillantes | Briones | Cabansag | Callanta | Chua | David|

De Leon | Gomez | Lopez | Macalino | Nostratis | Padilla | Reynon | Santos | Tan |Velasco
4E / 4F - 2018-2019
Page 216 of 920
LABOR REVIEW DIGEST
Atty. Joyrich Golangco

11. SEAPOWER SHIPPING ENT. INC., vs. HEIRS OF WARREN M. SABANAL, represented by ELVIRA ONG-
SABANAL
GR No: 198544 Date: June 19, 2017
Ponente: JARDELEZA, J.:

DOCTRINE:
The POEA standard employment contract for Filipino seafarers exempts the employer from liability for death or
injury resulting from the seafarer's willful act BUT evidence of insanity or mental sickness (the seafarer suffered
from complete deprivation of intelligence in committing the act or complete absence of the power to discern the
consequences of his action) may be presented by the heirs as a counter-defense.

FACTS:
Petitioner Sea Power hired the deceased Warren Sabanal as 3rd Mate. However, during voyage, Sabanal
started exhibiting unusual behavior as it appeared that he had problems with his brother in the Philippines. Per
captain’s orders, double guards were assigned to Sabanal, they reported that the latter wanted to board a life
boat, citing danger in the ship's prow. Later, the captain observed that Sabanal's condition was "rather better" and
he "did not appear to have any problems while performing simple assigned tasks (correcting maps and collecting
and typing the crew's declarations)." Hence, Sabanal was allowed to return to the deck for some fresh air but once
on deck, he suddenly ran to the stem and jumped to the sea. His body was never recovered despite the rescue
attempts.

The widow Elvira alleged that Seapower told her to wait for a period of 7 to 10 years before Sabanal can be
declared dead. Relying on this, Elvira went back to in late 2004 or early 2005 to claim whatever benefits she was
entitled to. Initially, Seapower informed her of entitlement to death benefits under the Social Security System, later
however, Seapower categorically denied any liability for Sabanal's death. Elvira filed a complaint for payment of
Sabanal' s death benefits with the Labor Arbiter.

LA RULING: dismissed Elvira's case on the grounds of prescription and lack of merit, and ruled that Elvira failed
to substantiate her claim that Seapower misled her to wait. Based on Seapower’s pieces of evidence, Sabanal
took his own life, hence, his death is not compensable.

NLRC RULING: affirmed LA, but ruled that the action had not prescribed because the prescriptive period only
began to run upon Seapower's categorical denial of Elvira's claim. Sabanal's suicide was established by
substantial evidence, hence, when the death of the seaman resulted from his own willful act, the death is
not compensable.

CA RULING: partly reversed the NLRC. In relying on Sabanal's strange conduct prior to jumping off ship, CA
concluded that "his actions were borne not by his willful disregard of his safety and of his life, but, on the contrary,
he became paranoid that the ship was in grave danger, that he wanted to save himself from the imagined
doom that was to befall the ship." Hence, Seapower must pay death benefits to Elvira.

ISSUE/S:
Whether Elvira was able to prove by substantial evidence Sabanal’s unusual behavior (that he was insane) prior
to the incident, so as to make his strange death is compensable. NO.

HELD:
The general rule under the 1989 POEA "Revised Standard Employment Contract Governing the Employment of
All Filipino Seamen On-Board Ocean-Going Vessels" (POEA-SEC) is that the employer is liable for death
compensation benefits when a seafarer dies during the term of employment. The exception under Part 2,
Section C (6) of the POEA-SEC – “exempts the employer from liability if it can successfully prove that the
Angeles| Bajana | Balladares | Brillantes | Briones | Cabansag | Callanta | Chua | David|

De Leon | Gomez | Lopez | Macalino | Nostratis | Padilla | Reynon | Santos | Tan |Velasco
4E / 4F - 2018-2019
Page 217 of 920
LABOR REVIEW DIGEST
Atty. Joyrich Golangco

seafarer's death was caused by an injury directly attributable to his deliberate or willful act.”

All the lower courts agreed that the Seapower’s evidence (ship log entries and master's report) sufficiently
established that Sabanal indeed jumped into the sea. However, evidence of insanity or mental sickness may
be presented by the heirs to negate the requirement of willfulness as a matter of counter-defense. But the
burden of evidence is then shifted to the claimant (Elvira) to prove that the seafarer was of unsound mind.

In Agile Maritime Resources, Inc. v. Siador, SC held that the insanity or mental illness required to be proven
must be one that deprived him of the full control of his senses; the seafarer's strange behavior alone is
insufficient to prove his insanity. In Crew/ink, Inc. v. Teringtering, SC held that the counter-defense of a
psychotic disorder or Mood Disorder Bipolar Type is unmeritorious when claimant presented no
evidence, witness, or any medical report to support the claim of insanity. SC added that homesickness
and/or family problems may result to depression, but the same does not necessarily equate to mental disorder.
Establishing the insanity of [a deceased seafarer] requires opinion testimony which may be given by a
witness who is intimately acquainted with the person claimed to be insane, or who has rational basis to
conclude that a person was insane based on the witness' own perception of the person, or who is
qualified as an expert, such as a psychiatrist.

APPLICATION - Elvira did not present any evidence to support her claim as she only relied on the strange
behavior detailed in the ship log and master's report. In order for insanity to prosper as a counter-defense, the
claimant must substantially prove that the seafarer suffered from complete deprivation of intelligence in
committing the act or complete absence of the power to discern the consequences of his action. Mere
abnormality of the mental faculties does not foreclose willfulness. The ship log and master's report
disproved insanity as it shows that Sabanal was still able to correct maps and type the declarations of the crew
hours before he jumped overboard; on captain’s observation, he did not appear to have any problems while
performing these simple tasks; on sailor-on-guard’s report, he did not show any signs of unrest immediately
before the incident. These circumstances, coupled with the legal presumption of sanity, tend to belie
Elvira's claim that Sabanal no longer exercised any control over his own senses and mental faculties.
Also, no liability can attach to Seapower in the absence of negligence and nonchalant attitude towards its
deceased seafarer. The records show that as soon as the ship captain became aware of Sabanal's unusual
behavior, he immediately assigned other sailors to watch him but they were unable to stop him from jumping
overboard because of the latter's brisk movement. They immediately undertook rescue maneuvers but their
diligent efforts proved futile.

Angeles| Bajana | Balladares | Brillantes | Briones | Cabansag | Callanta | Chua | David|

De Leon | Gomez | Lopez | Macalino | Nostratis | Padilla | Reynon | Santos | Tan |Velasco
4E / 4F - 2018-2019
Page 218 of 920
LABOR REVIEW DIGEST
Atty. Joyrich Golangco

12. GOVERNMENT SERVICE INSURANCE SYSTEM (GSIS), Petitioner vs. APOLINARIO C. PAUIG,
Respondent

GR No: 210328 Date: January 30, 2017

Ponente: PERALTA, J

Doctrine:
Compulsory coverage under the GSIS had previously and consistently included regular and permanent
employees, and expressly excluded casual, substitute or temporary employees from its retirement insurance plan.
Therefore, Pauig's casual and temporary service in the government from February 12, 1964 to July 18, 1977 must
necessarily be excluded from the creditable period of service for retirement purposes.

FACTS:
 Respondent Apolinario C. Pauig (Pauig) was the Municipal Agriculturist of the Municipality of San Pablo,
lsabela.
 He started in the government service on February 12, 1964 as Emergency Laborer on casual status.
Later, he became a temporary employee from July 5, 1972 to July 18, 1977.
 On July 19, 1977, he became a permanent employee, and on August 1, 1977, he became a GSIS
member, as indicated in his Information for Membership.
 Thereafter, on November 3, 2004, he retired from the service upon reaching the mandatory retirement
age of sixty-five (65) years old.
 But when he filed his retirement papers with the GSIS-Cauayan, the latter processed his claim based on
a Record of Creditable Service (RCS) and a Total Length of Service of only twenty-seven (27) years.
 Disagreeing with the computation, Pauig wrote a letter-complaint to the GSIS, arguing that his first
fourteen (14) years in the government service had been erroneously omitted.
 The GSIS ratiocinated that Pauig's first fourteen (14) years in the government were excluded in the
computation of his retirement benefits because during those years, no premium payments were remitted to it.
 Under the Premium-Based Policy of the GSIS which took effect on August 1, 2003, only periods of
service where premium payments were made and duly remitted to the System shall be included in the
computation of retirement benefits.
 Aggrieved, Pauig filed a case before the RTC of Cabagan, Isabela.

RTC: Directing the GSIS to credit under Policy and Procedural Guidelines No. 171-03 the casual/temporary
service from February 10, 1964 to July 18, 1977 in government of the plaintiff Apolinario C. Pauig as creditable
service for retirement purposes upon payment of the premium contributions and interest thereon in accordance
with the provisions thereof.
Respondent's Contention: Pauig insists that retirement laws must be liberally construed in favor of the retirees
because the intention is to provide for their sustenance, and hopefully even comfort, when they no longer have the
stamina to continue earning their livelihood.
After devoting the best years of his life to public service, Pauig asserts that he deserves the appreciation of a
grateful government as best concretely expressed in a generous retirement gratuity commensurate with the value
and length of his services.
That generosity, he argues, is the least he should expect now that his work is done and his youth is gone. Even as
he feels the weariness in his bones and glimpses the approach of the lengthening shadows, he should be able to
savor the fruits of his toil.

ISSUE:

Angeles| Bajana | Balladares | Brillantes | Briones | Cabansag | Callanta | Chua | David|

De Leon | Gomez | Lopez | Macalino | Nostratis | Padilla | Reynon | Santos | Tan |Velasco
4E / 4F - 2018-2019
Page 219 of 920
LABOR REVIEW DIGEST
Atty. Joyrich Golangco

Whether or not the GSIS should include Pauig's first fourteen (14) years in government service for the calculation
of the latter's retirement benefits claim.

HELD:

 No. The Court ruled that the doctrine of liberal construction cannot be applied in this case, where the
law invoked is clear, unequivocal and leaves no room for interpretation or construction. To uphold
Pauig's position will contravene the very words of the law, and will defeat the ends which it seeks to
attain.
 Compulsory coverage under the GSIS had previously and consistently included regular and permanent
employees, and expressly excluded casual, substitute or temporary employees from its retirement
insurance plan.
 A permanent appointment is one issued to a person who has met the requirements of the position to
which appointment is made, in accordance with the provisions of the Civil Service Act and the Rules and
Standards, while temporary appointment is made in the absence of appropriate eligibles and it becomes
necessary in the public interest to fill a vacancy.
 Casual employment, on the other hand, is not permanent but occasional, unpredictable, sporadic and
brief in nature. 11
 Based on the records, Pauig began his career in the government on February 12, 1964 as Emergency
Laborer on a casual status. Then, he became a temporary employee from July 5, 1972 to July 18, 1977.
However, the Court notes that it was not until 1997 that the compulsory membership in the GSIS was
extended to employees other than those on permanent status.
 Pauig cited the case of GSIS v. CSC, 13 where the Court ruled that the basis for the provision of
retirement benefits is service to the government.
 Unfortunately, Pauig's reliance on the aforecited case is misplaced.
 True, in GS/S v. CSC, the Court allowed the claimants to avail of their retirement benefits although no
deductions were made from their salaries during the disputed periods when they were paid on a per diem
basis.
 However, unlike in the case at bar, deductions were actually made from claimant's fixed salary before
and after the short controversial period. She assumed in all good faith that she continued to be covered
by the GSIS insurance benefits considering that, in fact and in practice, the deductions are virtually
mandatorily made from all government employees on an essentially involuntary basis.
 More importantly, neither of the claimants in this case of GSIS v. CSC was a casual or temporary
employee like Pauig, both of them being elective officials. 14
 Here, the primordial reason why there were no deductions during those fourteen (14) years was because
Pauig was not yet a GSIS member at that time. There was thus no legal obligation to pay the premium as
no basis for the remittance of the same existed.
 And since only periods of service where premium payments were actually made and duly remitted to the
GSIS shall be included in the computation of retirement benefits, said disputed period of fourteen (14)
years must corollarily be removed from Pauig's creditable service.
 Therefore, Pauig's casual and temporary service in the government from February 12, 1964 to July 18,
1977 must necessarily be excluded from the creditable period of service for retirement purposes.

__________________________________________________________________________

Angeles| Bajana | Balladares | Brillantes | Briones | Cabansag | Callanta | Chua | David|

De Leon | Gomez | Lopez | Macalino | Nostratis | Padilla | Reynon | Santos | Tan |Velasco
4E / 4F - 2018-2019
Page 220 of 920
LABOR REVIEW DIGEST
Atty. Joyrich Golangco

13. C.F. SHARP CREW MANAGEMENT vs. RHUDEL CASTILLO


GR No: 208215 Date: April 19, 2017
Ponente: Peralta, J.

Doctrine: the fit to work declaration in the seafarer's PEME cannot be a conclusive proof to show that he was free
from any ailment prior to his deployment

while the law recognizes that an illness may be disputably presumed to be work-related, the seafarer or the
claimant must still show a reasonable connection between the nature of work on board the vessel and the illness
contracted or aggravated

FACTS:

Respondent was hired by C.F. Sharp Crew Management on behalf of its foreign principal as a security guard on
board the vessel MV Norwegian Sun. Prior to his deployment, respondent underwent a Pre-employment Medical
Examination (PEME) and was pronounced fit to work. However, while on board the vessel he suffered from
difficulty of breathing and had a brief seizure attack causing him to fall from his bed. He was immediately treated
by the ship doctor. Respondent was repatriated and was referred to the company-designated physicians at
Comprehensive Marine Medical Services for further treatment, evaluation and management.

A Medical Progress Report was issued by Dr. Ong-Salvador stating that respondent is suffering from "right
parietal cavernoma" and the condition is deemed to be idiopathic, thus, it is not work-related. On the otherhand,
Dr. Pobre issued a Certification indicating that respondent is suffering from Cavernoma and the illness is a
congenital disorder and not work-related.

Respondent filed a Complaint for permanent and total disability benefits, damages and attorney's fees.
Respondent alleged that he is entitled to a maximum disability compensation of US$120,000.00 under the
Norwegian CBA. Respondent further alleged that even after all the examinations, he is still suffering from the
illnesses and is disabled up to the present.

LA / NLRC RULING:
The LA dismissed the complaint. The LA opined that while the illness of respondent is disputably presumed to be
work-related, petitioners have substantially disputed the presumption of work-connection with the submission of a
certification from the company physicians categorically stating that respondent's illness is idiopathic and
congenital in etiology, and as such, could not have been caused by working conditions aboard the vessel. Also,
the LA noted that no copy of the alleged Norwegian CBA was shown by respondent.

The LA stated that respondent did not submit any evidence or certification that his illness is work-related or work-
aggravated. The LA ratiocinated that the fact that the illness may have manifested during the period of
respondent's contract is inadequate to justify the grant of disability compensation. The POEA- SEC mandates that
the causal connection between the illness and nature of work performed should also be proven.
A motion for reconsideration was filed by respondent, but the same was denied by the NLRC.CA RULING:
Aggrieved, respondent filed a petition for certiorari before the CA. The CA reversed the Decision of the NLRC and
held that petitioners have not overcome the disputable presumption of work-relatedness of the disease due to the
conflicting statements of the petitioners' physicians as to the cause of respondent's illness.

Petitioners filed MR but it was denied by the CA.

APPEAL TO THE SC:

Angeles| Bajana | Balladares | Brillantes | Briones | Cabansag | Callanta | Chua | David|

De Leon | Gomez | Lopez | Macalino | Nostratis | Padilla | Reynon | Santos | Tan |Velasco
4E / 4F - 2018-2019
Page 221 of 920
LABOR REVIEW DIGEST
Atty. Joyrich Golangco

Petitioner's Contention:
Petitioners argued that in order to overturn the opinion and findings of the company-designated physician, the
opinion of respondent's physician must be supported by a third doctor's opinion without which, the company-
designated physician's opinion shall prevail. and that the burden to prove that an illness is work-related belongs to
respondent. And considering that the illness is not work-related, the same is not compensable whether or not
respondent is not able to work for more than 120 days.

Respondent's Contention:
Stated that he was presumed fit at the time he entered into a contract with the petitioners as revealed by the
results of the PEME. He argued that he is entitled to total permanent disability benefits because he was found and
declared as unfit to work by his private physician and that there is a disputable presumption that his illness is
work-related. He also argued that he is considered total and permanently disabled as he was unable to work for
more than 120 days.

ISSUE/S:
1. Ihether or not respondent is entitled to total and permanent disability benefits.

HELD:
1. No. Entitlement of seamen on overseas work to disability benefits is a matter governed, not only by medical
findings, but by law and by contract. Considering that respondent was hired in 2008, the 2000 POEA-SEC
applies.

The illness of respondent, cavernoma, is not included in the list of occupational diseases under Section 32-A of
the POEA-SEC. However, Section 20(B) (4) of the contract provides that those illnesses not listed in Section 32
are disputably presumed as work-related.

In the case at bar, petitioners' physician, Dr. Pobre, declared that the illness of respondent which is cavemova is
not work-related as the same is congenital in nature, while petitioners' other physician Dr. Salvador-Ong declared
the same as idiopathic in its causation and, thus, not work-related.

Petitioners' physicians differ in their view on the causation of respondent's illness, but both are one in declaring
that the illness is not work-related, as opposed to the statement of respondent's physician Dr. Vicaldo that the
illness is work-related.

The conflicting findings of the company's doctor and the seafarer's physician often stir suits for disability
compensation. As an extrajudicial measure of settling their differences, the POEA-SEC gives the parties the
option of agreeing jointly on a third doctor whose assessment shall break the impasse and shall be the final and
binding diagnosis. The POEA-SEC provides for a procedure to resolve the conflicting findings of a company
designated physician and personal physician. If a doctor appointed by the seafarer disagrees with the
assessment, a third doctor may be agreed jointly between the Employer and the seafarer. The third doctor's
decision shall be final and binding on both parties.

The medical certificate issued by Dr. Vicaldo was not based on results from medical tests and procedures. While
Dr. Ong-Salvador and Dr. Pobre are familiar with respondent's medical history and condition, thus, their medical
opinion on whether respondent's illness is work-aggravated/-related deserve more credence as opposed to Dr.
Vicaldo's unsupported conclusions.

Furthermore, while the law recognizes that an illness may be disputably presumed to be work-related, the
seafarer or the claimant must still show a reasonable connection between the nature of work on board the vessel
and the illness contracted or aggravated.
Angeles| Bajana | Balladares | Brillantes | Briones | Cabansag | Callanta | Chua | David|

De Leon | Gomez | Lopez | Macalino | Nostratis | Padilla | Reynon | Santos | Tan |Velasco
4E / 4F - 2018-2019
Page 222 of 920
LABOR REVIEW DIGEST
Atty. Joyrich Golangco

Here, assuming that cavernoma is not idiopathic, respondent did not adduce proof to show a reasonable
connection between his work as Security Guard and his cavernoma. There was no showing how the demands
and nature of his job vis-a-vis the ship's working conditions increased the risk of contracting cavernoma. It must
be stressed that respondent was hired by petitioners on a 10-month contract on June 6, 2008. While on board the
vessel, he suffered from difficulty of breathing and other symptoms of his current illness. When respondent got
sick, he was on board only for 3 months. Because of this short span of time, then the presentation of evidence
showing the relation between respondent's work as Security Guard and his illness becomes all the more crucial.

Since respondent's illness is not work-related, this Court need not labor on respondent's argument that his illness
must be deemed total and permanent since he was unable to work for more than 120 days. Such should be read
in relation to the POEA-SEC which, among others, provide that an illness should be work-related.

Let it be stressed that the seafarer's inability to resume his work after the lapse of more than 120 days from the
time he suffered an injury and/or illness is not a magic wand that automatically warrants the grant of total and
permanent disability benefits in his favor. Both law and evidence must be on his side.

Moreover, respondent argued that he was presumed fit at the time he entered into a contract with the petitioners
as revealed by the results of the PEME. The fact that respondent passed the company's PEME is of no moment.
We have ruled that in the past the PEME is not exploratory in nature. It was not intended to be a totally in-depth
and thorough examination of an applicant's medical condition. The PEME merely determines whether one is fit to
work at sea or fit for sea service; it does not state the real state of health of an applicant. In short, the fit to work
declaration in the seafarer's PEME cannot be a conclusive proof to show that he was free from any ailment prior
to his deployment.

The respondent is not entitled to total and permanent disability benefits for his failure to refute the company
designated physicians' findings that his illness was not work-related. The CA, thus, erred in finding grave abuse of
discretion on the part of the NLRC when the latter affirmed the LA's Decision not to grant permanent and total
disability benefits to the respondent despite insufficient evidence to justify this grant. We note that petitioners
shouldered all the expenses in connection with respondent's medical treatment, and respondent was, likewise,
paid his sickness wages.

Other Notes/ SC Pronouncements:

__________________________________________________________________________

Angeles| Bajana | Balladares | Brillantes | Briones | Cabansag | Callanta | Chua | David|

De Leon | Gomez | Lopez | Macalino | Nostratis | Padilla | Reynon | Santos | Tan |Velasco
4E / 4F - 2018-2019
Page 223 of 920
LABOR REVIEW DIGEST
Atty. Joyrich Golangco

14. SSS v Favila

GR No.: 170195 Date: March 28, 2011

Ponente: Del Castillo, J.

DOCTRINE:

For a spouse to qualify as a primary beneficiary, he/she must not only be a legitimate spouse but also dependent
upon the member for support.

FACTS:

Respondent Teresita Favila filed a claim with the SSS for pension benefits, and averred therein that after she was
married to Florante Favila (Florante) on January 17, 1970, the latter designated her as the sole beneficiary in the
E-1 Form he submitted before petitioner Social Security System (SSS), Quezon City Branch on June 30,
1970.When they begot their children Jofel, Floresa and Florante II, her husband likewise designated each one of
them as beneficiaries.Teresa further averred that when Florante died on February 1, 1997, his pension benefits
under the SSS were given to their only minor child at that time, Florante II, but only until his emancipation at age
21.Believing that as the surviving legal wife she is likewise entitled to receive Florantes pension benefits, Teresa
subsequently filed her claim for said benefits before the SSS.The SSS, however, denied the claim.

SSC Ruling: The surviving spouses entitlement to an SSS members death benefits is dependent on two factors
which must concur at the time of the latters death, to wit:(1) legality of the marital relationship; and (2) dependency
for support.As to dependency for support, the SSC opined that same is affected by factors such as separation de
facto of the spouses, marital infidelity and such other grounds sufficient to disinherit a spouse under the law.Thus,
although Teresa is the legal spouse and one of Florantes designated beneficiaries, the SSC ruled that she is
disqualified from claiming the death benefits because she was deemed not dependent for support from Florante
due to marital infidelity.

CA Ruling: reversed the SSCs decision.

ISSUE/S:

Whether Teresita is a primary beneficiary in contemplation of the Social Security Law, to be entitled to death
benefits accruing from the death of Florante.

Angeles| Bajana | Balladares | Brillantes | Briones | Cabansag | Callanta | Chua | David|

De Leon | Gomez | Lopez | Macalino | Nostratis | Padilla | Reynon | Santos | Tan |Velasco
4E / 4F - 2018-2019
Page 224 of 920
LABOR REVIEW DIGEST
Atty. Joyrich Golangco

HELD:

Under Sec. 8(e) and (k) of RA 1161, for a spouse to qualify as a primary beneficiary under paragraph (k) thereof,
he/she must not only be a legitimate spouse but also a dependent as defined under paragraph (e), that is, one
who is dependent upon the member for support. There is no question that Teresa was Florantes legal wife. What
is at point, however, is whether Teresa is dependent upon Florante for support in order for her to fall under the
term "dependent spouse" under Section 8(k) of RA 1161.

In Re: Application for Survivors Benefits of Manlavi, the Court defined "dependent" as "one who derives his or her
main support from another or relying on, or subject to, someone else for support; not able to exist or sustain
oneself, or to perform anything without the will, power or aid of someone else.

In SSS v. Aguas, "the obvious conclusion is that a wife who is already separated de facto from her husband
cannot be said to be dependent for support upon the husband, absent any showing to the contrary. Conversely, if
it is proved that the husband and wife were still living together at the time of his death, it would be safe to presume
that she was dependent on the husband for support, unless it is shown that she is capable of providing for
herself."

In this case, aside from Teresita’s bare allegation that she was dependent upon her husband for support and her
misplaced reliance on the presumption of dependency by reason of her valid and then subsisting marriage with
Florante, Teresita has not presented sufficient evidence to discharge her burden of proving that she was
dependent upon her husband for support at the time of his death. She could have done this by submitting
affidavits of reputable and disinterested persons who have knowledge that during her separation with Florante,
she does not have a known trade, business, profession or lawful occupation from which she derives income
sufficient for her support and such other evidence tending to prove her claim of dependency.

On the contrary, what is clear is that she and Florante had already been separated for about 17 years prior to the
latters death as Florante was in fact, living with his common law wife when he died. Whoever claims entitlement to
the benefits provided by law should establish his or her right thereto by substantial evidence. Hence, for Teresita’s
failure to show that despite their separation, she was dependent upon Florante for support at the time of his death,
Teresa cannot qualify as a primary beneficiary. Hence, she is not entitled to the death benefits accruing on
account of Florante’s death.

Angeles| Bajana | Balladares | Brillantes | Briones | Cabansag | Callanta | Chua | David|

De Leon | Gomez | Lopez | Macalino | Nostratis | Padilla | Reynon | Santos | Tan |Velasco
4E / 4F - 2018-2019
Page 225 of 920
LABOR REVIEW DIGEST
Atty. Joyrich Golangco

15. MENDOZA vs. PEOPLE OF THE PHILIPPINES


G.R. No. 183891; October 19, 2011
PONENTE: Brion, J.

DOCTRINE: RA No. 9903 grants condonation only to employers with delinquent contributions or pending cases
for their delinquencies and who pay their delinquencies within the six (6)-month period set by the law. Mere
payment of unpaid contributions does not suffice; it is payment within, and only within, the six (6)-month availment
period that triggers the applicability of RA No. 9903.

FACTS: This petition is a Motion for Reconsideration filed by petitioner Romaricp J. Mendoza seeking for the
reversal of SC Decision dated Aug. 3, 2010, affirming the conviction issued by both the trial and appellate courts
for the petitioner’s violation of Section 22(a) and (d), in relation to Section 28 of RA No. 8282 or the Social
Security Act of 1997.

Highlights of the Aug 3, 2010 Decision:


Petitioner readily admitted during trial that he did not remit the SSS premium contributions of his employees at
Summa Alta Tierra Industries, Inc. from August 1998 to July 1999, in the amount of ₱239,756.80; inclusive of
penalties, this unremitted amount totaled to ₱421,151.09. The petitioner contends that during this period, Summa
Alta Tierra Industries, Inc. shut down as a result of the general decline in the economy. The petitioner pleaded
good faith and lack of criminal intent as his defenses.

Petitioner’s Argument: His designation in the Information was a "proprietor," thus, he was without criminal
liability since "proprietors" are not among the corporate officers specifically enumerated in Section 28(f) of RA No.
8282 to be criminally liable for the violation of its provisions.

SC held: Conviction is proper by proof beyond reasonable doubt since: first, the remittance of employee
contributions to the SSS is mandatory under RA No. 8282; and second, the failure to comply with a special law
being malum prohibitum, the defenses of good faith and lack of criminal intent are immaterial.
As to the petitioner’s argument, the Court rules that to sustain the petitioner’s argument would be to allow the
unscrupulous to conveniently escape liability merely through the creative use of managerial titles.

IN THE PRESENT MOTION FOR RECONSIDERATION:


Petitioner’s Argument: Pending his appeal with the Court of Appeals (CA), he voluntarily paid the SSS the amount
of ₱239,756.80 to settle his delinquency. That during the pendency of the petitioner’s case before the Court, then
President Gloria Macapagal-Arroyo signed RA No. 9903 into law. RA No. 9903 mandates the effective withdrawal
of all pending cases against employers who would remit their delinquent contributions to the SSS within a
specified period, viz., within six months after the law’s effectivity. The petitioner claims that in view of RA No. 9903
and its implementing rules, the settlement of his delinquent contributions in 2007 entitles him to an acquittal. He
invokes the equal protection clause in support of his plea.

ISSUE: (1) Whether or not petitioner should be acquitted from conviction by virtue of RA No. 9903,
(2) Whether or not RA No. 9903 is violative of the equal protection clause
(3) Whether or not the petioner is entitled of waiver of accrued penalties pursuant to RA No. 9903

SC RULING ON THE MR:


(1) NO. The petitioner’s arguments supporting his prayer for acquittal fail to convince us. The petitioner cannot
benefit from the terms of RA No. 9903, which condone only employers who pay their delinquencies within six
months from the law’s effectivity.
We note that the petitioner does not ask for the reversal of his conviction based on the authority of RA No. 9903;
he avoids making a straightforward claim because this law plainly does not apply to him or to others in the same
Angeles| Bajana | Balladares | Brillantes | Briones | Cabansag | Callanta | Chua | David|

De Leon | Gomez | Lopez | Macalino | Nostratis | Padilla | Reynon | Santos | Tan |Velasco
4E / 4F - 2018-2019
Page 226 of 920
LABOR REVIEW DIGEST
Atty. Joyrich Golangco

situation. The clear intent of the law is to grant condonation only to employers with delinquent contributions or
pending cases for their delinquencies and who pay their delinquencies within the six (6)-month period set by the
law. Mere payment of unpaid contributions does not suffice; it is payment within, and only within, the six (6)-month
availment period that triggers the applicability of RA No. 9903.
True, the petitioner’s case was pending with us when RA No. 9903 was passed. Unfortunately for him, he paid his
delinquent SSS contributions in 2007. By paying outside of the availment period, the petitioner effectively placed
himself outside the benevolent sphere of RA No. 9903. This is how the law is written: it condones employers —
and only those employers — with unpaid SSS contributions or with pending cases who pay within the six (6)-
month period following the law’s date of effectivity. Dura lex, sed lex.
(2) NO. On the matter of equal protection, the Court cited People v. Cayat, which provides that, “It is an
established principle of constitutional law that the guaranty of the equal protection of the laws is not violated by a
legislation based on reasonable classification. And the classification, to be reasonable, (1) must rest on
substantial distinctions; (2) must be germane to the purposes of the law; (3) must not be limited to existing
conditions only; and (4) must apply equally to all members of the same class.”
RA No. 9903 creates two classifications of employers delinquent in remitting the SSS contributions of their
employees: (1) those delinquent employers who pay within the six (6)-month period (the former group), and (2)
those delinquent employers who pay outside of this availment period (the latter group). The creation of these two
classes is obvious and unavoidable when Section 2 and the last proviso of Section 4 of the law are read together.
The same provisions show the law’s intent to limit the benefit of condonation to the former group only; had RA No.
9903 likewise intended to benefit the latter group, which includes the petitioner, it would have expressly declared
so. Laws granting condonation constitute an act of benevolence on the government’s part, similar to tax amnesty
laws; their terms are strictly construed against the applicants. Since the law itself excludes the class of employers
to which the petitioner belongs, no ground exists to justify his acquittal.
The difference in the dates of payment of delinquent contributions provides a substantial distinction between the
two classes of employers. In limiting the benefits of RA No. 9903 to delinquent employers who pay within the six
(6)-month period, the legislature refused to allow a sweeping, non-discriminatory condonation to all delinquent
employers, lest the policy behind RA No. 8282 be undermined.1avvphi1
(3) YES. The one benefit the petitioner can obtain from RA No. 9903 is the waiver of his accrued penalties, which
remain unpaid in the amount of ₱181,394.29. This waiver is derived from the last proviso of Section 4 of RA No.
9903:
Provided, further, That for reason of equity, employers who settled arrears in contributions before the effectivity of
this Act shall likewise have their accrued penalties waived.
This proviso is applicable to the petitioner who settled his contributions long before the passage of the law.
Applied to the petitioner, therefore, RA No. 9903 only works to allow a waiver of his accrued penalties, but not the
reversal of his conviction.

Angeles| Bajana | Balladares | Brillantes | Briones | Cabansag | Callanta | Chua | David|

De Leon | Gomez | Lopez | Macalino | Nostratis | Padilla | Reynon | Santos | Tan |Velasco
4E / 4F - 2018-2019
Page 227 of 920
LABOR REVIEW DIGEST
Atty. Joyrich Golangco

16. Signey vs. SSS


G.R. No. 173582; January 28, 2008
Tinga, J.

FACTS: Rodolfo Signey, Sr., a member of the SSS, died. In his members records, he had designated Yolanda
Signey (petitioner) as primary beneficiary and his four children with her as secondary beneficiaries. Petitioner filed
a claim for death benefits with the public respondent SSS. She revealed in her SSS claim that the deceased had a
common-law wife, Gina Servano, with whom he had two minor children namely, Ginalyn Servano (Ginalyn) and
Rodelyn Signey.

Petitioners declaration was confirmed when Gina herself filed a claim for the same death benefits in which she
also declared that both she and petitioner were common-law wives of the deceased and that Editha Espinosa was
the legal wife.

In addition, Editha also filed an application for death benefits with the SSS stating that she was the legal wife of
the deceased. The deceased’s child with Editha had already predeceased him.

The SSS denied the death benefit claim of petitioner. However, it recognized Ginalyn and Rodelyn, the minor
children of the deceased with Gina, as the primary beneficiaries under the SSS Law. The SSS also found that the
marriage between petitioner and the deceased was null and void because of a prior subsisting marriage between
the deceased and Editha, as confirmed with the Local Civil Registry of Cebu City.

Thereafter, petitioner filed a petition with the SSC in which she attached a waiver of rights executed by Editha
whereby the latter waived any/all claims from National Trucking Forwarding Corporation (NTFC) under the
supervision of National Development Corporation (NDC), Social Security System (SSS) and other
(i)nsurance(b)enefits due to the deceased Rodolfo Signey Sr.

ISSUE: whether or not petitioner and her children are entitled to the death benefits

HELD: NO. Section 8(e) and (k) of R.A. No. 8282 provides:

For the purposes of this Act, the following terms shall, unless the context indicates otherwise, have the following
meanings:

xxx

(e) Dependents. The dependent shall be the following:

(1) The legal spouse entitled by law to receive support from the member;

2) The legitimate, legitimated, or legally adopted, and illegitimate child who is unmarried, not
gainfully employed and has not reached twenty-one years (21) of age, or if over twenty-one (21)
years of age, he is congenitally or while still a minor has been permanently incapacitated and incapable
of self-support, physically or mentally; and

3) The parent who is receiving regular support from the member.

xxx

Angeles| Bajana | Balladares | Brillantes | Briones | Cabansag | Callanta | Chua | David|

De Leon | Gomez | Lopez | Macalino | Nostratis | Padilla | Reynon | Santos | Tan |Velasco
4E / 4F - 2018-2019
Page 228 of 920
LABOR REVIEW DIGEST
Atty. Joyrich Golangco

(k) Beneficiaries The dependent spouse until he or she remarries, the dependent legitimate, legitimated or legally
adopted, and illegitimate children, who shall be the primary beneficiaries of the member: Provided, That the
dependent illegitimate children shall be entitled to fifty percent (50%) of the share of the legitimate, legitimated or
legally adopted children: Provided, further, That in the absence of the dependent legitimate, legitimated or legally
adopted children of the member, his/her dependent illegitimate children shall be entitled to one hundred percent
(100%) of the benefits. In their absence, the dependent parents who shall be the secondary beneficiaries of the
member. In the absence of all of the foregoing, any other person designated by the member as his/her
secondary beneficiary.

Whoever claims entitlement to the benefits provided by law should establish his or her right thereto by substantial
evidence. Since petitioner is disqualified to be a beneficiary and because the deceased has no legitimate child, it
follows that the dependent illegitimate minor children of the deceased shall be entitled to the death benefits as
primary beneficiaries. The SSS Law is clear that for a minor child to qualify as a dependent, the only requirements
are that he/she must be below 21 years of age, not married nor gainfully employed.

Had the legitimate child of the deceased and Editha survived and qualified as a dependent under the SSS Law,
Ginalyn and Rodelyn would have been entitled to a share equivalent to only 50% of the share of the said
legitimate child. Since the legitimate child of the deceased predeceased him, Ginalyn and Rodelyn, as the only
qualified primary beneficiaries of the deceased, are entitled to 100% of the benefits.

-------------------------

ARTICLE 218
Definition of Labor Dispute

1. CITIBANK v. CA
G.R. No. 108961, November 27, 1998
Pardo J.

DOCTRINE:

Article 212, Paragraph l of the Labor Code provides the definition of a "labor dispute". It "includes any controversy
or matter concerning terms or conditions of employment or the association or representation of persons in
negotiating, fixing, maintaining, changing or arranging the terms and conditions of employment, regardless of
whether the disputants stand in the proximate relation of employer and employee."

FACTS:

Citibank contracted El Toro Security Agency, Inc. to provide it with the services of security guards to safeguard
and protect the premises and property of Citibank. On April 22, 1990, the contract between Citibank and El Toro
expired.

Citibank Integrated Guards Labor Alliance-SEGA-TUPAS/FSM (CIGLA) filed with the NCMB a request for
preventive mediation citing Citibank as respondent. The issues involved were unfair labor practice, dismissal of
union officers/members, and union busting.

Citibank served on El Toro a written notice that the bank would not renew the service agreement with the latter.
Simultaneously, Citibank hired Golden Pyramid Security Agency.

Angeles| Bajana | Balladares | Brillantes | Briones | Cabansag | Callanta | Chua | David|

De Leon | Gomez | Lopez | Macalino | Nostratis | Padilla | Reynon | Santos | Tan |Velasco
4E / 4F - 2018-2019
Page 229 of 920
LABOR REVIEW DIGEST
Atty. Joyrich Golangco

On the same date, CIGLA filed a manifestation with the NCMB that it was converting its request for preventive
mediation into a notice of strike. Thereafter, security guards of El Toro who were replaced by guards of the
Golden Pyramid Security Agency.

CIGLA considered the non-renewal of El Toro's service agreement with Citibank as a lockout and/or a mass
dismissal. They threatened to go on strike against Citibank and picket its premises. In fact, security guards
formerly assigned to Citibank under the expired agreement loitered around and near the Citibank premises in
large groups of from twenty (20) and at times fifty (50) persons.

On June 14, 1990, respondent CIGLA filed a notice of strike directed at the premises of the Citibank main office.

Petitioner Citibank filed with the RTC of Makati a complaint for injunction and damages. The complaint sought to
enjoin CIGLA and any person claiming membership therein from striking or otherwise disrupting the operations of
the bank. CIGLA filed with the trial court a motion to dismiss the complaint. The motion alleging that the RTC had
no jurisdiction, this being labor dispute.

RTC RULING:

The trial court denied respondent CIGLA's motion to dismiss. CIGLA filed with the trial court a motion for
reconsideration of the above-mentioned order. The trial court denied the motion.

CIGLA filed with the trial court its answer to the complaint, and averred as special and affirmative defense lack of
jurisdiction of the court over the subject matter of the case.

CA Ruling:

On May 24, 1991, respondent CIGLA filed with the Court of Appeals a petition for certiorari with preliminary
injunction assailing the validity of the proceedings had before the regional trial court.
The Court of Appeals ruled in CIGLA's favor. It also denied Citibank’s filed a motion for reconsideration.

ISSUE/S:

Whether it is the labor tribunal or the regional trial court that has jurisdiction over the subject matter of the
complaint filed by Citibank with the trial court.

HELD:

 It has been decided also that the Labor Arbiter has no jurisdiction over a claim filed where no employer-employee
relationship existed between a company and the security guards assigned to it by a security service contractor. In
this case, it was the security agency El Toro that recruited, hired and assigned the watchmen to their place of
work. It was the security agency that was answerable to Citibank for the conduct of its guards.

 The question arises. Is there a labor dispute between Citibank and the security guards, members of respondent
CIGLA, regardless of whether they stand in the relation of employer and employees? Article 212, Paragraph l of
the Labor Code provides the definition of a "labor dispute". It "includes any controversy or matter concerning
terms or conditions of employment or the association or representation of persons in negotiating, fixing,
maintaining, changing or arranging the terms and conditions of employment, regardless of whether the disputants
stand in the proximate relation of employer and employee."

Angeles| Bajana | Balladares | Brillantes | Briones | Cabansag | Callanta | Chua | David|

De Leon | Gomez | Lopez | Macalino | Nostratis | Padilla | Reynon | Santos | Tan |Velasco
4E / 4F - 2018-2019
Page 230 of 920
LABOR REVIEW DIGEST
Atty. Joyrich Golangco

If at all, the dispute between Citibank and El Toro security agency is one regarding the termination or non-renewal
of the contract of services. This is a civil dispute. El Toro was an independent contractor. Thus, no employer-
employee relationship existed between Citibank and the security guard members of the union in the security
agency who were assigned to secure the bank's premises and property. Hence, there was no labor dispute and
no right to strike against the bank.

Angeles| Bajana | Balladares | Brillantes | Briones | Cabansag | Callanta | Chua | David|

De Leon | Gomez | Lopez | Macalino | Nostratis | Padilla | Reynon | Santos | Tan |Velasco
4E / 4F - 2018-2019
Page 231 of 920
LABOR REVIEW DIGEST
Atty. Joyrich Golangco

2. PAL vs NLRC
GR No: 120567 Date: March 20, 1998
Ponente: Martinez, J.

Doctrine:
The term "labor dispute" is defined as "any controversy or matter concerning terms and conditions of employment
or the association or representation of persons in negotiating, fixing, maintaining, changing, or arranging the terms
and conditions of employment regardless of whether or not the disputants stand in the proximate relation of
employers and employees."

FACTS:
Private respondents are flight stewards of the petitioner. Both were dismissed from the service for their alleged
involvement in the April 3, 1993 currency smuggling in Hong Kong. Aggrieved by said dismissal, private
respondents filed with the NLRC a petition for injunction.

NLRC RULING:
NLRC issued a temporary mandatory injunction enjoining petitioner to cease and desist from enforcing its
February 22, 1995 Memorandum of dismissal.
(1) private respondents cannot be validly dismissed on the strength of petitioner's Code of Discipline which was
declared illegal by this Court in the case of PAL, Inc. vs. NLRC, (G.R. No. 85985), promulgated August 13, 1993,
for the reason that it was formulated by the petitioner without the participation of its employees as required in R.A.
6715, amending Article 211 of the Labor Code; (2) the whimsical, baseless and premature dismissals of private
respondents which "caused them grave and irreparable injury" is enjoinable as private respondents are left "with
no speedy and adequate remedy at law'"except the issuance of a temporary mandatory injunction; (3) the NLRC
is empowered under Article 218 (e) of the Labor Code not only to restrain any actual or threatened commission of
any or all prohibited or unlawful acts but also to require the performance of a particular act in any labor dispute,
which, if not restrained or performed forthwith, may cause grave or irreparable damage to any party; and (4) the
temporary mandatory power of the NLRC was recognized by this Court in the case of Chemo-Technicshe Mfg.,
Inc. Employees Union,DFA, et.al. vs. Chemo-Technische Mfg., Inc. [G.R. No. 107031, January 25,1993].

APPEAL TO THE SC:

Petitioner's Contention:
NLRC it has no jurisdiction to issue an injunction or restraining order since this may be issued only under Article
218 of the Labor Code if the case involves or arises from labor disputes

ISSUE/S:
Can the National Labor Relations Commission (NLRC), even without a complaint for illegal dismissal filed before
the labor arbiter, entertain an action for injunction and issue such writ enjoining petitioner Philippine Airlines, Inc.
from enforcing its Orders of dismissal against private respondents, and ordering petitioner to reinstate the private
respondents to their previous positions?

HELD:
NO.
In labor cases, Article 218 of the Labor Code empowers the NLRC-
"(e) To enjoin or restrain any actual or threatened commission of any or all prohibited or unlawful acts or to require
the performance of a particular act in any labor dispute which, if not restrained or performed forthwith, may
cause grave or irreparable damage to any party or render ineffectual any decision in favor of such party; x x
x."

Angeles| Bajana | Balladares | Brillantes | Briones | Cabansag | Callanta | Chua | David|

De Leon | Gomez | Lopez | Macalino | Nostratis | Padilla | Reynon | Santos | Tan |Velasco
4E / 4F - 2018-2019
Page 232 of 920
LABOR REVIEW DIGEST
Atty. Joyrich Golangco

Complementing the above-quoted provision, Sec. 1, Rule XI of the New Rules of Procedure of the NLRC,
pertinently provides as follows:
"Section 1. Injunction in Ordinary Labor Dispute.-A preliminary injunction or a restraining order may be granted by
the Commission through its divisions pursuant to the provisions of paragraph (e) of Article 218 of the Labor Code,
as amended, when it is established on the bases of the sworn allegations in the petition that the acts complained
of, involving or arising from any labor dispute before the Commission, which, if not restrained or performed
forthwith, may cause grave or irreparable damage to any party or render ineffectual any decision in favor of such
party.
xxx xxx xxx
The foregoing ancillary power may be exercised by the Labor Arbiters only as an incident to the cases
pending before them in order to preserve the rights of the parties during the pendency of the case, but excluding
labor disputes involving strikes or lockout. [7] (Emphasis Ours)
From the foregoing provisions of law, the power of the NLRC to issue an injunctive writ originates from "any
labor dispute" upon application by a party thereof, which application if not granted "may cause grave or
irreparable damage to any party or render ineffectualany decision in favor of such party."
The term "labor dispute" is defined as "any controversy or matter concerning terms and conditions of
employment or the association or representation of persons in negotiating, fixing, maintaining, changing,
or arranging the terms and conditions of employment regardless of whether or not the disputants stand in
the proximate relation of employers and employees."
The term "controversy" is likewise defined as "a litigated question; adversary proceeding in a court of
law; a civil action or suit, either at law or in equity; a justiciable dispute."
A "justiciable controversy" is "one involving an active antagonistic assertion of a legal right on one side and a
denial thereof on the other concerning a real, and not a mere theoretical question or issue."
Taking into account the foregoing definitions, it is an essential requirement that there must first be a labor
dispute between the contending parties before the labor arbiter. In the present case, there is no
labor dispute between the petitioner and private respondents as there has yet been no complaint for illegal
dismissal filed with the labor arbiter by the private respondents against the petitioner.
The petition for injunction directly filed before the NLRC is in reality an action for illegal dismissal. This is
clear from the allegations in the petition which prays for: reinstatement of private respondents; award of full
backwages, moral and exemplary damages; and attorney's fees. As such, the petition should have been filed with
the labor arbiter.
On the other hand, the NLRC shall have exclusive appellate jurisdiction over all cases decided by labor
arbiters as provided in Article217(b) of the Labor Code. In short, the jurisdiction of the NLRC in illegal dismissal
cases is appellate in nature and, therefore, it cannot entertain the private respondents' petition for injunction which
challenges the dismissal orders of petitioner. Article 218(e) of the Labor Code does not provide blanket authority
to the NLRC or any of its divisions to issue writs of injunction, considering that Section 1 of Rule XI of the New
Rules of Procedure of the NLRC makes injunction only an ancillary remedy in ordinary labor disputes"
Thus, the NLRC exceeded its jurisdiction when it issued the assailed Order granting private respondents'
petition for injunction and ordering the petitioner to reinstate private respondents.

Other Notes/ SC Pronouncements:

Angeles| Bajana | Balladares | Brillantes | Briones | Cabansag | Callanta | Chua | David|

De Leon | Gomez | Lopez | Macalino | Nostratis | Padilla | Reynon | Santos | Tan |Velasco
4E / 4F - 2018-2019
Page 233 of 920
LABOR REVIEW DIGEST
Atty. Joyrich Golangco

Managerial Employee

1. Penaranda v. Baganga Plywood Corporation


G.R. No. 159577, May 3, 2006
Panganiban, C.J.

Doctrine:

Managerial employees and members of the managerial staff are exempted from the provisions of the Labor Code
on labor standards. Since petitioner belongs to this class of employees, he is not entitled to overtime pay and
premium pay for working on rest days.

Facts:

Petitioner Charlito Peñaranda was hired as an employee of Baganga Plywood Corporation (BPC) to take charge
of the operations and maintenance of its steam plant boiler. In May 2001, Peñaranda filed a Complaint for illegal
dismissal with money claims against BPC and its general manager, Hudson Chua, before the NLRC.

Peñaranda alleges that he was employed by respondent on March 15, 1999 with a monthly salary of P5,000.00 as
Foreman/Boiler Head/Shift Engineer until he was illegally terminated on December 19, 2000.

Upon the other hand, respondent allege that complainant’s separation from service was done pursuant to Art. 283
of the Labor Code. The respondent was on temporary closure due to repair and general maintenance and it
applied for clearance with the DOLE to shut down and to dismiss employees. When respondent partially reopened
in January 2001, Peñaranda failed to reapply.

LA Ruling:
The labor arbiter ruled that there was no illegal dismissal and that petitioner’s Complaint was premature because
he was still employed by BPC. The temporary closure of BPC’s plant did not terminate his employment. hence, he
need not reapply when the plant reopened.

NLRC Ruling:
Respondents filed an appeal to the NLRC, which deleted the award of overtime pay and premium pay for working
on rest days. According to the Commission, petitioner was not entitled to these awards because he was a
managerial employee.

CA Ruling:

In its Resolution dated January 27, 2003, the CA dismissed Peñaranda’s Petition for Certiorari. The CA also dnied
his motion for reconsideration on the ground that petitioner still failed to submit the pleadings filed before the
NLRC.

Issues:

WON Peñaranda is a managerial employee.

Held:

Angeles| Bajana | Balladares | Brillantes | Briones | Cabansag | Callanta | Chua | David|

De Leon | Gomez | Lopez | Macalino | Nostratis | Padilla | Reynon | Santos | Tan |Velasco
4E / 4F - 2018-2019
Page 234 of 920
LABOR REVIEW DIGEST
Atty. Joyrich Golangco

 Article 82 of the Labor Code exempts managerial employees from the coverage of labor standards. Labor
standards provide the working conditions of employees, including entitlement to overtime pay and premium pay
for working on rest days.29 Under this provision, managerial employees are "those whose primary duty consists of
the management of the establishment in which they are employed or of a department or subdivision."

The Implementing Rules of the Labor Code state that managerial employees are those who meet the following
conditions:

(1) Their primary duty consists of the management of the establishment in which they are employed or of a
department or subdivision thereof;
(2) They customarily and regularly direct the work of two or more employees therein;
(3) They have the authority to hire or fire other employees of lower rank; or their suggestions and
recommendations as to the hiring and firing and as to the promotion or any other change of status of other
employees are given particular weight."

The Court disagrees with the NLRC’s finding that petitioner was a managerial employee. However, petitioner was
a member of the managerial staff, which also takes him out of the coverage of labor standards. Like managerial
employees, officers and members of the managerial staff are not entitled to the provisions of law on labor
standards. The Implementing Rules of the Labor Code define members of a managerial staff as those with the
following duties and responsibilities:

(1) The primary duty consists of the performance of work directly related to management policies of the employer;
(2) Customarily and regularly exercise discretion and independent judgment;
(3) (i) Regularly and directly assist a proprietor or a managerial employee whose primary duty consists of the
management of the establishment in which he is employed or subdivision thereof; or (ii) execute under general
supervision work along specialized or technical lines requiring special training, experience, or knowledge; or (iii)
execute under general supervision special assignments and tasks; and
(4) who do not devote more than 20 percent of their hours worked in a workweek to activities which are not
directly and closely related to the performance of the work described in paragraphs (1), (2), and (3) above."33

 Petitioner supervised the engineering section of the steam plant boiler. His work involved overseeing the
operation of the machines and the performance of the workers in the engineering section. This work necessarily
required the use of discretion and independent judgment to ensure the proper functioning of the steam plant
boiler. As supervisor, petitioner is deemed a member of the managerial staff.

Noteworthy, even petitioner admitted that he was a supervisor. In his Position Paper, he stated that he was the
foreman responsible for the operation of the boiler.36 The term foreman implies that he was the representative of
management over the workers and the operation of the department. Petitioner’s evidence also showed that he
was the supervisor of the steam plant.38 His classification as supervisor is further evident from the manner his
salary was paid. He belonged to the 10% of respondent’s 354 employees who were paid on a monthly basis; the
others were paid only on a daily basis.

Hence, the Court found no justification to award overtime pay and premium pay for rest days to petitioner.

Angeles| Bajana | Balladares | Brillantes | Briones | Cabansag | Callanta | Chua | David|

De Leon | Gomez | Lopez | Macalino | Nostratis | Padilla | Reynon | Santos | Tan |Velasco
4E / 4F - 2018-2019
Page 235 of 920
LABOR REVIEW DIGEST
Atty. Joyrich Golangco

2. SAMAHANG MANGGAGAWA SA CHARTER CHEMICAL SOLIDARITY OF UNIONS IN THE PHILIPPINES


FOR EMPOWERMENT AND REFORMS (SMCC-SUPER), ZACARRIAS JERRY VICTORIO-Union President
vs. CHARTER CHEMICAL and COATING CORPORATION
G.R. No. 169717; March 16, 2011
DEL CASTILLO, J.:

DOCTRINE: supervisory employees and security guards shall not be eligible for membership in a labor
organization of the rank-and-file employees but may join, assist or form separate labor organizations of their own

FACTS:

On February 19, 1999, Samahang Manggagawa sa Charter Chemical Solidarity of Unions in the Philippines for
Empowerment and Reforms (petitioner union) filed a petition for certification election among the regular rank-and-
file employees of Charter Chemical and Coating Corporation (respondent company) with the Mediation Arbitration
Unit of the DOLE, National Capital Region.
On April 14, 1999, respondent company filed an Answer with Motion to Dismiss on the ground that petitioner
union is not a legitimate labor organization because of (1) failure to comply with the documentation requirements
set by law, and (2) the inclusion of supervisory employees within petitioner union.

Med-Arbiter’s Ruling: The Med-Arbiter ruled that petitioner union is not a legitimate labor organization. The Med-
Arbiter further held that the list of membership of petitioner union consisted of 12 batchman, mill operator and
leadman who performed supervisory functions. Under Article 245 of the Labor Code, said supervisory employees
are prohibited from joining petitioner union which seeks to represent the rank-and-file employees of respondent
company.

Department of Labor and Employment’s Ruling: DOLE initially issued a Decision in favor of respondent
company dismissing petitioner union’s appeal on the ground that the latter’s petition for certification election was
filed out of time. On motion for reconsideration, however, the DOLE reversed its earlier ruling.

ISSUE:
Whether or not the alleged mixture of rank-and-file and supervisory employees of petitioner union’s
membership is a ground for the cancellation of petitioner union’s legal personality and dismissal of [the] petition for
certification election.

RULING:

NO. The mixture of rank-and-file and supervisory employees in petitioner union does not nullify its legal
personality as a legitimate labor organization. The job descriptions indicate that the aforesaid employees exercise
recommendatory managerial actions which are not merely routinary but require the use of independent judgment,
hence, falling within the definition of supervisory employees under Article 212(m) of the Labor Code. For this

Angeles| Bajana | Balladares | Brillantes | Briones | Cabansag | Callanta | Chua | David|

De Leon | Gomez | Lopez | Macalino | Nostratis | Padilla | Reynon | Santos | Tan |Velasco
4E / 4F - 2018-2019
Page 236 of 920
LABOR REVIEW DIGEST
Atty. Joyrich Golangco

reason, we are constrained to agree with the Med-Arbiter, as upheld by the appellate court, that petitioner union
consisted of both rank-and-file and supervisory employees.

Nonetheless, the inclusion of the aforesaid supervisory employees in petitioner union does not divest it of its
status as a legitimate labor organization. Under Section 1 of the Rules and Regulations Implementing R.A. No.
6715 (1989 Amended Omnibus Rules) supervisory employees and security guards shall not be eligible for
membership in a labor organization of the rank-and-file employees but may join, assist or form separate labor
organizations of their own.

3. PAMELA FLORENTINA JUMUAD v. HI-FLYER FOOD, INC. and/or JESUS R. MONTEMAYOR.


GR No: 187887 Date: September 7, 2011
Ponente: Reyes, J.

Doctrine: Pursuant to the Courts ruling in Lima Land, Inc. v. Cuevas, as long as there is some basis for such loss
of confidence, such as when the employer has reasonable ground to believe that the employee concerned is
responsible for the purported misconduct, and the nature of his participation therein renders him unworthy of the
trust and confidence demanded of his position, a managerial employee may be dismissed.

FACTS:
Pamela Florentina Jumuad was employed as Area Manager in Visayas by Hi-Flyer, Inc., the company managing
Kentucky Fried Chicken stores throughout the country. Aside from being responsible in monitoring her
subordinates, Jumuad was tasked to: 1) be highly visible in the restaurants under her jurisdiction; 2) monitor and
Angeles| Bajana | Balladares | Brillantes | Briones | Cabansag | Callanta | Chua | David|

De Leon | Gomez | Lopez | Macalino | Nostratis | Padilla | Reynon | Santos | Tan |Velasco
4E / 4F - 2018-2019
Page 237 of 920
LABOR REVIEW DIGEST
Atty. Joyrich Golangco

support day-to-day operations; and 3) ensure that all the facilities and equipment at the restaurant were properly
maintained and serviced. As area manager, Jumuad was allowed to avail of Hi-Flyers car loan program, wherein
forty (40%) percent of the total loanable amount would be subsidized by Hi-Flyer and the remaining
sixty (60%) percent would be deducted from her salary. It was also agreed that in the event that she would resign
or would be terminated prior to the payment in full of the said car loan, she could opt to surrender the car to Hi-
Flyer or to pay the full balance of the loan.
Hi Flyer conducted a food safety, service and sanitation audit at KFC-Gaisano. The audit, denominated as
CHAMPS Excellence Review (CER), revealed several sanitation violations, such as the presence of rodents and
the use of a defective chiller for the storage of food. Hi-Flyer audited the accounts of KFC-Bohol amid reports that
certain employees were covering up cash shortages. As a result, the following irregularities were discovered: 1)
cash shortage amounting to ₱62,290.85; 2) delay in the deposits of cash sales by an average of three days; 3) the
presence of two sealed cash-for-deposit envelopes containing paper cut-outs instead of cash; 4) falsified entries
in the deposit logbook; 5) lapses in inventory control; and 6) material product spoilage. In her report regarding the
incident, Jumuad disclaimed any fault in the incident by pointing out that she was the one responsible for the
discovery of this irregularity. Hi-Flyer conducted another CER, this time at its KFC-Cocomall branch. Grout and
leaks at the branchs kitchen wall, dried up spills from the marinator, as well as a live rat under postmix, and signs
of rodent gnawing/infestation were found.
Seeking to hold Jumuad accountable for the irregularities uncovered in the branches under her supervision, Hi-
Flyer sent Jumuad an Irregularities Report and Notice of Charges. Jumuad submitted her written explanation. Hi-
Flyer held an administrative hearing where Jumuad appeared with counsel. Apparently not satisfied with her
explanations, Hi-Flyer served her a Notice of Dismissal. This prompted Jumuad to file a complaint against Hi-Flyer
and/or Jesus R. Montemayor (Montemayor) for illegal dismissal before the NLRC on October 17, 2005, praying for
reinstatement and payment of separation pay, 13th month pay, service incentive leave, moral and exemplary
damages, and attorneys fees. Jumuad also sought the reimbursement of the amount equivalent to her forty
percent (40%) contribution to Hi-Flyers subsidized car loan program.

LA RULING:
LA found that Jumuad was not completely blameless for the anomalies discovered, she was of the view that the
employers prerogative to dismiss or layoff an employee must be exercised without abuse of discretion and should
be tempered with compassion and understanding. Thus, the dismissal was too harsh considering the
circumstances. After finding that no serious cause for termination existed, the LA ruled that Jumuad was illegally
dismissed.
NLRC: Affirmed LA ruling.

COURT OF APPEALS:
CA was of the opinion that the requirements of substantive and procedural due process were complied with
affording Jumuad an opportunity to be heard first, when she submitted her written explanation and then, when she
was informed of the decision and the basis of her termination.

PETITION TO THE SC:


ISSUE/S:
1. Whether or not Jumuad was illegally dismissed
HELD:
NO. As correctly noted by the appellate court, Jumuad executed management policies and had the
power to discipline the employees of KFC branches in her area. She recommended actions on employees
to the head office. Pertinent is Article 212 (m) of the Labor Code defining a managerial employee as one who is
vested with powers or prerogatives to lay down and execute management policies and/or hire, transfer, suspend,
lay off, recall, discharge, assign or discipline employees.
Based on established facts, the mere existence of the grounds for the loss of trust and confidence
justifies petitioners dismissal. Pursuant to the Courts ruling in Lima Land, Inc. v. Cuevas, as long as there is some
Angeles| Bajana | Balladares | Brillantes | Briones | Cabansag | Callanta | Chua | David|

De Leon | Gomez | Lopez | Macalino | Nostratis | Padilla | Reynon | Santos | Tan |Velasco
4E / 4F - 2018-2019
Page 238 of 920
LABOR REVIEW DIGEST
Atty. Joyrich Golangco

basis for such loss of confidence, such as when the employer has reasonable ground to believe that the employee
concerned is responsible for the purported misconduct, and the nature of his participation therein renders him
unworthy of the trust and confidence demanded of his position, a managerial employee may be dismissed.
As the employer, Hi-Flyer has the right to regulate, according to its discretion and best judgment, all
aspects of employment, including work assignment, working methods, processes to be followed, working
regulations, transfer of employees, work supervision, lay-off of workers and the discipline, dismissal and recall of
workers. Management has the prerogative to discipline its employees and to impose appropriate penalties on
erring workers pursuant to company rules and regulations.
So long as they are exercised in good faith for the advancement of the employers interest and not for the
purpose of defeating or circumventing the rights of the employees under special laws or under valid agreements,
the employers exercise of its management prerogative must be upheld. In this case, Hi-Flyer exercised in good
faith its management prerogative as there is no dispute that it has lost trust and confidence in her and her
managerial abilities, to its damage and prejudice. Her dismissal, was therefore, justified.

Angeles| Bajana | Balladares | Brillantes | Briones | Cabansag | Callanta | Chua | David|

De Leon | Gomez | Lopez | Macalino | Nostratis | Padilla | Reynon | Santos | Tan |Velasco
4E / 4F - 2018-2019
Page 239 of 920
LABOR REVIEW DIGEST
Atty. Joyrich Golangco

ARTICLE 224
Jurisdiction of Labor Arbiter

1. PEOPLES BROADCASTING SERVICE (BOMBO RADYO PHILS., INC.) v. THE SECRETARY OF THE
DEPARTMENT OF LABOR AND EMPLOYMENT

DOCTRINE: If a complaint is brought before the DOLE to give effect to the labor standards provisions of the
Labor Code or other labor legislation, and there is a finding by the DOLE that there is an existing employer-
employee relationship, the DOLE exercises jurisdiction to the exclusion of the NLRC. If the DOLE finds that there
is no employer-employee relationship, the jurisdiction is properly with the NLRC. If a complaint is filed with the
DOLE, and it is accompanied by a claim for reinstatement, the jurisdiction is properly with the Labor Arbiter, under
Art. 217(3) of the Labor Code, which provides that the Labor Arbiter has original and exclusive jurisdiction over
those cases involving wages, rates of pay, hours of work, and other terms and conditions of employment, if
accompanied by a claim for reinstatement. If a complaint is filed with the NLRC, and there is still an existing
employer-employee relationship, the jurisdiction is properly with the DOLE. The findings of the DOLE, however,
may still be questioned through a petition for certiorari under Rule 65 of the Rules of Court
.
FACTS: Jandeleon Juezan filed a complaint against petitioner with the Department of Labor and Employment
(DOLE), for illegal deduction, nonpayment of service incentive leave, 13th month pay, premium pay for holiday
and rest day and illegal diminution of benefits, delayed payment of wages and non-coverage of SSS, PAG-IBIG
and Philhealth. The DOLE Regional Director found that private respondent was an employee of petitioner, and
was entitled to his money claims. When the matter was brought before the CA it was held that PBS was accorded
due process as it had been given the opportunity to be heard, and that the DOLE Secretary had jurisdiction over
the matter, as the jurisdictional limitation imposed by Article 129 of the Labor Code on the power of the DOLE
Secretary under Art. 128(b) of the Code had been repealed by Republic Act No. (RA) 7730. However, the SC
found that there was no employer-employee relationship between PBS and and private respondent. It was held
that while the DOLE may make a determination of the existence of an employer-employee relationship, this
function could not be co-extensive with the visitorial and enforcement power provided in Art. 128(b) of the Labor
Code, as amended by RA 7730. The National Labor Relations Commission (NLRC) was held to be the primary
agency in determining the existence of an employer-employee relationship. This was the interpretation of the
Court of the clause in cases where the relationship of employer-employee still exists in Art. 128(b). From this
Decision, PAO filed a Motion for Clarification of Decision. The PAO sought to clarify as to when the visitorial and
enforcement power of the DOLE be not considered as coextensive with the power to determine the existence of
an employer-employee relationship. The DOLE also sought the same clarification.

ISSUE: Is the NLRC the sole body with jurisdiction to determine the existence of an employer-employee
relationship?
SC RULING: NO. No procedure was laid down where the DOLE would only make a preliminary finding, that the
power was primarily held by the NLRC. The law did not say that the DOLE would first seek the NLRCs
determination of the existence of an employer-employee relationship, or that should the existence of the
employer-employee relationship be disputed, the DOLE would refer the matter to the NLRC. The DOLE must
have the power to determine whether or not an employer-employee relationship exists, and from there to decide
whether or not to issue compliance orders.

Angeles| Bajana | Balladares | Brillantes | Briones | Cabansag | Callanta | Chua | David|

De Leon | Gomez | Lopez | Macalino | Nostratis | Padilla | Reynon | Santos | Tan |Velasco
4E / 4F - 2018-2019
Page 240 of 920
LABOR REVIEW DIGEST
Atty. Joyrich Golangco

2. EX-BATAAN VETERANS SECURITY AGENCY, INC., (EBVSAI) v. THE SECRETARY OF LABOR


DOCTRINE: While it is true that under Articles 129 and 217 of the Labor Code, the LA has jurisdiction to hear and
decide cases where the aggregate money claims of each employee exceeds P5,000.00, said provisions of law do
not contemplate nor cover the visitorial and enforcement powers of the Secretary of Labor or his duly authorized
representatives. Rather, said powers are defined and set forth in Article 128 of the Labor Code.
FACTS: Private respondents are EBVSAI's employees who instituted a complaint for underpayment of wages
against EBVSAI before the Regional Office (RO) of DOLE. Consequently, RO conducted a complaint inspection of
EBVSAI’s Plant where several labor law violations were noted. On the same day, the RO issued a notice of
hearing requiring EBVSAI and private respondents to attend. After the hearing, the Regional Director (RD)
ordered EBVSAI to pay Php 763,927.85 to the affected employees. EBVSAI filed a motion for reconsideration and
alleged that under Articles 129 and 217(6) of the Labor Code, the Labor Arbiter, not the Regional Director, has
exclusive and original jurisdiction over the case because the individual monetary claim of private respondents
exceeds P5,000. RD denied the motion stating that, pursuant to RA 7730, the limitations under Articles 129 and
217(6) of the Labor Code no longer apply to the Secretary of Labor's visitorial and enforcement powers under
Article 128(b). The Secretary of Labor or his duly authorized representatives are now empowered to hear and
decide, in a summary proceeding, any matter involving the recovery of any amount of wages and other monetary
claims arising out of employer-employee relations at the time of the inspection.
DOLE SECRETARY RULING: It affirmed the Director’s decision on the ground that pursuant to RA 7730, the
Court's decision in the Servando case is no longer controlling insofar as the restrictive effect of Article 129 on the
visitorial and enforcement power of the Secretary of Labor is concerned.
CA RULING: affirmed DOLE Secretary ruling
ISSUE: Whether the Secretary of Labor or his duly authorized representatives have jurisdiction over the money
claims of private respondents which exceed P5,000?
SC RULING: YES. In Allied Investigation Bureau, Inc. v. Sec. of Labor, SC ruled that while it is true that under
Articles 129 and 217 of the Labor Code, the LA has jurisdiction to hear and decide cases where the aggregate
money claims of each employee exceeds P5,000.00, said provisions of law do not contemplate nor cover the
visitorial and enforcement powers of the Secretary of Labor or his duly authorized representatives. Rather, said
powers are defined and set forth in Article 128 of the Labor Code (as amended by R.A. No. 7730) thus: (b)
Notwithstanding the provisions of Article[s] 129 and 217 of this Code to the contrary, and in cases where the
relationship of employer-employee still exists, the Secretary of Labor and Employment or his duly authorized
representatives shall have the power to issue compliance orders to give effect to [the labor standards provisions
of this Code and other] labor legislation based on the findings of labor employment and enforcement officers or
industrial safety engineers made in the course of inspection. However, if the labor standards case is covered by
the exception clause in Article 128(b) of the Labor Code, then the RD will have to endorse the case to the
appropriate Arbitration Branch of the NLRC. In order to divest the RD or his representatives of jurisdiction, the
following elements must be present: (a) that the employer contests the findings of the labor regulations officer and
raises issues thereon; (b) that in order to resolve such issues, there is a need to examine evidentiary matters; and
(c) that such matters are not verifiable in the normal course of inspection. The rules also provide that the employer
shall raise such objections during the hearing of the case or at any time after receipt of the notice of inspection
results.
In this case, the RD validly assumed jurisdiction over the money claims of private respondents even if the claims
exceeded P5,000 because such jurisdiction was exercised in accordance with Article 128(b) of the Labor Code
and the case does not fall under the exception clause. EBVSAI did not contest the findings of the labor regulations
officer during the hearing or after receipt of the notice of inspection results. It was only in its supplemental motion
for reconsideration before the RD that EBVSAI questioned the findings of the labor regulations officer and
presented documentary evidence to controvert the claims of private respondents. But even if this was the case,
the RD and the Secretary of Labor still looked into and considered EBVSAI's documentary evidence and found
that such did not warrant the reversal of the order.

Angeles| Bajana | Balladares | Brillantes | Briones | Cabansag | Callanta | Chua | David|

De Leon | Gomez | Lopez | Macalino | Nostratis | Padilla | Reynon | Santos | Tan |Velasco
4E / 4F - 2018-2019
Page 241 of 920
LABOR REVIEW DIGEST
Atty. Joyrich Golangco

3. ARSENIO LOCSIN vs. NISSAN LEASE PHILS.


GR No: 185567 Date: October 20, 2010
Ponente: Brion, J.

Doctrine: A corporate officer’s dismissal is always a corporate act, or an intra-corporate controversy which arises
between a stockholder and a corporation so that RTC should exercise jurisdiction based on Section 5(c) of PD
902-A.
FACTS: Locsin was elected Executive Vice President and Treasurer (EVP/Treasurer) of NCLPI. Locsin held this
position for 13 years, having been re-elected every year since 1992, until January 21, 2005, when he was
nominated and elected Chairman of NCLPI’s Board of Directors. A little over seven (7) months after his election
as Chairman of the Board, the NCLPI Board held a special meeting at the Manila Polo Club. One of the items of
the agenda was the election of a new set of officers. Locsin was neither re-elected Chairman nor reinstated to his
previous position as EVP/Treasurer.
Locsin filed a complaint for illegal dismissal with prayer for reinstatement, payment of backwages, damages and
attorney’s fees before the Labor Arbiter against NCLPI and Banson, who was then President of NCLPI.
Instead of filing their position paper, NCLPI and Banson filed a Motion to Dismiss, on the ground that the Labor
Arbiter did not have jurisdiction over the case since the issue of Locsin’s removal as EVP/Treasurer involves an
intra-corporate dispute.
LA RULING: Labor Arbiter denied the Motion to Dismiss, holding that the office acquired "jurisdiction to arbitrate
and/or decide the instant complaint finding extant in the case an employer-employee relationship."
CA RULING: NCLPI elevated the case to the CA through a Petition for Certiorari under Rule 65 of the Rules of
Court. The CA ruled that Locsin was a corporate officer; the issue of his removal as EVP/Treasurer is an intra-
corporate dispute under the RTC’s jurisdiction. The CA defined "corporate officers as those officers of a
corporation who are given that character either by the Corporation Code or by the corporations’ by-laws."
Petitioner’s contention: Locsin submits that he is a regular employee of NCLPI since - as he argued before the
Labor Arbiter and the CA - his relationship with the company meets the "four-fold test." He concludes that the
Labor Arbiter and the NLRC – not the RTC (as NCLPI posits) – has jurisdiction to decide the controversy.
Respondent’s contention: Nissan submits that the CA correctly ruled that the Labor Arbiter does not have
jurisdiction over Locsin’s complaint for illegal dismissal. In support, Nissan maintains that Locsin is a corporate
officer and not an employee.
ISSUE: Whether or not Locsin is a corporate officer thereby excluding him from the coverage of the Labor
Code.
HELD:
YES. Locsin is a corporate officer.
He was undeniably Chairman and President, and was elected to these positions by the Nissan board pursuant to
its By-laws. Given Locsin’s status as a corporate officer, the RTC, not the Labor Arbiter or the NLRC, has
jurisdiction to hear the legality of the termination of his relationship with Nissan. As previously held, a corporate
officer’s dismissal from service is an intra-corporate dispute. A corporate officer’s dismissal is always a corporate
act, or an intra-corporate controversy which arises between a stockholder and a corporation so that the RTC
should exercise jurisdiction over it. Furthermore, it is provided in , Section 5(c) of Presidential Decree No. 902-A,
Subsection 5.2 that the Security and Exchange Commission’s jurisdiction over all cases enumerated under
Section 5 of Presidential Decree No. 902-A is hereby transferred to the Courts of general jurisdiction or the
appropriate Regional Trial Court.

Angeles| Bajana | Balladares | Brillantes | Briones | Cabansag | Callanta | Chua | David|

De Leon | Gomez | Lopez | Macalino | Nostratis | Padilla | Reynon | Santos | Tan |Velasco
4E / 4F - 2018-2019
Page 242 of 920
LABOR REVIEW DIGEST
Atty. Joyrich Golangco

4. OSCAR REYES vs. HON. REGIONAL TRIAL COURT OF MAKATI, BRANCH 142, ZENITH INSURANCE
CORPORATION and RODRIGO REYES
GR No: 165744 Date: August 11, 2008
Ponente: Brion, J.

Doctrine: To determine whether a case involves an intra-corporate controversy, and is to be heard and decided
by the branches of the RTC specifically designated by the Court to try and decide such cases, two elements must
concur: (a) the status or relationship of the parties; and (2) the nature of the question that is the subject of their
controversy.
FACTS: Oscar and private respondent Rodrigo C. Reyes (Rodrigo) are two of the four children of the spouses
Pedro and Anastacia Reyes. Pedro, Anastacia, Oscar, and Rodrigo each owned shares of stock of Zenith
Insurance Corporation (Zenith), a domestic corporation established by their family. Pedro died in 1964, while
Anastacia died in 1993. Although Pedro's estate was judicially partitioned among his heirs sometime in the 1970s,
no similar settlement and partition appear to have been made with Anastacia's estate, which included her
shareholdings in Zenith. As of June 30, 1990, Anastacia owned 136,598 shares of Zenith; Oscar and Rodrigo
owned 8,715,637 and 4,250 shares, respectively. Zenith and Rodrigo filed a complaint with the Securities and
Exchange Commission (SEC) against Oscar. The complaint stated that it is "a derivative suit initiated and filed by
the complainant Rodrigo C. Reyes to obtain an accounting of the funds and assets of ZENITH INSURANCE
CORPORATIONwhich are now or formerly in the control, custody, and/or possession of respondent [herein
petitioner Oscar] and to determine the stocks of deceased spouses Pedro and Anastacia Reyes that were
arbitrarily and fraudulently appropriated [by Oscar] for himself.
When Republic Act (R.A.) No. 8799P.D.) No. 902-A was transferred to the RTC designated as a special
commercial court. The records of Rodrigo's SEC case were thus turned over to the RTC, Branch 142, Makati, and
docketed as Civil Case No. 00-1553. .) No. 902-A was transferred to the RTC designated as a special commercial
court. The records of Rodrigo's SEC case were thus turned over to the RTC, Branch 142, Makati, and docketed
as Civil Case No. 00-1553.
Petitioner’s Contention: Oscar filed a Motion to Declare Complaint as Nuisance or Harassment Suit. He claimed
that the complaint is a mere nuisance or harassment suit and should, according to the Interim Rules of Procedure
for Intra-Corporate Controversies, be dismissed; and that it is not a bona fide derivative suit as it partakes of the
nature of a petition for the settlement of estate of the deceased Anastacia that is outside the jurisdiction of a
special commercial court.
RTC Ruling: The RTC denied Oscar’s motion. It ruled that it was not a derivative suit and thus, the petition should
be threshed out in a petition for settlement of estate.
CA Ruling: Affirmed the RTC ruling.
ISSUE: Whether or not the RTC has jurisdiction over Rodrigo’s complaint.
HELD:
NO. To determine whether a case involves an intra-corporate controversy, and is to be heard and decided by the
branches of the RTC specifically designated by the Court to try and decide such cases, two elements must
concur: (a) the status or relationship of the parties; and (2) the nature of the question that is the subject of their
controversy.
The first element requires that the controversy must arise out of intra-corporate or partnership relations between
any or all of the parties and the corporation, partnership, or association of which they are stockholders, members
or associates; between any or all of them and the corporation, partnership, or association of which they are
stockholders, members, or associates, respectively; and between such corporation, partnership, or association
and the State insofar as it concerns their individual franchises. The second element requires that the dispute
among the parties be intrinsically connected with the regulation of the corporation. If the nature of the controversy
involves matters that are purely civil in character, necessarily, the case does not involve an intra-corporate
controversy.
The transfer of title by means of succession, though effective and valid between the parties involved (i.e., between
the decedent's estate and her heirs), does not bind the corporation and third parties. The transfer must be
Angeles| Bajana | Balladares | Brillantes | Briones | Cabansag | Callanta | Chua | David|

De Leon | Gomez | Lopez | Macalino | Nostratis | Padilla | Reynon | Santos | Tan |Velasco
4E / 4F - 2018-2019
Page 243 of 920
LABOR REVIEW DIGEST
Atty. Joyrich Golangco

registered in the books of the corporation to make the transferee-heir a stockholder entitled to recognition as such
both by the corporation and by third parties. Rodrigo must, therefore, hurdle two obstacles before he can be
considered a stockholder of Zenith with respect to the shareholdings originally belonging to Anastacia. First, he
must prove that there are shareholdings that will be left to him and his co-heirs, and this can be determined only in
a settlement of the decedent's estate. No such proceeding has been commenced to date. Second, he must
register the transfer of the shares allotted to him to make it binding against the corporation.
Whether as an individual or as a derivative suit, the RTC - sitting as special commercial court - has no jurisdiction
to hear Rodrigo's complaint since what is involved is the determination and distribution of successional rights to
the shareholdings of Anastacia Reyes. Rodrigo's proper remedy, under the circumstances, is to institute a special
proceeding for the settlement of the estate of the deceased Anastacia Reyes, a move that is not foreclosed by the
dismissal of his present complaint.

Angeles| Bajana | Balladares | Brillantes | Briones | Cabansag | Callanta | Chua | David|

De Leon | Gomez | Lopez | Macalino | Nostratis | Padilla | Reynon | Santos | Tan |Velasco
4E / 4F - 2018-2019
Page 244 of 920
LABOR REVIEW DIGEST
Atty. Joyrich Golangco

5. OKOL v. SLIMMERS WORLD INTERNATIONAL, BEHAVIOR MODIFICATIONS, INC., and RONALD


JOSEPH MOY
G.R. No. 160146; December 11, 2009

Facts:
- the Bureau of Customs seized seven Precor elliptical machines and seven Precor treadmills belonging to or
consigned to Slimmers World.
- The shipment of the equipment was placed under the names of Okol and two customs brokers for a value less
than US$500. For being undervalued, the equipment were seized.

- Slimmers World International operating under the name Behavior Modifications, Inc. (Slimmers World) employed
petitioner Leslie Okol (Okol) as a management trainee on 15 June 1992.
- She rose up the ranks to become Head Office Manager and then Director and Vice President from 1996 until her
dismissal on 22 September 1999
- prior to Okols dismissal, Slimmers World preventively suspended Okol due to the said incident
- 2 September 1999, Okol received a memorandum that her suspension had been extended from 2 September until
1 October 1999 pending the outcome of the investigation
- 17 September 1999, Okol received another memorandum from Slimmers World requiring her to explain why no
disciplinary action should be taken against her in connection with the equipment seized
- 19 September 1999, Okol filed her written explanation. However, Slimmers World found Okols explanation to be
unsatisfactory.
- Through a letter dated 22 September 1999 signed by its president Ronald Joseph Moy (Moy), Slimmers World
terminated Okols employment
- CRUX OF THE CASE: Okol filed a complaint with the Arbitration branch of the NLRC against Slimmers World,
Behavior Modifications, Inc. and Moy (collectively called respondents) for illegal suspension, illegal dismissal,
unpaid commissions, damages and attorneys fees, with prayer for reinstatement and payment of backwages.
- respondents filed a Motion to Dismiss asserted that the NLRC had no jurisdiction over the subject matter of the
complaint
- LA granted the motion to dismiss. The labor arbiter ruled that Okol was the vice-president of Slimmers World at
the time of her dismissal. Since it involved a corporate officer, the dispute was an intra-corporate controversy
falling outside the jurisdiction of the Arbitration branch.
- Okol filed an appeal with the NLRC. In a Resolution dated 29 May 2001, the NLRC reversed and set aside the
labor arbiters order

- MR to the NLRC, NLRC not only decided the case on the merits but did so in the absence of position papers from
both parties.
- the appellate court set aside the NLRCs Resolution dated 29 May 2001 and affirmed the labor arbiters Order
dated 20 March 2000. The Court of Appeals ruled that the case, being an intra-corporate dispute, falls within the
jurisdiction of the regular courts pursuant to Republic Act No. 8799.

ISSUE:
- whether or not the NLRC has jurisdiction over the illegal dismissal case filed by petitioner.

HELD:
 The petition lacks merit.
 Petitioner asserts that even as vice-president, the work that she performed conforms to that of an employee rather
than a corporate officer. Mere title or designation in a corporation will not, by itself, determine the existence of an
employer-employee relationship. It is the four-fold test, namely (1) the power to hire, (2) the payment of wages, (3)
the power to dismiss, and (4) the power to control, which must be applied.
Angeles| Bajana | Balladares | Brillantes | Briones | Cabansag | Callanta | Chua | David|

De Leon | Gomez | Lopez | Macalino | Nostratis | Padilla | Reynon | Santos | Tan |Velasco
4E / 4F - 2018-2019
Page 245 of 920
LABOR REVIEW DIGEST
Atty. Joyrich Golangco

 The issue revolves mainly on whether petitioner was an employee or a corporate officer of Slimmers World.
 Section 25 of the Corporation Code enumerates corporate officers as the president, secretary, treasurer
and such other officers as may be provided for in the by-laws.
 In Tabang v. NLRC, we held that an office is created by the charter of the corporation and the officer is
elected by the directors or stockholders. On the other hand, an employee usually occupies no office and
generally is employed not by action of the directors or stockholders but by the managing officer of the
corporation who also determines the compensation to be paid to such employee.
 respondents attached the General Information Sheet (GIS) dated 14 April 1998, Minutes of the meeting of the
Board of Directors dated 14 April 1997 and Secretarys Certificate, and the Amended By-Laws dated 1 August
1994 of Slimmers World as submitted to the SEC to show that petitioner was a corporate officer whose rights
do not fall within the NLRCs jurisdiction.
 The GIS and minutes of the meeting of the board of directors indicated that petitioner was a member of the
board of directors, holding one subscribed share of the capital stock, and an elected corporate officer.
 Prior to its amendment, Section 5(c) of Presidential Decree No. 902-A[19] (PD 902-A) provided that intra-
corporate disputes fall within the jurisdiction of the Securities and Exchange Commission (SEC):
o Sec. 5. In addition to the regulatory and adjudicative functions of the Securities and Exchange Commission over
corporations, partnerships and other forms of associations registered with it as expressly granted under existing
laws and decrees, it shall have original and exclusive jurisdiction to hear and decide cases involving:
xx
c) Controversies in the election or appointments of directors, trustees, officers or managers of such
corporations, partnerships or associations.
 Subsection 5.2, Section 5 of Republic Act No. 8799, which took effect on 8 August 2000, transferred to regional
trial courts the SECs jurisdiction over all cases listed in Section 5 of PD 902-A:
o 5.2. The Commissions jurisdiction over all cases enumerated under Section 5 of Presidential Decree No. 902-A is
hereby transferred to the Courts of general jurisdiction or the appropriate Regional Trial Court.
xxx
 It is a settled rule that jurisdiction over the subject matter is conferred by law.
 The determination of the rights of a director and corporate officer dismissed from his employment as well as the
corresponding liability of a corporation, if any, is an intra-corporate dispute subject to the jurisdiction of the regular
courts. Thus, the appellate court correctly ruled that it is not the NLRC but the regular courts which have
jurisdiction over the present case.
 WHEREFORE, we DENY the petition. We AFFIRM

Angeles| Bajana | Balladares | Brillantes | Briones | Cabansag | Callanta | Chua | David|

De Leon | Gomez | Lopez | Macalino | Nostratis | Padilla | Reynon | Santos | Tan |Velasco
4E / 4F - 2018-2019
Page 246 of 920
LABOR REVIEW DIGEST
Atty. Joyrich Golangco

6. RURAL BANK OF CORON, ET AL CORPORATIONS v CORTES


G.R. No. 164888 December 6, 2006

Facts:
- Then still single Annalisa Cortes, respondent, was hired as clerk of the Rural Bank of Coron (Manila Office) by
Virgilio Garcia, "founder" of petitioner corporations
- Virgilio died, his son Victor took over the management of the corporations.
- Anita, the wife of Victor Garcia, was also involved in the management of the corporations.
- Respondent later married Anita’s brother Eduardo Cortes.
- Respondent later became the Financial Assistant, Personnel Officer and Corporate Secretary of The Rural Bank
of Coron, Personnel Officer of CDI, and also Personnel Officer and Disbursing Officer of The Empire Cold Storage
Development Corporation (ECSDC). She simultaneously received salaries from these corporations.
- Petitioner Sandra Garcia Escat, a daughter of Virgilio Garcia who was previously residing in Spain, examined the
financial books of the corporations and found out that respondent was involved in several anomalies, drawing
petitioners to terminate respondent’s services on November 23, 1998 in petitioner corporations.
- Respondent’s counsel conveyed respondent’s willingness to abide by the decision to terminate her but reminded
them that she was entitled to separation pay equivalent to 11 months salary as well as to the other benefits
provided by law in her favor.
- Respondent’s counsel thus demanded the payment of respondent’s unpaid salary for the months of October and
November 1998, separation pay equivalent to 12 months salary, 13th month pay and other benefits
- As the demand remained unheeded, respondent filed a complaint for illegal dismissal and non-payment of
salaries and other benefits, docketed as NLRC-NCR Case
- Petitioners moved for the dismissal of the complaint on the ground of lack of jurisdiction, contending that the case
was an intra-corporate controversy involving the removal of a corporate officer, respondent being the Corporate
Secretary of the Rural Bank of Coron, Inc., hence, cognizable by the Securities and Exchange Commission (SEC)
pursuant to Section 5 of PD 902-A
- Labor Arbiter noted as follows:
o It is to be noted that complainant, aside from her being Corporate Secretary of Rural Bank of Coron, complainant
was likewise appointed as Financial Assistant & Personnel Officer of all respondents herein, whose services
w[ere] terminated on 23 November 1998, hence, the instant complaint.
Verily, a Financial Assistant & Personnel Officer is not a Corporate Officer of the [petitioners’]
corporation, thus, pursuant to Article 217 of the Labor Code, as amended, the instant case falls within the ambit of
original and exclusive jurisdiction of this Office.
- August 13, 2001, the 10th or last day of the period of appeal, petitioners filed a Notice of Appeal and Motion for
Reduction of Bond to which they attached a Memorandum on Appeal.
- In their Motion for Reduction of Bond, petitioners alleged that the corporations were under financial distress and
the Rural Bank of Coron was under receivership. They thus prayed that the amount of bond be substantially
reduced, preferably to one half thereof or even lower.

- CRUX of the Case: By Resolution of October 16, 200113, the NLRC, while noting that petitioners timely filed the
appeal, held that the same was not accompanied by an appeal bond, a mandatory requirement under Article 223
of the Labor Code and Section 6, Rule VI of the NLRC New Rules of Procedure.
- It also noted that the Motion for Reduction of Bond was "premised on self-serving allegations." It accordingly
dismissed the appeal.
- Filed an MR but denied.
- The CA appellate court dismissed the petition for lack of merit.

ISSUE:
Angeles| Bajana | Balladares | Brillantes | Briones | Cabansag | Callanta | Chua | David|

De Leon | Gomez | Lopez | Macalino | Nostratis | Padilla | Reynon | Santos | Tan |Velasco
4E / 4F - 2018-2019
Page 247 of 920
LABOR REVIEW DIGEST
Atty. Joyrich Golangco

- Whether LA has jurisdiction.

HELD:
 While, indeed, respondent was the Corporate Secretary of the Rural Bank of Coron, she was also its Financial
Assistant and the Personnel Officer of the two other petitioner corporations.
 The Labor Arbiter has thus jurisdiction over respondent’s complaint.

ISSUE:
- whether petitioners’ appeal before the NLRC was perfected

HELD:
 As correctly ruled by the Court of Appeals, however, the cited cases of the petitioners are not in point.
o … The appellant in Taberrah filed a motion to fix appeal bond instead of posting an appeal bond; and the
Supreme Court relaxed the requirement considering that the labor arbiter’s decision did not contain a computation
of the monetary award.
o In Cosico, the appeal bond posted was of insufficient amount but the Supreme Court ruled that provisions of the
Labor Code on requiring a bond on appeal involving monetary awards must be given liberal interpretation in line
with the desired objective of resolving controversies on their merits.
 Herein, no appeal bond, whether sufficient or not, was ever filed by the petitioners.
 Petitioners invoke the aforementioned holding in Star Angel that there is a distinction between the filing of an
appeal within the reglementary period and its perfection, and that the appeal may be perfected after the
said reglementary period.
 Indeed, Star Angel held that the filing of a motion for reduction of appeal bond necessarily stays the reglementary
period for appeal. However, in this case, the motion for reduction of appeal bond, which was incorporated in
the appeal memorandum, was filed only on the tenth or final day of the reglementary period.
 Under such circumstance, the motion for reduction of appeal bond can no longer be deemed to have stayed
the appeal, and the petitioner faces the risk, as had happened in this case, of summary dismissal of the
appeal for non-perfection.
 Moreover, the reference in Star Angel to the distinction between the period to file the appeal and to perfect the
appeal has been pointedly made only once by this Court in Gensoli v. NLRC thus, it has not acquired the sheen
of venerability reserved for repeatedly-cited cases. The distinction, if any, is not particularly evident or material in
the Labor Code; hence, the reluctance of the Court to adopt such doctrine. Moreover, the present provision in
the NLRC Rules of Procedure, that "the filing of a motion to reduce bond shall not stop the running of the
period to perfect appeal" flatly contradicts the notion expressed in Star Angel that there is a distinction
between the filing an appeal and perfecting an appeal.
 Ultimately, the disposition of Star Angel was premised on the ruling that a motion for reduction of the appeal bond
necessarily stays the period for perfecting the appeal, and that the employer cannot be expected to perfect the
appeal by posting the proper bond until such time the said motion for reduction is resolved. The unduly
stretched-out distinction between the period to file an appeal and to perfect an appeal was not material to
the resolution of Star Angel, and this could be properly considered as obiter dictum.
 Petitioners also characterize the appeal bond requirement as a technical rule, and that the dismissal of an appeal
on purely technical grounds is frowned upon.
 However, Article 223, which prescribes the appeal bond requirement, is a rule of jurisdiction and not of
procedure. There is a little leeway for condoning a liberal interpretation thereof, and certainly none premised on
the ground that its requirements are mere technicalities.
 It must be emphasized that there is no inherent right to an appeal in a labor case, as it arises solely from grant of
statute, namely the Labor Code.
 We have indeed held that the requirement for posting the surety bond is not merely procedural but
jurisdictional and cannot be trifled with. Non-compliance with such legal requirements is fatal and has the effect
Angeles| Bajana | Balladares | Brillantes | Briones | Cabansag | Callanta | Chua | David|

De Leon | Gomez | Lopez | Macalino | Nostratis | Padilla | Reynon | Santos | Tan |Velasco
4E / 4F - 2018-2019
Page 248 of 920
LABOR REVIEW DIGEST
Atty. Joyrich Golangco

of rendering the judgment final and executory. The petitioners cannot be allowed to seek refuge in a liberal
application of rules for their act of negligence
 In the case at bar, petitioner did not post a full or partial appeal bond within the prescribed period, thus, no
appeal was perfected from the decision of the Labor Arbiter. For this reason, the decision sought to be
appealed to the NLRC had become final and executory and therefore immutable.
 Petition is Denied.

Angeles| Bajana | Balladares | Brillantes | Briones | Cabansag | Callanta | Chua | David|

De Leon | Gomez | Lopez | Macalino | Nostratis | Padilla | Reynon | Santos | Tan |Velasco
4E / 4F - 2018-2019
Page 249 of 920
LABOR REVIEW DIGEST
Atty. Joyrich Golangco

7. Patricia Halagueña v. Philippine Airlines


G.R. No. 172013; Oct. 2, 2009
Peralta, J.:
DOCTRINE: Not every dispute between an employer and employee involves matters that only labor arbiters and
the NLRC can resolve in the exercise of their adjudicatory or quasi-judicial powers. The jurisdiction of labor
arbiters and the NLRC under Article 217 of the Labor Code is limited to disputes arising from an employer-
employee relationship which can only be resolved by reference to the Labor Code, other labor statutes, or their
collective bargaining agreement.
Actions between employees and employer where the employer-employee relationship is merely incidental and the
cause of action precedes from a different source of obligation is within the exclusive jurisdiction of the regular
court.

FACTS:
Petitioner Patricia Halagueña and other petitioners are female flight attendants of respondent Philippine Airlines,
and members of the Flight Attendants and Stewards Association of the Philippines (FASAP) which was the
exclusive bargaining representative of the flight attendants. They were all employed on different dates prior to
Nov. 22, 1996.
In July 2001, respondent and FASAP entered into a CBA incorporating into its terms a provision providing for the
compulsory retirement at the age of 55 for females, and 60 for the males. July 2003, Petitioners and several
others wrote to respondent that the same was discriminatory and demanded equal treatment with their male
counterparts. July 2004, Dr. Andiuza, FASAP President, submitted proposed changes and manifested the
willingness to renegotiate the same. Petitioners thus filed a Special Civil Action for Declaratory Relief with a
Prayer for the Issuance of a TRO and Preliminary Injunction before the RTC of Makati to have Section 144
(Compulsory Retirement Provision) declared invalid.

RTC RULING: The RTC upheld its jurisdiction ruling that the allegations in the complaint do not make out a labor
dispute as the said allegations do not show that an employer-employee relationship exists. The RTC enjoined the
implementation of Sec. 144 via preliminary injunction. Respondent filed an MR but the same was denied.
CA RULING: A Petition for Certiorari Relief with a Prayer for the Issuance of a TRO and Preliminary Injunction
was filed before the CA praying that the decision of the court a quo upholding its own jurisdiction be annulled and
set aside. The CA sided with the petitioners and ruled that the RTC had no jurisdiction over the matter.
APPEAL TO THE SC:
Petitioner’s Contention: that the RTC has jurisdiction in all civil actions in which the subject of the litigation is
incapable of pecuniary estimation and in all cases not within the exclusive jurisdiction of any court, tribunal,
person or body exercising judicial or quasi-judicial functions.
Respondent’s Contention: that the labor tribunals have jurisdiction over the present case, as the controversy
partakes of a labor dispute. The dispute concerns the terms and conditions of petitioners' employment in PAL,
specifically their retirement age.
ISSUE: Whether the RTC jurisdiction.
RULING: YES. Jurisdiction of the court is determined on the basis of the material allegations of the complaint and
the character of the relief prayed for irrespective of whether plaintiff is entitled to such relief. the allegations in the
petition for declaratory relief plainly show that petitioners' cause of action is the annulment of Sec. 144,
questioning among others: validity of the same based on the Constitution, the Labor Code, Republic Act 6725 (An
Act Strengthening Prohibition on Discrimination Against Women), and the Convention of the Elimination of All
forms of Discrimination Against Women (CEDAW).
The primary issue raised is whether Sec. 144 is unlawful and or unconstitutional. The subject of litigation is
incapable of pecuniary estimation, exclusively cognizable by the RTC, pursuant to Section 19 (1) of Batas
Pambansa Blg. 129, as amended. Being an ordinary civil action, the same is beyond the jurisdiction of labor
tribunals. TIEHDC

Angeles| Bajana | Balladares | Brillantes | Briones | Cabansag | Callanta | Chua | David|

De Leon | Gomez | Lopez | Macalino | Nostratis | Padilla | Reynon | Santos | Tan |Velasco
4E / 4F - 2018-2019
Page 250 of 920
LABOR REVIEW DIGEST
Atty. Joyrich Golangco

The Court cited Gorg Grotjahn GMBH & Co. v. Isnani where it held that not every dispute between an employer
and employee involves matters that only labor arbiters and the NLRC can resolve in the exercise of their
adjudicatory or quasi-judicial powers. The jurisdiction of labor arbiters and the NLRC under Article 217 of the
Labor Code is limited to disputes arising from an employer-employee relationship which can only be resolved by
reference to the Labor Code, other labor statutes, or their collective bargaining agreement.
Actions between employees and employer where the employer-employee relationship is merely incidental and the
cause of action precedes from a different source of obligation is within the exclusive jurisdiction of the regular
court.
Thus, where the principal relief sought is to be resolved not by reference to the Labor Code or other labor
relations statute or a collective bargaining agreement but by the general civil law, the jurisdiction over the dispute
belongs to the regular courts of justice and not to the labor arbiter and the NLRC. In such situations, resolution of
the dispute requires expertise, not in labor management relations nor in wage structures and other terms and
conditions of employment, but rather in the application of the general civil law. Clearly, such claims fall outside the
area of competence or expertise ordinarily ascribed to labor arbiters and the NLRC and the rationale for granting
jurisdiction over such claims to these agencies disappears.
The Court held the grievance machinery and voluntary arbitrators do not have the power to determine and settle
the issues at hand. They have no jurisdiction and competence to decide constitutional issues relative to the
questioned compulsory retirement age. Their exercise of jurisdiction is futile, as it is like vesting power to someone
who cannot wield it.

Angeles| Bajana | Balladares | Brillantes | Briones | Cabansag | Callanta | Chua | David|

De Leon | Gomez | Lopez | Macalino | Nostratis | Padilla | Reynon | Santos | Tan |Velasco
4E / 4F - 2018-2019
Page 251 of 920
LABOR REVIEW DIGEST
Atty. Joyrich Golangco

8. Santiago v. CF Sharp Crew Management


G.R. No. 162419
July 10, 2007
Tinga, J.:

DOCTRINE: Thus, even before the start of any employer-employee relationship, contemporaneous with the
perfection of the employment contract was the birth of certain rights and obligations, the breach of which may give
rise to a cause of action against the erring party. The jurisdiction of labor arbiters is not limited to claims arising
from employer-employee relationships, but also includes those “by virtue of any law or contract involving Filipino
workers for overseas deployment including claims for actual, moral, exemplary and other forms of damages”
FACTS:
Paul Santiago was a seafarer for Smith Bell Management for about 5 years. On Feb 3, 1998 he signed a new
contract of employment for a duration of 9 months. A week before his scheduled departure, Capt. Pacifico
Fernandez, respondent’s VP sent a facsimile to the captain MSV Seaspread requesting that Santiago be denied
boarding because of a call from his wife and other anonymous callers saying that he would jump ship in Canada,
like his brother did in Japan.
Petitioner thus filed an action for illegal dismissal, with damages against respondent and foreign principal Cable
and Wireless Marine Ltd.
LA RULING: Ordered respondents to pay 7209USD plus 10% attorneys fees, payable in PHP at the exchange
rate prevailing at the time of payment, but dismissed all other claims.
NLRC RULING: The NLRC vacated the monetary awards and ruled that there was no valid employer-employee
relationship as the Standard Terms and Conditions Governing the Employment of Filipino Seafarers on Board
Ocean Going Vessels (POEA Standard Contract) provided the employment contract shall commence upon actual
departure of the seafarer from the airport or seaport at the point of hire and with a POEA-approved contract.
CA RULING: According to the appellate court, petitioner is not entitled to actual damages because damages are
not recoverable by a worker who was not deployed by his agency within the period prescribed in the POEA Rules.
It agreed with the NLRC's Finding that petitioner's non-deployment was a valid exercise of respondent's
management prerogative. It added that since petitioner had not departed from the Port of Manila, no employer-
employee relationship between the parties arose and any claim for damages against the so-called employer could
have no leg to stand on.
APPEAL TO THE SC
Petitioner’s Agrument: Hinged on the Migrant Worker’s Act providing that claims or disputes of the Overseas
Filipino Worker by virtue of a contract fall within the jurisdiction of the Labor Arbiter of the NLRC.
Respondent’s Argument: No jurisdiction.
ISSUE: Whether or not the Labor Arbiters have jurisdiction over the matter.
RULING: YES. There is no question that the parties entered into an employment contract on 3 February 1998,
whereby petitioner was contracted by respondent to render services on board "MSV Seaspread" for the
consideration of US$515.00 per month for nine (9) months, plus overtime pay.
At this junction a distinction must be made between the perfection of the employment contract and the
commencement of the employer-employee relationship. The perfection of the contract, which in this case
coincided with the date of execution thereof, occurred when petitioner and respondent agreed on the object and
the cause, as well as the rest of the terms and conditions therein. The commencement of the employer-employee
relationship, as earlier discussed, would have taken place had petitioner been actually deployed from the point of
hire. Thus, even before the start of any employer-employee relationship, contemporaneous with the perfection of
the employment contract was the birth of certain rights and obligations, the breach of which may give rise to a
cause of action against the erring party. Thus, if the reverse had happened, that is the seafarer failed or refused to
be deployed as agreed upon, he would be liable for damages.
The jurisdiction of labor arbiters is not limited to claims arising from employer-employee relationships, but also
includes those “by virtue of any law or contract involving Filipino workers for overseas deployment including
claims for actual, moral, exemplary and other forms of damages”. Thus, even before the start of any employer-
Angeles| Bajana | Balladares | Brillantes | Briones | Cabansag | Callanta | Chua | David|

De Leon | Gomez | Lopez | Macalino | Nostratis | Padilla | Reynon | Santos | Tan |Velasco
4E / 4F - 2018-2019
Page 252 of 920
LABOR REVIEW DIGEST
Atty. Joyrich Golangco

employee relationship, contemporaneous with the perfection of the employment contract was the birth of certain
rights and obligations, the breach of which may give rise to a cause of action against the erring party.

Angeles| Bajana | Balladares | Brillantes | Briones | Cabansag | Callanta | Chua | David|

De Leon | Gomez | Lopez | Macalino | Nostratis | Padilla | Reynon | Santos | Tan |Velasco
4E / 4F - 2018-2019
Page 253 of 920
LABOR REVIEW DIGEST
Atty. Joyrich Golangco

9. Atlas Farms Inc. vs NLRC


GR No: 142244 Date: November 18, 2002
Ponente: Quisumbing, J.

Doctrine:
Where the dispute is just in the interpretation, implementation or enforcement stage, it may be
referred to the grievance machinery set up in the CBA, or brought to voluntary arbitration. But, where
there was already actual termination, with alleged violation of the employees rights, it is already
cognizable by the labor arbiter.

FACTS:

Private respondent Jaime O. dela Pea was employed as a veterinary aide by petitioner. He was among
several employees terminated in July 1989. On July 8, 1989, he was re-hired by petitioner and given the
additional job of feedmill operator. He was instructed to train selected workers to operate the feedmill. Co-
respondent Marcial I. Abion was a carpenter/mason and a maintenance man whose employment by petitioner.
In 1993, Pea was allegedly caught urinating and defecating on company premises not intended for the
purpose. The farm manager of petitioner issued a formal notice directing him to explain within 24 hours why
disciplinary action should not be taken against him. Pea refused, however, to receive the formal notice. He never
bothered to explain. Thus, a notice of termination with payment of his monetary benefits was sent to him.
On the other hand Abion allegedly, caused the clogging of the fishpond drainage resulting in damages
worth several hundred thousand pesos when he improperly disposed of the cut grass and other waste materials
into the ponds drainage system. Petitioner sent a written notice to Abion, requiring him to explain what happened,
otherwise, disciplinary action would be taken against him. He refused to receive the notice and give an
explanation, according to petitioner. Consequently, the company terminated his services. He acknowledged
receipt of a written notice of dismissal, with his separation pay.
Pea and Abion filed separate complaints for illegal dismissal that were later consolidated. Both claimed
that their termination from service was due to petitioners suspicion that they were the leaders in a plan to form a
union to compete and replace the existing management-dominated union.

LA/RTC/NLRC RULING:
LA: The labor arbiter dismissed their complaints on the ground that the grievance machinery in the
collective bargaining agreement (CBA) had not yet been exhausted. Private respondents availed of the grievance
process, but later on refiled the case before the NLRC in Region IV. They alleged lack of sympathy on petitioners
part to engage in conciliation proceedings.
In a decision dated January 30, 1996, the labor arbiter dismissed the complaint for lack of merit, finding
that the case was one of illegal dismissal and did not involve the interpretation or implementation of any CBA
provision. He stated that Article 217 (c) of the Labor Code was inapplicable to the case. Further, the labor arbiter
found that although both complainants did not substantiate their claims of illegal dismissal, there was proof that
private respondents voluntarily accepted their separation pay and petitioners financial assistance.
NLRC: Reversed the LA decision

CA RULING:
The appellate court denied the petition and affirmed the NLRC resolution with some modifications, thus: 1) The
private respondents can not be reinstated, due to their acceptance of the separation pay offered by the petitioner;
2) The private respondents are entitled to their full back wages; and, 3) The amount of the separation pay
received by private respondents from petitioner shall not be deducted from their full back wages.
An MR was filed by herein petitioner but was denied by the CA hence the current petition before the SC.

Angeles| Bajana | Balladares | Brillantes | Briones | Cabansag | Callanta | Chua | David|

De Leon | Gomez | Lopez | Macalino | Nostratis | Padilla | Reynon | Santos | Tan |Velasco
4E / 4F - 2018-2019
Page 254 of 920
LABOR REVIEW DIGEST
Atty. Joyrich Golangco

APPEAL TO THE SC:

Petitioner's Contention:
Petitioner contends that the dismissal of private respondents was for a just and valid cause, pursuant to
the provisions of the companys rules and regulations. It also alleges lack of jurisdiction on the part of the labor
arbiter, claiming that the cases should have been resolved through the grievance machinery, and eventually
referred to voluntary arbitration, as prescribed in the CBA.

Respondent's Contention:
For their part, private respondents contend that they were illegally dismissed from employment because
management discovered that they intended to form another union, and because they were vocal in asserting their
rights. In any case, according to private respondents, the petition involves factual issues that cannot be properly
raised in a petition for review on certiorari under Rule 45 of the Revised Rules of Court.

ISSUE/S:
1. Whether private respondents were legally and validly dismissed
2. Whether the labor arbiter and the NLRC had jurisdiction to decide complaints for illegal dismissal

HELD:
1. NO.
The first issue primarily involves questions of fact, which can serve as basis for the conclusion that private
respondents were legally and validly dismissed. The burden of proving that the dismissal of private respondents
was legal and valid falls upon petitioner. The NLRC found that petitioner failed to substantiate its claim that both
private respondents committed certain acts that violated company rules and regulations, hence we find no factual
basis to say that private respondents dismissal was in order. We see no compelling reason to deviate from the
NLRC ruling that their dismissal was illegal, absent a showing that it reached its conclusion arbitrarily. Moreover,
factual findings of agencies exercising quasi-judicial functions are accorded not only respect but even finality,
aside from the consideration here that this Court is not a trier of facts.

2. YES.
As held in Vivero vs. CA, petitioner cannot arrogate into the powers of Voluntary Arbitrators the original
and exclusive jurisdiction of Labor Arbiters over unfair labor practices, termination disputes, and claims for
damages, in the absence of an express agreement between the parties in order for Article 262 of the Labor Code
[Jurisdiction over other labor disputes] to apply in the case at bar.

Records show, that private respondents sought without success to avail of the grievance procedure in
their CBA.[16] On this point, petitioner maintains that by so doing, private respondents recognized that their cases
still fell under the grievance machinery. According to petitioner, without having exhausted said machinery, the
private respondents filed their action before the NLRC, in a clear act of forum-shopping. However, it is worth
pointing out that private respondents went to the NLRC only after the labor arbiter dismissed their original
complaint for illegal dismissal. Under these circumstances private respondents had to find another
avenue for redress. We agree with the NLRC that it was petitioner who failed to show proof that it took steps to
convene the grievance machinery after the labor arbiter first dismissed the complaints for illegal dismissal and
directed the parties to avail of the grievance procedure under Article VII of the existing CBA. They could not now
be faulted for attempting to find an impartial forum, after petitioner failed to listen to them and after the
intercession of the labor arbiter proved futile. The NLRC had aptly concluded in part that private respondents had
already exhausted the remedies under the grievance procedure. It erred only in finding that their cause of action
was ripe for arbitration.
Angeles| Bajana | Balladares | Brillantes | Briones | Cabansag | Callanta | Chua | David|

De Leon | Gomez | Lopez | Macalino | Nostratis | Padilla | Reynon | Santos | Tan |Velasco
4E / 4F - 2018-2019
Page 255 of 920
LABOR REVIEW DIGEST
Atty. Joyrich Golangco

Coming to the merits of the petition, the NLRC found that petitioner did not comply with the requirements
of a valid dismissal. For a dismissal to be valid, the employer must show that: (1) the employee was
accorded due process, and (2) the dismissal must be for any of the valid causes provided for by law. No
evidence was shown that private respondents refused, as alleged, to receive the notices requiring them to show
cause why no disciplinary action should be taken against them. Without proof of notice, private respondents
who were subsequently dismissed without hearing were also deprived of a chance to air their side at the
level of the grievance machinery. Given the fact of dismissal, it can be said that the cases were effectively
removed from the jurisdiction of the voluntary arbitrator, thus placing them within the jurisdiction of the
labor arbiter. Where the dispute is just in the interpretation, implementation or enforcement stage, it may
be referred to the grievance machinery set up in the CBA, or brought to voluntary arbitration. But, where
there was already actual termination, with alleged violation of the employees rights, it is already
cognizable by the labor arbiter.

Other Notes/ SC Pronouncements:


_________________________________________________________________________

Angeles| Bajana | Balladares | Brillantes | Briones | Cabansag | Callanta | Chua | David|

De Leon | Gomez | Lopez | Macalino | Nostratis | Padilla | Reynon | Santos | Tan |Velasco
4E / 4F - 2018-2019
Page 256 of 920
LABOR REVIEW DIGEST
Atty. Joyrich Golangco

10. Perpetual Help Credit Cooperative Inc. vs Faburada


GR No: 121948 Date: October 8, 2001
Ponente: Sandoval-Gutierrez, J.

Doctrine: (main doctrine under the article)


With respect to the issue on jurisdiction, the Court held that disputes about payment of wages, overtime pay, rest
day and termination of employment are within the original and exclusive jurisdiction of the labor arbiter.

FACTS:

On January 3, 1990, Benedicto Faburada, Sisinita Vilar, Imelda Tamayo and Harold Catipay, private
respondents, filed a complaint against the Perpetual Help Credit Cooperative, Inc. (PHCCI), petitioner, with the
Arbitration Branch, Department of Labor and Employment (DOLE), Dumaguete City, for illegal dismissal, premium
pay on holidays and rest days, separation pay, wage differential, moral damages, and attorney's fees.

Respondents worked regularly on regular working hours, were assigned specific duties, were paid
regular wages and made to accomplish daily time records just like any other regular employee. They worked
under the supervision of the cooperative manager. But unfortunately, they were dismissed.

Forthwith, petitioner PHCCI filed a motion to dismiss the complaint on the ground that there is no
employer-employee relationship between them as private respondents are all members and co-owners of the
cooperative. Furthermore, private respondents have not exhausted the remedies provided in the cooperative by-
laws. On September 3, 1990, petitioner filed a supplemental motion to dismiss alleging that Article 121 of R.A. No.
6939, otherwise known as the Cooperative Development Authority Law which took effect on March 26, 1990,
requires conciliation or mediation within the cooperative before a resort to judicial proceeding.

LA/RTC/NLRC RULING:
LA: denied to motion to dismiss, holding that the case is impressed with employer-employee relationship and that
the law on cooperatives is subservient to the Labor Code. The LA declared that respondents were illegally
dismissed.
NLRC: AFFIRMED the decision of the LA

APPEAL TO THE SC:


Petitioner filed a petition for Certiorari assailing the decision of the NLRC which affirmed the LA
decision

Petitioner's Contention:
Petitioner contended that private respondents were mere volunteer workers, not regular employees.
Thus, they cannot sue them. Petitioner also questioned the jurisdiction of the Labor Arbiter.
Respondent's Contention:
Respondents’ contention was that they were regular employees of petitioner and were illegally dismissed
by the latter

ISSUE/S:
1. Whether or not there was employer employee relationship between the parties
2. Whether or not the LA has jurisdiction over the case

HELD:
1. YES.

Angeles| Bajana | Balladares | Brillantes | Briones | Cabansag | Callanta | Chua | David|

De Leon | Gomez | Lopez | Macalino | Nostratis | Padilla | Reynon | Santos | Tan |Velasco
4E / 4F - 2018-2019
Page 257 of 920
LABOR REVIEW DIGEST
Atty. Joyrich Golangco

In determining the existence of an employer-employee relationship, the following elements are


considered: (1) the selection and engagement of the worker or the power to
hire; (2) the power to dismiss; (3) the payment of wages by whatever means; and (4) the
power to control the worker's conduct, with the latter assuming primacy in the overall consideration. No particular
form of proof is required to prove the existence of an employer-employee relationship. Any competent and
relevant evidence may show the
relationship.

Necessarily, this leads us to the issue of whether or not private respondents are regular employees.
Article 280 of the Labor Code provides for three kinds of employees: (1) regular employees or those who have
been engaged to perform activities which are usually necessary or desirable in the usual business or trade of the
employer; (2) project employees or those whose employment has been fixed for a specific project or undertaking,
the completion or termination of which has been determined at the time of the engagement of the employee or
where the work or service to be performed is seasonal
in nature and the employment is for the duration of the season; and (3) casual employees
or those who are neither regular nor project employees. The employees who are deemed
regular are: (a) those who have been engaged to perform activities which are usually
necessary or desirable in the usual trade or business of the employer; and (b) those casual employees
who have rendered at least one (1) year of service, whether such service is continuous or broken, with
respect to the activity in which they are employed. Undeniably, private respondents were rendering services
necessary to the day-to-day operations of petitioner PHCCI. This fact alone qualified them as regular employees.

All of them, except Harold D. Catipay, worked with petitioner for more than one (1) year: Benedicto
Faburada, for one and a half (1 1/2) years; Sisinita Vilar, for two (2) years; and Imelda C. Tamayo, for two (2)
years and two (2) months. That Benedicto Faburada worked only on a part-time basis, does not mean that he is
not a regular employee. One's regularity of employment is not determined by the number of hours one works but
by the nature and by the length of time one has been in that particular job. Petitioner's contention that private
respondents are mere volunteer workers, not regular employees, must necessarily fail. Its invocation of San Jose
City Electric Cooperative vs. Ministry of Labor and Employment (173 SCRA 697, 703 (1989) is misplaced. The
issue in this case is whether or not the employees-members of a cooperative can organize themselves for
purposes of collective bargaining, not whether or not the members can be employees. Petitioner missed the point.
As regular employees or workers, private respondents are entitled to security of
tenure. Thus, their services may be terminated only for a valid cause, with observance of due process.

2. YES.
As aptly stated by the Solicitor General in his comment, P.D. 175 does not provide for a grievance
machinery where a dispute or claim may first be submitted. LOI 23 refers to instructions to the Secretary of Public
Works and Communications to implement immediately the recommendation of the Postmaster General for the
dismissal of some employees of the Bureau of Post. Obviously, this LOI has no relevance to the instant case.
Article 121 of Republic Act No. 6938 (Cooperative Code of the Philippines) provides the procedure how
cooperative disputes are to be resolved, thus:
"ART. 121. Settlement of Disputes. — Disputes among members, officers, directors, and committee members,
and intra-cooperative disputes shall, as far as practicable, be settled amicably in accordance with the conciliation
or mediation mechanisms embodied in the bylaws of the cooperative, and in applicable laws.
Should such a conciliation/mediation proceeding fail, the matter shall be settled in a court of competent
jurisdiction."

Complementing this Article is Section 8 of R.A. No. 6939 (Cooperative Development Authority Law)
which reads:

Angeles| Bajana | Balladares | Brillantes | Briones | Cabansag | Callanta | Chua | David|

De Leon | Gomez | Lopez | Macalino | Nostratis | Padilla | Reynon | Santos | Tan |Velasco
4E / 4F - 2018-2019
Page 258 of 920
LABOR REVIEW DIGEST
Atty. Joyrich Golangco

SEC. 8 Mediation and Conciliation. — Upon request of either or both parties, the Authority shall mediate and
conciliate disputes within a cooperative or between cooperatives: Provided, That if no mediation or conciliation
succeeds within three (3) months from request thereof, a certificate of non-resolution shall be issued by the
Commission prior to the filing of appropriate action before the proper courts.

The above provisions apply to members, officers and directors of the cooperative
involved in disputes within a cooperative or between cooperatives.
There is no evidence that private respondents are members of petitioner PHCCI and even if they
are, the dispute is about payment of wages, overtime pay, rest day and termination of employment. Under
Art. 217 of the Labor Code, these disputes are within the original and exclusive jurisdiction of the Labor
Arbiter.

As illegally dismissed employees, private respondents are therefore entitled to reinstatement without loss
of seniority rights and other privileges and to full backwages, inclusive of allowances, plus other benefits or their
monetary equivalent computed from the time their compensation was withheld from them up to the time of their
actual reinstatement. Since they were dismissed after March 21, 1989, the effectivity date of R.A. 6715 10 they
are granted full backwages, meaning, without deducting from their backwages the earnings derived by them
elsewhere during the period of their illegal dismissal. If reinstatement is no longer feasible, as when the
relationship between petitioner and private respondents has become strained, payment of their separation pay in
lieu of reinstatement is in order.
__________________________________________________________________

Angeles| Bajana | Balladares | Brillantes | Briones | Cabansag | Callanta | Chua | David|

De Leon | Gomez | Lopez | Macalino | Nostratis | Padilla | Reynon | Santos | Tan |Velasco
4E / 4F - 2018-2019
Page 259 of 920
LABOR REVIEW DIGEST
Atty. Joyrich Golangco

11. Pastor Dionisio V. Austria vs. National Labor Relations Commission (Fourth Division)
GR No: 124382 Date: August 16, 1999
Ponente: Kapunan, J.

Doctrine:

An ecclesiastical affair is "one that concerns doctrine, creed, or form of worship of the church, or the adoption and
enforcement within a religious association of needful laws and regulations for the government of the membership,
and the power of excluding from such associations those deemed unworthy of membership." Based on this
definition, an ecclesiastical affair involves the relationship between the church and its members and relate to
matters of faith, religious doctrines, worship and governance of the congregation. To be concrete, examples of
this so-called ecclesiastical affairs to which the State cannot meddle are proceedings for excommunication,
ordinations of religious ministers, administration of sacraments and other activities with attached religious
significance.

The case at bar does not concern an ecclesiastical or purely religious affair as to bar the State from taking
cognizance of the same. While the matter at hand relates to the church and its religious minister it does not ipso
facto give the case a religious significance. Simply stated, what is involved here is the relationship of the church
as an employer and the minister as an employee. It is purely secular and has no relation whatsoever with the
practice of faith, worship or doctrines of the church. In this case, petitioner was not excommunicated or expelled
from the membership of the SDA but was terminated from employment. Indeed, the matter of terminating an
employee, which is purely secular in nature, is different from the ecclesiastical act of expelling a member from the
religious congregation.

FACTS:

Private Respondent Central Philippine Union Mission Corporation of the Seventh-Day Adventists (SDA) is a
religious corporation represented in this case by the other private respondents, officers of the SDA. Petitioner
Dionisio V. Austria was a district pastor of the SDA until his services were terminated.

Petitioner was asked to admit accountability and responsibility for the church tithes and offerings collected by his
wife, Mrs. Thelma Austria. Petitioner refused since it was private respondents Pastor Gideon Buhat and Mr.
Eufronio Ibesate who authorized his wife to collect.

Thereafter, petitioner went to the office of Pastor Buhat, the president of the Negros Mission to persuade him to
convene the Executive Committee to settle his dispute with private respondent Pastor David Rodrigo. Pastor
Buhat denied the request since some committee members were out of town and there was no
quorum. Thereafter, the two exchanged heated arguments.

A fact-finding committee was created to investigate petitioner. Petitioner later received a letter of dismissal citing
misappropriation of denominational funds, willful breach of trust, serious misconduct, gross and habitual neglect of
duties, and commission of an offense against the person of employers duly authorized representative, as grounds
for the termination of his services.

Petitioner then filed a complaint before the Labor Arbiter for illegal dismissal against the SDA and its officers and
prayed for reinstatement with backwages and benefits, moral and exemplary damages and other labor law
benefits.

LA/RTC/NLRC RULING:

Angeles| Bajana | Balladares | Brillantes | Briones | Cabansag | Callanta | Chua | David|

De Leon | Gomez | Lopez | Macalino | Nostratis | Padilla | Reynon | Santos | Tan |Velasco
4E / 4F - 2018-2019
Page 260 of 920
LABOR REVIEW DIGEST
Atty. Joyrich Golangco

Labor Arbiter Cesar D. Sideo rendered a decision in favor of petitioner ordering respondents to immediately
reinstate complainant Pastor Dionisio Austria to his former position

The NLRC initially vacated the findings of the Labor Arbiter and dismissed the case for want of merit. However,
upon a motion for reconsideration, the NLRC reinstated the decision of the Labor Arbiter. The NLRC, without
ruling on the merits of the case, reversed itself once again, sustained the argument posed by private respondents
that the Labor Arbiter has no jurisdiction over the complaint filed by petitioner due to the constitutional provision on
the separation of church and state since the case allegedly involved and ecclesiastical affair to which the State
cannot interfere. and, accordingly, dismissed the complaint of petitioner.

APPEAL TO THE SC:

Petition for certiorari under Rule 65 assailing the dismissal of the case for illegal dismissal filed by petitioner
against private respondents for lack of jurisdiction.

Respondent's Contention:

Private respondents contend that by virtue of the doctrine of separation of church and state, the Labor Arbiter and
the NLRC have no jurisdiction to entertain the complaint filed by petitioner. Since the matter at bar allegedly
involves the discipline of a religious minister, it is to be considered a purely ecclesiastical affair to which the State
has no right to interfere.

ISSUE/S:
1. Whether or not the termination of the services of petitioner is an ecclesiastical affair, and, as such, involves the
separation of church and state.
2. Whether or not such termination is valid.

HELD:
1. No. The case at bar does not concern an ecclesiastical or purely religious affair as to bar the State from taking
cognizance of the same.

An ecclesiastical affair is one that concerns doctrine, creed, or form or worship of the church, or the adoption and
enforcement within a religious association of needful laws and regulations for the government of the membership,
and the power of excluding from such associations those deemed unworthy of membership. Based on this
definition, an ecclesiastical affair involves the relationship between the church and its members and relate to
matters of faith, religious doctrines, worship and governance of the congregation. To be concrete, examples of
this so-called ecclesiastical affairs to which the State cannot meddle are proceedings for excommunication,
ordinations of religious ministers, administration of sacraments and other activities with which attached religious
significance. The case at bar does not even remotely concern any of the abovecited examples. While the matter
at hand relates to the church and its religious minister it does not ipso facto give the case a religious
significance. Simply stated, what is involved here is the relationship of the church as an employer and the
minister as an employee. It is purely secular and has no relation whatsoever with the practice of faith, worship or
doctrines of the church.

It is clear that when the SDA terminated the services of petitioner, it was merely exercising its management
prerogative to fire an employee which it believes to be unfit for the job. As such, the State, through
the Labor Arbiter and the NLRC, has the right to take cognizance of the case and to determine whether the SDA,
as employer, rightfully exercised its management prerogative to dismiss an employee. This is in consonance with
the mandate of the Constitution to afford full protection to labor.

Angeles| Bajana | Balladares | Brillantes | Briones | Cabansag | Callanta | Chua | David|

De Leon | Gomez | Lopez | Macalino | Nostratis | Padilla | Reynon | Santos | Tan |Velasco
4E / 4F - 2018-2019
Page 261 of 920
LABOR REVIEW DIGEST
Atty. Joyrich Golangco

Under the Labor Code, the provision which governs the dismissal of employees, is comprehensive enough to
include religious corporations, such as the SDA, in its coverage. Article 278 of the Labor Code on post-
employment states that "the provisions of this Title shall apply to all establishments or undertakings, whether for
profit or not." Obviously, the cited article does not make any exception in favor of a religious corporation.

2. NO. Petitioner was terminated from service without just or lawful cause.

The requisites for a valid dismissal are: (a) the employee must be afforded due process, i.e., he must be given an
opportunity to be heard and to defend himself, and; (b) the dismissal must be for a valid cause as provided in
Article 282 of the Labor Code. Without the concurrence of this twin requirements, the termination would, in the
eyes of the law, be illegal.

Before the services of an employee can be validly terminated, Article 277 (b) of the Labor Code and Section 2,
Rule XXIII, Book V of the Rules Implementing the Labor Code further require the employer to furnish the
employee with two (2) written notices, to wit: (a) a written notice served on the employee specifying the ground or
grounds for termination, and giving to said employee reasonable opportunity within which to explain his side; and,
(b) a written notice of termination served on the employee indicating that upon due consideration of all the
circumstances, grounds have been established to justify his termination.

Private respondent failed to substantially comply with the above requirements. With regard to the first notice, the
letter which notified petitioner and his wife to attend the meeting cannot be construed as the written charge
required by law. The said letter never categorically stated the particular acts or omissions on which petitioner's
impending termination was grounded. In fact, the letter never even mentioned that petitioner would be subject to
investigation.

While admittedly, private respondents complied with the second requirement, the notice of termination, this does
not cure the initial defect of lack of the proper written charge required by law.

In the letter of termination, private respondents enumerated the following as grounds for the dismissal of
petitioner, namely: misappropriation of denominational funds, willful breach of trust, serious misconduct, gross and
habitual neglect of duties, and commission of an offense against the person of employer's duly authorized
representative.

a. The validity of the dismissal based on the ground of willful breach of trust cannot be sustained.

Settled is the rule that under Article 282 (c) of the Labor Code, the breach of trust must be willful. A breach is
willful if it is done intentionally, knowingly and purposely, without justifiable excuse, as distinguished from an act
done carelessly, thoughtlessly, heedlessly or inadvertently. It must rest on substantial grounds and not on the
employer's arbitrariness, whims, caprices or suspicion; otherwise, the employee would eternally remain at the
mercy of the employer.

The records show that there were only six (6) instances when petitioner personally collected and received from
the church treasurers the tithes, collections, and donations for the church. The stenographic notes of the Auditor
and a witness for private respondents, show that Pastor Austria was able to remit all his collections to the
treasurer.

b. The grounds of serious misconduct and commission of an offense against the person of the employer’s
duly authorized representative are unmeritorious and do not warrant petitioner’s dismissal from the service.

Angeles| Bajana | Balladares | Brillantes | Briones | Cabansag | Callanta | Chua | David|

De Leon | Gomez | Lopez | Macalino | Nostratis | Padilla | Reynon | Santos | Tan |Velasco
4E / 4F - 2018-2019
Page 262 of 920
LABOR REVIEW DIGEST
Atty. Joyrich Golangco

Misconduct has been defined as improper or wrong conduct. It is the transgression of some established and
definite rule of action, a forbidden act, a dereliction of duty, willful in character, and implies wrongful intent and not
mere error in judgment. For misconduct to be considered serious it must be of such grave and aggravated
character and not merely trivial or unimportant. Based on this standard, the act of petitioner in banging the attaché
case on the table, throwing the telephone and scattering the books in the office of Pastor Buhat, although
improper, cannot be considered as grave enough to be considered as serious misconduct. After all, though
petitioner committed damage to property, he did not physically assault Pastor Buhat or any other pastor.

c. Petitioner did not commit gross and habitual neglect of duties.

Private respondents failed to prove culpability on the part of petitioner. Petitioner's rise from the ranks disclose
that he was actually a hard-worker. Private respondents' evidence, which consisted of petitioner's Worker's
Reports, revealed how petitioner travelled to different churches to attend to the faithful under his care. Indeed, he
labored hard for the SDA, but, in return, he was rewarded with a dismissal from the service for a non-existent
cause.

Angeles| Bajana | Balladares | Brillantes | Briones | Cabansag | Callanta | Chua | David|

De Leon | Gomez | Lopez | Macalino | Nostratis | Padilla | Reynon | Santos | Tan |Velasco
4E / 4F - 2018-2019
Page 263 of 920
LABOR REVIEW DIGEST
Atty. Joyrich Golangco

12. Department of Foreign Affairs v. NLRC


GR No: 113191 Date: September 18, 1996
Ponente: Vitug, J.

Doctrine:

The stipulations of the Charter and Headquarters Agreement should be able to establish that, except in the
specified cases of borrowing and guarantee operations, as well as the purchase, sale and underwriting of
securities, the ADB enjoys immunity from legal process of every form. The Bank's officers, on their part, enjoy
Angeles| Bajana | Balladares | Brillantes | Briones | Cabansag | Callanta | Chua | David|

De Leon | Gomez | Lopez | Macalino | Nostratis | Padilla | Reynon | Santos | Tan |Velasco
4E / 4F - 2018-2019
Page 264 of 920
LABOR REVIEW DIGEST
Atty. Joyrich Golangco

immunity in respect of all acts performed by them in their official capacity. The Charter and the Headquarters
Agreement granting these immunities and privileges are treaty covenants and commitments voluntarily assumed
by the Philippine government which must be respected.

FACTS:

Private respondent Jose C. Magnayi initiated a case for illegal dismissal against Asian Development Bank and the
latter’s violation of the “labor-only” contracting law. ADB and DFA notified respondent Labor Arbiter that the ADB,
as well as its officers, were covered by an immunity from legal process except for borrowings, guaranties or the
sale of securities.

The Labor Arbiter took cognizance of the complaint on the impression that the ADB had waived its diplomatic
immunity from suit and rendered a decision declaring the complainant as a regular employee and the termination
of his services as illegal.

The DFA referred the matter to the NLRC. The latter denied request for investigation considering that the dispute
was within the jurisdiction of Labor Arbiters. Dissatisfied, the DFA lodged the instant petition for certiorari.

ISSUE:
Whether the dispute is covered by the diplomatic immunity extended to ADB as an international organization.

HELD:
YES. The ADB was correct in invoking its immunity from suit under the Charter and the Headquarters Agreement.

Article 50(1) of the Charter provides:


"The Bank shall enjoy immunity from every form of legal process, except in cases arising out of or in
connection with the exercise of its powers to borrow money, to guarantee obligations, or to buy and sell or
underwrite the sale of securities."

Under Article 55 thereof —


"All Governors, Directors, alternates, officers and employees of the Bank, including experts performing
missions for the Bank: "(1) shall be immune from legal process with respect of acts performed by them in their
official capacity, except when the Bank waives the immunity."

Like provisions are found in the Headquarters Agreement. Thus, its Section 5 reads:
"The Bank shall enjoy immunity from every form of legal process, except in cases arising out of, or in
connection with, the exercise of its powers to borrow money, to guarantee obligations, or to buy and sell or
underwrite the sale of securities."

And, with respect to certain officials of the bank, Section 44 of the agreement states:
"Governors, other representatives of Members, Directors, the President, Vice-President and executive
officers as may be agreed upon between the Government and the Bank shall enjoy, during their stay in the
Republic of the Philippines in connection with their official duties with the Bank:
"(b) Immunity from legal process of every kind in respect of words spoken or written and all acts done by
them in their official capacity."

The above stipulations of both the Charter and Headquarters Agreement should be able to establish that, except
in the specified cases of borrowing and guarantee operations, as well as the purchase, sale and underwriting of
securities, the ADB enjoys immunity from legal process of every form. The Bank's officers, on their part, enjoy
immunity in respect of all acts performed by them in their official capacity. The Charter and the Headquarters
Angeles| Bajana | Balladares | Brillantes | Briones | Cabansag | Callanta | Chua | David|

De Leon | Gomez | Lopez | Macalino | Nostratis | Padilla | Reynon | Santos | Tan |Velasco
4E / 4F - 2018-2019
Page 265 of 920
LABOR REVIEW DIGEST
Atty. Joyrich Golangco

Agreement granting these immunities and privileges are treaty covenants and commitments voluntarily assumed
by the Philippine government which must be respected.

Angeles| Bajana | Balladares | Brillantes | Briones | Cabansag | Callanta | Chua | David|

De Leon | Gomez | Lopez | Macalino | Nostratis | Padilla | Reynon | Santos | Tan |Velasco
4E / 4F - 2018-2019
Page 266 of 920
LABOR REVIEW DIGEST
Atty. Joyrich Golangco

13. PNB v Florence Cabansag


GR No: 157010 Date: June 21, 2005
Ponente: PANGANIBAN, J.:

Doctrine:
The Court reiterates the basic policy that all Filipino workers, whether employed locally or overseas, enjoy the
protective mantle of Philippine labor and social legislations. Our labor statutes may not be rendered ineffective by
laws or judgments promulgated, or stipulations agreed upon, in a foreign country.

FACTS:
Florence Cabansag (Cabansag) arrived in Singapore as a tourist. She applied for employment as Branch Credit
Officer, with the Singapore Branch of the PNB, a private banking corporation organized and existing under the
laws of the Philippines and was hired. Cabansag was issued an Employment Pass by the Ministry of manpower of
Singapore and an overseas employment certificate issued by the POEA thru the Phil embassy in Singapore.
Barely 3 months in office, Cabansag was told several times to resign by the general manager of the bank for
reasons such as cost-cutting measure of the bank and that the bank needed a Chinese-speaking credit officer but
Cabansag refuse to submit her letter of resignation. On April 20, 1999, she received a letter from the general
manager terminating her employment with the Bank.
Cabansag filed with the LA of QC a case for illegal dismissal.

LA/RTC/NLRC RULING:
LA ruled that PNB was guilty of Illegal dismissal and devoid of due process. Upon appeal, the NLRC affirmed the
decision, but reduced the moral damages to P100,000 and the exemplary damages to P50,000.

CA RULING:
The appellate court found that the Contract had actually been processed by the Philippine Embassy in Singapore
and approved by the Philippine Overseas Employment Administration (POEA), which then used that Contract as a
basis for issuing an Overseas Employment Certificate in favor of respondent. According to the CA, even though
respondent secured an employment pass from the Singapore Ministry of Employment, she did not thereby waive
Philippine labor laws, or the jurisdiction of the labor arbiter or the NLRC over her Complaint for illegal dismissal.
Finally, the CA held that petitioner had failed to establish a just cause for the dismissal of respondent. The bank
had also failed to give her sufficient notice and an opportunity to be heard and to defend herself. The CA ruled
that she was consequently entitled to reinstatement and back wages, computed from the time of her dismissal up
to the time of her reinstatement.

APPEAL TO THE SC:


Petitioner's Contention:

Respondent's Contention:

(Only issues and pertinent rulings were discussed.)

ISSUE/S:
1. Whether or not the arbitration branch of the NLRC in the National Capital Region has jurisdiction over the
instant controversy
2. Whether or not the respondent was illegally dismissed

HELD:
1. Yes, the NLRC has jurisdiction over the controversy as provided under Art 217(now 224) of the Labor and
Section 10 of RA 8042. Based on the foregoing provisions, labor arbiters clearly have original and exclusive
Angeles| Bajana | Balladares | Brillantes | Briones | Cabansag | Callanta | Chua | David|

De Leon | Gomez | Lopez | Macalino | Nostratis | Padilla | Reynon | Santos | Tan |Velasco
4E / 4F - 2018-2019
Page 267 of 920
LABOR REVIEW DIGEST
Atty. Joyrich Golangco

jurisdiction over claims arising from employer-employee relations, including termination disputes involving all
workers, among whom are overseas Filipino workers (OFW). Whether employed locally or overseas, all Filipino
workers enjoy the protective mantle of Philippine labor and social legislation, contract stipulations to the contrary
notwithstanding. This pronouncement is in keeping with the basic public policy of the State to afford protection to
labor, promote full employment, ensure equal work opportunities regardless of sex, race or creed, and regulate
the relations between workers and employers.

2.. Yes, respondent was illegally dismissed It must be noted that respondent was already a regular employee at
the time of her dismissal, because her three-month probationary period of employment had already ended. As a
regular employee, respondent was entitled to all rights, benefits and privileges provided under our labor laws. One
of her fundamental rights is that she may not be dismissed without due process of law. The twin requirements of
notice and hearing constitute the essential elements of procedural due process, and neither of these elements can
be eliminated without running afoul of the constitutional guarantee. In this case, all that petitioner tendered to
respondent was a notice of her employment termination effective the very same day, together with the equivalent
of a one-month pay. This Court has already held that nothing in the law gives an employer the option to substitute
the required prior notice and opportunity to be heard with the mere payment of 30 days salary

Other Notes/ SC Pronouncements:


14. Banez v. Hon. Valdevilla and Oro Marketing, Inc.
GR No: G.R. No. 128024 Date: May 9, 2000
Ponente: GONZAGA-REYES, J.:

Doctrine: The Labor Arbiter has jurisdiction to award not only the reliefs provided by labor laws, but also damages
governed by the Civil Code.

FACTS:
Petitioner was the sales operations manager of private respondent in its branch in Iligan City. In 1993, private
respondent "indefinitely suspended" petitioner and the latter filed a complaint for illegal dismissal with the National
Labor Relations Commission ("NLRC") in Iligan City.

LA found petitioner to have been illegally dismissed and ordered the payment of separation pay in lieu of
reinstatement, and of backwages and attorney's fees.
Upon appeal, the NLRC dismissed the appeal for being filed out of time. Elevated by petition for certiorari before
this Court, the case was dismissed for failure to show grave abuse of discretion on the part of the NLRC.

Private respondent filed a complaint for damages before the RTC. Petitioner filed a motion to dismiss on the
ground that the action for damages, having arisen from an employer-employee relationship, was squarely under
the exclusive original jurisdiction of the NLRC under Article 217(a)(now 224), paragraph 4 of the Labor Code and
is barred by reason of the final judgment in the labor case.

RTC RULING:
RTC ruled that it has jurisdiction over the subject matter. While seemingly the cause of action arose from
employer-employee relations, the employer's claim for damages is grounded on the nefarious activities of
defendant causing damage and prejudice to plaintiff as alleged in paragraph 7 of the complaint. The Court
believes that there was a breach of a contractual obligation, which is intrinsically a civil dispute.

APPEAL TO THE SC:


By way of assignment of errors, the petition reiterates the grounds raised in the Motion to Dismiss namely, lack of
jurisdiction over the subject matter of the action, res judicata, splitting of causes of action, and forum-shopping.

Angeles| Bajana | Balladares | Brillantes | Briones | Cabansag | Callanta | Chua | David|

De Leon | Gomez | Lopez | Macalino | Nostratis | Padilla | Reynon | Santos | Tan |Velasco
4E / 4F - 2018-2019
Page 268 of 920
LABOR REVIEW DIGEST
Atty. Joyrich Golangco

ISSUE/S:
1. Whether or not the RTC has jurisdiction over the subject matter

HELD:
1. No, the RTC has no jurisdiction over the subject matter. Art. 217(now 224) of the LC provides that it has
jurisdiction on cases, among others, for claims for actual, moral, exemplary and other forms of damages arising
from the employer-employee relations. This Court in a number of occasions had applied the jurisdictional
provisions of Article 217(now 224) to claims for damages filed by employees, we hold that by the designating
clause "arising from the employer-employee relations" Article 217(now 224) should apply with equal force to the
claim of an employer for actual damages against its dismissed employee, where the basis for the claim arises
from or is necessarily connected with the fact of termination, and should be entered as a counterclaim in the illegal
dismissal case. In this case, private respondent would not have taken issue with petitioner's "doing business of his
own" had the latter not been concurrently its employee. Thus, the damages alleged in the complaint below are:
first, those amounting to lost profits and earnings due to petitioner's abandonment or neglect of his duties as sales
manager, having been otherwise preoccupied by his unauthorized installment sale scheme; and second, those
equivalent to the value of private respondent's property and supplies which petitioner used in conducting his
"business ". Further, the issue of actual damages has been settled in the labor case, which is now final and
executory.

Other Notes/ SC Pronouncements:


This is, of course, to distinguish from cases of actions for damages where the employer-employee relationship is
merely incidental and the cause of action proceeds from a different source of obligation. Thus, the jurisdiction of
regular courts was upheld where the damages, claimed for were based on tort, malicious prosecution, or breach
of contract, as when the claimant seeks to recover a debt from a former employee or seeks liquidated damages in
enforcement of a prior employment contract.
__________________________________________________________________________

Angeles| Bajana | Balladares | Brillantes | Briones | Cabansag | Callanta | Chua | David|

De Leon | Gomez | Lopez | Macalino | Nostratis | Padilla | Reynon | Santos | Tan |Velasco
4E / 4F - 2018-2019
Page 269 of 920
LABOR REVIEW DIGEST
Atty. Joyrich Golangco

15. MA. ISABEL T. SANTOS, represented by ANTONIO P. SANTOS vs. SERVIER PHILIPPINES, INC. and
NLRC
GR No: 166377 Date: November 28, 2008
Ponente: NACHURA, J.:

DOCTRINE:
Albeit designated as "unpaid balance of the retirement package," the tax deductions in petitioner’s benefits is a
money claim arising from the employer-employee relationship, which clearly falls within the jurisdiction of the
Labor Arbiter and the NLRC under Article 217 of the Labor Code.

FACTS:
Petitioner Ma. Isabel Santos was the Human Resource Manager of respondent Servier, she attended a
company meeting in Paris, France. After the meeting, she dined with her family and friends at Leon des Bruxelles,
a restaurant known for mussels as specialty. During the dinner, she complained of stomach pain, vomited, and fell
into a coma for 21 days. The Paris hospital found that the probable cause was her "alimentary allergy" to mussels.
During her continued medical treatment in the Philippines, respondent paid all her hospitalization expenses and
salaries, but since she had not mentally and physically recovered to resume her work, the latter was constrained
to terminate her based on Article 284 of the Labor Code. As a consequence of her termination, respondent
offered her a retirement package of P1,063,841.76 consisting of disability retirement benefits, insurance, health
care, and educational assistance (voluntary). The unpaid balance was withheld by respondent allegedly for
taxation purposes.

Petitioner filed with NLRC the case for unpaid separation pay, retirement benefits, and damages under Article
284.

LA RULING: dismissed the complaint:


(a) Denied the separation pay – already been integrated in the retirement plan; (b) Refused to rule on the legality
of the deductions - the issue was beyond the jurisdiction of the NLRC.

NLRC RULING: reversed LA, and ruled that petitioner was not retired from the service pursuant to law, CBA, or
other employment contract; rather, she was dismissed from employment due to a disease/disability under
Article 284. Hence, the amounts she received as retirement benefits shall be treated as separation pay. On
issue of illegal deduction, petitioner could file a tax refund with the BIR.

CA RULING: affirmed the NLRC.

APPEAL TO SC: Petitioner changed her position that she is not entitled to separation pay but to retirement
pay under to Section 4, Article 5 of the Retirement Plan, on disability retirement. She also demanded the
payment of the amount allegedly deducted for taxation purposes.

Respondent’s contention: the legality of the deduction from the total benefits cannot be taken cognizance of by
this Court since the issue was not raised during the early stage of the proceedings.

ISSUE/S:
1. Whether the LA and NLRC have jurisdiction over the alleged illegal deductions on the retirement benefits?
YES. (Main issue)
2. Whether petitioner is also entitled to her separation pay under the law despite having partially claimed her
disability retirement benefits under the Retirement Plan of the company? NO.
3. Whether the retirement benefits are subject to withholding tax? YES, SUBJECT TO EXEMPTION.
HELD:
Angeles| Bajana | Balladares | Brillantes | Briones | Cabansag | Callanta | Chua | David|

De Leon | Gomez | Lopez | Macalino | Nostratis | Padilla | Reynon | Santos | Tan |Velasco
4E / 4F - 2018-2019
Page 270 of 920
LABOR REVIEW DIGEST
Atty. Joyrich Golangco

1. Contrary to the Labor Arbiter and NLRC’s conclusions, petitioner’s claim for illegal deduction falls within the
tribunal’s jurisdiction. Records reveal that as early in the Labor Arbiter, she already raised the legality of said
deduction, albeit designated as "unpaid balance of the retirement package." The issue of deduction for tax
purposes is intertwined with the main issue of whether or not petitioner’s benefits have been fully given her. It is a
money claim arising from the employer-employee relationship, which clearly falls within the jurisdiction of
the Labor Arbiter and the NLRC under Article 217 of the Labor Code.

2. The rule in Aquino v. National Labor Relations Commission that “the receipt of retirement benefits does
not bar the retiree from receiving separation pay” is only true if there is no specific prohibition against the
payment of both benefits in the retirement plan and/or in the CBA. The two benefits are not mutually
exclusive, as separation pay is a statutory right designed to provide the employee with the wherewithal during
the period that he/she is looking for another employment; while the retirement benefits are intended to help the
employee enjoy the remaining years of his life, lessening the burden of worrying about his financial support, and
are a form of reward for his loyalty and service to the employer.

APPLICATION – Section 2, Article 12 of the Retirement Plan bars the petitioner from claiming additional
benefits under the Labor Code; or under any present or future legislation; under any other contract; or CBA with
the Company, on top of that provided for in the Plan. There being such a provision, petitioner is entitled only
to either the separation pay under the law or retirement benefits under the Plan, and not both.

3. Section 32 (B) (6) (a) of the NIRC provides for the exclusion of retirement benefits from gross income: (6)
Retirement Benefits, Pensions, Gratuities, etc. – (a) Retirement benefits received under Republic Act 7641 and
those received by officials and employees of private firms, whether individual or corporate, in accordance with a
reasonable private benefit plan maintained by the employer: Provided, that the retiring official or employee
has been in the service of the same employer for at least ten (10) years AND is not less than fifty (50)
years of age at the time of his retirement: Provided further, That the benefits granted under this
subparagraph shall be availed of by an official or employee only once. x x x.

Elements for the retirement benefits to be exempt from the withholding tax:
(1) a reasonable private benefit plan is maintained by the employer; (2) the retiring official or employee has been
in the service of the same employer for at least ten (10) years; (3) the retiring official or employee is not less than
fifty (50) years of age at the time of his retirement; and (4) the benefit had been availed of only once.

APPLICATION – Petitioner was qualified for disability retirement. At the time of such retirement, petitioner was
only 41 years of age; and had been in the service for more or less eight (8) years. The above provision is not
applicable for failure to comply with the age and length of service requirements. Thus, respondent cannot
be faulted for deducting from petitioner’s total retirement benefits the amount for taxation purposes.

Angeles| Bajana | Balladares | Brillantes | Briones | Cabansag | Callanta | Chua | David|

De Leon | Gomez | Lopez | Macalino | Nostratis | Padilla | Reynon | Santos | Tan |Velasco
4E / 4F - 2018-2019
Page 271 of 920
LABOR REVIEW DIGEST
Atty. Joyrich Golangco

16. PEPSI COLA DISTRIBUTORS OF THE PHILIPPINES, INC., represented by its Plant General Manager
ANTHONY B. SIAN, et al vs. HON. LOLITA O. GAL-LANG, et al.
GR No: 89621 Date: September 24, 1991
Ponente: CRUZ, J.

DOCTRINE:
Not every controversy involving workers and their employers can be resolved only by the labor arbiters. Absence
of “reasonable causal connection" between the civil action for damages for malicious prosecution and the relations
of the parties as employer and employees; the action for damages is a matter which the labor arbiter has no
competence to resolve as the applicable law is not the Labor Code but the Revised Penal Code.

FACTS:
Angeles| Bajana | Balladares | Brillantes | Briones | Cabansag | Callanta | Chua | David|

De Leon | Gomez | Lopez | Macalino | Nostratis | Padilla | Reynon | Santos | Tan |Velasco
4E / 4F - 2018-2019
Page 272 of 920
LABOR REVIEW DIGEST
Atty. Joyrich Golangco

Private respondents were dismissed employees of petitioner suspected of complicity in the irregular disposition
of empty Pepsi Cola bottles. The criminal complaints filed against them for theft and falsification were dismissed
by the MTC, ruling that they were intended "to harass the poor employees." The dismissal was also affirmed by
the Provincial Prosecutor "for lack of evidence to establish even a slightest probability that all the respondents
have committed the crime imputed against them."

Respondents filed with RAB-NLRC a complaint for illegal dismissal, and with RTC a civil action for damages
for malicious prosecution. The civil action was initially dismissed, but was later reinstated by the public
respondent ruling that it was "distinct from the labor case for damages now pending before the labor courts."

Petitioner’s contention:
RTC had no jurisdiction over the civil action for damages because it involved employee-employer relations that
were exclusively cognizable by the labor arbiter under Article 217 of the Labor Code and the ruling in *Getz
Corporation v. Court of Appeals.

ISSUE/S:
Whether it is the RTC, not the LA, that has jurisdiction over the civil action for damages for malicious prosecution?
YES.

HELD:
ART. 217. Jurisdiction of Labor Arbiters and the Commission. xxx
(3) All money claims of workers xxx

The “money claims of workers" referred to embraces money claims that arise out of or in connection with the
employer-employee relationship, or some aspect or incident of such relationship. To put the case under the
provisions of Article 217, there must be a "reasonable causal connection" between the claim asserted and
employee-employer relations. Absent such a link, the complaint will be cognizable by the regular courts of
justice in the exercise of their civil and criminal jurisdiction.

In San Miguel Corporation v. NLRC - It is the character of the principal relief sought that appears essential.
Where such principal relief is to be granted under labor legislation or a collective bargaining agreement, the
case should fall within the jurisdiction of the Labor Arbiter and the NLRC, even though a claim for damages
might be asserted as an incident to such claim. Where the claim to the principal relief sought is to be resolved not
by reference to the Labor Code or other labor relations statute or a collective bargaining agreement but by the
general civil law, the jurisdiction over the dispute belongs to the regular courts and not to the Labor Arbiter and
the NLRC. In such situations, resolution of the dispute requires expertise, not in labor management relations nor
in wage structures and other terms and conditions of employment, but rather in the application of the general civil
law.

APPLICATION – It does not appear that there is a "reasonable causal connection" between the complaint and the
relations of the parties as employer and employees. The complaint did not arise from such relations and in
fact could have arisen independently of an employment relationship between the parties. No such
relationship or any unfair labor practice is asserted. What the employees are alleging is that the petitioners acted
with bad faith when they filed the criminal complaints.

Other notes: *Getz Corporation v. Court of Appeals not applicable because what was involved there was a
claim arising from the alleged illegal dismissal of an employee, who chose to complain to the regular court and not
to the labor arbiter.

Angeles| Bajana | Balladares | Brillantes | Briones | Cabansag | Callanta | Chua | David|

De Leon | Gomez | Lopez | Macalino | Nostratis | Padilla | Reynon | Santos | Tan |Velasco
4E / 4F - 2018-2019
Page 273 of 920
LABOR REVIEW DIGEST
Atty. Joyrich Golangco

17. 7K CORPORATION, Petitioner, vs. EDDIE ALBARICO, Respondent.


GR No: 182295 Date: June 26, 2013
Ponente: SERENO, CJ.

Doctrine: Although the general rule under the Labor Code gives the labor arbiter exclusive and original
jurisdiction over termination disputes, it also recognizes exceptions. One of the exceptions is provided in Article
262 of the Labor Code. Voluntary arbitrators may, by agreement of the parties, assume jurisdiction over a
termination dispute such as the present case, contrary to the assertion of petitioner that they may not.
FACTS:

 When he was dismissed on 5 April 1993, respondent Eddie Albarico (Albarico) was a regular employee of
petitioner 7K Corporation, a company selling water purifiers.
 Because of his good performance, his employment was regularized.

 He was also promoted several times: from salesman, he was promoted to senior sales representative and then to
acting team field supervisor.
 7K Corporation terminated Albarico’s employment allegedly for his poor sales performance.

 Respondent had to stop reporting for work, and he subsequently submitted his money claims against petitioner
for arbitration before the National Conciliation and Mediation Board (NCMB).

 The issue for voluntary arbitration before the NCMB, according to the parties’ Submission Agreement dated 19
April 1993, was whether respondent Albarico was entitled to the payment of separation pay and the sales
commission reserved for him by the corporation.

 While the NCMB arbitration case was pending, respondent Albarico filed a Complaint against petitioner
corporation with the Arbitration Branch of the National Labor Relations Commission (NLRC) for illegal
dismissal with money claims for overtime pay, holiday compensation, commission, and food and travelling
allowances.

LA: The Complaint was decided by the labor arbiter in favor of respondent Albarico, who was awarded separation
pay in lieu of reinstatement, backwages and attorney’s fees.

NLRC: The labor arbiter’s Decision was vacated by the NLRC for forum shopping on the part of respondent
Albarico, because the NCMB arbitration case was still pending. The NLRC Decision, which explicitly stated that
the dismissal was without prejudice to the pending NCMB arbitration case, became final after no appeal was
taken.
Petitioner corporation filed its Position Paper in the NCMB arbitration case. It denied that respondent was
terminated from work, much less illegally dismissed. The corporation claimed that he had voluntarily stopped
reporting for work after receiving a verbal reprimand for his sales performance; hence, it was he who was guilty of
abandonment of employment.

NCMB: NCMB voluntary arbitrator rendered a Decision finding petitioner corporation liable for illegal dismissal.
The termination of respondent Albarico, by reason of alleged poor performance, was found invalid. The arbitrator
explained that the promotions, increases in salary, and awards received by respondent belied the claim that the
latter was performing poorly. It was also found that Albarico could not have abandoned his job, as the
abandonment should have been clearly shown.

Angeles| Bajana | Balladares | Brillantes | Briones | Cabansag | Callanta | Chua | David|

De Leon | Gomez | Lopez | Macalino | Nostratis | Padilla | Reynon | Santos | Tan |Velasco
4E / 4F - 2018-2019
Page 274 of 920
LABOR REVIEW DIGEST
Atty. Joyrich Golangco

CA: The CA affirmed the Decision of the voluntary arbitrator, but eliminated the award of attorney’s fees for
having been made without factual, legal or equitable justification. Petitioner’s Motion for Partial Reconsideration
was denied as well.

Petitioner's Contentions:
A. Claims that under Article 217 of the Labor Code, original and exclusive jurisdiction over termination
disputes, such as the present case, is lodged only with the labor arbiter of the NLRC.

B. Assuming that the voluntary arbitrator has jurisdiction over the present termination dispute, the latter should
have limited his decision to the issue contained in the Submission Agreement of the parties – the issue of whether
respondent Albarico was entitled to separation pay and to the sales commission the latter earned before being
terminated.

ISSUE/S:
Whether the CA committed reversible error in finding that the voluntary arbitrator properly assumed jurisdiction to
decide the issue of the legality of the dismissal of respondent as well as the latter’s entitlement to backwages,
even if neither the legality nor the entitlement was expressly claimed in the Submission Agreement of the parties.

HELD:
NO. Although the general rule under the Labor Code gives the labor arbiter exclusive and original jurisdiction over
termination disputes, it also recognizes exceptions. One of the exceptions is provided in Article 262 of the Labor
Code.

Voluntary arbitrators may, by agreement of the parties, assume jurisdiction over a termination dispute such as the
present case, contrary to the assertion of petitioner that they may not.

We rule that although petitioner correctly contends that separation pay may in fact be awarded for reasons other
than illegal dismissal, the circumstances of the instant case lead to no other conclusion than that the claim of
respondent Albarico for separation pay was premised on his allegation of illegal dismissal. Thus, the voluntary
arbitrator properly assumed jurisdiction over the issue of the legality of his dismissal.
True, under the Labor Code, separation pay may be given not only when there is illegal dismissal. In fact, it is also
given to employees who are terminated for authorized causes, such as redundancy, retrenchment or installation
of labor-saving devices under Article 283 of the Labor Code. Additionally, jurisprudence holds that separation pay
may also be awarded for considerations of social justice, even if an employee has been terminated for a just
cause other than serious misconduct or an act reflecting on moral character. The Court has also ruled that
separation pay may be awarded if it has become an established practice of the company to pay the said benefit to
voluntarily resigning employees or to those validly dismissed for non-membership in a union as required in a
closed-shop agreement.

The above circumstances, however, do not obtain in the present case. In fact, even in instances in which
separation pay is awarded in consideration of social justice, the issue of the validity of the dismissal still needs to
be resolved first. Only when there is already a finding of a valid dismissal for a just cause does the court then
award separation pay for reason of social justice. The other circumstances when separation pay may be awarded
are not present in this case.

The foregoing findings indisputably prove that the issue of separation pay emanates solely from respondent’s
allegation of illegal dismissal. In fact, petitioner itself acknowledged the issue of illegal dismissal in its position
paper submitted to the NCMB. Moreover, we note that even the NLRC was of the understanding that the NCMB
arbitration case sought to resolve the issue of the legality of the dismissal of the respondent. In fact, the identity of

Angeles| Bajana | Balladares | Brillantes | Briones | Cabansag | Callanta | Chua | David|

De Leon | Gomez | Lopez | Macalino | Nostratis | Padilla | Reynon | Santos | Tan |Velasco
4E / 4F - 2018-2019
Page 275 of 920
LABOR REVIEW DIGEST
Atty. Joyrich Golangco

the issue of the legality of his dismissal, which was previously submitted to the NCMB, and later submitted to the
NLRC, was the basis of the latter’s finding of forum shopping and the consequent dismissal of the case before it.
In fact, petitioner also implicitly acknowledged this when it filed before the NLRC its Motion to Dismiss
respondent’s Complaint on the ground of forum shopping. Thus, it is now estopped from claiming that the issue
before the NCMB does not include the issue of the legality of the dismissal of respondent. Besides, there has to
be a reason for deciding the issue of respondent’s entitlement to separation pay. To think otherwise would lead to
absurdity, because the voluntary arbitrator would then be deciding that issue in a vacuum. The arbitrator would
have no basis whatsoever for saying that Albarico was entitled to separation pay or not if the issue of the legality
of respondent’s dismissal was not resolve first.

Moreover, we have ruled in Sime Darby Pilipinas, Inc. v. Deputy Administrator Magsalin that a voluntary arbitrator
has plenary jurisdiction and authority to interpret an agreement to arbitrate and to determine the scope of his own
authority when the said agreement is vague — subject only, in a proper case, to the certiorari jurisdiction of this
Court. Having established that the issue of the legality of dismissal of Albarico was in fact necessarily – albeit not
explicitly – included in the Submission Agreement signed by the parties, this Court rules that the voluntary
arbitrator rightly assumed jurisdiction to decide the said issue.

Consequently, we also rule that the voluntary arbitrator may award backwages upon a finding of illegal dismissal,
even though the issue of entitlement thereto is not explicitly claimed in the Submission Agreement. Backwages, in
general, are awarded on the ground of equity as a form of relief that restores the income lost by the terminated
employee by reason of his illegal dismissal.

Angeles| Bajana | Balladares | Brillantes | Briones | Cabansag | Callanta | Chua | David|

De Leon | Gomez | Lopez | Macalino | Nostratis | Padilla | Reynon | Santos | Tan |Velasco
4E / 4F - 2018-2019
Page 276 of 920
LABOR REVIEW DIGEST
Atty. Joyrich Golangco

18. ULIUS KAWACHI and GAYLE KAWACHI, Petitioners, vs. DOMINIE DEL QUERO and HON. JUDGE
MANUEL R. TARO, Metropolitan Trial Court, Branch 43, Quezon City, Respondents.
GR No: 163768 Date: March 27, 2007
Ponente: TINGA, J.

Doctrine:
The legality of the act of dismissal is intimately related to the issue of the legality of the manner by which that act
of dismissal was performed. Thus, NLRC has jurisdiction over private respondent’s complaint for illegal dismissal
and damages arising therefrom. She cannot be allowed to file a separate or independent civil action for damages
where the alleged injury has a reasonable connection to her termination from employment. Consequently, the
action for damages filed before the MeTC must be dismissed.

FACTS:
Private respondent Dominie Del Quero charged A/J Raymundo Pawnshop, Inc., Virgilio Kawachi and petitioner
Julius Kawachi with illegal dismissal, non-execution of a contract of employment, violation of the minimum wage
law, and non-payment of overtime pay. The complaint was filed before the National Labor Relations Commission
(NLRC).
The complaint essentially alleged that Virgilio Kawachi hired private respondent as a clerk of the pawnshop and
that on certain occasions, she worked beyond the regular working hours but was not paid the corresponding
overtime pay. The complaint also narrated an incident on 10 August 2002, wherein petitioner Julius Kawachi
scolded private respondent in front of many people about the way she treated the customers of the pawnshop and
afterwards terminated private respondent’s employment without affording her due process.
On 7 November 2002, private respondent Dominie Del Quero filed an action for damages against petitioners
Julius Kawachi and Gayle Kawachi before the MeTC of Quezon City.
Petitioners moved for the dismissal of the complaint on the grounds of lack of jurisdiction and forum-shopping or
splitting causes of action. At first, the MeTC granted petitioners’ motion and ordered the dismissal of the complaint
for lack of jurisdiction in an Order dated 2 January 2003.
MeTC: Upon private respondent’s motion, the MeTC reconsidered and set aside the order of dismissal in an
Order dated 3 March 2003. It ruled that no causal connection appeared between private respondent’s cause of
action and the employer-employee relations between the parties.
RTC: RTC issued the assailed Resolution, upholding the jurisdiction of the MeTC over private respondent’s
complaint for damages.
The RTC held that private respondent’s action for damages was based on the alleged tortious acts committed by
her employers and did not seek any relief under the Labor Code.
Petitioners argue that the NLRC has jurisdiction over the action for damages because the alleged injury is work-
related. They also contend that private respondent should not be allowed to split her causes of action by filing the
action for damages separately from the labor case.
Private respondent maintains that there is no causal connection between her cause of action and the employer-
employee relations of the parties.

ISSUE/S:
Whether or not NLRC has jurisdiction over the action for damages.

HELD:
YES. The jurisdictional controversy of the sort presented in this case has long been settled by this Court.
Article 217(a) of the Labor Code, as amended, clearly bestows upon the Labor Arbiter original and exclusive
jurisdiction over claims for damages arising from employer-employee relations —in other words, the Labor Arbiter
has jurisdiction to award not only the reliefs provided by labor laws, but also damages governed by the Civil Code.
In the 1999 case of San Miguel Corporation v. Etcuban, the Court noted what was then the current trend, and still
is, to refer worker-employer controversies to labor courts, unless unmistakably provided by the law to be
Angeles| Bajana | Balladares | Brillantes | Briones | Cabansag | Callanta | Chua | David|

De Leon | Gomez | Lopez | Macalino | Nostratis | Padilla | Reynon | Santos | Tan |Velasco
4E / 4F - 2018-2019
Page 277 of 920
LABOR REVIEW DIGEST
Atty. Joyrich Golangco

otherwise. Because of the trend, the Court noted further, jurisprudence has developed the "reasonable causal
connection rule." Under this rule, if there is a reasonable causal connection between the claim asserted and the
employer-employee relations, then the case is within the jurisdiction of our labor courts. In the absence of such
nexus, it is the regular courts that have jurisdiction.
In San Miguel Corporation, the Court upheld the labor arbiter’s jurisdiction over the employees’ separate action for
damages, which also sought the nullification of the so-called "contract of termination" and noted that the
allegations in the complaint were so carefully formulated as to avoid a semblance of employer-employee relations.
In the instant case, the allegations in private respondent’s complaint for damages show that her injury was the
offshoot of petitioners’ immediate harsh reaction as her administrative superiors to the supposedly sloppy manner
by which she had discharged her duties.
Petitioners’ reaction culminated in private respondent’s dismissal from work in the very same incident. The
incident on 10 August 2002 alleged in the complaint for damages was similarly narrated in private respondent’s
Affidavit-Complaint supporting her action for illegal dismissal before the NLRC. Clearly, the alleged injury is
directly related to the employer-employee relations of the parties.
Where the employer-employee relationship is merely incidental and the cause of action proceeds from a different
source of obligation, the Court has not hesitated to uphold the jurisdiction of the regular courts. Where the
damages claimed for were based on tort, malicious prosecution, or breach of contract, as when the claimant
seeks to recover a debt from a former employee or seeks liquidated damages in the enforcement of a prior
employment contract, the jurisdiction of regular courts was upheld. The scenario that obtains in this case is
obviously different. The allegations in private respondent’s complaint unmistakably relate to the manner of her
alleged illegal dismissal.
For a single cause of action, the dismissed employee cannot be allowed to sue in two forums: one, before the
labor arbiter for reinstatement and recovery of back wages or for separation pay, upon the theory that the
dismissal was illegal; and two, before a court of justice for recovery of moral and other damages, upon the theory
that the manner of dismissal was unduly injurious or tortious. Suing in the manner described is known as "splitting
a cause of action," a practice engendering multiplicity of actions. It is considered procedurally unsound and
obnoxious to the orderly administration of justice.

Angeles| Bajana | Balladares | Brillantes | Briones | Cabansag | Callanta | Chua | David|

De Leon | Gomez | Lopez | Macalino | Nostratis | Padilla | Reynon | Santos | Tan |Velasco
4E / 4F - 2018-2019
Page 278 of 920
LABOR REVIEW DIGEST
Atty. Joyrich Golangco

19. GILDA LUNZAGA vs. ALBAR SHIPPING AND TRADING CORP. et/. al.,
GR No: G.R. No. 200476 Date: April 18, 2012
Ponente:

Doctrine: It has been said this time and again that the perfection of an appeal within the period fixed by
the rules is mandatory and jurisdictional. But, it is always in the power of this Court to suspend its own
rules, or to except a particular case from its operation, whenever the purposes of justice require it. Strong
compelling reasons such as serving the ends of justice and preventing a grave miscarriage thereof
warrant the suspension of the rules.
FACTS:

Romeo Lunzaga, a seaman assigned as a Chief Engineer for respondent Albar Shipping’s vessel (MV Lake Aru)
suffered a heart attack while on board and died when he was repatriated to the Philippines.

Petitioner Glda Lunzaga, claiming to be the surviving spouse of Romeo, filed with the NLRC a complaint against
Albar for payment of death benefits, damages and attorney's fees. She was the designated heir in Romeo's OFW
Verification Sheet and PhilHealth Information Sheet.

However, the Lunzaga siblings (the children of Romeo from his first marriage that was judicially declared null and
void) opposed the claim and claimed that Gilda is not entitled to the death benefits of Romeo, as she had a
subsisting marriage when she married him. They claim that her marriage with Romeo was, therefore, bigamous.
During the mandatory conferences of the parties before the LA, respondent Albar signified its willingness to pay
Romeo's death benefits in the amount of USD 55,547.44. However, Gilda and the Lunzaga siblings could not
agree as to the sharing of the benefits.

LA RULING:
The LA issued an Order temporarily dismissing the complaint and directing the parties to file their case with the
regular courts.
Gilda received a copy of the August 28, 2009 Order of the LA on September 28, 2009. Gilda's appeal to the NLRC
was, however, filed only on October 9, 2009, one day past the 10-day period for filing an appeal from the decision
of the Labor Arbiter.

NLRC RULING:
Thus, the NLRC dismissed the appeal for having been filed beyond the reglementary period.

CA RULING:
The CA affirmed the NLRC Decision which dismissed the appeal of petitioner for having been filed out of time. It
also affirmed the LA Order which ruled that jurisdiction over the instant controversy is with the regular courts and
not with the NLRC.
Gilds’s MR was denied. Hence, this petition.

APPEAL TO THE SC:

Petitioner's Contention: (not stated categorically in the case)

The La erred in dismissing the case because it has the jurisdiction to determine whether the heirs of Romeo are
entitled to receive death benefits from Albar Shipping?
The NLRC erred in dismissing the case for being filed one day late.

Respondent's Contention:
Angeles| Bajana | Balladares | Brillantes | Briones | Cabansag | Callanta | Chua | David|

De Leon | Gomez | Lopez | Macalino | Nostratis | Padilla | Reynon | Santos | Tan |Velasco
4E / 4F - 2018-2019
Page 279 of 920
LABOR REVIEW DIGEST
Atty. Joyrich Golangco

(no specific contention by Albar Shipping)

ISSUE/S:
1. Does the LA have jurisdiction over the identification of the proper beneficiary of the deceased? – NO. NLRC
2. Does the LA have jurisdiction to determine whether the heirs of Romeo are entitled to receive his death benefits
from Albar. - YES
3. Is the NLRC correct in dismissing the appeal for having been filed one day late? – No.

HELD:
1. No. The LA and CA are correct in ruling that the issue of who is the proper beneficiary of Romeo is properly
within the jurisdiction of the regular courts.
2. Yes.The LA has jurisdiction over this issue and the case itself, involving as it does a claim arising from an
employer-employee relationship. And while the LA has no jurisdiction to determine who among the alleged heirs is
entitled to receive Romeo's death benefits, it should have made a ruling holding Albar liable for the claim.

3. No. The SC ruled that the ends of justice would be best served with the admission of the appeal for the
complete ventilation of the issues in the case. Considering that Albar admitted its liability to the heirs of Romeo for
his death benefits, the NLRC should have given due course to the meritorious appeal.

Evidently, the NLRC and the CA erred in not giving due course to the appeal due to a one-day delay of its filing,
considering the apparent merit of the appeal as shown by the admission of Albar.
Verily, Albar is liable to the heirs of Romeo for the amount of USD 55,547.44. Albar hereby is ordered to deposit
this amount in an escrow account under the control of the NLRC in order to protect the interests of Romeo's heirs.
The parties claiming to be the beneficiaries of Romeo are directed to file the appropriate action with a trial court to
determine the true and legal heirs of Romeo entitled to receive the disability benefits. The amount in the escrow
account will only be released to the legal heirs per the decision of a trial court.
The Petition is granted.

Angeles| Bajana | Balladares | Brillantes | Briones | Cabansag | Callanta | Chua | David|

De Leon | Gomez | Lopez | Macalino | Nostratis | Padilla | Reynon | Santos | Tan |Velasco
4E / 4F - 2018-2019
Page 280 of 920
LABOR REVIEW DIGEST
Atty. Joyrich Golangco

20. CONTINENTAL MICRONESIA INC. vs JOSEPH BASSO


GR No.: 178382-83 Date: September 23, 2015
Ponente: Jardeleza, J.

DOCTRINE:

Under the doctrine of forum non conveniens, a Philippine court in a conflict-of-laws case may assume jurisdiction
if it chooses to do so, provided, that the following requisites are met: (1) that the Philippine Court is one to which
the parties may conveniently resort to; (2) that the Philippine Court is in a position to make an intelligent decision
as to the law and the facts; and (3) that the Philippine Court has or is likely to have power to enforce its decision.

FACTS:

Petitioner Continental Micronesia is a foreign corporation organized and existing under the laws of and domiciled
in the United States of America. It is licensed to do business in the Philippines. Respondent, a US citizen residing
in the Philippines, accepted an offer to be a General Manager position by Mr. Braden, Managing Director-Asia of
Continental Airlines. On November 7, 1992, CMI took over the Philippine operations of Continental, with
respondent retaining his position as General Manager. Thereafter, respondent received a letter from Mr. Schulz,
who was then CMI’s Vice President of Marketing and Sales, informing him that he has agreed to work in CMI as a
consultant on an “as needed basis.” Respondent wrote a counter-proposal that was rejected by CMI.

Respondent then filed a complaint for illegal dismissal against the petitioner corporation. Alleging the presence of
foreign elements, CMI filed a Motion to Dismiss on the ground of lack of jurisdiction over the person of CMI and
the subject matter of the controversy.

LA RULING: The LA agreed with CMI that the employment contract was executed in the US “since the letter-offer
was under the Texas letterhead and the acceptance of Complainant was returned there.” Thus, applying the
doctrine of lex loci celebrationis, US laws apply. Also, applying lex loci contractus, the Labor Arbiter ruled that the
parties did not intend to apply Philippine laws.

NLRC RULING: The NLRC ruled that the Labor Arbiter acquired jurisdiction over the case when CMI voluntarily
submitted to his office’s jurisdiction by presenting evidence, advancing arguments in support of the legality of its
acts, and praying for reliefs on the merits of the case.

CA RULING: The Court of Appeals ruled that the Labor Arbiter and the NLRC had jurisdiction over the subject
matter of the case and over the parties.

ISSUE/S:

Whether the labor tribunals have jurisdiction over the case.

HELD:

Yes. The Court ruled that the labor tribunals had jurisdiction over the parties and the subject matter of the case.
The employment contract of Basso was replete with references to US laws, and that it originated from and was
returned to the US, do not automatically preclude our labor tribunals from exercising jurisdiction to hear and try
this case.

On the other hand, jurisdiction over the person of CMI was acquired through the coercive process of service of
summons. CMI never denied that it was served with summons. CMI has, in fact, voluntarily appeared and
Angeles| Bajana | Balladares | Brillantes | Briones | Cabansag | Callanta | Chua | David|

De Leon | Gomez | Lopez | Macalino | Nostratis | Padilla | Reynon | Santos | Tan |Velasco
4E / 4F - 2018-2019
Page 281 of 920
LABOR REVIEW DIGEST
Atty. Joyrich Golangco

participated in the proceedings before the courts. Though a foreign corporation, CMI is licensed to do business in
the Philippines and has a local business address here. The purpose of the law in requiring that foreign
corporations doing business in the country be licensed to do so, is to subject the foreign corporations to the
jurisdiction of our courts.

Where the facts establish the existence of foreign elements, the case presents a conflicts-of-laws issue. Under the
doctrine of forum non conveniens, a Philippine court in a conflict-of-laws case may assume jurisdiction if it
chooses to do so, provided, that the following requisites are met: (1) that the Philippine Court is one to which the
parties may conveniently resort to; (2) that the Philippine Court is in a position to make an intelligent decision as to
the law and the facts; and (3) that the Philippine Court has or is likely to have power to enforce its decision. All
these requisites are present here.

Angeles| Bajana | Balladares | Brillantes | Briones | Cabansag | Callanta | Chua | David|

De Leon | Gomez | Lopez | Macalino | Nostratis | Padilla | Reynon | Santos | Tan |Velasco
4E / 4F - 2018-2019
Page 282 of 920
LABOR REVIEW DIGEST
Atty. Joyrich Golangco

21. World’s Best Gas Inc. vs Vital


GR No.: 211588 Date: September 9, 2015
Ponente: Perlas-Bernabe, J.

DOCTRINE:

A cause of action from employer-employee relations, involving an amount exceeding P 5,000.00, fall under the
jurisdiction of the Labor Arbiter pursuant to Article 217 of the Labor Code.

FACTS:

Mr. Vital was one of the incorporators of WBGI, holding P500,000.00 worth of shares of stocks therein. As a
separate business venture, respondents (Mr. and Mrs. Vital) sourced LPG from WBGI and distributed the same
through ERJ Enterprises owned by them. The respondents have outstanding balance with WBGI for unpaid LPG
amounted to P923,843.59.Thereafter, Mr. Vital was appointed as Internal Auditor and Personnel Manager.

Upon his mandatory retirement, WBGI computed his retirement benefits. WBGI also agreed to acquire his shares
of stocks.After offsetting his shares of stocks against ERJ Enterprises’ P923,843.59 outstanding balance to WBGI,
Vital claimed that the unpaid salaries and separation pay due him amounted to P845,000.00 and P250,000.00,
respectively, leaving a net amount of P671,156.41 payable to him. WBGI rejected his claim and contended that
after offsetting, Vital actually owed it P369,156.19.

Mr. Vital filed a complaint for non-payment of separation and retirement benefits, underpayment of
salaries/wages and 13th month pay, illegal reduction of salary and benefits, and damages. WBGI averred that LA
has no jurisdiction over the complaint having no existing employer-employee relationship between the parties, and
that Mr. Vital is a mere incorporator and stockholder.

The LA dismissed the case for lack of jurisdiction. The LA found that the issues between Vital and WBGI are intra-
corporate in nature as they arose between the relations of a stockholder and the corporation, and not from an
employee and employer relationship.

The RTC, acting as a special commercial court, oppositely found that Vital was an employee of WBGI and
thereby, upheld his claim of unpaid salaries and separation pay.

The CA ruled that Mr. Vital is an employee of WBGI.

ISSUE/S:
Whether RTC has jurisdiction over the case.

HELD:
The petition is partly meritorious.

The Court pointed out that the case involves three distinct causes of action, namely,

(1) Vital’s claim for P845,000.00 and P250,000.00 in unpaid salaries and separation pay;

(2) the P923,843.59 in arrearages payable to WBGI from ERJ Enterprises, which was admitted by Vital but not
claimed by WBGI; and

(3) Vital’s claim of P500,000.00 due from WBGI’s acquisition of Vital’s shares of stocks.
Angeles| Bajana | Balladares | Brillantes | Briones | Cabansag | Callanta | Chua | David|

De Leon | Gomez | Lopez | Macalino | Nostratis | Padilla | Reynon | Santos | Tan |Velasco
4E / 4F - 2018-2019
Page 283 of 920
LABOR REVIEW DIGEST
Atty. Joyrich Golangco

The RTC’s adjudication of the first cause of action was improper since the same is one which arose from Vital and
WBGI’s employer-employee relations, involving an amount exceeding P 5,000.00, hence, belonging to the
jurisdiction of the Labor Arbiters pursuant to Article 217 of the Labor Code

On the contrary, RTC has:

(a) general jurisdiction to adjudicate on the P923,843.59 in arrearages payable to WBGI from ERJ Enterprises,
which was admitted by Vital but not claimed by WBGI; and

(b) special jurisdiction, as a special commercial court, to adjudicate on Vital’s claim of P500,000.00 from WBGI’s
acquisition of his shares of stocks.

Angeles| Bajana | Balladares | Brillantes | Briones | Cabansag | Callanta | Chua | David|

De Leon | Gomez | Lopez | Macalino | Nostratis | Padilla | Reynon | Santos | Tan |Velasco
4E / 4F - 2018-2019
Page 284 of 920
LABOR REVIEW DIGEST
Atty. Joyrich Golangco

22. SAME CASE AS IN NUMBER 20

23. ALLAN M. MENDOZA vs OFFICERS OF MANILA WATER EMPLOYEES UNION (MWEU)


G.R. No. 201595; January 25, 2016
Del Castillo, J.
Doctrine: It is true that some of petitioner’s causes of action constitute intra-union cases cognizable by the BLR
under Article 226 of the Labor Code. However, petitioner’s charge of unfair labor practices falls within
the original and exclusive jurisdiction of the Labor Arbiters, pursuant to Article 217 of the Labor Code. In addition,
Article 247 of the same Code provides that "the civil aspects of all cases involving unfair labor practices, which
may include claims for actual, moral, exemplary and other forms of damages, attorney’s fees and other affirmative
relief, shall be under the jurisdiction of the Labor Arbiters.
FACTS: Petitioner was a member of the Manila Water Employees Union (MWEU), a Department of Labor and
Employment (DOLE)-registered labor organization consisting of rank-and-file employees within Manila Water
Company (MWC). The 24 private respondents herein were MWEU officers during the period material to this
Petition, with Borela as President.
In an April 2007 letter, MWEU informed petitioner of his failure to pay the union dues (union dues is increased
from P100 to P200, and the increased amount was not paid). By reason of such failure, petitioner and several
others violated Section 1(g), Article IX of the MWEU’s Constitution and By-Laws. In turn, Borela referred the
charge to the MWEU grievance committee for investigation. The grievance committee recommended that
petitioner be suspended for 30 days.
Petitioner indicated their intention to appeal the same to the General Membership Assembly in accordance with
Section 2(g), Article V of the union’s Constitution and By-Laws. Petitioner’s appeal was denied, stating that the
prescribed period for appeal had expired.
Petitioner was again charged for non-payment of union dues for the second and third time and was eventually
meted with penalty of expulsion from the union. His pleas for an appeal to the General Membership Assembly
were once more unheeded.
In 2008, petitioner joined another union and was elected union President. Other MWEU members were inclined to
join this new union, but MWEU director Torres threatened that they would not get benefits from the new CBA.
Petitioner then filed a Complaint against respondents for unfair labor practices, damages, and attorney’s fees
before the National Labor Relations Commission (NLRC).
LABOR ARBITER RULING: The filing of the instant case is still premature. Section 5, Article X-Investigation
Procedures and Appeal Process of the Union Constitution and By-Laws provides that:
Section 5. Any dismissed and/or expelled member shall have the rights to appeal to the Executive Board within
seven (7) days from the date of notice of the said dismissal and/or expulsion, which in [turn] shall be referred to
the General Membership Assembly. In case of an appeal, a simple majority of the decision of the Executive Board
is imperative. The same shall be approved/disapproved by a majority vote of the general membership assembly in
a meeting duly called for the purpose.
Thus, instant case is referred back to the Union for the General Assembly to act or deliberate complainant’s
appeal on the decision of the Executive Board.
Petitioner appealed before the NLRC.
NLRC RULING: NLRC ruled that the Commission lacks the jurisdictional competence to act on this case. Article
217 of the Labor Code, as amended, specifically enumerates the cases over which the Labor Arbiters and the
Commission have original and exclusive jurisdiction. That the disputes and conflict fall under the jurisdiction of the
Bureau of Labor Relations, as these are inter/intra-union disputes.
Petitioner filed a Petition for Certiorari with the CA
COURT OF APPEALS RULING: The petition lacks merit. CA also held that Petitioner’s causes of action against
MWEU are inter/intra-union disputes cognizable by the BLR whose functions and jurisdiction are largely confined
to union matters, collective bargaining registry, and labor education.
Petitioner’s Argument: Petitioner avers that the Labor Arbiter, the NLRC, and the CA failed to rule on his
accusation of unfair labor practices and simply dismissed his complaint on the ground that his causes of action
Angeles| Bajana | Balladares | Brillantes | Briones | Cabansag | Callanta | Chua | David|

De Leon | Gomez | Lopez | Macalino | Nostratis | Padilla | Reynon | Santos | Tan |Velasco
4E / 4F - 2018-2019
Page 285 of 920
LABOR REVIEW DIGEST
Atty. Joyrich Golangco

are intra- or inter-union in nature; that admittedly, some of his causes of action involved intra- or inter-union
disputes, but other acts of respondents constitute unfair labor practices. Specifically, petitioner claims that he was
suspended and expelled from MWEU illegally as a result of the denial of his right to appeal his case to the general
membership assembly in accordance with the union’s constitution and by-laws.
Respondent’s Arguments: That findings of fact of the CA are final and conclusive; that the Labor Arbiter, NLRC,
and CA are one in declaring that there is no unfair labor practices committed against petitioner; that petitioner’s
other allegations fall within the jurisdiction of the BLR, as they refer to intra- or inter-union disputes between the
parties.
ISSUE/S: 1. WON the NLRC has jurisdiction over the complaint despite the fact that the petition contains
allegations referring to inter/intra-union conflicts
2. WON respondent committed of unfair labor practices
SC RULING:
1. YES. It is true that some of petitioner’s causes of action constitute intra-union cases cognizable by the BLR
under Article 226 of the Labor Code. However, petitioner’s charge of unfair labor practices falls within
the original and exclusive jurisdiction of the Labor Arbiters, pursuant to Article 217 of the Labor Code. In addition,
Article 247 of the same Code provides that "the civil aspects of all cases involving unfair labor practices, which
may include claims for actual, moral, exemplary and other forms of damages, attorney’s fees and other affirmative
relief, shall be under the jurisdiction of the Labor Arbiters."
2. YES, the respondents are guilty of unfair labor practices.
The primary concept of unfair labor practices is stated in Article 247 of the Labor Code, which states:
Article 247. Concept of unfair labor practice and procedure for prosecution thereof. –– Unfair labor practices
violate the constitutional right of workers and employees to self-organization, are inimical to the legitimate
interests of both labor and management, including their right to bargain collectively and otherwise deal with each
other in an atmosphere of freedom and mutual respect, disrupt industrial peace and hinder the promotion of
healthy and stable labor-management relations.
"In essence, [unfair labor practice] relates to the commission of acts that transgress the workers’ right to
organize."50"[A]ll the prohibited acts constituting unfair labor practice in essence relate to the workers’ right to self-
organization."51 "[T]he term unfair labor practice refers to that gamut of offenses defined in the Labor Code which,
at their core, violates the constitutional right of workers and employees to self-organization."
As members of the governing board of MWEU, respondents are presumed to know, observe, and apply the
union’s constitution and by-laws. Thus, their repeated violations thereof and their disregard of petitioner’s rights as
a union member – their inaction on his two appeals which resulted in his suspension, disqualification from running
as MWEU officer, and subsequent expulsion without being accorded the full benefits of due process – connote
willfulness and bad faith, a gross disregard of his rights thus causing untold suffering, oppression and, ultimately,
ostracism from MWEU. "Bad faith implies breach of faith and willful failure to respond to plain and well understood
obligation.
Notes: ___________________________________________________________

Angeles| Bajana | Balladares | Brillantes | Briones | Cabansag | Callanta | Chua | David|

De Leon | Gomez | Lopez | Macalino | Nostratis | Padilla | Reynon | Santos | Tan |Velasco
4E / 4F - 2018-2019
Page 286 of 920
LABOR REVIEW DIGEST
Atty. Joyrich Golangco

24. HIJO RESOURCES CORPORATION v. MEJARES


G.R. No. 208986, January 13, 2016
Carpio, J.:

DOCTRINE: Although, it is undisputed that the Med-Arbiter has the authority to determine the existence of an
employer-employee relationship between the parties in a petition for certification election, his decision, however,
does not operate as res judicata and does not foreclose all further dispute between the parties as to the existence
or non-existence of an employer-employee relationship between them in the NLRC.

FACTS: Respondents Mejares, Baluran, Jr., Saycon, and Cucharo (respondents) were among the complainants,
represented by their labor union named "Nagkahiusang Mamumuo ng Bit, Djevon, at Raquilla Farms sa Hijo
Resources Corporation" (NAMABDJERA-HRC), who filed with the NLRC an illegal dismissal case against
petitioner Hijo Resources Corporation (HRC).

HRC is formerly known as Hijo Plantation Incorporated (HPI). HRC is the owner of agricultural lands in Madum,
Tagum, Davao del Norte, which were planted primarily with Cavendish bananas.

Complainants claimed that they were employed by HPI as farm workers in HPI's plantations occupying various
positions as area harvesters, packing house workers, loaders, or labelers. Complainants were absorbed by HRC,
but they were working under the contractor-growers: Bit Farm, Djevon Farm, Raquilla Farm. Complainants
asserted that these contractor-growers received compensation from HRC and were under the control of HRC.
They further alleged that the contractor-growers did not have their own capitalization, farm machineries, and
equipment.

Complainants formed their union NAMABDJERA-HRC, which was later registered with the Department of Labor
and Employment (DOLE). On 24 August 2007, NAMABDJERA-HRC filed a petition for certification election before
the DOLE.

When HRC learned that complainants formed a union, the three contractor-growers filed with the DOLE a notice
of cessation of business operations. In September 2007, complainants were terminated from their employment on
the ground of cessation of business operations by the contractor-growers of HRC. Complainants, represented by
NAMABDJERA-HRC, filed a case for unfair labor practices, illegal dismissal, and illegal deductions with prayer for
moral and exemplary damages and attorney's fees before the NLRC.

MED ARBITER: Issued an Order, dismissing NAMABDJERA-HRC's petition for certification election on the
ground that there was no employer-employee relationship between complainants and HRC. Complainants did not
appeal the Order of Med-Arbiter but pursued the illegal dismissal case they filed.

HRC moved to dismiss the complaint for illegal dismissal anchored on the following arguments: (1) Lack of
jurisdiction under the principle of res judicata; and (2) The Order of the Med-Arbiter finding that complainants were
not employees of HRC, which complainants did not appeal, had become final and executory.

LABOR ARBITER RULING: Labor Arbiter denied the motion to dismiss, ruling that res judicata does not apply.
Citing the cases of Manila Golf & Country Club, Inc. v. IAC and Sandoval Shipyards, Inc. v. Pepito, the Labor
Arbiter ruled that the decision of the Med-Arbiter in a certification election case, by the nature of that proceedings,
does not foreclose further dispute between the parties as to the existence or non-existence of employer-employee
relationship between them. Med-Arbiter findings does not bar the Labor Arbiter from making his own independent
finding on the same issue. The non-litigious nature of the proceedings before the Med-Arbiter does not prevent
the Labor Arbiter from hearing and deciding the case. Labor Arbiter ordered the parties to file their position
papers.
Angeles| Bajana | Balladares | Brillantes | Briones | Cabansag | Callanta | Chua | David|

De Leon | Gomez | Lopez | Macalino | Nostratis | Padilla | Reynon | Santos | Tan |Velasco
4E / 4F - 2018-2019
Page 287 of 920
LABOR REVIEW DIGEST
Atty. Joyrich Golangco

HRC filed with the NLRC a petition for certiorari with a prayer for temporary restraining order, seeking to nullify
Orders of Labor Arbiter.

NLRC RULING: The NLRC ruled otherwise, holding that Labor Arbiter gravely abused her discretion in denying
HRC's motion to dismiss. The NLRC held that the Med-Arbiter Order dismissing the certification election case on
the ground of lack of employer-employee relationship between HRC and complainants constitutes res
judicata under the concept of conclusiveness of judgment, and thus, warrants the dismissal of the case.

COURT OF APPEALS RULING: The Court of Appeals held that the certification proceedings before the Med-
Arbiter are non-adversarial and merely investigative. On the other hand, under Article 217 of the Labor Code, the
Labor Arbiter has original and exclusive jurisdiction over illegal dismissal cases. Although the proceedings before
the Labor Arbiter are also described as non-litigious, the Court of Appeals noted that the Labor Arbiter is given
wide latitude in ascertaining the existence of employment relationship. Thus, unlike the Med-Artbiter, the Labor
Arbiter may conduct clarificatory hearings and even avail of ocular inspection to ascertain facts speedily.

ISSUE/S:
1. Whether or not the Med-Arbiter has authority to determine the existence of employer-employee relationship
2. Whether or not the Labor Arbiter is bound by the ruling of the Med-Arbiter regarding existence or non-existence of
ER-EE Relationship

SC RULING:
1. Yes. Med-Arbiter has the authority to determine the existence of an employer-employee relationship between
the parties in a petition for certification election.
Under Article 226 of the Labor Code, as amended, the Bureau of Labor Relations (BLR), of which the med-arbiter
is an officer, has the following jurisdiction -
"ART. 226. Bureau of Labor Relations. - The Bureau of Labor Relations and the Labor Relations Divisions in the
regional offices of the Department of Labor shall have original and exclusive authority to act, at their own initiative
or upon request of either or both parties, on all inter-union and intra-union conflicts, and all disputes, grievances or
problems arising from or affecting labor-management relations in all workplaces whether agricultural or non-
agricultural, except those arising from the implementation or interpretation of collective bargaining agreements
which shall be the subject of grievance procedure and/or voluntary arbitration.

From the foregoing, the BLR has the original and exclusive jurisdiction to inter alia, decide all disputes, grievances
or problems arising from or affecting labor-management relations in all workplaces whether agricultural or non-
agricultural. Necessarily, in the exercise of this jurisdiction over labor-management relations, the med-arbiter has
the authority, original and exclusive, to determine the existence of an employer-employee relationship between
the parties.

Once there is a determination as to the existence of such a relationship, the med-arbiter can then decide the
certification election case. As the authority to determine the employer-employee relationship is necessary and
indispensable in the exercise of jurisdiction by the med-arbiter, his finding thereon may only be reviewed and
reversed by the Secretary of Labor who exercises appellate jurisdiction under Article 259 of the Labor Code, as
amended, which provides -

"ART. 259. Appeal from certification election orders. - Any party to an election may appeal the order or results of
the election as determined by the Med-Arbiter directly to the Secretary of Labor and Employment on the ground
that the rules and regulations or parts thereof established by the Secretary of Labor and Employment for the
conduct of the election have been violated. Such appeal shall be decided within fifteen (15) calendar days."

Angeles| Bajana | Balladares | Brillantes | Briones | Cabansag | Callanta | Chua | David|

De Leon | Gomez | Lopez | Macalino | Nostratis | Padilla | Reynon | Santos | Tan |Velasco
4E / 4F - 2018-2019
Page 288 of 920
LABOR REVIEW DIGEST
Atty. Joyrich Golangco

c
2. NO. The Labor Arbiter is not bound by the decision of the Med-Arbiter. In Sandoval Case, the DOLE
Undersecretary reversed the finding of the Med-Arbiter in a certification election case and ruled that there was no
employer-employee relationship between the members of the petitioner union and Sandoval Shipyards, Inc. (SSI),
since the former were employees of the subcontractors.

The Court also cited the ruling in the Manila Golf case that the decision in a certification election case, by the very
nature of that proceeding, does not foreclose all further dispute between the parties as to the existence or non-
existence of an employer-employee relationship between them.
Notes: ___________________________________________________________

Angeles| Bajana | Balladares | Brillantes | Briones | Cabansag | Callanta | Chua | David|

De Leon | Gomez | Lopez | Macalino | Nostratis | Padilla | Reynon | Santos | Tan |Velasco
4E / 4F - 2018-2019
Page 289 of 920
LABOR REVIEW DIGEST
Atty. Joyrich Golangco

25. Case Title: Milan vs. NLRC


GR No: 202961 Date: February 04, 2015
Ponente: Leonen, J.

Doctrine: The National Labor Relations Commission has jurisdiction to determine, preliminarily, the parties’ rights
over a property, when it is necessary to determine an issue related to rights or claims arising from an employer-
employee relationship.

FACTS: Petitioners are respondent Solid Mills, Inc.’s employees. They are represented by the National
Federation of Labor Unions (NAFLU), their collective bargaining agent. As Solid Mills’ employees, petitioners and
their families were allowed to occupy SMI Village, a property owned by Solid Mills. According to Solid Mills, this
was out of liberality and for the convenience of its employees and on the condition that the employees would
vacate the premises anytime the Company deems it.

Petitioners were informed that Solid Mills would cease its operations due to serious business losses. NAFLU
recognized Solid Mills’ closure due to serious business losses in the memorandum of agreement which provided
for Solid Mills’ grant of separation pay less accountabilities, accrued sick leave benefits, vacation leave benefits,
and 13th month pay to the employees.

Petitioners were required to sign a memorandum of agreement with release and quitclaim before their vacation
and sick leave benefits, 13th month pay, and separation pay would be released. Employees who signed the
memorandum of agreement were considered to have agreed to vacate SMI Village, and to the demolition of the
constructed houses inside as condition for the release of their termination benefits and separation pay. Petitioners
refused to sign the documents and demanded to be paid their benefits and separation pay.

Petitioners filed complaints before the Labor Arbiter for alleged non-payment of benefits. Solid Mills argue that
petitioners’ complaint was premature because they had not vacated its property.

LA/RTC/NLRC RULING: Petitioners’ possession is not by virtue of any employer-employee relationship. It is a


civil issue, which is outside the jurisdiction of the Labor Arbiter.

NLRC: The termination of Solid Mills and petitioners employer-employee relationship made it incumbent upon
petitioners to turn over the property to Solid Mills.

CA RULING: The Court of Appeals ruled that Solid Mills’ act of allowing its employees to make temporary
dwellings in its property was a liberality on its part. It may be revoked any time at its discretion. As a consequence
of Solid Mills’ closure and the resulting termination of petitioners, the employer-employee relationship between
them ceased to exist. There was no more reason for them to stay in Solid Mills’ property.

APPEAL TO THE SC:

Petitioner's Contention: Petitioners point out that the NLRC had no jurisdiction to declare that petitioners act of
withholding possession of respondent Solid Mills property is illegal. The regular courts have jurisdiction over this
issue. It is independent from the issue of payment of petitioners’ monetary benefits.

Respondent's Contention:

Angeles| Bajana | Balladares | Brillantes | Briones | Cabansag | Callanta | Chua | David|

De Leon | Gomez | Lopez | Macalino | Nostratis | Padilla | Reynon | Santos | Tan |Velasco
4E / 4F - 2018-2019
Page 290 of 920
LABOR REVIEW DIGEST
Atty. Joyrich Golangco

Respondents Solid Mills argue that petitioners’ failure to turn over respondent Solid Mills’ property constituted an
unsatisfied accountability for which reason petitioners’ benefits could rightfully be withheld.

ISSUE/S: Whether or not the LA and NLRC had jurisdiction over the issue on possession of the properties

HELD: YES. The National Labor Relations Commission has jurisdiction to determine, preliminarily, the parties’
rights over a property, when it is necessary to determine an issue related to rights or claims arising from an
employer-employee relationship.

Claims arising from an employer-employee relationship are not limited to claims by an employee. Employers may
also have claims against the employee, which arise from the same relationship. As a rule a claim only needs to be
sufficiently connected to the labor issue raised and must arise from an employer-employee relationship for the
labor tribunals to have jurisdiction.

In this case, respondent Solid Mills claims that its properties are in petitioners’ possession by virtue of their status
as its employees. Respondent Solid Mills allowed petitioners to use its property as an act of liberality. Put in other
words, it would not have allowed petitioners to use its property had they not been its employees. The return of its
properties in petitioners’ possession by virtue of their status as employees is an issue that must be resolved to
determine whether benefits can be released immediately. The issue raised by the employer is, therefore,
connected to petitioners’ claim for benefits and is sufficiently intertwined with the parties’ employer-employee
relationship.

Other Notes/ SC Pronouncements:

_____________________________________________________________________

Angeles| Bajana | Balladares | Brillantes | Briones | Cabansag | Callanta | Chua | David|

De Leon | Gomez | Lopez | Macalino | Nostratis | Padilla | Reynon | Santos | Tan |Velasco
4E / 4F - 2018-2019
Page 291 of 920
LABOR REVIEW DIGEST
Atty. Joyrich Golangco

26. Case Title: Saudi Arabia Airlines vs. Rebesencio


GR No: 198587 Date: January 14, 2015
Ponente: Leonen, J.

Doctrine: Apart from the constitutional policy on the fundamental equality before the law of men and women, it is
settled that contracts relating to labor and employment are impressed with public interest. Article 1700 of the Civil
Code provides that "[t]he relation between capital and labor are not merely contractual. They are so impressed
with public interest that labor contracts must yield to the common good."

As the present dispute relates to (what the respondents allege to be) the illegal termination of respondents'
employment, this case is immutably a matter of public interest and public policy. Consistent with clear
pronouncements in law and jurisprudence, Philippine laws properly find application in and govern this case.

FACTS: Saudi Arabian Airlines is a foreign corporation established and existing under the laws of Jeddah,
Kingdom of Saudi Arabia. Respondents (complainants before the Labor Arbiter) were recruited and hired by
Saudia as Permanent Flight Attendants.

Respondents continued their employment with Saudia until they were separated from service on various dates in
2006. Respondents contended that the termination of their employment was illegal. They alleged that the
termination was made solely because they were pregnant.

Respondents filed a Complaint against Saudia and its officers for illegal dismissal before the Labor Arbiter. Saudia
assailed the jurisdiction of the Labor Arbiter. It claimed that all the determining points of contact referred to foreign
law and insisted that the Complaint ought to be dismissed on the ground of forum non conveniens.

Saudia asserts that Philippine courts and/or tribunals are not in a position to make an intelligent decision as to the
law and the facts. This is because respondents' Cabin Attendant contracts require the application of the laws of
Saudi Arabia, rather than those of the Philippines.

LA/RTC/NLRC RULING: Executive Labor Arbiter Fatima Jambaro-Franco rendered the Decision32 dismissing
respondents' Complaint. The dispositive portion of this Decision reads:
WHEREFORE, premises' considered, judgment is hereby rendered DISMISSING the instant complaint for lack of
jurisdiction/merit

On respondents' appeal, the National Labor Relations Commission's Sixth Division reversed the ruling of
Executive Labor Arbiter Jambaro-Franco. It explained that "[considering that complainants-appellants are OFWs,
the Labor Arbiters and the NLRC has [sic] jurisdiction to hear and decide their complaint for illegal termination.

CA RULING:
The CA upheld the ruling of the NLRC

APPEAL TO THE SC:

Petitioner's Contention: See facts

Angeles| Bajana | Balladares | Brillantes | Briones | Cabansag | Callanta | Chua | David|

De Leon | Gomez | Lopez | Macalino | Nostratis | Padilla | Reynon | Santos | Tan |Velasco
4E / 4F - 2018-2019
Page 292 of 920
LABOR REVIEW DIGEST
Atty. Joyrich Golangco

ISSUE/S: Whether or not the Philippine tribunals have jurisdiction over the case

HELD: YES. A choice of law governing the validity of contracts or the interpretation of its provisions does not
necessarily imply forum non conveniens. Choice of law and forum non conveniens are entirely different matters.

Choice of law provisions are an offshoot of the fundamental principle of autonomy of contracts. In contrast, forum
non conveniens is a device akin to the rule against forum shopping. It is designed to frustrate illicit means for
securing advantages and vexing litigants that would otherwise be possible if the venue of litigation (or dispute
resolution) were left entirely to the whim of either party.

Likewise, contractual choice of law is not determinative of jurisdiction. Stipulating on the laws of a given
jurisdiction as the governing law of a contract does not preclude the exercise of jurisdiction by tribunals elsewhere.
The reverse is equally true: The assumption of jurisdiction by tribunals does not ipso facto mean that it cannot
apply and rule on the basis of the parties' stipulation.

Forum non conveniens finds no application and does not operate to divest Philippine tribunals of jurisdiction and
to require the application of foreign law.

While a Philippine tribunal (acting as the forum court) is called upon to respect the parties' choice of governing
law, such respect must not be so permissive as to lose sight of considerations of law, morals, good customs,
public order, or public policy that underlie the contract central to the controversy.

Counter-balancing the principle of autonomy of contracting parties is the equally general rule that provisions of
applicable law, especially provisions relating to matters affected with public policy, are deemed written inta the
contract. Put a little differently, the governing principle is that parties may not contract away applicable provisions
of law especially peremptory provisions dealing with matters heavily impressed with public interest.

This does not only mean that the Philippines shall not countenance nor lend legal recognition and approbation to
measures that discriminate on the basis of one's being male or female. It imposes an obligation to actively engage
in securing the fundamental equality of men and women.

So informed and animated, we emphasize the glaringly discriminatory nature of Saudia's policy. As argued by
respondents, Saudia's policy entails the termination of employment of flight attendants who become pregnant. At
the risk of stating the obvious, pregnancy is an occurrence that pertains specifically to women. Saudia's policy
excludes from and restricts employment on the basis of no other consideration but sex.

We do not lose sight of the reality that pregnancy does present physical limitations that may render difficult the
performance of functions associated with being a flight attendant. Nevertheless, it would be the height of iniquity
to view pregnancy as a disability so permanent and immutable that, it must entail the termination of one's
employment. It is clear to us that any individual, regardless of gender, may be subject to exigencies that limit the
performance of functions. However, we fail to appreciate how pregnancy could be such an impairing occurrence
that it leaves no other recourse but the complete termination of the means through which a woman earns a living.

Apart from the constitutional policy on the fundamental equality before the law of men and women, it is settled that
contracts relating to labor and employment are impressed with public interest. Article 1700 of the Civil Code
provides that "[t]he relation between capital and labor are not merely contractual. They are so impressed with
public interest that labor contracts must yield to the common good."

Angeles| Bajana | Balladares | Brillantes | Briones | Cabansag | Callanta | Chua | David|

De Leon | Gomez | Lopez | Macalino | Nostratis | Padilla | Reynon | Santos | Tan |Velasco
4E / 4F - 2018-2019
Page 293 of 920
LABOR REVIEW DIGEST
Atty. Joyrich Golangco

As the present dispute relates to (what the respondents allege to be) the illegal termination of respondents'
employment, this case is immutably a matter of public interest and public policy. Consistent with clear
pronouncements in law and jurisprudence, Philippine laws properly find application in and govern this case.
'Moreover, as this premise for Saudia's insistence on the application forum non conveniens has been shattered, it
follows that Philippine tribunals may properly assume jurisdiction over the present controversy.

Other Notes/ SC Pronouncements:

______________________________________________________________________

Angeles| Bajana | Balladares | Brillantes | Briones | Cabansag | Callanta | Chua | David|

De Leon | Gomez | Lopez | Macalino | Nostratis | Padilla | Reynon | Santos | Tan |Velasco
4E / 4F - 2018-2019
Page 294 of 920
LABOR REVIEW DIGEST
Atty. Joyrich Golangco

27. Case Title: Amecos Innovations vs. Lopez


GR No: 178055 Date: January 2, 2014
Ponente: Del Castillo, J.

Angeles| Bajana | Balladares | Brillantes | Briones | Cabansag | Callanta | Chua | David|

De Leon | Gomez | Lopez | Macalino | Nostratis | Padilla | Reynon | Santos | Tan |Velasco
4E / 4F - 2018-2019
Page 295 of 920
LABOR REVIEW DIGEST
Atty. Joyrich Golangco

Doctrine: Under Article 217(a)(4) of the Labor Code, claims for actual, moral, exemplary and other forms
of damages arising from employer-employee relationship are under the jurisdiction of the Labor Arbiters or the
National Labor Relations Commission

FACTS: Amecos Innovations, Inc. (Amecos) was complained by the Social Security System (SSS) for alleged
delinquency in the remittance of SSS contributions. Amecos attributed its failure to remit the SSS contributions to
Eliza R. Lopez(Lopez) claiming that it hired Lopez but she refused to provide Amecos with her SSS Number.
Hence, Amecos no longer enrolled Lopez with the SSS and did not deduct her corresponding contributions up to
the time of her termination. The complaint was withdrawn upon settlement of the obligation by Amecos. Lopez did
not heed the demands of Amecos, thus, the latter filed a complaint for sum of money and damages against Lopez
before the Regional Trial Court (RTC).
Amecos claimed that because of Lopez’s misrepresentation, they suffered actual damages by way of settlement
and payment of its obligations with the SSS. Amecos’ contention is that the employer-employee relationship
between Amecos and Lopez is merely incidental, and does not necessarily place their dispute within the exclusive
jurisdiction of the labor tribunals but the true source of Lopez’s obligation is derived from Articles 19, 22, and 2154
of the Civil Code.

LA/RTC/NLRC RULING:

MTC dismissed the case for lack of jurisdiction. RTC affirmed decision of MTC:

“The RTC affirmed the view taken by the MeTC that under Article 217(a)(4) of the Labor Code, claims for actual,
moral, exemplary and other forms of damages arising from employer-employee relationship are under the
jurisdiction of the Labor Arbiters or the National Labor Relations Commission (NLRC); that since petitioners and
respondent were in an employer-employee relationship at the time, the matter of SSS contributions was thus an
integral part of that relationship; and as a result, petitioners’ cause of action for recovery of damages from
respondent falls under the jurisdiction of the Labor Arbiters, pursuant to Article 217(a)(4) of the Labor Code.”

CA RULING:
CA affirmed decision of RTC and denied MR

APPEAL TO THE SC:

Petitioner's Contention: petitioners argue that their Complaint is one for recovery of a sum of money and
damages, thus, jurisdiction resides with the regular courts, that the employer-employee relationship between
Amecos and respondent is merely incidental, and does not necessarily place their dispute within the exclusive
jurisdiction of the labor tribunals.

Respondent’s Contention: jurisdiction over petitioners’ case lies with the Labor Arbiter, as their cause of action
remains necessarily connected to and arose from their employer-employee relationship. At any rate, respondent
insists that petitioners, as employers, have the legal duty to enroll her with the SSS as their employee and to pay
or remit the necessary contributions.

ISSUE/S: WON the the regular courts have jurisdiction over claims for reimbursement arising from an EE-ER
relationship?

Angeles| Bajana | Balladares | Brillantes | Briones | Cabansag | Callanta | Chua | David|

De Leon | Gomez | Lopez | Macalino | Nostratis | Padilla | Reynon | Santos | Tan |Velasco
4E / 4F - 2018-2019
Page 296 of 920
LABOR REVIEW DIGEST
Atty. Joyrich Golangco

HELD: No. SSS contributions and recovery of damages arising from employee-employer relationship is under the
jurisdiction of the Labor Arbiters. This Court holds that as between the parties, Article 217(a)(4) of the Labor Code
is applicable. Said provision bestows upon the Labor Arbiter original and exclusive jurisdiction over claims for
damages arising from employer-employee relations. The observation that the matter of SSS contributions
necessarily flowed from the employer-employee relationship between the parties shared by the lower courts and
the Court of Appeals (CA) is correct; thus, Amecos claims should have been referred to the labor tribunals. In this
connection, it is noteworthy to state that the Labor Arbiter has jurisdiction to award not only the reliefs provided by
labor laws, but also damages governed by the Civil Code.
At the same time, it cannot be assumed that since the dispute concerns the payment of SSS premiums,
petitioners’ claim should be referred to the Social Security Commission (SSC) pursuant to Republic Act No. 1161,
as amended by Republic Act No. 8282.35 As far as SSS is concerned, there is no longer a dispute with respect to
petitioners’ accountability to the System; petitioners already settled their pecuniary obligations to it. Since there is
no longer any dispute regarding coverage, benefits, contributions and penalties to speak of, the SSC need not be
unnecessarily dragged into the picture.36 Besides, it cannot be made to act as a collecting agency for petitioners’
claims against the respondent; the Social Security Law should not be so interpreted, lest the SSC be swamped
with cases of this sort.

Angeles| Bajana | Balladares | Brillantes | Briones | Cabansag | Callanta | Chua | David|

De Leon | Gomez | Lopez | Macalino | Nostratis | Padilla | Reynon | Santos | Tan |Velasco
4E / 4F - 2018-2019
Page 297 of 920
LABOR REVIEW DIGEST
Atty. Joyrich Golangco

28. Case Title: Cacho vs. Balagtas


GR No: 202974 Date: February 7, 2018
Ponente: Del Castillo, J.

Facts:
Respondent Virginia D. Balagtas filed a complaint of constructive dismissal against petitioners North Star
International Travel, Inc. (North Star) and itsPresident Norma D. Cacho (Cacho) before the Labor Arbiter.
Balagtas after 14 years of service in the said corporation, was placed under30 days preventive suspension
pursuant to a Board Resolution passed by the Board of Directors of the respondent Corporation due to her alleged
questionable transactions. While under preventive suspension, she wrote a letter to Norma Cacho informing the
latter that she was assuming her position as Executive Vice-President/Chief Executive Officer effective on that
date; however, she was prevented from re-assuming her position.
Consequently, she filed a complaint claiming that she was constructively and illegally dismissed effective on April
12, 2004. In their defense, Cacho and North Star averred that preventive suspension was meant to prevent
Balagtas from influencing potential witnesses and to protect the respondent corporation's property. Subsequently,
the Board of Directors constituted an investigation committeetasked with the duty to impartially assess the
charges against petitioner. Cacho, et al. alleged that Balagtas violated her suspension when, on several
occasions, she went to the corporation's office and insisted on working despite respondent Norma Cacho's
protestation. They asserted that petitioner was not illegally dismissed but was merely placed under preventive
suspension.

LA/RTC/NLRC RULING:
The Labor Arbiter found that Balagtas was illegally dismissed from North Star but the latter appealed to the NLRC
for lack of jurisdiction. They contend that Balagtas was never dismissed and alleged that she was a corporate
officer, incorporator, and member of the North Star's Board of Directors. The Labor Arbiter found that Balagtas
was illegally dismissed from North Star but the latter appealed to the NLRC for lack of jurisdiction. They contend
that Balagtas was never dismissed and alleged that she was a corporate officer, incorporator, and member of the
North Star's Board of Directors.

CA RULING:
However, the CA affirmed the Labor Arbiter’s Decision and set aside the Decision of the NLRC.

ISSUE:
WON the present case is an intra-corporate controversy within the jurisdiction of the regular courts or an ordinary
labor dispute that the Labor Arbiter may properly take cognizance of.

HELD:
Respondent Balagtas's dismissal is an intra-corporate controversy.

A two-tier test must be employed to determine whether an intra-corporate controversy exists in the present case,
viz.: (a) the relationship test, and (b) the nature of the controversy test.

A dispute is considered an intra-corporate controversy under the relationship test when the relationship between
or among the disagreeing parties is anyone of the following: (a) between the corporation, partnership, or
association and the public; (b) between the corporation, partnership, or association and its stockholders, partners,
members, or officers; ( c) between the corporation, partnership, or association and the State as far as its
franchise, permit or license to operate is concerned; and ( d) among the stockholders, partners, or associates

Angeles| Bajana | Balladares | Brillantes | Briones | Cabansag | Callanta | Chua | David|

De Leon | Gomez | Lopez | Macalino | Nostratis | Padilla | Reynon | Santos | Tan |Velasco
4E / 4F - 2018-2019
Page 298 of 920
LABOR REVIEW DIGEST
Atty. Joyrich Golangco

themselves. We must now determine whether or not the Executive Vice President position is a corporate office so
as to establish the intra-corporate relationship between the parties.

One shall be considered a corporate officer only if two conditions are met, viz.: ( 1) the position occupied was
created by charter/by-laws, and (2) the officer was elected (or appointed) by the corporation's board of directors
tooccupy said position.

The Executive Vice President position is one of the corporate offices provided in petitioner North Star's By-laws.
Section 25 of the Corporation Code explicitly provides for the election of the corporation's president, treasurer,
secretary, and such other officers as may be provided for in the by-laws. In interpreting this provision, the Court
has ruled that if the position is other than the corporate president, treasurer, or secretary, it must be expressly
mentioned in the bylaws in order to be considered as a corporate office. North Star’s by-laws provides that there may be
one or more vice president positions in petitioner North Star and, by virtue of its by-laws, all such positions shall be
corporate offices.

The next question is whether or not the phrase "one or more vice president" in the above-cited provision of the by-
laws includes the Executive Vice President position held by respondent Balagtas.

The use of the phrase "one or more" in relation to the establishment of vice president positions without particular
exception indicates an intention to give petitioner North Star's Board ample freedom to make several vice
president positions available as it may deem fit and in consonance with sound business practice. To require that
particular designation/variation of each vice-president (i.e., executive vice president) be specified and enumerated
is to invalidate the by-laws' true intention and to encroach upon petitioner North Star's inherent right and authority
to adopt its own set of rules and regulations to govern its internal affairs. By name, the Executive Vice President
position is embraced by the phrase "one or more vice president" in North Star's by-laws.2.

Respondent Balagtas was appointed by the Board as petitioner North Star's Executive Vice President While a
corporate office is created by an express provision either in the Corporation Code or the By-laws, what makes one
a corporate officer is his election or appointment thereto by the board of directors. Thus, there must be
documentary evidence to prove that the person alleged to be a corporate officer was appointed by action or with
approval of the board. Petitioners Cacho and North Star assert that respondent Balagtas was elected as
Executive Vice President by the Board as evidenced by the Secretary's Certificate dated April 22, 2003.

The above-cited Secretary's Certificate overcomes respondent Balagtas's contention that she was merely the Executive
Vice President by name and was never empowered to exercise the functions of a corporate officer. Notably, she did not
offer any proof to show that her duties, functions, and compensation were all determined by petitioner Cacho as
petitioner NorthStar's President. Respondent Balagtas also denies her status as one of petitioner North Star's
corporate officers because she was not listed as such in petitioner NorthStar's 2003 General Information Sheet
(GIS). But the GIS neither governs nor establishes whether or not a position is an ordinary or corporate office. At
best, if one is listed in the GIS as an officer of a corporation, his/her position as indicated therein could only be
deemed a regular office, and not a corporate office as it is defined under the Corporation Code.

To be considered an intra-corporate controversy, the dismissal of a corporate officer must have something to do
with the duties and responsibilities attached to his/her corporate office or performed in his/her official capacity.
The termination complained of is intimately and inevitably linked to respondent Balagtas's role as petitioner North
Star's Executive Vice President: first, the alleged misappropriations were committed by respondent Balagtas in
her capacity as vice president, one of the officer sresponsible for approving the disbursements and signing the
checks. And, second, these alleged misappropriations breached petitioners Cacho's and North Star's trust and
confidence specifically reposed m respondent Balagtas as vice president. That all these incidents are adjuncts

Angeles| Bajana | Balladares | Brillantes | Briones | Cabansag | Callanta | Chua | David|

De Leon | Gomez | Lopez | Macalino | Nostratis | Padilla | Reynon | Santos | Tan |Velasco
4E / 4F - 2018-2019
Page 299 of 920
LABOR REVIEW DIGEST
Atty. Joyrich Golangco

of her corporate office lead the Court to conclude that respondent Balagtas's dismissal is an intra-corporate
controversy, not a mere labor dispute.

All told, the issue in the present case is an intra-corporate controversy, a matter outside the Labor Arbiter's
jurisdiction.

Angeles| Bajana | Balladares | Brillantes | Briones | Cabansag | Callanta | Chua | David|

De Leon | Gomez | Lopez | Macalino | Nostratis | Padilla | Reynon | Santos | Tan |Velasco
4E / 4F - 2018-2019
Page 300 of 920
LABOR REVIEW DIGEST
Atty. Joyrich Golangco

29. PAL vs ALPAP


GR No: 200088 Date: February 26, 2018
Ponente: Martires, J.

Doctrine:
Where the plaintiff’s cause of action for damages arose out of or was necessarily intertwined with an alleged
unfair labor practice, the jurisdiction is exclusively with the labor tribunal. Likewise, where the damages separately
claimed by the employer were allegedly incurred as a consequence of strike or picketing of the union, such
complaint for damages is deeply rooted in the labor dispute between the parties and within the exclusive
jurisdiction of the labor arbiter.

FACTS:
The present case arose from a labor dispute between petitioner Philippine Airlines, Inc. (PAL) and respondent
Airline Pilots' Association of the Philippines (ALPAP), a duly registered labor organization and the exclusive
bargaining agent of all commercial pilots of PAL. ALPAP filed with the Department of Labor and
Employment (DOLE) a notice of strike alleging that PAL committed unfair labor practice. The Secretary of
DOLE (SOLE) assumed jurisdiction over the dispute and thereafter prohibited ALPAP from staging a strike and
committing any act that could exacerbate the dispute. Despite the prohibition by the SOLE, ALPAP staged a strike
on 5 June 1998. A return-to-work order was issued by the SOLE on 7 June 1998, but ALPAP defied the same and
went on with their strike. Consequently, on 1 June 1999, the SOLE issued a resolutionwhich declared the illegality
of the strike staged by ALPAP and the loss of employment status of the officers who participated in the strike.

The SOLE's resolution was upheld by the CA in CA-G.R. SP No. 54880. The matter was eventually elevated to
this Court in G.R. No. 152306. In a Resolution, dated 10 April 2002, the Court dismissed ALPAP's petition for
failure to show that the CA committed grave abuse of discretion or a reversible error. The resolution attained
finality on 29 August 2002.

On 22 April 2003, or almost eight (8) months :from the finality of the Court's 10 April 2002 Resolution, PAL filed
before the LA a complaint for damages against ALPAP, as well as some of its officers and members.

PAL alleged, among others, that on 6 June 1998, the second day of the illegal strike conducted by ALPAP, its
striking pilots abandoned three (3) PAL aircraft, as follows: (i) PR 730 bound for Paris, France, at Bangkok,
Thailand; (ii) PR 741 bound for Manila, at Bangkok, Thailand; and (iii) PR 104 bound for Manila, at San Francisco,
California, U.S.A. Because of the deliberate and malicious abandonment of the said flights, its passengers were
stranded, and rendered PAL liable for violation of its contract of carriage. Thus, PAL was compelled to incur
expenses by way of hotel accommodations, meals for the stranded passengers, airport parking fees, and other
operational expenses. PAL further alleged that its operation was crippled by the illegal strike resulting in several
losses from ticket refunds, extraordinary expenses to cope with the shutdown situation, and lost income from the
cancelled domestic and international flights. PAL claimed that, as a result of the illegal strike, it suffered actual
damages in the amount of P731,078,988.59. PAL further prayed that it be awarded P300,000,000.00 and
P3,000,000.00 as exemplary damages and attorney's fees, respectively.

LA RULING:
LA dismissed PAL's complaint. It ruled that it had no jurisdiction to resolve the issue on damages. It noted that the
SOLE did not certify the controversy for compulsory arbitration to the NLRC nor in any occasion did the parties
agree to refer the same to voluntary arbitration under Article 263(h) of the Labor Code. Hence, jurisdiction to
resolve all issues arising from the labor dispute, including the claim for damages arising from the illegal strike, was
left with the SOLE to the exclusion of all other fora.

NLRC RULING:
Angeles| Bajana | Balladares | Brillantes | Briones | Cabansag | Callanta | Chua | David|

De Leon | Gomez | Lopez | Macalino | Nostratis | Padilla | Reynon | Santos | Tan |Velasco
4E / 4F - 2018-2019
Page 301 of 920
LABOR REVIEW DIGEST
Atty. Joyrich Golangco

NLRC affirmed with modification the LA's 22 April 2008 decision. It ruled that labor tribunals have no jurisdiction
over the claims interposed by PAL. It opined that the reliefs prayed for by PAL should have been ventilated before
the regular courts considering that they are based on the tortuous acts allegedly committed by the respondents. It
explained that the airline pilots' refusal to fly their assigned aircrafts constitutes breach of contractual obligation
which is intrinsically a civil dispute.

CA RULING:
CA partially granted PAL's petition. It ruled that while the NLRC correctly sustained the LA's dismissal of the
complaint for lack of jurisdiction, it declared that the NLRC gravely abused its discretion when it affirmed the LA's
pronouncement that PAL's cause of action had already prescribed.

The appellate court concurred with the NLRC's opinion that exclusive jurisdiction over PAL's claim for damages
lies with the regular courts and not with the SOLE. It ratiocinated that while Article 263(g) of the Labor Code vests
in the SOLE the authority to resolve all questions and controversies arising from a labor dispute over which it
assumed jurisdiction, said authority must be interpreted to cover only those causes of action which are based on
labor laws. Stated differently, causes of action based on an obligation or duty not provided under the labor laws
are beyond the SOLE's jurisdiction. It continued that only those issues that arise from the assumed labor dispute,
which has a direct causal connection to the employer-employee relationship between the parties, will fall under
the jurisdiction of the SOLE. It pointed out that the damages caused by the wilful acts of the striking pilots in
abandoning their aircraft are recoverable under civil law and are thus within the jurisdiction of the regular courts.

ISSUE/S:
Whether the NLRC and the Labor Arbiter have jurisdiction over PAL's claims against the respondents for
damages incurred as a consequence of the latter's actions during the illegal strike

HELD:
YES.
Under Article 217 [now Article 224] of the Labor Code, as amended by Section 9 of R.A. No. 6715, the LA and the
NLRC have jurisdiction to resolve cases involving claims for damages arising from employer-employee
relationship, to wit:
ART. 217. Jurisdiction of Labor Arbiters and the Commission-- (a) Except as otherwise provided under this Code,
the Labor Arbiters shall have original and exclusive jurisdiction to hear and decide, within thirty (30) calendar days
after the submission of the case by the parties for decision without extension, even in the absence of stenographic
notes, the following cases involving all workers, whether agricultural or nonagricultural:
1. Unfair labor practice cases;
2. Termination disputes;
3. If accompanied with a claim for reinstatement, those cases that workers may file involving wages, rates of pay,
hours of work and other terms and conditions of employment
4. Claims for actual, moral, exemplary and other forms of damages arising from employer-employee
relations;
5. Cases arising from any violation of Article 264 of this Code including questions involving the legality of
strikes and lockouts; and
6. Except claims for Employees Compensation, Social Security, Medicare and maternity benefits, all other claims,
arising from employer-employee relations, including those of persons in domestic or household service, involving
an amount exceeding five thousand pesos (P5,000.00) regardless of whether accompanied with a claim for
reinstatement.

It is settled, however, that not every controversy or money claim by an employee against the employer or vice-
versa falls within the jurisdiction of the labor arbiter. Intrinsically, civil disputes, although involving the claim of an
employer against its employees, are cognizable by regular courts.
Angeles| Bajana | Balladares | Brillantes | Briones | Cabansag | Callanta | Chua | David|

De Leon | Gomez | Lopez | Macalino | Nostratis | Padilla | Reynon | Santos | Tan |Velasco
4E / 4F - 2018-2019
Page 302 of 920
LABOR REVIEW DIGEST
Atty. Joyrich Golangco

To determine whether a claim for damages under paragraph 4 of Article 217 is properly cognizable by the labor
arbiter, jurisprudence has evolved the "reasonable connection rule" which essentially states that the claim for
damages must have reasonable causal connection with any of the claims provided for in that article. A money
claim by a worker against the employer or vice-versa is within the exclusive jurisdiction of the labor arbiter only if
there is a "reasonable causal connection" between the claim asserted and employee-employer relations. Only if
there is such a connection with the other claims can the claim for damages be considered as arising from
employer-employee relations. Absent such a link, the complaint will be cognizable by the regular courts.

The appellate court was of the opinion that, applying the reasonable connection rule, PAL's claims for damages
have no relevant connection whatsoever to the employer-employee relationship between the parties. Thus, the
claim is within the exclusive jurisdiction of the regular courts. It explained that Article 217 of the Labor Code does
not include a claim for damages wherein the employer-employee relation is merely incidental, and where the claim
is largely civil in character.

The appellate court is mistaken.


The Court agrees with PAL that its claim for damages has reasonable connection with its employer-employee
relationship with the respondents. Contrary to the pronouncements made by the appellate court, PAL's cause of
action is not grounded on mere acts of quasi-delict. The claimed damages arose from the illegal strike and acts
committed during the same which were in tum closely related and intertwined with the respondents' allegations of
unfair labor practices against PAL. This could not even be disputed as even the appellate court recognized this
fact.

Since the loss and injury from which PAL seeks compensation have reasonable causal connection with the
alleged acts of unfair labor practice, a claim provided for in Article 217 of the Labor Code, the question of
damages becomes a labor controversy and is therefore an employment relationship dispute.

Jurisprudence dictates that where the plaintiffs cause of action for damages arose out of or was necessarily
intertwined with an alleged unfair labor practice, the jurisdiction is exclusively with the labor tribunal. Likewise,
where the damages separately claimed by the employer were allegedly incurred as a consequence of strike or
picketing of the union, such complaint for damages is deeply rooted in the labor dispute between the parties and
within the exclusive jurisdiction of the labor arbiter. Consequently, the same should be dismissed by ordinary
courts for lack of jurisdiction.

From the foregoing, it is clear that the regular courts do not have jurisdiction over PAL's claim of damages, the
same being intertwined with its labor dispute with the respondents over which the SOLE had assumed jurisdiction.
It is erroneous, therefore, for the CA to even suggest that PAL's complaint should have been ventilated before the
trial court.

Other Notes/ SC Pronouncements:


_________________________________________________________________________

Angeles| Bajana | Balladares | Brillantes | Briones | Cabansag | Callanta | Chua | David|

De Leon | Gomez | Lopez | Macalino | Nostratis | Padilla | Reynon | Santos | Tan |Velasco
4E / 4F - 2018-2019
Page 303 of 920
LABOR REVIEW DIGEST
Atty. Joyrich Golangco

30. LRTA vs Alvarez


GR No: 188047 Date: November 28, 2016
Ponente: Jardeleza, J.

Doctrine: (main doctrine under the article)


For having conducted business through a private corporation, GOCC must submit itself to the provisions
governing private corporations, including the Labor Code. Thus, the money claim brought against it falls under the
original and exclusive jurisdiction of the LA.

FACTS:
LRTA is a government-owned and controlled corporation created by virtue of Executive Order No. 603, for the
purpose of the construction, operation, maintenance, and/or lease of light rail transit system in the
Philippines. Private respondents Bienvenido R. Alvarez, Carlos S. Velasco, Ascencion A. Gargalicano, Marlon E.
Aguinaldo, Petronilo T. Legaspi, Bonifacio A. Estopia, Andre A. Dela Merced, Jose Novier D. Bayot, Rolando C.
Amazona and Marlino G. Herrera (private respondents) are former employees of Meralco Transit Organization,
Inc. (METRO).

On June 8, 1984, METRO and LRTA entered into an agreement called "Agreement for the Management and
Operation of the Light Rail Transit System" (AMO-LRTS) for the operation and management of the light rail transit
system. LRTA shouldered and provided for all the operating expenses of METRO. Also, METRO signed a
Collective Bargaining Agreement (CBA) with its employees wherein provisions on wage increases and benefits
were approved by LRTA's Board of Directors.

However, on April 7, 1989, the Commission on Audit (COA) nullified and voided the AMO-LRTS. To resolve the
issue, LRTA decided to acquire METRO by purchasing all of its shares of stocks on June 8, 1989. METRO, thus,
became a wholly-owned subsidiary of LRTA. Since then, METRO has been renamed to Metro Transit
Organization, Inc. Also, by virtue of the acquisition, LRTA appointed the new set of officers, from chairman to
members of the board, and top management of METRO. LRTA and METRO declared and continued the
implementation of the AMO-LRTS and the non-interruption of employment relations of the employees of METRO.
They likewise continued the establishment and funding of the Metro, Inc. Employees Retirement Plan which
covers the past services of all METRO regular employees from the date of their employment. They confirmed that
all CBAs remained in force and effect. LRTA then sanctioned the CBA's of the union of rank and file employees
and the union of supervisory employees.

On November 17, 1997, the METRO general manager (who was appointed by LRTA) announced in a
memorandum that its board of directors approved the severance/resignation benefit of METRO employees at one
and a half (1 1/2) months salaries for every year of service.

On July 25, 2000, the union of rank and file employees of METRO declared a strike over a retirement fund
dispute. By virtue of its ownership of METRO, LRTA assumed the obligation to update the Metro, Inc. Employees
Retirement Fund with the Bureau of Treasury.

A few months later, or on September 30, 2000, LRTA stopped the operation of METRO. On April 5, 2001,
METRO's Board of Directors approved the release and payment of the first fifty percent (50%) of the severance
pay to the displaced METRO employees, including private respondents, who were issued certifications of eligibility
for severance pay along with the memoranda to receive the same.

Upon the request of the COA corporate auditor assigned at LRTA, COA issued an Advisory Opinion through its
Legal Department, and an Advise (sic) from Chairman Guillermo N. Carague, that LRTA is liable, as owner of its
wholly-owned subsidiary METRO, to pay the severance pay of the latter's employees.
Angeles| Bajana | Balladares | Brillantes | Briones | Cabansag | Callanta | Chua | David|

De Leon | Gomez | Lopez | Macalino | Nostratis | Padilla | Reynon | Santos | Tan |Velasco
4E / 4F - 2018-2019
Page 304 of 920
LABOR REVIEW DIGEST
Atty. Joyrich Golangco

LRTA earmarked an amount of P271,000,000.00 for the severance pay of METRO employees in its approved
corporate budget for the year 2002. However, METRO only paid the first fifty percent (50%) of the severance pay
of private respondents.
Private respondents repeatedly and formally asked LRTA, being the principal owner of METRO, to pay the
balance of their severance pay, but to no avail. Thus, they filed a complaint before the Arbitration Branch of the
NLRC, docketed as NLRC NCR Case No. 00-08-09472-04, praying for the payment of 13thmonth pay, separation
pay, and refund of salary deductions, against LRTA and METRO.
LA RULING:
Labor Arbiter ruled in favor of private respondents.
NLRC RULING:
NLRC dismissed METRO's appeal for failure to file the required appeal bond:Therefore, the NLRC ruled that the
appealed Decision of the LA (as regards METRO) is declared final and executory.

CA RULING:
The CA denied LRTA's petition. First, the CA ruled that since LRTA failed to comply with the mandatory appeal
bond, it lost its right to appeal. Consequently, the LA's ruling already became final and executory.

On the merits of the case, the CA noted that the monetary claims emanated from the CBA; hence, the controversy
must be settled in light of the CBA. As the CBA controls, it is clear that LRTA has to pay the remaining fifty
percent (50%) of the retirement benefits due to the private respondents. The CA held that whether the NLRC has
jurisdiction to hear the case, the result would be the same: that LRTA has financial obligations to private
respondents.

Finally, on the issue of jurisdiction, the CA found that METRO, even if it is a subsidiary of LRTA, remains a private
corporation. This being the case, the money claim brought against it falls under the original and exclusive
jurisdiction of the LA. Also, the CA agreed with the NLRC that the principle of stare decisisapplies to this case.
The NLRC applied the CA's Decision in Malanao, ruling that LRTA is liable for the fifty percent (50%) balance of
the separation pay of the private respondents therein.

APPEAL TO THE SC:


Petitioner's Contention:
LA and NLRC do not have jurisdiction over the case. LRTA cites Light Rail Transit Authority v. Venus, Jr. (Venus)
to support its claim.

ISSUE/S:
Whether LRTA can be made liable by the labor tribunals for private respondents' money claim despite the
absence of an employer-employee relationship, and though LRTA is a government-owned and controlled
corporation

HELD:
YES.
In Mendoza, this Court upheld the jurisdiction of the labor tribunals over LRTA, citing Philippine National Bank v.
Pabalan:

x x x By engaging in a particular business thru the instrumentality of a corporation, the government divests itself
pro hac vice of its sovereign character, so as to render the corporation subject to the rules of law governing
private corporations.
This Court further ruled that LRTA must submit itself to the provisions governing private corporations, including
the Labor Code, for having conducted business through a private corporation, in this case, METRO.
Angeles| Bajana | Balladares | Brillantes | Briones | Cabansag | Callanta | Chua | David|

De Leon | Gomez | Lopez | Macalino | Nostratis | Padilla | Reynon | Santos | Tan |Velasco
4E / 4F - 2018-2019
Page 305 of 920
LABOR REVIEW DIGEST
Atty. Joyrich Golangco

In this case, the NLRC accordingly declared, "[LRTA's] contractual commitments with [METRO] and its employees
arose out of its business relations with [METRO] which is private in nature. Such private relation was not changed
notwithstanding the subsequent acquisition by [LRTA] of full ownership of [METRO] and take-over of its business
operations at LRT."

In view of the foregoing, we rule that the CA did not err when it upheld the jurisdiction of the labor tribunals over
private respondents' money claims against LRTA.

Angeles| Bajana | Balladares | Brillantes | Briones | Cabansag | Callanta | Chua | David|

De Leon | Gomez | Lopez | Macalino | Nostratis | Padilla | Reynon | Santos | Tan |Velasco
4E / 4F - 2018-2019
Page 306 of 920
LABOR REVIEW DIGEST
Atty. Joyrich Golangco

ARTIICLE 225
1. PAL v. NLRC
G.R. No. 120567, March 20, 1998
Martinez J.

DOCTRINE:

On Injuctive Powers: The foregoing ancillary power may be exercised by the Labor Arbiters only as an incident
to the cases pending before them in order to preserve the rights of the parties during the pendency of the case,
but excluding labor disputes involving strikes or lockout.

It is an essential requirement that there must first be a labor dispute between the contending parties before the
labor arbiter. In the present case, there is no labor dispute betweenthe petitioner and private respondents as there
has yet been no complaint for illegal dismissal filed with the labor arbiter by the private respondents against the
petitioner.

FACTS:

Private respondents are flight stewards of the petitioner. Both were dismissed from the service for their alleged
involvement in the April 3, 1993 currency smuggling in Hong Kong.

Aggrieved by said dismissal, private respondents filed with the NLRC a petitionfor injunction.

NLRC RULING:

On April 3, 1995, the NLRC issued a temporary mandatory injunction enjoining petitioner to cease and desist from
enforcing its February 22, 1995 Memorandum of Dismissal. In support of the issuance of the writ of temporary
injunction, the NLRC adopted the view that: (1) private respondents cannot be validly dismissed on the strength of
petitioner's Code of Discipline which was declared illegal by this Court in the case of PAL, Inc. vs. NLRC, (G.R.
No. 85985), promulgated August 13, 1993, for the reason that it was formulated by the petitioner without the
participation of its employees as required in R.A. 6715, amending Article 211 of the Labor Code; (2) the
whimsical, baseless and premature dismissals of private respondents which "caused them grave and irreparable
injury" is enjoinable as private respondents are left "with no speedy and adequate remedy at law'"except the
issuance of a temporary mandatory injunction; (3) the NLRC is empowered under Article 218 (e) of the Labor
Code not only to restrain any actual or threatened commission of any or all prohibited or unlawful acts but also to
require the performance of a particular act in any labor dispute, which, if not restrained or performed forthwith,
may cause grave or irreparable damage to any party; and (4) the temporary mandatory power of the NLRC was
recognized by this Court in the case of Chemo-Technicshe Mfg., Inc. Employees Union,DFA, et.al. vs. Chemo-
Technische Mfg., Inc. [G.R. No. 107031, January 25,1993].

On May 4, 1995, petitioner moved for reconsideration. One of its arguments is that the NLRC erred in granting a
temporary injunction order when it has no jurisdiction to issue an injunction or restraining order since this may be
issued only under Article 218 of the Labor Code if the case involves or arises from labor disputes. The NLRC
denied petitioner's motion for reconsideration.

ISSUE/S:

Can the NLRC, even without a complaint for illegal dismissal filed before the labor arbiter, entertain an action for
injunction and issue such writ enjoining petitioner Philippine Airlines, Inc. from enforcing its Orders of dismissal

Angeles| Bajana | Balladares | Brillantes | Briones | Cabansag | Callanta | Chua | David|

De Leon | Gomez | Lopez | Macalino | Nostratis | Padilla | Reynon | Santos | Tan |Velasco
4E / 4F - 2018-2019
Page 307 of 920
LABOR REVIEW DIGEST
Atty. Joyrich Golangco

against private respondents, and ordering petitioner to reinstate the private respondents to their previous
positions?

HELD:
 In labor cases, Article 218 of the Labor Code empowers the NLRC-

"(e) To enjoin or restrain any actual or threatened commission of any or all prohibited or unlawful acts or to require
the performance of a particular act in any labor dispute which, if not restrained or performed forthwith, may
cause grave or irreparable damage to any party or render ineffectual any decision in favor of such party; x x
x."

Complementing the above-quoted provision, Sec. 1, Rule XI of the New Rules of Procedure of the NLRC,
pertinently provides as follows:

"Section 1. Injunction in Ordinary Labor Dispute.-A preliminary injunction or a restraining order may be granted by
the Commission through its divisions pursuant to the provisions of paragraph (e) of Article 218 of the Labor Code,
as amended, when it is established on the bases of the sworn allegations in the petition that the acts complained
of, involving or arising from any labor dispute before the Commission, which, if not restrained or performed
forthwith, may cause grave or irreparable damage to any party or render ineffectual any decision in favor of such
party.”

The foregoing ancillary power may be exercised by the Labor Arbiters only as an incident to the cases pending
before them in order to preserve the rights of the parties during the pendency of the case, but excluding labor
disputes involving strikes or lockout.

It is an essential requirement that there must first be a labor dispute between the contending parties before the
labor arbiter. In the present case, there is no labor dispute betweenthe petitioner and private respondents as there
has yet been no complaint for illegal dismissal filed with the labor arbiter by the private respondents against the
petitioner.

 The petition for injunction directly filed before the NLRC is in reality an action for illegal dismissal. This is clear
from the allegations in the petition which prays for: reinstatement of private respondents; award of full backwages,
moral and exemplary damages; and attorney's fees. As such, the petition should have been filed with the labor
arbiter who has the original and exclusive jurisdiction to hear and decide the case.

Thus, the NLRC exceeded its jurisdiction when it issued the assailed Order granting private respondents' petition
for injunction and ordering the petitioner to reinstate private respondents.

Other Notes/ SC Pronouncements:

2. Landbank v. Listana
G.R. No. 182758, May 30, 2011
Ponente

Doctrine:

The purpose of the injunction bond is to protect the defendant against loss or damage by reason of the injunction
in case the court finally decides that the plaintiff was not entitled to it, and the bond is usually conditioned
accordingly

Angeles| Bajana | Balladares | Brillantes | Briones | Cabansag | Callanta | Chua | David|

De Leon | Gomez | Lopez | Macalino | Nostratis | Padilla | Reynon | Santos | Tan |Velasco
4E / 4F - 2018-2019
Page 308 of 920
LABOR REVIEW DIGEST
Atty. Joyrich Golangco

Facts:

Severino Listana owned a parcel of land in Sorsogon, covered by Transfer Certificate of Title No. T-20193.
Listana voluntarily sold the property to the Department of Agrarian Reform under the Comprehensive Agrarian
Reform Law of 1988.

The DARAB of Sorsogon commenced summary administrative proceedings to determine the amount of just
compensation for the property. The DARAB set the amount at P10,956,963.25. PARAD issued a writ of execution
ordering Land Bank Manager and Agrarian Operations Center Head Alex A. Lorayes to pay Listana
₱10,956,963.25. Lorayes refused. Thus, Listana filed with the PARAD a motion for contempt against Lorayes.

LBP filed with the RTC acting as SAC, a petition for judicial determination of the amount of just compensation for
the property. LBP prayed that the amount be fixed at ₱5,871,689.03.

The SAC dismissed LBP’s petition for judicial determination of the amount of just compensation for the property.

The PARAD ordered the issuance of an alias writ of execution, ordering LBP to pay Listana ₱10,956,963.25.

LBP filed with the RTC a petition for injunction with application for the issuance of a writ of preliminary injunction
enjoining PARAD from implementing the warrant of arrest against Lorayes for indirect contempt.

The RTC enjoined the PARAD from implementing the warrant of arrest pending final determination of the amount
of just compensation for the property. LBP posted a ₱5,644,773.02 cash bond. Listana filed with the RTC a
motion for reconsideration. The RTC denied the motion. Listana filed with the Court of Appeals a petition for
certiorari.

CA Ruling:

In its 11 December 2001 Decision, the Court of Appeals set aside the 29 January and 2 April 2001 Orders of the
RTC.

SC Ruling:

LBP filed with the Court a petition for review on certiorari under Rule 45 of the Rules of Court. In Land Bank of the
Philippines v. Listana, Sr., the Court set aside the Decision of the Court of Appeals. The Court declared void all
proceedings that stemmed from Listana’s motion for contempt.

On 26 May 2004, LBP filed with the RTC a motion9 to withdraw the ₱5,644,773.02 cash bond.

RTC Ruling:

Tthe RTC denied LBP’s motion to withdraw the P5,644,773.02 cash bond. It ruled that contrary to Land Bank’s
conclusion, this Court holds otherwise that the cash bond did not become moot and academic upon the finality of
the Supreme Court’s decision dated August 5, 2003. This is so because the underlying reason for the posting of
the cash bond still remains despite the decision of the Supreme Court upholding the unconstitutionality of the
order of arrest issued by PARAD. And that reason is the distinctive fact that the cash bond was put up in order to
secure any damages that the private respondent Listana may incur by reason of the issuance of the injunction
order. The damages being referred to, that is — the legal right of Mr. Listana to be justly and promptly paid of his

Angeles| Bajana | Balladares | Brillantes | Briones | Cabansag | Callanta | Chua | David|

De Leon | Gomez | Lopez | Macalino | Nostratis | Padilla | Reynon | Santos | Tan |Velasco
4E / 4F - 2018-2019
Page 309 of 920
LABOR REVIEW DIGEST
Atty. Joyrich Golangco

expropriated property — was not effectively extinguished by the mere decision of the Supreme Court declaring the
illegality of the order of arrest issued by the PARAD against Mr. Alex Lorayes.

LBP filed a motion for reconsideration. The RTC denied the motion. LBP filed with the Court of Appeals a petition
for certiorari under Rule 65 of the Rules of Court.

CA Ruling:

The Court of Appeals dismissed LBP’s petition and affirmed in toto the RTC’s Orders. LBP filed a motion for
reconsideration. The Court of Appeals denied the motion.

Issue:

WON LBP can withdraw the cash bond.

Held:

 An applicant for preliminary injunction is required to file a bond executed to the party or person enjoined, to the
effect that the applicant will pay to such party or person all damages which he may sustain by reason of the
injunction.

As correctly ruled by the lower courts, the ₱5,644,773.02 bond shall answer for the damages Listana may sustain
if the courts finally uphold the ₱10,956,963.25 just compensation set by the DARAB. In Republic v. Caguioa, the
Court held that, "The purpose of the injunction bond is to protect the defendant against loss or damage by reason
of the injunction in case the court finally decides that the plaintiff was not entitled to it, and the bond is usually
conditioned accordingly."

 In Republic v. Court of Appeals, it was held that Special Agrarian Courts are given original and exclusive
jurisdiction over two categories of cases, to wit: (1) all petitions for the determination of just compensation;
and (2) the prosecution of all criminal offenses under R.A. No. 6657. x x x The DAR, as an administrative
agency, cannot be granted jurisdiction over cases of eminent domain and over criminal cases. The
valuation of property in eminent domain is essentially a judicial function which is vested with the Special
Agrarian Courts and cannot be lodged with administrative agencies.20

Thus, as a rule, the DARAB’s decision setting the amount of just compensation is merely preliminary and not
executory if challenged before the SAC. Execution pending "appeal" of the DARAB decision is allowed only on
meritorious grounds.21 Even then, it is the SAC, not the DARAB, that can grant execution pending "appeal"
because the SAC has original and exclusive jurisdiction over just compensation cases. The determination of the
amount of just compensation is a judicial function that cannot be usurped by administrative agencies.

In the present case, LBP filed with the SAC a petition for determination of the amount of just compensation on 6
September 1999. The PARAD issued the alias writ of execution and warrant of arrest on 27 November 2000 and
3 January 2001, respectively. The writ of execution and warrant of arrest were invalid because the 14 October
1998 Decision of the DARAB setting the amount at ₱10,956,963.25 was merely preliminary and not executory.

Angeles| Bajana | Balladares | Brillantes | Briones | Cabansag | Callanta | Chua | David|

De Leon | Gomez | Lopez | Macalino | Nostratis | Padilla | Reynon | Santos | Tan |Velasco
4E / 4F - 2018-2019
Page 310 of 920
LABOR REVIEW DIGEST
Atty. Joyrich Golangco

3. FEDERICO S. ROBOSA, ROLANDO E. PANDY, NOEL D. ROXAS, ALEXANDER ANGELES, VERONICA


GUTIERREZ, FERNANDO EMBAT, and NANETTE H. PINTO vs. NLRC
G.R. No. 176085; February 8, 2012
BRION, J.

DOCTRINE: The labor arbiter or the Commission is empowered or has jurisdiction to hold the offending party or
parties in direct or indirect contempt.

Angeles| Bajana | Balladares | Brillantes | Briones | Cabansag | Callanta | Chua | David|

De Leon | Gomez | Lopez | Macalino | Nostratis | Padilla | Reynon | Santos | Tan |Velasco
4E / 4F - 2018-2019
Page 311 of 920
LABOR REVIEW DIGEST
Atty. Joyrich Golangco

FACTS:

Federico S. Robosa, Rolando E. Pandy, Noel D. Roxas, Alexander Angeles, Veronica Gutierrez, Fernando Embat
and Nanette H. Pinto (petitioners) were rank-and-file employees of respondent Chemo-Technische Manufacturing,
Inc. (CTMI), the manufacturer and distributor of Wella products. They were officers and members of the CTMI
Employees Union-DFA (union). Respondent Procter and Gamble Philippines, Inc. (P & GPI) acquired all the
interests, franchises and goodwill of CTMI during the pendency of the dispute.

On July 15, 1991, CTMI, through its President and General Manager Franklin R. de Luzuriaga, issued a
memorandum announcing that territories were demobilized. On the same day, CTMI issued another
memorandum informing the company’s sales representatives and sales drivers of the new system in the Salon
Business Groups selling operations. The union asked for the withdrawal and deferment of CTMIs directives,
branding them as union busting acts constituting unfair labor practice. CTMI ignored the request. Instead, it issued
on July 23, 1991 a notice of termination of employment to the sales drivers, due to the abolition of the sales driver
positions. On August 1, 1991, the union and its affected members filed a complaint for illegal dismissal and unfair
labor practice, with a claim for damages, against CTMI, De Luzuriaga and other CTMI officers.

The Compulsory Arbitration Proceedings: The labor arbiter handling the case denied the unions motion for a
stay order on the ground that the issues raised by the petitioners can best be ventilated during the trial on the
merits of the case.

NLRC: It directed CTMI, De Luzuriaga and other company executives to cease and desist from dismissing any
member of the union and from implementing the subject memorandums and to return to status quo.

Allegedly, the respondents did not comply with the NLRCs August 23, 1991 resolution. They instead moved to
dissolve the TRO and opposed the unions petition for preliminary injunction. The union opposed the motion and
urgently moved to cite the responsible CTMI officers in contempt of court. Meanwhile, the NLRC heard the
contempt charge. On October 31, 2000, it issued a resolution dismissing the charge. It ordered the labor arbiter to
proceed hearing the main case on the merits.

CA: It opined that the dismissal is not subject to review by an appellate court.

ISSUE:
Whether or not the NLRC has contempt powers;

RULING:

Article 218 of the Labor Code, the NLRC (and the labor arbiters) may hold any offending party in contempt,
directly or indirectly, and impose appropriate penalties in accordance with law. The penalty for direct contempt
consists of either imprisonment or fine, the degree or amount depends on whether the contempt is against the
Commission or the labor arbiter. The Labor Code, however, requires the labor arbiter or the Commission to deal
with indirect contempt in the manner prescribed under Rule 71 of the Rules of Court.

Rule 71 of the Rules of Court does not require the labor arbiter or the NLRC to initiate indirect contempt
proceedings before the trial court. This mode is to be observed only when there is no law granting them contempt
powers. As is clear under Article 218(d) of the Labor Code, the labor arbiter or the Commission is empowered or
has jurisdiction to hold the offending party or parties in direct or indirect contempt. The petitioners, therefore, have
not improperly brought the indirect contempt charges against the respondents before the NLRC.

Angeles| Bajana | Balladares | Brillantes | Briones | Cabansag | Callanta | Chua | David|

De Leon | Gomez | Lopez | Macalino | Nostratis | Padilla | Reynon | Santos | Tan |Velasco
4E / 4F - 2018-2019
Page 312 of 920
LABOR REVIEW DIGEST
Atty. Joyrich Golangco

4. Landbank v. Listana
G.R. No. 182758, May 30, 2011
Ponente

Doctrine:

The purpose of the injunction bond is to protect the defendant against loss or damage by reason of the injunction
in case the court finally decides that the plaintiff was not entitled to it, and the bond is usually conditioned
accordingly

Facts:

Severino Listana owned a parcel of land in Sorsogon, covered by Transfer Certificate of Title No. T-20193.
Listana voluntarily sold the property to the Department of Agrarian Reform under the Comprehensive Agrarian
Reform Law of 1988.

The DARAB of Sorsogon commenced summary administrative proceedings to determine the amount of just
compensation for the property. The DARAB set the amount at P10,956,963.25. PARAD issued a writ of execution
ordering Land Bank Manager and Agrarian Operations Center Head Alex A. Lorayes to pay Listana
₱10,956,963.25. Lorayes refused. Thus, Listana filed with the PARAD a motion for contempt against Lorayes.

LBP filed with the RTC acting as SAC, a petition for judicial determination of the amount of just compensation for
the property. LBP prayed that the amount be fixed at ₱5,871,689.03.

The SAC dismissed LBP’s petition for judicial determination of the amount of just compensation for the property.

The PARAD ordered the issuance of an alias writ of execution, ordering LBP to pay Listana ₱10,956,963.25.

LBP filed with the RTC a petition for injunction with application for the issuance of a writ of preliminary injunction
enjoining PARAD from implementing the warrant of arrest against Lorayes for indirect contempt.

The RTC enjoined the PARAD from implementing the warrant of arrest pending final determination of the amount
of just compensation for the property. LBP posted a ₱5,644,773.02 cash bond. Listana filed with the RTC a
motion for reconsideration. The RTC denied the motion. Listana filed with the Court of Appeals a petition for
certiorari.

CA Ruling:

In its 11 December 2001 Decision, the Court of Appeals set aside the 29 January and 2 April 2001 Orders of the
RTC.

SC Ruling:

LBP filed with the Court a petition for review on certiorari under Rule 45 of the Rules of Court. In Land Bank of the
Philippines v. Listana, Sr., the Court set aside the Decision of the Court of Appeals. The Court declared void all
proceedings that stemmed from Listana’s motion for contempt.

On 26 May 2004, LBP filed with the RTC a motion9 to withdraw the ₱5,644,773.02 cash bond.

Angeles| Bajana | Balladares | Brillantes | Briones | Cabansag | Callanta | Chua | David|

De Leon | Gomez | Lopez | Macalino | Nostratis | Padilla | Reynon | Santos | Tan |Velasco
4E / 4F - 2018-2019
Page 313 of 920
LABOR REVIEW DIGEST
Atty. Joyrich Golangco

RTC Ruling:

Tthe RTC denied LBP’s motion to withdraw the P5,644,773.02 cash bond. It ruled that contrary to Land Bank’s
conclusion, this Court holds otherwise that the cash bond did not become moot and academic upon the finality of
the Supreme Court’s decision dated August 5, 2003. This is so because the underlying reason for the posting of
the cash bond still remains despite the decision of the Supreme Court upholding the unconstitutionality of the
order of arrest issued by PARAD. And that reason is the distinctive fact that the cash bond was put up in order to
secure any damages that the private respondent Listana may incur by reason of the issuance of the injunction
order. The damages being referred to, that is — the legal right of Mr. Listana to be justly and promptly paid of his
expropriated property — was not effectively extinguished by the mere decision of the Supreme Court declaring the
illegality of the order of arrest issued by the PARAD against Mr. Alex Lorayes.

LBP filed a motion for reconsideration. The RTC denied the motion. LBP filed with the Court of Appeals a petition
for certiorari under Rule 65 of the Rules of Court.

CA Ruling:

The Court of Appeals dismissed LBP’s petition and affirmed in toto the RTC’s Orders. LBP filed a motion for
reconsideration. The Court of Appeals denied the motion.

Issue:

WON LBP can withdraw the cash bond.

Held:

 An applicant for preliminary injunction is required to file a bond executed to the party or person enjoined, to the
effect that the applicant will pay to such party or person all damages which he may sustain by reason of the
injunction.

As correctly ruled by the lower courts, the ₱5,644,773.02 bond shall answer for the damages Listana may sustain
if the courts finally uphold the ₱10,956,963.25 just compensation set by the DARAB. In Republic v. Caguioa, the
Court held that, "The purpose of the injunction bond is to protect the defendant against loss or damage by reason
of the injunction in case the court finally decides that the plaintiff was not entitled to it, and the bond is usually
conditioned accordingly."

 In Republic v. Court of Appeals, it was held that Special Agrarian Courts are given original and exclusive
jurisdiction over two categories of cases, to wit: (1) all petitions for the determination of just compensation;
and (2) the prosecution of all criminal offenses under R.A. No. 6657. x x x The DAR, as an administrative
agency, cannot be granted jurisdiction over cases of eminent domain and over criminal cases. The
valuation of property in eminent domain is essentially a judicial function which is vested with the Special
Agrarian Courts and cannot be lodged with administrative agencies.20

Thus, as a rule, the DARAB’s decision setting the amount of just compensation is merely preliminary and not
executory if challenged before the SAC. Execution pending "appeal" of the DARAB decision is allowed only on
meritorious grounds.21 Even then, it is the SAC, not the DARAB, that can grant execution pending "appeal"
because the SAC has original and exclusive jurisdiction over just compensation cases. The determination of the
amount of just compensation is a judicial function that cannot be usurped by administrative agencies.

Angeles| Bajana | Balladares | Brillantes | Briones | Cabansag | Callanta | Chua | David|

De Leon | Gomez | Lopez | Macalino | Nostratis | Padilla | Reynon | Santos | Tan |Velasco
4E / 4F - 2018-2019
Page 314 of 920
LABOR REVIEW DIGEST
Atty. Joyrich Golangco

In the present case, LBP filed with the SAC a petition for determination of the amount of just compensation on 6
September 1999. The PARAD issued the alias writ of execution and warrant of arrest on 27 November 2000 and
3 January 2001, respectively. The writ of execution and warrant of arrest were invalid because the 14 October
1998 Decision of the DARAB setting the amount at ₱10,956,963.25 was merely preliminary and not executory.

Angeles| Bajana | Balladares | Brillantes | Briones | Cabansag | Callanta | Chua | David|

De Leon | Gomez | Lopez | Macalino | Nostratis | Padilla | Reynon | Santos | Tan |Velasco
4E / 4F - 2018-2019
Page 315 of 920
LABOR REVIEW DIGEST
Atty. Joyrich Golangco

5. CRISPIN S. FRONDOZO,* DANILO M. PEREZ, JOSE A. ZAFRA, ARTURO B. VITO, CESAR S. CRUZ,
NAZARIO C. DELA CRUZ, and LUISITO R. DILOY v. MANILA ELECTRIC COMPANY.
GR No: 178379 Date: August 22, 2017
Ponente: Carpio, J.

Doctrine: The 2005 Revised Rules of Procedure of the NLRC did not make a distinction between decisions or
resolutions decided by the Labor Arbiter and those decided by the Commission in certified cases when an order of
reinstatement is involved. Thus, even when the employer had perfected an appeal, the Labor Arbiter must issue a
writ of execution for actual or payroll reinstatement of the employees illegally dismissed from the service. The
Court of Appeals also cited Article 223 of the Labor Code which provides that the reinstatement aspect of the
Labor Arbiter's Decision is immediately executory.

FACTS:
The case originated from a Notice of Strike (first strike) filed on 16 May 1991 by the MERALCO Employees and
Workers Association (MEWA), composed of MERALCO’s rank-and-file employees, on the ground of Unfair Labor
Practice (ULP). Conciliation conferences conducted by the National Conciliation and Mediation Board (NCMB)
failed to settle the dispute and resulted to a strike staged by MEWA on 6 June 1991. In an Order dated 6 June
1991, then Acting Secretary·Nieves R. Confesor of the Department of Labor and Employment (DOLE) certified the
labor dispute to the NLRC for compulsory arbitration, ordered all the striking workers to return to work, and
directed MERALCO to accept the striking workers back to work under the same terms and conditions existing
prior to the work stoppage.
On 26 July 1991, MERALCO terminated the services of Crispin S. Frondozo (Frondozo ), Danilo M. Perez
(Perez), Jose A. Zafra (Zafra), Arturo B. Vito (Vito ). Cesar S. Cruz (Cruz), Nazario C. dela Cruz (N. dela Cruz),
Luisito R. Diloy (Diloy), and Danilo D. Dizon (Dizon) for having committed unlawful acts and violence during the
strike.
On 25 July 1991, MEWA filed a second Notice of Strike (second strike) on the ground of discrimination and union
busting that resulted to the dismissal from employment of 25 union officers and workers. Then DOLE Secretary
Ruben D. Torres issued an Order dated 8 August 1991 that certified the issues raised in the second strike to the
NLRC for consolidation with the first strike and strictly enjoined any strike or lockout pending resolution of the
labor dispute. The Order also directed MERALCO to suspend the effects of termination of the employees and re-
admit the employees under the same terms and conditions without loss of seniority rights.
The labor dispute resulted to the filing of two complaints for illegal dismissal. The NLRC consolidated the two
illegal dismissal cases with NLRC NCR CC No. 000021-91 (In the Matter of the Labor Dispute at the Manila
Electric Company) and NLRC NCR Case No. 00-05-03381-93 (MEWA v. MERALCO). NLRC modified its final
decision ordering respondent MANILA ELECTRIC COMPANY to reinstate to their former or equivalent positions
DANILO DIZON and LUISITO DILOY, without loss of seniority rights and payment of backwages computed from
the time of their dismissal.
Two petitions for certiorari were filed before the Court of Appeals. Pending the case in CA, the NLRC issued an
entry of judgment. MERALCO informed the NLRC of the payroll reinstatement of the 14 respondents. Meanwhile,
the Court of Appeals Special Second Division found that the strike of 6-8 June 1991 was illegal because it
occurred despite an assumption order by the DOLE Secretary and because of the commission of illegal acts
marred with violence and coercion. In view thereof, MERALCO stopped their payroll reinstatement. Labor Arbiter
Guerrero ruled that the Court of Appeals’ 30 May 2003 Decision had not attained finality and as such,
respondents should be reinstated from the time they were removed from the payroll until their actual/payroll
reinstatement based on their latest salary prior to their dismissal. Court of Appeals Fourteenth Division rendered
its decision that respondent MERALCO is ordered to pay the petitioners full backwages computed from July 26,
1991, when they were illegally dismissed, up to the date of their actual reinstatement in the service. The
respondents moved for the issuance of an Alias Writ of Execution for the satisfaction of their accrued wages
arising from the recall of their payroll reinstatement. On 10 June 2004, Labor Arbiter Guerrero granted the motion.
Angeles| Bajana | Balladares | Brillantes | Briones | Cabansag | Callanta | Chua | David|

De Leon | Gomez | Lopez | Macalino | Nostratis | Padilla | Reynon | Santos | Tan |Velasco
4E / 4F - 2018-2019
Page 316 of 920
LABOR REVIEW DIGEST
Atty. Joyrich Golangco

On 14 June 2004, a Second Alias Writ of Execution was issued directing the Sheriff to cause the reinstatement of
the respondents and to collect backwages from 14 December 2001 to 15 January 2003 and from 1 June 2003 to 1
June 2004. MERALCO filed a motion to quash the Second Alias Writ of Execution but it was denied on 2 July
2004. On 20 July 2004, the Sheriff reported that the amount of ₱2,879,967.53 garnished funds had been delivered
to and deposited with the NLRC Cashier for the satisfaction of the monetary award. However, the reinstatement
portion of the judgment remained unimplemented due to the failure of MERALCO to reinstate the respondents.
Meanwhile, MERALCO filed two motions before the NLRC: (1) a motion for reconsideration and/or appeal filed on
5 July 2004 assailing the 10 June 2004 Order of Labor Arbiter Guerrero granting the issuance of the Second Alias
Writ of Execution and directing the payment of backwages of ₱2,851,453 to respondents and ordering their
reinstatement actually or in the payroll, which was accompanied by a bond equivalent to the amount of the
accrued backwages; and (2) an urgent motion for the issuance of a temporary restraining order and/or preliminary
injunction filed on 13 July 2004 directed against the Second Alias Writ of Execution pending the resolution of its
first motion.

NLRC RULING: The NLRC granted the prayer for preliminary injunction of MERALCO. The NLRC considered the
difficulty in proceeding with the execution given the conflicting decisions of the Court of Appeals' Special Second
Division in CA-G.R. SP No. 72480 and the Court of Appeals' Fourteenth Division in CA-G.R.

COURT OF APPEALS RULING: NLRC ruling upheld. CA concurred with the NLRC's pronouncement that
MERALCO has no speedy and adequate remedy in the ordinary course of law for the preservation of its rights and
interests, at least insofar only and solely as to avoid the injurious consequences of the 2nd alias writ of
execution relative to the reinstatement aspect of the final decision in CA-G.R. No. SP 72509.

PETITION TO THE SC:

Petitioner's Contention: Petitioners alleged that the Court of Appeals committed grave abuse of discretion in
upholding the 28 February 2006 and 26 May 2006 Resolutions of the NLRC, in not passing upon the issues of
reinstatement and release of the garnished amount against MERALCO, and in ruling that the Decision in CA-G.R.
SP No. 72480 is considered a bar in the implementation of the Decision in CA-G.R. SP No. 72509.

ISSUE/S:
1. Whether the Court of Appeals committed a reversible error in upholding the NLRC in issuing the writ of
preliminary injunction prayed for by MERALCO.
The Ruling
HELD: Petition has no merit. The Court of Appeals cited the 2005 Revised Rules of Procedure of the NLRC which
provides that "[u]pon issuance of the entry of judgment, the Commission, motu proprio or upon motion by the
proper party, may cause the execution of the judgment in the certified case." According to the Court of Appeals,
the 2005 Revised Rules of Procedure of the NLRC did not make a distinction between decisions or resolutions
decided by the Labor Arbiter and those decided by the Commission in certified cases when an order of
reinstatement is involved. Thus, even when the employer had perfected an appeal, the Labor Arbiter must issue a
writ of execution for actual or payroll reinstatement of the employees illegally dismissed from the service. The
Court of Appeals also cited Article 223 of the Labor Code which provides that the reinstatement aspect of the
Labor Arbiter's Decision is immediately executory.
In this case, the applicable rule is Article 263 of the Labor Code and the NLRC Manual on Execution of Judgment,
as amended by Resolution No. 02-02, series of 2002. Section 1, Rule III of the NLRC Manual on Execution of
Judgment provides:
Section 1. Execution Upon Final Judgment or Order. Execution shall issue only upon a judgment or order that
finally disposes of an action or proceeding, except in specific instances where the law provides for execution
pending appeal.
Article 263(i) of the Labor Code, on the other hand, provides:
Angeles| Bajana | Balladares | Brillantes | Briones | Cabansag | Callanta | Chua | David|

De Leon | Gomez | Lopez | Macalino | Nostratis | Padilla | Reynon | Santos | Tan |Velasco
4E / 4F - 2018-2019
Page 317 of 920
LABOR REVIEW DIGEST
Atty. Joyrich Golangco

(i) The Secretary of Labor and Employment, the Commission or the voluntary arbitrator shall decide or resolve the
dispute within thirty (30) calendar days from the date of the assumption of jurisdiction or the certification or
submission of the dispute, as the case may be. The decision of the President, the Secretary of Labor and
Employment, the Commission or the voluntary arbitrator shall be final and executory ten (10) calendar days after
receipt thereof by the parties.
A judicial review of the decisions of the NLRC may be filed before the Court of Appeals via a petition
for certiorariunder Rule 65 of the Rules of Court but the petition shall not stay the execution of the assailed
decision unless a restraining order is issued by the Court of Appeals.
The situation in this case is analogous to a change in the situation of the parties making execution unjust or
inequitable. MERALCO's refusal to reinstate petitioners and to pay their backwages is justified by the 30 May
2003 Decision in CA-G.R. SP No. 72480. On the other hand, petitioners' insistence on the execution of judgment
is anchored on the 27 January 2004 Decision of the Court of Appeals' Fourteenth Division in CA-G.R. SP No.
72509. Given this situation, we see no reversible error on the part of the Court of Appeals in holding that the
NLRC did not commit grave abuse of discretion in suspending the proceedings.
Clearly, the NLRC did not act in a capricious, whimsical, arbitrary, or despotic manner. It suspended the
proceedings because it cannot revise or modify the conflicting Decisions of the Court of Appeals.

Final ruling: The Supreme Court's Third Division adopted the findings and conclusions reached by the Court of
Appeals in CA-G.R. SP No. 72480 which dismissed petitioners from the service.

Angeles| Bajana | Balladares | Brillantes | Briones | Cabansag | Callanta | Chua | David|

De Leon | Gomez | Lopez | Macalino | Nostratis | Padilla | Reynon | Santos | Tan |Velasco
4E / 4F - 2018-2019
Page 318 of 920
LABOR REVIEW DIGEST
Atty. Joyrich Golangco

ARTICLE 227
1. MERALCO v. GALA
DOCTRINE: It is the spirit and intention of labor legislation that the NLRC and the labor arbiters shall use every
reasonable means to ascertain the facts in each case speedily and objectively, without regard to technicalities of
law or procedure, provided due process is duly observed.
FACTS: On March 2, 2006, respondent Jan Carlo Gala commenced employment with the petitioner Meralco
Electric Company (Meralco) as a probationary lineman. On July 27, 2006, barely four months on the job, Gala was
dismissed for alleged complicity in pilferages of Meralcos electrical supplies, particularly, for the incident which
took place on May 25, 2006. On that day, Gala and other Meralco workers were instructed to replace a worn-out
electrical pole at the Pacheco Subdivision in Valenzuela City. While the Meralco crew was at work, one Noberto
Bing Llanes, a non-Meralco employee, arrived. He appeared to be known to the Meralco foremen as they were
seen conversing with him. Llanes boarded the trucks, without being stopped, and took out what were later found
as electrical supplies. Aside from Gala, the foremen and the other linemen who were at the worksite when the
pilferage happened were later charged with misconduct and dishonesty for their involvement in the incident.
Unknown to Gala and the rest of the crew, a Meralco surveillance task force was monitoring their activities and
recording everything with a Sony video camera. Meralco called for an investigation of the incident and asked Gala
to explain. Gala denied involvement in the pilferage, contending that even if his superiors might have committed a
wrongdoing, he had no participation in what they did. Despite Galas explanation, Meralco proceeded with the
investigation and eventually terminated his employment on July 27, 2006. Gala responded by filing an illegal
dismissal complaint against Meralco.
LA RULING: dismissed the complaint for lack of merit.
NLRC RULING: reversed the labor arbiters ruling. It found that Gala had been illegally dismissed.
CA RULING: concurred with the NLRC that Gala had been illegally dismissed.
ISSUES: 1. Should the Court dismiss the petition outright based on procedural grounds?
2. Was Gala illegally dismissed by petitioner Meralco?
SC RULING: 1. NO. Gala would want the petition to be dismissed outright on procedural grounds, claiming that
the Verification and Certification, Secretarys Certificate and Affidavit of Service accompanying the petition do not
contain the details of the Community Tax Certificates of the affiants, and that the lawyers who signed the petition
failed to indicate their updated MCLE certificate numbers, in violation of existing rules. We stress at this point that
it is the spirit and intention of labor legislation that the NLRC and the labor arbiters shall use every reasonable
means to ascertain the facts in each case speedily and objectively, without regard to technicalities of law or
procedure, provided due process is duly observed. In keeping with this policy and in the interest of substantial
justice, we deem it proper to give due course to the petition, especially in view of the conflict between the findings
of the labor arbiter, on the one hand, and the NLRC and the CA, on the other. As we said in S.S. Ventures
International, Inc. v. S.S. Ventures Labor Union, the application of technical rules of procedure in labor cases may
be relaxed to serve the demands of substantial justice. 2. NO. Contrary to the conclusions of the CA and the
NLRC, there is substantial evidence supporting Meralcos position that Gala had become unfit to continue his
employment with the company. Gala was found, after an administrative investigation, to have failed to meet the
standards expected of him to become a regular employee and this failure was mainly due to his undeniable
knowledge, if not participation, in the pilferage activities done by their group, all to the prejudice of the Company’s
interests.

Angeles| Bajana | Balladares | Brillantes | Briones | Cabansag | Callanta | Chua | David|

De Leon | Gomez | Lopez | Macalino | Nostratis | Padilla | Reynon | Santos | Tan |Velasco
4E / 4F - 2018-2019
Page 319 of 920
LABOR REVIEW DIGEST
Atty. Joyrich Golangco

2. NATIONWIDE SECURITY AND ALLIED SERVICES vs. COURT OF APPEALS


GR No: 155844 Date: July 14, 2008
Ponente: Quisumbing, J.
Doctrine: The right to appeal is a statutory right and one who seeks to avail of the right must comply with the
statute or the rules. The rules, particularly the requirements for perfecting an appeal within the reglementary
period specified in the law, must be strictly followed as they are considered indispensable interdictions against
needless delays and for the orderly discharge of judicial business.
FACTS: Labor Arbiter Manuel M. Manansala found petitioner Nationwide Security and Allied Services, Inc., a
security agency, not liable for illegal dismissal in two cases involving eight security guards who were employees of
the petitioner. The Labor Arbiter directed the petitioner to pay the aforementioned security guards ₱81,750.00 in
separation pay, ₱8,700.00 in unpaid salaries, ₱93,795.68 for underpayment and 10% attorney’s fees based on
the total monetary award.
Dissatisfied with the decision, petitioner appealed to the NLRC which dismissed its appeal for two reasons — first,
for having been filed beyond the reglementary period within which to perfect the appeal and second, for filing an
insufficient appeal bond.
NLRC Ruling: Petitioner filed a Motion for Reconsideration which was denied. They elevated the case to the CA .
CA Ruling: The Court of Appeals dismissed the case, holding that in a special action for certiorari, the burden is
on petitioner to prove not merely reversible error, but grave abuse of discretion amounting to lack of or excess of
jurisdiction on the part of public respondent NLRC.
Petitioner’s contention: Petitioner contends that the Court of Appeals erred when it dismissed its case based on
technicalities.
Respondent’s contention: Private respondents contend that the appeal to the NLRC had not been perfected,
since the appeal was filed outside the reglementary period, and the bond was insufficient.
ISSUE: Whether or not technicalities in Labor cases must prevail over the spirit and intention of the Labor Code
as provided in Art. 221.
HELD:
The petition lacks merit.
Both the NLRC and the Court of Appeals found that petitioner received the decision of the Labor Arbiter on July
16, 1999. This factual finding is supported by sufficient evidence, and we take it as binding on us. Petitioner then
simultaneously filed its "Appeal Memorandum", "Notice of Appeal" and "Motion to Reduce Bond", by registered
mail on July 29, 1999, under Registry Receipt No. 003098. These were received by the NLRC on July 30, 1999.
The appeal to the NLRC should have been perfected, as provided by its Rules, within a period of 10 days from
receipt by petitioner of the decision on July 16, 1999. Clearly, the filing of the appeal--three days after July 26,
1999--was already beyond the reglementary period and in violation of the NLRC Rules and the pertinent Article on
Appeal in the Labor Code.
Failure to perfect an appeal renders the decision final and executory. The right to appeal is a statutory right and
one who seeks to avail of the right must comply with the statute or the rules. The rules, particularly the
requirements for perfecting an appeal within the reglementary period specified in the law, must be strictly followed
as they are considered indispensable interdictions against needless delays and for the orderly discharge of
judicial business. It is only in highly meritorious cases that this Court will opt not to strictly apply the rules and thus
prevent a grave injustice from being done.

Angeles| Bajana | Balladares | Brillantes | Briones | Cabansag | Callanta | Chua | David|

De Leon | Gomez | Lopez | Macalino | Nostratis | Padilla | Reynon | Santos | Tan |Velasco
4E / 4F - 2018-2019
Page 320 of 920
LABOR REVIEW DIGEST
Atty. Joyrich Golangco

3. DIAMOND TAXI AND/OR BRYAN ONG v LLAMAS


G.R. No. 190724 March 12, 2014

Facts:
- Felipe Llamas, Jr. worked as a taxi driver for petitioner Diamond Taxi, owned and operated by petitioner Bryan
Ong.
- July 18, 2005, Llamas filed before the Labor Arbiter (LA) a complaint for illegal dismissal against the petitioners.
- petitioners denied dismissing Llamas.
- Claimed that Llamas had been absent without official leave for several days, beginning July 14, 2005 until August
1, 2005.
- The petitioners submitted a copy of the attendance logbook to prove that Llamas had been absent on these cited
dates.
- They also pointed out that Llamas committed several traffic violations in the years 2000-2005 and that they had
issued him several memoranda for acts of insubordination and refusal to heed management instructions.
- They argued that these acts, constitute grounds for the termination of Llamas’ employment.
- Llamas failed to seasonably file his position paper.
- November 29, 2005, the LA rendered a decision dismissing Llamas’ complaint for lack of merit.
- LA held that Llamas was not dismissed, legally or illegally. Rather, the LA declared that Llamas left his job and
had been absent for several days without leave.
- January 5, 2006 Llamas received a copy of this LA decision.
- December 20, 2005, he filed a position paper.
- Llamas claimed that he failed to seasonably file his position paper because his previous counsel, despite his
repeated pleas, had continuously deferred compliance with the LA’s orders for its submission.
- Llamas alleged that he had a misunderstanding with Aljuver Ong, Bryan’s brother and operations manager of
Diamond Taxi, on July 13, 2005
- When he reported for work on July 14, 2005, Bryan refused to give him the key to his assigned taxi cab unless he
would sign a prepared resignation letter.
- He did not sign the resignation letter.
- He reported for work again on July 15 and 16, 2005, but Bryan insisted that he sign the resignation letter prior to
the release of the key to his assigned taxi cab.
- Thus, he filed the illegal dismissal complaint.
- January 16, 2006, Llamas filed before the LA a motion for reconsideration of its November 29, 2005 decision.

- CRUX OF THE CASE: The LA treated Llamas’ motion as an appeal per Section 15, Rule V of the 2005 Revised
Rules of Procedure of the NLRC (2005 NLRC Rules) (the governing NLRC Rules of Procedure at the time Llamas
filed his complaint before the LA).
- May 30, 2006 resolution, the NLRC dismissed for non-perfection Llamas’ motion for reconsideration treated as an
appeal.
- The NLRC pointed out that Llamas failed to attach the required certification of non-forum shopping per Section 4,
Rule VI of the 2005 NLRC Rules.
- Llamas moved to reconsider the May 30, 2006 NLRC resolution; he attached the required certification of non-
forum shopping.
- When the NLRC denied his motion for reconsideration in its August 31, 2006 resolution, Llamas filed before the
CA a petition for certiorari.
- August 13, 2008 decision the CA reversed and set aside the assailed NLRC resolution
- CA pointed out that non-compliance with the requirement on the filing of a certificate of non-forum shopping, while
mandatory, may nonetheless be excused upon showing of manifest equitable grounds proving substantial
compliance.

Angeles| Bajana | Balladares | Brillantes | Briones | Cabansag | Callanta | Chua | David|

De Leon | Gomez | Lopez | Macalino | Nostratis | Padilla | Reynon | Santos | Tan |Velasco
4E / 4F - 2018-2019
Page 321 of 920
LABOR REVIEW DIGEST
Atty. Joyrich Golangco

- In order to determine if cogent reasons exist to suspend the rules of procedure, the court must first examine the
substantive aspect of the case.
- The CA pointed out that the petitioners failed to prove overt acts showing Llamas’ clear intention to abandon his
job.
- petitioners’ act amounted to constructive dismissal.
- The CA additionally noted that Llamas immediately filed the illegal dismissal case that proved his desire to return
to work and negates the charge of abandonment.
- CA found equitable grounds to take exception from the rule on certificate of non-forum shopping, it declared that
the NLRC had acted with grave abuse of discretion when it dismissed Llamas’ appeal purely on a technicality
- CA ordered the petitioners to pay Llamas separation pay, full backwages and other benefits due the latter from the
time of the dismissal up to the finality of the decision.
- CA awarded separation pay in lieu of reinstatement because of the resulting strained work relationship between
Llamas and Bryan following the altercation between the former and the latter’s brother.

ISSUE:
- Whether the CA erred when it encroached on the NLRC’s exclusive jurisdiction to review the merits of the LA’s
decision.
- To the petitioners, the CA should have limited its action in determining whether grave abuse of discretion attended
the NLRC’s dismissal of Llamas’ appeal; finding that it did, the CA should have remanded the case to the NLRC
for further proceedings.

HELD:
 We do not find the petition meritorious.
 In this Rule 45 petition for review on certiorari, we review the legal errors that the CA may have committed in
the assailed decision, in contrast with the review for jurisdictional error undertaken in an original certiorari action.
 In reviewing the legal correctness of the CA decision in a labor case made under Rule 65 of the Rules of Court,
we examine the CA decision in the context that it determined the presence or the absence of grave abuse
of discretion in the NLRC decision before it and not on the basis of whether the NLRC decision, on the merits
of the case, was correct.
 we have to be keenly aware that the CA undertook a Rule 65 review, not a review on appeal, of the challenged
NLRC decision.
 In question form, the question that we ask is: Did the CA correctly determine whether the NLRC committed
grave abuse of discretion in ruling on the case?
 Court’s jurisdiction in a Rule 45 petition for review on certiorari is limited to resolving only questions of law.
 A question of law arises when the doubt or controversy concerns the correct application of law or jurisprudence
to a certain set of facts.
 a question of fact exists when the doubt or controversy concerns the truth or falsehood of facts.
 petition before us involves mixed questions of fact and law, with the core issue being one of fact.
 Whether the CA, in ruling on the labor case before it under an original certiorari action, can make its own factual
determination requires the consideration and application of law and jurisprudence; it is essentially a question of
law that a Rule 45 petition properly addresses.
 this legal issue is inextricably linked with and cannot be resolved without the definitive resolution of the
core factual issue – whether Llamas abandoned his work or had been constructively dismissed.
 As a proscribed question of fact, we generally cannot address this issue, except to the extent necessary to
determine whether the CA correctly found that the NLRC acted with grave abuse of discretion in dismissing
Llamas’ appeal on purely technical grounds.
 For raising mixed questions of fact and law, we DENY the petition outright.
 Even if this error were to be disregarded, however, we would still deny the petition as we find the CA legally
correct in reversing the NLRC’s resolution on the ground of grave abuse of discretion.
Angeles| Bajana | Balladares | Brillantes | Briones | Cabansag | Callanta | Chua | David|

De Leon | Gomez | Lopez | Macalino | Nostratis | Padilla | Reynon | Santos | Tan |Velasco
4E / 4F - 2018-2019
Page 322 of 920
LABOR REVIEW DIGEST
Atty. Joyrich Golangco

ISSUE:
- CRUX OF THE CASE: Moreover, the petitioners point out that the NLRC did not gravely abuse its discretion when
it rejected Llamas’ appeal.
- they did not constructively dismiss Llamas.
- They maintain that Llamas no longer reported for work because of the several liabilities he incurred that would
certainly have, in any case, warranted his dismissal.

HELD:
 The CA has ample authority to make its own factual determination
 We agree that remanding the case to the NLRC for factual determination and decision of the case on the merits
would have been, ordinarily, a prudent approach.
 Nevertheless, the CA’s action on this case was not procedurally wrong and was not without legal and
jurisprudential basis.
 In this jurisdiction, courts generally accord great respect and finality to factual findings of administrative agencies,
i.e., labor tribunals, in the exercise of their quasi-judicial function.
 These findings, however, are not infallible.
 This doctrine espousing comity to administrative findings of facts cannot preclude the courts from reviewing and,
when proper, disregarding these findings of facts when shown that the administrative body committed grave
abuse of discretion by capriciously, whimsically or arbitrarily disregarding evidence or circumstances of
considerable importance that are crucial or decisive of the controversy.
 Accordingly, we do not find erroneous the course that the CA took in resolving Llamas’ certiorari petition.
 The CA may resolve factual issues by express legal mandate and pursuant to its equity jurisdiction.

HELD:
 CRUX OF THE CASE: The NLRC committed grave abuse of discretion in dismissing Llamas’ appeal on
mere technicality
 Article 229 of the Labor Code states that decisions (or awards or orders) of the LA shall become final and
executory unless appealed to the NLRC within 10 calendar days from receipt of the decision.
 Consistent with Article 229, Section 1, Rule VI of the 2005 NLRC Rules also provides for a 10-day period for
appealing the LA’s decision.
 Under Section 4(a), Rule VI of the 2005 NLRC Rules, the appeal shall be in the form of a verified
memorandum of appeal and accompanied by proof of payment of the appeal fee, posting of cash or surety
bond (when necessary), certificate of non-forum shopping, and proof of service upon the other parties.
 Failure of the appealing party to comply with any or all of these requisites within the reglementary period will
render the LA’s decision final and executory.
 Indisputably, Llamas did not file a memorandum of appeal from the LA’s decision.
 Instead, he filed, within the 10-day appeal period, a motion for reconsideration.
 Under Section 15, Rule V of the 2005 NLRC Rules, motions for reconsideration from the LA’s decision are
not allowed; they may, however, be treated as an appeal provided they comply with the requirements for
perfecting an appeal.
 The NLRC dismissed Llamas’ motion for reconsideration treated as an appeal for failure to attach the required
certificate of non-forum shopping per Section 4(a), Rule VI of the 2005 NLRC Rules.
 The requirement for a sworn certification of non-forum shopping was prescribed by the Court under Revised
Circular 28-91, as amended by Administrative Circular No. 04-94, to prohibit and penalize the evils of
forum shopping.

Angeles| Bajana | Balladares | Brillantes | Briones | Cabansag | Callanta | Chua | David|

De Leon | Gomez | Lopez | Macalino | Nostratis | Padilla | Reynon | Santos | Tan |Velasco
4E / 4F - 2018-2019
Page 323 of 920
LABOR REVIEW DIGEST
Atty. Joyrich Golangco

 A careful consideration of the circumstances of the case, however, convinces us that the NLRC should, indeed,
have given due course to Llamas’ appeal despite the initial absence of the required certificate.
 We note that in his motion for reconsideration of the NLRC’s May 30, 2006 resolution, Llamas attached the
required certificate of non-forum shopping.
 Faced with these circumstances, i.e.,
o Llamas’ subsequent compliance with the certification-against-forum-shopping requirement;
o the utter negligence and inattention of Llamas’ former counsel to his pleas and cause, and his vigilance in
immediately securing the services of a new counsel;
o Llamas’ filing of his position paper before he learned and received a copy of the LA’s decision; the absence of a
meaningful opportunity for Llamas to present his case before the LA; and the clear merits of his case (that our
subsequent discussion will show),
 the NLRC should have relaxed the application of procedural rules in the broader interests of substantial
justice.
 Indeed, while the requirement as to the certificate of non-forum shopping is mandatory, this requirement should
not, however, be interpreted too literally and thus defeat the objective of preventing the undesirable
practice of forum-shopping.
 Article 227 of the Labor Code, "the Commission and its members and the Labor Arbiters shall use every and all
reasonable means to ascertain the facts in each case speedily and objectively and without regard to
technicalities of law or procedure, all in the interest of due process."
 Consistently, we have emphasized that "rules of procedure are mere tools designed to facilitate the
attainment of justice. A strict and rigid application which would result in technicalities that tend to
frustrate rather than promote substantial justice should not be allowed x x x. No procedural rule is
sacrosanct if such shall result in subverting justice.”
 Ultimately, what should guide judicial action is that a party is given the fullest opportunity to establish the
merits of his action or defense rather than for him to lose life, honor, or property on mere technicalities.
 "the dismissal of an employee’s appeal on purely technical ground is inconsistent with the constitutional mandate
on protection to labor."
 Under the Constitution and the Labor Code, the State is bound to protect labor and assure the rights of
workers to security of tenure – tenurial security being a preferred constitutional right that, under these
fundamental guidelines, technical infirmities in labor pleadings cannot defeat.
 In this case, Llamas’ action against the petitioners concerned his job, his security of tenure.
 This is a property right of which he could not and should not be deprived of without due process.
 But, more importantly, it is a right that assumes a preferred position in our legal hierarchy.

HELD:
 Llamas did not abandon his work; he was constructively dismissed
 "Abandonment is the deliberate and unjustified refusal of an employee to resume his employment."
 It is a form of neglect of duty that constitutes just cause for the employer to dismiss the employee.
 To constitute abandonment of work, 2 elements must concur:
o (1) the employee must have failed to report for work or must have been absent without valid or justifiable
reason; and
o (2) there must have been a clear intention on the part of the employee to sever the employer-employee
relationship manifested by some overt act.
 The employee’s absence must be accompanied by overt acts that unerringly point to the employee’s clear
intention to sever the employment relationship.
 And, to successfully invoke abandonment, whether as a ground for dismissing an employee or as a defense, the
employer bears the burden of proving the employee’s unjustified refusal to resume his employment.
 Mere absence of the employee is not enough.
Angeles| Bajana | Balladares | Brillantes | Briones | Cabansag | Callanta | Chua | David|

De Leon | Gomez | Lopez | Macalino | Nostratis | Padilla | Reynon | Santos | Tan |Velasco
4E / 4F - 2018-2019
Page 324 of 920
LABOR REVIEW DIGEST
Atty. Joyrich Golangco

 we agree that the petitioners unerringly failed to prove the alleged abandonment.
 They did not present proof of some overt act of Llamas that clearly and unequivocally shows his intention to
abandon his job.
 We note that, aside from their bare allegation, the only evidence that the petitioners submitted to prove
abandonment were the photocopy of their attendance logbook and the July 15, 2005 memorandum that they
served on Llamas regarding the July 13, 2005 incident.
 These pieces of evidence, even when considered collectively, indeed failed to prove the clear and unequivocal
intention, on Llamas’ part, that the law requires to deem as abandonment Llamas’ absence from work.
 Llamas lost no time in filing the illegal dismissal case against them.
 To recall, he filed the complaint on July 18, 2005 or only two days from the third time he was refused
access to his assigned taxi cab on July 16, 2005.
 Clearly, Llamas could not be deemed to have abandoned his work for, as we have previously held, the
immediate filing by the employee of an illegal dismissal complaint is proof enough of his intention to return to work
and negates the employer's charge of abandonment
 Llamas as constructively dismissed for the petitioners' failure to prove the alleged just cause -abandonment -
for his dismissal.
 Constructive dismissal exists when there is cessation of work because continued employment is rendered
impossible, unreasonable or unlikely.
 Constructive dismissal is a dismissal in disguise or an act amounting to dismissal but made to appear as if it
were not.
 In constructive dismissal cases, the employer is, concededly, charged with the burden of proving that its
conduct and action were for valid and legitimate grounds.
 The petitioners' persistent refusal to give Llamas the key to his assigned taxi cab, on the condition that he should
first sign the resignation letter, rendered, without doubt, his continued employment impossible, unreasonable and
unlikely; it, thus, constituted constructive dismissal.
 we hereby DENY the petition. We AFFIRM the decision dated August 13, 2008

Angeles| Bajana | Balladares | Brillantes | Briones | Cabansag | Callanta | Chua | David|

De Leon | Gomez | Lopez | Macalino | Nostratis | Padilla | Reynon | Santos | Tan |Velasco
4E / 4F - 2018-2019
Page 325 of 920
LABOR REVIEW DIGEST
Atty. Joyrich Golangco

4. Sara Lee v. Macatlang


G.R. No. 180147
Jan. 14, 2015
Perez, J.:

DOCTRINE:
 A compromise must not be contrary to law, morals, good customs and public policy; and must have been freely
and intelligently executed by and between the parties.
 The 10% requirement set in McBurniz v. Ganzon pertains to the reasonable amount which the NLRC would
accept as the minimum of the bond that should accompany the motion to reduce bond in order to suspend the
period to perfect an appeal under the NLRC rules. The 10% is based on the judgment award and should in no
case be construed as the minimum amount of bond to be posted in order to perfect appeal.
FACTS:
This case arises from a motion for reconsideration filed by Sara Lee from a judgment rendered by the Supreme
Court.
Aris Philippines permanently ceased operations on 9 October 1995 displacing 5,984 rank- and-file employees. On
26 October 1995, Fashion Accessories Phils. Inc. (FAPI) was incorporated prompting former Aris employees to
file a case for illegal dismissal on the allegations that FAPI was a continuing business of Aris. Sara Lee
Corporations, Sara Lee Philippines and Cesar Cruz were impleaded as defendants being major stockholders of
FAPI and officers of Aris, respectively.

LA RULING: Found the dismissals to be illegal and awarded monetary benefits amounting to P3.4 BILLION
pesos. The Petitioner Corporations filed a Notice of Appeal together with a Motion to Reduce Appeal Bond. The
amount of 4.5 million was accepted with an order to post an additional 4.5 million bond.

CA RULING: The Aris employees filed a Petition for Review before the Court of Appeals raising that the amount
should have been an equivalent of the monetary award. The CA changed the amount to 1 BILLION- later reduced
725 Million by the Supreme Court.

APPEAL TO THE SC
Petitioner’s Argument: Seek to enforce the 10% minimum bond amount in the McBurnie v. Ganzon case;
enforce a compromise agreement denominated as a “Motion to Admit Confession of Judgment”
Respondent’s Argument: Sustains rulings of the Court.
ISSUES:
1. Whether or not there is merit in claiming the 10% bond amount.
2. Whether or not there the compromise is valid.
RULING:
1. NO. The Corporations gravely misappreciated the ruling in McBurnie. The 10% requirement pertains to the
reasonable amount which the NLRC would accept as the minimum of the bond that should accompany the motion
to reduce bond in order to suspend the period to perfect an appeal under the NLRC rules. The 10% is based on
the judgment award and should in no case be construed as the minimum amount of bond to be posted in order to
perfect appeal. There is no room for a different interpretation when McBurniemade it clear that the percentage of
bond set is provisional, thus:
The foregoing shall not be misconstrued to unduly hinder the NLRC's exercise of its discretion, given that the
percentage of bond that is set by this guideline shall be merely provisional. The NLRC retains its authority and
duty to resolve the motion and determine the final amount of bond that shall be posted by the appellant, still in
accordance with the standards of "meritorious grounds" and "reasonable amount." Should the NLRC, after
considering the motion's merit, determine that a greater amount or the full amount of the bond needs to be posted

Angeles| Bajana | Balladares | Brillantes | Briones | Cabansag | Callanta | Chua | David|

De Leon | Gomez | Lopez | Macalino | Nostratis | Padilla | Reynon | Santos | Tan |Velasco
4E / 4F - 2018-2019
Page 326 of 920
LABOR REVIEW DIGEST
Atty. Joyrich Golangco

by the appellant, then the party shall comply accordingly. The appellant shall be given a period of 10 days from
notice of the NLRC order within which to perfect the appeal by posting the required appeal bond.
2. NO. A compromise must not be contrary to law, morals, good customs and public policy; and must have been
freely and intelligently executed by and between the parties. A compromise agreement is valid as long as the
consideration is reasonable and the employee signed the waiver voluntarily, with a full understanding of what he
was entering into.
A review of the compromise agreement shows a gross disparity between the amount offered by the Corporations
compared to the judgment award. The judgment award is P3,453,664,710.86 or each employee is slated to
receive P577,149.85. On the other hand, the P342,284,800.00 compromise is to be distributed among 5,984
employees which would translate to only P57,200.00 per employee. From this amount, P8,580.00 as attorney's
fees will be deducted, leaving each employee with a measly P48,620.00. In fact, the compromised amount
roughly comprises only 10% of the judgment award.
We note that the compromise is a mere 6% of the contingent sum that may be received by petitioners and the
minuscule amount is certainly questionable because it does not represent a true and fair amount which a
reasonable agent may bargain for his principal.

Angeles| Bajana | Balladares | Brillantes | Briones | Cabansag | Callanta | Chua | David|

De Leon | Gomez | Lopez | Macalino | Nostratis | Padilla | Reynon | Santos | Tan |Velasco
4E / 4F - 2018-2019
Page 327 of 920
LABOR REVIEW DIGEST
Atty. Joyrich Golangco

5. Dela Rosa Liner Inc. and/or Rosauro Dela Rosa, Sr. and Nora Dela Rosa v. Borela
GR No: 207286 Date: July 29, 2015
Ponente: Brion, J.:

Doctrine:
The test to determine whether causes of action are identical is to ascertain whether the same evidence
would support both actions, or whether there is an identity in the facts essential to the maintenance of the
two actions. If the same facts or evidence would support both actions, then they are considered the same;
a judgment in the first case would be a bar to the subsequent action.

FACTS:
Dela Rosa Liner et al. are the petitioners in this case against private respondents Calixto B. Borela and
Estelo A. Amarille.
On Sept. 23, 2011, Borela, bus driver, and Amarille, conductor, filed separate complaints against
petitioners, for underpayment/non-payment of salaries, holiday pay, overtime pay, SIL, 13th month pay, sick leave
and vacation leave, NSD, illegal deductions, and violation of Wage Order Nos. 13, 14, 15 and 16.
The case was filed with the Labor Arbiter, Danna A. Castillon on sept 23. 2011.
Petitioners seek the dismissal of the case on the ground of forum shopping. They alleged that on Sept
28, 2011, the CA 13th Division disposed of a similar case between the parties (CA-GR SP no. 118038) after they
entered into a compromise agreement which covered all claims and causes of action they had against each other
in relation to the respondents’ employment.
Respondents opposed the motion, contending that the causes of action in the present case are different
from the causes of action settled in the case cited by petitioner.

LA/RTC/NLRC RULING:
LA: Castillon upheld petitioners’ position and dismissed the complaint
NLRC: Reversed the decision of the LA, stating that forum shopping could not have been committed by the
respondents’ as there was no identity of causes of action between the two cases and RES JUDICATA would
not bar the current petition. First complaint was for illegal dismissal and unfair labor practice; while the second
complaint was based on the petitioners’ alleged nonpayment/underpayment of their salaries and monetary
benefits, and violation of wage orders. MR was denied by the NLRC.

CA RULING:
Denied the petition. CA found no grave abuse of discretion in the NLRC ruling that there was no presence of
forum shopping and res judicata as bars to the respondents’ subsequent money claims against the petitioners.
MR was denied by the CA.

APPEAL TO THE SC:


Petitioner's Contention: that the CA erred in upholding that there was no forum shopping nor res judicata
that would bar the second complaint.
Respondent's Contention: the petition should not prosper as it was belatedly filed. They claimed that the
last day to file the current petition was June 12, 2013 but the petition was notarized only on June 13, 2013
which means that it was filed only on that day. And that their second complaint is based on a different
cause of action than that of the first which was already settled by the parties.

ISSUE/S:
1. Whether or not the petition was belatedly filed
2. Whether or not respondents committed forum shopping when they filed the second complaint

HELD:
Angeles| Bajana | Balladares | Brillantes | Briones | Cabansag | Callanta | Chua | David|

De Leon | Gomez | Lopez | Macalino | Nostratis | Padilla | Reynon | Santos | Tan |Velasco
4E / 4F - 2018-2019
Page 328 of 920
LABOR REVIEW DIGEST
Atty. Joyrich Golangco

1. Yes, current petition was timely filed. Under Sec. 1 of Rule 22 of the Rules of Court, when the last day on
which a pleading is due falls on a Sunday, Saturday, or a legal holiday, the filing of the pleading on the next
working day is deemed on time. The filing of the petition in this case was valid since June 12, 2013, the last day of
filing was a legal holiday (Independence Day)
2. No, the CA 15th Div committed no reversible error in affirming the NLRC decision.
We concur with the CA that forum shopping and res judicata are not applicable in the present case.
There is no identity of rights asserted and reliefs prayed for, and the judgment rendered in the previous action will
not amount to res judicata in the action now under consideration.
There is also no identity of causes of action in the first complaint and in the second complaint. In Yap v.
Chua, we held that the test to determine whether causes of action are identical is to ascertain whether the
same evidence would support both actions, or whether there is an identity in the facts essential to the
maintenance of the two actions. If the same facts or evidence would support both actions, then they are
considered the same; a judgment in the first case would be a bar to the subsequent action. Under the
circumstances of the case before us, sufficient basis exists for the NLRC's and CA's conclusions that there is no
identity of causes of action between the respondents' two complaints against the petitioners. The first complaint
involved illegal dismissal/suspension, unfair labor practice with prayer for damages and attorney's fees; while the
second complaint (the subject of the present appeal) involves claims for labor standards benefits — the
petitioners' alleged violation of Wage Orders Nos. 13, 14, 15 and 16; nonpayment of respondents' sick and
vacation leave pays, 13th-month pay, service incentive leave benefit, overtime pay, and night shift differential.
To put it simply, the facts or the evidence that would determine whether respondents were illegally
dismissed, illegally suspended, or had been the subject of an unfair labor practice act by the petitioners
are not the same facts or evidence that would support the charge of non-compliance with labor standards
benefits and several wage orders. We thus cannot find a basis for petitioners' claim that "the same action had
been settled . . . ."
We likewise cannot accept the compromise agreement's application "to all claims and damages or
losses either party may have against each other whether those damages or losses are known or
unknown, foreseen or unforeseen."
This coverage is too sweeping and effectively excludes any claims by the respondents against
the petitioners, including those that by law and jurisprudence cannot be waived without appropriate
consideration such as nonpayment or underpayment of overtime pay and wages.

Other Notes/ SC Pronouncements:

We note that in the illegal dismissal case where the compromise agreement took place, the NLRC 6th
Division (acting on the appeal from the LA's ruling) awarded Borela P442,550.00 in backwages; P20,000.00 in
moral and exemplary damages, plus 10% attorney's fees; and to Amarille P215,775.00 in back wages and
P50,000.00 in moral and exemplary damages, plus 10% attorney's fees.
Although the NLRC reconsidered these awards and eventually granted financial assistance of
P10,000.00 each to Borela and Amarille, it is reasonable to regard the amounts they received as a fair
compromise in the settlement of the first complaint in relation with the initial NLRC award, indicated above, before
its reconsideration. To be sure, the parties, especially the respondents, could not have considered the P10,000.00
financial assistance or their labor standards claims, particularly the alleged violation of the wage orders, as a
factor in their effort to settle the case amicably. The compromise agreement, it should be emphasized, was
executed on September 8, 2011, while the labor standards complaint was filed only on September 23, 2011.

Angeles| Bajana | Balladares | Brillantes | Briones | Cabansag | Callanta | Chua | David|

De Leon | Gomez | Lopez | Macalino | Nostratis | Padilla | Reynon | Santos | Tan |Velasco
4E / 4F - 2018-2019
Page 329 of 920
LABOR REVIEW DIGEST
Atty. Joyrich Golangco

6. Magsaysay Maritime Corporation v. Cynthia De Jesus


GR No: 203943 Date: August 30, 2017
Ponente: J. Leonen

Doctrine:
A conditional settlement of a judgment award may be treated as a compromise agreement and a judgment on the
merits of the case if it turns out to be highly prejudicial to one of the parties.

FACTS:
Petitioner Magsaysay Maritime Corporation, the local manning agent of Princess Cruise Lines, Limited, hired
Bernardine De Jesus as an Accommodation Supervisor for the cruise ship Regal Princess. Bernardine boarded
Regal Princess and he eventually disembarked 10 months later after his contract of employment ended.
Bernardine was soon diagnosed with Aortic Aneurysm and he had a coronary angiography. He also underwent a
Left Axillofemoral Bypass but later died.

Respondent Cynthia De Jesus, Bernardine's widow, filed a complaint against Magsaysay for payment of death
benefits, medical expenses, sickness allowance, damages, and attorney's fees. Cynthia and Magsaysay were
unable to amicably settle the case; hence, they were directed to submit their respective position papers.

LA/RTC/NLRC RULING:
Labor Arbiter granted Cynthia's complaint and directed Magsaysay to pay her claims for death benefits, additional
benefits, burial expenses, and attorney's fees.

The National Labor Relations Commission denied Magsaysay's appeal and upheld the Labor Arbiter's finding that
Bernardine's cardio-vascular disease was work-related. It also noted that while the general rule in compensability
of death is that a seafarer's death must have occurred during the term of the employment contract, an exception
to this rule is when a seafarer contracted an illness while under the contract and this illness caused his death

Magsaysay paid Cynthia an amount as conditional satisfaction of the judgment award against it and without
prejudice to its Petition for Certiorari pending before the Court of Appeals. In light of the conditional settlement
between the parties, the Labor Arbiter considered the case closed and terminated but without prejudice to
Magsaysay's pending petition before the Court of Appeals.

CA RULING:
The Court of Appeals dismissed the petition for being moot and academic.

APPEAL TO THE SC:


Petitioners filed their Petition for Review on Certiorari where they continue to assert that the Court of Appeals
erred in dismissing their Petition for Certiorari for being moot and academic. Petitioners emphasize that Leonis
Navigation v. Villamater stated that if the Court of Appeals grants a petition for certiorari, the assailed decision of
the National Labor Relations Commission will become void ab initio and will never attain finality. Petitioners
maintain that Leonis ruled that even if the employer voluntarily pays the judgment award, the seafarer's
beneficiary is estopped from claiming that the controversy has ended with the Labor Arbiter's Order closing and
terminating the case. This is because the beneficiary acknowledged that the payment received was without
prejudice to the final outcome of the petition for certiorari pending before the Court of Appeals.

ISSUE/S:
1. Whether or not the payment of money judgment has rendered the Petition for Certiorari before the Court of
Appeals moot and academic.
2. Whether or not the award of death benefits was proper.
Angeles| Bajana | Balladares | Brillantes | Briones | Cabansag | Callanta | Chua | David|

De Leon | Gomez | Lopez | Macalino | Nostratis | Padilla | Reynon | Santos | Tan |Velasco
4E / 4F - 2018-2019
Page 330 of 920
LABOR REVIEW DIGEST
Atty. Joyrich Golangco

HELD:
1. Yes. In the instant case, the parties entered into a compromise agreement when they executed a Conditional
Satisfaction of Judgment Award. However, the prohibition on the part of respondent to pursue any of the available
legal remedies should the Court of Appeals or Supreme Court reverse the judgment award of the labor tribunals
or prosecute any other suit or action in another country puts the seafarer's beneficiaries at a grave disadvantage.
Thus, Career Philippines is applicable and the Court of Appeals did not err in treating the conditional settlement as
an amicable settlement, effectively rendering the Petition for Certiorari moot and academic.

Article 2028 of the Civil Code defines a compromise agreement as "a contract whereby the parties, by making
reciprocal concessions, avoid a litigation or put an end to one already commenced." Parties freely enter into a
compromise agreement, making it a judgment on the merits of the case with the effect of res judicata upon them.

While the general rule is that a valid compromise agreement has the power to render a pending case moot and
academic, being a contract, the parties may opt to modify the legal effects of their compromise agreement to
prevent the pending case from becoming moot.

In the Conditional Satisfaction of Judgment Award, respondent acknowledged receiving a conditional payment of
the judgment award from the petitioners. Both parties agreed that the payment of the judgment award was without
prejudice to the pending certiorari proceedings before the Court of Appeals and was only made to prevent the
imminent execution being undertaken by respondent and the National Labor Relations Commission. Finally, in the
event the judgment award of the labor tribunals is reversed by the Court of Appeals or by this Court, respondent
agreed to return whatever she would have received back to petitioners and in the same vein, if the Court of
Appeals or this Court affirms the decisions of the labor tribunals, petitioners shall pay respondent the balance of
the judgment award without need of demand.

Respondent then signed a Receipt of Payment where she reiterated the undertakings she took in the Conditional
Satisfaction of Judgement Award. However, in the Affidavit of Heirship, respondent was prohibited from seeking
further redress against petitioners, making the compromise agreement ultimately prejudicial to respondent

2. Yes. Section 20 (A) of the POEA-SEC requires that for a seafarer to be entitled to death benefits, he must have
suffered a work-related death during the term of his contract.

Against the self-serving denials of the respondents, complainant has shown that her husband, prior to his death,
suffered chest pains while on board and reported his condition but he was not allowed to seek medical attention.
When he was repatriated, he asked the respondents anew for medical check up but his request was again denied.
Having substantially established that the causative circumstances leading to her husband's death had transpired
during his employment, the complainant is entitled to the death compensation and other benefits under the POEA
Standard Contract. Probability and not the ultimate degree of certainty is the test of proof in compensation
proceedings.

Angeles| Bajana | Balladares | Brillantes | Briones | Cabansag | Callanta | Chua | David|

De Leon | Gomez | Lopez | Macalino | Nostratis | Padilla | Reynon | Santos | Tan |Velasco
4E / 4F - 2018-2019
Page 331 of 920
LABOR REVIEW DIGEST
Atty. Joyrich Golangco

ARTICLE 229
1. Islriz Trading v Capada et. al.
GR No: 168501 Date: January 31, 2011
Ponente: DEL CASTILLO, J.:

Doctrine: We reiterate in this petition the settled view that employees are entitled to their accrued salaries during
the period between the Labor Arbiters order of reinstatement pending appeal and the resolution of the National
Labor Relations Commission (NLRC) overturning that of the Labor Arbiter. Otherwise stated, even if the order of
reinstatement of the Labor Arbiter is reversed on appeal, the employer is still obliged to reinstate and pay the
wages of the employee during the period of appeal until reversal by a higher court or tribunal.

FACTS:
Respondents were drivers and helpers of Islriz Trading, a gravel and sand business owned and operated by
petitioner Victor Hugo Lu. Claiming that they were illegally dismissed, respondents filed a Complaint for illegal
dismissal and non-payment of overtime pay, holiday pay, rest day pay, allowances and separation pay against
petitioner on August 9, 2000 before the Labor Arbiter. On his part, petitioner imputed abandonment of work
against respondents.

LA/RTC/NLRC RULING:
LA ruled that ISLRIZ TRADING was guilty of illegal dismissal and ordered the reinstatement of complainants to
their former positions without loss of seniority rights and the payment of full backwages from date of dismissal to
actual reinstatement.
Upon appeal, the NLRC ordered respondents reinstatement but without backwages because respondents failure
to continue working for petitioner was neither caused by termination nor abandonment of work.
Upon refusal of petitioner to reinstate the respondents, respondents prayed for computation of award of
backwages and that an Alias Writ of Execution for its enforcement be issued. Thereafter, the personal properties
of petitioner were levied to satisfy the writ of execution issued.

CA RULING:
The CA quoted the June 3, 2004 Order of Labor Arbiter Castillon and agreed with her ratiocination that pursuant
to Article 223(now 229) of the Labor Code, what is sought to be enforced by the subject Writ of Execution is the
accrued salaries owing to respondents by reason of the reinstatement order of the Labor Arbiter.

APPEAL TO THE SC:


Petitioner contends that the assailed Decision and Resolution of the CA are contrary to law and jurisprudence.
This is because in upholding the issuance of the questioned Writ of Execution for the enforcement of respondents
accrued salaries, said Decision and Resolution, in effect, altered the NLRC Resolution which only decreed
respondents reinstatement without backwages. Moreover, he posits that Article 223(now 229) of the Labor Code
only applies when an employee has been illegally dismissed from work. And since in this case the NLRC ruled
that respondents failure to continue working for petitioner was not occasioned by termination, there is no illegal
dismissal to speak of, hence, said provision of the Labor Code does not apply. Lastly, petitioner claims that the
computation of respondents accrued salaries in the total amount of P1,110,665.60 has no legal and factual bases
since as repeatedly pointed out by him, the NLRC Resolution reversing the Labor Arbiters Decision has already
ordered respondents reinstatement without backwages after it found that there was no illegal termination.

Respondents, on the other hand, maintain that the CA did not err in applying Article 223(now 229) of the Labor
Code to the instant case. They thus contend that the computation of their accrued salaries covering the period
during which they were supposed to have been reinstated or from January 1, 2002 to January 30, 2004, should be
upheld since same merely applied Article 223(now 229). In sum, respondents believe that the assailed Decision
and Resolution of the CA are in accord with law and jurisprudence.
Angeles| Bajana | Balladares | Brillantes | Briones | Cabansag | Callanta | Chua | David|

De Leon | Gomez | Lopez | Macalino | Nostratis | Padilla | Reynon | Santos | Tan |Velasco
4E / 4F - 2018-2019
Page 332 of 920
LABOR REVIEW DIGEST
Atty. Joyrich Golangco

ISSUE/S:
1. Whether respondents may collect their wages during the period between the Labor Arbiters order of
reinstatement pending appeal and the NLRC Resolution overturning that of the Labor Arbiter.

HELD:
1. Yes, even if the order of reinstatement of the Labor Arbiter is reversed on appeal, it is obligatory on the part of
the employer to reinstate and pay the wages of the dismissed employee during the period of appeal until reversal
by the higher court or tribunal. It likewise settled the view that the Labor Arbiters order of reinstatement is
immediately executory and the employer has to either re-admit them to work under the same terms and conditions
prevailing prior to their dismissal, or to reinstate them in the payroll, and that failing to exercise the options in the
alternative, employer must pay the employees salaries.
It then provided for the two-fold test in determining whether an employee is barred from recovering his accrued
wages, to wit: (1) there must be actual delay or that the order of reinstatement pending appeal was not executed
prior to its reversal; and (2) the delay must not be due to the employers unjustified act or omission. If the delay is
due to the employers unjustified refusal, the employer may still be required to pay the salaries notwithstanding the
reversal of the Labor Arbiters Decision.
To apply the two-fold test:
Was there an actual delay or was the order of reinstatement pending appeal executed prior to its reversal? As can
be recalled, Labor Arbiter Gan issued his Decision ordering respondents reinstatement on December 21, 2001.
Then, petitioner appealed said decision to the NLRC. On April 22, 2002, a Writ of Execution was issued by Labor
Arbiter Gan. However, until the issuance of the September 5, 2002 NLRC Resolution overturning Labor Arbiter
Gan’s Decision, petitioner still failed to reinstate respondents or effect payroll reinstatement in accordance with
Article 223(now 229) of the Labor Code. It cannot therefore be denied that there was an actual delay in the
execution of the reinstatement aspect of the Decision of Labor Arbiter Gan prior to the issuance of the NLRC
Resolution overturning the same.

Now, the next question is: Was the delay not due to the employers unjustified act or omission? Islriz Trading did
not undergo rehabilitation or was under any analogous situation which would justify petitioners non-exercise of the
options provided under Article 223(now 229) of the Labor Code. Notably, what petitioner gave as reason in not
immediately effecting reinstatement after he was served with the Writ of Execution dated April 22, 2002 was that
he would first refer the matter to his counsel as he could not effectively act on the order of execution without the
latter’s advice. Petitioner, however, without any satisfactory reason, failed to fulfill this promise and respondents
remained to be not reinstated until the NLRC resolved petitioners appeal. Evidently, the delay in the execution of
respondents reinstatement was due to petitioner’s unjustified refusal to effect the same.
Hence, the conclusion is that respondents have the right to collect their accrued salaries during the period
between the Labor Arbiters Decision ordering their reinstatement pending appeal and the NLRC Resolution
overturning the same because petitioner’s failure to reinstate them either actually or through payroll was due to
petitioners unjustified refusal to effect reinstatement.

Other Notes/ SC Pronouncements:


[T]he refund doctrine easily demonstrates how a favorable decision by the Labor Arbiter could harm, more than
help, a dismissed employee. The employee, to make both ends meet, would necessarily have to use up the
salaries received during the pendency of the appeal, only to end up having to refund the sum in case of a final
unfavorable decision. It is mirage of a stop-gap leading the employee to a risky cliff of insolvency.
_________________________________________________________________________

Angeles| Bajana | Balladares | Brillantes | Briones | Cabansag | Callanta | Chua | David|

De Leon | Gomez | Lopez | Macalino | Nostratis | Padilla | Reynon | Santos | Tan |Velasco
4E / 4F - 2018-2019
Page 333 of 920
LABOR REVIEW DIGEST
Atty. Joyrich Golangco

2. CESAR V. GARCIA, et al. vs. KJ COMMERCIAL and REYNALDO QUE


GR No: 196830 Date: February 29, 2012
Ponente: CARPIO, J.:

DOCTRINE:
The bond requirement on appeals involving a monetary award in labor cases under Art. 223 may be relaxed only
when there is substantial compliance with the Rules of Procedure of the NLRC or when the appellant shows
willingness to post a partial bond.

FACTS:
Petitioners were employed as truck drivers and truck helpers by respondent KJ Commercial, they demanded
for a P40 daily salary increase. To pressure respondent, they stopped working; abandoned their trucks at the
Angeles| Bajana | Balladares | Brillantes | Briones | Cabansag | Callanta | Chua | David|

De Leon | Gomez | Lopez | Macalino | Nostratis | Padilla | Reynon | Santos | Tan |Velasco
4E / 4F - 2018-2019
Page 334 of 920
LABOR REVIEW DIGEST
Atty. Joyrich Golangco

Plant Station in Pangasinan; and blocked other workers from reporting to work.
Petitioners filed with the LA a complaint for illegal dismissal, underpayment of salary and non-payment of
service incentive leave and thirteenth month pay.

LA RULING: petitioners were illegally dismissed

ON APPEAL TO NLRC: respondent filed with NLRC a motion to reduce appeal bond and posted a PARTIAL
AMOUNT P50K cash bond.

NLRC RULING: dismissed appeal as respondent failed to perfect the same for failure to post the required bond.

ON MR TO NLRC: respondent posted the FULL AMOUNT P2,562,930 surety bond that was granted by NLRC.

NLRC RULING: reversed LA, and ruling that it is within its power and discretion to grant or deny a motion to
reduce appeal bond, it is not precluded from reconsidering its earlier decision when it finds meritorious
ground to serve the ends of justice.

CA RULING: affirmed NLRC decisions

Petitioner’s contention:
The NLRC and CA decisions are void for lack of jurisdiction because the LA Decision having reached finality
for respondent’s failure to perfect an appeal as the motion to reduce bond did not stop the running of the period
to appeal.

ISSUE/S:
Whether respondent’s delayed posting of the appeal bond substantially complied with the requirement of posting
of a cash or surety bond to perfect an appeal? YES.

HELD:
Under Rules of Procedure of the NLRC, a motion to reduce bond to stop the running of the period to perfect
an appeal shall only be entertained when:
(1) The motion is founded on meritorious ground; and
(2) A bond of reasonable amount in relation to the monetary award is posted.

Exceptional circumstances that warrant the relaxation of the rule that the posting of the bond to be an
indispensable requirement to perfect an employers appeal: (a) counsels reliance on the footnote of the notice of
the decision of the labor arbiter that the aggrieved party may appeal within ten (10) working days; (b) fundamental
consideration of substantial justice; (c) prevention of miscarriage of justice or of unjust enrichment, as where the
tardy appeal is from a decision granting separation pay which was already granted in an earlier final decision; and
(d) special circumstances of the case combined with its legal merits or the amount and the issue involved.

In Ong v. Court of Appeals, the bond requirement on appeals may be relaxed when there is substantial
compliance with the Rules of Procedure of the NLRC or when the appellant shows willingness to post a
partial bond.

APPLICATION - KJ Commercial showed willingness to post a partial bond of P50K cash bond. Aside from posting
a partial bond, it immediately posted the full amount of the bond when it filed its motion for reconsideration of the
NLRCs Decision.

Angeles| Bajana | Balladares | Brillantes | Briones | Cabansag | Callanta | Chua | David|

De Leon | Gomez | Lopez | Macalino | Nostratis | Padilla | Reynon | Santos | Tan |Velasco
4E / 4F - 2018-2019
Page 335 of 920
LABOR REVIEW DIGEST
Atty. Joyrich Golangco

3. MARIANO ONG, doing business under the name and style MILESTONE METAL
MANUFACTURING, petitioner, vs. THE COURT OF APPEALS, CONRADO DABAC, BERNABE TAYACTAC,
MANUEL ABEJUELLA, LOLITO ABELONG, RONNIE HERRERO, APOLLO PAMIAS, JAIME ONGUTAN,
NOEL ATENDIDO, CARLOS TABBAL, JOEL ATENDIDO, BIENVENIDO EBBER, RENATO ABEJUELLA,
LEONILO ATENDIDO, JR., LODULADO FAA and JAIME LOZADA, respondents.
GR No: 152494 Date: September 22, 2004
Ponente: YNARES-SANTIAGO, J

Doctrine:
Time and again it has been held that the right to appeal is not a natural right or a part of due process, it is merely
a statutory privilege, and may be exercised only in the manner and in accordance with the provisions of law. The
party who seeks to avail of the same must comply with the requirements of the rules. Failing to do so, the right to
appeal is lost. The posting of cash or surety bond is not only mandatory but jurisdictional as well, and non-
compliance therewith is fatal and has the effect of rendering the judgment final and executory. This requirement is
intended to discourage employers from using the appeal to delay, or even evade, their obligation to satisfy their
employee’s just and lawful claims.

FACTS:

Petitioner is the sole proprietor of Milestone Metal Manufacturing (Milestone), which manufactures, among others,
wearing apparels, belts, and umbrellas. Sometime in May 1998, the business suffered very low sales and
productivity because of the economic crisis in the country.
Hence, it adopted a rotation scheme by reducing the workdays of its employees to three days a week or less for
an indefinite period.
On separate dates, the 15 respondents filed before the National Labor Relations Commission (NLRC) complaints
for illegal dismissal, underpayment of wages, non-payment of overtime pay, holiday pay, service incentive leave
pay, 13th month pay, damages, and attorneys fees against petitioner.
Petitioner claimed that 9 of the 15 respondents were not employees of Milestone but of Protone Industrial
Corporation which, however, stopped its operation due to business losses.Further, he claims that respondents
Manuel Abuela, Lolita Abelong, Ronnie Herrero, Carlos Tabbal, Conrado Dabac, and Lodualdo Faa were not
dismissed from employment; rather, they refused to work after the rotation scheme was adopted.
Anent their monetary claims, petitioner presented documents showing that he paid respondents’ minimum wage,
13th month pay, holiday pay, and contributions to the SSS, Medicare, and Pag-Ibig Funds.
LA: rendered a decision awarding to the respondents the aggregate amount of P1,111,200.40 representing their
wage differential, holiday pay, service incentive leave pay and 13th month pay, plus 10% thereof as attorneys
fees. Further, petitioner was ordered to pay the respondents separation pay equivalent to month salary for every
year of service due to the indefiniteness of the rotation scheme and strained relations caused by the filing of the
complaints.
Petitioner filed with the NLRC a notice of appeal with a memorandum of appeal and paid the docket fees
therefor. However, instead of posting the required cash or surety bond, he filed a motion to reduce the appeal
bond.
NLRC: denied the motion to reduce bond and dismissed the appeal for failure to post cash or surety bond within
the reglementary period. Petitioners motion for reconsideration was likewise denied.
CA: the petition was dismissed and thereafter the motion for reconsideration was likewise dismissed for lack of
merit.

ISSUE/S:
Whether or not NLRC acted with grave abuse of discretion in dismissing the appeal for non-perfection of appeal
although a motion to reduce appeal bond was seasonably filed.
Angeles| Bajana | Balladares | Brillantes | Briones | Cabansag | Callanta | Chua | David|

De Leon | Gomez | Lopez | Macalino | Nostratis | Padilla | Reynon | Santos | Tan |Velasco
4E / 4F - 2018-2019
Page 336 of 920
LABOR REVIEW DIGEST
Atty. Joyrich Golangco

HELD:

NO. Time and again it has been held that the right to appeal is not a natural right or a part of due process, it
is merely a statutory privilege, and may be exercised only in the manner and in accordance with the provisions of
law. The party who seeks to avail of the same must comply with the requirements of the rules. Failing to do so, the
right to appeal is lost.
Article 223 of the Labor Code, as amended, sets forth the rules on appeal from the Labor Arbiters monetary
award:
ART. 223. Appeal. Decisions, awards, or orders of the Labor Arbiter are final and executory unless appealed to
the Commission by any or both parties within ten (10) calendar days from receipt of such decisions, awards, or
orders. x x x.
In case of a judgment involving a monetary award, an appeal by the employer may be perfected only upon the
posting of a cash or surety bond issued by a reputable bonding company duly accredited by the Commission in
the amount equivalent to the monetary award in the judgment appealed from. (Emphasis ours)
In the case at bar, petitioner received the decision of the Labor Arbiter on January 6, 2000. He filed his notice of
appeal with memorandum of appeal and paid the corresponding appeal fees on January 17, 2000, the last day of
filing the appeal. However, in lieu of the required cash or surety bond, he filed a motion to reduce bond alleging
that the amount of P1,427,802,04 as bond is unjustified and prohibitive and prayed that the same be reduced to
a reasonable level. The NLRC denied the motion and consequently dismissed the appeal for non-perfection.
Petitioner now contends that he was deprived of the chance to post bond because the NLRC took 102 days to
decide his motion. Petitioners argument is unavailing.
While, Section 6, Rule VI of the NLRCs New Rules of Procedure allows the Commission to reduce the amount of
the bond, the exercise of the authority is not a matter of right on the part of the movant but lies within the sound
discretion of the NLRC upon showing of meritorious grounds.
After careful scrutiny of the motion to reduce appeal bond, we agree with the Court of Appeals that the NLRC did
not act with grave abuse of discretion when it denied petitioners motion for the same failed to either elucidate why
the amount of the bond was unjustified and prohibitive or to indicate what would be a reasonable level.
In Calabash Garments, Inc. v. NLRC, it was held that a substantial monetary award, even if it runs into millions,
does not necessarily give the employer-appellant a meritorious case and does not automatically warrant a
reduction of the appeal bond.
Even granting arguendo that petitioner has meritorious grounds to reduce the appeal bond, the result would have
been the same since he failed to post cash or surety bond within the prescribed period.
The above-cited provisions explicitly provide that an appeal from the Labor Arbiter to the NLRC must be perfected
within ten calendar days from receipt of such decisions, awards or orders of the Labor Arbiter. In a judgment
involving a monetary award, the appeal shall be perfected only upon (1) proof of payment of the required appeal
fee; (2) posting of a cash or surety bond issued by a reputable bonding company; and (3) filing of a memorandum
of appeal. A mere notice of appeal without complying with the other requisites mentioned shall not stop the
running of the period for perfection of appeal. The posting of cash or surety bond is not only mandatory but
jurisdictional as well, and non-compliance therewith is fatal and has the effect of rendering the judgment final and
executory. This requirement is intended to discourage employers from using the appeal to delay, or even evade,
their obligation to satisfy their employee’s just and lawful claims.
The fact that the NLRC took 102 days to resolve the motion will not help petitioners case. The NLRC Rules clearly
provide that the filing of the motion to reduce bond shall not stop the running of the period to perfect
appeal. Petitioner should have seasonably filed the appeal bond within the ten-day reglementary period following
the receipt of the order, resolution or decision of the NLRC to forestall the finality of such order, resolution or
decision. In the alternative, he should have paid only a moderate and reasonable sum for the premium, as was
held in Biogenerics Marketing and Research Corporation v. NLRC

Angeles| Bajana | Balladares | Brillantes | Briones | Cabansag | Callanta | Chua | David|

De Leon | Gomez | Lopez | Macalino | Nostratis | Padilla | Reynon | Santos | Tan |Velasco
4E / 4F - 2018-2019
Page 337 of 920
LABOR REVIEW DIGEST
Atty. Joyrich Golangco

While the bond requirement on appeals involving monetary awards has been relaxed in certain cases, this can
only be done where there was substantial compliance of the Rules or where the appellants, at the very least,
exhibited willingness to pay by posting a partial bond.
__________________________________________________________________________

Angeles| Bajana | Balladares | Brillantes | Briones | Cabansag | Callanta | Chua | David|

De Leon | Gomez | Lopez | Macalino | Nostratis | Padilla | Reynon | Santos | Tan |Velasco
4E / 4F - 2018-2019
Page 338 of 920
LABOR REVIEW DIGEST
Atty. Joyrich Golangco

4. Rosewood Processing vs NLRC


GR Nos: G.R. Nos. 116476-84 Date: May 21, 1998
Ponente: PANGANIBAN, J:

Doctrine: The appeal of a decision involving a monetary award in labor cases may be perfected "only upon the
posting of a cash or surety bond."

A motion to reduce the bond is a substantial compliance with the Labor Code. This holding is consistent
with the norm that letter-perfect rules must yield to the broader interest of substantial justice.
Angeles| Bajana | Balladares | Brillantes | Briones | Cabansag | Callanta | Chua | David|

De Leon | Gomez | Lopez | Macalino | Nostratis | Padilla | Reynon | Santos | Tan |Velasco
4E / 4F - 2018-2019
Page 339 of 920
LABOR REVIEW DIGEST
Atty. Joyrich Golangco

FACTS:
All the complainants were employed by Veterans Philippine Scout Security Agency [security agency] as security
guards. A complaint for illegal dismissal, underpayment of wages, and for nonpayment of overtime pay, legal
holiday pay, premium pay for holiday and rest day, thirteenth month pay, cash bond deposit, unpaid wages and
damages was filed against Veterans Philippine Scout Security Agency and/or Sergio Jamila IV. Thereafter,
petitioner Rosewood Processing was impleaded as a third-party respondent by the security agency.

LA RULING:
The LA noted the failure of the security agency to present evidence to refute the complainants' allegation. Further,
it impleaded petitioner Rosewood Processing as third-party respondent, contending that its actions were primarily
caused by petitioner's noncompliance with its obligations under the contract for security services, and the
subsequent cancellation of the said contract.

Although the security agency could lawfully place the complainants on floating status for a period not exceeding
six months, the act was "illegal" because the former had issued a newspaper advertisement for new security
guards. Since the relation between the complainants and the agency was already strained, the labor arbiter
ordered the payment of separation pay in lieu of reinstatement.

The LA held petitioner jointly and severally liable with the security agency as the complainants' indirect employer
under Articles 106, 107 and 109 of the Labor Code and ordered the respondents Veterans Philippine Scout
Security Agency, Sergio Jamila IV, and third-party respondent Rosewood Processing, Inc. to pay jointly and
severally complainants P789,154.39.

NLRC RULING:

The NLRC dismissed the appeal filed by petitioner for failure of the petitioner to file the required appeal bond
within the reglementary period. The NLRC denied MR. Hence, this special civil action for certiorari against the
NLRC.
APPEAL TO THE SC:

Petitioner's Contention:
Petitioner claims to have received the LA's Decision on April 6, 1993. On April 16, 1993, it filed, together with its
memorandum on appeal and notice of appeal, a motion to reduce the appeal bond accompanied by a surety bond
for 50,000 pesos issued by Prudential Guarantee and Assurance, Inc.

Respondent's Contention:
The solicitor general argues for the affirmation of the assailed Resolution for the sole reason that the appeal
bond, even if it was filed on time, was defective, as it was not in an amount "equivalent to the monetary
award in the judgment appealed from.

ISSUE/S
1. Did the filing of a motion to reduce the appeal bond constitute a substantial compliance? – Yes.
2. Is the indirect employer, Rosewood Processing, solidarily liable with the security agency for the payment of
back wages, wage differential, and separation pay? - Yes

HELD:
1. Yes. The petitioner's motion to reduce the bond is a substantial compliance with the Labor Code. This holding
is consistent with the norm that letter-perfect rules must yield to the broader interest of substantial justice.

Angeles| Bajana | Balladares | Brillantes | Briones | Cabansag | Callanta | Chua | David|

De Leon | Gomez | Lopez | Macalino | Nostratis | Padilla | Reynon | Santos | Tan |Velasco
4E / 4F - 2018-2019
Page 340 of 920
LABOR REVIEW DIGEST
Atty. Joyrich Golangco

Where a decision may be made to rest on informed judgment rather than rigid rules, the equities of the case must
be accorded their due weight because labor determinations should not only be "secundum rationem (according to
reason) but also secundum caritatem (according to a charitable heart)." A judicious reading of the memorandum
of appeal would have made it evident to NLRC that the recourse was meritorious.
The NLRC acted with grave abuse of discretion in peremptorily dismissing the appeal without passing upon — in
fact, ignoring — the motion to reduce the appeal bond.

2. Yes. Under the Labor Code, an employer is solidarily liable for legal wages due security guards for the period of
time they were assigned to it by its contracted security agency. However, in the absence of proof that the
employer itself committed the acts constitutive of illegal dismissal or conspired with the security agency in the
performance of such acts, the employer shall not be liable for back wages and/or separation pay arising as a
consequence of such unlawful termination.
Other Notes/ SC Pronouncements:
______________________________________________________________________

Angeles| Bajana | Balladares | Brillantes | Briones | Cabansag | Callanta | Chua | David|

De Leon | Gomez | Lopez | Macalino | Nostratis | Padilla | Reynon | Santos | Tan |Velasco
4E / 4F - 2018-2019
Page 341 of 920
LABOR REVIEW DIGEST
Atty. Joyrich Golangco

5. FILIPINAS SYSTEMS INC. vs NLRC


GR No.: 153850 Date: December 11, 2003
Ponente: Puno, J.

DOCTRINE:
Art. 223 of the Labor Code and Section 1 of the NLRC Rules of Procedure provide a ten (10) day period from
receipt of the decision of the Arbiter to file an appeal together with an appeal bond if the decision involves a
monetary award. The late filing of the bond divested the NLRC of its jurisdiction to entertain petitioner’s appeal.

FACTS:

A complaint for illegal dismissal and monetary claims were filed by private respondents against their employer,
Filipinas Systems, Inc. (Filsystems for brevity). Filsystems failed to file its position paper in spite of the order of the
Labor Arbiter prompting the Labor Arbiter to decide in favor of respondents in the illegal dismissal complaints and
awarded their monetary claims.
Filsystems appealed to the NLRC submitting for the first time evidence showing that respondents were project
employees whose dismissal was due to the discontinuation of the project they were assigned.Respondents
Questioned the jurisdiction of the NLRC over the appeal as petitioner belatedly filed the appeal bond however, the
NLRC assumed jurisdiction and remanded the case to the Labor Arbiter for further proceedings.
Respondents’motion for reconsideration was denied so they appealed to the CA via a petition for Certiorari. The
CA Held that the NLRC lacks jurisdiction over the appeal for late filing of the appeal bond and reinstated the Labor
Arbiter’s decision. Peitioner’s motion for reconsideration was denied.

ISSUE/S:

1. Whether the NLRC acquired jurisdiction over Petitioners’ appeal.

HELD:
NO.
Art.223 of the Labor Code and Section 1 of the NLRC Rules of Procedure provide a ten(10)day period from
receipt of the decision of the Arbiter to file an appeal together with an appeal bond if the decision involves a
monetary award. Records showed that petitioners received a copy of the Arbiters decision on October 31. Their
memorandum of appeal was dated November 9, but their appeal bond was e1ected only on November 17. No
partial payment of the bond was made within the reglementary period nor did they submit an explanation for its
late filing. The late filing of the bond divested the NLRC of its jurisdiction to entertain petitioner’s appeal.

Further, petitioners failed to submit their evidence to the Labor Arbiter despite opportunities given them and only
submitted the evidence to the NLRC when the decision was adverse to their interest.The court dismissed the
petition and the decision of the Labor Arbiter was reinstated with the modification that if reinstatement of
respondents is not feasible, petitioner should pay their separation pay in accordance with law.

Angeles| Bajana | Balladares | Brillantes | Briones | Cabansag | Callanta | Chua | David|

De Leon | Gomez | Lopez | Macalino | Nostratis | Padilla | Reynon | Santos | Tan |Velasco
4E / 4F - 2018-2019
Page 342 of 920
LABOR REVIEW DIGEST
Atty. Joyrich Golangco

6. BUENAOBRA vs LIM KING GUAN


G.R. No. 150147; January 20, 2004
Corono, J.

DOCTRINE: The provision of Article 223 (now 229) of the Labor Code requiring the posting of bond on appeals
involving monetary awards must be given liberal interpretation in line with the desired objective of resolving
controversies on the merits. If only to achieve substantial justice, strict observance of the reglementary periods
may be relaxed if warranted

FACTS: Petitioners (Buenaobra et al) were employees of private respondent Unix International Export
Corporation (UNIX), a corporation engaged in the business of manufacturing bags, wallets and the like.
Sometime in 1991 and 1992, petitioners filed several cases against UNIX and its incorporators and officers for
unfair labor practice, illegal lockout/dismissal, underpayment of wages, holiday pay, proportionate 13 th month pay,
unpaid wages, interest, moral and exemplary damages and attorneys fees.
Labor Arbiter Patis rendered a decision ordering respondent Unix Export Corporation to pay complainants
backwages, separation pay, wage differentials, regular holiday pay differentials; and proportionate 13 th month pay
for 1990.
There being no appeal by respondents or petitioners, the decision of labor arbiter eventually became final and
executory. However, petitioners complained that the decision could not be executed because UNIX allegedly
diverted, invested and transferred all its money, assets and properties to respondent Fuji Zipper Manufacturing
Corporation (FUJI) whose stockholders and officers were also those of UNIX.
Petitioners filed another complaint against respondents UNIX, its corporate officers and stockholders of record,
and FUJI. Petitioners mainly prayed that respondents UNIX and FUJI be held jointly and severally held liable for
the payment of the monetary awards ordered by labor arbiter de Vera.
LA Ruling: Ordered a judgment of piercing the veil of corporate fiction of the two respondent sister corporations
which by virtue of this Decision are now considered as mere associations of persons jointly and severally pay the
subject amount of P8,233,880.30 out of the properties and unpaid subscription on subscribed Capital Stock of the
Board of Directors, Corporate Officers, Incorporators and Stockholders of said respondent corporations, plus the
amount of P3,000,000.00 and P1,000,000.00 in the form of moral and exemplary damages, respectively, as well
as 10% attorneys fees from any recoverable amounts.
RESPONDENT’S ARGUMENT: Private respondents FUJI filed a memorandum on appeal and a motion to
dispense with the posting of a cash or surety appeal bond on the ground that they were not the employers of
petitioners. They alleged that they could not be held responsible for petitioners claims and to require them to post
the bond would be unjust and unfair, and not sanctioned by law.
NLRC RULING: NLRC denied respondent’s motion to be exempted from posting appeal bond. Respondents are
hereby directed to post cash or surety bond within an unextendible period of ten (10) days upon receipt.
Otherwise the appeal shall be dismissed.
Petitioners filed a petition in the Court of Appeals imputing grave abuse of discretion to the NLRC, Third Division
when it allowed private respondents to post the mandated cash or surety bond four months after the filing of their
memorandum on appeal.
COURT OF APPEALS RULING: Court of Appeals dismissed the petition for lack of merit.
ISSUE: Whether or not the posting of a bond beyond the reglementary period to perfect appeal may be allowed
SC RULING:
YES. The provision of Article 223 (now 229) of the Labor Code requiring the posting of bond on appeals involving
monetary awards must be given liberal interpretation in line with the desired objective of resolving controversies
on the merits. If only to achieve substantial justice, strict observance of the reglementary periods may be relaxed if
warranted. The NLRC, Third Division could not be said to have abused its discretion in requiring the posting of
bond after it denied private respondents motion to be exempted therefrom.
It is true that the perfection of an appeal in the manner and within the period prescribed by law is not only
mandatory but jurisdictional, and failure to perfect an appeal has the effect of making the judgment final and
Angeles| Bajana | Balladares | Brillantes | Briones | Cabansag | Callanta | Chua | David|

De Leon | Gomez | Lopez | Macalino | Nostratis | Padilla | Reynon | Santos | Tan |Velasco
4E / 4F - 2018-2019
Page 343 of 920
LABOR REVIEW DIGEST
Atty. Joyrich Golangco

executory. However, technicality should not be allowed to stand in the way of equitably and completely resolving
the rights and obligations of the parties. We have allowed appeals from the decisions of the labor arbiter to the
NLRC, even if filed beyond the reglementary period, in the interest of justice. The facts and circumstances of the
instant case warrant liberality considering the amount involved and the fact that petitioners already obtained a
favorable judgment on February 23, 1993 against their employer UNIX.
It is only fair and just that respondent FUJI be afforded the opportunity to be heard on appeal before the
NLRC, specially in the light of labor arbiter Patis later decision holding FUJI jointly and severally liable with UNIX
in the payment of the monetary awards adjudged by labor arbiter de Vera against UNIX.

Angeles| Bajana | Balladares | Brillantes | Briones | Cabansag | Callanta | Chua | David|

De Leon | Gomez | Lopez | Macalino | Nostratis | Padilla | Reynon | Santos | Tan |Velasco
4E / 4F - 2018-2019
Page 344 of 920
LABOR REVIEW DIGEST
Atty. Joyrich Golangco

7. Case Title: Lepanto Consolidated Mining Corp. vs. Icao


GR No: 196047 Date: January 15, 2014
Ponente: Sereno, C.J.

Doctrine: Therefore, once the appeal is finally decided and no award needs to be satisfied, the bond is
automatically released. Since the money is now unencumbered, the employer who posted it should now have
unrestricted access to the cash which he may now use as he pleases as appeal bond in another case, for
instance.

FACTS: The instant petition stemmed from a complaint for illegal dismissal and damages filed by private
respondent Belio Icao against petitioners Lepanto Consolidated Mining Company (LCMC) and its Chief Executive
Officer Felipe Yap before the Arbitration Branch of the NLRC.

Private respondent essentially alleged in his complaint that he was an employee of petitioner LCMC assigned as a
lead miner in its underground mine in Paco, Mankayan, Benguet. Ica was charged and terminated for allegedly
having stolen gold bearing highgrade ores from the company premises. LA ruled that LCMC is liable for illegal
dismissal.

Petitioner and its CEO filed an Appearance with Memorandum of Appeal before the NLRC. Instead of posting the
required appeal bond in the form of a cash bond or a surety bond in an amount equivalent to the monetary award
of P345,879.45 adjudged in favor of Icao, they filed a Consolidated Motion For Release Of Cash Bond And To
Apply Bond Subject For Release As Payment For Appeal Bond (Consolidated Motion). They requested therein
that the NLRC release the cash bond of P401,610.84, which they had posted in the separate case Dangiw
Siggaao v. LCMC, and apply that same cash bond to their present appeal bond liability. They reasoned that since
this Court had already decided Dangiw Siggaao in their favor, and that the ruling therein had become final and
executory, the cash bond posted therein could now be released. They also cited financial difficulty as a reason for
resorting to this course of action.

LA/RTC/NLRC RULING: The labor arbiter rendered a Decision holding petitioner and its CEO liable for illegal
dismissal and ordering them to pay respondent Icao P345,879.45, representing his full backwages and separation
pay.

The NLRC dismissed the appeal of petitioner LCMC for failure to post the appeal bond. The rules are clear.
Appeals from decision involving a monetary award maybe [sic] perfected only upon posting of a cash or surety-
bond within the ten (10) day reglementary period for filing an appeal. Failure to file and post the required appeal
bond within the said period results in the appeal not being perfected and the appealed judgment becomes final
and executory.

CA RULING: CA issued its assailed Decision15 affirming the Order of the NLRC First Division, which had
dismissed the appeal of petitioner and the latter’s CEO. According to the CA, they failed to comply with the
requirements of law and consequently lost the right to appeal.

APPEAL TO THE SC:

Petitioner's Contention: See facts

Respondent's Contention: See facts

Angeles| Bajana | Balladares | Brillantes | Briones | Cabansag | Callanta | Chua | David|

De Leon | Gomez | Lopez | Macalino | Nostratis | Padilla | Reynon | Santos | Tan |Velasco
4E / 4F - 2018-2019
Page 345 of 920
LABOR REVIEW DIGEST
Atty. Joyrich Golangco

ISSUE/S: Whether or not petitioner complied with the appeal bond requirement under the Labor Code and the
NLRC Rules by filing a Consolidated Motion to release the cash bond it posted in another case

HELD: YES. While it is true that the procedure undertaken by petitioner is not provided under the Labor Code or
in the NLRC Rules, we answer the question in the affirmative. We reiterate our pronouncement in Araneta v.
Rodas, where the Court said that when the law does not clearly provide a rule or norm for the tribunal to follow in
deciding a question submitted, but leaves to the tribunal the discretion to determine the case in one way or
another, the judge must decide the question in conformity with justice, reason and equity, in view of the
circumstances of the case. Applying this doctrine, we rule that petitioner substantially complied with the
mandatory requirement of posting an appeal bond for the reasons explained below.

First, there is no question that the appeal was filed within the 10-day reglementary period. Except for the alleged
failure to post an appeal bond, the appeal to the NLRC was therefore in order.

Under the Rule VI, Section 6 of the 2005 NLRC Rules, "[a] cash or surety bond shall be valid and effective from
the date of deposit or posting, until the case is finally decided, resolved or terminated, or the award satisfied."
Hence, it is clear that a bond is encumbered and bound to a case only for as long as 1) the case has not been
finally decided, resolved or terminated; or 2) the award has not been satisfied. Therefore, once the appeal is
finally decided and no award needs to be satisfied, the bond is automatically released. Since the money is now
unencumbered, the employer who posted it should now have unrestricted access to the cash which he may now
use as he pleases as appeal bond in another case, for instance. This is what petitioner simply did.

Angeles| Bajana | Balladares | Brillantes | Briones | Cabansag | Callanta | Chua | David|

De Leon | Gomez | Lopez | Macalino | Nostratis | Padilla | Reynon | Santos | Tan |Velasco
4E / 4F - 2018-2019
Page 346 of 920
LABOR REVIEW DIGEST
Atty. Joyrich Golangco

8. Case Title: Bergonio vs. SEAIR (South East Asian Airlines)


GR No: 195227 Date: April 21, 2014

Ponente: Brion, J.

DOCTRINE: Payment of accrued wages despite reversal of decision: An employer, who, despite the Labor
Arbiter’s order of reinstatement, did not reinstate the employee during the pendency of the appeal up to the
reversal by a higher tribunal may still be held liable for the accrued wages of the employee, i.e., the unpaid salary
accruing up to the time the higher tribunal reverses the decision. The rule, therefore, is that an employee may still
recover the accrued wages up to and despite the reversal by the higher tribunal. This entitlement of the employee
to the accrued wages proceeds from the immediate and self-executory nature of the reinstatement aspect of the
LA’s decision.

Exception. By way of exception to the above rule, an employee may be barred from collecting the accrued wages
if shown that the delay in enforcing the reinstatement pending appeal was without fault on the part of the
employer. To determine whether an employee is thus barred, two tests must be satisfied: (1) actual delay or the
fact that the order of reinstatement pending appeal was not executed prior to its reversal; and (2) the delay must
not be due to the employer’s unjustified act or omission. Note that under the second test, the delay must be
without the employer’s fault. If the delay is due to the employer’s unjustified refusal, the employer may still be
required to pay the salaries notwithstanding the reversal of the LA’s decision.

FACTS: On April 30, 2004, a complaint before the Labor Arbiter (LA) for illegal dismissal and illegal suspension was
filed by the petitioners against respondents South East Asian Airlines (SEAIR) and Irene Dornier as SEAIR’s President. On May
31, 2005, the LA found the petitioners illegally dismissed and ordered the respondents to reinstate the petitioners
with full back wages. There was failure on the part of the Respondent to reinstate the said employees, prompting
the application for the Writ of Execution, the latter was subsequently granted by the court in favor of
the petitioners.
On February 21, 2006, respondents issued a Memorandum directing the petitioners to report on work before
February 24, 2006. However, petitioners failed to report to work alleging that only one of them received the
Memorandum on February 23, 2005. The Memorandum also instructed them to report to Clark, Pampanga
instead of NAIA-Domestic Airport in Pasay which according to the petitioners was a violation of Art. 223 (par.3) of
the Labor Code as it changes the terms and conditions of the petitioners’ employment prior to the dismissal.
On May 31, 2005, respondents appealed to the NLRC which was subsequently dismissed for failure to perfect an
appeal. Respondents then filed a petition for certiorari which was granted by the Court of Appeals (CA), the latter
ruling in favor of the respondents and declaring the dismissal
valid. The CA also ruled that the delay of the execution of the reinstatement order was not due to the respondent’s
unjustified act but to petitioners’ refusal to comply with the February 21, 2006 Memorandum of return-to-work. This led the
petitioners to file a petition for review in the Supreme Court.

LA/RTC/NLRC RULING: LA granted the petitioners' motion; it directed Metrobank-San Lorenzo to release the
P1,900,000.00 garnished amount. The LA found valid and meritorious the respondents' claim for accrued wages
in view of the respondents' refusal to reinstate the petitioners despite the final and executory nature of the
reinstatement aspect of its (LA's) May 31, 2005 decision. The LA noted that as of the December 18, 2007 CA
decision (that reversed the illegal dismissal findings of the LA), the petitioners' accrued wages amounted to
P3,078,366.33.

In its July 16, 2008 resolution, the NLRC affirmed in toto the LA's March 13, 2008 order. The NLRC afterwards
denied the respondents' motion for reconsideration for lack of merit.

Angeles| Bajana | Balladares | Brillantes | Briones | Cabansag | Callanta | Chua | David|

De Leon | Gomez | Lopez | Macalino | Nostratis | Padilla | Reynon | Santos | Tan |Velasco
4E / 4F - 2018-2019
Page 347 of 920
LABOR REVIEW DIGEST
Atty. Joyrich Golangco

CA RULING: The CA agreed that the reinstatement aspect of the LA's decision is immediately executory even
pending appeal, such that the employer is obliged to reinstate and pay the wages of the dismissed employee
during the period of appeal until the decision (finding the employee illegally dismissed including the reinstatement
order) is reversed by a higher court.

Petitioner’s Contention: petitioners argue that the computation of their accrued wages stopped when they failed
to report for work on February 24, 2006.
Petitioners point out that the February 21, 2006 Memorandum directed them to report for work at Clark Field,
Angeles, Pampanga instead of at the NAIA-Domestic Airport in Pasay City where they had been assigned.
Petitioners claim that the delay in their reinstatement was in fact due to the respondents' unjustified acts and that
the respondents never really complied with the LA's reinstatement order

Petitioner’s Contention: Respondents maintain that the petitioners were validly dismissed and that they
complied with the LA's reinstatement order when it directed the petitioners to report back to work, which directive
the petitioners did not heed.
Respondents point out that their relationship had been so strained that reinstatement was no longer possible.
Despite this strained relationship, the respondents point out that they still required the petitioners to report back to
work if only to comply with the LA's reinstatement order. Instead of reporting for work as directed, the petitioners,
however, insisted for a payroll reinstatement, which option the law grants to them (the respondents) as employer.
Also, contrary to the petitioners' claim, the Memorandum directed them to report at Clark Field, Pampanga only for
a re-orientation of their respective duties and responsibilities.
Respondents claim that the delay in the petitioners' reinstatement was in fact due to the latter's refusal to report
for work after the issuance of the February 21, 2006 Memorandum in addition to their strained relationship.

ISSUE: WON the respondent SEAIR complied with the LA decision on the reinstatement of employees which will
negate petitioners’ claim for accrued back wages?

HELD: No. As provided for in Art. 223(3) of the Labor Code, the reinstatement of anemployee found illegally
dismissed is immediately executory even during the pendency of the employer’s appeal from the decision. Under
this provision, the employer must reinstate the employee – either by physically admitting him under the conditions
prevailing prior to his dismissal, and paying his wages, or within the option of the employer to reinstate him in the
payroll. A dismissed employee whose case was favored by the LA is entitled to receive wages pending appeal
upon reinstatement, which reinstatement is immediately executory. Unless the appellate tribunal issues a
restraining order, the LA is duty bound to implement the order of reinstatement and the employer has no option
but to comply with it.
Moreover, and equally worth emphasizing, is that an order of reinstatement issued by the LA is self-
executory, i.e., the dismissed employee need not even apply for and the LA need not even issue a writ of
execution to trigger the employer's duty to reinstate the dismissed employee.

The employer is duty-bound to reinstate the employee, failing which, the employer is liable instead to pay the
dismissed employee's salary. The Court's consistent and prevailing treatment and interpretation of the
reinstatement order as immediately enforceable, in fact, merely underscores the right to security of tenure of
employees that the Constitution protects.

The employer is obliged to pay the dismissed employee's salary if he refuses to reinstate until actual
reinstatement or reversal by a higher tribunal; circumstances that may bar an employee from receiving
the accrued wages.

As we amply discussed above, an employer is obliged to immediately reinstate the employee upon the LA's
finding of illegal dismissal; if the employer fails, it is liable to pay the salary of the dismissed employee. Of course,
Angeles| Bajana | Balladares | Brillantes | Briones | Cabansag | Callanta | Chua | David|

De Leon | Gomez | Lopez | Macalino | Nostratis | Padilla | Reynon | Santos | Tan |Velasco
4E / 4F - 2018-2019
Page 348 of 920
LABOR REVIEW DIGEST
Atty. Joyrich Golangco

it is not always the case that the LA's finding of illegal dismissal is, on appeal by the employer, upheld by the
appellate court. After the LA's decision is reversed by a higher tribunal, the employer's duty to reinstate the
dismissed employee is effectively terminated. This means that an employer is no longer obliged to keep the
employee in the actual service or in the payroll. The employee, in turn, is not required to return the wages that he
had received prior to the reversal of the LA's decision.

Angeles| Bajana | Balladares | Brillantes | Briones | Cabansag | Callanta | Chua | David|

De Leon | Gomez | Lopez | Macalino | Nostratis | Padilla | Reynon | Santos | Tan |Velasco
4E / 4F - 2018-2019
Page 349 of 920
LABOR REVIEW DIGEST
Atty. Joyrich Golangco

9. LOON, et.al vs POWERMASTER, INC.


GR No: 189404 Date: December 11, 2013
Ponente: Brion, J.

Doctrine:
Revocation of the bonding company's authority has a prospective application.

FACTS:
Respondents Power Master, Inc. and Tri-C General Services employed and assigned the petitioners as janitors
and leadsmen in various Philippine Long Distance Telephone Company (PLDT) offices in Metro Manila area.
Subsequently, the petitioners filed a complaint for money claims against Power Master, Inc., Tri-C General
Services and their officers, the spouses Homer and Carina Alumisin (collectively, the respondents). The
petitioners alleged in their complaint that they were not paid minimum wages, overtime, holiday, premium, service
incentive leave, and thirteenth month pays. They further averred that the respondents made them sign blank
payroll sheets. On June 11, 2001, the petitioners amended their complaint and included illegal dismissal as their
cause of action. They claimed that the respondents relieved them from service in retaliation for the filing of their
original complaint.
Notably, the respondents did not participate in the proceedings before the Labor Arbiter except on April 19, 2001
and May 21, 2001 when Mr. Romulo Pacia, Jr. appeared on the respondents’ behalf. The respondents’
counsel also appeared in a preliminary mandatory conference on July 5, 2001. However, the respondents
neither filed any position paper nor proffered pieces of evidence in their defense despite their knowledge of the
pendency of the case.
LA RULING:
The LA awarded the petitioners salary differential, service incentive leave, and thirteenth month pays. In
awarding these claims, the LA stated that the burden of proving the payment of these money claims rests with the
employer. The LA also awarded attorney’s fees in favor of the petitioners, pursuant to Article 111 of the Labor
Code. However, the LA denied the petitioners’ claims for backwages, overtime, holiday, and premium pays.
The LA observed that the petitioners failed to show that they rendered overtime work and worked on holidays and
rest days without compensation. The LA further concluded that the petitioners cannot be declared to have been
dismissed from employment because they did not show any notice of termination of employment. They were also
not barred from entering the respondents’ premises.

NLRC RULING:
The NLRC affirmed the LA’s awards of holiday pay and attorney’s fees. It also maintained that the LA acquired
jurisdiction over the persons of the respondents through their voluntary appearance.
However, it allowed the respondents to submit pieces of evidence for the first time on appeal on the
ground that they had been deprived of due process. It found that the respondents did not actually receive the
LA’s processes. It also admitted the respondents’ unverified supplemental appeal on the ground that technicalities
may be disregarded to serve the greater interest of substantial due process. Furthermore, the Rules of Court do
not require the verification of a supplemental pleading.
The NLRC also vacated the LA’s awards of salary differential, thirteenth month and service incentive leave
pays. In so ruling, it gave weight to the pieces of evidence attached to the memorandum on appeal and the
supplemental appeal. It maintained that the absence of the petitioners’ signatures in the payrolls was not an
indispensable factor for their authenticity. It pointed out that the payment of money claims was further evidenced
by the list of employees with ATM cards. It also found that the petitioners’ signatures were not forged. It took
judicial notice that many people use at least two or more different signatures.
The NLRC further ruled that the petitioners were lawfully dismissed on grounds of serious misconduct and
willful disobedience. It found that the petitioners failed to comply with various memoranda directing them to
transfer to other workplaces and to attend training seminars for the intended reorganization and reshuffling.
Angeles| Bajana | Balladares | Brillantes | Briones | Cabansag | Callanta | Chua | David|

De Leon | Gomez | Lopez | Macalino | Nostratis | Padilla | Reynon | Santos | Tan |Velasco
4E / 4F - 2018-2019
Page 350 of 920
LABOR REVIEW DIGEST
Atty. Joyrich Golangco

CA RULING:
CA affirmed the NLRC’s ruling

APPEAL TO THE SC
Petitioner’s contention
Respondents posted a bond from a surety that was not accredited by this Court and by the NLRC. In effect, the
respondents failed to perfect their appeal before the NLRC. They further insist that the NLRC should not have
admitted the respondents’ unverified supplemental appeal.

Respondent’s contention
Petitioners only raised the issue of the validity of the appeal bond for the first time on appeal

ISSUE/S:
1. Whether the respondents perfected their appeal before the NLRC despite revocation of the bonding company’s
authority
2. Whether the NLRC properly allowed the respondents’ supplemental appeal
HELD:
1.YES
The respondents perfected their appeal with the NLRC because the revocation of the bonding company's
authority has a prospective application.
Paragraph 2, Article 223 of the Labor Code provides that "[i]n case of a judgment involving a monetary award, an
appeal by the employer may be perfected only upon the posting of a cash or surety bond issued by a reputable
bonding company duly accredited by the Commission in the amount equivalent to the monetary award in the
judgment appealed from."
Contrary to the respondents’ claim, the issue of the appeal bond’s validity may be raised for the first time on
appeal since its proper filing is a jurisdictional requirement. The requirement that the appeal bond should be
issued by an accredited bonding company is mandatory and jurisdictional. The rationale of requiring an appeal
bond is to discourage the employers from using an appeal to delay or evade the employees' just and lawful
claims. It is intended to assure the workers that they will receive the money judgment in their favor upon the
dismissal of the employer’s appeal.
In the present case, the respondents filed a surety bond issued by Security Pacific Assurance
Corporation (Security Pacific) on June 28, 2002. At that time, Security Pacific was still an accredited bonding
company. However, the NLRC revoked its accreditation on February 16, 2003. Nonetheless, this subsequent
revocation should not prejudice the respondents who relied on its then subsisting accreditation in good faith.
In Del Rosario v. Philippine Journalists, Inc., we ruled that a bonding company’s revocation of authority is
prospective in application.
However, the respondents should post a new bond issued by an accredited bonding company in compliance with
paragraph 4, Section 6, Rule 6 of the NLRC Rules of Procedure. This provision states that "[a] cash or surety
bond shall be valid and effective from the date of deposit or posting, until the case is finally decided, resolved
or terminated or the award satisfied."
2.YES
The CA correctly ruled that the NLRC properly gave due course to the respondents’ supplemental appeal.
The CA also correctly ruled that the NLRC properly gave due course to the respondents’ supplemental appeal.
Neither the laws nor the rules require the verification of the supplemental appeal. Furthermore, verification is a
formal, not a jurisdictional, requirement. It is mainly intended for the assurance that the matters alleged in the
pleading are true and correct and not of mere speculation. Also, a supplemental appeal is merely an addendum to
the verified memorandum on appeal that was earlier filed in the present case; hence, the requirement for
verification has substantially been complied with.

Angeles| Bajana | Balladares | Brillantes | Briones | Cabansag | Callanta | Chua | David|

De Leon | Gomez | Lopez | Macalino | Nostratis | Padilla | Reynon | Santos | Tan |Velasco
4E / 4F - 2018-2019
Page 351 of 920
LABOR REVIEW DIGEST
Atty. Joyrich Golangco

The respondents also timely filed their supplemental appeal on January 3, 2003. The records of the case show
that the petitioners themselves agreed that the pleading shall be filed until December 18, 2002. The NLRC further
extended the filing of the supplemental pleading until January 3, 2003 upon the respondents’ motion for extension.
Other Notes/ SC Pronouncements:
__________________________________________________________________________

10. Mcburnie v. Ganzon


G.R. Nos. 178034 & 178117, G R. Nos. 186984-85, October 17, 2013
Reyes, J.

Doctrine:

Furthermore, on the matter of the filing and acceptance of motions to reduce appeal bond, as provided in Section
6, Rule VI of the 2011 NLRC Rules of Procedure, the Court hereby RESOLVES that henceforth, the following
guidelines shall be observed:

(a) The filing of a motion to reduce appeal bond shall be entertained by the NLRC subject to the following
conditions: (1) there is meritorious ground; and (2) a bond in a reasonable amount is posted;
(b) For purposes of compliance with condition no. (2), a motion shall be accompanied by the posting of a
provisional cash or surety bond equivalent to ten percent (10,) of the monetary award subject of the appeal,
exclusive of damages and attorney's fees;
(c) Compliance with the foregoing conditions shall suffice to suspend the running of the 1 0-day reglementary
period to perfect an appeal from the labor arbiter's decision to the NLRC;
(d) The NLRC retains its authority and duty to resolve the motion to reduce bond and determine the final amount
of bond that shall be posted by the appellant, still in accordance with the standards of meritorious grounds and
reasonable amount; and
(e) In the event that the NLRC denies the motion to reduce bond, or requires a bond that exceeds the amount of
the provisional bond, the appellant shall be given a fresh period of ten 1 0) days from notice of the NLRC order
within which to perfect the appeal by posting the required appeal bond.

Facts:

McBurnie instituted a complaint for illegal dismissal and other monetary claims against the respondents. McBurnie
claimed that on May 11, 1999, he signed a five-year employment agreement with the company EGI as an
Executive Vice-President. He performed work for the company until sometime in November 1999, when he
figured in an accident that compelled him to go back to Australia. While in Australia, he was informed by
respondent Ganzon that his services were no longer needed.

The respondents opposed the complaint, contending that their agreement with McBurnie was to jointly invest in
and establish a company for the management of hotels. They did not intend to create an employer-employee
relationship.

LA and NLRC Ruling:

The LA declared McBurnie as having been illegally dismissed from employment, and thus entitled to receive from
the respondents the following amounts: (a) US$985,162.00 as salary and benefits for the unexpired term of their
Angeles| Bajana | Balladares | Brillantes | Briones | Cabansag | Callanta | Chua | David|

De Leon | Gomez | Lopez | Macalino | Nostratis | Padilla | Reynon | Santos | Tan |Velasco
4E / 4F - 2018-2019
Page 352 of 920
LABOR REVIEW DIGEST
Atty. Joyrich Golangco

employment contract, (b) ₱2,000,000.00 as moral and exemplary damages, and (c) attorney’s fees equivalent to
10% of the total monetary award. Respondents appealed the LA’s Decision to the NLRC.

On November 5, 2004, they filed their Memorandum of Appeal and Motion to Reduce Bond, and posted an appeal
bond in the amount of ₱100,000.00. The respondents contended in their Motion to Reduce Bond, inter alia, that
the monetary awards of the LA were null and excessive, allegedly with the intention of rendering them incapable
of posting the necessary appeal bond.

On March 31, 2005, the NLRC denied the motion to reduce bond, explaining that "in cases involving monetary
award, an employer seeking to appeal the [LA’s] decision to the Commission is unconditionally required by Art.
223, Labor Code to post bond in the amount equivalent to the monetary award x x x." Thus, the NLRC required
from the respondents the posting of an additional bond in the amount of ₱54,083,910.00.

When their motion for reconsideration was denied, the respondents decided to elevate the matter to the Court of
Appeals (CA) via the Petition for Certiorari and Prohibition.

In the meantime, in view of the respondents’ failure to post the required additional bond, the NLRC dismissed their
appeal. The respondents’ motion for reconsideration was denied. This prompted the respondents to file with the
CA the Petition for Certiorari.

CA Ruling:

The CA rendered its Decision allowing the respondents’ motion to reduce appeal bond and directing the NLRC to
give due course to their appeal

On the issue of the NLRC’s denial of the respondents’ motion to reduce appeal bond, the CA ruled that the NLRC
committed grave abuse of discretion in immediately denying the motion without fixing an appeal bond in an
amount that was reasonable, as it denied the respondents of their right to appeal from the decision of the LA.

On the issue of the NLRC’s dismissal of the appeal on the ground of the respondents’ failure to post the additional
appeal bond, the CA also found grave abuse of discretion on the part of the NLRC, explaining that an appeal bond
in the amount of ₱54,083,910.00 was prohibitive and excessive.

McBurnie filed a motion for reconsideration. At the same time, the respondents moved that the appeal be resolved
on the merits by the CA. The CA issued a Resolution denied both motions. McBurnie then filed with the Court the
Petition for Review on Certiorari.

In the meantime, the NLRC, acting on the CA’s order of remand, accepted the appeal from the LA’s decision. It
reversed and set aside the Decision of the LA, and entered a new one dismissing McBurnie’s complaint. Hence,
McBurnie went to the SC.

SC Ruling:

The SC explained that the respondents’ failure to post a bond equivalent in amount to the LA’s monetary award
was fatal to the appeal. Although an appeal bond may be reduced upon motion by an employer, the following
conditions must first be satisfied: (1) the motion to reduce bond shall be based on meritorious grounds; and (2) a
reasonable amount in relation to the monetary award is posted by the appellant. Unless the NLRC grants the
motion to reduce the cash bond within the 10-day reglementary period to perfect an appeal from a judgment of the
LA, the employer is mandated to post the cash or surety bond securing the full amount within the said 10-day

Angeles| Bajana | Balladares | Brillantes | Briones | Cabansag | Callanta | Chua | David|

De Leon | Gomez | Lopez | Macalino | Nostratis | Padilla | Reynon | Santos | Tan |Velasco
4E / 4F - 2018-2019
Page 353 of 920
LABOR REVIEW DIGEST
Atty. Joyrich Golangco

period. The respondents’ initial appeal bond of ₱100,000.00 was grossly inadequate compared to the LA’s
monetary award.

The Court’s Decision became final and executory. On March 27, 2012, the respondents filed a Motion for Leave to
File Attached Third Motion for Reconsideration, with an attached Motion for Reconsideration with Motion to Refer
These Cases to the Honorable Court En Banc.

Issue:

 WON the respondent is entitled to a reduction of appeal bond.

Held:

 To clarify, the prevailing jurisprudence on the matter provides that the filing of a motion to reduce bond, coupled
with compliance with the two conditions emphasized in Garcia v. KJ Commercial78 for the grant of such motion,
namely, (1) a meritorious ground, and (2) posting of a bond in a reasonable amount, shall suffice to suspend the
running of the period to perfect an appeal from the labor arbiter’s decision to the NLRC. To require the full amount
of the bond within the 10-day reglementary period would only render nugatory the legal provisions which allow an
appellant to seek a reduction of the bond.

The NLRC has full discretion to grant or deny the motion to reduce bond, and it may rule on the motion beyond
the 10-day period within which to perfect an appeal. Obviously, at the time of the filing of the motion to reduce
bond and posting of a bond in a reasonable amount, there is no assurance whether the appellant’s motion is
indeed based on "meritorious ground" and whether the bond he or she posted is of a "reasonable amount." Thus,
the appellant always runs the risk of failing to perfect an appeal.

In order to give full effect to the provisions on motion to reduce bond, the appellant must be allowed to wait for the
ruling of the NLRC on the motion even beyond the 10-day period to perfect an appeal. If the NLRC grants the
motion and rules that there is indeed meritorious ground and that the amount of the bond posted is reasonable,
then the appeal is perfected. If the NLRC denies the motion, the appellant may still file a motion for
reconsideration as provided under Section 15, Rule VII of the Rules. If the NLRC grants the motion for
reconsideration and rules that there is indeed meritorious ground and that the amount of the bond posted is
reasonable, then the appeal is perfected. If the NLRC denies the motion, then the decision of the labor arbiter
becomes final and executory.

 As the Court, nonetheless, remains firm on the importance of appeal bonds in appeals from monetary awards of
LAs, we stress that the NLRC, pursuant to Section 6, Rule VI of the NLRC Rules of Procedure, shall only accept
motions to reduce bond that are coupled with the posting of a bond in a reasonable amount. Time and again, we
have explained that the bond requirement imposed upon appellants in labor cases is intended to ensure the
satisfaction of awards that are made in favor of appellees, in the event that their claims are eventually sustained
by the courts. On the part of the appellants, its posting may also signify their good faith and willingness to
recognize the final outcome of their appeal.

It is in this light that the Court finds it necessary to set a parameter for the litigants’ and the NLRC’s guidance on
the amount of bond that shall hereafter be filed with a motion for a bond’s reduction. To ensure that the provisions
of Section 6, Rule VI of the NLRC Rules of Procedure that give parties the chance to seek a reduction of the
appeal bond are effectively carried out, without however defeating the benefits of the bond requirement in favor of
a winning litigant, all motions to reduce bond that are to be filed with the NLRC shall be accompanied by the
posting of a cash or surety bond equivalent to 10% of the monetary award that is subject of the appeal, which
Angeles| Bajana | Balladares | Brillantes | Briones | Cabansag | Callanta | Chua | David|

De Leon | Gomez | Lopez | Macalino | Nostratis | Padilla | Reynon | Santos | Tan |Velasco
4E / 4F - 2018-2019
Page 354 of 920
LABOR REVIEW DIGEST
Atty. Joyrich Golangco

shall provisionally be deemed the reasonable amount of the bond in the meantime that an appellant’s motion is
pending resolution by the Commission. In conformity with the NLRC Rules, the monetary award, for the purpose
of computing the necessary appeal bond, shall exclude damages and attorney’s fees. Only after the posting of a
bond in the required percentage shall an appellant’s period to perfect an appeal under the NLRC Rules be
deemed suspended.

The foregoing shall not be misconstrued to unduly hinder the NLRC’s exercise of its discretion, given that the
percentage of bond that is set by this guideline shall be merely provisional. The NLRC retains its authority and
duty to resolve the motion and determine the final amount of bond that shall be posted by the appellant, still in
accordance with the standards of "meritorious grounds" and "reasonable amount". Should the NLRC, after
considering the motion’s merit, determine that a greater amount or the full amount of the bond needs to be posted
by the appellant, then the party shall comply accordingly. The appellant shall be given a period of 10 days from
notice of the NLRC order within which to perfect the appeal by posting the required appeal bond.

 The requirement on the existence of a "meritorious ground" delves on the worth of the parties’ arguments, taking
into account their respective rights and the circumstances that attend the case. The condition was emphasized in
University Plans Incorporated v. Solano,95 wherein the Court held that while the NLRC’s Revised Rules of
Procedure "allows the [NLRC] to reduce the amount of the bond, the exercise of the authority is not a matter of
right on the part of the movant, but lies within the sound discretion of the NLRC upon a showing of meritorious
grounds."96 By jurisprudence, the merit referred to may pertain to an appellant’s lack of financial capability to pay
the full amount of the bond,the merits of the main appeal such as when there is a valid claim that there was no
illegal dismissal to justify the award,98 the absence of an employer-employee relationship, prescription of claims,
and other similarly valid issues that are raised in the appeal. For the purpose of determining a "meritorious
ground", the NLRC is not precluded from receiving evidence, or from making a preliminary determination of the
merits of the appellant’s contentions.

 As regards the requirement on the posting of a bond in a "reasonable amount," the Court holds that the final
determination thereof by the NLRC shall be based primarily on the merits of the motion and the main appeal.

Given the circumstances in this case and the merits of the respondents’ arguments before the NLRC, the Court
holds that the respondents had posted a bond in a "reasonable amount", and had thus complied with the
requirements for the perfection of an appeal from the LA’s decision.

 Furthermore, on the matter of the filing and acceptance of motions to reduce appeal bond, as provided in Section
6, Rule VI of the 2011 NLRC Rules of Procedure, the Court hereby RESOLVES that henceforth, the following
guidelines shall be observed:

(a) The filing of a motion to reduce appeal bond shall be entertained by the NLRC subject to the following
conditions: (1) there is meritorious ground; and (2) a bond in a reasonable amount is posted;
(b) For purposes of compliance with condition no. (2), a motion shall be accompanied by the posting of a
provisional cash or surety bond equivalent to ten percent (10,) of the monetary award subject of the appeal,
exclusive of damages and attorney's fees;
(c) Compliance with the foregoing conditions shall suffice to suspend the running of the 1 0-day reglementary
period to perfect an appeal from the labor arbiter's decision to the NLRC;
(d) The NLRC retains its authority and duty to resolve the motion to reduce bond and determine the final amount
of bond that shall be posted by the appellant, still in accordance with the standards of meritorious grounds and
reasonable amount; and

Angeles| Bajana | Balladares | Brillantes | Briones | Cabansag | Callanta | Chua | David|

De Leon | Gomez | Lopez | Macalino | Nostratis | Padilla | Reynon | Santos | Tan |Velasco
4E / 4F - 2018-2019
Page 355 of 920
LABOR REVIEW DIGEST
Atty. Joyrich Golangco

(e) In the event that the NLRC denies the motion to reduce bond, or requires a bond that exceeds the amount of
the provisional bond, the appellant shall be given a fresh period of ten 1 0) days from notice of the NLRC order
within which to perfect the appeal by posting the required appeal bond.

Angeles| Bajana | Balladares | Brillantes | Briones | Cabansag | Callanta | Chua | David|

De Leon | Gomez | Lopez | Macalino | Nostratis | Padilla | Reynon | Santos | Tan |Velasco
4E / 4F - 2018-2019
Page 356 of 920
LABOR REVIEW DIGEST
Atty. Joyrich Golangco

11. WATERFRONT CEBU CITY CASINO HOTEL, INC. AND MARCO PROTACIO vs. ILDEBRANDO
LEDESMA
G.R. No. 197556; March 25, 2015
VILLARAMA, JR., J.

DOCTRINE: When a party to a suit appears by counsel, service of every judgment and all orders of the court
must be sent to the counsel. This is so because notice to counsel is an effective notice to the client, while notice to
the client and not his counsel is not notice in law. Receipt of notice by the counsel of record is the reckoning point
of the reglementary period.

FACTS:

Respondent was employed as a House Detective at Waterfront located at Salinas Drive, Cebu City. On the basis
of the complaints filed before Waterfront by Christe6 Mandal, a supplier of a concessionaire of Waterfront, and
Rosanna Lofranco, who was seeking a job at the same hotel, Ledesma was dismissed from employment. From
the affidavits and testimonies of Christe Mandal and Rosanna Lofranco during the administrative hearings
conducted by Waterfront, the latter found, among others, that Ledesma kissed and mashed the breasts of Christe
Mandal inside the hotel’s elevator, and exhibited his penis and asked Rosanna Lofranco to masturbate him at the
conference room of the hotel. On August 12, 2008, Ledesma filed a complaint for illegal dismissal.

LA: The LA found that the allegations leveled against Ledesma are mere concoctions, and concluded that
Ledesma was illegally dismissed.

NLRC: Reversed the ruling of the LA and held that Ledesma’s acts of sexual overtures to Christe Mandal and
Rosanna Lofranco constituted grave misconduct justifying his dismissal from employment.

Ledesma, now assisted by a new counsel, filed a motion for leave to file amended petition, and sought the
admission of his Amended Petition for Certiorari. Ledesma contended that his receipt on March 24, 2010 (and not
the receipt on March 15, 2010 by Atty. Abellana), is the reckoning date of the 60-day reglementary period within
which to file the petition. Hence, Ledesma claims that the petition was timely filed on May 17, 2010.

ISSUE:
Whether or not the petition to file an amended petition for certiorari was filed out of time.

RULING:

The unjustified failure of Ledesma to file his petition for certiorari before the CA within the 60-day period is a
ground for the outright dismissal of said petition. Ledesma erroneously asserted in his petition for certiorari filed
before the CA, that the 60th day is May 15, 2010, counted from March 15, 2010. In computing a period, the first
day shall be excluded, and the last included; hence, the last day to file his petition for certiorari is on May 14,
2010, a Friday. Ledesma therefore belatedly filed his petition on May 17, 2010.

When a party to a suit appears by counsel, service of every judgment and all orders of the court must be sent to
the counsel. This is so because notice to counsel is an effective notice to the client, while notice to the client and
not his counsel is not notice in law. Receipt of notice by the counsel of record is the reckoning point of the
reglementary period.

Angeles| Bajana | Balladares | Brillantes | Briones | Cabansag | Callanta | Chua | David|

De Leon | Gomez | Lopez | Macalino | Nostratis | Padilla | Reynon | Santos | Tan |Velasco
4E / 4F - 2018-2019
Page 357 of 920
LABOR REVIEW DIGEST
Atty. Joyrich Golangco

12.IBM Nestle v. Nestle Phils.


GR No: 198675 Date: September 23, 2015
Ponente: Peralta, J.

Doctrine: Compromise Agreements having been sanctioned by the court, it is entered as a determination of a
controversy and has the force and effect of a judgment. It is immediately executory and not appealable, except for
vices of consent or forgery.

FACTS:
Petitioner union staged a strike against herein respondent company's Ice Cream and Chilled Products Division,
citing, as grounds, respondent's alleged violation of the collective bargaining agreement (CBA), dismissal of union
officers and members, discrimination and other unfair labor practice (ULP) acts.
As a consequence, respondent filed with the National Labor Relations Commission (NLRC) a Petition for
Injunction with Prayer for Issuance of Temporary Restraining Order, Free Ingress and Egress Order, and
Deputization Order which was granted. Respondent filed a Petition to Declare Strike Illegal.
Department of Labor and Employment (DOLE) Acting Secretary, issued an Order assuming jurisdiction
over the strike and certifying the same to the NLRC.
Petitioner union filed a petition for certiorari with this Court, questioning the above order of the Acting
DOLE Secretary.
However, after a series of conciliation meetings and discussions between the parties, they agreed to
resolve their differences and came up with a compromise which was embodied in a Memorandum of Agreement
(MOA) dated August 4, 1998.
On August 6, 1998, the parties filed a Joint Motion to Dismiss stating that they are no longer interested in
pursuing the petition for injunction filed by respondent as a consequence of the settlement of their dispute.
On October 12, 1998, the NLRC issued its Decision approving the parties' compromise agreement and
granting their Joint Motion to Dismiss.
On January 25, 2010, or after a lapse of more than eleven (11) years from the time of execution of the
subject MO A, petitioners filed with the NLRC a Motion for Writ of Execution contending that they have not been
paid the amounts they are entitled to in accordance with the MOA.
Respondent filed its Opposition to the Motion for Writ of Execution contending that petitioners' remedy is
already barred by prescription because, under the 2005 Revised Rules of the NLRC, a decision or order may be
executed on motion within five (5) years from the date it becomes final and executory and that the same decision
or order may only be enforced by independent action within a period often (10) years from the date of its finality.

NLRC RULING: NLRC promulgated its Resolution denying petitioners' application for the issuance of a writ of
execution on the ground of prescription.

CA RULING:CA issued the first of its questioned Resolutions dismissing petitioners' certiorari petition on the
ground that it is a wrong mode of appeal. The CA held that petitioners' appeal involves a pure question of law
which should have been taken directly to this Court via a petition for review on certiorari under Rule 45 of the
Rules of Court.

PETITION TO THE SC:


Petitioners' basic contention is that respondent cannot invoke the defense of prescription because it is guilty of
deliberately causing delay in paying petitioners' claims and that petitioners, on the other hand, are entitled to
protection under the law because they had been vigilant in exercising their right as provided for under the subject
MOA.

ISSUE/S:
1. whether or not petitioners' claim for payment is barred by prescription
Angeles| Bajana | Balladares | Brillantes | Briones | Cabansag | Callanta | Chua | David|

De Leon | Gomez | Lopez | Macalino | Nostratis | Padilla | Reynon | Santos | Tan |Velasco
4E / 4F - 2018-2019
Page 358 of 920
LABOR REVIEW DIGEST
Atty. Joyrich Golangco

HELD:
There is no dispute that the compromise agreement between herein petitioner union, representing its
officers and members, and respondent company was executed on August 4, 1998 and was subsequently
approved via the NLRC Decision dated October 12, 1998. However, considering petitioners' allegation that the
terms and conditions of the agreement have not been complied with by respondent, petitioners should have
moved for the issuance of a writ of execution.
Having been sanctioned by the court, it is entered as a determination of a controversy and has the force
and effect of a judgment. It is immediately executory and not appealable, except for vices of consent or forgery.
The non-fulfillment of its terms and conditions justifies the issuance of a writ of execution; in such an instance,
execution becomes a ministerial duty of the court. Stated differently, a decision on a compromise agreement is
final and executory. Such agreement has the force of law and is conclusive between the parties. It transcends its
identity as a mere contract binding only upon the parties thereto, as it becomes a judgment that is subject to
execution in accordance with the Rules.

The most relevant rule in the instant case is Section 8, Rule XI, 2005 Revised Rules of Procedure of the NLRC
which states that:cralawlawlibrary

Section 8. Execution By Motion or By Independent Action. - A decision or order may be executed on motion within
five (5) years from the date it becomes final and executory. After the lapse of such period, the judgment shall
become dormant, and may only be enforced by an independent action within a period of ten (10) years from date
of its finality.
chanrobleslaw

In the same manner, pertinent portions of Sections 4 (a) and 6, Rule III, of the NLRC Manual on Execution of
Judgment, provide as follows:cralawlawlibrary

Section 4. Issuance of a Writ: - Execution shall issue upon an order, resolution or decision that finally disposes of
the actions or proceedings and after the counsel of record and the parties have been duly furnished with the
copies of the same in accordance with the NLRC Rules of Procedure, provided:cralawlawlibrary

a) The Commission or Labor Arbiter shall, motu proprio or upon motion of any interested party, issue a writ of
execution on a judgment only within five (5) years from the date it becomes final and executory, x x x

xxx xxx xxx


Section 6. Execution by Independent Action. - A judgment after the lapse of five (5) years from the date it
becomes final and executory and before it is barred by prescription, may only be enforced by an independent
action.c

13. MANILA MINING CORPORATION v. LOWITO AMOR

DOCTRINE: Although appeal is an essential part of our judicial process, it has been held, time and again, that the
right thereto is not a natural right or a part of due process but is merely a statutory privilege. Thus, the perfection
of an appeal in the manner and within the period prescribed by law is not only mandatory but also jurisdictional
and failure of a party to conform to the rules regarding appeal will render the judgment final and executory.

FACTS: Respondents Lowito Amor, Rollybie Ceredon, Julius Cesar, Ronito Martinez and Fermin Tabili, Jr. were
regular employees of petitioner Manila Mining Corporation. When the mine tailings being pumped into TP No. 7
reached the maximum level in December 2000, petitioner temporarily shut down its mining operations pending
approval of its application to increase said facilty’s capacity by the Department of Environment and Natural
Resources-Environment Management Bureau (DENR-EMB).
Angeles| Bajana | Balladares | Brillantes | Briones | Cabansag | Callanta | Chua | David|

De Leon | Gomez | Lopez | Macalino | Nostratis | Padilla | Reynon | Santos | Tan |Velasco
4E / 4F - 2018-2019
Page 359 of 920
LABOR REVIEW DIGEST
Atty. Joyrich Golangco

On 27 July 2001, petitioner served a notice, informing its employees and the Department of Labor and
Employment Regional Office No. XII (DOLE) of the temporary suspension of its operations for six months and the
temporary lay-off of two-thirds of its employees. After the lapse of said period, petitioner notified the DOLE on 11
December 2001 that it was extending the temporary shutdown of its operations for another six months.
Respondents filed the complaint for constructive dismissal and monetary claims. Executive Labor Arbiter
Benjamin E. Pelaez rendered a Decision holding petitioner liable for constructive dismissal in view of the
suspension of its operations beyond the six-month period.
Aggrieved, petitioner filed its memorandum of appeal before the NLRC and moved for the reduction of the appeal
bond to ₱100,000.00, on the ground that its financial losses in the preceding years had rendered it unable to put
up one in cash and/or surety equivalent to the monetary award. In opposition, respondents moved for the
dismissal of the appeal in view of the fact that, despite receipt of the appealed decision on 24 November 2004,
petitioner mailed their copy of the memorandum of appeal only on 7 February 2005. NLRC ruled that, under
Article 283 of the Labor Code, respondents were not even entitled to separation pay considering the eventual
closure of their employer’s business due to serious business losses or financial reverses. CA ruled that petitioner
failed to perfect its appeal therefrom considering that the copy of its 3 December 2004 Memorandum of Appeal
intended for respondents was served the latter by registered mail only on 7 February 2005. Aside from posting an
unusually smaller sum as appeal bond, petitioner was likewise faulted for replenishing the check it issued only on
1 April 2005 or 24 days before the rendition of the assailed NLRC Decision.

ISSUE:Whether the petitioners’ appeal filed with the NLRC was fatally defective.

HELD: Insofar as appeals from decisions of the Labor Arbiter are concerned, Article 223 of the Labor Code of the
Philippines provides that, "decisions, awards, or orders of the Labor Arbiter are final and executory unless
appealed to the [NLRC] by any or both parties within ten (10) calendar days from the receipt of such decisions,
awards or orders." In case of a judgment involving a monetary award, the same provision mandates that, "an
appeal by the employer may be perfected only upon the posting of a cash or surety bond issued by a reputable
bonding company duly accredited by the [NLRC] in the amount equivalent to the monetary award in the judgment
appealed from." Alongside the requirement that "the appellant shall furnish a copy of the memorandum of appeal
to the other party."
Having received the Labor Arbiter’s Decision on 24 November 2004, petitioner had ten (10) calendar
days or until 4 December 2004 within which to perfect an appeal. Considering that the latter date fell on a
Saturday, petitioner had until the next working day, 6 December 2004, within which to comply with the
requirements for the perfection of its appeal. Our perusal of the record shows that, despite bearing the date 3
December 2004, petitioner’s memorandum of appeal was subscribed before Notary Public Ronald Rex Recidoro
only on 6 December 2004. Without proof as to the actual date of filing of said pleading being presented by both
parties, the CA discounted the timeliness of its filing in light of the established fact that the copy thereof intended
for respondents was only served by registered mail on 7 February 2005.
As allegation is not evidence, however, the rule is settled that the burden of evidence lies with the party
who asserts the affirmative of an issue. As the parties claiming the non-perfection of petitioner’s appeal, it was,
therefore, respondents who had the burden of proving that said memorandum of appeal was, indeed, filed out of
time. By and of itself, the fact that the copy of memorandum of appeal intended for respondents was served upon
them by registered mail only on 7 February 2005 does not necessarily mean that petitioner’s appeal from the
Labor Arbiter’s decision was filed out of time. On the principle that justice should not be sacrificed for technicality,
it has been ruled that the failure of a party to serve a copy of the memorandum to the opposing party is not a
jurisdictional defect and does not bar the NLRC from entertaining the appeal. Considering that such an omission
is merely regarded as a formal lapse or an excusable neglect, the CA reversibly erred in ruling that, under the
circumstances, petitioner could not have filed its appeal earlier than 7 February 2005.
The question regarding the appeal bond rises from the record which shows that, in addition to its
memorandum of appeal, petitioner filed a 6 December 2004 motion for the reduction of the appeal bond on the
Angeles| Bajana | Balladares | Brillantes | Briones | Cabansag | Callanta | Chua | David|

De Leon | Gomez | Lopez | Macalino | Nostratis | Padilla | Reynon | Santos | Tan |Velasco
4E / 4F - 2018-2019
Page 360 of 920
LABOR REVIEW DIGEST
Atty. Joyrich Golangco

ground that the cash equivalent of the monetary award and/or cost of the surety bond have proven to be
prohibitive in view of the tremendous business losses it allegedly sustained. As supposed measure of its good
faith in complying with the Rules, petitioner attached to its motion Philam Bank Check No. 0000627153, dated 6
December2004, in the amount of ₱100,000.00 only. Respondent correctly called attention to the fact that the
check submitted by petitioner was dishonored upon presentment for payment, thereby rendering the tender
thereof ineffectual.

Angeles| Bajana | Balladares | Brillantes | Briones | Cabansag | Callanta | Chua | David|

De Leon | Gomez | Lopez | Macalino | Nostratis | Padilla | Reynon | Santos | Tan |Velasco
4E / 4F - 2018-2019
Page 361 of 920
LABOR REVIEW DIGEST
Atty. Joyrich Golangco

14. ILAW BUKLOD NG MANGGAGAWA NG NESTLE PHILS. vs. NESTLE PHILS.


GR No: 198675 Date: September 23, 2015
Ponente: Peralta, J.

Doctrine: A judgment may be executed on motion within five years from the date of its entry or from the date it
becomes final and executory. After the lapse of such time, and before it is barred by the statute of limitations, a
judgment may be enforced by action. If the prevailing party fails to have the decision enforced by a mere motion
after the lapse of five years from the date of its entry (or from the date it becomes final and executory), the said
judgment is reduced to a mere right of action in favor of the person whom it favors and must be enforced, as are
all ordinary actions, by the institution of a complaint in a regular form.
FACTS: Petitioner union staged a strike against herein respondent company's Ice Cream and Chilled Products
Division, citing, as grounds, respondent's alleged violation of the collective bargaining agreement (CBA), dismissal
of union officers and members, discrimination and other unfair labor practice (ULP) acts. respondent filed with the
National Labor Relations Commission (NLRC) a Petition for Injunction with Prayer for Issuance of Temporary
Restraining Order, Free Ingress and Egress Order, and Deputization Order.
a temporary restraining order was issued by the NLRC. Thereafter, the NLRC issued a preliminary injunction.
Respondent filed a Petition to Declare Strike Illegal. The Department of Labor and Employment (DOLE) Acting
Secretary, issued an Order assuming jurisdiction over the strike and certifying the same to the NLRC. Petitioner
union filed a petition for certiorari with this Court, questioning the above order of the Acting DOLE Secretary. After
a series of conciliation meetings and discussions between the parties, they agreed to resolve their differences and
came up with a compromise which was embodied in a Memorandum of Agreement. The parties filed a Joint
Motion to Dismiss stating that they are no longer interested in pursuing the petition for injunction filed by
respondent as a consequence of the settlement of their dispute. The NLRC issued its Decision approving the
parties' compromise agreement and granting their Joint Motion to Dismiss.
After a lapse of more than eleven (11) years from the time of execution of the subject MO A, petitioners filed with
the NLRC a Motion for Writ of Execution contending that they have not been paid the amounts they are entitled to
in accordance with the MOA.
Petitioner’s Contention: Petitioners' basic contention is that respondent cannot invoke the defense of
prescription because it is guilty of deliberately causing delay in paying petitioners' claims and that petitioners, on
the other hand, are entitled to protection under the law because they had been vigilant in exercising their right as
provided for under the subject MOA.
Respondent’s Contention: Respondent filed its Opposition to the Motion for Writ of Execution contending that
petitioners' remedy is already barred by prescription because, under the 2005 Revised Rules of the NLRC, a
decision or order may be executed on motion within five (5) years from the date it becomes final and executory
and that the same decision or order may only be enforced by independent action within a period often (10) years
from the date of its finality.
The NLRC in its Resolution denied petitioners' application for the issuance of a writ of execution on the ground of
prescription. Petitioners filed a Motion for Reconsideration but the NLRC in its Resolution dated dismissed it for
lack of merit. Petitioners then filed a petition for certiorari with the CA questioning the above Resolutions of the
NLRC. The CA dismissed the petition for certiorari on the ground that it is a wrong mode of remedy.
ISSUE: whether or not petitioners' claim is already barred by prescription.
HELD:
YES. The five-and ten-year periods provided by law and the rules are more than sufficient to enable petitioners to
enforce their right under the subject MOA. In this case, it is clear that the judgment of the NLRC, having been
based on a compromise embodied in a written contract, was immediately executory upon its issuance on October
12, 1998. Thus, it could have been executed by motion within five (5) years. It was not. Nonetheless, it could have
been enforced by an independent action within the next five (5) years, or within ten (10) years from the time the
NLRC Decision was promulgated. It was not. Therefore, petitioners' right to have the NLRC judgment executed by
mere motion as well as their right of action to enforce the same judgment had prescribed by the time they filed
their Motion for Writ of Execution on January 25, 2010.
Angeles| Bajana | Balladares | Brillantes | Briones | Cabansag | Callanta | Chua | David|

De Leon | Gomez | Lopez | Macalino | Nostratis | Padilla | Reynon | Santos | Tan |Velasco
4E / 4F - 2018-2019
Page 362 of 920
LABOR REVIEW DIGEST
Atty. Joyrich Golangco

There are instances in which this Court allowed execution by motion even after the lapse of five years upon
meritorious grounds. However, in instances when this Court allowed execution by motion even after the lapse of
five years, there is, invariably, only one recognized exception, i.e., when the delay is caused or occasioned by
actions of the judgment debtor and/or is incurred for his benefit or advantage. In the present case, there is no
indication that the delay in the execution of the MOA, as claimed by petitioners, was caused by respondent nor
was it incurred at its instance or for its benefit or advantage.

Angeles| Bajana | Balladares | Brillantes | Briones | Cabansag | Callanta | Chua | David|

De Leon | Gomez | Lopez | Macalino | Nostratis | Padilla | Reynon | Santos | Tan |Velasco
4E / 4F - 2018-2019
Page 363 of 920
LABOR REVIEW DIGEST
Atty. Joyrich Golangco

15. HALITE v SS VENTURES INTERNATIONAL, INC.


G.R. No. 195109 February 4, 2015

Facts:
- Respondent SS Ventures International, Inc. is a domestic corporation duly engaged in the business of
manufacturing footwear products for local sales and export abroad.
- Petitioners Andy Balite (Balite), Monaliza Bihasa (Bihasa) and Delfin Anzaldo (Anzaldo) were regular employees
of the respondent company until their employments were severed for violation of various company policies.
- Balite was issued a Show Cause Memorandum by the respondent company on 4 August 2005 charging him with
the following infractions:
o (1) making false reports, malicious and fraudulent statements and rumor-mongering against the company;
o (2) threatening and intimidating co-workers;
o (3) refusing to cooperate in the conduct of investigation; and
o (4) gross negligence in the care and use of the company property resulting in the damage of the finished products.
- After respondent found Balite’s explanation insufficient, he was dismissed from employment, through a Notice of
Termination
- Bihasa, on the other hand, was charged with absence without leave on two occasions and with improper behavior,
stubbornness, arrogance and uncooperative attitude towards superiors and employees. Bihasa was likewise
terminated from the service
- Anzaldo was also dismissed from employment after purportedly giving him due process. The records of the
infractions he committed as well as the date of his termination, however, are not borne by the records.
- the three employees charged respondents with illegal dismissal and recovery of backwages, 13th month pay and
attorney’s fees before the Labor Arbiter.
- In refuting the allegations of the petitioners, respondents averred that petitioners were separated from
employment for just causes and after affording them procedural due process of law.
- Labor Arbiter rendered a Decision in favor of petitioners and held that respondents are liable for illegal dismissal
for failing to comply with the procedural and substantive requirements in terminating employment.

- CRUX OF THE CASE: respondents interposed an appeal by filing a Notice of Appeal and paying the
corresponding appeal fee. However, instead of filing the required appeal bond equivalent to the total amount of
the monetary award which is ₱490,308.00, respondents filed a Motion to Reduce the Appeal Bond to ₱100,000.00
and appended therein a manager’s check bearing the said amount. Respondents cited financial difficulty as
justification for their inability to post the appeal bond in full owing to the partial shutdown of respondent company’s
operations.
- NLRC dismissed the appeal filed by the respondents for non-perfection. The NLRC ruled that posting of an appeal
bond equivalent to the monetary award is indispensable for the perfection of the appeal and the reduction of the
appeal bond
- Court of Appeals reversed the NLRC Decision and allowed the relaxation of the rule on posting of the appeal
bond. According to the appellate court, there was substantial compliance with the rules for the perfection of an
appeal because respondents seasonably filed their Memorandum of Appeal and posted an appeal bond in the
amount of ₱100,000.00.

ISSUE:
- Petitioners, in assailing the appellate court’s decision, argue that posting of an appeal bond in full is not only
mandatory but a jurisdictional requirement that must be complied within order to confer jurisdiction upon the
NLRC.
- They posit that the posting of an insufficient amount of appeal bond, as in this case, resulted to the non-perfection
of the appeal rendering the decision of the Labor Arbiter final and executory.

Angeles| Bajana | Balladares | Brillantes | Briones | Cabansag | Callanta | Chua | David|

De Leon | Gomez | Lopez | Macalino | Nostratis | Padilla | Reynon | Santos | Tan |Velasco
4E / 4F - 2018-2019
Page 364 of 920
LABOR REVIEW DIGEST
Atty. Joyrich Golangco

HELD:
 The pertinent rule on the matter is Article 223 of the Labor Code, as amended, which sets forth the rules on
appeal from the Labor Arbiter’s monetary award:
 ART. 223. Appeal.– Decisions, awards, or orders of the Labor Arbiter are final and executory unless appealed to
the Commission by any or both parties within ten (10) calendar days from receipt of such decisions, awards, or
orders. x x x.
xxxx
In case of a judgment involving a monetary award, an appeal by the employer may be perfected only
upon the posting of a cash or surety bond issued by a reputable bonding company duly accredited by the
Commission in the amount equivalent to the monetary award in the judgment appealed from.
 Implementing the aforestated provisions of the Labor Code are the provisions of Rule VI of the 2011 Rules of
Procedure of the NLRC on perfection of appeals which read:
o Section. 1. Periods of Appeal. - Decisions, awards or orders of the Labor Arbiter shall be final and executory
unless appealed to the Commission by any or both parties within ten (10)calendar days from receipt thereof. x x x
If the 10th day or the 5th day, as the case may be, falls on a Saturday, Sunday or holiday, the last day to perfect
the appeal shall be the first working day following such Saturday, Sunday or holiday.
xxxx
Section 4. Requisites for Perfection of Appeal. – (a) The appeal shall be:
(1) filed within the reglementary period as provided in Section 1 of this Rule;
(2) verified by the appellant himself/herself in accordance with Section 4, Rule 7 of the Rules of Court ,as
amended;
(3) in the form a of a memorandum of appeal which shall state the grounds relied upon and the arguments in
support thereof; the relief prayed for; and with a statement of the date when the appellant received the appealed
decision, award or order;
(4) in three (3) legibly typewritten or printed copies; and
(5) accompanied by:
i) proof of payment of the required appeal fee and legal research fee;
ii) posting of cash or surety bond as provided in Section 6 of this Rule; and
iii) proof of service upon the other parties.
xxxx
(b) A mere notice of appeal without complying with the other requisites aforestated shall not stop the
running of the period for perfecting an appeal.
xxxx

Section 5. Appeal Fee. - The appellant shall pay the prevailing appeal fee and legal research fee to the Regional
Arbitration Branch or Regional Office of origin, and the official receipt of such payment shall form part of the
records of the case.

Section 6. Bond. - In case the decision of the Labor Arbiter, or the Regional Director involves a monetary award,
an appeal by the employer shall be perfected only upon the posting of a bond, which shall either be in the form of
cash deposit or surety bond equivalent in amount to the monetary award, exclusive of damages and attorney’s
fees.

xxxx

The Commission through the Chairman may on justifiable grounds blacklist a bonding company,
notwithstanding its accreditation by the Supreme Court.
 These statutory and regulatory provisions explicitly provide that an appeal from the Labor Arbiter to the NLRC
must be perfected within ten calendar days from receipt of such decisions, awards or orders of the Labor
Angeles| Bajana | Balladares | Brillantes | Briones | Cabansag | Callanta | Chua | David|

De Leon | Gomez | Lopez | Macalino | Nostratis | Padilla | Reynon | Santos | Tan |Velasco
4E / 4F - 2018-2019
Page 365 of 920
LABOR REVIEW DIGEST
Atty. Joyrich Golangco

Arbiter. In a judgment involving a monetary award, the appeal shall be perfected only upon (1) proof of
payment of the required appeal fee; (2) posting of a cash or surety bond issued by a reputable bonding
company; and (3) filing of a memorandum of appeal.
 In McBurnie v. Ganzon, we harmonized the provision on appeal that its procedures are fairly applied to both
the petitioner and the respondent, assuring by such application that neither one or the other party is unfairly
favored.
 We pronounced that the posting of a cash or surety bond in an amount EQUIVALENT TO 10% of the
monetary award pending resolution of the motion to reduce appeal bond shall be deemed sufficient to
perfect an appeal
 The rule We set in McBurniewas clarified by the Court in Sara Lee Philippines v. Ermilinda Macatlang.
Considering the peculiar circumstances in Sara Lee, We determined what is the reasonable amount of appeal
bond. We underscored the fact that the amount of 10% of the award is not a permissible bond but is only such
amount that shall be deemed reasonable in the meantime that the appellant’s motion is pending resolution by the
Commission. The actual reasonable amount yet to be determined is necessarily a bigger amount.
 We sustain the Court of Appeals in so far as it increases the amount of the required appeal bond. But we deem it
reasonable to reduce the amount of the appeal bond to ₱725 Million. This directive already considers that the
award if not illegal, is extraordinarily huge and that no insurance company would be willing to issue a bond
for such big money. The amount of ₱725 Million is approximately 25% of the basis above calculated. It is a
balancing of the constitutional obligation of the state to afford protection to labor which, specific to this case, is
assurance that in case of affirmance of the award, recovery is not negated; and on the other end of the
spectrum, the opportunity of the employer to appeal.
 By reducing the amount of the appeal bond in this case, the employees would still be assured of at least
substantial compensation, in case a judgment award is affirmed. On the other hand, management will not be
effectively denied of its statutory privilege of appeal.
 In line with Sara Lee and the objective that the appeal on the merits to be threshed out soonest by the NLRC, the
Court holds that the appeal bond posted by the respondent in the amount of ₱100,000.00 which is equivalent to
around 20% of the total amount of monetary bond is sufficient to perfect an appeal.
 With the employer's demonstrated good faith in filing the motion to reduce the bond on demonstrable
grounds coupled with the posting of the appeal bond in the requested amount, as well as the filing of the
memorandum of appeal, the right of the employer to appeal must be upheld. This is in recognition of the
importance of the remedy of appeal, which is an essential part of our judicial system and the need to ensure that
every party litigant is given the amplest opportunity for the proper and just disposition of his cause freed from the
constraints of technicalities.
 PETITION IS DENIED.

Angeles| Bajana | Balladares | Brillantes | Briones | Cabansag | Callanta | Chua | David|

De Leon | Gomez | Lopez | Macalino | Nostratis | Padilla | Reynon | Santos | Tan |Velasco
4E / 4F - 2018-2019
Page 366 of 920
LABOR REVIEW DIGEST
Atty. Joyrich Golangco

16. Turks Shawarma Company and Gem Zeñaros v. Pajaron


G.R. No. 207156
Jan. 16, 2017
Del Castillo, J.:

DOCTRINE: the reduction of the appeal bond is allowed, subject to the following conditions: (1) the motion to
reduce the bond shall be based on meritorious grounds; and (2) a reasonable amount in relation to the monetary
award is posted by the appellant. Compliance with these two conditions will stop the running of the period to
perfect an appeal. ETHIDa

FACTS:
Petitioners hired Feliciano Pajaron and Larry Carbonilla in 2007 as service crew and head crew, respectively. In
April 2015, both would file complaints their respective complaints against their employers for constructive and
actual illegal dismissal, non-payment of overtime pay, holiday pay, holiday premium, rest day premium, service
incentive leave pay and 13th month pay against petitioners. Both Complaints were consolidated.
Pajaron alleged that Zeñarosa asked him to sign a piece of paper stating that he was receiving the correct amount
of wages and that he had no claim against petitioners. He refused to sign the same and that that caused his
dismissal. As for Carbonilla he had an altercation with his supervisor, causing his immediate termination. He was
also asked to sign a piece of paper acknowledging a debt of 7000php.
Both Pajaron and Carbonilla claim that there was no just or authorized cause for their dismissal. Petitioners on the
other hand claim that both abandoned their work, with Pajaron being a habitual absentee for extended periods
and that he was simply rehired on the spot; while Carbonilla habitually misbehaved and left employment without
settling an unpaid obligation of 78,900php, prompting the filing of the charge of estafa against him.

LA RULING: Ruled in favor of both Pajaron and Carbonilla, awarding respectively the amounts of about 148k and
49k.

NLRC PROCEEDINGS+RULING: Due to alleged unavailability of counsel, Zeñarosa filed on his own a Notice of
Appeal with Memorandum, with a Motion to Reduce Bond. He posted a 15,000php bond maintaining that he
cannot pay the full amount as he was a mere backyard entrepreneur. The NLRC denied the motion and ruled that
financial difficulty was not a ground for reduction, and that the same was unsubstantiated. Petitioners thru new
counsel questioned the harsh outright dismissal of their Motion and filed a new Motion for Reconsideration. The
NLRC denied the Motion for Reconsideration and that the posting of the remaining balance would amount to
extending the period allowed by the rules into 3 months.
CA RULING: Dismissed a Petition for Certiorari and ruled that there was no grave abuse of discretion on the part
of the NLRC.
ISSUE: Whether the refusal to grant the motion amounted to grave abuse.
RULING: NO. The Supreme Court cited Secs. 4 and 6 of the NLRC Rules of Procedure, with the latter providing:
No motion to reduce bond shall be entertained except on meritorious grounds, and upon the posting of a bond in a
reasonable amount. The mere fiing of a motion to reduce bond without complying with the requisites in the
preceding paragraphs shall not stop the running of the period to perfect an appeal.
"It is clear from both the Labor Code and the NLRC Rules of Procedure that there is legislative and administrative
intent to strictly apply the appeal bond requirement, and the Court should give utmost regard to this intention." The
posting of cash or surety bond is therefore mandatory and jurisdictional; failure to comply with this requirement
renders the decision of the Labor Arbiter final and executory.
the reduction of the appeal bond is allowed, subject to the following conditions: (1) the motion to reduce the bond
shall be based on meritorious grounds; and (2) a reasonable amount in relation to the monetary award is posted
by the appellant. Compliance with these two conditions will stop the running of the period to perfect an appeal.
The NLRC correctly held that the supposed ground cited in the motion is not well-taken for there was no evidence
to prove Zenarosa's claim that the payment of the full amount of the award would greatly affect his business due
Angeles| Bajana | Balladares | Brillantes | Briones | Cabansag | Callanta | Chua | David|

De Leon | Gomez | Lopez | Macalino | Nostratis | Padilla | Reynon | Santos | Tan |Velasco
4E / 4F - 2018-2019
Page 367 of 920
LABOR REVIEW DIGEST
Atty. Joyrich Golangco

to �nancial setbacks. Besides, "the law does not require outright payment of the total monetary award; [the
appellant has the option to post either a cash or surety bond.

Angeles| Bajana | Balladares | Brillantes | Briones | Cabansag | Callanta | Chua | David|

De Leon | Gomez | Lopez | Macalino | Nostratis | Padilla | Reynon | Santos | Tan |Velasco
4E / 4F - 2018-2019
Page 368 of 920
LABOR REVIEW DIGEST
Atty. Joyrich Golangco

Reinstatement aspect of LA’s Decision


1. Pioneer Texturizing Corporation v. NLRC
GR No: 118651 Date: Oct. 16, 1997
Ponente: Francisco, J.

Doctrine:
Loss of confidence as a ground: To be a valid ground, it must be shown that the employee concerned is
responsible for the misconduct or infraction and that the nature of his participation therein rendered him
absolutely unworthy of the trust and confidence demanded by his position.

Immediately executory judgement of reinstatement: The provision of Article 223 is clear that an award for
reinstatement shall be immediately executory even pending appeal and the posting of a bond by the
employer shall not stay the execution for reinstatement. The legislative intent is quite obvious, i.e., to
make an award of reinstatement immediately enforceable, even pending appeal.

FACTS:

Private respondent Lourdes A. de Jesus is petitioners' reviser/trimmer since 1980. As reviser/trimmer, de


Jesus based her assigned work on a paper note posted by petitioners.
De Jesus mistakenly trimmed the cloths’ ribs on P.O. Number 3853 to which herein petitioner suspended
her for 30 days and required her to explain why no disciplinary action should be taken against her for dishonesty
and tampering of official records and documents with the intention of cheating as P.O. No. 3853 allegedly did not
require trimming. De Jesus explained that she merely committed a mistake as P.O. 3853 has the same style as
P.O. 3824 which has an attached price list for trimming the ribs and admitted that she may have been negligent in
presuming that the same work has to be done, but not for dishonesty or tampering. Nevertheless, De Jesus was
terminated by petitioner’s personnel on Sept. 18, 1992.
On September 22, 1992, de Jesus filed a complaint for illegal dismissal against petitioners.
LA/RTC/NLRC RULING:
LA: The Labor Arbiter who heard the case noted that de Jesus was amply accorded procedural due process in
her termination from service. Nevertheless, after observing that de Jesus made some further trimming on P.O. No.
3853 and that her dismissal was not justified, the Labor Arbiter held petitioners guilty of illegal dismissal.
Petitioners were accordingly ordered to reinstate de Jesus to her previous position without loss of seniority rights
and with full backwages from the time of her suspension on August 19, 1992.

NLRC: The NLRC declared that the status quo between them should be maintained and affirmed the Labor
Arbiter's order of reinstatement, but without backwages.
APPEAL TO THE SC:
Petitioner's Contention: Petitioners insist that the NLRC gravely abused its discretion in holding that de Jesus is
entitled to reinstatement to her previous position for she was not illegally dismissed in the first place. In support
thereof, petitioners quote portions of the NLRC decision which stated that "respondents [petitioners herein] cannot
be entirely faulted for dismissing the complainant" and that there was "no illegal dismissal to speak of in the case
at bar".

ISSUE/S:
1. Whether or not de Jesus was illegally dismissed
2. Whether or not an order for reinstatement needs a writ of execution.
HELD:
1. No. Petitioners simply failed, both before the Labor Arbiter and the NLRC, to discharge the burden of proof and
to validly justify de Jesus' dismissal from service. The law, in this light, directs the employers, such as herein
petitioners, not to terminate the services of an employee except for a just or authorized cause under the Labor
Angeles| Bajana | Balladares | Brillantes | Briones | Cabansag | Callanta | Chua | David|

De Leon | Gomez | Lopez | Macalino | Nostratis | Padilla | Reynon | Santos | Tan |Velasco
4E / 4F - 2018-2019
Page 369 of 920
LABOR REVIEW DIGEST
Atty. Joyrich Golangco

Code. Lack of a just cause in the dismissal from service of an employee, as in this case, renders the dismissal
illegal, despite the employer's observance of procedural due process. And while the NLRC stated that "there was
no illegal dismissal to speak of in the case at bar" and that petitioners cannot be entirely faulted therefor, said
statements are inordinate pronouncements which did not remove the assailed dismissal from the realm of
illegality. Neither can these pronouncements preclude us from holding otherwise.
We also find the imposition of the extreme penalty of dismissal against de Jesus as certainly harsh and
grossly disproportionate to the negligence committed, especially where said employee holds a faithful and an
untarnished twelve-year service record.
Equally unmeritorious is petitioners' assertion that the dismissal is justified on the basis of loss of
confidence. To be a valid ground, it must be shown that the employee concerned is responsible for the
misconduct or infraction and that the nature of his participation therein rendered him absolutely unworthy
of the trust and confidence demanded by his position. In this case, petitioners were unsuccessful in
establishing their accusations of dishonesty and tampering of records with intention of cheating. Indeed, even if
petitioners' allegations against de Jesus were true, they just the same failed to prove that her position needs the
continued and unceasing trust of her employers. The breach of trust must be related to the performance of the
employee's functions. Surely, de Jesus who occupies the position of a reviser/trimmer does not require the
petitioners' perpetual and full confidence.

2. No writ of execution is required for an order for reinstatement under Article 223. Article 224 states that
the need for a writ of execution applies only within five (5) years from the date a decision, an order or award
becomes final and executory. It cannot relate to an award or order of reinstatement still to be appealed or pending
appeal which Article 223 contemplates. The provision of Article 223 is clear that an award for reinstatement
shall be immediately executory even pending appeal and the posting of a bond by the employer shall not
stay the execution for reinstatement. The legislative intent is quite obvious, i.e., to make an award of
reinstatement immediately enforceable, even pending appeal. To require the application for and issuance
of a writ of execution as prerequisites for the execution of a reinstatement award would certainly betray
and run counter to the very object and intent of Article 223, i.e., the immediate execution of a
reinstatement order. The reason is simple. An application for a writ of execution and its issuance could be
delayed for numerous reasons. A mere continuance or postponement of a scheduled hearing, for instance, or an
inaction on the part of the Labor Arbiter or the NLRC could easily delay the issuance of the writ thereby setting at
naught the strict mandate and noble purpose envisioned by Article 223. In other words, if the requirements of
Article 224 were to govern, as we so declared in Maranaw, then the executory nature of a reinstatement order or
award contemplated by Article 223 will be unduly circumscribed and rendered ineffectual.
_________________________________________________________________________

2. Alejandro Roquero v. Philippine Airlines, Inc.


GR No: 152329 Date: April 22, 2003
Ponente: J. Puno

Doctrines:

An employee instigated to take drugs has no right to be reinstated to his position. He took the drugs fully knowing
that he was on duty and more so that it is prohibited by company rules. Instigation is only a defense against
criminal liability. It cannot be used as a shield against dismissal from employment especially when the position
involves the safety of human lives.

The order of reinstatement is immediately executory. The unjustified refusal of the employer to reinstate a
dismissed employee entitles him to payment of his salaries effective from the time the employer failed to reinstate
him despite the issuance of a writ of execution. Unless there is a restraining order issued, it is ministerial upon the
Labor Arbiter to implement the order of reinstatement.
Angeles| Bajana | Balladares | Brillantes | Briones | Cabansag | Callanta | Chua | David|

De Leon | Gomez | Lopez | Macalino | Nostratis | Padilla | Reynon | Santos | Tan |Velasco
4E / 4F - 2018-2019
Page 370 of 920
LABOR REVIEW DIGEST
Atty. Joyrich Golangco

FACTS:
Alejandro Roquero, along with Rene Pabayo, were ground equipment mechanics of respondent Philippine
Airlines, Inc. (PAL). From the evidence on record, it appears that Roquero and Pabayo were caught red-handed
possessing and using Methampethamine Hydrochloride or shabu in a raid conducted by PAL security officers and
NARCOM personnel.

Roquero and Pabayo received a notice of administrative charge for violating the PAL Code of Discipline.
They assailed their arrest and asserted that they were instigated by PAL to take the drugs. Roquero and Pabayo
were dismissed by PAL. Thus, they filed a case for illegal dismissal.

LA/RTC/NLRC RULING:
The Labor Arbiter upheld the dismissal of Roquero and Pabayo. It found both parties at fault, PAL for applying
means to entice the complainants into committing the infraction and the complainants for giving in to the
temptation and eventually indulging in the prohibited activity. Nonetheless, the Labor Arbiter awarded separation
pay and attorney’s fees to the complainants.

While the case was on appeal with the NLRC, the complainants were acquitted by the RTC in the criminal case
which charged them with conspiracy for possession and use of a regulated drug in violation of Section 16, Article
III of Republic Act 6425, on the ground of instigation.

The NLRC ruled in favor of complainants as it likewise found PAL guilty of instigation. It ordered reinstatement to
their former positions but without backwages. Complainants did not appeal from the decision but filed a motion for
a writ of execution of the order of reinstatement. The Labor Arbiter granted the motion but PAL refused to execute
the said order on the ground that they have filed a Petition for Review before the Supreme Court. In accordance
with the case of St. Martin Funeral Home vs. NLRC and Bienvenido Aricayos, PAL’s petition was referred to the
Court of Appeals.

CA RULING:
PAL and Pabayo filed a Motion to Withdraw/Dismiss the case with respect to Pabayo, after they voluntarily
entered into a compromise agreement.

The Court of Appeals later reversed the decision of the NLRC and reinstated the decision of the Labor Arbiter
insofar as it upheld the dismissal of Roquero. However, it denied the award of separation pay and attorney’s fees
to Roquero on the ground that one who has been validly dismissed is not entitled to those benefits.

APPEAL TO THE SC:


Petition for Review on Certiorari under Rule 45

ISSUE/S:
1. Whether or not the instigated employee shall be solely responsible for an action arising from the instigation
perpetrated by the employer and shall not be reinstated.
2. Can the executory nature of the decision, more so the reinstatement aspect of a labor tribunals order be halted by
a petition having been filed in higher courts without any restraining order or preliminary injunction having been
ordered in the meantime?
3. Would the employer who refused to reinstate an employee despite a writ duly issued be held liable to pay the
salary of the subject employee from the time that he was ordered reinstated up to the time that the reversed
decision was handed down?

Angeles| Bajana | Balladares | Brillantes | Briones | Cabansag | Callanta | Chua | David|

De Leon | Gomez | Lopez | Macalino | Nostratis | Padilla | Reynon | Santos | Tan |Velasco
4E / 4F - 2018-2019
Page 371 of 920
LABOR REVIEW DIGEST
Atty. Joyrich Golangco

HELD:
1. Yes. Even if Petitioner was instigated to take drugs he has no right to be reinstated to his position. He took the
drugs fully knowing that he was on duty and more so that it is prohibited by company rules. Instigation is only a
defense against criminal liability. It cannot be used as a shield against dismissal from employment especially
when the position involves the safety of human lives.

There is also no question that petitioner Roquero is guilty of serious misconduct for possessing and using shabu.
He violated Chapter 2, Article VII, section 4 of the PAL Code of Discipline.

2. No. Article 223 (3rd paragraph) of the Labor Code, as amended by Section 12 of Republic Act No. 6715, and
Section 2 of the NLRC Interim Rules on Appeals under RA No. 6715, Amending the Labor Code, provide that an
order of reinstatement by the Labor Arbiter is immediately executory even pending appeal.

3. Yes. The order of reinstatement is immediately executory. The unjustified refusal of the employer to reinstate a
dismissed employee entitles him to payment of his salaries effective from the time the employer failed to reinstate
him despite the issuance of a writ of execution. Unless there is a restraining order issued, it is ministerial upon the
Labor Arbiter to implement the order of reinstatement. In the case at bar, no restraining order was granted. Thus,
it was mandatory on PAL to actually reinstate Roquero or reinstate him in the payroll. Having failed to do so, PAL
must pay Roquero the salary he is entitled to, as if he was reinstated, from the time of the decision of the NLRC
until the finality of the decision of this Court.

Even if the order of reinstatement of the Labor Arbiter is reversed on appeal, it is obligatory on the part of the
employer to reinstate and pay the wages of the dismissed employee during the period of appeal until reversal by
the higher court. On the other hand, if the employee has been reinstated during the appeal period and such
reinstatement order is reversed with finality, the employee is not required to reimburse whatever salary he
received for he is entitled to such, more so if he actually rendered services during the period.

Angeles| Bajana | Balladares | Brillantes | Briones | Cabansag | Callanta | Chua | David|

De Leon | Gomez | Lopez | Macalino | Nostratis | Padilla | Reynon | Santos | Tan |Velasco
4E / 4F - 2018-2019
Page 372 of 920
LABOR REVIEW DIGEST
Atty. Joyrich Golangco

3. Air Philippines Corp. v. Enrico Zamora


GR No: 148247 Date: August 7, 2006
Ponente: AUSTRIA-MARTINEZ, J.:

Doctrine:
Hence, even if the order of reinstatement of the Labor Arbiter is reversed on appeal, it is obligatory on the part of
the employer to reinstate and pay the wages of the dismissed employee during the period of appeal until reversal
by the higher court. On the other hand, if the employee has been reinstated during the appeal period and such
reinstatement order is reversed with finality, the employee is not required to reimburse whatever salary he
received for he is entitled to such, more so if he actually rendered services during the period.

FACTS:
Enrico Zamora (Zamora) was employed with Air Philippines Corporation (APC) as a Flight Deck Crew. He applied
for promotion to the position of airplane captain and underwent the requisite training program. After completing
training, he inquired about his promotion but APC did not act on it; instead, it continued to give him assignments
as flight deck crew. Thus, Zamora filed a Complaint with the Labor Arbiter. He argued that the act of APC of
withholding his promotion rendered his continued employment with it oppressive and unjust. He therefore asked
that APC be held liable for constructive dismissal.

APC denied that it dismissed complainant. It pointed out that, when the complaint was filed on May 14, 1997,
complainant was still employed with it. It was only on May 22, 1997 that complainant stopped reporting for work,
not because he was forced to resign, but because he had joined a rival airline, Grand Air.

LA/RTC/NLRC RULING:
LA ruled in favor of Zamora ordering the respondent to reinstate complainant to his position as Captain without
loss of seniority right immediately upon receipt thereof, pay the full backwages, moral and exemplary damages,
and attorney’s fees.

The NLRC granted the appeal in a Resolution dated February 10, 1999. It held that no dismissal, constructive or
otherwise, took place for it was Zamora himself who voluntarily terminated his employment by not reporting for
work and by joining a competitor Grand Air. However, it ordered APC to pay complainant his unpaid salaries and
allowances in the total amount of P198,502.30 within fifteen (15) days from receipt of this resolution for the reason
that the grant of salaries and allowances to complainant arose from the order of his reinstatement which is
executory even pending appeal of respondent questioning the same, pursuant to Article 223(now 229) of the
Labor Code.

CA RULING:
After filing of Petition for Certiorari, the CA dismissed the petition for failure of petitioner to x x x attach copies of all
pleadings (such complaint, answer, position paper) and other material portions of the record as would support the
allegations therein x x x. APC filed a MR and attached to it the pleadings and portions of the case record required
by the CA. Zamora filed an Opposition to Motion for Reconsideration. CA denied the MR and the resolution of
dismissal stands.

APPEAL TO THE SC:


Petitioner insists, for the deficiency was excusable: pleadings and other portions of the case records were not
attached to the petition because these documents had no bearing on the sole issue raised therein, which was,
whether the NLRC committed grave abuse of discretion in awarding unpaid salaries to respondent despite having
adjudged the latter at fault for abandonment of employment.
Angeles| Bajana | Balladares | Brillantes | Briones | Cabansag | Callanta | Chua | David|

De Leon | Gomez | Lopez | Macalino | Nostratis | Padilla | Reynon | Santos | Tan |Velasco
4E / 4F - 2018-2019
Page 373 of 920
LABOR REVIEW DIGEST
Atty. Joyrich Golangco

Respondent disagrees. He argues that the requirements under Section 1, Rule 65 are mandatory and
jurisdictional; petitioner’s failure to comply with them was a valid ground for the dismissal of its petition.

ISSUE/S:
1. Whether or not the lack of certified true copies of the pleadings and material portions of the case record
warrants the dismissal of a petition for certiorari.
2. Whether or not the NLRC committed grave abuse of discretion in holding petitioner liable to respondent for
P198,502.30.

HELD:
1. NO, the lack of certified true copies of the pleadings and material portions of the case record did not warrant the
dismissal of a petition for certiorari. As a general rule, a petition lacking copies of essential pleadings and portions
of the case record may be dismissed. This rule, however, is not petrified. There are, however, guideposts it must
follow.

First, not all pleadings and parts of case records are required to be attached to the petition. Only those which are
relevant and pertinent must accompany it. The test of relevancy is whether the document in question will support
the material allegations in the petition, whether said document will make out a prima facie case of grave abuse of
discretion as to convince the court to give due course to the petition.

Second, even if a document is relevant and pertinent to the petition, it need not be appended if it is shown that the
contents thereof can also found in another document already attached to the petition. Thus, if the material
allegations in a position paper are summarized in a questioned judgment, it will suffice that only a certified true
copy of the judgment is attached.

Third, a petition lacking an essential pleading or part of the case record may still be given due course or reinstated
(if earlier dismissed) upon showing that petitioner later submitted the documents required, or that it will serve the
higher interest of justice that the case be decided on the merits.

In this case, the pleadings and other documents it required of petitioner were not at all relevant to the petition.
Also, the attachments of petitioner to its petition for certiorari were already sufficient even without the pleadings
and portions of the case record. It was therefore unreasonable of the Court of Appeals to have dismissed it. More
so that petitioner later corrected the purported deficiency by submitting copies of the pleadings and other
documents.

2.
No, we hold that the NLRC did not commit grave abuse of discretion in holding petitioner liable to respondent for
P198,502.30. The premise of the award of unpaid salary to respondent is that prior to the reversal by the NLRC of
the decision of the Labor Arbiter, the order of reinstatement embodied therein was already the subject of an alias
writ of execution even pending appeal. Hence, even if the order of reinstatement of the Labor Arbiter is reversed
on appeal, it is obligatory on the part of the employer to reinstate and pay the wages of the dismissed employee
during the period of appeal until reversal by the higher court. On the other hand, if the employee has been
reinstated during the appeal period and such reinstatement order is reversed with finality, the employee is not
required to reimburse whatever salary he received for he is entitled to such, more so if he actually rendered
services during the period.

Other Notes/ SC Pronouncements:

Angeles| Bajana | Balladares | Brillantes | Briones | Cabansag | Callanta | Chua | David|

De Leon | Gomez | Lopez | Macalino | Nostratis | Padilla | Reynon | Santos | Tan |Velasco
4E / 4F - 2018-2019
Page 374 of 920
LABOR REVIEW DIGEST
Atty. Joyrich Golangco

Then, by and pursuant to the same power (police power), the State may authorize an immediate implementation,
pending appeal, of a decision reinstating a dismissed or separated employee since that saving act is designed to
stop, although temporarily since the appeal may be decided in favor of the appellant, a continuing threat or danger
to the survival or even the life of the dismissed or separated employee and his family.
_________________________________________________________________________

Angeles| Bajana | Balladares | Brillantes | Briones | Cabansag | Callanta | Chua | David|

De Leon | Gomez | Lopez | Macalino | Nostratis | Padilla | Reynon | Santos | Tan |Velasco
4E / 4F - 2018-2019
Page 375 of 920
LABOR REVIEW DIGEST
Atty. Joyrich Golangco

4. LUNESA LANSANGAN AND ROCITA CENDAÑA vs. AMKOR TECHNOLOGY PHILIPPINES, INC.
GR No: 177026 Date: January 30, 2009
Ponente: CARPIO MORALES, J.

DOCTRINE:
Roquero vs. PAL and Article 223 of LC do not apply where there is no finding of illegal dismissal, as in the
present case. Article 223 concerns itself with an interim relief, granted to a dismissed or separated employee
while the case for illegal dismissal is pending appeal, as what happened in Roquero.

FACTS:
Petitioners Lansangan and Cendaña are supervisory employees of respondent Amkor Technology
Philippines. An anonymous e-mail was sent to the respondent’s General Manager detailing allegations of
malfeasance on the part of petitioners for stealing company time. Upon investigation, petitioners admitted in
handwritten letters their wrongdoing that they were swiping another employees ID card or requesting another
employee to swipe ones ID card to gain personal advantage and/or in the interest of cheating. Respondent
terminated petitioners for extremely serious offenses as defined in its Code of Discipline. Hence, the petitioners to
filed a complaint for illegal dismissal with the LA.

LA RULING *(October 20, 2004)


a) Dismissal for a valid cause under Article 282 of LC - offense of dishonesty punishable as a serious form of
misconduct and fraud or breach of trust.
b) Ordered petitioners’ reinstatement to their former positions without backwages - as a measure of equitable and
compassionate relief due to their prior unblemished employment records, show of remorse, harshness of the
penalty and respondent’s defective attendance monitoring system.

Petitioners, without appealing LA’s ruling, filed a motion for the issuance of a writ of reinstatement. After a
series of oppositions, motions and orders, the LA issued an alias writ of execution garnishing respondent’s bank
account. Respondent filed a motion for the quashal of the alias writ of execution and lifting of the notice of
garnishment.

NLRC RULING **(June 30, 2005)


a) Deleted the reinstatement portion on the LA’s decision.
b) Set aside the Alias Writ of Execution and Notice of Garnishment.

CA RULING:
a) Affirmed that petitioners were guilty of misconduct and the like.
b) Ordered to pay petitioners backwages without qualification and deduction from date of *LA’s decision up to the
date of the **NLRC’s Decision citing Article 223 of LC and Roquero v. PAL.

Petitioner’s Contention:
CA’s Order limiting the payment of backwages from the date of LA’s decision up to the date of the NLRC’s
Decision is contrary to the case of Roquero v. PAL – “where the employer was ordered to pay the wages to the
employee from the time the reinstatement order was issued until the finality SC’ decision in favor of the latter.”
The payment of backwages should not be computed only up to the promulgation by the NLRC of its
decision.

Respondent’s Contention:
Reliance on Roquero v. PAL is misplaced in view of the glaring factual differences between said case and the
present case.

Angeles| Bajana | Balladares | Brillantes | Briones | Cabansag | Callanta | Chua | David|

De Leon | Gomez | Lopez | Macalino | Nostratis | Padilla | Reynon | Santos | Tan |Velasco
4E / 4F - 2018-2019
Page 376 of 920
LABOR REVIEW DIGEST
Atty. Joyrich Golangco

ISSUE/S:
Whether the petitioners can avail of the interim relief granted under Art. 223 of LC in the absence of any finding of
illegal dismissal?

HELD:

NO. Roquero, as well as Article 223 of the Labor Code which the CA also relied, finds no application in the
present case. Article 223 concerns itself with an interim relief, granted to a dismissed or separated employee
while the case for illegal dismissal is pending appeal, as what happened in Roquero. It does not apply where there
is no finding of illegal dismissal, as in the present case.
Article 223 of the LC –
“In any event, the decision of the Labor Arbiter reinstating a dismissed or separated employee, insofar as the
reinstatement aspect is concerned, shall immediately be executory, pending appeal. The employee shall either be
admitted back to work under the same terms and conditions prevailing prior to his dismissal or separation or, at
the option of the employer, merely reinstated in the payroll. The posting of a bond by the employer shall not stay
the execution for reinstatement provided herein.”

LA decision finding that the petitioners’ dismissal was valid because they committed dishonesty as a form of
serious misconduct and fraud, or breach of trust had become final. Petitioners not having appealed the same
before the NLRC as in fact they even moved for the execution of the reinstatement aspect of the decision. It bears
recalling that it was only respondent which assailed the LA decision to the NLRC to solely question the propriety
of the order for reinstatement, and it succeeded.

Article 279 of the LC –


“In cases of regular employment, the employer shall not terminate the services of an employee except for a just
cause or when authorized by this Title. An employee who is unjustly dismissed from work shall be entitled to
reinstatement without loss of seniority rights and other privileges and to his full backwages, inclusive of
allowances, and to his other benefits or their monetary equivalent computed from the time his compensation was
withheld from him up to the time of his actual reinstatement.”

Angeles| Bajana | Balladares | Brillantes | Briones | Cabansag | Callanta | Chua | David|

De Leon | Gomez | Lopez | Macalino | Nostratis | Padilla | Reynon | Santos | Tan |Velasco
4E / 4F - 2018-2019
Page 377 of 920
LABOR REVIEW DIGEST
Atty. Joyrich Golangco

5. MARILOU S. GENUINO v. NATIONAL LABOR RELATIONS COMMISSION, CITIBANK,


N.A., WILLIAM FERGUSON, and AZIZ RAJKOTWALA
GR Nos. 142732-33 Date: December 4, 2007
Ponente: VELASCO, JR., J.
Doctrine: If the decision of the labor arbiter is later reversed on appeal upon the finding that the ground for
dismissal is valid, then the employer has the right to require the dismissed employee on payroll reinstatement to
refund the salaries s/he received while the case was pending appeal, or it can be deducted from the accrued
benefits that the dismissed employee was entitled to receive from his/her employer under existing laws, collective
bargaining agreement provisions, and company practices. However, if the employee was reinstated to work during
the pendency of the appeal, then the employee is entitled to the compensation received for actual services
rendered without need of refund.
FACTS:
Citibank is an American banking corporation duly licensed to do business in the Philippines. William Ferguson
was the Manila Country Corporate Officer and Business Head of the Global Finance Bank of Citibank while Aziz
Rajkotwala was the International Business Manager for the Global Consumer Bank of Citibank.

Genuino was employed by Citibank sometime in January 1992 as Treasury Sales Division Head with the rank of
Assistant Vice-President. On August 23, 1993, Citibank sent Genuino a letter charging her with knowledge
and/or involvement in transactions which were irregular or even fraudulent. In the same letter, Genuino was
informed she was under preventive suspension.
Genuino wrote Citibank on September 13, 1993 and asked the bank the following:

a. Confront our client with the factual and legal basis of your charges, and afford her an
opportunity to explain;
b. Substantiate your charge of fraudulent transactions against our client; or if the same cannot
be substantiated;
c. Correct/repair/compensate the damage you have caused our client.

On September 13, 1993, Citibank, through Victorino P. Vargas, its Country Senior Human Resources Officer, sent
a letter to Genuino, the relevant portions of which read:

As you are well aware, the bank served you a letter dated August 23, 1993 advising you that
ongoing investigations show that you are involved and/or know of irregular transactions which are at the
very least in conflict with the banks interest, and, may even be fraudulent in nature.
In view of the foregoing, you are hereby directed to explain in writing three (3) days from your
receipt hereof why your employment should not be terminated in view of your involvement in these
irregular transactions. You are also directed to appear in an administrative investigation of the matter
which is set on Tuesday, Sept. 21, 1993 at 2:00 P.M. at the HR Conference Room,
6thFloor, Citibank Center. You may bring your counsel if you so desire.

Genuinos counsel replied through a letter dated September 17, 1993, demanding for a bill of particulars regarding
the charges against Genuino. Citibanks counsel replied on September 20, 1993, as follows:

1.2. [T]he bank has no intention of converting the administrative investigation of this case to a full blown
trial. What it is prepared to do is give your client, as required by law and Supreme Court decisions, an
opportunity to explain her side on the issue of whether she violated the conflict of interest ruleeither in
writing (which could be in the form of a letter-reply to the September 13, 1993 letter to Citibank, N.A.) or
in person, in the administrative investigation which is set for tomorrow afternoon vis--vis the bank
clients/parties mentioned in the letter of Citibank, N.A.

Angeles| Bajana | Balladares | Brillantes | Briones | Cabansag | Callanta | Chua | David|

De Leon | Gomez | Lopez | Macalino | Nostratis | Padilla | Reynon | Santos | Tan |Velasco
4E / 4F - 2018-2019
Page 378 of 920
LABOR REVIEW DIGEST
Atty. Joyrich Golangco

Genuino did not appear in the administrative investigation held on September 21, 1993. Her lawyers wrote a letter
to Citibanks counsel asking what bank clients funds were diverted from the bank and invested in other companies,
the specific amounts involved, the manner by which and the date when such diversions were purportedly
affected.In reply, Citibanks counsel noted Genuinos failure to appear in the investigation and gave Genuino up
to September 23, 1993 to submit her written explanation. Genuino did not submit her written explanation.
On September 27, 1993, Citibank informed Genuino of the result of their investigation. It found that Genuino with
Santos used facilities of Genuinos family corporation, namely, Global Pacific, personally and actively participated
in the diversion of bank clients funds to products of other companies that yielded interests higher than what
Citibank products offered, and that Genuino and Santos realized substantial financial gains, all in violation of
existing company policy and the Corporation Code, which for your information, carries a penal sanction.

Genuinos employment was terminated by Citibank on grounds of (1) serious misconduct, (2) willful breach of the
trust reposed upon her by the bank, and (3) commission of a crime against the bank.
Genuino filed before the Labor Arbiter a Complaint against Citibank docketed as NLRC Case No. 00-10-06450-93
for illegal suspension and illegal dismissal with damages and prayer for temporary restraining order and/or writ of
preliminary injunction.

LA: found the dismissal of the complainant Marilou S. Genuino to be without just cause and in violation
of her right to due process.
NLRC: reversed LA’s decision and held the dismissal of the complainant valid and legal on the ground
of serious misconduct and breach of trust and confidence.

ISSUE/S:
Whether or not the dismissal of Genuino is for a just cause and in accordance with due process.

HELD:
We affirm that Genuino was dismissed for just cause but without the observance of due process.
The Labor Arbiter found that Citibank failed to adequately notify Genuino of the charges against her. On the
contrary, the NLRC held that the function of a notice to explain is only to state the basic facts of the employers
charges, which x x x the letters of September 13 and 17, 1993 in question have fully served.

We agree with the CA that the dismissal was valid and legal, and with its modification of the NLRC ruling that PhP
5,000 is due Genuino for failure of Citibank to observe due process. The Implementing Rules and Regulations of
the Labor Code provide that any employer seeking to dismiss a worker shall furnish the latter a written notice
stating the particular acts or omissions constituting the grounds for dismissal. The purpose of this notice is to
sufficiently apprise the employee of the acts complained of and enable him/her to prepare his/her defense. In this
case, the letters dated August 23, September 13 and 20, 1993 sent by Citibank did not identify the particular
acts or omissions allegedly committed by Genuino.

While the bank gave Genuino an opportunity to deny the truth of the allegations in writing and participate in the
administrative investigation, the fact remains that the charges were too general to enable Genuino to intelligently
and adequately prepare her defense.
The two-notice requirement of the Labor Code is an essential part of due process. The first notice informing the
employee of the charges should neither be pro-forma nor vague. It should set out clearly what the employee is
being held liable for. The employee should be afforded ample opportunity to be heard and not mere opportunity.
Since the notice of charges given to Genuino is inadequate, the dismissal could not be in accordance with due
process.

While we hold that Citibank failed to observe procedural due process, we nevertheless find Genuinos dismissal
justified. In view of Citibanks failure to observe due process, however, nominal damages are in order but the
Angeles| Bajana | Balladares | Brillantes | Briones | Cabansag | Callanta | Chua | David|

De Leon | Gomez | Lopez | Macalino | Nostratis | Padilla | Reynon | Santos | Tan |Velasco
4E / 4F - 2018-2019
Page 379 of 920
LABOR REVIEW DIGEST
Atty. Joyrich Golangco

amount is hereby raised to PhP 30,000 pursuant to Agabon v. NLRC.The NLRCs order for payroll reinstatement is
set aside.

Thus, the award of PhP 5,000 to Genuino as indemnity for non-observance of due process under the CAs March
31, 2000 Resolution in CA-G.R. SP No. 51532 is increased to PhP 30,000.

Anent the directive of the NLRC in its September 3, 1994 Decision ordering Citibank to pay the salaries due to the
complainant from the date it reinstated complainant in the payroll (computed at P60,000.00 a month, as found by
the Labor Arbiter) up to and until the date of this decision, the Court hereby cancels said award in view of its
finding that the dismissal of Genuino is for a legal and valid ground.

Ordinarily, the employer is required to reinstate the employee during the pendency of the appeal pursuant to Art.
223, paragraph 3 of the Labor Code, which states:

In any event, the decision of the Labor Arbiter reinstating a dismissed or separated employee,
insofar as the reinstatement aspect is concerned, shall immediately be executory, even pending
appeal. The employee shall either be admitted back to work under the same terms and conditions
prevailing prior to his dismissal or separation or, at the option of the employer, merely reinstated in the
payroll. The posting of a bond by the employer shall not stay the execution for reinstatement provided
herein.

If the decision of the labor arbiter is later reversed on appeal upon the finding that the ground for
dismissal is valid, then the employer has the right to require the dismissed employee on payroll
reinstatement to refund the salaries s/he received while the case was pending appeal, or it can be
deducted from the accrued benefits that the dismissed employee was entitled to receive from his/her
employer under existing laws, collective bargaining agreement provisions, and company
practices. However, if the employee was reinstated to work during the pendency of the appeal, then the
employee is entitled to the compensation received for actual services rendered without need of refund.
Considering that Genuino was not reinstated to work or placed on payroll reinstatement, and her dismissal is
based on a just cause, then she is not entitled to be paid the salaries stated in item no. 3 of the fallo of the
September 3, 1994 NLRC Decision.

Angeles| Bajana | Balladares | Brillantes | Briones | Cabansag | Callanta | Chua | David|

De Leon | Gomez | Lopez | Macalino | Nostratis | Padilla | Reynon | Santos | Tan |Velasco
4E / 4F - 2018-2019
Page 380 of 920
LABOR REVIEW DIGEST
Atty. Joyrich Golangco

6. Juanito Garcia and Alberto Dumago vs PAL


GR No: Date:
Ponente: CARPIO-MORALES, J :

Doctrine:

FACTS:

Petitioners Alberto J. Dumago and Juanito A. Garcia were Aircraft Furnishers Master "C" and Aircraft Inspector,
respectively, assigned in the PAL Technical Center. They were allegedly caught in the act of sniffing shabu when
a team of company security personnel and law enforcers raided the PAL Technical Center's Toolroom. Thus, after
due notice, they were dismissed for violating PAL's Code of Discipline.

Before the promulgation of the LA's decision, the SEC placed PAL, which was suffering from severe financial
losses, under Receivership.

LA RULING:

The LA ruled in favor of the petitioners, thus ordering PAL to, inter alia, to immediately comply with the
reinstatement aspect of the decision.

NLRC RULING:

Upon appeal to the NLRC, it reversed the LA decision and dismissed petitioners' complaint for lack of merit.

Petitioners' Motion for Reconsideration was denied and an Entry of Judgment was issued. Subsequently, the LA
issued a Writ of Execution respecting the reinstatement aspect of his Decision, and subsequently he issued a
Notice of Garnishment. Respondent thereupon moved to quash the Writ and to lift the Notice while petitioners
moved to release the garnished amount.

CA RULING:

The CA nullified the NLRC Resolutions on two grounds, that: (1) a subsequent finding of a valid dismissal
removes the basis for implementing the reinstatement aspect of a labor arbiter's decision, and (2) the impossibility
to comply with the reinstatement order due to corporate rehabilitation provides a reasonable justification for the
failure to exercise the options under Article 223 of the Labor Code.

APPEAL TO THE SC:

Petitioner's Contention:\Respondent's Contention:

ISSUE/S:
1. Does the subsequent finding of a valid dismissal removes the basis for implementing the reinstatement
aspect of a LA's decision? - No
2. If there is a reversal of the LA's decision, may the employee still collect the accrued wages if it is shown
that delay to reinstate was without fault of the employer?- No.

Angeles| Bajana | Balladares | Brillantes | Briones | Cabansag | Callanta | Chua | David|

De Leon | Gomez | Lopez | Macalino | Nostratis | Padilla | Reynon | Santos | Tan |Velasco
4E / 4F - 2018-2019
Page 381 of 920
LABOR REVIEW DIGEST
Atty. Joyrich Golangco

HELD:
1. No. The Court reaffirms the prevailing principle that even if the order of reinstatement of the Labor Arbiter is
reversed on appeal, it is obligatory on the part of the employer to reinstate and pay the wages of the dismissed
employee during the period of appeal until reversal by the higher court. It settles the view that the Labor Arbiter's
order of reinstatement is immediately executory and the employer has to either re-admit them to work under the
same terms and conditions prevailing prior to their dismissal, or to reinstate them in the payroll, and that failing to
exercise the options in the alternative, employer must pay the employee's salaries.

2. No. After the LA's decision is reversed by a higher tribunal, the employee may be barred from collecting the
accrued wages, if it is shown that the delay in enforcing the reinstatement pending appeal was without fault on the
part of the employer.

The test is two-fold:


(1) there must be actual delay or the fact that the order of reinstatement pending appeal was not executed prior to its
reversal; and
(2) the delay must not be due to the employer's unjustified act or omission. If the delay is due to the employer's
unjustified refusal, the employer may still be required to pay the salaries notwithstanding the reversal of the LA's
decision.

In this case, Respondent was, during the period material to the case, effectively deprived of the alternative
choices under Article 223 of the Labor Code, not only by virtue of the statutory injunction but also in view of the
interim relinquishment of management control to give way to the full exercise of the powers of the rehabilitation
receiver. Had there been no need to rehabilitate, respondent may have opted for actual physical reinstatement
pending appeal to optimize the utilization of resources. Then again, though the management may think this wise,
the rehabilitation receiver may decide otherwise, not to mention the subsistence of the injunction on claims.

In sum, the obligation to pay the employee's salaries upon the employer's failure to exercise the alternative
options under Article 223 of the Labor Code is not a hard and fast rule, considering the inherent constraints of
corporate rehabilitation.

Petition is partially granted.

Note:

Quisumbing, J.,with Separate Opinion.

The principle of unjust enrichment does not apply.


First, the provision on reinstatement pending appeal is in accord with the social justice philosophy of our
Constitution. It is meant to afford full protection to labor as it aims to stop (albeit temporarily, since the appeal may
be decided in favor of the employer) a continuing threat or danger to the survival or even the life of the dismissed
employee and his family.
Second, the provision on reinstatement pending appeal partakes of a special law that must govern the instant
case. The provision of the Civil Code on unjust enrichment, being of general application, must give way.

Rehabilitation merely provides for the automatic stay of all pending actions or the suspension of payments of the
distressed corporation to prevent the dissipation of its assets; it does not relieve the corporation of its obligations.
Upon its successful rehabilitation, it must settle in full all claims previously suspended.

Angeles| Bajana | Balladares | Brillantes | Briones | Cabansag | Callanta | Chua | David|

De Leon | Gomez | Lopez | Macalino | Nostratis | Padilla | Reynon | Santos | Tan |Velasco
4E / 4F - 2018-2019
Page 382 of 920
LABOR REVIEW DIGEST
Atty. Joyrich Golangco

Applying the foregoing rule, we cannot adhere to the posture taken by the majority. Just because PAL was under
rehabilitation did not necessarily mean that immediately executory orders such as reinstatement pending appeal
will be put to naught. That would in effect nullify the relief given to the employee when all the law seeks to do is
suspend it.

Velasco, Jr.,J.,I concur in the result. With separate opinion.

When the NLRC rendered its reversal decision and held the petitioner's dismissal from PAL valid, it had in effect
removed the legal basis for petitioners' reinstatement. Accordingly, as there is no more basis for reinstatement,
the payment of unearned wages during the appeal, therefore, has no legal basis either.

In the case at bar, PAL did not reinstate the petitioners due to corporate rehabilitation, doubtless a justifiable
cause. Thus, it was incumbent for the employees to procure a writ of execution to compel reinstatement. If
PALdisobeyed, then they could have asked the labor arbiter to cite the airline in contempt. They did not. They only
got the writ after the NLRC decision annulling the arbiter's decision has become final. In this situation, they are not
clearly entitled to the wages that could have been due to them during the appeal period.

Brion, J.,with concurring and dissenting opinion.

Concurs with the ponencia on the first issue, but dissent from the conclusion on the corporate rehabilitation issue.
Thus, he voted to GRANT the petition and order the respondent PAL to pay the petitioners the salaries due them
prior to the NLRC's reversal of the labor arbiter's decision.

__________________________________________________________________________

Angeles| Bajana | Balladares | Brillantes | Briones | Cabansag | Callanta | Chua | David|

De Leon | Gomez | Lopez | Macalino | Nostratis | Padilla | Reynon | Santos | Tan |Velasco
4E / 4F - 2018-2019
Page 383 of 920
LABOR REVIEW DIGEST
Atty. Joyrich Golangco

7. MT. CARMEL COLLEGE v. JOCELYN RESUENA, et. al.


GR No.: 173076 Date: October 10, 2007
Ponente: CHICO-NAZARIO, J.

DOCTRINE:

Angeles| Bajana | Balladares | Brillantes | Briones | Cabansag | Callanta | Chua | David|

De Leon | Gomez | Lopez | Macalino | Nostratis | Padilla | Reynon | Santos | Tan |Velasco
4E / 4F - 2018-2019
Page 384 of 920
LABOR REVIEW DIGEST
Atty. Joyrich Golangco

An illegally dismissed employee is entitled to two reliefs: backwages and reinstatement. The two reliefs provided
are separate and distinct. In instances where reinstatement is no longer feasible because of strained relations
between the employee and the employer, separation pay is granted. In effect, an illegally dismissed employee is
entitled to either reinstatement, if viable, or separation pay if reinstatement is no longer viable, and backwages.

FACTS:

Petitioner is a private educational institution administered by the Carmelite Fathers at New Escalante, Negros
Occidental while respondents were the employees of petitioner. On November 1997 respondents, together with
several faculty members, non-academic personnel, and other students, participated in a protest action against
petitioner. Because of this, respondents were terminated by petitioner on 15 May 1998. Thus, petitioner filed
separate complaints before Regional Arbitration Branch VI of the NLRC in Bacolod City, charging petitioner with
illegal dismissal and claimed 13th month pay, separation pay, damages and attorney's fees.

LA RULING: Labor Arbiter Drilon found that they were not illegally dismissed but ordered that they be awarded
13th month pay, separation pay and attorney’s fees in the amount of P334,875.47.

NLRC RULING: the NLRC reversed the findings of the Labor Arbiter ruling that the termination of respondents
was illegal and ordering the payment of back wages of respondents from 15 May 1998 up to 25 May 1999. It
further directed the reinstatement of respondents or payment of separation pay, with back wages.

CA RULING: The CA affirmed NLRC’s decision.

ISSUE/S:

1. Whether reinstatement in the instant case is self-executory and does not need a writ of execution for its
enforcement.

2. Whether the continuing award of backwages is proper.

HELD:
1. NO.

An order for reinstatement must be specifically declared and cannot be presumed; like back wages, it is a
separate and distinct relief given to an illegally dismissed employee. There being no specific order for
reinstatement and the order being for complainant’s separation, there can be no basis for the award of
salaries/back wages during the pendency of appeal. This Court had declared in the aforesaid case that
reinstatement during appeal is warranted only when the Labor Arbiter himself rules that the dismissed employee
should be reinstated. But this was precisely because on appeal to the NLRC, it found that there was no illegal
dismissal; thus, neither reinstatement nor back wages may be awarded.

2. YES.

An illegally dismissed employee is entitled to two reliefs: back wages and reinstatement. The two reliefs provided
are separate and distinct. In instances where reinstatement is no longer feasible because of strained relations
between the employee and the employer, separation pay is granted. In effect, an illegally dismissed employee is
entitled to either reinstatement, if viable, or separation pay if reinstatement is no longer viable, and back wages.
The normal consequences of respondents’ illegal dismissal, then, are reinstatement without loss of seniority
rights, and payment of back wages computed from the time compensation was withheld up to the date of actual
reinstatement. Where reinstatement is no longer viable as an option, separation pay equivalent to one (1)month
Angeles| Bajana | Balladares | Brillantes | Briones | Cabansag | Callanta | Chua | David|

De Leon | Gomez | Lopez | Macalino | Nostratis | Padilla | Reynon | Santos | Tan |Velasco
4E / 4F - 2018-2019
Page 385 of 920
LABOR REVIEW DIGEST
Atty. Joyrich Golangco

salary for every year of service should be awarded as an alternative. The payment of separation pay is in addition
to payment of back wages. Concomitantly, it is evident that respondents backwages should not be limited to the
period from 15 May 1998 to 25 May 1999. The backwages due respondents must be computed from the time they
were unjustly dismissed until their actual reinstatement to their former position or upon petitioners payment of
separation pay to them if reinstatement is no longer feasible. Thus, until petitioner actually implements the
reinstatement aspect of the NLRC Decision dated 30 October 2001, as affirmed in the Court of Appeals Decision
dated 17 March 2004 in CA-G.R. SP No. 80639, its obligation to respondents, insofar as accrued backwages and
other benefits are concerned, continues to accumulate.

Angeles| Bajana | Balladares | Brillantes | Briones | Cabansag | Callanta | Chua | David|

De Leon | Gomez | Lopez | Macalino | Nostratis | Padilla | Reynon | Santos | Tan |Velasco
4E / 4F - 2018-2019
Page 386 of 920
LABOR REVIEW DIGEST
Atty. Joyrich Golangco

8. BUENVIAJE VS CA
G.R. No. 147806. November 12, 2002
Puno, J.

DOCTRINE: Under R.A. 6715, employees who are illegally dismissed are entitled to “full backwages”. Full
Backwages means backwages that is to be awarded to an illegally dismissed employee, should not, as a general
rule, be diminished or reduced by the earnings derived by him elsewhere during the period of his illegal dismissal.

FACTS: Petitioners were former employees of Cottonway Marketing Corp. (Cottonway), hired as promo girls for
their garment products. After their services were terminated as the company was allegedly suffering business
losses, petitioners filed with the National Labor Relations Commission (NLRC) a complaint for illegal dismissal,
underpayment of salary, and non-payment of premium pay for rest day, service incentive leave pay and thirteenth
month pay against Cottonway Marketing Corp. and Network Fashion Inc./JCT International Trading
LABOR ARBITER RULING: LA Protasio issued a Decision finding petitioners' retrenchment valid and ordering
Cottonway to pay petitioners' separation pay and their proportionate thirteenth month pay.
NLRC RULING: On appeal, the NLRC reversed the Decision of the Labor Arbiter and ordered the reinstatement
of petitioners without loss of seniority rights and other privileges, and payment of appropriate allowances and
benefits
Cottonway filed with the NLRC a manifestation stating that they have complied with the order of reinstatement by
sending notices requiring the petitioners to return to work, but to no avail; and consequently, they sent letters to
petitioners informing them that they have lost their employment for failure to comply with the return to work order,
and that the petitioners have found new employment.
Petitioners filed with the NLRC a motion for execution of its Decision on the ground that it had become final and
executor.
Labor Arbiter Protasio issued another Order declaring that the award of backwages and proportionate thirteenth
month pay to petitioners should be limited from the time of their illegal dismissal up to the time they received the
notice of termination sent by the company upon their refusal to report for work despite the order of reinstatement.
However, LA’s order was set aside by the Commission ruling that its Decision has become final and executory
and it is the ministerial duty of the Labor Arbiter to issue the corresponding writ of execution to effect full and
unqualified implementation of said decision. The Commission thus ordered that the records of the case be
remanded to the Labor Arbiter for execution.
Hence, Cottonway filed a petition for certiorari with the Court of Appeals seeking the reversal of the ruling of the
NLRC and the reinstatement of the Order of LA.
COURT OF APPEALS RULING: The appellate court granted the petition. It ruled that petitioners' reinstatement
was no longer possible as they deliberately refused to return to work despite the notice given by
Cottonway. Additionally, it ruled that it would be unjust and inequitable then to require petitioner to pay private
respondents their backwages even after the latter were validly terminated when in fact petitioner dutifully complied
with the reinstatement aspect of the decision. Thus, the period within which the monetary award of private
respondents should be based is limited up to the time of private respondents' receipt of the respective notices of
termination.
ISSUES: 1. Should the computation of petitioners' backwages be limited from the time they were illegally
dismissed until they received the notice of termination sent by Cottonway as held by CA, or whether it should be
computed from the time of their illegal dismissal until their actual reinstatement as argued by the petitioners.
2. Whether the failure of the Petitioners to return to work despite notice result to abandonment and thus became a
proper ground for dismissal.
SC RULING:
1. Under R.A. 6715, employees who are illegally dismissed are entitled to full backwages, inclusive of
allowances and other benefits or their monetary equivalent, computed from the time their actual compensation
was withheld from them up to the time of their actual reinstatement. If reinstatement is no longer possible, the
backwages shall be computed from the time of their illegal termination up to the finality of the decision.
Angeles| Bajana | Balladares | Brillantes | Briones | Cabansag | Callanta | Chua | David|

De Leon | Gomez | Lopez | Macalino | Nostratis | Padilla | Reynon | Santos | Tan |Velasco
4E / 4F - 2018-2019
Page 387 of 920
LABOR REVIEW DIGEST
Atty. Joyrich Golangco

The evident legislative intent as expressed in Rep. Act No. 6715, above-quoted, backwages to be awarded to an
illegally dismissed employee, should not, as a general rule, be diminished or reduced by the earnings derived by
him elsewhere during the period of his illegal dismissal. The underlying reason for this ruling is that the employee,
while litigating the legality (illegality) of his dismissal, must still earn a living to support himself and family, while full
backwages have to be paid by the employer as part of the price or penalty he has to pay for illegally dismissing
his employee. The clear legislative intent of the amendment in Rep. Act No. 6715 is to give more benefits to
workers than was previously given them under the Mercury Drug rule or the "deduction of earnings elsewhere"
rule.
In this case, there is no question whether or not there is illegal dismissal as the decision of the NLRC has
become final and executory.
2. NO. Petitioners' alleged failure to return to work cannot be made the basis for their termination. Such failure
does not amount to abandonment which would justify the severance of their employment. To warrant a valid
dismissal on the ground of abandonment, the employer must prove the concurrence of two elements: (1) the
failure to report for work or absence without valid or justifiable reason, and (2) a clear intention to sever the
employer-employee relationship.
The facts of this case do not support the claim of Cottonway that petitioners have abandoned their desire to
return to their previous work at said company. Cottonway, before finally deciding to dispense with their services,
did not give the petitioners the opportunity to explain why they were not able to report to work. The records also
do not bear any proof that all the petitioners received a copy of the letters. Cottonway merely claimed that some of
them have left the country and some have found other employment. It should also be expected that petitioners
would seek other means of income to tide them over during the time that the legality of their termination is under
litigation. Furthermore, petitioners never abandoned their suit against Cottonway.
It appears that the supposed notice sent by Cottonway to the petitioners demanding that they report back to
work immediately was only a scheme to remove the petitioners for good.

Angeles| Bajana | Balladares | Brillantes | Briones | Cabansag | Callanta | Chua | David|

De Leon | Gomez | Lopez | Macalino | Nostratis | Padilla | Reynon | Santos | Tan |Velasco
4E / 4F - 2018-2019
Page 388 of 920
LABOR REVIEW DIGEST
Atty. Joyrich Golangco

9. Case Title: Pfizer, Inc. vs. Velasco


GR No: 177467 Date: March 9, 2011
Ponente: Leonardo-De Castro, J.

Doctrine: (1) To reiterate, under Article 223 of the Labor Code, an employee entitled to reinstatement "shall either
be admitted back to work under the same terms and conditions prevailing prior to his dismissal or separation or, at
the option of the employer, merely reinstated in the payroll."

(2) The provision of Article 223 is clear that an award [by the Labor Arbiter] for reinstatement shall be immediately
executory even pending appeal and the posting of a bond by the employer shall not stay the execution for
reinstatement.

Even if the order of reinstatement of the Labor Arbiter is reversed on appeal, it is obligatory on the part of the
employer to reinstate and pay the wages of the dismissed employee during the period of appeal until reversal by
the higher court.

FACTS: Geraldine Velasco was employed with petitioner PFIZER, INC. as Professional Health Care
Representative. Sometime in April 2003, Velasco had a medical work up for her high-risk pregnancy and was
subsequently advised bed rest which resulted in her extending her leave of absence from March to July 2003.

While Velasco was still on leave, PFIZER through its Area Sales Manager, herein petitioner Ferdinand Cortez,
personally served Velasco a "Show-cause Notice." Aside from mentioning about an investigation on her possible
violations of company work rules regarding "unauthorized deals and/or discounts in money or samples, the notice
also advised her that she was being placed under "preventive suspension" for 30 days. Velasco sent a letter to
Cortez denying the charges.

Velasco filed a complaint for illegal suspension with money claims before the Regional Arbitration Branch. Then
PFIZER informed Velasco of its "Management Decision" terminating her employment.

LA/RTC/NLRC/CA RULING: The Labor Arbiter and NLRC rendered decisions declaring the dismissal of Velasco
illegal, ordering her reinstatement with backwages. PFIZER ordered respondent to report to its main office in
Makati City while knowing fully well that respondent’s previous job had her stationed in Baguio City (respondent’s
place of residence).

CA ruled in favor of the validity of respondent’s dismissal from employment but directed PFIZER to pay
respondent her wages from the date of the Labor Arbiter’s Decision up to the Court of Appeals Decision.

PFIZER challenges the award of wages from the date of the Labor Arbiter’s decision ordering her reinstatement
until November 23, 2005, when the Court of Appeals rendered its decision declaring Velasco’s dismissal valid.
APPEAL TO THE SC:

Petitioner's Contention: In PFIZER’s view, it should no longer be required to pay wages considering that (1) it
had already previously paid an enormous sum to respondent under the writ of execution issued by the Labor
Arbiter; (2) it was allegedly ready to reinstate respondent as of July 1, 2005 but it was respondent who
unjustifiably refused to report for work

Respondent's Contention: (not discussed. Respondent’s decision to claim separation pay over reinstatement
had no legal effect, not only because there was no genuine compliance by the employer to the reinstatement
order but also because the employer chose not to act on said claim. If it was PFIZER’s position that respondent’s
Angeles| Bajana | Balladares | Brillantes | Briones | Cabansag | Callanta | Chua | David|

De Leon | Gomez | Lopez | Macalino | Nostratis | Padilla | Reynon | Santos | Tan |Velasco
4E / 4F - 2018-2019
Page 389 of 920
LABOR REVIEW DIGEST
Atty. Joyrich Golangco

act amounted to a "resignation" it should have informed respondent that it was accepting her resignation and that
in view thereof she was not entitled to separation pay. PFIZER did not respond to respondent’s demand at all. As
it was, PFIZER’s failure to effect reinstatement and accept respondent’s offer to terminate her employment
relationship with the company meant that, prior to the Court of Appeals’ reversal in the November 23, 2005
Decision, PFIZER’s liability for backwages continued to accrue for the period not covered by the writ of execution
dated May 24, 2005 until November 23, 2005.)

ISSUE/S:
1. Whether or not the reinstatement of Velasco to petitioner’s Makati Office is valid

2. Whether or not the CA committed an error when it ordered Pfizer to pay Velasco wages from the date of the
Labor Arbiter’s decision ordering her reinstatement until November 23, 2005, when the Court of Appeals rendered
its decision declaring Velasco’s dismissal valid.

HELD:
1. NO. To reiterate, under Article 223 of the Labor Code, an employee entitled to reinstatement "shall either be
admitted back to work under the same terms and conditions prevailing prior to his dismissal or separation or, at
the option of the employer, merely reinstated in the payroll."

It is established in jurisprudence that reinstatement means restoration to a state or condition from which one had
been removed or separated. The person reinstated assumes the position he had occupied prior to his dismissal.
Reinstatement presupposes that the previous position from which one had been removed still exists, or that there
is an unfilled position which is substantially equivalent or of similar nature as the one previously occupied by the
employee.

Applying the foregoing principle to the case before us, it cannot be said that with PFIZER’s June 27, 2005 Letter,
in belated fulfillment of the Labor Arbiter’s reinstatement order, it had shown a clear intent to reinstate respondent
to her former position under the same terms and conditions nor to a substantially equivalent position. To begin
with, the return-to-work order PFIZER sent respondent is silent with regard to the position or the exact nature of
employment that it wanted respondent to take up as of July 1, 2005. Even if we assume that the job awaiting
respondent in the new location is of the same designation and pay category as what she had before, it is plain
from the text of PFIZER’s June 27, 2005 letter that such reinstatement was not "under the same terms and
conditions" as her previous employment, considering that PFIZER ordered respondent to report to its main office
in Makati City while knowing fully well that respondent’s previous job had her stationed in Baguio City
(respondent’s place of residence) and it was still necessary for respondent to be briefed regarding her work
assignments and responsibilities, including her relocation benefits.

The Court is cognizant of the prerogative of management to transfer an employee from one office to another
within the business establishment, provided that there is no demotion in rank or diminution of his salary, benefits
and other privileges and the action is not motivated by discrimination, made in bad faith, or effected as a form of
punishment or demotion without sufficient cause. Likewise, the management prerogative to transfer personnel
must be exercised without grave abuse of discretion and putting to mind the basic elements of justice and fair
play. There must be no showing that it is unnecessary, inconvenient and prejudicial to the displaced employee.27

The June 27, 2005 return-to-work directive implying that respondent was being relocated to PFIZER’s Makati
main office would necessarily cause hardship to respondent, a married woman with a family to support residing in
Baguio City. However, PFIZER, as the employer, offered no reason or justification for the relocation such as the
filling up of respondent’s former position and the unavailability of substantially equivalent position in Baguio City. A
Angeles| Bajana | Balladares | Brillantes | Briones | Cabansag | Callanta | Chua | David|

De Leon | Gomez | Lopez | Macalino | Nostratis | Padilla | Reynon | Santos | Tan |Velasco
4E / 4F - 2018-2019
Page 390 of 920
LABOR REVIEW DIGEST
Atty. Joyrich Golangco

transfer of work assignment without any justification therefor, even if respondent would be presumably doing the
same job with the same pay, cannot be deemed faithful compliance with the reinstatement order. In other words,
in this instance, there was no real, bona fide reinstatement to speak of prior to the reversal by the Court of
Appeals of the finding of illegal dismissal.

2. NO. At the outset, we note that PFIZER’s previous payment to respondent of the amount of ₱1,963,855.00
(representing her wages from December 5, 2003, or the date of the Labor Arbiter decision, until May 5, 2005) that
was successfully garnished under the Labor Arbiter’s Writ of Execution dated May 26, 2005 cannot be considered
in its favor. Not only was this sum legally due to respondent under prevailing jurisprudence but also this
circumstance highlighted PFIZER’s unreasonable delay in complying with the reinstatement order of the Labor
Arbiter. A perusal of the records, including PFIZER’s own submissions, confirmed that it only required respondent
to report for work on July 1, 2005, as shown by its Letter20 dated June 27, 2005, which is almost two years from
the time the order of reinstatement was handed down in the Labor Arbiter’s Decision dated December 5, 2003.

An award or order of reinstatement is immediately self-executory without the need for the issuance of a writ of
execution. The provision of Article 223 is clear that an award [by the Labor Arbiter] for reinstatement shall be
immediately executory even pending appeal and the posting of a bond by the employer shall not stay the
execution for reinstatement. The legislative intent is quite obvious, i.e., to make an award of reinstatement
immediately enforceable, even pending appeal. To require the application for and issuance of a writ of execution
as prerequisites for the execution of a reinstatement award would certainly betray and run counter to the very
object and intent of Article 223, i.e., the immediate execution of a reinstatement order.

Even if the order of reinstatement of the Labor Arbiter is reversed on appeal, it is obligatory on the part of the
employer to reinstate and pay the wages of the dismissed employee during the period of appeal until reversal by
the higher court. On the other hand, if the employee has been reinstated during the appeal period and such
reinstatement order is reversed with finality, the employee is not required to reimburse whatever salary he
received for he is entitled to such, more so if he actually rendered services during the period.

Angeles| Bajana | Balladares | Brillantes | Briones | Cabansag | Callanta | Chua | David|

De Leon | Gomez | Lopez | Macalino | Nostratis | Padilla | Reynon | Santos | Tan |Velasco
4E / 4F - 2018-2019
Page 391 of 920
LABOR REVIEW DIGEST
Atty. Joyrich Golangco

10. WENPHIL v. ABING April 7, 2014


G.R. No. 207983
Ponente: Brion, J.:

DOCTRINE: The period for computing the backwages due to the respondents during the period of appeal should
end on the date that a higher court reversed the labor arbitration ruling of illegal dismissal.

FACTS: This case stemmed from a complaint for illegal dismissal filed by the respondents against Wenphil,
docketed as NLRC NCR Case No. 30-03-00993-00. LA Bartolabac ruled that the respondents had been illegally
dismissed by Wenphil. According to the LA, the allegation of serious misconduct against the respondents had no
factual and legal basis. Consequently, LA Bartolabac ordered Wenphil to immediately reinstate the respondents to
their respective positions or to equivalent ones, whether actuall or in the payroll. Also, the LA ordered Wenphil to
pay the respondents their backwages until the date of their actual reinstatement.

Wenphil appealed to the NLRC.

In the meantime, the respondents moved for the immediate execution of the LA’s decision. NLRC issued a
resolution affirming LA Bartolabac’s decision with modifications. Instead of ordering the respondents’
reinstatement, the NLRC directed Wenphil to pay the respondents their respective separation pay at the rate of
one (1) month salary for every year of service. Also, the NLRC found that while the respondents had been illegally
dismissed, they had not been illegally suspended. Thus, the period from February 3 to February 28, 2000 during
which the respondents were on preventive suspension – was excluded by the NLRC in the computation of the
respondents’ backwages.

CA rendered its decision reversing the NLRC’s finding that the respondents had been illegally dismissed.
According to the CA, there was enough evidence to show that the respondents had been guilty of serious
misconduct; thus, their dismissal was for a valid cause.

SC, in G.R. No. 162447, denied the respondents petition for review on certiorari and affirmed the CA’s decision
and resolution. The respondents did not file any motion for reconsideration to question the SC’s decision; thus, the
decision became final and executory.

Sometime after the SC’s decision in G.R. No. 162447 became final and executory, the respondents filed with LA
Bartolabac a motion for computation and issuance of writ of execution. The respondents asserted in this motion
that although the CA’s ruling on the absence of illegal dismissal (as affirmed by the SC) was adverse to them,
under the law and settled jurisprudence, they were still entitled to backwages from the time of their dismissal until
the NLRC’s decision finding them to be illegally dismissed was reversed with finality.

LA RULING: LA Bartolabac granted the respondents’ motion and, in an order, directed Wenphil to pay each
complainant their salaries on reinstatement covering the period from Feb 15, 2002, the date Wenphil last paid the
respondents’ respective salaries, until Nov. 8, 2002 when the NLRC’s decision finding the respondents illegally
dismissed became final and executory.

NLRC RULING: Affirmed LA

CA RULING: CA, in setting aside the NLRC’s rulings, relied on the case of Pfizer v. Velasco (G.R. No. 177467,
March 9, 2011, 645 SCRA 135) where the Supreme Court ruled that the backwages of the dismissed employee
should be granted during the period of appeal until reversal by a higher court. Since the first CA decision that
found the respondents had not been illegally dismissed was promulgated on Aug. 27, 2003, then the reversal by
the higher court was effectively made on Aug. 27, 2003
Angeles| Bajana | Balladares | Brillantes | Briones | Cabansag | Callanta | Chua | David|

De Leon | Gomez | Lopez | Macalino | Nostratis | Padilla | Reynon | Santos | Tan |Velasco
4E / 4F - 2018-2019
Page 392 of 920
LABOR REVIEW DIGEST
Atty. Joyrich Golangco

ISSUE: Which computation is correct, the LA’s or the CA’s?

HELD: That of CA.

Among these views, the commanding one is the rule in Pfizer, which merely echoes the rulings the Supreme
Court (SC) made in the cases of Roquero v. Philippine Airlines (G.R. No. 152329, 449 Phil. 437 (2003)) and
Garcia v. Philippine Airlines (G.R. No. 164856, January 20, 2009, 576 SCRA 479) that the period for computing
the backwages due to the respondents during the period of appeal should end on the date that a higher court
reversed the labor arbitration ruling of illegal dismissal. In this case, the higher court that first reversed the NLRC’s
ruling was not the SC but rather the CA. In this light, the CA was correct when it found that that the period of
computation should end on Aug. 27, 2003. The date when the SC’s decision became final and executory need not
matter as the rule in Roquero, Garcia and Pfizer merely referred to the date of reversal, not the date of the
ultimate finality of such reversal.

As a last minor detail, we do not agree with the CA that the date of computation should start on Feb. 15, 2002.
Rather, it should be on Feb. 16, 2002. The respondents themselves admitted in their motion for computation and
issuance of writ of execution that the last date when they were paid their backwages was on Feb. 15, 2002. To
start the computation on the same date would result to a duplication of wages for this day; thus, computation
should start on the following date – Feb. 16, 2002.

Angeles| Bajana | Balladares | Brillantes | Briones | Cabansag | Callanta | Chua | David|

De Leon | Gomez | Lopez | Macalino | Nostratis | Padilla | Reynon | Santos | Tan |Velasco
4E / 4F - 2018-2019
Page 393 of 920
LABOR REVIEW DIGEST
Atty. Joyrich Golangco

11. Smarticle Communicatons v. Solidum (Santos)

Angeles| Bajana | Balladares | Brillantes | Briones | Cabansag | Callanta | Chua | David|

De Leon | Gomez | Lopez | Macalino | Nostratis | Padilla | Reynon | Santos | Tan |Velasco
4E / 4F - 2018-2019
Page 394 of 920
LABOR REVIEW DIGEST
Atty. Joyrich Golangco

ARTICLE 230
1. Sy, et al. v. Fairland Knitcraft Co.
G.R. No. 182915, December 12, 2011
Del Castillo J.

DOCTRINE:
Angeles| Bajana | Balladares | Brillantes | Briones | Cabansag | Callanta | Chua | David|

De Leon | Gomez | Lopez | Macalino | Nostratis | Padilla | Reynon | Santos | Tan |Velasco
4E / 4F - 2018-2019
Page 395 of 920
LABOR REVIEW DIGEST
Atty. Joyrich Golangco

When a party to a suit appears by counsel, service of every judgment and all orders of the court must be sent to
the counsel. This is so because notice to counsel is an effective notice to the client, while notice to the client and
not his counsel is not notice in law. Receipt of notice by the counsel of record is the reckoning point of the
reglementary period.

FACTS:

These are two cosolidated petitions involcing Fairland, Weesan, and the workers hired by Weesan. Fairland is a domestic
corporation engaged in garments business, while Susan de Leon is the owner/proprietress of Weesan Garments. On the
other hand, the complaining workers are sewers, trimmers, helpers, a guard and a secretary who were hired by Weesan.

On December 23, 2002, workers Sy, Penullar, Aguinaldo, Aniano, Dela Pea and Matias filed with the Arbitration Branch of the
NLRC a Complaint for underpayment and/or non-payment of wages, overtime pay, premium pay for holidays, 13th month pay
and other monetary benefits against Susan/Weesan. In January 2003, other workers also filed similar complaints. Eventually
all the cases were consolidated as they involved the same causes of action.

Thereafter, Weesan filed before the DOLE-NCR a report on its temporary closure for a period of not less than six
months. The workers amended their Complaint to include the charge of illegal dismissal. They impleaded Fairland and its
manager, Debbie Manduabas (Debbie), as additional respondents.

LA and NLRC RULING:

The LA found that the allegations leveled against Ledesma are mere concoctions, and concluded that Ledesma
was illegally dismissed.

On appeal to the NLRC, the latter reversed the ruling of the LA and held that Ledesma’s acts of sexual overtures
to Christe Mandal and Rosanna Lofranco constituted grave misconduct justifying his dismissal from employment.
The NLRC denied Ledesma’s motion for reconsideration.

CA RULING in CA-G.R. SP No. 93204:

On July 25, 2007, the CA’s First Division denied Fairlands petition.

Fairland filed its Motion for Reconsideration as well as a Motion for Voluntary Inhibition of Associate Justices Celia C. Librea-
Leagogo and Regalado E. Maambong from handling the case. As the Motion for Voluntary Inhibition was granted, the case
was transferred to the CAs Special Ninth Division.

The CAs Special Ninth Division reversed the First Divisions ruling. It held that the labor tribunals did not acquire jurisdiction over
the person of Fairland, and even assuming they did, Fairland is not liable to the workers since Weesan is not a mere labor-only
contractor but a bona fide independent contractor. The Special Ninth Division thus annulled and set aside the assailed NLRC
Decision and Resolution insofar as Fairland is concerned and excluded the latter therefrom.

Aggrieved, the workers filed before the SC their Petition for Review on Certiorari.

CA RULING in CA-G.R. SP No. 93860:

With regard to Susan’s petition, the CA Special Ninth Division issued a Resolution temporarily restraining the NLRC from
enforcing its assailed Decision and thereafter the CA Special Eighth Division issued a writ of preliminary prohibitory injunction.

Angeles| Bajana | Balladares | Brillantes | Briones | Cabansag | Callanta | Chua | David|

De Leon | Gomez | Lopez | Macalino | Nostratis | Padilla | Reynon | Santos | Tan |Velasco
4E / 4F - 2018-2019
Page 396 of 920
LABOR REVIEW DIGEST
Atty. Joyrich Golangco

On July 20, 2009, the Special Former Special Eighth Division of the CA denied the petition. Susan moved for reconsideration
which was denied by the CA.

Hence, she filed before the SC a Petition for Review on Certiorari which was denied. Susan and Fairland filed their respective
Motions for Reconsideration. But before said motions could be resolved, the Court ordered the consolidation of Susans petition
with that of the workers.
ISSUE/S:

In G.R. No. 189658:

WON Weesan is a labor-only contractor.


WON respondents were illegally dismissed.

In G.R. No. 182915:

Whether the NLRC acquired jurisdiction over the respondent.


Whether the NLRC decision became final and executory.
Whether respondent is solidarily liable with Weesan and Susan De Leon.

HELD:

 The SC granted the workers petition (G.R. No. 182915) but denied the petition of Susan (G.R. No. 189658).

 Susan/Weesan is a mere labor-only contractor: Here, there is no question that the workers, majority of whom are sewers,
were recruited by Susan/Weesan and that they performed activities which are directly related to Fairlands principal business of
garments. What must be determined is whether Susan/Weesan has substantial capital or investment in the form of tools,
equipment, machineries, work premises, among others.

We have examined the records but found nothing therein to show that Weesan has investment in the form of tools, equipment
or machineries. The records show that Fairland has to furnish Weesan with sewing machines for it to be able to provide the
sewing needs of the former. Also, save for the Balance Sheets purportedly submitted by Weesan to the BIR indicating its fixed
assets in the amount of P243,000.00, Weesan was unable to show that apart from the borrowed sewing machines, it owned
and possessed any other tools, equipment, and machineries necessary to its being a contractor or sub-contractor for
garments. Neither was Weesan able to prove that it has substantial capital for its business.

Likewise significant is the fact that there is doubt as to who really owns the work premises occupied by Weesan. As may be
recalled, the workers emphasized in their Appeal Memorandum filed with the NLRC that Susan/Weesan was a labor-only
contractor and that Fairland was its principal. To buttress this, they alleged that the work premises utilized by Weesan is owned
by Fairland, which significantly, was not in the business of renting properties. They also advanced that there was no showing
that Susan/Weesan paid any rentals for the use of the
premises.They contended that all thatSusan had was a Mayors Permit for Weesan indicating 715 Ricafort Street,
Tondo, Manila as its address.

 The National Labor Relations Commission and the Court of Appeals did not err in their findings of illegal dismissal:
Indeed, Article 283 of the Labor Code allows as a mode of termination of employment the closure or termination of business.
Closure or cessation of business is the complete or partial cessation of the operations and/or shut-down of the establishment
of the employer. It is carried out to either stave off the financial ruin or promote the business interest of the employer. The
decision to close business [or to temporarily suspend operation] is a management prerogative exclusive to the employer, the
exercise of which no court or tribunal can meddle with, except only when the employer fails to prove compliance with the
Angeles| Bajana | Balladares | Brillantes | Briones | Cabansag | Callanta | Chua | David|

De Leon | Gomez | Lopez | Macalino | Nostratis | Padilla | Reynon | Santos | Tan |Velasco
4E / 4F - 2018-2019
Page 397 of 920
LABOR REVIEW DIGEST
Atty. Joyrich Golangco

requirements of Art. 283, to wit: a) that the closure/cessation of business is bona fide, i.e., its purpose is to advance the interest
of the employer and not to defeat or circumvent the rights of employees under the law or a valid agreement; b) that written
notice was served on the employees and the DOLE at least one month before the intended date of closure or cessation of
business; and c) in case of closure/cessation of business not due to financial losses, that the employees affected have been
given separation pay equivalent to month pay for every year of service or one month pay, whichever is higher.

Here, Weesan filed its Establishment Termination Report allegedly due to serious business losses and other economic
reasons. However, we are mindful of the doubtful character of Weesans application for closure given the circumstances
surrounding the same.

It bears stressing that [t]he burden of proving that x x x a temporary suspension is bona fide falls upon the employer. Clearly
here, Susan/Weesan was not able to discharge this burden. The documents Weesan submitted to support its claim of severe
business losses cannot be considered as proof of financial crisis to justify the temporary suspension of its operations since they
clearly appear to have not been duly filed with the BIR. Weesan failed to satisfactorily explain why the Income Tax Returns and
financial statements it submitted do not bear the signature of the receiving officers. Also hard to ignore is the absence of the
mandatory 30-day prior notice to the workers.

 Although not served with summons, jurisdiction over Fairland and Debbie was acquired through their voluntary
appearance: From the records, it appears that Atty. Geronimo first entered his appearance on behalf of Susan/Weesan in the
hearing held on April 3, 2003. Being then newly hired, he requested for an extension of time within which to file a position paper
for said respondents. On the next scheduled hearing on April 28, 2003, Atty. Geronimo again asked for another extension to
file a position paper for all the respondents considering that he likewise entered his appearance for Fairland. Thereafter, said
counsel filed pleadings such as Respondents Position Paper and Respondents Consolidated Reply on behalf of all the
respondents namely, Susan/Weesan, Fairland and Debbie. The fact that Atty. Geronimo entered his appearance for Fairland
and Debbie and that he actively defended them before the Labor Arbiter raised the presumption that he is authorized to appear
for them.As held in Santos, it is unlikely that Atty. Geronimo would have been so irresponsible as to represent Fairland and
Debbie if he were not in fact authorized. As an officer of the Court, Atty. Geronimo is presumed to have acted with due
propriety. Moreover, [i]t strains credulity that a counsel who has no personal interest in the case would fight for and defend a
case with persistence and vigor if he has not been authorized or employed by the party concerned.

 Fairland is Weesan's principal: In addition to our discussion in G.R. No. 189658 with respect to the finding that
Susan/Weesan is a mere labor-only contractor which we find to be likewise significant here, a careful examination of the
records reveals other telling facts that Fairland is Susan/Weesans principal, to wit: (1) aside from sewing machines, Fairland
also lent Weesan other equipment such as fire extinguishers, office tables and chairs, and plastic chairs; (2) no proof
evidencing the contractual arrangement between Weesan and Fairland was ever submitted by Fairland; (3) while both
Weesan and Fairland assert that the former had other clients aside from the latter, no proof of Weesans contractual
relationship with its other alleged client is extant on the records; and (4) there is no showing that any of the workers were
assigned to other clients aside from Fairland.Moreover, as found by the NLRC and affirmed by both the Special Former
Special Eighth Division in CA-G.R. SP No. 93860 and the First Division in CA-G.R. SP No. 93204, the activities, the manner of
work and the movement of the workers were subject to Fairlands control. It bears emphasizing that factual findings of quasi-
judicial agencies like the NLRC, when affirmed by the Court of Appeals, as in the present case, are conclusive upon the parties
and binding on this Court.

Fairland, therefore, as the principal employer, is solidarily liable with Susan/Weesan, the labor-only contractor, for the rightful
claims of the employees. Under this set-up, Susan/Weesan, as the "labor-only" contractor, is deemed an agent of the principal,
Fairland, and the law makes the principal responsible to the employees of the "labor-only" contractor as if the principal itself
directly hired or employed the employees.

Angeles| Bajana | Balladares | Brillantes | Briones | Cabansag | Callanta | Chua | David|

De Leon | Gomez | Lopez | Macalino | Nostratis | Padilla | Reynon | Santos | Tan |Velasco
4E / 4F - 2018-2019
Page 398 of 920
LABOR REVIEW DIGEST
Atty. Joyrich Golangco

2. YUPANGCO COTTON MILLS, INC. vs. COURT OF APPEALS


G.R. No. 126322; January 16, 2002
PARDO, J.

DOCTRINE: A third party whose property has been levied upon by a sheriff to enforce a decision against a judgment debtor is afforded
with several alternative remedies to protect its interests. The third party may avail himself of alternative remedies cumulatively, and one will
not preclude the third party from availing himself of the other alternative remedies in the event he failed in the remedy first availed of.

FACTS:

Petitioner contended that a sheriff of the NLRC “erroneously and unlawfully levied” certain properties which it claims as its own. It filed a 3rd
party claim with the Labor Arbiter and recovery of property and damages with the RTC. The RTC dismissed the case. In the CA, the court
dismissed the petition on the ground of forums hopping and that the proper remedy was appeal in due course, not certiorari or mandamus.
Petitioner filed a MFR and argued that the filing of a complaint for accion reinvindicatoria with the RTC was proper because it is a remedy
specifically granted to an owner (whose properties were subjected to a writ of execution to enforce a decision rendered in a labor dispute in
which it was not a party). The MFR was denied. Hence, petitioner filed this appeal.

ISSUE:
Whether or not the CA has jurisdiction over the case.

RULING:

A third party whose property has been levied upon by a sheriff to enforce a decision against a judgment debtor is afforded with several
alternative remedies to protect its interests. The third party may avail himself of alternative remedies cumulatively, and one will not preclude
the third party from availing himself of the other alternative remedies in the event he failed in the remedy first availed of. Thus, a third party
may avail himself of the following alternative remedies: a) File a third party claim with the sheriff of the Labor Arbiter, and b) If the third party
claim is denied, the third party may appeal the denial to the NLRC. Even if a third party claim was denied, a third party may still file a proper
action with a competent court to recover ownership of the property illegally seized by the sheriff. The filing of a third party claim with the Labor
Arbiter and the NLRC did not preclude the petitioner from filing a subsequent action for recovery of property and damages with the Regional
Trial Court. And, the institution of such complaint will not make petitioner guilty of forum shopping.

Angeles| Bajana | Balladares | Brillantes | Briones | Cabansag | Callanta | Chua | David|

De Leon | Gomez | Lopez | Macalino | Nostratis | Padilla | Reynon | Santos | Tan |Velasco
4E / 4F - 2018-2019
Page 399 of 920
LABOR REVIEW DIGEST
Atty. Joyrich Golangco

3. ANDO v. CAMPO
G.R. No. 184007 February 16, 2011
NACHURA, J.

Doctrine: Regular courts have no jurisdiction to hear and decide questions which arise from and are
incidental to the enforcement of decisions, orders, or awards rendered in labor cases by appropriate
officers and tribunals of the Department of Labor and Employment. It is, first and foremost, the NLRC
Manual on the Execution of Judgment that governs any question on the execution of a judgment of that
body. However, the power of the NLRC, or the courts, to execute its judgment extends only to
properties unquestionably belonging to the judgment debtor alone.

FACTS:
Petitioner was the president of Premier Allied and Contracting Services, Inc. (PACSI), an independent
labor contractor. Respondents were hired by PACSI as pilers or haulers tasked to manually carry bags
of sugar from the warehouse of Victorias Milling Company and load them on trucks. On June 1998,
respondents were dismissed from employment. They filed a case for illegal dismissal and with money
claims.
Labor Arbiter Pura promulgated a decision, ruling in respondents’ favor. PACSI and petitioner were
directed to pay a total of P422, 702.28, representing respondents’ separation pay and the award of
attorney’s fees. Petitioner and PACSI appealed to the NLRC. The NLRC ruled that petitioner failed to
perfect his appeal because he did not pay the supersedeas bond. It also affirmed the Labor Arbiter’s
decision with modification of the award for separation pay to four other employees who were similarly
situated. Upon finality of the decision, respondents moved for its execution. To answer for the monetary
award, the NLRC Acting Sheriff issued a Notice of Sale on Execution of Personal Property over the
property in the name of "Paquito V. Ando x x x married to Erlinda S. Ando."
This prompted petitioner to file an action for prohibition and damages with prayer for the issuance of a
TRO before the Regional Trial Court of Bacolod City. Petitioner claimed that the property belonged to
him and his wife, not to the corporation, and, hence, could not be subject of the execution sale. Since it
is the corporation that was the judgment debtor, execution should be made on properties.

RTC RULING: The RTC issued an Order denying the prayer for a TRO, holding that the trial court had
no jurisdiction to try and decide the case. The RTC ruled that, pursuant to the NLRC Manual on the
Execution of Judgment, petitioner’s remedy was to file a third-party claim with the NLRC Sheriff. Despite
lack of jurisdiction, however, the RTC went on to decide the merits of the case. Petitioner did not file a
motion for reconsideration of the RTC Order. Instead, he filed a petition for certiorari under Rule 65
before the CA.

CA RULING: The CA affirmed the RTC Order in so far as it dismissed the complaint on the ground that
it had no jurisdiction over the case, and nullified all other pronouncements in the same Order.

PETITION TO THE SC:

ISSUE/S:
1. Whether or not regular courts have jurisdiction over the enforcement of decisions, orders or awards
rendered in labor cases?

HELD:
1. None. NO. The Court has long recognized that regular courts have no jurisdiction to hear and decide
questions which arise from and are incidental to the enforcement of decisions, orders, or awards
Angeles| Bajana | Balladares | Brillantes | Briones | Cabansag | Callanta | Chua | David|

De Leon | Gomez | Lopez | Macalino | Nostratis | Padilla | Reynon | Santos | Tan |Velasco
4E / 4F - 2018-2019
Page 400 of 920
LABOR REVIEW DIGEST
Atty. Joyrich Golangco

rendered in labor cases by appropriate officers and tribunals of the Department of Labor and
Employment. To hold otherwise is to sanction splitting of jurisdiction which is obnoxious to the orderly
administration of justice. Thus, it is, first and foremost, the NLRC Manual on the Execution of Judgment
that governs any question on the execution of a judgment of that body. Petitioner need not look further
than that. The Rules of Court apply only by analogy or in a suppletory character. The NLRC Manual on
the Execution of Judgment deals specifically with third-party claims in cases brought before that body. It
defines a third-party claim as one where a person, not a party to the case, asserts title to or right to
the possession of the property levied upon. It also sets out the procedure for the filing of a third-party
claim. There is no doubt in our mind that petitioner’s complaint is a third- party claim within the
cognizance of the NLRC. Petitioner may indeed be considered a "third party" in relation to the property
subject of the execution vis-à-vis the Labor Arbiter’s decision. There is no question that the property
belongs to petitioner and his wife, and not to the corporation. It can be said that the property belongs to
the conjugal partnership, not to petitioner alone. Thus, the property belongs to a third party, i.e., the
conjugal partnership. At the very least, the Court can consider that petitioner’s wife is a third party within
contemplation of the law. The broad powers granted to the Labor Arbiter and to the National Labor
Relations Commission by Articles 217, 218 and 224 of the Labor Code can only be interpreted as
vesting in them jurisdiction over incidents arising from, in connection with or relating to labor disputes, as
the controversy under consideration, to the exclusion of the regular courts. There is no denying that the
present controversy arose from the complaint for illegal dismissal. The subject matter of petitioner’s
complaint is the execution of the NLRC decision. Execution is an essential part of the proceedings
before the NLRC. Jurisdiction, once acquired, continues until the case is finally terminated, and there can
be no end to the controversy without the full and proper implementation of the commission’s directives.

Petitioner claims that the property sought to be levied does not belong to PACSI, the judgment debtor,
but to him and his wife. Since he was sued in a representative capacity, and not in his personal
capacity, the property could not be made to answer for the judgment obligation of the corporation.
Moreover, the power of the NLRC, or the courts, to execute its judgment extends only to properties
unquestionably belonging to the judgment debtor alone. A sheriff, therefore, has no authority to attach
the property of any person except that of the judgment debtor.

Angeles| Bajana | Balladares | Brillantes | Briones | Cabansag | Callanta | Chua | David|

De Leon | Gomez | Lopez | Macalino | Nostratis | Padilla | Reynon | Santos | Tan |Velasco
4E / 4F - 2018-2019
Page 401 of 920
LABOR REVIEW DIGEST
Atty. Joyrich Golangco

4. PAL v. Bichara
DOCTRINE: Where the writ of execution is not in harmony with and exceeds the judgment which gives it life, the
writ has pro tanto no validity. A companion to this rule is the principle of immutability of final judgments, which
states that a final judgment may no longer be altered, amended or modified, even if the alteration, amendment or
modification is meant to correct what is perceived to be an erroneous conclusion of fact or law and regardless of
what court renders it.
FACTS: On October 28, 1968, PAL hired Bichara as a flight attendant. Sometime in 1971, PAL implemented a
retrenchment program. By April of that year, Bichara voluntarily resigned. On May 15, 1975, he was rehired. In
1993, he was included in PAL's Purser Upgrading Program. As flight purser, he was required to take five (5) check
rides for his performance evaluation and earn at least an 85% rating for each ride. However, Bichara failed in the
two (2) check rides(83.46% and 80.63%). Consequently, he was demoted to flight steward. He appealed his
demotion to PAL, but no action was taken; hence, he filed a complaint for illegal demotion before the NLRC. LA
issued a decision declaring the demotion as illegal, and accordingly, ordered PAL to reinstate him to his position
as flight purser. PAL filed an appeal before the NLRC and later before the CA, both of which, however, upheld
LA’s finding.
During the pendency of the illegal demotion case, PAL implemented retrenchment program that resulted in the
termination of Bichara's employment. This prompted him, along with more than 1,400 other attendants,
represented by the Flight Attendants and Stewards Association of the Philippines (FASAP), to file a complaint for
unfair labor practice, illegal retrenchment with claims for reinstatement against PAL. This case, which was
appealed all the way to the SC, remains pending as of this time. On July 9, 2005, Bichara reached the 60
year-old compulsory retirement age under the PAL-FASAP CBA. On January 31, 2008, Bichara filed a motion
for execution of LA’s June 16, 1997 Decision, which PAL opposed by arguing that the "complaint for illegal
demotion x x x was overtaken by supervening events, i.e., the retrenchment of [Bichara] in 1998 and his having
reached [the] compulsory retirement age in 2005."
LA granted Bichara's motion for execution declaring that, notwithstanding the pendency before this Court of the
illegal retrenchment case, Bichara's termination was invalid, given that: (a) PAL did not use a fair and reasonable
criteria in effecting the retrenchment; (b) PAL disregarded the labor arbiters' rulings in the illegal demotion and
illegal retrenchment cases which were both immediately executory; and (c) retrenchment was made during the
pendency of the illegal demotion case without the permission of the court where the case was pending.
NLRC reversed and set aside LA’s Order and denied the motion for execution for being moot and academic,
considering Bichara's compulsory retirement in 2005, without prejudice to the latter's entitlement to backwages
and retirement benefits of a flight steward pursuant to the FASAP case. NLRC ruled that Bichara's reinstatement
could have taken effect, if at all, only on January 31, 2008 when he sought the execution of the said relief. It
declared that reinstatement is no longer possible as the same was rendered moot and academic when he
compulsorily retired in 2005.
CA reversed and set aside the NLRC's ruling stating that when reinstatement is not possible due to over age,
payment of separation pay is in place. It also ruled that the retrenchment was illegal and thereby stated that
"[f]light attendants who have reached their compulsory retirement age of retirement shall receive backwages up to
the date of their retirement only," should be made to apply. Thus, instead of separation pay, Bichara is entitled to
backwages from the time of his retrenchment up to the time he reached the compulsory retirement age of 60.
ISSUE: Whether or not the CA erred in reversing the NLRC's Decision and thereby awarding Bichara the
aforementioned monetary awards.
HELD: The petition is partly meritorious. A judgment should be implemented according to the terms of its
dispositive portion is a long and well-established rule. As such, where the writ of execution is not in harmony
with and exceeds the judgment which gives it life, the writ has pro tanto no validity. A companion to this
rule is the principle of immutability of final judgments, which states that a final judgment may no longer be
altered, amended or modified, even if the alteration, amendment or modification is meant to correct what is
perceived to be an erroneous conclusion of fact or law and regardless of what court renders it. Any attempt to
insert, change or add matters not clearly contemplated in the dispositive portion violates the rule on immutability of
judgments. But like any other rule, this principle has exceptions, namely: (1) the correction of clerical errors; (2)
Angeles| Bajana | Balladares | Brillantes | Briones | Cabansag | Callanta | Chua | David|

De Leon | Gomez | Lopez | Macalino | Nostratis | Padilla | Reynon | Santos | Tan |Velasco
4E / 4F - 2018-2019
Page 402 of 920
LABOR REVIEW DIGEST
Atty. Joyrich Golangco

the so-called nunc pro tunc entries which cause no prejudice to any party; (3) void judgments; and (4) whenever
circumstances transpire after the finality of the decision rendering its execution unjust and inequitable.

In this case, the final judgment sought to be executed is LA's June 16, 1997 Decision, which was confined to the
directive that PAL reinstate Bichara as a flight purser in view of his illegal demotion.

Evidently, LA went beyond the terms of the June 16, 1997 Decision when he, in his February 4, 2009 Order,
directed the issuance of a writ of execution ordering the payment of separation pay in lieu of reinstatement.
Unlike the cases cited by the CA, which all involved illegal dismissal cases, it would not be proper to accord such
relief in this case since, in those cases, the awards of separation pay in lieu of reinstatement were all hinged on
the validity of the employee's dismissal. Here, the validity of Bichara's termination is the subject matter of a
separate case, i.e., the FASAP case, which is still pending before this Court, and is also beyond the ambit of the
illegal demotion proceedings. Hence, LA exceeded his authority when he ruled on this issue and directed PAL to
pay Bichara separation pay in lieu of reinstatement.

PAL's supervening retrenchment of its employees, which included Bichara, in July 1998, and his compulsory
retirement in July 2005, however, prevent the enforcement of the reinstatement of Bichara to the position of flight
purser under the June 16, 1997 Decision. Nonetheless, since this Decision had already settled the illegality of
Bichara's demotion with finality, this Court finds that Bichara should, instead, be awarded the salary
differential of a flight purser from a flight steward from the time of his illegal demotion on March 21, 1994 up
until the time he was retrenched in July 1998. Notably, unlike LA's award of separation pay in lieu of
reinstatement, the award of salary differential is not dependent on the validity of his termination, as it is, in fact,
intrinsically linked to the illegality of Bichara's demotion. Hence, with this direct relation, there should be no
obstacle in rendering this award.

Further, it should be pointed out that the principle of immutability of judgments, from which the above-stated rule
on writ of executions proceed, allow courts, as an exception, to recognize circumstances that transpire after the
finality of the decision which would render its execution unjust and inequitable and act accordingly. Thus, in view
of the supervening events above-mentioned, this Court deems the award of salary differential to be the just and
equitable award under the circumstances herein prevailing. Jurisprudence holds that courts may modify or alter
the judgment to harmonize the same with justice and the facts when after judgment has been rendered and the
latter has become final, facts and circumstances transpire which render its execution impossible or unjust, as in
this case.

As a last point, it deserves mentioning that since Bichara's illegal demotion has been finally decreed, he should be
entitled to (a) backwages, at the salary rate of a flight purser, from the time of retrenchment in July 1998 up until
his compulsory retirement in July 2005; (b) retirement benefits of a flight purser in accordance with the existing
CBA at the time of Bichara's retirement; and (c) attorney's fees, moral, and exemplary damages, if any, but only
if this Court, in the FASAP case, finally rules that the subject retrenchment is invalid. Otherwise, he should
only be entitled to the above-stated salary differential, as well as the corresponding separation pay required under
the relevant CBA, or Article 297 (formerly Article 283) of the Labor Code if no such CBA provision exists. The
awards of backwages, and retirement benefits, including attorney's fees, moral, and exemplary damages, if any,
cannot, however, be executed in these proceedings since they are incidents which pertain to the illegal
retrenchment case, hence, executable only when the FASAP case is finally concluded.

Angeles| Bajana | Balladares | Brillantes | Briones | Cabansag | Callanta | Chua | David|

De Leon | Gomez | Lopez | Macalino | Nostratis | Padilla | Reynon | Santos | Tan |Velasco
4E / 4F - 2018-2019
Page 403 of 920
LABOR REVIEW DIGEST
Atty. Joyrich Golangco

5. JOSE GUILLERMO vs. CRISANTO USON


GR No: 198967 Date: March 07, 2016
Ponente: Peralta, J.

Doctrine:
FACTS: Respondent Crisanto P. Uson (Uson) began his employment with Royal Class Venture Phils., Inc. (Royal
Class Venture) as an accounting clerk. He was promoted to the position of accounting supervisor, with a salary of
Phpl3, 000.00 a month, until he was allegedly dismissed from employment on December 20, 2000. Uson filed with
the Sub-Regional Arbitration Branch No. 1, Dagupan City, of the NLRC a Complaint for Illegal Dismissal, with
prayers for backwages, reinstatement, salaries and 13th month pay, moral and exemplary damages and attorney's
fees against Royal Class Venture.
Royal Class Venture did not make an appearance in the case despite its receipt of summons.
Labor Arbiter decided in favor of the complainant Uson and ordered Respondent Royal Class Venture to reinstate
him to his former position and pay his backwages, 13th month pay as well as moral and exemplary damages and
attorney's fees.
Royal Class Venture, as the losing party, did not file an appeal of the decision. Upon Uson's motion, a Writ of
Execution was issued to implement the Labor Arbiter's decision. An Alias Writ of Execution was issued. But with
the judgment still unsatisfied, a Second Alias Writ of Execution was issued. Uson again, filed a Motion for Alias
Writ of Execution and to Hold Directors and Officers of Respondent Liable for Satisfaction of the decision.
Labor Arbiter issued an Order granting the motion filed by Uson. The order held that officers of a corporation are
jointly and severally liable for the obligations of the corporation to the employees and there is no denial of due
process in holding them so even if the said officers were not parties to the case when the judgment in favor of the
employees was rendered.
Thus, the Labor Arbiter pierced the veil of corporate fiction of Royal Class Venture and held herein petitioner Jose
Emmanuel Guillermo (Guillermo), in his personal capacity, jointly and severally liable with the corporation for the
enforcement of the claims of Uson. Guillermo filed, by way of special appearance, a Motion for
Reconsideration/To Set Aside the Order.
LA Ruling: The LA sustained the decision of the LAs before her. The LA also castigated Guillermo for his
unexplained absence in the prior proceedings despite notice, effectively putting responsibility on Guillermo for the
case's outcome against him.

NLRC Ruling: The NLRC denied Guillermo’s Memorandum of Appeal with Prayer for Preliminary Injunction.
Hence, Guillermo filed with the CA a Petition for Certiorari assailing the NLRC’s decision.
CA Ruling: The appellate court found that summons was in fact served on Guillermo as President and General
Manager of Royal Class Venture, which was how the Labor Arbiter acquired jurisdiction over the company. But,
Guillermo subsequently refused to receive all notices of hearings and conferences as well as the order to file
Royal Class Venture's position paper. Then, it was learned during execution that Royal Class Venture had been
dissolved. The court held Guillermo liable, citing jurisprudence that hold the president of the corporation liable for
the latter's obligation to illegally dismissed employees.
Guillermo filed a Motion for Reconsideration but was denied. Hence, this petition.
ISSUE: Whether an officer of a corporation may be included as judgment obligor in a labor case for the first time
only after the decision of the Labor Arbiter had become final and executor.
HELD:
The petition is denied.
A corporation is still an artificial being invested by law with a personality separate and distinct from that of its
stockholders and from that of other corporations to which it may be connected.It is not in every instance of inability
to collect from a corporation that the veil of corporate fiction is pierced, and the responsible officials are made
liable. Personal liability attaches only when, as enumerated by the said Section 31 of the Corporation Code, there
is a wilfull and knowing assent to patently unlawful acts of the corporation, there is gross negligence or bad faith in
directing the affairs of the corporation, or there is a conflict of interest resulting in damages to the corporation. The
Angeles| Bajana | Balladares | Brillantes | Briones | Cabansag | Callanta | Chua | David|

De Leon | Gomez | Lopez | Macalino | Nostratis | Padilla | Reynon | Santos | Tan |Velasco
4E / 4F - 2018-2019
Page 404 of 920
LABOR REVIEW DIGEST
Atty. Joyrich Golangco

doctrine of piercing the corporate veil is held to apply only in three (3) basic areas, namely: ( 1) defeat of public
convenience as when the corporate fiction is used as a vehicle for the evasion of an existing obligation; (2) fraud
cases or when the corporate entity is used to justify a wrong, protect fraud, or defend a crime; or (3) alter
ego cases, where a corporation is merely a farce since it is a mere alter ego or business conduit of a person, or
where the corporation is so organized and controlled and its affairs are so conducted as to make it merely an
instrumentality, agency, conduit or adjunct of another corporation. In the absence of malice, bad faith, or a specific
provision of law making a corporate officer liable, such corporate officer cannot be made personally liable for
corporate liabilities.
It also bears emphasis that in cases where personal liability attaches, not even all officers are made accountable.
Rather, only the "responsible officer," i.e., the person directly responsible for and who "acted in bad faith" in
committing the illegal dismissal or any act violative of the Labor Code, is held solidarily liable, in cases wherein the
corporate veil is pierced.
The veil of corporate fiction can be pierced, and responsible corporate directors and officers or even a separate
but related corporation, may be impleaded and held answerable solidarily in a labor case, even after final
judgment and on execution, so long as it is established that such persons have deliberately used the corporate
vehicle to unjustly evade the judgment obligation, or have resorted to fraud, bad faith or malice in doing so. When
the shield of a separate corporate identity is used to commit wrongdoing and opprobriously elude responsibility,
the courts and the legal authorities in a labor case have not hesitated to step in and shatter the said shield and
deny the usual protections to the offending party, even after final judgment. The key element is the presence of
fraud, malice or bad faith. Bad faith, in this instance, does not connote bad judgment or negligence but imports a
dishonest purpose or some moral obliquity and conscious doing of wrong; it means breach of a known duty
through some motive or interest or ill will; it partakes of the nature of fraud.
The records of the present case bear allegations and evidence that Guillermo, the officer being held liable, is the
person responsible in the actual running of the company and for the malicious and illegal dismissal of the
complainant; he, likewise, was shown to have a role in dissolving the original obligor company in an obvious
"scheme to avoid liability" which jurisprudence has always looked upon with a suspicious eye in order to protect
the rights of labor.
As for Guillermo's assertion that the case is an intra-corporate controversy, the Court sustains the finding of the
appellate court that the nature of an action and the jurisdiction of a tribunal are determined by the allegations of
the complaint at the time of its filing, irrespective of whether or not the plaintiff is entitled to recover upon all or
some of the claims asserted therein. Although Uson is also a stockholder and director of Royal Class Venture, it is
settled in jurisprudence that not all conflicts between a stockholder and the corporation are intra-corporate; an
examination of the complaint must be made on whether the complainant is involved in his capacity as a
stockholder or director, or as an employee. If the latter is found and the dispute does not meet the test of what
qualities as an intra-corporate controversy, then the case is a labor case cognizable by the NLRC and is not within
the jurisdiction of any other tribunal.

Angeles| Bajana | Balladares | Brillantes | Briones | Cabansag | Callanta | Chua | David|

De Leon | Gomez | Lopez | Macalino | Nostratis | Padilla | Reynon | Santos | Tan |Velasco
4E / 4F - 2018-2019
Page 405 of 920
LABOR REVIEW DIGEST
Atty. Joyrich Golangco

6. DUTCH MOVERS INC. V LEQUIN, ET AL


G.R. No. 210032, April 25, 2017
Facts:
- Dutch Movers, Inc. (DMI) a domestic corporation engaged in hauling liquefied petroleum gas, employed Lequin as
truck driver and the rest of respondents as helpers.
- On December 28, 2004, Cesar Lee, through the Supervisor Nazario Furio, informed them that DMI would cease
its hauling operation for no reason; as such, they requested DMI to issue a formal notice regarding the matter but
to no avail.
- Later, upon respondents' request, the DOLE NCR issued a certification revealing that DMI did not file any notice
of business closure.
- Thus, respondents argued that they were illegally dismissed as their termination was without cause and only on
the pretext of closure.
- LA Aliman D. Mangandog dismissed the case for lack of cause of action
- NLRC reversed and set aside the LA Decision. It ruled that respondents were illegally dismissed because DMI
simply placed them on standby, and no longer provide them with work
- NLRC Decision became final and executory on December 30, 2007. And, on February 14, 2008, the NLRC issued
an Entry of Judgment on the case
- respondents filed a Motion for Writ of Execution.
- Later, they submitted a Reiterating Motion for Writ of Execution with Updated Computation of Full Backwages.
- Pending resolution of these motions, respondents filed a Manifestation and Motion to Implead stating that upon
investigation, they discovered that DMI no longer operates.
- They, nonetheless, insisted that petitioners - who managed and operated DMI, and consistently represented to
respondents that they were the owners of DMI - continue to work at Toyota Alabang, which they (petitioners) also
own and operate.
- They further averred that the Articles of Incorporation (AOI) of DMI ironically did not include petitioners as its
directors or officers; and those named directors and officers were persons unknown to them. They likewise
claimed that per inquiry with the SEC and the DOLE, they learned that DMI did not file any notice of business
closure
- respondents prayed that petitioners, and the officers named in DMI's AOI, which included Spouses Smith, be
impleaded, and be held solidarity liable with DMI in paying the judgment awards
- Smith alleged that as part of their services as lawyers, they lent their names to petitioners to assist them in
incorporating DMI. Allegedly, after such undertaking, spouses Smith promptly transferred their supposed rights in
DMI in favor of petitioners
- LA Savari issued an Order holding petitioners liable for the judgment awards
- respondents filed anew a Reiterating Motion for Writ of Execution and Approve[d) Updated Computation of Full
Backwages.
- LA Savari issued a Writ of Execution
- Petitioners moved to quash the Writ of Execution contending that the April 1, 2009 LA Order was void because
the LA has no jurisdiction to modify the final and executory NLRC Decision
- LA Savari denied petitioners' Motion to Quash because it did not contain any ground that must be set forth in such
motion.
- NLRC quashed the Writ of Execution insofar as it held petitioners liable to pay the judgment awards
- NLRC ruled that the Writ of Execution should only pertain to DMI since petitioners were not held liable to pay the
awards under the final and executory NLRC Decision.
- CA reversed and set aside the NLRC Resolutions, and accordingly affirmed the Writ of Execution impleading
petitioners as party-respondents liable to answer for the judgment awards.
- CRUX OF THE CASE: The CA ratiocinated that as a rule, once a judgment becomes final and executory, it
cannot anymore be altered or modified; however, an exception to this rule is when there is a supervening
event, which renders the execution of judgment unjust or impossible.
Angeles| Bajana | Balladares | Brillantes | Briones | Cabansag | Callanta | Chua | David|

De Leon | Gomez | Lopez | Macalino | Nostratis | Padilla | Reynon | Santos | Tan |Velasco
4E / 4F - 2018-2019
Page 406 of 920
LABOR REVIEW DIGEST
Atty. Joyrich Golangco

ISSUE:
- Whether petitioners are personally liable to pay the judgment awards in favor of respondents
HELD:
 After a thorough review of the records, the Court finds that contrary to petitioners' claim, Valderrama v.
National Labor Relations Commission, and David v. Court of Appeals are applicable here.
 CRUX OF THE CASE: In said cases, the Court held that the principle of immutability of judgment, or the rule
that once a judgment has become final and executory, the same can no longer be altered or modified and
the court's duty is only to order its execution, is not absolute.
 One of its exceptions is when there is a supervening event occurring after the judgment becomes final and
executory, which renders the decision unenforceable.
 A supervening event refers to facts that transpired after a judgment has become final and executory, or to
new situation that developed after the same attained finality.
 Supervening events include matters that the parties were unaware of before or during trial as they were not
yet existing during that time.
 supervening events transpired in this case after the NLRC Decision became final and executory, which
rendered its execution impossible and unjust.
 Like in Valderrama, during the execution stage, DMI ceased its operation, and the same did not file any
formal notice regarding it.
 Added to this, in their Opposition to the Motion to Implead, spouses Smith revealed that they only lent their
names to petitioners, and they were included as incorporators just to assist the latter in forming DMI; after
such undertaking, spouses Smith immediately transferred their rights in DMI to petitioners, which proved that
petitioners were the ones in control of DMI, and used the same in furthering their business interests.
 In considering the foregoing events, the Court is not unmindful of the basic tenet that a corporation has a
separate and distinct personality from its stockholders, and from other corporations it may be connected
with.
 However, such personality may be disregarded, or the veil of corporate fiction may be pierced attaching
personal liability against responsible person if the corporation's personality "is used to defeat public
convenience, justify wrong, protect fraud or defend crime, or is used as a device to defeat the labor laws x
x x."
 By responsible person, we refer to an individual or entity responsible for, and who acted in bad faith in
committing illegal dismissal or in violation of the Labor Code; or one who actively participated in the
management of the corporation.
 Also, piercing the veil of corporate fiction is allowed where a corporation is a mere alter ego or a conduit of a
person, or another corporation.
 Here, the veil of corporate fiction must be pierced and accordingly, petitioners should be held personally
liable for judgment awards because the peculiarity of the situation shows that they controlled DMI; they actively
participated in its operation such that DMI existed not as a separate entity but only as business conduit of
petitioners.
 As will be shown be shown below, petitioners controlled DMI by making it appear to have no mind of its own, and
used DMI as shield in evading legal liabilities, including payment of the judgment awards in favor of respondents.
 First, petitioners and DMI jointly filed their Position Paper, Reply, and Rejoinder in contesting respondents'
illegal dismissal. Perplexingly, petitioners argued that they were not part of DMI and were not privy to its
dealings; yet, petitioners, along with DMI, collectively raised arguments on the illegal dismissal case against
them.
 Stated differently, petitioners denied having any participation in the management and operation of DMI; however,
they were aware of and disclosed the circumstances surrounding respondents' employment, and propounded
arguments refuting that respondents were illegally dismissed.

Angeles| Bajana | Balladares | Brillantes | Briones | Cabansag | Callanta | Chua | David|

De Leon | Gomez | Lopez | Macalino | Nostratis | Padilla | Reynon | Santos | Tan |Velasco
4E / 4F - 2018-2019
Page 407 of 920
LABOR REVIEW DIGEST
Atty. Joyrich Golangco

 Second, Spouses Smith categorically identified petitioners as the owners and managers of DMI. In their
Motion to Quash, however, petitioners neither denied the allegation of spouses Smith nor adduced evidence to
establish that they were not the owners and managers of DMI. They simply insisted that they could not be
held personally liable because of the immutability of the final and executory NLRC Decision, and of the
separate and distinct personality of DMI.
 Third, piercing the veil of corporate fiction is allowed, and responsible persons may be impleaded, and be held
solidarily liable even after final judgment and on execution, provided that such persons deliberately used
the corporate vehicle to unjustly evade the judgment obligation, or resorted to fraud, bad faith, or malice in
evading their obligation.
 Clearly, petitioners should be held liable for the judgment awards as they resorted to such scheme to
countermand labor laws by causing the incorporation of DMI but without any indication that they were part thereof.
While such device to defeat labor laws may be deemed ingenious and imaginative, the Court will not hesitate to
draw the line, and protect the right of workers to security of tenure, including ensuring that they will receive the
benefits they deserve when they fall victims of illegal dismissal.
 Finally, it appearing that respondents' reinstatement is no longer feasible by reason of the closure of DMI,
then separation pay should be awarded to respondents instead.

Angeles| Bajana | Balladares | Brillantes | Briones | Cabansag | Callanta | Chua | David|

De Leon | Gomez | Lopez | Macalino | Nostratis | Padilla | Reynon | Santos | Tan |Velasco
4E / 4F - 2018-2019
Page 408 of 920
LABOR REVIEW DIGEST
Atty. Joyrich Golangco

ARTICLE 232
1. Employees Union of Bayer Phils, Federation of Free Workers, and Juanito Facundo v. Bayer
Philippines, Lonishen (President)
G.R. No. 146530; Jan. 17, 2005
Callejo, Sr., J.:

DOCTRINE: It must be remembered that a CBA is entered into in order to foster stability and mutual cooperation
between labor and capital. An employer should not be allowed to rescind unilaterally its CBA with the duly certified
bargaining agent it had previously contracted with, and decide to bargain anew with a different group if there is no
legitimate reason for doing so and without first following the proper procedure. The act of the employer in doing so
is gives rise to the question of whether or not an unfair labor practice has been committed.
FACTS:
Employees Union of Bayer Philippines (EUBP), an affiliate of the Federation of Free Workers (FFW), is the
exclusive bargaining agent of all rank-and-file employees of Bayer Philippines (Bayer). In 1997, thru EUPB’s
President Juanito Facundo, EUPB negotiated with Bayer a CBA. Not finding Bayer’s 9.9% wage-increase
agreeable, EUPB declared and staged a strike prompting the Secretary of Labor to assume jurisdiction over the
dispute.
Pending the dispute, Avelina Remigio and 27 other union members, without authority from their union leaders,
accepted the wage-increase proposal. EUPB’s grievance committee questioned the move and reprimanded
Remigio. The Secretary issued an arbitral award ordering EUPB and Bayer to register the CBA, which was
registered with the Industrial Relations Division of the DOLE-NCR. Meanwhile, the rift between Facundo’s
leadership and Remigio’s group broadened. During a company-sponsored seminar, Remigio solicited signatures
which aimed to: (1) disaffiliate from FFW, (2) rename the union as Reformed Employees Union of Bayer
Philippines (REUBP), (3) adopt a new constitution and by-laws for the union, (4) abolish all existing officer
positions in the union and elect a new set of interim officers, and (5) authorize REUBP to administer the CBA
between EUBP and Bayer. The resolution was signed by 147 of 257 local union members.
This would be the start of a power tug-of-war between the two groups, with EUBP demanding that union dues be
remitted to them, and REUBP demanding that remittances be made to it, and that management stop transacting
with EUBP. EUPB then filed a case for ULP against Bayer for non-remittance of union dues. EUPB then sent two
letters to Bayer asking for grievance conference. Facundo did not attend, but sent two EUBP officers to inform
REUBP and management that a preventive mediation conference between the two groups have been scheduled.
The two groups failed to settle their issues before the NCMB. Facundo sent two more letters two the management
and Lonishen (company President) to discuss the failure to settle issues but both requests remained unheeded. It
was at this point, during the pendency of the first ULP case, that Bayer decided to remit the accumulated dues to
REUBP. EUBP then filed a complaint against Remigio et. al. before the Industrial Relations Division of the DOLE
complaining that their actions endanger the life of the union.
The first ULP case was dismissed because, according to the Labor Arbiter, the root cause of the conflict was an
intra-union conflict over which it had no jurisdiction.
Despite requests to renegotiate what was the current CBA at that time (1997-2001 CBA), management instead
opted to negotiate with Remigio’s group, and that the two agreed to sign a new CBA. EUBP immediately filed an
injunction with the NLRC asserting their authority as exclusive bargaining agent, and included later in an amended
complaint the issue of the gross violation of the CBA particularly the contract bar rule because of management’s
decision to negotiate with REUBP.
The Regional Director of the case filed before the Industrial Relations Division dismissed the case for failure to
exhaust the reliefs within the union and ordered the conduct of a referendum to determine which of the two groups
should be recognized as union officers. The BLR reversed this decision, but the ruling came late as the CBA
between Bayer and REUBP had already been signed and ratified at this point.
A second ULP was filed before the NLRC.
LA RULING: Dismissed because of lack of jurisdiction due to the issue involving an intra-union dispute.
NLRC Ruling: Also dismissed, together with the injunction on the same grounds.
Angeles| Bajana | Balladares | Brillantes | Briones | Cabansag | Callanta | Chua | David|

De Leon | Gomez | Lopez | Macalino | Nostratis | Padilla | Reynon | Santos | Tan |Velasco
4E / 4F - 2018-2019
Page 409 of 920
LABOR REVIEW DIGEST
Atty. Joyrich Golangco

CA Ruling: Affirmed the dismissal, and held that the employees had the right to change their bargaining unit after
the 5 year term of their CBA, provided it was done within the 60-day freedom period.
APPEAL TO THE SC:
Petitioner’s Contention: Las & NLRC have jurisdiction because the primary issue is the ULP of Bayer in
contracting with REUBP.
Respondent’s Contention: There is NO jurisdiction as the issues should first have been ventilated with
voluntary arbitrators.
ISSUE: Whether there is jurisdiction.
RULING: YES. The petitioners do not seek a determination of whether it is the Facundo group (EUBP) or the
Remigio group (REUBP) which is the true set of union officers. Instead, the issue raised pertained only to the
validity of the acts of management in light of the fact that it still has an existing CBA with EUBP. The dismissal of
the second ULP complaint was therefore erroneous.
It must be remembered that a CBA is entered into in order to foster stability and mutual cooperation between labor
and capital. An employer should not be allowed to rescind unilaterally its CBA with the duly certified bargaining
agent it had previously contracted with, and decide to bargain anew with a different group if there is no legitimate
reason for doing so and without first following the proper procedure. If such behavior would be tolerated,
bargaining and negotiations between the employer and the union will never be truthful and meaningful, and no
CBA forged after arduous negotiations will ever be honored or be relied upon.
When an employer proceeds to negotiate with a splinter union despite the existence of its valid CBA with the duly
certi�ed and exclusive bargaining agent, the former indubitably abandons its recognition of the latter and
terminates the entire CBA.
Respondents cannot claim good faith to justify their acts. They knew that Facundo's group represented the duly-
elected officers of EUBP. Moreover, they were cognizant of the fact that even the DOLE Secretary himself had
recognized the legitimacy of EUBP's mandate by rendering an arbitral award ordering the signing of the 1997-
2001 CBA between Bayer and EUBP. Respondents were likewise well-aware of the pendency of the intra-union
dispute case, yet they still proceeded to turn over the collected union dues to REUBP and to effusively deal with
Remigio. The totality of respondents' conduct, therefore, reeks with anti-EUBP animus.

Angeles| Bajana | Balladares | Brillantes | Briones | Cabansag | Callanta | Chua | David|

De Leon | Gomez | Lopez | Macalino | Nostratis | Padilla | Reynon | Santos | Tan |Velasco
4E / 4F - 2018-2019
Page 410 of 920
LABOR REVIEW DIGEST
Atty. Joyrich Golangco

2. Atty. Allan S. Montaño vs. Atty. Ernesto C. Verceles


GR No: 168583 Date: July 26, 2010
Ponente: Del Castillo, J.

Doctrine:
The Federation/Union's Constitution and By-Laws govern the relationship between and among its
members. They are akin to ordinary contracts in that their provisions have obligatory force upon the
federation/union and its member. What has been expressly stipulated therein shall be strictly binding on
both.

FACTS:

Atty. Montaño worked as legal assistant of Federation of Free Workers (FFW) on Oct. 1, 1994 and
eventually became the employees’ union president in July 19917. He was likewise designated officer-in-charge of
FFW Legal Center on November of 1998. Atty. Verceles on the other hand was the president of University of the
East Employees’ Association (UEEA-FFW) which is an affiliate union of FFW.
During the 21st National Convention and Election of National Officers of FFW, Atty. Montaño was
nominated for the position of National Vice-President. In a letter dated May 25, 2001, however, the Commission
on Elections (FFW COMELEC), informed him that he is not qualified for the position as his candidacy violates the
1998 FFW Constitution and By-Laws, particularly Section 76 of Article XIX 8 and Section 25 (a) of Article VIII, 9
both in Chapter II thereof. Atty. Montaño thus Fled an Urgent Motion for Reconsideration praying that his name
be included in the official list of candidates.
Election ensued on May 26-27, 2001 in the National Convention held at Subic
International Hotel, Olongapo City. Despite the pending motion for reconsideration with
the FFW COMELEC, and strong opposition and protest of respondent Atty. Ernesto C.
Verceles (Atty. Verceles), a delegate to the convention,the convention delegates allowed Atty. Montaño's
candidacy. He emerged victorious and was proclaimed as the National Vice-President.
On May 28, 2001, through a letter to the Chairman of FFW COMELEC, Atty. Verceles reiterated his
protest over Atty. Montaño's candidacy. On June 18, 2001, Atty. Verceles sent a follow-up letter to the President
of FFW requesting for immediate action on his protest.
Ony July 31, 2001, Atty. Verceles filed before the Bureau of Labor Relations (BLR) a petition for the
nullification of the election of Atty. Montaño.
Atty. Montaño filed his Comment with Motion to Dismiss on the grounds that the RD of the DOLE and not
the BLR has jurisdiction over the case; that the filing was premature due to the pending and unresolved protest
before the FFW COMELEC; and that Atty. Verceles has no legal standing to initiate the petition.
Atty. Verceles alleged that as already ruled by the FFW Comelec, Atty. Montaño is not qualified to run for
position because Sec. 76 of Art. XIX of the FFW Constitution and By-Laws prohibits federation employees from
sitting in its Governing Board.

BLR RULING:
BLR dismissed the petition for lack of merit on May 8, 2002. It upheld its jurisdiction over intra-union
dispute cases and affirmed Atty. Verceles’ legal personality to institute the action as president of an affiliate union
of FFW. BLR ruled that there were no grounds to hold petitioner unqualified for National Vice-President of FFW. It
opined that there was sufficient compliance with the requirements laid down by section 26 of Art. VIII
(which it said to be the applicable provision and not Sec 76 of Art. XIX) and that the convention delegates
unanimously decided that Atty. Montaño was qualified to run for the position. MR was denied by the BLR.

CA RULING:
Case was elevated to the CA via a petition for certiorari on the ground that the Convention had no
authority under the FFW Constitution and By-laws to overrule or set aside the FFW COMELEC’s Decision.
Angeles| Bajana | Balladares | Brillantes | Briones | Cabansag | Callanta | Chua | David|

De Leon | Gomez | Lopez | Macalino | Nostratis | Padilla | Reynon | Santos | Tan |Velasco
4E / 4F - 2018-2019
Page 411 of 920
LABOR REVIEW DIGEST
Atty. Joyrich Golangco

On May 28, 2004, CA set aside the BLR’s Decision. CA ruled that petitioner did not possess the
qualification requirement under par. (d) of Section 26 that candidates must be an officer or member of a legitimate
labor organization. According to the CA, since petitioner, as legal assistant employed by FFW, is considered as
confidential employee, consequently he is ineligible to join FFW Staff Association, the rank-and-file union of FFW.
An MR was filed by petitioner claiming that the CA erred in stating that FFW Staff Association, which he
is an officer and member, is not a legitimate labor organization. He asserted that the legitimacy of the union was
never raised as an issue. The FFW Association also sought intervention believing that it will be prejudiced by the
CA decision since its legal existence was put at stake. CA denied both the MR and motion for
intervention/clarification.

APPEAL TO THE SC:

Petitioner's Contention: Atty. Montaño contends that the CA gravely erred in upholding the jurisdiction of the
BLR; in not declaring as premature the petition in view of the pending protest before FFW COMELEC; in not
Fnding that the petition violated the rule on non-forum shopping; in not dismissing the case for being moot in view
of the appointment of Atty. Verceles as NLRC Commissioner; and in granting the petition to annul his election as
FFW National Vice-President on the ground that FFW Staff Association is not a legitimate labor organization.

ISSUE/S:
1. Whether or not BLR has jurisdiction over the case
2. Whether or not the petition to annul Atty. Montaño’s election was prematurely filed.
3. Whether or not Atty. Montaño is qualified to run as National-VP of FFW

HELD:
1. Yes. The BLR has jurisdiction over intra-union disputes involving a federation.
Section 226 of the Labor Code 28 clearly provides that the BLR and the Regional Directors of DOLE
have concurrent jurisdiction over inter-union and intra-union disputes. Such disputes include the conduct or
nulliFcation of election of union and workers' association oGcers. 29 There is, thus, no doubt as to the BLR's
jurisdiction over the instant dispute involving member-unions of a federation arising from disagreement over the
provisions of the federation's constitution and by-laws.
2. No. It is true that under the Implementing Rules, redress must first be sought within the organization itself in
accordance with its constitution and by-laws. However, this requirement is not absolute but yields to exception
under varying circumstances. In the case at bench, Atty. Verceles made his protest over Atty. Montaño's
candidacy during the plenary session before the holding of the election proceedings. The FFW COMELEC,
notwithstanding its reservation and despite objections from certain convention delegates, allowed Atty. Montaño's
candidacy and proclaimed him winner for the position. Under the rules, the committee on election shall endeavor
to settle or resolve all protests during or immediately after the close of election proceedings and any protest left
unresolved shall be resolved by the committee within five days after the close of the election proceedings. A day
or two after the election, Atty. Verceles made his written/formal protest over Atty. Montaño's
candidacy/proclamation with the FFW COMELEC. He exhausted the remedies under the constitution and by-
laws to have his protest acted upon by the proper forum and even asked for a formal hearing on the
matter. Still, the FFW COMELEC failed to timely act thereon. Thus, Atty. Verceles had no other recourse
but to take the next available remedy to protect the interest of the union he represents as well as the
whole federation, especially so that Atty. Montaño, immediately after being proclaimed, already assumed and
started to perform the duties of the position. Consequently, Atty. Verceles properly sought redress from the BLR
so that the right to due process will not be violated. To insist on the contrary is to render the exhaustion of
remedies within the union as illusory and vain.
Angeles| Bajana | Balladares | Brillantes | Briones | Cabansag | Callanta | Chua | David|

De Leon | Gomez | Lopez | Macalino | Nostratis | Padilla | Reynon | Santos | Tan |Velasco
4E / 4F - 2018-2019
Page 412 of 920
LABOR REVIEW DIGEST
Atty. Joyrich Golangco

3. No.
To begin with, FFW COMELEC is vested with authority and power, under the FFW Constitution and By-
Laws, to screen candidates and determine their qualifications and eligibility to run in the election and to adopt and
promulgate rules concerning the conduct of elections. The Committee is also regarded as the final arbiter of all
election protests. From the foregoing, FFW COMELEC, undeniably, has sufficient authority to adopt its own
interpretation of the explicit provisions of the federation's constitution and by-laws and unless it is shown
to have committed grave abuse of discretion, its decision and ruling will not be interfered with. The FFW
Constitution and By-laws are clear that no member of the Governing Board shall at the same time perform
functions of the rank-and-file staff. The BLR erred in disregarding this clear provision. The FFW COMELEC's
ruling which considered Atty. Montaño's candidacy in violation of the FFW Constitution is therefore correct.

Other Notes/ SC Pronouncements:


Petition was moot and academic as the challenged term of office held and served by petitioner expired on 2006.
The Court delved into the merits of the case notwithstanding supervening events as the issue is capable of
repetition.

The allegation regarding certification against forum shopping was belatedly raised. It was raised for the first time
only on MR of the decision of the CA.
3. Emilio Diokno et. al. v. Hon. Hans Leo Cacdac
GR No: 168475 Date: July 4, 2007
Ponente: J. Chico-Nazario

Doctrine:
The BLR has the original and exclusive jurisdiction on all inter-union and intra-union conflicts. Since Article 226 of
the Labor Code has declared that the BLR shall have original and exclusive authority to act on all inter-union and
intra-union conflicts, there should be no more doubt as to its jurisdiction. As defined, an intra-union conflict would
refer to a conflict within or inside a labor union, while an inter-union controversy or dispute is one occurring or
carried on between or among unions

FACTS:
The First Line Association of Meralco Supervisory Employees (FLAMES) is a legitimate labor organization which
is the supervisory union of Meralco. Petitioners and private respondents are members of FLAMES. The FLAMES
Executive Board created the Committee on Election (COMELEC) for the conduct of its union elections

The COMELEC was composed of petitioner Dante M. Tong as its chairman, and petitioners Jaime C. Mendoza
and Romeo M. Macapulay as members. Subsequently, private respondents Jimmy S. Ong, Nardito C. Alvarez,
Alfredo J. Escall, and Jaime T. Valeriano filed their respective certificates of candidacy. The COMELEC rejected
Ong’s candidacy on the ground that he was not a member of FLAMES. Meanwhile, the others’ certificates of
candidacy were similarly rejected on the basis of the exclusion of their department from the scope of the existing
CBA for being confidential employees.

Private respondents Ong, et. al., filed a Petition before the Med-Arbitration Unit of DOLE for the nullification of the
order of the COMELEC which disallowed their candidacy. Petitioners filed a Petition with the COMELEC seeking
the disqualification of private respondents Edgardo Daya, Pablo Lucas, Leandro Tabilog, Reynaldo Espiritu, Jose
Vito, Antonio de Luna, Armando Yalung, Edwin Layug, Nards Pabilona, Reynaldo Reyes, Evangeline Escall,
Alberto Alcantara, Rogelio Cervitillo, Marcelino Morelos, and Faustino Ermino (Daya, et. al.) alleging that the latter
allowed themselves to be assisted by non-union members.

COMELEC RULING:
Angeles| Bajana | Balladares | Brillantes | Briones | Cabansag | Callanta | Chua | David|

De Leon | Gomez | Lopez | Macalino | Nostratis | Padilla | Reynon | Santos | Tan |Velasco
4E / 4F - 2018-2019
Page 413 of 920
LABOR REVIEW DIGEST
Atty. Joyrich Golangco

The COMELEC officially disqualified Daya, et. al. to participate in the FLAMES elections. It also proclaimed some
of herein petitioners as winners of the elections.

MED-ARBITER RULING:
Private respondents Daya, et al., along with Ong, et al., filed with the Med-Arbitration Unit a Petition to: a) Nullify
Order of Disqualification; b) Nullify Election Proceedings and Counting of Votes; c) Declare Failure of Election;
and d) Declare Holding of New Election to be Controlled and Supervised by the DOLE.

The Med-Arbiter issued a Decision in favor of Daya, et al. and reversed the disqualification imposed by the
COMELEC against them. Finally, the Petition of Ong, et al., seeking to declare themselves as bona fide members
of FLAMES was ordered dismissed.

BLR RULING:
On appeal to the Director of the BLR, public respondent Director Hans Leo J. Cacdac, he affirmed in toto the
assailed decision of the Med-Arbiter. He echoed the interpretation of the Med-Arbiter that the COMELEC
erroneously resorted to the CBL provision which refers to the expulsion of a member from the union on valid
grounds and with due process, along with the requisite 2/3 vote of the Executive Board, but not to the
disqualification of a member as a candidate in a union election. The COMELEC cannot disqualify a candidate on
the same grounds for expulsion of members, which power is vested by the CBL on the Executive Board.

CA RULING:
The appellate court held that the provision relied upon by the COMELEC concerns the dismissal and/or expulsion
of union members, which power is vested in the FLAMES Executive Board, and not the COMELEC. It affirmed the
finding of the BLR Director that the COMELEC, in disqualifying private respondents Daya, et al., committed a
procedural shortcut without the requisite 2/3 vote of the Executive Board dismissing and/or expelling private
respondents.

APPEAL TO THE SC:


Petitioners contend that the instant Petition falls under the exceptions to the rule that the Supreme Court is not a
trier of facts. They implore this Court to make factual determination anent the conduct of the elections. They also
question the jurisdiction of the BLR because of the failure of private respondents Daya, et al., to exhaust
administrative remedies within the union. It is the stance of petitioner that Article 226 of the Labor Code which
grants power to the BLR to resolve inter-union and intra-union disputes is dead law, and has been amended by
Section 14 of Republic Act No. 6715, whereby the conciliation, mediation and voluntary arbitration functions of the
BLR had been transferred to the National Conciliation and Mediation Board.

Private respondents Daya, et al., maintain that the Petition they filed before the DOLE-NCR Med-Arbiter
questioning the disqualification order of the COMELEC and seeking the nullification of the 7 May 2003 election
involves an intra-union dispute which is within the jurisdiction of the BLR. They further claim that the COMELEC,
in disqualifying them, mistakenly relied on a provision in the FLAMES CBL that addresses the expulsion of
members from the union, and no expulsion proceedings were held against them.

ISSUE:
1. Whether or not the BLR has jurisdiction.
2. Whether private respondents prematurely sought the BLR’s jurisdiction on the ground that they failed to
exhaust administrative remedies within the union.

HELD:
1. Yes. The issue of the propriety of the disqualification of private respondents is an intra-union dispute as it is one
which involves a dispute within or inside FLAMES, a labor union. The help of non-members of the union in their
Angeles| Bajana | Balladares | Brillantes | Briones | Cabansag | Callanta | Chua | David|

De Leon | Gomez | Lopez | Macalino | Nostratis | Padilla | Reynon | Santos | Tan |Velasco
4E / 4F - 2018-2019
Page 414 of 920
LABOR REVIEW DIGEST
Atty. Joyrich Golangco

election campaign to the detriment of FLAMES, the same does not detract from the real character of the
controversy. It remains as one which involves the grievance over the constitution and bylaws of a union, and it is a
controversy involving members of the union.

Article 226 of the Labor Code provides that:

ART. 226. BUREAU OF LABOR RELATIONS. The Bureau of Labor Relations and the Labor Relations Divisions
in the regional offices of the Department of Labor shall have original and exclusive authority to act, at their own
initiative or upon request of either or both parties, on all inter-union and intra-union conflicts, and all disputes,
grievances or problems arising from or affecting labor-management relations in all workplaces whether agricultural
or nonagricultural, except those arising from the implementation or interpretation of collective bargaining
agreements which shall be the subject of grievance procedure and/or voluntary arbitration.

The Bureau shall have fifteen (15) working days to act on labor cases before it, subject to extension by agreement
of the parties.

The amendment to Article 226, as couched in Republic Act No. 6715, which is relied upon by petitioners in arguing
that the BLR had been divested of its jurisdiction, simply reads, thus:

Sec. 14. The second paragraph of Article 226 of the same Code is likewise hereby amended to read as follows:

"The Bureau shall have fifteen (15) calendar days to act on labor cases before it, subject to extension by
agreement of the parties."

Bautista v. Court of Appeals, interpreting Article 226 of the Labor Code, was explicit in declaring that the BLR has
the original and exclusive jurisdiction on all inter-union and intra-union conflicts. Since Article 226 of the Labor
Code has declared that the BLR shall have original and exclusive authority to act on all inter-union and intra-union
conflicts, there should be no more doubt as to its jurisdiction. As defined, an intra-union conflict would refer to a
conflict within or inside a labor union, while an inter-union controversy or dispute is one occurring or carried on
between or among unions. More specifically, an intra-union dispute is defined under Section (z), Rule I of the
Rules Implementing Book V of the Labor Code, viz:

(z) Intra-Union Dispute refers to any conflict between and among union members, and includes all disputes or
grievances arising from any violation of or disagreement over any provision of the constitution and by-laws of a
union, including cases arising from chartering or affiliation of labor organizations or from any violation of the rights
and conditions of union membership provided for in the Code.

2. No. The case falls under the exception to the rule of exhaustion of administrative remedies.

Before a party is allowed to seek the intervention of the court, it is a pre-condition that he should have availed of
all the means of administrative processes afforded him. Hence, if a remedy within the administrative machinery
can still be resorted to by giving the administrative officer concerned every opportunity to decide on a matter that
comes within his jurisdiction when such remedy should be exhausted first before the courts judicial power can be
sought. The premature invocation of courts judicial intervention is fatal to ones cause of action.

Verily, there are exceptions to the applicability of the doctrine. Among the established exceptions are: 1) when the
question raised is purely legal; 2) when the administrative body is in estoppel; 3) when the act complained of is
patently illegal; 4) when there is urgent need for judicial intervention; 5) when the claim involved is small; 6) when
irreparable damage will be suffered; 7) when there is no other plain, speedy, and adequate remedy; 8) when
Angeles| Bajana | Balladares | Brillantes | Briones | Cabansag | Callanta | Chua | David|

De Leon | Gomez | Lopez | Macalino | Nostratis | Padilla | Reynon | Santos | Tan |Velasco
4E / 4F - 2018-2019
Page 415 of 920
LABOR REVIEW DIGEST
Atty. Joyrich Golangco

strong public interest is involved; 9) when the subject of the proceeding is private land; 10) in quo
warranto proceedings; and 11) where the facts show that there was a violation of due process. As aptly
determined by the BLR Director, private respondents Daya, et al., were prejudiced by the disqualification order of
the COMELEC. They endeavored to seek reconsideration, but the COMELEC failed to act thereon. The
COMELEC was also found to have refused to receive their written protest. The foregoing facts sustain the finding
that private respondents Daya, et al., were deprived of due process. Hence, it becomes incumbent upon private
respondents Daya, et al., to seek the aid of the BLR. To insist on the contrary is to render their exhaustion of
remedies within the union as illusory and vain. These antecedent circumstances convince this Court that there
was proper application by the Med-Arbiter of the exception to the rule of exhaustion of administrative remedies, as
affirmed by the BLR Director, and upheld by the Court of Appeals.

Angeles| Bajana | Balladares | Brillantes | Briones | Cabansag | Callanta | Chua | David|

De Leon | Gomez | Lopez | Macalino | Nostratis | Padilla | Reynon | Santos | Tan |Velasco
4E / 4F - 2018-2019
Page 416 of 920
LABOR REVIEW DIGEST
Atty. Joyrich Golangco

ARTICLE 233
1. Magbanua v. Uy
GR No: 161003 Date: May 6, 2005
Ponente: PANGANIBAN, J.:

Doctrine:
Rights may be waived through a compromise agreement, notwithstanding a final judgment that has already
settled the rights of the contracting parties. To be binding, the compromise must be shown to have been
voluntarily, freely and intelligently executed by the parties, who had full knowledge of the judgment. Furthermore,
it must not be contrary to law, morals, good customs and public policy.

FACTS:
Petitioners filed before the LA a complaint against Uy. LA ruled in favor of the petitioners. Thereafter, petitioners
filed a Motion for Issuance of Writ of Execution while Uy filed a Manifestation requesting that the cases be
terminated and closed, stating that the judgment award as computed had been complied with to the satisfaction of
[petitioners]. Said Manifestation was also signed by the eight (8) [petitioners]. Together with the Manifestation is a
Joint Affidavit dated May 5, 1997 of [petitioners], attesting to the receipt of payment from [respondent] and waiving
all other benefits due them in connection with their complaint.

LA/RTC/NLRC RULING:
On February 27, 1998, the Labor Arbiter issued an order denying the motion for issuance of writ of execution and
[considered] the cases closed and terminated x x x.
On appeal, the [National Labor Relations Commission (hereinafter NLRC)] reversed the Labor Arbiter and
directed the immediate issuance of a writ of execution, holding that a final and executory judgment can no longer
be altered and that quitclaims and releases are normally frowned upon as contrary to public policy.

CA RULING:
The CA held that compromise agreements may be entered into even after a final judgment. Thus, petitioners
validly released respondent from any claims, upon the voluntary execution of a waiver pursuant to the
compromise agreement.

APPEAL TO THE SC:


Petitioners vehemently argue that a compromise of a final judgment is invalid under Article 2040 of the Civil Code.

ISSUE/S:
1. Whether the compromise agreement was valid even if entered into after a final judgment
2. Whether the absence of their counsel and the labor arbiter when they executed the waiver invalidates the
quitclaim

HELD:
1. Yes, the compromise agreement was valid even if entered into after a final judgment. The Court noted that
Article 2040 impliedly allowed such agreements; there was no limitation as to when these should be entered into.
In Jesalva v. Bautista, the court upheld a compromise agreement that covered cases pending trial, on appeal, and
with final judgment. Further, the validity of the agreement is determined by compliance with the requisites and
principles of contracts, not by when it was entered into. In this case, petitioners voluntarily entered into the
compromise agreement, as shown by the following facts: (1) they signed respondents Manifestation (filed with the
labor arbiter) that the judgment award had been satisfied; (2) they executed a Joint Affidavit dated May 5, 1997,
attesting to the receipt of payment and the waiver of all other benefits due them; and (3) 6 of the 8 petitioners filed
a Manifestation with the labor arbiter on October 20, 1997, requesting that the cases be terminated because of
Angeles| Bajana | Balladares | Brillantes | Briones | Cabansag | Callanta | Chua | David|

De Leon | Gomez | Lopez | Macalino | Nostratis | Padilla | Reynon | Santos | Tan |Velasco
4E / 4F - 2018-2019
Page 417 of 920
LABOR REVIEW DIGEST
Atty. Joyrich Golangco

their receipt of payment in full satisfaction of their claims. These circumstances also reveal that respondent has
already complied with its obligation pursuant to the compromise agreement. Having already benefited from the
agreement, estoppel bars petitioners from challenging it.

2. No, absence of their counsel and the labor arbiter when they executed the waiver does invalidates the
quitclaim. Under the NLRC’s New Rules of Procedure, should the parties arrive at any agreement as to the whole
or any part of the dispute, the same shall be reduced to writing and signed by the parties and their respective
counsel, or authorized representative, if any, before the Labor Arbiter.
The settlement shall be approved by the Labor Arbiter after being satisfied that it was voluntarily entered into by
the parties and after having explained to them the terms and consequences thereof.
A compromise agreement entered into by the parties not in the presence of the Labor Arbiter before whom the
case is pending shall be approved by him, if after confronting the parties, particularly the complainants, he is
satisfied that they understand the terms and conditions of the settlement and that it was entered into freely and
voluntarily by them and the agreement is not contrary to law, morals, and public policy.
The labor arbiters absence when the waivers were executed was remedied upon compliance with the above
procedure. The Court observes that the arbiter made searching questions during the pre-execution conference to
ascertain whether petitioners had voluntarily and freely executed the waivers. Likewise, there was evidence that
they made an intelligent choice, considering that the contents of the written waivers had been explained to them.
The labor arbiters absence when those waivers were executed does not, therefore, invalidate them.
Other Notes/ SC Pronouncements:
2. AL ARELLANO, SOLOMON, et al. vs. POWERTECH CORPORATION, WILLIE CABOBOS and CA
G.R. No. 150861 Date: January 22, 2008
Ponente: REYES, R.T., J.:

DOCTRINE:
Collusion is a species of fraud that Article 227 of the LC empowers the NLRC to void a compromise agreement for
fraud. Any compromise settlement, including those involving labor standard laws, voluntarily agreed upon by the
parties with the assistance of the Bureau or the regional office of the Department of Labor, shall be final and
binding upon the parties. The NLRC or any court shall not assume jurisdiction over issues involved therein except
in case of non-compliance thereof or if there is prima facie evidence that the settlement was obtained through
fraud, misrepresentation, or coercion.

FACTS:

Petitioners are among the respondent’s 52 illegally dismissed employees. On their behalf, their union filed a
complaint for illegal dismissal and other money claims against of respondent Powertech. The LA declared
that they were illegally terminated and granted their monetary claims in the total amount of P2,538,728.84. On
appeal to the NLRC, the petitioners appointed Carlos Gestiada who was one of the complainants in the suit as
their attorney-in-fact evidenced by an SPA. Thereafter, Gestiada executed a quitclaim, release, and waiver in
the latter’s favor in consideration of P150K check allegedly as compromise amount for all of petitioners.

Powertech filed a motion for the withdrawal of the appeal and cash bond which was granted in the NLRC
Resolution. The check, however, bounced due to a stop payment order of Powertech. Petitioners moved to
nullify the release and quitclaim for lack of consideration, which was granted by NLRC. The appeal was
reinstated and Powertech was ordered to post a cash or surety bond for the monetary judgment less the amount it
had previously posted.

Gestiada terminated the services of their union’s counsel, Atty. Evangelista, and hired Atty. Felipe instead. A day
later, Powertech paid P150K cash to Gestiada purportedly as compromise amount for all of petitioners.
Powertech and Gestiada filed with the NLRC a joint motion to dismiss based on the compromise agreement.
Angeles| Bajana | Balladares | Brillantes | Briones | Cabansag | Callanta | Chua | David|

De Leon | Gomez | Lopez | Macalino | Nostratis | Padilla | Reynon | Santos | Tan |Velasco
4E / 4F - 2018-2019
Page 418 of 920
LABOR REVIEW DIGEST
Atty. Joyrich Golangco

It was opposed by Atty. Evangelista alleging that the compromise agreement is void, that he was illegally
terminated as counsel for the other petitioners without their consent, and that the P150K cash was received by
Gestiada in collusion with Powertech as payment solely for his own backwages and other monetary claims, and
such amount did not cover the monetary claims of the other petitioners in this case. Gestiada admitted those facts
in his letter addressed to Atty. Evangelista.

NLRC RULING:
Denied the joint motion to dismiss. It held that a waiver of money claims must be regarded as a personal right;
hence, the protective rule that for a compromise dealing with their judgment to be validly entered into there must
be personal and specific individual consent given by the workers. Moreover, assuming that Gestiada had the
authority to enter into a compromise agreement in behalf of the other complainants, the Quitclaim and Release
cannot be recognized as a valid and binding undertaking as the consideration P150K as opposed to the total
monetary award in the amount P2M is clearly unconscionable and is thus void for being contrary to public policy.
CA RULING:
Reversed the NLRC and upheld the validity of the compromise agreement. NLRC’s act of dismissing the appeal
and declaring the compromise agreement void is a grave abuse of discretion. Fundamental is the rule that a
compromise agreement entered into in good faith by workers and their employer to resolve a pending controversy
is valid and binding on the agreeing parties.

ISSUE:
Whether the NLRC by declaring that the compromise agreement entered between the Gestiada and Powertech as
null and void acted with grave abuse of discretion.

HELD:
NO. The NLRC justifiably declared the compromise agreement as void, and the CA gravely erred in upholding the
compromise agreement. CA decision was premised on the compromise agreement entered into in good faith. But
it is clear that there is ample evidence indicating that Powertech was negotiating in bad faith and, worse, it
colluded with Gestiada in shortchanging, nay, fraudulently depriving petitioners of their just share in the award. SC
rebuke Powertech’s unscrupulous and despicable act of using an apparently valid compromise agreement to
evade payment of its legal obligation to petitioners. We will not allow employers to make a mockery of our legal
system by using legal means to perpetrate fraud. This should serve as a warning to parties in labor cases to
endeavor to achieve a just and equitable resolution of their disputes and to enter into compromise agreements in
good faith.

There would have been no opportunity for collusion between Powertech and Gestiada without the blanket
authority given by petitioners to Gestiada in the special power of attorney. This should serve as a caveat to
principals, particularly to laborers in labor disputes, to be wary of giving too broad an authority to their agents. The
powers of the agent may be circumscribed either (a) by putting a clause in the special power of attorney providing
a minimum amount upon which the agent may compromise on behalf of the principal or (b) by providing that some
acts of the agent are conditional and subject to the approval of the principal. These conditions may impose
additional burden on the negotiating parties. But it will better protect them since the agent will only be authorized
to settle for an amount predictably acceptable to the principal, and the third party will have full knowledge of the
terms and conditions the principal would not disown or disclaim.
3. Case Title: PHILIPPINE JOURNALISTS, INC., BOBBY DELA CRUZ, ARNOLD BANARES and ATTY. RUBY
RUIZ BRUNO,petitioners, vs.NATIONAL LABOR RELATIONS COMMISSION, HON. COMMS. LOURDES
JAVIER, TITO GENILO and ERNESTO VERCELES, JOURNAL EMPLOYEES UNION, and THE COURT OF
APPEALS, respondents.
GR No.: 166421 Date: September 5, 2006
Ponente: CALLEJO, SR., J

Angeles| Bajana | Balladares | Brillantes | Briones | Cabansag | Callanta | Chua | David|

De Leon | Gomez | Lopez | Macalino | Nostratis | Padilla | Reynon | Santos | Tan |Velasco
4E / 4F - 2018-2019
Page 419 of 920
LABOR REVIEW DIGEST
Atty. Joyrich Golangco

Doctrine: Thus, contrary to the allegation of petitioners, the execution and subsequent approval by the NLRC of
the agreement forged between it and the respondent Union did not render the NLRC resolution ineffectual, nor
rendered it "moot and academic." The agreement becomes part of the judgment of the court or tribunal, and as a
logical consequence, there is an implicit waiver of the right to appeal.
CA was correct in holding that the compromise agreement pertained only to the "monetary obligation" of the
employer to the dismissed employees, and in no way affected the Resolution in NCMB-NCR-NS-03-087-00 dated
May 31, 2001 where the NLRC made the pronouncement that there was no basis for the implementation of
petitioners' retrenchment program.
FACTS:
The Philippine Journalists, Inc. (PJI) is a domestic corporation engaged in the publication and sale of newspapers
and magazines. The exclusive bargaining agent of all the rank-and-file employees in the company is the Journal
Employees Union (Union for brevity).
Sometime in April 2005, the Union filed a notice of strike before the National Conciliation and Mediation Board
(NCMB), claiming that PJI was guilty of unfair labor practice. PJI was then going to implement a retrenchment
program due to "over-staffing or bloated work force and continuing actual losses sustained by the company for the
past three years resulting in negative stockholders equity of P127.0 million." The Secretary of the Department of
Labor and Employment (DOLE) certified4 the labor dispute to the National Labor Relations Commission (NLRC)
for compulsory arbitration pursuant to Article 263 (g) of the Labor Code. The case was docketed as NCMB-NCR-
NS-03-087-00.
PJI filed a motion to dismiss, contending that the Secretary of Labor had no jurisdiction to assume over the case
and thus erred in certifying it to the Commission. The NLRC denied the motion. PJI, thereafter, filed a Motion to
Defer Further Proceedings, alleging, among others, that the filing of its position paper might jeopardize attempts to
settle the matter extrajudicially, which the NLRC also denied. The case was, thereafter, submitted for decision.
NLRC: declared that the 31 complainants were illegally dismissed and that there was no basis for the
implementation of petitioner's retrenchment program; ordered their reinstatement "to their former position without
loss of seniority rights and other benefits, with payment of unpaid salaries, bonuses and backwages from the date
of dismissal up to the actual date of reinstatement plus 10% of the total monetary award as attorney's fees."
Thereafter, the parties executed a Compromise Agreement8 dated July 9, 2001, where PJI undertook to
reinstate the 31 complainant-employees effective July 1, 2001 without loss of seniority rights and benefits; 17 of
them who were previously retrenched were agreed to be given full and complete payment of their respective
monetary claims, while 14 others would be paid their monetary claims minus what they received by way of
separation pay.
The compromise agreement was submitted to the NLRC for approval. The compromise agreement was approved
and NCMB-NCR-NS-03-087-00 was deemed closed and terminated.
In the meantime however, the Union filed another notice of strike claiming, among others, that some 29
employees were illegally dismissed and that the salaries and benefits of 50 others had been illegally reduced.
NLRC ruled that the complainants were not illegally dismissed. The May 31, 2001 Resolution declaring the
retrenchment program illegal did not attain finality as "it had been academically mooted by the compromise
agreement entered into between both parties on July 9, 2001." According to the Commission, it was on the basis
of this agreement that the July 25, 2002 Resolution which declared the case closed and terminated was issued.
Pursuant to Article 223 of the Labor Code, this later resolution attained finality upon the expiration of ten days
from both parties' receipt thereof. Thus, the May 31, 2001 Resolution could not be made the basis to justify the
alleged continued employment regularity of the 29 complainants subsequent to their retrenchment.
CA: held that the NLRC gravely abused its discretion in ruling for PJI. The compromise agreement referred only to
the award given by the NLRC to the complainants in the said case, that is, the obligation of the employer to the
complainants. The CA pointed out that the NLRC Resolution nevertheless declared that respondent failed to
prove the validity of its retrenchment program, which according to it, stands even after the compromise agreement
was executed; it was the reason why the agreement was reached in the first place.
Petitioners point out that a compromise agreement is the product of free will and consent of the parties and that
such agreement can be entered into during any stage of the case. They insist that the compromise agreement
Angeles| Bajana | Balladares | Brillantes | Briones | Cabansag | Callanta | Chua | David|

De Leon | Gomez | Lopez | Macalino | Nostratis | Padilla | Reynon | Santos | Tan |Velasco
4E / 4F - 2018-2019
Page 420 of 920
LABOR REVIEW DIGEST
Atty. Joyrich Golangco

effectively abandoned all findings of facts and its necessary consequences in favor of the amicable settlement.
The compromise agreement was thereafter approved on July 25, 2001 by the NLRC. As clearly stated in Article
223 of the Labor Code, it is the Resolution dated July 25, 2001 that attained finality after the expiration of the ten-
day period, and not the abandoned and mooted Resolution dated May 31, 2001.
For their part, private respondents claim that the appellate court did not commit any reversible error, and that
the assailed decision is borne out by the evidence on record. Since the dismissal of the retrenched employees
has been declared illegal, the 29 dismissed employees enjoy the status of regular and permanent employees who
cannot be dismissed except for cause; hence, the CA correctly ordered their reinstatement.
ISSUE/S:
Whether the compromise agreement entered into by the parties mooted the NLRC resolution finding that the
union members had been illegally dismissed.
HELD:
NO. Article 227 (now 233) of the Labor Code of the Philippines authorizes compromise agreements voluntarily
agreed upon by the parties, in conformity with the basic policy of the State "to promote and emphasize the
primacy of free collective bargaining and negotiations, including voluntary arbitration, mediation and conciliation,
as modes of settling labor or industrial disputes."
In AFP Mutual Benefit Association, Inc. v. Court of Appeals, the Court held that once approved, the agreement
becomes more than a mere contract binding upon the parties, considering that it has been entered as the court's
determination of the controversy and has the force and effect of any other judgment. The Court went on to state:
“Adjective law governing judicial compromises annunciate that once approved by the court, a judicial compromise
is not appealable and it thereby becomes immediately executory but this rule must be understood to refer and
apply only to those who are bound by the compromise and, on the assumption that they are the only parties to the
case, the litigation comes to an end except only as regards to its compliance and the fulfillment by the parties of
their respective obligations thereunder. The reason for the rule, said the Court in Domingo v. Court of Appeals
[325 Phil. 469], is that when both parties so enter into the agreement to put a close to a pending litigation
between them and ask that a decision be rendered in conformity therewith, it would only be "natural to
presume that such action constitutes an implicit waiver of the right to appeal" against that decision. The
order approving the compromise agreement thus becomes a final act, and it forms part and parcel of the
judgment that can be enforced by a writ of execution unless otherwise enjoined by a restraining order.”
Thus, contrary to the allegation of petitioners, the execution and subsequent approval by the NLRC of the
agreement forged between it and the respondent Union did not render the NLRC resolution ineffectual, nor
rendered it "moot and academic." The agreement becomes part of the judgment of the court or tribunal, and as a
logical consequence, there is an implicit waiver of the right to appeal.
A careful perusal of the wordings of the compromise agreement will show that the parties agreed that the only
issue to be resolved was the question of the monetary claim of several employees. The prayer of the parties in the
compromise agreement which was submitted to the NLRC reads:
“WHEREFORE, premises considered, it is respectfully prayed that the Compromise Settlement be noted and
considered; that the instant case [be] deemed close[d] and terminated and that the Decision dated May 31, 2001
rendered herein by this Honorable Commission be deemed to be fully implemented insofar as concerns the thirty-
one (31) employees mentioned in paragraphs 2c and 2d hereof; and, that the only issue remaining to be resolved
be limited to the question of the monetary claim raised in the motion for clarification by the seven employees
mentioned in paragraph 2e hereof.”
The agreement was later approved by the NLRC. The case was considered closed and terminated and the
Resolution dated May 31, 2001 fully implemented insofar as the employees "mentioned in paragraphs 2c and 2d
of the compromise agreement" were concerned. Hence, the CA was correct in holding that the compromise
agreement pertained only to the "monetary obligation" of the employer to the dismissed employees, and in no way
affected the Resolution in NCMB-NCR-NS-03-087-00 dated May 31, 2001 where the NLRC made the
pronouncement that there was no basis for the implementation of petitioners' retrenchment program.

Angeles| Bajana | Balladares | Brillantes | Briones | Cabansag | Callanta | Chua | David|

De Leon | Gomez | Lopez | Macalino | Nostratis | Padilla | Reynon | Santos | Tan |Velasco
4E / 4F - 2018-2019
Page 421 of 920
LABOR REVIEW DIGEST
Atty. Joyrich Golangco

To reiterate, the rule is that when judgment is rendered based on a compromise agreement, the judgment
becomes immediately executory, there being an implied waiver of the parties' right to appeal from the
decision.43 The judgment having become final, the Court can no longer reverse, much less modify it.

Angeles| Bajana | Balladares | Brillantes | Briones | Cabansag | Callanta | Chua | David|

De Leon | Gomez | Lopez | Macalino | Nostratis | Padilla | Reynon | Santos | Tan |Velasco
4E / 4F - 2018-2019
Page 422 of 920
LABOR REVIEW DIGEST
Atty. Joyrich Golangco

1. Colegio de San Juan de Letran vs Association of Employees and Faculty of Letran


GR No: Date:
Ponente:

Doctrine: the mere filing of a petition for certification election does not ipso facto justify the suspension of
negotiation by the employer.

FACTS:
Association of Employees and Faculty of Letran, initiated the renegotiation of its CBA with petitioner Colegio de
San Juan de Letran for the last 2 years of the CBA's 5 year lifetime from 1989-1994. In the same year, the union
elected a new set of officers wherein private respondent Eleanor Ambas emerged as the newly elected President.

Thereafter, the parties agreed to disregard the unsigned CBA and to start negotiation on a new 5-year CBA
starting 1994-1999. On February 7, 1996, the union submitted its proposals to petitioner, which notified the union
six days later, or on February 13, 1996, that the same had been submitted to its Board of Trustees.

In the meantime, Ambas was informed through a letter that her work schedule was being changed from Monday
to Friday to Tuesday to Saturday. Ambas protested and requested management to submit the issue to a
grievance machinery under the old CBA. On March 29, 1996, the union received petitioner's letter dismissing
Ambas for alleged insubordination. Hence, the union amended its notice of strike to include Ambas' dismissal.

Both parties resumed the negotiations however, petitioner stopped the negotiations after it purportedly
received information that a new group of employees had filed a petition for certification election.

Then, the union finally struck. The public respondent DOLE Sec assumed jurisdiction and ordered all striking
employees, including the union president, to return to work, and for petitioner to accept them back under the same
terms and conditions before the actual strike. Petitioner readmitted the striking members except Ambas.

DOLE SEC RULING:

Public respondent DOLE Secretary issued an order declaring petitioner guilty of unfair labor practice on two
counts and directing the reinstatement of private respondent Ambas with backwages. Petitioner filed a MR which
was denied. Petitioner sought a review of the order of the DOLE Sec before the CA.

CA RULING:
The CA dismissed the petition and affirmed the findings of the Secretary of Labor and Employment. Hence, the
present petition.

APPEAL TO THE SC:


Petitioner's Contention:
Petitioner argues that since it has already submitted the union's proposals to the Board of Trustees and that a
series of conferences had already been undertaken to discuss the ground rules for negotiation such should
already be considered as acts indicative of its intention to bargain.

Petitioner also claims that the suspension of negotiation was proper since by the filing of the petition for
certification election the issue on majority representation of the employees has arose.

Respondent's Contention:
The petitioner is guilty of unfair labor practice,
Angeles| Bajana | Balladares | Brillantes | Briones | Cabansag | Callanta | Chua | David|

De Leon | Gomez | Lopez | Macalino | Nostratis | Padilla | Reynon | Santos | Tan |Velasco
4E / 4F - 2018-2019
Page 423 of 920
LABOR REVIEW DIGEST
Atty. Joyrich Golangco

ISSUE/S:
1. Was the petitioner is guilty of unfair labor practice by refusing to bargain with the union when it unilaterally
suspended the ongoing negotiations for a new CBA upon mere information that a petition for certification has
been filed by another legitimate labor organization. - yes
2. Was the dismissal of Ambas valid? - No.

HELD:
1. The Supreme Court denied the petition. According to the Court, petitioner's actuation showed a lack of sincere
desire to negotiate rendering it guilty of unfair labor practice. Petitioner's utter lack of interest in bargaining with the
union is obvious in its failure to make a timely reply to the proposals presented by the latter. More than a month
after the proposals were submitted by the union, petitioner still had not made any counter-proposals. Said inaction
on the part of petitioner prompted the union to file its second notice of strike on March 13, 1996. Petitioner could
only offer a feeble explanation that the Board of Trustees had not yet convened to discuss the matter as its
excuse for failing to file its reply. Petitioner clearly violated Article 250 of the Labor Code governing the
procedure in collective bargaining.

In order to allow the employer to validly suspend the bargaining process there must be a valid petition for
certification election raising a legitimate representation issue. Hence, the mere filing of a petition for
certification election does not ipso facto justify the suspension of negotiation by the employer. The petition
must first comply with the provisions of the Labor Code and its Implementing Rules. Foremost is that a petition for
certification election must be filed during the sixty-day freedom period.

The "Contract Bar Rule" under Section 3, Rule XI, Book V, of the Omnibus Rules Implementing the Labor Code,
provides that: ". . . If a collective bargaining agreement has been duly registered in accordance with Article 231 of
the Code, a petition for certification election or a motion for intervention can only be entertained within sixty (60)
days prior to the expiry date of such agreement.

In the case at bar, the lifetime of the previous CBA was from 1989-1994. The petition for certification election by
ACEC, allegedly a legitimate labor organization, was filed with the Department of Labor and Employment (DOLE)
only on May 26, 1996. Clearly, the petition was filed outside the sixty-day freedom period. Hence, the filing thereof
was barred by the existence of a valid and existing collective bargaining agreement. Consequently, there is no
legitimate representation issue and, as such, the filing of the petition for certification election did not constitute a
bar to the ongoing negotiation

2. No. Ambas was not validly terminated. The dismissal was effected in violation of the employees' right to self-
organization. The factual backdrop of the termination of Ms. Ambas led to no other conclusion that she was
dismissed in order to strip the union of a leader who would fight for the right of her coworkers at the bargaining
table. The record showed that it was when Ms. Ambas was the union president and during the period of tense and
difficult negotiations when her work schedule was altered from Mondays to Fridays to Tuesdays to Saturdays.
When she did not budge, although her schedule was changed, she was outrightly dismissed for alleged
insubordination.

Angeles| Bajana | Balladares | Brillantes | Briones | Cabansag | Callanta | Chua | David|

De Leon | Gomez | Lopez | Macalino | Nostratis | Padilla | Reynon | Santos | Tan |Velasco
4E / 4F - 2018-2019
Page 424 of 920
LABOR REVIEW DIGEST
Atty. Joyrich Golangco

Angeles| Bajana | Balladares | Brillantes | Briones | Cabansag | Callanta | Chua | David|

De Leon | Gomez | Lopez | Macalino | Nostratis | Padilla | Reynon | Santos | Tan |Velasco
4E / 4F - 2018-2019
Page 425 of 920
LABOR REVIEW DIGEST
Atty. Joyrich Golangco

Article 240
1. MARIWASA SIAM CERAMICS, INC., vs. THE SECRETARY OF THE DEPARTMENT OF LABOR AND
EMPLOYMENT, et al
GR No.: 183317 Date: December 21, 2009
Ponente: Nachura, J.

DOCTRINE:

Article 234 of the Labor Code merely requires a 20% minimum membership during the application for union
registration. It does not mandate that a union must maintain the 20% minimum membership requirement all
throughout its existence.

FACTS:

On May 2005, private respondent Samahan Ng Mga Manggagawa Sa Mariwasa Siam Ceramics, Inc. (SMMSC-
Independent) was issued a Certificate of Registration as a legitimate labor organization by the Department of
Labor and Employment (DOLE), Region IV-A.

On June 2005, petitioner Mariwasa Siam Ceramics, Inc. filed a Petition for Cancellation of Union Registration
against private respondent, claiming that the latter violated Article 234 of the Labor Code for not complying with
the 20% requirement and that it committed massive fraud and misrepresentation in violation of Article 239 of the
same code.

The Regional Director of DOLE IV-A issued an Order granting the petition, revoking the registration of respondent,
and delisting it from the roster of active labor unions.

SMMSC-Independent appealed to the Bureau of Labor Relations. BLR ruled in favor of the respondent, thus, they
remain in the roster of legitimate labor organizations.

The petitioner appealed and insisted that private respondent failed to comply with the 20% union membership
requirement for its registration as a legitimate labor organization because of the disaffiliation from the total number
of union members of 102 employees who executed affidavits recanting their union membership

Hence, this petition for review on certiorari under Rule 45 of the Rules of Court.

ISSUE/S:

1) Whether or not there was failure to comply with the 20% union membership requirement;

2) Whether or not the withdrawal of 31 union members affected the petition for certification election insofar as the
30% requirement is concerned.

HELD:

The Supreme Court DENIED the petition.

On the first issue, while it is true that the withdrawal of support may be considered as a resignation from the union,
the fact remains that at the time of the union’s application for registration, the affiants were members of
Angeles| Bajana | Balladares | Brillantes | Briones | Cabansag | Callanta | Chua | David|

De Leon | Gomez | Lopez | Macalino | Nostratis | Padilla | Reynon | Santos | Tan |Velasco
4E / 4F - 2018-2019
Page 426 of 920
LABOR REVIEW DIGEST
Atty. Joyrich Golangco

respondent and they comprised more than the required 20% membership for purposes of registration as a labor
union. Article 234 of the Labor Code merely requires a 20% minimum membership during the application for union
registration. It does not mandate that a union must maintain the 20% minimum membership requirement all
throughout its existence.

On the second issue, it appears undisputedly that the 31 union members had withdrawn their support to the
petition before the filing of said petition. The distinction must be that withdrawals made before the filing of the
petition are presumed voluntary unless there is convincing proof to the contrary, whereas withdrawals made after
the filing of the petition are deemed involuntary. Therefore, following jurisprudence, the employees were not totally
free from the employer’s pressure and so the voluntariness of the employees’ execution of the affidavits becomes
suspect.

The cancellation of a union’s registration doubtless has an impairing dimension on the right of labor to self-
organization. For fraud and misrepresentation to be grounds for cancellation of union registration under the Labor
Code, the nature of the fraud and misrepresentation must be grave and compelling enough to vitiate the consent
of a majority of union members.
___________________________________________________________________

2. ELECTROMAT MANUFACTURING RECORDING CORPORATION vs LAGUNZAD


G. R. No. 172699; July 27, 2011
Brion, J.:

DOCTRINE: The issuance of D.O. 40-03 is a valid exercise of delegated powers as it merely implemented the
intent of the law – that in imposing lesser requirements in the case of a branch or local of a registered federation
or national union is to encourage the affiliation of a local union with a federation or national union in order to
increase the local union’s bargaining powers respecting terms and conditions of labor.

FACTS: Private respondent Nagkakaisang Samahan ng Manggagawa ng Electromat-Wasto (union) applied for
registration with the Bureau of Labor Relations (BLR) submitting several supporting documents for application: (1)
copies of its ratified constitution and by-laws (CBL); (2) minutes of the CBLs adoption and ratification; (3) minutes
of the organizational meetings; (4) names and addresses of the union officers; (5) list of union members; (6) list of
rank-and-file employees in the company; (7) certification of non-existence of a collective bargaining agreement
(CBA) in the company; (8) resolution of affiliation with WASTO, a labor federation; (9) WASTOs resolution of
acceptance; (10) Charter Certificate; and (11) Verification under oath.
Angeles| Bajana | Balladares | Brillantes | Briones | Cabansag | Callanta | Chua | David|

De Leon | Gomez | Lopez | Macalino | Nostratis | Padilla | Reynon | Santos | Tan |Velasco
4E / 4F - 2018-2019
Page 427 of 920
LABOR REVIEW DIGEST
Atty. Joyrich Golangco

The BLR thereafter issued the union a Certification of Creation of Local Chapter (equivalent to the certificate of
registration of an independent union), pursuant to Department Order No. (D.O.) 40-03

On October 1, 2003, the petitioner Electromat Manufacturing and Recording Corporation (company) filed a
petition for cancellation of the unions registration certificate, for the unions failure to comply with Article 234 (now
240) of the Labor Code. It argued that D.O. 40-03 is an unconstitutional diminution of the Labor Codes union
registration requirements under Article 234.

Acting Director Ciriaco A. Lagunzad of the Department of Labor and Employment (DOLE)-National Capital Region
dismissed the petition. BLR Director affirmed the dismissal.

Petitioner’s Argument: Petitioner specifically assails the constitutionality of Section 2(E), Rule III of D.O. 40-03
which provides:

“The report of creation of a chartered local shall be accompanied by a charter certificate issued
by the federation or national union indicating the creation or establishment of the chartered local.”

The company points out that D.O. 40-03 delisted some of the requirements under Article 234 (currently 240) of the
Labor Code for the registration of a local chapter The company contends that the enumeration of the requirements
for union registration under the law is exclusive and should not be diminished, and that the same requirements
should apply to all labor unions whether they be independent labor organizations, federations or local chapters. It
adds that in making a different rule for local chapters, D.O. 40-03 expanded or amended Article 234 of the Labor
Code, resulting in an invalid exercise by the DOLE of its delegated rule-making power. It thus posits that the
unions certificate of registration which was issued in violation of the letters of Article 234 of the Labor Code is void
and of no effect, and that the CA committed grave abuse of discretion when it affirmed the unions existence.

CA Ruling: Dismissed the petition and affirmed the assailed BLR ruling. CA noted that there are sufficient
safeguards found in other provisions of the Labor Code to prevent commission of fraud in the union. It pointed out
that D.O. 40-03 was issued by the DOLE pursuant to its rule-making power under the law.

ISSUE: Whether or not the D.O. 40-03 is a valid exercise of the rule-making power of the DOLE.

SC RULING: YES. D.O. 40-03 represents an expression of the governments implementing policy on trade
unionism. It builds upon the old rules by further simplifying the requirements for the establishment of locals or
chapters. The Court sees nothing contrary to the law or the Constitution in the adoption by the Secretary of Labor
and Employment of D.O. 40-03 as this department order is consistent with the intent of the government to
encourage the affiliation of a local union with a federation or national union to enhance the locals bargaining
power. If changes were made at all, these were those made to recognize the distinctions made in the law itself
between federations and their local chapters, and independent unions; local chapters seemingly have lesser
requirements because they and their members are deemed to be direct members of the federation to which they
are affiliated, which federations are the ones subject to the strict registration requirements of the law.
___________________________________________________________________

Angeles| Bajana | Balladares | Brillantes | Briones | Cabansag | Callanta | Chua | David|

De Leon | Gomez | Lopez | Macalino | Nostratis | Padilla | Reynon | Santos | Tan |Velasco
4E / 4F - 2018-2019
Page 428 of 920
LABOR REVIEW DIGEST
Atty. Joyrich Golangco

3. Case Title: Eagleridge Golf & Country Club vs. CA


GR No: 178989 Date: March 18, 2010
Ponente: Velasco, Jr., J.

Doctrine: As aptly found by the BLR Director, the Union already had 30 members when it applied for registration,
for the admission of new members is neither prohibited by law nor was it concealed in its application for
registration. Eagle Ridges contention is flawed when it equated the requirements under Art. 234(b) and (c) of the
Labor Code. Par. (b) clearly required the submission of the minutes of the organizational meetings and the list of
workers who participated in the meetings, while par. (c) merely required the list of names of all the union members
comprising at least 20% of the bargaining unit. The fact that EREU had 30 members when it applied for
registration on December 19, 2005 while only 26 actually participated in the organizational meeting is borne by the
records.

FACTS: On December 6, 2005, at least 20% of Eagle Ridges rank-and-file employees the percentage threshold
required under Article 234(c) of the Labor Code for union registration had a meeting where they organized
themselves into an independent labor union, named Eagle Ridge Employees Union (EREU), elected a set of
officers, and ratified their constitution and by-laws.

On December 19, 2005, EREU formally applied for registration. In time, DOLE RO IV granted the application and
issued EREU Registration Certificate.

The EREU then filed a petition for certification election in Eagle Ridge Golf & Country Club. Eagle Ridge opposed
this petition, followed by its filing of a petition for the cancellation of its Reg. Cert. Eagle Ridge’s petition ascribed
misrepresentation, false statement, or fraud to EREU in connection with the adoption of its constitution and by-
laws, the numerical composition of the Union, and the election of its officers.

Going into specifics, Eagle Ridge alleged that the EREU declared in its application for registration having 30
members, when the minutes of its December 6, 2005 organizational meeting showed it only had 26 members. The
misrepresentation was exacerbated by the discrepancy between the certification issued by the Union secretary
and president that 25 members actually ratified the constitution and by-laws on December 6, 2005 and the fact
that 26 members affixed their signatures on the documents, making one signature a forgery.

Finally, Eagle Ridge contended that six employees who attended the organizational meeting had manifested the
desire to withdraw from the union. The six executed individual affidavits attesting that they arrived late at said
meeting which they claimed to be drinking spree; that they did not know that the documents they signed on that
occasion pertained to the organization of a union; and that they now wanted to be excluded from the Union. The
withdrawal of the six, Eagle Ridge maintained, effectively reduced the union membership to 20 or 21, either of
which is below the mandatory minimum 20% membership requirement under Art. 234(c) of the Labor Code.
Reckoned from 112 rank-and-file employees of Eagle Ridge, the required number would be 22 or 23 employees.

DOLE REGIONAL DIRECTOR RULING: The DOLE Regional Director, Region IV-A, focusing on the question of
misrepresentation, issued an Order finding for Eagle Ridge.

Angeles| Bajana | Balladares | Brillantes | Briones | Cabansag | Callanta | Chua | David|

De Leon | Gomez | Lopez | Macalino | Nostratis | Padilla | Reynon | Santos | Tan |Velasco
4E / 4F - 2018-2019
Page 429 of 920
LABOR REVIEW DIGEST
Atty. Joyrich Golangco

BLR: The Eagle Ridge Employees Union (EREU) shall remain in the roster of legitimate organizations. In finding
for the Union, the BLR Director found as without basis allegations of misrepresentation or fraud as ground for
cancellation of EREUs registration.

CA RULING: The appellate court, in a terse two-page Resolution,dismissed Eagle Ridges petition for being
deficient (lack of verification and certification of non-forum shopping)

APPEAL TO THE SC:

Petitioner's Contention: Eagle Ridge cites the grounds provided under Art. 239(a) and (c) of the Labor Code for
its petition for cancellation of the EREUs registration.

Respondent's Contention: On the other hand, the Union asserts bona fide compliance with the registration
requirements under Art. 234 of the Code, explaining the seeming discrepancies between the number of
employees who participated in the organizational meeting and the total number of union members at the time it
filed its registration, as well as the typographical error in its certification which understated by one the number of
union members who ratified the unions constitution and by-laws.

ISSUE/S: Whether or not EREU legally complied with the requirements in its formation

HELD: YES. Before their amendment by Republic Act No. 9481 on June 15, 2007, the then governing Art. 234
(on the requirements of registration of a labor union) and Art. 239 (on the grounds for cancellation of union
registration) of the Labor Code respectively provided as follows:

ART. 234. REQUIREMENTS OF REGISTRATION. Any applicant labor organization, association or group of
unions or workers shall acquire legal personality and shall be entitled to the rights and privileges granted by law to
legitimate labor organizations upon issuance of the certificate of registration based on the following requirements:

(a) Fifty pesos (P50.00) registration fee;

(b) The names of its officers, their addresses, the principal address of the labor organization, the minutes of the
organizational meetings and the list of workers who participated in such meetings;

(c) The names of all its members comprising at least twenty percent (20%) of all the employees in the bargaining
unit where it seeks to operate;

xxxx

ART. 239. GROUNDS FOR CANCELLATION OF UNION REGISTRATION. The following shall constitute grounds
for cancellation of union registration:

(a) Misrepresentation, false statements or fraud in connection with the adoption or ratification of the constitution
and by-laws or amendments thereto, the minutes of ratification, and the list of members who took part in the
ratification;

xxxx
Angeles| Bajana | Balladares | Brillantes | Briones | Cabansag | Callanta | Chua | David|

De Leon | Gomez | Lopez | Macalino | Nostratis | Padilla | Reynon | Santos | Tan |Velasco
4E / 4F - 2018-2019
Page 430 of 920
LABOR REVIEW DIGEST
Atty. Joyrich Golangco

A scrutiny of the records fails to show any misrepresentation, false statement, or fraud committed by EREU to
merit cancellation of its registration.

First. The Union submitted the required documents attesting to the facts of the organizational meeting on
December 6, 2005, the election of its officers, and the adoption of the Unions constitution and by-laws.

Second. The members of the EREU totaled 30 employees when it applied on December 19, 2005 for registration.
The Union thereby complied with the mandatory minimum 20% membership requirement under Art. 234(c). Of
note is the undisputed number of 112 rank-and-file employees in Eagle Ridge, as shown in the Sworn Statement
of the Union president and secretary and confirmed by Eagle Ridge in its petition for cancellation.

Third. The Union has sufficiently explained the discrepancy between the number of those who attended the
organizational meeting showing 26 employees and the list of union members showing 30. The difference is due to
the additional four members admitted two days after the organizational meeting as attested to by their duly
accomplished Union Membership forms. Consequently, the total number of union members, as of December 8,
2005, was 30, which was truthfully indicated in its application for registration on December 19, 2005.

As aptly found by the BLR Director, the Union already had 30 members when it applied for registration, for the
admission of new members is neither prohibited by law nor was it concealed in its application for registration.

Fourth. The difference between the number of 26 members, who ratified the Unions constitution and by-laws, and
the 25 members shown in the certification of the Union secretary as having ratified it, is, as shown by the factual
antecedents, a typographical error. It was an insignificant mistake committed without malice or prevarication. The
list of those who attended the organizational meeting shows 26 members, as evidenced by the signatures beside
their handwritten names. Thus, the certifications understatement by one member, while not factual, was clearly an
error, but neither a misleading one nor a misrepresentation of what had actually happened.

Fifth. The six affiants of the affidavits of retraction were not presented in a hearing before the Hearing Officer
(DOLE Regional Director), as required under the Rules Implementing Book V of the Labor Code covering Labor
Relations. For their non-presentation and consonant to the above-quoted rule, the six affidavits of retraction are
inadmissible as evidence against the Union in the instant case (hearsay).

Sixth. The fact that six union members, indeed, expressed the desire to withdraw their membership through their
affidavits of retraction will not cause the cancellation of registration on the ground of violation of Art. 234(c) of the
Labor Code requiring the mandatory minimum 20% membership of rank-and-file employees in the employees
union.
Twenty percent (20%) of 112 rank-and-file employees in Eagle Ridge would require a union membership of at
least 22 employees. When the EREU filed its application for registration on December 19, 2005, there were
clearly 30 union members. Thus, when the certificate of registration was granted, there is no dispute that the
Union complied with the mandatory 20% membership requirement.

Besides, it cannot be argued that the six affidavits of retraction retroact to the time of the application of registration
or even way back to the organizational meeting. Prior to their withdrawal, the six employees in question were
bona fideunion members. More so, they never disputed affixing their signatures beside their handwritten names
during the organizational meetings. While they alleged that they did not know what they were signing, it bears
stressing that their affidavits of retraction were not re-affirmed during the hearings of the instant case rendering
them of little, if any, evidentiary value.

Angeles| Bajana | Balladares | Brillantes | Briones | Cabansag | Callanta | Chua | David|

De Leon | Gomez | Lopez | Macalino | Nostratis | Padilla | Reynon | Santos | Tan |Velasco
4E / 4F - 2018-2019
Page 431 of 920
LABOR REVIEW DIGEST
Atty. Joyrich Golangco

With the withdrawal of six union members, there is still compliance with the mandatory membership requirement
under Art. 234(c), for the remaining 24 union members constitute more than the 20% membership requirement of
22 employees.

Other Notes/ SC Pronouncements:


___________________________________________________________________

Certificate of Registration of a Labor Organization

1. TAGAYTAY HIGHLANDS INTERNATIONAL GOLF CLUB INCORPORATED v. TAGAYTAY HIGHLANDS


EMPLOYEES UNION-PGTWO January 22, 2003
G.R. No. 142000
Ponente: CARPIO MORALES, J:
GROUNDS FOR CANCELLATION OF UNION REGISTRATION
DOCTRINE: After a certificate of registration is issued to a union, its legal personality cannot be subject to
collateral attack. It may be questioned only in an independent petition for cancellation in accordance with Section
5 of Rule V, Book IV of the "Rules to Implement the Labor Code." The grounds for cancellation of union
registration are provided for under Article 239 of the Labor Code. The inclusion in a union of disqualified
employees is not among the grounds for cancellation, unless such inclusion is due to misrepresentation, false
statement or fraud under the circumstances enumerated in Sections (a) and (c) of Article 139 of above-quoted
Article 239 of the Labor Code. THEU, having been validly issued a certificate of registration, should be considered
to have already acquired juridical personality which may not be assailed collaterally. As for petitioner's allegation
that some of the signatures in the petition for certification election were obtained through fraud, false statement
and misrepresentation, the proper procedure is, as reflected above, for it to file a petition for cancellation of the
certificate of registration, and not to intervene in a petition for certification election.
FACTS: On October 16, 1997, the Tagaytay Highlands Employees Union (THEU)– Philippine Transport and
General Workers Organization (PTGWO), Local Chapter No. 776, a legitimate labor organization said to represent
majority of the rank- and-file employees of Tagaytay Highlands International Golf Club Incorporated (THIGCI),
filed a petition for certification election before the DOLE Mediation- Arbitration Unit, Regional Branch No. IV.
THIGCI, in its Comment, opposed THEU’s petition for certification election on the ground that the list of union
members submitted by it was defective and fatally flawed as it included the names and signatures of supervisors,
resigned, terminated and absent without leave (AWOL) employees, as well as employees of The Country Club,
Inc., a corporation distinct and separate from THIGCI; and that out of the 192 signatories to the petition, only 71
were actual rank-and-file employees of THIGCI. THIGCI thus submitted a list of the names of its 71 actual rank-
and-file employees to the petition for certification election. And it therein incorporated a tabulation showing the
number of signatories to said petition whose membership in the union was being questioned as disqualified and
the reasons for disqualification.
THEU asserted that it complied with all the requirements for valid affiliation and inclusion in the roster of legitimate
labor organizations pursuant to DOLE Department Order No. 9, series of 1997, on account of which it was duly
granted a Certification of Affiliation by DOLE on October 10, 1997; and that Section 5, Rule V of said Department
Order provides that the legitimacy of its registration cannot be subject to collateral attack, and for as long as there
is no final order of cancellation, it continues to enjoy the rights accorded to a legitimate organization. Therefore,
the Med-Arbiter should, pursuant to Article 257 of the Labor Code and Section 11, Rule XI of DOLE Department
Order No. 09, automatically order the conduct of a certification election.
On January 28, 1998, DOLE Med-Arbiter Anastacio Bactin ordered the holding of a certification election.
THIGCI appealed to the Office of the DOLE Secretary which, by Resolution of June 4, 1998, set aside the said
Med-Arbiter’s Order and accordingly dismissed the petition for certification election on the ground that there is a
"clear absence of community or mutuality of interests," it finding that THEU sought to represent two separate
bargaining units (supervisory employees and rank-and- file employees) as well as employees of two separate and
distinct corporate entities.
Angeles| Bajana | Balladares | Brillantes | Briones | Cabansag | Callanta | Chua | David|

De Leon | Gomez | Lopez | Macalino | Nostratis | Padilla | Reynon | Santos | Tan |Velasco
4E / 4F - 2018-2019
Page 432 of 920
LABOR REVIEW DIGEST
Atty. Joyrich Golangco

Upon Motion for Reconsideration by THEU, DOLE Undersecretary Rosalinda Dimalipis-Baldoz, by authority of the
DOLE Secretary, issued DOLE Resolution of November 12, 1998 setting aside the June 4, 1998 Resolution
dismissing the petition for certification election. She held that since THEU is a local chapter, the twenty percent
(20%) membership requirement is not necessary for it to acquire legitimate status, hence, "the alleged retraction
and withdrawal of support by 45 of the 70 remaining rank-and-file members . . . cannot negate the legitimacy it
has already acquired before the petition". THIGCI’s Motion for Reconsideration was denied by the DOLE
Undersecretary hence it filed a petition for certiorari with the CA.
The CA denied THIGCI’s Petition for Certiorari and affirmed the DOLE Resolution dated November 12, 1998. It
held that while a petition for certification election is an exception to the innocent bystander rule, hence, the
employer may pray for the dismissal of such petition on the basis of lack of mutuality of interests of the members
of the union as well as lack of employer-employee relationship and petitioner failed to adduce substantial
evidence to support its allegations.
ISSUE: Whether the union’s legal personality can be subject to collateral attack after a certificate of registration is
issued
HELD: NO, Petition is DENIED, and the records of the case are remanded to the office of origin. While above-
quoted Article 245 expressly prohibits supervisory employees from joining a rank-and-file union, it does not
provide what would be the effect if a rank-and-file union counts supervisory employees among its members, or
vice-versa.
Citing Toyota which held that "a labor organization composed of both rank-and-file and supervisory employees is
no labor organization at all," and the subsequent case of Progressive Development Corp. – Pizza Hut v.
Ledesma20 which held that:
"The Labor Code requires that in organized and unorganized establishments, a petition for certification election
must be filed by a legitimate labor organization. The acquisition of rights by any union or labor organization,
particularly the right to file a petition for certification election, first and foremost, depends onwhether or not the
labor organization has attained the status of a legitimate labor organization.
In the case before us, the Med-Arbiter summarily disregarded the petitioner’s prayer that the former look into the
legitimacy of the respondent Union by a sweeping declaration that the union was in the possession of a charter
certificate so that ‘for all intents and purposes, Sumasaklaw sa Manggagawa sa Pizza Hut (was) a legitimate
organization,’" (Underscoring and emphasis supplied).”
We also do not agree with the ruling of the respondent Secretary of Labor that the infirmity in the membership of
the respondent union ca n b e re m e d ie d in "the pr e - el ect i on con f er en ce thru the exclusion-inclusion
proceedings wherein those employees who are occupying rank-and-file positions will be excluded from the list of
eligible voters." After a certificate of registration is issued to a union, its legal personality cannot be subject to
collateral attack. It may be questioned only in an independent petition for cancellation in accordance with Section
5 of Rule V, Book IV of the "Rules to Implement the Labor Code" (Implementing Rules) which section reads:
Sec. 5. Effect of registration. The labor organization or workers’ association shall be deemed registered and
vested with legal personality on the date of issuance of its certificate of registration. Such legal personality cannot
thereafter be subject to collateral attack, but may be questioned only in an independent petition for cancellation in
accordance with these Rules. (Emphasis supplied)

The inclusion in a union of disqualified employees is not among the grounds for cancellation, unless such
inclusion is due to misrepresentation, false statement or fraud under the circumstances enumerated in Sections
(a) and (c) of Article 239 of above-quoted Article 239 of the Labor Code.
THEU, having been validly issued a certificate of registration, should be considered to have already acquired
juridical personality which may not be assailed collaterally.
As for petitioner’s allegation that some of the signatures in the petition for certification election were obtained
through fraud, false statement and misrepresentation, the proper procedure is, as reflected above, for it to file a
petition for cancellation of the certificate of registration, and not to intervene in a petition for certification election.
Regarding the alleged withdrawal of union members from participating in the certification election, this Court’s
following ruling is instructive:
Angeles| Bajana | Balladares | Brillantes | Briones | Cabansag | Callanta | Chua | David|

De Leon | Gomez | Lopez | Macalino | Nostratis | Padilla | Reynon | Santos | Tan |Velasco
4E / 4F - 2018-2019
Page 433 of 920
LABOR REVIEW DIGEST
Atty. Joyrich Golangco

"‘[T]he best forum for determining whether there were indeed retractions from some of the laborers is in
thecertification election itself wherein the workers can freely express their choice in a secret ballot.’ Suffice it to
say that the will of the rank-and-file employees should in every possible instance be determined by secret ballot
rather than by administrative or quasi-judicial inquiry. Such representation and certification election cases are not
to be taken as contentious litigations for suits but as mere investigations of a non-adversary, fact-finding character
as to which of the competing unions represents the genuine choice of the workers to be their sole and exclusive
collective bargaining representative with their employer."
As for the lack of mutuality of interest argument of petitioner, it, at all events, does not lie given, as found by the
court a quo, its failure to present substantial evidence that the assailed employees are actually occupying
supervisory positions.
While petitioner submitted a list of its employees with their corresponding job titles and ranks, there is nothing
mentioned about the supervisors’ respective duties, powers and prerogatives that would show that they can
effectively recommend managerial actions which require the use of independent judgment.
As this Court put it in Pepsi-Cola Products Philippines, Inc. v. Secretary of Labor:
Designation should be reconciled with the actual job description of subject employees x x x The mere fact that an
employee is designated manager does not necessarily make him one. Otherwise, there would be an absurd
situation where one can be given the title just to be deprived of the right to be a member of a union. In the case of
National Steel Corporation vs. Laguesma (G. R. No. 103743, January 29, 1996), it was stressed that:
What is essential is the nature of the employee’s function and not the nomenclature or titlegiven to the job which
determines whether the employee has rank-and-file or managerial status or whether he is a supervisory
employee. (Emphasis supplied).
_____________________________________________________________________

Angeles| Bajana | Balladares | Brillantes | Briones | Cabansag | Callanta | Chua | David|

De Leon | Gomez | Lopez | Macalino | Nostratis | Padilla | Reynon | Santos | Tan |Velasco
4E / 4F - 2018-2019
Page 434 of 920
LABOR REVIEW DIGEST
Atty. Joyrich Golangco

2. SS Ventures International Inc. vs SS Ventures Labor Union


GR No: 161690 Date: July 23, 2008
Ponente: Velasco, Jr., J.

Doctrine: The cancellation of a union’s registration doubtless has an impairing dimension on the right of labor to
self-organization. Accordingly, we can accord concurrence to the following apt observation of the BLR: [F]or fraud
and misrepresentation [to be grounds for] cancellation of union registration under Article 239 [of the Labor Code],
the nature of the fraud and misrepresentation must be grave and compelling enough to vitiate the consent of a
majority of union members.

FACTS:
Petitioner S.S. Ventures International, Inc. (Ventures), a PEZA-registered export firm is in the business of
manufacturing sports shoes. Respondent S.S. Ventures Labor Union (Union), on the other hand, is a labor
organization registered with the Department of Labor and Employment (DOLE) under Certificate of Registration
No. RO300-00-02-UR-0003. The Union filed with DOLE-Region III a petition
for certification election in behalf of the rank-and-file employees of Ventures.
Five hundredforty two (542) signatures, 82 of which belong to terminated Ventures employees, appeared on the
basic documents supporting the petition. Ventures filed a Petition to cancel the Unions certificate of registration
invoking the grounds set forth in Article 239(a) of the Labor Code, alleging the following:
(1) The Union deliberately and maliciously included the names of more or less 82 former employees no
longer connected with Ventures in its list of members who attended the organizational meeting and in the
adoption/ratification of its constitution and by-laws held on January 9, 2000 in Mariveles, Bataan; and the Union
forged the signatures of these 82 former employees to make it appear they took part in the organizational meeting
and adoption and ratification of the constitution;
(2) The Union maliciously twice entered the signatures of three persons namely: Mara Santos, Raymond
Balangbang, and Karen Agunos;
(3) No organizational meeting and ratification actually took place; and
(4) The Unions application for registration was not supported by at least 20% of the rank-and-file
employees of Ventures, or 418 of the total 2,197-employee complement. Since more or less 82 of the
500[3] signatures were forged or invalid, then the remaining valid signatures would only be 418, which is very much
short of the 439 minimum (2197 total employees x 20% = 439.4) required by the Labor Code.

In its Answer with Motion to Dismiss, the Union denied committing the imputed acts of fraud or forgery
and alleged that: (1) the organizational meeting actually took place on January 9, 2000 at the Shoe City basketball
court in Mariveles; (2) the 82 employees adverted to in Ventures petition were qualified Union members for,
although they have been ordered dismissed, the one-year prescriptive period to question their dismissal had not
yet lapsed; (3) it had complied with the 20%-member registration requirement since it had 542 members; and (4)
the double signatures were inadvertent human error.

In its supplemental reply memorandum, Ventures cited other instances of fraud and misrepresentation,
claiming that the affidavits executed by 82 alleged Union members show that they were deceived into signing
paper minutes or were harassed to signing their attendance in the organizational meeting. Ventures added that
some employees signed the affidavits denying having attended such meeting.

RD: Granted the petition


BLR: Reversed RD’s decision
Angeles| Bajana | Balladares | Brillantes | Briones | Cabansag | Callanta | Chua | David|

De Leon | Gomez | Lopez | Macalino | Nostratis | Padilla | Reynon | Santos | Tan |Velasco
4E / 4F - 2018-2019
Page 435 of 920
LABOR REVIEW DIGEST
Atty. Joyrich Golangco

CA: Dismissed Venture’s petition

Petitioner's Contention:
 Ventures faults both the BLR and the CA in finding that there was no fraud or misrepresentation on the part of
the Union sufficient to justify cancellation of its registration. In this regard, Ventures makes much of, first, the
separate hand-written statements of 82 employees who, in gist, alleged that they were unwilling or harassed
signatories to the attendance sheet of the organizational meeting.
 Ventures draws attention to the inclusion of 82 individuals to the list of participants in the January 9,
2000 organizational meeting. Ventures submits that the 82, being no longer connected with the company, should
not have been counted as attendees in the meeting and the ratification proceedings immediately afterwards.

ISSUE/S:
1. Whether or not the Union must be denied certificate of registration based on fraud and misrepresentations due to
the inclusion of the separate hand-written statements of 82 employees who, in gist, alleged that they were
unwilling or harassed signatories to the attendance sheet of the organizational meeting?
2. Whether or not the inclusion of the 82 individuals, who were no longer connected with the company, the list of
participants in the January 9, 2000 meeting and the ratification proceedings fatal to the registration?

HELD:
1. NO
As aptly noted by both the BLR and CA, these mostly undated written statements submitted by Ventures
on March 20, 2001, or seven months after it filed its petition for cancellation of registration, partake of the nature of
withdrawal of union membership executed after the Unions filing of a petition for certification election on March 21,
2000. We have in precedent cases said that the employees withdrawal from a labor union made before the filing
of the petition for certification election is presumed voluntary, while withdrawal after the filing of such petition is
considered to be involuntary and does not affect the same. Now then, if a withdrawal from union membership
done after a petition for certification election has been filed does not vitiate such petition, is it not but logical to
assume that such withdrawal cannot work to nullify the registration of the union? Upon this light, the Court is
inclined to agree with the CA that the BLR did not abuse its discretion nor gravely err when it concluded that the
affidavits of retraction of the 82 members had no evidentiary weight.

It cannot be over-emphasized that the registration or the recognition of a labor union after it has
submitted the corresponding papers is not ministerial on the part of the BLR. Far from it. After a labor organization
has filed the necessary registration documents, it becomes mandatory for the BLR to check if the requirements
under Art. 234 of the Labor Code have been sedulously complied with. If the unions application is infected by
falsification and like serious irregularities, especially those appearing on the face of the application and its
attachments, a union should be denied recognition as a legitimate labor organization. Prescinding from these
considerations, the issuance to the Union of Certificate of Registration No. RO300-00-02-UR-0003 necessarily
implies that its application for registration and the supporting documents thereof are prima facie free from any
vitiating irregularities.

2. NO
The assailed inclusion of the said 82 individuals to the meeting and proceedings adverted to is not really
fatal to the Unions cause for, as determined by the BLR, the allegations of falsification of signatures or
misrepresentation with respect to these individuals are without basis. The Court need not delve into the question
of whether these 82 dismissed individuals were still Union members qualified to vote and affix their signature on
its application for registration and supporting documents. Suffice it to say that, as aptly observed by the CA, the
procedure for acquiring or losing union membership and the determination of who are qualified or disqualified to
be members are matters internal to the union and flow from its right to self-organization.
Angeles| Bajana | Balladares | Brillantes | Briones | Cabansag | Callanta | Chua | David|

De Leon | Gomez | Lopez | Macalino | Nostratis | Padilla | Reynon | Santos | Tan |Velasco
4E / 4F - 2018-2019
Page 436 of 920
LABOR REVIEW DIGEST
Atty. Joyrich Golangco

To our mind, the relevancy of the 82 individuals active participation in the Unions organizational meeting
and the signing ceremonies thereafter comes in only for purposes of determining whether or not the Union, even
without the 82, would still meet what Art. 234(c) of the Labor Code requires to be submitted, to wit:

Art. 234. Requirements of Registration. Any applicant labor organization x x x shall acquire legal
personality and shall be entitled to the rights and privileges granted by law to legitimate labor
organizations upon issuance of the certificate of registration based on the following requirements:

xxxx

(c) The names of all its members comprising at least twenty percent (20%) of all the employees in the
bargaining unit where it seeks to operate.
The BLR, based on its official records, answered the poser in the affirmative. Wrote the BLR:

It is imperative to look into the records of respondent union with this Bureau pursuant to our role
as a central registry of union and CBA records under Article 231 of the Labor Code and Rule XVII of the
rules implementing Book V of the Labor Code, as amended x x x.

In its union records on file with this Bureau, respondent union submitted the names of [542]
members x x x. This number easily complied with the 20% requirement, be it 1,928 or 2,202 employees
in the establishment. Even subtracting the 82 employees from 542 leaves 460 union members, still
within 440 or 20% of the maximum total of 2,202 rank-and-file employees.

Whatever misgivings the petitioner may have with regard to the 82 dismissed employees is
better addressed in the inclusion-exclusion proceedings during a pre-election conference x x x. The
issue surrounding the involvement of the 82 employees is a matter of membership or voter
eligibility. It is not a ground to cancel union registration. (Emphasis added.)

The bare fact that three signatures twice appeared on the list of those who participated in the organizational
meeting would not, to our mind, provide a valid reason to cancel Certificate of Registration No. RO300-00-02-UR-
0003. As the Union tenably explained without rebuttal from Ventures, the double entries are no more than normal
human error, effected without malice. Even the labor arbiter who found for Ventures sided with the Union in its
explanation on the absence of malice.

The cancellation of a union’s registration doubtless has an impairing dimension on the right of labor to self-
organization. Accordingly, we can accord concurrence to the following apt observation of the BLR: [F]or fraud and
misrepresentation [to be grounds for] cancellation of union registration under Article 239 [of the Labor Code], the
nature of the fraud and misrepresentation must be grave and compelling enough to vitiate the consent of a
majority of union members.

In its Comment, the Union points out that for almost seven (7) years following the filing of its petition, no
certification election has yet been conducted among the rank-and-file employees. If this be the case, the delay
has gone far enough and can no longer be allowed to continue. The CA is right when it said that Ventures should
not interfere in the certification election by actively and persistently opposing the certification election of
the Union. A certification election is exclusively the concern of employees and the employer lacks the legal
personality to challenge it. In fact, jurisprudence frowns on the employers interference in a certification election for
such interference unduly creates the impression that it intends to establish a company union.

Angeles| Bajana | Balladares | Brillantes | Briones | Cabansag | Callanta | Chua | David|

De Leon | Gomez | Lopez | Macalino | Nostratis | Padilla | Reynon | Santos | Tan |Velasco
4E / 4F - 2018-2019
Page 437 of 920
LABOR REVIEW DIGEST
Atty. Joyrich Golangco

Article 245-248
1. Heritage Hotel Manila v. NUWHRAIN-HHMSC
G.R. No. 178296, January 12, 2011
Nachura J.

DOCTRINE:

Once jurisdiction is acquired by the court, it remains with it until the full termination of the case. Thus, jurisdiction
remained with the BLR despite the BLR Director's inhibition. When the DOLE Secretary resolved the appeal, she
merely stepped into the shoes of the BLR Director and performed a function that the latter could not himself
perform. She did so pursuant to her power of supervision and control over the BLR.

FACTS:

Respondent filed with the DOLE-NCR a petition for certification election. The Med-Arbiter granted the petition. On
appeal, the DOLE Secretary, affirmed the Med-Arbiter's order and remanded the case to the Med-Arbiter for the
holding of a pre-election conference. Petitioner filed a motion for reconsideration but it was denied.

The pre-election conference was not held as initially scheduled; it was held a year later. Petitioner moved to
archive or to dismiss the petition due to alleged repeated non-appearance of respondent. The latter agreed to
suspend proceedings until further notice.

Subsequently, petitioner discovered that respondent had failed to submit to the BLR its annual financial report for
several years and the list of its members since it filed its registration papers in 1995. Consequently, petitioner filed
a Petition for Cancellation of Registration of respondent, on the ground of the non-submission of the said
documents.

On June 1, 2000, petitioner reiterated its request by filing a Motion to Dismiss or Suspend the [Certification
Election] Proceedings, arguing that the dismissal or suspension of the proceedings is warranted, considering that
the legitimacy of respondent is seriously being challenged in the petition for cancellation of registration.

Nevertheless, the certification election pushed through on June 23, 2000. Respondent emerged as the winner.

On June 28, 2000, petitioner filed a Protest with Motion to Defer Certification of Election Results and Winner,
stating that the certification election held on June 23, 2000 was an exercise in futility because, once respondent's
registration is cancelled, it would no longer be entitled to be certified as the exclusive bargaining agent of the
supervisory employees.

Meanwhile, respondent filed its Answer to the petition for the cancellation of its registration. Respondent prayed
for the dismissal of the petition for the following reasons: (a) petitioner is estopped from questioning respondent's
status as a legitimate labor organization as it had already recognized respondent as such during the pre-election
conferences; (b) petitioner is not the party-in-interest, as the union members are the ones who would be
disadvantaged by the non-submission of financial reports; (c) it has already complied with the reportorial
requirements, having submitted its financial statements for 1996, 1997, 1998, and 1999, its updated list of officers,
and its list of members for the years 1995, 1996, 1997, 1998, and 1999; (d) the petition is already moot and
academic, considering that the certification election had already been held, and the members had manifested their
will to be represented by respondent.

Angeles| Bajana | Balladares | Brillantes | Briones | Cabansag | Callanta | Chua | David|

De Leon | Gomez | Lopez | Macalino | Nostratis | Padilla | Reynon | Santos | Tan |Velasco
4E / 4F - 2018-2019
Page 438 of 920
LABOR REVIEW DIGEST
Atty. Joyrich Golangco

The Med-Arbiter held that the pendency of a petition for cancellation of registration is not a bar to the holding of a
certification election. Thus, the Med-Arbiter dismissed petitioner's protest, and certified respondent as the sole and
exclusive bargaining agent of all supervisory employees.

Petitioner subsequently appealed the said Order to the DOLE Secretary. The appeal was later dismissed by
DOLE Secretary Patricia A. Sto. Tomas. Petitioner moved for reconsideration, but the motion was also denied.

In the meantime, the DOLE-NCR finally resolved the petition for cancellation of registration. While finding that
respondent had indeed failed to file financial reports and the list of its members for several years, he, nonetheless,
denied the petition, ratiocinating that freedom of association and the employees' right to self-organization are
more substantive considerations.

In view of Director Cacdac's inhibition, DOLE Secretary Sto. Tomas took cognizance of the appeal. She dismissed
the appeal, holding that the constitutionally guaranteed freedom of association and right of workers to self-
organization outweighed respondent's noncompliance with the statutory requirements to maintain its status as a
legitimate labor organization.

Petitioner filed a motion for reconsideration, but the motion was likewise denied in a resolution.

Petitioner filed a petition for certiorari with the CA, raising the issue of whether the DOLE Secretary acted with
grave abuse of discretion in taking cognizance of the appeal and affirming the dismissal of its petition for
cancellation of respondent's registration. The CA denied the petition.

ISSUE/S:

WON the SOLE properly assumed jurisdiction over the appeal of the Regional Director's Decision in the
Cancellation Petition.

WON the Court of Appeals gravely erred in affirming the dismissal of the Cancellation Petition.

HELD:

Other Notes/ SC Pronouncements:

 In the instant case, the appeal was filed by petitioner with the BLR, which, undisputedly, acquired jurisdiction over
the case. Once jurisdiction is acquired by the court, it remains with it until the full termination of the case.

Thus, jurisdiction remained with the BLR despite the BLR Director's inhibition. When the DOLE Secretary resolved
the appeal, she merely stepped into the shoes of the BLR Director and performed a function that the latter could
not himself perform. She did so pursuant to her power of supervision and control over the BLR.

It is true that the power of control and supervision does not give the Department Secretary unbridled authority to
take over the functions of his or her subordinate. Such authority is subject to certain guidelines which are stated in
Book IV, Chapter 8, Section 39(1)(a) of the Administrative Code of 1987. However, in the present case, the DOLE
Secretary's act of taking over the function of the BLR Director was warranted and necessitated by the latter's
inhibition from the case and the objective to "maintain the integrity of the decision, as well as the Bureau itself."

Angeles| Bajana | Balladares | Brillantes | Briones | Cabansag | Callanta | Chua | David|

De Leon | Gomez | Lopez | Macalino | Nostratis | Padilla | Reynon | Santos | Tan |Velasco
4E / 4F - 2018-2019
Page 439 of 920
LABOR REVIEW DIGEST
Atty. Joyrich Golangco

 Articles 238 and 239 of the Labor Code give the Regional Director ample discretion in dealing with a petition for
cancellation of a union's registration, particularly, determining whether the union still meets the requirements
prescribed by law. It is sufficient to give the Regional Director license to treat the late filing of required documents
as sufficient compliance with the requirements of the law. After all, the law requires the labor organization to
submit the annual financial report and list of members in order to verify if it is still viable and financially sustainable
as an organization so as to protect the employer and employees from fraudulent or fly-by-night unions. With the
submission of the required documents by respondent, the purpose of the law has been achieved, though
belatedly.

We cannot ascribe abuse of discretion to the Regional Director and the DOLE Secretary in denying the petition for
cancellation of respondent's registration. The union members and, in fact, all the employees belonging to the
appropriate bargaining unit should not be deprived of a bargaining agent, merely because of the negligence of the
union officers who were responsible for the submission of the documents to the BLR.

It is undisputed that appellee failed to submit its annual financial reports and list of individual members in
accordance with Article 239 of the Labor Code. However, the existence of this ground should not necessarily lead
to the cancellation of union registration. Article 239 recognizes the regulatory authority of the State to exact
compliance with reporting requirements. Yet there is more at stake in this case than merely monitoring union
activities and requiring periodic documentation thereof.

The more substantive considerations involve the constitutionally guaranteed freedom of association and right of
workers to self-organization. Also involved is the public policy to promote free trade unionism and collective
bargaining as instruments of industrial peace and democracy. An overly stringent interpretation of the statute
governing cancellation of union registration without regard to surrounding circumstances cannot be allowed.
Otherwise, it would lead to an unconstitutional application of the statute and emasculation of public policy
objectives. Worse, it can render nugatory the protection to labor and social justice clauses that pervades the
Constitution and the Labor Code.

Moreover, submission of the required documents is the duty of the officers of the union. It would be unreasonable
for this Office to order the cancellation of the union and penalize the entire union membership on the basis of the
negligence of its officers.

Angeles| Bajana | Balladares | Brillantes | Briones | Cabansag | Callanta | Chua | David|

De Leon | Gomez | Lopez | Macalino | Nostratis | Padilla | Reynon | Santos | Tan |Velasco
4E / 4F - 2018-2019
Page 440 of 920
LABOR REVIEW DIGEST
Atty. Joyrich Golangco

1. Republic vs. Kawashima Textile


GR No: 160352 Date: July 23, 2008
Ponente: AUSTRIA-MARTINEZ, J.

Doctrine: a labor organization composed of both rank-and-file and supervisory employees is no labor
organization at all. It cannot, for any guise or purpose, be a legitimate labor organization. Not being one,
an organization which carries a mixture of rank-and-file and supervisory employees cannot possess any
of the rights of a legitimate labor organization, including the right to file a petition for certification election
for the purpose of collective bargaining. It becomes necessary, therefore, anterior to the granting of an
order allowing a certification election, to inquire into the composition of any labor organization whenever
the status of the labor organization is challenged on the basis of Article 245 of the Labor Code

FACTS:

On January 24, 2000, KFWU filed with DOLE Regional Office No. IV, a Petition for Certification Election to
be conducted in the bargaining unit composed of 145 rank-and-file employees of respondent. Attached to
its petition are a Certificate of Creation of Local/Chapter6 issued on January 19, 2000 by DOLE Regional
Office No. IV, stating that it [KFWU] submitted to said office a Charter Certificate issued to it by the
national federation Phil. Transport & General Workers Organization (PTGWO), and a Report of Creation of
Local/Chapter.

Respondent filed a Motion to Dismiss the petition on the ground that KFWU did not acquire any legal
personality because its membership of mixed rank-and-file and supervisory employees violated Article
245 of the Labor Code, and its failure to submit its books of account

Med-Arbiter Bactin found KFWU’s legal personality defective and dismissed its petition for certification
election, stating that, Since petitioner’s members are mixture of rank and file and supervisory employees,
petitioner union, at this point [in] time, has not attained the status of a legitimate labor organization.
Petitioner should first exclude the supervisory employees from it membership before it can attain the
status of a legitimate labor organization.
Respondent filed with DOLE Regional Office No. IV a Petition for Cancellation of Charter/Union
Registration of KFWU,13 the final outcome of which, unfortunately, cannot be ascertained from the
records.

KFWU appealed to the DOLE which granted the appeal; ordered the case be remanded to the office of
origin for the immediate conduct of certification election xxx CA reversed. MR denied. Hence, this
petition.

ISSUE/S:
Whether a mixed membership of rank-and-file and supervisory employees in a union is a ground for the
dismissal of a petition for certification election in view of the amendment brought about by D.O. 9, series
of 1997, which deleted the phraseology in the old rule that “[t]he appropriate bargaining unit of the rank-
and-file employee shall not include the supervisory employees and/or security guards;”

Angeles| Bajana | Balladares | Brillantes | Briones | Cabansag | Callanta | Chua | David|

De Leon | Gomez | Lopez | Macalino | Nostratis | Padilla | Reynon | Santos | Tan |Velasco
4E / 4F - 2018-2019
Page 441 of 920
LABOR REVIEW DIGEST
Atty. Joyrich Golangco

HELD:

The key to the closure that petitioner seeks could have been Republic Act (R.A.) No. 9481 [AN ACT
STRENGTHENING THE WORKERS’ CONSTITUTIONAL RIGHT TO SELF-ORGANIZATION, AMENDING FOR
THE PURPOSE PRESIDENTIAL DECREE NO. 442, AS AMENDED, OTHERWISE KNOWN AS THE LABOR
CODE OF THE PHILIPPINES] Sections 8 and 9 (See NOTES)

However, R.A. No. 9481 took effect only on June 14, 2007;26 hence, it applies only to labor representation cases
filed on or after said date.27 As the petition for certification election subject matter of the present petition was filed
by KFWU on January 24, 2000,28 R.A. No. 9481 cannot apply to it. There may have been curative labor
legislations29 that were given retrospective effect,30 but not the aforecited provisions of R.A. No. 9481, for
otherwise, substantive rights and interests already vested would be impaired in the process.

Instead, the law and rules in force at the time of the filing by KFWU of the petition for certification election on
January 24, 2000 are R.A. No. 6715,32 amending Book V of Presidential Decree (P.D.) No. 442 (Labor
Code),33as amended, and the Rules and Regulations Implementing R.A. No. 6715,34 as amended by
Department Order No. 9, series of 1997.35

One area of contention has been the composition of the membership of a labor organization, specifically whether
there is a mingling of supervisory and rank-and-file employees and how such questioned mingling affects its
legitimacy.

Effective 1989, R.A. No. 6715 restored the prohibition against the questioned mingling in one labor organization,
viz:

Sec. 18. Article 245 of the same Code, as amended, is hereby further amended to read as follows
“Art. 245. Ineligibility of managerial employees to join any labor organization; right of supervisory employees.
Managerial employees are not eligible to join, assist or form any labor organization. Supervisory employees shall
not be eligible for membership in a labor organization of the rank-and-file employees but may join, assist or form
separate labor organizations of their own.” (Emphasis supplied)

Unfortunately, just like R.A. No. 875, R.A. No. 6715 omitted specifying the exact effect any violation of the
prohibition would bring about on the legitimacy of a labor organization.

Thus, when the issue of the effect of mingling was brought to the fore in Toyota,48 the Court, citing Article 245 of
the Labor Code, as amended by R.A. No. 6715, held:

Clearly, based on this provision, a labor organization composed of both rank-and-file and supervisory employees
is no labor organization at all. It cannot, for any guise or purpose, be a legitimate labor organization. Not being
one, an organization which carries a mixture of rank-and-file and supervisory employees cannot possess any of
the rights of a legitimate labor organization, including the right to file a petition for certification election for the
purpose of collective bargaining. It becomes necessary, therefore, anterior to the granting of an order allowing a
certification election, to inquire into the composition of any labor organization whenever the status of the labor
organization is challenged on the basis of Article 245 of the Labor Code xxxx

In the case at bar, as respondent union’s membership list contains the names of at least twenty-seven (27)
supervisory employees in Level Five positions, the union could not, prior to purging itself of its supervisory
employee members, attain the status of a legitimate labor organization. Not being one, it cannot possess the
requisite personality to file a petition for certification election.

Angeles| Bajana | Balladares | Brillantes | Briones | Cabansag | Callanta | Chua | David|

De Leon | Gomez | Lopez | Macalino | Nostratis | Padilla | Reynon | Santos | Tan |Velasco
4E / 4F - 2018-2019
Page 442 of 920
LABOR REVIEW DIGEST
Atty. Joyrich Golangco

But then, on June 21, 1997, the 1989 Amended Omnibus Rules was further amended by Department Order No. 9,
series of 1997 (1997 Amended Omnibus Rules). Specifically, the requirement under Sec. 2(c) of the 1989
Amended Omnibus Rules – that the petition for certification election indicate that the bargaining unit of rank-and-
file employees has not been mingled with supervisory employees – was removed.

Consequently, the Court reverses the ruling of the CA and reinstates that of the DOLE granting the petition for
certification election of KFWU.

Abaria v. NLRC
G.R. No. 154113; Dec. 7, 2011
Peralta, J.:

DOCTRINE: The rights enumerated in Art. 242 (now 251) of the Labor Code pertain to Legitimate Labor
Organizations.

FACTS:

The Metro Cebu Community Hospital, Inc. (MCCHI; now the Visayas Community Medical Center) owns and
operated the Metro Cebu Community Hospital (MCCH), while the National Federation of Labor (NFL) was the
exclusive bargaining agent of the MCCH. The NFL Regional Director for the area was Atty. Alforque and Paterno
A. Lumapguid was President of NFL-MCCH Chapter. In the CBA effective from January 1994 until December 31,
1995, the signatories were Sheila E. Buot as Board of Trustees Chairman, Rev. Iyoy as MCCH Administrator and
Atty. Fernando Yu as Legal Counsel of NFL, while Perla Nava, President of Nagkahiusang Mamumuo sa MCCH
(NAMA-MCCH-NFL) signed the Proof of Posting.

Dec 1995, Nava wrote to Rev. Iyoy as hospital administrator, expressing the union’s desire to renew the CBA and
attached a statement of proposals, endorsed by 153 union members. At this point Atty. Alforque informed Fr.
Nava that the NFL was never informed of the proposed CBA, nor had it authorized Nava et. al. to negotiate a new
CBA. Following this incident, Nava and several others had their union membership for violations the Consti and
By-Laws after due process. Thereafter, Nava led a series of mass actions which included black and red armbands
and headbands, marching around the premises, uttering slanderous words against the hospital and its board, and
putting up signs of protest. The NFL immediately disowned the concerted actions. Nava and her group were
immediately placed under preventive suspension. For their part they insisted the MCCH comply with its duty to
bargain collectively. At this point the DOLE issued certifications to the effect that NAMA-MCCH-NFL that there

Angeles| Bajana | Balladares | Brillantes | Briones | Cabansag | Callanta | Chua | David|

De Leon | Gomez | Lopez | Macalino | Nostratis | Padilla | Reynon | Santos | Tan |Velasco
4E / 4F - 2018-2019
Page 443 of 920
LABOR REVIEW DIGEST
Atty. Joyrich Golangco

was nothing in their records showing that the same was an LLO. Because of this, MCCH issued notices to the
striking members requiring them to explain why they should not be dismissed.

NAMA-MCCH-NFL filed a notice of strike but the same was denied by the NCMB but the same was denied for
want of legal personality, not be an LLO. Despite the same, Nava and her group staged an illegal strike, going to
the extent of barricading the hospital. The illegal strike resulted in the dismissal of the striking workers. Thereafter,
several actions for illegal dismissal and other claims were filed- more particular charging MCCH with an Unfair
Labor Practice.

Executive LA: The LA dismissed the action and found ground for the ULP, declaring the strike and picketing
activities as illegal for being conducted by a group not qualifying as a legitimate Labor Organization.

NLRC RULING: Upheld the LA except for modifying the several decisions awarding separation pay and attorney’s
fees.

CA RULING: Upheld LA and NLRC.

APPEAL TO THE SC:

Petitioner’s Contention: that the lower bodies erred in not holding that MCCH was guilty of a ULP for their unjust
refusal to collectively bargain..

Respondent’s Contention: that there was no duty to collectively bargain.

ISSUE: Whether the NMA-MCCH-NFL had the right to collectively bargain.

RULING: NO. Records of the NCMB and DOLE Region 7 confirmed that NAMA-MCCH-NFL had not registered as
a labor organization, having submitted only its charter certificate as an affiliate or local chapter of NFL. Not being a
legitimate labor organization, NAMA- MCCH-NFL is not entitled to those rights granted to a legitimate labor
organization under Art. 242 (now 251).

NAMA-MCCH-NFL is not the labor organization certified or designated by the majority of the rank-and-file hospital
employees to represent them in the CBA negotiations but the NFL, as evidenced by CBAs concluded in 1987,
1991 and 1994. While it is true that a local union has the right to disaffiliate from the national federation, NAMA-
MCCH-NFL has not done so as there was no any effort on its part to comply with the legal requisites for a valid
disafiliation during the "freedom period" 39 or the last 60 days of the last year of the CBA, through a majority vote
in a secret balloting in accordance with Art. 241 (d). Nava and her group simply demanded that MCCHI directly
negotiate with the local union which has not even registered as one. Thus, it cannot demand the rights of a
legitimate labor organization, as it is not a legitimate labor organization in the first place.

Article 263
1. UNION OF FILIPRO EMPLOYEES - DRUG, FOOD AND ALLIED INDUSTRIES UNIONS - KILUSANG MAYO
UNO (UFE-DFA-KMU) vs NESTLÉ PHILIPPINES, INCORPORATED
G.R. Nos. 158930-31; March 3, 2008
Angeles| Bajana | Balladares | Brillantes | Briones | Cabansag | Callanta | Chua | David|

De Leon | Gomez | Lopez | Macalino | Nostratis | Padilla | Reynon | Santos | Tan |Velasco
4E / 4F - 2018-2019
Page 444 of 920
LABOR REVIEW DIGEST
Atty. Joyrich Golangco

NESTLÉ PHILIPPINES, INCORPORATED vs UNION OF FILIPRO EMPLOYEES - DRUG, FOOD AND ALLIED
INDUSTRIES UNIONS - KILUSANG MAYO UNO (UFE-DFA-KMU)
G.R. Nos. 158944-45; March 3, 2008
CHICO-NAZARIO, J.:

DOCTRINE:
While the law makes it an obligation for the employer and the employees to bargain collectively with each other,
such compulsion does not include the commitment to precipitately accept or agree to the proposals of the other.
All it contemplates is that both parties should approach the negotiation with an open mind and make reasonable
effort to reach a common ground of agreement.

FACTS: UFE-DFA-KMU was the sole and exclusive bargaining agent of the rank-and-file employees of Nestlé
belonging to the latter’s Alabang and Cabuyao plants. As the existing collective bargaining agreement (CBA)
between Nestlé and UFE-DFA-KMU was to end the Presidents of the Alabang and Cabuyao Divisions of UFE-
DFA-KMU informed Nestlé of their intent to "open new Collective Bargaining Negotiation for the year 2001-2004.
In response thereto, Nestle, through a letter, stated that "unilateral grants, one-time company grants, company-
initiated policies and programs, which include, but are not limited to the Retirement Plan, Incidental Straight Duty
Pay and Calling Pay Premium, are by their very nature not proper subjects of CBA negotiations and therefore
shall be excluded therefrom."
After series of dialogue between the parties, Nestlé requested the National Conciliation and Mediation Board
(NCMB) to conduct preventive mediation proceedings between it and UFE-DFA-KMU. However, such conciliation
is futile and the UFE-DFA-KMU filed a two Notices of Strike on different occasion, first due to a bargaining
deadlock pertaining to economic issues, i.e., "retirement (plan), panel composition, costs and attendance, and
CBA", and another second on Nestlé’s alleged unfair labor practices, that is, bargaining in bad faith by setting pre-
conditions in the ground rules and/or refusing to include the issue of the Retirement Plan in the CBA negotiations.
The employee members of UFE-DFA-KMU at Nestlé’s Cabuyao Plant went on strike. Secretary of DOLE
assumed jurisdiction over the subject labor dispute and directed the members of UFE-DFA-KMU to return-to-
work within twenty-four (24) hours from receipt of such Order.
In an attempt to finally resolve the crippling labor dispute between the parties, then Acting Secretary of the DOLE,
came out with an Order which (1) recognizes the present Retirement Plan at the Nestlé Cabuyao Plant, and (2)
Union’s charge of unfair labor practice against the Company is hereby dismissed for lack of merit; among others.
Unsatified, UFE-DFA-KMU went to the Court of Appeals via another Petition for Certiorari seeking to annul the
Orders. CA ruled entirely in favor of UFE-DFA-KMU. Both parties appealed from the decision via Petitions for
Review on Certiorari under Rule 45.
G.R. No. 158930-31 - was filed by UFE-DFA-KMU against Nestlé seeking to reverse the Court of Appeals
Decision insofar as the appellate court’s failure to find Nestlé guilty of unfair labor practice was concerned, arguing
that Nestlé’s "refusal to bargain on a very important CBA economic provision constitutes unfair labor practice." It
explains that Nestlé set as a precondition for the holding of collective bargaining negotiations the non-inclusion of
the issue of Retirement Plan.
G.R. No. 158944-45 - was instituted by Nestlé against UFE-DFA-KMU likewise looking to annul and set aside the
part of the Court of Appeals Decision declaring that the Retirement Plan was a valid collective bargaining issue

ISSUE’s: Whether or not Nestle committed unfair labor practice when it considers some economic benefits as
unilaterally granted and therefore excluded from CBA negotiations.

HELD: NO. Nestle is not guilty of Unfair Labor Practice.


The duty to bargain collectively is mandated by the Labor Code. Obviously, the purpose of collective bargaining is
the reaching of an agreement resulting in a contract binding on the parties; but the failure to reach an agreement
after negotiations have continued for a reasonable period does not establish a lack of good faith. The statutes
Angeles| Bajana | Balladares | Brillantes | Briones | Cabansag | Callanta | Chua | David|

De Leon | Gomez | Lopez | Macalino | Nostratis | Padilla | Reynon | Santos | Tan |Velasco
4E / 4F - 2018-2019
Page 445 of 920
LABOR REVIEW DIGEST
Atty. Joyrich Golangco

invite and contemplate a collective bargaining contract, but they do not compel one. The duty to bargain does not
include the obligation to reach an agreement.
For a charge of unfair labor practice to prosper, it must be shown that Nestlé was motivated by ill will, "bad faith,
or fraud, or was oppressive to labor, or done in a manner contrary to morals, good customs, or public policy, and,
of course, that social humiliation, wounded feelings, or grave anxiety resulted x x x" in disclaiming unilateral grants
as proper subjects in their collective bargaining negotiations. While the law makes it an obligation for the employer
and the employees to bargain collectively with each other, such compulsion does not include the commitment to
precipitately accept or agree to the proposals of the other. All it contemplates is that both parties should approach
the negotiation with an open mind and make reasonable effort to reach a common ground of agreement.
Herein, the union merely bases its claim of refusal to bargain on a letter written by Nestlé where the latter laid
down its position that "unilateral grants, one-time company grants, company-initiated policies and programs,
which include, but are not limited to the Retirement Plan, Incidental Straight Duty Pay and Calling Pay Premium,
are by their very nature not proper subjects of CBA negotiations and therefore shall be excluded therefrom." But
as we have stated in this Court’s Decision, said letter is not tantamount to refusal to bargain. In thinking to exclude
the issue of Retirement Plan from the CBA negotiations, Nestlé, cannot be faulted for considering the same
benefit as unilaterally granted, considering that eight out of nine bargaining units have allegedly agreed to treat
the Retirement Plan as a unilaterally granted benefit. This is not a case where the employer exhibited an
indifferent attitude towards collective bargaining, because the negotiations were not the unilateral activity of the
bargaining representative. Nestlé’s desire to settle the dispute and proceed with the negotiation being evident in
its cry for compulsory arbitration is proof enough of its exertion of reasonable effort at good-faith bargaining.
In the case at bar, Nestle never refused to bargain collectively with UFE-DFA-KMU. The corporation simply
wanted to exclude the Retirement Plan from the issues to be taken up during CBA negotiations, on the postulation
that such was in the nature of a unilaterally granted benefit.
Also, a meticulous review of the record and pleadings of the cases at bar shows that, of the two notices of strike
filed by UFE-DFA-KMU before the NCMB, it was only on the second that the ground of unfair labor practice was
alleged. A perusal of the allegations and arguments raised by UFE-DFA-KMU in the Memorandum (in G.R. Nos.
158930-31) will readily disclose the need for the presentation of evidence other than its bare contention of unfair
labor practice in order to make certain the propriety or impropriety of the ULP charge hurled against Nestlé. Under
Rule XIII, Sec. 4, Book V of the Implementing Rules of the Labor Code:
x x x. In cases of unfair labor practices, the notice of strike shall as far as practicable, state the acts complained
of and the efforts to resolve the dispute amicably." (Emphasis supplied.)
In the case at bar, except for the assertion put forth by UFE-DFA-KMU, neither the second Notice of Strike nor the
records of these cases substantiate a finding of unfair labor practice. It is not enough that the union believed that
the employer committed acts of unfair labor practice when the circumstances clearly negate even a prima facie
showing to warrant such a belief. Herein, no proof was presented to exemplify bad faith on the part of Nestlé apart
from mere allegation. Accordingly, since UFE-DFA-KMU failed to proffer substantial evidence that would
overcome the legal presumption of good faith on the part of Nestlé, the award of moral and exemplary damages is
unavailing.
————————————————————————

Angeles| Bajana | Balladares | Brillantes | Briones | Cabansag | Callanta | Chua | David|

De Leon | Gomez | Lopez | Macalino | Nostratis | Padilla | Reynon | Santos | Tan |Velasco
4E / 4F - 2018-2019
Page 446 of 920
LABOR REVIEW DIGEST
Atty. Joyrich Golangco

2. UST Faculty Union vs. UST


GR No: 180892 Date: April 7, 2009
Ponente: Velasco, Jr., J.

Doctrine: It is not the duty or obligation of respondents [employer] to inquire into the validity of the election of the
Gamilla Group. Such issue is properly an intra-union controversy subject to the jurisdiction of the med-arbiter of
the DOLE. Respondents could not have been expected to stop dealing with the Gamilla Group on the mere
accusation of the Mariño Group that the former was not validly elected into office.

FACTS: University of Santo Tomas Faculty Union (USTFU) wrote a letter to all its members informing them of a
General Assembly (GA) that was to be held on October 5, 1996. The letter contained an agenda for the GA which
included an election of officers. The then incumbent president of the USTFU was Atty. Eduardo J. Mariño, Jr.

On October 2, 1996, Fr. Rodel Aligan, O.P., Secretary General of the UST, issued a Memorandum allowing the
request of the Faculty Clubs of the university to hold a convocation on October 4, 1996.

Members of the faculties of the university attended the convocation, including members of the USTFU, without the
participation of the members of the UST administration. Also during the convocation, an election for the officers of
the USTFU was conducted by a group called the Reformist Alliance. Upon learning that the convocation was
intended to be an election, members of the USTFU walked out. Meanwhile, an election was conducted among
those present, and Gil Gamilla and other faculty members (Gamilla Group) were elected as the president and
officers, respectively, of the union. Such election was communicated to the UST administration. Thus, there were
two (2) groups claiming to be the USTFU: the Gamilla Group and the group led by Atty. Mariño, Jr. (Mariño
Group).

The Mariño Group filed a complaint for ULP against the UST with the Arbitration Branch of the NLRC praying for
the nullification of the election of the Gamilla Group as officers of the USTFU. Then, a CBA was entered into by
the Gamilla Group and the UST. The CBA superseded an existing CBA entered into by the UST and USTFU.

The med-arbiter issued a Resolution, declaring the election of the Gamilla group as null and void and ordering that
this group cease and desist from performing the duties and responsibilities of USTFU officers. The med-arbiter
issued a Resolution, declaring the election of the Gamilla group as null and void and ordering that this group
cease and desist from performing the duties and responsibilities of USTFU officers. This Resolution was appealed
to the BLR which affirmed the Resolution of the med-arbiter. His Resolution was then appealed to this Court which
rendered its November 16, 1999 Decision

Thus, USTFU then filed a Manifestation with the Arbitration Branch of the NLRC informing it of the Decision of the
Court. Thereafter, the Arbitration Branch of the NLRC issued a Decision dismissing the complaint for lack of merit.

The complaint was dismissed on the ground that USTFU failed to establish with clear and convincing evidence
that indeed UST was guilty of ULP. The acts of UST which USTFU complained of as ULP were the following: (1)
allegedly calling for a convocation of faculty members which turned out to be an election of officers for the faculty
union; (2) subsequently dealing with the Gamilla Group in establishing a new CBA.

LA/RTC/NLRC/CA RULING: The labor arbiter ruled that when the new CBA was entered into, (1) the Gamilla
Group presented more than sufficient evidence to establish that they had been duly elected as officers of the
USTFU; and (2) the ruling of the med-arbiter that the election of the Gamilla Group was null and void was not yet
final and executory. Thus, UST was justified in dealing with and entering into a CBA with the Gamilla Group,
including helping the Gamilla Group in securing the USTFU office.
Angeles| Bajana | Balladares | Brillantes | Briones | Cabansag | Callanta | Chua | David|

De Leon | Gomez | Lopez | Macalino | Nostratis | Padilla | Reynon | Santos | Tan |Velasco
4E / 4F - 2018-2019
Page 447 of 920
LABOR REVIEW DIGEST
Atty. Joyrich Golangco

APPEAL TO THE SC:

Petitioner's Contention: Petitioner claims that the labor arbiter, NLRC, and CA should have found that UST is
guilty of ULP. Petitioner enumerates the acts constituting ULP as follows: (1) respondents provided the Gamilla
Group with the facilities and forum to conduct elections, in the guise of a convocation; and (2) respondents
transacted business with the Gamilla Group such as the processing of educational and hospital benefits,
deducting USTFU dues from the faculty members without turning over the dues to the Mariño Group, and entering
into a CBA with them.

Respondent's Contention: (not discussed)

ISSUE/S:
1. Whether or not UST was in connivance with the Gamilla group in the election of the former as officers
2. Whether or not UST committed ULP in recognizing the Gamilla Group instead of the Marino Group as the
USTFU

HELD: 1. NO. Petitioner avers that: "Indeed, Respondents, under the guise of a faculty convocation, ordered the
suspension of classes and required the faculty members to attend the supposed faculty convocation which was to
be held at the Education Auditorium of the University of Santo Tomas." An examination of the Memorandum dated
October 2, 1996 would, however, rebut such allegation. In no way can the contents of the memorandum be
interpreted to mean that faculty members were required to attend the convocation. Not one coercive term was
used in the memorandum to show that the faculty club members were compelled to attend such convocation. And
the phrase "we are allowing them to hold a convocation" negates any idea that the UST would participate in the
proceedings. More importantly, USTFU itself even admitted that during the October 4, 1996 convocation/election,
not a single University Official was present.

In other words, the Memorandum dated October 2, 1996 does not support a claim that UST organized the
convocation in connivance with the Gamilla Group.

2. NO. In the instant case, until our Decision that the Gamilla Group was not validly elected into office, there was
no reason to believe that the members of the Gamilla Group were not the validly elected officers and directors of
USTFU. To reiterate, the Gamilla Group submitted a Letter dated October 4, 1996 whereby it informed Fr.
Rolando De La Rosa that its members were the newly elected officers and directors of USTFU.

More important though is the fact that the records are bereft of any evidence to show that the Mariño Group
informed the UST of their objections to the election of the Gamilla Group. In fact, there is even no evidence to
show that the scheduled elections on October 5, 1996 that was supposed to be presided over by the Mariño
Group ever pushed through. Instead, petitioner filed a complaint with the med-arbiter praying for the nullification of
the election of the Gamilla Group.

As such, there was no reason not to recognize the Gamilla Group as the new officers and directors of USTFU.
UST was obligated to deal with the USTFU, as the recognized representative of the bargaining unit, through the
Gamilla Group. UST’s failure to negotiate with the USTFU would have constituted ULP.

It is not the duty or obligation of respondents [employer] to inquire into the validity of the election of the Gamilla
Group. Such issue is properly an intra-union controversy subject to the jurisdiction of the med-arbiter of the DOLE.
Angeles| Bajana | Balladares | Brillantes | Briones | Cabansag | Callanta | Chua | David|

De Leon | Gomez | Lopez | Macalino | Nostratis | Padilla | Reynon | Santos | Tan |Velasco
4E / 4F - 2018-2019
Page 448 of 920
LABOR REVIEW DIGEST
Atty. Joyrich Golangco

Respondents could not have been expected to stop dealing with the Gamilla Group on the mere accusation of the
Mariño Group that the former was not validly elected into office.
————————————————————————

Angeles| Bajana | Balladares | Brillantes | Briones | Cabansag | Callanta | Chua | David|

De Leon | Gomez | Lopez | Macalino | Nostratis | Padilla | Reynon | Santos | Tan |Velasco
4E / 4F - 2018-2019
Page 449 of 920
LABOR REVIEW DIGEST
Atty. Joyrich Golangco

3. GENERAL MILLING CORPORATION, petitioner, vs. HON. COURT OF APPEALS, GENERAL MILLING
CORPORATION INDEPENDENT LABOR UNION (GMC-ILU), and RITO MANGUBAT, respondents.

Doctrine:

FACTS: In its two plants located at Cebu City and Lapu-Lapu City, petitioner General Milling Corporation (GMC)
employed 190 workers. They were all members of private respondent General Milling Corporation Independent
Labor Union (union, for brevity), a duly certified bargaining agent. A day before the expiration of the CBA, the
union sent GMC a proposed CBA, with a request that a counter-proposal be submitted within ten (10) days. GMC
had received collective and individual letters from workers who stated that they had withdrawn from their union
membership, on grounds of religious affiliation and personal differences. Believing that the union no longer had
standing to negotiate a CBA, GMC did not send any counter-proposal. Thus, the union filed, on July 2, 1992, a
complaint against GMC with the NLRC, Arbitration Division, Cebu City. The complaint alleged unfair labor practice
on the part of GMC for: (1) refusal to bargain collectively; (2) interference with the right to selforganization; and (3)
discrimination. The labor arbiter dismissed the case with the recommendation that a petition for certification
election be held to determine if the union still enjoyed the support of the workers.
NLRC ordered GMC to abide by the CBA draft that the union proposed for a period of two (2) years. NLRC
pointed out that upon the effectivity of Rep. Act No. 6715, the duration of a CBA, insofar as the representation
aspect is concerned, is five (5) years which, in the case of GMC-Independent Labor Union was from December 1,
1988 to November 30, 1993. All other provisions of the CBA are to be renegotiated not later than three (3) years
after its execution. Thus, the NLRC held that respondent union remained as the exclusive bargaining agent with
the right to renegotiate the economic provisions of the CBA. Consequently, it was unfair labor practice for GMC
not to enter into negotiation with the union.

ISSUE/S: Did the company commit ULP?

RULING: Yes. The law mandates that the representation provision of a CBA should last for five years. The
relation between labor and management should be undisturbed until the last 60 days of the fifth year. Hence, it is
indisputable that when the union requested for a renegotiation of the economic terms of the CBA on November
29, 1991, it was still the certified collective bargaining agent of the workers, because it was seeking said
renegotiation within five (5) years from the date of effectivity of the CBA on December 1, 1988. The unions
proposal was also submitted within the prescribed 3-year period from the date of effectivity of the CBA, albeit just
before the last day of said period. It was obvious that GMC had no valid reason to refuse to negotiate in good faith
with the union. For refusing to send a counter-proposal to the union and to bargain anew on the economic terms
of the CBA, the company committed an unfair labor practice under Article 248 of the Labor Code.
————————————————————————

Angeles| Bajana | Balladares | Brillantes | Briones | Cabansag | Callanta | Chua | David|

De Leon | Gomez | Lopez | Macalino | Nostratis | Padilla | Reynon | Santos | Tan |Velasco
4E / 4F - 2018-2019
Page 450 of 920
LABOR REVIEW DIGEST
Atty. Joyrich Golangco

4. KIOK LOY, doing business in the name of SWEDEN ICE CREAM PLANT vs NLRC
GR No: L-54334 Date:January 22, 1986
Ponente: Cuevas, J.

Doctrine: While it is a mutual obligation of the parties to bargain, the employer, however, is not under any legal
duty to initiate contract negotiation. The mechanics of collective bargaining is set in motion only when the
following jurisdictional preconditions are present, namely, (1) possession of the status of majority representation of
the employees' representative in accordance with any of the means of selection or designation provided for by the
Labor Code; (2) proof of majority representation; and (3) a demand to bargain under Article 251, par. (a) of the
New Labor Code . ... all of which preconditions are undisputedly present in the instant case.
From the over-all conduct of petitioner company in relation to the task of negotiation, there can be no doubt that
the Union has a valid cause to complain against its (Company's) attitude, the totality of which is indicative of the
latter's disregard of, and failure to live up to, what is enjoined by the Labor Code — to bargain in good faith.

FACTS:
In a certification election held on October 3, 1978, the Pambansang Kilusang Paggawa (Union for short), a
legitimate late labor federation, won and was subsequently certified in a resolution dated November 29, 1978 by
the Bureau of Labor Relations as the sole and exclusive bargaining agent of the rank-and-file employees of
Sweden Ice Cream Plant (Company for short). The Company's motion for reconsideration of the said resolution
was denied on January 25, 1978.
Thereafter, and more specifically on December 7, 1978, the Union furnished the Company with two copies of its
proposed collective bargaining agreement. At the same time, it requested the Company for its counter proposals.
Eliciting no response to the aforesaid request, the Union again wrote the Company reiterating its request for
collective bargaining negotiations and for the Company to furnish them with its counter proposals. Both requests
were ignored and remained unacted upon by the Company.
Left with no other alternative in its attempt to bring the Company to the bargaining table, the Union, on February
14, 1979, filed a "Notice of Strike", with the Bureau of Labor Relations (BLR) on ground of unresolved economic
issues in collective bargaining.
Conciliation proceedings then followed during the thirty-day statutory cooling-off period. But all attempts towards
an amicable settlement failed, prompting the Bureau of Labor Relations to certify the case to the National Labor
Relations Commission (NLRC) for compulsory arbitration pursuant to Presidential Decree No. 823, as amended.
The labor arbiter, Andres Fidelino, to whom the case was assigned, set the initial hearing for April 29, 1979. For
failure however, of the parties to submit their respective position papers as required, the said hearing was
cancelled and reset to another date. Meanwhile, the Union submitted its position paper. The Company did not,
and instead requested for a resetting which was granted. The Company was directed anew to submit its financial
statements for the years 1976, 1977, and 1978.
The case was further reset to May 11, 1979 due to the withdrawal of the Company's counsel of record, Atty.
Rodolfo dela Cruz. On May 24, 1978, Atty. Fortunato Panganiban formally entered his appearance as counsel for
the Company only to request for another postponement allegedly for the purpose of acquainting himself with the
case. Meanwhile, the Company submitted its position paper on May 28, 1979.
When the case was called for hearing on June 4, 1979 as scheduled, the Company's representative, Mr. Ching,
who was supposed to be examined, failed to appear. Atty. Panganiban then requested for another postponement
which the labor arbiter denied. He also ruled that the Company has waived its right to present further evidence
and, therefore, considered the case submitted for resolution.

NLRC RULING: Sweden Ice Cream is guilty of unjustified refusal to bargain, in violation of Section (g) Article 248
(now Article 249), of P.D. 442, as amended. Further, the draft proposal for a collective bargaining agreement
attached and made an integral part of this decision, sent by the Union (Private respondent) to the respondent
(petitioner herein) and which is found to be reasonable under the premises, is declared to be the collective
agreement which should govern the relationship between the parties herein.
Angeles| Bajana | Balladares | Brillantes | Briones | Cabansag | Callanta | Chua | David|

De Leon | Gomez | Lopez | Macalino | Nostratis | Padilla | Reynon | Santos | Tan |Velasco
4E / 4F - 2018-2019
Page 451 of 920
LABOR REVIEW DIGEST
Atty. Joyrich Golangco

Petitioner's Contention:
Assailing the aforesaid decision contending that the National Labor Relations Commission acted without or in
excess of its jurisdiction or with grave abuse of discretion amounting to lack of jurisdiction in rendering the
challenged decision. This Court dismissed the petition for lack of merit. Upon motion of the petitioner, however,
the Resolution of dismissal was reconsidered and the petition was given due course in a Resolution dated April 1,
1981.
Petitioner Company now maintains that its right to procedural due process has been violated when it was
precluded from presenting further evidence in support of its stand and when its request for further postponement
was denied. Petitioner further contends that the National Labor Relations Commission's finding of unfair labor
practice for refusal to bargain is not supported by law and the evidence considering that it was only on May 24,
1979 when the Union furnished them with a copy of the proposed Collective Bargaining Agreement and it was
only then that they came to know of the Union's demands; and finally, that the Collective Bargaining Agreement
approved and adopted by the National Labor Relations Commission is unreasonable and lacks legal basis.

ISSUE/S:
Whether or not the Petitioner Company is guilty of unfair labor practice?

HELD: YES
Collective bargaining which is defined as negotiations towards a collective agreement, is one of the democratic
frameworks under the New Labor Code, designed to stabilize the relation between labor and management and to
create a climate of sound and stable industrial peace. It is a mutual responsibility of the employer and the Union
and is characterized as a legal obligation. So much so that Article 249, par. (g) of the Labor Code makes it an
unfair labor practice for an employer to refuse "to meet and convene promptly and expeditiously in good faith for
the purpose of negotiating an agreement with respect to wages, hours of work, and all other terms and conditions
of employment including proposals for adjusting any grievance or question arising under such an agreement and
executing a contract incorporating such agreement, if requested by either party.
While it is a mutual obligation of the parties to bargain, the employer, however, is not under any legal duty to
initiate contract negotiation. The mechanics of collective bargaining is set in motion only when the following
jurisdictional preconditions are present, namely, (1) possession of the status of majority representation of the
employees' representative in accordance with any of the means of selection or designation provided for by the
Labor Code; (2) proof of majority representation; and (3) a demand to bargain under Article 251, par. (a) of the
New Labor Code . ... all of which preconditions are undisputedly present in the instant case.
From the over-all conduct of petitioner company in relation to the task of negotiation, there can be no doubt that
the Union has a valid cause to complain against its (Company's) attitude, the totality of which is indicative of the
latter's disregard of, and failure to live up to, what is enjoined by the Labor Code — to bargain in good faith.
We are in total conformity with respondent NLRC's pronouncement that petitioner Company is GUILTY of unfair
labor practice. It has been indubitably established that (1) respondent Union was a duly certified bargaining agent;
(2) it made a definite request to bargain, accompanied with a copy of the proposed Collective Bargaining
Agreement, to the Company not only once but twice which were left unanswered and unacted upon; and (3) the
Company made no counter proposal whatsoever all of which conclusively indicate lack of a sincere desire to
negotiate. A Company's refusal to make counter proposal if considered in relation to the entire bargaining
process, may indicate bad faith and this is specially true where the Union's request for a counter proposal is left
unanswered. Even during the period of compulsory arbitration before the NLRC, petitioner Company's approach
and attitude-stalling the negotiation by a series of postponements, non-appearance at the hearing conducted, and
undue delay in submitting its financial statements, lead to no other conclusion except that it is unwilling to
negotiate and reach an agreement with the Union. Petitioner has not at any instance, evinced good faith or
willingness to discuss freely and fully the claims and demands set forth by the Union much less justify its
opposition thereto.

Angeles| Bajana | Balladares | Brillantes | Briones | Cabansag | Callanta | Chua | David|

De Leon | Gomez | Lopez | Macalino | Nostratis | Padilla | Reynon | Santos | Tan |Velasco
4E / 4F - 2018-2019
Page 452 of 920
LABOR REVIEW DIGEST
Atty. Joyrich Golangco

The case at bar is not a case of first impression, for in the Herald Delivery Carriers Union (PAFLU) vs. Herald
Publications the rule had been laid down that "unfair labor practice is committed when it is shown that the
respondent employer, after having been served with a written bargaining proposal by the petitioning Union, did not
even bother to submit an answer or reply to the said proposal This doctrine was reiterated anew in Bradman vs.
Court of Industrial Relations wherein it was further ruled that "while the law does not compel the parties to reach
an agreement, it does contemplate that both parties will approach the negotiation with an open mind and make a
reasonable effort to reach a common ground of agreement”.
————————————————————————

Angeles| Bajana | Balladares | Brillantes | Briones | Cabansag | Callanta | Chua | David|

De Leon | Gomez | Lopez | Macalino | Nostratis | Padilla | Reynon | Santos | Tan |Velasco
4E / 4F - 2018-2019
Page 453 of 920
LABOR REVIEW DIGEST
Atty. Joyrich Golangco

5. Colegio De San Juan De Letran v. AEFL


G.R. No. 141471, September 18, 2000
Kapunan J.

DOCTRINE:

FACTS:

Salvador Abtria, then President of respondent union, Association of Employees and Faculty of Letran, initiated the
renegotiation of its Collective Bargaining Agreement with petitioner for the last two (2) years of the CBA's five
(5) year lifetime from 1989-1994. On the same year, the union elected a new set of officers wherein private
respondent Eleanor Ambas emerged as the newly elected President.

Ambas wanted to continue the renegotiation of the CBA but petitioner claimed that the CBA was already prepared
for signing by the parties. The parties submitted the disputed CBA to a referendum by the union members, who
eventually rejected the said CBA.

On January 18, 1996, the parties agreed to disregard the unsigned CBA and to start negotiation on a new five-
year CBA starting 1994-1999. On February 7, 1996, the union submitted its proposals to petitioner, which notified
the union that the same had been submitted to its Board of Trustees. In the meantime, Ambas was informed from
her superior that her work schedule was being changed from Monday to Friday to Tuesday to Saturday. Ambas
protested and requested management to submit the issue to a grievance machinery under the old CBA.

Due to petitioner's inaction, the union filed a notice of strike on March 13, 1996. On March 29, 1996, the union
received petitioner's letter dismissing Ambas for alleged insubordination. Hence, the union amended its notice of
strike to include Ambas' dismissal.

On April 20, 1996, both parties again discussed the ground rules for the CBA renegotiation. However, petitioner
stopped the negotiations after it purportedly received information that a new group of employees had filed a
petition for certification election.

On June 18, 1996, the union finally struck.

SOLE/CA RULING:
The SOLE issued an order declaring petitioner guilty of unfair labor practice on two counts and directing the
reinstatement of private respondent Ambas with backwages. Petitioner filed a motion for reconsideration which
was denied.

The CA affirmed.

ISSUE/S:
• Whether petitioner is guilty of unfair labor practice by refusing to bargain with the union when it unilaterally
suspended the ongoing negotiations for a new Collective Bargaining Agreement (CBA) upon mere information
that a petition for certification has been filed by another legitimate labor organization?
• Whether the termination of the union president amounts to an interference of the employees' right to self-
organization?

HELD:
• Petitioner's utter lack of interest in bargaining with the union is obvious in its failure to make a timely reply to the
proposals presented by the latter. More than a month after the proposals were submitted by the union, petitioner
Angeles| Bajana | Balladares | Brillantes | Briones | Cabansag | Callanta | Chua | David|

De Leon | Gomez | Lopez | Macalino | Nostratis | Padilla | Reynon | Santos | Tan |Velasco
4E / 4F - 2018-2019
Page 454 of 920
LABOR REVIEW DIGEST
Atty. Joyrich Golangco

still had not made any counter-proposals. This inaction on the part of petitioner prompted the union to file its
second notice of strike on March 13, 1996. Petitioner could only offer a feeble explanation that the Board of
Trustees had not yet convened to discuss the matter as its excuse for failing to file its reply. This is a clear
violation of Article 250 of the Labor Code governing the procedure in collective bargaining.

• Moreover, the series of events that transpired after the filing of the first notice of strike in January 1996 show
petitioner's resort to delaying tactics to ensure that negotiation would not push through. Thus, on February 15,
1996, or barely a few days after the union proposals for the new CBA were submitted, the union president was
informed by her superior that her work schedule was being changed from Mondays to Fridays to Tuesdays to
Saturdays. A request from the union president that the issue be submitted to a grievance machinery was
subsequently denied. Thereafter, the petitioner and the union met on March 27, 1996 to discuss the ground rules
for negotiation. However, just two days later, or on March 29, 1996, petitioner dismissed the union president for
alleged insubordination. In its final attempt to thwart the bargaining process, petitioner suspended the negotiation
on the ground that it allegedly received information that a new group of employees called the Association of
Concerned Employees of Colegio (ACEC) had filed a petition for certification election. Clearly, petitioner tried to
evade its duty to bargain collectively.

• Petitioner, likewise, claims that the suspension of negotiation was proper since by the filing of the petition for
certification election the issue on majority representation of the employees has arose. According to petitioner, the
authority of the union to negotiate on behalf of the employees was challenged when a rival union filed a petition
for certification election. Citing the case of Lakas Ng Manggagawang Makabayan v. Marcelo
Enterprises,[7] petitioner asserts that in view of the pendency of the petition for certification election, it had no duty
to bargain collectively with the union.

We disagree. In order to allow the employer to validly suspend the bargaining process there must be a valid
petition for certification election raising a legitimate representation issue. Hence, the mere filing of a petition for
certification election does not ipso facto justify the suspension of negotiation by the employer. The petition must
first comply with the provisions of the Labor Code and its Implementing Rules. Foremost is that a petition for
certification election must be filed during the sixty-day freedom period. The "Contract Bar Rule" under Section 3,
Rule XI, Book V, of the Omnibus Rules Implementing the Labor Code, provides that: " . If a collective bargaining
agreement has been duly registered in accordance with Article 231 of the Code, a petition for certification election
or a motion for intervention can only be entertained within sixty (60) days prior to the expiry date of such
agreement." The rule is based on Article 232,[8] in relation to Articles 253, 253-A and 256 of the Labor Code. No
petition for certification election for any representation issue may be filed after the lapse of the sixty-day freedom
period. The old CBA is extended until a new one is signed. The rule is that despite the lapse of the formal
effectivity of the CBA the law still considers the same as continuing in force and effect until a new CBA shall have
been validly executed.[9] Hence, the contract bar rule still applies.[10] The purpose is to ensure stability in the
relationship of the workers and the company by preventing frequent modifications of any CBA earlier entered into
by them in good faith and for the stipulated original period.

In the case at bar, the lifetime of the previous CBA was from 1989-1994. The petition for certification election by
ACEC, allegedly a legitimate labor organization, was filed with the Department of Labor and Employment
(DOLE) only on May 26, 1996. Clearly, the petition was filed outside the sixty-day freedom
period. Hence, the filing thereof was barred by the existence of a valid and existing collective bargaining
agreement. Consequently, there is no legitimate representation issue and, as such, the filing of the
petition for certification election did not constitute a bar to the ongoing negotiation.

• Concerning the issue on the validity of the termination of the union president, we hold that the dismissal was
effected in violation of the employees' right to self-organization.

Angeles| Bajana | Balladares | Brillantes | Briones | Cabansag | Callanta | Chua | David|

De Leon | Gomez | Lopez | Macalino | Nostratis | Padilla | Reynon | Santos | Tan |Velasco
4E / 4F - 2018-2019
Page 455 of 920
LABOR REVIEW DIGEST
Atty. Joyrich Golangco

The factual backdrop of the termination of Ms. Ambas leads us to no other conclusion that she was dismissed in
order to strip the union of a leader who would fight for the right of her co-workers at the bargaining
table. Ms. Ambas, at the time of her dismissal, had been working for the petitioner for ten (10) years
already. In fact, she was a recipient of a loyalty award.Moreover, for the past ten (10) years her working
schedule was from Monday to Friday. However, things began to change when she was elected as union
president and when she started negotiating for a new CBA. Thus, it was when she was the union
president and during the period of tense and difficult negotiations when her work schedule was altered
from Mondays to Fridays to Tuesdays to Saturdays. When she did not budge, although her schedule was
changed, she was outrightly dismissed for alleged insubordination.

We quote with approval the following findings of the Secretary of Labor on this matter, to wit:

"Admittedly, management has the prerogative to discipline its employees for insubordination. But when the
exercise of such management right tends to interfere with the employees' right to self-organization, it
amounts to union-busting and is therefore a prohibited act. The dismissal of Ms. Ambas was clearly
designed to frustrate the Union in its desire to forge a new CBA with the College that is reflective of the
true wishes and aspirations of the Union members. Her dismissal was merely a subterfuge to get rid of
her, which smacks of a pre-conceived plan to oust her from the premises of the College. It has the effect
of busting the Union, stripping it of its strong-willed leadership. When management refused to treat the
charge of insubordination as a grievance within the scope of the Grievance Machinery, the action of the
College in finally dismissing her from the service became arbitrary, capricious and whimsical, and
therefore violated Ms. Ambas' right to due process."
————————————————————————

Angeles| Bajana | Balladares | Brillantes | Briones | Cabansag | Callanta | Chua | David|

De Leon | Gomez | Lopez | Macalino | Nostratis | Padilla | Reynon | Santos | Tan |Velasco
4E / 4F - 2018-2019
Page 456 of 920
LABOR REVIEW DIGEST
Atty. Joyrich Golangco

6. PAL vs. PALEA


G. R. No. 142399, March 12, 2008
CHICO-NAZARIO, J.:

DOCTRINE: The benefits of a CBA extend to the laborers and employees in the collective bargaining unit,
including those who do not belong to the chosen bargaining labor organization.

FACTS:
On 6 February 1987, petitioner PAL and respondent PALEA entered into a CBA[covering the period of 1986-1989.
Part of said agreement required petitioner PAL to pay its rank and file employees certain bonuses. On 22 April
1988, prior to the payment of the 13th month pay (mid-year bonus), petitioner PAL released a guideline
implementing Section 3 of the CBA. Respondent PALEA assailed the implementation of the foregoing guideline
on the ground that all employees of PAL, regular or non-regular, must be paid their 13th month pay. In response
thereto, petitioner PAL informed respondent PALEA that rank and file employees who were regularized after 30
April 1988 were not entitled to the 13th month pay as they were already given their Christmas bonuses on 9
December 1988 per the Implementing Rules of Presidential Decree No. 851.

Disagreeing with petitioner PAL, respondent PALEA filed a labor complaint for unfair labor practice against
petitioner PAL before the NLRC on 1 March 1989. The complaint interposed that the cut-off period for
regularization should not be used as the parameter for granting the 13th month pay considering that the law does
not distinguish the status of employment but the law covers all employees. Respondent PALEA further disputes
petitioner PALs allegations and maintains that the benefits to all employees in the collective bargaining unit,
including those who do not belong to the chosen bargaining labor organization, applies.

Labor Arbiter rendered a Decision dismissing the respondent PALEAs complaint for lack of merit. The Labor
Arbiter ruled that petitioner PAL was not guilty of unfair labor practice in withholding the grant of the 13th Month
Pay or Mid Year Bonus to the concerned employees. NLRC is convinced that the 13th month pay or mid-year
bonus is distinct from the Christmas Bonus, and although petitioner PAL already paid its employees the latter, it
must likewise pay them the former. CA affirmed the decision of the NLRC.

ISSUE:
Whether or not the CBA is applicable even to the non-members of the chosen bargaining labor organization.

RULING:
It is a well-settled doctrine that the benefits of a CBA extend to the laborers and employees in the collective
bargaining unit, including those who do not belong to the chosen bargaining labor organization. Otherwise, it
would be a clear case of discrimination. Hence, to be entitled to the benefits under the CBA, the employees must
be members of the bargaining unit, but not necessarily of the labor organization designated as the bargaining
agent.

At this point, the allegation of petitioner PAL that the non-regular employees do not belong to the collective
bargaining unit and are thus not covered by the CBA is unjustified and unsubstantiated. It is apparent to us that
petitioner PAL excludes certain employees from the benefits of the CBA only because they have not yet achieved
regular status by the cut-off date, 30 April 1988. There is no showing that the non-regular status of the concerned
employees by said cut-off date sufficiently distinguishes their interests from those of the regular employees so as
to exclude them from the collective bargaining unit and the benefits of the CBA.
————————————————————————

Angeles| Bajana | Balladares | Brillantes | Briones | Cabansag | Callanta | Chua | David|

De Leon | Gomez | Lopez | Macalino | Nostratis | Padilla | Reynon | Santos | Tan |Velasco
4E / 4F - 2018-2019
Page 457 of 920
LABOR REVIEW DIGEST
Atty. Joyrich Golangco

Article 265
1. FVC Labor Union-Phil Transport and General Workers Organization (FVCLU-PTGWO) v. Sama-Samang
Nagkakaisang Manggagawa sa FVC-Solidarity of Independent and General Labor Organizations
(SANAMA-FVC-SIGLO)
G.R. No. 176249 Date: November 27, 2009
Ponent : Brion, J.

Doctrine: While the parties may agree to extend the CBAs original five -year term together with all other
CBA provisions, any such amendment or term in excess of five years will not carry with it a change in
the union’s exclusive collective bargaining status. By express provision of the above-quoted Article 253-
A, the exclusive bargaining status cannot go beyond five years and the representation status is a legal
matter not for the workplace parties to agree upon.

FACTS:
On December 22, 1997, petitioner FVCLU-PTGWO, the recognized bargaining agent of the rank-and-file
employees of the FVC Philippines, Inc., signed a five-year collective bargaining agreement with the
company to cover the period from February 1, 1998 to January 30, 2003. At the end of the 3rd year of
the five-year term and pursuant to the CBA, FVCLU-PTGWO and the company entered into the
renegotiation of the CBA and modified, among other provisions, the CBAs duration. The renegotiated CB
provides that the renegotiation agreement shall take effect beginning February 1, 2001 and until May 31,
2003 thus extending the original five-year period of the CBA by four months. However, on January 21,
2003, nine days before the January 30, 2003 expiration of the original five-year CBA, the responded
SANAMA-FVC-SIGLO filed before the Department of Labor and Employment a petition for certification
election for the same rank-and-file unit covered by the FVCLU-PTGWO CBA. Hence, FVCLU-PTGWO
moved to dismiss the petition on the ground that the certification election petition was filed outside the
freedom period or outside of the sixty (60) days before the expiration of the CBA on May 31, 2003. The
Med-Arbiter dismissed the petition on the ground that it was filed outside the 60-day period counted
from the May 31, 2003 expiry date of the amended CBA. SANAMA-SIGLO appealed the Med-Arbiters
Order to the DOLE Secretary, contending that the filing of the petition on January 21, 2003 was within
60-days from the January 30, 2003 expiration of the original CBA term. The DOLE Secretary sustained
SANAMA-SIGLOs position, thereby setting aside the decision of the Med-Arbiter. However, the DOLE
Acting Secretary later reversed the decision upon FVCLU-PTGWO’s motion for reconsideration and ruled
that the amended CBA (which extended the representation aspect of the original CBA by four [4]
months) had been ratified by members of the bargaining unit some of whom later organized themselves
as SANAMA-SIGLO, the certification election applicant. Since these SANAMA-SIGLO members fully
accepted and in fact received the benefits arising from the amendments, the Acting Secretary
rationalized that they also accepted the extended term of the CBA and cannot now file a petition for
certification election based on the original CBA expiration date.

PETITION TO THE SC:

ISSUE/S:
Whether or not the certification election petition was filed within the freedom period of sixty (60) days
prior to the expiration of the CBA.

HELD:
Yes.
Article 253-A of the Labor Code covers this situation and it provides: Terms of a collective bargaining
agreement.

Angeles| Bajana | Balladares | Brillantes | Briones | Cabansag | Callanta | Chua | David|

De Leon | Gomez | Lopez | Macalino | Nostratis | Padilla | Reynon | Santos | Tan |Velasco
4E / 4F - 2018-2019
Page 458 of 920
LABOR REVIEW DIGEST
Atty. Joyrich Golangco

Any Collective Bargaining Agreement that the parties may enter into, shall, insofar as the representation
aspect is concerned, be for a term of five (5) years. No petition questioning the majority status of the
incumbent bargaining agent shall be entertained and no certification election shall be conducted by the
Department of Labor and Employment outside of the sixty day period immediately before the date of
expiry of such five-year term of the Collective Bargaining Agreement. All other provisions of the
Collective Bargaining Agreement shall be renegotiated not later than three (3) years after its execution.
This Labor Code provision is implemented through Book V, Rule VIII of the Rules Implementing the
Labor Code which states:
Sec. 14. Denial of the petition; grounds.
The Med-Arbiter may dismiss the petition on any of the following grounds: x x x x (b) the petition was
filed before or after the freedom period of a duly registered collective bargaining agreement; provided
that the sixty-day period based on the original collective bargaining agreement shall not be affected by
any amendment, extension or renewal of the collective bargaining agreement.

FVCLU-PTGWO has taken the view that its exclusive representation status should fully be in step with
the term of the CBA and that this status can be challenged only within 60 days before the expiration of
this term. Thus, when the term of the CBA was extended, its exclusive bargaining status was similarly
extended so that the freedom period for the filing of a petition for certification election should be
counted back from the expiration of the amended CBA term. We hold this FVCLU-PTGWO position to be
correct, but only with respect to the original five-year term of the CBA which, by law, is also the
effective period of the union’s exclusive bargaining representation status. While the parties may agree to
extend the CBAs original five-year term together with all other CBA provisions, any such amendment or
term in excess of five years will not carry with it a change in the union’s exclusive collective bargaining
status. By express provision of the above-quoted Article 253-A, the exclusive bargaining status cannot go
beyond five years and the representation status is a legal matter not for the workplace parties to agree
upon. In other words, despite an agreement for a CBA with a life of more than five years, either as an
original provision or by amendment, the bargaining unions exclusive bargaining status is effective only for
five years and can be challenged within sixty (60) days prior to the expiration of the CBAs first five
years. The negotiated extension of the CBA term has no legal effect on the FVCLU-PTGWOs exclusive
bargaining representation status which remained effective only for five years ending on the original expiry
date of January 30, 2003.
————————————————————————

Angeles| Bajana | Balladares | Brillantes | Briones | Cabansag | Callanta | Chua | David|

De Leon | Gomez | Lopez | Macalino | Nostratis | Padilla | Reynon | Santos | Tan |Velasco
4E / 4F - 2018-2019
Page 459 of 920
LABOR REVIEW DIGEST
Atty. Joyrich Golangco

2. SMCEU-PTGWO vs Hon. Confesor


G.R. No. 111262. September 19, 1996

Doctrine: Article 253-A states that the CBA has a term of five (5) years instead of three years, before the
amendment of the law as far as the representation aspect is concerned. All other provisions of the CBA shall be
negotiated not later than three (3) years after its execution. The "representation aspect" refers to the identity and
majority status of the union that negotiated the CBA as the exclusive bargaining representative of the appropriate
bargaining unit concerned. "All other provisions" simply refers to the rest of the CBA, economic as well as non-
economic provisions, except representation.

FACTS: Petitioner-union entered into a CBA with SMC to take effect upon the expiration of the previous CBA or
on June 30, 1989 until June 30, 1992. The CBA also stated that the term of the Agreement insofar as the
representation aspect is concerned, shall be for 5 years from July 1, 1989 to June 30, 1994. Hence, the freedom
period for purposes of such representation shall be sixty (60) days prior to June 30, 1994.
After June 30, 1992, the CBA was renegotiated in accordance with the terms of the CBA and Article 253-A of the
Labor Code. Negotiations started sometime in July, 1992 with the two parties submitting their respective
proposals and counterproposals. During the negotiations, the petitioner-union insisted that the bargaining unit of
SMC should still include the employees of the spun-off corporations: Magnolia and SMFI; and that the
renegotiated terms of the CBA shall be effective only for the remaining period of two years or until June 30, 1994.
SMC, on the other hand, contended that the members/employees who had moved to Magnolia and SMFI,
automatically ceased to be part of the bargaining unit at the SMC. Furthermore, the CBA should be effective for
three years in accordance with Art. 253-A of the Labor Code.

ISSUE/S:
1) WON the duration of the renegotiated terms of the CBA is to be effective for three years or for only two years;
and
2) WON the bargaining unit of SMC includes also the employees of the Magnolia and SMFI.

RULING:
1) The renegotiated terms of the CBA shall be for three (3) years.
Article 253-A states that the CBA has a term of five (5) years instead of three years, before the amendment of the
law as far as the representation aspect is concerned. All other provisions of the CBA shall be negotiated not later
than three (3) years after its execution. The "representation aspect" refers to the identity and majority status of the
union that negotiated the CBA as the exclusive bargaining representative of the appropriate bargaining unit
concerned. "All other provisions" simply refers to the rest of the CBA, economic as well as non- economic
provisions, except representation.
From the congressional discussions, the legislators were more inclined to have the period of effectivity for three
(3) years insofar as the economic as well as non-economic provisions are concerned, except representation.
Obviously, the framers of the law wanted to maintain industrial peace and stability by having both management
and labor work harmoniously together without any disturbance. Thus, no outside union can enter the
establishment within five (5) years and challenge the status of the incumbent union as the exclusive bargaining
agent. Likewise, the terms and conditions of employment (economic and non-economic) cannot be questioned by
the employers or employees during the period of effectivity of the CBA.
The CBA is a contract between the parties and the parties must respect the terms and conditions of the
agreement. Notably, the framers of the law did not give a fixed term as to the effectivity of the terms and
conditions of employment. It can be gleaned from their discussions that it was left to the parties to fix the period.
As a matter of policy the parties are encouraged to enter into a renegotiated CBA with a term which would
coincide with the aforesaid five (5) year term of the bargaining representative. In the event however, that the
parties, by mutual agreement, enter into a renegotiated contract with a term of three (3) years or one which does
not coincide with the said 5-year term, and said agreement is ratified by majority of the members in the bargaining
Angeles| Bajana | Balladares | Brillantes | Briones | Cabansag | Callanta | Chua | David|

De Leon | Gomez | Lopez | Macalino | Nostratis | Padilla | Reynon | Santos | Tan |Velasco
4E / 4F - 2018-2019
Page 460 of 920
LABOR REVIEW DIGEST
Atty. Joyrich Golangco

unit, the subject contract is valid and legal and therefore, binds the contracting parties. The same will however not
adversely affect the right of another union to challenge the majority status of the incumbent bargaining agent
within sixty (60) days before the lapse of the original five (5) year term of the CBA.

2) No. As a result of the spin-offs:


1. Each of the companies are run by, supervised and controlled by different management teams including
separate human resource/personnel managers.
2. Each Company enforces its own administrative and operational rules and policies and are not dependent on
each other in their operations.
3. Each entity maintains separate financial statements and are audited separately from each other.
Indubitably, therefore, Magnolia and SMFI became distinct entities with separate juridical personalities. Thus, they
can not belong to a single bargaining unit. There are various factors which must be satisfied and considered in
determining the proper constituency of a bargaining unit. (1) will of the employees (Globe Doctrine); (2) affinity
and unit of employees' interest, such as substantial similarity of work and duties, or similarity of compensation and
working conditions; (3) prior collective bargaining history; and (4) employment status, such as temporary,
seasonal and probationary employees. Even assuming in gratia argumenti that at the time of the election they
were regular employees of San Miguel, nonetheless, these workers are no By RICKY BOY CABATU/ 3B/ L-
100355 longer connected with San Miguel Corporation in any manner because Magnolia has ceased to be a
division of San Miguel Corporation and has been formed into a separate corporation with a personality of its own.
This development, which was brought to our attention by private respondents, necessarily renders moot and
academic any further discourse on the propriety of the elections which petitioners impugn via the recourse.
————————————————————————

Angeles| Bajana | Balladares | Brillantes | Briones | Cabansag | Callanta | Chua | David|

De Leon | Gomez | Lopez | Macalino | Nostratis | Padilla | Reynon | Santos | Tan |Velasco
4E / 4F - 2018-2019
Page 461 of 920
LABOR REVIEW DIGEST
Atty. Joyrich Golangco

3. HONGKONG BANK INDEPENDENT LABOR UNION vs. HSBC


G.R. No. 218390 February 28, 2018
Ponente: Velasco, Jr. J.

Doctrine: Although jurisprudence recognizes the validity of the exercise by an employer of its management
prerogative and will ordinarily not interfere with such, this prerogative is not absolute and is subject to limitations
imposed by law, collective bargaining agreement, and general principles of fair play and justice. A collective
bargaining agreement or CBA is the negotiated contract between a legitimate labor organization and the employer
concerning wages, hours of work and all other terms and conditions of employment in a bargaining unit. As in all
contracts, the parties in a CBA may establish such stipulations, clauses, terms and conditions as they may deem
convenient provided these are not contrary to law, morals, good customs, public order or public policy. Thus,
where the CBA is clear and unambiguous, it becomes the law between the parties and compliance therewith is
mandated by the express policy of the law.

FACTS:
In 2001, the Bangko Sentral ng Pilipinas (BSP) issued the Manual of Regulations for Banks
(MoRB). ||| (Hongkong Bank Independent Labor Union v. Hongkong and Shanghai Banking Corp. Limited, G.R.
No. 218390, [February 28, 2018]). Section X338 thereof which reads:
Banks may provide financial assistance to their officers and employees, as part of their fringe benefits program, to
meet housing, transportation, household and personal needs of their officers and employees. Financing plans
and amendments thereto shall be with prior approval of the BSP.
Pursuant to the above-cited provision, respondent Hongkong and Shanghai Banking Corporation Limited (HSBC),
on March 12, 2003, submitted its Financial Assistance Plan (Plan) to the BSP for approval. The Plan allegedly
contained a credit checking proviso stating that "[r]epayment defaults on existing loans and adverse information
on outside loans will be considered in the evaluation of loan applications." The BSP approved the Plan on May 5,
2003. Said Plan was later amended thrice, all of which amendments were approved by the BSP.|
Hongkong Bank Independent Labor Union (HBILU), the incumbent bargaining agent of HSBC's rank-and-file
employees, entered into a CBA with the bank covering the period from April 1, 2010 to March 31, 2012 regarding
salary loans. When the CBA was about to expire, the parties started negotiations for a new one to cover the
period from April 1, 2012 to March 31, 2017. During the said negotiations, HSBC proposed amendments to the
above-quoted Article XI allegedly to align the wordings of the CBA with its BSP-approved Plan.
Particularly, HSBC proposed the deletion of Article XI, Section 4 (Credit Ratio) of the CBA, and the amendment of
Sections 1 to 3 of the same. HBILU vigorously objected to the proposed amendments, claiming that their
insertions would curtail its members' availment of salary loans. This, according to the Union, violates the existing
exceptions set forth in BSP Circular 423, Series of 2004, and Section X338.3 of the MoRB. In view of HBILU's
objection, HSBC withdrew its proposed amendments and, consequently, Article XI remained unchanged.
Despite the withdrawal of the proposal, HSBC sent an e-mail to its employees on April 20, 2012 concerning the
enforcement of the Plan. HBILU member Vince Mananghaya (Mananghaya) applied for a loan under the
provisions of Article XI of the CBA. His first loan application in March 2012 was approved, but adverse findings
from the external checks on his credit background resulted in the denial of his September application. HBILU then
raised the denial as a grievance issue with the National Conciliation Mediation Board (NCMB).

Petitioner’s Contention: Petitioners argued that the imposition of an additional requirement — the external credit
checking prior to approval of any loan application under Article XI of the CBA — is not sanctioned under the CBA.
The Union emphasized that under the terms of Article XI, there is no such requirement and that it cannot,
therefore, be unilaterally imposed by HSBC.

Respondent’s Contention: HSBC countered that the external credit check conducted in line with Mananghaya's
loan application was merely an implementation of the BSP-approved Plan. The adoption of the
Plan, HSBC stressed, is a condition sine qua non for any loan grant under Section X338 of the MoRB. Moreover,
Angeles| Bajana | Balladares | Brillantes | Briones | Cabansag | Callanta | Chua | David|

De Leon | Gomez | Lopez | Macalino | Nostratis | Padilla | Reynon | Santos | Tan |Velasco
4E / 4F - 2018-2019
Page 462 of 920
LABOR REVIEW DIGEST
Atty. Joyrich Golangco

the Credit Check policy has been in place since 2003, and is a sound practice in the banking industry to protect
the interests of the public and preserve confidence in banks.
The issue was then submitted for resolution by the NCMB Panel of Accredited Voluntary Arbitrators (the Panel).|||

NCMB Ruling: The NCMB ruled in favor of HSBC. It held that respondent, as an employer, has the right to issue
and implement guidelines for the availment of loan accommodations under the CBA as part of its management
prerogative. The repeated use of the term "qualified employees" in Article XI of the CBA was deemed indicative of
room for the adoption of further guidelines in the availment of the benefits thereunder. The Panel also agreed
that HSBC's Plan is not a new policy as it has already been approved by the BSP as early as 2003. Thus, the
Panel ruled that the salary loan provisions under Article XI of the CBA must be read in conjunction with the
provisions of the Plan.

CA Ruling: The CA sustained the findings and conclusions of the NCMB-PVA in toto on the ratiocination
that HSBC was merely complying with Section X338 of the MoRB when it submitted the Plan to BSP. When BSP,
in turn, approved the said Plan, HSBC became legally bound to enforce its provisions, including the conduct of
external credit checks on its loan applicants. The appellate court further ruled that the Plan should be deemed
incorporated in the CBA because it is a regulatory requirement of BSP without which the salary loan provisions of
the CBA are rendered inoperative.

ISSUE: Whether or not the HSBC could validly enforce the credit-checking requirement under its BSP-
approved Plan in processing the salary loan applications of covered employees even when the said
requirement is not recognized under the CBA.||

HELD:
NO. Although jurisprudence recognizes the validity of the exercise by an employer of its management prerogative
and will ordinarily not interfere with such, this prerogative is not absolute and is subject to limitations imposed by
law, collective bargaining agreement, and general principles of fair play and justice.
A collective bargaining agreement or CBA is the negotiated contract between a legitimate labor organization and
the employer concerning wages, hours of work and all other terms and conditions of employment in a bargaining
unit. As in all contracts, the parties in a CBA may establish such stipulations, clauses, terms and conditions as
they may deem convenient provided these are not contrary to law, morals, good customs, public order or public
policy. Thus, where the CBA is clear and unambiguous, it becomes the law between the parties and compliance
therewith is mandated by the express policy of the law. It is clear from the arguments and evidence submitted that
the Plan was never made part of the CBA. As a matter of fact, HBILU vehemently rejected the Plan's incorporation
into the agreement. Due to this lack of consensus, the bank withdrew its proposal and agreed to the retention of
the original provisions of the CBA. The subsequent implementation of the Plan's external credit check provisions
in relation to employee loan applications under Article XI of the CBA was then an imposition solely by HSBC.
In this respect, this Court is of the view that tolerating HSBC's conduct would be tantamount to allowing a blatant
circumvention of Article 253 of the Labor Code. It would contravene the express prohibition against the unilateral
modification of a CBA during its subsistence and even thereafter until a new agreement is reached. It would
unduly license HSBC to add, modify, and ultimately further restrict the grant of Salary Loans beyond the
terms of the CBA by simply adding stringent requirements in its Plan, and having the said Plan approved
by BSP in the guise of compliance with the MoRB.
|||
————————————————————————

Angeles| Bajana | Balladares | Brillantes | Briones | Cabansag | Callanta | Chua | David|

De Leon | Gomez | Lopez | Macalino | Nostratis | Padilla | Reynon | Santos | Tan |Velasco
4E / 4F - 2018-2019
Page 463 of 920
LABOR REVIEW DIGEST
Atty. Joyrich Golangco

Article 267
1. INTERNATIONAL SCHOOL ALLIANCE OF EDUCATORS (ISAE) v. QUISUMBING, as SOLE,
INTERNATIONAL SCHOOL INC.
G.R. No. 128845. June 1, 2000

Doctrine:

Facts:
- Accordingly, the Private respondent School hires both foreign and local teachers as members of its faculty,
classifying the same into two: (1) foreign-hires and (2) local-hires.
- The School grants foreign-hires certain benefits not accorded local-hires.
- These include housing, transportation, shipping costs, taxes, and home leave travel allowance. Foreign-hires are
also paid a salary rate twenty-five percent (25%) more than local-hires.
- The School justifies the difference on two "significant economic disadvantages" foreign-hires have to endure,
namely: (a) the "dislocation factor" and (b) limited tenure.
- When negotiations for a new collective bargaining agreement were held on June 1995, petitioner International
School Alliance of Educators, "a legitimate labor union and the collective bargaining representative of all faculty
members" of the School, contested the difference in salary rates between foreign and local-hires.
- This issue, as well as the question of whether foreign-hires should be included in the appropriate bargaining unit,
eventually caused a deadlock between the parties.
- On September 7, 1995, petitioner filed a notice of strike.
- The failure of the National Conciliation and Mediation Board to bring the parties to a compromise prompted the
DOLE to assume jurisdiction over the dispute.
- DOLE Acting Secretary DECISION:
o an Order resolving the parity and representation issues in favor of the School.
- SOLE DECISION:
o DOLE Secretary Leonardo A. Quisumbing subsequently denied petitioner's motion for reconsideration in an
Order.

ISSUE:
- Whether the point-of-hire classification for the distinction in salary rates should be upheld.

HELD:
• SUPREME COURT says NO.
• In Re: Article 267 (formerly Article 255) NO DECISION in connection with this Article.
• The Court recognizes in this jurisdiction the long honored legal truism of "equal pay for equal work." Persons
who work with substantially equal qualifications, skill, effort and responsibility, under similar conditions, should be
paid similar salaries. This rule applies to the School, its "international character" notwithstanding

ISSUE:
- whether foreign-hires should be included in the appropriate bargaining unit

HELD:
• In Re: Article 267 (formerly Article 255) NO DECISION in connection with this Article Medyo related.
• We agree, however, that foreign-hires do not belong to the same bargaining unit as the local-hires.
• A bargaining unit is "a group of employees of a given employer, comprised of all or less than all of the entire
body of employees, consistent with equity to the employer indicate to be the best suited to serve the reciprocal
rights and duties of the parties under the collective bargaining provisions of the law."
• The factors in determining the appropriate collective bargaining unit are
o (1) the will of the employees (Globe Doctrine);
Angeles| Bajana | Balladares | Brillantes | Briones | Cabansag | Callanta | Chua | David|

De Leon | Gomez | Lopez | Macalino | Nostratis | Padilla | Reynon | Santos | Tan |Velasco
4E / 4F - 2018-2019
Page 464 of 920
LABOR REVIEW DIGEST
Atty. Joyrich Golangco

o (2) affinity and unity of the employees' interest, such as substantial similarity of work and duties, or similarity of
compensation and working conditions (Substantial Mutual Interests Rule);
o (3) prior collective bargaining history; and
o (4) similarity of employment status.
• The basic test of an asserted bargaining unit's acceptability is whether or not it is fundamentally the combination
which will best assure to all employees the exercise of their collective bargaining rights.
• It does not appear that foreign-hires have indicated their intention to be grouped together with local-
hires for purposes of collective bargaining.
• The collective bargaining history in the School also shows that these groups were always treated separately.
• Foreign-hires have limited tenure; local-hires enjoy security of tenure.
• Although foreign-hires perform similar functions under the same working conditions as the local-hires, foreign-
hires are accorded certain benefits not granted to local-hires. These benefits, such as housing, transportation,
shipping costs, taxes, and home leave travel allowance, are reasonably related to their status as foreign-hires,
and justify the exclusion of the former from the latter.
• To include foreign-hires in a bargaining unit with local-hires would not assure either group the exercise
of their respective collective bargaining rights.
• petition is GIVEN DUE COURSE. The petition is hereby GRANTED IN PART.
————————————————————————

Angeles| Bajana | Balladares | Brillantes | Briones | Cabansag | Callanta | Chua | David|

De Leon | Gomez | Lopez | Macalino | Nostratis | Padilla | Reynon | Santos | Tan |Velasco
4E / 4F - 2018-2019
Page 465 of 920
LABOR REVIEW DIGEST
Atty. Joyrich Golangco

2. National Association of Free Trade Unions (NAFTU) v. Mainit Lumber Development Company Worker’s
Union- United Lumber General Workers of the Philippines (MALDECOWU-ULGWP)
G.R. No. 79526; Dec. 21, 1990
Paras, J.:

DOCTRINE: The basic test of an asserted bargaining unit's acceptability is whether or not it is fundamentally the
combination which will best assure to all employees the exercise of their collective bargaining rights. Otherwise
stated: the test of grouping is community or mutuality of interests.

FACTS:
Mainit Lumber Development Company Worker’s Union- United Lumber General Workers of the Philippines
(MALDECOWU-ULGWP- or simply THE UNION) filed with the Regional Office of the Ministry of Labor and
Employment in Cagayan De Oro City a Petition for Certification Election to determine the sole and exclusive
bargaining representative among the rank and file workers of Mainit Lumber Development Company Inc.
(MALDECO), a duly organized, registered and existing corporation engaged in the business of logging and saw-
mill operations employing approximately 136 rank and file employees/workers. During the hearing on the petition,
tge National Association of Free Trade Unions (NAFTU) intervened to request a postponement. During the
scheduled hearing the parties submitted their respective position papers were filed.

The Med-Arbiter granted the request but NAFTU appealed on the ground that Mainit Lumber was composed of
two bargaining units- the Sawmills Division and the Logging Division. The Bureau of Labor Relations affirmed the
decision and a SINGLE CERTIFICATION ELECTION was held for the two divisions on separate dates, in their
respective working premises. The Union garnered 146 votes, while NAFTU received 2 votes.

NAFTU filed an election protest and alleging massive vote buying as well as threats of force against the lives of 25
applicants. MALDECO filed a Manifestation supporting NAFTU’s position.

Med-Arbiter & BLR: The Med-Arbiter dismissed the election protest, and the said decision was appealed to the
BLR in Manila which also denied the appeal. Hence this Petition.

APPEAL TO THE SC:

Petitioner’s Contention: That the certification election was improperly held for reason that only one certification
election was held for different bargaining units.

ISSUE: Whether the Certification Election was validly held.

RULING: NO. While the existence of a bargaining history is a factor that may be reckoned with in determining the
appropriate bargaining unit, the same is not decisive or conclusive. Other factors must be considered. The test of
grouping is community or mutuality of interests. This is so because "the basic test of an asserted bargaining unit's
acceptability is whether or not it is fundamentally the combination which will best assure to all employees the
exercise of their collective bargaining rights."

Thus, Petitioner’s assertion that there must be two bargaining units for the reason that the employer MALDECO
was composed of two bargaining units, the Sawmill Division in Butuan City and the Logging Division, in Zapanta
Valley, Kitcharao, Agusan Norte, about 80 kilometers distant from each other and in fact, had then two separate
CBA's, does not bind considering that the election showed different results.

Angeles| Bajana | Balladares | Brillantes | Briones | Cabansag | Callanta | Chua | David|

De Leon | Gomez | Lopez | Macalino | Nostratis | Padilla | Reynon | Santos | Tan |Velasco
4E / 4F - 2018-2019
Page 466 of 920
LABOR REVIEW DIGEST
Atty. Joyrich Golangco

Significantly, out of two hundred and one (201) employees of MALDECO, one hundred seventy five (175)
consented and supported the petition for certification election, thereby confirming their desire for one bargaining
representative.
————————————————————————

Angeles| Bajana | Balladares | Brillantes | Briones | Cabansag | Callanta | Chua | David|

De Leon | Gomez | Lopez | Macalino | Nostratis | Padilla | Reynon | Santos | Tan |Velasco
4E / 4F - 2018-2019
Page 467 of 920
LABOR REVIEW DIGEST
Atty. Joyrich Golangco

Article 268
1. PICOP RESOURCES, INCORPORATED (PRI), Represented in this Petition by MR. WILFREDO D.
FUENTES, in his capacity as Senior Vice-President and Resident Manager, petitioner, vs. RICARDO
DEQUILLA, ELMO PABILANDO, CESAR ATIENZA and ANICETO ORBETA, JR., and NAMAPRI-SPFI,
respondents.
GR No: 172666 Date: December 7, 2011
Ponente: Mendoza, J.

Doctrine: An existing CBA cannot constitute a bar to a filing of a petition for certification election. When
there is a representational issue, the status quo provision in so far as the need to await the creation of a
new agreement will not apply.

FACTS:
Ricardo Dequilla, Cesar Atienza and Aniceto Orbeta (private respondents) were regular rank-and-Cle employees
of Picop Resources, Inc. (PICOP) and members of the NAMAPRI-SPFL, a duly registered labor organization and
existing bargaining agent of the PICOP rank-and-Cle employees. PICOP and NAMAPRI-SPFL had a collective
bargaining agreement (CBA) which would expire on May 22, 2000.
PICOP was advised by the late Atty. Proculo P. Fuentes, then National Pres. of SPFL to terminate employees of
PICOP for committing acts of disloyalty, specifically campaigning, supporting, and signing a petition for
certification of a rival union, Federation of Free Workers Union (FFW) before the 60-days “freedom period” and
during the effectivity of the CBA. The employees were ordered through a memorandum to submit an explanation
letter within 72 hours as to why their employment not be terminated due to the alleged acts of disloyalty. On
October 16, 2000, PICOP served a notice of termination due to acts of disloyalty to 31 of the 46 employees
requested by Atty. Fuentes to be terminated. Private respondents were among the 31 employees dismissed from
employment by PICOP on November 16, 2000.
Private respondents filed a complaint before the NLRC Regional Arbitration Branch No. XIII, Butuan City, for
Unfair Labor Practice and Illegal Dismissal with money claims, damages and attorney's fees.

LA RULING:
On June 9, 2001 declared that respondents were illegally dismissed

NLRC RULING:
Dismissed the appeal filed by PICOS but on the latter’s motion for reconsideration it reversed and set aside its
November 19, 2002 Resolution and dismissed the case based on lack of merit.

CA RULING:
CA reversed the decision of the NLRC and reinstated the June 9, 2001 Decision of the LA stating that the act of
private respondents in signing an authorization for the filing of petition for certification election of FFW was not a
sufficient ground for termination of employment.

APPEAL TO THE SC:

Petitioner's Contention: PICOP basically argues that Article 253 of the Labor Code applies in this case. Article
253 of the Labor Code provides that the terms and conditions of a CBA remain in full force and effect even
beyond the 5-year period when no new CBA has yet been reached. It further argues that private respondents
were not denied due process when they were terminated. Finally, it claims that the decision of the NLRC on the
issues raised was not without merit.

Respondent's Contention: Private respondents adopted and repleaded the ruling of the CA in their Comment on
this petition.
Angeles| Bajana | Balladares | Brillantes | Briones | Cabansag | Callanta | Chua | David|

De Leon | Gomez | Lopez | Macalino | Nostratis | Padilla | Reynon | Santos | Tan |Velasco
4E / 4F - 2018-2019
Page 468 of 920
LABOR REVIEW DIGEST
Atty. Joyrich Golangco

ISSUE/S:
1. Whether or not respondents were illegally dismissed
2. Whether or not Art. 253 (now 265) or Art. 256 (now 268) should apply in this case

HELD:
1. YES
Their mere act of signing an authorization for a petition for certification election before the freedom period does
not necessarily demonstrate union disloyalty. It is far from being within the definition of "acts of disloyalty" as
PICOP would want the Court to believe. The act of "signing an authorization for a petition for certification election"
is not disloyalty to the union per se considering that the petition for certification election itself was filed during the
freedom period which started on March 22, 2000.
Moreover, as correctly ruled by the CA, the records are bereft of proof of any contemporaneous acts of
resignation or withdrawal of union membership or nonpayment of union dues. Neither is there proof that private
respondents joined FFW. The fact is, private respondents remained in good standing with their union,
NAMAPRISPFL.

2. Art. 256 (now 268) should apply


At the expiration of the freedom period, the employer shall continue to recognize the majority status of
the incumbent bargaining agent where no petition for certification election is filed.
Applying the same provision, it can be said that while it is incumbent for the employer to continue to recognize the
majority status of the incumbent bargaining agent even after the expiration of the freedom period, they could
only do so when no petition for certification election was filed. The reason is, with a pending petition for
certification, any such agreement entered into by management with a labor organization is fraught with
the risk that such a labor union may not be chosen thereafter as the collective bargaining representative.
The provision for status quo is conditioned on the fact that no certification election was filed during the
freedom period. Any other view would render nugatory the clear statutory policy to favor certification election as
the means of ascertaining the true expression of the will of the workers as to which labor organization would
represent them.
Moreover, the last sentence of Article 253 which provides for automatic renewal pertains only to the economic
provisions of the CBA, and does not include representational aspect of the CBA. An existing CBA cannot
constitute a bar to a filing of a petition for certification election. When there is a representational issue,
the status quo provision in so far as the need to await the creation of a new agreement will not apply.
Otherwise, it will create an absurd situation where the union members will be forced to maintain membership by
virtue of the union security clause existing under the CBA and, thereafter, support another union when filing a
petition for certification election. If we apply it, there will always be an issue of disloyalty whenever the employees
exercise their right to self-organization. The holding of a certification election is a statutory policy that should not
be circumvented, or compromised.

Angeles| Bajana | Balladares | Brillantes | Briones | Cabansag | Callanta | Chua | David|

De Leon | Gomez | Lopez | Macalino | Nostratis | Padilla | Reynon | Santos | Tan |Velasco
4E / 4F - 2018-2019
Page 469 of 920
LABOR REVIEW DIGEST
Atty. Joyrich Golangco

2. NATIONAL UNION OF WORKERS IN HOTELS, RESTAURANTS AND ALLIED INDUSTRIES- MANILA


PAVILION HOTEL CHAPTER v. SEC. OF LABOR AND EMPLOYMENT
GR No: 181531 Date: July 31, 2009
Ponente: Carpio-Morales, J.

Doctrines:
In a certification election, all rank and file employees in the appropriate bargaining unit, whether probationary or
permanent are entitled to vote.

The period of reckoning in determining who shall be included in the list of eligible voters is, in cases where a
timely appeal has been filed from the Order of the Med-Arbiter, the date
when the Order of the Secretary of Labor and Employment, whether affirming or denying the appeal,
becomes final and executory.

The conduct of a certification election has a two-fold objective: to determine the appropriate bargaining unit and to
ascertain the majority representation of the bargaining representative, if the employees desire to be represented
at all by anyone.

FACTS:
A certification election was conducted among the rank-and-file employees of respondent Holiday Inn Manila
Pavilion Hotel. In view of the significant number of segregated votes (22 out of 346 total votes cast), contending
unions, petitioner NUHWHRAIN-MPHC and respondent Holiday Inn Manila Pavilion Hotel Labor Union
(HIMPHLU) referred the case back to the Med-Arbiter, which ruled for the opening of 17 out of the 22 segregated
votes, specially those cast by the 11 dismissed employees and those cast by the six supposedly supervisory
employees of the Hotel.

Petitioner, which garnered 151 votes, appealed to the Secretary of Labor and Employment (SOLE), arguing that
the votes of the probationary employees should have been opened considering that probationary employee
Gatbonton’s vote was tallied. And petitioner averred that respondent HIMPHLU, which garnered 169 votes, should
not be immediately certified as the bargaining agent, as the opening of the 17 segregated ballots would push the
number of valid votes cast to 338 (151 + 169 + 1 + 17), hence, the 169 votes which HIMPHLU garnered would be
one vote short of the majority which would then become 169.

SOLE RULING:
The SOLE affirmed the Med-Arbiter’s order. It held that pursuant to Section 5, Rule IX of the Omnibus Rules
Implementing the Labor Code on exclusion and inclusion of voters in a certification election, the probationary
employees cannot vote, as at the time the Med-Arbiter issued the Order granting the petition for the conduct of the
certification election, the six probationary employees were not yet hired, hence, they could not vote.

With respect to the votes cast by the 11 dismissed employees, they could be considered since their dismissal was
still pending appeal. As to the votes cast by the six alleged supervisory employees, their votes should be counted
since their promotion took effect months after the issuance of the Order of the Med-Arbiter, hence, they were still
considered as rank-and-file.

Respecting Gatbonton’s vote, the same could be the basis to include the votes of the other probationary
employees, as the records show that during the pre-election conferences, there was no disagreement as to his
inclusion in the voters list, and neither was it timely challenged when he voted on election day, hence, the Election
Officer could not then segregate his vote.

Angeles| Bajana | Balladares | Brillantes | Briones | Cabansag | Callanta | Chua | David|

De Leon | Gomez | Lopez | Macalino | Nostratis | Padilla | Reynon | Santos | Tan |Velasco
4E / 4F - 2018-2019
Page 470 of 920
LABOR REVIEW DIGEST
Atty. Joyrich Golangco

Even if the 17 votes of the dismissed and supervisory employees were to be counted and presumed to be in favor
of petitioner, still, the same would not suffice to overturn the 169 votes garnered by HIMPHLU.

CA RULING:
The CA affirmed the ruling of the SOLE.

ISSUE/S:
1. Whether employees on probationary status at the time of the certification elections should be allowed to vote.
2. Whether HIMPHLU was able to obtain the required majority for it to be certified as the exclusive bargaining
agent.

HELD:
1. YES. The inclusion of Gatbonton’s vote was proper not because it was not questioned but because
probationary employees have the right to vote in a certification election. The votes of the six other probationary
employees should thus also have been counted.

As Airtime Specialists, Inc. v. Ferrer-Calleja holds:

In a certification election, all rank and file employees in the appropriate bargaining unit, whether
probationary or permanent are entitled to vote. This principle is clearly stated in Art. 255 of the Labor Code
which states that the labor organization designated or selected by the majority of the employees in an appropriate
bargaining unit shall be the exclusive representative of the employees in such unit for purposes of collective
bargaining. Collective bargaining covers all aspects of the employment relation and the resultant CBA negotiated
by the certified union binds all employees in the bargaining unit. Hence, all rank and file employees, probationary
or permanent, have a substantial interest in the selection of the bargaining representative. The Code makes no
distinction as to their employment status as basis for eligibility in supporting the petition for certification
election. The law refers to all the employees in the bargaining unit. All they need to be eligible to support
the petition is to belong to the bargaining unit.

Prescinding from the principle that all employees are, from the first day of their employment, eligible for
membership in a labor organization, it is evident that the period of reckoning in determining who shall be included
in the list of eligible voters is, in cases where a timely appeal has been filed from the Order of the Med-Arbiter, the
date when the Order of the Secretary of Labor and Employment, whether affirming or denying the appeal,
becomes final and executory.

The filing of an appeal to the SOLE from the Med-Arbiters Order stays its execution, in accordance with Sec. 21,
and rationally, the Med-Arbiter cannot direct the employer to furnish him/her with the list of eligible voters pending
the resolution of the appeal.

During the pendency of the appeal, the employer may hire additional employees. To exclude the employees hired
after the issuance of the Med-Arbiters Order but before the appeal has been resolved would violate the guarantee
that every employee has the right to be part of a labor organization from the first day of their service.

In the present case, records show that the probationary employees, including Gatbonton, were included in the list
of employees in the bargaining unit submitted by the Hotel on May 25, 2006 in compliance with the directive of the
Med-Arbiter after the appeal and subsequent motion for reconsideration have been denied by the SOLE,
rendering the Med-Arbiter’s August 22, 2005 Order final and executory 10 days after the March 22, 2007
Resolution (denying the motion for reconsideration of the January 22 Order denying the appeal), and rightly
so. Because, for purposes of self-organization, those employees are deemed eligible to vote.

Angeles| Bajana | Balladares | Brillantes | Briones | Cabansag | Callanta | Chua | David|

De Leon | Gomez | Lopez | Macalino | Nostratis | Padilla | Reynon | Santos | Tan |Velasco
4E / 4F - 2018-2019
Page 471 of 920
LABOR REVIEW DIGEST
Atty. Joyrich Golangco

2. NO. HIMPHLU should not be certified as the exclusive bargaining agent. Under the double-majority rule, for
there to be a valid certification election, majority of the bargaining unit must have voted, AND the winning union
must have garnered majority of the valid votes cast.

Prescinding from the Courts ruling that all the probationary employees votes should be deemed valid votes while
that of the supervisory employees should be excluded, it follows that the number of valid votes cast would
increase from 321 to 337. Under Art. 256 of the Labor Code, the union obtaining the majority of the valid votes
cast by the eligible voters shall be certified as the sole and exclusive bargaining agent of all the workers in the
appropriate bargaining unit. This majority is 50% + 1. Hence, 50% of 337 is 168.5 + 1 or at least 170.

HIMPHLU obtained 169 while petitioner received 151 votes. Clearly, HIMPHLU was not able to obtain a majority
vote.

The true importance of ascertaining the number of valid votes cast is for it to serve as basis for computing the
required majority, and not just to determine which union won the elections. The opening of the segregated but
valid votes has thus become material. To be sure, the conduct of a certification election has a two-fold
objective: to determine the appropriate bargaining unit and to ascertain the majority representation of the
bargaining representative, if the employees desire to be represented at all by anyone.

Having declared that no choice in the certification election conducted obtained the required majority, it follows that
a run-off election must be held to determine which between HIMPHLU and petitioner should represent the rank-
and-file employees.
————————————————————————

Angeles| Bajana | Balladares | Brillantes | Briones | Cabansag | Callanta | Chua | David|

De Leon | Gomez | Lopez | Macalino | Nostratis | Padilla | Reynon | Santos | Tan |Velasco
4E / 4F - 2018-2019
Page 472 of 920
LABOR REVIEW DIGEST
Atty. Joyrich Golangco

3. Coca-Cola Bottlers Philippines, Inc. (CCBPI) v. Ilocos Professional And Technical Employees Union
(IPTEU)
GR No: 193798 Date: September 09, 2015
Ponente: PERALTA, J.

Doctrine:
Access to vital labor information is the imperative consideration of a confidential employee. An employee must
assist or act in a confidential capacity and obtain confidential information relating to labor relations policies.
Exposure to internal business operations of the company is not per se a ground for the exclusion in the bargaining
unit.

FACTS:
IPTEU filed a verified Petition for certification election seeking to represent a bargaining unit consisting of
approximately twenty-two (22) rank-and-file professional and technical employees of CCBPI Ilocos Norte Plant.
CCBPI prayed for the denial and dismissal of the petition, arguing that it composed of supervisory employees and
confidential employees; hence, ineligible for inclusion as members of IPTEU. It also sought to cancel and revoke
the registration of IPTEU for failure to comply with the twenty percent (20%) membership requirement based on all
the supposed employees in the bargaining unit it seeks to operate.

Med-Arbiter's Ruling:
Med-Arbiter granted IPTEU'S petition on the ground that the union members are rank-and-file employees and not
occupying positions that are supervisory or confidential in nature.

On election day, only sixteen (16) of the twenty-two (22) employees in the IPTEU list voted. However, no votes
were canvassed. CCBPI filed and registered a Protest questioning the conduct and mechanics of the election and
a Challenge to Votes on the ground that the voters are supervisory and confidential employees.

Med-Arbiter in resolving the protest found that the voters are rank-and-file employees holding positions that are
not confidential in nature, and who are not, or used to be, members of Ilocos Monthlies Union (IMU) due to the
reclassification of their positions by CCBPI and have been excluded from the CBA entered into by IMU and
CCBPI from 1997 to 2005. Consequently, the challenged votes were opened and canvassed. After garnering 14
out of the 16 votes cast, IPTEU was proclaimed as the sole and exclusive bargaining agent of the rank-and-file
exempt workers in CCBPI Ilocos Norte Plant.

Department of Labor and Employment's Ruling:


The SOLE held that, as shown by the certification of the IMU President and the CBAs forged between CCBPI and
IMU from 1997 to 2007, the 22 employees sought to be represented by IPTEU are not part of IMU and are
excluded from its CBA coverage; that even if the 16 challenged voters may have access to information which are
confidential from the business standpoint, the exercise of their right to self-organization could not be defeated
because their common functions do not show that there exist a confidential relationship within the realm of labor
relations.

CA RULING:
CA denied the petition.

ISSUE/S:
1. Whether or not the IPTEU is the sole and exclusive bargaining agent of the purported rank-and-file exempt
employees in the Ilocos Plant
2. Whether the 16 voters are confidential employees

Angeles| Bajana | Balladares | Brillantes | Briones | Cabansag | Callanta | Chua | David|

De Leon | Gomez | Lopez | Macalino | Nostratis | Padilla | Reynon | Santos | Tan |Velasco
4E / 4F - 2018-2019
Page 473 of 920
LABOR REVIEW DIGEST
Atty. Joyrich Golangco

HELD:
1. YES. Under 268 of the Labor Code, the labor union receiving the majority of the valid votes cast shall be the
exclusive bargaining agent of all the workers in the unit. In this case, the union garnered 14 out of the 16 votes
cast. Therefore, IPTEU is the sole and exclusive bargaining agent of the 22 rank-and-file employees.

2. NO. Employees who encounter or handle trade secrets and financial information are not automatically classified
as confidential employees. It was admitted that the subject employees encounter and handle financial as well as
physical production data and other information which are considered vital and important from the business
operations' standpoint. Nevertheless, it was opined that such information is not the kind of information that is
relevant to collective bargaining negotiations and settlement of grievances as would classify them as confidential
employees. Access to vital labor information is the imperative consideration. An employee must assist or act in a
confidential capacity and obtain confidential information relating to labor relations policies. Exposure to internal
business operations of the company is not per se a ground for the exclusion in the bargaining unit.
————————————————————————

Angeles| Bajana | Balladares | Brillantes | Briones | Cabansag | Callanta | Chua | David|

De Leon | Gomez | Lopez | Macalino | Nostratis | Padilla | Reynon | Santos | Tan |Velasco
4E / 4F - 2018-2019
Page 474 of 920
LABOR REVIEW DIGEST
Atty. Joyrich Golangco

Article 273-274
1. MIGUELA SANTUYO, et al. vs. REMERCO GARMENTS MANUFACTURING, INC. and/or VICTORIA
REYES
G.R. No. 174420 Date: March 22, 2010
Ponente: CORONA, J.

DOCTRINE:
Pursuant to Articles 217 in relation to Articles 260 and 261 of the Labor Code, the LA should have referred
first the labor dispute involving the implementation of the CBA to the grievance machinery provided therein and, if
unresolved within 7 days, it shall automatically be referred to voluntary arbitration.

FACTS:
Petitioners were employed as sewers by respondent RGMI, and members of KMM Kilusan (union). After the
union staged an illegal strike against RGMI, petitioners were among those recalled to their employment on the
condition that they would no longer be paid a daily rate but on a piece-rate basis. Without allowing RGMI to
normalize its operations, the union filed a notice of strike in NCMB on the ground that RGMI committed an unfair
labor practice. RGMI violated the existing CBA by conducting a time and motion study and chaning the salary
scheme from a daily rate to piece-rate basis without consulting it. RGMI filed a notice of lockout in the NCMB.
While the union and RGMI were undergoing conciliation in the NCMB, RGMI transferred its factory site. Hence,
the union went on strike and blocked the entry to RGMIs (new) premises. DOLE Secretary assumed jurisdiction
pursuant to Article 263(g) of LC and ordered RGMIs striking workers to return to work immediately.

DOLE Secretary Order: (Neither the union nor RGMI appealed the aforementioned order)
a) Ordered all employees to return to work and RGMI to pay its employees their unpaid salaries on the piece-rate
basis.
b) RGMI did not lock out its employees inasmuch as it informed them of the transfer of the worksite.
c) The employees would receive higher wages if they were paid on a piece-rate rather than on a daily rate basis.
Since the new salary scheme would be more advantageous to the employees, despite the provisions of the CBA,
the change in salary scheme was validated.

While the conciliation proceedings between the union and RGMI were pending, petitioners filed before the LA a
complaint for illegal dismissal against RGMI and demanded the payment of their accrued salaries and the
monetary equivalent of benefits they were entitled to under the CBA allegedly withheld by RGMI.

LA RULING:
In favor of petitioners. RGMI did not pay petitioners their salaries and deprived them of the benefits they were
entitled to under the CBA.

NLRC RULING:
Affirmed the LA decision.

CA RULING:
Reversed the NLRC decision on the ground that the LA had no jurisdiction over the complaint.

Respondents’ contention:
LA had no jurisdiction over the complaint because it involved the implementation of the CBA. The alleged
violations of the CBA should be resolved according to the grievance procedure laid out therein.

Petitioners’ contentions:

Angeles| Bajana | Balladares | Brillantes | Briones | Cabansag | Callanta | Chua | David|

De Leon | Gomez | Lopez | Macalino | Nostratis | Padilla | Reynon | Santos | Tan |Velasco
4E / 4F - 2018-2019
Page 475 of 920
LABOR REVIEW DIGEST
Atty. Joyrich Golangco

LA had jurisdiction inasmuch as the complaint was for illegal dismissal. The order of the Secretary of Labor was
inapplicable to them because despite being members of the union, they were not among those who went on
strike.

ISSUES:
(1) Whether the LA has jurisdiction over the labor dispute involving the manner of ascertaining employees’
salaries, a matter that was governed by the existing CBA.

(2) Whether the order of the Secretary of Labor is applicable to petitioners, who despite being members of the
union, were not among those who went on strike.

HELD:

(1) NO. The issue in this case is not a simple case of illegal dismissal but a labor dispute involving the
implementation of the CBA. Pursuant to Articles 217 in relation to Articles 260 and 261 of the Labor Code, the
LA should have referred the matter to the grievance machinery provided in the CBA. Because the labor arbiter
clearly did not have jurisdiction over the subject matter, his decision was void.

Under Article 217(c) of the LC, the jurisdiction of LA and NLRC:


(c) Cases arising from the interpretation or implementation of collective bargaining agreements and those arising
from the interpretation or enforcement of company personnel policies shall be disposed of by the Labor Arbiter by
referring the same to the grievance machinery and voluntary arbitration as may be provided in said agreements.

Article 260 of LC clarifies that such disputes involving the implementation of CBAs must be referred first to the
grievance machinery provided therein and, if unresolved within 7 days, they shall automatically be referred to
voluntary arbitration.

Under Article 261 of LC:


The Voluntary Arbitrator or panel of Voluntary Arbitrators shall have original and exclusive jurisdiction to hear and
decide all unresolved grievances arising from the interpretation or implementation of the Collective Bargaining
Agreement and those arising from the interpretation or enforcement of company personnel policies referred to in
the immediately preceding Article. Accordingly, violations of a Collective Bargaining Agreement, except those
which are gross in character, shall no longer be treated as unfair labor practice and shall be resolved as
grievances under the Collective Bargaining Agreement. For purposes of this Article, gross violations of a
Collective Bargaining Agreement shall mean flagrant and/or malicious refusal to comply with the economic
provisions of such agreement.

(2) YES. Article 263(g) of LC gives the Secretary of Labor discretion to assume jurisdiction over a labor dispute
likely to cause a strike or a lockout in an industry indispensable to the national interest and to decide the
controversy or to refer the same to the NLRC for compulsory arbitration. In doing so, the Secretary of Labor shall
resolve all questions and controversies in order to settle the dispute. His power is therefore plenary and
discretionary in nature to enable him to effectively and efficiently dispose of the issue.

The Secretary of Labor resolved the labor dispute between the union and RGMI in his order. Since neither the
union nor RGMI appealed the said order, it became final and executory. Settled is the rule that unions are the
agent of its members for the purpose of securing just and fair wages and good working conditions. Since
petitioners were part of the bargaining unit represented by the union and members thereof, the order of the
Secretary of Labor applies to them. Since the union was the bargaining agent of petitioners, the complaint was
barred under the principle of conclusiveness of judgments. The parties to a case are bound by the findings in a

Angeles| Bajana | Balladares | Brillantes | Briones | Cabansag | Callanta | Chua | David|

De Leon | Gomez | Lopez | Macalino | Nostratis | Padilla | Reynon | Santos | Tan |Velasco
4E / 4F - 2018-2019
Page 476 of 920
LABOR REVIEW DIGEST
Atty. Joyrich Golangco

previous judgment with respect to matters actually raised and adjudged therein. Hence, the LA should have
dismissed the complaint on the ground of res judicata.
————————————————————————

Angeles| Bajana | Balladares | Brillantes | Briones | Cabansag | Callanta | Chua | David|

De Leon | Gomez | Lopez | Macalino | Nostratis | Padilla | Reynon | Santos | Tan |Velasco
4E / 4F - 2018-2019
Page 477 of 920
LABOR REVIEW DIGEST
Atty. Joyrich Golangco

2. LBERT TENG, doing business under the firm name ALBERT TENG FISH TRADING, and EMILIA TENG-
CHUA, Petitioners, vs. ALFREDO S. PAHAGAC, EDDIE D. NIPA, ORLANDO P. LAYESE, HERNAN Y.
BADILLES and ROGER S. PAHAGAC, Respondents.
GR No.: 169704 Date: November 17, 2010
Ponente: BRION, J.:

Doctrine: Article 262-A of the Labor Code does not prohibit the filing of a motion for reconsideration. In Coca-
Cola Bottlers Phil., Inc., Sales Force Union-PTGWO-Balais v. Coca-Cola Bottlers Philippines, Inc., we likewise
ruled that the VA’s decision may still be reconsidered on the basis of a motion for reconsideration seasonably filed
within 10 days from receipt thereof. The seasonable filing of a motion for reconsideration is a mandatory
requirement to forestall the finality of such decision

FACTS:
Albert Teng Fish Trading is engaged in deep sea fishing and, for this purpose, owns boats (basnig), equipment,
and other fishing paraphernalia. As owner of the business, Teng claims that he customarily enters into joint
venture agreements with master fishermen (maestros) who are skilled and are experts in deep sea fishing; they
take charge of the management of each fishing venture, including the hiring of the members of its complement.
He avers that the maestros hired the respondent workers as checkers to determine the volume of the fish caught
in every fishing voyage.4
On February 20, 2003, the respondent workers filed a complaint for illegal dismissal against Albert Teng Fish
Trading, Teng, and Chua before the NCMB, Region Branch No. IX, Zamboanga City.
They asserted that sometime in September 2002, Teng expressed his doubts on the correct volume of fish caught
in every fishing voyage. In December 2002, Teng informed them that their services had been terminated.
In his defense, Teng maintained that he did not have any hand in hiring the respondent workers; the maestros,
rather than he, invited them to join the venture. According to him, his role was clearly limited to the provision of the
necessary capital, tools and equipment, consisting of basnig, gears, fuel, food, and other supplies.
The VA rendered a decision in Teng’s favor and declared that no employer-employee relationship existed
between Teng and the respondent workers.
On July 21, 2003, the respondent-workers elevated the case to the CA. In its decision of September 21, 2004, the
CA reversed the VA’s decision after finding sufficient evidence showing the existence of employer-employee
relationship.
Teng contends that the VA’s decision is not subject to a motion for reconsideration in the absence of any specific
provision allowing this recourse under Article 262-A of the Labor Code. He cites the 1989 Procedural Guidelines,
which, as the VA declared, does not provide the remedy of a motion for reconsideration. He claims that after the
lapse of 10 days from its receipt, the VA’s decision becomes final and executory unless an appeal is taken. He
argues that when the respondent workers received the VA’s decision on June 12, 2003, they had 10 days, or until
June 22, 2003, to file an appeal. As the respondent workers opted instead to move for reconsideration, the 10-day
period to appeal continued to run; thus, the VA’s decision had already become final and executory by the time
they assailed it before the CA on July 21, 2003.
Teng further insists that the VA was correct in ruling that there was no employer-employee relationship between
him and the respondent workers. What he entered into was a joint venture agreement with the maestros, where
Teng’s role was only to provide basnig, gears, nets, and other tools and equipment for every fishing voyage.

ISSUE/S:
1. WON Article 262-A precludes the filing of a motion for reconsideration of the VA’s decision within the 10-day
period.
2. WON there exists an employer-employee relationship between Teng and the respondent workers.

HELD:
1. Article 262-A of the Labor Code does not prohibit the filing of a motion for reconsideration.
Angeles| Bajana | Balladares | Brillantes | Briones | Cabansag | Callanta | Chua | David|

De Leon | Gomez | Lopez | Macalino | Nostratis | Padilla | Reynon | Santos | Tan |Velasco
4E / 4F - 2018-2019
Page 478 of 920
LABOR REVIEW DIGEST
Atty. Joyrich Golangco

In Coca-Cola Bottlers Phil., Inc., Sales Force Union-PTGWO-Balais v. Coca-Cola Bottlers Philippines, Inc., we
likewise ruled that the VA’s decision may still be reconsidered on the basis of a motion for reconsideration
seasonably filed within 10 days from receipt thereof.28 The seasonable filing of a motion for reconsideration is a
mandatory requirement to forestall the finality of such decision.29 We further cited the 1989 Procedural Guidelines
which implemented Article 262-A, viz:
[U]nder Section 6, Rule VII of the same guidelines implementing Article 262-A of the Labor Code, this Decision, as
a matter of course, would become final and executory after ten (10) calendar days from receipt of copies of the
decision by the parties x x x unless, in the meantime, a motion for reconsideration or a petition for review to the
Court of Appeals under Rule 43 of the Rules of Court is filed within the same 10-day period.
These rulings fully establish that the absence of a categorical language in Article 262-A does not preclude the
filing of a motion for reconsideration of the VA’s decision within the 10-day period. Teng’s allegation that the VA’s
decision had become final and executory by the time the respondent workers filed an appeal with the CA thus
fails. We consequently rule that the respondent workers seasonably filed a motion for reconsideration of the VA’s
judgment, and the VA erred in denying the motion because no motion for reconsideration is allowed.
The Court notes that despite our interpretation that Article 262-A does not preclude the filing of a motion for
reconsideration of the VA’s decision, a contrary provision can be found in Section 7, Rule XIX of the Department
of Labor’s Department Order (DO) No. 40, series of 2003:
Rule XIX
Section 7. Finality of Award/Decision. – The decision, order, resolution or award of the voluntary arbitrator or
panel of voluntary arbitrators shall be final and executory after ten (10) calendar days from receipt of the copy of
the award or decision by the parties and it shall not be subject of a motion for reconsideration.
We are surprised that neither the VA nor Teng cited DO 40-03 and the 2005 Procedural Guidelines as authorities
for their cause, considering that these were the governing rules while the case was pending and these directly and
fully supported their theory. Had they done so, their reliance on the provisions would have nevertheless been
unavailing for reasons we shall now discuss.
By disallowing reconsideration of the VA’s decision, Section 7, Rule XIX of DO 40-03 and Section 7 of the 2005
Procedural Guidelines went directly against the legislative intent behind Article 262-A of the Labor Code. These
rules deny the VA the chance to correct himself and compel the courts of justice to prematurely intervene with the
action of an administrative agency entrusted with the adjudication of controversies coming under its special
knowledge, training and specific field of expertise. In this era of clogged court dockets, the need for specialized
administrative agencies with the special knowledge, experience and capability to hear and determine promptly
disputes on technical matters or intricate questions of facts, subject to judicial review, is indispensable. In
Industrial Enterprises, Inc. v. Court of Appeals, we ruled that relief must first be obtained in an administrative
proceeding before a remedy will be supplied by the courts even though the matter is within the proper jurisdiction
of a court.

2. There exists an employer-employee relationship between Teng and the respondent workers.
We agree with the CA’s finding that sufficient evidence exists indicating the existence of an employer-employee
relationship between Teng and the respondent workers.
While Teng alleged that it was the maestros who hired the respondent workers, it was his company that issued to
the respondent workers identification cards (IDs) bearing their names as employees and Teng’s signature as the
employer. Generally, in a business establishment, IDs are issued to identify the holder as a bona fide employee of
the issuing entity.
For the 13 years that the respondent workers worked for Teng, they received wages on a regular basis, in addition
to their shares in the fish caught. The worksheet showed that the respondent workers received uniform amounts
within a given year, which amounts annually increased until the termination of their employment in 2002. Teng’s
claim that the amounts received by the respondent workers are mere commissions is incredulous, as it would
mean that the fish caught throughout the year is uniform and increases in number each year.
More importantly, the element of control – which we have ruled in a number of cases to be a strong indicator of
the existence of an employer-employee relationship – is present in this case. Teng not only owned the tools and
Angeles| Bajana | Balladares | Brillantes | Briones | Cabansag | Callanta | Chua | David|

De Leon | Gomez | Lopez | Macalino | Nostratis | Padilla | Reynon | Santos | Tan |Velasco
4E / 4F - 2018-2019
Page 479 of 920
LABOR REVIEW DIGEST
Atty. Joyrich Golangco

equipment, he directed how the respondent workers were to perform their job as checkers; they, in fact, acted as
Teng’s eyes and ears in every fishing expedition.
Teng cannot hide behind his argument that the respondent workers were hired by the maestros. To consider the
respondent workers as employees of the maestros would mean that Teng committed impermissible labor-only
contracting. As a policy, the Labor Code prohibits labor-only contracting
There is "labor-only" contracting where the person supplying workers to an employer does not have
substantial capital or investment in the form of tools, equipment, machineries, work premises, among
others, and the workers recruited and placed by such persons are performing activities which are directly
related to the principal business of such employer. In such cases, the person or intermediary shall be
considered merely as an agent of the employer who shall be responsible to the workers in the same manner and
extent as if the latter were directly employed by him.
In the present case, the maestros did not have any substantial capital or investment.1avvphi1 Teng admitted that
he solely provided the capital and equipment, while the maestros supplied the workers. The power of control over
the respondent workers was lodged not with the maestros but with Teng. As checkers, the respondent workers’
main tasks were to count and classify the fish caught and report them to Teng. They performed tasks that were
necessary and desirable in Teng’s fishing business. Taken together, these incidents confirm the existence of a
labor-only contracting which is prohibited in our jurisdiction, as it is considered to be the employer’s attempt to
evade obligations afforded by law to employees.
Accordingly, we hold that employer-employee ties exist between Teng and the respondent workers. A finding that
the maestros are labor-only contractors is equivalent to a finding that an employer-employee relationship exists
between Teng and the respondent workers. As regular employees, the respondent workers are entitled to all the
benefits and rights appurtenant to regular employment.
The respondent worker’s allegation that Teng summarily dismissed them on suspicion that they were not reporting
to him the correct volume of the fish caught in each fishing voyage was never denied by Teng. Unsubstantiated
suspicion is not a just cause to terminate one’s employment under Article 28250 of the Labor Code. To allow an
employer to dismiss an employee based on mere allegations and generalities would place the employee at the
mercy of his employer, and would emasculate the right to security of tenure.51 For his failure to comply with the
Labor Code’s substantive requirement on termination of employment, we declare that Teng illegally dismissed the
respondent workers.
————————————————————————

Angeles| Bajana | Balladares | Brillantes | Briones | Cabansag | Callanta | Chua | David|

De Leon | Gomez | Lopez | Macalino | Nostratis | Padilla | Reynon | Santos | Tan |Velasco
4E / 4F - 2018-2019
Page 480 of 920
LABOR REVIEW DIGEST
Atty. Joyrich Golangco

3. SAMAHAN NG MGA MANGGAGAWA SA HYATT (SAMASAH-NUWHRAIN) vs HON. VOLUNTARY


ARBITRATOR BUENAVENTURA C. MAGSALIN and HOTEL ENTERPRISES OF THE PHILIPPINES, INC., et
al
GR No: 164939 Date: June 6, 2011
Ponente: Villarama, Jr.

Doctrine:
The decision or award of the voluntary arbitrator or panel of arbitrators should is appealable to the CA under Rule
43 of the Rules of Court.

FACTS:
Petitioner SAMASAH-NUWHRAIN is a duly registered union and the certified bargaining representative of the
rank-and-file employees of Hyatt Regency Manila, a five-star hotel owned and operated by respondent Hotel
Enterprises of the Philippines, Inc.

Angelito Caragdag, a waiter at the hotel and a director of the union, committed various violations (refused to be
frisked by the security personnel, threatening, intimidating, coercing, and provoking to a fight your superior for
reasons directly connected with his discharge of official duty, left his work assignment during official hours without
prior permission) of the company's code of discipline which led to his suspension on different occasions.

Because of the succession of infractions he committed, the HRD required Caragdag to explain why the hotel's
OSDA 4.32 (Committing offenses which are penalized with 3 suspensions during a 12-month period) should not
be enforced against him. An investigation board was formed after receipt of Caragdag's written explanation, and
the matter was set for hearing. However, despite notice of the scheduled hearing, both Caragdag and the Union
President failed to attend. Thereafter, the investigating board resolved on the said date to dismiss Caragdag for
violation of OSDA 4.32. Caragdag appealed but the investigating board affirmed its resolution after hearing. The
hotel then sent Caragdag a notice of dismissal.

Caragdag's dismissal was questioned by petitioner, and the dispute was referred to voluntary arbitration upon
agreement of the parties.

VOLUNTARY ARBITRATOR RULING:


The VA ruled that the three separate suspensions of Caragdag are valid, his dismissal is legal and OSDA 4.32 of
Hyatt's Code of Discipline is reasonable. However, for humanitarian considerations, Hyatt was ordered to grant
financial assistance of 100,000 to Caragdag.

CA RULING:
The petitioner assailed the decision of the VA to the CA in a petition for certiorari which the CA dismissed outright
for being the wrong remedy. Petitioner filed a MR, but the motion was denied by the CA.

The respondent also filed a petition for review with the CA on the ground that the VA committed a grievous error in
awarding financial assistance to Caragdag despite his finding that the dismissal due to serious misconduct was
valid. The CA deleted said award of financial assistance.

APPEAL TO THE SC:

Petitioner's Contention:
Petitioner argues that because decisions rendered by VAs are issued under Title VII-A of the Labor Code, they
are not covered by Rule 43 of the 1997 Rules of Civil Procedure. Section 2, petitioner points out, expressly

Angeles| Bajana | Balladares | Brillantes | Briones | Cabansag | Callanta | Chua | David|

De Leon | Gomez | Lopez | Macalino | Nostratis | Padilla | Reynon | Santos | Tan |Velasco
4E / 4F - 2018-2019
Page 481 of 920
LABOR REVIEW DIGEST
Atty. Joyrich Golangco

provides that Rule 43 shall not apply to judgments or final orders issued under the Labor Code of the Philippines.
Hence, a petition for certiorari under Rule 65 is the proper remedy for questioning the decision of the VA.

Also, petitioner argues that, while it is true that the award of financial assistance is given only for dismissals due to
causes specified under Articles 283 and 284 of the Labor Code, as amended, the SC has, by way of exception,
allowed the grant of financial assistance to an employee dismissed for just causes based on equity.

Respondent's Contention:
Respondent maintains that the CA acted correctly in dismissing the petition for certiorari for being the wrong mode
of appeal. Section 1 of Rule 43 clearly states that it is the governing rule with regard to appeals from awards,
judgments, final orders or resolutions of voluntary arbitrators.

Respondent further asserts that the CA correctly deleted the award of financial assistance erroneously granted to
Caragdag considering that he was found guilty of serious misconduct and other acts adversely reflecting on his
moral character.

ISSUE/S:
1. whether the CA erred in dismissing outright the petition for certiorari filed before it on the ground that the same
is an improper mode of appeal? - No
2. whether the CA erred in deleting the award of financial assistance in the amount of P100,000.00 to Caragdag. -
No

HELD:
3. No. The decision or award of the voluntary arbitrator or panel of arbitrators should is appealable to the CA, in
line with the procedure outlined in Revised Administrative Circular No. 1-95 (now in Rule 43 of the 1997 Rules of
Civil Procedure), just like those of the quasi-judicial agencies, boards and commissions enumerated therein, and
consistent with the original purpose to provide a uniform procedure for the appellate review of adjudications of all
quasi-judicial entities.

Hence, upon receipt of the VA's Resolution denying petitioners motion for reconsideration, petitioner should have
filed with the CA, within the 15-day reglementary period, a petition for review, not a petition for certiorari.

2. No. Severance compensation, or whatever name it is called, on the ground of social justice shall be allowed
only when the cause of the dismissal is other than serious misconduct or for causes which reflect adversely on the
employees moral character.

The policy of social justice is not intended to countenance wrongdoing simply because it is committed by the
underprivileged. At best it may mitigate the penalty but it certainly will not condone the offense. Compassion for
the poor is an imperative of every humane society but only when the recipient is not a rascal claiming an
undeserved privilege. Social justice cannot be permitted to be refuge of scoundrels any more than can equity be
an impediment to the punishment of the guilty.

Caragdags dismissal being due to serious misconduct, it follows that he should not be entitled to financial
assistance. To rule otherwise would be to reward him for the grave misconduct he committed. We must
emphasize that social justice is extended only to those [33] who deserve its compassion.

Petition is denied.
————————————————————————

Angeles| Bajana | Balladares | Brillantes | Briones | Cabansag | Callanta | Chua | David|

De Leon | Gomez | Lopez | Macalino | Nostratis | Padilla | Reynon | Santos | Tan |Velasco
4E / 4F - 2018-2019
Page 482 of 920
LABOR REVIEW DIGEST
Atty. Joyrich Golangco

4. BARONDA vs COURT OF APPEALS AND HIDECO SUGAR MILLING CO., INC.


GR No.: 161006 Date: October 14, 2015

DOCTRINE:
The timely filing of an MR or of an Appeal forestalls the finality of the desicion or award of the VA but the
reinstatement aspect of said decision remains executory regardless of the filing of such MR or Appeal. The
immediate reinstatement of the employee pending appeal has bee introduce by Section 12 of RA No. 6751 which
amended Article 223 of the Labor Code.

FACTS:
Hideco Sugar Milling employed petitioner as a mud press truck driver. He hit HIDECO’s transmission lines while
operating a dumptruck, causing a factory-wide blackout which restoration had cost HIDECO. HIDECO then served
a notice of offense requiring him to issue an explanation within 3 days from notice. After compliance, he was
found guilty of negligence and the management recommended his dismissal.

The resident manager served his notice of termination effective at the close of officer hours that day. The
petitioner filed in the Office of the Voluntary Arbitrator of the NCMB in Tacloban City, a complaint for Illegal
Dismissal against HIDECO.

The VA held the dismissal illegal with an order for reinstatement, holding said separation from service as a work
suspension without pay and commanded him to pay on installment basis, the damages sustained by HIDECO
from the incident. HIDECO’s MR was denied and HIDECO subsequently reinstated him.

ISSUE/S:
Whether or not the reinstatement aspect of the Voluntary Arbitrator’s decision was executory pending appeal.

HELD:
YES. The timely filing of an MR or of an Appeal forestalls the finality of the desicion or award of the VA but the
reinstatement aspect of said decision remains executory regardless of the filing of such MR or Appeal. The
immediate reinstatement of the employee pending appeal has bee introduce by Section 12 of RA No. 6751 which
amended Article 223 of the Labor Code.

The reinstatement order of the VA shold have the same authority, force and effect as that of the Labor Arbiter, not
only to encourage parties to settle their disputes through this mode, but to enforce the constitutional mandate to
protect labor, provide security of tenure and to enhance social justice.
————————————————————————

Angeles| Bajana | Balladares | Brillantes | Briones | Cabansag | Callanta | Chua | David|

De Leon | Gomez | Lopez | Macalino | Nostratis | Padilla | Reynon | Santos | Tan |Velasco
4E / 4F - 2018-2019
Page 483 of 920
LABOR REVIEW DIGEST
Atty. Joyrich Golangco

5. NYK-FIL SHIP MANAGEMENT, INCORPORATED vs DABU


G.R. No. 225142, September 13, 2017
Peralta, J.:

DOCTRINE: Despite Rule 43 providing for a 15-day period to appeal, the Voluntary Arbitrator's decision must be
appealed before the Court of Appeals within 10 calendar days from receipt of the decision as provided in the
Labor Code.

FACTS: Petitioner NYK-Fil Ship Management, Inc., a local manning agent acting for and in behalf of its foreign
principal NYK Ship Management Pte. Ltd. Singapore, hired respondent Gener G. Dabu to work as oiler for nine
months on board the vessel M/V Hojin.
As soon as respondent embarked the vessel and discharged his duty as oiler he had palpitations, pains all over
the body, numbness of hands and legs, lack of sleep and nervousness. Upon consultation with a doctor, he was
diagnosed with elevated blood sugar level and was suffering from diabetes mellitus, and declared him unfit for sea
duty. He was repatriated to Manila and upon his arrival, he was immediately referred to the company-designated
physician at NGC Medical Specialist Clinic, Inc. who examined him and showed a similar result.
The company-designated physician declared that respondent's diabetes mellitus is not work-related. Respondent
wrote letters to petitioner appealing for the continuation of his treatment since his sickness was work-related
taking into account his 23 years of working in petitioner's various vessels.
Respondent also consulted 2 more doctors/specialists who both declared him to be unfit to work. Respondent
sought payment of disability benefits, damages and attorney's fees from petitioner, but was denied. He requested
for a grievance proceedings in accordance with the CBA, however, the parties did not reach any settlement. He
then filed a notice to arbitrate with the National Conciliation Mediation Board (NCMB), and the parties were
required to submit their position papers.
On November 28, 2014 NCMB-Panel of Voluntary Arbitrators (PVA) rendered a decision ordering the
respondents, jointly and severally, to pay complainant the following amounts for Disability compensation and
attorney’s fees.

Petitioner received a copy of the PVA decision on February 9, 2015 and filed with the CA a petition for review
under Rule 43 of the Rules of Court on February 24, 2015.

CA Ruling: CA initially granted the petition of Petitioner and reversed and set aside NCMB-PVA decision.
However, upon private respondent's motion for reconsideration, CA eventually recalled its decision and entered a
new dismissing the petition for having been filed out of time.

ISSUE: Whether or not the Petition for Review with the CA is filed out of time

SC Ruling: YES. The Labor Code provides under Art 276: xxx
The award or decision of the Voluntary Arbitrator or Panel of Voluntary Arbitrators shall contain the facts and the
law on which it is based. It shall be final and executory after ten (10) calendar days from receipt of the copy of the
award or decision by the parties.
and Section 6, Rule VII of the NCMB Procedural Guidelines in the conduct of voluntary arbitration proceedings
provides:
Section 6. Finality of Award or Decisions. - Awards or decisions of voluntary arbitrator become final and executory
after ten (10) calendar days from receipt of copies of the award or decision by the parties.
Clearly, the decision of the voluntary arbitrator becomes final and executory after 10 days from receipt thereof.
The proper remedy to reverse or modify a voluntary arbitrators' or panel of voluntary arbitrators' decision is to
appeal the award or decision via a petition under Rule 43 of the 1997 Rules of Civil Procedure. And under
Section 4 of Rule 43, the period to appeal to the CA is 15 days from receipt of the decision. Notwithstanding, since
Article 276 of the Labor Code expressly provides that the award or decision of the voluntary arbitrator shall be
Angeles| Bajana | Balladares | Brillantes | Briones | Cabansag | Callanta | Chua | David|

De Leon | Gomez | Lopez | Macalino | Nostratis | Padilla | Reynon | Santos | Tan |Velasco
4E / 4F - 2018-2019
Page 484 of 920
LABOR REVIEW DIGEST
Atty. Joyrich Golangco

final and executory after ten (10) calendar days from receipt of the decision by the parties, the appeal of the VA
decision to the CA must be filed within 10 days.
It is true that Rule 43, Section 4 of the Rules of Court provides for a 15-day reglementary period for filing an
appeal:
However, in a long line of cases, the Court denied petitioners' petitions for review on certiorari since petitioners
failed to appeal the Voluntary Arbitrator's decision within the 15-day reglementary period under Rule 43. In these
cases, the Court of Appeals had no jurisdiction to entertain the appeal assailing the Voluntary Arbitrator's decision.
Thus, despite Rule 43 providing for a 15-day period to appeal, we rule that the Voluntary Arbitrator's decision must
be appealed before the Court of Appeals within 10 calendar days from receipt of the decision as provided in the
Labor Code.
We ruled that Article 276 of the Labor Code allows the appeal of decisions rendered by Voluntary Arbitrators.
Statute provides that the Voluntary Arbitrator's decision "shall be final and executory after ten (10) calendar days
from receipt of the copy of the award or decision by the parties." Being provided in the statute, this 10-day period
must be complied with; otherwise, no appellate court will have jurisdiction over the appeal. This absurd situation
occurs when the decision is appealed on the 11th to 15th day from receipt as allowed under the Rules, but which
decision, under the law, has already become final and executory.
The rule, therefore, is that a Voluntary Arbitrator's award or decision shall be appealed before the Court of
Appeals within 10 days from receipt of the award or decision. Should the aggrieved party choose to file a motion
for reconsideration with the Voluntary Arbitrator, the motion must be filed within the same 10-day period since a
motion for reconsideration is filed "within the period for taking an appeal."
————————————————————————

Angeles| Bajana | Balladares | Brillantes | Briones | Cabansag | Callanta | Chua | David|

De Leon | Gomez | Lopez | Macalino | Nostratis | Padilla | Reynon | Santos | Tan |Velasco
4E / 4F - 2018-2019
Page 485 of 920
LABOR REVIEW DIGEST
Atty. Joyrich Golangco

Article 278-279
1. Abaria vs. NLRC
GR No: 154113 Date: December 7, 2011
Ponente: Villarama, Jr. J.

Doctrine: Considering their persistence in holding picketing activities despite the declaration that their union was
not duly registered as a legitimate labor organization, and their filing of the notice of strike and conducting a strike
vote notwithstanding that their union has no legal personality to negotiate with MCCHI for collective bargaining
purposes, there is no question that NAMA-MCCH-NFL officers knowingly participated in the illegal strike.

FACTS: Metro Cebu Community Hospital, Inc. (MCCHI), presently known as the Visayas Community Medical
Center (VCMC), is a non-stock, non-profit corporation organized under the laws of the Republic of the Philippines.
It operates the Metro Cebu Community Hospital (MCCH), a tertiary medical institution located at Osmea
Boulevard, Cebu City. MCCH is owned by the United Church of Christ in the Philippines (UCCP) and Rev.
Gregorio P. Iyoy is the Hospital Administrator.

The National Federation of Labor (NFL) is the exclusive bargaining representative of the rank-and-file employees
of MCCHI.

On December 6, 1995, Nava, President of Nagkahiusang Mamumuo sa MCCH (NAMA-MCCH-NFL), wrote Rev.
Iyoy expressing the unions desire to renew the CBA, attaching to her letter a statement of proposals
signed/endorsed by 153 union members. However, MCCHI returned the CBA proposal for Nava to secure first the
endorsement of the legal counsel of NFL, Atty. Alforque, as the official bargaining representative of MCCHI
employees. Meanwhile, Atty. Alforque informed MCCHI that the proposed CBA submitted by Nava was never
referred to NFL and that NFL has not authorized any other legal counsel or any person for collective bargaining
negotiations.

Thereafter, several union members led by Nava and her group launched a series of mass actions such as wearing
black and red armbands/headbands, marching around the hospital premises and putting up placards, posters and
streamers. Atty. Alforque immediately disowned the concerted activities being carried out by union members
which are not sanctioned by NFL.

MCCHI directed the union officers led by Nava to submit within 48 hours a written explanation why they should not
be terminated for having engaged in illegal concerted activities amounting to strike, and placed them under
immediate preventive suspension.

(DOLE) Regional Office No. 7 issued certifications stating that there is nothing in their records which shows that
NAMA-MCCH-NFL is a registered labor organization. MCCHI then sent individual notices to all union members
asking them to submit within 72 hours a written explanation why they should not be terminated for having
supported the illegal concerted activities of NAMA-MCCH-NFL which has no legal personality as per DOLE
records.

NAMA-MCCH-NFL filed a Notice of Strike but the same was deemed not filed for want of legal personality on the
part of the filer. Despite such rebuff, Nava and her group still conducted a strike vote during which an
overwhelming majority of union members approved the strike.

Unfazed, the striking union members held more mass actions. The means of ingress to and egress from the
hospital were blocked so that vehicles carrying patients and employees were barred from entering the premises.
Employees and patients reported acts of intimidation and harassment perpetrated by union leaders and members.

Angeles| Bajana | Balladares | Brillantes | Briones | Cabansag | Callanta | Chua | David|

De Leon | Gomez | Lopez | Macalino | Nostratis | Padilla | Reynon | Santos | Tan |Velasco
4E / 4F - 2018-2019
Page 486 of 920
LABOR REVIEW DIGEST
Atty. Joyrich Golangco

The City Government of Cebu ordered the demolition of the structures and obstructions put up by the picketing
employees of MCCHI along the sidewalk, having determined the same as a public nuisance or nuisance per se.
The striking employees were then terminated. Thereafter, several complaints for illegal dismissal and unfair labor
practice were filed by the terminated employees against MCCHI.

LA/RTC/NLRC/CA RULING: Labor Arbiter Belarmino found no basis for the charge of unfair labor practice and
declared the strike and picketing activities illegal having been conducted by NAMA-MCCH-NFL which is not a
legitimate labor organization. The termination of union leaders was upheld as valid

APPEAL TO THE SC:

Petitioner's Contention: In their collective response/statement it was explained that the picketing employees
wore armbands to protest MCCHIs refusal to bargain; it was also contended that MCCHI cannot question the legal
personality of the union which had actively assisted in CBA negotiations and implementation.

Respondent's Contention: The wildcat concerted activities spearheaded by the Nava group is illegal without a
valid Notice of Strike

ISSUE/S: Whether or not NAMA-MCCH-NFL committed illegal strike

HELD: Yes. To prove majority support of the employees, NAMA-MCCH-NFL presented the CBA proposal
allegedly signed by 153 union members. However, the petition signed by said members showed that the
signatories endorsed the proposed terms and conditions without stating that they were likewise voting for or
designating the NAMA-MCCH-NFL as their exclusive bargaining representative. In any case, NAMA-MCCH-NFL
at the time of submission of said proposals was not a duly registered labor organization, hence it cannot legally
represent MCCHIs rank-and-file employees for purposes of collective bargaining. Hence, even assuming that
NAMA-MCCH-NFL had validly disaffiliated from its mother union, NFL, it still did not possess the legal personality
to enter into CBA negotiations.

Furthermore, the strike was illegal due to the commission of the following prohibited activities: (1) violence,
coercion, intimidation and harassment against non-participating employees; and (2) blocking of free ingress to and
egress from the hospital, including preventing patients and their vehicles from entering the hospital and other
employees from reporting to work, the putting up of placards with a statement advising incoming patients to
proceed to another hospital because MCCHI employees are on strike/protest.

The findings of the Executive Labor Arbiter and NLRC, as sustained by the appellate court, clearly established
that the striking union members created so much noise, disturbance and obstruction that the local government
authorities eventually ordered their removal for being a public nuisance.

Considering their persistence in holding picketing activities despite the declaration that their union was not duly
registered as a legitimate labor organization, and their filing of the notice of strike and conducting a strike vote
notwithstanding that their union has no legal personality to negotiate with MCCHI for collective bargaining
purposes, there is no question that NAMA-MCCH-NFL officers knowingly participated in the illegal strike. The CA
therefore did not err in ruling that the termination of union officers.

Angeles| Bajana | Balladares | Brillantes | Briones | Cabansag | Callanta | Chua | David|

De Leon | Gomez | Lopez | Macalino | Nostratis | Padilla | Reynon | Santos | Tan |Velasco
4E / 4F - 2018-2019
Page 487 of 920
LABOR REVIEW DIGEST
Atty. Joyrich Golangco

With respect to the dismissed union members, although MCCHI submitted photographs taken at the picket line, it
did not individually name those striking employees and specify the illegal act committed by each of them. As to the
affidavits executed by non-striking employees, they identified mostly union officers as the persons who blocked
the hospital entrance, harassed hospital employees and patients whose vehicles were prevented from entering
the premises. Only some of these witnesses actually named a few union members who committed similar acts of
harassment and coercion. Consequently, we find no error committed by the CA when it modified the decision of
the NLRC and ruled that the dismissal of union members who merely participated in the illegal strike was illegal.
————————————————————————

Angeles| Bajana | Balladares | Brillantes | Briones | Cabansag | Callanta | Chua | David|

De Leon | Gomez | Lopez | Macalino | Nostratis | Padilla | Reynon | Santos | Tan |Velasco
4E / 4F - 2018-2019
Page 488 of 920
LABOR REVIEW DIGEST
Atty. Joyrich Golangco

2. YSS EMPLOYEES UNIONPHILIPPINE TRANSPORT AND GENERAL WORKERS ORGANIZATION v. YSS


LABORATORIES, INC., G.R. No. 155125 December 4, 2009 CHICO-NAZARIO, J.

DOCTRINE: Art. 263. Strikes, picketing, and lockouts. (g) When, in his opinion, there exists a labor dispute
causing or likely to cause a strike or lockout in an industry indispensable to the national interest, the Secretary of
Labor and Employment may assume jurisdiction over the dispute and decide it or certify the same to the
Commission for compulsory arbitration. Such assumption or certification shall have the effect of automatically
enjoining the intended or impending strike or lockout as specified in the assumption or certification order. If one
has already taken place at the time of assumption or certification, all striking or locked out employees shall
immediately return to work and the employer shall immediately resume operations and readmit all workers under
the same terms and conditions prevailing before the strike or lockout. The Secretary of Labor and Employment or
the Commission may seek the assistance of law enforcement agencies to ensure compliance with this provision
as well as with such orders as he may issue to enforce the same.

FACTS: In order to arrest escalating business losses, YSS Laboratories, a domestic corporation engaged in
Pharmaceutical business implemented a retrenchment program which affected 11 employees purportedly chosen
in accordance with the reasonable standards established by the company. Of the 11 employees sought to be
retrenched, nine were officers and members of YSSEU, a duly registered labor organization and the sole and
exclusive bargaining representative of the rank-and-file employees of YSS. Claiming that YSS Laboratories was
guilty of discrimination and union-busting in carrying out the said retrenchment program, YSSEU decided to hold a
valid strike. A number of conciliation proceedings were made by the NCMB-NCR but still the dispute was not
resolved. This prompted the Sectary of Labor to intervene. Finding that the labor dispute was inimical to the
national interest, it certifies the case to the NLRC for compulsory arbitration and issued two orders: first, directing
all striking workers to return to work and for the Company to accept them back under the same terms and
conditions of employment prior to the strike; second, that the nine retrenched employees be included in the return
to work order. Aggrieved, YSS Laboratories filed a petition under Rule 65 before the Court of Appeals in which the
latter reversed the orders of the Secretary of Labor and granted the petition of YSS Laboratories. The appellate
court found that YSS Laboratories validly carried out its retrenchment program, which effectively severed the
concerned employees’ employment with the company. Hence, YSSEU comes to this petition.

ISSUE: Whether or not the retrenched employees should be excluded from the coverage of the return-to-
workorder.

HELD: No. The Orders of the Secretary of Labor, certifying the labor dispute involving the herein parties to the
NLRC for compulsory arbitration, and enjoining YSSEU to return to work and YSS Laboratories to admit them
under the same terms and conditions prevailing before the strike, were issued pursuant to Article 263(g) of the
Labor Code. Said provision reads:
Art. 263. Strikes, picketing, and lockouts.
x x x x (g) When, in his opinion, there exists a labor dispute causing or likely to cause a strike or lockout in an
industry indispensable to the national interest, the Secretary of Labor and Employment may assume jurisdiction
over the dispute and decide it or certify the same to the Commission for compulsory arbitration. Such assumption
or certification shall have the effect of automatically enjoining the intended or impending strike or lockout as
specified in the assumption or certification order. If one has already taken place at the time of assumption or
certification, all striking or locked out employees shall immediately return to work and the employer shall
immediately resume operations and readmit all workers under the same terms and conditions prevailing
before the strike or lockout.
It should be noted that the primary reason why the strike was conducted in the first place was to protest the
implementation of the retrenchment program, which clearly discriminated against union officers and members.
YSS Laboratories’ vigorous insistence on the exclusion of the retrenched employees from the coverage of the
return-to-work order seriously impairs the authority of the Secretary of Labor to forestall a labor dispute that he
Angeles| Bajana | Balladares | Brillantes | Briones | Cabansag | Callanta | Chua | David|

De Leon | Gomez | Lopez | Macalino | Nostratis | Padilla | Reynon | Santos | Tan |Velasco
4E / 4F - 2018-2019
Page 489 of 920
LABOR REVIEW DIGEST
Atty. Joyrich Golangco

deems inimical to the national economy. Accordingly, when the Secretary of Labor directed YSS Laboratories to
accept all the striking workers back to work, the Secretary did not exceed his jurisdiction, or gravely abuse the
same, said the Supreme Court. Hence, the petition is granted. The orders of the Secretary of Labor and
Employment are hereby reinstated.
————————————————————————

Angeles| Bajana | Balladares | Brillantes | Briones | Cabansag | Callanta | Chua | David|

De Leon | Gomez | Lopez | Macalino | Nostratis | Padilla | Reynon | Santos | Tan |Velasco
4E / 4F - 2018-2019
Page 490 of 920
LABOR REVIEW DIGEST
Atty. Joyrich Golangco

3. NUWHRAIN-DUSIT HOTEL NIKKO CHAPTER vs SECRETARY OF LABOR AND EMPLOYMENT


GR No: 166295 Date: November 11, 2008
Ponente: Velasco, Jr., J.

Doctrine:
Art. 212(o) of the Labor Code defines a strike as "any temporary stoppage of work by the concerted action of
employees as a result of an industrial or labor dispute."
In Toyota Motor Phils. Corp. Workers Association (TMPCWA) v. National Labor Relations Commission, we cited
the various categories of an illegal strike, to wit:
Noted authority on labor law, Ludwig Teller, lists six (6) categories of an illegal strike, viz.:
(1) [when it] is contrary to a specific prohibition of law, such as strike by employees performing governmental
functions; or
(2) [when it] violates a specific requirement of law[, such as Article 263 of the Labor Code on the requisites of a
valid strike]; or
(3) [when it] is declared for an unlawful purpose, such as inducing the employer to commit an unfair labor practice
against non-union employees; or
(4) [when it] employs unlawful means in the pursuit of its objective, such as a widespread terrorism of non-strikers
[for example, prohibited acts under Art. 264(e) of the Labor Code]; or
(5) [when it] is declared in violation of an existing injunction[, such as injunction, prohibition, or order issued by the
DOLE Secretary and the NLRC under Art. 263 of the Labor Code]; or
(6) [when it] is contrary to an existing agreement, such as a no-strike clause or conclusive arbitration clause.

FACTS:
The Union is the certified bargaining agent of the regular rank-and-file employees of Dusit Hotel Nikko (Hotel), a
five star service establishment owned and operated by Philippine Hoteliers, Inc. located in Makati City. Chiyuki
Fuijimoto and Esperanza V. Alvez are impleaded in their official capacities as the Hotel's General Manager and
Director of Human Resources, respectively.
The Union submitted its Collective Bargaining Agreement (CBA) negotiation proposals to the Hotel. As
negotiations ensued, the parties failed to arrive at mutually acceptable terms and conditions. Due to the
bargaining deadlock, the Union filed a Notice of Strike on the ground of the bargaining deadlock with the National
Conciliation and Mediation Board (NCMB). Thereafter, conciliation hearings were conducted which proved
unsuccessful. Consequently, a Strike Vote was conducted by the Union on which it was decided that the Union
would wage a strike.
Soon thereafter, in the afternoon of January 17, 2002, the Union held a general assembly at its office located in
the Hotel's basement, where some members sported closely cropped hair or cleanly shaven heads. The next day,
or on January 18, 2002, more male Union members came to work sporting the same hair style. The Hotel
prevented these workers from entering the premises claiming that they violated the Hotel's Grooming Standards.
In view of the Hotel's action, the Union staged a picket outside the Hotel premises. Later, other workers were also
prevented from entering the Hotel causing them to join the picket. For this reason the Hotel experienced a severe
lack of manpower which forced them to temporarily cease operations in three restaurants.
Subsequently, the Hotel issued notices to Union members, preventively suspending them and charging them with
the following offenses: (1) violation of the duty to bargain in good faith; (2) illegal picket; (3) unfair labor practice;
(4) violation of the Hotel's Grooming Standards; (5) illegal strike; and (6) commission of illegal acts during the
illegal strike. The next day, the Union filed with the NCMB a second Notice of Strike on the ground of unfair labor
practice and violation of Article 248(a) of the Labor Code on illegal lockout. In the meantime, the Union officers
and members submitted their explanations to the charges alleged by the Hotel, while they continued to stage a
picket just inside the Hotel's compound.
On January 26, 2002, the Hotel terminated the services of twenty-nine (29) Union officers and sixty-one (61)
members; and suspended eighty-one (81) employees for 30 days, forty-eight (48) employees for 15 days, four (4)
employees for 10 days, and three (3) employees for five days. On the same day, the Union declared a strike.
Angeles| Bajana | Balladares | Brillantes | Briones | Cabansag | Callanta | Chua | David|

De Leon | Gomez | Lopez | Macalino | Nostratis | Padilla | Reynon | Santos | Tan |Velasco
4E / 4F - 2018-2019
Page 491 of 920
LABOR REVIEW DIGEST
Atty. Joyrich Golangco

Starting that day, the Union engaged in picketing the premises of the Hotel. During the picket, the Union officials
and members unlawfully blocked the ingress and egress of the Hotel premises.
Consequently, on January 31, 2002, the Union filed its third Notice of Strike with the NCMB, this time on the
ground of unfair labor practice and union-busting.
On the same day, the Secretary, through her January 31, 2002 Order, assumed jurisdiction over the labor dispute
and certified the case to the NLRC for compulsory arbitration. The Secretary ordered the consolidation of the two
Notices of Strike - NCMB-NCR-NS-12-369-01 and NCMB-NCR-NS-01-019-02 - and CERTIFIES the entire labor
dispute covered by these Notices and the intervening events, to the NATIONAL LABOR RELATIONS
COMMISSION for compulsory arbitration pursuant to Article 263 (g) of the Labor Code, as amended, under the
following terms: the Hotel is given the option, in lieu of actual reinstatement, to merely reinstate the dismissed or
suspended workers in the payroll in light of the special circumstances attendant to their reinstatement.
Pursuant to the Secretary's Order, the Hotel, on February 1, 2002, issued an Inter-Office Memorandum, directing
some of the employees to return to work, while advising others not to do so, as they were placed under payroll
reinstatement. Unhappy with the Secretary's January 31, 2002 Order, the Union moved for reconsideration, but
the same was denied per the Secretary's subsequent March 15, 2002 Order. Affronted by the Secretary's January
31, 2002 and March 15, 2002 Orders, the Union filed a Petition for Certiorari with the CA which was docketed as
CA-G.R. SP No. 70778.

NLRC RULING:
It ordered the Hotel and the Union to execute a CBA within 30 days from the receipt of the decision. The NLRC
also held that the January 18, 2002 concerted action was an illegal strike in which illegal acts were committed by
the Union; and that the strike violated the "No Strike, No Lockout" provision of the CBA, which thereby caused the
dismissal of 29 Union officers and 61 Union members. The NLRC ordered the Hotel to grant the 61 dismissed
Union members financial assistance in the amount of ½ month's pay for every year of service or their retirement
benefits under their retirement plan whichever was higher. The NLRC explained that the strike which occurred on
January 18, 2002 was illegal because it failed to comply with the mandatory 30-day cooling-off period10 and
the seven-day strike ban, as the strike occurred only 29 days after the submission of the notice of strike on
December 20, 2001 and only four days after the submission of the strike vote on January 14, 2002. The NLRC
also ruled that even if the Union had complied with the temporal requirements mandated by law, the strike would
nonetheless be declared illegal because it was attended by illegal acts committed by the Union officers and
members.

CA RULING:
CA affirmed NLRC decision. The CA ratiocinated that the Union failed to demonstrate that the NLRC committed
grave abuse of discretion and capriciously exercised its judgment or exercised its power in an arbitrary and
despotic manner.

Petitioner's Contention: The Union maintains that the mass picket conducted by its officers and members did
not constitute a strike and was merely an expression of their grievance resulting from the lockout effected by the
Hotel management.

Respondent's Contention: The Hotel argues that the Union's deliberate defiance of the company rules and
regulations was a concerted effort to paralyze the operations of the Hotel, as the Union officers and members
knew pretty well that they would not be allowed to work in their bald or cropped hair style. For this reason, the
Hotel argues that the Union committed an illegal strike on January 18, 2002 and on January 26, 2002.

ISSUE/S:
Whether or not the Union is liable for conducting illegal strike?

HELD: YES
Angeles| Bajana | Balladares | Brillantes | Briones | Cabansag | Callanta | Chua | David|

De Leon | Gomez | Lopez | Macalino | Nostratis | Padilla | Reynon | Santos | Tan |Velasco
4E / 4F - 2018-2019
Page 492 of 920
LABOR REVIEW DIGEST
Atty. Joyrich Golangco

Art. 212(o) of the Labor Code defines a strike as "any temporary stoppage of work by the concerted action of
employees as a result of an industrial or labor dispute."
In Toyota Motor Phils. Corp. Workers Association (TMPCWA) v. National Labor Relations Commission, we cited
the various categories of an illegal strike, to wit:
Noted authority on labor law, Ludwig Teller, lists six (6) categories of an illegal strike, viz.:
(1) [when it] is contrary to a specific prohibition of law, such as strike by employees performing governmental
functions; or
(2) [when it] violates a specific requirement of law[, such as Article 263 of the Labor Code on the requisites of a
valid strike]; or
(3) [when it] is declared for an unlawful purpose, such as inducing the employer to commit an unfair labor practice
against non-union employees; or
(4) [when it] employs unlawful means in the pursuit of its objective, such as a widespread terrorism of non-strikers
[for example, prohibited acts under Art. 264(e) of the Labor Code]; or
(5) [when it] is declared in violation of an existing injunction[, such as injunction, prohibition, or order issued by the
DOLE Secretary and the NLRC under Art. 263 of the Labor Code]; or
(6) [when it] is contrary to an existing agreement, such as a no-strike clause or conclusive arbitration clause.18
With the foregoing parameters as guide and the following grounds as basis, we hold that the Union is liable for
conducting an illegal strike for the following reasons:
First, the Union's violation of the Hotel's Grooming Standards was clearly a deliberate and concerted action to
undermine the authority of and to embarrass the Hotel and was, therefore, not a protected action. The
appearances of the Hotel employees directly reflect the character and well-being of the Hotel, being a five-star
hotel that provides service to top-notch clients. Being bald or having cropped hair per se does not evoke negative
or unpleasant feelings. The reality that a substantial number of employees assigned to the food and beverage
outlets of the Hotel with full heads of hair suddenly decided to come to work bald-headed or with cropped hair,
however, suggests that something is amiss and insinuates a sense that something out of the ordinary is afoot.
Obviously, the Hotel does not need to advertise its labor problems with its clients. It can be gleaned from the
records before us that the Union officers and members deliberately and in apparent concert shaved their heads or
cropped their hair. This was shown by the fact that after coming to work on January 18, 2002, some Union
members even had their heads shaved or their hair cropped at the Union office in the Hotel's basement. Clearly,
the decision to violate the company rule on grooming was designed and calculated to place the Hotel
management on its heels and to force it to agree to the Union's proposals.
In view of the Union's collaborative effort to violate the Hotel's Grooming Standards, it succeeded in forcing the
Hotel to choose between allowing its inappropriately hair styled employees to continue working, to the detriment of
its reputation, or to refuse them work, even if it had to cease operations in affected departments or service units,
which in either way would disrupt the operations of the Hotel. This Court is of the opinion, therefore, that the act of
the Union was not merely an expression of their grievance or displeasure but, indeed, a calibrated and calculated
act designed to inflict serious damage to the Hotel's finances or its reputation. Thus, we hold that the Union's
concerted violation of the Hotel's Grooming Standards which resulted in the temporary cessation and disruption of
the Hotel's operations is an unprotected act and should be considered as an illegal strike.
Second, the Union's concerted action which disrupted the Hotel's operations clearly violated the CBA's "No Strike,
No Lockout" provision, which reads: “The Union agrees that there shall be no strikes, walkouts, stoppage or slow-
down of work, boycott, refusal to handle accounts, picketing, sit-down strikes, sympathy strikes or any other form
of interference and/or interruptions with any of the normal operations of the HOTEL during the life of this
Agreement.” The facts are clear that the strike arose out of a bargaining deadlock in the CBA negotiations with the
Hotel. The concerted action is an economic strike upon which the afore-quoted "no strike/work stoppage and
lockout" prohibition is squarely applicable and legally binding.19
Third, the Union officers and members' concerted action to shave their heads and crop their hair not only violated
the Hotel's Grooming Standards but also violated the Union's duty and responsibility to bargain in good faith. By
shaving their heads and cropping their hair, the Union officers and members violated then Section 6, Rule XIII of
the Implementing Rules of Book V of the Labor Code.20 This rule prohibits the commission of any act which will
Angeles| Bajana | Balladares | Brillantes | Briones | Cabansag | Callanta | Chua | David|

De Leon | Gomez | Lopez | Macalino | Nostratis | Padilla | Reynon | Santos | Tan |Velasco
4E / 4F - 2018-2019
Page 493 of 920
LABOR REVIEW DIGEST
Atty. Joyrich Golangco

disrupt or impede the early settlement of the labor disputes that are under conciliation. Since the bargaining
deadlock is being conciliated by the NCMB, the Union's action to have their officers and members' heads shaved
was manifestly calculated to antagonize and embarrass the Hotel management and in doing so effectively
disrupted the operations of the Hotel and violated their duty to bargain collectively in good faith.
Fourth, the Union failed to observe the mandatory 30-day cooling-off period and the seven-day strike
ban before it conducted the strike on January 18, 2002. The NLRC correctly held that the Union failed to observe
the mandatory periods before conducting or holding a strike. Records reveal that the Union filed its Notice of
Strike on the ground of bargaining deadlock on December 20, 2001. The 30-day cooling-off period should have
been until January 19, 2002. On top of that, the strike vote was held on January 14, 2002 and was submitted to
the NCMB only on January 18, 2002; therefore, the 7-day strike ban should have prevented them from holding a
strike until January 25, 2002. The concerted action committed by the Union on January 18, 2002 which resulted in
the disruption of the Hotel's operations clearly violated the above-stated mandatory periods.
Last, the Union committed illegal acts in the conduct of its strike. The NLRC ruled that the strike was illegal since,
as shown by the pictures21 presented by the Hotel, the Union officers and members formed human barricades
and obstructed the driveway of the Hotel. There is no merit in the Union's argument that it was not its members
but the Hotel's security guards and the police officers who blocked the driveway, as it can be seen that the guards
and/or police officers were just trying to secure the entrance to the Hotel. The pictures clearly demonstrate the
tense and highly explosive situation brought about by the strikers' presence in the Hotel's driveway.
What then are the consequent liabilities of the Union officers and members for their participation in the
illegal strike?
Regarding the Union officers and members' liabilities for their participation in the illegal picket and strike, Art.
264(a), paragraph 3 of the Labor Code provides that "[a]ny union officer who knowingly participates in an
illegal strike and any worker or union officer who knowingly participates in the commission of illegal acts
during a strike may be declared to have lost his employment status x x x." The law makes a distinction
between union officers and mere union members. Union officers may be validly terminated from employment for
their participation in an illegal strike, while union members have to participate in and commit illegal acts for them
to lose their employment status. Thus, it is necessary for the company to adduce proof of the participation of the
striking employees in the commission of illegal acts during the strikes.
Clearly, the 29 Union officers may be dismissed pursuant to Art. 264(a), par. 3 of the Labor Code which imposes
the penalty of dismissal on "any union officer who knowingly participates in an illegal strike." We, however,
are of the opinion that there is room for leniency with respect to the Union members. It is pertinent to note that the
Hotel was able to prove before the NLRC that the strikers blocked the ingress to and egress from the Hotel. But it
is quite apparent that the Hotel failed to specifically point out the participation of each of the Union members in the
commission of illegal acts during the picket and the strike. For this lapse in judgment or diligence, we are
constrained to reinstate the 61 Union members.
Further, we held in one case that union members who participated in an illegal strike but were not identified to
have committed illegal acts are entitled to be reinstated to their former positions but without backwages. We then
held in G & S Transport Corporation v. Infante:
With respect to backwages, the principle of a "fair day's wage for a fair day's labor" remains as the basic factor in
determining the award thereof. If there is no work performed by the employee there can be no wage or pay
unless, of course, the laborer was able, willing and ready to work but was illegally locked out, suspended or
dismissed or otherwise illegally prevented from working. While it was found that respondents expressed their
intention to report back to work, the latter exception cannot apply in this case. In Philippine Marine Officer's Guild
v. Compañia Maritima, as affirmed in Philippine Diamond Hotel and Resort v. Manila Diamond Hotel Employees
Union, the Court stressed that for this exception to apply, it is required that the strike be legal, a situation that does
not obtain in the case at bar.28
In this light, we stand by our recent rulings and reinstate the 61 Union members without backwages.
————————————————————————

Angeles| Bajana | Balladares | Brillantes | Briones | Cabansag | Callanta | Chua | David|

De Leon | Gomez | Lopez | Macalino | Nostratis | Padilla | Reynon | Santos | Tan |Velasco
4E / 4F - 2018-2019
Page 494 of 920
LABOR REVIEW DIGEST
Atty. Joyrich Golangco

4. Jackbilt Industries v. JEWU-NAFLU-KMU


G.R. Nos. 171618-19, March 20, 2009
Corona J.

DOCTRINE:

The principle of conclusiveness of judgment, embodied in Section 47(c), Rule 39 of the Rules of Court, holds that
the parties to a case are bound by the findings in a previous judgment with respect to matters actually raised and
adjudged therein.

FACTS:
Due to the Asian economic crisis on the construction industry beginning 1997, petitioner Jackbilt Industries, Inc.
decided to temporarily stop its business of producing concrete hollow blocks, compelling most of its employees to
go on leave for six months.

Respondent Jackbilt Employees Workers Union-NAFLU-KMU immediately protested the temporary shutdown.
The shutdown was allegedly motivated by anti-union sentiments.

Accordingly, on March 9, 1998, respondent went on strike. Its officers and members picketed petitioners main
gates and deliberately prevented persons and vehicles from going into and out of the compound.

On March 19, 1998, petitioner filed a petition for injunction with a prayer for the issuance of a TRO in the NLRC.
The NLRC granted the petition.

The reports of both the implementing officer and the investigating labor arbiter revealed, however, that respondent
union violated the April 14, 1998 order. Union members, on various occasions, stopped and inspected private
vehicles entering and exiting petitioners production facility. Thus, in a decision dated July 17, 1998, the NLRC
ordered the issuance of a writ of preliminary injunction.

Meanwhile, petitioner sent individual memoranda to the officers and members of respondent who participated in
the strike ordering them to explain why they should not be dismissed for committing illegal acts in the course of a
strike. Respondent repeatedly ignored petitioners memoranda. Thus, petitioner dismissed the concerned officers
and members and barred them from entering its premises effective June 1, 1998.

Aggrieved, respondent filed complaints for illegal lockout, runaway shop and damages, unfair labor practice,
illegal dismissal and attorneys fees, and refusal to bargain on behalf of its officers and members against petitioner
and its corporate officers. It argued that there was no basis for the temporary partial shutdown as it was
undertaken by petitioner to avoid its duty to bargain collectively.

LA/NLRC RULING:
In a decision dated October 15, 1999, the labor arbiter dismissed the complaints for illegal lockout and unfair
labor practice for lack of merit. However, because petitioner did not file a petition to declare the strike illegal before
terminating respondents officers and employees, it was found guilty of illegal dismissal.

On December 28, 2000, the NLRC, on appeal, modified the decision of the labor arbiter. It held that only petitioner
should be liable for monetary awards granted to respondents officers and members.

Both petitioner and respondent moved for reconsideration but they were denied for lack of merit.

Angeles| Bajana | Balladares | Brillantes | Briones | Cabansag | Callanta | Chua | David|

De Leon | Gomez | Lopez | Macalino | Nostratis | Padilla | Reynon | Santos | Tan |Velasco
4E / 4F - 2018-2019
Page 495 of 920
LABOR REVIEW DIGEST
Atty. Joyrich Golangco

Aggrieved, petitioner assailed the decision of the NLRC via a petition for certiorari in the CA.

CA Ruling:
The CA dismissed the petition but modified the December 28, 2000 decision of the NLRC. Because most of
affected employees were union members, the CA held that the temporary shutdown was moved by anti-union
sentiments. Petitioner was therefore guilty of unfair labor practice and, consequently, was ordered to pay
respondents officers and employees backwages from March 9, 1998 ( to October 4, 1998 and separation pay of
one month salary for every year of credited service.

Petitioner moved for reconsideration but it was denied.

ISSUE/S:
Whether or not the filing of a petition with the labor arbiter to declare a strike illegal is a
condition sine qua non for the valid termination of employees who commit an illegal act in
the course of such strike.

HELD:

• The principle of conclusiveness of judgment, embodied in Section 47(c), Rule 39 of the Rules of Court, holds
that the parties to a case are bound by the findings in a previous judgment with respect to matters actually raised
and adjudged therein.

Article 264(e) of the Labor Code prohibits any person engaged in picketing from obstructing the free ingress to
and egress from the employers premises. Since respondent was found in the July 17, 1998 decision of the NLRC
to have prevented the free entry into and exit of vehicles from petitioners compound, respondents officers and
employees clearly committed illegal acts in the course of the March 9, 1998 strike.

The use of unlawful means in the course of a strike renders such strike illegal. Therefore, pursuant to the principle
of conclusiveness of judgment, the March 9, 1998 strike was ipso facto illegal. The filing of a petition to
declare the strike illegal was thus unnecessary.

————————————————————————
5. NCMB PRIMER ON STRIKES AND LOCKOUTS
————————————————————————

Angeles| Bajana | Balladares | Brillantes | Briones | Cabansag | Callanta | Chua | David|

De Leon | Gomez | Lopez | Macalino | Nostratis | Padilla | Reynon | Santos | Tan |Velasco
4E / 4F - 2018-2019
Page 496 of 920
LABOR REVIEW DIGEST
Atty. Joyrich Golangco

6. AIRLINE PILOTS ASSOCIATION OF THE PHILIPPINES v. PHILIPPINE AIRLINES, INC.


G.R. No. 168382 Date: June 6, 2011.
Del Castillo, J.

Doctrine: There is no necessity to conduct a proceeding to determine the participants in the illegal strike or those
who refused to heed the return to work order because the ambiguity can be cured by reference to the body of the
decision and the pleadings filed.

FACTS:
ALPAP filed a notice of strike against respondent PAL with the DOLE claiming that PAL committed unfair
labor practice. DOLE Secretary assumed jurisdiction over the labor dispute. DOLE Secretary ordered that
all strikes and lockouts at the Philippine Airlines, Inc., whether actual or impending, are hereby strictly
prohibited. The parties are also enjoined from committing any act that may exacerbate the situation.
However, ALPAP went on strike. DOLE issue a return-to-work order on June 7, 1998. However, it was
only on June 26, 1998 when ALPAP officers and members reported back to work as shown in a
logbook signed by each of them. As a consequence, PAL refused to accept the returning pilots for their
failure to comply immediately with the return-to-work order. DOLE declared the strike conducted by
ALPAP illegal and pronouncing the loss of employment status of its officers and members who
participated in the strike in defiance of the June 7, 1998 return-to-work order. This was eventually
elevated to the SC and attained finality on August 29, 2002. Despite such, on January 13, 2003, ALPAP
filed before DOLE a Motion requesting to conduct an appropriate legal proceeding to determine who
among its officers and members should be reinstated or deemed to have lost their employment with
PAL for their actual participation in the strike. PAL, in its comment argued that the motions cannot
legally prosper since the DOLE Secretary has no authority to reopen or review a final judgment of the
Supreme Court. DOLE concludes that the case has indeed been resolved with finality by the highest
tribunal of the land, the Supreme Court. CA affirmed.

PETITION TO THE SC:

ISSUE/S:
Whether or not the CA committed grave mistake in declaring that the resolution of the DOLE has
already taken up and resolved the issue of who among the ALPAP members are deemed to have lost
their employment status

HELD:
NO. ALPAP seeks for a conduct of a proceeding to determine who among its members and officers
actually participated in the illegal strike because, it insists, the June 1, 1999 DOLE Resolution did not
make such determination. However, as correctly ruled by DOLE and affirmed by the CA, such
proceeding would entail a reopening of a final judgment which could not be permitted by this Court.
Settled in law is that once a decision has acquired finality, it becomes immutable and unalterable, thus
can no longer be modified in any respect. Subject to certain recognized exceptions, the principle of
immutability leaves the judgment undisturbed as nothing further can be done except to execute it.
True, the dispositive portion of the DOLE Resolution does not specifically enumerate the names of those
who actually participated in the strike but only mentions that those strikers who failed to heed the
return-to-work order are deemed to have lost their employment. This omission, however, cannot prevent
an effective execution of the decision. Any ambiguity may be clarified by reference primarily to the body
of the decision or supplementary to the pleadings previously filed in the case, as in this case. A review
of the records reveals that in NCMB NCR NS 12-514-97, the DOLE Secretary declared the ALPAP
officers and members to have lost their employment status based on either of two grounds, viz: their
participation in the illegal strike on June 5, 1998 or their defiance of the return-to-work order of the
Angeles| Bajana | Balladares | Brillantes | Briones | Cabansag | Callanta | Chua | David|

De Leon | Gomez | Lopez | Macalino | Nostratis | Padilla | Reynon | Santos | Tan |Velasco
4E / 4F - 2018-2019
Page 497 of 920
LABOR REVIEW DIGEST
Atty. Joyrich Golangco

DOLE Secretary. The records of the case unveil the names of each of these returning pilots. The
logbook with the heading Return To Work Compliance/ Returnees bears their individual signature
signifying their conformity that they were among those workers who returned to work only on June 26,
1998 or after the deadline imposed by DOLE.
————————————————————————

Angeles| Bajana | Balladares | Brillantes | Briones | Cabansag | Callanta | Chua | David|

De Leon | Gomez | Lopez | Macalino | Nostratis | Padilla | Reynon | Santos | Tan |Velasco
4E / 4F - 2018-2019
Page 498 of 920
LABOR REVIEW DIGEST
Atty. Joyrich Golangco

7. VCMC v. Yballe

Doctrine: The law makes a distinction between union members and union officers. A worker merely participating
in an illegal strike may not be terminated from employment. It is only when he commits illegal acts during a strike
that he may be declared to have lost employment status. In contrast, a union officer may be terminated from
employment for knowingly participating in an illegal strike or participates in the commission of illegal acts during a
strike.
FACTS: Respondents were hired as staff nurses and midwives by petitioner. NFL is the exclusive bargaining
representatives of the rank-and-file employees of MCCH. Nava, president of Nagkahiusang Mamumuno sa MCCH
(NAMA-MCCH-NFL) wrote Rev. Iyoy, MCCH Administrator, expressing the union’s desire to renew the CBA
attaching a statement of proposal. However, MCCHI returned the CBA proposal for Nava to secure first the
endorsement of the legal counsel of NFL as the official bargaining representative of MCCHI employees.
Meanwhile, Atty. Alforque informed MCCHI that the proposed CBA submitted by Nava was never referred to NFL
and that NFL has not authorized any other legal counsel or any person for collective bargaining negotiations.
Atty. Alforque suspended the union membership of Nava, et al for serious violation of the Constition and By-
Laws. The next day, several union members led by Nava and her group launched a series of mass actions such
as wearing black and red armbands/headbands, marching around the hospital premises and putting up placards,
posters and streamers. Atty. Alforque immediately disowned the concerted activities being carried out by union
members which are not sanctioned by NFL.
DOLE RO issued certifications stating that there is nothing in their records which shows that NAMA-MCCH- NFL
is a registered labor organization. MCCHI then sent individual notices to all union members asking them to submit
within 72 hours a written explanation why they should not be terminated for having supported the illegal concerted
activities of NAMA-MCCH-NFL which has no legal personality as per DOLE records.
Nava group refused to submit to investigation so MCCH sent termination letter to union leaders and other
members who participated in the strike and picketing activities. With the intensified atmosphere of violence and
animosity within the hospital premises as a result of continued protest activities by union members, MCCHI
suffered heavy losses due to low patient admission rates. The hospital’s suppliers also refused to make further
deliveries on credit. MCCHI filed a petition for injunction in the NLRC. Several complaints for illegal dismissal and
unfair labor practice were filed by the terminated employees against MCCHI.

LA RULING: LA dismissed the claim of unfair labor practice and illegal dismissal and declaring the termination of
the respondents as an offshoot of the illegal strike, but directed the Hospital to pay the respondents their
separation pay.

NLRC RULING: NLRC affirmed the decision of LA but deleted the award of separation pay.

CA RULING: CA initially affirmed decision of NLRC. But, on certiorari, CA reversed and set aside NLRC’s
decision.

ISSUE: Were the respondents illegally dismissed?

RULING: Yes. The law makes a distinction between union members and union officers. A worker merely
participating in an illegal strike may not be terminated from employment. It is only when he commits illegal acts
during a strike that he may be declared to have lost employment status. In contrast, a union officer may be
terminated from employment for knowingly participating in an illegal strike or participates in the commission of
illegal acts during a strike.
The CA found that respondents’ participation was limited to the wearing of armband and thus, declared
respondents’ termination as invalid in the absence of any evidence that they committed any illegal act during the
strike.

Angeles| Bajana | Balladares | Brillantes | Briones | Cabansag | Callanta | Chua | David|

De Leon | Gomez | Lopez | Macalino | Nostratis | Padilla | Reynon | Santos | Tan |Velasco
4E / 4F - 2018-2019
Page 499 of 920
LABOR REVIEW DIGEST
Atty. Joyrich Golangco

With respect to backwages, the principle of a "fair day’s wage for a fair day’s labor" remains as the basic factor in
determining the award thereof. If there is no work performed by the employee there can be no wage or pay
unless, of course, the laborer was able, willing and ready to work but was illegally locked out, suspended or
dismissed or otherwise illegally prevented from working.
The alternative relief for union members who were dismissed for having participated in an illegal strike is the
payment of separation pay in lieu of reinstatement under the following circumstances: (a) when reinstatement can
no longer be effected in view of the passage of a long period of time or because of the realities of the situation; (b)
reinstatement is inimical to the employer’s interest; (c) reinstatement is no longer feasible; (d) reinstatement does
not serve the best interests of the parties involved; (e) the employer is prejudiced by the workers’ continued
employment; (f) facts that make execution unjust or inequitable have supervened; or (g) strained relations
between the employer and employee.
Considering that 15 years had lapsed from the onset of this labor dispute, and in view of strained relations that
ensued, in addition to the reality of replacements already hired by the hospital which had apparently recovered
from its huge losses, and with many of the petitioners either employed elsewhere, already old and sickly, or
otherwise incapacitated, separation pay without back wages is the appropriate relief.

Angeles| Bajana | Balladares | Brillantes | Briones | Cabansag | Callanta | Chua | David|

De Leon | Gomez | Lopez | Macalino | Nostratis | Padilla | Reynon | Santos | Tan |Velasco
4E / 4F - 2018-2019
Page 500 of 920
LABOR REVIEW DIGEST
Atty. Joyrich Golangco

7. DANILO ESCARIO,ET. AL. v. NLRC


G.R. No. 160302; September 27, 2010

FACTS: In the morning of March 13, 1993, all the officers and some 200 members of the Union walked out of
PINAs premises and proceeded to the barangay office to show support for Juanito Caete, an officer of the Union
charged with oral defamation by Aurora Manor, PINAs personnel manager, and Yolanda Fabella, Manors’
secretary.
INA filed a complaint for unfair labor practice (ULP) and damages. LA declared the incident an illegal walkout
constituting ULP and that all the Unions officers, except Caete, had thereby lost their employment. However, the
Union thereafter filed a notice of strike, claiming that PINA was guilty of union busting through the constructive
dismissal of its officers. The strike was thereafter held. PINA retaliated by charging the petitioners with ULP and
abandonment of work.
LA the again declared the strike to be illegal.
ISSUE: Are petitioners entitled to full backwages from the date of dismissal until the date of actual reinstatement
due to their not being found to have abandoned their jobs in an illegal strike?
RULING: No. The petitioners contend that they are entitled to full backwages by virtue of their reinstatement, and
submit that applicable to their situation is Article 279, not the third paragraph of Article 264(a), both of the Labor
Code.
We do not agree with the petitioners. Article 279 provides:
By its use of the phrase unjustly dismissed, Article 279 refers to a dismissal that is unjustly done, that is, the
employer dismisses the employee without observing due process, either substantive or procedural. Substantive
due process requires the attendance of any of the just or authorized causes for terminating an employee as
provided under Article 278 (termination by employer), or Article 283 (closure of establishment and reduction of
personnel), or Article 284 (disease as ground for termination), all of the Labor Code; while procedural due process
demands compliance with the twin-notice requirement.[17]
In contrast, the third paragraph of Article 264(a) states:
Art. 264. Prohibited activities. (a) xxx
Any worker whose employment has been terminated as a consequence of an unlawful lockout shall be entitled to
reinstatement with full backwages. Any union officer who knowingly participates in an illegal strike and any worker
or union officer who knowingly participates in the commission of illegal acts during a strike may be declared to
have lost his employment status; Provided, That mere participation of a worker in a lawful strike.
————————————————————————

Angeles| Bajana | Balladares | Brillantes | Briones | Cabansag | Callanta | Chua | David|

De Leon | Gomez | Lopez | Macalino | Nostratis | Padilla | Reynon | Santos | Tan |Velasco
4E / 4F - 2018-2019
Page 501 of 920
LABOR REVIEW DIGEST
Atty. Joyrich Golangco

8. TABANGAO SHELL REFINERY EMPLOYEES ASSOC. vs. PILIPINAS SHELL


G.R. No. 170007 April 07, 2014
Ponente: Leonardo-De Castro, J.
Doctrine: The power of the Secretary of Labor and Employment to assume jurisdiction over this dispute includes
and extends to all questions and controversies arising from the said dispute, such as, but not limited to the union's
allegation of bad faith bargaining. It also includes and extends to the various unresolved provisions of the new
CBA such as compensation, particularly the matter of annual wage increase or yearly lump sum payment in lieu of
such wage increase, whether or not there was deadlock in the negotiations.| As there is already an existing
controversy on the matter of wage increase, the Secretary of Labor and Employment need not wait for a deadlock
in the negotiations to take cognizance of the matter. That is the significance of the power of the Secretary of Labor
and Employment under Article 263 (g) of the Labor Code to assume jurisdiction over a labor dispute causing or
likely to cause a strike or lockout in an industry indispensable to the national interest.

FACTS: In anticipation of the expiration on April 30, 2004 of the 2001-2004 Collective Bargaining Agreement
(CBA) between the petitioner and the respondent Pilipinas ShellPetroleum Corporation, the parties started
negotiations for a new CBA. The union proposed a 20% annual across-the-board basic salary increase for the
next three years that would be covered by the new CBA. In lieu of the annual salary increases, the company
made a counter-proposal to grant all covered employees a lump sum amount of P80,000.00 yearly for the three-
year period of the new CBA.||| The union requested the company to present its counter-proposal in full detail,
similar to the presentation by the union of its economic proposal. The company explained that the lump sum
amount was based on its affordability for the corporation, the then current salary levels of the members of the
union relative to the industry, and the then current total pay and benefits package of the employees.||| the union
lowered its proposal to 12% annual across-the-board increase for the next three years. For its part, the company
increased its counter-proposal to a yearly lump sum payment of P88,000.00 for the next three years. |||
The union requested financial data for the manufacturing class of business in the Philippines. It also requested
justification for the company's counter-offer. In response, the company stated that financial measures
for Tabangao were available in the refinery scorecard regularly cascaded by the management to the employees.
The company reiterated that its counter-offer is based on its affordability for the company, comparison with the
then existing wage levels of allied industry, and the then existing total pay and benefits package of the
employees.||| However, the union remained unconvinced and asked for additional documents to justify the
company's counter-offer.|||
the company proposed the declaration of a deadlock and recommended that the help of a third party be sought.
The union replied that they would formally answer the proposal of the company a day after the signing of the
official minutes of the meeting. On that same day, however, the union filed a Notice of Strike in the National
Conciliation and Mediation Board (NCMB), alleging bad faith bargaining on the part of the company. The NCMB
immediately summoned the parties for the mandatory conciliation-mediation proceedings but the parties failed to
reach an amicable settlement.||| during the cooling off period, the union conducted the necessary strike vote. The
members of the union, who participated in the voting, unanimously voted for the holding of a strike. Upon being
aware of this development, the company filed a Petition for Assumption of Jurisdiction with the Secretary of Labor
and Employment. the then Secretary of Labor and Employment, Patricia Sto. Tomas, granted the petition of the
company. The Secretary of Labor and Employment took notice of the Notice of Strike filed by the union in the
NCMB which charged the company with unfair labor practice consisting of bad faith in bargaining negotiations.
The Secretary of Labor and Employment also found that the intended strike would likely affect the company's
capacity to provide petroleum products to the company's various clientele, including the transportation sector, the
energy sector, and the manufacturing and industrial sectors.|||
Convinced that such a strike would have adverse consequences on the national economy, the Secretary of Labor
and Employment ruled that the labor dispute between the parties would cause or likely to cause a strike in an
industry indispensable to the national interest.|||
The union filed a Petition for Certiorari with the Court of Appeals.

Angeles| Bajana | Balladares | Brillantes | Briones | Cabansag | Callanta | Chua | David|

De Leon | Gomez | Lopez | Macalino | Nostratis | Padilla | Reynon | Santos | Tan |Velasco
4E / 4F - 2018-2019
Page 502 of 920
LABOR REVIEW DIGEST
Atty. Joyrich Golangco

SOLE DECISION: The Secretary of Labor and Employment held that there was already deadlock although the
ground for the first Notice of Strike was unfair labor practice for bargaining in bad faith. the Secretary of Labor and
Employment ruled that the circumstances — 41 CBA meetings showing "reasonable efforts at good faith
bargaining" without arriving at a CBA — show that there was effectively a bargaining deadlock between the
parties. F
Petitioner’s Contention: Petitioners insist that the corporation is guilty of unfair labor practice through bad faith
bargaining. According to the union, bad faith bargaining and a CBA deadlock cannot legally co-exist because an
impasse in negotiations can only exist on the premise that both parties are bargaining in good faith. Besides, there
could have been no deadlock between the parties as the union had not given its consent to it, pursuant to item 8
of the ground rules governing the parties' negotiations which required mutual consent for a declaration of
deadlock. The union also posits that its filing of a CBA deadlock case against the company was a separate and
distinct case and not an offshoot of the company's unfair labor practice through bargaining in bad faith.|||

ISSUE: WHETHER OR NOT THE COMPANY WAS GUILTY OF BARGAINING IN BAD FAITH.

HELD: NO.
Pursuant to Article 263 (i) of the Labor Code, therefore, the Decision dated June 8, 2005 of the Secretary of Labor
and Employment became final and executory after the lapse of the period provided under the said provision.
Moreover, neither party further questioned the Decision dated June 8, 2005 of the Secretary of Labor and
Employment.
The Decision dated June 8, 2005 of the Secretary of Labor and Employment already considered and ruled upon
the issues being raised by the union in this petition. In particular, the said Decision already passed upon the issue
of whether there was already an existing deadlock between the union and the company when the Secretary of
Labor and Employment assumed jurisdiction over their labor dispute. The said Decision also answered the issue
of whether the company was guilty of bargaining in bad faith. As the Decision dated June 8, 2005 of the Secretary
of Labor and Employment already settled the said issues with finality, the union cannot once again raise those
issues in this Court through this petition without violating the principle of res judicata, particularly in the concept of
conclusiveness of judgment.
While the purpose of collective bargaining is the reaching of an agreement between the employer and the
employee's union resulting in a binding contract between the parties, the failure to reach an agreement after
negotiations continued for a reasonable period does not mean lack of good faith. The laws invite and contemplate
a collective bargaining contract but do not compel one.|||
More importantly, the union only caused confusion in the proceedings before the Secretary of Labor and
Employment when it questioned the latter's assumption of jurisdiction over the labor dispute between the union
and the company on the ground that the "Secretary erred in assuming jurisdiction over the 'CBA' case when it
[was] not the subject matter of the notice of strike" because the case was "all about 'ULP' in the form of bad faith
bargaining." For the union, the Secretary of Labor and Employment should not have touched the issue of the CBA
as there was no CBA deadlock at that time, and should have limited the assumption of jurisdiction to the charge of
unfair labor practice for bargaining in bad faith||. The union is wrong. As discussed above, there was already an
actual existing deadlock between the parties. What was lacking was the formal recognition of the existence of
such a deadlock because the union refused a declaration of deadlock.
————————————————————————

Angeles| Bajana | Balladares | Brillantes | Briones | Cabansag | Callanta | Chua | David|

De Leon | Gomez | Lopez | Macalino | Nostratis | Padilla | Reynon | Santos | Tan |Velasco
4E / 4F - 2018-2019
Page 503 of 920
LABOR REVIEW DIGEST
Atty. Joyrich Golangco

9. ASIA BREWERY v. TPMA (TUNAY NA PAGKAKAISA NG MGA MANGGAGAWA SA ASIA)


G.R. Nos. 171594-96. September 18, 2013

Facts:
- Respondent union and petitioner corporation had been negotiating for a new collective bargaining agreement
(CBA) for the years 2003-2006 since the old CBA expired last July 2003.
- After about 18 sessions or negotiations, the parties were still unable to reconcile their differences on their
respective positions on most items, particularly on wages and other economic benefits.
- the union declared a deadlock. Respondent union filed a notice of strike with the National Conciliation and
Mediation Board (NCMB). However, the parties did not come to terms even before the NCMB.
- union conducted a strike vote. Out of the 840 union members, 768 voted in favor of holding a strike.
- corporation then petitioned the Secretary of the DOLE to assume jurisdiction over the parties’ labor dispute,
invoking Article 263 (g) of the Labor Code
- union filed before Court of Appeals a petition for injunction, which sought to enjoin the respondent Secretary of
Labor from assuming jurisdiction over the labor dispute
- DECISION OF SOLE, In Re: Assumption of Jurisdiction
o public respondent, through Undersecretary/ Acting Secretary Imson, issued an order assuming jurisdiction over
the labor dispute
- union filed another petition for certiorari with Court of Appeals imputing bad faith and grave abuse of discretion to
the Secretary of Labor.
- DECISION OF SOLE, In Re: labor dispute:
o Secretary of Labor Patricia Sto. Tomas resolved the deadlock between the parties.
- parties executed and signed the Collective Bargaining Agreement with a term from August 1, 2003 to July 31,
2006
- Union filed a petition for certiorari assailing the arbitral award and imputing grave abuse of discretion upon the
public respondent
- DECISION OF CA, IN Re: Certiorari
o CA rendered the first assailed Decision affirming with modification the arbitral award of the Secretary of Labor

ISSUE:
- Petitioner raises the issue specifically on the non-dismissal of the petition for the grant of the arbitral award
despite the lack of authority of those who instituted it

HELD:
- PETITION is WITHOUT MERIT.
- MEDYO RELATED LANG sa ARTICLE 278-279 of the LABOR CODE.
- The Secretary’s Certificate attached to the Petition in CA-G.R. SP No. 83168 stated that the union’s board of
directors held a special meeting on March 23, 2004 and unanimously passed a resolution authorizing Perez
to file a Petition before the CA to question the Secretary of Labor’s arbitral award.
- While petitioner corporation claims that the proper procedure for calling such a meeting was not followed,
it presented no proof to establish the same.
- Miranda, the union president who allegedly did not call for and preside over the said meeting, did not come out
to contest the validity of the aforesaid resolution or Secretary’s Certificate.
- Similarly, petitioner corporation’s claim that the aforesaid resolution was still ineffective at the time of the filing of
the subject Petition is unsubstantiated.
- A fair reading of the provisions which petitioner corporation cited in the union’s constitution and by-laws,
particularly Article VIII, Section 211 thereof, would show that the same refers to decisions of the board of directors
regarding the laws or rules that would govern the union, hence, the necessity of a two-week prior notice to the
affected parties before they become effective.

Angeles| Bajana | Balladares | Brillantes | Briones | Cabansag | Callanta | Chua | David|

De Leon | Gomez | Lopez | Macalino | Nostratis | Padilla | Reynon | Santos | Tan |Velasco
4E / 4F - 2018-2019
Page 504 of 920
LABOR REVIEW DIGEST
Atty. Joyrich Golangco

- These provisions have not been shown to apply to resolutions granting authority to individuals to
represent the union in court cases.
- Besides, even if we assume that these provisions in the union’s constitution and by-laws apply to the subject
resolution, the continuing silence of the union, from the time of its adoption to the filing of the Petition with the
CA and up to this point in these proceedings, would indicate that such defect, if at all present, in the authority
of Perez to file the subject Petition, was impliedly ratified by respondent union itself.
- As to the two labor cases allegedly divesting Perez of the authority to file the subject Petition, an examination of
the same would show that they did not affect the legal capacity of Perez to file the subject Petition.
- The first labor case (i.e., RO400-0407-AU-002,12 RO400-0409-AU-006,13 and RO400-0412-AU-00114)
involved the move of Perez and other union members to amend the union’s Constitution and By-Laws in order to
include a provision on recall elections and to conduct a recall elections
- In that case, the Med-Arbiter, in his January 25, 2005 Order, ruled that the amendment sought to be
introduced was not validly ratified by the requisite two-thirds vote from the union membership. As a result,
the recall elections held on June 26, 2004 was annulled.
- The second labor case (i.e., NLRC NCR CC No. 000282-0417 and NLRC-RAB IV-12-20200-04-L18) involved
the strike staged by Perez and other union members on October 4, 2004. There, the National Labor Relations
Commission, in its March 2006 Decision, ruled that the strike was illegal and, as a consequence, Perez and
the other union members were declared to have lost their employment status.
- These two labor cases had no bearing on the legal capacity of Perez to represent the union in CA-G.R. SP
No. 83168 because
o (1) they did not nullify the authority granted to Perez in the March 23, 2004 resolution of the union’s board of
directors to file the subject Petition, and
o (2) the material facts of these cases occurred and the Decisions thereon were rendered after the subject
Petition was already filed with the CA on April 1, 2004.

————————————————————————
10. DO 40-G-3 series of 2010
————————————————————————
11. DO 40-H-3 series of 2013
————————————————————————
12. Operation Guidelines of DO 40-G-3 series of 2010
————————————————————————

Angeles| Bajana | Balladares | Brillantes | Briones | Cabansag | Callanta | Chua | David|

De Leon | Gomez | Lopez | Macalino | Nostratis | Padilla | Reynon | Santos | Tan |Velasco
4E / 4F - 2018-2019
Page 505 of 920
LABOR REVIEW DIGEST
Atty. Joyrich Golangco

13. Escario et. al. v. NLRC


GR No: 160302 Date: September 27, 2010
Ponente: BERSAMIN, J.

Doctrine:
Conformably with the long honored principle of a fair days wage for a fair days labor, employees dismissed for
joining an illegal strike are not entitled to backwages for the period of the strike even if they are reinstated by
virtue of their being merely members of the striking union who did not commit any illegal act during the strike.

FACTS:
The petitioners were among the regular employees of respondent Pinakamasarap Corporation (PINA), a
corporation engaged in manufacturing and selling food seasoning. They were members of petitioner Malayang
Samahan ng mga Manggagawa sa Balanced Foods (Union).
At 8:30 in the morning of March 13, 1993, all the officers and some 200 members of the Union walked out of
PINAs premises and proceeded to the barangay office to show support for an officer of the Union charged with
oral defamation by Aurora Manor, PINAs personnel manager, and Yolanda Fabella, Manor’s secretary.
As a result of the walkout, PINA preventively suspended all officers of the Union because of the March 13, 1993
incident. PINA terminated the officers of the Union after a month.

On April 14, 1993, PINA filed a complaint for unfair labor practice (ULP) and damages. LA ruled that the March
13, 1993 incident was an illegal walkout constituting ULP.
On April 28, 1993, the Union filed a notice of strike, claiming that PINA was guilty of union busting through the
constructive dismissal of its officers and held a strike.
PINA retaliated by charging the petitioners with ULP and abandonment of work, stating that they had violated
provisions on strike of the collective bargaining agreement (CBA).
LA ruled that the strike was illegal.

NLRC Ruling:
On appeal, the NLRC sustained the finding that the strike was illegal, but reversed the LAs ruling that there was
abandonment. NLRC ordered the reinstatement of the subject employees to their former positions but without
backwages.

CA RULING:
Petitioners claimed that the NLRC gravely abused its discretion in not awarding backwages pursuant to Article
279 of the Labor Code, and in not declaring their strike as a good faith strike.
CA applied the third paragraph of Article 264(a) instead of Article 279 of the Labor Code, explaining that the only
instance under Article 264 when a dismissed employee would be reinstated with full backwages was when he was
dismissed by reason of an illegal lockout.

APPEAL TO THE SC:


Petitioner’s contention:
The petitioners contend that they are entitled to full backwages by virtue of their reinstatement, and submit that
applicable to their situation is Article 279, not the third paragraph of Article 264(a), both of the Labor Code.

ISSUE/S:
1. Whether the petitioners are entitled to full backwages from the date of dismissal until the date of actual
reinstatement due to their not being found to have abandoned their jobs

Angeles| Bajana | Balladares | Brillantes | Briones | Cabansag | Callanta | Chua | David|

De Leon | Gomez | Lopez | Macalino | Nostratis | Padilla | Reynon | Santos | Tan |Velasco
4E / 4F - 2018-2019
Page 506 of 920
LABOR REVIEW DIGEST
Atty. Joyrich Golangco

HELD:
NO. As a general rule, backwages are granted to indemnify a dismissed employee for his loss of earnings during
the whole period that he is out of his job. Considering that an illegally dismissed employee is not deemed to have
left his employment, he is entitled to all the rights and privileges that accrue to him from the employment.
However, backwages are not granted to employees participating in an illegal strike simply accords with the reality
that they do not render work for the employer during the period of the illegal strike. If there is no work performed
by the employee there can be no wage or pay unless, of course, the laborer was able, willing and ready to work
but was illegally locked out, suspended or dismissed or otherwise illegally prevented from working. In this case,
petitioners herein do not deny their participation in the strike. As such, they did not suffer any loss of earnings
during their absence from work. Their reinstatement sans backwages is in order, to conform to the policy of a fair
days wage for a fair days labor.
————————————————————————

Angeles| Bajana | Balladares | Brillantes | Briones | Cabansag | Callanta | Chua | David|

De Leon | Gomez | Lopez | Macalino | Nostratis | Padilla | Reynon | Santos | Tan |Velasco
4E / 4F - 2018-2019
Page 507 of 920
LABOR REVIEW DIGEST
Atty. Joyrich Golangco

14. UNIVERSITY OF SAN AGUSTIN EMPLOYEES UNION VS. CA


G.R. No. 169632 Date: March 28, 2006
Ponente: GARCIA, J.

DOCTRINE:
Art. 263 of LC, “such assumption or certification of the SOLE shall have the effect of automatically enjoining the
intended or impending strike or lockout as specified in the assumption or certification order. If one has already
taken place at the time of assumption or certification, all striking or locked out employees shall immediately return
to work and the employer shall immediately resume operations and readmit all workers under the same terms and
conditions prevailing before the strike or lockout.” The phrase "immediately return to work" indicates an almost
instantaneous or automatic compliance for a striker to return to work once an AJO has been duly served.

FACTS:
The petitioner Union and respondent University agreed on a 5-year CBA, the economic provisions of which
are effective for 3 years. After the lapse of 3 years, the parties negotiated on the economic provisions but did not
agree on the terms during the remaining 2 years of the CBA and beyond. They did not agree on the computation
of tuition incremental proceeds (TIP) that shall be the basis for the increase of salaries. Later, the Union declared
a bargaining deadlock and thereafter filed a Notice of Strike with the NCMB. It was opposed by the University
through a Motion to Strike-out the Notice of Strike and Refer the Dispute to Voluntary Arbitration on the ground
that the CBA contained a "no-strike, no-lockout" provision, and a grievance machinery for settling disputes,
including a voluntary arbitration mechanism should the grievance machinery fail to settle the dispute. The NCMB
failed to resolve the Motion. Hence, both parties made a joint request for the Secretary of Labor (SOLE) to
assume jurisdiction over the dispute.

After the Secretary of Labor assumed jurisdiction, any strike or lockout was strictly enjoined. But the one day after,
and on the same day that the Assumption of Jurisdiction Order (AJO) was supposedly served to both parties, the
Union staged a strike. Union members refused to receive a copy of the AJO assailing that only the Union
President is authorized to receive the same. The Union filed a Petition Declare Illegal Strike and Loss of
Employment Status of the striking employees, which Petition was filed at the NLRC. Such Petition was later on
consolidated with the case pending before the Secretary.

SOLE RULING:
Dismissed the petition to declare illegal strike after resolving the economic issues.

CA RULING:
Reversed the SOLE decision. It directed the parties to refer the economic issues of the CBA to voluntary
arbitration as they were proper subject of the grievance machinery embodied in the parties' CBA. The strike
conducted by Union was illegal and its officers were deemed to have lost their employment status.

ISSUES:
(1) Whether the strike conducted by the petitioner Union was illegal.

(2) Whether the issue on the formula in computing the TIP share of the employees is a labor dispute that should
have been forwarded to Voluntary Arbitration.

RULING:

(1) YES. Under Art. 263 of LC:

Angeles| Bajana | Balladares | Brillantes | Briones | Cabansag | Callanta | Chua | David|

De Leon | Gomez | Lopez | Macalino | Nostratis | Padilla | Reynon | Santos | Tan |Velasco
4E / 4F - 2018-2019
Page 508 of 920
LABOR REVIEW DIGEST
Atty. Joyrich Golangco

Such assumption or certification of the SOLE shall have the effect of automatically enjoining the intended or
impending strike or lockout as specified in the assumption or certification order. If one has already taken place at
the time of assumption or certification, all striking or locked out employees shall immediately return to work and
the employer shall immediately resume operations and readmit all workers under the same terms and conditions
prevailing before the strike or lockout.

The phrase "immediately return to work" indicates an almost instantaneous or automatic compliance for a striker
to return to work once an AJO has been duly served. Therefore, the act of the striking employees is violative of
the foregoing provision.

(2) YES. Economic benefits, which included the issue on the formula in computing the TIP share of the
employees, is one that arises from the interpretation or implementation of the CBA, and these matters should be
referred to a Voluntary Arbitrator, as provided in Art. 261 and 262 of the Labor Code. The peculiar facts of the
instant case show that the University was deprived of a remedy that would have enjoined the Union strike and
was left without any recourse except to invoke the jurisdiction of the SOLE.
————————————————————————

Angeles| Bajana | Balladares | Brillantes | Briones | Cabansag | Callanta | Chua | David|

De Leon | Gomez | Lopez | Macalino | Nostratis | Padilla | Reynon | Santos | Tan |Velasco
4E / 4F - 2018-2019
Page 509 of 920
LABOR REVIEW DIGEST
Atty. Joyrich Golangco

15. PHILIPPINE DIAMOND HOTEL AND RESORT, INC. (MANILA DIAMOND HOTEL) V. MANILA DIAMOND
HOTEL EMPLOYEES UNION
GR No.: 158075 Date: June 30, 2006
Ponente: CARPIO MORALES, J.:

Doctrine: Even if the purpose of a strike is valid, the strike may still be held illegal where the means employed are
illegal. Thus, the employment of violence, intimidation, restraint or coercion in carrying out concerted activities
which are injurious to the rights to property renders a strike illegal. And so is picketing or the obstruction to the
free use of property or the comfortable enjoyment of life or property, when accompanied by intimidation, threats,
violence, and coercion as to constitute nuisance

FACTS:
The Court of Appeals, by the assailed decision of November 21, 2002, declared the strike staged by respondent,
Manila Diamond Hotel Employees Union (the union), illegal and its officers to have lost their employment status. It
ordered, however, among other things, the reinstatement and payment of backwages to its members.

On November 11, 1996, the union, which was registered on August 19, 1996 before the Department of Labor and
Employment (DOLE) filed a Petition for Certification Election before the DOLE-National Capital Region (NCR)
seeking certification as the exclusive bargaining representative of its members.

The DOLE-NCR denied the unions petition as it failed to comply with legal requirements, specifically Section 2,
Rule V, Book V of the Rules and Regulations Implementing the Labor Code, and was seen to fragment the
employees of petitioner. Through its president Kimpo, the union later notified petitioner of its intention to negotiate,
by Notice to Bargain, a Collective Bargaining Agreement (CBA) for its members.

Acting on the notice, the Hotel, through its Human Resource Development Manager Mary Anne Mangalindan,
advised the union that since it was not certified by the DOLE as the exclusive bargaining agent, it could not be
recognized as such.

The union clarified that it sought to bargain for its members only, and declared that [the
Hotels] refusal to bargain [would prompt] the union to engage in concerted activities to protect and assert its rights
under the Labor Code. The union went on to file a Notice of Strike on September 29, 1997 with the National
Conciliation and Mediation Board (NCMB) due to unfair labor practice (ULP) in that the Hotel refused to bargain
with it and the rank-and-file employees were being harassed and prevented from joining it.

Conciliation conferences were immediately conducted by the NCMB on October 6, 13, and 20, 1997 during which
the union insisted on the adoption of a CBA for its members.

In the meantime, or on or about November 7, 1997, Kimpo filed before the Arbitration Branch a complaint for ULP
against petitioner. More conferences took place between petitioner and the union before the NCMB.

In the early morning of November 29, 1997, however, the union suddenly went on strike. The following day, the
National Union of Workers in the Hotel, Restaurant and Allied Industries (NUWHRAIN) joined the strike and
openly extended its support to the union. At about this time, Hotel supervisors Vicente T. Agustin (Agustin) and
Rowena Junio (Rowena) failed to report for work and were, along with another supervisor, Mary Grace U. de Leon
(Mary Grace), seen participating in and supporting the strike.

Petitioner thus filed on December 1, 1997 a petition for injunction before the National Labor Relations Commission
(NLRC) to enjoin further commission of illegal acts by the strikers.

Angeles| Bajana | Balladares | Brillantes | Briones | Cabansag | Callanta | Chua | David|

De Leon | Gomez | Lopez | Macalino | Nostratis | Padilla | Reynon | Santos | Tan |Velasco
4E / 4F - 2018-2019
Page 510 of 920
LABOR REVIEW DIGEST
Atty. Joyrich Golangco

Mary Grace, who was directed to explain her participation in the strike, alleged that she was merely trying to
pacify the group. Petitioner, finding her explanation arrogant and unsatisfactory as her active participation in the
strike was confirmed by an eye witness, terminated her services, by communication sent on December 9, 1997,
drawing her to file a complaint for illegal dismissal against petitioner. Agustin, who was also terminated, filed a
similar complaint against the Hotel.

By Order of December 8, 1998, the NLRC thus issued a Temporary Restraining Order (TRO) directing the strikers
to immediately cease and desist from obstructing the free ingress and egress from the Hotel premises.

The service upon the strikers of the TRO notwithstanding, they refused to dismantle the tent they put up at the
employees entrance to the Hotel, prompting the Hotels security guards to, on December 10, 1997, dismantle the
same during which the strikers as well as the guards were hit by rocks coming from the direction of the
construction site at the nearby Land Bank Plaza, resulting to physical injuries to some of them. Despite the efforts
of the NCMB, which was joined by the Department of Tourism, to conciliate the parties, the same proved futile.

By Resolution of November 19, 1999, the NLRC declared that the strike was illegal and that the
union officers and members who were reinstated to the Hotels payroll were deemed to have lost their employment
status. And it dismissed the complaints filed by Mary Grace, Agustin, and Rowena as well as the union’s
complaint for ULP.

On appeal by the union, the Court of Appeals affirmed the NLRC Resolution dismissing the complaints of Mary
Grace, Agustin and Rowena and of the union. It modified the NLRC Resolution, however, by ordering
the reinstatement with back wages of union members. In so ruling, the appellate court noted that petitioner
failed to establish by convincing and substantial evidence that the union members who participated in the illegal
strike committed illegal acts, and although petitioner presented photographs of the striking employees, the strikers
who allegedly committed illegal acts were not named or identified.

ISSUE/S:
1. WON the strike is illegal.
2. WON there is substantial proof that the strikers-members of respondent who participated in the illegal strike
committed illegal acts.

HELD:
1. As did the NLRC and the Court of Appeals, this Court finds the strike illegal.
The union (hereafter referred to as respondent) is admittedly not the exclusive representative of the majority of the
employees of petitioner, hence, it could not demand from petitioner the right to bargain collectively in their behalf.

Respondent insists, however, that it could validly bargain in behalf of its members, relying on Article 242 of the
Labor Code. Respondent’s reliance on said article, a general provision on the rights of legitimate labor
organizations, is misplaced, for not every legitimate labor organization possesses the rights mentioned
therein. Article 242 (a) must be read in relation to above-quoted Article 255.

It bears noting that the goal of the DOLE is geered towards a single employer wide unit which is more to the
broader and greater benefit of the employees working force. The philosophy is to avoid fragmentation of the
bargaining unit so as to strengthen the employees bargaining power with the management. To veer away from
such goal would be contrary, inimical and repugnant to the objectives of a strong and dynamic unionism.

Petitioner’s refusal to bargain then with respondent cannot be considered a ULP to justify the staging of the strike.

Angeles| Bajana | Balladares | Brillantes | Briones | Cabansag | Callanta | Chua | David|

De Leon | Gomez | Lopez | Macalino | Nostratis | Padilla | Reynon | Santos | Tan |Velasco
4E / 4F - 2018-2019
Page 511 of 920
LABOR REVIEW DIGEST
Atty. Joyrich Golangco

The Court notes that respondent violated Article 264 which proscribes the staging of a strike on the ground of ULP
during the pendency of cases involving the same grounds for the strike.

Further, the photographs taken during the strike, as well as the Ocular Inspection Report of the NLRC
representative, show that the strikers, with the use of ropes and footed placards, blockaded the driveway to the
Hotels points of entrance and exit, making it burdensome for guests and prospective guests to enter the Hotel,
thus violating Article 264 (e) of the Labor Code.

Furthermore, the photographs indicate that indeed the strikers held noise barrage and threatened guests with
bodily harm.

Finally, the police reports mention about the strikers exploding of firecrackers, causing the guests to panic and
transfer to other areas of the Hotel. It is doctrinal that the exercise of the right of private sector employees to strike
is not absolute.

Even if the purpose of a strike is valid, the strike may still be held illegal where the means employed are
illegal. Thus, the employment of violence, intimidation, restraint or coercion in carrying out concerted activities
which are injurious to the rights to property renders a strike illegal. And so is picketing or the obstruction to the
free use of property or the comfortable enjoyment of life or property, when accompanied by intimidation, threats,
violence, and coercion as to constitute nuisance.

2. As the appellate court correctly held, the union officers should be dismissed for staging and participating in the
illegal strike, following paragraph 3, Article 264(a) of the Labor Code which provides that . . .[a]ny union
officer who knowingly participates in an illegal strike and any worker or union officer who knowingly participates
in the commission of illegal acts during strike may be declared to have lost his employment status . . .

An ordinary striking worker cannot, thus be dismissed for mere participation in an illegal strike. There must be
proof that he committed illegal acts during a strike, unlike a union officer who may be dismissed by mere
knowingly participating in an illegal strike and/or committing an illegal act during a strike.

The appellate court found no convincing and substantial proof, however, that the strikers-members of respondent
who participated in the illegal strike committed illegal acts.

This Court finds otherwise. As reflected above, the photographs show that some of the workers-strikers who
joined the strike indeed committed illegal acts blocking the free ingress to and egress from the Hotel, holding
noise barrage, threatening guests, and the like. The strikers were, in a list attached to petitioners Position
Paper filed with the NLRC, named.

The list failed to specifically identify the ones who actually committed illegal acts, however. Such being the case, a
remand of the case to the Labor Arbiter, through the NLRC, is in order for the purpose only of determining the
respective liabilities of the strikers listed by petitioner. Those proven to have committed illegal acts during the
course of the strike are deemed to have lost their employment, unless they have been readmitted by the Hotel,
whereas those not clearly shown to have committed illegal acts should be reinstated.
————————————————————————

Angeles| Bajana | Balladares | Brillantes | Briones | Cabansag | Callanta | Chua | David|

De Leon | Gomez | Lopez | Macalino | Nostratis | Padilla | Reynon | Santos | Tan |Velasco
4E / 4F - 2018-2019
Page 512 of 920
LABOR REVIEW DIGEST
Atty. Joyrich Golangco

16. Sukhothai Cuisine and Restaurant vs. Court of Appeals


GR No: 150437 Date: July 17, 2006
Ponente: Austria-Martinez, J.

Doctrine:
once jurisdiction over the labor dispute has been properly acquired by competent authority, that jurisdiction should
not be interfered with by the application of the coercive processes of a strike

strikes staged in violation of agreements providing for arbitration are illegal, since these agreements must be
strictly adhered to and respected if their ends are to be achieved
in case of alleged union busting, the three remaining requirements — notice, strike vote, and seven-day report
period — cannot be dispensed with

FACTS:
The PLAC Local 460 Sukhothai Restaurant Chapter (Union) filed a Notice of Strike with the NCMB on the ground
of ULPdue to alleged acts of harassment, fault-finding, and union busting through coercion and interference with
union affairs.||In a conciliation conference, Sukhothai agreed and guaranteed that there will be no termination of
the employees during the pendency of the case, with the reservation of the management prerogative to issue
memos to erring employees violation of company policies. On the following day, a Strike Vote was conducted and
the results of the vote were submitted to the NCMB.
Sukhothai and the Union agreed to submit the issue for voluntary arbitration. During the pendency of the voluntary
arbitration proceedings, Sukhothai, dismissed a union member, due to an alleged petty quarrel with a co-
employee. In view of this termination, the Union filed with the NLRC a complaint for illegal dismissal.
Subsequently, another union member, Lanorias was relieved from his post, and his employment terminated.
Subsequently, the Union protested Lanorias’ dismissal. Shortly thereafter, respondents staged a wildcat strike.
A Notice of Strike was re-filed by the private respondents and the protest was converted into a "sit-down strike."
On the next day, it was transformed into an "actual strike."|||
A complaint for illegal strike filed with the NLRC by the petitioner against private respondents seeking to declare
the strike illegal, and to declare respondents, who participated in the commission of illegal acts, to have lost their
employment status.

LA RULING:
The LA ruled that the strike is illegal because the Notice of Strike and the Strike Vote referred to a prior dispute
submitted for voluntary arbitration.

NLRC RULING:
Upon appeal to the NLRC, it ruled that the strike is legal and that Sukhothai is guilty of Union Busting.

CA RULING:
Upon appeal to the CA, it affirmed the NLRC decision.

APPEAL TO THE SC:

Petitioner's Contention:
The strike is illegal.
Respondent's Contention:
Respondents insist that the filing of the notice of strike, the submission of the results of the vote to the NCMB, and
their observation of the 15-day cooling-off period in case of unfair labor practice as well as the seven-day
reporting period of the results of the strike vote, all satisfy the mandatory requirements under Article 263 of the
Labor Code.
Angeles| Bajana | Balladares | Brillantes | Briones | Cabansag | Callanta | Chua | David|

De Leon | Gomez | Lopez | Macalino | Nostratis | Padilla | Reynon | Santos | Tan |Velasco
4E / 4F - 2018-2019
Page 513 of 920
LABOR REVIEW DIGEST
Atty. Joyrich Golangco

ISSUE/S:
1. Is the strike staged by the private respondents illegal? – YES

HELD:
1. The undisputed fact, however, is that at the time the strike was staged in June 1999, voluntary arbitration
between the parties was ongoing. The issue to be resolved under those proceedings pertained to the very same
issues stated in the Notice of Strike of December 3, 1998: the commission of unfair labor practices, such as acts
of harassment, fault-finding, and union busting through coercion and interference with union affairs.||
The SC held that strikes staged in violation of agreements providing for arbitration are illegal, since these
agreements must be strictly adhered to and respected if their ends are to be achieved. The rationale of the
prohibition under Article 264 is that once jurisdiction over the labor dispute has been properly acquired by
competent authority, that jurisdiction should not be interfered with by the application of the coercive processes of a
strike.
Indeed it is among the chief policies of the State to promote and emphasize the primacy of free collective
bargaining and negotiations, including voluntary arbitration, mediation, and conciliation, as modes of settling labor,
or industrial disputes.
The alleged dismissals of Lucente and respondent Lanorias, both union members, which allegedly triggered the
wildcat strike, are not sufficient grounds to justify the radical recourse on the part of the private respondents. The
questions that surround their dismissal, as private respondents so affirm, are connected to the alleged breach of
the "guarantee" by the petitioner not to dismiss its employees during the pendency of the arbitration case, the very
questions which they also link to the other incidents of unfair labor practices allegedly committed by the petitioner
— these matters should have been raised and resolved in the voluntary arbitration proceedings that were
commenced precisely to address them.
For failing to exhaust all steps in the arbitration proceedings by virtue of the Submission Agreement, in view of the
proscription under Article 264 of the Labor Code and the prevailing state policy as well as its underlying rationale,
this Court declares that the strike staged by the private respondents is illegal.
————————————————————————

Angeles| Bajana | Balladares | Brillantes | Briones | Cabansag | Callanta | Chua | David|

De Leon | Gomez | Lopez | Macalino | Nostratis | Padilla | Reynon | Santos | Tan |Velasco
4E / 4F - 2018-2019
Page 514 of 920
LABOR REVIEW DIGEST
Atty. Joyrich Golangco

17. BILFLEX PHIL. INC. LABOR UNION et al. Vs FILFLEX INDUSTRIAL AND MANUFACTURING
CORPORATION AND BILFLEX (PHILS.), INC.
GR No.: 155679 (511 SCRA 247) Date: December 19, 2006
PONENTE: Carpio Morales, J.

DOCTRINE:
Any union officer who knowingly participates in an illegal strike and any worker or union who knowingly
participates in the commission of illegal acts during a strike may be declared to have lost his employment status.
FACTS:

Biflex Philippines Inc. Labor Union and Filflex Industrial and Manufacturing Labor Union are the
respective collective bargaining agents of the employees of the sister companies Biflex and Filflex which are
engaged in the garment business. They are situated in one big compound and they have a common entrance.
On October 24, 1990, the labor sector staged a welga ng bayan to protest against oil price hike; the unions staged
a work stoppage which lasted for several days, prompting the companies to file a petition to declare the work
stoppage illegal for failure to comply with procedural requirements.

The Labor Arbiter held that the strike is illegal and declared the officers of the union to have lost
their employment status.

ISSUE/S:
Whether or not the staged strike is illegal and a ground for the lost of employment status of the union officers.

HELD:
Article 264 (a) of the Labor Code states that any union officer who knowingly participates in an illegal strike and
any worker or union who knowingly participates in the commission of illegal acts during a strike may be declared
to have lost his employment status.

Thus, a union officer may be declared to have lost his employment status if he knowingly participates in
an illegal strike and in this case, the strike is declared illegal by the court because the means employed by the
union are illegal.

Here, the unions blocked the egress and ingress of the company premises thus, a violation of Article 264 (e) of
the Labor Code which would affect the strike as illegal even if assuming arguendo that the unions had complied
with legal formalities and thus, the termination of the employees was valid.

The court said that the legality of a strike is determined not only by compliance with its legal formalities but also by
means by which it is carried out.
————————————————————————

Angeles| Bajana | Balladares | Brillantes | Briones | Cabansag | Callanta | Chua | David|

De Leon | Gomez | Lopez | Macalino | Nostratis | Padilla | Reynon | Santos | Tan |Velasco
4E / 4F - 2018-2019
Page 515 of 920
LABOR REVIEW DIGEST
Atty. Joyrich Golangco

18. SANTA ROSA COCA-COLA PLANT EMPLOYEES


UNION vs COCA-COLA BOTTLERS PHILS., INC.,
G.R. Nos. 164302-03; January 24, 2007
Callejo, Sr., J.:

DOCTRINE: Strike as a temporary stoppage of work by the concerted action of employees as a result of an
industrial or labor dispute and it encompasses not only concerted work stoppages, but also slowdowns, mass
leaves, sit-downs, attempts to damage, destroy or sabotage plant equipment and facilities, and similar activities.
While, Picketing involves merely the marching to and fro at the premises of the employer, usually accompanied by
the display of placards and other signs making known the facts involved in a labor dispute.

FACTS: The Sta. Rosa Coca-Cola Plant Employees Union (Union) is the sole and exclusive bargaining
representative of the regular daily paid workers and the monthly paid non-commission-earning employees of the
Coca-Cola Bottlers Philippines, Inc. (Company) in its Sta. Rosa, Laguna plant. The individual petitioners are Union
officers, directors, and shop stewards.

The Union and the Company had entered into a three-year Collective Bargaining Agreement (CBA) Upon its
expiration, the Union desired to continue the CBA and to renegotiate its terms. The CBA meetings commenced
where The Union insisted that representatives from the Alyansa ng mga Unyon sa Coca-Cola be allowed to sit
down as observers in the CBA meetings. The Union officers and members also insisted that their wages be based
on their work shift rates. For its part, the Company was of the view that the members of the Alyansa were not
members of the bargaining unit. The Alyansa was a mere aggregate of employees of the Company in its various
plants; and is not a registered labor organization. Thus, an impasse ensued.

The Union, filed a Notice of Strike, as amended, on the grounds of: (a) unfair labor practice for the companys
refusal to bargain in good faith; and (b) interference with the exercise of their right to self-organization.

Union members, officers and members of the Board of Directors, and shop stewards, individually filed applications
for leave of absence but they were not granted by reason that operations in the plant would come to a complete
stop since there were no sufficient trained contractual employees who would take over. A day before the mass
action, some Union members wore gears, red tag cloths stating YES KAMI SA STRIKE as headgears and on the
different parts of their uniform, shoulders and chests.

The Office of the Mayor issued a permit to the Union, allowing it to conduct a mass protest action within the
perimeter of the Coca-Cola plant. Thus, the Union officers and members held a picket along the front perimeter of
the plant resulting to absence of a number of company employees and personnel. As a result, only one of the
three bottling lines operated during the day shift. The volume of production for the day was short by 60,000
physical case[s] versus budget.

Respondent’s allegations: Company filed a Petition to Declare Strike Illegal alleging that the Union picketed the
premises of the Company in clear violation of Article 262 of the Labor Code; because of the slowdown in the work,
the Company suffered losses; the mass/protest action conducted was clearly a strike and since the Union did not
observe the requirements mandated by law the strike was therefore illegal and there being a direct violation of the
CBA, the Unions action constituted an unfair labor practice; and the officers who knowingly participated in the
commission of illegal acts during the strike should be declared to have lost their employment status.

Petitiner’s Answer: The Union filed an Answer with a Motion to Dismiss and/or to Suspend Proceedings alleging
therein that the mass action conducted by its officers and members was not a strike but just a valid exercise of
their right to picket, which is part of the right of free expression as guaranteed by the Constitution. The workers,
including the petitioners, merely marched to and fro at the side of the highway near one of the gates of the Sta.
Angeles| Bajana | Balladares | Brillantes | Briones | Cabansag | Callanta | Chua | David|

De Leon | Gomez | Lopez | Macalino | Nostratis | Padilla | Reynon | Santos | Tan |Velasco
4E / 4F - 2018-2019
Page 516 of 920
LABOR REVIEW DIGEST
Atty. Joyrich Golangco

Rosa Plant. After 3 hours, everyone returned to work according to their respective shifting schedules. Petitioners
also point out that members belonging to the IBM-KMU at the San Fernando Coca-Cola bottling plant staged
simultaneous walkout from their work assignments for two consecutive days. However, the Secretary of Labor and
Employment (SOLE) declared that the walkout was considered a mass action, not a strike, and the officers of the
IBM-KMU were only meted a three-day suspension.

Lastly, petitioners aver that even assuming that they had indeed staged a strike, the penalty of dismissal is too
harsh. They insist that they acted in good faith.

Labor Arbiter Decision: Granted the petition of the Company. LA declared that the mass leave was actually a
strike by reason that did not report for their usual work. Instead, they all assembled in front of the Sta. Rosa Plant
and picketed the premises. Very clearly, there was a concerted action here on the part of the respondents brought
about a temporary stoppage of work at two out of three bottling lines at the Sta. Rosa Plant. Also, the strike
conducted by the Union was illegal since there was no showing that the Union complied with requirement of a
valid strike.

NLRC Decision: Affirmed the decision of the Labor Arbiter.

CA Decision: Union filed a petition for certiorari in the CA. CA dismissed petition for lack of merit. MR also
dismissed.

ISSUE/s:
1. Whether the mass action staged by the Union was a strike;
2. If in the affirmative, whether it was legal; and
3. Whether the individual officers and shop stewards of petitioner Union should be dismissed from their
employment.

HELD:

1. Yes, the concerted action was not merely a picket but a strike.

In Article 212(o) of the Labor Code defines strike as a temporary stoppage of work by the concerted
action of employees as a result of an industrial or labor dispute. The term strike encompasses not only concerted
work stoppages, but also slowdowns, mass leaves, sit-downs, attempts to damage, destroy or sabotage plant
equipment and facilities, and similar activities.

Picketing involves merely the marching to and fro at the premises of the employer, usually accompanied
by the display of placards and other signs making known the facts involved in a labor dispute. As applied to a
labor dispute, to picket means the stationing of one or more persons to observe and attempt to observe. The
purpose of pickets is said to be a means of peaceable persuasion.

Petitioners insistence to stage a strike is


evident in the fact that an amended notice to strike was filed even as respondent moved to dismiss the first notice.
The basic elements of a strike are present in this case: 106 members of petitioner Union, whose respective
applications for leave of absence on days of supposed strike were disapproved, opted not to report for work on
said date, and gathered in front of the company premises to hold a mass protest action. Petitioners deliberately
absented themselves and instead wore red ribbons, carried placards with slogans. They marched to and fro in
front of the companys premises during working hours. Thus, petitioners engaged in a concerted activity which
already affected the companys operations. The mass concerted activity constituted a strike. What is definitive of

Angeles| Bajana | Balladares | Brillantes | Briones | Cabansag | Callanta | Chua | David|

De Leon | Gomez | Lopez | Macalino | Nostratis | Padilla | Reynon | Santos | Tan |Velasco
4E / 4F - 2018-2019
Page 517 of 920
LABOR REVIEW DIGEST
Atty. Joyrich Golangco

whether the action staged by petitioners is a strike and not merely a picket is the totality of the circumstances
surrounding the situation.

2. The Strike was illegal. Since strikes cause disparity effects not only on the relationship between labor and
management but also on the general peace and progress of society, the law has provided limitations on the right
to strike. For a strike to be valid, the following procedural requisites provided by Art. 263 of the Labor Code must
be observed: (a) a notice of strike filed with the DOLE 30 days before the intended date thereof, or 15 days in
case of unfair labor practice; (b) strike vote approved by a majority of the total union membership in the bargaining
unit concerned obtained by secret ballot in a meeting called for that purpose, (c) notice given to the DOLE of the
results of the voting at least seven days before the intended strike. These requirements are mandatory and the
failure of a union to comply therewith renders the strike illegal. It is clear in this case that petitioners totally ignored
the statutory requirements and embarked on their illegal strike.

3. Yes. The ruling of the CA affirming the decisions of the NLRC and the Labor Arbiter ordering the
dismissal of the petitioners-officers, directors and shop stewards of petitioner Union is correct.

However, distinction should be made. A worker merely participating in an illegal strike may not be terminated from
employment. It is only when he commits illegal acts during a strike that he may be declared to have
lost employment status. For knowingly participating in an illegal strike or participates in the commission of illegal
acts during a strike, the law provides that a union officer may be terminated from employment. The law grants the
employer the option of declaring a union officer who participated in an illegal strike as having lost his employment.
It possesses the right and prerogative to terminate the union officers from service.
————————————————————————

Angeles| Bajana | Balladares | Brillantes | Briones | Cabansag | Callanta | Chua | David|

De Leon | Gomez | Lopez | Macalino | Nostratis | Padilla | Reynon | Santos | Tan |Velasco
4E / 4F - 2018-2019
Page 518 of 920
LABOR REVIEW DIGEST
Atty. Joyrich Golangco

19. Manila Hotel Employees Association (MHEA) vs. Manila Hotel Corp.
GR No: 154591 Date: March 5, 2007
Ponente: Chico-Nazario, J.

Doctrine: The Court has consistently ruled in a long line of cases spanning several decades that once the SOLE
assumes jurisdiction over a labor dispute, such jurisdiction should not be interfered with by the application of the
coercive processes of a strike or lockout. Defiance of the assumption order or a return-to work order by a striking
employee, whether a union officer or a member, is an illegal act and, therefore, a valid ground for loss of
employment status.

FACTS: MHEA filed a Notice of Strike with the NCMB against Manila Hotel on the grounds of unfair labor
practices. Upon the petition of Manila Hotel, the SOLE certified the labor dispute to the NLRC for compulsory
arbitration. Specifically, the Order enjoined any strike or lockout and the parties were ordered to cease and desist
from committing any acts that may exacerbate the situation. The parties and their counsels were served copies of
the said Order. MHEA filed a Motion for Reconsideration assailing the validity of said Order.

The case was set for mandatory conference. During the conference, the parties were advised of the certification
order, which prohibited them from taking any action that would exacerbate the situation.

At the instance of the MHEA officers, the hearing of the case was reset to 29 February 2000 due to the absence
of the counsel for MHEA. On 10 February 2000, the MHEA conducted a strike despite the clear terms of the Order
issued by the SOLE.

After the strike was conducted, both parties filed various motions and pleadings before the NLRC. Manila Hotel
filed a complaint with Prayer for Injunction and/or Temporary Restraining Order alleging that MHEA conducted an
illegal strike, blocked all ingress and egress of the hotel premises, harassed and intimidated company officers,
non-striking employees, customers and suppliers. In addition, it sought a declaration that the strike was illegal and
that, consequently, the striking employees lost their employment.

The NLRC issued an Order directing the striking workers to return to work immediately and the hotel to accept
them back under the same terms and conditions of employment. The parties, through their counsels, received the
said Order the same day. The NLRC received a copy of the Compliance filed by Manila Hotel on 14 February
2000, manifesting that only six striking employees complied with the return-to-work Order and were reinstated.

In response to the NLRCs return-to-work order MHEA filed an Urgent Manifestation and Motion to Set Aside
Order. It alleged that the Motion for Reconsideration questioning the validity of the Order of the SOLE which
certified the case to the NLRC, was still pending with the SOLE. The said motion had prevented the said Order of
the SOLE from becoming final and executory. Thus, it alleged that the NLRC had not acquired jurisdiction over the
labor dispute pending the resolution of the Motion for Reconsideration filed before the SOLE.

NLRC RULING: The strike held by MHEA was illegal for its defiance of the return-to-work order. However, it
determined that only the union officers were deemed to have lost their employment. It ruled that there was no
evidence showing who among the striking employees were actually notified of the return-to-work order, and
therefore, such employees have not forfeited their employment. But in view of the antagonism on both sides, the
NLRC awarded a severance pay equivalent to one-month salary to the returning union members for every year of
service, instead of ordering Manila Hotel to reinstate them.

Angeles| Bajana | Balladares | Brillantes | Briones | Cabansag | Callanta | Chua | David|

De Leon | Gomez | Lopez | Macalino | Nostratis | Padilla | Reynon | Santos | Tan |Velasco
4E / 4F - 2018-2019
Page 519 of 920
LABOR REVIEW DIGEST
Atty. Joyrich Golangco

CA RULING: Both the incumbent officers and members of the Union involved in the illegal strike are declared to
have lost their employment status. The award of severance compensation to the striking members of the union is
consequently DELETED.

APPEAL TO THE SC:

Petitioner's Contention: MHEA, on the other hand, sought the reversal of the Decision on the ground that the
NLRC had no jurisdiction over the case and that they were deprived of due process.

Respondent's Contention: Manila Hotel filed a Motion for Partial Reconsideration which sought the deletion of
the award of severance compensation to the union members who participated in the illegal strike.

ISSUE/S: whether or not the pending resolution of the MR filed against the certification by the SOLE of the case
to the NLRC deprives the NLRC of jurisdiction from filing a return-to-work order

HELD: NO. MHEA members seek their reinstatement after participating in an illegal strike, that is, a strike that
was conducted after receiving an Order of assumption by the SOLE certifying the dispute to the NLRC for
compulsory arbitration. Worse still, the strikers failed to comply with the return-to-work Order, issued by the NLRC,
despite receipt thereof. The law explicitly prohibits such acts.

The assumption of jurisdiction by the SOLE over labor disputes causing or likely to cause a strike or lockout in an
industry indispensable to the national interest is in the nature of a police power measure.

The Hotel is engaged in the hotel and restaurant business and one of the de luxe hotels operating in Metro Manila
catering mostly to foreign tourist groups and businessmen. It serves as venue for local and international
conventions and conferences. It also provides substantial contribution to the government coffers in the form of
foreign exchange earnings and tax payments. Undoubtedly, a work stoppage thereat will adversely affect the
Hotel, its employees, the industry, and the economy as a whole.

A return-to-work order is immediately executory notwithstanding the filing of a motion for reconsideration. It must
be strictly complied with even during the pendency of any petition questioning its validity.

The very nature of a return-to-work order issued in a certified case lends itself to no other construction. The
certification attests to the urgency of the matter, affecting as it does an industry indispensable to the national
interest. The order is issued in the exercise of the courts compulsory power of arbitration, and therefore must be
obeyed until set aside. To say that its [return-to-work order] effectivity must await affirmance on a motion for
reconsideration is not only to emasculate it but indeed to defeat its import, for by then the deadline fixed for the
return to work would, in the ordinary course, have already passed and hence can no longer be affirmed insofar as
the time element it concerned.

Returning to work in this situation is not a matter of option or voluntariness but of obligation.
————————————————————————

Angeles| Bajana | Balladares | Brillantes | Briones | Cabansag | Callanta | Chua | David|

De Leon | Gomez | Lopez | Macalino | Nostratis | Padilla | Reynon | Santos | Tan |Velasco
4E / 4F - 2018-2019
Page 520 of 920
LABOR REVIEW DIGEST
Atty. Joyrich Golangco

20. G&S TRANSPORT CORPORATION v. INFANTE 533 SCRA 289 TING, J.:

Doctrine:

FACTS: - Petitioner was the exclusive coupon taxi concessionaire at the NAIA for 5 years. Under such contract
the taxi units were given a garage located at the Duty Free compound just opposite NAIA.
NAIA sent a letter to the NAIA Service Taxi Employees Union (Union) demanding the dismissal from employment
Ricardo Gonzales and Ephraim Alzaga (both drivers of herein petitioner) on the grounds of disloyalty and
unbecoming of a union member inimical to the interests of the Union. Petitioner terminated said drivers. This
caused several drivers to stop driving their taxis apparently to sympathize with their dismissed colleagues.
Petitioner claims that this brought stoppage to its business operation and is one that squarely falls under the
concept of an illegal strike at the work premises. Petitioner filed an action for illegal strike before the LA against 37
drivers (among those included private respondents Infante, Borbo and Castañeda). The 37 drivers filed an illegal
dismissal case but most of them filed their affidavits of desistance.
Defense of Infante and Borbo: they reported to work but found no taxis in their garage and found out that on that
same day the protest to sympathize with their colleagues ensued. The day after the protest they did not report
back to work, as it was their day-off. However, when they returned to work they were refused entry by the security
guard because their names did not appear in the list of drivers allowed to work by the petitioner.
Defense of Castañeda: He was on a sick leave. But when I reported back to work it was on the same day that
the strike occurred. The following day, he reported back to work but was refused entry by the guard.
Decision of LA: concerted action by the drivers is illegal strike. Drivers undertook those collective actions without
first filing a notice to strike and taking a strike vote, and in violation of the no strike – no lock out clause in the
CBA. The LA further noted that when the strike occurred there was no labor dispute (as Article 212 mandates that
such stoppage of work by employees be a result of an industrial of labor disputes). LA found respondents
PARTICIPATED in the illegal strike BUT did not affirm their dismissal. Instead, ordered that separation pay be
given in lieu of reinstatement but without backwages. NLRC affirmed in toto LA’s ruling.
CA’s Decision: It remanded it to the LA for the computation of backwages and other monetary benefits. It further
noted that the dismissal of respondents was illegal. CA relied on a certification from SEC that petitioner was still
operational and that the LA and NLRC ruling for separation pay instead of reinstatement amounted to grave
abuse of discretion.

ISSUES:
1. Was there an illegal strike?
2. Whether the order for the payment of separation pay, in lieu of reinstatement without backwages, was proper.

HELD:
1. YES. Article 212 of the Labor Code defines strike as any temporary stoppage of work by the concerted action of
employees as a result of an industrial or labor dispute. A valid strike therefore presupposes the existence of a
labor dispute. The strike undertaken by respondents took the form of a sit-down strike, or more aptly termed as a
sympathetic strike, where the striking employees have no demands or grievances of their own, but they strike for
the purpose of directly or indirectly aiding others, without direct relation to the advancement of the interest of the
strikers. It is indubitable that an illegal strike in the form of a sit-down strike occurred in petitioner’s premises, as a
show of sympathy to the two employees who were dismissed by petitioner. The office telegram sent to individual
respondents informing them to return to work went unheeded. Respondents failed to satisfactorily explain their
conspicuous absence following the day of the purported illegal strike.

What is the effect of their participation? Article 264 of the Labor Code, in providing for the consequences of an
illegal strike, makes a distinction between union officers and members who participated therein. Thus,
knowingly participating in an illegal strike is a valid ground for termination of employment of a union
officer. The law, however, treats differently mere union members. Mere participation in an illegal strike is
Angeles| Bajana | Balladares | Brillantes | Briones | Cabansag | Callanta | Chua | David|

De Leon | Gomez | Lopez | Macalino | Nostratis | Padilla | Reynon | Santos | Tan |Velasco
4E / 4F - 2018-2019
Page 521 of 920
LABOR REVIEW DIGEST
Atty. Joyrich Golangco

not a sufficient ground for termination of the services of the union members. There must be proof that he
committed illegal acts during the strike and the striker who participated in the commission of illegal act
must be identified. Proof beyond reasonable doubt is not required. Substantial evidence available under
the attendant circumstances, which may justify the imposition of the penalty of dismissal, may suffice.

In the case at bar, this Court is not convinced that the affidavits of petitioners witnesses constitute substantial
evidence to establish that illegal acts were committed by respondents. Nowhere in their affidavits did
these witnesses cite the particular illegal acts committed by each individual respondent during the strike.
Notably, no questions during the hearing were asked relative to the supposed illegal acts.

As adverted to earlier, no matter by what term the respondents complainants used in describing their concerted
action, i.e. [,] protest, sympathy or mere expression, their joint action have successfully paralyzed the
operations of G & S Transport, and this is considered a strike.

It can now therefore be concluded that the acts of respondents do not merit their dismissal from employment
because it has not been substantially proven that they committed any illegal act while participating in the
illegal strike.

2. YES. Respondents dismissal from work could not be any clearer than the refusal of petitioner to admit them
back as they signified their intention to go back to work..

With respect to backwages, the principle of a "fair day’s wage for a fair day’s labor" remains as the basic factor in
determining the award thereof. If there is no work performed by the employee there can be no wage or
pay unless, of course, the laborer was able, willing and ready to work but was illegally locked out,
suspended or dismissed or otherwise illegally prevented from working. While it was found that
respondents expressed their intention to report back to work, the latter exception cannot apply in this
case. In Philippine Marine Officers’ Guild v. Compañia Maritima, as affirmed in Philippine Diamond Hotel
and Resort v. Manila Diamond Hotel Employees Union, the Court stressed that for this exception to
apply, it is required that the strike be legal, a situation that does not obtain in the case at bar. Under the
circumstances, respondents’ reinstatement without backwages suffices for the appropriate relief. If
reinstatement is no longer possible, given the lapse of considerable time from the occurrence of the
strike, the award of separation pay of one (1) month salary for each year of service, in lieu of
reinstatement, is in order.

Section 4, Rule I of the Rules Implementing Book VI of the Labor Code provides: Reinstatement to former
position.—(a) An employee who is separated from work without just cause shall be reinstated to
his former position, unless such position no longer exists at the time of his reinstatement, in
which case he shall be given a substantially equivalent position in the same establishment
without loss of seniority rights.

The above-quoted rule enunciates reinstatement as the standard relief. However, in this case, seventeen (17)
years have elapsed since respondents were illegally dismissed. In Association of Independent Unions in
the Philippines v. NLRC, where more than eight (8) years have passed since the petitioners therein
staged an illegal strike and were found to have been unlawfully terminated, an award of separation pay
equivalent to one (1) month pay for every year of service, in lieu of reinstatement, was deemed more
practical and appropriate to all the parties concerned. We adopt the same tack in this case.

———————————————————————

Angeles| Bajana | Balladares | Brillantes | Briones | Cabansag | Callanta | Chua | David|

De Leon | Gomez | Lopez | Macalino | Nostratis | Padilla | Reynon | Santos | Tan |Velasco
4E / 4F - 2018-2019
Page 522 of 920
LABOR REVIEW DIGEST
Atty. Joyrich Golangco

21. Steel Corporation of the Philippines vs SCP Employees’ Union National Federation of Labor Unions
GR No: 169829-30 Date: April 16, 2008
Ponente: Azcuna, J.

Doctrine:
The strike undertaken by the officers of respondent union is patently illegal for the following reasons: (1) it is a
union-recognition-strike which is not sanctioned by labor laws; (2) it was undertaken after the dispute had been
certified for compulsory arbitration; and (3) it was in violation of the Secretarys return-to-work order.

FACTS:
Petitioner Steel Corporation of the Philippines (SCP) is engaged in manufacturing construction materials,
supplying approximately 50% of the domestic needs for roofing materials. SCP-Federated Union of the Energy
Leaders General and Allied Services (FUEL-GAS) filed a petition for Certification Election in its bid to represent
the rank-and-file employees of the petitioner. Respondent SCP Employees Union (SCPEU) National Federation of
Labor Unions (NAFLU) intervened, seeking to participate and be voted for in such election but the same was
denied for having been filed out of time.

A consent election was conducted, with FUEL-GAS and NO UNION as choices. Said election was however
declared a failure because less than a majority of the rank-and-file employees cast their votes. FUEL-GAS filed an
Election Protest claiming that the certification election was characterized by and replete with irregularities. NAFLU,
the mother federation of respondent, filed a petition for Certification Election for and on behalf of its affiliate,
seeking to represent the rank-and-file employees of petitioner. The Med-Arbiter denied the election protest of
FUEL-GAS and granted the petition for certification election filed by NAFLU and further ordered the conduct of the
election with NAFLU and NO UNION as choices. Both petitioner and FUEL-GAS appealed to the Secretary of
Labor, which appeals were later consolidated.

DOLE Undersecretary rendered a consolidated decision ordering the conduct of a certification election with FUEL-
GAS, respondent and NO UNION as choices. Unsatisfied, petitioner and FUEL-GAS appealed to the CA by way
of certiorari. The certification election, as ordered by the Med-Arbiter, proceeded. FUEL-GAS participated without
prejudice to the decision of the CA in its pending petition. In said election, respondent emerged as winner; hence,
the second election protest filed by FUEL-GAS.

CA rendered a Decision which annulled and set aside the decision and resolution of the Undersecretary. The CA
further directed the holding of a certification election with FUEL-GAS and NO UNION as choices, to the exclusion
of respondent.

The Med-Arbiter dismissed FUEL-GAS election protest but deferred the request of respondent to be declared
winner in the certification election until final resolution of the pending petitions with the CA. Not satisfied with the
deferment of their certification as winner, respondent appealed to the Labor Secretary. It further filed a
Manifestation before the CA pointing out that in the April 14, 2000 certification election, it emerged as winner, and
thus, the election should be considered as an intervening event sufficient to bar another certification election. The
CA, however, dismissed said manifestation on December 28, 2000.

Meanwhile, on October 16, 2000, the Undersecretary rendered a Decision certifying respondent as the exclusive
bargaining agent of petitioner’s employees. Petitioner and FUEL-GAS timely filed motions for reconsideration of
the aforesaid decision.

As a consequence of its certification as the exclusive bargaining agent, respondent sent to petitioner CBA
proposals. Petitioner, however, held in abeyance any action on the proposals in view of its pending motion for
reconsideration.
Angeles| Bajana | Balladares | Brillantes | Briones | Cabansag | Callanta | Chua | David|

De Leon | Gomez | Lopez | Macalino | Nostratis | Padilla | Reynon | Santos | Tan |Velasco
4E / 4F - 2018-2019
Page 523 of 920
LABOR REVIEW DIGEST
Atty. Joyrich Golangco

Finding no justification in petitioners refusal to bargain with it, respondent filed a Notice of Strike with the
NCMB. The union raised the issue of unfair labor practice (ULP) allegedly committed by petitioner for the latters
refusal to bargain with it.

FUEL-GAS moved for the conduct of a certification election pursuant to the CA decision. On February 27, 2001,
the Undersecretary affirmed its October 16, 2000 decision.

On March 16, 2001, the labor dispute was certified to the National Labor Relations Commission (NLRC) for
compulsory arbitration, which case was docketed as Cert. Case No. 000200-01. Again, on April 2, 2001, another
Notice of Strike was filed by respondent for non-recognition as a certified union; refusal to bargain; discrimination
against union officers and members; harassment and intimidation; and illegal dismissal, which was later
consolidated with the certified case.

On December 13, 2001, acting on the January 19, 2001 petition for certification election, the Med-Arbiter
recommended the holding of another certification election but with respondent and FUEL-GAS as contenders. The
decision was appealed to the Labor Secretary. The Labor Secretary in turn dismissed the motion to conduct
certification election in a Resolution dated October 17, 2002.

NLRC RULING: in Cert. Case No. 000200-01, the NLRC issued a Resolution dated April 17, 2002, declaring
petitioner as having no obligation to recognize respondent as the certified bargaining agent; dismissing the charge
of unfair labor practice; declaring as illegal the strike held by the union; and declaring the loss of employment of
the officers of the union.

On May 20, 2002, respondent filed another Notice of Strike alleging as grounds, petitioners refusal to bargain
and union busting. The notice was later dismissed and respondent was enjoined from holding a strike.

On January 7, 2003, respondent filed another Notice of Strike on the grounds of refusal to bargain and union
busting. Respondent thereafter went on strike on February 4, 2003. On February 7, 2003, the Labor Secretary
certified the dispute to the NLRC and directed the employees to return to work. The second certified case was
docketed as NLRC NCR CC No. 00253-03. On September 8, 2003, the NLRC rendered a Decision ordering
petitioner to bargain collectively with respondent as the duly certified bargaining agent. In addition, it ordered the
reinstatement of the employees who were dismissed in connection with the February 4, 2003 strike, without loss
of seniority rights and diminution of salary. Petitioner filed a motion for reconsideration but it was denied in the
Resolution dated January 26, 2004. The decision and resolution became the subject of a petition before the CA in
CA-G.R. SP No. 82314.

Meantime, in the first certified case, Cert. Case No. 000200-01, the NLRC, in a Decision dated February 12,
2003 opted to resolve the parties respective motions for reconsideration collectively. In said decision, the NLRC
modified its earlier resolution by ordering the reinstatement of the union officers whom it previously ordered
terminated, which in effect denied petitioners motion for partial reconsideration. Petitioner filed a motion for
reconsideration but it was denied in a Resolution dated June 30, 2003. These decision and resolution became the
subject of a petition before the CA in CA-G.R. SP No. 79446.

CA RULING:
CA rendered a Decision denying the petition in CA-G.R. SP No. 79446 while partially granting the petition in CA-
G.R. SP No. 82314.

Petitioner's Contention:

Angeles| Bajana | Balladares | Brillantes | Briones | Cabansag | Callanta | Chua | David|

De Leon | Gomez | Lopez | Macalino | Nostratis | Padilla | Reynon | Santos | Tan |Velasco
4E / 4F - 2018-2019
Page 524 of 920
LABOR REVIEW DIGEST
Atty. Joyrich Golangco

• Petitioner contends that the February 2003 strike held by respondent is illegal. To buttress its claim, petitioner
argues that respondent has no right to demand that it bargain with the latter. Its refusal to recognize respondent
as the bargaining representative of its employees is based on the directive of the CA in CA-G.R. SP No. 55721 to
conduct another certification election. Petitioner maintains that respondent never denied that its purpose for
holding the strike was to force it to recognize the latter over the other union. Since the strike is a union-
recognition-strike, it is illegal.
• Petitioner further argues that the strike was manifestly illegal for it was in gross violation of the Labor Code,
particularly Art. 264, which expressly prohibits the declaration of a strike over an issue that is pending arbitration
between the parties. Since the labor dispute in the first certified case, Cert. Case No. 000200-01, was still pending
compulsory arbitration at the time of the strike on February 4, 2003, and since the said strike was based
substantially on the same grounds, i.e., the alleged refusal by petitioner to recognize the union, the strike is illegal
by express provision of the law.
• Petitioner also contends that the union officers who participated in the illegal strike are all deemed to have lost
their employment. Unlike ordinary members of the union, whose dismissal requires that the employer prove that
they committed illegal acts, mere participation of the union officers in an illegal strike warrants their termination
from employment. Consequently, since the strike was illegal, it follows that the termination from employment of
the union officers was warranted.

ISSUE/S:
1. Whether or not the strike participated in by the officers of the respondent union is valid?
2. Whether or not the termination of such officers from employment by reason of such participation is valid?

HELD:
1. NO.
The strike is a legitimate weapon in the human struggle for a decent existence. It is considered as the most
effective weapon in protecting the rights of the employees to improve the terms and conditions of their
employment. But to be valid, a strike must be pursued within legal bounds. The right to strike as a means for the
attainment of social justice is never meant to oppress or destroy the employer. The law provides limits for its
exercise.

In the instant case, the strike undertaken by the officers of respondent union is patently illegal for the following
reasons: (1) it is a union-recognition-strike which is not sanctioned by labor laws; (2) it was undertaken after the
dispute had been certified for compulsory arbitration; and (3) it was in violation of the Secretarys return-to-work
order.

Respondents’ notices of strike were founded on petitioners continued refusal to bargain with it. It thus staged the
strike to compel petitioner to recognize it as the collective bargaining agent, making it a union-recognition-
strike. As its legal designation implies, this kind of strike is calculated to compel the employer to recognize ones
union and not other contending groups, as the employees bargaining representative to work out a collective
bargaining agreement despite the striking unions doubtful majority status to merit voluntary recognition and lack of
formal certification as the exclusive representative in the bargaining unit.

The certification election that was conducted where respondent emerged as winner, not having been recognized
as valid, it has no authority to represent the rank and file employees of petitioner. Thus, it could not ask petitioner
to bargain with it. As the issue of its identity had been the subject of a separate case which had been settled by
the court with finality, petitioner cannot, therefore, be faulted in refusing to bargain. Neither could this Court
sustain respondents imputation of unfair labor practice and union busting against petitioner. With more reason,
this Court cannot sustain the validity of the strike staged on such basis.

Angeles| Bajana | Balladares | Brillantes | Briones | Cabansag | Callanta | Chua | David|

De Leon | Gomez | Lopez | Macalino | Nostratis | Padilla | Reynon | Santos | Tan |Velasco
4E / 4F - 2018-2019
Page 525 of 920
LABOR REVIEW DIGEST
Atty. Joyrich Golangco

Even if this Court were to uphold the validity of respondents purpose or objective in staging a strike, still, the strike
would be declared illegal for having been conducted in utter defiance of the Secretarys return-to-work order and
after the dispute had been certified for compulsory arbitration. Although ostensibly there were several notices of
strike successively filed by respondent, these notices were founded on substantially the same grounds petitioners
continued refusal to recognize it as the collective bargaining representative.

Article 263(g) of the Labor Code provides:

When, in his opinion, there exists a labor dispute causing or likely to cause a strike or lockout in an industry
indispensable to the national interest, the Secretary of Labor and Employment may assume jurisdiction over the
dispute and decide it or certify the same to the Commission for compulsory arbitration. Such assumption or
certification shall have the effect of automatically enjoining the intended or impending strike or lockout as specified
in the assumption or certification order. If one has already taken place at the time of assumption or certification, all
striking or locked out employees shall immediately return to work and the employer shall immediately resume
operations and readmit all workers under the same terms and conditions prevailing before the strike or
lockout. The Secretary of Labor and Employment or the Commission may seek the assistance of law enforcement
agencies to ensure the compliance with this provision as well as with such orders as he may issue to enforce the
same. x x x.

The powers granted to the Secretary under Article 263(g) of the Labor Code have been characterized as an
exercise of the police power of the State, aimed at promoting the public good. When the Secretary exercises
these powers, he is granted great breadth of discretion to find a solution to a labor dispute. The most obvious of
these powers is the automatic enjoining of an impending strike or lockout or its lifting if one has already taken
place.

The moment the Secretary of Labor assumes jurisdiction over a labor dispute in an industry indispensable to
national interest, such assumption shall have the effect of automatically enjoining the intended or impending
strike. It was not even necessary for the Secretary of Labor to issue another order directing a return to work. The
mere issuance of an assumption order by the Secretary of Labor automatically carries with it a return-to-work
order, even if the directive to return to work is not expressly stated in the assumption order.

A return-to-work order imposes a duty that must be discharged more than it confers a right that may be
waived. While the workers may choose not to obey, they do so at the risk of severing their relationship with their
employer.

Says the Labor Code:

Art. 264. Prohibited activities.


xxx

No strike or lockout shall be declared after assumption of jurisdiction by the President or the Secretary or after
certification or submission of the dispute to compulsory or voluntary arbitration or during the pendency of cases
involving the same grounds for the strike or lockout.

Returning to work in this situation is not a matter of option or voluntariness but of obligation. The worker must
return to his job together with his co-workers so that the operations of the company can be resumed and it can
continue serving the public and promoting its interest. This extraordinary authority given to the Secretary of Labor
is aimed at arriving at a peaceful and speedy solution to labor disputes, without jeopardizing national
interests. Regardless of their motives, or the validity of their claims, the striking workers must cease and/or desist
Angeles| Bajana | Balladares | Brillantes | Briones | Cabansag | Callanta | Chua | David|

De Leon | Gomez | Lopez | Macalino | Nostratis | Padilla | Reynon | Santos | Tan |Velasco
4E / 4F - 2018-2019
Page 526 of 920
LABOR REVIEW DIGEST
Atty. Joyrich Golangco

from any and all acts that undermine or tend to undermine this authority of the Secretary of Labor, once an
assumption and/or certification order is issued. They cannot, for instance, ignore return-to-work orders, citing
unfair labor practices on the part of the company, to justify their action.

Respondent, in the instant case, after the assumption of jurisdiction and certification of the dispute to the NLRC
for compulsory arbitration, filed notices of strike and staged the strike obviously contrary to the provisions of labor
laws. Worse, it filed not one but several notices of strike which resulted in two certified cases which were earlier
consolidated. These disputes could have been averted had respondent respected the CAs decision. That way, the
collective bargaining agent would have been determined and petitioner could have been compelled to
bargain. Respondent, through its officers, instead opted to use the weapon of strike to force petitioner to
recognize it as the bargaining agent. The strike, having been staged after the dispute had been certified for
arbitration and contrary to the return-to-work order, became a prohibited activity, and was thus illegal.

Strikes exert disquieting effects not only on the relationship between labor and management, but also on the
general peace and progress of
society, not to mention the economic well-being of the State. It is a weapon that can either breathe life to or
destroy the union and members in their struggle with management for a more equitable due of their
labors. Hence, the decision to wield the weapon of strike must therefore rest on a rational basis, free from
emotionalism, unswayed by the tempers and tantrums of a few, and firmly focused on the legitimate interest of the
union which should not however be antithetical to the public welfare. In every strike staged by a union, the general
peace and progress of society and public welfare are involved.

2. Article 264 of the Labor Code further provides:

Art. 264. Prohibited activities. x x x

Any workers whose employment has been terminated as a consequence of an unlawful lockout shall be entitled to
reinstatement with full back wages. Any union officer who knowingly participates in an illegal strike and any worker
or union officer who knowingly participates in the commission of illegal acts during a strike may be declared to
have lost his employment status: Provided, that mere participation of a worker in a lawful strike shall not constitute
sufficient ground for termination of his employment, even if a replacement had been hired by the employer during
such lawful strike. x x x.

It bears stressing that the law makes a distinction between union members and union officers. A worker merely
participating in an illegal strike may not be terminated from employment. It is only when he commits illegal acts
during a strike that he may be declared to have lost employment status. For knowingly participating in an illegal
strike or participating in the commission of illegal acts during a strike, the law provides that a union officer may be
terminated from employment. The law grants the employer the option of declaring a union officer who participated
in an illegal strike as having lost his employment. It possesses the right and prerogative to terminate the union
officers from service. Otherwise, the workers will simply refuse to return to their work and cause a standstill in the
company operations while retaining the positions they refuse to discharge and preventing management from filling
up their positions.
————————————————————————

Angeles| Bajana | Balladares | Brillantes | Briones | Cabansag | Callanta | Chua | David|

De Leon | Gomez | Lopez | Macalino | Nostratis | Padilla | Reynon | Santos | Tan |Velasco
4E / 4F - 2018-2019
Page 527 of 920
LABOR REVIEW DIGEST
Atty. Joyrich Golangco

22. Chris Garments Corp. v. Sto. Tomas


G.R. No. 167426, January 12, 2009
Quisumbing, J.

DOCTRINE:
It is settled that the filing of a motion for reconsideration is a prerequisite to the filing of a special civil action for
certiorari to give the lower court the opportunity to correct itself. This rule, however, admits of exceptions, such as
when a motion for reconsideration would be useless under the circumstances.

Under Department Order No. 40-03, Series of 2003, the decision of the Secretary of Labor and Employment shall
be final and executory after ten days from receipt thereof by the parties and that it shall not be subject of a motion
for reconsideration.

FACTS:
Petitioner Chris Garments Corporation is engaged in the manufacture and export of quality garments and apparel.
Respondent Chris Garments Workers Union PTGWO filed a petition for certification election seeking to represent
petitioners rank-and-file employees not covered by its CBA with the SMCGC-SUPER, the certified bargaining
agent of the rank-and-file employees.

Petitioner moved to dismiss the petition. It argued that it has an existing CBA with SMCGC-SUPER which bars
any petition for certification election prior to the 60-day freedom period. It also contended that the union members
are not its regular employees since they are direct employees of qualified and independent contractors.

The Med-Arbiter dismissed the petition. The Med-Arbiter ruled that there was no employer-employee relationship
between the parties since the union itself admitted that its members are agency employees. The Med-Arbiter also
held that even if the union members are considered direct employees of petitioner, the petition for certification
election will still fail due to the contract bar rule under Article 232 of the Labor Code. Nevertheless, the Med-
Arbiter ruled that the union may avail of the CBA benefits by paying agency fees to SMCGC-SUPER.

The Secretary of Labor and Employment affirmed the decision of the Med-Arbiter. She ruled that petitioner failed
to prove that the union members are employees of qualified and independent contractors with substantial capital
or investment and added that petitioner had the right to control the performance of the work of such
employees. She also noted that the union members are garment workers who performed activities directly related
to petitioners main business. Thus, the union members may be considered part of the bargaining unit of
petitioners rank-and-file employees. However, she held that the petition could not be entertained except during the
60-day freedom period. She also found no reason to split petitioners bargaining unit.

The union filed a second petition for certification election. The Med-Arbiter dismissed the petition on the ground
that it was barred by a prior judgment. On appeal, the Secretary of Labor and Employment affirmed the decision
of the Med-Arbiter

The union filed a third petition for certification election. The Med-Arbiter dismissed the petition on the grounds that
no employer-employee relationship exists between the parties and that the case was barred by a prior
judgment. On appeal, the Secretary of Labor and Employment granted the petition.

A certification election was conducted on June 21, 2005 among petitioners rank-and-file employees where
SMCGC-SUPER emerged as the winning union.

ISSUE/S:

Angeles| Bajana | Balladares | Brillantes | Briones | Cabansag | Callanta | Chua | David|

De Leon | Gomez | Lopez | Macalino | Nostratis | Padilla | Reynon | Santos | Tan |Velasco
4E / 4F - 2018-2019
Page 528 of 920
LABOR REVIEW DIGEST
Atty. Joyrich Golangco

• Is a motion for reconsideration necessary before a party can file a petition for certiorari from the decision of the
Secretary of Labor and Employment?

• Is the case barred by res judicata or conclusiveness of judgment?

HELD:
• First. It is settled that the filing of a motion for reconsideration is a prerequisite to the filing of a special civil action
for certiorari to give the lower court the opportunity to correct itself. This rule, however, admits of exceptions, such
as when a motion for reconsideration would be useless under the circumstances.

Under Department Order No. 40-03, Series of 2003, the decision of the Secretary of Labor and Employment shall
be final and executory after ten days from receipt thereof by the parties and that it shall not be subject of a motion
for reconsideration.

In this case, the Decision dated January 18, 2005 of the Secretary of Labor and Employment was received by
petitioner on January 25, 2005. It would have become final and executory on February 4, 2005, the tenth day from
petitioners receipt of the decision. However, petitioner filed a petition for certiorari with the Court of Appeals on
even date. Clearly, petitioner availed of the proper remedy since Department Order No. 40-03 explicitly prohibits
the filing of a motion for reconsideration. Such motion becomes dispensable and not at all necessary.

• Second. The doctrine of res judicata provides that a final judgment or decree on the merits by a court of
competent jurisdiction is conclusive of the rights of the parties or their privies in all later suits on points and
matters determined in the former suit. The elements of res judicata are: (1) the judgment sought to bar the new
action must be final; (2) the decision must have been rendered by a court having jurisdiction over the subject
matter and the parties; (3) the disposition of the case must be a judgment on the merits; and (4) there must be as
between the first and second action, identity of parties, subject matter, and causes of action.

In the instant case, there is no dispute as to the presence of the first three elements of res judicata. The
Resolution dated December 27, 2002 of the Secretary of Labor and Employment on the first petition for
certification election became final and executory. It was rendered on the merits and the Secretary of Labor and
Employment had jurisdiction over the case. Now, is the fourth element identity of parties, subject matter, and
causes of action between the first and third petitions for certification election present? We hold in the negative.

The Secretary of Labor and Employment dismissed the first petition as it was filed outside the 60-day freedom
period. At that time therefore, the union has no cause of action since they are not yet legally allowed to challenge
openly and formally the status of SMCGC-SUPER as the exclusive bargaining representative of the bargaining
unit. Such dismissal, however, has no bearing in the instant case since the third petition for certification election
was filed well within the 60-day freedom period. Otherwise stated, there is no identity of causes of action to speak
of since in the first petition, the union has no cause of action while in the third, a cause of action already exists for
the union as they are now legally allowed to challenge the status of SMCGC-SUPER as exclusive bargaining
representative.
————————————————————————

Angeles| Bajana | Balladares | Brillantes | Briones | Cabansag | Callanta | Chua | David|

De Leon | Gomez | Lopez | Macalino | Nostratis | Padilla | Reynon | Santos | Tan |Velasco
4E / 4F - 2018-2019
Page 529 of 920
LABOR REVIEW DIGEST
Atty. Joyrich Golangco

23. UNIVERSITY OF IMMACULATE CONCEPTION vs. SECRETARY OF LABOR


G.R. NOS. 178085 – 178086, September 14, 2015
JARDELEZA, J.:

DOCTRINE: Confidential employees should be excluded from the bargaining unit and disqualified from joining any
union: employees should not be placed in a position involving a potential conflict of interests.

FACTS:
UIC is a non-stock, non-profit educational institution. Private respondent [the Union] is the certified sole bargaining
agent of UIC’s rank and file employees. On 20 June 1994, the Union filed a notice of strike on the grounds of
bargaining deadlock and unfair labor practice. On 20 July 1994, the National Conciliation and Mediation Board
(NCMB) called the parties to a conference where they agreed that an increase be granted to the workers in the
amount equivalent to: seventy-five percent (75%) of increment on the tuition fee for the first year, eighty percent
(80%) for the second year, and eighty percent (80%) for the third year.

On the same occasion, the UIC demanded the exclusion of secretaries, registrars, accounting personnel and
guidance counselors from the bargaining unit, on account of their being confidential employees. When the parties
agreed to submit this particular issue to voluntary arbitration, the arbitration panel sustained the UIC on 08
November 1994. The Union’s motion for reconsideration thereto was denied by the arbitration panel on 08
February 1995.

Accordingly, the UIC gave the affected employees the option to choose between keeping their positions or
resigning from the Union. When they elected to keep both their positions and their union membership, UIC sent
them notices of termination on 21 February 1995, which led into a notice of strike filed by the Union on 10 March
1995. UIC cites willful disobedience and "loss of confidence" as the grounds for dismissing the Respondent
Employees.

ISSUE:
Whether or not a confidential employee’s refusal to vacate his or her union membership is a valid ground for
dismissal

RULING:
As a preliminary matter, we clarify that the issue of whether or not the Respondent Employees are confidential
employees has long been settled and its reexamination is already barred by res judicata. In VA Case No. XI-354-
02-94 (the "Arbitration Case"), the panel of voluntary arbitrators had already determined that the Respondent
Employees are confidential employees who must be excluded from the bargaining unit. The just causes for
terminating an employee, confidential or not, are enumerated in Article 282 of the Labor Code.
Generally, employers are given wide latitude in terminating the services of employees who perform functions
which by their nature require the employer's full trust and confidence. To constitute a valid ground for dismissal, it
is sufficient that there be some reasonable basis, supported by substantial evidence, for such loss of confidence.
Nonetheless, employers do not have unbridled authority to dismiss employees by simply invoking Article 282(c).
The loss of confidence must be genuine and cannot be used as a subterfuge for causes which are illegal,
improper and unjust.

The Court held that the willful act of refusing to leave the Union is sufficient basis for UIC to lose its trust and
confidence on Respondent Employees. There was just cause for dismissing the Respondent Employees. This
conclusion follows the same reasoning why the Court finally adopted the doctrine that confidential employees
should be excluded from the bargaining unit and disqualified from joining any union: employees should not be
placed in a position involving a potential conflict of interests.

Angeles| Bajana | Balladares | Brillantes | Briones | Cabansag | Callanta | Chua | David|

De Leon | Gomez | Lopez | Macalino | Nostratis | Padilla | Reynon | Santos | Tan |Velasco
4E / 4F - 2018-2019
Page 530 of 920
LABOR REVIEW DIGEST
Atty. Joyrich Golangco

If Respondent Employees were allowed to retain their union membership, UIC would not be assured of their
loyalty because of the apparent conflict between the employees’ personal interests and their duty as confidential
employees. Such a result is likely to create an atmosphere of distrust between UIC and the confidential
employees, and it would be nigh unreasonable to compel UIC to continue in employment persons whom it no
longer trusts to handle delicate matters.
————————————————————————

Angeles| Bajana | Balladares | Brillantes | Briones | Cabansag | Callanta | Chua | David|

De Leon | Gomez | Lopez | Macalino | Nostratis | Padilla | Reynon | Santos | Tan |Velasco
4E / 4F - 2018-2019
Page 531 of 920
LABOR REVIEW DIGEST
Atty. Joyrich Golangco

24. ERROL RAMIREZ, JULITO APAS, RICKY ROSELO AND ESTEBAN MISSION, JR. VS. POLYSON
INDUSTRIES, INC. AND WILSON S. YU
G.R. No. 207898 Date: October 19, 2016
NACHURA, J.

Doctrine: Any union officer who knowingly participates in an illegal strike and any worker or union officer who
knowingly participates in the commission of illegal acts during a strike may be declared to have lost his
employment status: Provided, That mere participation of a worker in a lawful strike shall not constitute sufficient
ground for termination of his employment, even if a replacement had been hired by the employer during such
lawful strike.

FACTS:
Petitioners were employees of Polyson and were officers of Obrero Pilipino (Obrero), the union of the employees
of Polyson. The instant case arose from a labor dispute, between herein petitioners and respondent corporation,
which was certified by the Secretary of the Department of Labor and Employment (DOLE) to the NLRC for
compulsory arbitration.
Polyson alleged that it received a notice of hearing from the DOLE with respect to the petition for certification
election filed by Obrero; Polyson, through counsel and management representative, met with the officers of
Obrero, led by the union president, herein petitioner Ramirez; Obrero asked that it be voluntarily recognized by
Polyson as the exclusive bargaining agent of the rank-and-file employees of Polyson, but the latter refused and
opted for a certification election; furious at such refusal, the Obrero officers threatened the management that the
union will show its collective strength in the coming days; Polyson then received a rush order from one of its
clients for the production of 100,000 pieces of plastic bags; the management of Polyson informed the operators of
its Cutting Section that they would be needing workers to work overtime because of the said order; the
supervisors approached the operators but were told that they would be unable to work overtime because they
have other commitments after their shift; the supervisors then requested that the operators set aside their time for
the following day to work beyond their regular shift; on June 8, 2011, five (5) operators indicated their desire to
work overtime; however, after their regular shift, three of the five workers did not work overtime which resulted in
the delay in delivery of the client's order and eventually resulted in the cancellation of the said order by reason of
such delay; when management asked the workers, who initially manifested their desire to work overtime, to
indicate in the time sheet the reason for their failure to do so, two of the three workers, namely, Leuland Visca
(Visca) and Samuel Tuting (Tuting) gave the same reason, to wit: "Ayaw nila/ng iba na mag-OT [overtime] ako";
the management then conducted an investigation and a hearing where Visca affirmed his previous claim that
petitioners were the ones who pressured him to desist from rendering overtime work; on even date, Tuting
executed a written statement claiming that herein petitioners induced or threatened them not to work overtime; the
management then gave notices to petitioners asking them to explain why no disciplinary action would be taken
against them; petitioners submitted their respective explanations to the management denying their liability; after
evaluation, the management informed petitioners that it has decided to terminate petitioners' employment on the
ground that they instigated an illegal concerted activity resulting in losses to the company.
Petitioners contended that they were terminated from their employment not because they induced or threatened
their co-employees not to render overtime work but because they established a union which sought to become the
exclusive bargaining agent of the rank-and-file employees of Polyson; that their termination was undertaken
without affording them substantive and procedural due process; and that Polyson is guilty of unfair labor practice.
Obrero filed a Notice of Strike with the National Conciliation and Mediation Board (NCMB) which was predicated
on various grounds, among which was the alleged illegal dismissal of herein petitioners. DOLE Secretary certified
the labor dispute to the NLRC for immediate compulsory arbitration where the parties were required to maintain
the status quo, in accordance with Article 263(g) of the Labor Code.
NLRC found that Polyson was able to present sufficient evidence to establish that petitioners' termination from
employment was for a valid cause, as they were found guilty of inducing or threatening their co-employees not to
render overtime work, and that petitioners' dismissal was in conformity with due process requirements.
Angeles| Bajana | Balladares | Brillantes | Briones | Cabansag | Callanta | Chua | David|

De Leon | Gomez | Lopez | Macalino | Nostratis | Padilla | Reynon | Santos | Tan |Velasco
4E / 4F - 2018-2019
Page 532 of 920
LABOR REVIEW DIGEST
Atty. Joyrich Golangco

ISSUE/S:
Whether petitioners' dismissal from their employment was valid

HELD:
In the present case, petitioners failed to convince this Court that the NLRC's findings that they instigated the
slowdown on June 8, 2011 are not reinforced by substantial evidence. Verily, said findings have to be maintained
and upheld.
The Court agrees with both the NLRC and the CA that petitioners are guilty of instigating their co-employees to
commit slowdown, an inherently and essentially illegal activity even in the absence of a no-strike clause in a
collective bargaining contract, or statute or rule.
The Court is not persuaded by petitioners' contention that they are not guilty of "illegal concerted activity" as they
claim that this term contemplates a "careful planning of a considerable number of participants to insure that the
desired result is attained." Nothing in the law requires that a slowdown be carefully planned and that it be
participated in by a large number of workers. The essence of this kind of strike is that the workers do not quit their
work but simply reduce the rate of work in order to restrict the output or delay the production of the employer.
As to petitioners' liability, the second paragraph of Article 264(a) of the Labor Code provides:

x x x Any union officer who knowingly participates in an illegal strike and any worker or union officer who
knowingly participates in the commission of illegal acts during a strike may be declared to have lost his
employment status: Provided, That mere participation of a worker in a lawful strike shall not constitute sufficient
ground for termination of his employment, even if a replacement had been hired by the employer during such
lawful strike.

———————————————————————

Article 292 (b)

1. ST. LUKES MEDICAL CENTER, INC. and ROBERT KUAN, Chairman vs. ESTRELITO NOTARIO
G.R. No. 152166 October 20, 2010

DOCTRINE: Where the dismissal was without just cause and there was no due process, Article 279 of the Labor
Code, as amended, mandates that the employee is entitled to reinstatement without loss of seniority rights and
other privileges and full backwages, inclusive of allowances and other benefits, or their monetary equivalent
computed from the time the compensation was not paid up to the time of actual reinstatement.
FACTS:
St. Luke’s employed respondent as In-House Security Guard, mainly monitoring the CCTVs in the hospital.
In December 1996, a foreigner from Marshall Island, Justin Tibon, reported that he lost his mint green travelling
bag, placed inside the cabinet of the room where his daughter was confined. It contained, among others, two (2)
Continental Airlines tickets, two (2) passports, and some clothes. When he brought this incident to security, the
cameras failed to record any theft incident at room 257.
In his letter dated January 6, 1997, respondent explained that on the subject dates, he was the only personnel on
duty as nobody wanted to assist him. Because of this, he decided to focus the cameras on the Old and New
Maternity Units, as these two units have high incidence of crime. Finding the written explanation to be
unsatisfactory, St. Luke’s served on respondent a copy of the Notice of Termination, dismissing him on the ground
of gross negligence/inefficiency under Section 1, Rule VII of its Code of Discipline.
Thus respondent filed a Complaint for illegal dismissal against petitioners, seeking reinstatement with payment of
full backwages from the time of his dismissal up to actual reinstatement, without loss of seniority rights and other
benefits.

Angeles| Bajana | Balladares | Brillantes | Briones | Cabansag | Callanta | Chua | David|

De Leon | Gomez | Lopez | Macalino | Nostratis | Padilla | Reynon | Santos | Tan |Velasco
4E / 4F - 2018-2019
Page 533 of 920
LABOR REVIEW DIGEST
Atty. Joyrich Golangco

LA dismissed respondents’ complaint, stating that a CCTV monitoring system is designed to focus on many areas
in a programmed and sequential manner and should not to be focused only on a specific area, unless the situation
requires it.
NLRC reversed the LA, stating that petitioners failed to submit proof that there was an existing Standard
Operating Procedure (SOP) in the CCTV monitoring system, particularly on the focusing procedure.
CA dismissed petitioners’ petition for certiorari, affirming that respondent’s negligence does not constitute
sufficient ground to terminate him. Moreover, it declared that petitioners failed to comply with the twin notice rule
and hearing as what they did was to require respondent to submit a written explanation, within 24 hours and,
thereafter, he was ordered dismissed, without affording him an opportunity to be heard.

ISSUE: WoN respondent was illegally dismissed.

HELD: Contrary to the stance of petitioners, respondent was illegally dismissed without just cause and compliance
with the notice requirement.
Article 282 (b) of the Labor Code provides that an employer may terminate an employment for gross and habitual
neglect by the employee of his duties. To effectuate a valid dismissal, the Labor two requirements must be met:
(1) the dismissal must be for any of the causes provided in Article 282 of the Labor Code; and (2) the employee
must be given an opportunity to be heard and defend himself. An employer can terminate the services of an
employee only for valid and just causes which must be supported by clear and convincing evidence.
A perusal of petitioner hospital’s CCTV Monitoring Guidelines, disseminated to all in-house security personnel,
reveals that that there is no categorical provision requiring an in-house security personnel to observe a rotation
sequence procedure in focusing the cameras so that the security monitoring would cover as many areas as
possible.
Petitioners question the findings of the CA that there was no compliance with the twin-notice rule and hearing,
while respondent maintains that they violated his right to due process. The employee must be furnished two
———————————————————————————————————————-

Angeles| Bajana | Balladares | Brillantes | Briones | Cabansag | Callanta | Chua | David|

De Leon | Gomez | Lopez | Macalino | Nostratis | Padilla | Reynon | Santos | Tan |Velasco
4E / 4F - 2018-2019
Page 534 of 920
LABOR REVIEW DIGEST
Atty. Joyrich Golangco

2. ARMANDO ALILING vs. WIDE WIDE WORLD EXPRESS CORP.


G.R. No. 185829 April 25, 2012
Ponente: Velasco, Jr. J.

Doctrine: Employees must be reminded that while probationary employees do not enjoy permanent status, they
enjoy the constitutional protection of security of tenure. They can only be terminated for cause or when they
otherwise fail to meet the reasonable standards made known to them by the employer at the time of their
engagement.
FACTS: Respondent Wide Wide World Express Corporation (WWWEC) offered to employ petitioner Armando
Aliling (Aliling) as "Account Executive (Seafreight Sales)," with the following compensation package: a monthly
salary of PhP 13,000, transportation allowance of PhP 3,000, clothing allowance of PhP 800, cost of living
allowance of PhP 500, each payable on a per month basis and a 14th month pay depending on the profitability
and availability of financial resources of the company. The offer came with a six (6)-month probation period
condition with this express caveat: "Performance during [sic] probationary period shall be made as basis for
confirmation to Regular or Permanent Status."
Training then started. However, instead of a Seafreight Sale assignment, WWWEC asked Aliling to handle
Ground Express (GX), a new company product launched on June 18, 2004 involving domestic cargo forwarding
service for Luzon. Marketing this product and finding daily contracts for it formed the core of Aliling’s new
assignment. Barely a month after, Manuel F. San Mateo III (San Mateo), WWWEC Sales and Marketing Director,
emailed Aliling to express dissatisfaction with the latter’s performance. Joseph R. Lariosa (Lariosa), Human
Resources Manager of WWWEC, asked Aliling to report to the Human Resources Department to explain his
absence taken without leave from September 20, 2004.
Aliling responded two days later. He denied being absent on the days in question, attaching to his reply-letter11 a
copy of his timesheet12 which showed that he worked from September 20 to 24, 2004. Aliling’s explanation came
with a query regarding the withholding of his salary corresponding to September 11 to 25, 2004.
Aliling wrote San Mateo stating: "Pursuant to your instruction on September 20, 2004, I hereby tender my
resignation effective October 15, 2004." While WWWEC took no action on his tender, Aliling nonetheless
demanded reinstatement and a written apology, claiming in a subsequent letter dated October 1, 200414 to
management that San Mateo had forced him to resign.
Lariosa’s response-letter of October 1, 2004, nformed Aliling that his case was still in the process of being
evaluated. On October 6, 2004, Lariosa again wrote, this time to advise Aliling of the termination of his services
effective as of that date owing to his "non-satisfactory performance" during his probationary period. Earlier,
however, or on October 4, 2004, Aliling filed a Complaint17 for illegal dismissal due to forced resignation,
nonpayment of salaries as well as damages with the NLRC against WWWEC.

LA RULING: The LA ruled that the grounds upon which complainant’s dismissal was based did not conform not
only the standard but also the compliance required under Article 281 of the Labor Code, Necessarily,
complainant’s termination is not justified for failure to comply with the mandate the law requires. The labor arbiter
explained that Aliling cannot be validly terminated for non-compliance with the quota threshold absent a prior
advisory of the reasonable standards upon which his performance would be evaluated.
Both parties appealed the above decision to the NLRC, which affirmed the Decision in toto in its Resolution dated
May 31, 2007. The separate motions for reconsideration were also denied by the NLRC in its Resolution dated
August 31, 2007.
CA RULING: The CA anchored its assailed action on the strength of the following premises: (a) respondents failed
to prove that Aliling’s dismal performance constituted gross and habitual neglect necessary to justify his dismissal;
(b) not having been informed at the time of his engagement of the reasonable standards under which he will
qualify as a regular employee, Aliling was deemed to have been hired from day one as a regular employee; and
(c) the strained relationship existing between the parties argues against the propriety of reinstatement.
Aliling’s motion for reconsideration was rejected by the CA through the assailed Resolution dated December 15,
2008. Hence, the instant petition.
Angeles| Bajana | Balladares | Brillantes | Briones | Cabansag | Callanta | Chua | David|

De Leon | Gomez | Lopez | Macalino | Nostratis | Padilla | Reynon | Santos | Tan |Velasco
4E / 4F - 2018-2019
Page 535 of 920
LABOR REVIEW DIGEST
Atty. Joyrich Golangco

ISSUE: WHETHER OR NOT ALILING WAS ILLEGALLY DISMISSED.

HELD: YES. To justify fully the dismissal of an employee, the employer must, as a rule, prove that the dismissal
was for a just cause and that the employee was afforded due process prior to dismissal. As a complementary
principle, the employer has the onus of proving with clear, accurate, consistent, and convincing evidence the
validity of the dismissal. WWWEC had failed to discharge its twin burden in the instant case.
First off, the attendant circumstances in the instant case aptly show that the issue of petitioner’s alleged failure to
achieve his quota, as a ground for terminating employment, strikes the Court as a mere afterthought on the part of
WWWEC. Consider: Lariosa’s letter of September 25, 2004 already betrayed management’s intention to dismiss
the petitioner for alleged unauthorized absences. Aliling was in fact made to explain and he did so satisfactorily.
But, lo and behold, WWWEC nonetheless proceeded with its plan to dismiss the petitioner for non-satisfactory
performance, although the corresponding termination letter dated October 6, 2004 did not even specifically state
Aliling’s "non-satisfactory performance," or that Aliling’s termination was by reason of his failure to achieve his set
quota. In fine, an employee’s failure to meet sales or work quotas falls under the concept of gross inefficiency,
which in turn is analogous to gross neglect of duty that is a just cause for dismissal under Article 282 of the Code.
However, in order for the quota imposed to be considered a valid productivity standard and thereby validate a
dismissal, management’s prerogative of fixing the quota must be exercised in good faith for the advancement of
its interest. The duty to prove good faith, however, rests with WWWEC as part of its burden to show that the
dismissal was for a just cause. WWWEC must show that such quota was imposed in good faith. This WWWEC
failed to do, perceptibly because it could not. The fact of the matter is that the alleged imposition of the quota was
a desperate attempt to lend a semblance of validity to Aliling’s illegal dismissal.
Respondent WWWEC miserably failed to prove the termination of petitioner was for a just cause nor was there
substantial evidence to demonstrate the standards were made known to the latter at the time of his engagement.
Hence, petitioner’s right to security of tenure was breached.
As earlier stated, to effect a legal dismissal, the employer must show not only a valid ground therefor, but also that
procedural due process has properly been observed. When the Labor Code speaks of procedural due process,
the reference is usually to the two (2)-written notice rule envisaged in Section 2 (III), Rule XXIII, Book V of the
Omnibus Rules Implementing the Labor Code. Here, the first and second notice requirements have not been
properly observed, thus tainting petitioner’s dismissal with illegality.
The adverted memo dated September 20, 2004 of WWWEC supposedly informing Aliling of the likelihood of his
termination and directing him to account for his failure to meet the expected job performance would have had
constituted the "charge sheet," sufficient to answer for the first notice requirement, but for the fact that there is no
proof such letter had been sent to and received by him. In fact, in his December 13, 2004 Complainant’s Reply
Affidavit, Aliling goes on to tag such letter/memorandum as fabrication. WWWEC did not adduce proof to show
that a copy of the letter was duly served upon Aliling. Clearly enough, WWWEC did not comply with the first notice
requirement.
Neither was there compliance with the imperatives of a hearing or conference. The Court need not dwell at length
on this particular breach of the due procedural requirement. Suffice it to point out that the record is devoid of any
showing of a hearing or conference having been conducted. On the contrary, in its October 1, 2004 letter to
Aliling, or barely five (5) days after it served the notice of termination, WWWEC acknowledged that it was still
evaluating his case. And the written notice of termination itself did not indicate all the circumstances involving the
charge to justify severance of employment.

——————————————————————————————————————-

Angeles| Bajana | Balladares | Brillantes | Briones | Cabansag | Callanta | Chua | David|

De Leon | Gomez | Lopez | Macalino | Nostratis | Padilla | Reynon | Santos | Tan |Velasco
4E / 4F - 2018-2019
Page 536 of 920
LABOR REVIEW DIGEST
Atty. Joyrich Golangco

3. PEREZ v PHILIPPINE TELEGRAPH AND TELEPHONE COMPANY


G.R. No. 152048 April 7, 2009

Doctrine:

Facts:
- Petitioners Felix B. Perez and Amante G. Doria were employed by respondent Philippine Telegraph and
Telephone Company (PT&T) as shipping clerk and supervisor, respectively, in PT&Ts Shipping Section
- The two were dismissed from the service for having falsified company documents.
- petitioners filed a complaint for illegal suspension and illegal dismissal.
- LABOR ARBITER DECISION:
o The labor arbiter found that the 30-day extension of petitioners suspension and their subsequent dismissal were
both illegal. He ordered respondents to pay petitioners their salaries during their 30-day illegal suspension, as well
as to reinstate them
- NLRC DECISION:
o National Labor Relations Commission (NLRC) reversed the decision of the labor arbiter. It ruled that petitioners
were dismissed for just cause, that they were accorded due process
- CA DECISION:
o Petitioners appealed to the Court of Appeals (CA). In its decision, the CA affirmed the NLRC decision insofar as
petitioners illegal suspension for 15 days and dismissal for just cause were concerned. However, it found that
petitioners were dismissed without due process.
-

ISSUE:
- Petitioners likewise contended that due process was not observed in the absence of a hearing in which they
could have explained their side and refuted the evidence against them.

HELD:
· RULING RELATED TO ARTICLE 292 (b) (formerly Article 277)
· In sum, the following are the guiding principles in connection with the hearing requirement in dismissal cases:
(a) ample opportunity to be heard means any meaningful opportunity (verbal or written) given to
the employee to answer the charges against him and submit evidence in support of his defense, whether
in a hearing, conference or some other fair, just and reasonable way.
(b) a formal hearing or conference becomes mandatory only when requested by the employee
in writing or substantial evidentiary disputes exist or a company rule or practice requires it, or when
similar circumstances justify it.
(c) the ample opportunity to be heard standard in the Labor Code prevails over the hearing or
conference requirement in the implementing rules and regulations.

· LONGER VERSION OF THE DECISION


· There is no need for a hearing or conference. We note a marked difference in the standards of due process to
be followed as prescribed in the Labor Code and its implementing rules.
· The Labor Code, on one hand, provides that an employer must provide the employee ample opportunity to be
heard and to defend himself with the assistance of his representative if he so desires:
o ART. 277. Miscellaneous provisions. x x x
(b) Subject to the constitutional right of workers to security of tenure and their right to be
protected against dismissal except for a just and authorized cause and without prejudice to the
requirement of notice under Article 283 of this Code, the employer shall furnish the worker whose
employment is sought to be terminated a written notice containing a statement of the causes for
termination and shall afford the latter ample opportunity to be heard and to defend himself with the
Angeles| Bajana | Balladares | Brillantes | Briones | Cabansag | Callanta | Chua | David|

De Leon | Gomez | Lopez | Macalino | Nostratis | Padilla | Reynon | Santos | Tan |Velasco
4E / 4F - 2018-2019
Page 537 of 920
LABOR REVIEW DIGEST
Atty. Joyrich Golangco

assistance of his representative if he so desires in accordance with company rules and regulations
promulgated pursuant to guidelines set by the Department of Labor and Employment. Any decision
taken by the employer shall be without prejudice to the right of the worker to contest the validity or
legality of his dismissal by filing a complaint with the regional branch of the National Labor Relations
Commission. The burden of proving that the termination was for a valid or authorized cause shall
rest on the employer. (emphasis supplied)
· The omnibus rules implementing the Labor Code, on the other hand, require a hearing and conference during
which the employee concerned is given the opportunity to respond to the charge, present his evidence or rebut
the evidence presented against him:
o Section 2. Security of Tenure. x x x
(d) In all cases of termination of employment, the following standards of due process shall
be substantially observed:
For termination of employment based on just causes as defined in Article 282 of the Labor
Code:
(i) A written notice served on the employee specifying the ground or grounds for termination,
and giving said employee reasonable opportunity within which to explain his side.
(ii) A hearing or conference during which the employee concerned, with the assistance of
counsel if he so desires, is given opportunity to respond to the charge, present his evidence or rebut the
evidence presented against him.
(iii) A written notice of termination served on the employee, indicating that upon due
consideration of all the circumstances, grounds have been established to justify his termination.
(emphasis supplied)
· Which one should be followed? Is a hearing (or conference) mandatory in cases involving the dismissal of an
employee? Can the apparent conflict between the law and its IRR be reconciled?
· At the outset, we reaffirm the time-honored doctrine that, in case of conflict, the law prevails over the
administrative regulations implementing it.
· The authority to promulgate implementing rules proceeds from the law itself. To be valid, a rule or regulation
must conform to and be consistent with the provisions of the enabling statute. As such, it cannot amend the law
either by abridging or expanding its scope.
· Article 277(b) of the Labor Code provides that, in cases of termination for a just cause, an employee must be
given ample opportunity to be heard and to defend himself. Thus, the opportunity to be heard afforded by law to
the employee is qualified by the word ample which ordinarily means considerably more than adequate or
sufficient.
· In this regard, the phrase ample opportunity to be heard can be reasonably interpreted as extensive enough to
cover actual hearing or conference. To this extent, Section 2(d), Rule I of the Implementing Rules of Book VI of
the Labor Code is in conformity with Article 277(b).
· Nonetheless, Section 2(d), Rule I of the Implementing Rules of Book VI of the Labor Code should not be taken to
mean that holding an actual hearing or conference is a condition sine qua non for compliance with the due
process requirement in termination of employment. The test for the fair procedure guaranteed under Article 277(b)
cannot be whether there has been a formal pretermination confrontation between the employer and the employee.
The ample opportunity to be heard standard is neither synonymous nor similar to a formal hearing. To confine the
employees right to be heard to a solitary form narrows down that right. It deprives him of other equally effective
forms of adducing evidence in his defense. Certainly, such an exclusivist and absolutist interpretation is overly
restrictive. The very nature of due process negates any concept of inflexible procedures universally applicable to
every imaginable situation.
· The standard for the hearing requirement, ample opportunity, is couched in general language revealing the
legislative intent to give some degree of flexibility or adaptability to meet the peculiarities of a given situation. To
confine it to a single rigid proceeding such as a formal hearing will defeat its spirit.
· Significantly, Section 2(d), Rule I of the Implementing Rules of Book VI of the Labor Code itself provides that the
so-called standards of due process outlined therein shall be observed substantially, not strictly. This is a
Angeles| Bajana | Balladares | Brillantes | Briones | Cabansag | Callanta | Chua | David|

De Leon | Gomez | Lopez | Macalino | Nostratis | Padilla | Reynon | Santos | Tan |Velasco
4E / 4F - 2018-2019
Page 538 of 920
LABOR REVIEW DIGEST
Atty. Joyrich Golangco

recognition that while a formal hearing or conference is ideal, it is not an absolute, mandatory or exclusive avenue
of due process.
· An employees right to be heard in termination cases under Article 277(b) as implemented by Section 2(d), Rule I
of the Implementing Rules of Book VI of the Labor Code should be interpreted in broad strokes. It is satisfied not
only by a formal face to face confrontation but by any meaningful opportunity to controvert the charges against
him and to submit evidence in support thereof.
· A hearing means that a party should be given a chance to adduce his evidence to support his side of the case
and that the evidence should be taken into account in the adjudication of the controversy. To be heard does not
mean verbal argumentation alone inasmuch as one may be heard just as effectively through written explanations,
submissions or pleadings. Therefore, while the phrase ample opportunity to be heard may in fact include an actual
hearing, it is not limited to a formal hearing only. In other words, the existence of an actual, formal trial-type
hearing, although preferred, is not absolutely necessary to satisfy the employees right to be heard.
· This Court has consistently ruled that the due process requirement in cases of termination of employment does
not require an actual or formal hearing. Thus, we categorically declared in Skippers United Pacific, Inc. v.
Maguad:[25]
o The Labor Code does not, of course, require a formal or trial type proceeding before an erring employee may be
dismissed. (emphasis supplied)
· In Autobus Workers Union v. NLRC,[26] we ruled:
The twin requirements of notice and hearing constitute the essential elements of due process.
Due process of law simply means giving opportunity to be heard before judgment is rendered. In fact,
there is no violation of due process even if no hearing was conducted, where the party was given a
chance to explain his side of the controversy. What is frowned upon is the denial of the opportunity to be
heard.
xxxxxxxx
A formal trial-type hearing is not even essential to due process. It is enough that the parties are
given a fair and reasonable opportunity to explain their respective sides of the controversy and to present
supporting evidence on which a fair decision can be based. This type of hearing is not even mandatory in
cases of complaints lodged before the Labor Arbiter. (emphasis supplied)
· In Solid Development Corporation Workers Association v. Solid Development Corporation, we had the occasion
to state:
o [W]ell-settled is the dictum that the twin requirements of notice and hearing constitute the essential elements of
due process in the dismissal of employees. It is a cardinal rule in our jurisdiction that the employer must furnish
the employee with two written notices before the termination of employment can be effected: (1) the first apprises
the employee of the particular acts or omissions for which his dismissal is sought; and (2) the second informs the
employee of the employers decision to dismiss him. The requirement of a hearing, on the other hand, is complied
with as long as there was an opportunity to be heard, and not necessarily that an actual hearing was conducted.
o In separate infraction reports, petitioners were both apprised of the particular acts or omissions constituting the
charges against them. They were also required to submit their written explanation within 12 hours from receipt of
the reports. Yet, neither of them complied. Had they found the 12-hour period too short, they should have
requested for an extension of time. Further, notices of termination were also sent to them informing them of the
basis of their dismissal. In fine, petitioners were given due process before they were dismissed. Even if no hearing
was conducted, the requirement of due process had been met since they were accorded a chance to explain their
side of the controversy. (emphasis supplied)
· Our holding in National Semiconductor HK Distribution, Ltd. v. NLRC is of similar import:
· That the investigations conducted by petitioner may not be considered formal or recorded hearings or
investigations is immaterial. A formal or trial type hearing is not at all times and in all instances essential to due
process, the requirements of which are satisfied where the parties are afforded fair and reasonable opportunity to
explain their side of the controversy. It is deemed sufficient for the employer to follow the natural sequence of
notice, hearing and judgment.

Angeles| Bajana | Balladares | Brillantes | Briones | Cabansag | Callanta | Chua | David|

De Leon | Gomez | Lopez | Macalino | Nostratis | Padilla | Reynon | Santos | Tan |Velasco
4E / 4F - 2018-2019
Page 539 of 920
LABOR REVIEW DIGEST
Atty. Joyrich Golangco

· The above rulings are a clear recognition that the employer may provide an employee with ample opportunity to
be heard and defend himself with the assistance of a representative or counsel in ways other than a formal
hearing. The employee can be fully afforded a chance to respond to the charges against him, adduce his
evidence or rebut the evidence against him through a wide array of methods, verbal or written.
· After receiving the first notice apprising him of the charges against him, the employee may submit a written
explanation (which may be in the form of a letter, memorandum, affidavit or position paper) and offer evidence in
support thereof, like relevant company records (such as his 201 file and daily time records) and the sworn
statements of his witnesses. For this purpose, he may prepare his explanation personally or with the assistance of
a representative or counsel. He may also ask the employer to provide him copy of records material to his defense.
His written explanation may also include a request that a formal hearing or conference be held. In such a case,
the conduct of a formal hearing or conference becomes mandatory, just as it is where there exist substantial
evidentiary disputes or where company rules or practice requires an actual hearing as part of employment
pretermination procedure. To this extent, we refine the decisions we have rendered so far on this point of law.
· This interpretation of Section 2(d), Rule I of the Implementing Rules of Book VI of the Labor Code reasonably
implements the ample opportunity to be heard standard under Article 277(b) of the Labor Code without unduly
restricting the language of the law or excessively burdening the employer. This not only respects the power vested
in the Secretary of Labor and Employment to promulgate rules and regulations that will lay down the guidelines for
the implementation of Article 277(b). More importantly, this is faithful to the mandate of Article 4 of the Labor Code
that [a]ll doubts in the implementation and interpretation of the provisions of [the Labor Code], including its
implementing rules and regulations shall be resolved in favor of labor.
———————————————————————————————————————-

Angeles| Bajana | Balladares | Brillantes | Briones | Cabansag | Callanta | Chua | David|

De Leon | Gomez | Lopez | Macalino | Nostratis | Padilla | Reynon | Santos | Tan |Velasco
4E / 4F - 2018-2019
Page 540 of 920
LABOR REVIEW DIGEST
Atty. Joyrich Golangco

4. Distribution and Control Products Inc. v. Jeffrey Santos


G.R. No. 212616; Jul. 10, 2017
Peralta, J.:

DOCTRINE: Requisites of Procedural Due Process: 1.) Written notice of the cause and accusation; 2.) Hearing;
and 3.) Written notice of Termination and the causes therefor.

FACTS:

Herein Private Respondent Santos in this case was the company driver of Petitioner Distribution and Control
Products Inc. having worked with the company since 2005. In Dec. 2010 he received a notice informing him that
he was being preventively suspended for a period of 30 days, because of the suspicion of his participation in the
loss of a circuit breaker and several other electrical products from the petitioner’s warehouse amounting to
602,000php. This was discovered after an audit conducted by the Petitioner. Respondent Santos was the only
other person who had free access to the warehouse other than the warehouseman, and Petitioner theorized that
the theft could only have been done by no less than two persons- on of them being Santos.

Santos supposedly inquired with the Human Resources Department why he was being suspended as he was
never given an opportunity to explain his side, but he was not offered any. Instead he was no longer accepted at
work after the lapse of the preventive suspension. Thereafter, Santos filed an action for constructive illegal
dismissal and payment of separation pay.

LA(2012): The LA found that respondent was illegally terminated and ordered his reinstatement with full
backwages amounting to around 297,000php and held that the Petitioner had failed to prove that the dismissal
was valid.

NLRC: Affirmed the decision with modification ordering that Petitioner be paid separation pay amounting to one
month’s salary for each year of service rendered.

CA: Affirmed the NLRC

APPEAL TO THE SC:

Petitioner’s Contention: That the ruling of the CA infringed on the right of an employer to dismiss an employee..

ISSUE: Whether the dismissal was legal.

RULING: NO.

In determining whether an employee's dismissal had been legal, the inquiry focuses on whether the dismissal
violated his right to substantial and procedural due process. An employee's right not to be dismissed without just
or authorized cause as provided by law, is covered by his right to substantial due process. Compliance with
procedure provided in the Labor Code, on the other hand, constitutes the procedural due process right of an
employee.

The violation of either the substantial due process right or the procedural due process right of an employee
produces different results. Termination without a just or authorized cause renders the dismissal invalid, and
entitles the employee to reinstatement without loss of seniority rights and other privileges and full backwages,
Angeles| Bajana | Balladares | Brillantes | Briones | Cabansag | Callanta | Chua | David|

De Leon | Gomez | Lopez | Macalino | Nostratis | Padilla | Reynon | Santos | Tan |Velasco
4E / 4F - 2018-2019
Page 541 of 920
LABOR REVIEW DIGEST
Atty. Joyrich Golangco

inclusive of allowances, and other benefits or their monetary equivalent computed from the time the compensation
was not paid up to the time of actual reinstatement.

An employee's removal for just or authorized cause but without complying with the proper procedure, on the other
hand, does not invalidate the dismissal. It obligates the erring employer to pay nominal damages to the employee,
as penalty for not complying with the procedural requirements of due process.

Thus, two separate inquiries must be made in resolving illegal dismissal cases: first, whether the dismissal had
been made in accordance with the procedure set in the Labor Code; and second, whether the dismissal had been
for just or authorized cause.

As to whether or not respondent was afforded procedural due process, the settled rule is that in termination
proceedings of employees, procedural due process consists of the twin requirements of notice and hearing. The
employer must furnish the employee with two written notices before the termination of employment can be
effected: (1) the first apprises the employee of the particular acts or omissions for which his dismissal is sought;
and (2) the second informs the employee of the employer's decision to dismiss him. The requirement of a hearing
is complied with as long as there was an opportunity to be heard, and not necessarily that an actual hearing was
conducted.

In Unilever Philippines, Inc. v. Rivera, the Court laid down the guidelines on how to comply with procedural due
process in terminating an employee:

(1) The first written notice to be served on the employees should contain the specific causes or grounds for
termination against them, and a directive that the employees are given the opportunity to submit their written
explanation within a reasonable period. "Reasonable opportunity" under the Omnibus Rules means every kind of
assistance that management must accord to the employees to enable them to prepare adequately for their
defense.
(2) After serving the first notice, the employers should schedule and conduct a hearing or conference wherein the
employees will be given the opportunity to: (1) explain and clarify their defenses to the charge against them; (2)
present evidence in support of their defenses; and (3) rebut the evidence presented against them by the
management.
(3) After determining that termination of employment is justified, the employers shall serve the employees a
written notice of termination indicating that: (1) all circumstances involving the charge against the employees have
been considered; and (2) grounds have been established to justify the severance of their employment

In this case in particular, all the courts and bodies a quo uniformly ruled that the petitioner had failed to afford
Santos with procedural due process as nothing therein indicated that he was required nor was given the
opportunity to explain his side, considering that he was being implicated in the theft of the subject circuit breakers
and other electrical products. It is true that petitioners conducted their own investigation but the same was made
without the participation of respondent.

Petitioners allege that they did not terminate respondent from his employment and that it was the latter who
actually decided to abandon his job, but the same was not upheld by the lower courts and bodies. This being the
case, the Court found no cogent reason to depart from the rulings a quo.
———————————————————————————————————————-

Angeles| Bajana | Balladares | Brillantes | Briones | Cabansag | Callanta | Chua | David|

De Leon | Gomez | Lopez | Macalino | Nostratis | Padilla | Reynon | Santos | Tan |Velasco
4E / 4F - 2018-2019
Page 542 of 920
LABOR REVIEW DIGEST
Atty. Joyrich Golangco

Article 294

1. Jenny M. Agabon and Virgilio C. Agabon vs NLRC, Riviera Home Developments Improvements, INC,
and Vicente Angeles
GR No: 158693 Date: November 17, 2004
Ponente: Ynares-Santiago, J.

Doctrine:
Where the employer had a valid reason to dismiss an employee but did not follow the due process requirement,
the dismissal may be upheld but the employer will be penalized to pay an indemnity to the employee. This
became known as the Wenphil or Belated Due Process Rule.

FACTS:
Private respondent Riviera Home Improvements, Inc. is engaged in the business of selling and installing
ornamental and construction materials. It employed petitioners Virgilio Agabon and Jenny Agabon as gypsum
board and cornice installers on January 2, 1992 until February 23, 1999 when they were dismissed for
abandonment of work.
Petitioners then filed a complaint for illegal dismissal and payment of money claims with the labor arbiter.

LA/RTC/NLRC RULING:
LA: Declared the petitioners illegally dismissed and ordered the respondent to pay monetary claims of P 56,
231.93 each to Jenny and Virgilio

NLRC: Reversed the decision finding that petitioners abandoned their work and were not entitled to backwages
and separation pay. The other money claims awarded by the Labor Arbiter were also denied for lack of evidence.

CA RULING:
CA: Ruled that the dismissal of the petitioners was not illegal because they had abandoned their employment but
ordered the payment of money claims.

APPEAL TO THE SC:

Petitioner's Contention: Petitioners assert that they were dismissed because the private respondent refused to
give them assignments unless they agreed to work on a pakyaw basis. They did not agree on this arrangement
because it would mean losing benefits as Social Security System (SSS) members. Petitioners also claim that
private respondent did not comply with the twin requirements of notice and hearing.

Respondent's Contention: That petitioners were not dismissed but abandoned their work. Petitioners did not report
for work because they had subcontracted to perform installation work for another company.

ISSUE/S:
1. Whether or not petitioners were illegally dismissed

HELD:
1. Petitioners were dismissed for just causes but without due process hence sanctions must be imposed on the
employer.
To dismiss an employee, the law requires not only the existence of a just and valid cause but also enjoins the
employer to give the employee the opportunity to be heard and to defend himself. Article 282 of the Labor Code
enumerates the just causes for termination by the employer. Abandonment is the deliberate and unjustified refusal
Angeles| Bajana | Balladares | Brillantes | Briones | Cabansag | Callanta | Chua | David|

De Leon | Gomez | Lopez | Macalino | Nostratis | Padilla | Reynon | Santos | Tan |Velasco
4E / 4F - 2018-2019
Page 543 of 920
LABOR REVIEW DIGEST
Atty. Joyrich Golangco

of an employee to resume his employment. It is a form of neglect of duty, hence, a just cause for termination of
employment by the employer. For a valid finding of abandonment, these two factors should be present: (1) the
failure to report for work or absence without valid or justifiable reason; and (2) a clear intention to sever employer-
employee relationship, with the second as the more determinative factor which is manifested by overt acts from
which it may be deduced that the employees has no more intention to work. The intent to discontinue the
employment must be shown by clear proof that It was deliberate and unjustified.
In February 1999, petitioners were frequently absent having subcontracted for an installation work for another
company. Subcontracting for another company clearly showed the intention to sever the employer-employee
relationship with private respondent. This was not the first time they did this. Private respondent at that time
warned petitioners that they would be dismissed if this happened again. Petitioners disregarded the warning and
exhibited a clear intention to sever their employer-employee relationship. The record of an employee is a relevant
consideration in determining the penalty that should be meted out to him. In Sandoval Shipyard v. Clave, we held
that an employee who deliberately absented from work without leave or permission from his employer, for the
purpose of looking for a job elsewhere, is considered to have abandoned his job. We should apply that rule with
more reason here where petitioners were absent because they were already working in another company.
The dismissal should be upheld because it was established that the petitioners abandoned their jobs to work for
another company. Private respondent, however, did not follow the notice requirements and instead argued that
sending notices to the last known addresses would have been useless because they did not reside there
anymore. Unfortunately for the private respondent, this is not a valid excuse because the law mandates the twin
notice requirements to the employees last known address. Thus, it should be held liable for non-compliance with
the procedural requirements of due process. Where the employer had a valid reason to dismiss an employee but
did not follow the due process requirement, the dismissal may be upheld but the employer will be penalized to pay
an indemnity to the employee. This became known as the Wenphil or Belated Due Process Rule. However, this
doctrine was changed in the case of Serrano. It was held that held that the violation by the employer of the notice
requirement in termination for just or authorized causes was not a denial of due process that will nullify the
termination. However, the dismissal is ineffectual and the employer must pay full backwages from the time of
termination until it is judicially declared that the dismissal was for a just or authorized cause. The fact that the
Serrano ruling can cause unfairness and injustice which elicited strong dissent has prompted us to revisit the
doctrine. After carefully analyzing the consequences of the divergent doctrines in the law on employment
termination, we believe that in cases involving dismissals for cause but without observance of the twin
requirements of notice and hearing, the better rule is to abandon the Serrano doctrine and to follow Wenphil by
holding that the dismissal was for just cause but imposing sanctions on the employer.
Where the dismissal is for a just cause, as in the instant case, the lack of statutory due process should not nullify
the dismissal, or render it illegal, or ineffectual. However, the employer should indemnify the employee for the
violation of his statutory rights, as ruled in Reta v. National Labor Relations Commission. The indemnity to be
imposed should be stiffer to discourage the abhorrent practice of "dismiss now, pay later," which we sought to
deter in the Serrano ruling. The sanction should be in the nature of indemnification or penalty and should depend
on the facts of each case, taking into special consideration the gravity of the due process violation of the
employer.

Other Notes/ SC Pronouncements:

J PUNO Dissenting:
In the hierarchy of rights of an employees, the right to security of tenure is high, if not the highest.
In the case at bar, where petitioners Jenny Agabon and Virgilio Agabon were dismissed from the service for
abandonment of work without the due process requirements of two (2) notices and hearing, I submit that the
dismissals should be nullified and set aside, and petitioners immediately reinstated without loss of seniority rights
and other privileges. This Court should protect labor and it should walk the talk.

Angeles| Bajana | Balladares | Brillantes | Briones | Cabansag | Callanta | Chua | David|

De Leon | Gomez | Lopez | Macalino | Nostratis | Padilla | Reynon | Santos | Tan |Velasco
4E / 4F - 2018-2019
Page 544 of 920
LABOR REVIEW DIGEST
Atty. Joyrich Golangco

Accordingly, I vote for the immediate REINSTATEMENT of petitioners Jenny M. Agabon and Virgilio C. Agabon,
without loss of their seniority rights and other privileges and with full backwages, and the REVERSION to the pre-
Wenphil Doctrine in resolving future labor cases.
J Panganiban:
J Panganiban believes that even if there was just or authorized cause for termination of employment, but due
process was not afforded the employee, the dismissal proceedings must be declared null and void. Consequently,
the employee must be reinstated and given full back wages and accruing benefits. Depending on the facts of each
case, damages as provided under applicable articles of the Civil Code may additionally be awarded.
An exception may be entertained if the employer could adequately prove that under
the peculiar circumstances of the case, there was no opportunity to comply with due process requirements; or
doing so would have been impractical or gravely adverse to the employer, as when the employee was caught in
Kagrante delicto. Under such circumstances, dismissal would not be illegal, and no award may properly be
granted.

J. Tinga Concurring:
HOLD that henceforth, dismissals for just cause may not be invalidated due to the failure to observe the due
process requirements under the Labor Code, and that the only indemnity award available to the employee
dismissed for just cause are damages under the Civil Code as duly proven. Any and all previous rulings and
statements of the Court inconsistent with this holding are now deemed INOPERATIVE.
N. B.: Mas mahaba pa yung dissenting and concurring opinions. I suggest basahin sya ng full text if may time,
divided talaga yung opinion nila.
———————————————————————————————————————-

Angeles| Bajana | Balladares | Brillantes | Briones | Cabansag | Callanta | Chua | David|

De Leon | Gomez | Lopez | Macalino | Nostratis | Padilla | Reynon | Santos | Tan |Velasco
4E / 4F - 2018-2019
Page 545 of 920
LABOR REVIEW DIGEST
Atty. Joyrich Golangco

2. JAKA FOOD PROCESSING CORP. v. DARWIN PACOT


GR No: 151378 Date: March 28, 2005
Ponente: Garcia, J.

Doctrines:
Where the ground for respondent’s dismissal is one of the authorized causes enumerated under Article 283 of the
Labor Code i.e., retrenchment, and it is likewise established that the employer failed to comply with the notice
requirement under the same Article, it is deemed proper to fix the indemnity at P50,000.00.

If the dismissal is based on an authorized cause under Article 283 but the employer failed to comply with the
notice requirement, the sanction should be stiffer because the dismissal process was initiated by the employer’s
exercise of his management prerogative. This is in comparison with the Agabon case where the employer was
required to pay the dismissed employees based on a just cause under Article 282 the amount of P30,000
representing nominal damages.

The dismissed employees are not entitled to separation pay when the closure of business or cessation of
operations is due to serious business losses or financial reverses duly proved.

Where the dismissal is for a cause, the lack of statutory due process should not nullify the dismissal, or render it
illegal, or ineffectual. However, the employer should indemnify the employee for the violation of his statutory
rights.

FACTS:
Respondents Darwin Pacot, Robert Parohinog, David Bisnar, Marlon Domingo, Rhoel Lescano and Jonathan
Cagabcab were earlier hired by petitioner JAKA Foods Processing Corporation (JAKA) until the latter terminated
their employment because the corporation was in dire financial straits. It is not disputed, however, that the
termination was effected without JAKA complying with the requirement under Article 283 of the Labor Code
regarding the service of a written notice upon the employees and the Department of Labor and Employment at
least one (1) month before the intended date of termination.

In time, respondents separately filed with the regional Arbitration Branch of the NLRC complaints for illegal
dismissal, underpayment of wages and nonpayment of service incentive leave and 13th month pay against JAKA
and its HRD Manager, Rosana Castelo.

LA/NLRC RULING:
The LA rendered a decision declaring the termination illegal and ordering JAKA and its HRD Manager to reinstate
respondents with full backwages, and separation pay if reinstatement is not possible.

On appeal, the NLRC affirmed in toto the LA ruling. Acting on a motion for reconsideration, the NLRC modified the
LA decision, and reversed and set aside the awards of backwages, service incentive leave pay. Each complainant
shall be entitled to a separation pay and indemnification.

CA RULING:
Applying the doctrine laid down in Serrano v. NLRC, the CA reversed and set aside the NLRC decision, ordering
respondent JAKA to pay petitioners separation pay, proportionate 13th month pay and full backwages from the
time their employment was terminated up to the time the Decision herein becomes final.

ISSUE:
What are the legal implications of a situation where an employee is dismissed for cause but such dismissal was
effected without the employer’s compliance with the notice requirement under the Labor Code?
Angeles| Bajana | Balladares | Brillantes | Briones | Cabansag | Callanta | Chua | David|

De Leon | Gomez | Lopez | Macalino | Nostratis | Padilla | Reynon | Santos | Tan |Velasco
4E / 4F - 2018-2019
Page 546 of 920
LABOR REVIEW DIGEST
Atty. Joyrich Golangco

1. Whether respondents are entitled to nominal damages.


2. Whether respondents are entitled to separation pay.

HELD:
1. YES. It is established that there was ground for respondent’s dismissal, i.e., retrenchment, which is one of the
authorized causes enumerated under Article 283 of the Labor Code. Likewise, it is established that JAKA failed to
comply with the notice requirement under the same Article. Considering the factual circumstances, it is deemed
proper to fix the indemnity at P50,000.00.

In the case of Agabon v. NLRC, the validity of the dismissal was upheld despite non-compliance with the notice
requirement of the Labor Code. However, the employer was required to pay the dismissed employees the amount
of P30,000 representing nominal damages for non-compliance with statutory due process.

The difference between Agabon and the instant case is that in the former, the dismissal was based on a just
cause under Article 282 of the Labor Code while in the present case, respondents were dismissed due to
retrenchment, which is one of the authorized causes under Article 283 of the same Code.

A dismissal for just cause under Article 282 implies that the employee concerned has committed, or is guilty of,
some violation against the employer, i.e. the employee has committed some serious misconduct, is guilty of some
fraud against the employer, or, as in Agabon, he has neglected his duties. Thus, it can be said that the employee
himself initiated the dismissal process.

On another breath, a dismissal for an authorized cause under Article 283 does not necessarily imply delinquency
or culpability on the part of the employee. Instead, the dismissal process is initiated by the employer’s exercise of
his management prerogative, i.e. when the employer opts to install labor saving devices, when he decides to
cease business operations or when, as in this case, he undertakes to implement a retrenchment program.

The clear-cut distinction between a dismissal for just cause under Article 282 and a dismissal for authorized cause
under Article 283 is further reinforced by the fact that in the first, payment of separation pay, as a rule, is not
required, while in the second, the law requires payment of separation pay.

Accordingly: (1) if the dismissal is based on a just cause under Article 282 but the employer failed to comply with
the notice requirement, the sanction to be imposed upon him should be tempered because the dismissal process
was, in effect, initiated by an act imputable to the employee; and (2) if the dismissal is based on an authorized
cause under Article 283 but the employer failed to comply with the notice requirement, the sanction should
be stiffer because the dismissal process was initiated by the employer’s exercise of his management prerogative.

2. NO. As a rule, in all cases of business closure or cessation of operation or undertaking of the employer, the
affected employee is entitled to separation pay. This is consistent with the state policy of treating labor as a
primary social economic force, affording full protection to its rights as well as its welfare. The exception is when
the closure of business or cessation of operations is due to serious business losses or financial reverses; duly
proved, in which case, the right of affected employees to separation pay is lost for obvious reasons.

Other Notes/ SC Pronouncements:


Separate Opinion of J. Tinga:
I have reservations in declaring as valid, dismissals for authorized cause wherein the employer failed to render the
appropriate notices as required under Article 283 of the Labor Code. It cannot be discounted that Article 283
explicitly provides that the dismissal for authorized cause shall be effected by serving the notices to the
employees and to the Department of Labor and Employment thirty days before the effective date of termination.
Angeles| Bajana | Balladares | Brillantes | Briones | Cabansag | Callanta | Chua | David|

De Leon | Gomez | Lopez | Macalino | Nostratis | Padilla | Reynon | Santos | Tan |Velasco
4E / 4F - 2018-2019
Page 547 of 920
LABOR REVIEW DIGEST
Atty. Joyrich Golangco

There is no such unequivocal language used in the Labor Code provisions governing dismissals for just cause,
particularly Articles 282 and 277(b). Thus, in my Separate Opinion in Agabon v. NLRC, I wrote that the same rule
should not obtain given the obvious difference between the failure to comply with the notice requirement in
dismissals for just cause, on one hand, and the similar failure for dismissals for authorized cause, on the other.

Nonetheless, the present petition does not submit as an issue the validity of a dismissal for authorized cause in
the absence of such notices. Instead, the petition seeks modification of theSerrano doctrine insofar as it required
the payment of backwages starting from the date of termination up to finality of judgment. Given the importance of
the issue of whether such dismissals are valid in the first place, I would prefer to confront the issue in a more
appropriate case, one wherein the question is squarely raised and fully ventilated in the pleadings. In the
meantime, I find the ruling of the majority acceptable as an interim solution, until the time when the issue is
properly raised and thoroughly litigated before this Court.

———————————————————————————————————————-

Angeles| Bajana | Balladares | Brillantes | Briones | Cabansag | Callanta | Chua | David|

De Leon | Gomez | Lopez | Macalino | Nostratis | Padilla | Reynon | Santos | Tan |Velasco
4E / 4F - 2018-2019
Page 548 of 920
LABOR REVIEW DIGEST
Atty. Joyrich Golangco

3. Culili v. Eastern Telecom Phils.


GR No: 165381 Date: February 9, 2011
Ponente: LEONARDO-DE CASTRO, J.

Doctrine:
There is redundancy when the service capability of the workforce is greater than what is reasonably required to
meet the demands of the business enterprise. A position becomes redundant when it is rendered superfluous by
any number of factors such as over-hiring of workers, decrease in volume of business, or dropping a particular
product line or service activity previously manufactured or undertaken by the enterprise.

FACTS:
Petitioner Nelson A. Culili (Culili) was employed by ETPI as a Senior Technician in the Customer Premises
Equipment Management Unit of the Service Quality Department
Respondent Eastern Telecommunications Philippines, Inc. (ETPI) is a telecommunications company engaged
mainly in the business of establishing commercial telecommunications systems and leasing of international
datalines or circuits that pass through the international gateway facility (IGF).
Due to business troubles and losses, ETPI was compelled to implement a Right-Sizing Program which consisted
of two phases: the first phase involved the reduction of ETPIs workforce to only those employees that were
necessary and which ETPI could sustain; the second phase entailed a company-wide reorganization which would
result in the transfer, merger, absorption or abolition of certain departments of ETPI.
Among the departments abolished was the Service Quality Department. The functions of the Customer Premises
Equipment Management Unit, were absorbed by the Business and Consumer Accounts Department. The abolition
of the Service Quality Department rendered the specialized functions of a Senior Technician unnecessary. As a
result, Culilis position was abolished due to redundancy and his functions were absorbed by Andre Andrada,
another employee already with the Business and Consumer Accounts Department.
ETPI informed Culili of his termination from employment thru a letter.
Culili filed a complaint against ETPI and its officers for illegal dismissal, unfair labor practice, and money claims
before the Labor Arbiter. Culili alleged that neither he nor the Department of Labor and Employment (DOLE) were
formally notified of his termination. Culili claimed that he only found out about it sometime in March 1999 when
Vice President Virgilio Garcia handed him a copy of the letter, after he was barred from entering ETPIs premises
by its armed security personnel when he tried to report for work.

LA Ruling:
LA rendered a decision finding ETPI guilty of illegal dismissal and unfair labor practice and ordered ETPI to pay
Culili a total amount of P2,744,379.41. LA did not believe that Culili’s functions were as limited as ETPI made it
appear to be, and held that ETPI failed to present any reasonable criteria to justify the declaration of Culili’s
position as redundant.

NLRC Ruling:
On appeal, the NLRC affirmed the Labor Arbiters decision but modified the amount of moral and exemplary
damages awarded.

CA RULING:
CA partially granted the appeal and ordered ETPI to pay Culili full backwages from the time his salaries were not
paid until the finality of this Decision plus separation pay in an amount equivalent to one (1) month salary for every
year of service. CA found that Culilis position was validly abolished due to redundancy. Further, the CA found that
ETPI failed to observe the standards of due process as required by our laws when it failed to properly notify both
Culili and the DOLE of Culili’s termination.

APPEAL TO THE SC:


Angeles| Bajana | Balladares | Brillantes | Briones | Cabansag | Callanta | Chua | David|

De Leon | Gomez | Lopez | Macalino | Nostratis | Padilla | Reynon | Santos | Tan |Velasco
4E / 4F - 2018-2019
Page 549 of 920
LABOR REVIEW DIGEST
Atty. Joyrich Golangco

Petitioner’s contention:
Culili asserted that he was illegally dismissed because there was no valid cause to terminate his employment. He
claimed that ETPI failed to prove that his position had become redundant and that ETPI was indeed incurring
losses. Culili further alleged that his functions as a Senior Technician could not be considered a superfluity
because his tasks were crucial and critical to ETPIs business.

Respondent’s contention:
ETPI averred that since Culili did not avail of the Special Retirement Program and his position was subsequently
declared redundant, it had no choice but to terminate Culili.

ISSUE/S:
1. Whether there was a valid cause to terminate Culili
2. Whether ETPI complied with procedural due process for termination

HELD:
1. YES. Under our laws, an employee may be terminated for reasons involving measures taken by the employer
due to business necessities specifically Art 298 of the Labor Code. There is redundancy when the service
capability of the workforce is greater than what is reasonably required to meet the demands of the business
enterprise. A position becomes redundant when it is rendered superfluous by any number of factors such as over-
hiring of workers, decrease in volume of business, or dropping a particular product line or service activity
previously manufactured or undertaken by the enterprise. Among the requisites of a valid redundancy program
are: (1) the good faith of the employer in abolishing the redundant position; and (2) fair and reasonable criteria in
ascertaining what positions are to be declared redundant, such as but not limited to: preferred status, efficiency,
and seniority. In the case at bar, ETPI was upfront with its employees about its plan to implement a Right-Sizing
Program. In deciding which positions to retain and which to abolish, ETPI chose on the basis of efficiency,
economy, versatility and flexibility. The records show that ETPI had sufficiently established not only its need to
reduce its workforce and streamline its organization, but also the existence of redundancy in the position of a
Senior Technician. ETPI explained how it failed to meet its business targets and the factors that caused this, and
how this necessitated it to reduce its workforce and streamline its organization.

2. NO. Under Section 2, Rule I, Book VI of the Rules Implementing the Labor Code, For termination of
employment as defined in Article 298 of the Labor Code, the requirement of due process shall be deemed
complied with upon service of a written notice to the employee and the appropriate Regional Office of the
Department of Labor and Employment at least thirty days before effectivity of the termination, specifying the
ground or grounds for termination. In this case, ETPI does not deny its failure to provide DOLE with a written
notice regarding Culili’s termination. In view of ETPI’s failure to comply with the notice requirements under the
Labor Code, Culili is entitled to nominal damages in addition to his separation pay.

Other Notes/ SC Pronouncements:


In Jaka Food Processing Corporation v. Pacot,, the Court held that: (1) if the dismissal is based on a just cause
under Article 282(now 297) but the employer failed to comply with the notice requirement, the sanction to be
imposed upon him should be tempered because the dismissal process was, in effect, initiated by an act imputable
to the employee; and (2) if the dismissal is based on an authorized cause under Article 283(now 298) but the
employer failed to comply with the notice requirement, the sanction should be stiffer because the dismissal
process was initiated by the employer's exercise of his management prerogative.

———————————————————————————————————————-

Angeles| Bajana | Balladares | Brillantes | Briones | Cabansag | Callanta | Chua | David|

De Leon | Gomez | Lopez | Macalino | Nostratis | Padilla | Reynon | Santos | Tan |Velasco
4E / 4F - 2018-2019
Page 550 of 920
LABOR REVIEW DIGEST
Atty. Joyrich Golangco

4. ANTONIO M. SERRANO v. GALLANT MARITIME SERVICES, INC. and MARLOW NAVIGATION CO., INC.
G.R. No. 167614 Date: March 24, 2009
Ponente: AUSTRIA-MARTINEZ

DOCTRINE:
Section 3, Article 13 of the Constitution does not directly bestow on the working class any actual enforceable right,
but merely clothes it with the status of a sector for whom the Constitution urges protection through executive or
legislative action and judicial recognition. Its utility is best limited to being an impetus not just for the executive and
legislative departments, but for the judiciary as well, to protect the welfare of the working class.

FACTS:

Petitioner Antonio Serrano, a Filipino seafarer, was hired by respondents Gallant Maritime Services, Inc. and
Marlow Navigation Co., Ltd. as a Chief Officer. The terms and conditions of his POEA-Approved Contract of
Employment:
· Duration of contract - 12 months;
· Basic monthly salary - US$1,400.00
· Hours of work - 48.0 hours per week
· Overtime - US$700 per month
· Vacation leave with pay - 7.00 days per month

On his departure date, Serrano was constrained to accept a downgraded employment contract for Second Officer
with a monthly salary of US$1,0000.00, upon the assurance and representation by respondents that he would be
made Chief Officer. When Serrano refused he was repatriated to the Philippines on May 26, 1998, serving only 2
months and 7 days of his contract, leaving an unexpired portion of 9 months and 23 days.

Serrano filed before the LA a complaint for constructive dismissal and payment of his money claims and moral
and exemplary damages and attorney’s fees.

LA RULING:
Dismissal was illegal. Serrano was awarded the amount representing his salary for 3 months of the unexpired
portion of the contract.

NLRC RULING:
Serrano was only awarded a total of $4,669.50 reducing the applicable salary rate to US$1400 since RA 8042
does not provide for the award of OT pay, which should be proven to have been actually performed, and for
vacation leave pay.

CA RULING:
Affirmed the NLRC decision as to the applicable salary rate. M

Petitioner’s contention:
He assailed the constitutionality of the clause “or for three (3) months for every year of the unexpired term,
whichever is less” under Sec. 10 of RA 8042 on the ground that it violates Sec. 3, Art. 13 of the Constitution.

ISSUE:
Whether Section 3 of Art. 13 of the Constitution bestow an actual enforceable right?

RULING:

Angeles| Bajana | Balladares | Brillantes | Briones | Cabansag | Callanta | Chua | David|

De Leon | Gomez | Lopez | Macalino | Nostratis | Padilla | Reynon | Santos | Tan |Velasco
4E / 4F - 2018-2019
Page 551 of 920
LABOR REVIEW DIGEST
Atty. Joyrich Golangco

NO. While all the provisions of the 1987 Constitution are presumed self-executing, there are some which this
Court has declared not judicially enforceable, Article 13 being one, particularly Section 3 thereof, the nature of
which, this Court, in Agabon v. National Labor Relations Commission has described to be not self-actuating. Thus,
the constitutional mandates of protection to labor and security of tenure may be deemed as self-executing in the
sense that these are automatically acknowledged and observed without need for any enabling legislation.
However, to declare that the constitutional provisions are enough to guarantee the full exercise of the rights
embodied therein, and the realization of ideals therein expressed, would be impractical, if not unrealistic. The
espousal of such view presents the dangerous tendency of being overbroad and exaggerated. The guarantees of
“full protection to labor” and “security of tenure,” when examined in isolation, are facially unqualified, and the
broadest interpretation possible suggests a blanket shield in favor of labor against any form of removal regardless
of circumstance. This interpretation implies an unimpeachable right to continued employment—a utopian notion,
doubtless—but still hardly within the contemplation of the framers. Subsequent legislation is still needed to define
the parameters of these guaranteed rights to ensure the protection and promotion, not only the rights of the labor
sector, but of the employers’ as well. Without specific and pertinent legislation, judicial bodies will be at a loss,
formulating their own conclusion to approximate at least the aims of the Constitution.

Section 3, Article 13 of the Constitution does not directly bestow on the working class any actual enforceable right,
but merely clothes it with the status of a sector for whom the Constitution urges protection through executive or
legislative action and judicial recognition. Its utility is best limited to being an impetus not just for the executive and
legislative departments, but for the judiciary as well, to protect the welfare of the working class.

In Central Bank Employee Association, Inc. v. Bangko Sentral ng Pilipinas, the judicial precept that when the
challenge to a statute is premised on the perpetuation of prejudice against persons favored by the Constitution
with special protection—such as the working class or a section thereof—the Court may recognize the existence of
a suspect classification and subject the same to strict judicial scrutiny.

The subject clause does not state or imply any definitive governmental purpose; and it is for that precise reason
that the clause violates not just petitioner’s right to equal protection, but also her right to substantive due process
under Section 1, Article III of the Constitution. The subject clause being unconstitutional, petitioner is entitled to his
salaries for the entire unexpired period of nine months and 23 days of his employment contract, pursuant to law
and jurisprudence prior to the enactment of R.A. No. 8042.
———————————————————————————————————————-

Angeles| Bajana | Balladares | Brillantes | Briones | Cabansag | Callanta | Chua | David|

De Leon | Gomez | Lopez | Macalino | Nostratis | Padilla | Reynon | Santos | Tan |Velasco
4E / 4F - 2018-2019
Page 552 of 920
LABOR REVIEW DIGEST
Atty. Joyrich Golangco

5. CLAUDIO S. YAP, Petitioner v. THENAMARIS SHIPS MANAGEMENT and INTERMARE MARITIME


AGENCIES, INC., Respondents
GR No: 179532 Date: May 30, 2011
Ponente: NACHURA, J.

Doctrine: The clause or for three months for every year of the unexpired term, whichever is less provided in the
5th paragraph of Section 10 of R.A. No. 8042 is unconstitutional for being violative of the rights of Overseas
Filipino Workers (OFWs) to equal protection of the laws.

FACTS:
[Petitioner] Claudio S. Yap was employed as electrician of the vessel, M/T SEASCOUT on 14 August 2001 by
Intermare Maritime Agencies, Inc. in behalf of its principal, Vulture Shipping Limited. The contract of employment
entered into by Yap and Capt. Francisco B. Adviento, the General Manager of Intermare, was for a duration of 12
months.

On 23 August 2001, Yapboarded M/T SEASCOUT and commenced his job as electrician. However, on or about
08 November 2001, the vessel was sold. The Philippine Overseas Employment Administration (POEA) was
informed about the sale on 06 December 2001 in a letter signed by Capt. Adviento.Yap, along with the other
crewmembers, was informed by the Master of their vessel that the same was sold and will be scrapped.

Yap received his seniority bonus, vacation bonus, extra bonus along with the scrapping bonus. However, with
respect to the payment of his wage, he refused to accept the payment of one-month basic wage. He insisted that
he was entitled to the payment of the unexpired portion of his contract since he was illegally dismissed from
employment. He alleged that he opted for immediate transfer but none was made.

[Respondents], for their part, contended that Yap was not illegally dismissed. They alleged that following the sale
of the M/T SEASCOUT, Yap signed off from the vessel on 10 November 2001 and was paid his wages
corresponding to the months he worked or until 10 November 2001 plus his seniority bonus, vacation bonus and
extra bonus. They further alleged that Yaps employment contract was validly terminated due to the sale of the
vessel and no arrangement was made for Yap’s transfer to Thenamaris’ other vessels.

Thus, Claudio S. Yap (petitioner) filed a complaint for Illegal Dismissal with Damages and Attorneys’ Fees before
the Labor Arbiter (LA).

LA rendered a decision in favor of petitioner, finding the latter to have been constructively and illegally dismissed
by respondents. LA opined that since the unexpired portion of petitioner’s contract was less than one year,
petitioner was entitled to his salaries for the unexpired portion of his contract for a period of nine months.

NLRC affirmed the LAs findings that petitioner was indeed constructively and illegally dismissed; that respondents
bad faith was evident on their willful failure to transfer petitioner to another vessel; and that the award of attorney’s
fees was warranted. However, the NLRC held that instead of an award of salaries corresponding to nine months,
petitioner was only entitled to salaries for three months as provided under Section 10 of Republic Act (R.A.) No.
8042.

NLRC reversed its earlier Decision, holding that there can be no choice to grant only three (3) months’ salary for
every year of the unexpired term because there is no full year of unexpired term which this can be applied. Thus,
complainant is entitled to his salary for the unexpired portion of employment contract.

Angeles| Bajana | Balladares | Brillantes | Briones | Cabansag | Callanta | Chua | David|

De Leon | Gomez | Lopez | Macalino | Nostratis | Padilla | Reynon | Santos | Tan |Velasco
4E / 4F - 2018-2019
Page 553 of 920
LABOR REVIEW DIGEST
Atty. Joyrich Golangco

CA affirmed the findings and ruling of the LA and the NLRC that petitioner was constructively and illegally
dismissed. However, the CA ruled that the NLRC erred in sustaining the LAs interpretation of Section 10 of R.A.
No. 8042. In this regard, the CA relied on the clause or for three months for every year of the unexpired term,
whichever is less provided in the 5th paragraph of Section 10 of R.A. No. 8042

ISSUE/S:

1. Whether or not Section 10 of R.A. 8042, to the extent that it affords an illegally dismissed migrant worker the
lesser benefit of salaries for the unexpired portion of his employment contract or for three (3) months for every
year of the unexpired term, whichever is less is constitutional?

2. Whether or not the tanker allowance should not be included in the computation of the lump-sum salary to be
awarded to petitioner which Respondents only raised as issue during the hearing of the present case?

HELD:
1. NO. In the meantime, while this case was pending, the Court declared as unconstitutional the clause or for
three months for every year of the unexpired term, whichever is less provided in the 5th paragraph of Section 10
of R.A. No. 8042 in the case of Serrano v. Gallant Maritime Services, Inc. on March 24, 2009 for being violative of
the rights of Overseas Filipino Workers (OFWs) to equal protection of the laws.

The Court concludes that the subject clause contains a suspect classification in that, in the computation of the
monetary benefits of fixed-term employees who are illegally discharged, it imposes a 3-month cap on the claim of
OFWs with an unexpired portion of one year or more in their contracts, but none on the claims of other OFWs or
local workers with fixed-term employment. The subject clause singles out one classification of OFWs and burdens
it with a peculiar disadvantage.

Moreover, this Court held therein that the subject clause does not state or imply any definitive governmental
purpose; hence, the same violates not just therein petitioner’s right to equal protection, but also his right to
substantive due process under Section 1, Article III of the Constitution. Consequently, petitioner therein was
accorded his salaries for the entire unexpired period of nine months and 23 days of his employment contract,
pursuant to law and jurisprudence prior to the enactment of R.A. No. 8042.
Thus, the present case should not be different from Serrano.

2. NO. In the same vein, we cannot subscribe to respondents postulation that the tanker allowance of US$130.00
should not be included in the computation of the lump-sum salary to be awarded to petitioner. First. It is only at
this late stage, more particularly in their Memorandum, that respondents are raising this issue. It was not raised
before the LA, the NLRC, and the CA.

Second. Respondent’s invocation of Serrano is unavailing. Indeed, we made the following pronouncements in
Serrano, to wit: The word salaries in Section 10(5) does not include overtime and leave pay. For seafarers like
petitioner, DOLE Department Order No. 33, series 1996, provides a Standard Employment Contract of Seafarers,
in which salary is understood as the basic wage, exclusive of overtime, leave pay and other bonuses; whereas
overtime pay is compensation for all work performed in excess of the regular eight hours, and holiday pay is
compensation for any work performed on designated rest days and holidays.

A close perusal of the contract reveals that the tanker allowance of US$130.00 was not categorized as a bonus
but was rather encapsulated in the basic salary clause, hence, forming part of the basic salary of petitioner.
Respondents themselves in their petition for certiorari before the CA averred that petitioner’s basic salary,
pursuant to the contract, was US$1,300.00 + US$130.00 tanker allowance. If respondents intended it differently,

Angeles| Bajana | Balladares | Brillantes | Briones | Cabansag | Callanta | Chua | David|

De Leon | Gomez | Lopez | Macalino | Nostratis | Padilla | Reynon | Santos | Tan |Velasco
4E / 4F - 2018-2019
Page 554 of 920
LABOR REVIEW DIGEST
Atty. Joyrich Golangco

the contract per se should have indicated that said allowance does not form part of the basic salary or, simply, the
contract should have separated it from the basic salary clause.

———————————————————————————————————————-

Angeles| Bajana | Balladares | Brillantes | Briones | Cabansag | Callanta | Chua | David|

De Leon | Gomez | Lopez | Macalino | Nostratis | Padilla | Reynon | Santos | Tan |Velasco
4E / 4F - 2018-2019
Page 555 of 920
LABOR REVIEW DIGEST
Atty. Joyrich Golangco

6. BANK OF LUBAO VS. MANABAT


GR No: 188722 Date: February 1, 2012
Ponente: Reyes, J.

Doctrine:
Under the doctrine of strained relations, the payment of separation pay is considered an acceptable alternative to
reinstatement when the latter option is no longer desirable or viable.

Employees who are illegally dismissed are entitled to full backwages, inclusive of allowances and other benefits or
their monetary equivalent, computed from the time their actual compensation was withheld from them up to the
time of their actual reinstatement.

If reinstatement is no longer possible, the backwages shall be computed from the time of their illegal termination
up to the finality of the decision

FACTS:

Manabat is an encoder hired by petitioner bank. During his employment, he was found guilty of malversation of
funds. In an administrative hearing it was found through the Bank's audit that Manabat conspired with another
employee in making fraudulent entries disguised as error corrections in the bank's computer. A criminal case for
qualified theft was filed against Manabat. Subsequently, his employment was terminated due to serious
misconduct tantamount to willful breach of trust.

Respondent then filed a complaint for illegal dismissal when the trial court dismissed the case against him for lack
of sufficient basis.

LA RULING:
The LA found that there respondent was illegally dismissed and further ordered that he be reinstated to his former
position. The petitioner bank ordered the respondent to report for work in compliance to the LA order but the
respondent refused.

NLRC RULING:
Upon appeal, the NLRC affirmed the LA's decision.

CA RULING:
The petitioner filed a Petition for Certiorari with the CA which was denied. The CA however held that due to the
doctrine of strained relations, the respondent is entitled to separation pay in lieu of reinstatement and backwages
from the time of illegal dismissal up to the finality of the decision.

APPEAL TO THE SC:

Petitioner's Contention:
The CA erred in ordering that the petitioner pay the respondent separation pay in lieu of reinstatement because
the appeal involved only the issue of the respondent's dismissal. The CA erred in granting backwages because
petitioner was in good faith when it terminated respondent's employment.

Respondent's Contention:
The CA did not er in ordering the payment of separation pay since there is already a strained relationship between
him and the petitioner bank.

Angeles| Bajana | Balladares | Brillantes | Briones | Cabansag | Callanta | Chua | David|

De Leon | Gomez | Lopez | Macalino | Nostratis | Padilla | Reynon | Santos | Tan |Velasco
4E / 4F - 2018-2019
Page 556 of 920
LABOR REVIEW DIGEST
Atty. Joyrich Golangco

ISSUE/S:
1. Did the CA err in ordering the petitioner to pay the respondent separation pay in lieu of reinstatement? - No.
2. Is the respondent entitled to payment of backwages? - Yes

HELD:
1. No. Under the law and prevailing jurisprudence, an illegally dismissed employee is entitled to reinstatement as
a matter of right. However, if reinstatement would only exacerbate the tension and strained relations between the
parties, or where the relationship between the employer and the employee has been unduly strained by reason of
their irreconcilable differences, particularly where the illegally dismissed employee held a managerial or key
position in the company, it would be more prudent to order payment of separation pay instead of reinstatement.

Under the doctrine of strained relations, the payment of separation pay is considered an acceptable alternative to
reinstatement when the latter option is no longer desirable or viable. In such cases, it should be proved that the
employee concerned occupies a position where he enjoys the trust and confidence of his employer; and that it is
likely that if reinstated, an atmosphere of antipathy and antagonism may be generated as to adversely affect the
efficiency and productivity of the employee concerned.

Here, the SC agree with the CA that the relations between the parties had been already strained thereby justifying
the grant of separation pay in lieu of reinstatement in favor of the respondent.

2. Yes. Petitioner is entitled to payment of backwages. Employees who are illegally dismissed are entitled to full
backwages, inclusive of allowances and other benefits or their monetary equivalent, computed from the time their
actual compensation was withheld from them up to the time of their actual reinstatement. But if reinstatement is no
longer possible, the backwages shall be computed from the time of their illegal termination up to the finality of the
decision.

Thus, when there is an order of reinstatement, the computation of backwages shall be reckoned from the time of
illegal dismisssal up to the time that the employee is actually reinstated to his former position.

The fact that the CA, in its decision, ordered the payment of separation pay in lieu of the respondent's
reinstatement would not entitle the latter to backwages, The decisions of the CA, unlike that of the LA, are not
immediately executory. Accordingly, the petitioner should only pay the respondent backwages from the date of his
illegal dismissal up to the date when the petitioner required the former to report to work.

The petition is partially granted.


———————————————————————————————————————-

Angeles| Bajana | Balladares | Brillantes | Briones | Cabansag | Callanta | Chua | David|

De Leon | Gomez | Lopez | Macalino | Nostratis | Padilla | Reynon | Santos | Tan |Velasco
4E / 4F - 2018-2019
Page 557 of 920
LABOR REVIEW DIGEST
Atty. Joyrich Golangco

7. ST. MARYS ACADEMY OF DIPOLOG CITY vs. TERESITA PALACIO, MARIGEN CALIBOD, LEVIE
LAQUIO, ELAINE MARIE SANTANDER, ELIZA SAILE, and MA. DOLORES MONTEDERAMOS

G.R. No. 164913 DATE: September 8, 2010


PONENTE: DEL CASTILLO, J.:

DOCTRINE:
The Court will not hesitate to defend the workers constitutional right to security of tenure. After all, the interest of
the workers is paramount as they are regarded with compassion under the policy of social justice.

FACTS:
On different dates in the late 1990s, petitioner hired respondents as classroom teachers and guidance counselor.
In separate letters dated March 31, 2000, owever, petitioner informed them that their re-application for school
year 000-2001 could not be accepted because they failed to pass the LET.

According to petitioner, as non-board passers, respondents could not continue practicing their teaching profession
pursuant to the DECS’ Memorandum No. 10, S. 1998 which requires incumbent teachers to register as
professional teachers pursuant to Sec. 27 of R.A. No. 7836, otherwise known as the Philippine Teachers
Professionalization Act of 1994.

Respondents filed a complaint contesting their termination as highly irregular and premature. They argued that
their security of tenure could not simply be trampled upon for their failure to register with the PRC or to pass the
LET prior to the deadline (September 19, 2009) set by RA 7836.

As to the dismissal, the petitioner justifies respondents’ termination by advancing that it would be difficult to hire
licensed teachers in the middle of the school year as respondents’ replacements

As to the separation pay, the petitioner contends that assuming respondents were illegally dismissed, they are
only entitled to an amount computed from the time of dismissal up to September 19, 2000 only. After September
19, 2000, respondents, according to petitioner, are already dismissible for cause for lack of the necessary license
to teach.

LA RULING: The LA adjudged petitioner guilty of illegal dismissal because it terminated the services of the
respondents on March 31, 2000 which was clearly prior to the September 19, 2000 deadline fixed by PRC for the
registration of
teachers as professional teachers, in violation of the doctrine regarding the prospective application of laws.

Thus, petitioner was ordered to reinstate the respondents or to pay them separation pay at the rate of month wage
for every year of service, plus limited backwages covering the period from March 31, 2000 to September 30,
2000.

NLRC RULING:The NLRC affirmed the decision of the LA.j

CA RULING: The CA affirmed the decision of the LA and the NLRC.

ISSUES:
4. Is the dismissal not valid even if the same was made in order to protect the interest of the Employer?
4. Is the amount of separation pay awarded to respondents correct?
HELD:

Angeles| Bajana | Balladares | Brillantes | Briones | Cabansag | Callanta | Chua | David|

De Leon | Gomez | Lopez | Macalino | Nostratis | Padilla | Reynon | Santos | Tan |Velasco
4E / 4F - 2018-2019
Page 558 of 920
LABOR REVIEW DIGEST
Atty. Joyrich Golangco

1.YES, the dismissal is not valid. It is incumbent upon this Court to afford full protection to labor. Thus, while we
take cognizance of the employers’ right to protect its interest, the same should be exercised in a manner which
does not infringe on the workers right to security of tenure. Under the policy of social justice, the law bends over
backward to accommodate the interests of the working class on the humane justification that those with less
privilege in life should have more in law. To reiterate, this Court will not hesitate to defend respondents’ right to
security of tenure. The premature dismissal from the service of respondents is unwarranted.

2.Yes, the amount of separation pay awarded to respondents is correct. Petitioner cannot possibly presume that
respondents could not timely comply with the requirements of the law. At any rate, we note that petitioner only
assailed the amount of backwages for the first time in its motion for reconsideration of the Decision of the CA.
Thus, the Court cannot entertain the issue for being belatedly raised. Hence, the award of limited backwages
covering the period from March 31, 2000 to September 30, 2000 as ruled by the Labor Arbiter and affirmed by
both the NLRC and CA is in order.
———————————————————————————————————————-

Angeles| Bajana | Balladares | Brillantes | Briones | Cabansag | Callanta | Chua | David|

De Leon | Gomez | Lopez | Macalino | Nostratis | Padilla | Reynon | Santos | Tan |Velasco
4E / 4F - 2018-2019
Page 559 of 920
LABOR REVIEW DIGEST
Atty. Joyrich Golangco

8. TOYOTA MOTOR PHILS. CORP. WORKERS ASSOC. (TMPCWA) NLRC, TOYOTA MOTOR PHIL CORP et
al
G.R. Nos. 158786 & 158789
October 19, 2007

FACTS: The Union filed a petition for certification election among the Toyota rank and file employees with the
National Conciliation and Mediation Board (NCMB). The Med-Arbiter denied the petition, but, on appeal, the
DOLE Secretary granted the Union’s prayer, and, through an Order, directed the immediate holding of the
certification election.

After Toyota’s plea for reconsideration was denied, the certification election was conducted. The Med-Arbiter’s
Order certified the Union as the sole and exclusive bargaining agent of all the Toyota rank and file
employees. Toyota challenged said Order via an appeal to the DOLE Secretary.

In the meantime, the Union submitted its CBA proposals to Toyota, but the latter refused to negotiate in view of its
pending appeal. Consequently, the Union filed a notice of strike with the NCMB based on Toyota’s refusal to
bargain.

STRIKE 1: On February 21, 135 Union officers and members failed to render the required overtime work, and
instead marched to and staged a picket in front of the BLR office. The Union, in a letter of the same date,
also requested that its members be allowed to be absent on February 22 to attend the hearing and instead work
on their next scheduled rest day. This request however was denied by Toyota.

Despite denial of the Union’s request, more than 200 employees staged mass actions on February 22 and 23 in
front of the BLR and the DOLE offices, to protest the partisan and anti-union stance of Toyota. Due to the
deliberate absence of a considerable number of employees on February 22 to 23, Toyota experienced acute lack
of manpower in its manufacturing and production lines, and was unable to meet its production goals resulting in
huge losses.

Toyota then sent individual letters to some 360 employees requiring them to explain within 24 hours why they
should not be dismissed. On, March 1, The Union nonetheless submitted an explanation in compliance with the
February 27 notices sent by Toyota to the erring employees. On March 16, Toyota terminated the employment of
227 employees for participation in concerted actions in violation of its Code of Conduct and for misconduct under
Article 282 of the Labor Code.

STRIKE 2: In reaction to the dismissal of its union members and officers, the Union went on strike on March 17.
Subsequently, from March 28 to April 12, the Union intensified its strike by barricading the gates of Toyota’s
Bicutan and Sta. Rosa plants. The strikers prevented workers who reported for work from entering the plants.

Toyota filed a petition to declare the strike illegal with the NLRC arbitration branch, and prayed that the erring
Union officers, directors, and members be dismissed. DOLE Secretary assumed jurisdiction over the labor dispute
and issued an Order certifying the labor dispute to the NLRC.

DOLE Secretary Order: directed all striking workers to return to work at their regular shifts. On the other hand, it
ordered Toyota to accept the returning employees under the same terms and conditions obtaining prior to the
strike or at its option, put them under payroll reinstatement. The parties were also enjoined from committing acts
that may worsen the situation.

The Union ended the strike on April 12. The union members and officers tried to return to work but were told that
Toyota opted for payroll-reinstatement authorized by the Order of the DOLE Secretary.
Angeles| Bajana | Balladares | Brillantes | Briones | Cabansag | Callanta | Chua | David|

De Leon | Gomez | Lopez | Macalino | Nostratis | Padilla | Reynon | Santos | Tan |Velasco
4E / 4F - 2018-2019
Page 560 of 920
LABOR REVIEW DIGEST
Atty. Joyrich Golangco

STRIKE 3: Meanwhile, on May 23, despite the issuance of the DOLE Secretary’s certification Order, several
payroll-reinstated members of the Union staged a protest rally in front of Toyota’s Bicutan Plant bearing placards
and streamers in defiance of the April 10 Order. Then, on May 28, around Union members staged another protest
action in front of the Bicutan Plant. At the same time, some payroll-reinstated employees picketed in front of the
Santa Rosa Plant’s main entrance, and were later joined by other Union members.

NLRC decision: strikes staged by the Union on February 21 to 23 (as the Union failed to comply with the
procedural requirements of a valid strike under Art. 263 of the Labor Code) and May 23 and 28 as illegal and
the dismissal of the 227 who participated in the illegal strike on February 21-23 is legal. Lastly, award of
severance compensation was given to the dismissed Union members

After the DOLE Secretary assumed jurisdiction over the Toyota dispute on April 10, the Union again staged strikes
on May 23 and 28. The NLRC found the strikes illegal as they violated Art. 264 of the Labor Code which
proscribes any strike or lockout after jurisdiction is assumed over the dispute by the President or the DOLE
Secretary.

Court of Appeals: Affirmed the assailed NLRC Decision and Resolution with a modification, however, of deleting
the award of severance compensation to the dismissed Union members. However, CA modified Decision by
reinstating severance compensation to the dismissed employees based on social justice.

ISSUE:
1. Whether the mass actions committed by the Union on different occasions are illegal strikes; and
2. Whether separation pay should be awarded to the Union members who participated in the illegal strikes.

HELD: 1. YES, there is illegal strike.


A strike means any temporary stoppage of work by the concerted action of employees as a result of an industrial
or labor dispute. A labor dispute, in turn, includes any controversy or matter concerning terms or conditions of
employment or the association or representation of persons in negotiating, fixing, maintaining, changing, or
arranging the terms and conditions of employment, regardless of whether the disputants stand in the proximate
relation of the employer and the employee.

Six (6) categories of an illegal strike are as follows:


(1) [when it] is contrary to a specific prohibition of law, such as strike by employees performing governmental
functions; or
(2) [when it] violates a specific requirement of law[, such as Article 263 of the Labor Code on the requisites of a
valid strike]; or
(3) [when it] is declared for an unlawful purpose, such as inducing the employer to commit an unfair labor practice
against non-union employees; or
(4) [when it] employs unlawful means in the pursuit of its objective, such as a widespread terrorism of non-strikers
[for example, prohibited acts under Art. 264(e) of the Labor Code]; or
(5) [when it] is declared in violation of an existing injunction[, such as injunction, prohibition, or order issued by the
DOLE Secretary and the NLRC under Art. 263 of the Labor Code]; or
(6) [when it] is contrary to an existing agreement, such as a no-strike clause or conclusive arbitration clause

As to STRIKE 1: The Court ruled that the protest actions undertaken by the Union officials and members on
February 21 to 23 are not valid and proper exercises of their right to assemble and ask government for redress of
their complaints, but are illegal strikes in breach of the Labor Code. The Union’s position is weakened by the lack
of permit from the City of Manila to hold “rallies.”

Angeles| Bajana | Balladares | Brillantes | Briones | Cabansag | Callanta | Chua | David|

De Leon | Gomez | Lopez | Macalino | Nostratis | Padilla | Reynon | Santos | Tan |Velasco
4E / 4F - 2018-2019
Page 561 of 920
LABOR REVIEW DIGEST
Atty. Joyrich Golangco

Shrouded as demonstrations, they were in reality temporary stoppages of work perpetrated through the concerted
action of the employees who deliberately failed to report for work on the convenient excuse that they will hold a
rally at the BLR and DOLE offices on February 21 to 23. The purported reason for these protest actions was to
safeguard their rights against any abuse which the med-arbiter may commit against their cause. However, the
Union failed to advance convincing proof that the med-arbiter was biased against them. The acts of the med-
arbiter in the performance of his duties are presumed regular. What comes to the fore is that the decision not to
work for two days was designed and calculated to cripple the manufacturing arm of Toyota. It becomes obvious
that the real and ultimate goal of the Union is to coerce Toyota to finally acknowledge the Union as the sole
bargaining agent of the company. This is not a legal and valid exercise of the right of assembly and to demand
redress of grievance. It is obvious that the February 21 to 23 concerted actions were undertaken without satisfying
the prerequisites for a valid strike under Art. 263 of the Labor Code.

As to STRIKE 2 (March 17 to April 12): Initially it was legal as the legal requirements were met. However, on
March 28 to April 12, the Union barricaded the gates of the Bicutan and Sta. Rosa plants and blocked the free
ingress to and egress from the company premises. Toyota employees, customers, and other people having
business with the company were intimidated and were refused entry to the plants. As earlier explained, these
strikes were illegal because unlawful means were employed. The acts of the Union officers and members are in
palpable violation of Art. 264(e), which proscribes acts of violence, coercion, or intimidation, or which obstruct the
free ingress to and egress from the company premises. Undeniably, the strikes from March 28 to April 12
(STRIKE 2) were illegal.

2. NO. The general rule is that when just causes for terminating the services of an employee under Art. 282
of the Labor Code exist, the employee is not entitled to separation pay.

As in any rule, there are exceptions. One exception where separation pay is given even though an employee is
validly dismissed is when the court finds justification in applying the principle of social justice well entrenched in
the 1987 Constitution. In one case, the Court laid down the rule that severance compensation shall be
allowed only when the cause of the dismissal is other than serious misconduct or that which reflects adversely on
the employee’s moral character.

Two (2) exceptions when the NLRC or the courts should not grant separation pay based on social justice:
1. serious misconduct (which is the first ground for dismissal under Art. 282, or
2. acts that reflect on the moral character of the employee.

Considering that the dismissal of the employees was due to their participation in the illegal strikes as well as
violation of the Code of Conduct of the company, the same constitutes serious misconduct. A serious misconduct
is a transgression of some established and definite rule of action, a forbidden act, a dereliction of duty, willful in
character, and implies wrongful intent and not mere error in judgment. Based on existing jurisprudence, the award
of separation pay to the Union officials and members in the instant petitions cannot be sustained.

———————————————————————————————————————-

Angeles| Bajana | Balladares | Brillantes | Briones | Cabansag | Callanta | Chua | David|

De Leon | Gomez | Lopez | Macalino | Nostratis | Padilla | Reynon | Santos | Tan |Velasco
4E / 4F - 2018-2019
Page 562 of 920
LABOR REVIEW DIGEST
Atty. Joyrich Golangco

9. BRISTOL MYERS SQUIBB (PHILS.), INC. VS. BABAN


GR No: G.R. No. 167449 Date: December 17, 2008
Ponente: Reyes, R.T., J

Doctrine: While the State can regulate the right of an employer to select and discharge his employees, an
employer cannot be compelled to continue the employment of an employee in whom there has been a legitimate
loss of trust and confidence.

FACTS: Petitioner Bristol Myers Squibb Philippines, Inc. hired respondent Richard Nixon A. Baban as district
manager of the company. He was assigned to handle the company's clients in Cagayan de Oro-Northern
Mindanao area and its immediate vicinities. His duties included the promotion of nutritional products of petitioner
to medical practitioners, sale to drug outlets and the supervision of territory managers detailed in his district.

While conducting a field audit in Mindanao, petitioner's auditor, Sheela Torreja, found twenty (20) packs of
"Mamacare" samples in the baggage compartment of a company car with an accompanying note with political
overtones:

"Even if I've lost (sic) thank you so much for the support. Bidding you farewell for 36 years of public service. Will
continue to help for the good of the city of Zamboanga.
Atty. Ricardo S. Baban, Jr."

Atty. Ricardo S. Baban, Jr., referred to in the note, is respondent's father who had served as councilor in
Zamboanga City for thirty-six (36) years but lost in his bid for the vice-mayoralty post in the May 11, 1998
elections. Apparently, respondent's father was thanking supporters through distribution of company sample
products.

The auditor reported the incident, prompting the company's Medical Sales Director, Ferdinand Sarfati, to issue a
Memorandum requiring respondent to explain in writing within seventy-two (72) hours from notice why he should
not be terminated for the infraction.

Respondent admitted that he had caused the attachment of the notes to the product samples. He argued that
there was no unauthorized distribution of the samples since he intended to give them only to doctors who
requested them.

Respondent then received under protest the company's memorandum dismissing him from employment.
Questioning the validity of his dismissal, respondent filed a complaint for illegal dismissal.

LA/NLRC RULING: In sustaining the validity of respondent's dismissal, the Labor Arbiter ruled that respondent
had violated company rules and regulations by his unauthorized use of its property. Petitioner is therefore justified
to declare respondent unworthy of the trust and confidence formerly imposed in him.

CA RULING: The dismissal was illegal. The CA reasoned that the right of a worker to security of tenure is
constitutionally guaranteed. It further declared that, "when a person has no property, his job may possibly be his
only possession or means of livelihood. Therefore, he should be protected against any arbitrary deprivation of his
job."

APPEAL TO THE SC:


Petitioner's Contention: Petitioner argues that respondent, an employee occupying a position of trust and
confidence, admitted attaching his father's political thank you note on the product samples. Respondent likewise
Angeles| Bajana | Balladares | Brillantes | Briones | Cabansag | Callanta | Chua | David|

De Leon | Gomez | Lopez | Macalino | Nostratis | Padilla | Reynon | Santos | Tan |Velasco
4E / 4F - 2018-2019
Page 563 of 920
LABOR REVIEW DIGEST
Atty. Joyrich Golangco

confirmed his intention to distribute them to his father's political supporters to thank them for their help in the last
election. The act constituted an infraction of company rules. Respondent had breached his employer's trust,
meriting a penalty of dismissal.

Respondent's Contention: Respondent admitted that he committed an honest mistake, an irresponsible act to
have succumbed to the suggestion of one Dr. Gustahan. He pleaded for consideration for the lapse, insisting that
he has not caused any damage nor injury to the image of the company as the samples were not, in fact,
distributed and that no gain was derived by him or his family.

ISSUE/S: whether or not Baban was illegally dismissed

HELD: NO. It is clear that Article 282(c) of the Labor Code allows an employer to terminate the services of an
employee for loss of trust and confidence. The right of employers to dismiss employees by reason of loss of trust
and confidence is well established in jurisprudence.

The first requisite for dismissal on the ground of loss of trust and confidence is that the employee concerned must
be one holding a position of trust and confidence. Verily, We must first determine if respondent holds such a
position.

There are two (2) classes of positions of trust. The first class consists of managerial employees. They are defined
as those vested with the powers or prerogatives to lay down management policies and to hire, transfer suspend,
lay-off, recall, discharge, assign or discipline employees or effectively recommend such managerial actions. The
second class consists of cashiers, auditors, property custodians, etc. They are defined as those who in the normal
and routine exercise of their functions, regularly handle significant amounts of money or property.

In this case, respondent was employed as district manager for Cagayan de Oro-North Mindanao and its
immediate vicinities. It is not the job title but the actual work that the employee performs. He was employed to
handle pharmaceutical products for distribution to medical practitioners and sale to drug outlets. As a result of his
handling of large amounts of petitioner's samples, respondent is, by law, an employee with a position of trust,
falling under the second class.

The second requisite is that there must be an act that would justify the loss of trust and confidence. Loss of trust
and confidence to be a valid cause for dismissal must be based on a willful breach of trust and founded on clearly
established facts.

Respondent's act of stapling a thank you note from his father warrants the loss of petitioner's trust and confidence.
As the supervisor of fellow medical representatives, he had the duty to set a good example to his colleagues. A
higher standard of confidence was reposed in him.

There is no doubt that respondent willfully breached the trust and confidence reposed in him by not asking for
permission before using company property for his own or another's benefit, as required in the Company Standards
of Business Conduct. Moreover, when respondent failed to turn over the samples left in his care and stapled the
political "thank you" note with the intention of distributing them to his father's supporters, he had, in effect
appropriated company property for personal gain and benefit.

__________________________________________________________________________

Angeles| Bajana | Balladares | Brillantes | Briones | Cabansag | Callanta | Chua | David|

De Leon | Gomez | Lopez | Macalino | Nostratis | Padilla | Reynon | Santos | Tan |Velasco
4E / 4F - 2018-2019
Page 564 of 920
LABOR REVIEW DIGEST
Atty. Joyrich Golangco

10. ARMANDO G. YRASUEGUI v. PHILIPPINE AIRLINES, INC


G.R. No. 168081 October 17, 2008
REYES, R.T., J.:

DOCTRINES:
Requisites for Bona Fide Occupational Qualification (BFOQ):
(1) the employment qualification is reasonably related to the essential operation of the job involved; and
(2) that there is factual basis for believing that all or substantially all persons meeting the qualification would be
unable to properly perform the duties of the job.
The prescribed weight of PAL is a continuing BFOQ. On board an aircraft, the body weight and size of a cabin
attendant are important factors to consider in case of emergency. Aircrafts have constricted cabin space, and
narrow aisles and exit doors. It would d be absurd to require airline companies to reconfigure the aircraft in order
to widen the aisles and exit doors just to accommodate overweight cabin attendants like ARMANDO. Thus, the
failure to maintain such weight is a valid ground for dismissal under the “analogous cases” of Article 297(e) of the
Labor Code (new numbering).
As a rule, a legally dismissed employee is not entitled to separation pay. This may be deduced from the language
of Article 294 of the Labor Code (new numbering) that "[a]n employee who is unjustly dismissed from work shall
be entitled to x x x”. Exceptionally, it is granted to a legally dismissed employee as an act "social justice," or based
on "equity." In both instances, it is required that the dismissal (1) was not for serious misconduct; and (2) does not
reflect on the moral character of the employee.

FACTS: Armando was a former international flight steward of PAL. He stands five feet and eight inches (5’8") with
a large body frame. The proper weight for a man of his height and body structure is from 147 to 166 pounds, the
ideal weight being 166 pounds, as mandated by the Cabin and Crew Administration Manual of PAL.
In 1984, PAL gave Armando an extended vacation leave in order for him to lose weight. After meeting the
required weight, Armando was allowed to return to work. But Armando’s weight problem recurred. He was 40
pounds beyond his ideal weight and his weight kept on increasing in the course of his leave without pay from
October 17, 1988 to February 1989. For a span of 5 years, PAL has been giving Armando vacation leave without
pay to give him time to get back in shape. PAL even scheduled several weight checks to monitor Armando’s
weight. Armando, on the other hand, would always reschedule and would only report for a couple of times. On the
last 3 weight checks that he had attended, Armando still remained overweight.
On November 13, 1992, PAL finally served Armando a Notice of Administrative Charge for violation of company
standards on weight requirements. In his Answer, Armando did not deny being overweight but claimed that his
violation, if any, had already been condoned by PAL since "no action has been taken by the company" regarding
his case "since 1988. He also claimed that PAL discriminated against him because "the company has not been
fair in treating the cabin crew members who are similarly situated." During the clarificatory hearing on December
1992, Armando manifested that he was undergoing a weight reduction program to lose at least two (2) pounds per
week so as to attain his ideal weight. Six months thereafter, Armando was formally informed by PAL that due to
his inability to attain his ideal weight, "and considering the utmost leniency" extended to him "which spanned a
period covering a total of almost five (5) years," his services were considered terminated "effective immediately."
Armando then filed a complaint for illegal dismissal.
LA RULING: Armando was ILLEGALLY DISMISSED
The LA held that the weight standards of PAL are reasonable in view of the nature of the job of Armando.
However, the weight standards need not be complied with under pain of dismissal since his weight did not hamper
the performance of his duties. Assuming that it did, Armando could be transferred to other positions where his
weight would not be a negative factor. Notably, other overweight employees, i.e., Mr. Palacios, Mr. Cui, and Mr.
Barrios, were promoted instead of being disciplined. PAL and Armando appealed to the NLRC.
NLRC RULING: AFFIRMED the decision of the LA

Angeles| Bajana | Balladares | Brillantes | Briones | Cabansag | Callanta | Chua | David|

De Leon | Gomez | Lopez | Macalino | Nostratis | Padilla | Reynon | Santos | Tan |Velasco
4E / 4F - 2018-2019
Page 565 of 920
LABOR REVIEW DIGEST
Atty. Joyrich Golangco

According to the NLRC, "obesity, or the tendency to gain weight uncontrollably regardless of the amount of food
intake, is a disease in itself." As a consequence, there can be no intentional defiance or serious misconduct by
petitioner to the lawful order of PAL for him to lose weight.
CA RULING: Armando was VALIDLY DISMISSED
The CA found Armando’s termination to be valid under Article 297(e) of the Labor Code (new numbering), which
covers the analogous cases. Thus, Armando was legally dismissed because he repeatedly failed to meet the
prescribed weight standards. It is obvious that the issue of discrimination was only invoked by petitioner for
purposes of escaping the result of his dismissal for being overweight.
Armando appealed to the SC. He claims that obesity is a physical abnormality and/or illness and that as a
disease, he has no control over it. Thus, his dismissal cannot fall under the analogous cases.

ISSUE: Was Armando’s dismissal valid?

SC RULING: YES. The obesity of Armando is a ground for dismissal under Article 297(e) of the Labor Code (new
numbering) since the weight standards of PAL constitute a continuing qualification of an employee in order to
keep the job.
The SC held that the obesity of Armando, when placed in the context of his work as flight attendant, becomes an
analogous cause under Article 297(e) of the Labor Code that justifies his dismissal from the service. His obesity
may not be unintended, but is nonetheless voluntary. As the CA correctly puts it, "voluntariness basically means
that the just cause is solely attributable to the employee without any external force influencing or controlling his
actions. This element runs through all just causes under Article 297, whether they be in the nature of a wrongful
action or omission. Gross and habitual neglect, a recognized just cause, is considered voluntary although it lacks
the element of intent found in Article 297(a), (c), and (d)."
By way of equity, the SC granted separation pay in favor of Armando equivalent to one-half (1/2) month’s pay for
every year of service. It includes regular allowances, which he might have been receiving.

———————————————————————————————————————-

Angeles| Bajana | Balladares | Brillantes | Briones | Cabansag | Callanta | Chua | David|

De Leon | Gomez | Lopez | Macalino | Nostratis | Padilla | Reynon | Santos | Tan |Velasco
4E / 4F - 2018-2019
Page 566 of 920
LABOR REVIEW DIGEST
Atty. Joyrich Golangco

3. 11. DREAMLAND HOTEL RESORT vs JOHNSON


4. GR No: 191455 Date: March 12, 2014
5. Ponente: Sereno, C. J.
6.
7. Doctrine:
8. Even the most reasonable employee would consider quitting his job after working for three months and
receiving only an insignificant fraction of his salaries. There was, therefore, not an abandonment of employment
nor a resignation in the real sense, but a constructive dismissal, which is defined as an involuntary resignation
resorted to when continued employment is rendered impossible, unreasonable or unlikely.
9.
10. FACTS:
11. Dreamland is a corporation engaged in the hotel, restaurant and allied businesses. Respondent Stephen
B. Johnson is an Australian citizen who came to the Philippines as a businessman/investor without the authority to
be employed as the employee/officer of any business as he was not able to secure his Alien Employment Permit.
12.
13. According to Dreamland, Johnson was able to convince Prentice, its president, to accept his offer to
invest in Dreamland and at the same time provide his services as Operations Manager of Dreamland with a
promise that he will secure an AEP and Tax Identification Number prior to his assumption of work. Sometime on
June 21, 2007, Prentice and Johnson entered into an Employment Agreement, which stipulates among others,
that Johnson shall serve as Operations Manager of Dreamland from August 1, 2007 and shall serve as such for a
period of three (3) years. Before entering into the said agreement, Prentice required the submission of the AEP
and TIN from Johnson. Johnson promised that the same shall be supplied within one (1) month from the signing
of the contract because the application for the TIN and AEP were still under process. Thus, it was agreed that the
efficacy of the said agreement shall begin after one (1) month. On or about October 8, 2007, Prentice asked on
several occasions the production of the AEP and TIN from Johnson. Johnson gave excuses and promised that he
is already in possession of the requirements. Believing the word of Johnson, Dreamland commenced a dry run of
its operations. Johnson worked as a hotel and resort Operations Manager only at that time. He worked for only
about three (3) weeks until he suddenly abandoned his work and subsequently resigned as Operations Manager
starting November 3, 2007. He never reported back to work despite several attempts of Prentice to clarify his
issues.
1. On the other hand, respondent Johnson averred that there is also no truth to the allegation that it was
him who "offered" and "convinced" petitioner Prentice to "invest" in and provide his services to petitioner
Dreamland Hotel Resort. The truth of the matter is that it was petitioners who actively advertised for a resort
manager for Dreamland Hotel. It was in response to these advertisements that private respondent Johnson
contacted petitioners to inquire on the terms for employment offered. It was Prentice who offered employment and
convinced Johnson to give out a loan, purportedly so the resort can be completed and operational by August
2007. Believing the representations of petitioner Prentice, private respondent Johnson accepted the employment
as Resort Manager and loaned money to petitioners [consisting of] his retirement pay in the amount of USD
100,000.00 to finish construction of the resort. From the start of August 2007, as stipulated in the Employment
Agreement, respondent Johnson already reported for work. It was then that he found out to his dismay that the
resort was far from finished. However, he was instructed to supervise construction and speak with potential
guests. He also undertook the overall preparation of the guestrooms and staff for the opening of the hotel, even
performing menial tasks. As Johnson was remained unpaid since August 2007 and he has loaned all his money to
petitioners, he asked for his salary after the resort was opened in October 2007 but the same was not given to him
by petitioners. Johnson became very alarmed with the situation as it appears that there was no intention to pay
him his salary, which he now depended on for his living as he has been left penniless. He was also denied the
benefits promised him as part of his compensation such as service vehicles, meals and insurance. Johnson was
also not given the authority due to him as resort manager. Prentice countermanded his orders to the staff at every
opportunity. Worse, he would even be berated and embarrassed in front of the staff. Prentice would go into
drunken tiffs, even with customers and [Johnson] was powerless to prohibit Prentice. It soon became clear to him
Angeles| Bajana | Balladares | Brillantes | Briones | Cabansag | Callanta | Chua | David|

De Leon | Gomez | Lopez | Macalino | Nostratis | Padilla | Reynon | Santos | Tan |Velasco
4E / 4F - 2018-2019
Page 567 of 920
LABOR REVIEW DIGEST
Atty. Joyrich Golangco

that he was only used for the money he loaned and there was no real intention to have him as resort manager of
Dreamland Hotel. Thus, on November 3, 2007, after another embarrassment was handed out by petitioner
Prentice in front of the staff, which highlighted his lack of real authority in the hotel and the disdain for him by
petitioners, respondent Johnson was forced to submit his resignation. In deference to the Employment Agreement
signed, he stated that he was willing to continue work for the three month period stipulated therein. However, in an
SMS or text message sent by Prentice to Johnson on the same day at around 8:20 pm, he was informed that "… I
consider [yo]ur resignation as immediate". Despite demand, petitioners refused to pay [Johnson] the salaries and
benefits due him.
2. Johnson filed a Complaint for illegal dismissal and non-payment of salaries, among others, against the
petitioners.
14. LA RULING:
15. Dismissed Johnson’s complaint for lack of merit with the finding that he voluntarily resigned from his
employment and was not illegally dismissed
16.
17. NLRC RULING
18. Reversed LA’s decision, constructive dismissal
19.
20. CA RULING:
21. Dismissed the petition
22.
23. ISSUE/S:
24. Whether or not Johnson was illegally dismissed
25.
26. HELD: YES
27. Even the most reasonable employee would consider quitting his job after working for three months and
receiving only an insignificant fraction of his salaries. There was, therefore, not an abandonment of employment
nor a resignation in the real sense, but a constructive dismissal, which is defined as an involuntary resignation
resorted to when continued employment is rendered impossible, unreasonable or unlikely.
3. With this, the Court brings to mind its earlier ruling in the case of SHS Perforated Materials, Inc. v.
Diaz where it held that:
4. "There is constructive dismissal if an act of clear discrimination, insensibility, or disdain by an employer
becomes so unbearable on the part of the employee that it would foreclose any choice by him except to forego his
continued employment. It exists where there is cessation of work because continued employment is rendered
impossible, unreasonable or unlikely, as an offer involving a demotion in rank and a diminution in pay."
5. It is impossible, unreasonable or unlikely that any employee, such as Johnson would continue working
for an employer who does not pay him his salaries. Applying the Court’s pronouncement in Duldulao v. CA, the
Court construes that the act of the petitioners in not paying Johnson his salaries for three months has become
unbearable on the latter’s part that he had no choice but to cede his employment with them. The Court quotes the
pertinent sections of Johnson’s resignation letter which reflects the real reason why he was resigning as
operations manager of the hotel:
6. I hereby tender my resignation to you, Mr[.] Wes Prentice, Dreamland Resort, Subic, Zambales,
Philippines.
7. Since joining Dreamland Resort & Hotel over three months ago I have put my heart and soul into the
business. I have donated many hours of my personal time. I have frequently worked seven days a week and
twelve to thirteen hours a day. I am now literally penniless, due totally to the fact that I have lent you and your
resort/hotel well over $200,000AU (approx 8million pesos) and your non-payment of wages to me from 1st August
2007 as per Employment Agreement. x x x.36 (Emphasis and underscoring ours)
8. The above preceding statement only goes to show that while it was Johnson who tendered his
resignation, it was due to the petitioners’ acts that he was constrained to resign. The petitioners cannot expect
Johnson to tolerate working for them without any compensation.
Angeles| Bajana | Balladares | Brillantes | Briones | Cabansag | Callanta | Chua | David|

De Leon | Gomez | Lopez | Macalino | Nostratis | Padilla | Reynon | Santos | Tan |Velasco
4E / 4F - 2018-2019
Page 568 of 920
LABOR REVIEW DIGEST
Atty. Joyrich Golangco

9. Since Johnson was constructively dismissed, he was illegally dismissed


28. Other Notes/ SC Pronouncements:
29. As to the reliefs granted to an employee who is illegally dismissed, Golden Ace Builders v.
Talde referring to Macasero v. Southern Industrial Gases Philippines is instructive:
10. Thus, an illegally dismissed employee is entitled to two reliefs: backwages and reinstatement. The two
reliefs provided are separate and distinct. In instances where reinstatement is no longer feasible because of
strained relations between the employee and the employer, separation pay is granted. In effect, an illegally
dismissed employee is entitled to either reinstatement, if viable, or separation pay if reinstatement is no longer
viable, and backwages.
11. The normal consequences of respondents’ illegal dismissal, then, are reinstatement without loss of
seniority rights, and payment of backwages computed from the time compensation was withheld up to the date of
actual reinstatement. Where reinstatement is no longer viable as an option, separation pay equivalent to one (1)
month salary for every year of service should be awarded as an alternative. The payment of separation pay is in
addition to payment of backwages.
12. The case of Golden Ace further provides:
13. "The accepted doctrine is that separation pay may avail in lieu of reinstatement if reinstatement is no
longer practical or in the best interest of the parties. Separation pay in lieu of reinstatement may likewise be
awarded if the employee decides not to be reinstated." x x x
14. Under the doctrine of strained relations, the payment of separation pay is considered an acceptable
alternative to reinstatement when the latter option is no longer desirable or viable. On one hand, such payment
liberates the employee from what could be a highly oppressive work environment.1âwphi1 On the other hand, it
releases the employer from the grossly unpalatable obligation of maintaining in its employ a worker it could no
longer trust.
15. While the Court agrees with the NLRC that the award of separation pay and unpaid salaries is warranted,
the Court does not lose sight of the fact that the employment contract states that Johnson's employment is for a
term of three years.
16. Accordingly, the award of backwages should be computed from November 3, 2007 to August 1, 2010 -
which is three years from August 1, 2007. Furthermore, separation pay is computed from the commencement of
employment up to the time of termination, including the imputed service for which the employee is entitled to
backwages.42 As one-month salary is awarded as separation pay for every year of service, including imputed
service, Johnson should be paid separation pay equivalent to his three-month salary for the three-year contract.
———————————————————————————————————————-

Angeles| Bajana | Balladares | Brillantes | Briones | Cabansag | Callanta | Chua | David|

De Leon | Gomez | Lopez | Macalino | Nostratis | Padilla | Reynon | Santos | Tan |Velasco
4E / 4F - 2018-2019
Page 569 of 920
LABOR REVIEW DIGEST
Atty. Joyrich Golangco

17. 12. Manila Water Company v. Del Rosario


18. G.R. No. 188747, January 29, 2014
19. Perez J.
20.
21. Doctrine
22. As a general rule, an employee who has been dismissed for any of the just causes enumerated under
Article 282 of the Labor Code is not entitled to a separation pay.
23.
24. In exceptional cases, however, the Court has granted separation pay to a legally dismissed employee as
an act of "social justice" or on "equitable grounds." In both instances, it is required that the dismissal (1) was not
for serious misconduct; and (2) did not reflect on the moral character of the employee.
25.
26. Facts
27.
28. Del Rosario was employed as Instrument Technician by MWSS. Sometime in 1996, MWSS was
reorganized pursuant to Republic Act No. 8041 or the National Water Crisis Act of 1995, and its implementing
guidelines − Executive Order No. 286. Because of the reorganization under R.A. No. 8041, Manila Water
absorbed some employees of MWSS including Del Rosario.
29.
30. Sometime in May 2000, Manila Water discovered that 24 water meters were missing in its stockroom.
Upon initial investigation, it appeared that Del Rosario and his co-employee were involved in the pilferage and the
sale of water meters to the company’s contractor. Consequently, Manila Water directed Del Rosario to explain in
writing within 72 hours why he should not be dealt with administratively for the loss of the said water meters. Del
Rosario confessed his involvement in the act charged and pleaded for forgiveness, promising not to commit
similar acts in the future.
31.
32. Manila Water conducted a hearing. During the formal investigation Del Rosario was found responsible for
the loss of the water meters and therefore liable for violating Section 11.1 of the Company’s Code of
Conduct. Manila Water proceeded to dismiss Del Rosario.
33.
34. This prompted Del Rosario to file an action for illegal dismissal claiming that his severance from
employment is without just cause. Del Rosario averred that his admission to the misconduct charged was not
voluntary but was coerced by the company.
35.
36. LA/NLRC Ruling
37.
38. The Labor Arbiter DISMISSED the case for lack of merit. Del Rosario was, however, awarded separation
pay. According to the Labor Arbiter, Del Rosario’s length of service for 21 years, without previous derogatory
record, warrants the award of separation pay.
39.
40. The NLRC dismissed the appeal interposed by Manila Water for its failure to append a certification
against forum shopping in its Memorandum of Appeal.
41.
42. CA Ruling
43.
44. The Court of Appeals reversed the NLRC Resolution and held that it committed a grave abuse of
discretion when it dismissed Manila Water’s appeal on mere technicality. The appellate court, however,
proceeded to affirm the decision of the Labor Arbiter awarding separation pay to Del Rosario. Considering that Del
Rosario rendered 21 years of service to the company without previous derogatory record, the appellate court
considered the granting of separation pay by the labor officer justified.
Angeles| Bajana | Balladares | Brillantes | Briones | Cabansag | Callanta | Chua | David|

De Leon | Gomez | Lopez | Macalino | Nostratis | Padilla | Reynon | Santos | Tan |Velasco
4E / 4F - 2018-2019
Page 570 of 920
LABOR REVIEW DIGEST
Atty. Joyrich Golangco

45.
46. Issue
47.
48. WON the award of separation pay to respondent who was dismissed for stealing the company’s property
which amounted to gross misconduct is proper.
49.
50. Held
51.
 As a general rule, an employee who has been dismissed for any of the just causes enumerated under
Article 282 of the Labor Code is not entitled to a separation pay.
52.
53. In exceptional cases, however, the Court has granted separation pay to a legally dismissed
employee as an act of "social justice" or on "equitable grounds." In both instances, it is required that the
dismissal (1) was not for serious misconduct; and (2) did not reflect on the moral character of the
employee.
54.
55. We hold that henceforth separation pay shall be allowed as a measure of social justice only in
those instances where the employee is validly dismissed for causes other than serious misconduct or
those reflecting on his moral character. Where the reason for the valid dismissal is, for example, habitual
intoxication or an offense involving moral turpitude, like theft or illicit sexual relations with a fellow worker,
the employer may not be required to give the dismissed employee separation pay, or financial
assistance, or whatever other name it is called, on the ground of social justice.
56.
 The attendant circumstances in the present case considered, we are constrained to deny Del Rosario
separation pay since the admitted cause of his dismissal amounts to serious misconduct. He is not only
responsible for the loss of the water meters in flagrant violation of the company’s policy but his act is in
utter disregard of his partnership with his employer in the pursuit of mutual benefits.
57.
58. Indubitably, the appellate court erred in awarding separation pay to Del Rosario without taking
into consideration that the transgression he committed constitutes a serious offense. The grant of
separation pay to a dismissed employee is determined by the cause of the dismissal. The years of
service may determine how much separation pay may be awarded. It is, however, not the reason why
such pay should be granted at all.

———————————————————————————————————————-

Angeles| Bajana | Balladares | Brillantes | Briones | Cabansag | Callanta | Chua | David|

De Leon | Gomez | Lopez | Macalino | Nostratis | Padilla | Reynon | Santos | Tan |Velasco
4E / 4F - 2018-2019
Page 571 of 920
LABOR REVIEW DIGEST
Atty. Joyrich Golangco

13. Nacar vs. Gallery Frames


GR No: 189871 Date: August 13, 2013
Ponente: PERALTA, J.

Doctrine: The backwages/separation pay shall be computed from the date of illegal dismissal until the date of the
decision of the Labor Arbiter. But if the employer appeals, then the end date shall be extended until the day when
the appellate court’s decision shall become final.

FACTS:

Petitioner Dacario Nacar filed a complaint for Constructive Dismissal before the NLRC against Respondent
Gallery Frames.

On October 15, 1998, the Labor Arbiter rendered a Decision3 in favor of petitioner and found that he was
dismissed from employment without a valid or just cause. Thus, petitioner was awarded backwages and
separation pay in lieu of reinstatement in the amount of ₱158,919.92.

When the case was referred back to the LA, petitioner filed a Motion for Correct Computation, praying that his
backwages be computed from the date of his dismissal on January 24, 1997 up to the finality of the Resolution of
the Supreme Court on May 27, 2002. Upon recomputation, the Computation and Examination Unit of the NLRC
arrived at an updated amount in the sum of ₱471,320.31.

Respondents filed a motion to quash the writ of execution on the ground that since the Labor Arbiter awarded
separation pay of ₱62,986.56 and limited backwages of ₱95,933.36, no more recomputation is required to be
made of the said awards. They claimed that after the decision becomes final and executory, the same cannot be
altered or amended anymore. LA denied, but the NLRC granted the motion upon appeal.

Petitioner filed an appeal for the said computation but the same was denied up to the court of appeals. Hence, the
petition.

Petitioner argues that notwithstanding the fact that there was a computation of backwages in the Labor Arbiter’s
decision, the same is not final until reinstatement is made or until finality of the decision, in case of an award of
separation pay. Petitioner maintains that considering that the October 15, 1998 decision of the Labor Arbiter did
not become final and executory until the April 17, 2002 Resolution of the Supreme Court in G.R. No. 151332 was
entered in the Book of Entries on May 27, 2002, the reckoning point for the computation of the backwages and
separation pay should be on May 27, 2002 and not when the decision of the Labor Arbiter was rendered on
October 15, 1998. Further, petitioner posits that he is also entitled to the payment of interest from the finality of the
decision until full payment by the respondents.

On their part, respondents assert that since only separation pay and limited backwages were awarded to
petitioner by the October 15, 1998 decision of the Labor Arbiter, no more recomputation is required to be made of
said awards. Respondents insist that since the decision clearly stated that the separation pay and backwages are
"computed only up to [the] promulgation of this decision," and considering that petitioner no longer appealed the
decision, petitioner is only entitled to the award as computed by the Labor Arbiter in the total amount of
₱158,919.92. Respondents added that it was only during the execution proceedings that the petitioner questioned
the award, long after the decision had become final and executory. Respondents contend that to allow the further
recomputation of the backwages to be awarded to petitioner at this point of the proceedings would substantially
vary the decision of the Labor Arbiter as it violates the rule on immutability of judgments.

Angeles| Bajana | Balladares | Brillantes | Briones | Cabansag | Callanta | Chua | David|

De Leon | Gomez | Lopez | Macalino | Nostratis | Padilla | Reynon | Santos | Tan |Velasco
4E / 4F - 2018-2019
Page 572 of 920
LABOR REVIEW DIGEST
Atty. Joyrich Golangco

ISSUE/S:
Whether a re-computation in the course of execution of the labor arbiter's original computation of the awards
made, pegged as of the time the decision was rendered and confirmed with modification by a final CA decision, is
legally proper.

HELD:

There are two parts of a decision when it comes to illegal dismissal cases (referring to cases where the dismissed
employee wins, or loses but wins on appeal). The first part is the ruling that the employee was illegally dismissed.
This is immediately final even if the employer appeals – but will be reversed if employer wins on appeal. The
second part is the ruling on the award of backwages and/or separation pay. For backwages, it will be computed
from the date of illegal dismissal until the date of the decision of the Labor Arbiter. But if the employer appeals,
then the end date shall be extended until the day when the appellate court’s decision shall become final. Hence,
as a consequence, the liability of the employer, if he loses on appeal, will increase – this is just but a risk that the
employer cannot avoid when it continued to seek recourses against the Labor Arbiter’s decision. This is also in
accordance with Article 279 of the Labor Code.

Anent the issue of award of interest in the form of actual or compensatory damages, the Supreme Court ruled that
the old case of Eastern Shipping Lines vs CA is already modified by the promulgation of the Bangko Sentral ng
Pilipinas Monetary Board Resolution No. 796 which lowered the legal rate of interest from 12% to 6%. Specifically,
the rules on interest are now as follows:

1. Monetary Obligations ex. Loans:

a. If stipulated in writing:

a.1. shall run from date of judicial demand (filing of the case)

a.2. rate of interest shall be that amount stipulated

b. If not stipulated in writing

b.1. shall run from date of default (either failure to pay upon extra-judicial demand or upon judicial demand
whichever is appropriate and subject to the provisions of Article 1169 of the Civil Code)

b.2. rate of interest shall be 6% per annum

2. Non-Monetary Obligations (such as the case at bar)

a. If already liquidated, rate of interest shall be 6% per annum, demandable from date of judicial or extra-judicial
demand (Art. 1169, Civil Code)

b. If unliquidated, no interest

Except: When later on established with certainty. Interest shall still be 6% per annum demandable from the date of
judgment because such on such date, it is already deemed that the amount of damages is already ascertained.

3. Compounded Interest

– This is applicable to both monetary and non-monetary obligations


Angeles| Bajana | Balladares | Brillantes | Briones | Cabansag | Callanta | Chua | David|

De Leon | Gomez | Lopez | Macalino | Nostratis | Padilla | Reynon | Santos | Tan |Velasco
4E / 4F - 2018-2019
Page 573 of 920
LABOR REVIEW DIGEST
Atty. Joyrich Golangco

– 6% per annum computed against award of damages (interest) granted by the court. To be computed from the
date when the court’s decision becomes final and executory until the award is fully satisfied by the losing party.

4. The 6% per annum rate of legal interest shall be applied prospectively:

– Final and executory judgments awarding damages prior to July 1, 2013 shall apply the 12% rate;

– Final and executory judgments awarding damages on or after July 1, 2013 shall apply the 12% rate for unpaid
obligations until June 30, 2013; unpaid obligations with respect to said judgments on or after July 1, 2013 shall still
incur the 6% rate.

———————————————————————————————————————-

Angeles| Bajana | Balladares | Brillantes | Briones | Cabansag | Callanta | Chua | David|

De Leon | Gomez | Lopez | Macalino | Nostratis | Padilla | Reynon | Santos | Tan |Velasco
4E / 4F - 2018-2019
Page 574 of 920
LABOR REVIEW DIGEST
Atty. Joyrich Golangco

14. BANI RURAL BANK INC. ENOC THEATER I AND II and/or RAFAEL DE GUZMAN vs. TERESA DE
GUZMAN, EDGAR C. TAN and TERESA G. TAN.
G.R. No. 170904 Date: November 13, 2013
NACHURA, J.

Doctrine: When reinstatement is ordered, the general concept under Article 279 of the Labor Code, as
amended, compute the backwages from the time of dismissal until the employee’s reinstatement.

FACTS:
Respondents were employees of Bani Rural Bank, Inc. and ENOC Theatre I and II who filed a
complaint for illegal dismissal against petitioners. NLRC ordered the petitioners to: Reinstate the two
complainants to their former positions, without loss o seniority rights and other benefits and privileges,
with backwages from the time o their dismissal (con-structive) until their actual reinstatement, less
earnings elsewhere. The March 17, 1995 resolution of the NLRC became final and executory and the
computation of the awards was remanded to the labor arbiter for execution purposes. The first
computation of the monetary award under the March 17, 1995 resolution of the NLRC First, Labor
Arbiter Gambito deducted the earnings derived by the respondents either from Bani Rural Bank, Inc. or
ENOC Theatre I and II. Second, Labor Arbiter Gambito fixed the period of backwages from the
respondents' illegal dismissal until August 25 1995 or the date when the respondents allegedly manifested
that they no longer wanted to be reinstated. The re-spondents appealed Labor Arbiter Gambito's
computation with the NLRC. The NLRC modified the terms of the March 17, 1995 resolution insofar as it
clarified the phrase less earnings elsewhere. The NLRC additionally awarded the payment of separation
pay, in lieu of reinstatement, under the following terms: The decision of this Commission is hereby
MODIFIED to the extent that: (1) the phrase earnings elsewhere in its dispositive portion shall exclude
the complainants' salaries from the Rural Bank of Mangantarem; and (2) in lieu of reinstatement, the
respondents are hereby ordered to pay the complainants separation pay equivalent to one month salary
for every year of service computed from the start of their employment up to the date of the finality of
the decision. The respond-ents filed a motion for reconsideration on whether the award of backwages
was still in-cluded in the judgment. The NLRC dismissed the motion for having been filed out of time.
On January 29, 1999, the July 31, 1998 decision of the NLRC lapsed to finality and became executory.
The second computation of the monetary awards under the July 31, 998 decision of the NLRC Labor
Arbiter Gambito computed the respondents back-wages only up to August 25, 1995. The NLRC ruled
that the computation of the re-spondents backwages should be until January 29, 1999 which was the
date when the July 31, 1998 decision attained finality.

ISSUE/S:
What is the basis for the computation of backwages

HELD:
The computation of the respondents' backwages must be from the time of the illegal dismissal
from employment until the finality of the decision ordering the payment of separation pay. The
respondents' backwages, therefore, must be computed from the time of their illegal dismissal until
January 29, 1999, the date of finality of the NLRC's July 31, 1998 Decision. As a final point, the CA’s
ruling must be modified to include legal interest commencing from the finality of the NLRC's July 31,
1998 decision.
The computation of backwages depends on the final awards adjudged as a consequence of
illegal dismissal, in that: First, when reinstatement is ordered, the general concept under Article 279 of
the Labor Code, as amended, compute the backwages from the time of dismissal until the employee’s
reinstatement. The computation of backwages (and similar benefits considered part of the backwages) can
even continue beyond the decision of the labor arbiter or NLRC and ends only when the employee is
Angeles| Bajana | Balladares | Brillantes | Briones | Cabansag | Callanta | Chua | David|

De Leon | Gomez | Lopez | Macalino | Nostratis | Padilla | Reynon | Santos | Tan |Velasco
4E / 4F - 2018-2019
Page 575 of 920
LABOR REVIEW DIGEST
Atty. Joyrich Golangco

actually reinstated. Second, when separation pay is ordered in lieu of reinstatement (in the event that
this aspect of the case is disputed) or reinstatement is waived by the employee (in the event that the
payment of separation pay, in lieu, is not disputed), backwages is computed from the time of dismissal
until the finality of the decision ordering separation pay. Third, when separation pay is ordered after the
finality of the decision ordering the reinstatement by reason of a supervening event that makes the
award of reinstatement no longer possible (as in the case), backwages is computed from the time of
dismissal until the finality of the decision ordering separation pay.
When there is an order of separation pay (in lieu of reinstatement or when the reinstatement
aspect is waived or subsequently ordered in light of a supervening event making the award of
reinstatement no longer possible), the employment relationship is terminated only upon the finality of the
decision ordering the separation pay. The finality of the decision cuts-off the employment relationship and
represents the final settlement of the rights and obligations of the parties against each other. Hence,
backwages no longer accumulate upon the finality of the decision ordering the payment of separation pay
since the employee is no longer entitled to any compensation from the employer by reason of the
severance of his employment.
The awards of reinstatement and separation pay are exclusive remedies; the change of awards
(from reinstatement to separation pay) under the NLRC’s decision on July 31, 1998 not only modified the
awards granted, but also changed the manner the respondents backwages is to be computed. The
respondents’ backwages can no longer be computed up to the point of reinstatement as there is no
longer any award of reinstatement to speak of.
———————————————————————————————————————-

Angeles| Bajana | Balladares | Brillantes | Briones | Cabansag | Callanta | Chua | David|

De Leon | Gomez | Lopez | Macalino | Nostratis | Padilla | Reynon | Santos | Tan |Velasco
4E / 4F - 2018-2019
Page 576 of 920
LABOR REVIEW DIGEST
Atty. Joyrich Golangco

15. URC v. Castillo


G.R. No. 189686 July 10, 2013

DOCTRINE:
As the rule now stands, the award of separation pay is authorized in the situations dealt with in Article 283 and
284 of the Labor Code, but not in terminations of employment based on instances enumerated in Article 282.
FACTS:
Petitioner Universal Robina Corporation (URC) terminated from employment as a Regional Sales Manager,
respondent Wilfredo Castillo (Castillo). He was dismissed for breach of trust and confidence and acts inimical to
the interest of URC after a finding that he signed 2 blank Charge Invoices despite full knowledge that the same
will be used by Liana’s Supermarket to cause the deduction of P72,000.00 against the account of URC for the gift
certificate that Castillo unduly received. Castillo filed a complaint for illegal dismissal against URC.
LA declared Castillo to have been illegally dismissed and ordered the payment of backwages and separation pay.
The NLRC reversed the decision of LA after finding that URC had more than sufficient proof that respondent
violated its trust. MR of Castillo was denied. On appeal, the CA upheld his dismissal but awarded him separation
pay "as a form of equitable relief” considering that he honorably served the company for about 23 years and this is
his only and first offense.

ISSUE: Whether Castillo is entitled to separation pay?

HELD: NO. Castillo is not entitled to separation pay.


As the rule now stands, the award of separation pay is authorized in the situations dealt with in Article 283 and
284 of the Labor Code, but not in terminations of employment based on instances enumerated in Article 282.
Labor adjudicatory officials and the CA must demur the award of separation pay based on social justice when an
employee’s dismissal is based on serious misconduct or willful disobedience; gross and habitual neglect of duty;
fraud or willful breach of trust; or commission of a crime against the person of the employer or his immediate
family. They must be most judicious and circumspect in awarding separation pay or financial assistance as the
constitutional policy to provide full protection to labor is not meant to be an instrument to oppress the employers.
In fine, we should be more cautious in awarding financial assistance to the undeserving and those who are
unworthy of the liberality of the law.
Indeed, Castillo has committed acts constituting willful breach of trust and confidence reposed on him by URC.
The principal charge against petitioner Castillo was hinged upon "unauthorized arrangements" which he allegedly
entered into. His neglect in signing the blank charge invoices and his improper receipt of gift certificates for his
personal gain, when taken together constitute a breach of the trust and confidence reposed on petitioner Castillo
by URC.
As a Regional Sales Manager, petitioner Castillo occupied a position or responsibility and as such, he should
have known that he placed the interests of the company at a disadvantage by signing the blank charge invoices.
Because of such act, private respondent URC was prejudiced by no less than P72,000.00. This alone is sufficient
cause for breach of trust and loss of confidence.
———————————————————————————————————————-

Angeles| Bajana | Balladares | Brillantes | Briones | Cabansag | Callanta | Chua | David|

De Leon | Gomez | Lopez | Macalino | Nostratis | Padilla | Reynon | Santos | Tan |Velasco
4E / 4F - 2018-2019
Page 577 of 920
LABOR REVIEW DIGEST
Atty. Joyrich Golangco

16. MINETTE BAPTISTA et al vs. ROSARIO VILLANUEVA, et. al


G.R. No. 194709 July 31, 2013
Mendoza, J.

DOCTRINE: The essence of due process is simply to be heard, or as applied to administrative proceedings, an
opportunity to explain one’s side, or an opportunity to seek a reconsideration of the action or ruling complained of.
Mere absence of a one-on-one confrontation between the petitioners and their complainants does not
automatically affect the validity of the proceedings before the Committee.

FACTS: Petitioners were former union members of Radio Philippines Network Employees Union (RPNEU). It was
the sole and exclusive bargaining agent of the rank and file employees of Radio Philippines Network (RPN), while
the respondents were the union’s elected officers and members.
A complaint for impeachment of their union president, Reynato Siozon, before the executive board of RPN, was
filed by the Petitioners due to their suspicion of union mismanagement. They re-lodged the impeachment
complaint, however, this time, against all the union officers and members of RPNEU before the DOLE. Complaints
were filed before the Chairman of RPNEU’s Committee on Grievance and Investigation (the Committee) against
petitioners citing as grounds the "commission of an act which violates RPNEU Constitution and By-Laws,
specifically, Article IX, Section 2.5 for urging or advocating that a member start an action in any court of justice or
external investigative body against the Union or its officer without first exhausting all internal remedies open to him
or available in accordance with the CBL.
Expulsion was the recommendation of the Committee to RPNEU’s Board of Directors. It was affirmed by the latter.
Thereafter, Petitioners were served with an expulsion notice. The Petitioners were informed of the termination of
their employment, enforcing Article II, Sec. 2 of their CBA which contained a union security clause.
A complaint for ULP was filed by the Petitioners against Respondents on the ground that they were denied
substantive and procedural due process when they were expelled from the union.

PETITIONER’S CONTENTION: Petitioners contend that the procedure should have been followed by the
Respondents in resolving the charges against them as provided in their Constitution. Further, they maintain that
any form of grievance would be referred only to the committee upon failure of the parties to settle amicably.

ISSUE: Whether or not the petitioners were denied of their right to substantive and procedural due process in their
expulsion.

HELD: NO. The expulsion of the Petitioners from the union was valid.
Workers’ and employers’ organizations shall have the right to draw up their constitutions. In this case, RPNEU’s
Constitution and By-Laws expressly mandate that before a party is allowed to seek the intervention of the court, it
is a pre-condition that he should have availed of all the internal remedies within the organization. Petitioners were
found to have violated such when they filed petitions for impeachment against their union officers and for audit
before the DOLE without first exhausting all internal remedies available within their organization. This act is a
ground for expulsion from union membership.
Petitioners’ expulsion from the union was not a deliberate attempt to curtail or restrict their right to organize, but
was triggered by the commission of an act, expressly sanctioned by their Constitution.
The SC also declared that the expulsion was free from infirmities. Under RPNEU’s Constitution, any charge
against the member must be submitted to the Board of Directors and not to the Committee. Although the
respondents filed the charge before the Committee, this supposed procedural flaw deemed cured when
petitioners were given the opportunity to be heard. Due process, as a constitutional precept, is satisfied when a
person was notified of the charge against him and was given an opportunity to explain or defend himself. In
administrative proceedings, the filing of charges and giving reasonable opportunity for the person so charged to
answer the accusations against him constitute the minimum requirements of due process. The essence of due

Angeles| Bajana | Balladares | Brillantes | Briones | Cabansag | Callanta | Chua | David|

De Leon | Gomez | Lopez | Macalino | Nostratis | Padilla | Reynon | Santos | Tan |Velasco
4E / 4F - 2018-2019
Page 578 of 920
LABOR REVIEW DIGEST
Atty. Joyrich Golangco

process is simply to be heard, or as applied to administrative proceedings, an opportunity to explain one’s side, or
an opportunity to seek a reconsideration of the action or ruling complained of.

———————————————————————————————————————-

Angeles| Bajana | Balladares | Brillantes | Briones | Cabansag | Callanta | Chua | David|

De Leon | Gomez | Lopez | Macalino | Nostratis | Padilla | Reynon | Santos | Tan |Velasco
4E / 4F - 2018-2019
Page 579 of 920
LABOR REVIEW DIGEST
Atty. Joyrich Golangco

17. BPI EMPLOYEES UNION-DAVAO v. BPI


G.R. No. 174912 July 24, 2013

DOCTRINE: Court held that it is management prerogative to farm out any of its activities, regardless of whether
such activity is peripheral or core in nature. What is of primordial importance is that the service agreement does
not violate the employee's right to security of tenure and payment of benefits to which he is entitled under the law.

Facts:
A merger between BPI and Far East Bank and Trust Company (FEBTC) took effect on April 10, 2000
with BPI as the surviving corporation. Thereafter, BPI’s cashiering function and FEBTC’s cashiering, distribution
and bookkeeping functions were handled by BOMC. Consequently, 12 former FEBTC employees were
transferred to BOMC to complete the latter’s service complement.
BPI Davao’s rank and file collective bargaining agent, BPI Employees Union-Davao City-FUBU (Union),
petitioner in this case, objected to the transfer of the functions and the 12 personnel to BOMC contending that the
functions rightfully belonged to the BPI employees and that the Union was deprived of membership of former
FEBTC personnel who, by virtue of the merger, would have formed part of the bargaining unit represented by the
Union pursuant to its union shop provision in the CBA.
NLRC came out with a resolution upholding the validity of the service agreement between BPI and
BOMC and dismissing the charge of ULP. It ruled that the engagement by BPI of BOMC to undertake some of its
activities was clearly a valid exercise of its management prerogative.
After the denial of its motion for reconsideration, the Union elevated its grievance to the CA via a petition
for certiorari under Rule 65.
Union filed a motion for reconsideration which was, however, denied by the CA.

Issue:
Whether or not the act of BPI to outsource the cashiering, distribution and bookkeeping functions to
BOMC is in conformity with the law and the existing.
The Union claims that a union shop agreement is stipulated in the existing CBA. It is unfair labor practice
for employer to outsource the positions in the existing bargaining unit, citing the case of Shell Oil

HELD:
Workers’ Union v. Shell Company of the Philippines, Ltd.
The Union’s reliance on the Shell Case is misplaced. The rule now is covered by Article 261 of the Labor
Code, which took effect on November 1, 1974. Article 261 provides:
ART. 261. Jurisdiction of Voluntary Arbitrators or panel of Voluntary Arbitrators. – x x x Accordingly,
violations of a Collective Bargaining Agreement, except those which are gross in character, shall no longer be
treated as unfair labor practice and shall be resolved as grievances under the Collective Bargaining Agreement.
For purposes of this article, gross violations of Collective Bargaining Agreement shall mean flagrant and/or
malicious refusal to comply with the economic provisions of such agreement.
Clearly, only gross violations of the economic provisions of the CBA are treated as ULP. Otherwise, they
are mere grievances.
The alleged violation of the union shop agreement in the CBA, even assuming it was malicious and
flagrant, is not a violation of an economic provision in the agreement. The provisions relied upon by the Union
were those articles referring to the recognition of the union as the sole and exclusive bargaining representative of
all rank-and-file employees, as well as the articles on union security, specifically, the maintenance of membership
in good standing as a condition for continued employment and the union shop clause.26 It failed to take into
consideration its recognition of the bank’s exclusive rights and prerogatives, likewise provided in the CBA, which
included the hiring of employees, promotion, transfers, and dismissals for just cause and the maintenance of
order, discipline and efficiency in its operations.

Angeles| Bajana | Balladares | Brillantes | Briones | Cabansag | Callanta | Chua | David|

De Leon | Gomez | Lopez | Macalino | Nostratis | Padilla | Reynon | Santos | Tan |Velasco
4E / 4F - 2018-2019
Page 580 of 920
LABOR REVIEW DIGEST
Atty. Joyrich Golangco

As far as the 12 former FEBTC employees are concerned, the Union failed to substantially prove that
their transfer, made to complete BOMC’s service complement, was motivated by ill will, anti-unionism or bad faith
so as to affect or interfere with the employees’ right to self-organization.
It is to be emphasized that contracting out of services is not illegal perse. It is an exercise of business
judgment or management prerogative. Absent proof that the management acted in a malicious or arbitrary
manner, the Court will not interfere with the exercise of judgment by an employer. In this case, bad faith cannot be
attributed to BPI because its actions were authorized by CBP Circular No. 1388, Series of 1993 issued by the
Monetary Board of the then Central Bank of the Philippines (now Bangko Sentral ng Pilipinas). The circular
covered amendments in Book I of the Manual of Regulations for Banks and Other Financial Intermediaries,
particularly on the matter of bank service contracts. A finding of ULP necessarily requires the alleging party to
prove it with substantial evidence. Unfortunately, the Union failed to discharge this burden.
In one case, the Court held that it is management prerogative to farm out any of its activities, regardless
of whether such activity is peripheral or core in nature. What is of primordial importance is that the service
agreement does not violate the employee's right to security of tenure and payment of benefits to which he is
entitled under the law. Furthermore, the outsourcing must not squarely fall under labor-only contracting where the
contractor or sub-contractor merely recruits, supplies or places workers to perform a job, work or service for a
principal or if any of the following elements are present:

i) The contractor or subcontractor does not have substantial capital or investment which relates to the
job, work or service to be performed and the employees recruited, supplied or placed by such contractor or
subcontractor are performing activities which are directly related to the main business of the principal; or

ii) The contractor does not exercise the right to control over the performance of the work of the
contractual employee.

———————————————————————————————————————-

Angeles| Bajana | Balladares | Brillantes | Briones | Cabansag | Callanta | Chua | David|

De Leon | Gomez | Lopez | Macalino | Nostratis | Padilla | Reynon | Santos | Tan |Velasco
4E / 4F - 2018-2019
Page 581 of 920
LABOR REVIEW DIGEST
Atty. Joyrich Golangco

18. Integrated Microelectronics v. Pionilla


G.R. No. 200222; Aug. 28, 2013
Perlas-Bernabe, J.:

DOCTRINE: As a general rule, an illegally dismissed employee is entitled to reinstatement (or separation pay, if
reinstatement is not viable) and payment of full backwages. In certain cases, however, the Court has carved out
an exception to the foregoing rule and thereby ordered the reinstatement of the employee without backwages on
account of the following: (a) the fact that dismissal of the employee would be too harsh of a penalty; and (b) that
the employer was in good faith in terminating the employee. Good faith on the part of the employer may preclude
or diminish the recovery of backwages.

FACTS:

Adonis Pionilla was hired as a production worker by Integrated Microelectronics (IMI) in 1996. On May 5, 2005, he
was required to explain an incident wherein he was seen boarding a company shuttlebus at the Alabang Terminal.
It was reported by the bus driver that both held a company ID (Company employees get free rides) in the name of
Pionilla.

A Conscience Committee was convened to deliberate on the matter wherein Pionilla admitted that he had two IDs
because his original one was lost, and but was able to secure a temporary ID later. It would appear that the
temporary ID was never surrendered despite finding the original. Pionilla admitted his shortcomings and
apologized for the infraction of the Company Rules and Regulations. He was dismissed by IMI, prompting the
filing of an illegal dismissal suit by Pionilla, believing that his punishment was too severe.

LA RULING: Ordered the reinstatement of Pionilla and the payment of backwages amounting to 417,818.78php.
Held that the act was impelled by a false notion of pakikisama and utang na loob and not by dishonesty. Held that
there was no breach of company security whatsoever and considered his 9 years of unblemished service.

NLRC RULING: Reversed the LA and held that Pionilla’s attitude was perverse.

CA RULING: Held that the penalty did not commensurate the offense and that the right of an employer to
discipline and employee remains subject to reasonable regulation.

APPEAL TO THE SC:


Petitioner’s Contention: An award of full backwages and reinstatement would be contrary to law and
jurisprudence.

ISSUE:. Whether or not the actions warranted dismissal.

RULING: NO. As a general rule, an illegally dismissed employee is entitled to reinstatement (or separation pay, if
reinstatement is not viable) and payment of full backwages. In certain cases, however, the Court has carved out
an exception to the foregoing rule and thereby ordered the reinstatement of the employee without backwages on
account of the following: (a) the fact that dismissal of the employee would be too harsh of a penalty; and (b) that
the employer was in good faith in terminating the employee. Citing the case of Cruz v. Minister of Labor and
Employment, the court held that while an employee’s guilt may be sufficiently established, dismissal may be too

Angeles| Bajana | Balladares | Brillantes | Briones | Cabansag | Callanta | Chua | David|

De Leon | Gomez | Lopez | Macalino | Nostratis | Padilla | Reynon | Santos | Tan |Velasco
4E / 4F - 2018-2019
Page 582 of 920
LABOR REVIEW DIGEST
Atty. Joyrich Golangco

drastic of a punishment. Denial of Backwages may be sufficient to penalize an employee for his or her infractions.
Good faith on the part of the employer may preclude or diminish the recovery of backwages.

In this case in particular, it was held that indeed Pionilla was too harshly punished for his infractions, while
recognizing that IMI acted in good faith for dealing with what it perceived to ba a threat to its security.

———————————————————————————————————————-

Angeles| Bajana | Balladares | Brillantes | Briones | Cabansag | Callanta | Chua | David|

De Leon | Gomez | Lopez | Macalino | Nostratis | Padilla | Reynon | Santos | Tan |Velasco
4E / 4F - 2018-2019
Page 583 of 920
LABOR REVIEW DIGEST
Atty. Joyrich Golangco

19. Golden Ace Builders and Arnold U. Azul vs Jose A. Talde


GR No: 187200 Date: May 5, 2010
Ponente: Carpio Morales, J.

Doctrine:
Under the doctrine of strained relations, the payment of separation pay is considered an acceptable alternative to
reinstatement when the latter option is no longer desirable or viable. It is entirely different from payment of
backwages since backwages represent compensation that should have been earned but were not collected
because of the unjust dismissal.

FACTS:

Jose A. Talde (respondent) was hired in 1990 as a carpenter by petitioner Golden Ace Builders of which its co-
petitioner Arnold Azul (Azul) is the owner-manager.
In February 1999, Azul, alleging the unavailability of construction projects, stopped giving work assignments to
respondent, prompting the latter to file a complaint for illegal dismissal with the Labor Arbiter. LA ruled in favor of
respondent.
Pending their appeal to the National Labor Relations Commission (NLRC) and in compliance with the Labor
Arbiter's Decision, petitioners, through counsel, advised respondent to report for work in the construction site
within 10 days from receipt thereof. Respondent submitted, however, on May 16, 2001 a manifestation to the
Labor Arbiter that actual animosities existed between him and petitioners and there had been threats to his life
and his family's safety, hence, he opted for the payment of separation pay. Petitioners denied the existence of any
such animosity.

LA/RTC/NLRC RULING:
LA: Ruled in favor of respondent and ordered his reinstatement without loss of seniority rights and other privileges
with payment of full backwages.

NLRC: Dismissed petitioners appeal holding that respondent was a regular employee and not a project employee
and that there was no valid ground for termination

CA RULING:
Dismissed the appeal and granted full backwages and separation pay to respondent.

APPEAL TO THE SC:

Petitioner's Contention: Petitioners assail the appellate court's award of separation pay. They assailed too as
contrary to prevailing jurisprudence the computation of backwages from the time of dismissal up to actual
reinstatement. They contend that, in effect, the appellate court modified an already final and executory decision.

ISSUE/S:
1. Whether or not the appellate court erred in awarding separation pay.
2. When does the computation for separation pay begin.

HELD:
1. No.
The basis for the payment of backwages is different from that for the award of Separation pay. Separation pay is
granted where reinstatement is no longer advisable because of strained relations between the employee and the
Angeles| Bajana | Balladares | Brillantes | Briones | Cabansag | Callanta | Chua | David|

De Leon | Gomez | Lopez | Macalino | Nostratis | Padilla | Reynon | Santos | Tan |Velasco
4E / 4F - 2018-2019
Page 584 of 920
LABOR REVIEW DIGEST
Atty. Joyrich Golangco

employer. Backwages represent compensation that should have been earned but were not collected because of
the unjust dismissal. The basis for computing backwages is usually the length of the employee's service while
that for separation pay is the actual period when the employee was unlawfully prevented from working.
Under the doctrine of strained relations, the payment of separation pay is considered an acceptable alternative to
reinstatement when the latter option is no longer desirable or viable. On one hand, such payment liberates the
employee from what could be a highly oppressive work environment. On the other hand, it releases the employer
from the grossly unpalatable obligation of maintaining in its employ a worker it could no longer trust. Strained
relations must be demonstrated as a fact, however, to be adequately supported by evidence — substantial
evidence to show that the relationship between the employer and the employee is indeed strained as a necessary
consequence of the judicial controversy.
In the present case, the Labor Arbiter found that actual animosity existed between petitioner Azul and respondent
as a result of the filing of the illegal dismissal case. Such finding, especially when affirmed by the appellate court
as in the case at bar, is binding upon the Court, consistent with the prevailing rules that this Court will not try facts
anew and that findings of facts of quasi-judicial bodies are accorded great respect, even finality. Clearly then,
respondent is entitled to backwages and separation pay as his reinstatement has been rendered impossible due
to strained relations.
2. The day he is deemed to have been actually separated.
The Court, however, does not find the appellate court's computation of separation pay in order. The appellate
court considered respondent to have served petitioner company for only eight years. Petitioner was hired in 1990,
however, as he must be considered to have been in the service not only until 1999, when he was unjustly
dismissed, but until June 30, 2005, the day he is deemed to have been actually separated (his reinstatement
having been rendered impossible) from petitioner company or for a total of 15 years.

———————————————————————————————————————-

Angeles| Bajana | Balladares | Brillantes | Briones | Cabansag | Callanta | Chua | David|

De Leon | Gomez | Lopez | Macalino | Nostratis | Padilla | Reynon | Santos | Tan |Velasco
4E / 4F - 2018-2019
Page 585 of 920
LABOR REVIEW DIGEST
Atty. Joyrich Golangco

20. METROGUARDS SECURITY AGENCY CORP. v. ALBERTO HILONGO


GR No: 215630 Date: March 9, 2015
Ponente: Villarama, Jr., J.

Doctrine:
The computation of the monetary consequences (backwages and separation pay) of the illegal dismissal should
be reckoned from its finality- when the CA decision became final and executory. Since the computation of the
monetary award was only up to the date of the LA Decision, there must be a computation of additional backwages
and separation pay.

A recomputation (or an original computation, if no previous computation has been made) is a part of the law –
specifically, Article 279 of the Labor Code and the established jurisprudence on this provision – that is read into
the decision. By the nature of an illegal dismissal case, the reliefs continue to add up until full satisfaction, as
expressed under Article 279 of the Labor Code. The recomputation of the consequences of illegal dismissal upon
execution of the decision does not constitute an alteration or amendment of the final decision being implemented.
.
FACTS:
The LA rendered a judgment finding the dismissal of respondent Alberto Hilongo illegal and ordering the
petitioners to pay complainant his backwages from the date of dismissal to the date of his decision and separation
pay.

NLRC RULING:
On appeal, the NLRC reversed the LA ruling.

CA RULING:
The CA reversed the NLRC decision and reinstated the LA decision.

Hilongo then filed a motion for entry of judgment and a motion for clarification of Decision/Resolution praying that
the CA’s Resolution be clarified and interpreted to include the amount of the award as stated in the LA’s Decision
and additional award computed from the date the CA denied petitioners’ motion for reconsideration.

The CA granted the motion for entry of judgment and noted Hilongo’s motion for clarification of
Decision/Resolution. The CA held that when an appellate court affirms the Labor Arbiter’s ruling, it is understood
that awards due to the illegally dismissed employee shall be recomputed in order to account for the period of time
that has lapsed from the rendition of the Labor Arbiter’s decision up to its finality.

The CA held that it is already settled that the computation of the monetary awards due to the illegally dismissed
employee must continue to run until the final termination of the case on appeal.

ISSUE/S:
Whether the re-computation of the consequences of illegal dismissal upon execution of the decision constitute an
alteration of the final decision being implemented.

HELD:
NO. A re- computation (or an original computation, if no previous computation has been made) is a part of the law
– specifically, Article 279 of the Labor Code and the established jurisprudence on this provision – that is read into
the decision. By the nature of an illegal dismissal case, the reliefs continue to add on until full satisfaction, as
expressed under Article 279 of the Labor Code. The re-computation of the consequences of illegal dismissal upon
execution of the decision does not constitute an alteration or amendment of the final decision being implemented.

Angeles| Bajana | Balladares | Brillantes | Briones | Cabansag | Callanta | Chua | David|

De Leon | Gomez | Lopez | Macalino | Nostratis | Padilla | Reynon | Santos | Tan |Velasco
4E / 4F - 2018-2019
Page 586 of 920
LABOR REVIEW DIGEST
Atty. Joyrich Golangco

The illegal dismissal ruling stands; only the computation of monetary consequences of this dismissal is affected
and this is not a violation of the principle of immutability of final judgments.

Thus, the April 30, 2010 Decision of the Labor Arbiter which ordered the payment of separation pay in lieu of
reinstatement, effectively ended the employment relationship of the parties on April 26, 2013, the date the CA
decision became final. Since the Labor Arbiter’s computation of Hilongo’s monetary award was up to the date of
his April 30, 2010 Decision only, the CA properly decreed the computation of additional back wages and
separation pay.

———————————————————————————————————————-

Angeles| Bajana | Balladares | Brillantes | Briones | Cabansag | Callanta | Chua | David|

De Leon | Gomez | Lopez | Macalino | Nostratis | Padilla | Reynon | Santos | Tan |Velasco
4E / 4F - 2018-2019
Page 587 of 920
LABOR REVIEW DIGEST
Atty. Joyrich Golangco

30. 21. Maersk-Filipinas Crewing, Inc., A.P. Moller Singapore Pte. Limited, And Jesus Agbayani V.
Toribio C. Avestruz
31. GR No: 207010 Date: February 18, 2015
Ponente: PERLAS-BERNABE, J
32. Doctrine:
33. It is well-settled that the burden of proving that the termination of an employee was for a just or
authorized cause lies with the employer. If the employer fails to meet this burden, the conclusion would be that the
dismissal was unjustified and, therefore, illegal.
34.
35. FACTS:
36. Maersk-Filipinas Crewing, Inc. (Maersk) hired Avestruz as a chief cook on board a vessel for a period of
6 months. On June 22, 2011, in the course of the weekly inspection of the vessel's galley, Captain Charles C.
Woodward (Captain Woodward) noticed that the cover of the garbage bin in the kitchen near the washing area
was oily. As part of Avestruz's job was to ensure the cleanliness of the galley, Captain Woodward called Avestruz
and asked him to stand near the garbage bin where the former took the latter's right hand and swiped it on the oily
cover of the garbage bin, telling Avestruz to feel it. Shocked, Avestruz remarked, "Sir if you are looking for [dirt],
you can find it[;] the ship is big. Tell us if you want to clean and we will clean it." Captain Woodward replied by
shoving Avestruz's chest, to which the latter complained and said, "Don't touch me," causing an argument to
ensue between them.
37.
38. Captain Woodward required Avestruz and Messman Kong to submit their written statements. On the very
same day, Captain Woodward informed Avestruz that he would be dismissed from service and be disembarked in
order to return to the Phils.
39.
40. Avestruz filed a complaint for illegal dismissal, payment for the unexpired portion of his contract,
damages, and attorney’s fees against petitioners. He alleged that no investigation or hearing was conducted nor
was he given the chance to defend himself before he was dismissed, and that Captain Woodward failed to
observe the provisions under Section 17 of the Philippine Overseas Employment Administration (POEA) Standard
Employment Contract (POEA-SEC) on disciplinary procedures.
41.
42. Petitioners maintained that during his stint on the vessel, Avestruz failed to attend to his tasks,
specifically to maintain the cleanliness of the galley, he was informed of his dismissal from service due to
insubordination, and Avestruz was dismissed for a just and valid cause and is, therefore, not entitled to recover
his salary for the unexpired portion of his contract.
43.
44. LA Ruling:
45. LA dismissed Avestruz's complaint for lack of merit. The LA found that he failed to perform his duty of
maintaining cleanliness in the galley, and that he also repeatedly failed to obey the directives of his superior,
which was tantamount to insubordination.
46.
47. NLRC Ruling:
48. NLRC sustained the validity of Avestruz's dismissal but found that petitioners failed to observe the
procedures laid down in Section 17 of the POEA-SEC or the two-notice rule.
49.
50. CA RULING:
51. CA found Avestruz to have been illegally dismissed on the ground that the conclusion of NLRC was not
supported by substantial evidence, there being no factual basis for the charge of insubordination.
52.
53. APPEAL TO THE SC:
54. Petitioner’s contention:
Angeles| Bajana | Balladares | Brillantes | Briones | Cabansag | Callanta | Chua | David|

De Leon | Gomez | Lopez | Macalino | Nostratis | Padilla | Reynon | Santos | Tan |Velasco
4E / 4F - 2018-2019
Page 588 of 920
LABOR REVIEW DIGEST
Atty. Joyrich Golangco

55. Petitioners maintain that Avestruz was dismissed on the ground of insubordination, consisting of his
"repeated failure to obey his superior's order to maintain cleanliness in the galley of the vessel" as well as his act
of "insulting a superior officer by words or deeds."
56.
57. ISSUE/S:
58. 1. Whether Avestruz was illegally dismissed?
59. 2. Was there compliance with the provisions of Section 17 of the POEA-SEC
60.
61. HELD:
62. 1.YES. It is well-settled that the burden of proving that the termination of an employee was for a just or
authorized cause lies with the employer. If the employer fails to meet this burden, the conclusion would be that the
dismissal was unjustified and, therefore, illegal. Insubordination, as a just cause for the dismissal of an employee,
necessitates the concurrence of at least two requisites: (1) the employee's assailed conduct must have been
willful, that is, characterized by a wrongful and perverse attitude; and (2) the order violated must have been
reasonable, lawful, made known to the employee, and must pertain to the duties which he had been engaged to
discharge. In this case, the contents of Captain Woodward's e-mails do not establish that Avestruz's conduct had
been willful, or characterized by a wrongful and perverse attitude. Therefore, Avestruz was illegally dismissed.
63.
64. 2. NO. An erring seaman is given a written notice of the charge against him and is afforded an
opportunity to explain or defend himself. Should sanctions be imposed, then a written notice of penalty and the
reasons for it shall be furnished the erring seafarer. It is only in the exceptional case of clear and existing danger
to the safety of the crew or vessel that the required notices are dispensed with; but just the same, a complete
report should be sent to the manning agency, supported by substantial evidence of the findings. In this case, there
is dearth of evidence to show that Avestruz had been given a written notice of the charge against him, or that he
was given the opportunity to explain or defend himself. Neither was Avestruz given a written notice of penalty and
the reasons for its imposition. Instead, Captain Woodward verbally informed him that he was dismissed from
service and would be disembarked from the vessel. Therefore, there was no compliance with the provisions of
Section 17 of the POEA-SEC which requires the "two-notice rule."

———————————————————————————————————————-

Angeles| Bajana | Balladares | Brillantes | Briones | Cabansag | Callanta | Chua | David|

De Leon | Gomez | Lopez | Macalino | Nostratis | Padilla | Reynon | Santos | Tan |Velasco
4E / 4F - 2018-2019
Page 589 of 920
LABOR REVIEW DIGEST
Atty. Joyrich Golangco

22. CONCEPCION A. VILLENA v. BATANGAS II ELECTRIC COOPERATIVE, INC. and GEORGE A. DIN
G.R. No. 205735 Date: February 4, 2015
Ponente: PERLAS-BERNABE, J.

DOCTRINE:
The SC is not unaware of its rulings wherein it pronounced that retirement pay and separation pay are not
mutually exclusive (unless there is a specific prohibition in the collective bargaining agreement [CBA] or retirement
plan against the payment of both benefits)—however Villena’s entitlement to retirement pay is NOT included as an
issue in an illegal dismissal case which had already been finally decided, it is quite absurd for Villena to submit a
“contemporaneous” claim for retirement pay on the execution phase of these proceedings. Hence, the plea to
include retirement pay in the execution of the final and executor CA Decision and NLRC Resolution, under the
phrase “other benefits,” CANNOT be granted.

FACTS:

In 1978, Petitioner Concepcion Villena was hired by respondent Batangas II Electric Cooperative, Inc. (BATELEC
II) as bookkeeper. She was promoted as Finance Manager in 1985. In 1994, she was demoted to the position of
Auditor, which caused her to file a complaint for constructive dismissal before the LA. After the case passed
through LA and NLRC, the CA ruled that Villena was illegally dismissed and she was entitled among others to any
other benefits pertaining to the position of Finance Manager at the time she was removed therefrom up to the date
of her actual reinstatement. The case was remanded to the NLRC for the computation of the total amount due.

LA—declared that Villena was entitled only to "salary differentials, 13th month pay, unused sick leave, leave of
absence" excluding from the computation claims for bonus, representation allowance, transportation benefits, and
attorney’s fees. Moreover, her claim for separation pay in lieu of reinstatement was denied.

NLRC—declared that Villena was entitled to her claim for separation pay in lieu of reinstatement, salary
differentials and other benefits, from the date of her dismissal up to the date of the payment of her separation pay,
and attorney’s fees equivalent to 10% of the totality of her award.

The NLRC Resolution became final and executory. Hence, Villena filed a motion for execution before the LA.

LA RULING:
Villena is entitled to the following benefits: (a) salary differentials; (b) 13th month pay; (c) 14th month pay; (d)
bonus cash gift; (e) unused sick leave; (f) leave of absence; (g) uniform allowance; (h) separation pay; (i)
representation allowance; (j) transportation allowance; (k) cellular phone allowance; (l) retirement pay; and (m)
attorney’s fees.

NLRC RULING:
Excluded from the computation of monetary awards the sums for representation, transportation, and cellular
phone usage allowances, as well as retirement pay. Villena was not able to prove that she was qualified to receive
representation allowance or that she was authorized to travel, and there was no basis for the award of cellular
phone allowance.

CA RULING:
Affirmed the NLRC disallowing the inclusion of allowances for representation, transportation, and cellular phone
usage as Villena did not perform her duties as Finance Manager not being a certified public accountant which is a
required qualification for such position.

BATELEC II’s contention:


Angeles| Bajana | Balladares | Brillantes | Briones | Cabansag | Callanta | Chua | David|

De Leon | Gomez | Lopez | Macalino | Nostratis | Padilla | Reynon | Santos | Tan |Velasco
4E / 4F - 2018-2019
Page 590 of 920
LABOR REVIEW DIGEST
Atty. Joyrich Golangco

Villena was not entitled to salary differentials, allowances and benefits of a Finance Manager, separation pay, and
allowances for representation, transportation, and cellular phone usage.

ISSUES:
(1) Is Villena entitled to the award of retirement pay? NO.
(2) Is Villena entitled to the award of representation, transportation, and cellular phone usage allowances? YES.

RULING:
(1) NO. In order for her retirement pay claim to be considered, Villena’s complaint should have contained
substantial allegations which would show that:
(a) She had applied for the same,
(b) Her application squares with the requirements of entitlement under the terms of the company’s retirement
plan.

However, based on the records, what she sought for in her illegal dismissal complaint were the reliefs of
reinstatement, payment of salary differentials, all benefits and allowances that she may have received as
Finance Manager, attorney’s fees, and damages (which does not include retirement pay).

Verily, the Court is not unaware of its rulings wherein it pronounced that retirement pay and separation pay
are not mutually exclusive (unless there is a specific prohibition in the collective bargaining agreement or
retirement plan against the payment of both benefits); however, with Villena’s entitlement to retirement pay
not included as an issue in an illegal dismissal case which had already been finally decided, it is quite absurd
for Villena to submit a "contemporaneous" claim for retirement pay on the execution phase of these
proceedings. In fine, the plea to include retirement pay in the execution under the phrase "other benefits,"
cannot be granted.

(2) YES. It is clear from BATELEC II’s pleadings and submissions that representation allowance, transportation
allowance, and cellular phone usage allowance are given to the Finance Manager/Department Manager as
part of their benefits, unlike the separate entitlement to retirement pay which may be recovered only upon a
meritorious subsequent application when the employee decides to retire. Consequently, these allowances
ought to be included in the "other benefits pertaining to the position of Finance Manager" to which Villena is
entitled to and which were awarded to her under the final and executory CA Decision and NLRC Resolution.
Hence, the claim of Villena is granted.
———————————————————————————————————————-

Angeles| Bajana | Balladares | Brillantes | Briones | Cabansag | Callanta | Chua | David|

De Leon | Gomez | Lopez | Macalino | Nostratis | Padilla | Reynon | Santos | Tan |Velasco
4E / 4F - 2018-2019
Page 591 of 920
LABOR REVIEW DIGEST
Atty. Joyrich Golangco

23. SANGWOO PHILIPPINES, INC. and/or SANG IK JANG, JISSO JANG, WISSO JANG and NORBERTO
TADEO, Petitioners, vs. SANGWOO PHILIPPINES, INC. EMPLOYEE UNION - OLALIA, represented by
PORFERIA SALIBONGCOGON, Respondents.
GR No: 173154 Date: December 9, 2013
Ponente: PERLAS-BERNABE, J.

Doctrine: In this case, the LA, NLRC, and the CA all consistently found that SPI indeed suffered from serious
business losses which resulted in its permanent shutdown and accordingly, held the company’s closure to be
valid. Perforce, without any cogent reason to deviate from the findings on the validity of SPI’s closure, the Court
thus holds that SPI is not obliged to give separation benefits to the minority employees pursuant to Article 297 of
the Labor Code as interpreted in the case of Galaxie.

FACTS:
On July 25, 2003, during the collective bargaining agreement (CBA) negotiations between Sangwoo Philippines,
Inc. Employees Union – Olalia (SPEU) and Sangwoo Philippines, Inc.(SPI), the latter filed with the Department of
Labor and Employment (DOLE) a letter-notice of temporary suspension of operations for one (1) month, beginning
September 15, 2003, due to lack of orders from its buyers. SPEU was furnished a copy of the said letter.
59. Negotiation son the CBA, however, continued and on September 10, 2003, the parties signed a
handwritten Memorandum of Agreement, which, among others, specified the employees’ wages and benefits for
the next two (2) years, and that in the event of a temporary shutdown, all machineries and raw materials would not
be taken out of the SPI premises.
60. On September 15, 2003, SPI temporarily ceased operations. Thereafter, it successively filed two (2)
letters with the DOLE, copy furnished SPEU, for the extension of the temporary shutdown until March 15, 2004.
61. Meanwhile, on October 28, 2003, SPEU filed a complaint for unfair labor practice, illegal closure, illegal
dismissal, damages and attorney’s fees before the Regional Arbitration Branch IV of the NLRC.
62. SPI offered separation benefits of one-half (½) month pay for every year of service to each of its
employees. 234 employees of SPI accepted the offer, received the said sums and executed
quitclaims.14 Thosewho refused the offer, i.e., the minority employees, were nevertheless given until March 25,
2004 to accept their checks and correspondingly, execute quitclaims. However, the minority employees did not
claim the said checks.
63. Labor Arbiter (LA) ruled in favor of SPI. The LA found that SPI was indeed suffering from serious
business losses–as evidenced by financial statements which were never contested by SPEU –and, as such,
validly discontinued its operations. LA ruled that since SPI’s closure of business was due to serious business
losses, it was not mandated by law to grant separation benefits to the minority employees.
64. NLRC sustained the ruling of the LA, albeit with modification. While it upheld SPI’s closure due to serious
business losses, it ruled that the members of SPEU are entitled to payment of separation pay equivalent to one-
half (½) month pay for every year of service. In this relation, the NLRC opined that since SPI already gave
separation benefits to 234 of its employees, the minority employees should not be denied of the same.
65. CA held that the minority employees were not entitled to separation pay considering that the company’s
closure was due to serious business losses. It pronounced that requiring an employer to be generous when it was
no longer in a position to be so would be oppressive and unjust. Nevertheless, the CA still ordered SPI to pay the
minority employees ₱15,000.00 each, representing the amount of financial assistance as contained in the Formal
Offer of Settlement.
66. ISSUE/S:
1. Whether or not the minority employees are entitled to separation pay.
2. Whether or not SPI complied with the notice requirement of Article 297 (formerly Article 283)26 of the Labor
Code.

HELD:

Angeles| Bajana | Balladares | Brillantes | Briones | Cabansag | Callanta | Chua | David|

De Leon | Gomez | Lopez | Macalino | Nostratis | Padilla | Reynon | Santos | Tan |Velasco
4E / 4F - 2018-2019
Page 592 of 920
LABOR REVIEW DIGEST
Atty. Joyrich Golangco

1. NO. Closure of business is the reversal of fortune of the employer whereby there is a complete cessation of
business operations and/or an actual locking-up of the doors of establishment, usually due to financial losses.
Closure of business, as an authorized cause for termination of employment,27 aims to prevent further financial
drain upon an employer who cannot pay anymore his employees since business has already stopped.28 In such a
case, the employer is generally required to give separation benefits to its employees, unless the closure is due to
serious business losses.

In this case, the LA, NLRC, and the CA all consistently found that SPI indeed suffered from serious business
losses which resulted in its permanent shutdown and accordingly, held the company’s closure to be valid. It is a
rule that absent any showing that the findings of fact of the labor tribunals and the appellate court are not
supported by evidence on record or the judgment is based on a misapprehension of facts, the Court shall not
examine a new the evidence submitted by the parties.31

Perforce, without any cogent reason to deviate from the findings on the validity of SPI’s closure, the Court thus
holds that SPI is not obliged to give separation benefits to the minority employees pursuant to Article 297 of the
Labor Code as interpreted in the case of Galaxie.

As such, SPI should not be directed to give financial assistance amounting to ₱15,000.00 to each of the minority
employees based on the Formal Offer of Settlement. If at all, such formal offer should be deemed only as a
calculated move on SPI’s part to further minimize the expenses that it will be bound to incur should litigation drag
on, and not as an indication that it was still financially sustainable. However, since SPEU chose not to accept, said
offer did not ripen into an enforceable obligation on the part of SPI from which financial assistance could have
been realized by the minority employees.
67. 2. NO. Article 297 of the Labor Code provides that before any employee is terminated due to closure of
business, it must give a one (1) month prior written notice to the employee and to the DOLE. In this relation, case
law instructs that it is the personal right of the employee to be personally informed of his proposed dismissal as
well as the reasons therefor; and such requirement of notice is not a mere technicality or formality which the
employer may dispense with.
68. Keeping with these principles, the Court finds that the LA, NLRC, and CA erred in ruling that SPI
complied with the notice requirement when it merely posted various copies of its notice of closure in conspicuous
places within the business premises.
69. As earlier explained, SPI was required to serve written notices of termination to its employees, which it,
however, failed to do. It is well to stress that while SPI had a valid ground to terminate its employees, i.e., closure
of business, its failure to comply with the proper procedure for termination renders it liable to pay the employee
nominal damages for such omission. Based on existing jurisprudence, an employer which has a valid cause for
dismissing its employee but conducts the dismissal with procedural infirmity is liable to pay the employee nominal
damages in the amount of ₱30,000.00 if the ground for dismissal is a just cause, or the amount of ₱50,000.00 if
the ground for dismissal is an authorized cause.35 However, case law exhorts that in instances where the payment
of such damages becomes impossible, unjust, or too burdensome, modification becomes necessary in order to
harmonize the disposition with the prevailing circumstances.36
70. Thus, in the case of Industrial Timber Corporation v. Ababon37 (Industrial Timber),the Court reduced the
amount of nominal damages awarded to employees from ₱50,000.00 to ₱10,000.00 since the authorized cause
of termination was the employer’s closure or cessation of business which was done in good faith and due to
circumstances beyond the employer’s control,viz.:38
71. In the determination of the amount of nominal damages which is addressed to the sound discretion of the
court, several factors are taken into account: (1) the authorized cause invoked, whether it was a retrenchment or a
closure or cessation of operation of the establishment due to serious business losses or financial reverses or
otherwise; (2) the number of employees to be awarded; (3) the capacity of the employers to satisfy the awards,
taken into account their prevailing financial status as borneby the records; (4) the employer’s grant of other

Angeles| Bajana | Balladares | Brillantes | Briones | Cabansag | Callanta | Chua | David|

De Leon | Gomez | Lopez | Macalino | Nostratis | Padilla | Reynon | Santos | Tan |Velasco
4E / 4F - 2018-2019
Page 593 of 920
LABOR REVIEW DIGEST
Atty. Joyrich Golangco

termination benefits in favor of the employees; and (5) whether there was a bona fide attempt to comply with the
notice requirements as opposed to giving no notice at all.
72. To be clear, the foregoing award should only obtain in favor of the minority employees and not for those
employees who already received sums equivalent to separation pay and executed quitclaims "releasing [SPI] now
and in the future any claims and obligation which may arise as results of [their] employment with the
company."39 For these latter employees who have already voluntarily accepted their dismissal, their executed
quitclaims practically erased the consequences of infirmities on the notice of dismissal, 40 at least as to them.
——————————————————————————————————————-

Angeles| Bajana | Balladares | Brillantes | Briones | Cabansag | Callanta | Chua | David|

De Leon | Gomez | Lopez | Macalino | Nostratis | Padilla | Reynon | Santos | Tan |Velasco
4E / 4F - 2018-2019
Page 594 of 920
LABOR REVIEW DIGEST
Atty. Joyrich Golangco

Article 295

65. 1. LYNVIL FISHING ENTERPRISES VS ARIOLA


66. GR No. 181974 Date: February 1, 2012
67. Ponente: Perez, J.
68.
69. Doctrine:
70. Any employee who has rendered at least one year of service, whether such service is continuous or
broken, shall be considered a regular employee with respect to the activity in which he is employed and his
employment shall continue while such activity exists.
71.
72. FACTS:
73. Lynvil is a company engaged in deep—sea fishing. The respondents were Lynvil’s employees whose
employments were terminated when Lynvil found out upon investigation that the respondents conspired with one
another and stole 8 tubs of tanigue fish and delivered them to another vessel to the prejudice of Lynvil. Lynvil
then dismissed the said employees from employment.
74.
75. Lynvil filed a criminal complaint against the dismissed employees and the City Prosec found probable
cause for the crime of qualified theft.
76.
77. The employees then filed a case for illegal dismissal.
78.
79. LA RULING:
80.
81. The LA ruled that the respondents were illegally dismissed and disregarded the resolution of the City
Prosec on the theft case. He reasoned that the Labor Office is governed by different rules for the determination of
the validity of the dismissal of employees. It further held that the procedural due process was not complied with
because mere notice to the dismissed employees fell short of the requirement of ample opportunity to present the
employees side.
82.
83. NLRC RULING:
84. On appeal, the NLRC found that the dismissed employees were merely contractual employees. Thus, it
reversed the LA decision and dismissed the complaint for lack of merit.
85.
86. CA RULING:
87. On appeal to the CA, it reinstated the LA decision and held that the allegations of theft did not warrant
the dismissal and that the dismissed employees were regular employees, hence, entitled to the twin-notice rule.
88.
89. APPEAL TO THE SC:
90.
91. Petitioner's Contention:
92. The filing of a criminal case before the Office of the Prosecutor is sufficient basis for a valid termination of
employment based on serious misconduct and/or loss of trust and confidence.
93.
94. Lynvil further contends that it cannot be guilty of illegal dismissal because the private respondents were
employed under a fixed-term contract which expired at the end of the voyage.
95.
96. Respondent's Contention:
97.

Angeles| Bajana | Balladares | Brillantes | Briones | Cabansag | Callanta | Chua | David|

De Leon | Gomez | Lopez | Macalino | Nostratis | Padilla | Reynon | Santos | Tan |Velasco
4E / 4F - 2018-2019
Page 595 of 920
LABOR REVIEW DIGEST
Atty. Joyrich Golangco

98. The private respondents contend that they became regular employees by reason of their continuous
hiring and performance of tasks necessary and desirable in the usual trade and business of Lynvil.
99.
100. ISSUE/S:
1. Were the respondents validly dismissed based on the filing of the criminal case against them? - Yes
2. Are the respondents regular employees?
101.
102. HELD:
1. Yes, but not based on findings of the City Prosec. Whichever way the public prosecutor disposes of a
complaint, the finding does not bind the labor tribunal. Lynvil cannot argue that since the Office of the
Prosecutor found probable cause for theft the LA must follow the finding as a valid reason for the termination
of respondents’ employment. The proof required for purposes that differ from one and the other are likewise
different.
103.
104. Nonetheless, even without reliance on the prosecutor’s finding, there was valid cause for respondents’
dismissal. In illegal dismissal cases, the employer bears the burden of proving that the termination was for a valid
or authorized cause. Just cause is required for a valid dismissal. The Labor Code provides that the employer may
terminate an employment based on fraud or willful breach of trust reposed on the employee. Breach of trust is
present in this case.
105.
106. 2. Jurisprudence, laid two conditions for the validity of a fixed-contract agreement between the employer
and employee:
107.
108. First, the fixed period of employment was knowingly and voluntarily agreed upon by the parties without
any force, duress, or improper pressure being brought to bear upon the employee and absent any other
circumstances vitiating his consent; or Second, it satisfactorily appears that the employer and the employee dealt
with each other on more or less equal terms with no moral dominance exercised by the former or the latter.
109.
110. In the context, however, of the facts that: (1) the respondents were doing tasks necessarily to Lynvils
fishing business with positions ranging from captain of the vessel to bodegero; (2) after the end of a trip, they will
again be hired for another trip with new contracts; and (3) this arrangement continued for more than ten years, the
clear intention is to go around the security of tenure of the respondents as regular employees. Thus, respondents
are considered regular employees.
111.
112. Having found that respondents are regular employees who may be, however, dismissed for cause as we
have so found in this case, there is a need to look into the procedural requirement of due process.
113.
114. It is required that the employer furnish the employee with two written notices: (1) a written notice served
on the employee specifying the ground or grounds for termination, and giving to said employee reasonable
opportunity within which to explain his side; and (2) a written notice of termination served on the employee
indicating that upon due consideration of all the circumstances, grounds have been established to justify his
termination.
115.
116. From the records, there was only one written notice which required respondents to explain within 5 days
why they should not be dismissed from the service. Alcovendas was the only one who signed the receipt of the
notice. The others, as claimed by Lynvil, refused to sign. The other employees argue that no notice was given to
them. Despite the inconsistencies, what is clear is that no final written notice or notices of termination were sent to
the employees.
117.

Angeles| Bajana | Balladares | Brillantes | Briones | Cabansag | Callanta | Chua | David|

De Leon | Gomez | Lopez | Macalino | Nostratis | Padilla | Reynon | Santos | Tan |Velasco
4E / 4F - 2018-2019
Page 596 of 920
LABOR REVIEW DIGEST
Atty. Joyrich Golangco

118. The twin requirements of notice and hearing constitute the elements of due process in cases of
employee's dismissal. The requirement of notice is intended to inform the employee concerned of the employer's
intent to dismiss and the reason for the proposed dismissal. Upon the other hand, the requirement of hearing
affords the employee an opportunity to answer his employer's charges against him and accordingly, to defend
himself therefrom before dismissal is effected. Obviously, the second written notice, as indispensable as the first,
is intended to ensure the observance of due process.
119.
120. Applying the rule to the facts at hand, we grant a monetary award of P50,000.00 as nominal damages,
121. __________________________________________________________________________

Angeles| Bajana | Balladares | Brillantes | Briones | Cabansag | Callanta | Chua | David|

De Leon | Gomez | Lopez | Macalino | Nostratis | Padilla | Reynon | Santos | Tan |Velasco
4E / 4F - 2018-2019
Page 597 of 920
LABOR REVIEW DIGEST
Atty. Joyrich Golangco

2. JOSE Y. SONZA vs. ABS-CBN BROADCASTING CORPORATION


GR No.: 138051 Date: JUNE 10, 2004
Ponente: Carpio, J.

DOCTRINE:

Being an exclusive talent does not by itself mean that one is an employee. In the broadcast industry, exclusivity is
not necessarily the same as control.

FACTS:

ABS-CBN signed an Agreement with Mel and Jay Management and Development Corporation (MJMDC), as
“AGENT” of Sonza, as President and GM, and Carmela Tiangco, as EVP and Treasurer. MJMDC agreed to
provide Sonza’s services exclusively to ABS-CBN as talent for radio and television.

On April 1, 1996, Sonza wrote to ABS-CBN’s President Eugenio Lopez III serving a notice of rescission of the
Agreement at their instance effective as of date thereby waiving and renouncing recovery of the remaining amount
stipulated but reserving the right to seek recovery to other benefits pursuant to Sonza’s resignation in view of
recent events concerning his programs and career due to ABS-CBN’s act and breach violative of the Agreement.

Sonza filed a complaint against ABS-CBN before the DOLE, NCR in QC on the ground that ABS-CBN did not pay
his salaries, separation pay, service incentive leave pay, 13th month pay, signing bonus, travel allowance, and
amounts due under the Employees Stock Option Plan (ESOP). ABS-CBN filed a Motion to Dismiss on the ground
that no employer-employee relationship existed between the parties. Meanwhile ABS-CBN continued to remit
Sonza’s monthly talent fees.

LA DECISION: Denied the motion to dismiss and directed the parties to file their position papers ruling that Sonza,
for having invoked a claim that he was an employee and was not paid certain claims, is sufficient enough to confer
jurisdiction over the case. On February 24, 1997, the position papers were submitted.

Sonza filed a Reply with Motion to Expunge the affidavits of ABS-CBN’s witnesses Soccoro Vidanes and Rolando
Cruz which stated that the prevailing practice in the tv and broadcast industry is to treat talents as independent
contractors. LA dismissed the complaint for lack of jurisdiction on the ground that a talent cannot be considered as
an employee by reason of the peculiar circumstances surrounding the engagement of his services:

(1) He was free to perform the services he undertook to render in accordance with his own style, his benefits
were very much higher than those given to employees, and he was not bound to render 8 hours of work
per day.
(2)Whatever benefits Sonza enjoyed arose from specific agreement by the parties and not by reason of the
employer-employee relationship. All these benefits are merely talent fees and other contractual benefits and
should not be deemed as salaries, wages, and/or other remuneration accorded to an employee, notwithstanding
the nomenclature appended to these benefits. Apropos to this is the rule that the term or nomenclature given to a
stipulated benefit is not controlling, but the intent of parties to the Agreement conferring such benefit.
(3)The fact that Sonza was made subject to ABS-CBN’s Rules and Regulations does not detract from the
absence of employer-employee relationship as such merely served as guidelines toward the achievement of the
mutually desired result without dictating the means or methods to be employed in attaining it.

NLRC DECISION: Affirmed the LA’s decision.

ISSUE/S:
Angeles| Bajana | Balladares | Brillantes | Briones | Cabansag | Callanta | Chua | David|

De Leon | Gomez | Lopez | Macalino | Nostratis | Padilla | Reynon | Santos | Tan |Velasco
4E / 4F - 2018-2019
Page 598 of 920
LABOR REVIEW DIGEST
Atty. Joyrich Golangco

Is Sonza an employee of ABS-CBN?

HELD:

NO. Independent contractors often present themselves to possess unique skills, expertise, or talent to distinguish
them from ordinary employees. The specific selection and hiring of Sonza because of his unique skills, talent, and
celebrity status not possessed by ordinary employees is a circumstance indicative, but not conclusive, of an
independent contractual relationship. If SONZA did not possess such unique skills, talent and celebrity status,
ABS- CBN would not have entered into the Agreement with SONZA but would have hired him through its
personnel department just like any other employee.

All the talent fees and benefits paid to Sonza were the result of negotiations that led to the Agreement. If Sonza
were ABS-CBN’s employee, there would be no need for the parties to stipulate on the benefits as such are
automatically incorporated into every employer-employee contract by the law. Whatever benefits Sonza enjoyed
arose from contract and not because of an employer-employee relationship. The payment of talent fees directly to
SONZA and not to MJMDC does not negate the status of SONZA as an independent contractor. The parties
expressly agreed on such mode of payment.

Sonza failed to show that ABS-CBN could terminate his services on grounds other than breach of contract, such
as retrenchment to prevent losses as provided under labor laws. During the life of the Agreement, even if it
suffered severe business losses, ABS-CBN could not retrench Sonza because it remained obligation to pay
Sonza’s talent fees. This circumstance indicates an independent contractual relationship between Sonza and
ABS-CBN. Even after ABS-CBN ceased broadcasting his programs, ABS-CBN still paid him his talent fees.

Furthermore, while Sonza did actually resign from ABS-CBN, he also, as president of MJMDC, rescinded the
Agreement.
The control test is the most important test the courts apply in distinguishing an employee from an independent
contractor. It is based on the extent of control the hirer exercises over a worker. The greater the supervision and
control the hirer exercises, the more likely the worker is deemed an employee. The converse holds true as well –
the less control the hirer exercises, the more likely the worker is considered as an independent contractor.

Control over the means and methods of work.


ABS-CBN engaged SONZA’s services specifically to cohost the “Mel & Jay” programs. ABS-CBN did not assign
any other work to SONZA. To perform his
work, SONZA only needed his skills and talent. How SONZA delivered his lines, appeared on television, and
sounded on radio were outside ABS-CBN’s control. SONZA did not have to render eight hours of work per day.

he Agreement required SONZA to attend only rehearsals and tapings of the shows, as well as pre- and post-
production staff meetings, as well as pre- and post-production staff meetings. ABS-CBN could not dictate the
contents of SONZA’s script. However, the Agreement prohibited SONZA from criticizing in his shows ABS-CBN or
its interests. The clear implication is that SONZA had a free hand on what to say or discuss in his shows provided
he did not attack ABS-CBN or its interests. ABS-CBN was not involved in the actual performance that produced
the finished product of Sonza’s work. ABS-CBN merely reserved the right to modify the program format and
airtime schedule “for more effective programming.” ABS-CBN’s sole concern was the quality of the shows and
their standing in the ratings. Clearly, ABS- CBN did not exercise control over the means and methods of
performance of SONZA’s work. A radio broadcast specialist who works under minimal supervision is an
independent contractor.

Subjected to rules and standards of performance.


Angeles| Bajana | Balladares | Brillantes | Briones | Cabansag | Callanta | Chua | David|

De Leon | Gomez | Lopez | Macalino | Nostratis | Padilla | Reynon | Santos | Tan |Velasco
4E / 4F - 2018-2019
Page 599 of 920
LABOR REVIEW DIGEST
Atty. Joyrich Golangco

The Agreement stipulates the Sonza shall abide with the rules and standards of performance covering talents of
ABS-CBN. One could still be an independent contractor although the hirer reserved certain supervision to insure
the attainment of the desired result. The hirer, however, must not deprive the one hired from performing his
services according to his own initiative.

Exclusivity clause.
Being an exclusive talent does not by itself mean that SONZA is an employee ofABS-CBN. In the broadcast
industry, exclusivity is not necessarily the same as control. The hiring of exclusive talents is a widespread and
accepted practice in the entertainment industry since the broadcast station normally spends substantial amounts
of money, time and effort in building up its talents and programs they appear in.

Sonza’s claim that the practice to treat talents as independent contractors is void for violating the right of labor to
security of tenure. The right of labor to security of tenure as guaranteed in the Constitution arises only if there is
an employer- employee relationship under labor laws. Not every performance of services for a fee creates an
employer- employee relationship. To hold that every person who renders services to another for a fee is an
employee - to give meaning to the security of tenure clause - will lead to absurd results.

The Court will not interpret the right of labor to security of tenure to compel artists and talents to render their
services only as employees. If radio and television program hosts can render their services only as employees,
the station owners and managers can dictate to the radio and television hosts what they say in their shows. This is
not conducive to freedom of the press.
———————————————————————————————————————-

Angeles| Bajana | Balladares | Brillantes | Briones | Cabansag | Callanta | Chua | David|

De Leon | Gomez | Lopez | Macalino | Nostratis | Padilla | Reynon | Santos | Tan |Velasco
4E / 4F - 2018-2019
Page 600 of 920
LABOR REVIEW DIGEST
Atty. Joyrich Golangco

3. CONSOLIDATED BROADCASTING SYSTEM, INC., vs DANNY OBERIO et. al


G.R. No. 168424; June 8, 2007
Ynares-Santiago, J.:

DOCTRINE: The test to determine whether employment is regular or not is the reasonable connection between
the particular activity performed by the employee in relation to the usual business or trade of the employer. Also,
if the employee has been performing the job for at least one year, even if the performance is not continuous or
merely intermittent, the law deems the repeated and continuing need for its performance as sufficient evidence of
the necessity, if not indispensability of that activity to the business.

FACTS: Respondents alleged that they were employed as drama talents by DYWB-Bombo Radyo, a radio station
owned and operated by petitioner Consolidated Broadcasting System, Inc. They reported for work daily for six
days in a week and were required to record their drama production in advance. Some of them were employed by
petitioner since 1974, while the latest one was hired in 1997. Their drama programs were aired not only
in Bacolod City but also in the sister stations of DYWB in the Visayas and Mindanao areas

Sometime in August 1998, petitioner reduced the number of its drama productions from 14 to 11, but was
opposed by respondents. After the negotiations failed, the latter sought the intervention of the Department of
Labor and Employment (DOLE), which upon inspection found out that petitioner is guilty of violation of labor
standard laws, such as underpayment of wages, 13thmonth pay, non-payment of service incentive leave pay, and
non-coverage of respondents under the Social Security System.

Thereafter, allegedly vexed by respondent’s complaint, petitioner suspended respondents Oberio and Delta for
minor lapses and the payment of their salaries were purportedly delayed, they were also prohibited to enter
company premises thus the filed a case for illegal dismissal, underpayment/non-payment of wages and benefits
plus damages against petitioner.

Labor Arbiter dismissed the case without prejudice while waiting for the decision of the Secretary of Labor on the
same issue of the existence of an employer-employee relationship between petitioner and respondents.

NLRC Decision: Respondents submitted the following to prove the existence of such ER-EE relationship, to wit:
time cards, identification cards, payroll, a show cause order of the station manager to respondent Danny Oberio
and memoranda either noted or issued by said manager. NLRC rendered a decision holding that respondents
were regular employees of petitioner who were illegally dismissed by the latter.

ISSUE/s:
1. Whether respondents were employees of petitioner; and
2. Whether their dismissal was illegal.

HELD:
1. Yes. Respondents employment with petitioner passed the four-fold test on employer-employee relations,
namely: (1) the selection and engagement of the employee, or the power to hire; (2) the payment of wages; (3)
the power to dismiss; and (4) the power to control the employee.

Petitioner failed to controvert with substantial evidence the allegation of respondents that they were hired by the
former on various dates from 1974 to 1997. Note that under Policy Instruction No. 40, petitioner is obliged to
execute the necessary contract specifying the nature of the work to be performed, rates of pay, and the programs
in which they will work. Moreover, project or contractual employees are required to be apprised of the project they

Angeles| Bajana | Balladares | Brillantes | Briones | Cabansag | Callanta | Chua | David|

De Leon | Gomez | Lopez | Macalino | Nostratis | Padilla | Reynon | Santos | Tan |Velasco
4E / 4F - 2018-2019
Page 601 of 920
LABOR REVIEW DIGEST
Atty. Joyrich Golangco

will undertake under a written contract. This was not complied with by the petitioner, justifying the reasonable
conclusion that no such contracts exist and that respondents were in fact regular employees.

In ABS-CBN v. Marquez, the Court held that the failure of the employer to produce the contract mandated by
Policy Instruction No. 40 is indicative that the so called talents or project workers are in reality, regular
employees.

Ironically, however, petitioner failed to adduce an iota proof that the requirements for program employment were
even complied with by it. It is basic that project or contractual employees are appraised of the project they will
work under a written contract, specifying, inter alia, the nature of work to be performed and the rates of pay and
the program in which they will work. Sadly, however, no such written contract was ever presented by the
petitioner. Petitioner is in the best of position to present these documents. And because none was presented, we
have every reason to surmise that no such written contract was ever accomplished by the parties, thereby belying
petitioners posture.

Moreover, the engagement of respondents for a period ranging from 2 to 25 years and the fact that their drama
programs were aired not only in Bacolod City but also in the sister stations of DYWB in the Visayas and Mindanao
areas, undoubtedly show that their work is necessary and indispensable to the usual business or trade of
petitioner. The test to determine whether employment is regular or not is the reasonable connection between the
particular activity performed by the employee in relation to the usual business or trade of the employer. Also, if
the employee has been performing the job for at least one year, even if the performance is not continuous or
merely intermittent, the law deems the repeated and continuing need for its performance as sufficient evidence of
the necessity, if not indispensability of that activity to the business. Thus, even assuming that respondents were
initially hired as project/contractual employees who were paid per drama or per project/contract, the engagement
of their services for 2 to 25 years justify their classification as regular employees, their services being deemed
indispensable to the business of petitioner

2. Yes, respondents were illegally dismissed. In labor cases, the employer has the burden of proving that the
dismissal was for a just cause; failure to show this would necessarily mean that the dismissal was unjustified and,
therefore, illegal. To allow an employer to dismiss an employee based on mere allegations and generalities would
place the employee at the mercy of his employer; and the right to security of tenure, which this Court is bound to
protect, would be unduly emasculated. In this case, petitioner merely contended that it was respondents who
ceased to report to work, and never presented any substantial evidence to support said allegation. Petitioner
therefore failed to discharge its burden, hence, respondents were correctly declared to have been illegally
dismissed.
———————————————————————————————————————-

Angeles| Bajana | Balladares | Brillantes | Briones | Cabansag | Callanta | Chua | David|

De Leon | Gomez | Lopez | Macalino | Nostratis | Padilla | Reynon | Santos | Tan |Velasco
4E / 4F - 2018-2019
Page 602 of 920
LABOR REVIEW DIGEST
Atty. Joyrich Golangco

4. OROZCO vs. CA
GR No: G.R. No. 155207 Date: August 13, 2008
Ponente: Nachura, J.

Doctrine: Not every form of control that the hiring party reserves to himself over the conduct of the party hired in
relation to the services rendered may be accorded the effect of establishing an employer-employee relationship
between them in the legal or technical sense of the term.

Logically, the line should be drawn between rules that merely serve as guidelines towards the achievement of the
mutually desired result without dictating the means or methods to be employed in attaining it, and those that
control or fix the methodology and bind or restrict the party hired to the use of such means. The first, which aim
only to promote the result, create no employer-employee relationship unlike the second, which address both the
result and the means used to achieve it.

FACTS: In March 1990, Philippine Daily Inquirer (PDI) engaged the services of petitioner Wilhelmina S. Orozco to
write a weekly column for its Lifestyle section. She religiously submitted her articles every week, except for a six-
month stint in New York City when she, nonetheless, sent several articles through mail.

On November 7, 1992, petitioner’s column appeared in the PDI for the last time. Petitioner claims that her then
editor, Ms. Lita T. Logarta, told her that respondent Leticia Jimenez Magsanoc, PDI Editor in Chief, wanted to stop
publishing her column for no reason at all.

On the other hand, PDI claims it agreed to cut down the number of columnists by keeping only those whose
columns were well-written, with regular feedback and following. In their judgment, petitioner’s column failed to
improve, continued to be superficially and poorly written, and failed to meet the high standards of the newspaper.
Hence, they decided to terminate petitioner’s column.

Aggrieved by the newspaper’s action, petitioner filed a complaint for illegal dismissal.

LA/NLRC RULING: Respondent company exercised full and complete control over the means and method by
which complainant’s work – that of a regular columnist – had to be accomplished. This control might not be found
in an instruction, verbal or oral, given to complainant defining the means and method she should write her column.
Rather, this control is manifested and certained (sic) in respondents’ admitted prerogative to reject any article
submitted by complainant for publication.

Moreover, this control is already manifested in column title, "Feminist Reflection" allotted complainant. Under this
title, complainant’s writing was controlled and limited to a woman’s perspective on matters of feminine interests.
That respondent had no control over the subject matter written by complainant is strongly belied by this
observation. Even the length of complainant’s articles were set by respondents.

If complainant did not have to report for work eight (8) hours a day, six (6) days a week, it is because her task was
mainly mental. Lastly, the fact that her articles were (sic) published weekly for three (3) years show that she was
respondents’ regular employee, not a once-in-a-blue-moon contributor.

CA RULING: The Court does not agree with NLRC’s conclusion. First, private respondent admitted that she was
and [had] never been considered by petitioner PDI as its employee. Second, it is not disputed that private
respondent had no employment contract with petitioner PDI. In fact, her engagement to contribute articles for
publication was based on a verbal agreement between her and the petitioner’s Lifestyle Section Editor. Moreover,
it was evident that/ private respondent was not required to report to the office eight (8) hours a day. Further, it is
Angeles| Bajana | Balladares | Brillantes | Briones | Cabansag | Callanta | Chua | David|

De Leon | Gomez | Lopez | Macalino | Nostratis | Padilla | Reynon | Santos | Tan |Velasco
4E / 4F - 2018-2019
Page 603 of 920
LABOR REVIEW DIGEST
Atty. Joyrich Golangco

not disputed that she stayed in New York for six (6) months without petitioner’s permission as to her leave of
absence nor was she given any disciplinary action for the same. These undisputed facts negate private
respondent’s claim that she is an employee of petitioner.

Moreover, with regards (sic) to the control test, the public respondent NLRC’s ruling that the guidelines given by
petitioner PDI for private respondent to follow, e.g. in terms of space allocation and length of article, is not the
form of control envisioned by the guidelines set by the Supreme Court. The length of the article is obviously
limited so that all the articles to be featured in the paper can be accommodated. Other than said given limitations,
if the same could be considered limitations, the topics of the articles submitted by private respondent were all her
choices. Thus, the petitioner PDI in deciding to publish private respondent’s articles only controls the result of the
work and not the means by which said articles were written.

APPEAL TO THE SC:

Petitioner's Contention: Petitioner argues that several factors exist to prove that respondents exercised control
over her and her work, namely:
a. As to the Contents of her Column – The PETITIONER had to insure that the contents of her column
hewed closely to the objectives of its Lifestyle Section and the over-all principles that the newspaper
projects itself to stand for. As admitted, she wanted to write about death in relation to All Souls Day but
was advised not to.
b. As to Time Control – The PETITIONER, as a columnist, had to observe the deadlines of the
newspaper for her articles to be published.
c. As to Control of Space – The PETITIONER was told to submit only two or three pages of article for the
column, (sic) "Feminist Reflections" per week.

Respondent's Contention: (see facts)

ISSUE/S: whether or not Petitioner is an employee of PDI and was thus illegally dismissed

HELD: NO. This Court has constantly adhered to the "four-fold test" to determine whether there exists an
employer-employee relationship between parties. The four elements of an employment relationship are: (a) the
selection and engagement of the employee; (b) the payment of wages; (c) the power of dismissal; and (d) the
employer’s power to control the employee’s conduct.

Of these four elements, it is the power of control which is the most crucial and most determinative factor, so
important, in fact, that the other elements may even be disregarded. In other words, the test is whether the
employer controls or has reserved the right to control the employee, not only as to the work done, but also as to
the means and methods by which the same is accomplished.
Petitioner has misconstrued the "control test," as did the Labor Arbiter and the NLRC.

Not every form of control that the hiring party reserves to himself over the conduct of the party hired in relation to
the services rendered may be accorded the effect of establishing an employer-employee relationship between
them in the legal or technical sense of the term.

Logically, the line should be drawn between rules that merely serve as guidelines towards the achievement of the
mutually desired result without dictating the means or methods to be employed in attaining it, and those that
control or fix the methodology and bind or restrict the party hired to the use of such means. The first, which aim

Angeles| Bajana | Balladares | Brillantes | Briones | Cabansag | Callanta | Chua | David|

De Leon | Gomez | Lopez | Macalino | Nostratis | Padilla | Reynon | Santos | Tan |Velasco
4E / 4F - 2018-2019
Page 604 of 920
LABOR REVIEW DIGEST
Atty. Joyrich Golangco

only to promote the result, create no employer-employee relationship unlike the second, which address both the
result and the means used to achieve it.

The main determinant therefore is whether the rules set by the employer are meant to control not just the results
of the work but also the means and method to be used by the hired party in order to achieve such results.

A careful examination reveals that the factors enumerated by the petitioner are inherent conditions in running a
newspaper. In other words, the so-called control as to time, space, and discipline are dictated by the very nature
of the newspaper business itself.

The Inquirer is the publisher of a newspaper of general circulation which is widely read throughout the country. As
such, public interest dictates that every article appearing in the newspaper should subscribe to the standards set
by the Inquirer, with its thousands of readers in mind. It is not, therefore, unusual for the Inquirer to control what
would be published in the newspaper. What is important is the fact that such control pertains only to the end
result, i.e., the submitted articles.

Petitioner has not shown that PDI, acting through its editors, dictated how she was to write or produce her articles
each week. Aside from the constraints presented by the space allocation of her column, there were no restraints
on her creativity; petitioner was free to write her column in the manner and style she was accustomed to and to
use whatever research method she deemed suitable for her purpose.

The perceived constraint on petitioner’s column was dictated by her own choice of her column’s perspective. The
column title "Feminist Reflections" was of her own choosing, as she herself admitted, since she had been known
as a feminist writer. Thus, respondent PDI, as well as her readers, could reasonably expect her columns to speak
from such perspective.

Respondent PDI did not dictate how she wrote or what she wrote in her column. Neither did PDI’s guidelines
dictate the kind of research, time, and effort she put into each column. In fact, petitioner herself said that she
received "no comments on her articles…except for her to shorten them to fit into the box allotted to her column."
Therefore, the control that PDI exercised over petitioner was only as to the finished product of her efforts, i.e., the
column itself, by way of either shortening or outright rejection of the column.

Aside from the control test, this Court has also used the economic reality test. This is especially appropriate when,
as in this case, there is no written agreement or contract on which to base the relationship. Petitioner’s main
occupation is not as a columnist for respondent but as a women’s rights advocate working in various women’s
organizations. Likewise, she herself admits that she also contributes articles to other publications. Thus, it cannot
be said that petitioner was dependent on respondent PDI for her continued employment in respondent’s line of
business.
———————————————————————————————————————-

Angeles| Bajana | Balladares | Brillantes | Briones | Cabansag | Callanta | Chua | David|

De Leon | Gomez | Lopez | Macalino | Nostratis | Padilla | Reynon | Santos | Tan |Velasco
4E / 4F - 2018-2019
Page 605 of 920
LABOR REVIEW DIGEST
Atty. Joyrich Golangco

5. WILLIAM UY CONSTRUCTION CORP. v. TRINIDAD


G.R. No. 183250 March 10, 2010 ABAD, J.:

REPEATED HIRING IN THE CONSTRUCTION BUSINESS OF A DRIVER DOES NOT RESULT INTO
REGULAR EMPLOYMENT.

DOCTRINE: Generally, length of service provides a fair yardstick for determining when an employee initially hired
on a temporary basis becomes a permanent one, entitled to the security and benefits of regularization. But this
standard will not be fair, if applied to the construction industry, simply because construction firms cannot
guarantee work and funding for its payrolls beyond the life of each project. And getting projects is not a matter of
course. Construction companies have no control over the decisions and resources of project proponents or
owners. There is no construction company that does not wish it has such control but the reality, understood by
construction workers, is that work depended on decisions and developments over which construction companies
have no say.

FACTS:
Trinidad claimed that he had been working with the latter company for 16 years since 1988 as driver of its service
vehicle, dump truck, and transit mixer. He had signed several employment contracts with the company that
identified him as a project employee although he had always been assigned to work on one project after another
with some intervals.

In 2004 petitioner company terminated him from work after it shut down operations because of lack of projects. He
learned later, however, that although it opened up a project in Batangas, it did not hire him back for that project.

Trinidad filed a case for illegal dismissal.

ISSUE: Whether repeated hiring in a construction business, for 16 years, will result into a status of regular
employment?

HELD: NO. Generally, length of service provides a fair yardstick for determining when an employee initially hired
on a temporary basis becomes a permanent one, entitled to the security and benefits of regularization. But this
standard will not be fair, if applied to the construction industry, simply because construction firms cannot
guarantee work and funding for its payrolls beyond the life of each project. And getting projects is not a matter of
course. Construction companies have no control over the decisions and resources of project proponents or
owners. There is no construction company that does not wish it has such control but the reality, understood by
construction workers, is that work depended on decisions and developments over which construction companies
have no say.

For this reason, the Court held in Caseres v. Universal Robina Sugar Milling Corporation that the repeated and
successive rehiring of project employees do not qualify them as regular employees, as length of service is not the
controlling determinant of the employment tenure of a project employee, but whether the employment has been
fixed for a specific project or undertaking, its completion has been determined at the time of the engagement of
the employee.

In this case, respondent Trinidads series of employments with petitioner company were co-terminous with its
projects. When its Boni Serrano-Katipunan Interchange Project was finished in December 2004, Trinidads
employment ended with it. He was not dismissed. His employment contract simply ended with the project for
which he had signed up. His employment history belies the claim that he continuously worked for the company.
Intervals or gaps separated one contract from another.

Angeles| Bajana | Balladares | Brillantes | Briones | Cabansag | Callanta | Chua | David|

De Leon | Gomez | Lopez | Macalino | Nostratis | Padilla | Reynon | Santos | Tan |Velasco
4E / 4F - 2018-2019
Page 606 of 920
LABOR REVIEW DIGEST
Atty. Joyrich Golangco

Parenthetically, the Social Security System should be able to alleviate the temporary unemployment of
construction workers, a problem that is inherent in the nature of their work.—————————————————
——————————————————————-

Angeles| Bajana | Balladares | Brillantes | Briones | Cabansag | Callanta | Chua | David|

De Leon | Gomez | Lopez | Macalino | Nostratis | Padilla | Reynon | Santos | Tan |Velasco
4E / 4F - 2018-2019
Page 607 of 920
LABOR REVIEW DIGEST
Atty. Joyrich Golangco

122. 6. DM CONSUNJI INC. vs JAMIN


123. GR No: 192514 Date: April 18, 2012
124. Ponente: Brion, J.
125.
126. Doctrine:
Once a project or work pool employee has been: (1) continuously, as opposed to intermittently, rehired by the
same employer for the same tasks or nature of tasks; and (2) these tasks are vital, necessary and indispensable
to the usual business or trade of the employer, then the employee must be deemed a regular employee.
127.
128. FACTS:
Petitioner D.M. Consunji, Inc. (DMCI), a construction company, hired respondent Estelito L. Jamin as a laborer.
Sometime in 1975, Jamin became a helper carpenter. Since his initial hiring, Jamin’s employment contract had
been renewed a number of times. On March 20, 1999, his work at DMCI was terminated due to the completion of
the SM Manila project. This termination marked the end of his employment with DMCI as he was not rehired
again. Jamin filed a complaint for illegal dismissal, with several money claims (including attorneys fees), against
DMCI and its President/General Manager, David M. Consunji. Jamin alleged that DMCI terminated his
employment without a just and authorized cause at a time when he was already 55 years old and had no
independent source of livelihood. He claimed that he rendered service to DMCI continuously for almost 31 years.
In addition to the schedule of projects (where he was assigned) submitted by DMCI to the labor arbiter, he alleged
that he worked for three other DMCI projects: Twin Towers, Ritz Towers, from July 29, 1980 to June 12, 1982;
New Istana Project, B.S.B. Brunei, from June 23, 1982 to February 16, 1984; and New Istana Project, B.S.B.
Brunei, from January 24, 1986 to May 25, 1986.

DMCI denied liability. It argued that it hired Jamin on a project-to-project basis, from the start of his engagement in
1968 until the completion of its SM Manila project on March 20, 1999 where Jamin last worked. With the
completion of the project, it terminated Jamins employment. It alleged that it submitted a report to the Department
of Labor and Employment (DOLE) everytime it terminated Jamins services.
129.
130. LA and NLRC RULING:
131. Dismissed complaint, Jamin was a project employee
132.
133. CA RULING:
134. Jamin was a regular employee
135.
136. ISSUE/S:
137. Whether or not Jamin was a regular employee
138.
139. HELD: YES
As earlier mentioned, Jamin worked for DMCI for almost 31 years, initially as a laborer and, for the most part, as a
carpenter. Through all those years, DMCI treated him as a project employee, so that he never obtained tenure.
On the surface and at first glance, DMCI appears to be correct. Jamin entered into a contract of employment
(actually an appointment paper to which he signified his conformity) with DMCI either as a field worker, a
temporary worker, a casual employee, or a project employee everytime DMCI needed his services and a
termination of employment paper was served on him upon completion of every project or phase of the project
where he worked. DMCI would then submit termination of employment reports to the DOLE, containing the names
of a number of employees including Jamin. The NLRC and the CA would later on say, however, that DMCI failed
to submit termination reports to the DOLE.

The CA pierced the cover of Jamin’s project employment contract and declared him a regular employee who had
been dismissed without cause and without notice. To reiterate, the CAs findings were based on: (1) Jamins
Angeles| Bajana | Balladares | Brillantes | Briones | Cabansag | Callanta | Chua | David|

De Leon | Gomez | Lopez | Macalino | Nostratis | Padilla | Reynon | Santos | Tan |Velasco
4E / 4F - 2018-2019
Page 608 of 920
LABOR REVIEW DIGEST
Atty. Joyrich Golangco

repeated and successive engagements in DMCIs construction projects, and (2) Jamins performance of activities
necessary or desirable in DMCIs usual trade or business.

We agree with the CA. In Liganza v. RBL Shipyard Corporation, the Court held that [a]ssuming, without granting[,]
that [the] petitioner was initially hired for specific projects or undertakings, the repeated re-hiring and continuing
need for his services for over eight (8) years have undeniably made him a regular employee. We find
the Liganza ruling squarely applicable to this case, considering that for almost 31 years, DMCI had repeatedly,
continuously and successively engaged Jamins services since he was hired on December 17, 1968 or for a total
of 38 times 35 as shown by the schedule of projects submitted by DMCI to the labor arbiter and three more
projects or engagements added by Jamin, which he claimed DMCI intentionally did not include in its schedule so
as to make it appear that there were wide gaps in his engagements. One of the three projects was local, the Ritz
Towers, from July 29, 1980 to June 12, 1982, while the other two were overseas the New Istana Project in Brunei,
Darussalam, from June 23, 1982 to February 16, 1984; and again, the New Istana Project, from January 24, 1986
to May 25, 1986.

We reviewed Jamin’s employment contracts as the CA did and we noted that while the contracts indeed show that
Jamin had been engaged as a project employee, there was an almost unbroken string of Jamins rehiring
from December 17, 1968 up to the termination of his employment on March 20, 1999. While the history of Jamin’s
employment (schedule of projects) relied upon by DMCI shows a gap of almost four years in his employment for
the period between July 28, 1980 (the supposed completion date of the Midtown Plaza project) and June 13, 1984
(the start of the IRRI Dorm IV project), the gap was caused by the companys omission of the three projects above
mentioned.

For not disclosing that there had been other projects where DMCI engaged his services, Jamin accuses the
company of suppressing vital evidence that supports his contention that he rendered service in the company’s
construction projects continuously and repeatedly for more than three decades. The non-disclosure might not
have constituted suppression of evidence it could just have been overlooked by the company but the oversight is
unfair to Jamin as the non-inclusion of the three projects gives the impression that there were substantial gaps not
only of several months but years in his employment with DMCI.

Thus, as Jamin explains, the Ritz Tower Project (July 29, 1980 to June 12, 1982) and the New Istana Project
(June 23, 1982 to February 16, 1984) would explain the gap between the Midtown Plaza project (September 3,
1979 to July 28, 1980) and the IRRI Dorm IV project (June 13, 1984 to March 12, 1985) and the other New Istana
Project (January 24, 1986 to May 25, 1986) would explain the gap between P. 516 Hanger (September 13, 1985
to January 23, 1986) and P. 516 Maint (May 26, 1986 to November 18, 1987).

To reiterate, Jamins employment history with DMCI stands out for his continuous, repeated and successive
rehiring in the companys construction projects. In all the 38 projects where DMCI engaged Jamins services, the
tasks he performed as a carpenter were indisputably necessary and desirable in DMCIs construction business. He
might not have been a member of a work pool as DMCI insisted that it does not maintain a work pool, but his
continuous rehiring and the nature of his work unmistakably made him a regular employee. In Maraguinot, Jr. v.
NLRC,[43] the Court held that once a project or work pool employee has been: (1) continuously, as opposed to
intermittently, rehired by the same employer for the same tasks or nature of tasks; and (2) these tasks are vital,
necessary and indispensable to the usual business or trade of the employer, then the employee must be deemed
a regular employee.

Further, as we stressed in Liganza, [r]espondent capitalizes on our ruling in D.M. Consunji, Inc. v. NLRC which
reiterates the rule that the length of service of a project employee is not the controlling test of employment tenure
but whether or not the employment has been fixed for a specific project or undertaking the completion or
termination of which has been determined at the time of the engagement of the employee.
Angeles| Bajana | Balladares | Brillantes | Briones | Cabansag | Callanta | Chua | David|

De Leon | Gomez | Lopez | Macalino | Nostratis | Padilla | Reynon | Santos | Tan |Velasco
4E / 4F - 2018-2019
Page 609 of 920
LABOR REVIEW DIGEST
Atty. Joyrich Golangco

Surely, length of time is not the controlling test for project employment. Nevertheless, it is vital in determining if the
employee was hired for a specific undertaking or tasked to perform functions vital, necessary and indispensable to
the usual business or trade of the employer. Here, [private] respondent had been a project employee several
times over. His employment ceased to be coterminous with specific projects when he was repeatedly re-hired due
to the demands of petitioners business. Without doubt, Jamins case fits squarely into the employment situation
just quoted.

Other Notes/ SC Pronouncements:


———————————————————————————————————————-

Angeles| Bajana | Balladares | Brillantes | Briones | Cabansag | Callanta | Chua | David|

De Leon | Gomez | Lopez | Macalino | Nostratis | Padilla | Reynon | Santos | Tan |Velasco
4E / 4F - 2018-2019
Page 610 of 920
LABOR REVIEW DIGEST
Atty. Joyrich Golangco

 7. Aro v. NLRC
 G.R. No. 174792 March 7, 2012
Peralta J.

DOCTRINE

FACTS

Several employees of Benthel Development Corporation filed a Complaint for illegal dismissal with various money
claims and prayer for damages against the latter. Thereafter, the Labor Arbiter found private respondent guilty of
illegal dismissal and ordering it to pay its thirty-six (36) employees P446,940.00 as separation pay.

The employees appealed from the said decision. The NLRC affirmed the decision of the Labor Arbiter, with the
modification that private respondent pay backwages computed from the respective dates of dismissal until finality
of the decision.

Aggrieved, private respondent filed a Petition for Certiorari with the CA assailing the January 12, 1999 decision of
the NLRC and the denial of its motion for reconsideration which was dismissed for non-payment of docket fees
and insufficiency of form. It filed a motion for reconsideration, but the latter was also denied.

Thus, private respondent filed with the SC a Petition for Review on Certiorari. In a Resolution dated September
20, 2000, this Court denied the petition for having been filed out of time and for non-payment of docket and other
lawful fees.

The employees filed a Motion for Execution before the Labor Arbiter of the January 12, 1999 decision. Thereafter,
the Labor Arbiter ordered for the issuance of a writ of execution directing the computation of the awards.

Afterwards, private respondent filed an appeal from the said Order with an urgent prayer for the issuance of a
temporary restraining order and/or preliminary injunction with public respondent NLRC. The NLRC held that the
appeal was premature.

In the meantime, fifteen (15) employees have executed Affidavits of Full Settlement after having settled amicably
with the private respondent. Labor Arbiter Violeta Ortiz-Bantug issued an Order dated July 31, 2003 for the
issuance of a writ of execution only for the payment of the claims of the twenty-one (21) remaining employees in
the total amount of P4,383,225.00, which included attorney's fees equivalent to ten (10%) percent of the sum
received as settlement by the fifteen (15) employees who had earlier settled with the private respondent.

Private respondent appealed to NLRC contending that the computation for backwages must be only until the
completion of the project and not until the finality of the decision. Public respondent affirmed the Order of Labor
Arbiter Bantug, but reduced the total amount to P4,073,858.00, inclusive of attorney's fees. Thereafter, the NLRC
admitted the affidavits of withdrawal, release/waiver and quitclaim executed by another group of fourteen (14)
employees, leaving unresolved only the claims of the petitioners herein.

As a recourse, private respondent filed a petition for certiorari with the CA, alleging that public respondent
committed grave abuse of discretion in promulgating its assailed decision and denying its motion for
reconsideration. The CA granted the petition, therefore, annulling and setting aside the decision and resolution of
the NLRC as to the award for backwages and remanded the case to the same public respondent for the proper
computation of the backwages due to each of the petitioners herein.

Angeles| Bajana | Balladares | Brillantes | Briones | Cabansag | Callanta | Chua | David|

De Leon | Gomez | Lopez | Macalino | Nostratis | Padilla | Reynon | Santos | Tan |Velasco
4E / 4F - 2018-2019
Page 611 of 920
LABOR REVIEW DIGEST
Atty. Joyrich Golangco

ISSUE

What is the reckoning date for the computation of the backwages of the petitioners.

HELD

 Article 280 of the Labor Code distinguishes a "project employee" from a "regular employee," thus:

Article 280. Regular and Casual Employment − The provisions of written agreement to the contrary
notwithstanding and regardless of the oral agreement of the parties, an employment shall be deemed to
be regular where the employee has been engaged to perform activities which are usually necessary or
desirable in the usual business or trade of the employer, except where the employment has been fixed
for a specific project or undertaking the completion or termination of which has been determined at the
time of the engagement of the employee or where the work or service to be performed is seasonal in
nature and the employment is for the duration of the season.

An employment shall be deemed to be casual if it is not covered by the preceding paragraph: Provided,
That, any employee who has rendered at least one year service, whether such service is continuous or
broken, shall be considered a regular employee with respect to the activity in which he is employed and
his employment shall continue while such activity exists.

 In Hanjin Heavy Industries and Construction Co. Ltd. v. Ibaez, this Court extensively discussed the above
distinction, thus:

x x x [T]he principal test for determining whether particular employees are properly characterized as
"project employees" as distinguished from "regular employees" is whether or not the project employees
were assigned to carry out a "specific project or undertaking," the duration and scope of which were
specified at the time the employees were engaged for that project.

In a number of cases, the Court has held that the length of service or the re-hiring of construction
workers on a project-to-project basis does not confer upon them regular employment status, since their
re-hiring is only a natural consequence of the fact that experienced construction workers are preferred.
Employees who are hired for carrying out a separate job, distinct from the other undertakings of the
company, the scope and duration of which has been determined and made known to the employees at
the time of the employment, are properly treated as project employees and their services may be lawfully
terminated upon the completion of a project. Should the terms of their employment fail to comply with this
standard, they cannot be considered project employees.

 Therefore, being project employees, petitioners are only entitled to full backwages, computed from the
date of the termination of their employment until the actual completion of the work. Illegally dismissed
workers are entitled to the payment of their salaries corresponding to the unexpired portion of their
employment where the employment is for a definite period. In this case, as found by the CA, the Cordova
Reef Village Resort project had been completed in October 1996 and private respondent herein had
signified its willingness, by way of concession to petitioners, to set the date of completion of the project as
March 18, 1997; hence, the latter date should be considered as the date of completion of the project for
purposes of computing the full backwages of petitioners.

———————————————————————————————————————-
Angeles| Bajana | Balladares | Brillantes | Briones | Cabansag | Callanta | Chua | David|

De Leon | Gomez | Lopez | Macalino | Nostratis | Padilla | Reynon | Santos | Tan |Velasco
4E / 4F - 2018-2019
Page 612 of 920
LABOR REVIEW DIGEST
Atty. Joyrich Golangco

Angeles| Bajana | Balladares | Brillantes | Briones | Cabansag | Callanta | Chua | David|

De Leon | Gomez | Lopez | Macalino | Nostratis | Padilla | Reynon | Santos | Tan |Velasco
4E / 4F - 2018-2019
Page 613 of 920
LABOR REVIEW DIGEST
Atty. Joyrich Golangco

8. Universal Robina Corp. vs. Acibo


GR No. 186439 January 15, 2014
BRION, J.

DOCTRINE: By way of an exception, paragraph 2, Article 280 of the Labor Code also considers regular a casual
employment arrangement when the casual employee’s engagement has lasted for at least one year, regardless of
the engagement’s continuity. The controlling test in this arrangement is the length of time during which the
employee is engaged.

FACTS:

The complainants were employees of URSUMCO. They were hired on various dates and on different
capacities. At the start of their respective engagements, the complainants signed contracts of employment for a
period of one (1) month or for a given season. URSUMCO repeatedly hired the complainants to perform the same
duties and, for every engagement, required the latter to sign new employment contracts for the same duration of
one month or a given season.

On August 23, 2002, the complainants filed before the LA complaints for regularization, entitlement to the benefits
under the existing Collective Bargaining Agreement (CBA),and attorney’s fees. LA dismissed the petition and held
that they were seasonal projects and not regular employees. The NLRC reversed the LA’s ruling; it declared the
complainants as regular URSUMCO employees and granted their monetary claims under the CBA. The NLRC
pointed out that the complainants performed activities which were usually necessary and desirable in the usual
trade or business of URSUMCO, and had been repeatedly hired for the same undertaking every season. CA
affirmed.

ISSUE:

Whether the respondents are regular employees of URSUMCO

HELD:

The petitioners maintain that the respondents are contractual or project/seasonal workers and not regular
employees of URSUMCO. They thus argue that the CA erred in applying the legal parameters and guidelines for
regular employment to the respondents’ case. They contend that the legal standards – length of the employee’s
engagement and the desirability or necessity of the employee’s work in the usual trade or business of the
employer – apply only to regular employees under paragraph 1, Article 280 of the Labor Code, and, under
paragraph 2 of the same article, to casual employees who are deemed regular by their length of service.

The respondents, the petitioners point out, were specifically engaged for a fixed and predetermined duration of, on
the average, one (1) month at a time that coincides with a particular phase of the company’s business operations
or sugar milling season. By the nature of their engagement, the respondents’ employment legally ends upon the
end of the predetermined period; thus, URSUMCO was under no legal obligation to rehire the respondents.

In their comment, the respondents maintain that they are regular employees of URSUMCO. Relying on the NLRC
and the CA rulings, they point out that they have been continuously working for URSUMCO for more than one
year, performing tasks which were necessary and desirable to URSUMCO’s business. Hence, under the above-
stated legal parameters, they are regular employees.

Angeles| Bajana | Balladares | Brillantes | Briones | Cabansag | Callanta | Chua | David|

De Leon | Gomez | Lopez | Macalino | Nostratis | Padilla | Reynon | Santos | Tan |Velasco
4E / 4F - 2018-2019
Page 614 of 920
LABOR REVIEW DIGEST
Atty. Joyrich Golangco

We disagree with the petitioners’ position. We find the respondents to be regular seasonal employees of
URSUMCO. By way of an exception, paragraph 2, Article 280 of the Labor Code also considers regular a casual
employment arrangement when the casual employee’s engagement has lasted for at least one year, regardless of
the engagement’s continuity. The controlling test in this arrangement is the length of time during which the
employee is engaged.

Clearly, therefore, the nature of the employment does not depend solely on the will or word of the employer or on
the procedure for hiring and the manner of designating the employee. Rather, the nature of the employment
depends on the nature of the activities to be performed by the employee, considering the nature of the employer’s
business, the duration and scope to be done, and, in some cases, even the length of time of the performance and
its continued existence.

———————————————————————————————————————-

Angeles| Bajana | Balladares | Brillantes | Briones | Cabansag | Callanta | Chua | David|

De Leon | Gomez | Lopez | Macalino | Nostratis | Padilla | Reynon | Santos | Tan |Velasco
4E / 4F - 2018-2019
Page 615 of 920
LABOR REVIEW DIGEST
Atty. Joyrich Golangco

9. GMA NETWORK, INC., vs. CARLOS P. PABRIGA et.al


G.R. No. 176419 November 27, 2013
Leonardo-De Castro, J.

Doctrine: To safeguard the rights of workers against the arbitrary use of the word “project” to prevent
employees from attaining the status of regular employees, employers claiming that their workers are
project employees should not only prove that the duration and scope of the employment was specified
at the time they were engaged, but also that there was indeed a project.

FACTS:
Respondents were engaged by petitioner GMA Network, Inc. to perform the following activities: 1)
manning of technical operations center: (a) responsibility for airing of local commercials, and (b)
logging/monitoring of national commercials (satellite); 2) acting as transmitter/VTR men: (a) prepare tapes
for local airing, (b) actual airing of commercials, (c) plugging of station promo, (d) logging of transmitter
reading, and (e) in case of power failure, start up generator set to resume program; 3) acting as
maintenance staff: (a) checking of equipment, (b) warming up of generator, (c) filling of oil, fuel, and
water in radiator, and 4) acting as cameramen. In a complaint for illegal dismissal, the petitioner
interchangeably characterized respondents’ employment as project and fixed-period/fixed-term employment.

NLRC RULING: The National Labor Relations Commission (NLRC), in reversing the decision of the labor
arbiter, held that they are regular employees.

PETITION TO THE SC:

ISSUE/S:
1. Whether or not the respondents are regular employees

HELD:
Yes. To safeguard the rights of workers against the arbitrary use of the word “project” to prevent
employees from attaining the status of regular employees, employers claiming that their workers are
project employees should not only prove that the duration and scope of the employment was specified
at the time they were engaged, but also that there was indeed a project.
The project could either be (1) a particular job or undertaking that is within the regular or usual
business of the employer company, but which is distinct and separate, and identifiable as such, from the
other undertakings of the company; or (2) a particular job or undertaking that is not within the regular
business of the corporation. As it was with regard to the distinction between a regular and casual
employee, the purpose of this requirement is to delineate whether or not the employer is in constant
need of the services of the specified employee. If the particular job or undertaking is within the regular
or usual business of the employer company and it is not identifiably distinct or separate from the other
undertakings of the company, there is clearly a constant necessity for the performance of the task in
question, and therefore said job or undertaking should not be considered a project.
Herein jobs and undertakings are clearly within the regular or usual business of the employer
company and are not identifiably distinct or separate from the other undertakings of the company. There
is no denying that the manning of the operations center to air commercials, acting as transmitter/VTR
men, maintaining the equipment, and acting as cameramen are not undertakings separate or distinct from
the business of a broadcasting company.
———————————————————————————————————————-

Angeles| Bajana | Balladares | Brillantes | Briones | Cabansag | Callanta | Chua | David|

De Leon | Gomez | Lopez | Macalino | Nostratis | Padilla | Reynon | Santos | Tan |Velasco
4E / 4F - 2018-2019
Page 616 of 920
LABOR REVIEW DIGEST
Atty. Joyrich Golangco

10. Pasos v. PNCC


G.R. No. 192394; July 03, 2013
DOCTRINE: The principal test used to determine whether employees are project employees is whether or not the
employees were assigned to carry out a specific project or undertaking, the duration or scope of which was
specified at the time the employees were engaged for that project.
FACTS: Petitioner Roy D. Pasos started working for respondent PNCC on April 26, 1996. He was designated as
"Clerk II (Accounting)" and was assigned to the "NAIA – II Project." It is stated in his employment contract that: his
contract maybe terminated at anytime for cause as provided for by law and/or existing Company Policy. This
maybe terminated if services are unsatisfactory, or when it shall no longer needed, as determined by the
Company. If services are still needed beyond the validity of this contract, the Company shall extend your services.
After services are terminated, the employee shall be under no obligation to re-employ with the Company nor shall
the Company be obliged to re-employ the employee. Petitioner’s employment, however, did not end on July 25,
1996 but was extended until August 4, 1998, or more than two years later, based on the "Personnel Action Form –
Project Employment" dated July 7, 1998. Despite the termination of his employment on October 19, 2000,
petitioner claims that his superior instructed him to report for work the following day, intimating to him that he will
again be employed for the succeeding SM projects. For purposes of reemployment, he then underwent a medical
examination which allegedly revealed that he had pneumonitis. Petitioner was advised by PNCC’s physician, Dr.
Arthur C. Obena, to take a 14-day sick leave. After serving his sick leave, petitioner claims that he was again
referred for medical examination where it was revealed that he contracted Koch’s disease. He was then required
13
to take a 60-day leave of absence. The following day, he submitted his application for sick leave but PNCC’s
Project Personnel Officer, Mr. R.S. Sanchez, told him that he was not entitled to sick leave because he was not a
regular employee. Petitioner claims that after he presented his medical clearance to the Project Personnel Officer
on even date, he was informed that his services were already terminated on October 19, 2000 and he was already
14
replaced due to expiration of his contract. This prompted petitioner on February 18, 2003 to file a complaint for
illegal dismissal against PNCC with a prayer for reinstatement and back wages. He argued that he is deemed a
regular employee of PNCC due to his prolonged employment as a project employee as well as the failure on the
part of PNCC to report his termination every time a project is completed. He further contended that his termination
without the benefit of an administrative investigation was tantamount to an illegal dismissal.
LA RULING: complainant had attained regular employment thereby making his termination from employment
illegal since it was not for any valid or authorized causes.
Petitioner submits that the CA erroneously concluded that he was a project employee when there are indicators
which point otherwise. He contends that even if he was just hired for the NAIA 2 Project from April 26, 1996 to
July 25, 1996, he was made to work until August 4, 1998. He also avers the DOLE had certified that he was not
among the employees listed in the termination reports submitted by PNCC which belies the photocopies of
termination reports attached by PNCC to its pleadings listing petitioner as one of the affected employees
ISSUE: Whether petitioner is a regular employee
RULING: This Court is convinced however that although he started as a project employee, he eventually became
a regular employee of PNCC. The principal test used to determine whether employees are project employees is
whether or not the employees were assigned to carry out a specific project or undertaking, the duration or scope
of which was specified at the time the employees were engaged for that project. In the case at bar, petitioner
worked continuously for more than two years after the supposed three-month duration of his project employment
for the NAIA II Project. While his appointment for said project allowed such extension since it specifically provided
that in case his "services are still needed beyond the validity of the contract, the Company shall extend his
services," there was no subsequent contract or appointment that specified a particular duration for the extension.
His services were just extended indefinitely until "Personnel Action Form – Project Employment" dated July 7,
1998 was issued to him which provided that his employment will end a few weeks later or on August 4, 1998.
While for first three months, petitioner can be considered a project employee of PNCC, his employment thereafter,

Angeles| Bajana | Balladares | Brillantes | Briones | Cabansag | Callanta | Chua | David|

De Leon | Gomez | Lopez | Macalino | Nostratis | Padilla | Reynon | Santos | Tan |Velasco
4E / 4F - 2018-2019
Page 617 of 920
LABOR REVIEW DIGEST
Atty. Joyrich Golangco

when his services were extended without any specification of as to the duration, made him a regular employee of
PNCC. And his status as a regular employee was not affected by the fact that he was assigned to several other
projects and there were intervals in between said projects since he enjoys security of tenure.

———————————————————————————————————————-

Angeles| Bajana | Balladares | Brillantes | Briones | Cabansag | Callanta | Chua | David|

De Leon | Gomez | Lopez | Macalino | Nostratis | Padilla | Reynon | Santos | Tan |Velasco
4E / 4F - 2018-2019
Page 618 of 920
LABOR REVIEW DIGEST
Atty. Joyrich Golangco

11. JAIME GAPAYAO vs. ROSARIO FULO, SOCIAL SECURITY SYSTEM and SOCIAL SECURITY
COMMISSION
G.R. No. 193493 June 13. 2013
SERENO, CJ.

DOCTRINE: Regular seasonal employees are those called to work from time to time. The nature of their
relationship with the employer is such that during the off season, they are temporarily laid off; but reemployed
during the summer season or when their services are needed. They are in regular employment because of the
nature of their job,and not because of the length of time they have worked.

FACTS: On 4 November 1997, Jaime Fulo (deceased) died of "acute renal failure secondary to 1st degree burn
70% secondary electrocution" while doing repairs at the residence and business establishment of petitioner.
Allegedly moved by his Christian faith, petitioner extended some financial assistance to private respondent.
Rosario Fulo executed an Affivadit of Desistance wherein she stated that she was not holding Petitioner liable for
the death of her husband, Jaime. She further stated that she was waiving her right and she desists from filing any
action, civil or criminal against the Petitioner.
A Compromise Agreement was executed by both parties. The sum of P40,000 was given as a complete and full
payment of all claims due the victim.
Rosario Fulo, thereafter, filed a claim for Social Security Benefits with the SSS-Sorsogon. It was discovered that
the deceased was not a registered member of the SSS. The SSS conducted an investigation which showed that
the deceased is an employee of petitioner and hence held petitioner liable for unpaid contributions due on behalf
of deceased Fulo and penalties and surcharges.
PETITIONER’S CONTENTION: Petitioner claims that Jaime Fulo is not an employee. According to Petitioner,
Fulo was an independent contractor whose tasks were not subject to petitioner’s control and supervision.
Assuming arguendo that the deceased was petitioner’s employee, he was still not entitled to be paid his SSS
premiums for the intervening period when he was not at work, as he was an "intermittent worker who was only
summoned every now and then as the need arose." Hence, petitioner insisted that he was under no obligation to
report the former’s demise to the SSS for social security coverage.
ISSUE: Whether or not Jaime Fulo was an employee of the Petitioner.
HELD: The three types of employees mentioned in the provision: (1) regular employees or those who have been
engaged to perform activities that are usually necessary or desirable in the usual business or trade of the
employer; (2) project employees or those whose employment has been fixed for a specific project or undertaking,
the completion or termination of which has been determined at the time of their engagement, or those whose work
or service is seasonal in nature and is performed for the duration of the season; and (3) casual employees or
those who are neither regular nor project employees.
Farm workers generally fall under the definition of seasonal employees. Seasonal employees may be considered
as regular employees. Regular seasonal employees are those called to work from time to time. The nature of their
relationship with the employer is such that during the off season, they are temporarily laid off; but reemployed
during the summer season or when their services are needed. They are in regular employment because of the
nature of their job, and not because of the length of time they have worked.
The deceased was indeed a farm worker who was in the regular employ of petitioner. From year to year, starting
January 1983 up until his death, the deceased had been working on petitioner’s land by harvesting abaca and
coconut, processing copra, and clearing weeds. His employment was continuous in the sense that it was done for
more than one harvesting season. Moreover, no amount of reasoning could detract from the fact that these tasks
were necessary or desirable in the usual business of petitioner.
The other tasks allegedly done by the deceased outside his usual farm work only bolster the existence of an
employer-employee relationship. As found by the SSC, the deceased was a construction worker in the building,
and a helper in the bakery, grocery, hardware, and piggery – all owned by petitioner. This fact only proves that
even during the off season, the deceased was still in the employ of petitioner.

Angeles| Bajana | Balladares | Brillantes | Briones | Cabansag | Callanta | Chua | David|

De Leon | Gomez | Lopez | Macalino | Nostratis | Padilla | Reynon | Santos | Tan |Velasco
4E / 4F - 2018-2019
Page 619 of 920
LABOR REVIEW DIGEST
Atty. Joyrich Golangco

12. Millennium Erectors v. Magallanes


G.R. No. 184362 November 15, 2010

DOCTRINE:
Facts:
Respondent Virgilio Magallanes started working in 1988 as a utility man for Laurencito Tiu (Tiu), Chief
Executive Officer of Millennium Erectors Corporation (petitioner), Tius family, and Kenneth Construction
Corporation. He was assigned to different construction projects undertaken by petitioner in Metro Manila, the last
of which was for a building in Libis, Quezon City. In July of 2004 he was told not to report for work anymore
allegedly due to old age, prompting him to file on August 6, 2004 an illegal dismissal complaint before the Labor
Arbiter.
Labor Arbiter ruled in favor of petitioner and dismissed the complaint, holding that respondent knew of
the nature of his employment as a project employee, he having executed an employment contract specifying
therein the name of and duration of the project from January 2003 until its completion.
On appeal, the National Labor Relations Commission (NLRC) set aside the Labor Arbiters Decision of
February 6, 2007 holding that respondent was a regular, not a project employee. MR also denied.
Court of Appeals, to which petitioner appealed, affirmed the NLRCs ruling by Decision.

Issue: Whether private respondent employee is a regular or project employee.

HELD:
NLRC did not err in treating respondents motion for reconsideration as an appeal, the presence of some
procedural flaws including the lack of verification and proof of service notwithstanding
On the merits of the case, the Court finds that, indeed, respondent was a regular, not a project
employee.
A project employee is one whose "employment has been fixed for a specific project or undertaking, the
completion or termination of which has been determined at the time of the engagement of the employee or where
the work or service to be performed is seasonal in nature and the employment is for the duration of the season."
Equipment Technical Services v. Court of Appeals emphasizes the difference between a regular
employee and a project employee:
As the Court has consistently held, the service of project employees are coterminus with the project and
may be terminated upon the end or completion of that project or project phase for which they were hired. Regular
employees, in contrast, enjoy security of tenure and are entitled to hold on to their work or position until their
services are terminated by any of the modes recognized under the Labor Code.
Petitioners various payrolls dating as early as 2001 show that respondent had been employed by it. As
aptly observed by the appellate court, these documents, rather than sustaining petitioners argument, only serve to
support respondents contention that he had been employed in various projects, if not for 16 years, at the very
least two years prior to his dismissal.
Assuming arguendo that petitioner hired respondent initially on a per project basis, his continued rehiring,
as shown by the sample payrolls converted his status to that of a regular employee. Following Cocomangas
Beach Hotel Resort v. Visca, the repeated and continuing need for respondents services is sufficient evidence of
the necessity, if not indispensability, of his services to petitioner's business and, as a regular employee, he could
only be dismissed from employment for a just or authorized cause.
Petitioner having failed to discharge its burden of proving that it terminated the services of respondent for
cause and with due process, the challenged decision must remain.

———————————————————————————————————————-

Angeles| Bajana | Balladares | Brillantes | Briones | Cabansag | Callanta | Chua | David|

De Leon | Gomez | Lopez | Macalino | Nostratis | Padilla | Reynon | Santos | Tan |Velasco
4E / 4F - 2018-2019
Page 620 of 920
LABOR REVIEW DIGEST
Atty. Joyrich Golangco

13. Caparoso v. NLRC


G.R. No. 155505; Feb. 15, 2007
Carpio, J.:

73. DOCTRINE: Fixed-term employment could not be said to be in circumvention of the law on security of
tenure, thus:
1. The fixed period of employment was knowingly and voluntarily agreed upon by the parties without any
force, duress, or improper pressure being brought to bear upon the employee and absent any other
circumstances vitiating his consent; or
2. It satisfactorily appears that the employer and the employee dealt with each other on more or less equal
terms with no moral dominance exercised by the former or the latter.

FACTS:
Composite Enterprises is engaged in the distribution and supply of confectioneries to various retail establishments
within the Philippines. Emilio M. Caparoso (Caparoso) and Joeve P. Quindipan (Quindipan) were Composite's
deliverymen. Caparoso alleged that he was hired in November 1998 while Quindipan alleged that he was hired on
intermittent basis since 1997. Quindipan further alleged that he had been working continuously with Composite
since August 1998.

In October 1999, Caparoso and Quindipan were dismissed from the service prompting them to file an illegal
dismissal case before the Labor Arbiter. Personel Mannager Edit Tan and Composite argued that both of herein
Petitioners were hired initially for three months, then on a month-to-month basis, with their final contract expiring
in October 1999.

LA RULING: Ordered the reinstatement of Petitioners, holding that they were regular employees by reason of the
nature of Composite’s business.

NLRC RULING: Reversed the LA and held the parties are not precluded from stipulating on the period of
employment.

CA RULING: Affirmed the NLRC, noting that the respondent’s manpower needs varied from month to month, and
that petitioners were merely hired to address manpower shortage.

APPEAL TO THE SC:


Petitioner’s Contention: They are regular employees.

ISSUE:. Whether or not employees hired on several fixed term contracts may become regular empoyees.
74. RULING: NO. A regular employee is (1) one who is engaged to perform activities that are necessary or
desirable in the usual trade or business of the employer, or (2) a casual employee who has rendered at least one
year of service, whether continuous or broken, with respect to the activity in which he is employed.
75. In Brent School v. Zamora, the Court upheld the validity of Fixed Term Employment contracts that Art.
280 (now 294) of the Labor Code on Security of Tenure has no application:
76. “where a fixed period of employment was agreed upon knowingly and voluntarily by the parties,
without any force, duress or improper pressure being brought to bear upon the employee and absent any
other circumstances vitiating his consent, or where it satisfactorily appears that the employer and
employee dealt with each other on more or less equal terms with no moral dominance whatever being
exercised by the former over the latter.”
77. Fixed-term employment could not be said to be in circumvention of the law on security of tenure, thus:
Angeles| Bajana | Balladares | Brillantes | Briones | Cabansag | Callanta | Chua | David|

De Leon | Gomez | Lopez | Macalino | Nostratis | Padilla | Reynon | Santos | Tan |Velasco
4E / 4F - 2018-2019
Page 621 of 920
LABOR REVIEW DIGEST
Atty. Joyrich Golangco

1. The fixed period of employment was knowingly and voluntarily agreed upon by the parties without any
force, duress, or improper pressure being brought to bear upon the employee and absent any other
circumstances vitiating his consent; or
2. It satisfactorily appears that the employer and the employee dealt with each other on more or less equal
terms with no moral dominance exercised by the former or the latter.
78. Given the same, the Petitioners could not have been illegally dismissed given that their termination was
based on a valid ground: the expiration of their contracts.
79. Other Rulings:
80. Petitioners' employment for less than six months can be considered probationary. Petitioners were hired
on 11 May 1999, initially for three months. After the expiration of their contracts, petitioners were hired on a
month-to-month basis. Their contracts of employment ended on 8 October 1999. Hence, they were employed for
a total of five months. Their employment did not even exceed six months to entitle them to become regular
employees.
———————————————————————————————————————-

Angeles| Bajana | Balladares | Brillantes | Briones | Cabansag | Callanta | Chua | David|

De Leon | Gomez | Lopez | Macalino | Nostratis | Padilla | Reynon | Santos | Tan |Velasco
4E / 4F - 2018-2019
Page 622 of 920
LABOR REVIEW DIGEST
Atty. Joyrich Golangco

14. SPOUSES ALWYN ONG LIM and EVELYN LUKANG LIM , petitioners, vs. LEGAZPI HOPE CHRISTIAN
SCHOOL/RAMON SIA/OMEGA SIA/HELEN SIA/CECILIO K. PEDRO, respondents.
GR No: 172818 Date: March 31, 2009
Ponente: Quisumbing J.

Doctrine: For a private school teacher to acquire permanent status in employment, the following requisites must
concur: (1) the teacher is a full-time teacher; (2) the teacher must have rendered three consecutive years of
service; and (3) such service must have been satisfactory.

FACTS:

Petitioner-spouses Alwyn Ong Lim and Evelyn Lukang Lim were hired in June 1999. Alwyn was assigned to teach
Mathematics, Geometry, Algebra and Trigonometry subjects in the high school department of Legazpi Hope
Christian School. Evelyn, on the other hand, was assigned to teach Chinese Language 1 and 2 and Chinese Math
subjects in the elementary department of the same school. Helen Sia, head teacher of the schools Chinese
department, verbally informed them that their employment were to be terminated, without giving reasons therefor.
Spouses Lim filed for illegal dismissal and monetary claims against the school. Ramon Sia, Vice Chairman of the
School’s Board of Directors, sent a letter to petitioners stating that their three-year probation had expired and that
the school management had decided to discontinue their employment.

LA/RTC/NLRC RULING:
LA: Ruled in favor of petitioners and ordered for their reinstatement.

NLRC: Reversed the decision of the LA and declared that petitioners were mere part-time teachers who did not
acquire permanent status; hence, their dismissal was valid. MR was denied.

CA RULING:
Affirmed the decision of the NLRC. Denied the MR filed by petitioners

APPEAL TO THE SC:

Petitioner's Contention: Spouses Lim contend that they were not issued any formal written probationary contract
and neither were they informed of reasonable standards under which they would be evaluated or rated in
connection with their supposed probationary period of employment. In the absence of such, they became regular
and permanent teaching personnel of the school. Petitioners further claim that they are full-time, not part-time,
teaching personnel. They claim to have no other outside remunerative occupation requiring regular hours of work
that will conflict with the working hours of the respondent school and in addition to their teaching jobs, they were
performing non-teaching functions like preparing lesson plans, checking of notebooks and test papers, assisting
during enrollment period, attending to school programs and other tasks. They were required to report as early as 7
a.m. until their respective classes ended.

Respondent's Contention: Respondents argue that under the Manual of Regulations for Private Schools, a full-
time instructor is one who has a teaching load of at least 15 hours a week or is paid on a full salary basis, while a
part-time instructor is one who has a teaching load of less than 15 hours a week. Thus, according to respondents,
since petitioners have a teaching load that is less than 15 hours a week then they are only part-time instructors
and do not enjoy security of tenure.

ISSUE/S:
1. Whether or not petitioners achieved permanent status
Angeles| Bajana | Balladares | Brillantes | Briones | Cabansag | Callanta | Chua | David|

De Leon | Gomez | Lopez | Macalino | Nostratis | Padilla | Reynon | Santos | Tan |Velasco
4E / 4F - 2018-2019
Page 623 of 920
LABOR REVIEW DIGEST
Atty. Joyrich Golangco

HELD:
1. NO.
In University of Sto. Tomas v. NLRC, 23 we ruled that for a private school teacher to acquire permanent status in
employment, the following requisites must concur: (1) the teacher is a full-time teacher; (2) the teacher must have
rendered three consecutive years of service; and (3) such service must have been satisfactory. The burden is on
petitioners to prove their affirmative allegation that they are permanent teaching personnel. However, there is not
enough evidence on record to show that their total working day is devoted to the school. There is no showing of
what the regular work schedule of a regular teacher in respondent school is. What is clear in the records is that
Evelyn and Alwyn spent two hours and four hours, respectively, but not the entire working day, at the respondent
school. They do not meet requirement "c" of Section 45 of the Manual. Hence, we sustain the Gndings of the
Court of Appeals that the petitioners are part-time teachers. Being part-time teachers, in accordance with
University of Sto. Tomas v. NLRC, they cannot acquire permanent status.
Considering that petitioners were new teachers, then in accordance with the above-quoted guidelines their
unwritten contracts were considered to be for one school year at a time, they being required to submit a letter of
re-application for each school year. After the end of each school year, the school did not have any obligation to
give them any teaching loads, they being part-time teachers. That respondents did not give any teaching
assignment to the petitioners after the school year 2001-2002 did not amount to an actionable violation of
petitioners' right. It did not amount to illegal dismissal.

Angeles| Bajana | Balladares | Brillantes | Briones | Cabansag | Callanta | Chua | David|

De Leon | Gomez | Lopez | Macalino | Nostratis | Padilla | Reynon | Santos | Tan |Velasco
4E / 4F - 2018-2019
Page 624 of 920
LABOR REVIEW DIGEST
Atty. Joyrich Golangco

15. D.M. CONSUNJI, INC. v. ANTONIO GOBRES


GR No: 169170 Date: August 8, 2010
Ponente: Peralta, J.

Doctrine:
If the termination is brought about by the completion of the contract or phase thereof, no prior notice is required.
This is because completion of the work or project automatically terminates the employment, in which case, the
employer is, under the law, only obliged to render a report to the DOLE on the termination of the employment.
Therefore, failure to give respondents advance notice of their termination is not a violation of any requirement of
procedural process; thus, there is no basis for the payment of nominal damages.

FACTS:
Respondents Antonio Gobres, Magellan Dalisay, Godofredo Paragsa, Emilio Aleta and Generoso Melo worked as
carpenters in the construction projects of petitioner D.M. Consunji, Inc., a construction company, on several
occasions and/or at various times. Their termination from employment for each project was reported to the
Department of Labor and Employment (DOLE). The respondents saw their names included in the Notice of
Termination posted on the bulletin board at the premises of their last project.

Respondents filed a Complaint with the Arbitration Branch of the NLRC against petitioner for illegal dismissal.

Petitioner countered that respondents, being project employees, their services were terminated when their phases
of work for which their services were engaged were completed or when the projects themselves were completed.

LA/NLRC RULING:
The LA dismissed the respondents’ complaint. It found that respondents were project employees, and that they
were dismissed from the last project they were assigned to when their respective phases of work were completed,
and that their dismissal was reported to the DOLE.

The NLRC affirmed the LA decision.

CA RULING:
The CA affirmed the decision of the NLRC in finding the dismissal valid but ordered petitioner to pay P20,000 as
nominal damages for non-compliance with the statutory due process, citing the case of Agabon v. NLRC.

The CA sustained the findings that respondents are project employees. However, although such, they were
entitled to know the reason for their dismissal and to be heard on whatever claims they might have.

ISSUE/S:
Whether or not respondents, as project employees, are entitled to nominal damages for lack of advance notice of
their dismissal.

HELD:
NO. Respondents were dismissed after the expiration of their respective project employment contracts, and due to
the completion of the phases of work respondents were engaged for. Hence, the requirements of due process or
prior notice when an employee is dismissed for just or authorized cause (under Articles 282 and 283 of the Labor
Code) prior to the completion of the project or phase thereof for which the employee was engaged do not apply
to this case. There being no requirement of prior notice of termination when the termination is brought about by
the completion of the contract or phase thereof for which the worker was hired, respondents are not entitled to
nominal damages for lack of advance notice of their termination.

Angeles| Bajana | Balladares | Brillantes | Briones | Cabansag | Callanta | Chua | David|

De Leon | Gomez | Lopez | Macalino | Nostratis | Padilla | Reynon | Santos | Tan |Velasco
4E / 4F - 2018-2019
Page 625 of 920
LABOR REVIEW DIGEST
Atty. Joyrich Golangco

The case of Agabon v. NLRC is not applicable to this case. Unlike in Agabon, respondents, in this case, were not
terminated for just cause under Article 282 of the Labor Code. Dismissal based on just causes contemplate acts
or omissions attributable to the employee. Instead, respondents were terminated due to the completion of the
phases of work for which their services were engaged.

As project employees, respondents’ termination is governed by Section 1 (c) and Section 2 (III), Rule XXIII
(Termination of Employment), Book V of the Omnibus Rules Implementing the Labor Code.

Section 1. Security of tenure. (a) In cases of regular employment, the employer shall not
terminate the services of an employee except for just or authorized causes as provided by law,
and subject to the requirements of due process.

xxxx

(c) In cases of project employment or employment covered by legitimate contracting or


sub-contracting arrangements, no employee shall be dismissed prior to the completion of the
project or phase thereof for which the employee was engaged, or prior to the expiration of the
contract between the principal and contractor, unless the dismissal is for just or authorized
cause subject to the requirements of due process or prior notice, or is brought about by the
completion of the phase of the project or contract for which the employee was engaged.

Further, Section 2 (III), Rule XXIII, Book V of the Omnibus Rules Implementing the Labor Code provides:

III. If the termination is brought about by the completion of the contract or phase thereof, no prior notice is
required. If the termination is brought about by the failure of an employee to meet the standards of the
employer in the case of probationary employment, it shall be sufficient that a written notice is served the
employee within a reasonable time from the effective date of termination.

———————————————————————————————————————-

Angeles| Bajana | Balladares | Brillantes | Briones | Cabansag | Callanta | Chua | David|

De Leon | Gomez | Lopez | Macalino | Nostratis | Padilla | Reynon | Santos | Tan |Velasco
4E / 4F - 2018-2019
Page 626 of 920
LABOR REVIEW DIGEST
Atty. Joyrich Golangco

140. 16. Yolanda M. Mercado, et. al. v. Ama Computer College-Parañaque City, Inc.
141. GR No: 183572 Date: April 13, 2010
Ponente: BRION, J
142. Doctrine:
143. In a situation where the probationary status overlaps with a fixed-term contract not specifically used for
the fixed term it offers, Article 281(now 296) should assume primacy and the fixed-period character of the contract
must give way.
144.
145. Cases dealing with employment on probationary status of teaching personnel are not governed solely by
the Labor Code as the law is supplemented, with respect to the period of probation, by special rules found in the
Manual of Regulations for Private Schools.
146.
147. FACTS:
148. The petitioners were faculty members who started teaching at AMACC on May 25, 1998. Respondent,
AMACC, is an educational institution engaged in computer-based education in the country. The petitioners
executed individual Teacher's Contracts for each of the trimesters that they were engaged to
149. teach, with the following common stipulation:
150. 1. POSITION. The TEACHER has agreed to accept a non-tenured appointment to work in the College of
. . . effective . . . to . . . or for the duration of the last term that the TEACHER is given a teaching load based on the
assignment duly approved by the DEAN/SAVP-COO. [Emphasis supplied]
151.
152. For the school year 2000-2001, AMACC implemented new faculty screening guidelines, set forth in its
Guidelines on the Implementation of AMACC Faculty Plantilla. The petitioners failed to obtain a passing rating
based on the performance standards; hence AMACC did not give them any salary increase. Because of AMACC's
action on the salary increases, the petitioners filed a complaint with the Arbitration Branch of the NLRC on July 25,
2000, for underpayment of wages, non-payment of overtime and overload compensation, 13th month pay, and for
discriminatory practices.
153.
154. On September 7, 2000, the petitioners individually received a memorandum from AMACC informing them
155. that with the expiration of their contract to teach, their contract would no longer be renewed.
156.
157. The petitioners amended their labor arbitration complaint to include the charge of illegal dismissal against
AMACC.
158. In their Position Paper, the petitioners claimed that their dismissal was illegal because it was made in
retaliation for their complaint for monetary benefits and discriminatory practices against AMACC. The petitioners
also contended that AMACC failed to give them adequate notice; hence, their dismissal was ineffectual. AMACC
contended in response that the petitioners worked under a contracted term under a non-tenured appointment and
were still within the three-year probationary period for teachers. Their contracts were not renewed for the following
term because they failed to pass the Performance Appraisal System for Teachers (PAST) while others failed to
comply with the other requirements for regularization, promotion, or increase in salary.
159.
160. LA Ruling:
161. LA declared that the petitioners had been illegally dismissed, and ordered AMACC to reinstate them to
their former positions without loss of seniority rights and to pay them full backwages, attorney's fees and 13th
month pay. The LA ruled that Article 281 of the Labor Code on probationary employment applied to the case; that
AMACC allowed the petitioners to teach for the first semester of school year 2000-200; that AMACC did not
specify who among the petitioners failed to pass the PAST and who among them did not comply with the other
requirements of regularization, promotions or increase in salary; and that the petitioners' dismissal could not be
sustained on the basis of AMACC's "vague and general allegations" without substantial factual basis.
162.
Angeles| Bajana | Balladares | Brillantes | Briones | Cabansag | Callanta | Chua | David|

De Leon | Gomez | Lopez | Macalino | Nostratis | Padilla | Reynon | Santos | Tan |Velasco
4E / 4F - 2018-2019
Page 627 of 920
LABOR REVIEW DIGEST
Atty. Joyrich Golangco

163. NLRC Ruling:


164. NLRC affrmed the LA's finding of illegal dismissal since the petitioners were terminated on the basis of
standards that were only introduced near the end of their probationary period. According to the NLRC, the
imposition of the new guidelines violates Section 6 (d) of Rule I, Book VI of the Implementing Rules of the Labor
Code, which provides that "in all cases of probationary employment, the employer shall make known to the
employee the standards under which he will qualify as a regular employee at the time of his
165. engagement."
166.
167. CA RULING:
168. CA ruled that under the Manual for Regulations for Private Schools, a teaching personnel in a private
educational institution (1) must be a full time teacher; (2) must have rendered three consecutive years of service;
and (3) such service must be satisfactory before he or she can acquire permanent status. CA effectively found
reasonable basis for AMACC not to renew the petitioners' contracts on the ground that petitioners they failed to
satisfy the school's standards for the school year 2000-2001 that measured their fitness and aptitude to teach as
regular faculty members.
169.
170. APPEAL TO THE SC:
171. Petitioner’s contention:
172. Petitioners argue that the applicable law on probationary employment, as explained by the LA, is Article
281 of the Labor Code which mandates a period of six (6) months as the maximum duration of the probationary
period unless there is a stipulation to the contrary; that the CA should not have disturbed the LA's conclusion that
the AMACC failed to support its allegation that they did not qualify under the new guidelines adopted for the
school year 2000-2001; and that they were illegally dismissed; their employment was terminated based on
standards that were not made known to them at the time of their engagement.
173.
174. Respondent’s contention:
175. AMACC notes that the petitioners raised no substantial argument in support of their petition and that the
CA correctly found that the petitioners were hired on a non-tenured basis and for a fixed or predetermined term.
AMACC stresses that the CA was correct in concluding that no actual dismissal transpired; it simply did not renew
the petitioners' respective employment contracts because of their poor performance and failure to satisfy the
school's standards.
176.
177. ISSUE/S:
178. 1. Should the teachers' probationary status be disregarded simply because the contracts were fixed-
term?
179. 2. Whether or not the petitioners were illegally dismissed
180.
181. HELD:
182. 1. NO. The fixed-term period of employment should be regarded as the probationary period for the
petitioners. Under the given facts where the school year is divided into trimesters, the school apparently utilizes its
fixed-term contracts as a convenient arrangement dictated by the trimestral system and not because the
workplace parties really intended to limit the period of their relationship to any fixed term and to finish this
relationship at the end of that term. In a situation where the probationary status overlaps with a fixed-term contract
not specifically used for the fixed term it offers, Article 281(now 296) should assume primacy and the fixed-period
character of the contract must give way. This conclusion is immeasurably strengthened by the petitioners' and the
AMACC's hardly concealed expectation that the employment on probation could lead to permanent status, and
that the contracts are renewable unless the petitioners fail to pass the school's standards.
183.
184. 2. YES. Under Art 281 of the Labor Code, the services of an employee who has been engaged on a
probationary basis may be terminated for a just cause or when he fails to qualify as a regular employee in
Angeles| Bajana | Balladares | Brillantes | Briones | Cabansag | Callanta | Chua | David|

De Leon | Gomez | Lopez | Macalino | Nostratis | Padilla | Reynon | Santos | Tan |Velasco
4E / 4F - 2018-2019
Page 628 of 920
LABOR REVIEW DIGEST
Atty. Joyrich Golangco

accordance with reasonable standards made known by the employer to the employee at the time of his
engagement. In this case, the standards were duly communicated to the petitioners and could be applied
beginning the 1st trimester of the school year 2000- 2001, glaring and very basic gaps in the school's evidence
still exist. The exact terms of the standards were never introduced as evidence; neither does the evidence show
how these standards were applied to the petitioners. Without these pieces of evidence (effectively, the finding of
just cause for the non-renewal of the petitioners' contracts), we have nothing to consider and pass upon as valid
or invalid for each of the petitioners. Inevitably, the non-renewal (or effectively, the termination of employment of
employees on probationary status) lacks the supporting finding of just cause that the law requires and, hence, is
illegal.

———————————————————————————————————————-

Angeles| Bajana | Balladares | Brillantes | Briones | Cabansag | Callanta | Chua | David|

De Leon | Gomez | Lopez | Macalino | Nostratis | Padilla | Reynon | Santos | Tan |Velasco
4E / 4F - 2018-2019
Page 629 of 920
LABOR REVIEW DIGEST
Atty. Joyrich Golangco

17. BRENT SCHOOL, INC., and REV. GABRIEL DIMACHE v. RONALDO ZAMORA, the Presidential
Assistant for Legal Affairs, Office of the President, and DOROTEO R. ALEGRE

G.R. No. L-48494 Date: February 5, 1990


Ponente: NARVASA, J.

Doctrine:
Stipulations in employment contracts providing for “term employment” or “fixed period employment” are valid when
the period where agreed upon knowingly, and voluntarily by the parties without force, duress or improper pressure
exerted on the employee; and when such stipulations were not designed to circumvent the laws on security of
tenure.

FACTS:
Respondent Doroteo Alegre was engaged as the athletic director by the petitioner Brent School, Inc. The contract
fixed a specific term for its existence at 5 years. Subsequent subsidiary agreements reiterated the same terms
and conditions including the expiry date as those contained in the original contract. 3 months before the expiration
of the stipulated period, Alegre was given a copy of the report by Brent School with the Department of Labor
advising of the termination of his services. The stated ground for termination was completion of contract and
expiration of the definite period. Alegre protested the announced termination of his services at the investigation
conducted by a Labor Conciliator.

Respondent’s contentions:
Alegre argued that although his contract did stipulate that the same would terminate after 5 years, since his
services were necessary and desirable in the usual business of his employer, and his employment had lasted for
5 years, he had acquired the status of a regular employee and could not be removed except for valid cause.

DOLE-RD RULING:
Required the reinstatement of Alegre as a permanent employee as recommended by the Labor Conciliator.

DOLE SECRETARY RULING:


Affirmed the Regional Director.

OFFICE OF THE PRESIDENT RULING:


Dismissed petitioner’s appeal.

ISSUE:
Was the fixed-period employment of Alegre valid? YES.

RULING:

YES. Since the entire purpose behind the development of legislation culminating in the present Article 280 of the
Labor Code is to prevent circumvention of the employee’s right to be secure in his tenure, the clause in said article
indiscriminately and completely ruling out all written or oral agreements conflicting with the concept of regular
employment as defined therein should be construed to refer to the substantive evil that the Code itself has singled
out: agreements entered into precisely to circumvent security of tenure. It should have no application to instances
where a fixed period of employment was agreed upon knowingly and voluntarily by the parties, without any force,
duress or improper pressure being brought to bear upon the employee and absent any other circumstances
vitiating his consent, or where it satisfactorily appears that the employer and employee dealt with each other on
more or less equal terms with no moral dominance whatever being exercised by the former over the latter. Unless
thus limited in its purview, the law would be made to apply to purposes other than those explicitly stated by its
Angeles| Bajana | Balladares | Brillantes | Briones | Cabansag | Callanta | Chua | David|

De Leon | Gomez | Lopez | Macalino | Nostratis | Padilla | Reynon | Santos | Tan |Velasco
4E / 4F - 2018-2019
Page 630 of 920
LABOR REVIEW DIGEST
Atty. Joyrich Golangco

framers; it thus becomes pointless and arbitrary, unjust in its effects and apt to lead to absurd and unintended
consequences.

By a reading of Article 319 the question is whether or not a voluntary agreement on a fixed term or period would
be valid where the employee “has been engaged to perform activities which are usually necessary or desirable in
the usual business or trade of the employer.” The definition seems a non sequitur. From the premise—that the
duties of an employee entail “activities which are usually necessary or desirable in the usual business or trade of
the employer”—the conclusion does not necessarily follow that the employer and employee should be forbidden to
stipulate any period of time for the performance of those activities. There is nothing essentially contradictory
between a definite period of an employment contract and the nature of the employee’s duties set down in that
contract as being “usually necessary or desirable in the usual business or trade of the employer.” The concept of
the employee’s duties as being “usually necessary or desirable in the usual business or trade of the employer” is
not synonymous with or identical to employment with a fixed term. Logically, the decisive determinant in term
employment should not be the activities that the employee is called upon to perform, but the day certain agreed
upon by the parties for the commencement and termination of their employment relationship, a day certain being
understood to be “that which must necessarily come, although it may not be known when.” Seasonal employment,
and employment for a particular project are merely instances of employment in which a period, where not
expressly set down, is necessarily implied.

———————————————————————————————————————-

Angeles| Bajana | Balladares | Brillantes | Briones | Cabansag | Callanta | Chua | David|

De Leon | Gomez | Lopez | Macalino | Nostratis | Padilla | Reynon | Santos | Tan |Velasco
4E / 4F - 2018-2019
Page 631 of 920
LABOR REVIEW DIGEST
Atty. Joyrich Golangco

18. PURE FOODS CORPORATON, petitioner, vs. NATIONAL LABOR RELATIONS COMMISSION,
RODOLFO CORDOVA, VIOLETA CRUSIS, ET AL.,*respondents.
GR No: 122653 Date: December 12, 1997
Ponente: DAVIDE, JR., J.

Doctrine: In the instant case, the private respondent’s activities consisted in the receiving, skinning, loining,
packing, and casing-up of tuna fish which were then exported by the petitioner. Indisputably, they were performing
activities which were necessary and desirable in petitioners business or trade. The fact that the petitioner
repeatedly and continuously hired workers to do the same kind of work as that performed by those whose
contracts had expired negates petitioners contention that those workers were hired for a specific project or
undertaking only.
1)
FACTS:
2)The private respondents (numbering 906) were hired by petitioner Pure Foods Corporation to work for a
fixed period of five months at its tuna cannery plant in Tambler, General Santos City. After the expiration of their
respective contracts of employment in June and July 1991, their services were terminated. They forthwith
executed a Release and Quitclaim stating that they had no claim whatsoever against the petitioner.
3)On 29 July 1991, the private respondents filed before the National Labor Relations Commission (NLRC)
Sub-Regional Arbitration Branch No. XI, General Santos City, a complaint for illegal dismissal against the
petitioner and its plant manager, Marciano Aganon.
LA handed down a decision [2] dismissing the complaint on the ground that the private respondents were mere
contractual workers, and not regular employees; hence, they could not avail of the law on security of tenure. The
termination of their services by reason of the expiration of their contracts of employment was, therefore, justified.
The Labor Arbiter also observed that an order for private respondent’s reinstatement would result in the
reemployment of more than 10,000 former contractual employees of the petitioner. Besides, by executing a
Release and Quitclaim, the private respondents had waived and relinquished whatever right they might have
against the petitioner.
4)NLRC affirmed the Labor Arbiter's decision. [3] However, on private respondent’s motion for
reconsideration, the NLRC rendered another decision on 30 January 1995 [4]vacating and setting aside its
decision of 28 October 1994 and holding that the private respondents and their co-complainants were regular
employees. It declared that the contract of employment for five months was a clandestine scheme employed by
[the petitioner] to stifle [private respondents] right to security of tenure and should therefore be struck down and
disregarded for being contrary to law, public policy, and morals. Hence, their dismissal on account of the
expiration of their respective contracts was illegal.
5)Accordingly, the NLRC ordered the petitioner to reinstate the private respondents to their former position
without loss of seniority rights and other privileges, with full back wages; and in case their reinstatement would no
longer be feasible, the petitioner should pay them separation pay equivalent to one-month pay or one-half-month
pay for every year of service, whichever is higher, with back wages and 10% of the monetary award as attorney’s
fees.
6)The petitioner submits that the private respondents are now estopped from questioning their separation
from petitioners employ in view of their express conformity with the five-month duration of their employment
contracts. Besides, they fell within the exception provided in Article 280 of the Labor Code which reads: [E]xcept
where the employment has been fixed for a specific project or undertaking the completion or termination of which
has been determined at the time of the engagement of the employee.
7)In its Comment, the Office of the Solicitor General (OSG) advances the argument that the private
respondents were regular employees, since they performed activities necessary and desirable in the business or
trade of the petitioner. The period of employment stipulated in the contracts of employment was null and void for
being contrary to law and public policy, as its purpose was to circumvent the law on security of tenure. The
expiration of the contract did not, therefore, justify the termination of their employment.

Angeles| Bajana | Balladares | Brillantes | Briones | Cabansag | Callanta | Chua | David|

De Leon | Gomez | Lopez | Macalino | Nostratis | Padilla | Reynon | Santos | Tan |Velasco
4E / 4F - 2018-2019
Page 632 of 920
LABOR REVIEW DIGEST
Atty. Joyrich Golangco

8)The private respondents, on the other hand, argue that contracts with a specific period of employment may
be given legal effect provided, however, that they are not intended to circumvent the constitutional guarantee on
security of tenure. They submit that the practice of the petitioner in hiring workers to work for a fixed duration of
five months only to replace them with other workers of the same employment duration was apparently to prevent
the regularization of these so-called casuals, which is a clear circumvention of the law on security of tenure.

ISSUE:
Whether employees hired for a definite period and whose services are necessary and desirable in the usual
business or trade of the employer are regular employees.

HELD:
9)Based on Article 280 of the Labor Code, the two kinds of regular employees are (1) those who are
engaged to perform activities which are necessary or desirable in the usual business or trade of the employer; and
(2) those casual employees who have rendered at least one year of service, whether continuous or broken, with
respect to the activity in which they are employed.[6]
10)In the instant case, the private respondent’s activities consisted in the receiving, skinning, loining,
packing, and casing-up of tuna fish which were then exported by the petitioner. Indisputably, they were performing
activities which were necessary and desirable in petitioners business or trade.
11)Contrary to petitioner's submission, the private respondents could not be regarded as having been hired
for a specific project or undertaking. The term specific project or undertaking under Article 280 of the Labor Code
contemplates an activity which is not commonly or habitually performed or such type of work which is not done on
a daily basis but only for a specific duration of time or until completion; the services employed are then necessary
and desirable in the employers usual business only for the period of time it takes to complete the project.[7]
12)The fact that the petitioner repeatedly and continuously hired workers to do the same kind of work as that
performed by those whose contracts had expired negates petitioners contention that those workers were hired for
a specific project or undertaking only.
13)Now on the validity of private respondents' five-month contracts of employment. In the leading case of
Brent School, Inc. v. Zamora, [8] which was reaffirmed in numerous subsequent cases, [9] this Court has upheld the
legality of fixed-term employment. It ruled that the decisive determinant in term employment should not be the
activities that the employee is called upon to perform but the day certain agreed upon by the parties for the
commencement and termination of their employment relationship. But, this Court went on to say that where from
the circumstances it is apparent that the periods have been imposed to preclude acquisition of tenurial security by
the employee, they should be struck down or disregarded as contrary to public policy and morals.
14)The petitioner does not deny or rebut private respondents' averments (1) that the main bulk of its
workforce consisted of its so-called casual employees; (2) that as of July 1991, casual workers numbered 1,835;
and regular employees, 263; (3) that the company hired casual every month for the duration of five months, after
which their services were terminated and they were replaced by other casual employees on the same five-month
duration; and (4) that these casual employees were actually doing work that were necessary and desirable in
petitioners usual business.
15)As a matter of fact, the petitioner even stated in its position paper submitted to the Labor Arbiter that,
according to its records, the previous employees of the company hired on a five-month basis numbered about
10,000 as of July 1990. This confirms private respondent’s allegation that it was really the practice of the company
to hire workers on a uniformly fixed contract basis and replace them upon the expiration of their contracts with
other workers on the same employment duration.
16)This scheme of the petitioner was apparently designed to prevent the private respondents and the other
casual employees from attaining the status of a regular employee. It was a clear circumvention of the employee’s
right to security of tenure and to other benefits like minimum wage, cost-of-living allowance, sick leave, holiday
pay, and 13th month pay. [11] Indeed, the petitioner succeeded in evading the application of labor laws. Also,
it saved itself from the trouble or burden of establishing a just cause for terminating employees by the simple
expedient of refusing to renew the employment contracts.
Angeles| Bajana | Balladares | Brillantes | Briones | Cabansag | Callanta | Chua | David|

De Leon | Gomez | Lopez | Macalino | Nostratis | Padilla | Reynon | Santos | Tan |Velasco
4E / 4F - 2018-2019
Page 633 of 920
LABOR REVIEW DIGEST
Atty. Joyrich Golangco

17)The five-month period specified in private respondents employment contracts having been imposed
precisely to circumvent the constitutional guarantee on security of tenure should, therefore, be struck down or
disregarded as contrary to public policy or morals. [12] To uphold the contractual arrangement between the
petitioner and the private respondents would, in effect, permit the former to avoid hiring permanent or regular
employees by simply hiring them on a temporary or casual basis, thereby violating the employee’s security of
tenure in their jobs.[13]
18)The execution by the private respondents of a Release and Quitclaim did not preclude them from
questioning the termination of their services. Generally, quitclaims by laborers are frowned upon as contrary to
public policy and are held to be ineffective to bar recovery for the full measure of the worker’s rights. [14] The
reason for the rule is that the employer and the employee do not stand on the same footing.[15]
19)Notably, the private respondents lost no time in filing a complaint for illegal dismissal. This act is hardly
expected from employees who voluntarily and freely consented to their dismissal.[16]
20)The NLRC was, thus, correct in finding that the private respondents were regular employees and that
they were illegally dismissed from their jobs. Under Article 279 of the Labor Code and the recent
jurisprudence, [17] the legal consequence of illegal dismissal is reinstatement without loss of seniority rights and
other privileges, with full back wages computed from the time of dismissal up to the time of actual reinstatement,
without deducting the earnings derived elsewhere pending the resolution of the case.
21)However, since reinstatement is no longer possible because the petitioner's tuna cannery plant had,
admittedly, been closed in November 1994,[18] the proper award is separation pay equivalent to one month pay or
one-half month pay for every year of service, whichever is higher, to be computed from the commencement of
their employment up to the closure of the tuna cannery plant. The amount of back wages must be computed from
the time the private respondents were dismissed until the time petitioner's cannery plant ceased operation.
———————————————————————————————————————-

Angeles| Bajana | Balladares | Brillantes | Briones | Cabansag | Callanta | Chua | David|

De Leon | Gomez | Lopez | Macalino | Nostratis | Padilla | Reynon | Santos | Tan |Velasco
4E / 4F - 2018-2019
Page 634 of 920
LABOR REVIEW DIGEST
Atty. Joyrich Golangco

185. 19. Leyte Geothermal Power Progressive Employees Union vs Phil National Oil Company Energy
Devt Corporation (PNOC-EDC)
186. GR No: 170351 Date: March 30, 2011
187. Ponente: Nachura, J.
188.
189. Doctrine:
190. the litmus test to determine whether an individual is a project employee lies in setting a fixed period of
employment involving a specific undertaking which completion or termination has been determined at the time of
the particular employees engagement.
191.
192. FACTS:
193. PNOC-EDC is a GOCC engaged in exploration, development, utilization, generation and distribution of
energy resources like geothermal energy. Petitioner is a legitimate labor organization. Among PNOC-EDC‘s
geothermal projects is the Leyte Geothermal Power Project.
194.
195. Majority of the employees hired by PNOC-EDC are members of petitioner. In view of that circumstance,
the petitioner demanded from the PNOC-EDC for it’s recognition as the collective bargaining agent of said
employees and for a CBA negotiation with it. However, PNOC-EDC did not heed such demands of the petitioner.
Sometime in 1998 when the project was about to be completed, PNOC-EDC served Notices of Termination of
Employment upon the employees who are members of the petitioner.
196.
197. The petitioners filed a notice of strike and the DOLE Sec. assumed jurisdiction. However, despite earnest
efforts to compromise, the dispute was not settled amicably.
198.
199. PNOC-EDC filed a Complaint for Strike Illegality, Declaration of Loss of Employment and Damages at
the NLRC in Tacloban City and at the same time, filed a Petition for Cancellation of Petitioners Certificate of
Registration.
200.
201. NLRC RULING:
202. The NLRC ruled that the petitioners are project employees. Petitioner Union filed a motion for
reconsideration of the NLRC decision, which was subsequently denied.
203.
204. CA RULING:
205. Petitioner Union filed a petition for certiorari before the CA, alleging grave abuse of discretion in the
decision of the NLRC. However, the CA dismissed the said petition.
206.
207. Petitioner's Contention:
208. Petitioner Union contends that its officers and members performed activities that were usually necessary
and desirable to respondents usual business.
209.
210. Respondent's Contention:
211. Petitioners are mere project employees.
212.
213. ISSUE/S:
214. 1. Whether the officers and members of petitioner Union are project employees of respondent.
215.
216. HELD:
1. Yes. The foregoing contemplates 4 kinds of employees:
(1) regular employees or those who have been engaged to perform activities which are usually necessary or
desirable in the usual business or trade of the employer;
Angeles| Bajana | Balladares | Brillantes | Briones | Cabansag | Callanta | Chua | David|

De Leon | Gomez | Lopez | Macalino | Nostratis | Padilla | Reynon | Santos | Tan |Velasco
4E / 4F - 2018-2019
Page 635 of 920
LABOR REVIEW DIGEST
Atty. Joyrich Golangco

(2) project employees or those whose employment has been fixed for a specific project or undertaking, the
completion or termination of which has been determined at the time of the engagement of the employee;
(3) seasonal employees or those who work or perform services which are seasonal in nature, and the
employment is for the duration of the season; and
(4) casual employees or those who are not regular, project, or seasonal employees. Jurisprudence has added a
fifth kind a fixed-term employee.
217.
218. Project employment contracts which fix the employment for a specific project or undertaking remain valid
under the law.
219.
220. In this case, the officers and the members of petitioner Union were specifically hired as project
employees for respondents Leyte Geothermal Power Project. Consequently, upon the completion of the project or
substantial phase thereof, the officers and the members of petitioner Union could be validly terminated.
221.
222. __________________________________________________________________________

Angeles| Bajana | Balladares | Brillantes | Briones | Cabansag | Callanta | Chua | David|

De Leon | Gomez | Lopez | Macalino | Nostratis | Padilla | Reynon | Santos | Tan |Velasco
4E / 4F - 2018-2019
Page 636 of 920
LABOR REVIEW DIGEST
Atty. Joyrich Golangco

20. ENGINEER LEONCIO V. SALAZAR, vs. NATIONAL LABOR RELATIONS COMMISSION (2nd Division)
and H. L. CARLOS CONSTRUCTION, CO. INC.
GR No.: 109210 Date: APRL 17, 1996
Ponente: Kapunan, J.

DOCTRINE:

As a project employee, petitioner’s services are deemed coterminous with the project, that is, petitioner’s services
may be terminated as soon as the project for which he was hired is completed.

FACTS:

On 17 April 1990, private respondent Carlos Construction, at a monthly salary of P4,500.00, employed Salazar as
construction/project engineer for the construction of a building in Cubao. Allegedly, by virtue of an oral contract,
petitioner would also receive a share in the profits after completion of the project and that petitioner’s services in
excess of 8 ours on regular days and services rendered on weekends and legal holidays shall be compensable
overtime.

On 16 April 1991, petitioner received a memorandum issued by private respondent’s project manager informing
him of the termination of his services effective on 30 April 1991.

On 13 September 1991, Salazar filed a complaint against private respondent for illegal dismissal, unfair labor
practice, illegal deduction, non-payment of wages, overtime rendered, service incentive leave pay, commission,
allowances, profit-sharing and separation pay with the NLRC-NCR Arbitration Branch, Manila.

The Labor Arbiter rendered a decision dismissing the instant case for lack of merit. Petitioner appealed to the
NLRC, where it affirmed in toto the decision of the Labor Arbiter. His MR was likewise dismissed. Hence the
instant petition.

ISSUE/S:

22)WON petitioner is entitled to overtime pay, premium pay for services rendered on rest days and holidays and
service incentive leave pay;
23)WON petitioner is entitled to a share in the profits of the construction project;.
3) WON petitioner rendered services from 1 May to 15 May 1991 and is, therefore, entitled to unpaid wages;
4) WON private respondent is liable to reimburse petitioner’s legal expenses and;
5) WON petitioner is entitled to separation pay.

HELD:

1. NO. Although petitioner cannot strictly be classified as a managerial employee, nonetheless he is still not
entitled to payment of the aforestated benefits because he falls squarely under another exempt category —
“officers or members of a managerial staff” as defined under sec. 2(c) of the abovementioned implementing rules:

Sec. 2. Exemption. — The provisions of this Rule shall not apply to the following persons if they
qualify for exemption under the condition set forth herein:xxx

(c) Officers or members of a managerial staff xxx

Angeles| Bajana | Balladares | Brillantes | Briones | Cabansag | Callanta | Chua | David|

De Leon | Gomez | Lopez | Macalino | Nostratis | Padilla | Reynon | Santos | Tan |Velasco
4E / 4F - 2018-2019
Page 637 of 920
LABOR REVIEW DIGEST
Atty. Joyrich Golangco

That petitioner was paid overtime benefits does not automatically and necessarily denote that petitioner is entitled
to such benefits.

b. NO. Petitioner insists that private respondent promised him a share in the profits after completion of the
construction project. It is because of this oral agreement, petitioner elucidates, that he agreed to a monthly salary
of P4,500.00, an amount which he claims is too low for a professional civil engineer like him with the rank of
project engineer.

We cannot accede to petitioner’s demand. Nowhere in the disbursement vouchers can we find even the remotest
hint of a profit-sharing agreement between petitioner and private respondent. Petitioner’s rationalization stretches
the imagination way too far.

Also, as said by the Labor Arbiter:


“As to the issue of profit sharing, we simply cannot grant the same on the mere basis of
complainant’s allegation that respondent verbally promised him that he is entitled to a share in the
profits derive(d) from the projects.
Benefits or privileges of this nature (are) usually in writing, besides complainant failed to (establish)
that said benefits or privileges (have) been given to any of respondent(‘s) employees as a matter
of practice or policy.”

c. YES. On April 30, he was advised by the Manager to continue supervising the finishing touches to the
building until May 15, the date appearing in the Certificate of Service as the date of the termination of the
contract between Salazar and the Company. But the Manager insists that Salazar’s services terminated at April
30 according to the Memorandum given the petitioner.

The purpose for which the said certificate was issued becomes irrelevant. The fact remains that private
respondent knowingly and voluntarily issued the certificate. Mere denials and self-serving statements to the effect
that petitioner allegedly promised not to use the certificate against private respondent are not sufficient to overturn
the same. Hence, private respondent is estopped from assailing the contents of its own certificate of service.

d. YES. During the construction of the building, a criminal complaint for unjust vexation was filed against the
officers of the owner of the building. Petitioner avers that he was implicated in the complaint for the sole reason
that he was the construction engineer of the project.

Although not directly implicated in the criminal complaint, Carlos Construction is nonetheless obligated to defray
petitioner’s legal expenses. Petitioner was included in the complaint not in his personal capacity but in his capacity
as project engineer of private respondent and the case arose in connection with his work as such. At the
construction site, petitioner is the representative of private respondent being its employee and he acts for and in
behalf of private respondent. Hence, the inclusion of petitioner in the complaint for unjust vexation, which was
work-related, is equivalent to inclusion of private respondent itself.

e. NO. On the last issue, we rule that petitioner is a project employee and, therefore, not entitled to
separation pay.

The applicable provision is Article 280 of the Labor Code which defines the term “project employee,” thus:

“Art. 280. Regular and Casual Employment. — The provisions of written agreement to the contrary
notwithstanding and regardless of the oral agreement of the parties, an employment shall be deemed to be
regular where the employee has been engaged to perform activities which are usually necessary or
Angeles| Bajana | Balladares | Brillantes | Briones | Cabansag | Callanta | Chua | David|

De Leon | Gomez | Lopez | Macalino | Nostratis | Padilla | Reynon | Santos | Tan |Velasco
4E / 4F - 2018-2019
Page 638 of 920
LABOR REVIEW DIGEST
Atty. Joyrich Golangco

desirable in the usual business or trade of the employer, except where the employment has been fixed for a
specific period or undertaking the completion or termination of which has been determined at the time of the
engagement of the employee or where the work or services to be performed is seasonal in nature and the
employment is for the duration of the season. (Emphasis ours.)”

In the case at bench, it was duly established that private respondent hired petitioner as project or construction
engineer specifically for its Monte de Piedad building project. Accordingly, as project employee, petitioner’s
services are deemed coterminous with the project, that is, petitioner’s services may be terminated as soon as the
project for which he was hired is completed. There can be no dispute that petitioner’s dismissal was due to the
completion of the construction of the building.

Angeles| Bajana | Balladares | Brillantes | Briones | Cabansag | Callanta | Chua | David|

De Leon | Gomez | Lopez | Macalino | Nostratis | Padilla | Reynon | Santos | Tan |Velasco
4E / 4F - 2018-2019
Page 639 of 920
LABOR REVIEW DIGEST
Atty. Joyrich Golangco

21. FONTERRA BRANDS PHILS. vs LARGADO


GR No. 205300, Mar 18, 2015
Velasco Jr., J.:

DOCTRINE: Fixed-term employment contracts are not limited, as they are under the present Labor Code, to those
by nature seasonal or for specific projects with predetermined dates of completion; they also include those to
which the parties by free choice have assigned a specific date of termination. The determining factor of such
contracts is not the duty of the employee but the day certain agreed upon by the parties for the commencement
and termination of the employment relationship.

FACTS: Petitioner Fonterra Brands Phils., Inc. (Fonterra) contracted the services of Zytron Marketing and
Promotions Corp. (Zytron) for the marketing and promotion of its milk and dairy products. Pursuant to the contract,
Zytron provided Fonterra with trade merchandising representatives (TMRs), including respondents Leonardo
Largado (Largado) and Teotimo Estrellado (Estrellado).

Fonterra sent Zytron a letter terminating its promotions contract, effective June 5, 2006. Fonterra then entered into
an agreement for manpower supply with A.C. Sicat Marketing and Promotional Services (A.C. Sicat). Desirous of
continuing their work as TMRs, respondents submitted their job applications with A.C. Sicat, which hired them for
a term of five (5) months.

When respondents' 5-month contracts with A.C. Sicat were about to expire, they allegedly sought renewal thereof,
but were allegedly refused. This prompted respondents to file complaints for illegal dismissal, regularization, non-
payment of service incentive leave and 13th month pay, and actual and moral damages, against petitioner, Zytron,
and A.C. Sicat.

LA Ruling: dismissed the complaint and ruled that respondents were not illegally dismissed. and they were
consecutively employed by Zytron and A.C. Sicat, not by Fonterra.

NLRC Ruling: affirmed the Labor Arbiter.


CA Ruling: CA found that A.C. Sicat satisfies the requirements of legitimate job contracting, but Zytron does not.
Additionally, the CA held that respondents were illegally dismissed since Fonterra itself failed to prove that their
dismissal is lawful. However, respondents' transfer to A.C. Sicat is tantamount to a completely new engagement
by another employer.

ISSUE/s:
i. Are the respondents are employees of the Zytron or A.C. Sicat?
ii. Considering the respondents’ type of employment, is there illegal dismissal?

HELD:
1. The Respondents are employees of A.C. Sicat. The termination of respondents' employment with Zytron was
brought about by the cessation of their contracts with the latter. By refusing to renew their contracts with Zytron,
respondents effectively resigned from the latter. This being the case, Zytron cannot be said to have illegally
dismissed respondents, contrary to the findings of the CA.

Furthermore, A.C. Sicat, as correctly ruled by CA is a legitimate labor-only contractor. Its Agreement with Fonterra
clearly sets forth that A.C. Sicat shall be liable for the wages and salaries of its employees or workers, including
benefits, premiums, and protection due them, as well as remittance to the proper government entities of all
withholding taxes, Social Security Service, and Medicare premiums, in accordance with relevant laws.

Angeles| Bajana | Balladares | Brillantes | Briones | Cabansag | Callanta | Chua | David|

De Leon | Gomez | Lopez | Macalino | Nostratis | Padilla | Reynon | Santos | Tan |Velasco
4E / 4F - 2018-2019
Page 640 of 920
LABOR REVIEW DIGEST
Atty. Joyrich Golangco

The appellate court further correctly held that Fonterra's issuance of Merchandising Guidelines, stock monitoring
and inventory forms, and promo mechanics, for compliance and use of A.C. Sicat's employees assigned to them,
does not establish that Fonterra exercises control over A.C. Sicat.

2. No, there was no illegal dismissal, since the respondents were merely fixed-term employees, more specifically
project employees, thus the termination of respondents' employment with the latter was simply brought about by
the expiration of their employment contracts.

Fixed-term employment contracts are not limited, as they are under the present Labor Code, to those by nature
seasonal or for specific projects with predetermined dates of completion; they also include those to which the
parties by free choice have assigned a specific date of termination. The determining factor of such contracts is not
the duty of the employee but the day certain agreed upon by the parties for the commencement and termination of
the employment relationship.

In the case at bar, it is clear that respondents were employed by A.C. Sicat as project employees. In their
employment contract with the latter, it is clearly stated that "[A.C. Sicat is] temporarily employing [respondents] as
TMR[s] under the following terms and conditions: The need for your service being only for a specific project, your
temporary employment will be for the duration only of said project of our client, namely to promote FONTERRA
BRANDS products xxx which is expected to be finished on or before Nov. 06, 2006.

Respondents, by accepting the conditions of the contract with A.C. Sicat, were well aware of and even acceded to
the condition that their employment thereat will end on said pre-determined date of termination. They cannot now
argue that they were illegally dismissed by the latter when it refused to renew their contracts after its expiration.
This is so since the non-renewal of their contracts by A.C. Sicat is a management prerogative, and failure of
respondents to prove that such was done in bad faith militates against their contention that they were illegally
dismissed. The expiration of their contract with A.C. Sicat simply caused the natural cessation of their fixed-term
employment there at.

Note: Requisites of Legitimate labor-only contractor:


5. The contractor or subcontractor carries on a distinct and independent business and undertakes to perform
the job, work or service on its own account and under its own responsibility according to its own manner
and method, and free from the control and direction of the principal in all matters connected with the
performance of the work except as to the results thereof;
5. The contractor or subcontractor has substantial capital or investment; and
5. The agreement between the principal and contractor or subcontractor assures the contractual employees
entitlement to all labor and occupational safety and health standards, free exercise of the right to self-
organization, security of tenure, and social and welfare benefit

Angeles| Bajana | Balladares | Brillantes | Briones | Cabansag | Callanta | Chua | David|

De Leon | Gomez | Lopez | Macalino | Nostratis | Padilla | Reynon | Santos | Tan |Velasco
4E / 4F - 2018-2019
Page 641 of 920
LABOR REVIEW DIGEST
Atty. Joyrich Golangco

22. Basan vs. Coca-Cola Bottlers Phils.


GR No: G.R. Nos. 174365-66 Date: February 4, 2015
Ponente: Peralta, J.

Doctrine: But, although the work to be performed is only for a specific projector seasonal, where a person thus
engaged has been performing the job for at least one year, even if the performance is not continuous or is merely
intermittent, the law deems the repeated and continuing need for its performance as being sufficient to indicate the
necessity or desirability of that activity to the business or trade of the employer. The employment of such person is
also then deemed to be regular with respect to such activity and while such activity exists.

FACTS: Petitioners Romeo Basan, Danilo Dizon, Jaime L. Tumabiao, Jr., Roberto Dela Rama,Jr., Ricky S. Nicolas,
Crispulo D. Donor, Galo Falguera filed a complaint for illegal dismissal with money claims against respondent Coca-
Cola Bottlers Philippines, alleging that respondent dismissed them without just cause and prior written notice required
by law.

Respondent corporation, however, countered that it hired petitioners as temporary route helpers to act as substitutes
for its absent regular route helpers merely for a fixed period in anticipation of the high volume of work in its plants or
sales offices. As such, petitioners’ claims have no basis for they knew that their assignment as route helpers was
temporary in duration.

LA/NLRC RULING: Labor Arbiter and NLRC ruled in favor of petitioners and found that since they were performing
activities necessary and desirable to the usual business of petitioner for more than the period for regularization,
petitioners are considered as regular employees, and thus, their dismissal was done contrary to law in the absence of
just cause and prior written notice. Thus, it ordered respondent to reinstate petitioners with full backwages from the
time their salaries were withheld until their actual reinstatement

CA RULING: That the respondents "performed duties which are necessary or desirable in the usual trade or business
of Coca-Cola," is of no moment. This is not the only standard for determining the status of one’s employment. Such
fact does not prevent them from being considered as fixed term employees of Coca-Cola whose engagement was
"fixed" for a specific period. The respondent’s repeated hiring for various periods (ranging from more than six months
for private respondent Basan to eight years in the case of private respondent Dizon) would not automatically
categorize them as REGULAR EMPLOYEES.

APPEAL TO THE SC:

Petitioner's Contention: Petitioners essentially maintain that contrary to the findings of the CA, they were continuously
hired by respondent company to perform duties necessary and desirable in the usual trade or business and are,
therefore, regular employees. They allege that if their services had really been engaged for fixed specific periods,
respondent should have at least provided the contracts of employment evidencing the same.

Respondent's Contention: For its part, respondent contends that petitioners were fixed-term employees who were
hired intermittently. Considering that the evidence presented showed that petitioners merely rendered their services
for periods of less than a year, respondent claims that petitioners could not have attained regular employment status.
It added that its failure to present petitioners’ employment contracts was due to a fire that destroyed its Manila Plant
where said contracts were kept. Nevertheless, respondent persistently asserts that where a fixed period of
employment was agreed upon knowingly and voluntarily by the petitioners, the duration of which was made known to
them at the time of their engagement, petitioners cannot now claim otherwise. In addition, it disagrees with the
contention that petitioners, as route helpers, were performing functions necessary or desirable to its business.

ISSUE/S: whether or not petitioners are regular employees of respondent

HELD: YES. The nature of work of route helpers hired by Coca Cola Bottlers Philippines, Inc. is necessary and
desirable in its usual business or trade thereby qualifying them as regular employees. Respondent workers, when
hired, would go with route salesmen on board delivery trucks and undertake the laborious task of loading and
unloading softdrink products of petitioner company to its various delivery points.

Angeles| Bajana | Balladares | Brillantes | Briones | Cabansag | Callanta | Chua | David|


De Leon | Gomez | Lopez | Macalino | Nostratis | Padilla | Reynon | Santos | Tan |Velasco
4E / 4F - 2018-2019
Page 642 of 920
LABOR REVIEW DIGEST
Atty. Joyrich Golangco

In determining whether an employment should be considered regular or non-regular, the applicable test is the
reasonable connection between the particular activity performed by the employee in relation to the usual business or
trade of the employer.

It is distinguished from a specific undertaking that is divorced from the normal activities required in carrying on the
particular business or trade. But, although the work to be performed is only for a specific projector seasonal, where a
person thus engaged has been performing the job for at least one year, even if the performance is not continuous or
is merely intermittent, the law deems the repeated and continuing need for its performance as being sufficient to
indicate the necessity or desirability of that activity to the business or trade of the employer. The employment of such
person is also then deemed to be regular with respect to such activity and while such activity exists.

The argument of petitioner that its usual business or trade is softdrink manufacturing and that the work assigned to
respondent workers as sales route helpers so involves merely "postproduction activities," one which is not
indispensable in the manufacture of its products, scarcely can be persuasive. If, as so argued by petitioner company,
only those whose work are directly involved in the production of softdrinks may be held performing functions
necessary and desirable in its usual business or trade, there would have then been no need for it to even maintain
regular truck sales route helpers. The nature of the work performed must be viewed from a perspective of the
business or trade in its entirety and not on a confined scope.

As route helpers who are engaged in the service of loading and unloading softdrink products of respondent company
to its various delivery points, which is necessary or desirable in its usual business or trade, petitioners are considered
as regular employees. That they merely rendered services for periods of less than a year is of no moment since for
as long as they were performing activities necessary to the business of respondent, they are deemed as regular
employees under the Labor Code, irrespective of the length of their service.

Angeles| Bajana | Balladares | Brillantes | Briones | Cabansag | Callanta | Chua | David|


De Leon | Gomez | Lopez | Macalino | Nostratis | Padilla | Reynon | Santos | Tan |Velasco
4E / 4F - 2018-2019
Page 643 of 920
LABOR REVIEW DIGEST
Atty. Joyrich Golangco

23. CONVOY MARKETING VS ALBIA

Doctrine: Labor Code – Art. 280 Regular and casual employment. - The provisions of written agreement to the
contrary notwithstanding and regardless of the oral agreement of the parties, an employment shall be deemed to be
regular where the employee has been engaged to perform activities which are usually necessary or desirable in the
usual business or trade of the employer, except where the employment has been fixed for a specific project or
undertaking the completion or termination of which has been determined at the time of the engagement of the
employee or where the work or service to be performed is seasonal in nature and the employment is for the duration
of the season.
An employment shall be deemed to be casual if it is not covered by the preceding paragraph: Provided, That any
employee who has rendered at least one year of service, whether such service is continuous or broken, shall be
considered a regular employee with respect to the activity in which he is employed and his employment shall
continue while such activity exists
Art. 282 Termination by employer. - An employer may terminate an employment for any of the following causes: (a)
Serious misconduct or willful disobedience by the employee of the lawful orders of his employer or representative in
connection with his work; x x x

FACTS:
• Oliver Albia started working for Convoy Marketing in 2001. In 2002, he became a delivery van driver for the
company.
• As a driver, he was paid P290 per trip regardless of the route. He was not given any additional or special pay
mandated by the Labor Code (vacation leave with pay, holiday pay, 13th month pay, service incentive pay, etc).
• On July 22, 2004 he drank beer with his coworkers after his shift ended for the day. He later returned to their
warehouse smelling like liquor.
• The incident was reported to Convoy’s logistics manager Arnold Laab who terminated Albia’s employment the very
next day, or on July 23, 2004.
• On July 26, 2004, Albia filed a complaint for illegal dismissal and nonpayment of wage benefits against Convoy
Marketing and Laab.
• Respondents contended that Albia was not an employee of the company but was only an independent contractor.
• To prove their contentions, respondents showed delivery service agreements and quitclaims signed by Albia saying
that he would never be considered an employee of Convoy and that once he received his wage he would no longer
be able to go after Convoy.
• Albia on the other hand argued that the quitclaims he signed are only for the money claims and not those arising
out of his employee-employer relationship with Convoy Marketing.
• July 24, 2006: Filing of complaint
• January 10, 2006: Labor Arbiter decided in favor of Convoy Marketing
• November 28, 2006: NLRC affirmed the decision of the Labor Arbiter
• May 31, 2010: CA reversed and set aside NLRC’s resolution
ISSUES:
1. Whether Albia is a regular employee of Convoy
2. Whether he was dismissed for a just cause
3. Whether the quitclaims he had signed are valid
HELD:
1. Yes. As stated in Article 280, ‘an employment shall be deemed to be regular where the employee has been
engaged to perform activities which are usually necessary or desirable in the usual business or trade of the employer’
and ‘any employee who has rendered at least one year of service, whether such service is continuous or broken,
shall be considered a regular employee’. Albia’s work was necessary in the usual course of Convoy’s business, and
his employment although broken lasted for more than one year.
Also, there is no question that Convoy controls or has reserved its right to control Albia’s conduct, not only as to the
result of his work but also as to the means and methods by which such result is to be accomplished.
2. No. “Misconduct is defined as the transgression of some established and definite rule of action, a forbidden act, a
dereliction of duty, willful in character, and implies wrongful “It is also clear that there was no valid grounds for the
termination of petitioner. His misconduct was not gross.
He was not guilty of any seriously offensive conduct, nor was there any untoward incident that occurred. The penalty
of dismissal was certainly not commensurate to the infraction committed.”
3. No. For a quitclaim to be valid, these requisites must be present:
1) the employee executes a deed of quitclaim voluntarily
2) there is no fraud or deceit on the part of any of the parties
3) the consideration of the quitclaim is credible and reasonable 4) the contract is not contrary to law, public order,
public policy, morals or good customs, or prejudicial to a third person with a right recognized by law.

Angeles| Bajana | Balladares | Brillantes | Briones | Cabansag | Callanta | Chua | David|


De Leon | Gomez | Lopez | Macalino | Nostratis | Padilla | Reynon | Santos | Tan |Velasco
4E / 4F - 2018-2019
Page 644 of 920
LABOR REVIEW DIGEST
Atty. Joyrich Golangco

Petitioners failed to prove that the consideration in their signed quitclaims was reasonable. Also, said quitclaims are
“contrary to law and public policy, as they preclude him from becoming a regular employee and acquiring tenurial
security”.
———————————————————————————————————————-

Angeles| Bajana | Balladares | Brillantes | Briones | Cabansag | Callanta | Chua | David|


De Leon | Gomez | Lopez | Macalino | Nostratis | Padilla | Reynon | Santos | Tan |Velasco
4E / 4F - 2018-2019
Page 645 of 920
LABOR REVIEW DIGEST
Atty. Joyrich Golangco

223. 24. JAMIAS vs NLRC


224. GR No: 159350 Date: March 9, 2016
225. Ponente: Bersamin, J.
226.
227. Doctrine:
228. A fixed term agreement, to be valid, must strictly conform with the requirements and conditions provided in
Article 280 of the Labor Code. The test to determine whether a particular employee is engaged as a project or regular
employee is whether or not the employee is assigned to carry out a specific project or undertaking, the duration or
scope of which was specified at the time of his engagement. There must be a determination of, or a clear agreement
on, the completion or termination of the project at the time the employee is engaged. Otherwise put, the fixed period
of employment must be knowingly and voluntarily agreed upon by the parties, without any force, duress or improper
pressure being brought to bear upon the employee and absent any other circumstances vitiating his consent, or it
must satisfactorily appear that the employer and employee dealt with each other on more or less equal terms with no
moral dominance whatsoever being exercised by the former on the latter.
229.
230. FACTS:
231. Respondent Innodata Philippines, Inc. (Innodata), a domestic corporation engaged in the business of data
processing and conversion for foreign clients, hired the petitioners on various dates. After their respective contracts
expired, petitioners filed a complaint for illegal dismissal claiming that Innodata had made it appear that they had
been hired as project employees in order to prevent them from becoming regular employees.
232.
233. LA/NLRC RULING:
234. Dismissed the complaint
235.
236. CA RULING:
237. Upheld NLRC
238.
239. Petitioner's Contention:
240. The petitioners argue that Innodata circumvented the security of tenure protected under Article 280 of
the Labor Code by providing a fixed term; and that they were regular employees because the work they performed
were necessary and desirable to the business of Innodata.
241. The petitioners posit that they should be accorded regular status because their work as editors and
proofreaders were usually necessary to Innodata's business of data processing.
242.
243. ISSUE/S:
244. Were the petitioners regular or project employees of Innodata?
245.
246. HELD: project employees
Article 280 (now 295) of the Labor Code contemplates three kinds of employees, namely: (a) regular employees; (b)
project employees; and (c) casuals who are neither regular nor project employees. The nature of employment of a
worker is determined by the factors provided in Article 280 of the Labor Code, regardless of any stipulation in the
contract to the contrary. Thus, in Brent School, Inc. v. Zamora, we explained that the clause referring to written
contracts should be construed to refer to agreements entered into for the purpose of circumventing the security of
tenure. Obviously, Article 280 does not preclude an agreement providing for a fixed term of employment knowingly
and voluntarily executed by the parties.

A fixed term agreement, to be valid, must strictly conform with the requirements and conditions provided in Article
280 of the Labor Code. The test to determine whether a particular employee is engaged as a project or regular
employee is whether or not the employee is assigned to carry out a specific project or undertaking, the duration or
scope of which was specified at the time of his engagement. There must be a determination of, or a clear agreement
on, the completion or termination of the project at the time the employee is engaged. Otherwise put, the fixed period
of employment must be knowingly and voluntarily agreed upon by the parties, without any force, duress or improper
pressure being brought to bear upon the employee and absent any other circumstances vitiating his consent, or it
must satisfactorily appear that the employer and employee dealt with each other on more or less equal terms with no
moral dominance whatsoever being exercised by the former on the latter.

The contracts of the petitioners indicated the one-year duration of their engagement as well as their respective
project assignments (i.e., Jamias being assigned to the CD-ROM project; Cruz and Matuguinas to the TSET
project). There is no indication that the petitioners were made to sign the contracts against their will. Neither did they
refute Innodata's assertion that it did not employ force, intimidate or fraudulently manipulate the petitioners into

Angeles| Bajana | Balladares | Brillantes | Briones | Cabansag | Callanta | Chua | David|


De Leon | Gomez | Lopez | Macalino | Nostratis | Padilla | Reynon | Santos | Tan |Velasco
4E / 4F - 2018-2019
Page 646 of 920
LABOR REVIEW DIGEST
Atty. Joyrich Golangco

signing their contracts and that the terms thereof had been explained and made known to them. Hence, the
petitioners knowingly agreed to the terms of and voluntarily signed their respective contracts.

That Innodata drafted the contracts with its business interest as the overriding consideration did not necessarily
warrant the holding that the contracts were prejudicial against the petitioners. The fixing by Innodata of the period
specified in the contracts of employment did not also indicate its ill-motive to circumvent the petitioners' security of
tenure. Indeed, the petitioners could not presume that the fixing of the one-year term was intended to evade or avoid
the protection to tenure under Article 280 of the Labor Code in the absence of other evidence establishing such
intention. This presumption must ordinarily be based on some aspect of the agreement other than the mere
specification of the fixed term of the employment agreement, or on evidence aliunde of the intent to evade.

Lastly, the petitioners posit that they should be accorded regular status because their work as editors and
proofreaders were usually necessary to Innodata's business of data processing.

We reject this position. For one, it would be unusual for a company like Innodata to undertake a project that had no
relationship to its usual business. Also, the necessity and desirability of the work performed by the employees are not
the determinants in term employment, but rather the "day certain" voluntarily agreed upon by the parties. As the CA
cogently observed in this respect:

There is proof to establish that Innodata's operations indeed rests upon job orders or undertakings coming from its
foreign clients. Apparently, its employees are assigned to projects - one batch may be given a fixed period of one
year, others, a slightly shorter duration, depending on the estimated time of completion of the particular job or
undertaking farmed out by the client to the company.

247. In fine, the employment of the petitioners who were engaged as project employees for a fixed term legally
ended upon the expiration of their contract. Their complaint for illegal dismissal was plainly lacking in merit.

Angeles| Bajana | Balladares | Brillantes | Briones | Cabansag | Callanta | Chua | David|


De Leon | Gomez | Lopez | Macalino | Nostratis | Padilla | Reynon | Santos | Tan |Velasco
4E / 4F - 2018-2019
Page 647 of 920
LABOR REVIEW DIGEST
Atty. Joyrich Golangco

25. Gadia v. Sykes Asia


G.R. No. 209499, January 28, 2015
Perlas-Bernabe J.

DOCTRINE:
Verily, for an employee to be considered project-based, the employer must show compliance with two (2) requisites,
namely that: (a) the employee was assigned to carry out a specific project or undertaking; and (b) the duration and
scope of which were specified at the time they were engaged for such project.

FACTS:
Sykes Asia, a BPO, provides support to its international clients by carrying on some of their operations, governed by
service contracts that it enters with them. Alltel Communications, Inc., a United States-based telecommunications
firm, contracted Sykes Asia’s services to accommodate the needs and demands of Alltel clients for its postpaid and
prepaid services. Thus, on different dates, Sykes Asia hired petitioners as customer service representatives, team
leaders, and trainers for the Alltel Project.
In 2009, Alltel terminated all support services provided by Sykes Asia related to the Alltel Project. Sykes Asia sent
each of the petitioners end-of-life notices, informing them of their dismissal from employment due to the termination
of the Alltel Project. Aggrieved, petitioners filed separate complaints for illegal dismissal.
LA RULING:
LA dismissed petitioners’ complaints for lack of merit. It found that petitioners are merely project-based employees.
Hence, the LA concluded that the cessation of the Alltel Project naturally resulted in the termination of petitioners’
employment in Sykes Asia.
NLRC RULING:
The NLRC ruled that petitioners are regular employees but were validly terminated due to redundancy. The NLRC
found that petitioners could not be properly characterized as project-based employees, ratiocinating that while it was
made known to petitioners that their employment would be co-terminus to the Alltel Project, it was neither determined
nor made known to petitioners, at the time of hiring, when the said project would end, be terminated, or be
completed. In this relation, the NLRC concluded that inasmuch as petitioners had been engaged to perform activities
which are necessary or desirable in respondents’ usual business or trade of BPO, petitioners should be deemed
regular employees of Sykes Asia. This notwithstanding, and in view of the cessation of the Alltel Project, the NLRC
found petitioners’ employment with Sykes Asia to be redundant; hence, declared that they were legally dismissed
from service and were only entitled to receive their respective separation pay.
CA RULING:
The CA annulled and set aside the ruling of the NLRC. It held that a perusal of petitioners’ respective employment
contracts readily shows that they were hired exclusively for the Alltel Project and that it was specifically stated therein
that their employment would be project-based.
ISSUE/S:
Whether or not the petitioners were merely project-based employees, and thus, validly dismissed from service.
HELD:
• In Omni Hauling Services, Inc. v. Bon, the Court extensively discussed how to determine whether an employee may
be properly deemed project-based or regular, to wit:

A project employee is assigned to a project which begins and ends at determined or determinable times. Unlike
regular employees who may only be dismissed for just and/or authorized causes under the Labor Code, the services
of employees who are hired as "project[-based] employees" may be lawfully terminated at the completion of the
project.

According to jurisprudence, the principal test for determining whether particular employees are properly characterised
as "project[-based] employees" as distinguished from "regular employees," is whether or not the employees were
assigned to carry out a "specific project or undertaking," the duration (and scope) of which were specified at the time
they were engaged for that project. The project could either be (1) a particular job or undertaking that is within the
regular or usual business of the employer company, but which is distinct and separate, and identifiable as such, from
the other undertakings of the company; or (2) a particular job or undertaking that is not within the regular business of
the corporation. In order to safeguard the rights of workers against the arbitrary use of the word "project" to prevent
employees from attaining a regular status, employers claiming that their workers are project[-based] employees
should not only prove that the duration and scope of the employment was specified at the time they were engaged,
but also, that there was indeed a project.

Angeles| Bajana | Balladares | Brillantes | Briones | Cabansag | Callanta | Chua | David|


De Leon | Gomez | Lopez | Macalino | Nostratis | Padilla | Reynon | Santos | Tan |Velasco
4E / 4F - 2018-2019
Page 648 of 920
LABOR REVIEW DIGEST
Atty. Joyrich Golangco

• Verily, for an employee to be considered project-based, the employer must show compliance with two (2) requisites,
namely that: (a) the employee was assigned to carry out a specific project or undertaking; and (b) the duration and
scope of which were specified at the time they were engaged for such project.

In this case, records reveal that Sykes Asia adequately informed petitioners of their employment status at the time of
their engagement, as evidenced by the latter’s employment contracts which similarly provide that they were
hired in connection with the Alltel Project, and that their positions were "project-based and as such is co-
terminus to the project." In this light, the CA correctly ruled that petitioners were indeed project-based
employees, considering that: (a) they were hired to carry out a specific undertaking, i.e., the Alltel Project;
and (b) the duration and scope of such project were made known to them at the time of their engagement,
i.e., "co-terminus with the project."

As regards the second requisite, the CA correctly stressed that "[t]he law and jurisprudence dictate that ‘the duration
of the undertaking begins and ends at determined or determinable times’" while clarifying that "[t]he phrase
‘determinable times’ simply means capable of being determined or fixed." In this case, Sykes Asia
substantially complied with this requisite when it expressly indicated in petitioners’ employment contracts
that their positions were "co-terminus with the project." To the mind of the Court, this caveat sufficiently
apprised petitioners that their security of tenure with Sykes Asia would only last as long as the Alltel Project
was subsisting. In other words, when the Alltel Project was terminated, petitioners no longer had any project
to work on, and hence, Sykes Asia may validly terminate them from employment. Further, the Court likewise
notes the fact that Sykes Asia duly submitted an Establishment Employment Report and an Establishment
Termination Report to the Department of Labor and Employment Makati-Pasay Field Office regarding the
cessation of the Alltel Project and the list of employees that would be affected by such cessation. As
correctly pointed out by the CA, case law deems such submission as an indication that the employment was
indeed project-based.

———————————————————————————————————————-

Angeles| Bajana | Balladares | Brillantes | Briones | Cabansag | Callanta | Chua | David|


De Leon | Gomez | Lopez | Macalino | Nostratis | Padilla | Reynon | Santos | Tan |Velasco
4E / 4F - 2018-2019
Page 649 of 920
LABOR REVIEW DIGEST
Atty. Joyrich Golangco

26. Innodata Knowledge Services, Inc. vs. Inting


GR No: 211892 Date: December 6, 2017
Ponente: PERALTA, J.

Doctrine: In order to safeguard the rights of workers against the arbitrary use of the word "project" which prevents
them from attaining regular status, employers claiming that their workers are project employees have the burden of
showing that: (a) the duration and scope of the employment was specified at the time they were engaged; and (b)
there was indeed a project.

FACTS:

Petitioner Innodata Knowledge Services, Inc. (IKSI) is a company engaged in data processing, encoding, indexing,
abstracting, typesetting, imaging, and other processes in the capture, conversion, and storage of data and
information. At one time, Applied Computer Technologies (ACT), a company based in the United States of America,
hired IKSI to review various litigation documents. Due to the nature of the job, ACT required IKSI to hire lawyers, or
at least, law graduates, to review various litigation documents, classify said documents into the prescribed
categories, and ensure that outputs are delivered on time. For this purpose, IKSI engaged the services of
respondents.

On January 7, 2010, however, respondents received a Notice of Forced Leave from IKSI informing them that they
shall be placed on indefinite forced leave effective that same day due to changes in business conditions, client
requirements, and specifications. Hence, respondents filed a complaint for illegal dismissal, reinstatement or
payment of separation pay, backwages, and damages against IKSI.

Subsequently, IKSI sent respondents separate notices dated May 27, 2010 informing them that due to the
unavailability of new work related to the product stream and uncertainties pertaining to the arrival of new workloads,
their project employment contracts would have to be terminated.

LA and NLRC declared that there was no Illegal Dismissal for they are only project employees. CA reversed the
decision.

ISSUE/S:
Whether or not the CA committed an error when it reversed the NLRC, which declared that respondent employees,
as mere project employees

HELD:

IKSI argued that based on the contract, it is undeniable that respondents’ employment was fixed for a specific project
or undertaking, with its completion or termination clearly determined at the time of the employee’s engagement.
Indeed, records would disclose that respondents signed employment contracts specifically indicating the Content
Supply Chain Project, also known as the ACT Project, as the project for which they were being hired, which was
expected to be completed after a maximum of five (5) years. However, sometime in November 2008, IKSI required
respondents to work on another project called "Bloomberg," which was not included in the original contracts that they
signed and without entering into a new project employment contracts. Such fact was never refuted by IKSI. During
that time, respondents were required to read and review decided cases in the United States of America and they
were no longer called Senior or Junior Reviewers, but referred to as Case Classifiers. Respondents initially opposed
working on said project but eventually agreed, in fear of losing their employment altogether. Months later, they were
again required to work on the ACT Project and reverted to their previous designation as Document Reviewers.

In the case of ALU-TUCP v. NLRC, the Court made a pronouncement on the two (2) categories of project
employees. The project for which project employees are hired would ordinarily have some relationship to the usual
business of the employer. There should be no difficulty in distinguishing the employees for a certain project from
ordinary or regular employees, as long as the duration and scope of the project were determined or specified at the
time of engagement of said project employees.

In order to safeguard the rights of workers against the arbitrary use of the word "project" which prevents them from
attaining regular status, employers claiming that their workers are project employees have the burden of showing
that: (a) the duration and scope of the employment was specified at the time they were engaged; and (b) there was
indeed a project. Therefore, as evident in Article 295, the litmus test for determining whether particular employees
are properly characterized as project employees, as distinguished from regular employees, is whether or not the

Angeles| Bajana | Balladares | Brillantes | Briones | Cabansag | Callanta | Chua | David|


De Leon | Gomez | Lopez | Macalino | Nostratis | Padilla | Reynon | Santos | Tan |Velasco
4E / 4F - 2018-2019
Page 650 of 920
LABOR REVIEW DIGEST
Atty. Joyrich Golangco

employees were assigned to carry out a specific project or undertaking, the duration and scope of which were
specified at the time the employees were engaged for that project.

Here, while IKSI was able to show the presence of a specific project, the ACT Project, in the contract and the alleged
duration of the same, it failed to prove, however, that respondents were in reality made to work only for that specific
project indicated in their employment documents and that it adequately informed them of the duration and scope of
said project at the time their services were engaged. It is well settled that a party alleging a critical fact must support
his allegation with substantial evidence, as allegation is not evidence. The fact is IKSI actually hired respondents to
work, not only on the ACT Project, but on other similar projects such as the Bloomberg. When respondents were
required to work on the Bloomberg project, without signing a new contract for that purpose, it was already outside of
the scope of the particular undertaking for which they were hired; it was beyond the scope of their employment
contracts. The fact that the same happened only once is inconsequential. What matters is that IKSI required
respondents to work on a project which was separate and distinct from the one they had signed up for. This act by
IKSI indubitably brought respondents outside the realm of the project employees category.

IKSI likewise fell short in proving that the duration of the project was reasonably determinable at the time
respondents were hired. As earlier mentioned, the employment contracts provided for "the duration of the Project,
which is expected to be completed after a maximum of five (5) years, or on or before"The NLRC upheld the same,
finding that the contracts clearly provided for the duration of the project which was expected to end after a maximum
of five (5) years, or on or before July 2, 2013. It is interesting to note, however, that the five (5)-year period is not
actually the duration of the project but merely that of the employment contract. Naturally, therefore, not all of
respondents' employment would end on July 2, 2013, as the completion of the five (5)-year period would depend on
when each employee was employed. This is precisely the reason why IKSI originally left a blank for the termination
date because it varied for each employee. If respondents were truly project employees, as IKSI claims and as found
by the NLRC, then the termination date would have been uniform for all of them.

Thus, while the CA erred in simply relying on the Court's rulings on previous cases involving Innodata Phils., Inc.
since there is no substantial proof that Innodata Phils., Inc. and herein petitioner, IKSI, are one and the same entity, it
would appear, however, that respondents indeed entered into fixed-term employment contracts with IKSI, contracts
with a fixed period of five (5) years. But project employment and fixed-term employment are not the same. While the
former requires a particular project, the duration of a fixed-term employment agreed upon by the parties may be any
day certain, which is understood to be "that which must necessarily come although it may not be known when." The
decisive determinant in fixed-term employment is not the activity that the employee is called upon to perform but the
day certain agreed upon by the parties for the commencement and termination of the employment relationship.

———————————————————————————————————————-

Angeles| Bajana | Balladares | Brillantes | Briones | Cabansag | Callanta | Chua | David|


De Leon | Gomez | Lopez | Macalino | Nostratis | Padilla | Reynon | Santos | Tan |Velasco
4E / 4F - 2018-2019
Page 651 of 920
LABOR REVIEW DIGEST
Atty. Joyrich Golangco

Management Prerogative

1. ZUELLIG FREIGHT AND CARGO SYSTEMS, vs. NATIONAL LABOR RELATIONS COMMISSION AND
RONALDO V. SAN MIGUEL
GR No. 157900 July 22, 2013
Ponente: Bersamin, J.

Doctrine: Article 283. “Closure of establishment and reduction of personnel. — The employer may also
terminate the employment of any employee due to the installation of labor-saving devices, redundancy,
retrenchment to prevent losses or the closing or cessation of operation of the establishment or undertaking
unless the closing is for the purpose of circumventing the provisions of this Title.”

FACTS:
Ronaldo San Miguel brought a complaint for unfair labor practice, illegal dismissal, non-payment of salaries
and moral damages against Zuellig Freight and Cargo Systems, formerly known as Zeta Brokerage
Corporation (Zeta). San Miguel alleged that he had been a checker/customs representative of Zeta since
December 16, 1985; that in January 1994, he and other employees of Zeta were informed that Zeta would
cease operations, and that all affected employees, including him, would be separated; Zeta informed him of
his termination. San Miguel contended that the amendments of the articles of incorporation of Zeta were for
the purpose of changing the corporate name, broadening the primary functions, and increasing the capital
stock; and that such amendments could not mean that Zeta had been thereby dissolved. Zeta countered
that San Miguel’s termination from Zeta had been for a cause authorized by the Labor Code; that its non-
acceptance of him had not been by any means irregular or discriminatory; that it had no obligation to
employ San Miguel in the exercise of its valid management prerogative.

LA RULING: San Miguel illegally dismissed. LA believes that there was merely a change of business name
and primary purpose and upgrading of stocks of the corporation. Zuellig and Zeta are therefore legally the
same person and entity.

CA RULING: There being no valid closure of business operations, the dismissal of private respondent San
Miguel on alleged authorized cause of cessation of business pursuant to Article 283 of the Labor Code,
was utterly illegal.

PETITION TO THE SC:

ISSUE/S:
1. WON the cessation of business by Zeta was a bona fide closure to be regarded as a valid ground for
the termination of employment of San Miguel

HELD:
NO.
Article 283. Closure of establishment and reduction of personnel. — The employer may also terminate the
employment of any employee due to the installation of labor-saving devices, redundancy, retrenchment to
prevent losses or the closing or cessation of operation of the establishment or undertaking unless the
closing is for the purpose of circumventing the provisions of this Title, x x x.
The amendments of the articles of incorporation of Zeta to change the corporate name to Zuellig Freight
and Cargo Systems, Inc. did not produce the dissolution of the former as a corporation. For sure, the
Corporation Code defined and delineated the different modes of dissolving a corporation, and amendment of
the articles of incorporation was not one of such modes. Zeta and petitioner remained one and the same
corporation. The change of name did not give petitioner the license to terminate employees of Zeta like
San Miguel without just or authorized cause.
Petitioner, despite its new name, was the mere continuation of Zeta's corporate being, and still held the
obligation to honor all of Zeta's obligations, one of which was to respect San Miguel's security of tenure.
The dismissal of San Miguel from employment on the pretext that petitioner, being a different corporation,
had no obligation to accept him as its employee, was illegal and ineffectual

———————————————————————————————————————-

Angeles| Bajana | Balladares | Brillantes | Briones | Cabansag | Callanta | Chua | David|


De Leon | Gomez | Lopez | Macalino | Nostratis | Padilla | Reynon | Santos | Tan |Velasco
4E / 4F - 2018-2019
Page 652 of 920
LABOR REVIEW DIGEST
Atty. Joyrich Golangco

2. Peckson v. Robinsons Supermarket Inc


Doctrine: It is the employer’s prerogative, based on its assessment and perception of its employees’ qualifications,
aptitudes, and competence, to move them around in the various areas of its business operations in order to ascertain
where they will function with maximum benefit to the company. An employee’s right to security of tenure does not
give him such a vested right in his position as would deprive the company of its prerogative to change his assignment
or transfer him where he will be most useful.
FACTS: Jenny Peckson first joined the Robinsons Supermarket Corporation (RSC) as a Sales Clerk holding the
position of Category Buyer when respondent Roena Sarte Assistant Vice-President, reassigned her to the position of
Provincial Coordinator. Claiming that her new assignment was a demotion because it was non- supervisory and
clerical in nature, the petitioner refused to turn over her responsibilities to the new Category Buyer, or to accept her
new responsibilities as Provincial Coordinator.
RSC, through Sarte, demanded an explanation from her for her refusal to accept her new assignment despite written
and verbal demands. Sarte cited a company rule, Offenses Subject to Disciplinary Action No. 4.07, which provided
that "[d]isobedience, refusal or failure to do assigned task or to obey superior’s/official’s orders/instructions, or to
follow established procedures or practices without valid reason" would be meted the penalty of suspension.
The petitioner ignored the 48-hour deadline to explain. Peckson stated in reply that she could not accept the position
of Provincial Coordinator since she saw it as a demotion. As it turned out, however, Peckson had already filed a
complaint for constructive dismissal.
Several tasks were asked of Peckson by Sarte which Peckson refused to heed.
Peckson argued before the LA that the true organizational chart of the RSC showed that the position of Category
Buyer was one level above that of the Provincial Coordinator.
The respondents in their position paper denied the correctness of the organizational chart presented by the
petitioner. They maintained that her transfer was not a demotion since the Provincial Coordinator occupied a "Level
5" position like the Category Buyer, with the same work conditions, salary and benefits. The respondents also raised
the petitioner’s record of habitual tardiness as far back as 1999, as well as poor performance rating. Also, that the
petitioner was suspended for seven (7) days in September and October 2005 for deliberately violating a company
policy after she was seen having lunch with a company supplier.
Sarte denied that the reassignment of the petitioner as Provincial Coordinator was motivated by a desire to besmirch
the name of the latter. She asserted that it was made in the exercise of management prerogative and sound
discretion.
LA: ruled that job reassignment or classification is a strict prerogative of the employer, and that the petitioner cannot
refuse her transfer since both positions commanded the same salary structure, high degree of responsibility and
impeccable honesty and integrity. LA found no showing of any illegal motive on the part of the respondents in
reassigning the petitioner.
A month after the above ruling, Peckson tendered her written "forced" resignation, wherein she complained that she
was being subjected to ridicule by clients and co-employees alike on account of her floating status since the time she
refused to accept her transfer. She likewise claimed that she was being compelled to accept the position of Provincial
Coordinator without due process.
ISSUE: WON the reassignment of Peckson amounted to demotion thus it was not a valid exercise of management
prerogative.
HELD: VALID exercise of management prerogative. This Court has consistently refused to interfere with the exercise
by management of its prerogative to regulate the employees’ work assignments, the working methods and the place
and manner of work.
Labor laws discourage interference with an employer’s judgment in the conduct of his business.
Under the doctrine of management prerogative, every employer has the inherent right to regulate, according to his
own discretion and judgment, all aspects of employment, including hiring, work assignments, working methods, the
time, place and manner of work, work supervision, transfer of employees, lay-off of workers, and discipline, dismissal,
and recall of employees. The only limitations to the exercise of this prerogative are those imposed by labor laws and
the principles of equity and substantial justice.
While the law imposes many obligations upon the employer, nonetheless, it also protects the employer’s right to
expect from its employees not only good performance, adequate work, and diligence, but also good conduct and
loyalty.
(Rural Bank of Cantila Inc v. Julve Jurisprudential Guidelines)
Concerning the transfer of employees, these are the following jurisprudential guidelines:
 (a) a transfer is a movement from one position to another of equivalent rank, level or salary without break in the
service or a lateral movement from one position to another of equivalent rank or salary;

 (b) the employer has the inherent right to transfer or reassign an employee for legitimate business purposes;

 (c) a transfer becomes unlawful where it is motivated by discrimination or bad faith or is effected as a form of

Angeles| Bajana | Balladares | Brillantes | Briones | Cabansag | Callanta | Chua | David|


De Leon | Gomez | Lopez | Macalino | Nostratis | Padilla | Reynon | Santos | Tan |Velasco
4E / 4F - 2018-2019
Page 653 of 920
LABOR REVIEW DIGEST
Atty. Joyrich Golangco

punishment or is a demotion without sufficient cause;

 (d) the employer must be able to show that the transfer is not unreasonable, inconvenient, or prejudicial to the
employee.

It is the employer’s prerogative, based on its assessment and perception of its employees’ qualifications, aptitudes,
and competence, to move them around in the various areas of its business operations in order to ascertain where
they will function with maximum benefit to the company. An employee’s right to security of tenure does not give him
such a vested right in his position as would deprive the company of its prerogative to change his assignment or
transfer him where he will be most useful.
We agree with the appellate court that the respondents are justified in moving the petitioner to another equivalent
position, which presumably would be less affected by her habitual tardiness or inconsistent attendance than if she
continued as a Category Buyer, a "frontline position" in the day-to-day business operations of a supermarket such as
Robinsons.
The managerial prerogative to transfer personnel must be exercised without grave abuse of discretion, bearing in
mind the basic elements of justice and fair play. Having the right should not be confused with the manner in which
that right is exercised. Thus, it cannot be used as a subterfuge by the employer to rid himself of an undesirable
worker.
There is substantial showing that the transfer of the petitioner from Category Buyer to Provincial Coordinator was not
unreasonable, inconvenient, or prejudicial to her. The petitioner failed to dispute that the job classifications of
Category Buyer and Provincial Coordinator are similar, or that they command a similar salary structure and
responsibilities. We agree with the NLRC that the Provincial Coordinator’s position does not involve mere clerical
functions but requires the exercise of discretion from time to time, as well as independent judgment, since the
Provincial Coordinator gives appropriate recommendations to management and ensures the faithful implementation
of policies and programs of the company. It even has influence over a Category Buyer because of its
recommendatory function that enables the Category Buyer to make right decisions on assortment, price and quantity
of the items to be sold by the store.
We also cannot sustain the petitioner’s claim that she was not accorded due process. In addition to verbal reminders
from Sarte, the petitioner was asked in writing twice to explain within 48 hours her refusal to accept her transfer. To
her discredit, her defiance constituted a neglect of duty, or an act of insubordination, per the LA.
———————————————————————————————————————-

Angeles| Bajana | Balladares | Brillantes | Briones | Cabansag | Callanta | Chua | David|


De Leon | Gomez | Lopez | Macalino | Nostratis | Padilla | Reynon | Santos | Tan |Velasco
4E / 4F - 2018-2019
Page 654 of 920
LABOR REVIEW DIGEST
Atty. Joyrich Golangco

3. RENATO GATBONTON vs. NLRC, MAPUA INSTITUTE OF TECHNOLOGY and JOSE CALDERON
G.R. No. 146779 January 23, 2006
Austria-Martinez, J.

DOCTRINE: Preventive suspension is a disciplinary measure for the protection of the company’s property pending
investigation of any alleged malfeasance or misfeasance committed by the employee. The employer may place the
worker concerned under preventive suspension if his continued employment poses a serious and imminent threat to
the life or property of the employer or of his co-workers. However, when it is determined that there is no sufficient
basis to justify an employee’s preventive suspension; the latter is entitled to the payment of salaries during the time
of preventive suspension.
FACTS: Renato Gatbonton is an associate professor of respondent Mapua Institute of Technology (MIT), Faculty of
Civil Engineering. A civil engineering student of respondent MIT filed a letter-complaint against petitioner for
unfair/unjust grading system, sexual harassment and conduct unbecoming of an academician. MIT, through its
Committee on Decorum and Investigation placed Petitioner under a 30-day Preventive Suspension. The Committee
believed that Petitioner’s continued stay during the investigation affects his performance as a faculty member, and as
well as the students learning; and that the suspension will allow petitioner to prepare himself for the investigation and
will prevent his influences to other members of the community.
Petitioner filed with the NLRC a complaint for illegal suspension, damages and attorney’s fees.
LA RULING: The LA declared the Preventive Suspension against the Petitioner as illegal. Respondents MIT are
directed to pay his wages during the period of his preventive suspension.
Both respondents and petitioner filed their appeal from the Labor Arbiters Decision, with petitioner questioning the
dismissal of his claim for damages.
NLRC RULING: The NLRC granted respondents appeal and set aside the Labor Arbiters decision. His motion for
reconsideration having been denied by the NLRC on December 13, 1999, petitioner filed a special civil action
for certiorari with the CA.
CA RULING: The CA affirmed the NLRC’s decision.
Hence, Petitioner filed a Motion for Reconsideration.
PETITIONER’S CONTENTION: Petitioner contends that his preventive suspension does not find any justification in
the Mapua Rules and Regulations considering that at the time of his suspension, the rules have not yet been
promulgated yet.
ISSUE: Whether or not the imposition of the 30-day preventive suspension is legal.
HELD: NO. The 30-day preventive suspension is illegal.
Preventive suspension is a disciplinary measure for the protection of the company’s property pending investigation of
any alleged malfeasance or misfeasance committed by the employee. The employer may place the worker
concerned under preventive suspension if his continued employment poses a serious and imminent threat to the life
or property of the employer or of his co-workers. However, when it is determined that there is no sufficient basis to
justify an employee’s preventive suspension; the latter is entitled to the payment of salaries during the time of
preventive suspension.
Under the Labor Code, petitioner’s preventive suspension finds no valid justification. As provided in Section 8, Rule
XXIII, Book V of the Omnibus Rules Implementing the Labor Code:
Sec. 8. Preventive Suspension. The employer may place the worker concerned under preventive suspension if his
continued employment poses a serious threat to the life or property of the employer or of his co-workers.
There is nothing on record which shows that respondent MIT imposed the preventive suspension on petitioner as his
continued employment poses a serious threat to the life or property of the employer or of his coworkers; therefore, his
preventive suspension is not justified.
The Mapua Rules is one of those issuances that should be published for its effectivity, since its purpose is to enforce
and implement R.A. No. 7877, which is a law of general application.[14] In fact, the Mapua Rules itself explicitly
required publication of the rules for its effectivity, as provided in Section 3, Rule IV (Administrative Provisions), which
states that [T]hese Rules and Regulations to implement the Anti-Sexual Harassment Act of 1995 shall take effect
fifteen (15) days after publication by the Committee. Thus, at the time of the imposition of petitioners preventive
suspension on January 11, 1999, the Mapua Rules were not yet legally effective, and therefore the suspension had
no legal basis.
Moreover, even assuming that the Mapua Rules are applicable, the Court finds that there is no sufficient basis to
justify his preventive suspension. Under the Mapua Rules, an accused may be placed under preventive suspension
during pendency of the hearing under any of the following circumstances:

(a) if the evidence of his guilt is strong and the school head is morally convinced that the continued stay of the
accused during the period of investigation constitutes a distraction to the normal operations of the institution; or

(b) the accused poses a risk or danger to the life or property of the other members of the educational community.

Angeles| Bajana | Balladares | Brillantes | Briones | Cabansag | Callanta | Chua | David|


De Leon | Gomez | Lopez | Macalino | Nostratis | Padilla | Reynon | Santos | Tan |Velasco
4E / 4F - 2018-2019
Page 655 of 920
LABOR REVIEW DIGEST
Atty. Joyrich Golangco

In petitioners case, there is no indication that petitioners preventive suspension may be based on the foregoing
circumstances. The Committee resolution does not show that evidence of petitioners guilt is strong and that the
school head is morally convinced that petitioners continued stay during the period of investigation constitutes a
distraction to the normal operations of the institution; or that petitioner poses a risk or danger to the life or property of
the other members of the educational community.

———————————————————————————————————————-

Angeles| Bajana | Balladares | Brillantes | Briones | Cabansag | Callanta | Chua | David|


De Leon | Gomez | Lopez | Macalino | Nostratis | Padilla | Reynon | Santos | Tan |Velasco
4E / 4F - 2018-2019
Page 656 of 920
LABOR REVIEW DIGEST
Atty. Joyrich Golangco

Article 296

1. Tamson’s Enterprises Inc v. CA


G.R. No. 192881 November 16, 2011

DOCTRINE: The law is clear that in all cases of probationary employment, the employer shall make known to the
employee the standards under which he will qualify as a regular employee at the time of his engagement. Where no
standards are made known to the employee at that time, he shall be deemed a regular employee.

Facts:
On February 24, 2007, four days before she completed her sixth month of working in Tamsons, Ng, the
Sales Project Manager, called her to a meeting with him and Lee. During the meeting, they informed Sy that her
services would be terminated due to inefficiency. She was asked to sign a letter of resignation and quitclaim. She
was told not to report for work anymore because her services were no longer needed.
Sy to file a case for illegal dismissal with claims for back wages, unpaid salary, service incentive leave,
overtime pay, 13th month pay, and moral and exemplary damages, and attorneys fees.
Executive Labor Arbiter rendered a decision in favor of Sy, stating that a termination, notwithstanding the
probationary status, must be for a just cause.
On appeal, the NLRC reversed the ELAs finding that Sy was terminated without just cause and without due
process and dismissed the case.
CA agreed with the ELAs conclusion that the termination of Sys services was illegal as there was no
evidence that a standard of performance had been made known to her and that she was accorded due process.

ISSUE: Whether the respondent is a probationary employee.

HELD:
The Court finds the petition devoid of merit.
The pertinent law governing the present case is Article 281 of the Labor Code which provides as follows:
Art. 281. Probationary employment. Probationary employment shall not exceed six months from the date the
employee started working, unless it is covered by an apprenticeship agreement stipulating a longer period. The
services of an employee who has been engaged in a probationary basis may be terminated for a just cause or when
he fails to qualify as a regular employee in accordance with reasonable standards made known by the employer to
the employee at the time of his engagement. An employee who is allowed to work after a probationary period shall be
considered a regular employee.
There is probationary employment where the employee upon his engagement is made to undergo a trial
period during which the employer determines his fitness to qualify for regular employment based on reasonable
standards made known to him at the time of engagement. The probationary employment is intended to afford the
employer an opportunity to observe the fitness of a probationary employee while at work, and to ascertain whether he
will become an efficient and productive employee. While the employer observes the fitness, propriety and efficiency
of a probationer to ascertain whether he is qualified for permanent employment, the probationer, on the other hand,
seeks to prove to the employer that he has the qualifications to meet the reasonable standards for permanent
employment. Thus, the word probationary, as used to describe the period of employment, implies the purpose of the
term or period, not its length.
On the basis of the aforequoted provisions and definition, there is no dispute that Sys employment with
Tamsons on September 1, 2006 was probationary in character. As a probationary employee, her employment status
was only temporary. Although a probationary or temporary employee with a limited tenure, she was still entitled to a
security of tenure.
It is settled that even if probationary employees do not enjoy permanent status, they are accorded the
constitutional protection of security of tenure. This means they may only be terminated for a just cause or when they
otherwise fail to qualify as regular employees in accordance with reasonable standards made known to them by the
employer at the time of their engagement. Consistently, in Mercado v. AMA Computer College-Paranaque City, Inc.,
this Court clearly stressed that:
Labor, for its part, is given the protection during the probationary period of knowing the company standards
the new hires have to meet during the probationary period, and to be judged on the basis of these standards, aside
from the usual standards applicable to employees after they achieve permanent status. Under the terms of the Labor
Code, these standards should be made known to the [employees] on probationary status at the start of their
probationary period, or xxx during which the probationary standards are to be applied. Of critical importance in
invoking a failure to meet the probationary standards, is that the [employer] should show as a matter of due process
how these standards have been applied. This is effectively the second notice in a dismissal situation that the law
requires as a due process guarantee supporting the security of tenure provision, and is in furtherance, too, of the

Angeles| Bajana | Balladares | Brillantes | Briones | Cabansag | Callanta | Chua | David|


De Leon | Gomez | Lopez | Macalino | Nostratis | Padilla | Reynon | Santos | Tan |Velasco
4E / 4F - 2018-2019
Page 657 of 920
LABOR REVIEW DIGEST
Atty. Joyrich Golangco

basic rule in employee dismissal that the employer carries the burden of justifying a dismissal. These rules ensure
compliance with the limited security of tenure guarantee the law extends to probationary employees.
Indeed, the Court recognizes the employers power to terminate as an exercise of management prerogative.
The petitioners, however, must be reminded that such right is not without limitations. In this connection, it is well to
quote the ruling of the Court in the case of Dusit Hotel Nikko v. Gatbonton, where it was written:
As Article 281 clearly states, a probationary employee can be legally terminated either: (1) for a just cause;
or (2) when the employee fails to qualify as a regular employee in accordance with the reasonable standards made
known to him by the employer at the start of the employment. Nonetheless, the power of the employer to terminate
an employee on probation is not without limitations. First, this power must be exercised in accordance with the
specific requirements of the contract. Second, the dissatisfaction on the part of the employer must be real and in
good faith, not feigned so as to circumvent the contract or the law; and third, there must be no unlawful discrimination
in the dismissal. In termination cases, the burden of proving just or valid cause for dismissing an employee rests on
the employer.
Here, the petitioners failed to convey to Sy the standards upon which she should measure up to be
considered for regularization and how the standards had been applied in her case. As correctly pointed out by Sy, the
dissatisfaction on the part of the petitioners was at best self-serving and dubious as they could not present concrete
and competent evidence establishing her alleged incompetence. Failure on the part of the petitioners to discharge
the burden of proof is indicative that the dismissal was not justified.
The law is clear that in all cases of probationary employment, the employer shall make known to the
employee the standards under which he will qualify as a regular employee at the time of his engagement. Where no
standards are made known to the employee at that time, he shall be deemed a regular employee. The standards
under which she would qualify as a regular employee not having been communicated to her at the start of her
probationary period, Sy qualified as a regular employee. As held by this Court in the very recent case of Hacienda
Primera Development Corporation v. Villegas,:
In this case, petitioner Hacienda fails to specify the reasonable standards by which respondents alleged
poor performance was evaluated, much less to prove that such standards were made known to him at the start of his
employment. Thus, he is deemed to have been hired from day one as a regular employee. Due process dictates that
an employee be apprised beforehand of the condition of his employment and of the terms of advancement therein.
Even on the assumption that Sy indeed failed to meet the standards set by them and made known to the
former at the time of her engagement, still, the termination was flawed for failure to give the required notice to Sy.
Section 2, Rule I, Book VI of the Implementing Rules provides:
Section 2. Security of tenure. (a) In cases of regular employment, the employer shall not terminate the
services of an employee except for just or authorized causes as provided by law, and subject to the requirements of
due process.
(b) The foregoing shall also apply in cases of probationary employment; Provided however, that in such
cases, termination of employment due to failure of the employee to qualify in accordance with the standards of the
employer made known to the former at the time of engagement may also be a ground for termination of employment.
xxx
(d) In all cases of termination of employment, the following standards of due process shall be substantially
observed:
xxx
If the termination is brought about by the completion of a contract or phase thereof, or by failure of an
employee to meet the standards of the employer in the case of probationary employment, it shall be sufficient that a
written notice is served the employee, within a reasonable time from the effective date of termination.
Being a regular employee whose termination was illegal, Sy is entitled to the twin relief of reinstatement and
backwages granted by the Labor Code. Article 279 provides that an employee who is unjustly dismissed from work
shall be entitled to reinstatement without loss of seniority rights and other privileges, to her full backwages, inclusive
of allowances, and to her other benefits or their monetary equivalent computed from the time her compensation was
withheld from her up to the time of actual reinstatement.

———————————————————————————————————————-

Angeles| Bajana | Balladares | Brillantes | Briones | Cabansag | Callanta | Chua | David|


De Leon | Gomez | Lopez | Macalino | Nostratis | Padilla | Reynon | Santos | Tan |Velasco
4E / 4F - 2018-2019
Page 658 of 920
LABOR REVIEW DIGEST
Atty. Joyrich Golangco

2. Hacienda Primera Development Corporation v. Michael Villegas


G.R. No. 186243; Apr. 11, 2011
Nachura, J.:
81. DOCTRINE: In all cases of probationary employment, the employer shall make known to the employee the
standards under which he will qualify as a regular employee at the time of his engagement. Where no standards are
made known to the employee at that time, he shall be deemed a regular employee. Failure to make known the
standard to be met by the probationary employee shall make such an employee a regular employee from the day of
hiring.
FACTS:

Petitioner Hacienda Primera Development Corporation (petitioner Hacienda) hired respondent Michael S. Villegas as
General Manager of Amorita Resort. He was hired as a probationary employee for three (3) months.

Reposndent started working on Jan. 1, 2007, but on Mar. 4, 2007 he received a call from Paramount Consultancy
and Management telling him to report back to Manila. There he was informed that his services had been terminated.
He requested a written notice of termination but none was given, prompting Villegas to file an action for Illegal
Dismissal before the Labor Arbiter.

Primera for its part explained that respondent's services were terminated because he failed to qualify for regular
employment. Specifically, it claimed that respondent failed to conceptualize and complete financial budgets, sales
projection, room rates, website development, and marketing plan in coordination with the Sales and Marketing
Manager.

LA RULING: Ordered the reinstatement of Villegas.

NLRC RULING: Reversed the LA and modified its judgment to remove the order for reinstatement, and ordering
instead that the expired portion of his contract be paid him.

CA RULING: Reinstated the Ruling of the LA but without an order for reinstatement noting the strained relations
between the parties, and awarded separation pay be paid instead.

APPEAL TO THE SC:


Petitioner’s Contention: Respondent was legally dismissed.

ISSUE:. Whether or not Villegas was illegally dismissed for failing to meet the standards of employment for
regularization.

82. RULING: YES. The dismissal of a probationary employee presupposes that the standards to be met were
made known to the employee. Probationary Employment is governed by the Labor Code together with its
Implementing Rules and Regulations.
83.
84. LABOR CODE, Implementing Rules of Book VI, Rule I, Section 6
85.
86. “Sec. 6. Probationary employment. — There is probationary employment where the
employee, upon his engagement, is made to undergo a trial period during which the employer determines
his ?tness to qualify for regular employment, based on reasonable standards made known to him at the time
of engagement.
87. Probationary employment shall be governed by the following rules:
88. xxx xxx xxx
89. (c)
90. The services of an employee who has been engaged on probationary basis may be terminated
only for a just or authorized cause, when he fails to qualify as a regular employee in accordance with the
reasonable standards prescribed by the employer.
91. (d) In all cases of probationary employment, the employer shall make known to the employee the
standards under which he will qualify as a regular employee at the time of his engagement. Where no
standards are made known to the employee at that time, he shall be deemed a regular employee.”
92. Thus, there are two grounds to legally terminate a probationary employee. It may be done either: a) for a
just cause; or b) when the employee fails to qualify as a regular employee in accordance with reasonable standards
made known by the employer to the employee at the start of the employment.

Angeles| Bajana | Balladares | Brillantes | Briones | Cabansag | Callanta | Chua | David|


De Leon | Gomez | Lopez | Macalino | Nostratis | Padilla | Reynon | Santos | Tan |Velasco
4E / 4F - 2018-2019
Page 659 of 920
LABOR REVIEW DIGEST
Atty. Joyrich Golangco

93.
94. In this case, petitioner Hacienda fails to specify the reasonable standards by which respondent's alleged
poor performance was evaluated, much less to prove that such standards were made known to him at the start of his
employment. Thus, he is deemed to have been hired from day one as a regular employee.
———————————————————————————————————————-

Angeles| Bajana | Balladares | Brillantes | Briones | Cabansag | Callanta | Chua | David|


De Leon | Gomez | Lopez | Macalino | Nostratis | Padilla | Reynon | Santos | Tan |Velasco
4E / 4F - 2018-2019
Page 660 of 920
LABOR REVIEW DIGEST
Atty. Joyrich Golangco

3. UNIVERSIDAD DE STA. ISABEL , petitioner, vs. MARVIN-JULIAN L.


SAMBAJON, JR., respondent.
GR No: 196280 & 196286 Date: April 2, 2014
Ponente: Villarama, J.

Doctrine: Notwithstanding the limited engagement of probationary employees, they are entitled to constitutional
protection of security of tenure during and before the end of the probationary period.

FACTS:
Universidad de Sta. Isabel (petitioner) is a non-stock, non-profit religious educational institution in Naga City.
Petitioner hired Marvin-Julian L. Sambajon, Jr. (respondent) as a full-time college faculty member with the rank of
Assistant Professor on probationary status, as evidenced by an Appointment Contract4 dated November 1, 2002,
effective November 1, 2002 up to March 30, 2003. After the aforesaid contract expired, petitioner continued to give
teaching loads to respondent who remained a full-time faculty member of the Department of Religious Education for
the two semesters of school-year (SY) 2003-2004 (June 1, 2003 to March 31, 2004); and two semesters of SY 2004-
2005 (June 2004 to March 31, 2005).5 Sometime in June 2003, respondent submitted his master’s degree for the
purpose of salary adjustment/increase. Subsequently, respondent’s salary was increased, as reflected in his pay
slips starting October 1-15, 2004.
In a letter dated October 15, 2004 addressed to the President of petitioner, Sr. Ma. Asuncion G. Evidente, D.C.,
respondent vigorously argued that his salary increase should be made effective as of June 2003 and demanded the
payment of his salary differential. The school replied that The Faculty Manual as revised for school year 2002-2003
provides (page 38) "Reranking is done every two years, hence the personnel hold their present rank for two years. It
also argued that personnel file shows that respondent were hired as a probationary teacher in the second semester
of school year 2002-2003. By October 2004, he will be completing four (4) semesters (two school years) of service.
Even permanent teachers are re-ranked only every two years, and he is not even a permanent teacher.
On Feb 26, 2005 respondent received a letter informing him that the school would no longer renew his probationary
appointment when it expires at the end of March 2005. On April 14, 2005, respondent filed a case for illegal dismissal
with the Labor Arbiter.

LA/RTC/NLRC RULING:
LA: Found petitioner guilty of illegal dismissal stating that there was no just or authorized cause in the termination of
respondent’s probationary employment. Petitioner appealed the decision to the NLRC.

NLRC: Affirmed the decision holding that respondent has acquired permanent status pursuant to Sections 91, 92 and
93 of the 1992 Manual of Regulations for Private Schools, in relation to Article 281 of the Labor Code, as amended.
Petitioner and respondent sought reconsideration, with the former contending that the NLRC resolved an issue not
raised in the appeal memorandum, while the latter asserted that the NLRC erred in not awarding him full back wages
so as to conform to the finding that he had acquired a permanent status. Both motions were denied by the NLRC
which ruled that regardless of whether or not the parties were aware of the rules for the acquisition of permanent
status by private school teachers, these rules applied to them and overrode their mistaken beliefs. As to respondent's
plea for back wages, the NLRC said the award of back wages was not done in this case because respondent did not
appeal the Labor Arbiter's decision.
CA RULING:
CA sustained the ruling of the NLRC but found it necessary to include the award of backwages to respondent.

APPEAL TO THE SC:

Petitioner's Contention: Petitioner ascribes grave error on the part of the CA in sustaining the NLRC which ruled that
respondent was dismissed without just or authorized cause at the time he had already acquired permanent or regular
status since petitioner allowed him to continue teaching despite the expiration of the first contract of probationary
employment for the second semester of SY 2002-2003. Petitioner at the outset underscores the fact that the NLRC
decided an issue which was not raised on appeal, i.e., whether respondent had attained regular status. It points out
that the Labor Arbiter's finding that respondent was dismissed while still a probationary employee was not appealed
by him, and hence such finding had already become final.
ISSUE/S:
1. Whether respondent's probationary employment was validly terminated by petitioner.
2. Whether the NLRC correctly resolved an issue not raised in petitioner's appeal memorandum

HELD:
1. NO, the petition was partly granted by the SC.

Angeles| Bajana | Balladares | Brillantes | Briones | Cabansag | Callanta | Chua | David|


De Leon | Gomez | Lopez | Macalino | Nostratis | Padilla | Reynon | Santos | Tan |Velasco
4E / 4F - 2018-2019
Page 661 of 920
LABOR REVIEW DIGEST
Atty. Joyrich Golangco

A probationary employee is one who is on trial by the employer during which the employer determines whether or not
said employee is qualified for permanent employment. A probationary appointment is made to afford the employer an
opportunity to observe the fitness of a probationary employee while at work, and to ascertain whether he will become
a proper and efficient employee. The word probationary as used to describe the period of employment implies the
purpose of the term or period, but not its length
ART. 281(now 296) Probationary Employment.–Probationary employment shall not exceed six (6) months from the
date the employee started working, unless it is covered by an apprenticeship agreement stipulating a longer period.
The services of an employee who has been engaged on a probationary basis may be terminated for a just cause or
when he fails to qualify as a regular employee in accordance with reasonable standards made known by the
employer to the employee at the time of his engagement. An employee who is allowed to work after a probationary
period shall be considered a regular employee.
The probationary employment of teachers in private schools is not governed purely by the Labor Code. The Labor
Code is supplemented with respect to the period of probation by special rules found in the Manual of Regulations for
Private Schools. On the matter of probationary period, Section 92 of the 1992 Manual of Regulations for Private
Schools regulations states:
Section 92. Probationary Period. – Subject in all instances to compliance with the Department and school
requirements, the probationary period for academic personnel shall not be more than three (3) consecutive years of
satisfactory service for those in the elementary and secondary levels, six (6) consecutive regular semesters of
satisfactory service for those in the tertiary level, and nine (9) consecutive trimesters of satisfactory service for those
in the tertiary level where collegiate courses are offered on a trimester basis.
Thus, it is the Manual of Regulations for Private Schools, and not the Labor Code, that determines whether or not a
faculty member in an educational institution has attained regular or permanent status.
There can be no dispute that the period of probation may be reduced if the employer, convinced of the fitness and
efficiency of a probationary employee, voluntarily extends a permanent appointment even before the three-year
period ends. Conversely, if the purpose sought by the employer is neither attained nor attainable within the said
period, the law does not preclude the employer from terminating the probationary employment on justifiable ground;
or, a shorter probationary period may be incorporated in a collective bargaining agreement. But absent any
circumstances which unmistakably show that an abbreviated probationary period has been agreed upon, the three-
year probationary term governs.
The circumstance that respondent’s services were hired on semester basis did not negate the applicable
probationary period, which is three school years or six consecutive semesters.
Notwithstanding the limited engagement of probationary employees, they are entitled to constitutional protection of
security of tenure during and before the end of the probationary period. The services of an employee who has been
engaged on probationary basis may be terminated for any of the following: (a) a just or (b) an authorized cause; and
(c) when he fails to qualify as a regular employee in accordance with reasonable standards prescribed by the
employer.
Respondent’s termination after five semesters of satisfactory service was illegal. Respondent therefore is entitled to
continue his three-year probationary period, such that from March 31, 2005, his probationary employment is deemed
renewed for the following semester (1st semester of SY 2005-2006). However, given the discordant relations that
had arisen from the parties' dispute, it can be inferred with certainty that petitioner had opted not to retain respondent
in its employ beyond the three-year period.
2. YES
In reviewing the Labor Arbiter's finding of illegal dismissal, the NLRC concluded that respondent had already attained
regular status after the expiration of his first appointment contract as probationary employee. Such conclusion was
but a logical result of the NLRC's own interpretation of the law. Since petitioner elevated the questions of the validity
of respondent's dismissal and the applicable probationary period under the aforesaid regulations, the NLRC did not
gravely abuse its discretion in fully resolving the said issues.

———————————————————————————————————————-

Angeles| Bajana | Balladares | Brillantes | Briones | Cabansag | Callanta | Chua | David|


De Leon | Gomez | Lopez | Macalino | Nostratis | Padilla | Reynon | Santos | Tan |Velasco
4E / 4F - 2018-2019
Page 662 of 920
LABOR REVIEW DIGEST
Atty. Joyrich Golangco

4. UNIVAC DEVELOPMENT, INC. v. WILLIAM SORIANO


GR No: 182072 Date: June 19, 2013
Ponente: Peralta, J.

Doctrine:
Petitioner’s failure to specify the reasonable standards by which respondent’s alleged poor performance was
evaluated as well as to prove that such standards were made known to him at the start of his employment, makes
respondent a regular employee. In other words, because of this omission on the part of petitioner, respondent is
deemed to have been hired from day one as a regular employee.

FACTS:
Respondent William Soriano was hired by petitioner Univac Development, Inc. on probationary basis as legal
assistant. Eight days prior to the completion of his six months probationary period, he was terminated from
employment due to the company’s cost-cutting measures. He allegedly asked for a thirty-day notice but his
termination was ordered to be effective immediately. He then filed a complaint for illegal dismissal against petitioner.

Petitioner denied the allegations of respondent and claimed instead that prior to his employment, respondent was
informed of the standards required for regularization. Petitioner also supposedly informed him of his duties and
obligations. Petitioner recalled that a company meeting was held where respondent allegedly expressed his intention
to leave the company because he wanted to review for the bar examinations. It was also in that meeting where he
was informed of his unsatisfactory performance in the company. Thus, when respondent did not report for work on,
petitioner assumed that he pushed through with his plan to leave the company. In other words, petitioner claimed that
respondent was not illegally dismissed from employment, rather, he in fact abandoned his job by his failure to report
for work.

LA/NLRC RULING:
The LA dismissed the respondent’s complaint for lack of merit and held that respondent was informed of his
unsatisfactory performance. Considering, however, that respondent was dismissed from employment eight days prior
to the end of his probationary period, he was entitled to eight days backwages.

The NLRC affirmed the LA decision in its entirety. Citing respondent’s educational background and knowledge of the
laws, he was presumed to know prior to employment the reasonable standards required for regularization. The
tribunal also gave credence to petitioner’s claim that a company meeting was held and that respondent was apprised
of his unsatisfactory performance. Hence, petitioner was found to have validly exercised management prerogative
when it terminated respondent’s probationary employment.

CA RULING:
The CA found petitioner to have been illegally dismissed from work, and ordered the payment of his full backwages
and separation pay in lieu of reinstatement. It gave more credence to respondent’s claim of illegal dismissal rather
than petitioner’s theory of abandonment. It held that petitioner failed to apprise respondent of the standards required
for regularization, coupled with the fact that it failed to make an evaluation of his performance, making his dismissal
illegal.

ISSUE/S:
1. Whether there was a violation of the doctrine of immutability of judgment when the respondent elevated the matter
to the CA.
2. Whether respondent was illegally dismissed from employment.

HELD:
1. NONE. Under Article 223 of the Labor Code, the decision of the NLRC becomes final and executory after the lapse
of ten calendar days from receipt thereof by the parties. However, the adverse party is not precluded from assailing
the decision via petition for certiorari under Rule 65 of the Rules of Court before the CA and then to this Court via a
petition for review under Rule 45. Thus, contrary to the contention of petitioner, there is no violation of the doctrine of
immutability of judgment when respondent elevated the matter to the CA which the latter consequently granted.

The power of the CA to review NLRC decisions via Rule 65 or Petition for Certiorari has long been settled. Pursuant
to the exercise of its original jurisdiction over Petitions for Certiorari, the CA is specifically given the power to pass
upon the evidence, if and when necessary, to resolve factual issues. Contrary to the contention of petitioner, the CA
can review the finding of facts of the NLRC and the evidence of the parties to determine whether the NLRC gravely
abused its discretion in finding that there was no illegal dismissal against respondent

Angeles| Bajana | Balladares | Brillantes | Briones | Cabansag | Callanta | Chua | David|


De Leon | Gomez | Lopez | Macalino | Nostratis | Padilla | Reynon | Santos | Tan |Velasco
4E / 4F - 2018-2019
Page 663 of 920
LABOR REVIEW DIGEST
Atty. Joyrich Golangco

2. YES. Petitioner’s failure to specify the reasonable standards by which respondent’s alleged poor performance was
evaluated as well as to prove that such standards were made known to him at the start of his employment, makes
respondent a regular employee. In other words, because of this omission on the part of petitioner, respondent is
deemed to have been hired from day one as a regular employee. To justify the dismissal of an employee, the
employer must, as a rule, prove that the dismissal was for a just cause and that the employee was afforded due
process prior to dismissal. Both of which were not observed in this case.

It is undisputed that respondent was hired as a probationary employee. As such, he did not enjoy a permanent
status. Nevertheless, he is accorded the constitutional protection of security of tenure which means that he can only
be dismissed from employment for a just cause or when he fails to qualify as a regular employee in accordance with
reasonable standards made known to him by the employer at the time of his engagement. However, petitioner failed
to present adequate evidence to substantiate its claim that respondent was apprised of the standards for
regularization. Aside from a bare allegation, it was not shown that a performance evaluation was conducted to prove
that his performance was indeed unsatisfactory.
———————————————————————————————————————-

Angeles| Bajana | Balladares | Brillantes | Briones | Cabansag | Callanta | Chua | David|


De Leon | Gomez | Lopez | Macalino | Nostratis | Padilla | Reynon | Santos | Tan |Velasco
4E / 4F - 2018-2019
Page 664 of 920
LABOR REVIEW DIGEST
Atty. Joyrich Golangco

248. 5. Abbott Laboratories, Philippines, Cecille A. Terrible, Edwin D. Feist, Maria Olivia T. Yabut-Misa,
Teresita C. Bernardo, And Allan G. Almazar v. Pearlie Ann F. Alcaraz
249. GR No: 192571 Date: July 23, 2013
Ponente: PERLAS-BERNABE, J
250. Doctrine:
251. A probationary employee, like a regular employee, enjoys security of tenure. However, in cases of
probationary employment, aside from just or authorized causes of termination, an additional ground is provided under
Article 296 of the Labor Code, i.e., the probationary employee may also be terminated for failure to qualify as a
regular employee in accordance with the reasonable standards made known by the employer to the employee at the
time of the engagement.
252.
253. FACTS:
254. Alcaraz was employed on a probationary basis for a period of 6 months by Abbot Laboratories(Abbott). On
March 3, 2005, petitioner Maria Olivia T. Yabut-Misa (Misa), Abbott's Human Resources (HR) Director, sent Alcaraz
an e-mail which contained an explanation of the procedure for evaluating the performance of probationary employees
and further indicated that Abbott had only one evaluation system for all of its employees. Alcaraz was also given
copies of Abbott's Code of Conduct and Probationary Performance Standards and Evaluation (PPSE) and
Performance Excellence Orientation Modules (Performance Modules) which she had to apply in line with her task of
evaluating the Hospira ALSU staff.
255.
256. On May 16, 2005, Alcaraz was called to a meeting with Walsh and Terrible where she was informed that
she failed to meet the regularization standards for the position of Regulatory Affairs Manager. Thereafter, Walsh and
Terrible requested Alcaraz to tender her resignation, else they be forced to terminate her services. She was also told
that, regardless of her choice, she should no longer report for work and was asked to surrender her ofiice
identification cards. She requested to be given one week to decide on the same, but to no avail.
257. On May 23, 2005, Walsh, Almazar, and Bernardo personally handed to Alcaraz a letter stating that her
services had been terminated effective May 19, 2005. The letter detailed the reasons for Alcaraz's termination —
particularly, that Alcaraz: (a) did not manage her time effectively; (b) failed to gain the trust of her staff and to build an
effective rapport with them; (c) failed to train her staff effectively; and (d) was not able to obtain the knowledge and
ability to make sound judgments on case processing and article review which were necessary for the proper
performance of her duties.
258.
259. Alcaraz filed complaint for illegal dismissal and damages against Abbott and its oficers, namely, Misa,
Bernardo, Almazar, Walsh, Terrible, and Feist. She claimed that she should have already been considered as a
regular and not a probationary employee given Abbott's failure to inform her of the reasonable standards for her
regularization upon her engagement as required under Article 295 of the Labor Code. In this relation, she contended
that while her employment contract stated that she was to be engaged on a probationary status, the same did not
indicate the standards on which her regularization would be based. She further averred that the individual petitioners
maliciously connived to illegally dismiss her when: (a) they threatened her with termination; (b) she was ordered not
to enter company premises even if she was still an employee thereof; and (c) they publicly announced that she
already resigned in order to humiliate her.
260.
261. On the contrary, petitioners maintained that Alcaraz was validly terminated from her probationary
employment given her failure to satisfy the prescribed standards for her regularization which were made known to her
at the time of her engagement.
262.
263. LA Ruling:
264. LA dismissed Alcaraz's complaint for lack of merit. The LA rejected Alcaraz's argument that she was not
informed of the reasonable standards to qualify as a regular employee considering her admissions that she was
briefed by Almazar on her work during her pre-employment orientation meeting and that she received copies of
Abbott's Code of Conduct and Performance Modules which were used for evaluating all types of Abbott employees.
As Alcaraz was unable to meet the standards set by Abbott as per her performance evaluation, the LA ruled that the
termination of her probationary employment was justified.
265.
266. NLRC Ruling:
267. NLRC ruled that there was no evidence showing that Alcaraz had been apprised of her probationary status
and the requirements which she should have complied with in order to be a regular employee. It held that Alcaraz's
receipt of her job description and Abbott's Code of Conduct and Performance Modules was not equivalent to her
being actually informed of the performance standards upon which she should have been evaluated on. It further
observed that Abbott did not comply with its own standard operating procedure in evaluating probationary

Angeles| Bajana | Balladares | Brillantes | Briones | Cabansag | Callanta | Chua | David|


De Leon | Gomez | Lopez | Macalino | Nostratis | Padilla | Reynon | Santos | Tan |Velasco
4E / 4F - 2018-2019
Page 665 of 920
LABOR REVIEW DIGEST
Atty. Joyrich Golangco

employees. The NLRC was also not convinced that Alcaraz was terminated for a valid cause given that petitioners'
allegation of Alcaraz's "poor performance" remained unsubstantiated.
268.
269. CA RULING:
270. CA affirmed the decision of the NLRC
271.
272. ISSUE/S:
273. 1. Whether or not Alcaraz was sufficiently informed of the reasonable standards to qualify her as a regular
employee
274. 2. Whether or not Alcaraz was validly terminated from her employment
275.
276. HELD:
277. 1. YES. The employer is made to comply with two (2) requirements when dealing with a probationary
employee: first, the employer must communicate the regularization standards to the probationary employee; and
second, the employer must make such communication at the time of the probationary employee's engagement. If the
employer fails to comply with either, the employee is deemed as a regular and not a probationary employee. In this
case, Abbott clearly conveyed to Alcaraz her duties and responsibilities as Regulatory Affairs Manager prior to,
during the time of her engagement, and the incipient stages of her employment.
278.
279. 2.YES. Section 2, Rule I, Book VI of the Implementing Rules of the Labor Code states that "[i]f the
termination is brought about by the . . . failure of an employee to meet the standards of the employer in case of
probationary employment, it shall be sufficient that a written notice is served the employee, within a reasonable time
from the effective date of termination." As the records show, Alcaraz's dismissal was effected through a letter dated
May 19, 2005 which she received on May 23, 2005 and again on May 27, 2005. Stated therein were the reasons for
her termination, i.e., that after proper evaluation, Abbott determined that she failed to meet the reasonable standards
for her regularization considering her lack of time and people management and decision-making skills, which are
necessary in the performance of her functions as Regulatory Affairs Manager. Undeniably, this written notice
sufiiciently meets the criteria set forth above, thereby legitimizing the cause and manner of Alcaraz's dismissal as a
probationary employee under the parameters set by the Labor Code.

Angeles| Bajana | Balladares | Brillantes | Briones | Cabansag | Callanta | Chua | David|


De Leon | Gomez | Lopez | Macalino | Nostratis | Padilla | Reynon | Santos | Tan |Velasco
4E / 4F - 2018-2019
Page 666 of 920
LABOR REVIEW DIGEST
Atty. Joyrich Golangco

280. 6. Colegio Del Santisimo Rosario And Sr. Zenaida S. Mofada v. Emmanuel Rojo
281. GR No: 170388 Date: September 4, 2013
Ponente: DEL CASTILLO, J
282. Doctrine:
283. Full-time teachers become regular or permanent employees once they have satisfactorily completed the
probationary period of three school years.
284. For teachers on probationary employment, in which case a fixed term contract is not specifically used for the
fixed term it offers, it is incumbent upon the school to have not only set reasonable standards to be followed by said
teachers in determining qualification for regular employment, the same must have also been communicated to the
teachers at the start of the probationary period, or at the very least, at the start of the period when they were to be
applied.
285.
286. FACTS:
287. Petitioner Colegio del Santisimo Rosario (CSR) hired respondent as a high school teacher on probationary
basis for the school years 1992-1993, 1993-1994 and 1994-1995. On April 5, 1995, CSR, through petitioner Sr.
Zenaida S. Mofada, OP (Mofada), decided not to renew respondent's services.
288.
289. Thus, on July 13, 1995, respondent filed a Complaint for illegal dismissal. He alleged that since he had
served three consecutive school years which is the maximum number of terms allowed for probationary employment,
he should be extended permanent employment. Citing paragraph 75 of the 1970 Manual of Regulations for Private
Schools (1970 Manual), respondent asserted that "full-time teachers who have rendered three (3) consecutive years
of satisfactory services shall be considered permanent."
290.
291. On the other hand, petitioners argued that respondent knew that his Teacher's Contract for school year
1994-1995 with CSR would expire on March 31, 1995. Accordingly, respondent was not dismissed but his
probationary contract merely expired and was not renewed. Petitioners also claimed that the "three years" mentioned
in paragraph 75 of the 1970 Manual refer to "36 months," not three school years. And since respondent served for
only three school years of 10 months each or 30 months, then he had not yet served the "three years" or 36 months
mentioned in paragraph 75 of the 1970 Manual.
292.
293. LA Ruling:
294. LA ruled that "three school years" means three years of 10 months, not 12 months. Considering that
respondent had already served for three consecutive school years, then he has already attained regular employment
status. Thus, the non-renewal of his contract for school year 1995-1996 constitutes illegal dismissal.
295.
296. NLRC Ruling:
297. NLRC affirmed the LA's Decision with modification. It held that after serving three school years, respondent
had attained the status of regular employment especially because CSR did not make known to respondent the
reasonable standards he should meet.
298.
299. CA RULING:
300. According to the CA, respondent has attained the status of a regular employee after he was employed for
three consecutive school years as a full-time teacher and had served CSR satisfactorily.
301.
302. APPEAL TO THE SC:
303. Petitioner’s contention:
304. Petitioners maintain that upon the expiration of the probationary period, both the school and the respondent
were free to renew the contract or let it lapse. Petitioners insist that a teacher hired for three consecutive years as a
probationary employee does not automatically become a regular employee upon completion of his third year of
probation. It is the positive act of the school — the hiring of the teacher who has just completed three consecutive
years of employment on probation for the next school year — that makes the teacher a regular employee of the
school.
305.
306. ISSUE/S:
307. Whether or not petitioner was illegally dismissed
308.
309. HELD:
310. YES. The scheme of fixed-term contract is a system that operates during the probationary period and for
this reason is subject to Article 281(now 296) of the Labor Code. Under Section 93 of the 1992 Manual, full-time
teachers who have satisfactorily completed their probationary period shall be considered regular or permanent. It

Angeles| Bajana | Balladares | Brillantes | Briones | Cabansag | Callanta | Chua | David|


De Leon | Gomez | Lopez | Macalino | Nostratis | Padilla | Reynon | Santos | Tan |Velasco
4E / 4F - 2018-2019
Page 667 of 920
LABOR REVIEW DIGEST
Atty. Joyrich Golangco

must be emphasized that mere rendition of service for three consecutive years does not automatically ripen into a
permanent appointment. It is incumbent upon the school to have not only set reasonable standards to be followed by
said teachers in determining qualification for regular employment, the same must have also been communicated to
the teachers at the start of the probationary period, or at the very least, at the start of the period when they were to be
applied. Should the teachers not have been apprised of such reasonable standards at the time specified above, they
shall be deemed regular employees. In this case, glaringly absent from petitioners' evidence are the reasonable
standards that respondent was expected to meet that could have served as proper guidelines for purposes of
evaluating his performance. Therefore, petitioner was illegally dismissed.

———————————————————————————————————————-

Angeles| Bajana | Balladares | Brillantes | Briones | Cabansag | Callanta | Chua | David|


De Leon | Gomez | Lopez | Macalino | Nostratis | Padilla | Reynon | Santos | Tan |Velasco
4E / 4F - 2018-2019
Page 668 of 920
LABOR REVIEW DIGEST
Atty. Joyrich Golangco

311. 7. Philippine Daily Inquirer, Inc. v. Leon M. Magtibay, Jr. And Philippine Daily Inquirer Employees
312. Union (PDIEU)
313. GR No: 164532 Date: July 27, 2007
314. Ponente: GARCIA, J
315.
316. Doctrine:
317. Within the limited legal six-month probationary period, probationary employees are still entitled to security of
tenure. It is expressly provided in the afore-quoted Article 296 that a probationary employee may be terminated only
on two grounds: (a) for just cause, or (b) when he fails to qualify as a regular employee in accordance with
reasonable standards made known by the employer to the employee at the time of his engagement.
318.
319. FACTS:
320. PDI hired Magtibay, on contractual basis, to assist, for a period of five months the regular phone operator.
Before the expiration of Magtibay's contractual employment, he and PDI agreed to a fifteen-day contract extension,
or from July 17, 1995 up to July 31, 1995, under the same conditions as the existing contract.
321. Thereafter, PDI announced the creation and availability of a new position for a second telephone operator
which Magtibay applied for. PDI chosed to hire Magtibay on a probationary basis for a period of six (6) months.
322. A week before the end the agreed 6-month probationary period, PDI officer Benita del Rosario handed
Magtibay his termination paper, grounded on his alleged failure to meet company standards. Aggrieved, Magtibay
immediately filed a complaint for illegal dismissal and damages before the Labor Arbiter. PDIEU later joined the fray
by filing a supplemental complaint for unfair labor practice.
323. Magtibay argued that he had become a regular employee by operation of law, considering that he had been
employed by and had worked for PDI for a total period of ten months and that he was not apprised at the beginning
of his employment of the performance standards of the company, hence, there was no basis for his dismissal.
324. PDI contended that the period covered by the contractual employment cannot be counted with or tacked to
the period for probation and that Magtibay was dismissed for violation of company rules and policies which were
allegedly made known to him at the start through an orientation seminar conducted by the company.
325.
326. LA Ruling:
LA ruled that the previous contractual employment, as later extended by 15 days, cannot be considered as part of his
subsequent probationary employment. Labor Arbiter further ruled that Magtibay's dismissal from his probationary
employment was for a valid reason. Albeit the basis for termination was couched in the abstract, i.e., "you did not
meet the standards of the company", there were three specific reasons for Magtibay's termination, to wit: (1) he
repeatedly violated the company rule prohibiting unauthorized persons from entering the telephone operator's room;
(2) he intentionally omitted to indicate in his application form his having a dependent child; and (3) he exhibited lack
of sense of responsibility by locking the door of the telephone operator's room.
327.
328. NLRC Ruling:
329. NLRC ruled that Magtibay was illegally dismissed because Magtibay's probationary employment had
ripened into a regular one.
330.
331. CA RULING:
332. CA agreed with the NLRC’s decision.
333.
334. Petitioner’s contention:
335. In claiming that it had adequately apprised Magtibay of the reasonable standards against which his
performance will be gauged for purposes of permanent employment, PDI cited the one-on-one seminar between
Magtibay and its Personnel Assistant and briefing made to him by his direct supervisor.
336.
337. ISSUE/S:
338. 1. Is failure to follow the employer's rules and regulations be deemed failure by said employee to meet the
standards of his employer?
339. 2. Whether the procedural due process of the labor code had been observed by the petitioner
340.
341. HELD:
342. 1. YES. Within the limited legal six-month probationary period, probationary employees are still entitled to
security of tenure. It is expressly provided in the afore-quoted Article 296 that a probationary employee may be
terminated only on two grounds: (a) for just cause, or (b) when he fails to qualify as a regular employee in
accordance with reasonable standards made known by the employer to the employee at the time of his engagement.
All employees, be they regular or probationary, are expected to comply with company-imposed rules and regulations,

Angeles| Bajana | Balladares | Brillantes | Briones | Cabansag | Callanta | Chua | David|


De Leon | Gomez | Lopez | Macalino | Nostratis | Padilla | Reynon | Santos | Tan |Velasco
4E / 4F - 2018-2019
Page 669 of 920
LABOR REVIEW DIGEST
Atty. Joyrich Golangco

else why establish them in the first place. Probationary employees unwilling to abide by such rules have no right to
expect, much less demand, permanent employment. It is on record that Magtibay committed obstinate infractions of
company rules and regulations, which in turn constitute sufficient manifestations of his inadequacy to meet
reasonable employment norms. Therefore, there is sufficient factual and legal basis, duly established by substantial
evidence, for PDI to legally terminate Magtibay's probationary employment effective upon the end of the 6- month
probationary period.
343.
2. YES. Unlike under the first ground for the valid termination of probationary employment which is for just cause, the
second ground does not require notice and hearing. Due process of law for this second ground consists of making
the reasonable standards expected of the employee during his probationary period known to him at the time of his
probationary employment. In this case, the orientation and briefing done by PDI with the respondent suffices the
requirement of the law that the employee must be apprise of the of the employment standards expected of him at the
time of his probationary employment.
———————————————————————————————————————-

Angeles| Bajana | Balladares | Brillantes | Briones | Cabansag | Callanta | Chua | David|


De Leon | Gomez | Lopez | Macalino | Nostratis | Padilla | Reynon | Santos | Tan |Velasco
4E / 4F - 2018-2019
Page 670 of 920
LABOR REVIEW DIGEST
Atty. Joyrich Golangco

8. RADIN C. ALCIRA vs. NLRC, MIDDLEBY PHILIPPINES CORPORATION, ET AL.


G.R. No. 149859 Date: June 9, 2004
Ponente: CORONA, J.:

DOCTRINES:
(1) The computation of the 6-month probationary period is reckoned from the date of appointment up to the same
calendar date of the 6th month following. Since the number of days in each particular month was irrelevant.—(2) An
employer is deemed to substantially comply with the rule on notification of standards if he apprises the employee that
he will be subjected to a performance evaluation on a particular date after his hiring.—(3) Even if probationary
employees do not enjoy permanent status, they are accorded the constitutional protection of security of tenure. This
means they may only be terminated for just cause or when they otherwise fail to qualify as regular employees in
accordance with reasonable standards made known to them by the employer at the time of their engagement.

FACTS:
Respondent Middleby Philippines Corporation (MPC) hired petitioner Radin Alcira on a probationary basis for 6
months. Presenting their respective copies of Alcira’s appointment paper, the parties claimed conflicting starting
dates of employment according to petitioner he started on May 20, 1996 while Respondent claims on May 27, 1996.
Both documents indicated Alciras’ employment status as probationary and a remark that after five months Alciras’
performance shall be evaluated and any adjustment in salary shall depend on his work performance. Unhappy with
petitioner’s performance, MPC terminated his services on November 20, 1996. Petitioner filed a complaint of illegal
dismissal with NLRC.

LA RULING:
Dismissed the complaint on the ground that: (1) respondents were able to prove that petitioner was apprised of the
standards for becoming a regular employee; (2) they showed that petitioner did not perform well in his assigned work
and his attitude was below par compared to the company’s standard required of him and (3) petitioner’s dismissal
was before his regularization.

NLRC RULING:
Affirmed the LA.

CA RULING:
Affirmed the NLRC.

Petitioner’s contention:
Under the terms of his contract, his probationary employment was only for 5 months, he already attained the status of
a regular employee when he was dismissed on November 20, 1996 because, having started work on May 20, 1996,
the 6-month probationary period ended on November 16, 1996; and that respondent never informed him of the
standards for regularization at the start of his employment.

ISSUE:
 Whether petitioner was allowed to work beyond his probationary period and was therefore already a regular
employee at the time of his alleged dismissal? NO.

 Whether respondent informed petitioner of the standards for regularization at the start of his employment? YES.

 Whether petitioner was illegally dismissed when respondent opted not to renew his contract on the last day of his
probationary employment? NO.

RULING:

(1) NO. In CALS Poultry Supply Corporation, et al. vs. Roco, the computation of the 6-month probationary period is
reckoned from the date of appointment up to the same calendar date of the 6th month following. Since the
number of days in each particular month was irrelevant, petitioner was still a probationary employee when
respondent opted not to regularize him on November 20, 1996.

(2) YES. Section 6 (d) of Rule 1 of the Implementing Rules of Book VI of the Labor Code (Department Order No. 10,
Series of 1997) provides that—in all cases of probationary employment, the employer shall make known to the
employee the standards under which he will qualify as a regular employee at the time of his engagement. Where
no standards are made known to the employee at that time, he shall be deemed a regular employee.

Angeles| Bajana | Balladares | Brillantes | Briones | Cabansag | Callanta | Chua | David|


De Leon | Gomez | Lopez | Macalino | Nostratis | Padilla | Reynon | Santos | Tan |Velasco
4E / 4F - 2018-2019
Page 671 of 920
LABOR REVIEW DIGEST
Atty. Joyrich Golangco

An employer is deemed to substantially comply with the rule on notification of standards if he apprises the
employee that he will be subjected to a performance evaluation on a particular date after his hiring. Respondent
substantially notified petitioner of the standards to qualify as a regular employee when it apprised him, at the
start of his employment, that it would evaluate his supervisory skills after five months.

(3) NO. It is settled that even if probationary employees do not enjoy permanent status, they are accorded the
constitutional protection of security of tenure. This means they may only be terminated for just cause or when
they otherwise fail to qualify as regular employees in accordance with reasonable standards made known to
them by the employer at the time of their engagement. But we have also ruled in Manlimos, et al. vs. NLRC that
this constitutional protection ends on the expiration of the probationary period. On that date, the parties are free
to either renew or terminate their contract of employment.
———————————————————————————————————————-

Angeles| Bajana | Balladares | Brillantes | Briones | Cabansag | Callanta | Chua | David|


De Leon | Gomez | Lopez | Macalino | Nostratis | Padilla | Reynon | Santos | Tan |Velasco
4E / 4F - 2018-2019
Page 672 of 920
LABOR REVIEW DIGEST
Atty. Joyrich Golangco

9. YOLANDA M. MERCADO, CHARITO S. DE LEON, DIANA R. LACHICA, MARGARITO M. ALBA, JR., and
FELIX A. TONOG v. AMA COMPUTER COLLEGE-PARAÑAQUE CITY, INC.
G.R. No. 183572 Date: April 13, 2010
Ponente: BRION, J.:

Doctrine:
The school’s system of fixed-term contract is a system that operates during the probationary period and for this
reason is subject to the terms of Article 281 of the Labor Code. Labor, for its part, is given the protection during the
probationary period of knowing the company standards the new hires have to meet during the probationary period,
and to be judged on the basis of these standards, aside from the usual standards applicable to employees after they
achieve permanent status. These standards should be made known to the teachers on probationary status at the
start of their probationary period, or at the very least, at the start of the semester or the trimester during which the
probationary standards are to be applied. These rules ensure compliance with the limited security of tenure
guarantee the law extends to probationary employees.

FACTS:
Petitioners Yolanda Mercado et al. are all former faculty members of respondent AMA Computer College (AMACC)
who started teaching on May 25, 1998. They executed individual Teachers Contracts for each of the trimesters that
they were engaged to teach, with the following common stipulation:

1. POSITION. The TEACHER has agreed to accept a non-tenured appointment to work in the College of xxx
effective xxx to xxx or for the duration of the last term that the TEACHER is given a teaching load based on the
assignment duly approved by the DEAN/SAVP-COO.

For the school year 2000-2001, AMACC implemented new Performance Appraisal System for Teachers (PAST).
Under the new screening guidelines, teachers were to be hired or maintained based on extensive teaching
experience, capability, potential, high academic qualifications and research background. The petitioners failed to
obtain a passing rating based on the performance standards. On September 7, 2000, petitioners individually received
a memorandum from AMACC, informing them that with the expiration of their contract to teach, their contract would
no longer be renewed.

Petitioners filed with the Arbitration Branch of the NLRC a complaint for illegal dismissal, underpayment of wages,
non-payment of overtime and overload compensation, 13th month pay, and for discriminatory practices against
AMACC.

LA RULING:
Petitioners had been illegally dismissed. Article 281 of the Labor Code on probationary employment applied to the
case.

NLRC RULING:
Affirmed LA ruling but the applicable law is Section 92 of the Manual of Regulations for Private Schools (which
mandates a probationary period of 9 consecutive trimesters of satisfactory service for academic personnel in the
tertiary level where collegiate courses are offered on a trimester basis), not Article 281 of the Labor Code (which
prescribes a probationary period of 6 months) as the LA ruled.

CA RULING:
Dismissed the petitioners’ complaint for illegal dismissal as they were hired on a non-tenured basis and for a fixed or
predetermined term.

Petitioners’ contention:
The applicable law on probationary employment is Article 281 of the Labor Code. AMACC failed to support its
allegation that they did not qualify under the new guidelines adopted for the school year 2000-2001; and that they
were illegally dismissed; their employment was terminated based on standards that were not made known to them at
the time of their engagement.

Respondent’s contention:
Petitioners were hired on a non-tenured basis and for a fixed or predetermined term. No actual dismissal transpired;
it simply did not renew the petitioners respective employment contracts because of their poor performance and failure
to satisfy the schools standards.

Angeles| Bajana | Balladares | Brillantes | Briones | Cabansag | Callanta | Chua | David|


De Leon | Gomez | Lopez | Macalino | Nostratis | Padilla | Reynon | Santos | Tan |Velasco
4E / 4F - 2018-2019
Page 673 of 920
LABOR REVIEW DIGEST
Atty. Joyrich Golangco

ISSUE:
Was the non-renewal of the contracts of petitioners on probationary status legal? NO.

RULING:

NO. The existence of the term-to-term contracts covering the petitioners employment is not disputed, nor is it
disputed that they were on probationary status not permanent or regular status from the time they were employed on
May 25, 1998 and until the expiration of their Teaching Contracts on September 7, 2000. Given the clear
constitutional and statutory intents, in a situation where the probationary status overlaps with a fixed-term contract
not specifically used for the fixed term it offers, Article 281 should assume primacy and the fixed-period character of
the contract must give way. This conclusion is immeasurably strengthened by the petitioners and the AMACCs hardly
concealed expectation that the employment on probation could lead to permanent status, and that the contracts are
renewable unless the petitioners fail to pass the schools standards.

The provision on employment on probationary status under the Labor Code is a primary example of the fine
balancing of interests between labor and management that the Code has institutionalized pursuant to the underlying
intent of the Constitution. On the one hand, employment on probationary status affords management the chance to
fully scrutinize the true worth of hired personnel before the full force of the security of tenure guarantee of the
Constitution comes into play. Based on the standards set at the start of the probationary period, management is
given the widest opportunity during the probationary period to reject hirees who fail to meet its own adopted but
reasonable standards. These standards, together with the just and authorized causes for termination of employment
the Labor Code expressly provides, are the grounds available to terminate the employment of a teacher on
probationary status.

Labor, for its part, is given the protection during the probationary period of knowing the company standards the new
hires have to meet during the probationary period, and to be judged on the basis of these standards, aside from the
usual standards applicable to employees after they achieve permanent status. Under the terms of the Labor Code,
these standards should be made known to the teachers on probationary status at the start of their probationary
period, or at the very least under the circumstances of the present case, at the start of the semester or the trimester
during which the probationary standards are to be applied. Of critical importance in invoking a failure to meet the
probationary standards, is that the school should show as a matter of due process how these standards have been
applied. These rules ensure compliance with the limited security of tenure guarantee the law extends to probationary
employees.

The school, however, cannot forget that its system of fixed-term contract is a system that operates during the
probationary period and for this reason is subject to the terms of Article 281 of the Labor Code. Unless this
reconciliation is made, the requirements of this Article on probationary status would be fully negated as the school
may freely choose not to renew contracts simply because their terms have expired. The inevitable effect of course is
to wreck the scheme that the Constitution and the Labor Code established to balance relationships between labor
and management.

AMACC did not renew the petitioners contracts because they failed to pass the Performance Appraisal System for
Teachers (PAST) and other requirements for regularization that the school undertakes to maintain its high academic
standards. The exact terms of the standards were never introduced as evidence; neither does the evidence show
how these standards were applied to the petitioners. Without these pieces of evidence (effectively, the finding of just
cause for the non-renewal of the petitioners contracts), we have nothing to consider and pass upon as valid or invalid
for each of the petitioners. Inevitably, the non-renewal (or effectively, the termination of employment of employees on
probationary status) lacks the supporting finding of just cause that the law requires and, hence, is illegal.

Angeles| Bajana | Balladares | Brillantes | Briones | Cabansag | Callanta | Chua | David|


De Leon | Gomez | Lopez | Macalino | Nostratis | Padilla | Reynon | Santos | Tan |Velasco
4E / 4F - 2018-2019
Page 674 of 920
LABOR REVIEW DIGEST
Atty. Joyrich Golangco

10. OYSTER PLAZA HOTEL, ROLITO GO, AND JENNIFER AMPEL vs. ERROL O. MELIVO
G.R. No. 217455 Date: October 05, 2016
Ponente: MENDOZA, J.

DOCTRINE:
Probation is the period during which the employer may determine if the employee is qualified for possible inclusion in
the regular force. The employer has the right or is at liberty to choose who will be hired and who will be denied
employment. In that sense, it is within the exercise of the right to select his employees that the employer may set or
fix a probationary period within which the latter may test and observe the conduct of the former before hiring him
permanently. An employee allowed to work beyond the probationary period is deemed a regular employee.

FACTS:
Respondent Errol Melivo was hired as a trainee room boy on probationary basis by petitioner Oyster Plaza. He was
made to sign an employment contract but he was not furnished a copy, after his 6-months contract expired and his
work ended, petitioner hired him again as a room boy, but without any employment contract or document. Months
after, his supervisor verbally told him that his contract was expiring, hence he must stop reporting for work.
Respondent filed before the NLRC a complaint for illegal dismissal with prayers for reinstatement and payment of
back wages, holiday pay, overtime pay, service incentive leave, and, 13th month pay against petitioners.

LA RULING:
Respondent was illegally dismissed considering that he had already rendered six (6) months of service, hence, he
had become a regular employee by operation of law. Having attained the regular employment status, he could only
be terminated for a valid cause.

NLRC RULING:
Affirmed the LA. The contract of employment failed to reveal the specific project or any phase of it where he was
employed; and that the petitioners failed to submit a report of his termination to the nearest public employment office,
as required under D.O. No, 19. The failure to file a termination report upon the alleged cessation of Melivo's
employment was an indication that he was not a project employee, but a regular employee. Hence, he was illegally
dismissed.

CA RULING:
Affirmed the NLRC.

Petitioners’ contention:
Melivo was not illegally dismissed because he was not a regular employee but merely a project employee.

ISSUE:
(1) Was respondent illegally terminated? YES.
(2) Was respondent a project employee? NO.

RULING:
(1) YES. Probation is the period during which the employer may determine if the employee is qualified for possible
inclusion in the regular force. The employer has the right or is at liberty to choose who will be hired and who will
be denied employment. In that sense, it is within the exercise of the right to select his employees that the
employer may set or fix a probationary period within which the latter may test and observe the conduct of the
former before hiring him permanently. An employee allowed to work beyond the probationary period is deemed a
regular employee.

Melivo was first hired as a trainee in August 2008 for 3 months. As a room boy, his performance was certainly
under observation. Hence, his probationary employment actually started in August 2008, at the same time he
started working as a trainee. Therefore, when he was re-hired as room boy after his training period sometime in
November 2008 he attained regular employment status. Assuming that the 3-month training period could be
considered a probationary period, the conclusion would still be the same. It should be remembered that Melivo
was again employed as a room boy in November 2008 under probationary status for five (5) months or until
March 2009. Records would show that Melivo had completed his probationary employment. Thus, when Oyster
Plaza re-hired him for the third time on April 7, 2009, he became its regular employee thereof.

As a regular employee, Melivo could only be dismissed for just or authorized causes after affording him the
procedural requirement of notice and hearing. The petitioners failed to adduce evidence that Melivo's dismissal

Angeles| Bajana | Balladares | Brillantes | Briones | Cabansag | Callanta | Chua | David|


De Leon | Gomez | Lopez | Macalino | Nostratis | Padilla | Reynon | Santos | Tan |Velasco
4E / 4F - 2018-2019
Page 675 of 920
LABOR REVIEW DIGEST
Atty. Joyrich Golangco

was for a just or authorized cause, or that he was sufficiently notified and given opportunity to be heard why his
employment should not be terminated. Hence, Melivo's dismissal was illegal.

(2) NO. Petitioners' contention that Melivo was hired as a project employee is untenable. Under Article 280 of the
Labor Code, as amended, a project employee is one whose employment has been fixed for a specific project or
undertaking, the completion or termination of which has been determined at the time of the engagement of the
employee. Here, the contract of employment failed to indicate the specific project or undertaking for which
Oyster Plaza sought Melivo's services. Moreover, as correctly noted by the NLRC, the petitioners failed to
submit a report of Melivo's termination to the nearest public employment office, as required under Section 2 of
D.O. No. 19.

———————————————————————————————————————-

Angeles| Bajana | Balladares | Brillantes | Briones | Cabansag | Callanta | Chua | David|


De Leon | Gomez | Lopez | Macalino | Nostratis | Padilla | Reynon | Santos | Tan |Velasco
4E / 4F - 2018-2019
Page 676 of 920
LABOR REVIEW DIGEST
Atty. Joyrich Golangco

ARTICLE 297 - TERMINATION BY EMPLOYER

1. LOLITA S. CONCEPCION v. MINEX IMPORT CORPORATION/MINERAMA CORPORATION, KENNETH


MEYERS, SYLVIA P. MARIANO, and VINA MARIANO
GR No: 153569 Date: January 24, 2012
Ponente: PERLAS-BERNABE, J.

Doctrine: The employer may validly dismiss for loss of trust and confidence an employee who commits an
act of fraud prejudicial to the interest of the employer. Neither a criminal prosecution nor a conviction
beyond reasonable doubt for the crime is a requisite for the validity of the dismissal. Nonetheless, the
dismissal for a just or lawful cause must still be made upon compliance with the requirements of due
process under the Labor Code; otherwise, the employer is liable to pay nominal damages as indemnity to the
dismissed employee.

FACTS:
Respondent Minex Import-Export Corporation (Minex) engaged in the retail of semi-precious stones, selling them in
kiosks or stalls installed in various shopping centers within Metro Manila. It employed the petitioner initially as a
salesgirl,1 rotating her assignment among nearly all its outlets. It made her a supervisor in July 1997, but did not
grant her any salary increase. On October 23, 1997, respondent Vina Mariano, an Assistant Manager of Minex,
assigned the petitioner to the SM Harrison Plaza kiosk with the instruction to hold the keys of the kiosk. Working
under her supervision there were salesgirls Cristina Calung and Lida Baquilar.
On November 9, 1997, a Sunday, the petitioner and her salesgirls had sales of crystal items totaling P39,194.50. At
the close of business that day, they conducted a cash-count of their sales proceeds, including those from the
preceding Friday and Saturday, and determined their total for the three days to be P50,912.00. The petitioner
wrapped the amount in a plastic bag and deposited it in the drawer of the locked wooden cabinet of the kiosk.
At about 9:30 am of November 10, 1997, the petitioner phoned Vina Mariano to report that the P50,912.00 was
missing, explaining how she and her salesgirls had placed the wrapped amount at the bottom of the cabinet the night
before, and how she had found upon reporting to work that morning that the contents of the cabinet were in disarray
and the money already missing.
Later, while the petitioner was giving a detailed statement on the theft to the security investigator of Harrison Plaza,
Vina and Sylvia Mariano, her superiors, arrived with a policeman who immediately placed the petitioner under arrest
and brought her to Precinct 9 of the Malate Police Station. There, the police investigated her.
On November 12, 1997, the petitioner complained against the respondents for illegal dismissal in the Department of
Labor and Employment. On November 14, 1997, Minex, through Vina, filed a complaint for qualified theft against the
petitioner in the Office of the City Prosecutor in Manila.
Vina declared that the petitioner did not call the office of Minex for the pick-up of the P39,194.50 cash sales on
Sunday, November 9, 1997, in violation of the standard operating procedure (SOP) requiring cash proceeds
exceeding P10,000.00 to be reported for pick-up if the amount could not be deposited in the bank.
Thus, petitioner was charged with qualified theft.

The Labor Arbiter rendered decision in favor of the complainant and against the respondents declaring the
dismissal of the latter from work illegal and ordering her reinstatement to her former work position with full
backwages.
NLRC reversed the decision of the Labor Arbiter on December 28, 2000, declaring that the petitioner had not been
dismissed, but had abandoned her job after being found to have stolen the proceeds of the sales; and holding that
even if she had been dismissed, her dismissal would be justifiable for loss of trust and confidence in the light of the
finding of probable cause by the DOJ and the City Prosecutor and the filing of the information for qualified theft
against her.
CA sustained the NLRC mainly because of the DOJ Secretary’s finding of probable cause for qualified theft.
The petitioner still argues, however, that there was no evidence at all upon which Minex could validly dismiss her
considering that she had not yet been found guilty beyond reasonable doubt of the crime of qualified theft.

ISSUE/S:
Whether or not the petitioner was terminated for a just and valid cause.
HELD:
Yes. To dismiss an employee, the law requires the existence of a just and valid cause. Article 282 of the Labor
Code enumerates the just causes for termination by the employer.
The petitioner’s argument is not novel. It has been raised and rejected many times before on the basis that neither
conviction beyond reasonable doubt for a crime against the employer nor acquittal after criminal prosecution was

Angeles| Bajana | Balladares | Brillantes | Briones | Cabansag | Callanta | Chua | David|


De Leon | Gomez | Lopez | Macalino | Nostratis | Padilla | Reynon | Santos | Tan |Velasco
4E / 4F - 2018-2019
Page 677 of 920
LABOR REVIEW DIGEST
Atty. Joyrich Golangco

indispensable. Nor was a formal charge in court for the acts prejudicial to the interest of the employer a pre-requisite
for a valid dismissal.
Indeed, the employer is not expected to be as strict and rigorous as a judge in a criminal trial in weighing all the
probabilities of guilt before terminating the employee. Unlike a criminal case, which necessitates a moral certainty of
guilt due to the loss of the personal liberty of the accused being the issue, a case concerning an employee suspected
of wrongdoing leads only to his termination as a consequence. The quantum of proof required for convicting an
accused is thus higher – proof of guilt beyond reasonable doubt – than the quantum prescribed for dismissing an
employee – substantial evidence. In so stating, we are not diminishing the value of employment, but only noting that
the loss of employment occasions a consequence lesser than the loss of personal liberty, and may thus call for a
lower degree of proof.
Here, no less than the DOJ Secretary found probable cause for qualified theft against the petitioner. That finding was
enough to justify her termination for loss of confidence. To repeat, her responsibility as the supervisor tasked to
oversee the affairs of the kiosk, including seeing to the secure handling of the sales proceeds, could not be ignored
or downplayed. The employer’s loss of trust and confidence in her was directly rooted in the manner of how she, as
the supervisor, had negligently handled the large amount of sales by simply leaving the amount inside the cabinet
drawer of the kiosk despite being aware of the great risk of theft. At the very least, she could have resorted to the
SOP of first seeking guidance from the main office on how to secure the amount if she could not deposit in the bank
due to that day being a Sunday.
However, petitioner plainly demonstrated how quickly and summarily her dismissal was carried out without first
requiring her to explain anything in her defense as demanded under Section 2 (d) of Rule I of the Implementing Rules
of Book VI of the Labor Code. Instead, the respondents forthwith had her arrested and investigated by the police
authorities for qualified theft. This, we think, was a denial of her right to due process of law, consisting in the
opportunity to be heard and to defend herself.14 In fact, their decision to dismiss her was already final even before
the police authority commenced an investigation of the theft, the finality being confirmed by no less than Sylvia
Mariano herself telling the petitioner during their phone conversation following the latter’s release from police custody
on November 11, 1997 that she (Sylvia) “no longer wanted to see” her.
The fact that the petitioner was the only person suspected of being responsible for the theft aggravated the denial of
due process. When the respondents confronted her in the morning of November 10, 1997 for the first time after the
theft, they brought along a police officer to arrest and hale her to the police precinct to make her answer for the theft.
They evidently already concluded that she was the culprit despite a thorough investigation of the theft still to be
made.
The fair and reasonable opportunity required to be given to the employee before dismissal encompassed not only the
giving to the employee of notice of the cause and the ability of the employee to explain, but also the chance to defend
against the accusation.
In view of the foregoing, we impose on the respondents the obligation to pay to the petitioner an indemnity in the form
of nominal damages of P30,000.00.

Angeles| Bajana | Balladares | Brillantes | Briones | Cabansag | Callanta | Chua | David|


De Leon | Gomez | Lopez | Macalino | Nostratis | Padilla | Reynon | Santos | Tan |Velasco
4E / 4F - 2018-2019
Page 678 of 920
LABOR REVIEW DIGEST
Atty. Joyrich Golangco

2. DREAMLAND HOTEL RESORT and WESTLEY J. PRENTICE,


vs. STEPHEN B. JOHNSON
GR No: 191455 Date: March 12, 2014
Ponente: REYES, J
Doctrine: There is constructive dismissal if an act of clear discrimination, insensibility, or disdain by an
employer becomes so unbearable on the part of the employee that it would foreclose any choice by him
except to forego his continued employment. It exists where there is cessation of work because continued
employment is rendered impossible, unreasonable or unlikely, as an offer involving a demotion in rank and a
diminution in pay.
The Court construes that the act of the petitioners in not paying Johnson his salaries for three months has
become unbearable on the latter’s part that he had no choice but to cede his employment with them. The
petitioners cannot expect Johnson to tolerate working for them without any compensation.
FACTS:

Dreamland is a corporation duly registered with the Securities and Exchange Commission on January 15, 2003 to
exist for a period of fifty [50] years. Prentice is its current President and Chief Executive Officer. It is engaged in the
hotel, restaurant and allied businesses.

Respondent Stephen B. Johnson is an Australian citizen who came to the Philippines as a businessman/investor
without the authority to be employed as the employee/officer of any business as he was not able to secure his Alien
Employment Permit ["AEP" for brevity].

Prentice alleged that Johnson was able to convince Prentice to accept his offer to invest in Dreamland and at the
same time provide his services as Operations Manager of Dreamland with a promise that he will secure an AEP and
Tax Identification Number ["TIN" for brevity] prior to his assumption of work. Prentice and Johnson entered into an
Employment Agreement. Prentice asked on several occasions the production of the AEP and TIN from Johnson.
Johnson gave excuses and promised that he is already in possession of the requirements. Believing the word of
Johnson, Dreamland commenced a dry run of its operations. Johnson worked as a hotel and resort Operations
Manager only at that time. He worked for only about three (3) weeks until he suddenly abandoned his work and
subsequently resigned as Operations Manager starting November 3, 2007. He never reported back to work despite
several attempts of Prentice to clarify his issues.

However, according to Johnson there is no truth to the allegation that it was him who "offered" and "convinced"
petitioner Prentice to "invest" in and provide his services to petitioner Dreamland Hotel Resort x x x. The truth of the
matter is that it was petitioners who actively advertised for a resort manager for Dreamland Hotel. That from the start
of August 2007, as stipulated in the Employment Agreement, respondent Johnson already reported for work. It was
then that he found out to his dismay that the resort was far from finished. However, he was instructed to supervise
construction and speak with potential guests. As he remained unpaid since August 2007 and he has loaned all his
money to petitioners, he asked for his salary after the resort was opened in October 2007 but the same was not given
to him by petitioners. Thus, on November 3, 2007, after another embarrassment was handed out by petitioner
Prentice in front of the staff, which highlighted his lack of real authority in the hotel and the disdain for him by
petitioners, respondent Johnson was forced to submit his resignation, x x x. In deference to the Employment
Agreement signed, [Johnson] stated that he was willing to continue work for the three month period stipulated
therein. However, in an SMS or text message sent by Prentice to [Johnson] on the same day at around 8:20 pm, he
was informed that "… I consider [yo]ur resignation as immediate". Despite demand, petitioners refused to pay
[Johnson] the salaries and benefits due him.5
On January 31, 2008, Johnson filed a Complaint for illegal dismissal and non-payment of salaries, among others,
against the petitioners.
LA rendered a Decision6 dismissing Johnson’s complaint for lack of merit with the finding that he voluntarily resigned
from his employment and was not illegally dismissed.
The NLRC reversed the decision. CA dismissed the petition for lack of proof of authority and affidavit of service of
filing as required by Section 13 of the 1997 Rules of Procedure. The subsequent motion for reconsideration filed by
the petitioners was likewise denied.

ISSUE/S:
Whether or not Johnson was constructively dismissed.

HELD:

Angeles| Bajana | Balladares | Brillantes | Briones | Cabansag | Callanta | Chua | David|


De Leon | Gomez | Lopez | Macalino | Nostratis | Padilla | Reynon | Santos | Tan |Velasco
4E / 4F - 2018-2019
Page 679 of 920
LABOR REVIEW DIGEST
Atty. Joyrich Golangco

Yes. As regards the NLRC findings that Johnson was constructively dismissed and did not abandon his work, the
Court is in consonance with this conclusion with the following basis:
“Even the most reasonable employee would consider quitting his job after working for three months and receiving
only an insignificant fraction of his salaries. There was, therefore, not an abandonment of employment nor a
resignation in the real sense, but a constructive dismissal, which is defined as an involuntary resignation resorted to
when continued employment is rendered impossible, unreasonable or unlikely x x x.”
The petitioners aver that considering that Johnson tendered his resignation and abandoned his work; it is his burden
to prove that his resignation was not voluntary on his part.32
With this, the Court brings to mind its earlier ruling in the case of SHS Perforated Materials, Inc. v. Diaz33 where it
held that:
"There is constructive dismissal if an act of clear discrimination, insensibility, or disdain by an employer becomes so
unbearable on the part of the employee that it would foreclose any choice by him except to forego his continued
employment. It exists where there is cessation of work because continued employment is rendered impossible,
unreasonable or unlikely, as an offer involving a demotion in rank and a diminution in pay."34
It is impossible, unreasonable or unlikely that any employee, such as Johnson would continue working for an
employer who does not pay him his salaries. Applying the Court’s pronouncement in Duldulao v. CA35, the Court
construes that the act of the petitioners in not paying Johnson his salaries for three months has become
unbearable on the latter’s part that he had no choice but to cede his employment with them. The petitioners
cannot expect Johnson to tolerate working for them without any compensation.
As to the reliefs granted to an employee who is illegally dismissed, Golden Ace Builders v. Talde37 referring to
Macasero v. Southern Industrial Gases Philippines38 is instructive:
Thus, an illegally dismissed employee is entitled to two reliefs: backwages and reinstatement. The two reliefs
provided are separate and distinct. In instances where reinstatement is no longer feasible because of strained
relations between the employee and the employer, separation pay is granted. In effect, an illegally dismissed
employee is entitled to either reinstatement, if viable, or separation pay if reinstatement is no longer viable, and
backwages.
The normal consequences of respondents’ illegal dismissal, then, are reinstatement without loss of seniority rights,
and payment of backwages computed from the time compensation was withheld up to the date of actual
reinstatement. Where reinstatement is no longer viable as an option, separation pay equivalent to one (1) month
salary for every year of service should be awarded as an alternative. The payment of separation pay is in addition to
payment of backwages.39 (Emphasis and underscoring supplied)
The case of Golden Ace further provides:
"The accepted doctrine is that separation pay may avail in lieu of reinstatement if reinstatement is no longer practical
or in the best interest of the parties. Separation pay in lieu of reinstatement may likewise be awarded if the employee
decides not to be reinstated." x x x
Under the doctrine of strained relations, the payment of separation pay is considered an acceptable alternative to
reinstatement when the latter option is no longer desirable or viable. On one hand, such payment liberates the
employee from what could be a highly oppressive work environment.1âwphi1 On the other hand, it releases the
employer from the grossly unpalatable obligation of maintaining in its employ a worker it could no longer trust.40
In the present case, the NLRC found that due to the strained relations between the parties, separation pay is to be
awarded to Johnson in lieu of his reinstatement.
The NLRC held that Johnson is entitled to backwages from November 3, 2007 up to the finality of the decision;
separation pay equivalent to one month salary; and unpaid salaries from August 1, 2007 to November 1, 2007
amounting to a total of ₱172,800.00.41
While the Court agrees with the NLRC that the award of separation pay and unpaid salaries is warranted, the Court
does not lose sight of the fact that the employment contract states that Johnson's employment is for a term of three
years.
Accordingly, the award of backwages should be computed from November 3, 2007 to August 1, 2010 - which is three
years from August 1, 2007. Furthermore, separation pay is computed from the commencement of employment up to
the time of termination, including the imputed service for which the employee is entitled to backwages.42 As one-
month salary is awarded as separation pay for every year of service, including imputed service, Johnson should be
paid separation pay equivalent to his three-month salary for the three-year contract.

Angeles| Bajana | Balladares | Brillantes | Briones | Cabansag | Callanta | Chua | David|


De Leon | Gomez | Lopez | Macalino | Nostratis | Padilla | Reynon | Santos | Tan |Velasco
4E / 4F - 2018-2019
Page 680 of 920
LABOR REVIEW DIGEST
Atty. Joyrich Golangco

3. GRAND ASIAN SHIPPING LINES, INC., EDUARDO P. FRANCISCO and WILLIAM HOW vs.
WILFREDO GALVEZ, JOEL SALES, CRISTITO GRUTA, DANILO ARGUELLES, RENATO BATAYOLA,
PATRICIO FRESMILLO,* JOVY NOBLE, EMILIO DOMINICO, BENNY NILMAO, and JOSE AUSTRAL
GR No: 178184 Date: January 29, 2014
Ponente: DEL CASTILLO, J.

Doctrine: The employer has broader discretion in dismissing managerial employees on the ground of loss of
trust and confidence than those occupying ordinary ranks. While plain accusations are not sufficient to
justify the dismissal of rank and file employees, the mere existence of a basis for believing that managerial
employees have breached the trust reposed on them by their employer would suffice to justify their
dismissal.

FACTS:

Petitioner Grand Asian Shipping Lines, Inc. (GASLI) is a domestic corporation engaged in transporting liquified
petroleum gas (LPG) from Petron Corporation’s refinery in Limay, Bataan to Petron’s Plant in Ugong, Pasig and
Petron’s Depot in Rosario, Cavite. Petitioners William How and Eduardo Francisco are its President and General
Manager, respectively. Respondents, on the other hand, are crewmembers of one of GASLI’s vessels, M/T Dorothy
Uno, with the following designations: Wilfredo Galvez (Galvez) as Captain; Joel Sales (Sales) as Chief Mate; Cristito
Gruta (Gruta) as Chief Engineer; Danilo Arguelles (Arguelles) as Radio Operator; Renato Batayola (Batayola),
Patricio Fresmillo (Fresmillo) and Jovy Noble (Noble) as Able Seamen; Emilio Dominico (Dominico) and Benny
Nilmao (Nilmao) as Oilers; and Jose Austral (Austral) as 2nd Engineer.

Sometime in January 2000, one of the vessel’s Oilers, Richard Abis (Abis), reported to GASLI’s Office and Crewing
Manager, Elsa Montegrico (Montegrico), an alleged illegal activity being committed by respondents aboard the
vessel. Abis revealed that after about four to five voyages a week, a substantial volume of fuel oil is unconsumed and
stored in the vessel’s fuel tanks. However, Gruta would misdeclare it as consumed fuel in the Engineer’s Voyage
Reports. Then, the saved fuel oil is siphoned and sold to other vessels out at sea usually at nighttime. Respondents
would then divide among themselves the proceeds of the sale. Abis added that he was hesitant at first to report
respondents’ illegal activities for fear for his life.

An investigation on the alleged pilferage was conducted. After audit and examination of the Engineer’s Voyage
Reports, GASLI’s Internal Auditor, Roger de la Rama (De la Rama), issued a Certification of Overstatement of Fuel
Oil Consumption9 for M/T Dorothy Uno stating that for the period June 30, 1999 to February 15, 2000 fuel oil
consumption was overstated by 6,954.3 liters amounting to ₱74,737.86.10
On February 11, 2000, a formal complaint11 for qualified theft was filed with the Criminal Investigation and Detection
Group (CIDG) at Camp Crame against respondents
In a letter20 dated April 14, 2000, the CIDG referred the case to the Office of the City Prosecutor of Manila, which,
after finding a prima facie case, filed the corresponding Information for Qualified Theft21 dated August 18, 2000 with
the Regional Trial Court (RTC) of Manila.
Meanwhile, GASLI placed respondents under preventive suspension. After conducting administrative hearings,
petitioners decided to terminate respondents from employment. Respondents (except Sales) were thus served with
notices22 informing them of their termination for serious misconduct, willful breach of trust, and commission of a
crime or offense against their employer.
Labor Arbiter rendered a Decision26 finding the dismissal of all 21 complainants illegal. As regards the dismissal of
herein respondents, the Labor Arbiter ruled that the filing of a criminal case for qualified theft against them did not
justify their termination from employment.

The NLRC found the appeal meritorious and ruled that petitioners presented sufficient evidence to show just causes
for terminating complainants’ employment and compliance with due process. Accordingly, complainants’ dismissal
was valid, with the exception of Sales. The NLRC adjudged petitioners to have illegally dismissed Sales as there was
absence of any record that the latter received any notice of suspension, administrative hearing, or termination.

The CA conformed with the Labor Arbiter’s ruling that petitioners’ evidence was inadequate to support the charge of
pilferage and justify respondents’ termination.

ISSUE/S:
Whether or not respondents were illegally dismissed.

HELD:

Angeles| Bajana | Balladares | Brillantes | Briones | Cabansag | Callanta | Chua | David|


De Leon | Gomez | Lopez | Macalino | Nostratis | Padilla | Reynon | Santos | Tan |Velasco
4E / 4F - 2018-2019
Page 681 of 920
LABOR REVIEW DIGEST
Atty. Joyrich Golangco

Galvez and Gruta were validly dismissed on the ground of loss of trust and confidence; there were no valid
grounds for the dismissal of Arguelles, Batayola, Fresnillo, Noble, Dominico, Nilmao and Austral.
As for respondents’ termination, which is loss of trust and confidence, distinction should be made between
managerial and rank and file employees. "[W]ith respect to rank-and-file personnel, loss of trust and confidence, as
ground for valid dismissal, requires proof of involvement in the alleged events x x x [while for] managerial employees,
the mere existence of a basis for believing that such employee has breached the trust of his employer would suffice
for his dismissal."55
In the case before us, Galvez, as the ship captain, is considered a managerial employee since his duties involve the
governance, care and management of the vessel.56 Gruta, as chief engineer, is also a managerial employee for he
is tasked to take complete charge of the technical operations of the vessel.57 As captain and as chief engineer,
Galvez and Gruta perform functions vested with authority to execute management policies and thereby hold positions
of responsibility over the activities in the vessel. Indeed, their position requires the full trust and confidence of their
employer for they are entrusted with the custody, handling and care of company property and exercise authority over
it.
Thus, we find that there is some basis for the loss of confidence reposed on Galvez and Gruta. The
certification issued by De la Rama stated that there is an overstatement of fuel consumption. Notably, while
respondents made self-serving allegations that the computation made therein is erroneous, they never questioned
the competence of De la Rama to make such certification. Neither did they question the authenticity and validity of
the certification. Thus, the fact that there was an overstatement of fuel consumption and that there was loss of a
considerable amount of diesel fuel oil remained unrefuted. Their failure to account for this loss of company property
betrays the trust reposed and expected of them. They had violated petitioners’ trust and for which their dismissal is
justified on the ground of breach of confidence.
As for Arguelles, Batayola, Fresnillo, Noble, Dominico, Nilmao and Austral, proof of involvement in the loss
of the vessel’s fuel as well as their participation in the alleged theft is required for they are ordinary rank and
file employees. And as discussed above, no substantial evidence exists in the records that would establish their
participation in the offense charged. This renders their dismissal illegal, thus, entitling them to reinstatement plus full
backwages, inclusive of allowances and other benefits, computed from the time of their dismissal up to the time of
actual reinstatement.
No evidence of Sales’ dismissal from employment.
The rule that the employer bears the burden of proof in illegal dismissal cases finds no application when the
employer denies having dismissed the employee.58 The employee must first establish by substantial evidence the
fact of dismissal59 before shifting to the employer the burden of proving the validity of such dismissal.
We give credence to petitioners’ claim that Sales was not dismissed from employment. Unlike the other respondents,
we find no evidence in the records to show that Sales was preventively suspended, that he was summoned and
subjected to any administrative hearing and that he was given termination notice. From the records, it appears Sales
was not among those preventively suspended on February 26, 2000. To bolster this fact, petitioners presented the
Payroll Journal Register for the period March 1-15, 200060 showing that Sales was still included in the payroll and
was not among those who were charged with an offense to warrant suspension. In fact, Sales’ signature in the Semi-
Monthly Attendance Report for February 26, 2000 to March 10, 200061 proves that he continued to work as Chief
Mate for the vessel M/T Dorothy Uno along with a new set of crewmembers. It is likewise worth noting that in the
Supplemental Complaint Affidavit of Montegrico, Sales was not included in the list of those employees who were
accused of having knowledge of the alleged pilferage. This only shows that he was never subjected to any
accusation or investigation as a prelude to termination. Hence, it would be pointless to determine the legality or
illegality of his dismissal because, in the first place, he was not dismissed from employment.
P.S.
Arguelles, Renato Batayola, Patricio Fresmillo, Jovy Noble, Emilio Dominico, Benny Nilmao, and Jose Austral are
DECLARED to have been illegally dismissed; hence, petitioners are ordered to reinstate them to their former position
or its equivalent without loss of seniority rights and to pay them full backwages, inclusive of allowances and other
benefits, computed from the time of dismissal up to the time of actual reinstatement, as well as 13th month pay for
the period of their illegal dismissal.
Petitioner Grand Asian Shipping Lines, Inc. is also ordered to pay respondents Wilfredo Galvez, Danilo Arguelles,
Renato Batayola, Patricio Fresnillo, Jovy Noble, Emilio Dominico, Benny Nilmao and Jose Austral unpaid salaries
from February 16 to 29, 2000, as computed by the Labor Arbiter; and to pay respondents Danilo Arguelles, Renato
Batayola, Patricio Fresmillo, Jovy Noble, Emilio Dominico and Benny Nilmao salary differentials plus double
indemnity, as computed by the Labor Arbiter. Ten percent (10%) of the monetary award should be added as and by
way of attorney’s fees. Interest at the rate of six percent (6%) per annum shall be imposed on all monetary awards
from date of finality of this Decision until full payment pursuant to Nacar v. Gallery Frames.78
Petitioners Eduardo P. Francisco and William How are absolved from the liability adjudged against petitioner Grand
Asian Shipping Lines, Inc.

Angeles| Bajana | Balladares | Brillantes | Briones | Cabansag | Callanta | Chua | David|


De Leon | Gomez | Lopez | Macalino | Nostratis | Padilla | Reynon | Santos | Tan |Velasco
4E / 4F - 2018-2019
Page 682 of 920
LABOR REVIEW DIGEST
Atty. Joyrich Golangco

Angeles| Bajana | Balladares | Brillantes | Briones | Cabansag | Callanta | Chua | David|


De Leon | Gomez | Lopez | Macalino | Nostratis | Padilla | Reynon | Santos | Tan |Velasco
4E / 4F - 2018-2019
Page 683 of 920
LABOR REVIEW DIGEST
Atty. Joyrich Golangco

4. MIRANT PHILIPPINES VS CARO


GR NO: 181490 DATE: APRIL 23, 2014
PONENTE: VILLARAMA, JR., J.

Doctrine: When a conflicting interest of labor and capital are weighed on the scales of social justice, the
heavier influence of the latter must be counter-balanced by the sympathy and compassion the law must
accord the underprivileged worker.

Company policies and regulations are generally valid and binding between the employer and the employee
unless shown to be grossly oppressive or contrary to law.

FACTS:
Petitioner corporation is a holding company that owns shares in Mirant Pagbilao. Respondent was an employee who
was terminated because of an alleged unjust refusal to submit himself to drug-testing.

The company conducted a random drug-testing wherein respondent was randomly as one of the participants.
However, on the day of the supposed drug-test, the respondent left the office due to an alleged emergency.

Respondent received a Show Cause Notice from petitioner corporation. He submitted a written explanation.
Thereafter, the Investigating Panel found respondent guilty of unjust refusal to submit to random drug testing.

Respondent filed a case for illegal dismissal.


LA RULING:
The LA ruled that the respondent was illegally dismissed.

NLRC RULING:
The NLRC affirmed the LA decision. Further, it stated that the offer of respondent to submit to another drug test the
following day, even at his expense, cannot operate to free him from liability. The respondent filed a motion for
reconsideration which was denied by the NLRC.

CA RULING:
Upon appeal to the CA it held that the singular fact material to this case was that respondent did not get himself
tested in clear disobedience of company instructions and policy. Despite such disobedience, however, the appellate
court considered the penalty of dismissal to be too harsh to be imposed on respondent. Thus, respondent was
illegally dismissed.

APPEAL TO THE SC:

Petitioner's Contention:

To the mind of petitioners, they are not liable for illegal dismissal because all the circumstances to prove that
respondent really eluded the random drug test were present and the respondent was therefore validly terminated for
cause after being properly accorded with due process.

ISSUE/S:
1. Was the respondent validly terminated from employment?

HELD:
1. No. While it is a management prerogative to terminate its erring employee for willful disobedience, the SC has
recognized that such penalty is too harsh depending on the circumstances of each case. "There must be
reasonable proportionality between, on the one hand, the willful disobedience by the employee and, on the other
hand, the penalty imposed therefor" x x x.

In this case, [petitioner corporation’s] own investigating panel has revealed that the penalty of dismissal is too harsh
to impose on [respondent], considering that this was the first time in his 10-year employment that the latter violated
its company policies. The investigating panel even suggested that a review be had of the company policy on the term
"unjustified refusal" to clearly define what constitutes a violation thereof.

Angeles| Bajana | Balladares | Brillantes | Briones | Cabansag | Callanta | Chua | David|


De Leon | Gomez | Lopez | Macalino | Nostratis | Padilla | Reynon | Santos | Tan |Velasco
4E / 4F - 2018-2019
Page 684 of 920
LABOR REVIEW DIGEST
Atty. Joyrich Golangco

The fact that petitioner corporation’s own Investigating Panel and its Vice President for Operations, Sliman, differed
in their recommendations regarding respondent’s case are first-hand proof that there, indeed, is ambiguity in the
interpretation and application of the subject drug policy.

It is not a mere jurisprudential principle, but an enshrined provision of law, that all doubts shall be resolved in favor of
labor. Applying this to the circumstances in the case at bar, it is not fair for this Court to allow an ambiguous policy to
prejudice the rights of an employee against illegal dismissal. To hold otherwise and sustain the stance of petitioner
corporation would be to adopt an interpretation that goes against the very grain of labor protection in this jurisdiction.
As correctly stated by the Labor Arbiter, "when a conflicting interest of labor and capital are weighed on the scales of
social justice, the heavier influence of the latter must be counter-balanced by the sympathy and compassion the law
must accord the underprivileged worker."

The penalty of termination imposed by petitioner corporation upon respondent fell short of being reasonable.
Company policies and regulations are generally valid and binding between the employer and the employee unless
shown to be grossly oppressive or contrary to law – as in the case at bar.

The [petitioner corporation’s] Anti-Drug Policy is excessive in terminating an employee for his "unjustified refusal" to
subject himself to the random drug test on first offense, without clearly defining what amounts to an "unjustified
refusal."

The SC recommended 4 working weeks’ suspension without pay as the reasonable penalty to be imposed on
[respondent] for his disobedience.

Angeles| Bajana | Balladares | Brillantes | Briones | Cabansag | Callanta | Chua | David|


De Leon | Gomez | Lopez | Macalino | Nostratis | Padilla | Reynon | Santos | Tan |Velasco
4E / 4F - 2018-2019
Page 685 of 920
LABOR REVIEW DIGEST
Atty. Joyrich Golangco

5. BLUER THAN BLUE JOINT VENTURES VS ESTEBAN


GR NO: 192582 DATE: APRIL 7, 2014
PONENTE: REYES, J.

Doctrine: With respect to rank-and-file personnel, loss of trust and confidence as ground for valid dismissal
requires proof of involvement in the alleged events in question, and that mere uncorroborated assertions
and accusations by the employer will not be sufficient.

FACTS:
Respondent was a sales clerk at petitioner’s EGG boutique at SM Marilao, Bulacan. Petitioner found out that some of
its employees including respondent had access to its point-of-sale (POS) system thru a universal password. Upon
imvestigation, respondent was dismissed from employment on the ground of loss of trust and confidence.
Respondent filed a complaint for illegal dismissal.

LA RULING:
The LA ruled that the respondent was illegally dismissed.

NLRC RULING:
The NLRC reversed the LA’s decision and dismissed the complaint for illegal dismissal for lack of merit.

CA RULING:
The CA reversed the NLRC decision and reinstated the LA decision.

APPEAL TO THE SC:

Petitioner's Contention:
The petitioner argues that it had just cause to terminate the employment of Esteban, that is, loss of trust and
confidence. Esteban, the petitioner believes, is a rank-and-file employee whose nature of work is reposed with trust
and confidence. Her unauthorized access to the POS system of the company and her dissemination of the
unauthorized password, which Esteban admitted, is a breach of trust and confidence, and justifies her dismissal.

Respondent's Contention:
Respondent avers that her main task is that of a sales clerk and not of a cashier. Hence, her position is not that of
trust and confidence.

ISSUE/S:
1. Do the acts of respondent constitute just cause to terminate her employment with the company on the ground of
loss of trust and confidence.

HELD:
1. No. Loss of trust and confidence is premised on the fact that the employee concerned holds a position of
responsibility, trust and confidence. The employee must be invested with confidence on delicate matters, such
as the custody, handling, care and protection of the employer’s property and funds. “With respect to rank-and-
file personnel, loss of trust and confidence as ground for valid dismissal requires proof of involvement in the
alleged events in question, and that mere uncorroborated assertions and accusations by the employer will not be
sufficient."

Loss of trust and confidence to be a valid cause for dismissal must be work related such as would show the
employee concerned to be unfit to continue working for the employer and it must be based on a willful breach of trust
and founded on clearly established facts. Such breach is willful if it is done intentionally, knowingly, and purposely,
without justifiable excuse as distinguished from an act done carelessly, thoughtlessly, heedlessly or inadvertently.
The loss of trust and confidence must spring from the voluntary or willful act of the employee, or by reason of some
blameworthy act or omission on the part of the employee.
In this case, the SC found that the acts committed by respondent do not amount to a willful breach of trust. The SC is
not saying that respondent is innocent of any breach of company policy. That she relayed the password to another
employee is likewise demonstrative of her mindless appreciation of her duties as a sales clerk in the petitioner’s
employ. But absent any showing that her acts were done with "moral perverseness" that would justify the claimed
loss of trust and confidence attendant to her job, the Court must sustain the conclusion that respondent was illegally
dismissed. As stated by the CA, "suspension would have sufficed as punishment, considering that the petitioner had
already been with the company for more than 2 years, and the petitioner apologized and readily admitted her mistake

Angeles| Bajana | Balladares | Brillantes | Briones | Cabansag | Callanta | Chua | David|


De Leon | Gomez | Lopez | Macalino | Nostratis | Padilla | Reynon | Santos | Tan |Velasco
4E / 4F - 2018-2019
Page 686 of 920
LABOR REVIEW DIGEST
Atty. Joyrich Golangco

in her written explanation, and considering that no clear and convincing evidence of loss or prejudice, which was
suffered by the [petitioner] from [Esteban’s] supposed infraction."

Angeles| Bajana | Balladares | Brillantes | Briones | Cabansag | Callanta | Chua | David|


De Leon | Gomez | Lopez | Macalino | Nostratis | Padilla | Reynon | Santos | Tan |Velasco
4E / 4F - 2018-2019
Page 687 of 920
LABOR REVIEW DIGEST
Atty. Joyrich Golangco

6. MANILA JOCKEY CLUB (MJCI) VS TRAJANO


GR NO. 160982 DATE: JUNE 26, 2013
PONENTE: BERSAMIN, J.

Doctrine:
To dismiss an employee based on speculation as to the damage the employer could have suffered would be
an injustice.

The loss of confidence should not be simulated in order to justify what would otherwise be, under the
provisions of law, an illegal dismissal. It should not be used as a subterfuge for causes which are illegal,
improper and unjustified. It must be genuine, not a mere afterthought to justify an earlier action taken in bad
faith.

FACTS:
MJCI had employed Trajano as a selling teller of betting tickets. She was placed under preventive suspension due to
a ticket cancellation incident. Trajano instituted a complaint for illegal dismissal against MJCI when she was no
longer permitted to work after the end of the preventive suspension.

LA RULING:
The LA dismissed the complaint for illegal dismissal upon finding that Trajano's gross negligence in the performance
of her job warranted the termination of her employment. |

NLRC RULING:
The NLRC reversed and set aside the LA decision and declared Trajano to have been illegally dismissed by MJCI
without just or authorized cause and without due process of law. It concluded that her cancellation of the ticket was
an honest mistake that did not constitute a serious misconduct or willful disobedience of the lawful orders of her
employer; that such cancellation did not amount to a gross and habitual neglect of duty because her mistake was
only her first offense in the nine years of service to MJCI; and that MJCI sustained no damage.

CA RULING:
The CA upheld the NLRC.

APPEAL TO THE SC:

Petitioner's Contention:
MJCI posits that Trajano held a position of trust and confidence and that the act of canceling the ticket was
unauthorized because it was done without the consent of the bettor. MJCI further contends that the unauthorized
cancellation of the ticket could have greatly prejudiced MJCI for causing damage to both its income and reputation.

Respondent's Contention
Trajano claims that he did not commit any gross dishonesty or any serious misconduct or habitual neglect of duties,
because what she committed was purely an honest mistake that did not merit the imposition of the penalty of
dismissal from the service.|

ISSUE/S:
1. Whether or not there was just cause when MJCI dismissed Trajano from the service.

HELD:
1. No. The valid termination of an employee may either be for just causes or for authorized causes under
the Labor Code.
Specifically, loss of the employer's trust and confidence is a just cause under Article 282 (c), a provision that
ideally applies only to cases involving an employee occupying a position of trust and confidence or to a situation
where the employee has been routinely charged with the care and custody of the employer's money or property.
As the records indicate, MJCI's prejudice remained speculative and unrealized. To dismiss an employee
based on speculation as to the damage the employer could have suffered would be an injustice. The injustice in the
case of Trajano would be greater if the supposed just cause for her dismissal was not even sufficiently established.
While MJCI as the employer understandably had its own interests to protect, and could validly terminate any
employee for a just cause, its exercise of the power to dismiss should always be tempered with compassion and
imbued with understanding, avoiding its abuse

Angeles| Bajana | Balladares | Brillantes | Briones | Cabansag | Callanta | Chua | David|


De Leon | Gomez | Lopez | Macalino | Nostratis | Padilla | Reynon | Santos | Tan |Velasco
4E / 4F - 2018-2019
Page 688 of 920
LABOR REVIEW DIGEST
Atty. Joyrich Golangco

In this case, as a selling teller, Trajano held a position of trust and confidence. Although the act complained
of — the unauthorized cancellation of the ticket (i.e.,unauthorized because it was done without the consent of the
bettor) — was related to her work as a selling teller, MJCI did not establish that the cancellation of the ticket was
intentional, knowing and purposeful on her part in order for her to have breached the trust and confidence
reposed in her by MJCI, instead of being only out of an honest mistake.||
The SC noted that the invocation of loss of trust and confidence as a ground for dismissing Trajano was made
belatedly. Such a belated invocation of loss of confidence broadly hints the ground as a mere afterthought to buttress
an otherwise baseless dismissal of the employee.

Angeles| Bajana | Balladares | Brillantes | Briones | Cabansag | Callanta | Chua | David|


De Leon | Gomez | Lopez | Macalino | Nostratis | Padilla | Reynon | Santos | Tan |Velasco
4E / 4F - 2018-2019
Page 689 of 920
LABOR REVIEW DIGEST
Atty. Joyrich Golangco

7. REXIE A. HORMILLOSA vs. COCA-COLA BOTTLERS PHILS., INC., represented by its Iloilo Plant Human
Resource Head, ROBERTO RICHARD H. DOLAR
GR No.: 198699 Date: OCTOBER 9, 2013
Ponente: Mendoza, J.

DOCTRINE:
The first requisite for dismissal on the ground of loss of trust and confidence is that the employee
concerned must be one holding a position of trust and confidence. The second requisite is that there must
be an act that would justify the loss of trust and confidence. It must be based on a willful breach of trust and
founded on clearly established facts.

FACTS:

In 1996, Hormillosa was employed as a route salesman by Coca-Cola Bottlers Phils., Inc. (CBPI). His duties
included, among others, selling CBPI’s soft drink products, either on cash or on credit basis; receiving payments from
proceeds of the sale or payments of past due or current accounts; issuing sales invoices; and receiving empty bottles
and cases of soft drinks. Due to their delicate position, route salesmen, like Hormillosa, were given a handbook
entitled, CCBPI Employee Code of Disciplinary Rules and Regulations.

In 1999, the then CBPI District Sales Supervisor, Raul S. Tiosayco III, conducted a verification and audit of the
accounts handled by Hormillosa. He discovered transactions in violation of the handbook, specifically "Fictitious sales
transactions; Falsification of company records, etc., among others. Hormillosa was then placed on grounded status
and investigated. He was subsequently terminated. In addition to his termination, CBPI also filed several criminal
cases against him citing his fraudulent acts. Even after the termination, more anomalies committed by Hormillosa
were discovered.

Hormillosa averred in his position paper that prior to his dismissal, he was a member of the Board of Directors of
CBPI’s employees union and he became its secretary. As secretary, he sent a copy of the new list of union officers to
the management with a warning that if CBPI would not stop harassing the members of the union, it would declare a
strike.

LA RULING: dismissed Hormillosa’s complaint for illegal dismissal, ruling that his termination was proper.

NLRC RULING: ordered the remand of the case to give Hormillosa the opportunity to confront the witnesses and
evidence against him.

CA RULING: nullified and set aside the NLRC decision and held that the dismissal of Hormillosa was valid.

ISSUE/S:

Whether Hormillosa was validly dismissed.

HELD:

YES. Article 282 of the Labor Code enumerates the just causes for the termination of employment of an employee by
the employer. The CA was correct when it ruled that Hormillosa’s employment was validly terminated under
paragraph (c). There was substantial evidence to justify his dismissal. It is clear that Article 282(c) of the Labor Code
allows an employer to terminate the services of an employee for loss of trust and confidence.

The first requisite for dismissal on the ground of loss of trust and confidence is that the employee concerned must be
one holding a position of trust and confidence.

There are two classes of positions of trust. The first class consists of managerial employees. They are defined as
those vested with the powers or prerogatives to lay down management policies and to hire, transfer suspend, lay-off,
recall, discharge, assign or discipline employees or effectively recommend such managerial actions.
The second class consists of cashiers, auditors, property custodians, etc. They are defined as those who in the
normal and routine exercise of their functions,
regularly handle significant amounts of money or property.

Angeles| Bajana | Balladares | Brillantes | Briones | Cabansag | Callanta | Chua | David|


De Leon | Gomez | Lopez | Macalino | Nostratis | Padilla | Reynon | Santos | Tan |Velasco
4E / 4F - 2018-2019
Page 690 of 920
LABOR REVIEW DIGEST
Atty. Joyrich Golangco

The second requisite is that there must be an act that would justify the loss of trust and confidence. It must be based
on a willful breach of trust and founded on clearly established facts. The basis for the dismissal must be clearly and
convincingly established but proof beyond reasonable doubt is not necessary.

Hormillosa, being a route salesman, falls under the second class.


Salesmen are highly individualistic personnel who have to be trusted and left essentially on their own. A high degree
of confidence is reposed on them because they are entrusted with funds or properties of their employer. Clearly,
Hormillosa occupies a position of trust. Hormillosa committed acts which warranted his dismissal from employment.
The falsified invoices remained unexplained by him. Evidently, Hormillosa disregarded and disobeyed the company
rules. The breach was willful. The tampering of the invoices were clear enough.

In the case at bench, the cause for the dismissal from employment of Hormillosa clearly falls under Article 282 of the
Labor Code. Therefore, he is not entitled to any separation pay.

Angeles| Bajana | Balladares | Brillantes | Briones | Cabansag | Callanta | Chua | David|


De Leon | Gomez | Lopez | Macalino | Nostratis | Padilla | Reynon | Santos | Tan |Velasco
4E / 4F - 2018-2019
Page 691 of 920
LABOR REVIEW DIGEST
Atty. Joyrich Golangco

8. ERIC V. CHUANICO vs. LEGACY CONSOLIDATED PLANS, INC.


GR No.: 181852 Date: OCTOBER 9, 2013
Ponente: Abad, J.

DOCTRINE:

Settled is the rule, however, that under Article 282(c) of the Labor Code, the breach of trust must be willful.
Ordinary breach will not be enough. A breach is willful if it is done intentionally and knowingly without any
justifiable excuse, as distinguished from an act done carelessly, thoughtlessly or inadvertently.

FACTS:

Legacy Plans hired petitioner Atty. Chuanico as Asst Vice- President for legal services. He was to serve as in-house
counsel for the company and its subsidiaries under the supervision Atty. Cruz, the Senior Vice-Pres for Legal Affairs.
In the same year, Legacy Plans merged with Consolidated Plans Philippines, Inc. to become Legacy Consolidated,
the respondent in this case. Its legal services unit served all its affiliates.

Subsequently Atty. Cruz wrote Atty. Chuanico a memorandum, requiring him to explain why no administrative action
should be taken against him for mishandling two cases.

First case: he was supposed to draft an answer to a complaint for Bank of East Asia (a Legacy Consolidated
affiliate) but he belatedly drafted a haphazard one that he gave to the handling lawyers without coursing it to his
superior.

His defense: that he was given only one day within which to finish the draft. While admitting that his superior had no
opportunity to review it for lack of time, he denied that the answer had been haphazardly done

Second case: Atty. Chuanico was required to prepare a complaint-affidavit for the Rural Bank of Parañaque (also an
affiliate) against a certain De Rama but he failed to do so.

His defense: that the case had not actually been turned over to him. It was originally assigned to Atty. Dennis
Amparo who later said that the complaint-affidavit could not be prepared because the Rural Bank had no witness.

Legacy Consolidated dismissed Atty. Chuanico with for serious misconduct, willful disobedience to lawful orders,
gross and habitual neglect of duties, and willful breach of trust. He filed complaint for illegal dismissal with claims for
his unpaid salary and 13th-month pay plus damages and attorney’s fees

LA RULING: Legacy Consolidated is guilty of illegal dismissal; entitled to full backwages, separation pay and 13th-
month pay
First charge: that Atty. Chuanico actually drafted an answer for Bank of East Asia but the company’s two new
lawyers did not like it and chose to file one that they themselves prepared; Legacy Consolidated neither bothered to
present Atty. Chuanico’s draft answer nor explained why it regarded the same as haphazardly done nor present
evidence that the bank filed a late answer on account of Atty. Chuanico’s fault.

Second charge: gave credence to Atty. Dennis Amparo’s sworn statement that it was to Atty. Cruz, not to Atty.
Chuanico, that he personally turned over the cases he was handling.

NLRC RULING: affirmed the LA’s Decision; Legacy Consolidated failed to present evidence to prove that Atty.
Chuanico violated some company rules or his superior’s order; employer gave him no notice of these alleged
violations.

CA RULING: NLRC committed grave abuse of discretion in holding Legacy guilty of illegal dismissal but affirmed, the
award to of13th-month pay. Atty. Chuanico moved for reconsideration but the CA denied his motion hence this
petition.

ISSUE/S:

Whether or not the CA erred in holding that the NLRC committed grave abuse of discretion in finding that Legacy
Consolidated illegally dismissed Atty. Chuanico for mishandling the two cases alleged to have been assigned to him.

Angeles| Bajana | Balladares | Brillantes | Briones | Cabansag | Callanta | Chua | David|


De Leon | Gomez | Lopez | Macalino | Nostratis | Padilla | Reynon | Santos | Tan |Velasco
4E / 4F - 2018-2019
Page 692 of 920
LABOR REVIEW DIGEST
Atty. Joyrich Golangco

HELD:

No. Atty. Chuanico was not a mere rank-and-file employee but an in-house counsel. Thus, Legacy Consolidated
enjoyed wide latitude in evaluating his work and attitude and in terminating his employment on the ground of loss of
trust and confidence. His mishandling of the cases assigned to him shows that he had been unfit to continue working
for his employer. But these are broad principles that do not themselves show when, where, and how Atty. Chuanico
betrayed the trust that Legacy Consolidated gave him as in-house counsel.

To be a valid cause for dismissal, the loss of trust must be based on a willful breach of such trust and founded on
clearly established facts. The company charged him with having mishandled two things that were assigned to him,
the drafting of an answer in one and the preparation of a complaint affidavit in the other. It failed to present proof,
however, of such mishandling.

In the first case: Legacy Consolidated did not bother to present the draft-answer Atty. Chuanico prepared and
demonstrate why it regarded the same as haphazardly done. Besides, as Atty. Chuanico said, he was given only one
day within which to finish the draft-answer and Legacy Consolidated did not contest this fact. Consequently, he could
not be expected to do more than an adequate pleading.
In the second case: Legacy Consolidated did not bother to present some note or logbook to refute this denial. It only
presented the sworn statement of the office secretary, supposedly competent, who relied merely on her memory for
ascertaining individual work assignments in a law practice that served a number of affiliated companies.

Besides, Atty. Amparo, the former handling lawyer of the Rural Bank case said in his sworn statement that he had
been unable to prepare the required complaint-affidavit because the bank could not produce a witness against De
Rama. Atty. Amparo further added that it was to Atty. Cruz, not to Atty. Chuanico, that he turned over the Rural
Bank's case.

The Court held in CAPANELA v. National Labor Relations Commission that the factual findings of quasi-judicial
bodies, which are triers of facts on matters within their expertise, should be considered, when supported by
substantial evidence, binding and conclusive on appellate courts.

Here the LA and the NLRC were in better positions to assess and evaluate the credibility of the parties' claims and
the weight to which the irrespective evidence is entitled.

Lastly Atty. Chuanico was dismissed due to willful breach of trust. Settled is the rule, however, that under Article 282(
c) of the Labor Code, the breach of trust must be willful. Ordinary breach will not be enough. A breach is willful if it is
done intentionally and knowingly without any justifiable excuse, as distinguished from an act done carelessly,
thoughtlessly or inadvertently. Willful breach was not proved in this case.

Angeles| Bajana | Balladares | Brillantes | Briones | Cabansag | Callanta | Chua | David|


De Leon | Gomez | Lopez | Macalino | Nostratis | Padilla | Reynon | Santos | Tan |Velasco
4E / 4F - 2018-2019
Page 693 of 920
LABOR REVIEW DIGEST
Atty. Joyrich Golangco

9. DUNCAN ASSOCIATION OF DETAILMAN-PTGWO and PEDRO A. TECSON vs. GLAXO WELLCOME


PHILIPPINES, INC.
GR No.: 162994 Date: SEPTEMBER 17, 2004
Ponente: Tinga, J.

DOCTRINE: The prohibition against personal or marital relationships with employees of competitor
companies upon Glaxo’s employees is reasonable under the circumstances because relationships of that
nature might compromise the interests of the company. In laying down the assailed company policy, Glaxo
only aims to protect its interests against the possibility that a competitor company will gain access to its
secrets and procedures.

FACTS:

Petitioner Tecson was hired by respondent Glaxo Wellcome Philippines, Inc. (Glaxo) as medical representative.He
signed a contract of employment which stipulates, among others, that he agrees to study and abide by existing
company rules; to disclose to management any existing or future relationship by consanguinity or affinity with co-
employees or employees of competing drug companies and should management find that such relationship poses a
possible conflict of interest, to resign from the company.

The Employee Code of Conduct of Glaxo similarly provides that if management perceives a conflict of interest or a
potential conflict between such relationship and the employee’s employment with the company, the management and
the employee will explore the possibility of a "transfer to another department in a noncounterchecking position" or
preparation for employment outside the company after six months.

Tecson was initially assigned to market Glaxo’s products in the Camarines Sur-Camarines Norte sales area.
Subsequently, Tecson entered into a romantic relationship with Bettsy, an employee of Astra Pharmaceuticals
(Astra), a competitor of Glaxo. Bettsy was Astra’s Branch Coordinator in Albay. She supervised the district managers
and medical representatives of her company and prepared marketing strategies for Astra in that area. Tecson and
bettsy got martied.

Tecson’s superiors informed him that his marriage to Bettsy gave rise to a conflict of interest. Tecson’s superiors
reminded him that he and Bettsy should decide which one of them would resign from their jobs, although they told
him that they wanted to retain him as much as possible because he was performing his job well. Tecson requested
for time to comply with the company policy against entering into a relationship with an employee of a competitor
company.

Glaxo transferred Tecson to the Butuan City-Surigao City-Agusan del Sur sales area. Tecson asked Glaxo to
reconsider its decision, but his request was denied. Tecson defied the transfer order and continued acting as medical
representative in the Camarines Sur- Camarines Norte sales area.

During the pendency of the grievance proceedings, Tecson was paid his salary, but was not issued samples of
products which were competing with similar products manufactured by Astra. He was also not included in product
conferences regarding such products.

For failure to resolve the issue at the grievance machinery level, they submitted the matter for voluntary arbitration.

NCMB RULING: declared as valid Glaxo’s policy on relationships between its employees and persons employed with
competitor companies, and affirming Glaxo’s right to transfer Tecson to another sales territory.

CA RULING: affirmed NCMB; held that it is a valid exercise of its management prerogatives. Petitioners contend that
Glaxo’s policy against employees marrying employees of competitor companies violates the equal protection clause
of the Constitution because it creates invalid distinctions among employees on account only of marriage claiming that
the policy restricts the employees’ right to marry

ISSUE/S:

Whether the Court of Appeals erred in ruling that Glaxo’s policy against its employees marrying employees from
competitor companies is valid, and in not holding that said policy violates the equal protectionclause of the
Constitution

Angeles| Bajana | Balladares | Brillantes | Briones | Cabansag | Callanta | Chua | David|


De Leon | Gomez | Lopez | Macalino | Nostratis | Padilla | Reynon | Santos | Tan |Velasco
4E / 4F - 2018-2019
Page 694 of 920
LABOR REVIEW DIGEST
Atty. Joyrich Golangco

HELD:

No. The stipulation in Tecson’s contract of employment with Glaxo being questioned by petitioners provides:

“10. You agree to disclose to management any existing or future relationship you may have, either by
consanguinity or affinity with co-employees or employees of competing drug companies.

Should it pose a possible conflict of interest in management discretion, you agree to resign voluntarily from
the Company as a matter of Company policy.”

The same contract also stipulates that Tescon agrees to abide by the existing company rules of Glaxo, and to study
and become acquainted with such policies. In this regard, the Employee Handbook of Glaxo expressly informs its
employees of its rules regarding conflict of interest:

“1. Conflict of Interest


Employees should avoid any activity, investment relationship, or interest that may run counter to the
responsibilities which they owe Glaxo Wellcome.

Specifically, this means that employees are expected:

 To avoid having personal or family interest, financial or otherwise, in any competitor supplier or
other businesses which may consciously or unconsciously influence their actions or decisions and thus
deprive Glaxo Wellcome of legitimate profit.

1.1. Employee Relationships

Employees with existing or future relationships either by consanguinity or affinity with


coemployees of competing drug companies are expected to disclose such relationship to
the Management. If management perceives a conflict or potential conflict of interest, every
effort shall be made, together by management and the employee, to arrive at a solution
within six (6) months, either by transfer to another department in a non-counter checking
position, or by career preparation toward outside employment after Glaxo Wellcome.
Employees must be prepared for possible resignation within six (6) months, if no other
solution is feasible.”

Glaxo has a right to guard its trade secrets, manufacturing formulas, marketing strategies and other confidential
programs and information from competitors, especially so that it and Astra are rival companies in the highly
competitive pharmaceutical industry.

The prohibition against personal or marital relationships with employees of competitor companies upon Glaxo’s
employees is reasonable under the circumstances because relationships of that nature might compromise the
interests of the company. In laying down the assailed company policy, Glaxo only aims to protect its interests against
the possibility that a competitor company will gain access to its secrets and procedures.

That Glaxo possesses the right to protect its economic interests cannot be denied. No less than the Constitution
recognizes the right of enterprises to adopt and enforce such a policy to protect its right to reasonable returns on
investments and to expansion and growth.

The challenged company policy does not violate the equal protection clause. Significantly, the company actually
enforced the policy after repeated requests to the employee to comply with the policy. Indeed, the application of the
policy was made in an impartial and even-handed manner, with due regard for the lot of the employee.
In any event, it is clear that Glaxo does not impose an absolute prohibition against relationships between its
employees and those of competitor companies. Its employees are free to cultivate relationships with and marry
persons of their own choosing. What the company merely seeks to avoid is a conflict of interest between the
employee and the company that may arise out of such relationships.

Angeles| Bajana | Balladares | Brillantes | Briones | Cabansag | Callanta | Chua | David|


De Leon | Gomez | Lopez | Macalino | Nostratis | Padilla | Reynon | Santos | Tan |Velasco
4E / 4F - 2018-2019
Page 695 of 920
LABOR REVIEW DIGEST
Atty. Joyrich Golangco

Further, Employee Code of Conduct and of its contracts with its employees, such as that signed by Tescon, was
made known to him prior to his employment. Tecson, therefore, was aware of that restriction when he signed his
employment contract and when he entered into a relationship with Bettsy. Since Tecson knowingly and voluntarily
entered into a contract of employment with Glaxo, the stipulations therein have the force of law between them and,
thus, should be complied with in good faith. He is therefore estopped from questioning said policy.

Angeles| Bajana | Balladares | Brillantes | Briones | Cabansag | Callanta | Chua | David|


De Leon | Gomez | Lopez | Macalino | Nostratis | Padilla | Reynon | Santos | Tan |Velasco
4E / 4F - 2018-2019
Page 696 of 920
LABOR REVIEW DIGEST
Atty. Joyrich Golangco

10. STAR PAPER CORPORATION VS SIMBOL


G.R. NO. 164774; APRIL 12, 2006
PONENTE: PUNO, J.,

DOCTRINE: In the Philippine jurisdiction, we employ the standard of reasonableness of the company policy
which is parallel to the bona fide occupational qualification requirement. This requirement of reasonableness
must be clearly established to uphold the questioned employment policy. The employer has the burden to
prove the existence of a reasonable business necessity.

FACTS:
Petitioner Star Paper Corporation is a corporation engaged in trading principally of paper products. Respondnets
Ronaldo D. Simbol (Simbol), Wilfreda N. Comia (Comia) and Lorna E. Estrella (Estrella) were all regular employees
of the company.

Respondents Simbol and Comia both married a co-employee in the petitioner company, and alleged that they were
compelled to resign from the company pursuant to a company policy that states “in case of two of our employees
(both singles [sic], one male and another female) developed a friendly relationship during the course of their
employment and then decided to get married, one of them should resign to preserve the policy stated above.”

As to respondent Estrella, she alleged that she had a relationship with co-worker Zulga who misrepresented himself
as a married but separated man. After he got her pregnant, she discovered that he was not separated. Thus, she
severed her relationship with him to avoid dismissal due to the company policy. However, she was still dismissed on
the ground of immoral conduct.

Respondents filed a complaint for unfair labor practice, constructive dismissal, separation pay and attorneys fees.
They averred that the aforementioned company policy is illegal and contravenes Article 134 (formerly 136) of the
Labor Code.

Petitioners allege that its policy may appear to be contrary to Article 134 of the Labor Code but it assumes a new
meaning if read together with the first paragraph of the rule. The rule does not require the woman employee to
resign. The employee spouses have the right to choose who between them should resign. Further, they are free to
marry persons other than co-employees. Hence, it is not the marital status of the employee, per se, that is being
discriminated. It is only intended to carry out its no-employment-for-relatives-within-the-third-degree-policy which is
within the ambit of the prerogatives of management.

LA/NLRC Ruling:
LA dismissed the complaint. LA opines that the company policy is a proper exercise management prerogative.

NLRC affirmed LA decision.

CA Ruling:
Court of Appeals reversed the NLRC decision and declared that the dismissal of respondents illegal and ordering
private respondents to reinstate petitioners to their former positions without loss of seniority rights with full backwages
from the time of their dismissal until actual reinstatement; and attorneys fees

ISSUE:
Whether or not the company policy is a proper exercise of management prerogative.

HELD:
NO. The questioned government policy is not a proper exercise of management prerogative and is in violation to
Article 134 of the Labor Code. In the Philippine jurisdiction, we employ the standard of reasonableness of the
company policy which is parallel to the bona fide occupational qualification requirement. This requirement of
reasonableness must be clearly established to uphold the questioned employment policy. The employer has the
burden to prove the existence of a reasonable business necessity.

The Court did not find a reasonable business necessity in the case at bar.

Petitioners failed to show how the marriage of Simbol, then a Sheeting Machine Operator, to Alma Dayrit, then an
employee of the Repacking Section, could be detrimental to its business operations. Neither did petitioners explain
how this detriment will happen in the case of Wilfreda Comia, then a Production Helper in the Selecting Department,

Angeles| Bajana | Balladares | Brillantes | Briones | Cabansag | Callanta | Chua | David|


De Leon | Gomez | Lopez | Macalino | Nostratis | Padilla | Reynon | Santos | Tan |Velasco
4E / 4F - 2018-2019
Page 697 of 920
LABOR REVIEW DIGEST
Atty. Joyrich Golangco

who married Howard Comia, then a helper in the cutter-machine. The policy is premised on the mere fear that
employees married to each other will be less efficient. If we uphold the questioned rule without valid justification, the
employer can create policies based on an unproven presumption of a perceived danger at the expense of an
employees right to security of tenure.

The failure of petitioners to prove a legitimate business concern in imposing the questioned policy cannot prejudice
the employees right to be free from arbitrary discrimination based upon stereotypes of married persons working
together in one company.
For failure of petitioners to present undisputed proof of a reasonable business necessity, we rule that the questioned
policy is an invalid exercise of management prerogative.

*(Example of reasonableness of the company policy which is parallel to the bona fide occupational qualification
requirement - Policy of a pharmaceutical company prohibiting its employees from marrying employees of any
competitor company. Company has a right to guard its trade secrets, manufacturing formulas, marketing strategies
and other confidential programs and information from competitors. (Duncan Association of Detailman-PTGWO and
Pedro Tecson v. Glaxo Wellcome Philippines, Inc))

Angeles| Bajana | Balladares | Brillantes | Briones | Cabansag | Callanta | Chua | David|


De Leon | Gomez | Lopez | Macalino | Nostratis | Padilla | Reynon | Santos | Tan |Velasco
4E / 4F - 2018-2019
Page 698 of 920
LABOR REVIEW DIGEST
Atty. Joyrich Golangco

11. ACE PROMOTION AND MARKETING CORPORATION vs REYNALDO URSABIA


G.R. NO. 171703 : September 22, 2006
YNARES-SANTIAGO, J.
DOCTRINE: Two elements must be present, namely: (1) the failure to report for work or absence without
valid or justifiable reason, and (2) a clear intention to sever the employer-employee relationship. The second
element is more determinative of the intent and must be evinced by overt acts. Mere absence, not being
sufficient, the burden of proof rests upon the employer to show that the employee clearly and deliberately
intended to discontinue his employment without any intention of returning.
FACTS: Petitioner Ace Promotion and Marketing Corporation, a company engaged in the promotion of various
consumer products, commodities, and goods, hired respondent Reynaldo Ursabia as a company driver assigned to
pick up the products of Nestle Philippines, Inc., for promotion and marketing.
On July 6, 2001, respondent failed to report for work. Petitioner issued a two (2) Memoranda with a subject of
“Violation of Company Rule” for alleged abandonment of work and “Damage on the vehicle” assigned to respondent.
Both memoranda required the respondent to explain why his services should not be terminated based on the
abovementioned grounds.
The first memorandum was served to the respondent personally but he refused acknowledge receipt of such. The
second memorandum was served through registered mail to respondent's last known address.
Thereafter, an anonymous note was discovered among the stocks of petitioner containing the words "(Good news)
be careful and save your (sic) life because there's a time to come everybody x x x will die." The examination
conducted by the PNP Crime Laboratory allegedly showed that the handwriting of respondent has significant
similarities with the said handwritten note
On August 6, 2011, Respondent went to petitioner's office and was served with a termination letter. Again,
respondent refused to receive the same, prompting petitioner serve it by registered mail to respondent's last known
address.
Meanwhile, the petitioner filed two criminal cases for Malicious Mischief and Grave Threats against the respondent.
Displeased with his termination, respondent filed a complaint for illegal dismissal and non-payment of other monetary
benefits.
Labor Arbiter: LA favored respondent declaring the complainant illegally dismissed from his employment because
no hearing was conducted to allow him (respondent) to confront petitioner's witnesses.
NLRC: Reversed LA’s decision. It ruled that the dismissal was valid because respondent is guilty of abandonment.
Court of Appeals: Set aside the decision of the NLRC and held that termination of respondent is not valid because
his failure to report for work for a single day did not constitute abandonment.
ISSUE/s: 1.Whether or not respondent was legally dismissed based on the ground cited by the petitioner;
2. Whether or not respondent was accorded procedural due process.
HELD: 1. NO, the grounds relied to by the petitioner – abandonment and destruction to property—are not sufficient
grounds to dismiss the respondent. To constitute a just and valid ground for dismissal, abandonment requires the
deliberate and unjustified refusal of the employee to resume his employment. Two elements must be present,
namely: (1) the failure to report for work or absence without valid or justifiable reason, and (2) a clear intention to
sever the employer-employee relationship. The second element is more determinative of the intent and must be
evinced by overt acts. Mere absence, not being sufficient, the burden of proof rests upon the employer to show that
the employee clearly and deliberately intended to discontinue his employment without any intention of returning.
In the instant case, the subsequent conduct of respondent after he failed to report for work on July 6, 2001, shows
that he had no intention to sever his employment with petitioner. Record shows that he went to work on July 9, 2001,
which enabled petitioner to personally serve him the memorandum of even date. While his act of loitering outside the
company premises cannot be considered as reporting for work, it shows an intention to make his services available
for petitioner. More importantly, he formally reported for work on August 6, 2001. All these show that respondent
never really wanted to quit his job. He may be guilty of going on absence without leave, but not abandonment
because the totality of his acts show a clear intention to return to work.
Likewise, the alleged damage on the company car assigned to respondent cannot justify his dismissal. Termination is
simply disproportionate to such infraction not only because the extent of the damage was never proved by petitioner
but more importantly, no substantial evidence was presented to establish the guilt of respondent. With regard to the
"anonymous note" purportedly written by the latter, petitioner failed to discharge the burden of proving that the same
was indeed a threat and that respondent was the author thereof.
The foregoing, notwithstanding, the Court finds that respondent should be dismissed for willful disobedience of the
memoranda issued by petitioner. To be validly dismissed on the ground of willful disobedience requires the
concurrence of at least two requisites: (1) the employee's assailed conduct must have been willful or intentional, the
willfulness being characterized by a wrongful and perverse attitude; and (2) the order violated must have been
reasonable, lawful, made known to the employee and must pertain to the duties which he had been engaged to
discharge.

Angeles| Bajana | Balladares | Brillantes | Briones | Cabansag | Callanta | Chua | David|


De Leon | Gomez | Lopez | Macalino | Nostratis | Padilla | Reynon | Santos | Tan |Velasco
4E / 4F - 2018-2019
Page 699 of 920
LABOR REVIEW DIGEST
Atty. Joyrich Golangco

In the instant case, the failure of respondent to answer the July 9 and 10, 2001 memoranda of petitioner is clearly
intentional. He reported to and loitered outside petitioner's premises but never made any oral or written reply to the
said memoranda. This shows respondent's wrongful and perverse attitude to defy the reasonable orders, which
undoubtedly pertain to his duties as an employee of petitioner.
Respondent had worked with petitioner for almost seven years yet he did not give the courtesy, if not gratitude due it
by complying with its directives and explaining his conduct either verbally or in writing
2. NO. Court finds that respondent was not afforded his procedural due process rights. In dismissing an employee,
the employer has the burden of proving that the former worker has been served two notices: (1) one to apprise him of
the particular acts or omissions for which his dismissal is sought, and (2) the other to inform him of his employer's
decision to dismiss him. The first notice must state that dismissal is sought for the act or omission charged against
the employee, otherwise, the notice cannot be considered sufficient compliance with the rules.
In the instant case, the just cause to terminate respondent was his willful disobedience to the July 9 and July 10,
2001 memoranda of petitioner. However, he was not given sufficient notice that his services will be terminated on
such grounds. Respondent defied two memoranda of petitioner. Hence, it is necessary that he be furnished with a
third memorandum informing him that his disobedience to the previous two memoranda may cause his dismissal.
Petitioner never sent a notice to respondent ordering him to explain his disobedience to the July 10, 2001
memorandum. Moreover, the final notice of termination of respondent failed to specify the ground for his dismissal. It
vaguely stated that he is being terminated for violation of company rules, which were not specified by petitioner.
The lack of statutory due process should not nullify the dismissal, or render it illegal, or ineffectual. However, the
employer should indemnify the employee for violation of his statutory rights.

Angeles| Bajana | Balladares | Brillantes | Briones | Cabansag | Callanta | Chua | David|


De Leon | Gomez | Lopez | Macalino | Nostratis | Padilla | Reynon | Santos | Tan |Velasco
4E / 4F - 2018-2019
Page 700 of 920
LABOR REVIEW DIGEST
Atty. Joyrich Golangco

12.CONCEPCION VS MINEX IMPORT CORPORATION/MINERAMA CORPORATION


G.R. NO. 153569; JANUARY 24, 2012
BERSAMIN, J.
DOCTRINE: The employer may validly dismiss for loss of trust and confidence an employee who commits an
act of fraud prejudicial to the interest of the employer. Neither a criminal prosecution nor a conviction
beyond reasonable doubt for the crime is a requisite for the validity of the dismissal. Nonetheless, the
dismissal for a just or lawful cause must still be made upon compliance with the requirements of due
process under the Labor Code; otherwise, the employer is liable to pay nominal damages as indemnity to the
dismissed employee.
FACTS: Respondent Minex Import-Export Corporation (Minex) engaged in the retail2 of semi-precious stones, selling
them in kiosks or stalls installed in various shopping centers within Metro Manila. It employed the petitioner, Lolita
Conception, initially as a salesgirl rotating her assignment among nearly all its outlets, until she was made a superior.
Respondent Vina Mariano, an Assistant Manager of Minex, assigned the petitioner to the SM Harrison Plaza kiosk
with the instruction to hold the keys of the kiosk. Working under her supervision there were salesgirls Cristina Calung
and Lida Baquilar.
On November 9, 1997, a Sunday, the petitioner and her salesgirls had sales of crystal items totaling P39,194.50. At
the close of business that day, they conducted a cash-count of their sales proceeds, including those from the
preceding Friday and Saturday, and determined their total for the three days to be P50,912.00. The petitioner
wrapped the amount in a plastic bag and deposited it in the drawer of the locked wooden cabinet of the kiosk.
At about 9:30 am of November 10, 1997, the petitioner phoned Vina Mariano to report that the P50,912.00 was
missing. Later, while the petitioner was giving a detailed statement on the theft to the security investigator of Harrison
Plaza, Vina and Sylvia Mariano, her superiors, arrived with a policeman who immediately placed the petitioner under
arrest and brought her to Precinct 9 of the Malate Police Station. There, the police investigated her. She was
detained for a day.
Thereafter, the petitioner complained against the respondents for illegal dismissal in the Department of Labor and
Employment. Minex, through Vina, filed a complaint for qualified theft against the petitioner in the Office of the City
Prosecutor in Manila. Vina declared that the petitioner did not call the office of Minex for the pick-up of
the P39,194.50 cash sales on Sunday, in violation of the standard operating procedure (SOP).
After the preliminary investigation, the Assistant Prosecutor rendered a resolution finding probable cause for qualified
theft. Thus, she was charged with qualified theft.
Labor Arbiter: declared that the dismissal of the latter from work illegal and ordering her reinstatement to her former
work position with full backwages.
NLRC: reversed the decision of the Labor Arbiter declaring that the petitioner had not been dismissed, but had
abandoned her job after being found to have stolen the proceeds of the sales; and holding that even if she had been
dismissed, her dismissal would be justifiable for loss of trust and confidence
Court of Appeals: CA sustained the NLRC mainly because of the DOJ Secretary’s finding of probable cause for
qualified theft Petitioner contends that there is no lawful cause for her dismissal since there is no direct evidence that
the Petitioner took the money from the drawer in the cabinet in the Kiosk. That such evidence is required for the
petitioner to be lawfully dismissed for the loss of the money of the Private Respondent corporation. Therefore,
considering that she had not yet been found guilty beyond reasonable doubt of the crime of qualified theft she cannot
be dismissed.
ISSUE/s:
1. Whether or not the petitioner was terminated for a just and valid cause.
2. Whether or not petitioner was afforded due process.
HELD: 1.YES. To dismiss an employee, the law requires the existence of a just and valid cause. Neither conviction
beyond reasonable doubt for a crime against the employer nor acquittal after criminal prosecution was indispensable.
Nor was a formal charge in court for the acts prejudicial to the interest of the employer a pre-requisite for a valid
dismissal.
The conviction of an employee in a criminal case is not indispensable to warrant his dismissal by his employer. If
there is sufficient evidence to show that the employee has been guilty of a breach of trust, or that his
employer has ample reason to distrust him, it cannot justly deny to the employer the authority to dismiss
such employee. It is not necessary for said court to find that an employee has been guilty of a crime beyond
reasonable doubt in order to authorize his dismissal.
Likewise, it is not necessary that the employer should await the employee’s final conviction in the criminal
case involving such fraud or breach of trust before it can terminate the employee’s services. In fact, even the
dropping of the charges or an acquittal of the employee therefrom does not preclude the dismissal of an employee for
acts inimical to the interests of the employer.
It is unfair to require an employer to first be morally certain of the guilt of the employee by awaiting a conviction
before terminating him when there is already sufficient showing of the wrongdoing. Here, no less than the DOJ

Angeles| Bajana | Balladares | Brillantes | Briones | Cabansag | Callanta | Chua | David|


De Leon | Gomez | Lopez | Macalino | Nostratis | Padilla | Reynon | Santos | Tan |Velasco
4E / 4F - 2018-2019
Page 701 of 920
LABOR REVIEW DIGEST
Atty. Joyrich Golangco

Secretary found probable cause for qualified theft against the petitioner. That finding was enough to justify her
termination for loss of confidence.
Also, the employer’s loss of trust and confidence in her was directly rooted in the manner of how she, as the
supervisor, had negligently handled the large amount of sales by simply leaving the amount inside the cabinet drawer
of the kiosk despite being aware of the great risk of theft. At the very least, she could have resorted to the SOP of
first seeking guidance from the main office on how to secure the amount if she could not deposit in the bank due to
that day being a Sunday.
2. NO. The petitioner plainly demonstrated how quickly and summarily her dismissal was carried out without first
requiring her to explain anything in her defense. Instead, the respondents forthwith had her arrested and investigated
by the police authorities for qualified theft. This, we think, was a denial of her right to due process of law, consisting in
the opportunity to be heard and to defend herself. In fact, their decision to dismiss her was already final even before
the police authority commenced an investigation of the theft.
The fair and reasonable opportunity required to be given to the employee before dismissal encompassed not only the
giving to the employee of notice of the cause and the ability of the employee to explain, but also the chance to defend
against the accusation. The violation of the petitioners’ right to statutory due process by the private respondent
warrants the payment of indemnity in the form of nominal damages.

Angeles| Bajana | Balladares | Brillantes | Briones | Cabansag | Callanta | Chua | David|


De Leon | Gomez | Lopez | Macalino | Nostratis | Padilla | Reynon | Santos | Tan |Velasco
4E / 4F - 2018-2019
Page 702 of 920
LABOR REVIEW DIGEST
Atty. Joyrich Golangco

13. ST. LUKE’S MEDICAL CENTER, INC. VS. NOTARIO


GR NO: 152166 DATE: OCTOBER 20, 2010
PONENTE: PERALTA, J.

Doctrine: Article 282 (b) of the Labor Code provides that an employer may terminate an employment for
gross and habitual neglect by the employee of his duties. Neglect of duty, to be a ground for dismissal, must
be both gross and habitual. Gross negligence connotes want of care in the performance of ones duties.
Habitual neglect implies repeated failure to perform ones duties for a period of time, depending upon the
circumstances. A single or isolated act of negligence does not constitute a just cause for the dismissal of
the employee.

To effectuate a valid dismissal from employment by the employer, the Labor Code has set twin requirements,
namely: (1) the dismissal must be for any of the causes provided in Article 282 of the Labor Code; and (2) the
employee must be given an opportunity to be heard and defend himself. This first requisite is referred to as
the substantive aspect, while the second is deemed as the procedural aspect.

FACTS: St. Lukes Medical Center, Inc. (petitioner hospital), located at Quezon City, employed respondent as In-
House Security Guard. In August 1996, Nimaya Electro Corporation installed a closed-circuit television (CCTV)
system in the premises of petitioner hospital to enhance its security measures and conducted an orientation seminar
for the in-house security personnel on the proper way of monitoring video cameras, subject to certain guidelines.

On December 30, 1996, respondent was on duty from 6:00 p.m. to 6:00 a.m. of the following day, December 31,
1996. His work consisted mainly of monitoring the video cameras. In the evening of December 30, 1996, Justin
Tibon, a foreigner from Majuro, Marshall Island, then attending to his 3-year-old daughter who was admitted at room
257, cardiovascular unit of petitioner hospital, reported to the management of petitioner hospital about the loss of his
mint green traveling bag. Acting on the complaint of Tibon, the Security Department of petitioner hospital conducted
an investigation. When the tapes of video camera recorder (VCR) no. 3 covering the subject period were reviewed, it
was shown that the VCR was focused on camera no. 2 (Old Maternity Unit), from 9:03 p.m. to 10:15 p.m. of
December 30, 1996, and camera no. 1 (New Maternity Unit), from 12:25 a.m. to 6:00 a.m. of December 31, 1996.
The cameras failed to record any incident of theft at room 257.

On January 6, 1997, petitioner hospital issued a Memorandum to respondent, the CCTV monitoring staff on duty,
directing him to explain in writing why no disciplinary action should be taken against him for violating the normal
rotation/sequencing process of the VCR and, consequently, failed to capture the theft of Tibon's traveling bag at
room 257.

Respondent explained that on the subject dates, he was the only personnel on duty as nobody wanted to assist him.
Because of this, he decided to focus the cameras on the Old and New Maternity Units, as these two units have high
incidence of crime. Finding the written explanation of respondent to be unsatisfactory, petitioner hospital served on
respondent a copy of the Notice of Termination, dismissing him on the ground of gross negligence/inefficiency under
Section 1, Rule VII of its Code of Discipline.

Respondent filed a Complaint for illegal dismissal against petitioner hospital and its Chairman, Robert Kuan, seeking
reinstatement. Petitioners countered that they validly dismissed respondent for gross negligence and observed due
process before terminating his employment.

LA RULING: Labor Arbiter dismissed respondents complaint for illegal dismissal against petitioners. He stated that a
CCTV monitoring system is designed to focus on many areas in a programmed and sequential manner and should
not to be focused only on a specific area, unless the situation requires it. He concluded that during respondents duty
from December 30 to 31, 1996, he was negligent in focusing the cameras at the Old and New Maternity Units only
and, consequently, the theft committed at room 257 was not recorded.

NLRC/CA RULING: NLRC reversed the Decision of the Labor Arbiter. It stated that petitioners failed to submit proof
that there was an existing Standard Operating Procedure (SOP) in the CCTV monitoring system, particularly on the
focusing procedure. It observed that respondent was not negligent when he focused the cameras on the Old and
New Maternity Units, as they were located near the stairways and elevators, which were frequented by many visitors
and, thus, there is the likelihood that untoward incidents may arise. If at all, it treated the matter as a single or
isolated act of simple negligence which did not constitute a just cause for the dismissal of an employee.

Angeles| Bajana | Balladares | Brillantes | Briones | Cabansag | Callanta | Chua | David|


De Leon | Gomez | Lopez | Macalino | Nostratis | Padilla | Reynon | Santos | Tan |Velasco
4E / 4F - 2018-2019
Page 703 of 920
LABOR REVIEW DIGEST
Atty. Joyrich Golangco

Even assuming that respondents act would constitute gross negligence, it ruled that the ultimate penalty of dismissal
was not proper as it was not habitual, and that there was no proof of pecuniary injury upon petitioner
hospital.Moreover, it declared that petitioners failed to comply with the twin notice rule and hearing as what they did
was to require respondent to submit a written explanation, within 24 hours and, thereafter, he was ordered dismissed,
without affording him an opportunity to be heard.

APPEAL TO THE SC:

Petitioner's Contention: Petitioners allege that, by not focusing the CCTV cameras on the different areas of the
hospital, respondent committed gross negligence which warrants his dismissal. According to them, there was no
need to prove that the act done was habitual, as the occurrence of the theft exposed them to possible law suit and,
additionally, there might be a repetition of a similar incident in the future if respondent would remain in their employ.

Respondent's Contention: Respondent maintains that he was not negligent in the discharge of his duties. He said
that there was no actual loss to petitioner hospital as no complaint or legal action was taken against them and that
the supposed complainant, Tibon, did not even report the matter to the police authorities.

ISSUE/S:
1. whether or not respondent was illegally dismissed
2. whether or not there was compliance with the notice requirement

HELD:
1. YES. Article 282 (b) of the Labor Code provides that an employer may terminate an employment for gross and
habitual neglect by the employee of his duties. Neglect of duty, to be a ground for dismissal, must be both gross and
habitual. Gross negligence connotes want of care in the performance of ones duties. Habitual neglect implies
repeated failure to perform ones duties for a period of time, depending upon the circumstances. A single or isolated
act of negligence does not constitute a just cause for the dismissal of the employee. Under the prevailing
circumstances, respondent exercised his best judgment in monitoring the CCTV cameras so as to ensure the
security within the hospital premises. Verily, assuming arguendo that respondent was negligent, although this Court
finds otherwise, the lapse or inaction could only be regarded as a single or isolated act of negligence that cannot be
categorized as habitual and, hence, not a just cause for his dismissal.

Petitioners anchor on the postulate that even a single or isolated act of negligence by respondent constitutes a just
cause for his dismissal as it engendered the possibility of a legal action that may be taken against them by the owner
of the lost items. This is purely speculative.

2. NO. To effectuate a valid dismissal from employment by the employer, the Labor Code has set twin requirements,
namely: (1) the dismissal must be for any of the causes provided in Article 282 of the Labor Code; and (2) the
employee must be given an opportunity to be heard and defend himself. This first requisite is referred to as the
substantive aspect, while the second is deemed as the procedural aspect.

The employee must be furnished two written notices: the first notice apprises the employee of the particular acts or
omissions for which his dismissal is sought, and the second is a subsequent notice, which informs the employee of
the employer's decision to dismiss him. The CA found that petitioner hospital failed to comply with the rule on twin
notice and hearing as it merely required respondent to give his written explanation within 24 hours and, thereafter,
ordered his dismissal.

Where the dismissal was without just cause and there was no due process, Article 279 of the Labor Code, as
amended, mandates that the employee is entitled to reinstatement without loss of seniority rights and other privileges
and full backwages, inclusive of allowances and other benefits, or their monetary equivalent computed from the time
the compensation was not paid up to the time of actual reinstatement.

Angeles| Bajana | Balladares | Brillantes | Briones | Cabansag | Callanta | Chua | David|


De Leon | Gomez | Lopez | Macalino | Nostratis | Padilla | Reynon | Santos | Tan |Velasco
4E / 4F - 2018-2019
Page 704 of 920
LABOR REVIEW DIGEST
Atty. Joyrich Golangco

14. COCA-COLA EXPORT CORP. VS. GACAYAN


GR NO: 149433 DATE: DECEMBER 15, 2010
PONENTE: LEONARDO-DE CASTRO, J.

Doctrine: It bears emphasizing that the right of an employer to dismiss its employees on the ground of loss
of trust and confidence must not be exercised arbitrarily. For loss of trust and confidence to be a valid
ground for dismissal, it must be substantial and founded on clearly established facts. Loss of confidence
must not be used as a subterfuge for causes which are improper, illegal or unjustified; it must be genuine,
not a mere afterthought, to justify earlier action taken in bad faith.

FACTS: Respondent Clarita P. Gacayan began working with petitioner on October 8, 1985. At the time her
employment was terminated on April 6, 1995, for alleged loss of trust and confidence, respondent was holding the
position of Senior Financial Accountant.

Under petitioners company policy, one of the benefits enjoyed by its employees was the reimbursement of meal and
transportation expenses incurred while rendering overtime work. This reimbursement was allowed only when the
employee worked overtime for at least four hours on a Saturday, Sunday or holiday, and for at least two hours on
weekdays. The maximum amount allowed to be reimbursed was one hundred fifty (P150.00) pesos. It was in
connection with this company policy that petitioner called the attention of respondent and required her to explain the
alleged alterations in three receipts which she submitted to support her claim for reimbursement of meal expenses, to
wit: 1) McDonalds Receipt No. 875493 dated October 1, 1994 for P111.00; 2) Shakeys Pizza Parlor Receipt No.
122658 dated November 20, 1994 for P174.06; and 3) Shakeys Pizza Parlor Receipt No. 41274 dated July 19, 1994
for P130.50.

McDonalds Receipt: Respondent explained that the alteration may have been made by the staff from McDonalds as
they sometimes make mistakes in issuing receipts. Respondent also narrated that her sister, Odette, sometimes
buys food for her and that she is not quite sure if the receipt in question was the correct one which Odette gave her.

Shakeys Receipt: Respondent explained that the delivery staff brought a wrong receipt as it did not correspond to the
food that she actually ordered. Respondent added that she asked the delivery staff to alter the receipt thinking that he
could just write the correct items ordered and sign the said receipt to authenticate the alterations made thereon. She
further stated that there was no intention on her part to commit fraud since she was just avoiding the hassle of
waiting for a replacement receipt.

Petitioner sent respondent a letter directing her to explain why she should not be subjected to disciplinary sanctions
for violating Section II, No. 15, paragraph (d) of the companys rules and regulations which punishes with dismissal
the submission of any fraudulent item of expense. Consequently, respondent submitted her explanation on January
4, 1995, and denied any personal knowledge in the commission of the alterations in the subject receipts.

Petitioner sent respondent a memorandum inviting her to a hearing and formal investigation. Thereafter, petitioner
dismissed respondent for fraudulently submitting tampered and/or altered receipts in support of her petty cash
reimbursements in gross violation of the companys rules and regulations.

Respondent filed a complaint for illegal dismissal

LA/RTC/NLRC RULING: Labor Arbiter ruled in favor of petitioner and dismissed respondents complaint for lack of
merit. Respondent was terminated for repeatedly submitting fraudulent items of expense, clearly in violation of
petitioners company rules and regulations which consequently resulted in loss of trust and confidence.

CA RULING: The Court of Appeals ruled that the penalty of dismissal imposed on respondent was too harsh and
further directed petitioner to immediately reinstate respondent to her former position, if possible, or a substantially
equivalent position without loss of seniority rights and with full backwages.

APPEAL TO THE SC:

Petitioner's Contention: According to the petitioner, respondents repeated submission of altered or tampered
receipts to support her claim for reimbursement constitutes a betrayal of the employers trust and confidence and a
serious misconduct, thus, giving cause for the termination of her employment with petitioner.

Angeles| Bajana | Balladares | Brillantes | Briones | Cabansag | Callanta | Chua | David|


De Leon | Gomez | Lopez | Macalino | Nostratis | Padilla | Reynon | Santos | Tan |Velasco
4E / 4F - 2018-2019
Page 705 of 920
LABOR REVIEW DIGEST
Atty. Joyrich Golangco

Respondent's Contention: Respondent averred that, assuming arguendo that she altered the receipts in question,
dismissal was too harsh a penalty for her considering that: (a) it was her first offense in her 9 years of service; (b) the
offense imputed was minor, as only the dates and items, not the amounts, were altered or the amounts involved were
very minimal; (c) the company did not suffer material damage, as she was really entitled to the P150.00 allowance
even without accompanying receipt; and (d) respondent acted without malice, as she really rendered (unpaid)
overtime work on those three dates.

ISSUE/S: whether or not the dismissal of respondents is proper

HELD: NO. The Labor Code mandates that before an employer may validly dismiss an employee from the service,
the requirement of substantial and procedural due process must be complied with. Under the requirement of
substantial due process, the grounds for termination of employment must be based on just or authorized causes.

After examining the records of the case, this Court finds that respondents dismissal from employment was not
grounded on any of the just causes enumerated under Article 282 of the Labor Code.

At the outset, it is important to note that the term trust and confidence is restricted to managerial employees. In
Samson v. National Labor Relations Commission, the Court, citing Section 2(b), Rule I, Book III of the Omnibus
Rules Implementing the Labor Code, enumerated the conditions for one to be properly considered a managerial
employee:

(1) Their primary duty consists of the management of the establishment in which they are employed or of a
department or sub-division thereof;

(2) They customarily and regularly direct the work of two or more employees therein; [and]

(3) They have the authority to hire or fire other employees of lower rank; or their suggestions and recommendations
as to the hiring and firing and as to the promotion or any other change of status of other employees are given
particular weight.

In the instant case, respondent was the Senior Financial Accountant with the Job Description of a Financial Project
Analyst. Respondent, among others, provides support in the form of financial analyses and evaluation of alternative
strategies or action plans to assist management in strategic and operational decision-making, x x x liaises with the
Bottler to comply with Corporate Bottler financial reporting requirements and to ensure Bottlers plans are aligned with
TCCECs, x x x and assists management on various initiatives on ad hoc basis.

In Nokom v. National Labor Relations Commission, this Court set the guidelines for the application of the doctrine of
loss of confidence

(a) Loss of confidence should not be simulated;

(b) It should not be used as a subterfuge for causes which are improper, illegal or unjustified;

(c) It may not be arbitrarily asserted in the face of overwhelming evidence to the contrary; and

(d) It must be genuine, not a mere afterthought to justify earlier action taken in bad faith.

In the instant case, the basis for terminating the employment of respondent was for gross violation of the companys
rules and regulations, as specified in the termination letter. Evidently, no mention was made regarding petitioners
alleged loss of trust and confidence in respondent. Neither was there any explanation nor discussion of the alleged
sensitive and delicate position of respondent requiring the utmost trust of petitioner. It was only in the Reply to
Respondents Comment that petitioner made mention of another ground for the dismissal of respondent, that of
serious misconduct, when she submitted altered or tampered receipts to support her claim for reimbursement. Such
allegation appears to be a mere afterthought, being tardily raised only in the Reply.

The misconduct to be serious must be of such grave and aggravated character and not merely trivial and
unimportant. Such misconduct, however serious, must nevertheless be in connection with the employees work to
constitute just cause for his separation. Thus, for misconduct or improper behavior to be a just cause for dismissal,
(a) it must be serious; (b) must relate to the performance of the employees duties; and (c) must show that the
employee has become unfit to continue working for the employer.

Angeles| Bajana | Balladares | Brillantes | Briones | Cabansag | Callanta | Chua | David|


De Leon | Gomez | Lopez | Macalino | Nostratis | Padilla | Reynon | Santos | Tan |Velasco
4E / 4F - 2018-2019
Page 706 of 920
LABOR REVIEW DIGEST
Atty. Joyrich Golangco

In this light, the alleged infractions of respondent could hardly be considered serious misconduct. It is well to stress
that in order to constitute serious misconduct which will warrant the dismissal of an employee, it is not sufficient that
the act or conduct complained of has violated some established rules or policies. It is equally important and required
that the act or conduct must have been done with wrongful intent. Such is, however, lacking in the instant case.

While this Court does not condone respondents act of submitting altered and/or tampered receipts to support her
claim for reimbursement, we nevertheless agree with the finding of the Court of Appeals that, under the attendant
facts, the dismissal meted out on respondent appears to be too harsh a penalty.

Angeles| Bajana | Balladares | Brillantes | Briones | Cabansag | Callanta | Chua | David|


De Leon | Gomez | Lopez | Macalino | Nostratis | Padilla | Reynon | Santos | Tan |Velasco
4E / 4F - 2018-2019
Page 707 of 920
LABOR REVIEW DIGEST
Atty. Joyrich Golangco

15. DOMINGO VS. RAYALA


GR NO: 155831 DATE: FEBRUARY 18, 2008
PONENTE: NACHURA, J.

Doctrine: Under the Labor Code, the Chairman of the NLRC shall hold office during good behavior until he or
she reaches the age of sixty-five, unless sooner removed for cause as provided by law or becomes
incapacitated to discharge the duties of the office.

In this case, it is the President of the Philippines, as the proper disciplining authority, who would determine
whether there is a valid cause for the removal of Rayala as NLRC Chairman. This power, however, is
qualified by the phrase for cause as provided by law.

FACTS: On November 16, 1998, Ma. Lourdes T. Domingo (Domingo), then Stenographic Reporter III at the NLRC,
filed a Complaint for sexual harassment against Rayala before Secretary Bienvenido Laguesma of the Department of
Labor and Employment (DOLE). Prior to the filing of the complaint, Domingo filed for leave of absence and asked to
be immediately transferred.

Upon receipt of the Complaint, the DOLE Secretary referred the Complaint to the OP, Rayala being a presidential
appointee. The OP, through then Executive Secretary Ronaldo Zamora, ordered Secretary Laguesma to investigate
the allegations in the Complaint and create a committee for such purpose.

OP: The Committee found Rayala guilty of the offense charged and the OP concurred therewith. Rayala is hereby
DISMISSED from the service. Rayala filed a Motion for Reconsideration, which the OP denied.

CA RULING: The CA held that there was sufficient evidence on record to create moral certainty that Rayala
committed the acts he was charged with. It also held that Rayalas dismissal was proper. But upon the filing of a MR,
CA modified its decision to the effect that the penalty of dismissal is DELETED and instead the penalty of suspension
from service for the maximum period of one (1) year is imposed upon Rayala.

Petitioner's Contention: Domingo assails the CAs resolution modifying the penalty imposed by the Office of the
President. She raises this issue:
The Court of Appeals erred in modifying the penalty for the respondent from dismissal to suspension from
service for the maximum period of one year. The President has the prerogative to determine the proper
penalty to be imposed on an erring Presidential appointee. The President was well within his power when he
fittingly used that prerogative in deciding to dismiss the respondent from the service.

The Republic further claims that, although AO 250 provides only a one (1) year suspension, it will not prevent the OP
from validly imposing the penalty of dismissal on Rayala. It argues that even though Rayala is a presidential
appointee, he is still subject to the Civil Service Law. Under the Civil Service Law, disgraceful and immoral conduct,
the acts imputed to Rayala, constitute grave misconduct punishable by dismissal from the service.

Respondent's Contention: Rayala attacks the penalty imposed by the OP. He alleges that under the pertinent Civil
Service Rules, disgraceful and immoral conduct is punishable by suspension for a period of six (6) months and one
(1) day to one (1) year.

ISSUE/S: whether or not Rayala should only be meted with the penalty of suspension

HELD: YES. It is noteworthy that the five CA Justices who deliberated on the case were unanimous in upholding the
findings of the Committee and the OP. They found the assessment made by the Committee and the OP to be a
meticulous and dispassionate analysis of the testimonies of the complainant (Domingo), the respondent (Rayala),
and their respective witnesses. They differed only on the appropriate imposable penalty.

That Rayala committed the acts complained of and was guilty of sexual harassment is, therefore, the common factual
finding of not just one, but three independent bodies: the Committee, the OP and the CA. It should be remembered
that when supported by substantial evidence, factual findings made by quasi-judicial and administrative bodies are
accorded great respect and even finality by the courts.

Under the Labor Code, the Chairman of the NLRC shall hold office during good behavior until he or she reaches the
age of sixty-five, unless sooner removed for cause as provided by law or becomes incapacitated to discharge the
duties of the office.

Angeles| Bajana | Balladares | Brillantes | Briones | Cabansag | Callanta | Chua | David|


De Leon | Gomez | Lopez | Macalino | Nostratis | Padilla | Reynon | Santos | Tan |Velasco
4E / 4F - 2018-2019
Page 708 of 920
LABOR REVIEW DIGEST
Atty. Joyrich Golangco

Under AO 250, the penalty for the first offense is suspension for six (6) months and one (1) day to one (1) year, while
the penalty for the second offense is dismissal. On the other hand, Section 22(o), Rule XVI of the Omnibus Rules
Implementing Book V of the Administrative Code of 1987 and Section 52 A(15) of the Revised Uniform Rules on
Administrative Cases in the Civil Service both provide that the first offense of disgraceful and immoral conduct is
punishable by suspension of six (6) months and one (1) day to one (1) year. A second offense is punishable by
dismissal.

In this case, it is the President of the Philippines, as the proper disciplining authority, who would determine whether
there is a valid cause for the removal of Rayala as NLRC Chairman. This power, however, is qualified by the phrase
for cause as provided by law. Thus, when the President found that Rayala was indeed guilty of disgraceful and
immoral conduct, the Chief Executive did not have unfettered discretion to impose a penalty other than the penalty
provided by law for such offense. As cited above, the imposable penalty for the first offense of either the
administrative offense of sexual harassment or for disgraceful and immoral conduct is suspension of six (6) months
and one (1) day to one (1) year. Accordingly, it was error for the Office of the President to impose upon Rayala the
penalty of dismissal from the service, a penalty which can only be imposed upon commission of a second offense.

Angeles| Bajana | Balladares | Brillantes | Briones | Cabansag | Callanta | Chua | David|


De Leon | Gomez | Lopez | Macalino | Nostratis | Padilla | Reynon | Santos | Tan |Velasco
4E / 4F - 2018-2019
Page 709 of 920
LABOR REVIEW DIGEST
Atty. Joyrich Golangco

SEXUAL HARASSMENT

16. PHILIPPINE AEOLUS AUTOMOTIVE UNITED CORPORATION and/or FRANCIS CHUA v. NATIONAL
LABOR RELATIONS COMMISSION and ROSALINDA C. CORTEZ, respondents.
G.R. No. 124617 April 28, 2000
BELLOSILLO, J.:

DOCTRINE:
The gravamen of the offense in sexual harassment is not the violation of the employee's sexuality but the
abuse of power by the employer.

FACTS:
Petitioner Philippine Aeolus Automotive United Corporation (PAAUC) is a corporation duly organized and
existing under Philippine laws, petitioner, Francis Chua is its President while private respondent Rosalinda C. Cortez
was a company nurse of Petitioner Corporation until her termination.

PAAUC dismissed Private Respondent from service on the ground of serious misconduct, gross habitual neglect and
fraud or willful breach of trust. Among the acts she allegedly committed is throwing a stapler at Plant Manager
William Chua, her superior and uttering invectives against him. She filed with the Labor Arbiter a complaint for illegal
dismissal, non-payment of annual service incentive leave, 13th month pay and damages against PAAUC and its
President Francis Chua. She claimed as a defense for the offense charged against her that William Chua manifested
a special liking for her. She claimed that William Chua would oftentimes invite her for a date, make sexual advances
– touching her hands, putting his arms around her shoulders, running his finger on her arms and telling her she
looked beautiful. The special treatment and sexual advances continued during her employment for 4 years but she
never reciprocated his flirtations, until finally, she noticed that his attitude toward her changed. He made her
understand that if she would not give in to his sexual advances he would cause her termination from the service; and
he made good his threat when he started harassing her.

LA RULING:
The Labor Arbiter rendered a decision holding the termination of Cortez as valid and legal, at the same time
dismissing her claim for damages for lack of merit.

NLRC RULING:
The NLRC disbelieved the explanation proffered by private respondent on the ground she never filed a
complaint against William Chua for more than 4 years.

ISSUE:
Whether or not William Chua committed acts of sexual harassment against Cortez?

SC RULING:
Yes. The gravamen of the offense in sexual harassment is not the violation of the employee's sexuality but the abuse
of power by the employer. Any employee, male or female, may rightfully cry "foul" provided the claim is well
substantiated. Strictly speaking, there is no time period within which he or she is expected to complain through the
proper channels. The time to do so may vary depending upon the needs, circumstances, and more importantly, the
emotional threshold of the employee.

Private respondent admittedly allowed four (4) years to pass before finally coming out with her employer's
sexual impositions. Not many women, especially in this country, are made of the stuff that can endure the agony and
trauma of a public, even corporate, scandal. If petitioner corporation had not issued the third memorandum that
terminated the services of private respondent, we could only speculate how much longer she would keep her silence.
Moreover, few persons are privileged indeed to transfer from one employer to another. The dearth of quality
employment has become a daily "monster" roaming the streets that one may not be expected to give up one's
employment easily but to hang on to it, so to speak, by all tolerable means. Perhaps, to private respondent's mind,
for as long as she could outwit her employer's ploys she would continue on her job and consider them as mere
occupational hazards. This uneasiness in her place of work thrived in an atmosphere of
tolerance for four (4) years, and one could only imagine the prevailing anxiety and resentment, if not bitterness, that
beset her all that time. But William Chua faced reality soon enough. Since he had no place in private respondent's
heart, so must she have no place in his office. So, he provoked her, harassed her, and finally dislodged her; and for
finally venting her pent-up anger for years, he "found" the perfect reason to terminate her.

Angeles| Bajana | Balladares | Brillantes | Briones | Cabansag | Callanta | Chua | David|


De Leon | Gomez | Lopez | Macalino | Nostratis | Padilla | Reynon | Santos | Tan |Velasco
4E / 4F - 2018-2019
Page 710 of 920
LABOR REVIEW DIGEST
Atty. Joyrich Golangco

Angeles| Bajana | Balladares | Brillantes | Briones | Cabansag | Callanta | Chua | David|


De Leon | Gomez | Lopez | Macalino | Nostratis | Padilla | Reynon | Santos | Tan |Velasco
4E / 4F - 2018-2019
Page 711 of 920
LABOR REVIEW DIGEST
Atty. Joyrich Golangco

TERMINATION BY EMPLOYER: Willful Disobedience (Transfer)

17. PHARMACIA and UPJOHN, INC. v. RICARDO P. ALBAYDA, JR.


G.R. No. 172724 August 23, 2010
PERALTA, J.:

DOCTRINE: Court has long stated that the objection to the transfer being grounded solely upon the personal
inconvenience or hardship that will be caused to the employee by reason of the transfer is not a valid reason
to disobey an order of transfer. Such being the case, respondent cannot adamantly refuse to abide by the
order of transfer without exposing himself to the risk of being dismissed. Hence, his dismissal was for just
cause in accordance with Article 282(a) of the Labor Code.

FACTS: Ricardo P. Albayda, Jr. (respondent) was an employee of Upjohn, Inc. (Upjohn) in 1978 and continued
working there until 1996 when a merger between Pharmacia and Upjohn was created. After the merger, respondent
was designated by petitioner Pharmacia and Upjohn (Pharmacia) as District Sales Manager assigned to District XI in
the Western Visayas area. During the period of his assignment, respondent settled in Bacolod City.

However, in December 1999, respondent received a Memorandum announcing the sales force structure for the year
2000. In the said memorandum, respondent was reassigned as District Sales Manager to District XII in the Northern
Mindanao area. One of the key areas covered in District XII is Cagayan de Oro City. In response to the
memorandum, respondent wrote a letter to Aleda Chu. Respondent said that he has always been assigned to the
Western Visayas area and that he felt that he could not improve the sales of products if he was assigned to an
unfamiliar territory. Respondent concluded that his transfer might be a way for his managers to dismiss him from
employment. Respondent added that he could not possibly accept his new assignment in Cagayan de Oro City
because he will be dislocated from his family; his wife runs an established business in Bacolod City; his eleven- year-
old daughter is studying in Bacolod City; and his two-year-old son is under his and his wifes direct care.

Chu explained to respondent that they are moving him to Cagayan de Oro City, because of their need of
respondent’s expertise to build the business there. Chu added that the district performed dismally in 1999 and,
therefore, they were confident that under respondent’s leadership, he can implement new ways and develop the
sales force to become better and more productive. Moreover, since respondent has been already in Bacolod and
Iloilo for 22 years, Chu said that exposure to a different market environment and new challenges will contribute to
respondent’s development as a manager. Finally, Chu stressed that the decision to transfer respondent was purely a
business decision.

Thereafter, Chu reassigned respondent to Manila, and he was directed to report to work to Manila within 5 days, but
this was unheeded. Because of this, petitioner dismissed him from work through a letter due to AWOL and
Insubordination.

Respondent filed a case for constructive dismissal with the Labor arbiter.

ISSUE: Is the dismissal valid?

HELD: Yes. Jurisprudence recognizes the exercise of management prerogative to transfer or assign employees from
one office or area of operation to another, provided there is no demotion in rank or diminution of salary, benefits, and
other privileges, and the action is not motivated by discrimination, made in bad faith, or effected as a form of
punishment or demotion without sufficient cause. In this case, the transfer from one district to another is duly
explained.

Moreover, the allegation of complainant that respondent’s family’s income will be affected because his wife who is
doing business in Bacolod City and earns P50,000.00, if true, should not be taken in consideration of his transfer.
What is contemplated here is the diminution of the salary of the complainant but not his wife. Besides, even if
complainant may accept his new assignment in Cagayan de Oro or in Metro Manila, his wife may still continue to do
her business in Bacolod City. Anyway, Bacolod City and Manila is just one (1) hour travel by plane.
Also, in respondent’s contract of employment, he agreed to be assigned to any work or workplace as may be
determined by the company whenever the operations require such assignment. As such, he is bound thereby. It is
also in the nature of sales force to be transferred whenever deemed fit by the company. Court has long stated that
the objection to the transfer being grounded solely upon the personal inconvenience or hardship that will be caused
to the employee by reason of the transfer is not a valid reason to disobey an order of transfer. Such being the case,

Angeles| Bajana | Balladares | Brillantes | Briones | Cabansag | Callanta | Chua | David|


De Leon | Gomez | Lopez | Macalino | Nostratis | Padilla | Reynon | Santos | Tan |Velasco
4E / 4F - 2018-2019
Page 712 of 920
LABOR REVIEW DIGEST
Atty. Joyrich Golangco

respondent cannot adamantly refuse to abide by the order of transfer without exposing himself to the risk of being
dismissed. Hence, his dismissal was for just cause in accordance with Article 282(a) of the Labor Code.

ON THE ISSUE OF SEPARATION PAY:


In the instant case, this Court rules that an award to respondent of separation pay by way of financial
assistance, equivalent to one-half (1/2) months pay for every year of service, is equitable. Although respondent's
actions constituted a valid ground to terminate his services, the same is to this Court's mind not so reprehensible as
to warrant complete disregard of his long years of service (22 years). It also appears that the same is respondent's
first offense. While it may be expected that petitioners will argue that respondent has only been in their service for
four years since the merger of Pharmacia and Upjohn took place in 1996, equity considerations dictate that
respondent's tenure be computed from 1978, the year when respondent started working for Upjohn.

Angeles| Bajana | Balladares | Brillantes | Briones | Cabansag | Callanta | Chua | David|


De Leon | Gomez | Lopez | Macalino | Nostratis | Padilla | Reynon | Santos | Tan |Velasco
4E / 4F - 2018-2019
Page 713 of 920
LABOR REVIEW DIGEST
Atty. Joyrich Golangco

TERMINATION BY EMPLOYER: Loss of Trust and Confidence

18. JAMES BEN L. JERUSALEM vs. HOCK, SUNNY YAP and JOSEFINA PICART
G.R. No. 169564 April 6, 2011
DEL CASTILLO, J.:

DOCTRINE: For breach of trust and confidence to become a valid ground for the dismissal of an employee,
the cause of loss of trust and confidence must be related to the performance of the employee’s duties.

FACTS: James Ben L. Jerusalem (James) was employed by Keppel Monte Bank (Keppel) as Assistant Vice-
President as was assigned as the head of the VISA Credit card department. Later, he was reassigned as the head of
the Marketing and Operations of the Jewelry Department.

During his leadership in the new assignment, James received from Jorge Javier (Jorge) a sealed envelope said to be
containing VISA Card application forms. Jorge is a Keppel Visa Card Holder since December 1998. James
immediately handed over the envelope with accomplished application forms to the VISA Credit Card Unit. All in all,
the VISA credit card applications referred by Jorge which James forwarded to the VISA Credit Card Unit numbered
67, all of which were subsequently approved. As it turned out, all the accounts under these approved applications
became past due.

Due to this, the bank suffered a loss of P7,961,619.82. James upon knowing the status of the accounts referred by
Jorge, sent a Memorandum6 to Roberto (head of the VISA Credit Card Department) recommending the filing of a
criminal case for estafa against Jorge. He further recommended that coordination with the other banks where Jorge
has deposits should be made promptly so that they can ask said banks to freeze Jorge’s accounts. James even
warned Keppel that immediate action should be taken while Jorge is still in the country.

However thereafter, James received a Notice to Explain from Keppel’s Vice President for Operations, Sunny Yap
(Sunny), why no disciplinary action should be taken against him for referring/endorsing fictitious VISA card
applicants.

James submitted his written explanation to Sunny. He pointed out that he had no participation in the processing of
the VISA card applications since he was no longer connected with the VISA Credit Card Unit at the time of such
transactions. He explained that he can only endorse the applications referred by Jorge to the VISA Credit Card Unit
because he was already transferred to Jewelry Department, as Head.

Despite such explanation, the Manager for Human Resources Department, Josefina Picart, handed to James a
Notice of Termination informing the latter that he was found guilty of breach of trust and confidence for knowingly and
maliciously referring, endorsing and vouching for VISA card applicants who later turned out to be impostors resulting
in financial loss to Keppel.

This prompted James to file before the Labor Arbiter a complaint for illegal dismissal.

ISSUE: Is the dismissal of James on the ground of loss of trust and confidence valid?

HELD: NO. "Loss of confidence as a just cause for termination of employment is premised on the fact that the
employee concerned holds a position of responsibility or trust and confidence. He must be invested with confidence
on delicate matters, such as custody handling or care and protection of the property and assets of the employer. And,
in order to constitute a just cause for dismissal, the act complained of must be work-related and shows that the
employee concerned is unfit to continue to work for the employer."

The first requisite for dismissal on the ground of loss of trust and confidence is that the employee concerned must be
holding a position of trust and confidence. In this case, there is no doubt that James held a position of trust and
confidence as Assistant Vice-President of the Jewelry Department.
The second requisite is that there must be an act that would justify the loss of trust and confidence. Loss of trust and
confidence, to be a valid cause for dismissal, must be based on a wilful breach of trust and founded on clearly
established facts. The basis for the dismissal must be clearly and convincingly established but proof beyond
reasonable doubt is not necessary. Keppel’s evidence against James fails to meet this standard.

The bank has not been able to show any concrete proof that indeed complainant had participated in the
approval of the questioned VISA CARD accounts. The records [are] bereft of any concrete showing that

Angeles| Bajana | Balladares | Brillantes | Briones | Cabansag | Callanta | Chua | David|


De Leon | Gomez | Lopez | Macalino | Nostratis | Padilla | Reynon | Santos | Tan |Velasco
4E / 4F - 2018-2019
Page 714 of 920
LABOR REVIEW DIGEST
Atty. Joyrich Golangco

complainant directed Ms. Gerena to approve the applications without passing through the process. The alleged
marginal notations in the applications were admittedly scribbled by Ms. Gerena. Even assuming that there are such
notations on the applications i.e., "c/o James Jerusalem", still, such notations to us can not be construed as a
directive coming from complainant to specifically do away with existing policy on the approval of applications for VISA
Card.

Of course, we concede to the fact that respondent had sustained losses on account of the so-called "credit card
scam" in the amount of P7,961,619.82 all coming from the accounts referred x x x by Mr. Jorge Javier, but no amount
of mind boggling can we infer that the mere act of handing the already accomplished forms for VISA CREDIT Card
could be interpreted as "Favorable endorsement" with instructions not to conduct the usual credit
investigation/verification of applicants. To lay the blame upon the complainant would be at the height of
injustice considering that at that time, he no longer has the authority to pass upon such applications. To
attribute such huge financial losses to one who is no longer connected with the VISA Card department
would be stretching too far, the import of the term "some basis." We simply could not see our way through how
respondent bank could have inferred that complainant made such instruction upon Ms. Gerena to forego the usual
process and have the applications approved without any direct evidence showing to be so.

Angeles| Bajana | Balladares | Brillantes | Briones | Cabansag | Callanta | Chua | David|


De Leon | Gomez | Lopez | Macalino | Nostratis | Padilla | Reynon | Santos | Tan |Velasco
4E / 4F - 2018-2019
Page 715 of 920
LABOR REVIEW DIGEST
Atty. Joyrich Golangco

19. RENO FOODS INC. vs NLM-KATIPUNAN


GR No: 164016 Date: March 15, 2010
Ponente: Del Castillo, J.

Doctrine:
Criminal conviction is not necessary to find just cause for employment termination. Otherwise stated, an employee’s
acquittal in a criminal case, especially one that is grounded on the existence of reasonable doubt, will not preclude a
determination in a labor case that he is guilty of acts inimical to the employer’s interests. Criminal cases require proof
beyond reasonable doubt while labor
disputes require only substantial evidence, which means such relevant evidence as a reasonable mind might accept
as adequate to justify a conclusion.

FACTS:
Petitioner Reno Foods, Inc. (Reno Foods) is a manufacturer of canned meat products of which Vicente Khu is the president and
is being sued in that capacity. Respondent NenitaCapor (Capor) was an employee of Reno Foods until her dismissal
on October 27, 1998.

It is a standard operating procedure of petitioner-company to subject all its employees to reasonable search of their belongings
upon leaving the company premises. On October 19, 1998, the guard on duty found six Reno canned goods wrapped in nylon
leggings inside Capors fabric clutch bag. The only other contents of the bag were money bills and a small plastic medicine
container.

Petitioners accorded Capor several opportunities to explain her side, often with the assistance of the union officers
of Nagkakaisang Lakas ng Manggagawa (NLM) Katipunan. In fact, after petitioners sent a Notice of Termination to Capor, she
was given yet another opportunity for reconsideration through a labor-management grievance conference held on November
17, 1999. Unfortunately, petitioners did not find reason to change its earlier decision to terminate Capors employment with the
company.

On December 8, 1998, petitioners filed a complaint-affidavit against Capor for qualified theft in the Office of the City
Prosecutor, Malabon-Navotas Substation. On April 5, 1999, a Resolution was issued finding probable cause for the crime
charged. Consequently, an Information was filed against Capor.

Meanwhile, the Nagkakaisang Lakas ng Manggagawa (NLM) Katipunan filed on behalf of Capor a complaint for illegal dismissal
and money claims against petitioners with the Head Arbitration Office of the National Labor Relations Commission (NLRC) for
the National Capital Region. The complaint prayed that Capor be paid her full backwages as well as moral and exemplary
damages.

LA and NLRC RULING:


Capor is guilty of serious misconduct which is a just cause for termination.

CA RULING:
Affirmed NLRC decision

Petitioner's Contention:
Petitioners argue that the dismissal of a criminal action should not carry a corresponding dismissal of the labor action since a
criminal conviction is unnecessary in warranting a valid dismissal for employment.

Respondent's Contention:
Capor thus claims that her acquittal in the criminal case proves that petitioners failed to present substantial evidence to justify her
termination from the company. She therefore asks for a finding of illegal dismissal and an award of separation pay equivalent to
one month pay for every year of service.

ISSUE/S:
Whether or not conviction in a criminal case is necessary to find just cause for termination of employment

HELD: NO
Capor was acquitted in Criminal Case No. 207-58-MN based on reasonable doubt. In his Decision, the trial judge entertained
doubts regarding the guilt of Capor because of two circumstances: (1) an ensuing labor dispute (though it omitted to state the
parties involved), and (2) the upcoming retirement of Capor. The trial judge made room for the possibility that these
circumstances could have motivated petitioners to plant evidence against Capor so as to avoid paying her retirement

Angeles| Bajana | Balladares | Brillantes | Briones | Cabansag | Callanta | Chua | David|


De Leon | Gomez | Lopez | Macalino | Nostratis | Padilla | Reynon | Santos | Tan |Velasco
4E / 4F - 2018-2019
Page 716 of 920
LABOR REVIEW DIGEST
Atty. Joyrich Golangco

benefits. The trial court did not categorically rule that the acts imputed to Capor did not occur. It did not find petitioner’s version of
the event as fabricated, baseless, or unreliable. It merely acknowledged that seeds of doubt have been planted in the jurors’
mind which, in a criminal case, is enough to acquit an accused based on reasonable doubt.
In Nicolas v. National Labor Relations Commission, we held that a criminal conviction is not necessary to find just
cause for employment termination. Otherwise stated, an employee’s acquittal in a criminal case, especially one that is grounded
on the existence of reasonable doubt, will not preclude a determination in a labor case that he is guilty of acts inimical to the
employer’s interests.
Criminal cases require proof beyond reasonable doubt while labor
disputes require only substantial evidence, which means such relevant evidence as a reasonable mind might accept as
adequate to justify a conclusion. The evidence in this case was reviewed by the appellate court and two labor tribunals endowed
with expertise on the matter the Labor Arbiter and the NLRC. They all found substantial evidence to conclude that Capor had
been validly dismissed for dishonesty or serious misconduct. It is settled that factual findings of quasi-judicial agencies are
generally accorded respect and finality so long as these are supported by substantial evidence. In the instant case, we find no
compelling reason to doubt the common findings of the three reviewing bodies.

Other Notes/ SC Pronouncements:


NOT ENTITLED TO SEPARATION PAY
Separation pay is only warranted when the cause for termination is not attributable to the employee’s fault, such as those
provided in Articles 283 and 284 of the Labor Code, as well as in cases of illegal dismissal in which reinstatement is no longer
feasible. It is not allowed when an employee is dismissed for just cause, such as serious misconduct.

We are not persuaded by Capor’s argument that despite the finding of theft, she should still be granted separation pay in light of
her long years of service with petitioners. While we sympathize with Capor’s plight, being of retirement age and having served
petitioners for 39 years, we cannot award any financial assistance in her favor because it is not only against the law but also a
retrogressive public policy.

Angeles| Bajana | Balladares | Brillantes | Briones | Cabansag | Callanta | Chua | David|


De Leon | Gomez | Lopez | Macalino | Nostratis | Padilla | Reynon | Santos | Tan |Velasco
4E / 4F - 2018-2019
Page 717 of 920
LABOR REVIEW DIGEST
Atty. Joyrich Golangco

20. LA ROSA vs AMBASSADOR HOTEL


GR No: 177059 Date: March 13, 2009
Ponente: Carpio Morales, J.

Doctrine:
Abandonment is a matter of intention and cannot lightly be inferred or legally presumed from certain
equivocal acts. For abandonment to exist, two requisites must concur: first, the employee must have failed
to report for work or must have been absent without valid or justifiable reason; and second, there must have
been a clear intention on the part of the employee to sever the employer-employee relationship as
manifested by some overt acts. The second element is the more determinative factor. Abandonment as a just
ground for dismissal thus requires clear, willful, deliberate, and unjustified refusal of the employee to
resume employment. Mere absence or failure to report for work, even after notice to return, is not
tantamount to abandonment.

FACTS:
Employees of Ambassador Hotel including herein petitioners filed before the NLRC several complaints for illegal
dismissal, illegal suspension, and illegal deductions against the hotel (respondent) and its manager, Yolanda L.
Chan. They alleged that, following their filing of complaints with the Department of Labor and Employment-NCR
which prompted an inspection of the hotel’s premises by a labor inspector, respondent was found to have been
violating labor standards laws and was thus ordered to pay them some money claims. This purportedly angered
respondent’s management which retaliated by suspending and/or constructively dismissing them by
drastically reducing their work days through the adoption of a work reduction/rotation scheme. Criminal cases for
estafa were likewise allegedly filed against several of the employees involved, some of which cases were eventually
dismissed by the prosecutor’s office for lack of merit.

LA/NLRC RULING:
Found respondent and its manager Yolanda L. Chan guilty of illegal dismissal

CA RULING:
Reversed NLRC decision

Petitioner's Contention:
Petitioners deny having abandoned their jobs. And they take exception to the appellate court’s finding that they did
not pray for reinstatement, they inviting attention to paragraph 14, page 5 of their verified position paper reading: "x x
x Hence they are entitled to reinstatement with full backwages, or in the alternative to full separation pay of one
month per year of service," as well as to their prayer in the pro-forma complaints filed before the labor arbiter asking
for the same relief.
Petitioners question as bereft of specific proof the appellate court’s ruling that the work reduction/rotation scheme
adopted by respondent was a valid exercise of management prerogative.
Finally, petitioners question the issuance by the appellate court of a TRO, and subsequently of a writ of preliminary
injunction conditioned on respondent’s posting of a bond which was lower than the judgment award, hence,
prejudicial to them.
ISSUE/S:
Whether or not petitioners abandoned their work and were not illegally dismissed

HELD: NO
Case law holds that constructive dismissal occurs when there is cessation of work because continued employment
is rendered impossible, unreasonable or unlikely; when there is a demotion in rank or diminution in pay or both; or
when a clear discrimination, insensibility, or disdain by an employer becomes unbearable to the employee.
Respondent’s sudden, arbitrary and unfounded adoption of the two-day work scheme which greatly reduced
petitioners’ salaries renders it liable for constructive dismissal.
Respecting the appellate court’s ruling that petitioners "simply disappeared" from their work, hence, they are guilty of
abandonment, the same does not lie.1avvphAbsence must be accompanied by overt acts unerringly pointing to the
fact that the employee simply does not want to work anymore. And the burden of proof to show that there was
unjustified refusal to go back to work rests on the employer.
Abandonment is a matter of intention and cannot lightly be inferred or legally presumed from certain equivocal acts.
For abandonment to exist, two requisites must concur: first, the employee must have failed to report for work or must
have been absent without valid or justifiable reason; and second, there must have been a clear intention on the part
of the employee to sever the employer-employee relationship as manifested by some overt acts. The second element
is the more determinative factor. Abandonment as a just ground for dismissal thus requires clear, willful, deliberate,

Angeles| Bajana | Balladares | Brillantes | Briones | Cabansag | Callanta | Chua | David|


De Leon | Gomez | Lopez | Macalino | Nostratis | Padilla | Reynon | Santos | Tan |Velasco
4E / 4F - 2018-2019
Page 718 of 920
LABOR REVIEW DIGEST
Atty. Joyrich Golangco

and unjustified refusal of the employee to resume employment. Mere absence or failure to report for work, even after
notice to return, is not tantamount to abandonment. (Emphasis and underscoring supplied)
Respondent, which has the onus of proving that petitioners abandoned their work, failed to discharge the same,
however.
Upon the other hand, petitioners’ immediate filing of complaints for illegal suspension and illegal dismissal after the
implementation of the questioned work scheme, which scheme was adopted soon after petitioners’ complaints
against respondent for violation of labor standards laws were found meritorious, negates respondent’s claim of
abandonment. An employee who takes steps to protest his dismissal cannot by logic be said to have abandoned his
work.

Angeles| Bajana | Balladares | Brillantes | Briones | Cabansag | Callanta | Chua | David|


De Leon | Gomez | Lopez | Macalino | Nostratis | Padilla | Reynon | Santos | Tan |Velasco
4E / 4F - 2018-2019
Page 719 of 920
LABOR REVIEW DIGEST
Atty. Joyrich Golangco

21. MARIBAGO RESORT vs DUAL


GR No: 180660 Date: July 20, 2010
Ponente: Perez, J.

Doctrine:
Respondent’s acts constitute serious misconduct which is a just cause for termination under the law. Theft
committed by an employee is a valid reason for his dismissal by the employer.

FACTS:
Petitioner Maribago is a corporation operating a resort hotel and restaurant in Barangay Maribago, Lapu-
Lapu City. It hired respondent Dual as waiter and promoted him later as outlet cashier of its Poolbar/Allegro
Restaurant.
On 9 January 2005, around 6:30 p.m., a group of Japanese guests and their companions dined at
Allegro. Captain waiter Alvin Hiyas took their dinner orders comprising of six (6) sets of lamb and six (6) sets of
fish. As per company procedure, Hiyas forwarded one copy of the order slip to the kitchen and another copy to
respondent. Pursuant to the order slip, fourteen (14) sets of dinner were prepared by the chef. Hiyas and waiter
Genaro Mission, Jr. served twelve (12) set dinners to the guests, and another two (2) sets to their guides free of
charge (total of 14 sets of dinner).
After dinner, at around 9:00 p.m., the guests asked for their bill. Since Hiyas was attending to other guests,
he gave a signal to Mission to give the bill. Mission asked respondent Dual for the sales transaction receipt and
presented this to the guests. The guests paid the amount indicated on the receipt and thereafter left in a hurry.
The receipt printed at 10:40 p.m. shows that only P3,036.00 was remitted by cashier Dual corresponding to
six (6) sets of dinner. In view of the discrepancy between the order slip and the receipt issued, petitioner Maribago,
through its Human Resource Development (HRD) manager, issued memoranda, all dated 12 January 2005, requiring
respondent Dual, Alvin Hiyas, Ernesto Avenido and Basilio Alcoseba to explain why they should not be penalized for
violating House Rule 4.1 (dishonesty in any nature).
On 14 January 2005, the concerned employees were requested to attend a clarificatory hearing to be
conducted on 15 January 2005. The hearing was attended by respondent Dual, Human Resource Manager Ignacio
Hermias, Jr., Chief Security Officer Roland Cubillan, Captain Waiter Hiyas, Chef Arman, Bartender Avenido, Room
Service Waiter Alcoseba, Butcher Ryan Alegrado, John Marollana, and union officials. This was followed by another
clarificatory hearing conducted on 16 January 2005. It was in the 16 January 2005 hearing that waiter Mission gave
his testimony.
During the clarificatory hearing, butcher Alegrado testified that waiter Alcoseba went to the butchery looking
for the order slip for table no. 113. At around 9:45 p.m., waiter Alcoseba caused the alteration of the order slip to
reflect that six (6) orders were cancelled. Alegrado allegedly asked Alcoseba if the cook was already aware of the
cancellation, to which the latter answered oo, kahibaw na (yes, he is already aware).
In his written explanation, Alcoseba stated that he was not privy to the cancellation of orders since he was
busy attending to his room service duty. He claims that he saw the cancelled food orders at the waiters station but
insists that he did not have any part in the alteration of the order slip. During the clarificatory hearing, however, he
admitted that he altered the order slip by cancelling six (6) set dinners.
After the investigation, respondent Dual was found guilty of dishonesty for his fabricated statements and for
asking one of the waiters (Mission) to corroborate his allegations. He was terminated per memorandum dated 22
January 2005. Alcoseba was also terminated for dishonesty based on his admission that he altered the order slip.
Dual filed a complaint for unfair labor practice, illegal dismissal, non-payment of 13th month and separation
pay, and damages before the NLRC.

LA RULING:
Termination was without valid cause and ruled that respondent is entitled to separation pay

NLRC RULING
Set aside LA’s decision

CA RULING:
Reversed NLRC decision

ISSUE/S:
Whether or not Dual was illegally dismissed

HELD: NO

Angeles| Bajana | Balladares | Brillantes | Briones | Cabansag | Callanta | Chua | David|


De Leon | Gomez | Lopez | Macalino | Nostratis | Padilla | Reynon | Santos | Tan |Velasco
4E / 4F - 2018-2019
Page 720 of 920
LABOR REVIEW DIGEST
Atty. Joyrich Golangco

The law requires that an employer shall not terminate the services of an employee except for a just or
authorized cause. Otherwise, an employee unjustly dismissed from work is entitled to reinstatement and full
backwages. The law also requires the employer to observe due process in termination
cases.http://sc.judiciary.gov.ph/jurisprudence/2010/july2010/180660.htm - _ftn33 In Agabon v. National Labor
Relations Commission, we ruled that violation of the employees statutory right to due process makes the employer
liable to pay indemnity in the form of nominal damages. The law further requires that the burden of proving the cause
for termination rests with the employer.
In this case, we are in agreement that petitioners evidence proved that respondent is guilty of dishonesty
and of stealing money entrusted to him as cashier.Instead of reporting P10,100.00 as payment by the guests for their
dinner, respondent cashier only reported P3,036.00 as shown by the receipt which he admitted to have issued. The
receipt which bears his name NITO was printed at 22:40 (10:40 p.m.) or 1 hour and 40 minutes after the guests had
left at 9:00 p.m. Two other receipts were issued for the same amount at 22:39:55 and 22:40:01. Moreover,
respondents claim that he received P3,100.00 only and gave Mission P64.00 as change is not shown by the receipt
that he issued. The issued receipt does not show that change was given. In addition, the amount indicated in the
receipt does not coincide with Duals contention that only four (4) dishes were cancelled and two (2) dishes were
given free of charge. If such were the case, then the amount charged to the guests should have been for eight (8)
sets of dinner and not six (6) sets. As established during the clarificatory hearing, twelve (12) sets of dinner were
served to guests and two (2) dinner sets were given to the tour guides free of charge. It is clearly indicated in the
altered order slip that six (6) out of the twelve (12) sets of dinner were cancelled.
The allegation of Dual that six (6) dinner sets were indeed cancelled as evidenced by the dishes he
allegedly saw in the utensil station is negated by the testimonies of the kitchen staff (Chef Armand Galica, Butcher
Alegrado and Dessert-in-charge John Marollano) that twelve (12) set meals were served and consumed. These
testimonies coincide with the claim of waiters Hiyas and Mission that fourteen (14) sets of dinner were served. The
serving of food eliminates the argument of cancellation.
The alibi of cancellation has no leg to stand on. The standard operating procedure of Maribago dictates that
in cases of cancellation, the order slip has to be countersigned by the attending waiter (which in this case should
have been Chief Waiter Hiyas) but such was not so in this case.
The foregoing facts explain why Dual and Alcoseba tried twice to convince Mission to cover up their
crime. They even asked Mission to take the fall by asking him to admit that he altered the order slip from twelve (12)
sets of dinner to six (6) sets.
In fine, what is damning to the cause of Dual is the receipt which he admittedly issued. The receipt was
issued long after the guests had left (9:00 p.m.) and after the alteration of the order slip (9:45 p.m.) was done. Such
fact led us to the conclusion that he consented to and participated in the anomaly.
Respondent’s acts constitute serious misconduct which is a just cause for termination under the law. Theft
committed by an employee is a valid reason for his dismissal by the employer. Although as a rule this Court leans
over backwards to help workers and employees continue with their employment or to mitigate the penalties imposed
on them, acts of dishonesty in the handling of company property, petitioner’s income in this case, are a different
matter.
Withal, the law, in protecting the rights of the laborers, authorizes neither oppression nor self-destruction of
the employer. While the Constitution is committed to the policy of social justice and the protection of the working
class, it should not be supposed that every labor dispute will be automatically decided in favor of labor. The
management also has its own rights, as such, are entitled to respect and enforcement in the interest of simple fair
play. Out of its concern for those with less privileges in life, the Supreme Court has inclined more often than not
toward the worker and upheld his cause in his conflicts with the employer. Such favoritism, however, has not blinded
the Court to the rule that justice is in every case for the deserving, to be dispensed in the light of the established facts
and applicable law and doctrine.
Regarding the due process requirement, petitioner had complied with it as clearly shown by the facts.

Angeles| Bajana | Balladares | Brillantes | Briones | Cabansag | Callanta | Chua | David|


De Leon | Gomez | Lopez | Macalino | Nostratis | Padilla | Reynon | Santos | Tan |Velasco
4E / 4F - 2018-2019
Page 721 of 920
LABOR REVIEW DIGEST
Atty. Joyrich Golangco

22. CENTURY CANNING CORP. V. RAMIL


G.R. NO. 171630, AUGUST 8, 2010
PERALTA J.

DOCTRINE:
The right of an employer to dismiss an employee on the ground that it has lost its trust and confidence in
him must not be exercised arbitrarily and without just cause. Loss of trust and confidence, to be a valid
cause for dismissal, must be based on a willful breach of trust and founded on clearly established facts. The
basis for the dismissal must be clearly and convincingly established, but proof beyond reasonable doubt is
not necessary. It must rest on substantial grounds and not on the employers arbitrariness, whim, caprice or
suspicion; otherwise, the employee would eternally remain at the mercy of the employer.

FACTS:

Century Canning Corporation employed Vicente Randy Ramil in August 1993 as technical specialist. His job
included, among others, the preparation of the purchase requisition (PR) forms and capital expenditure (CAPEX)
forms, as well as the coordination with the purchasing department regarding technical inquiries on needed products
and services of petitioner's different departments.

On March 3, 1999, respondent prepared a CAPEX form for external fax modems and terminal server for signature of
Executive Vice-President Ricardo T. Po. The forms, however, were incomplete. The following day, with the form
apparently signed by Po, respondent transmitted it to the Main Office. The Purchasing Officer processed the paper
and found that some details in the CAPEX form were left blank. She also doubted the genuineness of the signature
of Po. She then transmitted the CAPEX form to Purchasing Manager Virgie Garcia and informed her of the
questionable signature of Po. Consequently, the request for the equipment was put on hold due to Po's forged
signature. However, due to the urgency of purchasing badly needed equipment, respondent was ordered to make
another CAPEX form, which was immediately transmitted to the Purchasing Department.

Suspecting him to have committed forgery, respondent was asked to explain in writing the events surrounding the
incident. He vehemently denied any participation in the alleged forgery. Respondent was suspended. He was later
terminated for loss of trust and confidence.

Respondent, then, filed a complaint for illegal dismissal before the NLRC.

LA/NLRC RULING:

LA Potenciano S. Canizares dismissed the complaint for lack of merit.

On appeal, the NLRC First Dvision declared respondent's dismissal to be illegal and directed petitioner to reinstate
respondent with full backwages and seniority rights and privileges. It found that petitioner failed to show clear and
convincing evidence that respondent was responsible for the forgery of the signature of Po in the CAPEX form.

Petitioner filed a motion for reconsideration. Thereafter, the NLRC reversed itself and upheld LA Canizares' dismissal
of his complaint. Respondent filed a motion for reconsideration, which was denied by the NLRC.

CA RULING:

The CA reinstated the earlier decision of the NLRC. It ordered petitioner to reinstate respondent, without loss of
seniority rights and privileges, and to pay respondent full backwages from the time his employment was terminated
on May 20, 1999 up to the time of the finality of its decision.

ISSUE:

WON the dismissal was for a just and authorized cause.

HELD:

(1) As correctly found by the NLRC in its original decision dated August 26, 2002, if respondent was the one who
forged the signature of Po in the CAPEX form, there was no need for him to endorse the same to Villanueva and
transmit it the next day. He could have easily forged the signature of Po on the same day that he prepared the

Angeles| Bajana | Balladares | Brillantes | Briones | Cabansag | Callanta | Chua | David|


De Leon | Gomez | Lopez | Macalino | Nostratis | Padilla | Reynon | Santos | Tan |Velasco
4E / 4F - 2018-2019
Page 722 of 920
LABOR REVIEW DIGEST
Atty. Joyrich Golangco

CAPEX form and submitted it on the very same day to petitioner's main office without passing through any officer
of petitioner.

(2) The law mandates that the burden of proving the validity of the termination of employment rests with the
employer. Failure to discharge this evidentiary burden would necessarily mean that the dismissal was not justified
and, therefore, illegal. Unsubstantiated suspicions, accusations, and conclusions of employers do not provide for
legal justification for dismissing employees. In case of doubt, such cases should be resolved in favor of labor,
pursuant to the social justice policy of labor laws and the Constitution.

(3) Petitioner based respondent's dismissal on its unsubstantiated suspicions and conclusion that since respondent
was the custodian and the one who prepared the CAPEX forms, he had the motive to commit the forgery.
However, as correctly found by the NLRC in its original Decision, respondent would not be benefited by the
purchase of the subject equipment. The equipment would be for the use of petitioner company.

(4) The right of an employer to dismiss an employee on the ground that it has lost its trust and confidence in him
must not be exercised arbitrarily and without just cause. Loss of trust and confidence, to be a valid cause for
dismissal, must be based on a willful breach of trust and founded on clearly established facts. The basis for the
dismissal must be clearly and convincingly established, but proof beyond reasonable doubt is not necessary. It
must rest on substantial grounds and not on the employers arbitrariness, whim, caprice or suspicion; otherwise,
the employee would eternally remain at the mercy of the employer.

Angeles| Bajana | Balladares | Brillantes | Briones | Cabansag | Callanta | Chua | David|


De Leon | Gomez | Lopez | Macalino | Nostratis | Padilla | Reynon | Santos | Tan |Velasco
4E / 4F - 2018-2019
Page 723 of 920
LABOR REVIEW DIGEST
Atty. Joyrich Golangco

23. TONGKO V. MANULIFE


G.R. NO. 167622, JUNE 29, 2010
BRION J.

DOCTRINE:
Under this legal situation, the only conclusion that can be made is that the absence of evidence showing
Manulife’s control over Tongko’s contractual duties points to the absence of any employer-employee
relationship between Tongko and Manulife. In the context of the established evidence, Tongko remained an
agent all along; although his subsequent duties made him a lead agent with leadership role, he was
nevertheless only an agent whose basic contract yields no evidence of means-and-manner control.

FACTS:
The contractual relationship between Tongko and Manulife had two basic phases. The first or initial phase began on
July 1, 1977, under a Career Agent’s Agreement that provided:
It is understood and agreed that the Agent is an independent contractor and nothing contained herein shall be
construed or interpreted as creating an employer-employee relationship between the Company and the Agent.
xxxx
a) The Agent shall canvass for applications for Life Insurance, Annuities, Group policies and other products offered
by the Company, and collect, in exchange for provisional receipts issued by the Agent, money due to or become due
to the Company in respect of applications or policies obtained by or through the Agent or from policyholders allotted
by the Company to the Agent for servicing, subject to subsequent confirmation of receipt of payment by the Company
as evidenced by an Official Receipt issued by the Company directly to the policyholder.
xxxx
The Company may terminate this Agreement for any breach or violation of any of the provisions hereof by the Agent
by giving written notice to the Agent within fifteen (15) days from the time of the discovery of the breach. No waiver,
extinguishment, abandonment, withdrawal or cancellation of the right to terminate this Agreement by the Company
shall be construed for any previous failure to exercise its right under any provision of this Agreement.
Either of the parties hereto may likewise terminate his Agreement at any time without cause, by giving to the other
party fifteen (15) days notice in writing.
Tongko additionally agreed (1) to comply with all regulations and requirements of Manulife, and (2) to maintain a
standard of knowledge and competency in the sale of Manulife’s products, satisfactory to Manulife and sufficient to
meet the volume of the new business, required by his Production Club membership.
The second phase started in 1983 when Tongko was named Unit Manager in Manulife’s Sales Agency Organization.
In 1990, he became a Branch Manager. In 1996, Tongko became a Regional Sales Manager.
Tongko’s gross earnings consisted of commissions, persistency income, and management overrides. Since the
beginning, Tongko consistently declared himself self-employed in his income tax returns. Thus, under oath, he
declared his gross business income and deducted his business expenses to arrive at his taxable business income.
Manulife withheld the corresponding 10% tax on Tongko’s earnings.
In 2001, Respondent Renato Vergel de Dios wrote Tongko a letter on concerns that were brought up during the
October 18, 2001 Metro North Sales Managers Meeting.

Subsequently, de Dios wrote Tongko another letter, dated December 18, 2001, terminating Tongko’s services.
Tongko responded by filing an illegal dismissal complaint.
LA/NLRC/CA RULING:
The LA decreed that no employer-employee relationship existed between the parties. However, the NLRC reversed
the labor arbiter’s decision on appeal; it found the existence of an employer-employee relationship and concluded
that Tongko had been illegally dismissed. In the petition for certiorari with the Court of Appeals (CA), the appellate
court found that the NLRC gravely abused its discretion in its ruling and reverted to the labor arbiter’s decision that no
employer-employee relationship existed between Tongko and Manulife.

FIRST SC RULING:
The SC reversed the CA ruling and found that an employment relationship existed between Tongko and Manulife.
Manulife filed a Motion for Reconsideration.

ISSUE:
WON there is an employment relationship.
HELD:
1. The primary evidence in the present case is the July 1, 1977 Agreement that governed and defined the
parties’ relations until the Agreement’s termination in 2001. This Agreement stood for more than two
decades and, based on the records of the case, was never modified or novated. It assumes primacy

Angeles| Bajana | Balladares | Brillantes | Briones | Cabansag | Callanta | Chua | David|


De Leon | Gomez | Lopez | Macalino | Nostratis | Padilla | Reynon | Santos | Tan |Velasco
4E / 4F - 2018-2019
Page 724 of 920
LABOR REVIEW DIGEST
Atty. Joyrich Golangco

because it directly dealt with the nature of the parties’ relationship up to the very end; moreover, both parties
never disputed its authenticity or the accuracy of its terms.

By the Agreement’s express terms, Tongko served as an "insurance agent" for Manulife, not as an
employee. To be sure, the Agreement’s legal characterization of the nature of the relationship cannot be
conclusive and binding on the courts; as the dissent clearly stated, the characterization of the juridical
relationship the Agreement embodied is a matter of law that is for the courts to determine. At the same time,
though, the characterization the parties gave to their relationship in the Agreement cannot simply be
brushed aside because it embodies their intent at the time they entered the Agreement, and they were
governed by this understanding throughout their relationship. At the very least, the provision on the absence
of employer-employee relationship between the parties can be an aid in considering the Agreement and its
implementation, and in appreciating the other evidence on record.

The parties’ legal characterization of their intent, although not conclusive, is critical in this case because this
intent is not illegal or outside the contemplation of law, particularly of the Insurance and the Civil Codes.
From this perspective, the provisions of the Insurance Code cannot be disregarded as this Code (as
heretofore already noted) expressly envisions a principal-agent relationship between the insurance
company and the insurance agent in the sale of insurance to the public. For this reason, we can take judicial
notice that as a matter of Insurance Code-based business practice, an agency relationship prevails in the
insurance industry for the purpose of selling insurance. The Agreement, by its express terms, is in
accordance with the Insurance Code model when it provided for a principal-agent relationship, and thus
cannot lightly be set aside nor simply be considered as an agreement that does not reflect the parties’ true
intent. This intent, incidentally, is reinforced by the system of compensation the Agreement provides, which
likewise is in accordance with the production-based sales commissions the Insurance Code provides.

2. That Tongko assumed a leadership role but nevertheless wholly remained an agent is the inevitable
conclusion that results from the reading of the Agreement (the only agreement on record in this case) and
his continuing role thereunder as sales agent, from the perspective of the Insurance and the Civil Codes and
in light of what Tongko himself attested to as his role as Regional Sales Manager. To be sure, this
interpretation could have been contradicted if other agreements had been submitted as evidence of the
relationship between Manulife and Tongko on the latter’s expanded undertakings. In the absence of any
such evidence, however, this reading – based on the available evidence and the applicable insurance and
civil law provisions – must stand, subject only to objective and evidentiary Labor Code tests on the existence
of an employer-employee relationship.

In applying such Labor Code tests, however, the enforcement of the Agreement during the course of the
parties’ relationship should be noted. From 1977 until the termination of the Agreement, Tongko’s
occupation was to sell Manulife’s insurance policies and products. Both parties acquiesced with the terms
and conditions of the Agreement. Tongko, for his part, accepted all the benefits flowing from the Agreement,
particularly the generous commissions.

Evidence indicates that Tongko consistently clung to the view that he was an independent agent selling
Manulife insurance products since he invariably declared himself a business or self-employed person in his
income tax returns. This consistency with, and action made pursuant to the Agreement were pieces of
evidence that were never mentioned nor considered in our Decision of November 7, 2008. Had they
been considered, they could, at the very least, serve as Tongko’s admissions against his interest. Strictly
speaking, Tongko’s tax returns cannot but be legally significant because he certified under oath the amount
he earned as gross business income, claimed business deductions, leading to his net taxable income. This
should be evidence of the first order that cannot be brushed aside by a mere denial. Even on a layman’s
view that is devoid of legal considerations, the extent of his annual income alone renders his claimed
employment status doubtful.

3. A glaring evidentiary gap for Tongko in this case is the lack of evidence on record showing that Manulife
ever exercised means-and-manner control, even to a limited extent, over Tongko during his ascent in
Manulife’s sales ladder. In 1983, Tongko was appointed unit manager. Inexplicably, Tongko never bothered
to present any evidence at all on what this designation meant. This also holds true for Tongko’s appointment
as branch manager in 1990, and as Regional Sales Manager in 1996. The best evidence of control – the
agreement or directive relating to Tongko’s duties and responsibilities – was never introduced as part of the
records of the case. The reality is, prior to de Dios’ letter, Manulife had practically left Tongko alone not only
in doing the business of selling insurance, but also in guiding the agents under his wing. As discussed

Angeles| Bajana | Balladares | Brillantes | Briones | Cabansag | Callanta | Chua | David|


De Leon | Gomez | Lopez | Macalino | Nostratis | Padilla | Reynon | Santos | Tan |Velasco
4E / 4F - 2018-2019
Page 725 of 920
LABOR REVIEW DIGEST
Atty. Joyrich Golangco

below, the alleged directives covered by de Dios’ letter, heretofore quoted in full, were policy directions and
targeted results that the company wanted Tongko and the other sales groups to realign with in their own
selling activities. This is the reality that the parties’ presented evidence consistently tells us.

What, to Tongko, serve as evidence of labor law control are the codes of conduct that Manulife imposes on
its agents in the sale of insurance. The mere presentation of codes or of rules and regulations, however, is
not per se indicative of labor law control as the law and jurisprudence teach us.

4. Even de Dios’ letter is not determinative of control as it indicates the least amount of intrusion into Tongko’s
exercise of his role as manager in guiding the sales agents. Strictly viewed, de Dios’ directives are merely
operational guidelines on how Tongko could align his operations with Manulife’s re-directed goal of being a
"big league player." The method is to expand coverage through the use of more agents. This requirement for
the recruitment of more agents is not a means-and-method control as it relates, more than anything else,
and is directly relevant, to Manulife’s objective of expanded business operations through the use of a bigger
sales force whose members are all on a principal-agent relationship. An important point to note here is that
Tongko was not supervising regular full-time employees of Manulife engaged in the running of the insurance
business; Tongko was effectively guiding his corps of sales agents, who are bound to Manulife through the
same Agreement that he had with Manulife, all the while sharing in these agents’ commissions through his
overrides. This is the lead agent concept mentioned above for want of a more appropriate term, since the
title of Branch Manager used by the parties is really a misnomer given that what is involved is not a specific
regular branch of the company but a corps of non-employed agents, defined in terms of covered territory,
through which the company sells insurance. Still another point to consider is that Tongko was not even
setting policies in the way a regular company manager does; company aims and objectives were simply
relayed to him with suggestions on how these objectives can be reached through the expansion of a non-
employee sales force.

5. As previously discussed, what simply happened in Tongko’s case was the grant of an expanded sales
agency role that recognized him as leader amongst agents in an area that Manulife defined. Whether this
consequently resulted in the establishment of an employment relationship can be answered by
concrete evidence that corresponds to the following questions:

a. as lead agent, what were Tongko’s specific functions and the terms of his additional engagement;
b. was he paid additional compensation as a so-called Area Sales Manager, apart from the commissions
he received from the insurance sales he generated;
c. what can be Manulife’s basis to terminate his status as lead agent;
d. can Manulife terminate his role as lead agent separately from his agency contract; and
e. to what extent does Manulife control the means and methods of Tongko’s role as lead agent?

Given this anemic state of the evidence, particularly on the requisite confluence of the factors determinative
of the existence of employer-employee relationship, the Court cannot conclusively find that the relationship
exists in the present case, even if such relationship only refers to Tongko’s additional functions. While a
rough deduction can be made, the answer will not be fully supported by the substantial evidence needed.

Under this legal situation, the only conclusion that can be made is that the absence of evidence showing
Manulife’s control over Tongko’s contractual duties points to the absence of any employer-employee
relationship between Tongko and Manulife. In the context of the established evidence, Tongko remained an
agent all along; although his subsequent duties made him a lead agent with leadership role, he was
nevertheless only an agent whose basic contract yields no evidence of means-and-manner control.

Angeles| Bajana | Balladares | Brillantes | Briones | Cabansag | Callanta | Chua | David|


De Leon | Gomez | Lopez | Macalino | Nostratis | Padilla | Reynon | Santos | Tan |Velasco
4E / 4F - 2018-2019
Page 726 of 920
LABOR REVIEW DIGEST
Atty. Joyrich Golangco

24. HOLY SPIRIT OF QC V. TAGUIAM


G.R. NO. 165565, JULY 14, 2008
QUISUMBING J.

DOCTRINE:

Notably, respondents negligence, although gross, was not habitual. In view of the considerable resultant
damage, however, we are in agreement that the cause is sufficient to dismiss respondent. Indeed, the
sufficiency of the evidence as well as the resultant damage to the employer should be considered in the
dismissal of the employee. In this case, the damage went as far as claiming the life of a child.

FACTS:

Respondent Corazon P. Taguiam was the Class Adviser of Grade 5 of School of the Holy Spirit of Quezon
City. On March 10, 2000, the class president requested permission to hold a year-end celebration at the school
grounds. The principal authorized the activity and allowed the pupils to use the swimming pool. In this connection,
respondent distributed the parents/guardians permit forms to the pupils.

Respondent admitted that Chiara Mae Federicos permit form was unsigned. Nevertheless, she concluded
that Chiara Mae was allowed by her mother to join the activity since her mother personally brought her to the school
with her packed lunch and swimsuit.

Unfortunately, Chiara Mae drowned. She was still alive when respondent rushed her to the General Malvar Hospital
where she was pronounced dead on arrival.

On May 23, 2000, petitioners issued a Notice of Administrative Charge to respondent for alleged gross negligence
and required her to submit her written explanation. Thereafter, petitioners conducted a clarificatory hearing which
respondent attended.

On July 31, 2000, petitioners dismissed respondent on the ground of gross negligence resulting to loss of trust and
confidence.

On July 25, 2001, respondent in turn filed a complaint against the school for illegal dismissal.

LA/NLRC/CA RULING:

The Labor Arbiter declared that respondent was validly terminated for gross neglect of duty. The NLRC affirmed the
dismissal of the complaint.

Aggrieved, respondent instituted a petition for certiorari before the Court of Appeals, which ruled in her favor.

ISSUE:

Whether respondents’ dismissal on the ground of gross negligence resulting to loss of trust and confidence was valid.

HELD:

(1) Under Article 282 of the Labor Code, gross and habitual neglect of duties is a valid ground for an employer
to terminate an employee. Gross negligence implies a want or absence of or a failure to exercise slight care
or diligence, or the entire absence of care. It evinces a thoughtless disregard of consequences without
exerting any effort to avoid them. Habitual neglect implies repeated failure to perform ones duties for a
period of time, depending upon the circumstances.

 Our perusal of the records leads us to conclude that respondent had been grossly negligent. First, it is
undisputed that Chiara Maes permit form was unsigned. Yet, respondent allowed her to join the activity
because she assumed that Chiara Maes mother has allowed her to join it by personally bringing her to the
school with her packed lunch and swimsuit.

Second, it was respondents responsibility as Class Adviser to supervise her class in all activities sanctioned
by the school. Thus, she should have coordinated with the school to ensure that proper safeguards, such as

Angeles| Bajana | Balladares | Brillantes | Briones | Cabansag | Callanta | Chua | David|


De Leon | Gomez | Lopez | Macalino | Nostratis | Padilla | Reynon | Santos | Tan |Velasco
4E / 4F - 2018-2019
Page 727 of 920
LABOR REVIEW DIGEST
Atty. Joyrich Golangco

adequate first aid and sufficient adult personnel, were present during their activity. She should have been
mindful of the fact that with the number of pupils involved, it would be impossible for her by herself alone to
keep an eye on each one of them.

As it turned out, since respondent was the only adult present, majority of the pupils were left unsupervised
when she followed the two pupils who sneaked out. In the light of the odds involved, respondent should
have considered that those who sneaked out could not have left the school premises since there were
guards manning the gates. The guards would not have allowed them to go out in their swimsuits and without
any adult accompanying them. But those who stayed at the pool were put at greater risk, when she left them
unattended by an adult.

1. Notably, respondents negligence, although gross, was not habitual. In view of the considerable resultant
damage, however, we are in agreement that the cause is sufficient to dismiss respondent. Indeed, the
sufficiency of the evidence as well as the resultant damage to the employer should be considered in the
dismissal of the employee. In this case, the damage went as far as claiming the life of a child.

Angeles| Bajana | Balladares | Brillantes | Briones | Cabansag | Callanta | Chua | David|


De Leon | Gomez | Lopez | Macalino | Nostratis | Padilla | Reynon | Santos | Tan |Velasco
4E / 4F - 2018-2019
Page 728 of 920
LABOR REVIEW DIGEST
Atty. Joyrich Golangco

25. YRASUEGUI VS. PHILIPPINE AIRLINES


GR NO: 168081 DATE: OCTOBER 17, 2008
PONENTE: REYES, J.

Doctrine: In fine, We hold that the obesity of petitioner, when placed in the context of his work as flight
attendant, becomes an analogous cause under Article 282(e) of the Labor Code that justifies his dismissal
from the service. His obesity may not be unintended, but is nonetheless voluntary. As the CA correctly puts
it, [v]oluntariness basically means that the just cause is solely attributable to the employee without any
external force influencing or controlling his actions. This element runs through all just causes under Article
282, whether they be in the nature of a wrongful action or omission. Gross and habitual neglect, a
recognized just cause, is considered voluntary although it lacks the element of intent found in Article 282(a),
(c), and (d)

FACTS:

Petitioner Armando G. Yrasuegui was a former international flight steward of Philippine Airlines, Inc. (PAL). He
stands five feet and eight inches (58) with a large body frame. The proper weight for a man of his height and body
structure is from 147 to 166 pounds, the ideal weight being 166 pounds, as mandated by the Cabin and Crew
Administration Manual of PAL.

The weight problem of petitioner dates back to 1984. Back then, PAL advised him to go on an extended vacation
leave from December 29, 1984 to March 4, 1985 to address his weight concerns. Apparently, petitioner failed to meet
the company’s weight standards, prompting another leave without pay from March 5, 1985 to November 1985.

After meeting the required weight, petitioner was allowed to return to work. But petitioners weight problem recurred.
He again went on leave without pay from October 17, 1988 to February 1989.

On April 26, 1989, petitioner weighed 209 pounds, 43 pounds over his ideal weight. In line with company policy, he
was removed from flight duty effective May 6, 1989 to July 3, 1989. He was formally requested to trim down to his
ideal weight and report for weight checks on several dates. He was also told that he may avail of the services of the
company physician should he wish to do so. He was advised that his case will be evaluated on July 3, 1989.

On February 25, 1989, petitioner underwent weight check. It was discovered that he gained, instead of losing, weight.
He was overweight at 215 pounds, which is 49 pounds beyond the limit. Consequently, his off-duty status was
retained. he was directed to report every two weeks for weight checks.

Petitioner failed to report for weight checks. Despite that, he was given one more month to comply with the weight
requirement. As usual, he was asked to report for weight check on different dates. He was reminded that his
grounding would continue pending satisfactory compliance with the weight standards.

On November 13, 1992, PAL finally served petitioner a Notice of Administrative Charge for violation of company
standards on weight requirements. He was given ten (10) days from receipt of the charge within which to file his
answer and submit controverting evidence.

Petitioner did not deny he was overweight but claimed that it was condoned by PAL because of inaction since 1988
and claimed that he was discriminated against employees that are similarly situated.

PAL terminated his employment, thus, he filed for Illegal dismissal.

The Labor Arbiter held that the weight standards of PAL are reasonable in view of the nature of the job of petitioner.
However, the weight standards need not be complied with under pain of dismissal since his weight did not hamper
the performance of his duties. Assuming that it did, petitioner could be transferred to other positions where his weight
would not be a negative factor. NLRC affirmed. CA reversed. the weight standards of PAL are meant to be a
continuing qualification for an employees position. The failure to adhere to the weight standards is an analogous
cause for the dismissal of an employee under Article 282(e) of the Labor Code in relation to Article 282(a). It is not
willful disobedience as the NLRC seemed to suggest. Said the CA, the element of willfulness that the NLRC decision
cites is an irrelevant consideration in arriving at a conclusion on whether the dismissal is legally proper. In other
words, the relevant question to ask is not one of willfulness but one of reasonableness of the standard and whether
or not the employee qualifies or continues to qualify under this standard.

Angeles| Bajana | Balladares | Brillantes | Briones | Cabansag | Callanta | Chua | David|


De Leon | Gomez | Lopez | Macalino | Nostratis | Padilla | Reynon | Santos | Tan |Velasco
4E / 4F - 2018-2019
Page 729 of 920
LABOR REVIEW DIGEST
Atty. Joyrich Golangco

Petitioner, though, advances a very interesting argument. He claims that obesity is a physical abnormality and/or
illness. Relying on Nadura v. Benguet Consolidated, Inc., he says his dismissal is illegal:

ISSUE/S:
WHETHER OR NOT THE COURT OF APPEALS GRAVELY ERRED IN HOLDING THAT PETITIONERS OBESITY
CAN BE A GROUND FOR DISMISSAL UNDER PARAGRAPH (e) OF ARTICLE 282 OF THE LABOR CODE OF
THE PHILIPPINES

HELD:

The reliance on Nadura is off-tangent. The factual milieu in Nadura is substantially different from the case at bar.
First, Nadura was not decided under the Labor Code. The law applied in that case was Republic Act (RA) No. 1787.
Second, the issue of flight safety is absent in Nadura, thus, the rationale there cannot apply here. Third, in Nadura,
the employee who was a miner, was laid off from work because of illness, i.e., asthma. Here, petitioner was
dismissed for his failure to meet the weight standards of PAL. He was not dismissed due to illness. Fourth, the issue
in Nadura is whether or not the dismissed employee is entitled to separation pay and damages. Here, the issue
centers on the propriety of the dismissal of petitioner for his failure to meet the weight standards of PAL. Fifth, in
Nadura, the employee was not accorded due process. Here, petitioner was accorded utmost leniency. He was given
more than four (4) years to comply with the weight standards of PAL.

In the case at bar, the evidence on record militates against petitioners claims that obesity is a disease. That he was
able to reduce his weight from 1984 to 1992 clearly shows that it is possible for him to lose weight given the proper
attitude, determination, and self-discipline. Indeed, during the clarificatory hearing on December 8, 1992, petitioner
himself claimed that [t]he issue is could I bring my weight down to ideal weight which is 172, then the answer is yes. I
can do it now.

True, petitioner claims that reducing weight is costing him a lot of expenses. However, petitioner has only himself to
blame. He could have easily availed the assistance of the company physician, per the advice of PAL. He chose to
ignore the suggestion. In fact, he repeatedly failed to report when required to undergo weight checks, without offering
a valid explanation. Thus, his fluctuating weight indicates absence of willpower rather than an illness.

Angeles| Bajana | Balladares | Brillantes | Briones | Cabansag | Callanta | Chua | David|


De Leon | Gomez | Lopez | Macalino | Nostratis | Padilla | Reynon | Santos | Tan |Velasco
4E / 4F - 2018-2019
Page 730 of 920
LABOR REVIEW DIGEST
Atty. Joyrich Golangco

26. JOHN HANCOCK LIFE INSURANCE VS. DAVIS


GR NO: 169549 DATE: SEPTEMBER 3, 2009
PONENTE: CORONA, J.

Doctrine: For an employee to be validly dismissed for a cause analogous to those enumerated in Article 282,
the cause must involve a voluntary and/or willful act or omission of the employee.

FACTS:

Respondent Joanna Cantre Davis was agency administration officer of petitioner John Hancock Life Insurance
Corporation.

On October 18, 2000, Patricia Yuseco, petitioners corporate affairs manager, discovered that her wallet was missing.
She immediately reported the loss of her credit cards to AIG and BPI Express. To her surprise, she was informed that
Patricia Yuseco had just made substantial purchases using her credit cards in various stores in the City of Manila.
She was also told that a proposed transaction in Abensons-Robinsons Place was disapproved because she gave the
wrong information upon verification.

Because loss of personal property among its employees had become rampant in its office, petitioner sought the
assistance of the National Bureau of Investigation (NBI). The NBI, in the course of its investigation, obtained a
security video from Abensons showing the person who used Yusecos credit cards. Yuseco and other witnesses
positively identified the person in the video as respondent.

Petitioner placed respondent under preventive suspension and instructed her to cooperate with its ongoing
investigation. Instead of doing so, however, respondent filed a complaint for illegal dismissal alleging that petitioner
terminated her employment without cause.

LA ruled that there was a valid cause of dismissal. NLRC affirmed. CA reversed the decision

Petitioner essentially argues that the ground for an employees dismissal need only be proven by substantial
evidence. Thus, the dropping of charges against an employee (specially on a technicality such as lack of proper
verification) or his subsequent acquittal does not preclude an employer from dismissing him due to serious
misconduct.

ISSUE/S:
Whether or not petitioner substantially proved the presence of valid cause for respondent’s termination.

HELD:
Misconduct involves the transgression of some established and definite rule of action, forbidden act, a dereliction of
duty, willful in character, and implies wrongful intent and not mere error in judgment. For misconduct to be serious
and therefore a valid ground for dismissal, it must be:

1. of grave and aggravated character and not merely trivial or unimportant and

2. connected with the work of the employee.

In this case, petitioner dismissed respondent based on the NBIs finding that the latter stole and used Yusecos credit
cards. But since the theft was not committed against petitioner itself but against one of its employees, respondents
misconduct was not work-related and therefore, she could not be dismissed for serious misconduct.

Nonetheless, Article 282(e) of the Labor Code talks of other analogous causes or those which are susceptible of
comparison to another in general or in specific detail. For an employee to be validly dismissed for a cause analogous
to those enumerated in Article 282, the cause must involve a voluntary and/or willful act or omission of the employee.
A cause analogous to serious misconduct is a voluntary and/or willful act or omission attesting to an employees
moral depravity. Theft committed by an employee against a person other than his employer, if proven by substantial
evidence, is a cause analogous to serious misconduct.

Angeles| Bajana | Balladares | Brillantes | Briones | Cabansag | Callanta | Chua | David|


De Leon | Gomez | Lopez | Macalino | Nostratis | Padilla | Reynon | Santos | Tan |Velasco
4E / 4F - 2018-2019
Page 731 of 920
LABOR REVIEW DIGEST
Atty. Joyrich Golangco

Did petitioner substantially prove the existence of valid cause for respondents separation? Yes. The labor arbiter and
the NLRC relied not only on the affidavits of the NBIs witnesses but also on that of respondent. They likewise
considered petitioners own investigative findings. Clearly, they did not merely adopt the findings of the NBI but
independently assessed evidence presented by the parties. Their conclusion (that there was valid cause for
respondents separation from employment) was therefore supported by substantial evidence.

Angeles| Bajana | Balladares | Brillantes | Briones | Cabansag | Callanta | Chua | David|


De Leon | Gomez | Lopez | Macalino | Nostratis | Padilla | Reynon | Santos | Tan |Velasco
4E / 4F - 2018-2019
Page 732 of 920
LABOR REVIEW DIGEST
Atty. Joyrich Golangco

27. HOCHENG PHILIPPINES CORP. VS. FARRALES


GR NO: 211497 DATE: MARCH 18, 2015
PONENTE: REYES, J.

Doctrine: To validly dismiss an employee, the law requires the employer to prove the existence of any of the
valid or authorized causes, which, as enumerated in Article 282 of the Labor Code, are: (a) serious
misconduct or willful disobedience by the employee of the lawful orders of his employer or the latter’s
representative in connection with his work; (b) gross and habitual neglect by the employee of his duties; (c)
fraud or willful breach by the employee of the trust reposed in him by his employer or his duly authorized
representative; (d) commission of a crime or offense by the employee against the person of his employer or
any immediate member of his family or his duly authorized representative; and (e) other causes analogous to
the foregoing.

FACTS:

Farrales was first employed by HPC on May 12, 1998 as Production Operator, followed by promotions as (1)
Leadman in 2004, (2) Acting Assistant Unit Chief in 2007, and (3) Assistant Unit Chief of Production in 2008, a
supervisory position with a monthly salary of 17,600.00. He was a consistent recipient of citations for outstanding
performance, as well as appraisal and year-end bonuses.

On December 2, 2009, a report reached HPC management that a motorcycle helmet of an employee, Reymar Solas
(Reymar), was stolen at the parking lot within its premises. A CCTV footage confirmed that it was Farrales who took
the missing helmet.

Later that day, HPC sent Farrales a notice to explain his involvement in the alleged theft. After due hearing, Farrales
was issued a notice of termination which provides that “stealing from the company, its employees and officials, or
from its contractors, visitors or clients,” is akin to serious misconduct and fraud or willful breach by the employee of
the trust reposed in him by his employer or duly authorized representative, which are just causes for termination of
employment under Article 282 of the Labor Code.

On March 25, 2010, Farrales filed a complaint for illegal dismissal, non-payment of appraisal and mid-year bonuses,
service incentive leave pay and 13th month pay. He also prayed for reinstatement, or in lieu thereof, separation pay
with full backwages, plus moral and exemplary damages and attorney’s fees. During the mandatory conference, HPC
paid Farrales 10,914.51, representing his 13th month pay for the period of January to February 2010 and vacation
leave/sick leave conversion. Farrales agreed to waive his claim for incentive bonus.

LA ruled in favor of Farrales, On appeal, NLRC reversed the ruling. On petition for certiorari to the CA, Farrales
sought to refute the NLRC’s factual finding that he committed theft, as well as to question NLRC’s jurisdiction over
HPC’s appeal for non-payment of appeal fees. But the CA found that HPC was able to perfect its appeal by posting a
bond equivalent to the monetary award of 897,893.37 and paying the appeal fees by postal money order.

ISSUE/S:
Whether or not theft is a valid/just cause for termination

HELD:

To validly dismiss an employee, the law requires the employer to prove the existence of any of the valid or authorized
causes,24 which, as enumerated in Article 282 of the Labor Code, are: (a) serious misconduct or willful disobedience
by the employee of the lawful orders of his employer or the latter’s representative in connection with his work; (b)
gross and habitual neglect by the employee of his duties; (c) fraud or willful breach by the employee of the trust
reposed in him by his employer or his duly authorized representative; (d) commission of a crime or offense by the
employee against the person of his employer or any immediate member of his family or his duly authorized
representative; and (e) other causes analogous to the foregoing.25 As a supervisorial employee, Farrales is
admittedly subject to stricter rules of trust and confidence, and thus pursuant to its management prerogative HPC
enjoys a wider latitude of discretion to assess his continuing trustworthiness, than if he were an ordinary rank-and-file
employee.26 HPC therefore insists that only substantial proof of Farrales’ guilt for theft is needed to establish the just
causes to dismiss him, as the NLRC lengthily asserted in its decision.

Article 4 of the Labor Code mandates that all doubts in the implementation and interpretation of the provisions thereof
shall be resolved in favor of labor. Consistent with the State’s avowed policy to afford protection to labor, as Article 3

Angeles| Bajana | Balladares | Brillantes | Briones | Cabansag | Callanta | Chua | David|


De Leon | Gomez | Lopez | Macalino | Nostratis | Padilla | Reynon | Santos | Tan |Velasco
4E / 4F - 2018-2019
Page 733 of 920
LABOR REVIEW DIGEST
Atty. Joyrich Golangco

of the Labor Code and Section 3, Article XIII of the 1987 Constitution have enunciated, particularly in relation to the
worker’s security of tenure, the Court held that “[t]o be lawful, the cause for termination must be a serious and grave
malfeasance to justify the deprivation of a means of livelihood. This is merely in keeping with the spirit of our
Constitution and laws which lean over backwards in favor of the working class, and mandate that every doubt must
be resolved in their favor.” Moreover, the penalty imposed on the erring employee ought to be proportionate to the
offense, taking into account its nature and surrounding circumstances.

The Court has always taken care, therefore, that the employer does not invoke any baseless justification, much less
management prerogative, as a subterfuge by which to rid himself of an undesirable worker, and thus in exceptional
cases the Court has never hesitated to delve into the NLRC’s factual conclusions where evidence was found
insufficient to support them, or too much was deduced from the bare facts submitted by the parties, or the LA and the
NLRC came up with conflicting positions, as is true in this case.

As aptly pointed out by the LA, while HPC has the onus probandi that the taking of Reymar’s helmet by Farrales was
with intent to gain, it failed to discharge this burden, as shown by the following circumstances: Farrales sought and
obtained the permission of Eric, his co-employee as well as barangay co-resident, to borrow his helmet; at the
parking lot, Farrales asked another employee, Andy, to fetch a yellow helmet from one of the parked motorcycles,
mistakenly thinking it belonged to Eric (whom he knew owned two helmets); the following day, November 28,
Farrales asked Eric why he had not dropped by his house to get his helmet, and Eric replied that Farrales got the
wrong helmet because he still had his other helmet with him; Farrales immediately sought the help of the company
guards to locate the owner of the yellow helmet, who turned out to be Reymar; Farrales apologized to Reymar for his
mistake, and his apology was promptly accepted. All these circumstances belie HPC’s claim that Farrales took
Reymar’s helmet with intent to gain, the LA said.

Angeles| Bajana | Balladares | Brillantes | Briones | Cabansag | Callanta | Chua | David|


De Leon | Gomez | Lopez | Macalino | Nostratis | Padilla | Reynon | Santos | Tan |Velasco
4E / 4F - 2018-2019
Page 734 of 920
LABOR REVIEW DIGEST
Atty. Joyrich Golangco

28. ROQUE B. BENITEZ and SANTA FE LABOR UNION-FEDERATION OF FREE WORKERS vs. SANTA
FE MOVING AND RELOCATION SERVICESNEDIT KURANGIL
GR No. 208163 Date: April 20, 2015
Ponente: Brion, J.

Doctrine: Benitez's offense constituted a serious misconduct as defined by law. His display of
insolent and disrespectful behavior, in utter disregard of the time and place of its occurrence, had
very much to do with his work. He set a bad example as a union officer and as a crew leader of a
vital division of the company.

FACTS:
Petitioners filed a complaint for ULP and illegal dismissal, with money claims against respondents. The
company is engaged in providing relocation and moving services, including visa, immigration and real estate
services. Benitez (the union’s Vice-President at the time), was its former packing and moving operator
(crew leader) since June 2001. Benitez alleged that the company served him a memorandum advising him
not to report for work effective immediately, thereby terminating his employment, supposedly on grounds of
serious misconduct or willful disobedience. He allegedly uttered abusive words against Kurangil during the
company’s Christmas Party. He bewailed that he was not given the opportunity to defend himself. Benitez
denies the accusation against him and argued that his dismissal constituted ULP as he was a union officer
and that it was undertaken to derail the conclusion of a collective bargaining agreement with the company.
He further argued that the penalty of dismissal is disproportionate to his alleged offense, considering that it
was committed during a casual gathering and had no connection to his work. The company required Benitez
to explain in writing why he should not be disciplined for serious misconduct and willful disobedience of its
lawful orders in connection with the incident. Benitez failed to comply and neither did he show remorse for
what he did. In view of Benitez’s failure to explain his side, the company issued a memorandum to
Benitez, terminating his employment effective on the same day, for clear violation of "Santa Fe Policy and
Procedure under Conduct and Behavior as well as Labor Code of the Philippines under Art. 282 – Serious
misconduct or willful disobedience by the employee of the lawful orders of his employer x x x."

LA RULING:Benitez is legally dismissed. Benitez, who was holding a position of trust and confidence as
packing and moving operator, committed a serious misconduct at the company’s Christmas Party by "hurling
obscene, insulting or offensive language against a superior," thereby losing the trust and confidence of his
employer.

PETITION TO THE SC:

ISSUE/S:
1. Is Benitez legally dismissed on the ground of serious misconduct?

HELD:
YES, The present case is distinguished with the case of Samson v. NLRC, where the SC held that
the employee was illegally dismiss
In Samson v. NLRC, it was held that: “The instant case [Samson v. NLRC] should be distinguished from the
previous cases where we held that the use of insulting and offensive language constituted gross misconduct
justifying an employee’s dismissal. In De la Cruz vs. NLRC, the dismissed employee shouted "sayang ang
pagka-professional mo!" and "putang ina mo" at the company physician when the latter refused to give him
a referral slip. In Autobus Workers’ Union (AWU) v. NLRC, the dismissed employee called his supervisor
"gago ka" and taunted the latter by saying "bakit anong gusto mo tang ina mo." In these cases, the
dismissed employees personally subjected their respective superiors to the foregoing verbal abuses. The
utter lack of respect for their superiors was patent. In contrast,when petitioner was heard to have uttered
the alleged offensive words against respondent company’s president and general manager, the latter was
not around.”
Here, the company acted swiftly, and decisively in Benitez's case, obviously and understandably,
because of the gravity and high visibility of his offense, which not only constituted a frontal verbal, and
nearly physical (the attempted beer bottle throwing), assault against Kurangil. Needless to say, Benitez's
outburst also caused grave embarrassment for the audience who witnessed the incident, including company
officials whom he likewise maligned, as well as company clients and guests. Under the foregoing
circumstances, we are convinced - as the Labor Arbiter, the NLRC and the CA had been - that Benitez's
offense constituted a serious misconduct as defined by law. His display of insolent and disrespectful

Angeles| Bajana | Balladares | Brillantes | Briones | Cabansag | Callanta | Chua | David|


De Leon | Gomez | Lopez | Macalino | Nostratis | Padilla | Reynon | Santos | Tan |Velasco
4E / 4F - 2018-2019
Page 735 of 920
LABOR REVIEW DIGEST
Atty. Joyrich Golangco

behavior, in utter disregard of the time and place of its occurrence, had very much to do with his work. He
set a bad example as a union officer and as a crew leader of a vital division of the company.

Angeles| Bajana | Balladares | Brillantes | Briones | Cabansag | Callanta | Chua | David|


De Leon | Gomez | Lopez | Macalino | Nostratis | Padilla | Reynon | Santos | Tan |Velasco
4E / 4F - 2018-2019
Page 736 of 920
LABOR REVIEW DIGEST
Atty. Joyrich Golangco

29. ST. LUKE’S MEDICAL CENTER, INC. v. MARIA THERESA V. SANCHEZ


GR No: 212054 Date: March 11, 2015
Ponente: Perlas-Bernabe, J.

Doctrine: Among the employer’s management prerogatives is the right to prescribe reasonable rules
and regulations necessary or proper for the conduct of its business or concern, to provide certain
disciplinary measures to implement said rules and to assure that the same would be complied with.
At the same time, the employee has the corollary duty to obey all reasonable rules, orders, and
instructions of the employer; and willful or intentional disobedience thereto, as a general rule,
justifies termination of the contract of service and the dismissal of the employee.

FACTS:
Respondent was hired by petitioner St. Luke’s Medical Center, Inc. (SLMC) as a Staff Nurse, and was
eventually assigned at SLMC, Quezon City’s Pediatric Unit until her termination for her purported violation
of SLMC’s Code of Discipline, particularly Section 1, Rule 1 on Acts of Dishonesty, i.e., Robbery, Theft,
Pilferage, and Misappropriation of Funds. Records reveal that at the end of her shift on May 29, 2011,
Sanchez passed through the SLMC Centralization Entrance/Exit where she was subjected to the standard
inspection procedure by the security personnel. In the course thereof, the Security Guard on-duty, SG
Manzanade, noticed a pouch in her bag and asked her to open the same. When opened, said pouch
contained the following assortment of medical stocks which were subsequently confiscated. Sanchez asked
SG Manzanade if she could just return the pouch inside the treatment room; however, she was not allowed
to do so. Instead, she was brought to the SLMC In-House Security Department (IHSD) where she was
directed to write an Incident Report explaining why she had the questioned items in her possession. She
complied with the directive and also submitted an undated handwritten letter of apology. An initial
investigation was also conducted by the SLMC Division of Nursing which thereafter served Sanchez a notice
to explain. Sanchez submitted an Incident Report Addendum, explaining that the questioned items came
from the medication drawers of patients who had already been discharged, and, as similarly practiced by
the other staff members, she started saving these items as excess stocks in her pouch, along with other
basic items that she uses during her shift. She then put the pouch inside the lowest drawer of the bedside
table in the treatment room for use in immediate procedures in case replenishment of stocks gets delayed.
However, on the day of the incident, she failed to return the pouch inside the medication drawer upon
getting her tri-colored pen and calculator and, instead, placed it inside her bag. Eventually, she forgot about
the same as she got caught up in work, until it was noticed by the guard on duty on her way out of
SMLC’s premises. Consequently, Sanchez was placed under preventive suspension until the conclusion of
the investigation by SLMC’s Employee and Labor Relations Department (ELRD) which, thereafter, required
her to explain why she should not be terminated from service for “acts of dishonesty” due to her
possession of the questioned items. In response, she submitted a letter which merely reiterated her claims
in her previous letter. She likewise requested for a case conference, which SLMC granted. After hearing her
side, SLMC informed Sanchez of its decision to terminate her employment.

LA RULING:Sanchez was validly dismissed for intentionally taking the property of SLMC’s clients for her
own personal benefit, which constitutes an act of dishonesty as provided under SLMC’s Code of Discipline.

NLRC RULING: Illegally dismissed. It declared that the alleged violation of Sanchez was a unique case,
considering that keeping excess hospital stocks or “hoarding” was an admitted practice amongst nurses in
the Pediatric Unit which had been tolerated by SLMC management for a long time.

CA RULING: Illegally dismissed.

PETITION TO THE SC:

ISSUE/S:
1. Whether or not respondent was illegally dismissed?

HELD:
NO. Respondent was legally dismissed.
Among the employer’s management prerogatives is the right to prescribe reasonable rules and
regulations necessary or proper for the conduct of its business or concern, to provide certain disciplinary
measures to implement said rules and to assure that the same would be complied with. At the same time,

Angeles| Bajana | Balladares | Brillantes | Briones | Cabansag | Callanta | Chua | David|


De Leon | Gomez | Lopez | Macalino | Nostratis | Padilla | Reynon | Santos | Tan |Velasco
4E / 4F - 2018-2019
Page 737 of 920
LABOR REVIEW DIGEST
Atty. Joyrich Golangco

the employee has the corollary duty to obey all reasonable rules, orders, and instructions of the employer;
and willful or intentional disobedience thereto, as a general rule, justifies termination of the contract of
service and the dismissal of the employee. Article 296 (formerly Article 282) of the Labor Code provides:
Article 296. Termination by Employer. - An employer may terminate an employment for any of the following
causes: (a) Serious misconduct or willful disobedience by the employee of the lawful orders of his employer
or his representative in connection with his work; x x x x Note that for an employee to be validly dismissed
on this ground, the employer’s orders, regulations, or instructions must be: (1) reasonable and lawful, (2)
sufficiently known to the employee, and (3) in connection with the duties which the employee has been
engaged to discharge.”
Section 1, Rule 1 of the SLMC Code of Discipline is further supplemented by the company policy
requiring the turn-over of excess medical supplies/items for proper handling and providing a restriction on
taking and bringing such items out of the SLMC premises without the proper authorization or “pass” from
the official concerned, which Sanchez was equally aware thereof. Nevertheless, Sanchez failed to turn-over
the questioned items and, instead, “hoarded” them, as purportedly practiced by the other staff members in
the Pediatric Unit. As it is clear that the company policies subject of this case are reasonable and lawful,
sufficiently known to the employee, and evidently connected with the latter’s work, the Court concludes that
SLMC dismissed Sanchez for a just cause.

Angeles| Bajana | Balladares | Brillantes | Briones | Cabansag | Callanta | Chua | David|


De Leon | Gomez | Lopez | Macalino | Nostratis | Padilla | Reynon | Santos | Tan |Velasco
4E / 4F - 2018-2019
Page 738 of 920
LABOR REVIEW DIGEST
Atty. Joyrich Golangco

30. MAERSK-FILIPINAS CREWING, INC., A.P. MOLLER SINGAPORE PTE. LIMITED, AND JESUS
AGBAYANI vs. TORIBIO C. AVESTRUZ
GR No: 207010 Date: February 18, 2015
Ponente: Torres, Jr., J.

Doctrine: An erring seaman is given a written notice of the charge against him and is afforded an
opportunity to explain or defend himself. Should sanctions be imposed, then a written notice of
penalty and the reasons for it shall be furnished the erring seafarer. It is only in the exceptional
case of clear and existing danger to the safety of the crew or vessel that the required notices are
dispensed with.

FACTS:
Maersk-Filipinas Crewing, Inc. (Maersk), on behalf of its foreign principal hired respondent Avestruz as Chief
Cook on board the vessel M/V Nedlloyd Drake for a period of six (6) months Avestruz boarded the vessel
on May 4, 2011. On June 22, 2011, in the course of the weekly inspection of the vessel’s galley, Captain
Charles C. Woodward noticed that the cover of the garbage bin in the kitchen near the washing area was
oily. Woodward called Avestruz. An altercation ensued. On the very same day, Captain Woodward informed
Avestruz that he would be dismissed from service and be disembarked in India. On July 3, 2011, Avestruz
was disembarked in Colombo, Sri Lanka and arrived in the Philippines on July 4, 2011. Consequently,
Avestruz filed a complaint for illegal dismissal, payment for the unexpired portion of his contract, damages,
and attorney’s fees against Maersk et al. and that Captain Woodward failed to observe the provisions under
Section 17 of the Philippine Overseas Employment Administration (POEA) Standard Employment Contract
(POEA-SEC) on disciplinary procedures. Maersk maintained that Avestruz was dismissed for a just and valid
cause and is, therefore, not entitled to recover his salary for the unexpired portion of his contract. They
contend that Avestruz was dismissed on the ground of insubordination, consisting of his “repeated failure to
obey his superior’s order to maintain cleanliness in the galley of the vessel” as well as his act of “insulting
a superior officer by words or deeds.” In support of this contention, petitioners presented as evidence the e-
mails sent by Captain Woodward, both dated June 22, 2011, and time-stamped 10:07 a.m. and 11:40 a.m.,
respectively.

CA RULING: The CA held in favor of Avestruz and directed petitioners to pay him, jointly and severally, the
full amount of his placement fee and deductions made, with interest at twelve percent (12%) per annum, as
well as his salaries for the unexpired portion of his contract, and attorney’s fees of ten percent (10%) of
the total award. All other money claims were denied for lack of merit

PETITION TO THE SC:

ISSUE/S:
1. WON Avestruz’ dismissal was proper pursuant to the POEA-SEC

HELD:
NO. An erring seaman is given a written notice of the charge against him and is afforded an opportunity
to explain or defend himself. Should sanctions be imposed, then a written notice of penalty and the reasons
for it shall be furnished the erring seafarer. It is only in the exceptional case of clear and existing danger
to the safety of the crew or vessel that the required notices are dispensed with; but just the same, a
complete report should be sent to the manning agency, supported by substantial evidence of the findings.
Here, there is dearth of evidence to show that Avestruz had been given a written notice of the
charge against him, or that he was given the opportunity to explain or defend himself. The statement given
by Captain Woodward requiring him to explain in writing the events that transpired at the galley in the
morning of June 22, 2011 hardly qualifies as a written notice of the charge against him, nor was it an
opportunity for Avestruz to explain or defend himself. While Captain Woodward claimed in his e-mail that he
conducted a “disciplinary hearing” informing Avestruz of his inefficiency, no evidence was presented to
support the same. Neither was Avestruz given a written notice of penalty and the reasons for its imposition.

Angeles| Bajana | Balladares | Brillantes | Briones | Cabansag | Callanta | Chua | David|


De Leon | Gomez | Lopez | Macalino | Nostratis | Padilla | Reynon | Santos | Tan |Velasco
4E / 4F - 2018-2019
Page 739 of 920
LABOR REVIEW DIGEST
Atty. Joyrich Golangco

31. WATERFRONT CEBU CITY CASINO HOTEL, INC. AND MARCO PROTACIO v. ILDEBRANDO LEDESMA
G.R. No. 197556 March 25, 2015
FACTS: Respondent was employed as a House Detective at Waterfront. On the basis of the complaints filed before
Waterfront by Christe Mandal, a supplier of a concessionaire of Waterfront, and Rosanna Lofranco, who was seeking
a job at the same hotel, Ledesma was dismissed from employment. From the affidavits and testimonies of Christe
Mandal and Rosanna Lofranco during the administrative hearings conducted by Waterfront, the latter found, among
others, that Ledesma kissed and mashed the breasts of Christe Mandal inside the hotel’s elevator, and exhibited his
penis and asked Rosanna Lofranco to masturbate him at the conference room of the hotel. Ledesma filed a
complaint for illegal dismissal.
LA RULING: The LA found that the allegations leveled against Ledesma are mere concoctions, and concluded that
Ledesma was illegally dismissed, and ordered the petitioner among others to reinstate Ledesma.
CA RULING: The CA entertained the petition and reinstated the decision of the LA.
ISSUE: Whether or not the improper behavior of Ledesma a just cause for dismissal
RULING: The CA ruled in favor of Ledesma since it believed his version that the complainants merely invented the
accusations against him because Waterfront failed to present as evidence the CCTV footages of the alleged
lascivious conduct of Ledesma inside the elevator and the conference room. But this argument was not even raised
by Ledesma himself and it was only the CA which utilized this as a justification to bolster its findings that Ledesma
did not commit any infraction. This being a labor case, the evidence required is only substantial evidence which was
adequately established here by the positive and credible testimonies of the complainants.

Notably, Ledesma never refuted, at the administrative investigation level at Waterfront, and even at the proceedings
before the LA, NLRC, and the CA, the allegations leveled against him by Rosanna Lofranco that, after deluding her to
perform a massage on him, Ledesma exhibited to her his penis and requested that he be masturbated while inside
the conference room of the hotel. If not for the position of Ledesma as a House Detective, he will not have access to
the conference room nor will he know that the premises is not monitored through a closed-circuit television,52 thus
giving him the untrammeled opportunity to accomplish his lewd design on the unsuspecting victim. Such acts of
Ledesma constituted misconduct or improper behavior53 which is a just cause for his dismissal.

Angeles| Bajana | Balladares | Brillantes | Briones | Cabansag | Callanta | Chua | David|


De Leon | Gomez | Lopez | Macalino | Nostratis | Padilla | Reynon | Santos | Tan |Velasco
4E / 4F - 2018-2019
Page 740 of 920
LABOR REVIEW DIGEST
Atty. Joyrich Golangco

32. MELVIN P. MALLO v. SOUTHEAST ASIAN COLLEGE, INC. AND EDITA ENATSU
G.R. No. 212861, October 14, 2015

DOCTRINE: As defined under established jurisprudence, abandonment is the deliberate and unjustified
refusal of an employee to resume his employment. It constitutes neglect of duty and is a just cause for
termination of employment under paragraph (b) of Article 282 [now Article 296] of the Labor Code. To
constitute abandonment, however, there must be a clear and deliberate intent to discontinue one's
employment without any intention of returning. In this regard, two elements must concur: (1) failure to report
for work or absence without valid or justifiable reason; and (2) a clear intention to sever the employer-
employee relationship, with the second element as the more determinative factor and being manifested by
some overt acts. Otherwise stated, absence must be accompanied by overt acts unerringly pointing to the
fact that the employee simply does not want to work anymore.

FACTS: The instant case arose from a complaint for, inter alia, unfair labor practice, illegal dismissal, underpayment
of salary/wages, damages, and attorney's fees filed by Mallo against respondents Southeast Asian College, Inc.
(SACI) and its Executive President/Chief Executive Officer, Edita F. Enatsu (Enatsu; collectively, respondents) before
the NLRC. Mallo alleged that SACI first hired him as a Probationary Full-Time Faculty Member of its College of
Nursing and Midwifery with the rank of Assistant Professor C for the Second Semester of School Year (SY) 2007-
2008 and, thereafter, his employment was renewed for the succeeding semesters until the Summer Semester of SY
2010-2011. On June 3 and 8, 2011, Mallo inquired about his teaching load for the First Semester of SY 2011-2012,
but SACI only responded that teaching assignments for the semester were yet to be given to faculty members.
Thereafter, on June 15, 2011, he learned from a co-professor that faculty meetings were conducted on June 9 and
10, 2011 whereby teaching loads were distributed to the professors. Upon learning of this development, Mallo went
again to SACI to confront the Dean of the College of Nursing, Dr. Clarita D. Curato (Dr. Curato). Claiming that he was
already a permanent employee of SACI, having been a professor of SACI for almost four (4) years since his first
teaching assignment in November 2007, Mallo demanded that he be given his corresponding teaching load.
However, Dr. Curato simply retorted that the school was under no obligation to give him any teaching loads for the
semester because he was merely a contractual employee. As such, Mallo was constrained to file the instant
complaint against respondents.
In their defense, respondents denied dismissing Mallo, maintaining that as early as April 2011 and as evidenced by
Dr. Curato's letter to the Medical Center Chief II of the National Center for Mental Health (NCMH), SACI already gave
Mallo his teaching load for the First Semester of SY 2011- 2012 - as Clinical Instructor for the College of Nursing's
Preceptorship Program, an on-the-job mentoring and ongoing clinical experience of students under the Nursing
Related Learning Experience (NLRE) curriculum, to be conducted at NCMH. Unfortunately, Mallo twice failed the
qualifying test required for the job. This notwithstanding, SACI endeavored to give Mallo a teaching load by
appointing him as a Clinical Instructor for Preceptorship Program to be conducted at the United Doctors Medical
Center (UDMC) instead, beginning June 23, 2011, which he accepted. However, a day before he was set to start as
a Clinical Instructor at UDMC, Mallo asked for a change in schedule, which was denied as it would entail a reshuffle
of the entire NLRE schedule of the school. On June 23 to 25, 2011, Mallo did not attend his classes at UDMC. This
prompted a S ACI official to contact Mallo if he would report for work the following day, to which the latter allegedly
replied in the negative as his schedule with SACI conflicted with his new employment. Thereafter, SACI never heard
from Mallo again until he filed the instant case.

ISSUE: W/N there was no illegal dismissal and that Mallo abandoned his job.

RULING: The petition is partly meritorious. At the outset, the Court notes that the LA, the NLRC, and the CA were
one in declaring that Mallo's employment with SACI had already attained the status of a regular employee. However,
a scrutiny of the records reveals that their factual findings differ as to whether or not Mallo was illegally dismissed or
had abandoned his job. In instances where there is a divergence in the findings of facts of the NLRC and that of the
CA, there is a need for the Court to review the records to determine which of them should be preferred as more
conformable to evidentiary facts, as in this case.

Here, Mallo insists that respondents illegally dismissed him because the latter failed to give him any teaching load for
the First Semester of SY 2011-2012. On the other hand, respondents vehemently deny Mallo's claims, maintaining
that they promptly gave him his teaching assignment and that the latter even initially accepted the same, but such
assignment was eventually turned down due to a conflict in schedule with his new employment in another school. His
dismissal was not illegal fully rests on the employer; the failure to discharge such onus would mean that the dismissal
was not justified and, therefore, illegal.

Angeles| Bajana | Balladares | Brillantes | Briones | Cabansag | Callanta | Chua | David|


De Leon | Gomez | Lopez | Macalino | Nostratis | Padilla | Reynon | Santos | Tan |Velasco
4E / 4F - 2018-2019
Page 741 of 920
LABOR REVIEW DIGEST
Atty. Joyrich Golangco

The records readily show that as early as April 2011, respondents already assigned Mallo a teaching load for the
First Semester of SY 2011-2012 as a Clinical Instructor for SACI students to be assigned at NCMH, which the latter
accepted. Unfortunately, Mallo failed the qualifying tests at NCMH twice, thus, virtually disqualifying him from
performing his work as SACFs Clinical Instructor thereat. Despite these developments, respondents were able to
remedy the situation, albeit belatedly, by assigning Mallo as a Clinical Instructor at UDMC instead, as shown in the
Tentative Faculty Loading dated June 24, 2011. In view of the foregoing, the Court is inclined to hold that
respondents never dismissed Mallo from his job. While the Court concurs with the CA that Mallo was not illegally
dismissed, the Court does not agree that he had abandoned his work. The concept of abandonment in labor law had
been thoroughly discussed in Tan Brothers Corporation of Basilan City v. Escudero: As defined under established
jurisprudence, abandonment is the deliberate and unjustified refusal of an employee to resume his employment. It
constitutes neglect of duty and is a just cause for termination of employment under paragraph (b) of Article 282 [now
Article 296] of the Labor Code. To constitute abandonment, however, there must be a clear and deliberate intent to
discontinue one's employment without any intention of returning. In this regard, two elements must concur: (1) failure
to report for work or absence without valid or justifiable reason; and (2) a clear intention to sever the employer-
employee relationship, with the second element as the more determinative factor and being manifested by some
overt acts. Otherwise stated, absence must be accompanied by overt acts unerringly pointing to the fact that the
employee simply does not want to work anymore. It has been ruled that the employer has the burden of proof to
show a deliberate and unjustified refusal of the employee to resume his employment without any intention of
returning. In this case, records are bereft of any indication that Mallo's absence from work was deliberate, unjustified,
and with a clear intent to sever his employment relationship with SACI. While respondents claim to have assigned
Mallo as Clinical Instructor at UDMC after failing the qualifying tests at NCMH, which assignment the latter initially
accepted, but eventually declined, there is no proof that Mallo was informed of such assignment. It bears stressing
that a party alleging a critical fact must support his allegation with substantial evidence for any decision based.

Angeles| Bajana | Balladares | Brillantes | Briones | Cabansag | Callanta | Chua | David|


De Leon | Gomez | Lopez | Macalino | Nostratis | Padilla | Reynon | Santos | Tan |Velasco
4E / 4F - 2018-2019
Page 742 of 920
LABOR REVIEW DIGEST
Atty. Joyrich Golangco

33. NAGUIT V. SAN MIGUEL CORPORATION


G.R. NO. 188839; JUNE 22, 2015

Doctrine: The settled rule is that fighting within company premises is a valid ground for the dismissal of an
employee.

FACTS: Petitioner was employed as a machine operator of SMC Metal Closure and Lithography Plant, a division of
herein respondent. Sometime in the afternoon of September 23, 2002, a petitioner and one Renato Rgeala, also an
employee of respondent got involved in an altercation in respondent’s Canlubang Plant. In his Position Paper,
petitioner claimed that Regala went to the Canlubang Plant to distribute antiunion materials that are libelous and
defamatory and that, as union steward, petitioner confronted Regala, which confrontation developed to a heated
exchange of words. Petitioner then elbowed Regala, hitting him in the face, causing hi, to lose his balance and the
fall to the ground.

As a consequence, Regala filed a complaint with respondent’s HR Department. Respondent then conducted and
administrative investigation giving both parties the opportunity to defend themselves. However, petitioner opted to
remain silent and did not address the charges against him. The company designated investigator submitted his report
and recommendation finding petitioner guilty of willful injury to another employee within company premises, which is
an infraction of the company’s rules and regulations. Respondent then served upon petitioner a letter informing him
of the termination of his employment on the basis of findings and recommendation of the investigator. Petitioner then
filed a complaint for illegal dismissal against respondent.
LA: Dismissed the case for lack of merit. NLRC: affirmed the decision of LA. CA: promulgated a resolution denying
petitioner’s motion for extension of time to file petition for certiorari. Citing amended provision of section 4 rule 65 of
Rules of Court. The CA held that the 60 day period to file certiorari is non-extendible.

ISSUE: W/N petitioner is illegally dismissed

RULING: No, there was a just cause for termination. The settled rule is that fighting within company premises is a
valid ground for the dismissal of an employee. Moreover, the act of assaulting another employee is a serious
misconduct which justifies the termination of employment. As noted by both the LA and the NLRC, substantial
evidence exists to show that petitioner committed acts which are tantamount to serious misconduct and willful
disobedience of company rules and regulations. Also, the Court agrees with respondent’s contention that if
petitioner’s long years of service would be regarded as a justification for moderating the penalty of dismissal, it will
actually become a prize for disloyalty, perverting the meaning of social justice and undermining the efforts of labor to
cleanse its rank of all undesirable. In addition, where the totality if the evidence was sufficient to warrant the dismissal
of the employee, the law warrants their dismissal without making any distinction between a first offender and habitual
delinquent.

Angeles| Bajana | Balladares | Brillantes | Briones | Cabansag | Callanta | Chua | David|


De Leon | Gomez | Lopez | Macalino | Nostratis | Padilla | Reynon | Santos | Tan |Velasco
4E / 4F - 2018-2019
Page 743 of 920
LABOR REVIEW DIGEST
Atty. Joyrich Golangco

34. MA. SOCORRO VILLAPANDO vs. COCOPLANS, INC.


G.R. No. 183129 May 30, 2016
Peralta, J.

DOCTRINE: To be a valid ground for dismissal, loss of trust and confidence must be based on a willful
breach of trust and founded on clearly established facts. A breach is willful if it is done intentionally,
knowingly and purposely, without justifiable excuse, as distinguished from an act done carelessly,
thoughtlessly, heedlessly or inadvertently. It must rest on substantial grounds and not on the employer's
arbitrariness, whims, caprices or suspicion; otherwise, the employee would eternally remain at the mercy of
the employer. Loss of confidence must not also be indiscriminately used as a shield by the employer against
a claim that the dismissal of an employee was arbitrary. And, in order to constitute a just cause for
dismissal, the act complained of must be work-related and show that the employee concerned is unfit to
continue working for the employer.
FACTS: Ma. Socorro R. Villapando, began working as a Financial Advisor for petitioner Cocoplans, Inc., (Coco
plans) in 1995. On October 11, 2000, she was eventually promoted to Division Head/Senior Sales Manager.
On November 4, 2002, however, her employment was terminated by Cocoplans, through its President, Caesar T.
Michelena, on the alleged ground that she was deliberately influencing people to transfer to another company
thereby breaching the trust and losing the confidence given to her by Cocoplans. Villapando filed an action for illegal
dismissal alleging that she was dismissed without the just cause mandated by law.
Villapando maintained that she was illegally dismissed for her employment was terminated on baseless and
untruthful grounds. According to her, Michelena simply wanted to oust her from the company because he felt that she
was sympathizing with the Vice-President for Marketing, Dario B. Martinez, an officer with whom Michelena had a
personal quarrel.That she was influencing the company's employees to transfer to another company, particularly,
Pioneer Allianz, was improbable and preposterous for she never invited nor encouraged anyone to leave the
company. In fact, up until the present time, not a single subordinate nor Villapando, herself, has transferred to said
other company. Villapando submitted a written statement9 signed by Ms. Milagros Perez, Senior Area Manager,
together with six (6) other officers of the company, wherein they attested that Villapando never influenced them to
resign or join another company. With respect to a contradictory Joint Affidavit 10 likewise executed by the same Ms.
Perez, together with Senior Area Manager David M. Sandoval, wherein they stated that Villapando, indeed,
motivated them to transfer to another company,
Villapando alleged that the written statement earlier signed by Ms. Perez belies the Joint Affidavit she subsequently
executed. Thus, the contents of the written statement should be controlling. In view of the baseless allegations the
company dismissed her on, Villapando prayed that her termination from employment be declared illegal and that she
be awarded full backwages, separation pay, and moral damages.
According to Cocoplans, Complainant has instigated the Sales Force of COCOPLANS in her area of responsibility, to
either slow down sales production or completely stop selling, then join a mass resignation and transfer to a
competitor company which was allegedly much better than COCO PLANS. Because of the persistent flow of
information that the Sales Force will proceed with their planned mass resignations as agitated by the Complainant,
the President of COCOPLANS confronted her on September 20, 2002 and when asked -
"Did you at any time during this year tell your people of leaving COCOPLANS for another company?"
The Complainant replied "Yes Sir!" thereby directly admitting the truth of the information received by the President
himself.
Cocoplans insist that Villapando's suspension and eventual termination was for just cause due to the fact that she
wilfully breached petitioners' trust in her when she deliberately encouraged her very own sales staff to move to
another company.
LA RULING: The Labor Arbiter ruled in favor of Villapando finding that she was illegally terminated from her
employment.1âwphi1 According to the Labor Arbiter, evidence clearly shows that the initial investigation conducted
by the Committee on Employee Discipline was merely to determine the truth about the allegations of Villapando in
her resignation letter that she was being forced to resign. But in Michelena's desire to terminate Villapando's
employment, he instructed the committee to expand the scope of investigation to her alleged acts of motivating her
subordinates to transfer to another company.
NLRC RULING: , The NLRC disagreed with the Labor Arbiter in its Decision holding that the matter of resignation is
a non-issue as the termination of Villapando's employment was affected for reasons other than her resignation.
According to the NLRC, the two essential elements of a lawful termination of employment, namely: (1) that the
employee be afforded due process, i.e., he must be given an opportunity to be heard and to defend himself; and (2)
that the dismissal must be for valid cause, are present in this case.
CA RULING: The CA disagreed with the NLRC and reinstated the Labor Arbiter's Decision, finding that while
Villapando was duly afforded the required due process mandated by law, the evidence adduced by herein petitioners
was not substantial enough to support their allegation that Villapando deliberately influenced people to transfer to

Angeles| Bajana | Balladares | Brillantes | Briones | Cabansag | Callanta | Chua | David|


De Leon | Gomez | Lopez | Macalino | Nostratis | Padilla | Reynon | Santos | Tan |Velasco
4E / 4F - 2018-2019
Page 744 of 920
LABOR REVIEW DIGEST
Atty. Joyrich Golangco

another company. The appellate court held that petitioners failed to discharge the burden of proving its just and valid
cause for dismissing Villapando. Thus, her dismissal was unjustified.
ISSUE: Whether or not Villapando was terminated for a valid and just cause.
HELD: NO.
Article 282 (now Article 297) ( c) of the Labor Code provides that an employer may terminate an employment for
fraud or willful breach by the employee of the trust reposed in him by his employer or duly authorized representative.
As firmly entrenched in our jurisprudence, loss of trust and confidence, as a just cause for termination of
employment, is premised on the fact that an employee concerned holds a position where greater trust is placed by
management and from whom greater fidelity to duty is correspondingly expected. The betrayal of this trust is the
essence of the offense for which an employee is penalize.29
To be a valid ground for dismissal, loss of trust and confidence must be based on a willful breach of trust and
founded on clearly established facts. A breach is willful if it is done intentionally, knowingly and purposely, without
justifiable excuse, as distinguished from an act done carelessly, thoughtlessly, heedlessly or inadvertently. It must
rest on substantial grounds and not on the employer's arbitrariness, whims, caprices or suspicion; otherwise, the
employee would eternally remain at the mercy of the employer. Loss of confidence must not also be indiscriminately
used as a shield by the employer against a claim that the dismissal of an employee was arbitrary. And, in order to
constitute a just cause for dismissal, the act complained of must be work-related and show that the employee
concerned is unfit to continue working for the employer.
In the instant case, the Court does not find the evidence presented by petitioners (Cocoplans) to be substantial
enough to discharge the burden of proving that Villapando was, indeed, dismissed for just cause.
In dismissing an employee for just cause, it must be shown that the employer fairly made a determination of just
cause in good faith, taking into consideration all of the evidence available to him. But as the appellate court noted,
the affidavit of Ms. Gurango was never presented before the investigation panel, merely surfacing only during the
proceedings before the Labor Arbiter, in spite of the fact that the same was supposedly executed as early as
September 9, 2002, an entire month before the time the Committee on Employee Discipline convened. Thus, not only
is there no showing that said affidavit was considered by petitioners in arriving at their decsiion to dismiss Villapando,
Villapando never had the opportunity to address the accusations stated therein. As such, the Court cannot consider
the same.

Angeles| Bajana | Balladares | Brillantes | Briones | Cabansag | Callanta | Chua | David|


De Leon | Gomez | Lopez | Macalino | Nostratis | Padilla | Reynon | Santos | Tan |Velasco
4E / 4F - 2018-2019
Page 745 of 920
LABOR REVIEW DIGEST
Atty. Joyrich Golangco

35. UNIVERSAL ROBINA SUGAR MILLING CORP. vs. ALBAY


G.R. No. 218172 March 16, 2016
Perlas-Bernabe, J.

DOCTRINE: Misconduct is defined as an improper or wrong conduct. It is a transgression of some


established and definite rule of action, a forbidden act, a dereliction of duty, willful in character, and implies
wrongful intent and not mere error in judgment. To constitute a valid cause for the dismissal within the text
and meaning of Article 282 of the Labor Code, the employee's misconduct must be serious, i.e., of such
grave and aggravated character, and not merely trivial or unimportant. Additionally, the misconduct must be
related to the performance of the employee's duties showing him to be unfit to continue working for the
employer. Further, and equally important and required, the act or conduct must have been performed with
wrongful intent. In other words, for serious misconduct to be a just cause for dismissal, the concurrence of
the following elements is required: (a) the misconduct must be serious; (b) it must relate to the performance
of the employee's duties showing that the employee has become unfit to continue working for the employer;
and (c) it must have been performed with wrongful intent.

FACTS: Respondents, members of the Nagkahiusang Mamumuo sa Ursumco-National Federation of Labor (the
Union), filed a complaint for illegal dismissal, unfair labor practice, and recovery of damages against petitioner before
the Sub-Regional Arbitration Branch No. VII, Dumaguete City of the NLRC.
Respondents alleged that sometime in 1997, the Union filed a complaint against petitioner for non-compliance with
Wage Order No. 3 issued by the Regional Tripartite Wages and Productivity Board before the Department of Labor
and Employment (DOLE). After due proceedings, the DOLE found petitioner liable to the members of the Union in the
total amount of P210,217.54 and, consequently, issued a Writ of Execution to enforce the said ruling. On September
11, 2003, DOLE Sheriff Ignacio Calinawan (Sheriff Calinawan) went to petitioner's premises to serve the writ to
petitioner's Personnel Manager, Jocelyn Teo (Teo), but the latter refused to comply by reason of petitioner's pending
appeal before the Secretary of Labor. Two (2) months later, or on November 12, 2003, Sheriff Calinawan went back
to petitioner's premises in another attempt to serve the writ of execution, this time, seeking the help of the Union
Officers, including respondents, in its enforcement. Despite Teo's refusal to receive the writ, Sheriff Calinawan and
respondents still effected a levy on one of petitioner's forklifts, took it outside the company premises, and deposited it
at the municipal hall for safekeeping.

Due to the foregoing incidents, petitioner issued a Notice of Offense dated November 18, 2003 to each of the
respondents, requiring them to explain in writing why no disciplinary action should be taken against them. Thereafter,
or on November 24, 2003, petitioner issued a Notice of Administrative Investigation13 to each of the respondents,
charging them of stealing company property, fraudulent acquisition or release to other persons of company property,
unauthorized possession/use of company property, unauthorized operation of company equipment, and serious
misconduct during official working hours or within company premises. On December 1, 2003, after due investigation,
petitioner furnished respondents with a Notice of Dismissal for being found guilty as charged. This prompted the filing
of the instant complaint.

LA RULING: The LA dismissed respondents' complaint for illegal dismissal for lack of merit. The LA found that
respondents' participation in the execution of the writ by Sheriff Calinawan, while legal, was tainted with arrogance
and lawlessness, considering that the same was effected with the use of force and intimidation. The LA highlighted
the fact that their act of assisting Sheriff Calinawan in an intimidating mob-like manner to divest the company of its
property was inimical to the interest of petitioner company.

NLRC RULING: The NLRC agreed with the LA that the manner in which respondents assisted in the execution of the
writ was arrogant and unlawful and, thus, deemed the legality of their termination as valid.

CA RULING: The CA reversed and set aside the NLRC ruling by declaring respondents to have been illegally
dismissed by petitioner. While the CA agrees with the finding that respondents violated company rules in the manner
by which they assisted Sheriff Calinawan in enforcing the writ of execution, it ruled that dismissal is too severe a
penalty for the infraction.

ISSUE: Whether or not the Respondents were illegally dismissed.

HELD: YES.
Misconduct is defined as an improper or wrong conduct. It is a transgression of some established and definite rule of
action, a forbidden act, a dereliction of duty, willful in character, and implies wrongful intent and not mere error in
judgment. To constitute a valid cause for the dismissal within the text and meaning of Article 282 of the Labor Code,

Angeles| Bajana | Balladares | Brillantes | Briones | Cabansag | Callanta | Chua | David|


De Leon | Gomez | Lopez | Macalino | Nostratis | Padilla | Reynon | Santos | Tan |Velasco
4E / 4F - 2018-2019
Page 746 of 920
LABOR REVIEW DIGEST
Atty. Joyrich Golangco

the employee's misconduct must be serious, i.e., of such grave and aggravated character, and not merely trivial or
unimportant. Additionally, the misconduct must be related to the performance of the employee's duties showing him
to be unfit to continue working for the employer. Further, and equally important and required, the act or conduct must
have been performed with wrongful intent. In other words, for serious misconduct to be a just cause for dismissal, the
concurrence of the following elements is required: (a) the misconduct must be serious; (b) it must relate to the
performance of the employee's duties showing that the employee has become unfit to continue working for the
employer; and (c) it must have been performed with wrongful intent.
Clearly, respondents committed some form of misconduct when they assisted Sheriff Calinawan in effecting the levy
on the forklift and depositing the same to the municipal hall for safekeeping as they operated the forklift and took it
out of company premises, all without the authority and consent from petitioner or any of its officers. However, as
correctly pointed out by the CA, respondents did not perform the said acts with intent to gain or with wrongful intent.
Rather, they were impelled by their belief - albeit misplaced - that they were merely facilitating the enforcement of a
favorable decision in a labor standards case in order to finally collect what is due them as a matter of right, which is
the balance of their unpaid benefits. In light of the foregoing, the Court upholds the right of petitioner to take the
appropriate disciplinary action against respondents, but nevertheless, holds that respondents should not have been
dismissed from service as a less punitive sanction, i.e., suspension, would have sufficed.
Further, considering the fact that respondents were mere equipment operators, technicians, and electricians, and
thus, not occupying managerial nor confidential positions, and that the incident concerning the forklift was only their
first offense in their 14-15 years of service, the Court agrees with the CA that they should have only been meted a
penalty that is less severe than dismissal, i.e., suspension. Hence, respondents could not be validly dismissed by
petitioner.

Angeles| Bajana | Balladares | Brillantes | Briones | Cabansag | Callanta | Chua | David|


De Leon | Gomez | Lopez | Macalino | Nostratis | Padilla | Reynon | Santos | Tan |Velasco
4E / 4F - 2018-2019
Page 747 of 920
LABOR REVIEW DIGEST
Atty. Joyrich Golangco

36. CEBU PEOPLE’S MULTI PURPOSE COOPERATIVE vs. NICERATO CARBONILLA, JR.
G.R. No. 212070 January 27, 2016
Perlas-Bernabe, J.

DOCTRINE: For misconduct to be considered as a just cause for termination, the following requisites must
concur: (a) the misconduct must be serious; (b) it must relate to the performance of the employee's duties
showing that the employee has become unfit to continue working for the employer; and (c) it must have been
performed with wrongful intent. |||
According to jurisprudence, loss of trust and confidence will validate an employee's dismissal when it is
shown that: (a) the employee concerned holds a position of trust and confidence; and (b) he performs an act
that would justify such loss of trust and confidence.|||
FACTS: On November 14, 2005, CPMPC hired Carbonilla, Jr. as a Credit and Collection Manager and, as such,
was tasked with the handling of the credit and collection activities of the cooperative, which included
recommending loan approvals, formulating and implementing credit and collection policies, and conducting
trainings.||| CPMPC underwent a reorganization whereby Carbonilla, Jr. was also assigned to perform the duties of
Human Resources Department (HRD) Manager, i.e., assisting in the personnel hiring, firing, and handling of labor
disputes. In 2008, he was appointed as Legal Officer and subsequently, held the position of Legal and Collection
Manager.
However, beginning February 2008, CPMPC, through its HRD Manager, Ma. Theresa R. Marquez (HRD Manager
Marquez), sent various memoranda to Carbonilla, Jr. seeking explanation on the various infractions he allegedly
committed.
Unconvinced by Carbonilla, Jr.'s explanations, CPMPC scheduled several clarificatory hearings, but the former
failed to attend despite due notice. Later, CPMPC conducted a formal investigation where it ultimately
found Carbonilla, Jr. to have committed acts prejudicial to CPMPC's interests. As such, CPMPC, CEO Quevedo,
sent Carbonilla, Jr. a Notice of Dismissal dated August 5, 2008 informing the latter of his termination on the
grounds of: (a) loss of trust and confidence;(b) gross disrespect; (c) serious misconduct; (d) gross
negligence; (e) commission of a crime of falsification/inducing Aguipo to violate the law or the Land Transportation
and Traffic Code; and (e) committing acts highly prejudicial to the interest of the cooperative.
Consequently, Carbonilla, Jr. filed the instant case for illegal dismissal, non-payment of salaries, 13th month pay,
as well as damages and backwages, against CPMPC, before the NLRC.
PETITIONER’S CONTENTION: CPMPC maintained that the totality of Carbonilla, Jr.'s infractions was sufficient to
warrant his dismissal, and that it had complied with the procedural due process in terminating him.|||
RESPONDENT’S CONTENTION: Carbonilla, Jr. denied the administrative charges against him, asserting that the
Management and Board of Directors of CPMPC merely orchestrated means to unjustly dismiss him from
employment.||

LA RULING: The Labor Arbiter (LA) dismissed Carbonilla, Jr.'s complaint for lack of merit. The LA found
that Carbonilla, Jr. committed a litany of infractions, the totality of which constituted just cause for the termination of
his employment||.

NLRC RULING: The NLRC affirmed the LA ruling. It found CPMPC to have substantially proven the existence of just
causes in dismissing Carbonilla, Jr., i.e., abuse of authority; disrespect to his colleagues and superiors; being remiss
in his duties; and commission of acts of misrepresentation. 62 It further held that Carbonilla, Jr. was given the
opportunity to present his side and to disprove the charges against him, but failed to do so.|||

CA RULING: The CA reversed and set aside the NLRC ruling and accordingly, ordered Carbonilla, Jr.'s
reinstatement and the remand of the case to the LA for the computation of his full backwages, inclusive of allowances
and other benefits, as well as attorney's fees. It held that the NLRC gravely abused its discretion in
declaring Carbonilla, Jr.'s dismissal as valid, considering that, other than CPMPC's series of memoranda and self-
serving allegations, it did not present substantial documents to support a conclusion that would warrant Carbonilla,
Jr.'s valid dismissal.|||

ISSUE: Whether or not Carbonilla, Jr. was legally dismissed.

HELD: YES.
Case law characterizes misconduct as a transgression of some established and definite rule of action, a forbidden
act, a dereliction of duty, willful in character and implies wrongful intent and not mere error in judgment. For
misconduct to be considered as a just cause for termination, the following requisites must concur: (a) the misconduct
must be serious; (b) it must relate to the performance of the employee's duties showing that the employee has
become unfit to continue working for the employer; and (c) it must have been performed with wrongful intent. |||

Angeles| Bajana | Balladares | Brillantes | Briones | Cabansag | Callanta | Chua | David|


De Leon | Gomez | Lopez | Macalino | Nostratis | Padilla | Reynon | Santos | Tan |Velasco
4E / 4F - 2018-2019
Page 748 of 920
LABOR REVIEW DIGEST
Atty. Joyrich Golangco

All of the foregoing requisites have been duly established in this case.|||
Indisputably, Carbonilla, Jr.'s demeanor towards his colleagues and superiors is serious in nature as it is not only
reflective of defiance but also breeds of antagonism in the work environment. Surely, within the bounds of law,
management has the rightful prerogative to take away dissidents and undesirables from the workplace. It should not
be forced to deal with difficult personnel, especially one who occupies a position of trust and confidence, as will be
later discussed, else it be compelled to act against the best interest of its business. Carbonilla, Jr.'s conduct is also
clearly work-related as all were incidents which sprung from the performance of his duties. Lastly, the misconduct
was performed with wrongful intent as no justifiable reason was presented to excuse the same. On the
contrary, Carbonilla, Jr. comes off as a smart aleck who would even go to the extent of dangling whatever knowledge
he had of the law against his employer in a combative manner. As succinctly put by CPMPC, "[e]very time
[Carbonilla, Jr.'s] attention was called for some inappropriate actions, he would always show his Book, Philippine Law
Dictionary and would ask the CEO or HRD Manager under what provision of the law he would be liable for the
complained action or omission." Irrefragably, CPMPC is justified in no longer tolerating the grossly discourteous
attitude of Carbonilla, Jr. as it constitutes conduct unbecoming of his managerial position and a serious breach of
order and discipline in the workplace||.
For another, Carbonilla, Jr.'s dismissal was also justified on the ground of loss of trust and confidence. According to
jurisprudence, loss of trust and confidence will validate an employee's dismissal when it is shown that: (a) the
employee concerned holds a position of trust and confidence; and (b) he performs an act that would justify such loss
of trust and confidence.|||
Records reveal that Carbonilla, Jr. occupied a position of trust and confidence as he was employed as Credit and
Collection Manager, and later on, as Legal and Collection Manager, tasked with the duties of, among others,
handling the credit and collection activities of the cooperative, which included recommending loan approvals,
formulating and implementing credit and collection policies, and conducting trainings. With such responsibilities, it is
fairly evident that Carbonilla, Jr. is a managerial employee within the ambit of the first classification of employees
afore-discussed. |||

Angeles| Bajana | Balladares | Brillantes | Briones | Cabansag | Callanta | Chua | David|


De Leon | Gomez | Lopez | Macalino | Nostratis | Padilla | Reynon | Santos | Tan |Velasco
4E / 4F - 2018-2019
Page 749 of 920
LABOR REVIEW DIGEST
Atty. Joyrich Golangco

37. VISAYAN ELECTRIC CO V VECO


G.R. NO. 205575, JULY 22, 2015

DOCTRINE: Settled is the rule that an employer cannot be compelled to retain an employee who is guilty of
acts inimical to the interests of the employer. A company has the right to dismiss its employee if only as a
measure of self-protection

Facts:
May 1, 2009, union members marched on the streets of Cebu City to protest VECO's refusal to comply with
the political and economic provisions of the CBA. Mahilum and other union officers were interviewed by the media,
and they handed out a document containing their grievances against VECO, the gist of which came out in local
newspapers. Following said incident, Mahilum was allegedly demoted as warehouse staff to isolate him and restrict
his movements. Other union officers were transferred to positions that will keep them away from the general union
membership.
Union filed another Notice of Strike with the NCMB against VECO on the grounds of unfair labor practice,
specifically union busting for the dismissal and/or suspension of its union president and officers, refusal to bargain
collectively, as well as non-observance of the grievance procedure in their CBA.
NLRC dismissed the charge of unfair labor practice against VECO for lack of merit, and declaring Mahilum's
dismissal from employment as legal.
NLRC ruled that Mahilum was terminated for a just and valid cause under Article 282 (c) of the Labor Code,
i.e., fraud or willful breach of trust by the employee of the trust reposed in him by his employer or duly authorized
representative, when he, together with some other union officers, caused the publication of a document which was
deemed to have dishonored and blackened the memory of former corporate officer Luis Alfonso Y. Aboitiz,
besmirched VECO's name and reputation, and exposed the latter to public hatred, contempt, and ridicule. MR
denied.
CA denied certiorari petition.

ISSUE:
Whether the petitioner is unjustly dismissed.

HELD:
The petition is not impressed with merit. The Court is not convinced.
Delving now into the merits of Mahilum's dismissal, the Court holds that the two requisites for a valid
dismissal from employment have been met, namely: (1) it must be for a just or authorized cause; and (2) the
employee must be afforded due process.
VECO anchored its termination of Mahilum on Article 282 (c) of the Labor Code and Articles 5.1 and 4.471
of VECO's Company Code of Discipline, which read as follows:
Article 282 (c) of tile Labor Code:
Art. 282. Termination By Employer. - An employer may terminate an employment for any of the following
causes:
xxxx
(c) fraud or willful breach of trust by the employee of the trust reposed in him by his employer or duly
authorized representative.
The Court has consistently held that "x x x loss of trust and confidence must be based on willful breach of
the trust reposed in the employee by his employer. Such breach is willful if it is done intentionally, knowingly, and
purposely, without justifiable excuse, as distinguished from an act done carelessly, thoughtlessly, heedlessly or
inadvertently. Moreover, it must be based on substantial evidence and not on the employer's whims or caprices or
suspicions, otherwise, the employee would eternally remain at the mercy of the employer. x x x. And, in order to
constitute a just cause for dismissal, the act complained of must be work-related and show that the employee
concerned is unfit to continue working for the employer. In addition, loss of confidence x x x is premised on the fact
that the employee concerned holds a position of responsibility, trust, and confidence or that the employee concerned
is entrusted with confidence with respect to delicate matters, such as handling or care and protection of the property
and assets of the employer. The betrayal of this trust is the essence of the offense for which an employee is
penalized."
Mahilum's attempt to rationalize his act as part of his "moral, legal or social duty x x x to make known his
legitimate perception" against VECO does not, in any way, detract from the indubitable fact that he intentionally,
knowingly, and purposely caused the aforequoted "disparaging publication." Neither can he hide behind the claim
that the press release was simply "an expression of a valid grievance." As the NLRC aptly pointed out, "(i)nstead of
him and the rest of the union officers bringing their sentiments and/or grievances against the management to the
proper forum, they intentionally, knowingly and purposefully breached their employer's trust, by issuing x x x

Angeles| Bajana | Balladares | Brillantes | Briones | Cabansag | Callanta | Chua | David|


De Leon | Gomez | Lopez | Macalino | Nostratis | Padilla | Reynon | Santos | Tan |Velasco
4E / 4F - 2018-2019
Page 750 of 920
LABOR REVIEW DIGEST
Atty. Joyrich Golangco

derogatory statements and causing their publication, apparently, to incite public condemnation against the latter." It
bears noting that, while petitioners harp on the refusal of VECO to follow the grievance machinery procedure under
the CBA, they conveniently forgot that they themselves shunned the very procedure to which they now hang by a
thread.
However, with the derogatory statements issued by Mahilum that were intended to incite, not just public
condemnation of VECO, but antagonism and obstruction against rate increases in electricity that it may be allowed,
by law, to fix, there can be no dispute that VECO, indeed, had lost its trust and confidence in Mahilum and his ability
to perform his tasks with utmost efficiency and loyalty expected of an employee entrusted to handle customers and
funds. Settled is the rule that an employer cannot be compelled to retain an employee who is guilty of acts
inimical to the interests of the employer. A company has the right to dismiss its employee if only as a
measure of self-protection.
Thus, Mahilum was terminated for a just and valid cause. Moreover, as declared by the NLRC, VECO
complied with the procedural due process requirements of furnishing Mahilum with two written notices before the
termination of employment can be effected. On May 8, 2009,83 Mahilum was apprised of the particular acts for which
his termination was sought; and, after due investigation, he was given a Notice of Decision84 on October 28, 2010
informing him of his dismissal from service.
The fact that Mahilum served the company for a considerable period of time will not help his cause. It is well
to emphasize that the longer an employee stays in the service of the company, the greater is his responsibility for
knowledge and compliance with the norms of conduct and the code of discipline in the company.

Angeles| Bajana | Balladares | Brillantes | Briones | Cabansag | Callanta | Chua | David|


De Leon | Gomez | Lopez | Macalino | Nostratis | Padilla | Reynon | Santos | Tan |Velasco
4E / 4F - 2018-2019
Page 751 of 920
LABOR REVIEW DIGEST
Atty. Joyrich Golangco

38. MANARPIIS V TEXAN PHILIPPINES


G.R. NO. 197011 JANUARY 28, 2015

DOCTRINE: Here, loss of confidence was belatedly raised by the respondents who initiated an investigation
on the alleged irregularities committed by petitioner only after the latter had questioned the legality of her
earlier dismissal due to the purported company closure.

Facts:
Claiming insurmountable losses, respondents served a written notice (July 27, 2000) addressed to all their
employees that TPI will cease operations by August 31, 2000.
On August 7, 2000, petitioner filed a complaint for illegal dismissal, non-payment of overtime pay, holiday
pay, service incentive leave pay, unexpired vacation leave and 13th month pay and with prayer for moral and actual
damages. Subsequently, petitioner amended her complaint to state the true date of her dismissal which is July 27,
2000 and not August 31, 2000. She averred that on the same day she was served with notice of company closure,
respondents barred her from reporting for work and paid her last salary up to the end of July 2000.
LA Melquiades Sol D. Del Rosario rendered a Decision declaring the dismissal of petitioner as illegal.
Respondents appealed to the NLRC which affirmed the LA’s decision. Their motion for reconsideration was
also denied.
In a petition for certiorari filed with the CA, respondents argued that the subsequent termination of petitioner
on the grounds of dishonesty, loss of confidence and abandonment, after TPI was able to regain financial viability,
was made in view of the fact that commission of the said offenses surfaced only during the audit investigation
conducted after notice of cessation of business operation was sent to the employees.
CA reversed the NLRC and ruled that petitioner was validly dismissed.

ISSUE:

HELD:
We reverse the CA and reinstate the LA’s decision as affirmed by the NLRC.
We have laid down the two elements which must concur for a valid abandonment, viz: (1) the failure to
report to work or absence without valid or justifiable reason, and (2) a clear intention to sever the employer employee
relationship, with the second element as the more determinative factor being manifested by some overt acts.
Abandonment as a just ground for dismissal requires the deliberate, unjustified refusal of the employee to
perform his employment responsibilities. Mere absence or failure to work, even after notice to return, is not
tantamount to abandonment.
Furthermore, it is well-settled that the filing by an employee of a complaint for illegal dismissal with a prayer
for reinstatement is proof enough of his desire to return to work, thus, negating the employer’s charge of
abandonment. An employee who takes steps to protest his dismissal cannot logically be said to have abandoned his
work.
Abandonment in this case was a trumped up charge, apparently to make it appear that petitioner was not
yet terminated when she filed the illegal dismissal complaint and to give a semblance of truth to the belated
investigation against the petitioner. Petitioner did not abandon her work but was told not to report for work anymore
after being served a written notice of termination of company closure on July 27, 2000 and turning over company
properties to respondent Rialubin-Tan.
On the issue of loss of confidence, we have held that proof beyond reasonable doubt is not needed to justify
the loss as long as the employer has reasonable ground to believe that the employee is responsible for the
misconduct and his participation therein renders him unworthy of the trust and confidence demanded of his
position.31 Nonetheless, the right of an employer to dismiss employees on the ground of loss of trust and confidence,
however, must not be exercised arbitrarily and without just cause. Unsupported by sufficient proof, loss of confidence
is without basis and may not be successfully invoked as a ground for dismissal. Loss of confidence as a ground for
dismissal has never been intended to afford an occasion for abuse by the employer of its prerogative, as it can easily
be subject to abuse because of its subjective nature, as in the case at bar, and the loss must be founded on clearly
established facts sufficient to warrant the employee’s separation from work.
Here, loss of confidence was belatedly raised by the respondents who initiated an investigation on the
alleged irregularities committed by petitioner only after the latter had questioned the legality of her earlier
dismissal due to the purported company closure. As correctly observed by the NLRC, assuming to be true that
respondents had not yet actually dismissed the petitioner, the notice of cessation of operations (memo dated July 27,
2000) addressed to all employees never mentioned the supposed charges against the petitioner who was also never
issued a separate memorandum to that effect. Moreover, the turn over of company properties by petitioner on the
same date as demanded by respondent Rialubin-Tan belies the latter’s claim that she verbally instructed the former
to continue reporting for work in view of the audit of the company’s finances. Indeed, considering the gravity of the

Angeles| Bajana | Balladares | Brillantes | Briones | Cabansag | Callanta | Chua | David|


De Leon | Gomez | Lopez | Macalino | Nostratis | Padilla | Reynon | Santos | Tan |Velasco
4E / 4F - 2018-2019
Page 752 of 920
LABOR REVIEW DIGEST
Atty. Joyrich Golangco

accusations of fraud against the petitioner, it is strange that respondents have not at least issued her a separate
memorandum on her accountability for the alleged business losses.
To prove the dishonesty imputed to petitioner, respondents submitted before the NLRC a letter dated
August 4, 2000 from one of TPI’s suppliers advising the company of a supposed double payment made in February
and March 2000. However, there is no showing that such payment was made or ordered by petitioner, and neither
was it shown that this overpayment was reflected in the account books of TPI. Respondents likewise failed to prove
their accusation that petitioner put up a competing business while she was still employed with TPI, and their bare
allegation that petitioner divulged confidential company matters to customers. As to the supposed failure of petitioner
to account for funds intended for "under the table" transactions at the Bureau of Customs, the same was never raised
before the labor tribunals and not a shred of evidence was presented by respondent to prove this allegation.
Apropos we recall our pronouncement in Lima Land, Inc., et al. v. Cuevas:
As a final note, the Court is wont to reiterate that while an employer has its own interest to protect, and
pursuant thereto, it may terminate a managerial employee for a just cause, such prerogative to dismiss or lay
off an employee must be exercised without abuse of discretion. Its implementation should be tempered with
compassion and understanding. The employer should bear in mind that, in the execution of the said prerogative,
what is at stake is not only the employee’s position, but his very livelihood, his very breadbasket. Indeed, the
consistent rule is that if doubts exist between the evidence presented by the employer and the employee, the
scales of justice must be tilted in favor of the latter. The employer must affirmatively show rationally adequate
evidence that the dismissal was for justifiable cause. Thus, when the breach of trust or loss of confidence alleged is
not borne by clearly established facts, as in this case, such dismissal on the cited grounds cannot be allowed.
The normal consequences of petitioner’s illegal dismissal are reinstatement without loss of seniority rights,
and payment of back wages computed from the time compensation was withheld up to the date of actual
reinstatement. Where reinstatement is no longer viable as an option, separation pay equivalent to one month salary
for every year of service should be awarded as an alternative. The payment of separation pay is in addition to
payment of back wages. Given the strained relations between the parties, the award of separation pay, in lieu of
reinstatement, is in order.

Angeles| Bajana | Balladares | Brillantes | Briones | Cabansag | Callanta | Chua | David|


De Leon | Gomez | Lopez | Macalino | Nostratis | Padilla | Reynon | Santos | Tan |Velasco
4E / 4F - 2018-2019
Page 753 of 920
LABOR REVIEW DIGEST
Atty. Joyrich Golangco

39. LEUS V ST SCHOLASTICA COLLEGE


G.R. NO. 187226 JANUARY 28, 2015

DOCTRINE: The petitioner’s pregnancy out of wedlock is not a disgraceful or immoral conduct since she and
the father of her child have no impediment to marry each other.

Facts:
In May 2001, SSCW hired the petitioner as an Assistant to SSCW’s Director of the Lay Apostolate and Community
Outreach Directorate.
Sometime in 2003, the petitioner and her boyfriend conceived a child out of wedlock. When SSCW learned
of the petitioner’s pregnancy, Sr. Edna Quiambao (Sr. Quiambao), SSCW’s Directress, advised her to file a
resignation letter effective June 1, 2003. In response, the petitioner informed Sr. Quiambao that she would not resign
from her employment just because she got pregnant without the benefit of marriage.
She was dismissed eventually and Thereupon, the petitioner filed a complaint for illegal dismissal with the
Regional Arbitration Branch of the NLRC in Quezon City against SSCW and Sr. Quiambao (respondents).
The LA found that there was a valid ground for the petitioner’s dismissal; that her pregnancy out of wedlock
is considered as a "disgraceful and immoral conduct."
NLRC issued a Resolution, which affirmed the LA Decision. MR denied.
CA rendered the herein assailed Decision,23 which denied the petition for certiorari filed by the petitioner.

ISSUE:
whether the petitioner’s pregnancy out of wedlock constitutes a valid ground to terminate her
employment.

HELD:
NO.
The labor tribunals’ respective conclusions that the petitioner’s pregnancy is a "disgraceful or immoral
conduct" were arrived at arbitrarily.
The CA and the labor tribunals affirmed the validity of the petitioner’s dismissal pursuant to Section 94(e) of
the 1992 MRPS, which provides that:
Sec. 94. Causes of Terminating Employment – In addition to the just causes enumerated in the Labor Code,
the employment of school personnel, including faculty, may be terminated for any of the following causes:
xxxx
e. Disgraceful or immoral conduct;
xxxx

The labor tribunals concluded that the petitioner’s pregnancy out of wedlock, per se, is "disgraceful and
immoral" considering that she is employed in a Catholic educational institution. In arriving at such conclusion, the
labor tribunals merely assessed the fact of the petitioner’s pregnancy vis-à-visthe totality of the circumstances
surrounding the same.
However, the Court finds no substantial evidence to support the aforementioned conclusion arrived at by the
labor tribunals. The fact of the petitioner’s pregnancy out of wedlock, without more, is not enough to
characterize the petitioner’s conduct as disgraceful or immoral. There must be substantial evidence to establish
that pre-marital sexual relations and, consequently, pregnancy out of wedlock, are indeed considered disgraceful or
immoral.
Otherwise stated, it is not the totality of the circumstances surrounding the conduct per se that determines
whether the same is disgraceful or immoral, but the conduct that is generally accepted by society as respectable
or moral. If the conduct does not conform to what society generally views as respectable or moral, then the conduct
is considered as disgraceful or immoral. Tersely put, substantial evidence must be presented, which would establish
that a particular conduct, viewed in light of the prevailing norms of conduct, is considered disgraceful or immoral.
Thus, the determination of whether a conduct is disgraceful or immoral involves a two-step process:
first, a consideration of the totality of the circumstances surrounding the conduct; and second, an assessment of the
said circumstances vis-à-visthe prevailing norms of conduct, i.e., what the society generally considers moral and
respectable.
Under these tests, two things may be concluded from the fact that an unmarried woman gives birth
out of wedlock:

(1) if the father of the child is himself unmarried, the woman is not ordinarily administratively liable for
disgraceful and immoral conduct. It may be a not-so-ideal situation and may cause complications for both mother and
child but it does not give cause for administrative sanction. There is no law which penalizes an unmarried mother

Angeles| Bajana | Balladares | Brillantes | Briones | Cabansag | Callanta | Chua | David|


De Leon | Gomez | Lopez | Macalino | Nostratis | Padilla | Reynon | Santos | Tan |Velasco
4E / 4F - 2018-2019
Page 754 of 920
LABOR REVIEW DIGEST
Atty. Joyrich Golangco

under those circumstances by reason of her sexual conduct or proscribes the consensual sexual activity between two
unmarried persons. Neither does the situation contravene any fundamental state policy as expressed in the
Constitution, a document that accommodates various belief systems irrespective of dogmatic origins.
(2) if the father of the child born out of wedlock is himself married to a woman other thanthe mother, then
there is a cause for administrative sanction against either the father or the mother. In sucha case, the "disgraceful
and immoral conduct" consists of having extramarital relations with a married person. The sanctity of marriage is
constitutionally recognized and likewise affirmed by our statutes as a special contract of permanent union.
Accordingly, judicial employees have been sanctioned for their dalliances with married persons or for their own
betrayals of the marital vow of fidelity.
In this case, it was not disputed that, like respondent, the father of her child was unmarried.
Therefore, respondent cannot be held liable for disgraceful and immoral conduct simply because she gave
birth to the child Christian Jeon out of wedlock.
The petitioner’s pregnancy out of wedlock is not a disgraceful or immoral conduct since she and the father
of her child have no impediment to marry each other.
The petitioner’s dismissal is not a valid exercise of SSCW’s management prerogative.
In sum, the Court finds that the petitioner was illegally dismissed as there was no just cause for the
termination of her employment. SSCW failed to adduce substantial evidence to establish that the petitioner’s conduct,
i.e., engaging in pre-marital sexual relations and conceiving a child out of wedlock, assessed in light of the prevailing
norms of conduct, is considered disgraceful or immoral. The labor tribunals gravely abused their discretion in
upholding the validity of the petitioner’s dismissal as the charge against the petitioner lay not on substantial evidence,
but on the bare allegations of SSCW. In turn, the CA committed reversible error in upholding the validity of the
petitioner’s dismissal, failing torecognize that the labor tribunals gravely abused their discretion in ruling for the
respondents.

The petitioner is entitled to separation pay, in lieu of actual reinstatement, full backwages and attorney’s
fees, but not to moral and exemplary damages.

Angeles| Bajana | Balladares | Brillantes | Briones | Cabansag | Callanta | Chua | David|


De Leon | Gomez | Lopez | Macalino | Nostratis | Padilla | Reynon | Santos | Tan |Velasco
4E / 4F - 2018-2019
Page 755 of 920
LABOR REVIEW DIGEST
Atty. Joyrich Golangco

40. CADIZ V. BRENT HOSPITAL AND COLLEGES


G.R. NO. 187417; FEB. 24, 2016
REYES, J.:
DOCTRINE: When Immorality is relied upon by an employer as a ground for dismissal from service, it must
have clearly defined the same in the terms and conditions of employment. The dictionary definition of
immorality, or more particularly “fornication” as any form of illicit relation does not hold water.
Jurisprudence has already set the standard of morality with which an act should be gauged — it is public
and secular, not religious.
The determination of whether a conduct is disgraceful or immoral involves a two-step process: first, a
consideration of the totality of the circumstances surrounding the conduct; and second , an assessment of
the said circumstances vis-à-vis the prevailing norms of conduct, i.e., what the society generally considers
moral and respectable.

FACTS:

Christine Joy Capin-Cadiz was the Human Resource Officer of the Brent Hospital and Colleges was at the time of her
indefinite suspension from employment in 2006. The cause of suspension was Cadiz's Unprofessionalism and
Unethical Behavior Resulting to Unwed Pregnancy. It appears that Cadiz became pregnant out of wedlock, and Brent
imposed the suspension until such time that she marries her boyfriend in accordance with law.

Cadiz filed with the LA a complaint for ULP, Constructive Dismissal, and Non-Payment of Wages and Damages.

LA RULING: the LA found that Cadiz's indefinite suspension amounted to a constructive dismissal; nevertheless, the
LA ruled that Cadiz was not illegally dismissed as there was just cause for her dismissal, that is, she engaged in
premarital sexual relations with her boyfriend resulting in a pregnancy out of wedlock. It noted not only the fact that
these relations did not only result in her pregnancy, but also the fact that Brent was an institution of the Episcopal
Church in the Philippines.

NLRC RULING: The Affirmed the decision of the LA

CA RULING: Dismissed her petition outright due to technical defects in the petition: (1) incomplete statement of
material dates; (2) failure to attach registry receipts; and (3) failure to indicate the place of issue of counsel's PTR
and IBP official receipts

APPEAL TO THE SC:

Petitioner’s Contention: Impregnation outside of wedlock should not be a ground for termination.

ISSUE:. Whether or not the respondents validly applied impregnation out of wedlock/ immorality as a ground for
termination.

RULING: NO. Setting aside the procedural matters, the court resorted to the then recently resolved case of Leus v
St. Scholastica’s College which bore similar factual antecedents- with the exception that the respondent in the said
case is a Catholic institution.

The Court ruled in Leus that the determination of whether a conduct is disgraceful or immoral involves a two-step
process: first , a consideration of the totality of the circumstances surrounding the conduct; and second , an
assessment of the said circumstances vis-à-vis the prevailing norms of conduct, i.e., what the society generally
considers moral and respectable.

In this case, the surrounding facts leading to Cadiz's dismissal are straightforward — she was employed as a human
resource officer in an educational and medical institution of the Episcopal Church of the Philippines; she and her
boyfriend at that time were both single; they engaged in premarital sexual relations, which resulted into pregnancy.
The labor tribunals characterized these as constituting disgraceful or immoral conduct. They also sweepingly
concluded that as Human Resource Officer, Cadiz should have been the epitome of proper conduct and her
indiscretion "surely scandalized the Brent community.”

The foregoing circumstances, however, do not readily equate to disgraceful and immoral conduct. Brent's Policy
Manual and Employee's Manual of Policies do not define what constitutes immorality; it simply stated immorality as a
ground for disciplinary action.

Angeles| Bajana | Balladares | Brillantes | Briones | Cabansag | Callanta | Chua | David|


De Leon | Gomez | Lopez | Macalino | Nostratis | Padilla | Reynon | Santos | Tan |Velasco
4E / 4F - 2018-2019
Page 756 of 920
LABOR REVIEW DIGEST
Atty. Joyrich Golangco

Jurisprudence has already set the standard of morality with which an act should be gauged — it is public and secular,
not religious. Whether a conduct is considered disgraceful or immoral should be made in accordance with the
prevailing norms of conduct, which, as stated in Leus, refer to those conducts which are proscribed because they are
detrimental to conditions upon which depend the existence and progress of human society. The fact that a
particular act does not conform to the traditional moral views of a certain sectarian institution is not sufficient reason
to qualify such act as immoral unless it, likewise, does not conform to public and secular standards.

Angeles| Bajana | Balladares | Brillantes | Briones | Cabansag | Callanta | Chua | David|


De Leon | Gomez | Lopez | Macalino | Nostratis | Padilla | Reynon | Santos | Tan |Velasco
4E / 4F - 2018-2019
Page 757 of 920
LABOR REVIEW DIGEST
Atty. Joyrich Golangco

41.RAYMOND SON V. UNIVERSITY OF SANTO TOMAS


G.R. NO. 211273; APR. 19, 2018
DEL CASTILLO, J.:

DOCTRINE: There can be no security of tenure to be spoken of when minimum standards of the law for such
security have not been met.
FACTS:

Petitioners Raymond A. Son (Son), Raymond S. Antiola (Antiola), and Wilfredo E. Pollarco (Pollarco) are full time
professors of the UST Colleges of Fine Arts and Design and Philosophy, and are members of the UST Faculty Union,
with which UST at the time had a Collective Bargaining Agreement (CBA).

Son and Antiola were hired in June, 2005, while Pollarco was employed earlier, or in June, 2004. Under their
respective appointment papers, petitioners were designated as "faculty member[s] on PROBATIONARY status,"
whose "accession to tenure status is conditioned by [ sic] your meeting all the requirements provided under existing
University rules and regulations and other applicable laws including, among others, possession of the [prerequisite]
graduate degree before the expiration of the probationary period and by your satisfactory performance of the duties
and responsibilities set forth in the job description hereto attached."

The UST-UST Faculty Union CBA provided that —

ARTICLE XV TENURE
Section 1. Tenured Faculty Member. — He is:

1. Teaching Faculty member, given a tenure track appointment upon hiring who has rendered six (6) consecutive
semesters of satisfactory service on a full-time basis, carrying fifteen-unit load (15) or more. Although a master's
degree is an entry requirement, a faculty member admitted to serve the University without a master's degree
shall finish his master's degree in five (5) semesters. If he does not finish his degree in five (5) semesters, he
shall be separated from service at the end of the fifth semester; however, if he is made to serve the University
further, in spite of the lack of a master's degree, he shall be deemed to have attained tenure.

The provision is in relation to the requirement Revised Manual of Regulations for Private Schools mandating that
faculty in Undergraduate courses must possess a minimum of a Master’s Degree.

Petitioners in this case did not have the said qualification, but were nonetheless engaged by UST subject to later
compliance as per the CBA.

In 2010, however, CHED Chairman Angeles issued Memorandum issued to the Presidents of various educational
instututions directing the strict implementation of the minimum qualification of a Master’s Degree as required by
Memorandum Order 40-08. UST in compliance wrote to the petitioners and other concerned faculty members
informing them of the university decision not to re-appoint those who fail to complete their masters degree, subject to
the sole exception of allowing via Written Appeal those who are due to defend or complete their Master’s Degrees.

Petitioners did not make the written appeal, laboring under the belief that their tenure was secure by reason of the
CBA due to their continued re-appointment notwithstanding the lack of Master’s degrees.. June 2010 petitioners
receiver termination/thank you letters signed by the Dean of CFAD, terminating their services by reason of their non-
compliance with the Written Appeal requirement. This prompted the petitioners to file an action for illegal dismissal,
ULP, and money claims against UST.

LA RULING: Ordered the reinstatement of Petitioners finding Illegal Dismissal and ULP on the part of UST, and
upheld the CBA on the tenurial security by default of UST.

NLRC RULING: Initially dismissed the appeal, and held that the only reason the petitioners were dismissed was the
non-compliance with the Written Appeal. The case was re-opened upon Motion for Reconsideration with the original
commissioners inhibiting themselves. The NLRC then ruled that the law prevailed over the CBA. The case was
reconsidered and re-assigned upon motion, with the NLRC ultimately ruling favor of Son, et. al. on the premise that
the 2008 directive of the CHED should not be given retroactive effect given that the CBA had been in effect since
2006.

Angeles| Bajana | Balladares | Brillantes | Briones | Cabansag | Callanta | Chua | David|


De Leon | Gomez | Lopez | Macalino | Nostratis | Padilla | Reynon | Santos | Tan |Velasco
4E / 4F - 2018-2019
Page 758 of 920
LABOR REVIEW DIGEST
Atty. Joyrich Golangco

CA RULING: Ruled in favor of UST, emphasizing that the freedom to contract was not absolute- and limited by the
law, which among others included the Minimum Requirement of Masteral Degrees.

APPEAL TO THE SC:

Petitioner’s Contention: that since they acquired tenure by application of the CBA provision, they may not be
removed except for cause; that respondents violated the twin-notice rule as petitioners were not given notice and an
opportunity to be heard prior to their separation (note: This should be the issue in 297, I think, but it was not
addressed by the Court)

Respondent’s Contention: that it was merely complying with the law.

ISSUE:. Whether or not the provisions of a CBA on Tenurial Security may override law..

RULING: NO. From a strict legal viewpoint, the parties are both in violation of the law: respondents, for maintaining
professors without the mandated masteral degrees, and for petitioners, agreeing to be employed despite knowledge
of their lack of the necessary qualifications. Petitioners cannot therefore insist to be employed by UST since they still
do not possess the required master's degrees; the fact that UST continues to hire and maintain professors without
the necessary master's degrees is not a ground for claiming illegal dismissal, or even reinstatement.

The waiver is precisely on the part of UST is contrary to law. Moreover, a waiver would prejudice the rights of the
students and the public, who have a right to expect that UST is acting within the bounds of the law, and provides
quality education by hiring only qualified teaching personnel.

It cannot be said either that in requiring petitioners to file a written appeal, respondents are guilty of bad faith and
malice for practically forcing the former to renounce their tenure. There is no tenure to speak of in the first place.

Angeles| Bajana | Balladares | Brillantes | Briones | Cabansag | Callanta | Chua | David|


De Leon | Gomez | Lopez | Macalino | Nostratis | Padilla | Reynon | Santos | Tan |Velasco
4E / 4F - 2018-2019
Page 759 of 920
LABOR REVIEW DIGEST
Atty. Joyrich Golangco

42. CASCO V. NLRC


G.R. NO. 200571; FEB. 19, 2008
BERSAMIN, J.:

DOCTRINE: Neglect of duty, as a ground for dismissal, must be both gross and habitual. 27 Gross
negligence implies a want or absence of or a failure to exercise slight care or diligence, or the entire absence
of care. It evinces a thoughtless disregard of consequences without exerting any effort to avoid them.
Habitual neglect implies repeated failure to perform one's duties for a period of time, depending upon the
circumstances.
As for loss of trust and confidence, there must some basis for such loss of confidence, such as when the
employer has reasonable ground to believe that the employee concerned is responsible for the purported
misconduct, and the nature of his participation therein renders him unworthy of the trust and confidence
demanded of his position.

FACTS:

Josephine Casco was engaged by Capitol Medical Center in 184 as a Staff Nurse in their recovery room. She worked
her way up the ranks of the hospital eventually becoming Nurse Supervisor of their Operating Room in 2002. The job
summary of a Nurse Supervisor of the Operating Room are as follows: a.) responsible for the supervision and
management of nurses and services at the Operating and Recovery Room; b.) plan all nursing and exercise
personnel management within the area, make decisions when problems arise in the unit; c.) accountable for losses,
equipment malfunction, breakage, patients and personnel.

In June 2006, Casco received from Capitol various equipment such as vaporizers, patient monitors and Pulse
Oximeters for the Operating Room. In January 2008 representative of Abbot Laboratories conducted a calibration of
the Operating Room's vaporizers. In the course of the calibration, it was discovered that several hospital equipment
in the Operating Room were missing. An investigation was launched in which Casco was allowed to explain herself,
to which she raised that never in her 23 years of service was there an incident similar to the loss; that she suggested
to management at an earlier that surveillance cameras be put in areas around the operating room, but only 2 were
placed; and that the premises under her jurisdiction had 7 doors, 3 of which could not be locked and hence was
constantly subject to ingress and egress. Notwithstanding the explanation on the ground of negligence and loss of
trust and confidence, Casco was dismissed in December of 2008, prompting her to file a suit for Illegal Dismissal in
2009.

LA RULING: the LA found in favor of Casco, and ruled that Capitol had not proven habituality in the neglect on the
part of Casco, and that an isolated case of negligence did not warrant her dismissal.

NLRC RULING: Reversed the decision of the LA, on the ground that Casco’s position as Nurse Supervisor qualified
her as a supervisor or as a manager upon whom trust and confidence is reposed- more specifically for having been
entrusted with the “custody” of the hospital equipment. MR was denied.

CA RULING: Upheld the NLRC.

APPEAL TO THE SC:

Petitioner’s Contention: That the isolated instance of negligence does not warrant dismissal; that the loss of trust
and confidence was not sufficiently proven.

ISSUES:.
1. Whether or not a single instance of negligence warrants dismissal.
2. Whether or not loss of trust and confidence was based on some sound basis.

RULING:
1. NO. Neglect of duty, as a ground for dismissal, must be both gross and habitual. 27 Gross negligence
implies a want or absence of or a failure to exercise slight care or diligence, or the entire absence of care. It
evinces a thoughtless disregard of consequences without exerting any effort to avoid them. Habitual neglect
implies repeated failure to perform one's duties for a period of time, depending upon the circumstances.

The burden of proving that the dismissal of the employees was for a valid and authorized cause rests on the
employer, who show by substantial evidence that the termination of the employment of the employee was

Angeles| Bajana | Balladares | Brillantes | Briones | Cabansag | Callanta | Chua | David|


De Leon | Gomez | Lopez | Macalino | Nostratis | Padilla | Reynon | Santos | Tan |Velasco
4E / 4F - 2018-2019
Page 760 of 920
LABOR REVIEW DIGEST
Atty. Joyrich Golangco

validly made; the failure to discharge this duty will mean that the dismissal was not justified and was,
therefore, illegal.

Capitol failed to discharge this duty, and the LA as well as the NLRC failed came to an erroneous
conclusion by merely relying on the Casco’s job description as “custodian” and being. The Court believes
that Casco should not be blamed for failing to secure the equipment fall upon her if access to the operating
room was not under her control. Instead this should be a matter addressed by that of the management, to
which security of the premises from unauthorized and undesirable personalities was of utmost importance.

2. NO. Loss of trust and confidence as a valid ground for dismissal is premised on the fact that the employee
holds a position whose functions may only be performed by someone who enjoys the trust and confidence of
the management this includes managerial personnel entrusted with confidence on delicate matters, such as
the custody, handling, or care and protection of the employer's property.
Loss of trust and confidence as a ground of dismissal has never been intended to afford an occasion for
abuse because of its subjective nature. It should not be used as a subterfuge for causes which are illegal,
improper, and unjustified. It must be genuine, not a mere afterthought intended to justify an earlier action
taken in bad faith. There must some basis for such loss of confidence, such as when the employer has
reasonable ground to believe that the employee concerned is responsible for the purported misconduct, and
the nature of his participation therein renders him unworthy of the trust and confidence demanded of his
position.

There is no reasonable basis for the loss of trust and confidence in Casco for the following reasons: Firstly,
she was not vested with the responsibility of safekeeping of the hospital equipment and machines. And,
secondly, the respondents did not adduce evidence showing that she had committed wilful and deliberate
acts that led to the loss. As such, her dismissal based on loss of trust and confidence should not be upheld.

Angeles| Bajana | Balladares | Brillantes | Briones | Cabansag | Callanta | Chua | David|


De Leon | Gomez | Lopez | Macalino | Nostratis | Padilla | Reynon | Santos | Tan |Velasco
4E / 4F - 2018-2019
Page 761 of 920
LABOR REVIEW DIGEST
Atty. Joyrich Golangco

43. STERLING PAPER PRODUCTS ENTERPRISES, INC., petitioner, vs. KMM-KATIPUNAN and RAYMOND Z.
ESPONGA, respondents.
GR No: 221493 Date: August 2, 2017
Ponente: Mendoza, J.

Doctrine: For misconduct or improper behavior to be a just cause for dismissal, the following elements must
concur: (a) the misconduct must be serious; (b) it must relate to the performance of the employee's duties
showing that the employee has become unfitt to continue working for the employer; and (c) it must have
been performed with wrongful intent.

FACTS:
On July 29, 1998, petitioner Sterling Paper Products Enterprises, Inc. (Sterling) hired respondent Raymond
Z. Esponga (Esponga), as machine operator.
An incident occurred on June 26, 2010 between their supervisor, Mercy Vinoya (Vinoya) and Esponga
wherein Vinoya prohibited Esponga from sleeping on the sheeter machine. Later on, when Vinoya was passing by,
Esponga uttered "Huwag maingay, puro bawal." When she confronted him, he retorted "Puro kayo bawal, bakit bawal
ba magpahinga?" Not contented, Esponga gave her supervisor the "dirty finger" sign and said "Wala ka pala eh, puro
ka dakdak. Baka pag ako nagsalita hindi mo kayanin.”. The incident was witnessed by Mylene Pesimo (Pesimo), who
executed a handwritten account thereon. Later that day, Esponga was found to have been not working as the
machine assigned to him was not running from 2:20 to 4:30 in the afternoon. Instead, he was seen to be having a
conversation with his co-employees, Bobby Dolor and Ruel Bertulfo. Additionally, he failed to submit his daily report
from June 21 to June 29, 2010. When asked to explain and requested to attend an administrative hearing regarding
the incident, Esponga failed to submit the same and to appear in the hearing. Petitioner rescheduled the hearing,
however, Esponga and his counsel still failed to attend.
Having found Esponga guilty of gross and serious misconduct, gross disrespect to superior and habitual
negligence, Sterling sent a termination notice, dated November 15, 2010. This prompted Esponga and KMM-
Katipunan (respondents) to file a complaint for illegal dismissal, unfair labor practice, damages, and attorney's fees
against Sterling.
LA/RTC/NLRC RULING:
LA: Ruled that Esponga was illegally dismissed. It held that Sterling failed to discharge the burden of proof for failure
to submit in evidence the company's code of conduct, which was used as basis to dismiss Esponga.

NLRC: Reversed and set aside the LA ruling. It declared that Esponga's dismissal was valid. The NLRC observed
that as a result of the June 26, 2010 incident, Esponga no longer performed his duties and simply spent the
remaining working hours talking with his co-workers. It opined that Esponga intentionally did all these infractions on
the same day to show his defiance and displeasure with Vinoya, who prohibited him from sleeping on the sheeter
machine. It concluded that these were all violations of the Company Code of Conduct and Discipline, and constituted
a valid cause for termination of employment under the Labor Code.

CA RULING:
Reinstated the LA ruling. It held that the utterances and gesture did not constitute serious misconduct but only simple
misconduct.

APPEAL TO THE SC:

Petitioner’s contention: Sterling argues that Esponga's utterance of foul and abusive language against his
supervisor, demonstrating a dirty finger, and defiance to perform his duties undeniably constitute serious misconduct.

Respondent’s Contention: Esponga replied that Sterling failed to establish the validity of his dismissal by clear and
convincing evidence. He insisted that if doubts exist between the evidence presented by the employer and the
employee, the scales of justice must be tilted in favor of the latter because the employer must affirmatively show
rationally adequate evidence that the dismissal was for a justifiable cause.

ISSUE/S:
1. Whether the cause of Esponga's dismissal amounts to Serious misconduct.

HELD:
1. YES.

Angeles| Bajana | Balladares | Brillantes | Briones | Cabansag | Callanta | Chua | David|


De Leon | Gomez | Lopez | Macalino | Nostratis | Padilla | Reynon | Santos | Tan |Velasco
4E / 4F - 2018-2019
Page 762 of 920
LABOR REVIEW DIGEST
Atty. Joyrich Golangco

Under Article 282 (now 297) (a) of the Labor Code, serious misconduct by the employee justifies
the employer in terminating his or her employment. Misconduct is defined as an improper or wrong conduct.
It is a transgression of some established and definite rule of action, a forbidden act, a dereliction of duty,
willful in character, and implies wrongful intent and not mere error in judgment. To constitute a valid cause
for the dismissal within the text and meaning of Article 282 of the Labor Code, the employee's misconduct
must be serious, i.e., of such grave and aggravated character and not merely trivial or unimportant.
Additionally, the misconduct must be related to the performance of the employee's duties showing him to be
unfit to continue working for the employer. Further, and equally important and required, the act or conduct
must have been performed with wrongful intent.
To summarize, for misconduct or improper behavior to be a just cause for dismissal, the
following elements must concur: (a) the misconduct must be serious; (b) it must relate to the
performance of the employee's duties showing that the employee has become unfit to continue
working for the employer; and (c) it must have been performed with wrongful intent.
It is well-settled in jurisprudence that accusatory and inflammatory language used by an employee
towards his employer or superior can be a ground for dismissal or termination.
Esponga's assailed conduct was related to his work. Vinoya did not prohibit him from taking a nap.
She merely reminded him that he could not do so on the sheeter machine for safety reasons. Esponga's
acts reflect an unwillingness to comply with reasonable management directives.
It must be noted that Esponga committed all the acts in front of his co-employees, which evidently
showed that he intended to disrespect and humiliate his supervisor.
Time and again, the Court has put emphasis on the right of an employer to exercise its
management prerogative in dealing with its affairs including the right to dismiss its erring employees. It is a
general principle of labor law to discourage interference with an employer's judgment in the conduct of his
business. As already noted, even as the law is solicitous of the welfare of the employees, it also recognizes
the employer's exercise of management prerogatives. As long as the company's exercise of judgment is in
good faith to advance its interest and not for the purpose of defeating or circumventing the rights of
employees under the laws or valid agreements, such exercise will be upheld.

Angeles| Bajana | Balladares | Brillantes | Briones | Cabansag | Callanta | Chua | David|


De Leon | Gomez | Lopez | Macalino | Nostratis | Padilla | Reynon | Santos | Tan |Velasco
4E / 4F - 2018-2019
Page 763 of 920
LABOR REVIEW DIGEST
Atty. Joyrich Golangco

44. RUTCHER T. DAGASDAS , petitioner, vs. GRAND PLACEMENT AND GENERAL SERVICES
CORPORATION, respondent
GR No: 205727 Date: January 18, 2017
Ponente: Del Castillo, J.

Doctrine:
To allow employers to reserve a right to terminate employees without cause is violative of this guarantee of
security of tenure under Art. 297.

A valid dismissal requires substantive and procedural due process. As regards the latter, the employer must
give the concerned employee at least two notices before his or her termination. Specifically, the employer
must inform the employee of the cause or causes for his or her termination, and thereafter, the employer's
decision to dismiss him. Aside from the notice requirement, the employee must be accorded the opportunity
to be heard.

FACTS:
Grand Placement and General Services Corp. (GPGS) is a licensed recruitment or placement agency in the
Philippines while Saudi Aramco (Aramco) is its counterpart in Saudi Arabia. On the other hand, Industrial &
Management Technology Methods Co. Ltd. (ITM) is the principal of GPGS, a company existing in Saudi Arabia. In
November 2007, GPGS, for and on behalf of ITM, employed Dagasdas as Network Technician. He was to be
deployed in Saudi Arabia under a one-year contract with a monthly salary of Saudi Riyal (SR) 5,112.00. In his Job
Offer it was indicated that he was accepted by Aramco and ITM for the position of “Supt.”
It was contended by Dagasdas that he actually applied for and was engaged as a Civil Engineer and that
the position in his contract as Network Technician was purportedly for the purpose of securing a visa for Saudi Arabia
because ITM could not support a visa application for civil engineers.
Upon his arrival in Saudi Arabia on Feb. 8, 2008 he signed a new employment contract which stipulated that
ITM contracted him as Superintendent or in any capacity within the scope of his abilities with salary SR 5,112 and
allowance of SR 2,045 per month. Under this new contract, Dagasdas was placed in a probationary 3 month period
and this new contract stipulated that it cancels all contracts prior to its date in any source.
Dagasdas was allegedly tasked with a work suited that of a Mechanical Engineer which were foreign to him,
seeing that he would not be able to perform well in his work, Dagasdas raised his concern with his superior. He was
then transferred to the Civil Engineering Department and was temporarily assigned as Civil Construction Engineer.
Dagasdas averred that on March 9, 2008, he was directed to exit the worksite but Rashid H. Siddiqui (Siddiqui), the
Site Coordinator Manager, advised him to remain in the premises, and promised to secure him the position he
applied for. However, before Dagasdas' case was investigated, Siddiqui had severed his employment with ITM.
In April 2008, Dagasdas returned to Al-Khobar and stayed at the ITM Office. Later, ITM gave him a
termination notice indicating that his last day of work was on April 30, 2008, and he was dismissed pursuant to clause
17.4.3 of his contract, which provided that ITM reserved the right to terminate any employee within the three-month
probationary period without need of any notice to the employee.
Before he was repatriated back to the Philippines, Dagasdas’ was made to sign a Statement of quitclaim
with Final Settlement indicating that ITM paid him all his salaries and benefits and that ITM was relieved from all
financial responsibilities due to Dagasdas.
On June 24, 2008, Dagasdas returned to the Philippines and thereafter he filed an illegal dismissal case
against GPGS, Aramco, and ITM.
Dagasdas claims that GPGS, Aramco, and ITM were guilty of misrepresentation which resulted in the
mismatch of work assigned to him. He also argued that although he was a project he was still entitled to security of
tenure for the duration of his contract and that GPGS, Aramco and ITM merely invented imaginary causes to
terminate him. Thus he claimed that he was dismissed without just cause and due process of law.
GPGS, Aramco, and ITM countered that Dagasdas was legally dismissed. They claim that Dagasdas was
aware that he was employed as Network Technician, that he was informed of his poor performance, and that he
conformed to his termination as evidenced his quitclaim. They also stressed that Dagasdas was a only a
probationary employee at that time.

LA/RTC/NLRC RULING:
LA: Dismissed the petition for lack of merit stating that Dagasdas accepted the stipulations of his new contract; that
the new contract, being more favorable to him (salary + allowance), was not prohibited by law; and that his employer
dismissed him as part of its management prerogative.

Angeles| Bajana | Balladares | Brillantes | Briones | Cabansag | Callanta | Chua | David|


De Leon | Gomez | Lopez | Macalino | Nostratis | Padilla | Reynon | Santos | Tan |Velasco
4E / 4F - 2018-2019
Page 764 of 920
LABOR REVIEW DIGEST
Atty. Joyrich Golangco

NLRC: Reversed the decision of the LA holding that GPGS erroneously recruited Dagasdas and that the dismissal
amounted to illegal termination because the mismatch between his qualifications and the job given him was no fault
of his.

CA RULING: Reversed the decision of the NLRC and reinstated the LA decision. The CA gave credence to
Dagasdas’ Statement of Quitclaim with Final Settlement.

APPEAL TO THE SC:


Petitioner's Contention: That he was terminated because of a “discipline mismatch”; that the new contract was not
favorable to him because it was inserted therein that he still had to undergo a 3-month probationary period; that the
new contract was not approved by the POEA; and that he signed the quitclaim only so he can afford the plane ticket
for his repatriation.

Respondent's Contention: On the other hand, GPGS maintains that Dagasdas was fully aware that he applied for
and was accepted as Network Technician. It also stresses that it was Dagasdas himself who decide to accept from
ITM a new job offer when he arrived in Saudi Arabia. It further declares that Dagasdas' quitclaim is valid as there is
no showing that he was compelled to sign it.

ISSUE/S:
1. Whether or not Dagasdas was validly dismissed

HELD:
1. Dagasdas was illegally dismissed.
Security of tenure remains even if employees, particularly the overseas Filipino workers (OFW), work in a
different jurisdiction. Since the employment contracts of OFWs are perfected in the Philippines, and following the
principle of lex loci contractus (the law of the place where the contract is made), these contracts are governed by our
laws, primarily the Labor Code of the Philippines and its implementing rules and regulations. At the same time, our
laws generally apply even to employment contracts of OFWs as our Constitution explicitly provides that the State
shall afford full protection to labor, whether local or overseas. Thus, even if a Filipino is employed abroad, he or she
is entitled to security of tenure, among other constitutional rights.
First, Dagasdas' new contract is in clear violation of his right to security of tenure. Under the Labor Code of
the Philippines the following are the just causes for dismissing an employee:
ARTICLE 297. [282] Termination by Employer. — An employer may terminate an employment for any of the
following causes: (a) Serious misconduct or willful disobedience by the employee of the lawful orders of his
employer or representative in connection with his work; (b) Gross and habitual neglect by the employee of
his duties; (c) Fraud or willful breach by the employee of the trust reposed in him by his employer or duly
authorized representative; (d) Commission of a crime or offense by the employee against the person of his
employer or any immediate member of his family or his duly authorized representative; and (e) Other
causes analogous to the foregoing.
However, per the notice of termination given to Dagasdas, ITM terminated him for violating clause 17.4.3 of
his new contract, viz.:
17.4 The Company reserves the right to terminate this agreement without serving any notice to the
Consultant in the following cases:
xxx xxx xxx
17.4.3 If the Consultant is terminated by company or its client within the probation period of 3 months.
Based on the foregoing, there is no clear justification for the dismissal of Dagasdas other than the exercise
of ITM's right to terminate him within the probationary period.
To allow employers to reserve a right to terminate employees without cause is violative of this
guarantee of security of tenure. Moreover, even assuming that Dagasdas was still a probationary employee
when he was terminated, his dismissal must still be with a valid cause. As regards a probationary employee, his
or her dismissal may be allowed only if there is just cause or such reason to conclude that the employee fails to
qualify as regular employee pursuant to reasonable standards made known to the employee at the time of
engagement.
Secondly, this new contract also breached Dagasdas' original contract as it was entered into even before
the expiration of the original contract approved by the POEA. Therefore, it cannot supersede the original contract; its
terms and conditions, including reserving in favor of the employer the right to terminate an employee without notice
during the probationary period, are void.
Third, under this new contract, Dagasdas was not afforded procedural due process when he was dismissed
from work. As cited above, a valid dismissal requires substantive and procedural due process. As regards the
latter, the employer must give the concerned employee at least two notices before his or her termination.

Angeles| Bajana | Balladares | Brillantes | Briones | Cabansag | Callanta | Chua | David|


De Leon | Gomez | Lopez | Macalino | Nostratis | Padilla | Reynon | Santos | Tan |Velasco
4E / 4F - 2018-2019
Page 765 of 920
LABOR REVIEW DIGEST
Atty. Joyrich Golangco

Specifically, the employer must inform the employee of the cause or causes for his or her termination, and
thereafter, the employer's decision to dismiss him. Aside from the notice requirement, the employee must be
accorded the opportunity to be heard.
Lastly, while it is shown that Dagasdas executed a waiver in favor of his employer, the same does not
preclude him from filing this suit. This is because waiver or quitclaim is looked upon with disfavor, and is frowned
upon for being contrary to public policy. Unless it can be established that the person executing the waiver voluntarily
did so, with full understanding of its contents, and with reasonable and credible consideration, the same is not a valid
and binding undertaking. Moreover, the burden to prove that the waiver or quitclaim was voluntarily executed is with
the employer. In this case, however, neither did GPGS nor its principal, ITM, successfully discharged its burden.
GPGS and/or ITM failed to show that Dagasdas indeed voluntarily waived his claims against the employer.

Angeles| Bajana | Balladares | Brillantes | Briones | Cabansag | Callanta | Chua | David|


De Leon | Gomez | Lopez | Macalino | Nostratis | Padilla | Reynon | Santos | Tan |Velasco
4E / 4F - 2018-2019
Page 766 of 920
LABOR REVIEW DIGEST
Atty. Joyrich Golangco

Angeles| Bajana | Balladares | Brillantes | Briones | Cabansag | Callanta | Chua | David|


De Leon | Gomez | Lopez | Macalino | Nostratis | Padilla | Reynon | Santos | Tan |Velasco
4E / 4F - 2018-2019
Page 767 of 920
LABOR REVIEW DIGEST
Atty. Joyrich Golangco

45. HOLCIM PHILIPPINES, INC., petitioner, vs. RENANTE J. OBRA, respondent.


GR No: 220998 Date: Aug. 8, 2016
Ponente: Perlas-Bernabe, J

Doctrine: To constitute a valid cause for dismissal within the text and meaning of Article 282 (now Article
297) of the Labor Code, the employee's misconduct must be serious, i.e., of such grave and aggravated
character and not merely trivial or unimportant, as in this case where the item which respondent tried to
takeout was practically of no value to petitioner.

FACTS:
Respondent was employed by petitioner as packhouse operator in its La Union Plant for nineteen (19)
years, from March 19, 1994 until August 8, 2013. As packhouse operator, respondent ensures the safe and efficient
operation of rotopackers. auto-bag placers, and cariramats, as well as their auxiliaries.
On July 10, 2013, at around 4 o'clock in the afternoon, respondent was about to exit Gate 2 of petitioner's
La Union Plant when the security guard on duty, Kristian Castillo (Castillo), asked him to submit himself and the
backpack he was carrying for inspection. Respondent refused and con2ded to Castillo that he has a piece of scrap
electrical wire in his bag. He also requested Castillo not to report the incident to the management, and asked the
latter if respondent could bring the scrap wire outside the company premises; otherwise, he will return it to his locker
in the Packhouse Office. However, Castillo did not agree, which prompted respondent to turn around and hurriedly go
back to the said office where he took the scrap wire out of his bag. Soon thereafter, a security guard arrived and
directed him to go to the Security Office where he was asked to write a statement regarding the incident. In his
statement respondent admitted the incident, but asserted that he had no intention to steal.
On July 16, 2013, respondent received a Notice of Gap requiring him to explain within five (5) days
therefrom why no disciplinary action, including termination, should be taken against him on account of the
abovementioned incident. He was also placed on preventive suspension for thirty (30) days effective immediately. In
a statement dated July 23, 2013, respondent reiterated that he had no intention to steal from petitioner and that the
scrap wire which he had asked from a contractor was already for disposal anyway. He also expressed his remorse
over the incident and asked that he be given a chance to correct his mistake. Meetings of petitioner's Review
Committee were thereafter conducted, with respondent and the security guards concerned in attendance.
Respondent was later on dismissed by petitioner for serious misconduct. Petitioner found incredulous
respondent’s claim that he was unaware that a gate pass is required to take out a piece of scrap wire since he had
been working thereat for 19 years. Respondent sought reconsideration and prayed for a lower penalty, especially
considering his length of service with petitioner but the same was denied by petitioner. Hence, respondent filed with
the NLRC a complaint for illegal dismissal and money claims.

LA/RTC/NLRC RULING:
LA: Dismissed the complaint and held that the latter was validly dismissed from service by petitioner for committing
the crime of theft, and therefore, not entitled to reinstatement, backwages, and other money claims.

NLRC: Reversed the LA ruling and held that the penalty imposed, dismissal, was unduly harsh since his
misconduct was not so gross to deserve such penalty. It found merit that the piece of electrical wire was already for
disposal, noting that petitioner never denied the same. It emphasized that petitioner did not suffer any damage since
the said wire never made it out of the company’s premises. That respondent did not hold a position of trust and
confidence and that he was remorseful of his mistake. Coupled with respondent’s length of service, the dismissal was
considered excessive and harsh. NLRC awarded backwages and separation pay in lieu of reinstatement.

CA RULING:
Affirmed the Ruling of the NLRC.

APPEAL TO THE SC:

Petitioner's Contention: Respondent was validly dismissed for serious misconduct

Respondent's Contention: Penalty of dismissal was excessive

ISSUE/S:
1. Whether respondent was validly dismissed

HELD:
3. Respondent was illegally dismissed.

Angeles| Bajana | Balladares | Brillantes | Briones | Cabansag | Callanta | Chua | David|


De Leon | Gomez | Lopez | Macalino | Nostratis | Padilla | Reynon | Santos | Tan |Velasco
4E / 4F - 2018-2019
Page 768 of 920
LABOR REVIEW DIGEST
Atty. Joyrich Golangco

There is no question that the employer has the inherent right to discipline, including that of dismissing its
employees for just causes. This right is, however, subject to reasonable regulation by the State in the exercise of its
police power.
The Court agrees with the CA and the NLRC that respondent's misconduct is not so gross as to deserve the
penalty of dismissal from service.
Neither can respondent's infraction be characterized as a serious misconduct which, under Article 282 (now
Article 297) of the Labor Code, is a just cause for dismissal. Misconduct is an improper or wrong conduct, or a
transgression of some established and definite rule of action, a forbidden act, a dereliction of duty, willful in character,
and implies wrongful intent and not mere error in judgment. To constitute a valid cause for dismissal within the
text and meaning of Article 282 (now Article 297) of the Labor Code, the employee's misconduct must be
serious, i.e., of such grave and aggravated character and not merely trivial or unimportant, as in this case
where the item which respondent tried to takeout was practically of no value to petitioner. Moreover, ill will or
wrongful intent cannot be ascribed to respondent. considering that, while he asked Castillo not to report the incident
to the management, he also volunteered the information that he had a piece of scrap wire in his bag and offered to
return it if the same could not possibly be brought outside the company premises sans a gate pass.

Other Notes/ SC Pronouncements:


SC found no strained relationship between parties which would make reinstatement impossible. It ordered
respondent’s reinstatement but without payment of backwages because respondent was not entirely faultless and
should not profit from a wrongdoing.

Angeles| Bajana | Balladares | Brillantes | Briones | Cabansag | Callanta | Chua | David|


De Leon | Gomez | Lopez | Macalino | Nostratis | Padilla | Reynon | Santos | Tan |Velasco
4E / 4F - 2018-2019
Page 769 of 920
LABOR REVIEW DIGEST
Atty. Joyrich Golangco

ARTICLE 298 – CLOSURE OF ESTABLISHMENT AND REDUCTION OF PERSONNEL

1. SPI TECHNOLOGIES, INC. AND LEA VILLANUEVA V. VICTORIA K. MAPUA


GR NO: G.R. NO. 191154 DATE: APRIL 7, 2014
PONENTE: REYES, J.

Doctrine:
For a valid implementation of a redundancy program, the employer must comply with the following
requisites: (1) written notice served on both the employee and the DOLE at least one month prior to the
intended date of termination; (2) payment of separation pay equivalent to at least one month pay or at least
one month pay for every year of service, whichever is higher; (3) good faith in abolishing the redundant
position; and (4) fair and reasonable criteria in ascertaining what positions are to be declared redundant.

FACTS:
Victoria K. Mapua alleged that she was hired by SPI Technologies, Inc. and was the Corporate Development's
Research/Business Intelligence Unit Head and Manager of the company. Sometime, the hard disk on Mapua's
laptop crashed, causing her to lose files and data. Mapua informed Nolan and her colleagues that she was working
on recovering the lost data and asked for their patience for any possible delay on her part in meeting deadlines.

Mapua retrieved the lost data with the assistance of National Bureau of Investigation Anti-Fraud and Computer
Crimes Division. Yet, Nolan informed Mapua that she was realigning Mapua's position to become a subordinate of
comanager Sameer Raina due to her missing a work deadline.

Mapua noticed that her colleagues began to ostracize and avoid her. Nolan and Raina started giving out majority of
her research work and other duties under Healthcare and Legal Division to the rank-and-file staff. Mapua lost about
95% of her work projects and job responsibilities.

Mapua consulted these work problems with SPI's Human Resource Director, Lea Villanueva, and asked if she can be
transferred to another department within SPI. Mapua was informed of an intra-office opening. However, due to
postponements not made by Mapua, her interview did not materialize.

The new organizational structure also showed that Mapua's level will be again downgraded because a new manager
will be hired and positioned between her rank and Raina's

Raina informed Mapua over the phone that her position was considered redundant and that she is terminated from
employment effective immediately. Villanueva notified Mapua through termination letter that she should cease
reporting for work the next day.

Mapua's lawyer, in a phone call, advised Villanueva that SPI violated Mapua's right to a 30-day notice. Mapua filed
with the LA a complaint for illegal dismissal, claiming reinstatement or if deemed impossible, for separation pay.

Mapua received through mail, a third Notice of Termination dated March 21, 2007 but the date of effectivity of the
termination was changed from March 21 to April 21, 2007. Also, a recruitment advertisement of SPI was published in
the Philippine Daily Inquirer which listed all vacancies in SPI, including a position for Marketing Communications
Manager under Corporate Support —
the same group where Mapua previously belonged.

LA/RTC/NLRC RULING:
The LA declared the termination illegal since the redundancy of Mapua’s position is in want of factual basis.

The NLRC reversed the decision and found SPI not guilty of illegal dismissal. The determination of whether Mapua’s
position as Corporate Development Manager is redundant essentially lies within the sound business management.

CA RULING:
The CA reinstated the decision of the LA

APPEAL TO THE SC:


Petition for Review on Certiorari

Petitioner's Contention:

Angeles| Bajana | Balladares | Brillantes | Briones | Cabansag | Callanta | Chua | David|


De Leon | Gomez | Lopez | Macalino | Nostratis | Padilla | Reynon | Santos | Tan |Velasco
4E / 4F - 2018-2019
Page 770 of 920
LABOR REVIEW DIGEST
Atty. Joyrich Golangco

SPI stated that the company regularly makes an evaluation and assessment of its corporate/organizational structure
due to the unexpected growth of its business along with its partnership with ePLDT and the acquisition of CyMed. As
a result, SPI underwent a reorganization of its structure with the objective of streamlining its operations. This was
embodied in an Inter-Office Memorandum. It was then discovered after assessment and evaluation that the duties of
a Corporate Development Manager could be performed/were actually being performed by other
officers/managers/departments of the company. As proof that the duties of Mapua are being/could be performed by
other SPI officers and employees, Villanueva executed an affidavit attesting that Mapua's functions are being
performed by other SPI managers and employees.

Respondent's Contention:
Mapua was dismissed from employment supposedly due to redundancy. However, she contended that her position
as Corporate Development Manager is not redundant. She cited that SPI was in fact actively looking for her
replacement after she was terminated. Furthermore, SPI violated her right to procedural due process when her
termination was
made effective on the same day she was notified of it.

ISSUE:
Whether the termination by reason of redundancy is valid.

HELD:
NO.

In Asian Alcohol Corporation v. NLRC pronounced that for a valid implementation of a redundancy program, the
employer must comply with the following requisites: (1) written notice served on both the employee and the DOLE at
least one month prior to the intended date of termination; (2) payment of separation pay equivalent to at least one
month pay or at least one month pay for every year of service, whichever is higher; (3) good faith in abolishing the
redundant position; and (4) fair and reasonable criteria in ascertaining what positions are to be declared redundant.

Date of Termination

Anent the first requirement which is written notice served on both the employee and the DOLE at least one month
prior to the intended date of termination, SPI had discharged the burden of proving that it submitted a notice to the
DOLE on March 21, 2007, stating therein that the effective date of termination is on April 21, 2007. It is, however,
quite peculiar that two kinds of notices were served to Mapua. One termination letter stated that its date of effectivity
is on the same day, March 21, 2007. The other termination letter sent through mail to Mapua's residence stated that
the effective date of her termination is on April 21, 2007.

Explaining the discrepancy, SPI alleged that the company served a notice to Mapua on March 21, 2007, which stated
that the effective date of termination is on April 21, 2007. However she refused to acknowledge or accept the letter.
Later on, Mapua requested for a copy of the said letter but due to inadvertence and oversight, a draft of the
termination letter bearing a wrong effectivity date was given to her. To correct the oversight, a copy of the original
letter was sent to her through mail. Curiously, there was neither allegation nor proof that the original letter was
misplaced or lost which would necessitate the drafting of a new one. SPI did not even explain in the second letter that
the same was being sent in lieu of the one given to her. Hence, SPI must shoulder the consequence of causing the
confusion brought by the variations of termination letters given to Mapua.

Validity of Redundancy Program

SPI asserted that an employer has the unbridled right to conduct its own business in order to achieve the results it
desires. To prove that Villanueva's functions are redundant, SPI submitted an Inter-Office Memorandum and affidavit.
The memorandum however made no mention that the position of the Corporate Development Manager or any other
position would be abolished or deemed redundant but only enumerated the various functions of a Corporate
Development Manager being performed by other SPI employees. This is not enough evidence to support the
company’s redundancy program.

Meanwhile, SPI admitted that caused the Inquirer advertisement for a Marketing Communications Manager position.
Instead of explaining how the functions of a Marketing Communications Manager differ from a Corporate
Development Manager, SPI hardly disputed Mapua when it stated that, "judging from the titles or designation of the
positions, it is obvious that the functions of one are entirely different from that of the other." SPI, being the employer,

Angeles| Bajana | Balladares | Brillantes | Briones | Cabansag | Callanta | Chua | David|


De Leon | Gomez | Lopez | Macalino | Nostratis | Padilla | Reynon | Santos | Tan |Velasco
4E / 4F - 2018-2019
Page 771 of 920
LABOR REVIEW DIGEST
Atty. Joyrich Golangco

has possession of valuable information concerning the functions of the offices within its organization. Nevertheless, it
did not even bother to differentiate the two positions.

Furthermore, on the assumption that the functions of a Marketing Communications Manager are different from that of
a Corporate Development Manager, it was not even discussed why Mapua was not considered for the position. While
SPI had no legal duty to hire Mapua as a Marketing Communications Manager, it could have clarified why she is not
qualified for that position. In fact, Mapua brought up the subject of transfer to Villanueva and Raina several times
prior to her termination but to no avail. There was even no showing that Mapua could not perform the duties of a
Marketing Communications Manager.

Angeles| Bajana | Balladares | Brillantes | Briones | Cabansag | Callanta | Chua | David|


De Leon | Gomez | Lopez | Macalino | Nostratis | Padilla | Reynon | Santos | Tan |Velasco
4E / 4F - 2018-2019
Page 772 of 920
LABOR REVIEW DIGEST
Atty. Joyrich Golangco

2. EUGENE S. ARABIT, ET. AL. V. JARDINE PACIFIC FINANCE, INC. (FORMERLY MB FINANCE)
GR NO: 181719 DATE: APRIL 21, 2014
PONENTE: BRION, J

Doctrine:
Redundancy exists where the services of an employee are in excess of what is reasonably demanded by the
actual requirements of the enterprise. A position is redundant where it is superfluous , and superfluity of a
position or positions may be the outcome of a number of factors, such as over hiring of workers, decreased
volume of business, or dropping of a particular product line or service activity previously manufactured or
undertaken by the enterprise. Redundancy does not need to be always triggered by a decline in the
business.

FACTS:
Petitioners were former regular employees of respondent Jardine Pacific Finance, Inc. (formerly MB Finance)
(Jardine) and petitioners were also officers and members of MB Finance Employees Association-FFW Chapter (the
Union), a legitimate labor union and the sole exclusive bargaining agent of the employees of Jardine.
On the claim of financial losses, Jardine decided to reorganize and implement a redundancy program among its
employees. The petitioners were among those affected by the redundancy program. Jardine thereafter hired
contractual employees to undertake the functions these employees used to perform.
Petitioners and the Union filed a complaint against Jardine with the NLRC for illegal dismissal and unfair labor
practice. Petitioners alleged before the LA that their dismissal was illegal and was tainted with bad faith as their
positions were not superfluous. They argued that if their positions had really been redundant, then Jardine should
have not hired contractual workers to replace them.
Jardine argued that it had to lay-off some of its employees and reorganize its ranks to eliminate positions that were in
excess of what its business required because it incurred substantial business losses from 1996-1999. Jardine
reasoned out that no bad faith took place since the hiring of contractual employees was a valid exercise of its
management prerogative.

LA Ruling:
LA held that the hiring of contractual employees to replace the petitioners directly contradicts the concept of
redundancy which involves the trimming down of the workforce because a task is being carried out by too many
people. The LA explained that the company's action was a circumvention of the right of the petitioners to security of
tenure. Jardine should have explained why the respective positions of the petitioners became superfluous in relation
to the other positions and employees of the company.

NLRC Ruling:
NLRC affirmed the LA’s decision.

CA RULING:
CA found that Jardine's act of hiring contractual employees in replacement of the petitioners does not run counter to
the argument that their positions are already superfluous. According to the CA, the hiring of contractual employees is
a management prerogative that Jardine has the right to exercise. In the absence of any showing of malice or
arbitrariness on the part of Jardine in implementing its redundancy program, the courts must not interfere with the
company's exercise of a bonafide management decision.

APPEAL TO THE SC:


Petitioner’s contention:
Petitioners assert that Jardine did not take any remedial measure before it implemented its redundancy program. It
simply hastily terminated the petitioners from the service without any basis or criteria in choosing the petitioners for
inclusion in the program. Further, Jardine's act of hiring contractual employees as their replacements is contrary to
Jardine's claim that there was redundancy.

ISSUE/S:
Is there a valid redundancy to qualify as an authorized cause of termination of an employee?

HELD:
NO. Redundancy exists where the services of an employee are in excess of what is reasonably demanded by the
actual requirements of the enterprise. A position is redundant where it is superfluous , and superfluity of a position or
positions may be the outcome of a number of factors, such as over hiring of workers, decreased volume of business,

Angeles| Bajana | Balladares | Brillantes | Briones | Cabansag | Callanta | Chua | David|


De Leon | Gomez | Lopez | Macalino | Nostratis | Padilla | Reynon | Santos | Tan |Velasco
4E / 4F - 2018-2019
Page 773 of 920
LABOR REVIEW DIGEST
Atty. Joyrich Golangco

or dropping of a particular product line or service activity previously manufactured or undertaken by the enterprise.
Redundancy does not need to be always triggered by a decline in the business. From this perspective, it is illogical
for Jardine to terminate the petitioners' employment and replace them with contractual employees. The replacement
effectively belies Jardine's claim that the petitioners' positions were abolished due to superfluity. Redundancy could
have been justified if the functions of the petitioners were transferred to other existing employees of the company.

Other Notes/ SC Pronouncements:


Retrenchment, on the other hand, is used interchangeably with the term "lay-off." It is the termination of employment
initiated by the employer through no fault of the employee's and without prejudice to the latter, resorted to by
management during periods of business recession, industrial depression, or seasonal fluctuations, or during lulls
occasioned by lack of orders, shortage of materials, conversion of the plant for a new production program or the
introduction of new methods or more efficient machinery, or of automation. Simply put, it is an act of the employer of
dismissing employees because of losses in the operation of a business, lack of work, and considerable reduction on
the volume of his business, a right consistently recognized and affirmed by this Court.

In Golden Thread Knitting Industries, Inc. v. NLRC, this Court laid down the principle that the employer must use fair
and reasonable criteria in the selection of employees who will be dismissed from employment due to redundancy.
Such fair and reasonable criteria may include the following, but are not limited to:
(a) less preferred status (e.g., temporary employee);
(b) efficiency; and
(c) seniority.

In Asian Alcohol Corp. v. NLRC, the Court set guidelines for redundancy to be characterized as validly undertaken by
the employer:
(1) written notice served on both the employees and the Department of Labor and Employment at least one month
prior to the intended date of retrenchment;
(2) payment of separation pay equivalent to at least one month pay or at least one month pay for every year of
service, whichever is higher;
(3) good faith in abolishing the redundant positions; and
(4) fair and reasonable criteria in ascertaining what positions are to be declared redundant and accordingly
abolished.

Angeles| Bajana | Balladares | Brillantes | Briones | Cabansag | Callanta | Chua | David|


De Leon | Gomez | Lopez | Macalino | Nostratis | Padilla | Reynon | Santos | Tan |Velasco
4E / 4F - 2018-2019
Page 774 of 920
LABOR REVIEW DIGEST
Atty. Joyrich Golangco

3. PHILIPPINE CARPET MANUFACTURING CORPORATION, ET AL. vs. IGNACIO B. TAGYAMON, ET AL.


G.R. No. 191475 Date: December 11, 2013
Ponente: PERALTA, J.

DOCTRINE:
The purchase of machinery and equipment after the termination, the declaration of cash dividends to
stockholders, the hiring of 100 new employees after the retrenchment, and the authorization of full blast
overtime work for six hours daily are inconsistent with PCMC’s claim that there was a slump in the demand
for its products which compelled them to implement the termination programs.

FACTS:
Respondents were the regular and permanent employees of petitioner Philippine Carpet Manufacturing
Corporation (PCMC), a corporation engaged in the business of manufacturing wool and yarn carpets and rugs. On
March 15, 2004, respondents received a uniformly worded Memorandum of dismissal on the ground of its voluntary
retirement and retrenchment programs as the company was constrained to reduce the number of its personnel “due
to the tremendous drop of production output since about the last quarter of 1994 up to the present.” Respondents
filed separate complaints for illegal dismissal.

LA RULING:
The complaint was dismissed.

NLRC RULING:
Affirmed the decision of the LA.

CA RULING:
The respondents were illegally dismissed.

Respondents’ contention:
They primarily relied on the SC decision in Philippine Carpet Employees Association (PHILCEA) v. Hon. Sto.
Tomas (Philcea case) as to the validity of the company’s retrenchment program. They further explained that PCMC
did not, in fact, suffer losses shown by its acts prior to and subsequent to their termination. They also insisted that
their acceptance of separation pay and signing of quitclaim is not a bar to the pursuit of illegal dismissal case.

Petitioner’s contention:
As an employer, it had no obligation to keep in its employ more workers than are necessary for the operation of his
business. Thus, there was an authorized cause for dismissal as an exercise of a necessary management
prerogative.

ISSUE:
Were the dismissals of the respondents illegal? YES.

RULING:
YES. In Philippine Carpet Employees Association (PHILCEA) v. Hon. Sto. Tomas—the illegality of the basis of
the implementation of both the voluntary retirement and retrenchment programs of petitioners had been thoroughly
ruled upon by the SC. In that case, the SC discussed that both retrenchment and redundancy are authorized causes
of termination but the petitioners failed to adduce clear and convincing evidence to prove the confluence of the
essential requisites for a valid retrenchment of its employees. In ascertaining the bases of the termination of
employees, SC took into consideration petitioners’ claim of business losses; the purchase of machinery and
equipment after the termination, the declaration of cash dividends to stockholders, the hiring of 100 new employees
after the retrenchment, and the authorization of full blast overtime work for six hours daily. These, said the Court, are
inconsistent with petitioners’ claim that there was a slump in the demand for its products that compelled them to
implement the termination programs. In arriving at its conclusions, the Court took note of petitioners’ net sales, gross
and net profits, as well as net income. The Court, thus, reached the conclusion that the retrenchment effected by
PCMC is invalid due to a substantive defect.

As quoted therein—“contrary to the claim of PCMC that it was experiencing business losses, in fact, it amassed
substantial earnings from 1999 to 2003. It found no need to appropriate its retained earnings except on March 23,
2001, when it appropriated P60,000,000 to increase production capacity. If PCMC were to be believed that it had to
retrench employees due to the debilitating slump in demand for its products resulting in severe losses, how could it
justify the purchase of P20,000,000 worth of machinery and equipment? There is likewise no justification for the

Angeles| Bajana | Balladares | Brillantes | Briones | Cabansag | Callanta | Chua | David|


De Leon | Gomez | Lopez | Macalino | Nostratis | Padilla | Reynon | Santos | Tan |Velasco
4E / 4F - 2018-2019
Page 775 of 920
LABOR REVIEW DIGEST
Atty. Joyrich Golangco

hiring of more than 100 new employees, more than the number of those who were retrenched, as well as the order
authorizing full blast overtime work for six hours daily. All these are inconsistent with the intransigent claim that
PCMC was impelled to retrench its employees precisely because of low demand for its products and other external
causes.
xxxx

That PCMC acted in bad faith in retrenching the 77 members of petitioner is buttressed by the fact that Diaz issued
his Memorandum announcing the cost-reduction program on March 9, 2004, after receipt of the February 10, 2004
letter of the Union president which included the proposal for additional benefits and wage increases to be
incorporated in the CBA for the ensuing year. Petitioner and its members had no inkling, before February 10, 2004,
that PCMC would terminate their employment. Moreover, PCMC failed to exhaust all other means to avoid further
losses without retrenching its employees, such as utilizing the latter's respective forced vacation leaves.
Respondents also failed to use fair and reasonable criteria in implementing the retrenchment program, and instead
chose to retrench 77 of the members of petitioner out of the dismissed 88 employees. Worse, PCMC hired new
employees and even rehired the others who had been "retrenched."
xxxx

In contrast, in this case, the retrenchment effected by PCMC is invalid due to a substantive defect, non-compliance
with the substantial requirements to effect a valid retrenchment; it necessarily follows that the termination of the
employment of petitioner Union's members on such ground is, likewise, illegal. As such, petitioner Union's members
are entitled to reinstatement with full backwages.

Angeles| Bajana | Balladares | Brillantes | Briones | Cabansag | Callanta | Chua | David|


De Leon | Gomez | Lopez | Macalino | Nostratis | Padilla | Reynon | Santos | Tan |Velasco
4E / 4F - 2018-2019
Page 776 of 920
LABOR REVIEW DIGEST
Atty. Joyrich Golangco

4. SANOH FULTON PHILS., INC. and MR. EDDIE JOSE


vs. EMMANUEL BERNARDO and SAMUEL TAGHOY

GR No: 187214 Date: August 14, 2013


Ponente: PEREZ, J.
Doctrine: We held in Lambert Pawnbrokers and Jewelry Corporation v. Binamira,20 that the losses must be
supported by sufficient and convincing evidence and the normal method of discharging this is by the
submission of financial statements duly audited by independent external auditors.21 It was aptly observed
by the appellate court that no financial statements or documents were presented to substantiate Sanoh’s
claim of loss of ₱7 million per month. And a business lull caused by lack of orders which could have
justified retrenchment was not shown by petitioner. As observed once more by the Court of Appeals,
petitioner failed to present proof of the extent of the reduced order and its contribution to the sustainability
of its business.

FACTS:

Sanoh is a domestic corporation engaged in the manufacture of automotive parts and wire condensers for home
appliances. Its Wire Condenser Department employed 61 employees. Respondents belonged to this department.
In view of job order cancellations relating to the manufacture of wire condensers by Matsushita, Sanyo and National
Panasonic, Sanoh decided to phase out the Wire Condenser Department. On 22 December 2003, the Human
Resources Manager of Sanoh informed the 17 employees, 16 of whom belonged to the Wire Condenser Department,
of retrenchment effective 22 January 2004. All 17 employees are union members.
A grievance conference was held where the affected employees were informed of the following grounds for
retrenchment:
1) Lack of local market.
2) Competition from imported products.
3) Phasing out of Wire Condenser Department.3
Two succeeding conciliation conferences were likewise held but the parties failed to reach an amicable settlement.
Thus, two (2) separate complaints for illegal dismissal were filed.
Sanoh on its part, filed a petition for declaration of the partial closure of its Wire Condenser Department and valid
retrenchment of the 17 employees, docketed as NLRC Case No. RAB-IV-01-18762-04-C.
During the course of the proceedings before the Labor Arbiter, 14 of the 17 employees executed individual
quitclaims. Hence, their interest in the cases was dismissed with prejudice. Only 3 employees, respondents
Emmanuel Bernardo and Samuel Taghoy, and Manny Santos persisted.
The complainants alleged that there was no valid cause for retrenchment and in effecting retrenchment, there was a
violation of the "first in-last out" and "last in-first out" (LIFO) policy embodied in the Collective Bargaining Agreement.
Sanoh, on the other hand, asserted that retrenchment was a valid exercise of management prerogative. Sanoh
averred that some employees who were hired much later were either assigned to other departments or were bound
by the terms of their job training agreement to stay with the company for 3 years.
Labor Arbiter rendered a Decision5 dismissing the complaint for illegal dismissal.
NLRC affirmed in toto the decision of the Labor Arbiter in its Resolution. The NLRC held that "the retrenchment x x x
was a valid exercise of management prerogative, more so, since the said decision was premised on the ‘permanent
lack of orders from major clients.’"8The NLRC found no violation of the company’s LIFO policy because the
employees involved were bound under a training agreement to render three (3) years of continuous service. The
NLRC also sustained the award of separation pay to the three (3) employees.
The Court of Appeals overturned the findings of the Labor Arbiter and the NLRC, and ruled that Sanoh failed to
prove the existence of substantial losses that would justify a valid retrenchment. The Court of Appeals also upheld
the quitclaim executed by complainant Manny Santos, thus he was deemed to have released Sanoh from his
monetary claims.

ISSUE/S:

Whether or not there was a valid retrenchment.

HELD:
No. After scouring the records, we are in full accord with the decision of the Court of Appeals.
In this case, there was no valid retrenchment. Nor was there a closure of business.
We are mindful of the principle that losses in the operation of the enterprise, lack of work, or considerable reduction
on the volume of business may justify an employer to reduce the work force. But a lull caused by lack of orders or

Angeles| Bajana | Balladares | Brillantes | Briones | Cabansag | Callanta | Chua | David|


De Leon | Gomez | Lopez | Macalino | Nostratis | Padilla | Reynon | Santos | Tan |Velasco
4E / 4F - 2018-2019
Page 777 of 920
LABOR REVIEW DIGEST
Atty. Joyrich Golangco

shortage of materials must be of such nature as would severely affect the continued business operations of the
employer to the detriment of all and sundry if not properly addressed.19
Sanoh asserts that cancelled orders of wire condensers led to the phasing out of the Wire Condenser Department
which triggered retrenchment. Sanoh presented the letters of cancellation given by Matsushita and Sanyo as
evidence of cancelled orders.1âwphi1 The evidence presented by Sanoh barely established the connection between
the cancelled orders and the projected business losses that may be incurred by Sanoh. Sanoh failed to prove that
these cancelled orders would severely impact on their production of wire condensers.
We held in Lambert Pawnbrokers and Jewelry Corporation v. Binamira,20 that the losses must be supported by
sufficient and convincing evidence and the normal method of discharging this is by the submission of financial
statements duly audited by independent external auditors.21 It was aptly observed by the appellate court that no
financial statements or documents were presented to substantiate Sanoh’s claim of loss of ₱7 million per month. And
a business lull caused by lack of orders which could have justified retrenchment was not shown by petitioner. As
observed once more by the Court of Appeals, petitioner failed to present proof of the extent of the reduced order and
its contribution to the sustainability of its business.
On the other hand, respondents’ refutations of the employer’s reason for retrenchment were supported by
documentary evidence. Respondents explained that Matsushita had four (4) outstanding orders of condensers of
refrigerators: Model 17-20, Model 1404, Model 802 and Model 602. It was only in March 2004 that Model 17-20 and
Model 1404 were phased out and only in July 2004 that Model 802 was phased out. However, Model 602 remained
and the order of Matsushita had been increased from 500 to 1600 units monthly from July 2004.22
With respect to the Sanyo account, respondent assert that Sanyo had sufficient stocks for three (3) months which
explained why it did not order from Sanyo. However, beginning February 2004, Sanyo resumed making orders.23
Respondents added that despite the cancellation of some orders by Matsushita and Sanyo, the additional orders
made by Concepcion Industries and Uni-Magma more than compensated the losses incurred on the cancelled
orders.24
Verily, Sanoh failed to discharge its burden of submitting competent proof to show the substantial business losses it
suffered warranting retrenchment. Contrarily, respondents amply proved that the cancelled orders did not seriously
create a dent on Sanoh’s financial standing. Respondents further presented the production target and actual
production of the Wire Condenser Department for the year 2005, to prove that the department had realized income
for that year.
Sanoh would then argue that it did not even have to prove business losses when it decided to close down the Wire
Condenser Department because the law recognizes the right of management to cease business operations. As
already stated, the burden of proving that the closure was bona fide, rests upon the employer. Sanoh made a
categorical statement that the Wire Condenser Department was totally closed. The documentary evidence presented
by respondents, however, negate Sanoh’s statement. In other words, Sanoh lacked bona fides even in its assertion
that Wire Condenser Department had closed down. Respondents disclose that this department had gone full blast in
its operations, even with substantial overtime operations immediately after their dismissal was effected. Moreover,
respondents assert that Sanoh still hired employees after the so-called retrenchment.
Respondents submitted the time sheets of the Wire Condenser Department for the months of January up to July
200425 which showed that some of the employees had been rendering overtime work after retrenchment was
effected presumably to compensate the lack of manpower in that department.
As the Wire Condenser Department is still in operation and no business losses were proven by Sanoh, the dismissal
of respondents was unlawful. Resultingly, respondents are entitled to reinstatement without loss of seniority rights
and other privileges and to full backwages, computed from the time the compensation was withheld up to the time of
actual reinstatement. Present law says that if reinstatement is not feasible, the payment of full backwages shall be
made from the date of dismissal until finality of judgment.
Verily, in this case, reinstatement is no longer practical in view of the length of time that had elapsed tl·01n the time
of respondents' dismissal.26 As held in EDI Staff Builders International Inc. v. Magsino, apart from backwages,
respondents should be awarded separation pay.

Angeles| Bajana | Balladares | Brillantes | Briones | Cabansag | Callanta | Chua | David|


De Leon | Gomez | Lopez | Macalino | Nostratis | Padilla | Reynon | Santos | Tan |Velasco
4E / 4F - 2018-2019
Page 778 of 920
LABOR REVIEW DIGEST
Atty. Joyrich Golangco

5. ANDRADA, ET AL VS NLRC AND SUBIC LEGEND RESORTS AND CASINO


GR NO: 173231 DATE: DECEMBER 28, 2007
PONENTE: VELASCO, JR. J.

Doctrine:
To provide full protection to labor, the employers' prerogative to bring down labor costs through
retrenchment must be exercised carefully and essentially as a measure of last resort. So should
managements' prerogative to declare the employees' services redundant not be used a weapon to frustrate
labor.

FACTS:
Petitioners were employees of Subic Legend Resorts and Casino. Legend sent notice to the DOLE of its intention to
retrench and terminate the employment of 34 of its employees, which include petitioners. Thereafter, Legend sent the
34 employees their respective notices of retrenchment, stating the same reasons for their retrenchment. It offered to
its employees the choice between temporary retrenchment, permanent retrenchment and immediate retrenchment.
The said 34 employees made their choices and signed quitclaims. Subsequently, the Labor and Employment Center
of the SBMA advertised that Legend International Resorts, Inc. was in need of employees for positions similar to
those vacated by petitioners.||

Thereafter, they filed with the NLRC a complaint for illegal dismissal based the said advertisement. Legend, however,
invoked management prerogative.

LA RULING:
The LA ruled that the petitioners were illegally dismissed finding that Legend failed to prove that it had suffered actual
losses or that there were redundant positions.

NLRC RULING:
The NLRC reversed the LA decision and ruled that the petitioners were validly terminated from employment due to
redundancy and not retrenchment.

CA RULING:
The CA affirmed the NLRC decision.

APPEAL TO THE SC:


Petitioner's Contention:
Petitioners argue that Legend failed to prove the legal and factual existence of the cause for dismissal, and that it
failed to comply with the requirements for the implementation of retrenchment. Petitioners further argue that the CA
abused its discretion in ruling that the employees were validly dismissed not because of retrenchment but for
redundancy.|||

Respondent's Contention:
Legend, in contrast, relied on its management prerogative to justify the termination of petitioners' employment.
Legend also relied on the CA's ruling that Legend sufficiently proved the existence of redundancy that justified
petitioners' dismissal from service.|||

ISSUE/S:
1. Was there a valid retrenchment? - No

HELD:
1. No. A company's exercise of its management prerogatives is not absolute. It cannot exercise its prerogative in a
cruel, repressive, or despotic manner.||| Under the Labor Code, retrenchment and redundancy are authorized causes
for separation from service. However, to protect labor, dismissals due to retrenchment or redundancy are subject to
strict requirements under Article 283 of the Labor Code. Retrenchment is an exercise of management's prerogative
to terminate the employment of its employees en masse, to either minimize or prevent losses, or when the company
is about to close or cease operations for causes not due to business losses.|||

Also, Legend failed to establish redundancy. Retrenchment and redundancy are two different concepts; they are not
synonymous and therefore should not be used interchangeably. Redundancy exists where the services of an
employee are in excess of what is reasonably demanded by the actual requirements of the enterprise. Retrenchment,

Angeles| Bajana | Balladares | Brillantes | Briones | Cabansag | Callanta | Chua | David|


De Leon | Gomez | Lopez | Macalino | Nostratis | Padilla | Reynon | Santos | Tan |Velasco
4E / 4F - 2018-2019
Page 779 of 920
LABOR REVIEW DIGEST
Atty. Joyrich Golangco

on the other hand, is used interchangeably with the term "lay-off.” It is an act of the employer of dismissing
employees because of losses in the operation of a business, lack of work, and considerable reduction on the volume
of his business, a right consistently recognized and affirmed by this Court.|||

It is however not enough for a company to merely declare that positions have become redundant. It must produce
adequate proof of such redundancy to justify the dismissal of the affected employees. |||

In this case, the pieces of evidence submitted by Legend are mere allegations and conclusions not supported by
other evidence. Legend did not even bother to illustrate or explain in detail how and why it considered petitioners'
positions superfluous or unnecessary.|||

Other Notes/ SC Pronouncements:

The requirements for retrenchment are:


 it is undertaken to prevent losses, which are not merely de minimis, but substantial, serious, actual, and
real, or if only expected, are reasonably imminent as perceived objectively and in good faith by the
employer;
 the employer serves written notice both to the employees and the DOLE at least one month prior to the
intended date of retrenchment; and
 the employer pays the retrenched employees separation pay equivalent to one month pay or at least 1/2
month pay for every year of service, whichever is higher.
The Court later added the requirements that the employer must use fair and reasonable criteria in ascertaining who
would be dismissed and . . . retained among the employees and that the retrenchment must be undertaken in good
faith. Except for the written notice to the affected employees and the DOLE, non-compliance with any of these
requirements renders the retrenchment illegal.|||

Angeles| Bajana | Balladares | Brillantes | Briones | Cabansag | Callanta | Chua | David|


De Leon | Gomez | Lopez | Macalino | Nostratis | Padilla | Reynon | Santos | Tan |Velasco
4E / 4F - 2018-2019
Page 780 of 920
LABOR REVIEW DIGEST
Atty. Joyrich Golangco

6. CULILI vs ETPI
GR No.: 165381 Date: FEBRUARY 9, 2011
Ponente: LEONARDO-DE CASTRO, J.

DOCTRINE:

The following evidence may be proffered to substantiate redundancy: the new staffing pattern, feasibility
studies/ proposal on the viability of the newly created positions, job description and the approval by the
management of the restructuring.

FACTS:

Culili was a Senior Technician in the Customer Premises Equipment Management Unit of the Service Quality
Department of Eastern Telecom (ETPI). Due to business troubles and losses, ETPI was compelled to implement a
Right-Sizing Program which consisted of two phases: the first phase involved the reduction of ETPI’s workforce to
only those employees that were necessary and which ETPI could sustain; the second phase entailed a company-
wide reorganization which would result in the transfer, merger, absorption or abolition of certain departments of ETPI.
ETPI offered the Special Retirement Program and the corresponding retirement package to the 102 employees who
qualified for the program.

Of all the employees who qualified to avail of the program, only Culili rejected the offer. After the successful
implementation of the first phase of the Right-Sizing Program, ETPI proceeded with the second phase which
necessitated the abolition, transfer and merger of a number of ETPI’s departments. The specialized functions of a
Senior Technician was deemed to be unnecessary.

As a result, Culili’s position was abolished due to redundancy and his functions were absorbed by another employee.
Later, he was informed of his termination from employment through a letter which was similar to the memo shown to
Culili by the union president weeks before Culili was dismissed. The memo was advising him of his dismissal
effective due to the Right-Sizing Program ETPI was going to implement to cut costs and avoid losses.

Culilialleged that neither he nor the Department of Labor and Employment (DOLE) were formally notified of his
termination. Culili claimed that he only found out about it after he was barred from entering ETPI’s premises by its
armed security personnel when he tried to report for work. Culili asserted that ETPI had contracted out the services
he used to perform to a labor-only contractor which not only proved that his functions had not become unnecessary,
but which also violated their CBA and the Labor Code. ETPI denied singling Culili out for termination.

ETPI averred that since Culili did not avail of the Special Retirement Program and his position was subsequently
declared redundant, it had no choice but to terminate Culili.

LA RULING: Rendered a decision finding, among others, ETPI guilty of illegal dismissal and unfair labor practice,
and ordering it to reinstate Culili with backwages and such other benefits due him, and damages and attorney’s fees.

NLRC RULING: Affirmed the LA’s decision but modified the amount when it failed to properly notify both Culili and
the DOLE of Culili’s termination.

CA RULING: partially granted ETPI’s petition and deleted the award for damages. CA found that Culili’s position was
validly abolished due to redundancy

ISSUE/S:

1. Whether Culili was validly dismissed;


2. Whether ETPI failed to properly comply with the procedural due process requirement in terminating Culili?

HELD:

1. Yes. There is redundancy when the service capability of the workforce is greater than what is reasonably
required to meet the demands of the business enterprise. A position becomes redundant when it is rendered
superfluous by any number of factors such as over-hiring of workers, decrease in volume of business, or dropping a
particular product line or service activity previously manufactured or undertaken by the enterprise.

Angeles| Bajana | Balladares | Brillantes | Briones | Cabansag | Callanta | Chua | David|


De Leon | Gomez | Lopez | Macalino | Nostratis | Padilla | Reynon | Santos | Tan |Velasco
4E / 4F - 2018-2019
Page 781 of 920
LABOR REVIEW DIGEST
Atty. Joyrich Golangco

This Court has been consistent in holding that the determination of whether or not an employees services are still
needed or sustainable properly belongs to the employer. Provided there is no violation of law or a showing that the
employer was prompted by an arbitrary or malicious act, the soundness or wisdom of this exercise of business
judgment is not subject to the discretionary review of the Labor Arbiter and the NLRC.

However, an employer cannot simply declare that it has become overmanned and dismiss its employees without
producing adequate proof to sustain its claim of redundancy. Among the requisites of a valid redundancy program
are:

1. the good faith of the employer in abolishing the redundant position; and
2. fair and reasonable criteria in ascertaining what positions are to be declared redundant, such as but not
limited to: preferred status, efficiency, and seniority.

This Court also held that the following evidence may be proffered to substantiate redundancy: the new staffing
pattern, feasibility studies/ proposal on the viability of the newly created positions, job description and the approval by
the management of the restructuring.
In the case at bar, ETPI was upfront with its employees about its plan to implement a Right-Sizing Program.

Even in the face of initial opposition from and rejection of the said program by ETEU, ETPI patiently negotiated with
ETEUs officers to make them understand ETPIs business dilemma and its need to reduce its workforce and
streamline its organization. This evidently rules out bad faith on the part of ETPI.

In deciding which positions to retain and which to abolish, ETPI chose on the basis of efficiency, economy, versatility
and flexibility. It needed to reduce its workforce to a sustainable level while maintaining functions necessary to keep it
operating. The records show that ETPI had sufficiently established not only its need to reduce its workforce and
streamline its organization, but also the existence of redundancy in the position of a Senior Technician. ETPI
explained how it failed to meet its business targets and the factors that caused this, and how this necessitated it to
reduce its workforce and streamline its organization. ETPI also submitted its old and new tables of organization and
sufficiently described how limited the functions of the abolished position of a Senior Technician were and how it
decided on whom to absorb these functions.

In the new table of organization that the management approved, one hundred twelve (112) employees were
redeployed and nine (9) positions were declared redundant. It is inconceivable that ETPI would effect a company-
wide reorganization of this scale for the mere purpose of singling out Culili and terminating him. If Culili’s position
were indeed indispensable to ETPI, then it would be absurd for ETPI, which was then trying to save its operations, to
abolish that one position which it needed the most. Contrary to Culilis assertions that ETPI could not do away with his
functions as long as it is in the telecommunications industry, ETPI did not abolish the functions performed by Culili as
a Senior Technician. What ETPI did was to abolish the position itself
for being too specialized and limited.

The functions of that position were then added to another employee whose functions were broad enough to absorb
the tasks of a Senior Technician.

2. Yes. ETPI, in effecting Culili’s termination, simply asked one of its guards to serve the required written
notice on Culili. Culili, on one hand, claims in his petition that this was handed to him by ETPI’s vice president, but
previously testified before the Labor Arbiter that this was left on his table.

Regardless of how this notice was served on Culili, this Court believes that ETPI failed to properly notify Culili about
his termination. Aside from the manner the written notice was served, a reading of that notice shows that ETPI failed
to properly inform Culili of the grounds for his termination. Since it has been established that Culili’s termination was
due to an authorized cause and cannot be considered unfair labor practice on the part of ETPI, his dismissal
is valid. However, in view of ETPI’s failure to comply with the notice requirements under the Labor Code, Culili is
entitled to nominal damages in addition to his separation pay.

As for the award of moral and exemplary damages in illegal dismissal cases, moral damages are awarded only
where the dismissal was attended by bad faith or fraud, or constituted an act oppressive to labor, or was done in a
manner contrary to morals, good customs or public policy. Exemplary damages may avail if the dismissal was
effected in a wanton, oppressive or malevolent manner to warrant an award for exemplary damages.

Angeles| Bajana | Balladares | Brillantes | Briones | Cabansag | Callanta | Chua | David|


De Leon | Gomez | Lopez | Macalino | Nostratis | Padilla | Reynon | Santos | Tan |Velasco
4E / 4F - 2018-2019
Page 782 of 920
LABOR REVIEW DIGEST
Atty. Joyrich Golangco

It is our considered view that Culili has failed to prove that his dismissal was orchestrated by the individual
respondents herein (ETPI officers) for the mere purpose of getting rid of him. In fact, most of them have not even
dealt with Culili personally. Moreover, it has been established that his termination was for an authorized cause, and
that there was no bad faith on the part of ETPI in implementing its Right-Sizing Program, which involved abolishing
certain positions and departments for redundancy. It is not enough that ETPI failed to comply with the due process
requirements to warrant an award of damages, there being no showing that the company’s and its officers’ acts were
attended with bad faith or were done oppressively.

Angeles| Bajana | Balladares | Brillantes | Briones | Cabansag | Callanta | Chua | David|


De Leon | Gomez | Lopez | Macalino | Nostratis | Padilla | Reynon | Santos | Tan |Velasco
4E / 4F - 2018-2019
Page 783 of 920
LABOR REVIEW DIGEST
Atty. Joyrich Golangco

7. ASUFRIN, JR., vs SAN MIGUEL CORPORATION


G.R. No. 156658. March 10, 2004
Ynares-Santiago, J.:
DOCTRINE: Redundancy, for purposes of the Labor Code, exists where the services of an employee are in
excess of what is reasonably demanded by the actual requirements of the enterprise. However, it is not
enough for a company to merely declare that it has become over-manned. It must produce adequate proof
that such is the actual situation to justify the dismissal of the affected employees for redundancy. It bears
stressing that whether it be by redundancy or retrenchment or any of the other authorized causes, no
employee may be dismissed without observance of the fundamentals of good faith.
FACTS: Coca Cola Plant, then a department of respondent San Miguel Beer Corporation (SMC), hired petitioner
Bonifacio Asufrin as a utility/miscellaneous worker who later on became a regular monthly paid employee promoted
as Stock Clerk.
Petitioner was initially assigned at the Sam-ug Sales office in Bacolod City. In the following years, there were series
of a reorganization which resulted to the abolition of the sales office, respondent also implemented a new marketing
system and pre-selling schems which resulted to the certain positions declared as redundant.
Affected employees of Sum-ag sales force were informed that they can avail of respondents early retirement
package pursuant to the retrenchment program, while those who will not avail of early retirement would be
redeployed or absorbed at the Brewery or other sales offices. Petitioner opted to remain and manifested his
willingness to be assigned to any job, considering that he had three children in college.
Petitioner was surprised when he was informed that his name was included in the list of employees who availed of
the early retirement package. Petitioner then filed a complaint for illegal dismissal with the NLRC.
Labor Arbiter: dismissed the complaint for lack of merit.
NLRC: Set aside the Labor Arbiters decision and ordered respondent SMC to reinstate petitioner to his former or
equivalent position with full backwages.
Court of Appeals: reversed the decision of the NLRC and reinstated the judgment of the Labor Arbiter dismissing
the complaint for illegal dismissal.
ISSUE/s : Whether or not the dismissal of petitioner is based on a just and authorized cause.
HELD: NO. Petitioners dismissal is declared illegal.
Respondent alleged that petitioner was dismissed on the ground of redundancy, one of the authorized causes for
dismissal. Redundancy in an employer’s personnel force necessarily or even ordinarily refers to duplication of work.
Redundancy, for purposes of the Labor Code, exists where the services of an employee are in excess of what is
reasonably demanded by the actual requirements of the enterprise. A position is redundant where it is superfluous,
and superfluity of a position or positions may be the outcome of a number of factors, such as over hiring of workers,
decreased volume of business, or dropping of a particular product line or service activity previously manufactured or
undertaken by the enterprise.
The determination that employees services are no longer necessary or sustainable and, therefore, properly
terminable is an exercise of business judgment of the employer. The wisdom or soundness of this judgment is not
subject to discretionary review of the Labor Arbiter and the NLRC, provided there is no violation of law and no
showing that it was prompted by an arbitrary or malicious act.
However, the Court is not persuaded by the respondent’s defense for the following reasons: The petitioner did not
accept early retirement plan offer of the respondent. Petitioner clearly manifested, through several letters his desire
to be redeployed to the Sta. Fe Brewery or any sales office and for any position not necessarily limited to that of a
warehouse checker. Also, it appears that despite respondents allegation that it ceased and closed down its
warehousing operations at the Sum-ag Sales Office, actually it is still used for warehousing activities and as a transit
point where buyers and dealers get their stocks. It then appears that there are still available positions for the
petitioner who expressed his desires to be employed to any position as long as he remains to be employed.
Lastly, in selecting employees to be dismissed, a fair and reasonable criteria must be used, such as but not limited to
(a) less preferred status, e.g. temporary employee; (b) efficiency; and (c) seniority. In the case at bar, no criterion
whatsoever was adopted by respondent in dismissing petitioner. Respondent has not shown how the cessation of
operations of the Sum-ag Sales Office, if there is such, contributed to the ways and means of improving effectiveness
of the organization with the end in view of efficiency and cutting distribution overhead and other related costs.
Respondent, thus, clearly resorted to sweeping generalization[s] in dismissing complainant.
It bears stressing that whether it be by redundancy or retrenchment or any of the other authorized causes, no
employee may be dismissed without observance of the fundamentals of good faith.
It is not difficult for employers to abolish positions in the guise of a cost-cutting measure and we should not be easily
swayed by such schemes which all too often reduce to near nothing what is left of the rubble of rights of our exploited
workers.
All told, to sustain the position taken by the appellate court would be to dilute the working mans most important right:
his constitutional right to security of tenure. While respondent may have offered a generous compensation package
to those whose services were terminated upon the implementation of the pre-selling scheme, the Court found such

Angeles| Bajana | Balladares | Brillantes | Briones | Cabansag | Callanta | Chua | David|


De Leon | Gomez | Lopez | Macalino | Nostratis | Padilla | Reynon | Santos | Tan |Velasco
4E / 4F - 2018-2019
Page 784 of 920
LABOR REVIEW DIGEST
Atty. Joyrich Golangco

an offer, in the face of the prevailing facts, anathema to the underlying principles which give life to our labor statutes
because it would be tantamount to likening an employer-employee relationship to a salesman and a purchaser of a
commodity.
Wherefore, respondent is ordered to reinstate him to his former or equivalent position, with full backwages computed
up to his actual reinstatement, and to pay petitioner the sum equivalent to ten percent (10%) of his total monetary
award as attorneys fees.

Angeles| Bajana | Balladares | Brillantes | Briones | Cabansag | Callanta | Chua | David|


De Leon | Gomez | Lopez | Macalino | Nostratis | Padilla | Reynon | Santos | Tan |Velasco
4E / 4F - 2018-2019
Page 785 of 920
LABOR REVIEW DIGEST
Atty. Joyrich Golangco

8. OCEAN EAST AGENCY CORP. VS. LOPEZ


GR NO: 194410 DATE: OCTOBER 14, 2015
PONENTE: PERALTA, J.

Doctrine: For the implementation of a redundancy program to be valid, the employer must comply with these
requisites: (1) written notice served on both the employee and the Department of Labor and Employment at
least one month prior to the intended date of retrenchment; (2) payment of separation pay equivalent to at
least one month pay or at least one month pay for every year of service, whichever is higher; (3) good faith in
abolishing the redundant positions; and (4) fair and reasonable criteria in ascertaining what positions are to
be declared redundant and accordingly abolished.

FACTS: Petitioner Ocean East Agency Corporation (Ocean East) is a manning agency engaged in recruitment and
deployment of Filipino seamen for overseas principals.

Respondent Allan I. Lopez was employed as Documentation Officer assigned to Ocean East's Operations
Department. Prior to his employment, Ocean East had already engaged the services of one Grace Reynolds as
Documentation Clerk.

The Documentation Clerks and Officer were tasked to perform the following functions: prepare the line-up of request
crew by various principals in close coordination with the Port Captain; assist in attending to various operational
expenses and disbursements; coordinate closely with deserving former crew members for pooling and/or immediate
employment, if so required; and supervise the preparation of the crew documents, such as travel documents and
clearances.

Ocean East served notice to Lopez that effective thirty (30) days later, his services will be terminated on the ground
of redundancy, as his position as Documentation Officer is but a duplication of those occupied by its two (2) other
personnel who were also exercising similar duties and functions.

Lopez filed a Complaint for illegal dismissal.

LA/RTC/NLRC RULING: The Labor Arbiter rendered a Decision dismissing the illegal dismissal complaint for lack of
merit. Citing the employer's management prerogative to abolish a position which it deems no longer necessary, the
Labor Arbiter held that it would be unfair to compel Ocean East to retain Lopez' position whose duties and functions
are likewise being performed by its 2 other employees. It also ruled that apart from the lack of evidence to support
the acts of discrimination and oppression that he imputed against petitioners, there is also no showing that the
streamlining of Ocean East's workforce was attended by malice and ill-will.

NLRC also pointed out that what took place was a reduction of personnel due to redundancy, not retrenchment;
hence, there was no need to prove business losses on the part of Ocean East.

CA RULING: The CA reversed the NLRC ruling – there was illegal dismissal. Oceaneast's main argument that
Lopez' dismissal was only resorted to after a study that was conducted to keep its organization more cost efficient is
not only insufficient but also'baseless and self-serving. There is nothing in the records that shows that indeed a study
was conducted which led to the termination of Lopez' services on the ground that his position has become redundant.
Neither was there any proof that Oceaneast had a concrete redundancy program that is reflective of any financial
loss or possible and obtainable substantial profits in case the program is implemented nor were there any named
factors considered by Oceaneast in undertaking the reduction program.

Petitioner's Contention: Petitioners claim to have clearly established that the functions of Lopez as a
Documentation Officer is virtually a duplication of the duties and responsibilities performed by Ocean East's two (2)
other Documentation Clerks. They call attention to the job descriptions presented by both parties and the letter sent
to Lopez which both confirm that his position is but a duplication, as its duties and functions were exercised similarly
by such clerks in the company's operations department.

Respondent's Contention:

ISSUE/S: whether or not Lopez was legally dismissed

Angeles| Bajana | Balladares | Brillantes | Briones | Cabansag | Callanta | Chua | David|


De Leon | Gomez | Lopez | Macalino | Nostratis | Padilla | Reynon | Santos | Tan |Velasco
4E / 4F - 2018-2019
Page 786 of 920
LABOR REVIEW DIGEST
Atty. Joyrich Golangco

HELD: NO. Redundancy exists when the service capability of the workforce is in excess of what is reasonably
needed to meet the demands of the enterprise. A redundant position is one rendered superfluous by any number of
factors, such as over hiring of workers, decreased volume of business, dropping of a particular product line
previously manufactured by the company, or phasing-out of a service activity previously undertaken by the
business. Under these factors, the employer has no legal obligation to keep in its payroll more employees than are
necessary for the operation of its business. Even if a business is doing well, an employer can still validly dismiss an
employee from the service due to redundancy if that employee's position has already become in excess of what the
employer's enterprise requires.

As an authorized cause for termination of employment, redundancy may be implemented subject only to strict
requirements spelled out in Article 283 of the Labor Code.

For the implementation of a redundancy program to be valid, the employer must comply with these requisites: (1)
written notice served on both the employee and the Department of Labor and Employment at least one month prior to
the intended date of retrenchment; (2) payment of separation pay equivalent to at least one month pay or at least one
month pay for every year of service, whichever is higher; (3) good faith in abolishing the redundant positions; and (4)
fair and reasonable criteria in ascertaining what positions are to be declared redundant and accordingly abolished.

The Court finds that petitioners failed to establish compliance with the first, third and fourth requisites for a valid
implementation of a redundancy program, thereby making Ocean East liable for illegal dismissal.

It is undisputed that Ocean East failed to comply with the first requisite of service of a written notice of termination to
the DOLE.

There is no merit in petitioners' contention that notice to the DOLE may already be dispensed with since there was no
more useful purpose for it, and he was already adequately compensated as required by law. Indeed, to dispense with
such notice would not only disregard a clear labor law provision that affords protection to an employee, but also
defeats its very purpose which is to give the DOLE the opportunity to ascertain the veracity of the alleged authorized
cause of termination.

With regard to petitioners' failure to establish the third and fourth requisites for a valid implementation of a
redundancy program, the Court stresses the importance of having fair and reasonable criteria, such as but not limited
to (a) less preferred status, e.g., temporary employee; (b) efficiency; and (c) seniority.The presence of such criteria
used by the employer shows good faith on its part and is evidence that the implementation of redundancy was
painstakingly done by the employer in order to properly justify the termination from the service of its
employees. Conversely, the absence of criteria in the selection of an employee to be dismissed and the erroneous
implementation of the criterion selected, both render invalid the redundancy because both have the ultimate effect of
illegally dismissing an employee.

While it is true that the characterization of an employee's services as superfluous or no longer necessary and,
therefore, properly terminable, is an exercise of business judgment on the part of the employer, the exercise of such
judgment must not violate the law, and must not be arbitrary or malicious. An employer cannot simply declare that it
has become overmanned and dismiss its employees without adequate proof to sustain its claim of redundancy. The
following evidence may be proffered to substantiate redundancy, to wit: the new staffing pattern, feasibility
studies/proposal on the viability of the newly-created positions, job description and the approval by the management
of the restructuring.

In this case, petitioners were able to establish through Ocean East's Quality Procedures Manual that Lopez' position
as a Documentation Officer was redundant because its duties and functions were similar to those of the
Documentation Clerks in its operations department. However, they failed to prove by substantial evidence their
observance of the fair and reasonable criteria of seniority and efficiency in ascertaining the redundancy of the
position of Documentation Officer, as well as good faith on their part in abolishing such position. Petitioners were
unable to justify why it was more efficient to terminate Lopez rather than its two other Documentation Clerks,
Reynolds and Hing. Also, while Reynolds was supposedly retained for being more senior than Lopez, petitioners
were silent on why they chose to retain Hing who was hired in 1996, instead of Lopez who was hired about eight (8)
years earlier in 1988.

Angeles| Bajana | Balladares | Brillantes | Briones | Cabansag | Callanta | Chua | David|


De Leon | Gomez | Lopez | Macalino | Nostratis | Padilla | Reynon | Santos | Tan |Velasco
4E / 4F - 2018-2019
Page 787 of 920
LABOR REVIEW DIGEST
Atty. Joyrich Golangco

9. BLUE EAGLE MANAGEMENT, INC. V. NAVAL


G.R. NO. 192488, APRIL 19, 2016

FACTS:
Petitioner BEMI is a domestic corporation engaged in operating a sports complex while Petitioners Bonoan and Dela
Rama were then the General Manager and Human Resources (HR) Manager, respectively, of petitioner BEMI.
Respondent was hired by BEMI as a member of its maintenance staff. During its first year of operation, BEMI
suffered financial losses. In an attempt to reduce its financial losses, the Management resolved to implement
retrenchment proceedings. the Managemente evaluated and identified five employees for retrenchment taking into
consideration the employees positions and tenures at BEMI. Respondent was included in the list because she was
one of the employees with the shortest tenures.
Before actually commencing retrenchment proceedings, Dela Rama separately met with each of the five employees
and presented to them the option of resigning instead with financial package. This option would also give the
employees free time to seek other employment while still receiving salary from petitioner BEMI. The five employees
decided to voluntarily resign instead and avail themselves of the financial package offered by BEMI. Thereafter, the
resigned emplo!ees, except for respondent, appeared at the premises of BEMI, completed their exit procedures,
received the amounts due them, and executed release waivers and quitclaims in favor of BEMI.
As respondent was finding it difficult to find new employment, she asked it it was possible for her to return to work for
BEMI. However, Bonoan replied that respondent's resignation had long been approved and that petitioner BEMI
would not be able to rehire respondent given the difficult financial position of the company. On the afternoon of March
3, 2006, respondent filed with the NLRC a complaint for illegal dismissal with prayer for reinstatement and payment
of backwages, damages, and attorneys fees.

Issue: WON respondent was illegally dismissed.

Held:
No, respondent was not illegally dismissed. As borne out of the financial statements for 2005 of petitioner BEMI,
there was ground for the company to implement a retrenchment of its emplo!ees at the time respondent resigned.
The evaluation and identification of the employees to be retrenched were jointly undertaken by based on fair and
reasonable criteria, i.e., the employees' positions and tenures at the company. Respondent was included in the final
list of five employees to be retrenched because she was one of the emplo!ees with the shortest tenures.
Because the five emplo!ees to be retrenched opted to voluntarily resign instead avail themselves of the financial
package offered, there was no more need for petitioner BEMI to comply with the notice requirement to the
Department of Labor and Employmentt said five employees were to receive more benefits than what the law
prescribed in case of retrenchment, particularly (a) full salary for February 2006 although they were no longer
required to report to work after submission of their resignation letters in mid-February 2006; (b) pro-rated 13th month
pay; and (c) financial assistance equivalent to one-month salary for every year of service.
The foregoing circumstances persuade the Court that no fraud or deception was employed upon respondent to
resign because petitioner BEMI was indeed about to implement in good faith a retrenchment of its employees in
order to advance its interest and not merely to defeat or circumvent the respondent's right to security of tenure.

Petitioners, moreover, were able to present respondent's resignation letter, written and signed in her own hand, the
material portion of which is reproduced below:

Ako ay magbibitiw sa akingposition bilang maintenance personnel sa Feb. 28, 2006. Makukuha ko ang aking huling
sweldo sa Feb. 28, 2006. At makukuha ko ang aking separation pay at pro-rated 13' month pay sa Mar so 2006.31

Both the Labor Arbiter and the Court of Appeals invoked the oft-repeated ruling of the Court that resignation is
inconsistent with the filing of the complaint for illegal dismissal.32 However, the employee's filing of the complaint for
illegal dismissal by itself is not sufficient to disprove that said employee voluntarily resigned. There must be other
attendant circumstances and/or submitted evidence which would raise a cloud of doubt as to the voluntariness of the
resignation.
Aside from respondent's bare allegations, there is no proof of such threat ever being made. While respondent
claimed that her husband's employment was also connected with petitioner BEMI, she did not provide any other
details. Without such details, there is no basis for determining the extent of control or influence petitioners actually
had over the employment of respondent's husband as to make said threat plausible. Therefore, it could not be said
that respondent's consent to execute the resignation letter was vitiated by coercion or intimidation. Pertinent herein
are the findings made by the Court in Gan v. Galderma Philippines, Inc.33 that:

Angeles| Bajana | Balladares | Brillantes | Briones | Cabansag | Callanta | Chua | David|


De Leon | Gomez | Lopez | Macalino | Nostratis | Padilla | Reynon | Santos | Tan |Velasco
4E / 4F - 2018-2019
Page 788 of 920
LABOR REVIEW DIGEST
Atty. Joyrich Golangco

Gari could not have been coerced. Coercion exists when there is a reasonable or well-grounded fear of an imminent
evil upon a person or his property or upon the person or property of his spouse, descendants or ascendants. Neither
do the facts of this case disclose that Gan was intimidated. In St. Michael Academy v. NLRC, we enumerated the
requisites for intimidation to vitiate one's consent, thus:
chanRoblesvirtualLawlibrary
x x x (1) that the intimidation caused the consent to be given; (2) that the threatened act be unjust or unlawful; (3) that
the threat be real or serious, there being evident disproportion between the evil and the resistance which all men can
offer, leading to the choice of doing the act which is forced on the person to do as the lesser evil; and (4) that it
produces a well-grounded fear from the fact that the person from whom it comes has the necessary means or ability
to inflict the threatened injury to his person or property x x x.
The instances of "harassment" alleged by Gan are more apparent than real. Aside from the need to treat his
accusations with caution for being self-serving due to lack of substantial documentary or testimonial evidence to
corroborate the same, the acts of "harassment," if true, do not suffice to be considered as "peculiar circumstances"
material to the execution of the subject resignation letter. (Citations omitted.)

It is inconsequential that the contents of respondent's resignation letter was dictated by petitioner Dela Rama and,
per the Labor Arbiter's observation, reads more of a quitclaim rather than a resignation letter, for as long as
respondent wrote down and signed said letter by her own volition. In Samaniego v. National Labor Relations
Commission,34 the Court accorded weight to the resignation letters of the employees because although said letters
were prepared by the company, the employees signed the same voluntarily. Granted that the employees
in Samaniego were managerial employees, while respondent in the present case was a rank and file employee, the
financial situation of petitioner BEMI, the need for retrenchment, and the option to voluntarily resign and the financial
package which respondent could avail herself of were duly explained to respondent during the meeting on February
20, 2006; and respondent's resignation letter was in Filipino, using simple terms which could be easily understood.

Angeles| Bajana | Balladares | Brillantes | Briones | Cabansag | Callanta | Chua | David|


De Leon | Gomez | Lopez | Macalino | Nostratis | Padilla | Reynon | Santos | Tan |Velasco
4E / 4F - 2018-2019
Page 789 of 920
LABOR REVIEW DIGEST
Atty. Joyrich Golangco

10. PNCC SKYWAY vs SECRETARY OF LABOR


GR No: 213299 Date: April 19, 2016
Ponente: Perlas-Bernabe, J.

Doctrine:
PSC complied with the mandated thirty (30)-day notice requirement. Although PSC informed its
employees that it would be turning over its operations to SOMCO not earlier than December 31, 2007, they
were duly notified that the effective date of their termination was set on January 31, 2008. In light of valid
business reasons, i.e., the transfer of operations to SOMCO pursuant to the ASTOA, PSC asked its
employees not to report for work beginning December 31, 2007 but were still retained on payroll until
January 31, 2008. Evidently, their employment with PSC did not cease by the sole reason that they were told
not to render any service. In addition, since the employees were not reporting for work although retained on
payroll, they had, in fact, more free time to look for job opportunities elsewhere after December 31, 2007 up
until January 31, 2008.

FACTS:
In October 1977, the Republic of the Philippines, through the TRB and the PNCC entered into a Toll
Operation Agreement (TOA) for the latter's operation and maintenance of the South Metro Manila Skyway (Skyway).
On November 27, 1995, a Supplemental TOA (STOA) was executed by the TRB, PNCC, and Citra Metro Manila
Tollways Corporation (CITRA), whereby CITRA, as an incoming investor, agreed, under a build-and-transfer
scheme, to finance, design, and construct the Skyway. However, PNCC retained the right to operate and maintain
the toll facilities, and for such purpose, undertook to incorporate a subsidiary company that would assume its rights
and obligations under the STOA. Thus, on December 15, 1998, PSC was incorporated as a subsidiary of PNCC to
operate the Skyway on PNCC's behalf. As such, it was tasked to maintain the toll facilities, ensure traffic safety, and
collect toll fees at the Skyway.
On July 18, 2007, the TRB, PNCC, and CITRA entered into an Amended STOA (ASTOA). Under the
ASTOA, the operation and management of the Skyway would be transferred from PSC to a new Replacement
Operator, which turned out to be the Skyway O & M Corporation (SOMCO). A transition period of 5 1/2 months was
provided commencing on the date of signing of the ASTOA until December 31, 2007, during which period, PSC
continued to operate the Skyway.
In line with the above-mentioned transfer, PSC, on December 28, 2007, issued termination letters to its
employees and filed a notice of closure with the DOLE - National Capital Region, advising them that it shall cease to
operate and maintain the Skyway, and that the services of the employees would be consequently terminated
effective January 31, 2008. In this regard, PSC offered its employees a separation package consisting of 250% of
their basic monthly salary for every year of service, gratuity pay of P40,000.00 each, together with all other remaining
benefits such as 13th month pay, rice subsidy, cash conversion of leave credits, and medical reimbursement.
On the same date, the PSC Employees Union (PSCEU) filed a Notice of Strike on the ground of unfair labor
practice resulting in union busting and dismissal of workers. On December 31, 2007, the DOLE Secretary intervened
and assumed jurisdiction over the labor incident.

SOLE RULING:
Dismissed the charges of unfair labor practice and union busting, as well as the counter-charges of illegal strike, but
ordered PSC to pay its terminated employees P30,000.00 each as indemnity after finding that the notices of their
dismissal were invalid.

The DOLE Secretary held that while there was a valid and sufficient legal basis for PSC's closure - as it was a mere
consequence of the termination of its contract to operate and maintain the Skyway in view of the amendment of the
STOA - PSC, nonetheless, foiled to comply with the thirty (30)-day procedural notice requirement in terminating its
employees, as provided under Article 283 (now, Article 298) of the Labor Code.

CA RULING:
Affirmed SOLE

ISSUE/S:
Whether or not PSC properly complied with the thirty (30)-day prior notice rule, which is the first prong of the
termination procedure under Article 298 (formerly Article 283) of the Labor Code

HELD: YES
As admitted by both parties, the PSC employees and the DOLE were notified on December 28, 2007 that
PSC intended to cease operations on January 31, 2008. The PSC employees and the DOLE were, therefore,

Angeles| Bajana | Balladares | Brillantes | Briones | Cabansag | Callanta | Chua | David|


De Leon | Gomez | Lopez | Macalino | Nostratis | Padilla | Reynon | Santos | Tan |Velasco
4E / 4F - 2018-2019
Page 790 of 920
LABOR REVIEW DIGEST
Atty. Joyrich Golangco

notified 34 days ahead of the impending closure of PSC. Clearly, the mere fact that PSC turned over the operation
and management of the Skyway to SOMCO and ceased business operations on December 31, 2007, should not be
taken to mean that the FSC employees were ipso facto terminated on the same date. The employees were
notified that despite the cessation of its operations on December 31, 2007 - which, as a consequence thereof, would
result in the needlessness of their services - the effective date of their termination from employment would be
on January 31, 2008.
That the effectivity of the PSC employees' termination is on January 31, 2008, and not on December 31,
2007, is lucidly evinced by the unrefuted fact that they were still paid their salaries and benefits for the whole month
of January 2008. Surely, it would go against the stream of practical business logic to retain employees on payroll a
month after they had already been terminated.
On top of that, it deserves mentioning that PSC undisputedly paid its dismissed employees separation pay
in amounts more than that required by law. As the records show, PSC's separation package to its employees was a
generous one consisting of no less than 250% of the basic monthly pay per year of service, a gratuity pay of
P40,000.00, rice subsidy, cash conversion of vacation and sick leaves and medical reimbursement. On the other
hand, the legally-mandated rate for separation pay provided under Article 298 (formerly, Article 283) of the Labor
Code, as amended, in cases such as the present, is equivalent to "one (1) month pay or at least one-half (1/2) month
pay for every year of service, whichever is higher."
Ultimately, it was within PSC's prerogative and discretion as employer to retain the services of its employees
for one month after the turnover date to SOMCO and to continue paying their salaries and benefits corresponding to
that period even when there is no more work to be done, if only "to ensure a smooth transition and gradual phasing in
of the new operator, which had yet to familiarize itself with the business."
PSC complied with the mandated thirty (30)-day notice requirement. Although PSC informed its employees
that it would be turning over its operations to SOMCO not earlier than December 31, 2007, they were duly notified
that the effective date of their termination was set on January 31, 2008. In light of valid business reasons, i.e., the
transfer of operations to SOMCO pursuant to the ASTOA, PSC asked its employees not to report for work beginning
December 31, 2007 but were still retained on payroll until January 31, 2008. Evidently, their employment with PSC
did not cease by the sole reason that they were told not to render any service.
In addition, since the employees were not reporting for work although retained on payroll, they had, in fact,
more free time to look for job opportunities elsewhere after December 31, 2007 up until January 31, 2008.
All told, considering that PSC had complied with Article 298 (formerly, Article 283) of the Labor Code, as
amended, the indemnity award in favor of the terminated employees was grossly improper and must therefore be
nullified, in this respect, the DOLE Secretary gravely abused its discretion and the CA erred in ruling otherwise.
When, a lower court or tribunal patently violates the Constitution, the law, or existing jurisprudence, grave abuse of
discretion is committed, as in this case.

Angeles| Bajana | Balladares | Brillantes | Briones | Cabansag | Callanta | Chua | David|


De Leon | Gomez | Lopez | Macalino | Nostratis | Padilla | Reynon | Santos | Tan |Velasco
4E / 4F - 2018-2019
Page 791 of 920
LABOR REVIEW DIGEST
Atty. Joyrich Golangco

11. GJT Rebuilders Machine Shop v. Ambos


G.R. No. 174184, January 28, 2015
Leonen J.

DOCTRINE:
To prove serious business losses, employers must present in evidence financial statements showing the net
losses suffered by the business within a sufficient period of time. Generally, it cannot be based on a single
financial statement showing losses. Absent this proof, employers closing their businesses must pay the
dismissed employees separation pay equivalent to one-month pay or to at least one-half-month pay for every
year of service, whichever is higher.
FACTS:
G.J.T. Rebuilders is a single proprietorship owned by the Trillana spouses. Ricardo Ambos (Ricardo), Russell Ambos
(Russell), and Benjamin Putian (Benjamin) worked as machinists. G.J.T. Rebuilders occupied a space in the FEA
Building in Shaw Boulevard, Mandaluyong City. On September 8, 1996, a fire partially destroyed the FEA Building.
Due to the damage sustained by the building, its owner notified its tenants to vacate their rented units by the end of
September 1996.
Despite the notice to vacate, G.J.T. Rebuilders continued its business in the condemned building. When the building
owner finally refused to accommodate it, G.J.T. Rebuilders left its rented space and closed the machine shop on
December 15, 1997. It then filed an Affidavit of Closure before the Department of Labor and Employment.
Ricardo, Russell, and Benjamin filed a Complaint for illegal dismissal before the Labor Arbiter.
LA/NLRC/CA Ruling:
Labor Arbiter Leda found no convincing proof of G.J.T. Rebuilders’ alleged serious business losses. Thus, Ricardo,
Russell, and Benjamin were entitled to separation pay under Article 283 of the Labor Code.
The National Labor Relations Commission found G.J.T. Rebuilders to have suffered serious business losses.
The Court of Appeals reversed the National Labor Relations Commission’s Decision, agreeing with Labor Arbiter
Leda that G.J.T. Rebuilders failed to prove its alleged serious business losses.

ISSUES:
Whether petitioners sufficiently proved that G.J.T. Rebuilders suffered from serious business losses.
HELD:

1. The decision to close one’s business is a management prerogative that courts cannot interfere
with.35 Employers can "lawfully close shop at anytime," even for reasons of their own. "Just as no law
forces anyone to go into business, no law can compel anybody to continue in it."

However, despite this management prerogative, employers closing their businesses must pay the affected
workers separation pay equivalent to one-month pay or to at least one-half-month pay for every year of
service, whichever is higher. The reason is that an employee dismissed, even for an authorized cause,
loses his or her means of livelihood.

2. The only time employers are not compelled to pay separation pay is when they closed their establishments
or undertaking due to serious business losses or financial reverses.

Serious business losses are substantial losses, not de minimis. "Losses" means that the business must
have operated at a loss for a period of time for the employer "to [have] perceived objectively and in good
faith" that the business’ financial standing is unlikely to improve in the future.

The burden of proving serious business losses is with the employer. The employer must show losses on the
basis of financial statements covering a sufficient period of time. The period covered must be sufficient for
the National Labor Relations Commission and this court to appreciate the nature and vagaries of the
business.

Aside from the obligation to pay separation pay, employers must comply with the notice requirement under
Article 283 of the Labor Code. Employers must serve a written notice on the affected employees and on the
Department of Labor and Employment at least one month before the intended date of closure. Failure to
comply with this requirement renders the employer liable for nominal damages.

3. We uphold G.J.T. Rebuilders’ decision to close its establishment as a valid exercise of its management
prerogative. G.J.T. Rebuilders closed its machine shop, believing that its "former customers . . . seriously

Angeles| Bajana | Balladares | Brillantes | Briones | Cabansag | Callanta | Chua | David|


De Leon | Gomez | Lopez | Macalino | Nostratis | Padilla | Reynon | Santos | Tan |Velasco
4E / 4F - 2018-2019
Page 792 of 920
LABOR REVIEW DIGEST
Atty. Joyrich Golangco

doubted [its] capacity . . . to perform the same quality [of service]" after the fire had partially damaged the
building where it was renting space.

Nevertheless, we find that G.J.T. Rebuilders failed to sufficiently prove its alleged serious business losses.

The financial statement G.J.T. Rebuilders submitted in evidence covers the fiscal years 1996 and 1997.
Based on the financial statement, G.J.T. Rebuilders earned a net income of 61,157.00 in 1996 and incurred
a net loss of 316,210.00 in 1997.

We find the two-year period covered by the financial statement insufficient for G.J.T. Rebuilders to have
objectively perceived that the business would not recover from the loss. In fact, in one of the two fiscal years
covered by the financial statement presented in evidence, G.J.T. Rebuilders earned a net income. We,
therefore, agree with the Labor Arbiter and the Court of Appeals that G.J.T. Rebuilders closed its machine
shop to prevent losses, not because of serious business losses.

Considering that G.J.T. Rebuilders failed to prove its alleged serious business losses, it must pay
respondents their separation pay equivalent to one-month pay or at least one-half-month pay for every year
of service, whichever is higher. In computing the period of service, a fraction of at least six months is
considered a year.

4. In addition to separation pay, G.J.T. Rebuilders must pay each of the respondents nominal damages for
failure to comply with the notice requirement under Article 283 of the Labor Code.

Notice of the eventual closure of establishment is a "personal right of the employee to be personally
informed of his [or her] proposed dismissal as well as the reasons therefor."69 The reason for this
requirement is to "give the employee some time to prepare for the eventual loss of his [or her] job."

The requirement "is not a mere technicality or formality which the employer may dispense with." Should
employers fail to properly notify their employees, they shall be liable for nominal damages even if they
validly closed their businesses.

Angeles| Bajana | Balladares | Brillantes | Briones | Cabansag | Callanta | Chua | David|


De Leon | Gomez | Lopez | Macalino | Nostratis | Padilla | Reynon | Santos | Tan |Velasco
4E / 4F - 2018-2019
Page 793 of 920
LABOR REVIEW DIGEST
Atty. Joyrich Golangco

12. FASAP vs. PAL


G.R. No. 178083, March 13, 2018
BERSAMIN, J.:

DOCTRINE: In determining the validity of a retrenchment, judicial notice may be taken of the financial losses
incurred by an employer undergoing corporate rehabilitation. In such a case, the presentation of audited
financial statements may not be necessary to establish that the employer is suffering from severe financial
losses.

FACTS:

Resolving the appeal of F ASAP, the Third Division of the Court promulgated its decision on July 22, 2008 reversing
the decision promulgated on August 23, 2006 by the Court of Appeals (CA) and entering a new one finding PAL
guilty of unlawful retrenchment. The Special Third Division was unconvinced by PAL’s change of theory in urging the
June 1998 Association of Airline Pilots of the Philippines (ALP AP) pilots' strike as the reason behind the immediate
retrenchment; and observed that the strike was a temporary occurrence that did not require the immediate and
sweeping retrenchment of around 1,400 cabin crew.

PAL insists that FASAP, while admitting PAL’s serious financial condition, only questioned before the Labor Arbiter
the alleged unfair and unreasonable measures in retrenching the employees; that FASAP categorically manifested
before the NLRC, the CA and this Court that PAL’s financial situation was not the issue but rather the manner of
terminating the 1,400 cabin crew; that the Court's disregard of FASAP's categorical admissions was contrary to the
dictates of fair play; that considering that the Labor Arbiter, the NLRC and the CA unanimously found PAL to have
experienced financial losses, the Court should have accorded such unanimous findings with respect and finality; that
its being placed under suspension of payments and corporate rehabilitation and receivership already sufficiently
indicated its grave financial condition; and that the Court should have also taken judicial notice of the suspension of
payments and monetary claims filed against PAL that had reached and had been consequently resolved by the
Court.

ISSUE:
Whether or not there was a valid retrenchment.

RULING:

Yes. Accordingly, the employer may resort to retrenchment in order to avert serious business losses. The burden of
proving serious financial losses rests upon the employer. However, in this case, FASAP’s express recognition of
PAL’s grave financial situation meant that such situation no longer needed to be proved, the same having become a
judicial admission in the context of the issues between the parties. By FASAP’s admission of PAL’s severe financial
woes, PAL was relieved of its burden to prove its dire financial condition to justify the retrenchment.

Furthermore, the July 22, 2008 decision recognized that PAL underwent corporate rehabilitation. Indeed, that a
company undergoes rehabilitation sufficiently indicates its fragile financial condition. After having been placed under
corporate rehabilitation and its rehabilitation plan having been approved by the SEC on June 23, 2008, PAL’s dire
financial predicament could not be doubted. Incidentally, the SEC’s order of approval came a week after PAL had
sent out notices of termination to the affected employees. It is thus difficult to ignore the fact that PAL had then been
experiencing difficulty in meeting its financial obligations long before its rehabilitation.

Also, PAL retrenched in good faith. Given PAL’s dire financial predicament, it becomes understandable that PAL was
constrained to finally implement the retrenchment program when the ALPAP pilots strike crippled a major part of
PAL’s operations. PAL also used fair and reasonable criteria in selecting the
employees to be retrenched pursuant to the CBA. PAL resorted to both efficiency rating and inverse seniority in
selecting the employees to be subject of termination.

Angeles| Bajana | Balladares | Brillantes | Briones | Cabansag | Callanta | Chua | David|


De Leon | Gomez | Lopez | Macalino | Nostratis | Padilla | Reynon | Santos | Tan |Velasco
4E / 4F - 2018-2019
Page 794 of 920
LABOR REVIEW DIGEST
Atty. Joyrich Golangco

13.LA CONSOLACION COLLEGE OF MANILA, SR., ET AL. VS. VIRGINIA PASCUA, M.D.
GR NO: 214744 DATE: MARCH 14, 2018

Doctrine: Retrenchment is normally resorted to by management during periods of business reverses and
economic difficulties occasioned by such events as recession, industrial depression, or seasonal
fluctuations. It is an act of the employer of reducing the work force because of losses in the operation of the
enterprise, lack of work, or considerable reduction on the volume of business. Retrenchment is, in many
ways, a measure of last resort when other less drastic means have been tried and found to be inadequate. A
lull caused by lack of orders or shortage of materials must be of such nature as would severely affect the
continued business operations of the employer to the detriment of all and sundry if not properly addressed.

FACTS:
La Consolacion (LC) engaged the services of Dr. Virginia Pascua (Dr. Pascua) as school physician. She started
working part-time before serving full-time from 2008.
Dr. Pascua was handed an Inter-Office Memo from HR of La Consolacion inviting her to a meeting concerning her
“working condition.” In that meeting she was handed a termination letter, explaining the reasons thereof and the
terms of her dismissal, including payment of separation pay.
The letter states as reason for termination:
- Current financial situation of LC caused by decease in enrollment; Hence, downsize health services staff
- Position of Dr. Pascua rendering 35 hours work in a week was eliminated
- Last resort to prevent serious business losses
Dr. Pascua pointed out that the part-time school physician should have been considered first for dismissal. LC could
have asked her to revert to part-time status instead. She also asked:
- Criteria for retrenchment selection
- Why her over a part-time physician
- She is the only one terminated
- What were the other cost-cutting measures done by school to abate its alleged losses than implementing
the drastic measure of termination
In the meantime, Dr. Pascua underwent LC’s clearance procedures and completed them. In the exit clearance, Dr.
Pascua reserved her right to question the validity / legality of her termination.
Dr. Pascua filed a complaint for illegal dismissal.

LA RULING: The Labor Arbiter (LA) held that Dr. Pascua was illegally terminated. LC failed to justify the criteria used
in terminating her employment.

NLRC RULING:
The National Labor Relations Commission (NLRC) reversed the LA Decision.
It explained the validity of the basis for dismissal that substantial drop in income justified the reduction of the larger of
the college expenses such as salaries and allowances of its more expensive staff members.

CA RULING: Reinstated LA decision.

PETITION TO THE SC:

ISSUE/S:
Whether or not by terminating a full-time physician instead of the part-time physician a criterion on seniority was
observed to make retrenchment valid.
Whether or not the reason to dismiss the highest paid employee first prevails over the fact that the employee
terminated was more senior and demonstrating exemplary performance

HELD:
The SC held further that LC’s failure was non-compliance with third substantive requisite of using fair and reasonable
criteria that considered the status and seniority of the retrenched employee.
According to SC there is no dispute about Dr. Pascua’s seniority and preferred status. Thus, her preeminence is a
necessary implication of the very criteria used by LC in retrenching her, i.e., that she was the highest paid employee
in health services division.
LC’s disregard of Dr. Pascua’s seniority and preferred status relative to a part-time employee indicates its resort to an
unfair and unreasonable criterion for retrenchment.
The SC stated that it may have made mathematical sense to dismiss the highest paid employee first. However,
appraising the propriety of retrenchment is not merely a matter of enabling an employer to augment financial

Angeles| Bajana | Balladares | Brillantes | Briones | Cabansag | Callanta | Chua | David|


De Leon | Gomez | Lopez | Macalino | Nostratis | Padilla | Reynon | Santos | Tan |Velasco
4E / 4F - 2018-2019
Page 795 of 920
LABOR REVIEW DIGEST
Atty. Joyrich Golangco

prospects. It is as much a matter of giving employees their just due. Employees who have earned their keep by
demonstrating exemplary performance and securing roles in their respective organizations cannot be summarily
disregarded by nakedly pecuniary considerations.

Angeles| Bajana | Balladares | Brillantes | Briones | Cabansag | Callanta | Chua | David|


De Leon | Gomez | Lopez | Macalino | Nostratis | Padilla | Reynon | Santos | Tan |Velasco
4E / 4F - 2018-2019
Page 796 of 920
LABOR REVIEW DIGEST
Atty. Joyrich Golangco

14. VETERANS FEDERATION OF THE PHILIPPINES V. MONTENEJO (2017 CASE)


Doctrine: As stated in the provision, an employer’s closure or cessation of business or operations is
regarded as an invalid ground for the termination of employment only when the closure or cessation is made
for the purpose of circumventing the tenurial rights of the employees.

Facts: VFP entered into a management agreement5 with VMDC. Under the said agreement, VMDC was to assume
exclusive management and operation of the VFPIA in exchange for forty percent (40%) of the lease rentals
generated from the area.

In managing and operating the VFPIA, VMDC hired its own personnel and employees. Among those hired by VMDC
were respondents Eduardo L. Montenejo, Mylene M. Bonifacio, Evangeline E. Valverde and Deana N. Pagal
(hereafter collectively referred to as “Montenejo, et al.”).6

The management agreement between VFP and VMDC had a term of five (5) years, or up to 4 January 1996, and is
renewable for another five (5) years.7 Subsequently, both parties acceded to extend the agreement up to 1998.8 After
1998, the agreement was again extended by VFP and VMDC albeit only on a month-to-month basis.

Then, in November 1999, the VFP board passed a resolution terminating the management agreement effective
December 31, 1999.9 VMDC conceded to the termination and eventually agreed to turn over to VFP the possession
of all buildings, equipment and other properties necessary to the operation of the VFPIA.

On January 3, 2000, the President of VMDC11 issued a memorandum12 informing the company’s employees of the
termination of their services effective at the close of office hours on January 31, 2000 “[i]n view of the termination of
the [management agreement].” True to the memorandum’s words, on January 31, 2000, VMDC dismissed all of its
employees and paid each his or her separation pay.

The Illegal Dismissal Complaint

Contending in the main that their dismissals had been effected without cause and observance of due process,
Montenejo, et al. filed before the Labor Arbiter (LA) a complaint for illegal dismissal, 13 money claims and damages.
They impleaded both VMDC and VFP as defendants in the complaint.
VMDC, for its part, denied the contention. It argued that the dismissals of Montenejo, et al. were valid as they were
due to an authorized cause-the cessation or closure of its business. VMDC claimed that the cessation of its
operations was but the necessary consequence of the termination of such agreement.
VFP, on the other hand, seconded the arguments of VMDC. In addition, however, VFP asserted that it could not, at
any rate, be held liable under the complaint because it is not the employer of Montenejo, et al.

Issue: Whether or not Montenejo et al illegally dismissed

Ruling: The first qualm of VFP is justified. The NLRC and the CA erred in ruling that Montenejo, et al. were illegally
dismissed.

Montenejo, et al. were dismissed as a result of the closure of VMDC. Contrary to the ruling of the NLRC and the CA,
there is ample support from the records to establish that VMDC did, in fact, close its operations. VMDC’s closure,
more importantly, qualifies as a bona fide cessation of operations or business as contemplated under Article 298 of
the Labor Code.25

The dismissals of Montenejo, et al. were, therefore, premised on an authorized cause. Being so, such dismissals are
valid and remain to be valid even though they suffer from a procedural defect. Consequently, Montenejo, et al. are
not entitled to the monetary awards (i.e., full backwages, separation pay in lieu of reinstatement, 13th month pay,
SILP and COLA) granted to them by the NLRC, but only to nominal damages on top of the separation pay under
Article 298 of the Labor Code.

Concept of Illegal Dismissal; Closure of Business as an Authorized Cause for the Termination of
Employment

In our jurisdiction, the right of an employer to terminate employment is regulated by law. Both the Constitution 26 and
our laws guarantee security of tenure to labor and, thus, an employee can only be validly dismissed from work if the
dismissal is predicated upon any of the just or authorized causes allowed under the Labor Code.27 Correspondingly,

Angeles| Bajana | Balladares | Brillantes | Briones | Cabansag | Callanta | Chua | David|


De Leon | Gomez | Lopez | Macalino | Nostratis | Padilla | Reynon | Santos | Tan |Velasco
4E / 4F - 2018-2019
Page 797 of 920
LABOR REVIEW DIGEST
Atty. Joyrich Golangco

a dismissal that is not based on either of the said causes is regarded as illegal and entitles the dismissed employee
to the payment of backwages and, in most cases, to reinstatement.28

One of the authorized causes for dismissal recognized under the Labor Code is the bona fide cessation of business
or operations by the employer. Article 298 of the Labor Code explicitly sanctions terminations due to the employer’s
cessation of business or operations-as long as the cessation is bona fide or is not made “for the purpose of
circumventing the [employees’ right to security of tenure]”-.

As stated in the provision, an employer’s closure or cessation of business or operations is regarded as an invalid
ground for the termination of employment only when the closure or cessation is made for the purpose of
circumventing the tenurial rights of the

To unmask the true intent of an employer when effecting a closure of business, it is important to consider not only the
measures adopted by the employer prior to the purported closure but also the actions taken by the latter after the
fact. For, as can be seen from the examples in the cited cases, the employer’s subsequent acts of suddenly reviving
a business it had just closed or surreptititiously continuing with its operation after announcing a shutdown are telltale
badges that the employer had no real intent to cease its business or operations and only seeks an excuse to
terminate employees capriciously.
Guided by the foregoing, we shall now address the issue at hand.

VMDC’s Closure Was Established; The Closure Is Bona Fide; The Dismissals of Montenejo, et al Are Based
on an Authorized Cause

In this case, the NLRC and the CA both ruled against the validity of the dismissals of Montenejo, et al. for the reason
that the dismissals were not proven to be based on any valid cause. The NLRC and the CA were disapproving of the
claim that the dismissals were due to the closure of VMDC, lamenting the lack of any evidence showing that VMDC
had formally closed its business.

We disagree.

Though not proclaimed in any formal document, the closure of VMDC was still duly proven in this case. The closure
can be inferred from other facts that were established by the records and/or were not refuted by the parties. These
facts are:

1. The fact that VMDC, on January 3, 2000, had turned over possession of all buildings, equipment and other
properties necessary to the operation of the VFPIA to VFP;33 and

2. The fact that, on January 31, 2000, VMDC had dismissed all of its officials and employees, which included
Montenejo, et al.34

The confluence of the above facts, to our mind, indicates that VMDC indeed closed shop or ceased operations
following the termination of its management agreement with VFP. The acts of VMDC in relinquishing all
properties required for its operations and in dismissing its entire workforce would have indubitably compromised its
ability to continue on with its operations and are, thus, the practical equivalents of a business closure. Hence, in
these regards, we hold that the closure of VMDC had been established.

Moreover, we find VMDC’s cessation of operations to be bona fide. None of the telltale badges of bad faith in
closures of business, as illustrated in our jurisprudence, was shown to be present in this case. Here, there is no
evidence on record that shows that VMDC-after dismissing its entire workforce and ceasing to operate-had revived
its business or had hired new employees to replace those dismissed.

Thus, it cannot be reasonably said that VMDC’s cessation of operations was just a ruse or had been implemented
merely as an excuse to terminate its employees.

The mere fact that VMDC could have chosen to continue operating despite the termination of its management
agreement with VFP is also of no consequence. The decision of VMDC to cease its operations after the termination
of the management agreement is, under the law, a lawful exercise by the company’s leadership of its management
prerogative that must perforce be upheld where, as in this case, there is an absence of showing that the cessation
was made for prohibited purposes.35 As Alabang Country Club, Inc. v. NLRC reminds:36

Angeles| Bajana | Balladares | Brillantes | Briones | Cabansag | Callanta | Chua | David|


De Leon | Gomez | Lopez | Macalino | Nostratis | Padilla | Reynon | Santos | Tan |Velasco
4E / 4F - 2018-2019
Page 798 of 920
LABOR REVIEW DIGEST
Atty. Joyrich Golangco

For any bona fide reason, an employer can lawfully close shop anytime. Just as no law forces anyone to go into
business, no law can compel anybody to continue the same. It would be stretching the intent and spirit of the law if a
court interferes with management’s prerogative to close or cease its business operations just because the business
is not suffering from any loss or because of the desire to provide the workers continued employment.

The validity of the closure of VMDC necessarily validates the dismissals of Montenejo, et al. that resulted therefrom.
The dismissals cannot be regarded as illegal because they were predicated upon an authorized cause recognized by
law.

Montenejo, et al. Are Not Entitled to Monetary Awards Adjudged in Their Favor by the NLRC; They Are Only
Entitled to Separation Pay Under Article 298 of the Labor Code

Since Montenejo, et al. had been validly dismissed, it becomes apparent that the monetary awards granted to them
by the NLRC, and affirmed by the CA, were not proper. We substantiate:

1. The awards for full backwages and separation pay in lieu of reinstatement cannot be sustained as these awards
are reserved by law, and jurisprudence, for employees who were illegally dismissed.37

2. The awards for 13th month pay, SILP and COLA, on the other hand, must also be invalidated as these are mere
components of the award for backwages and were, thus, made by the NLRC and the CA in consideration of the
illegality of the dismissals of Montenejo, et al. The 13th month pay, SILP and COLA that were awarded by the
NLRC and the CA refer to the benefits that Montenejo, et al. would be entitled to had they not been illegally
dismissed and are computed from the time of their dismissals up to the time the judgment declaring their dismissals
illegal becomes final.38 The awards, in other words, were not due to any failure on the part of VMDC to pay
13th month pay, SILP and COLA to Montenejo, et al. during the subsistence of their employer-employee relationship.

For having been terminated by reason of the employer’s closure of operations that was not due to serious business
losses or financial reverses, Montenejo, et al. are, however, entitled to be paid separation pay pursuant to Article 298
of the Labor Code. The records in this regard, though, reveal that Montenejo, et al. have already received their
respective separation pays from VMDC.39

Failure of VMDC to File a Notice of Closure with the DOLE Does Not Invalidate the Dismissals of Montenejo,
et al.; Such Procedural Lapse Only Gives Rise to Liability for Nominal Damages
Anent the failure of VMDC to file a notice of closure with the DOLE, we find our rulings in Agabon v.
NLRC40 and Jaka Food Processing Corporation v. Pacot41 to be apt.

To recall, Agabon laid out the rule that when a dismissal is based on a just cause but is implemented without
observance of the statutory notice requirements, the dismissal should be upheld as valid but the employer must
thereby pay an indemnity to the employee in the amount of P30,000. Jaka, on the other hand, expounded
on Agabonin two (2) ways:

1. First, Jaka extended the application of the Agabon doctrine to dismissals that were based on authorizedcauses but
have been effected without observance of the notice requirements. Thus, similar to Agabon, the dismissals under
such circumstances will also be regarded as valid while the employer shall likewise be required to pay an indemnity
to the employee; and

2. Second, Jaka increased the amount of indemnity payable by the employer in cases where the dismissals are
based on authorized causes but have been effected without observance of the notice requirements. It fixed the
amount of indemnity in the mentioned scenario to P50,000.

Verily, the failure of VMDC to file a notice of closure with the DOLE does not render the dismissals of Montenejo, et
al., which were based on an authorized cause, illegal. Following Agabon and Jaka, such failure only entitles
Montenejo, et al. to recover nominal damages from VMDC in the amount of P50,000 each, on top of the separation
pay they already received.

II

The NLRC and the CA also erred in ruling that VFP may be held solidarily liable with VMDC for any monetary award
that may be found due to Montenejo, et al. We find that, contrary to the holding of the NLRC and the CA, the
application of the doctrine of piercing the veil of corporate fiction is not justified by the facts of this case.

Angeles| Bajana | Balladares | Brillantes | Briones | Cabansag | Callanta | Chua | David|


De Leon | Gomez | Lopez | Macalino | Nostratis | Padilla | Reynon | Santos | Tan |Velasco
4E / 4F - 2018-2019
Page 799 of 920
LABOR REVIEW DIGEST
Atty. Joyrich Golangco

Accordingly, the liability for the award of nominal damages-the only award that Montenejo, et al. are entitled to in this
case-ought to rest exclusively upon their employer, VMDC.

Doctrine of Piercing the Veil of Corporate Fiction Does Not Apply to This Case

The NLRC and the CA’s stance is based on their submission that the doctrine of piercing the veil of corporate fiction
is applicable to this case, i.e., that VFP and VMDC could, for purposes of satisfying any monetary award that may be
due to Montenejo, et al., be treated as one and the same entity. According to the two tribunals, the doctrine may be
applied to this case because VFP apparently owns almost all of the shares of stock of VMDC. In this regard, both the
NLRC and the CA cite the Closing Agreement42 of VFP and VMDC which states that:

NOW THEREFORE, for and in consideration of the foregoing premises the [VFP] and the [VMDC] hereby agree to
terminate the [management agreement] for the development and management of the [VFPIA] in Taguig effective on 3
January 2000, subject to the following conditions:

1. The [VMDC] agrees that the [VFP] is the majority stockholder of the [VMDC] and that all its original
incorporators have endorsed all their shares of stock to the [VFP] except one (1) qualifying share each to be
able to sit as Director in the Board of Directors of the [VMDC]. (Emphasis supplied)

We disagree with the submission.

The doctrine of piercing the veil of corporate fiction is a legal precept that allows a corporation’s separate personality
to be disregarded under certain cirumstances, so that a corporation and its stockholders or members, or a
corporation and another related corporation could be treated as a single entity. The doctrine is an equitable principle,
it being meant to apply only in situations where the separate corporate personality of a corporation is being abused or
being used for wrongful purposes.43 As Manila Hotel Corporation v. NLRC44 explains:

Piercing the veil of corporate entity is an equitable remedy. It is resorted to when the corporate fiction is used to
defeat public convenience, justify wrong, protect fraud or defend a crime. It is done only when a corporation is a mere
alter ego or business conduit of a person or another corporation. (Citations omitted)
In Concept Builders, Inc. v. NLRC,45 we laid down the following test to determine when it would be proper to apply
the doctrine of piercing the veil of corporate fiction:

1. Control, not mere majority or complete stock control, but complete domination, not only of finances but of
policy and business practice in respect to the transaction attacked so that the corporate entity as to this transaction
had at the time no separate mind, will or existence of its own;

2. Such control must have been used by the defendant to commit fraud or wrong, to perpetuate the violation
of a statutory or other positive legal duty, or dishonest and unjust act in contravention of plaintiffs legal
rights; and

3. The aforesaid control and breach of duty must proximately cause the injury or unjust loss complained of.

The absence of any one of these elements prevents piercing the corporate veil. In applying the instrumentality
or alter ego doctrine, the courts are concerned with reality and not form, with how the corporation operated and the
individual defendant’s relationship to that operation. (Emphasis supplied and citations omitted).

Relative to the Concept Builders test are the following critical ruminations from Rufina Luy Lim v. CA:46
Mere ownership by a single stockholder or by another corporation of all or nearly all of the capital stock of a
corporation is not of itself a sufficient reason for disregarding the fiction of separate corporate personalities.
Moreover, to disregard the separate juridical personality of a corporation,the wrong-doing must be clearly and
convincingly established. It cannot be presumed. (Citations omitted)
Utilizing the foregoing standards, it becomes clear that the NLRC and the CA were mistaken in their application of the
doctrine to the case at bench. The sole circumstance used by both to justify their disregard of the separate
personalities of VFP and VMDC is the former’s alleged status as the majority stockholder of the latter. Completely
absent, however, both from the decisions of the NLRC and the CA as well as from the records of the instant case
itself, is any circumstance which establishes that VFP had complete control or domination over the
“finances[,]... policy and business practice” of VMDC. Worse, even assuming that VFP had such kind of control over
VMDC, there is likewise no evidence that the former had used the same to “commit fraud or wrong, to perpetuate the

Angeles| Bajana | Balladares | Brillantes | Briones | Cabansag | Callanta | Chua | David|


De Leon | Gomez | Lopez | Macalino | Nostratis | Padilla | Reynon | Santos | Tan |Velasco
4E / 4F - 2018-2019
Page 800 of 920
LABOR REVIEW DIGEST
Atty. Joyrich Golangco

violation of a statutory or other positive legal duty, or dishonest and unjust act in contravention of [another’s] legal
rights. ”
Given the absence of any convincing proof of misuse or abuse of the corporate shield, we, thus, find the application
of the doctrine of piercing the veil of corporate fiction to the present case to be unwarranted, if not utterly improper.
Consequently, we must also reject, for being erroneous, the pronouncement that VFP may be held solidarily liable
with VMDC for any monetary award that may be adjudged in favor of Montenejo, et al. in this case.

Angeles| Bajana | Balladares | Brillantes | Briones | Cabansag | Callanta | Chua | David|


De Leon | Gomez | Lopez | Macalino | Nostratis | Padilla | Reynon | Santos | Tan |Velasco
4E / 4F - 2018-2019
Page 801 of 920
LABOR REVIEW DIGEST
Atty. Joyrich Golangco

15. PNCC SKYWAY CORP. vs. SECRETARY OF LABOR and PNCC SKYWAY TRAFFIC MANAGEMENT, and
SECURITY DIVISION WORKERS ORGANIZATION
G.R. No. 196110 February 6, 2017
Peralta, J.

DOCTRINE: The required written notice under Article 283 of the Labor Code is to inform the employees of the
specific date of termination or closure of business operations, and must be served upon them at least one
(1) month before the date of effectivity to give them sufficient time to make the necessary arrangements. The
purpose of this requirement is to give employees time to prepare for the eventual loss of their jobs, as well
as to give DOLE the opportunity to ascertain the veracity of the alleged cause of termination.
FACTS: the Philippine National Construction Corporation (PNCC) was awarded by the Toll Regulatory
Board (TRB) with the franchise of constructing, operating and maintaining the north and south expressways,
including the South Metro Manila Skyway (Skyway). On December 15, 1998, it created petitioner PNCC Skyway
Corporation (PSC) for the purpose of taking charge of its traffic safety, maintaining its facilities and collecting toll.
Eight years later, or on July 18, 2007, the Citra Metro Manila Tollway Corporation (Citra), a private investor under a
build-and-transfer scheme, entered into an agreement with the TRB and the PNCC to transfer the operation of the
Skyway from petitioner PSC to the Skyway O & M Corporation (SOMCO). The said transfer provided for a five-month
transition period from July 2007 until the full turn-over of the Skyway at 10:00 p.m. of December 31, 2007 upon which
petitioner PSC will close its operation.
On December 28, 2007, or three (3) days before the full transfer of the operation of the Skyway to SOMCO, petitioner
PSC served termination letters to its employees, many of whom were members of private respondent PNCC Skyway
Traffic Management and Security Division Worker's Organization (Union). According to the letter, PSC has no choice
but to close its operations resulting in the termination of its employees effective January 31, 2008. However, the
employees are entitled to receive separation pay amounting to 250% of the basic monthly pay for every year of
service, among others things. Petitioner PSC, likewise, served a notice of termination to the Department of Labor and
Employment (DOLE)
Private respondent Union, immediately upon receipt of the termination letters, filed a Notice of Strike before the
DOLE alleging that the closure of the operation of PSC is tantamount to union-busting because it is a means of
terminating employees who are members thereof. Furthermore, the notices of termination were served on its
employees three (3) days before petitioner PSC ceases its operations, thereby violating the employees' right to due
process. Private respondent Union, thus, prayed that petitioner PSC be held guilty of unfair labor practice and illegal
dismissal. It, likewise, prayed for the reinstatement of all dismissed employees, along with the award of backwages,
moral and exemplary damages, and attorney's fees.
PSC denied that the closure of its operation was intended to remove employees who are members of private
respondent Union. Instead, it claimed that it was done in good faith and in the exercise of management prerogative,
considering that it was anchored on an agreement between the TRB, the PNCC and the private investor Citra. PSC
likewise denied that it had violated the right to due process of its employees, considering that the notices of
termination were served on December 28, 2007 while the termination was effective only on January 31, 2008. PSC
alleged that the Union was guilty of an illegal strike when it started a strike on the same day it filed a notice of strike
on December 28, 2007.
Public respondent Secretary of Labor and Employment (SOLE), in its assailed Decision, found that there was an
authorized cause for the closure of the operation of PSC albeit it failed to comply with the procedural requirements
set forth under Article 283 (now Art. 298) of the Labor Code.
PSC filed a Petition for Certiorari alleging grave abuse of discretion amounting to lack or excess of jurisdiction on the
part of the SOLE when it additionally directed payment of an additional ₱30,000.00 to PSC's former employees
pursuant to Article 283 of the Labor Code.
The Court of Appeals dismissed PSC's petition. The appellate court held that the Secretary of Labor was correct in
saying that the extension of the employee's employment in paper only and the payment of the employee's salaries for
said period cannot substitute for the PSC's failure to comply with the due process requirements. Thus, the SOLE
cannot be said to have acted capriciously or whimsically, in the exercise of his official duties.

ISSUE: Whether or not PSC violated Art. 283 (now Art. 298) of the Labor Code with regard to procedural
requirements stated therein.

HELD: YES.
Under Article 283 of the Labor Code, three requirements are necessary for a valid cessation of business operations:
(a) service of a written notice to the employees and to the DOLE at least one month before the intended date thereof;
(b) the cessation of business must be bona fide in character; and (c) payment to the employees of termination pay
amounting to one month pay or at least one-half month pay for every year of service, whichever is higher.

Angeles| Bajana | Balladares | Brillantes | Briones | Cabansag | Callanta | Chua | David|


De Leon | Gomez | Lopez | Macalino | Nostratis | Padilla | Reynon | Santos | Tan |Velasco
4E / 4F - 2018-2019
Page 802 of 920
LABOR REVIEW DIGEST
Atty. Joyrich Golangco

In the instant case, while both the SOLE and the appellate court found the closure of PSC's business operation to
be bona fide, the required notices were, however, served on the employees and the DOLE only three (3) days before
the closure of the company. PSC contends that it had substantially complied with the one (1) month notice
requirement since the termination of its employees was made effective only on January 31, 2008, or more than one
(1) month after it had given the notice of termination on December 28, 2007. It insists that they have in fact paid the
affected employees for the said period covered by the supposed one-month notice.
The required written notice under Article 283 of the Labor Code is to inform the employees of the specific date of
termination or closure of business operations, and must be served upon them at least one (1) month before the date
of effectivity to give them sufficient time to make the necessary arrangements. The purpose of this requirement is to
give employees time to prepare for the eventual loss of their jobs, as well as to give DOLE the opportunity to
ascertain the veracity of the alleged cause of termination. Thus, considering that the notices of termination were given
merely three (3) days before the cessation of the PSC's operation, it defeats the very purpose of the required notice
and the mandate of Article 283 of the Labor Code. Neither the payment of employees' salaries for the said one-
month period13 nor the employees' alleged actual knowledge of the ASTOA is sufficient to replace the formal and
written notice required by the law.

Angeles| Bajana | Balladares | Brillantes | Briones | Cabansag | Callanta | Chua | David|


De Leon | Gomez | Lopez | Macalino | Nostratis | Padilla | Reynon | Santos | Tan |Velasco
4E / 4F - 2018-2019
Page 803 of 920
LABOR REVIEW DIGEST
Atty. Joyrich Golangco

ARTICLE 299 - DISEASE AS GROUND FOR TERMINATION

1. SY, ET AL V. CA
G.R. NO. 142293. FEBRUARY 27, 2003

DOCTRINE: The employer is required to furnish an employee with two written notices before the latter is
dismissed: (1) the notice to apprise the employee of the particular acts or omissions for which his dismissal
is sought, which is the equivalent of a charge; and (2) the notice informing the employee of his dismissal, to
be issued after the employee has been given reasonable opportunity to answer and to be heard on his
defense.

Facts:
Sometime in 1958, private respondent Jaime Sahot[5] started working as a truck helper for petitioners
family-owned trucking business named Vicente Sy Trucking. In 1965, he became a truck driver of the same family
business, renamed T. Paulino Trucking Service, later 6Bs Trucking Corporation in 1985, and thereafter known as
SBT Trucking Corporation since 1994. Throughout all these changes in names and for 36 years, private respondent
continuously served the trucking business of petitioners.
In April 1994, Sahot was already 59 years old. He had been incurring absences as he was suffering from
various ailments. Particularly causing him pain was his left thigh, which greatly affected the performance of his task
as a driver. He inquired about his medical and retirement benefits with the Social Security System (SSS) on April 25,
1994, but discovered that his premium payments had not been remitted by his employer.
At the end of his week-long absence, Sahot applied for extension of his leave for the whole month of June,
1994. It was at this time when petitioners allegedly threatened to terminate his employment should he refuse to go
back to work.
At this point, Sahot found himself in a dilemma. He was facing dismissal if he refused to work, But he could
not retire on pension because petitioners never paid his correct SSS premiums. The fact remained he could no
longer work as his left thigh hurt abominably. Petitioners ended his dilemma. They carried out their threat and
dismissed him from work, effective June 30, 1994. He ended up sick, jobless and penniless.
Sahot filed with the NLRC NCR Arbitration Branch, a complaint for illegal dismissal.
NLRC NCR Arbitration Branch, through Labor Arbiter Ariel Cadiente Santos, ruled that there was no illegal
dismissal in Sahots case.
On appeal, the National Labor Relations Commission modified the judgment of the Labor Arbiter.
the appellate court affirmed with modification the judgment of the NLRC. It held that private respondent was
indeed an employee of petitioners since 1958.

ISSUE:
Whether or not there was valid dismissal.

HELD:
In termination cases, the burden is upon the employer to show by substantial evidence that the termination
was for lawful cause and validly made. Article 277(b) of the Labor Code puts the burden of proving that the dismissal
of an employee was for a valid or authorized cause on the employer, without distinction whether the employer admits
or does not admit the dismissal. For an employees dismissal to be valid, (a) the dismissal must be for a valid cause
and (b) the employee must be afforded due process.
Article 284 of the Labor Code authorizes an employer to terminate an employee on the ground of disease,
viz:
Art. 284. Disease as a ground for termination- An employer may terminate the services of an employee who
has been found to be suffering from any disease and whose continued employment is prohibited by law or prejudicial
to his health as well as the health of his co-employees: xxx
However, in order to validly terminate employment on this ground, Book VI, Rule I, Section 8 of the Omnibus
Implementing Rules of the Labor Code requires:
Sec. 8. Disease as a ground for dismissal- Where the employee suffers from a disease and his continued
employment is prohibited by law or prejudicial to his health or to the health of his co-employees, the employer shall
not terminate his employment unless there is a certification by competent public health authority that the disease is of
such nature or at such a stage that it cannot be cured within a period of six (6) months even with proper medical
treatment. If the disease or ailment can be cured within the period, the employer shall not terminate the employee but
shall ask the employee to take a leave. The employer shall reinstate such employee to his former position
immediately upon the restoration of his normal health.
As this Court stated in Triple Eight integrated Services, Inc. vs. NLRC, the requirement for a medical
certificate under Article 284 of the Labor Code cannot be dispensed with; otherwise, it would sanction the unilateral

Angeles| Bajana | Balladares | Brillantes | Briones | Cabansag | Callanta | Chua | David|


De Leon | Gomez | Lopez | Macalino | Nostratis | Padilla | Reynon | Santos | Tan |Velasco
4E / 4F - 2018-2019
Page 804 of 920
LABOR REVIEW DIGEST
Atty. Joyrich Golangco

and arbitrary determination by the employer of the gravity or extent of the employees illness and thus defeat the
public policy in the protection of labor.
In the case at bar, the employer clearly did not comply with the medical certificate requirement before
Sahots dismissal was effected. In the same case of Sevillana vs. I.T. (International) Corp., we ruled:
Since the burden of proving the validity of the dismissal of the employee rests on the employer, the latter
should likewise bear the burden of showing that the requisites for a valid dismissal due to a disease have been
complied with. In the absence of the required certification by a competent public health authority, this Court has ruled
against the validity of the employees dismissal. It is therefore incumbent upon the private respondents to prove by
the quantum of evidence required by law that petitioner was not dismissed, or if dismissed, that the dismissal was not
illegal; otherwise, the dismissal would be unjustified. This Court will not sanction a dismissal premised on mere
conjectures and suspicions, the evidence must be substantial and not arbitrary and must be founded on clearly
established facts sufficient to warrant his separation from work.

In addition, we must likewise determine if the procedural aspect of due process had been complied with by
the employer.
From the records, it clearly appears that procedural due process was not observed in the separation of
private respondent by the management of the trucking company. The employer is required to furnish an employee
with two written notices before the latter is dismissed: (1) the notice to apprise the employee of the particular acts or
omissions for which his dismissal is sought, which is the equivalent of a charge; and (2) the notice informing the
employee of his dismissal, to be issued after the employee has been given reasonable opportunity to answer and to
be heard on his defense. These, the petitioners failed to do, even only for record purposes. What management did
was to threaten the employee with dismissal, then actually implement the threat when the occasion presented itself
because of private respondents painful left thigh.
All told, both the substantive and procedural aspects of due process were violated. Clearly, therefore,
Sahots dismissal is tainted with invalidity.
On the last issue, as held by the Court of Appeals, respondent Jaime Sahot is entitled to separation pay.
The law is clear on the matter. An employee who is terminated because of disease is entitled to separation pay
equivalent to at least one month salary or to one-half month salary for every year of service, whichever is greater
xxx.[34] Following the formula set in Art. 284 of the Labor Code, his separation pay was computed by the appellate
court at P2,080 times 36 years (1958 to 1994) or P74,880.

Angeles| Bajana | Balladares | Brillantes | Briones | Cabansag | Callanta | Chua | David|


De Leon | Gomez | Lopez | Macalino | Nostratis | Padilla | Reynon | Santos | Tan |Velasco
4E / 4F - 2018-2019
Page 805 of 920
LABOR REVIEW DIGEST
Atty. Joyrich Golangco

2. UNION MOTOR CORPORATION V. NLRC


G.R. NO. 159738; DEC. 9, 2004
CALLEJO, SR., J.:

DOCTRINE: Medical or Dental Certificated need not be notarized to be accorded probative value.

For continued absence due to medical reason to equate to neglect of duty, the same should not merely be
gross but also habitual. Gross negligence implies a want or absence of or failure to exercise slight care or
diligence, or the entire absence of care. It evinces a thoughtless disregard of consequences without exerting
any effort to avoid them.

FACTS:

Alejandro Etis was hired by Union Motor Corp as automotive mechanic at the service department of their Paco
branch in 1993. In 1994, he was transferred to the Caloocan City Branch, where his latest monthly salary was
P6,330.00. During his employment, he was awarded the "Top Technician" for the month of May in 1995 and
Technician of the Year (1995). He also became a member of the Exclusive P40,000.00 Club and received the Model
Employee Award in the same year.

On the 22nd of September 1997, Etis made a call to Rosita dela Cruz, the company nurse, to inform her that he had
to take a sick leave due to a painful and unbrearable toothache. He called the next day to inform her that he could not
report for work as he had to consult a doctor. The doctor he consulted referred him to a dentist in the person of Dr.
Rodolfo Pamor, who scheduled him of a tooth extraction on Sept. 27, hoping that by that time the inflammation would
have subsided. A certain Mr. Dumagan, security personnel for Union dropped by Etis’ residence to confirm Etis’
condition- which he did. Unfortunately, the swelling did not subside by Sept. 27 and the procedure had to be
rescheduled, with Etis being unable to report in that time being.

On Oct. 2, Etis received an Office Memorandum terminating his services for his failure to give proper notice for his
absences, as per the company policy of terminating employees who incur more than 5 days of consecutive absence
without proper notice. Etis reported to work as soon as able, but he was denied entry and work. The proceedings
initiated by his labor union before the NCMB having failed, he was constrained to file an action before the LA for
illegal dismissal.

LA RULING: Ruled in favor of Union Motor. Held that his failure to report for work for 10 consecutive days was gross
neglect of duty, and that his unnotarized medical certificated were self-service and devoid of any probative value.

NLRC RULING: Reversed the LA and held that Etis had sufficiently substantiated his medical condition as, and that
the absence of 10 consecutive absences did not amount to gross or habitual neglect.

CA RULING: Affirmed the NLRC.

APPEAL TO THE SC:

Petitioner’s Contention: That the NLRC and the CA erred giving the medical certificates evidentiary weight, and that
Etis was not illegally dismissed.

ISSUES:.
 Whether or not the satisfy neglect as a ground for dismissal.
 Whether or not the medical certificates should be accorded evidentiary weight.

RULINGS:

1. NO. To warrant removal from service, the negligence should not merely be gross but also habitual. Gross
negligence implies a want or absence of or failure to exercise slight care or diligence, or the entire absence
of care. It evinces a thoughtless disregard of consequences without exerting any effort to avoid them.

Angeles| Bajana | Balladares | Brillantes | Briones | Cabansag | Callanta | Chua | David|


De Leon | Gomez | Lopez | Macalino | Nostratis | Padilla | Reynon | Santos | Tan |Velasco
4E / 4F - 2018-2019
Page 806 of 920
LABOR REVIEW DIGEST
Atty. Joyrich Golangco

The petitioner has not sufficiently shown that the respondent had willfully disobeyed the company rules and
regulations. The petitioner also failed to prove that the respondent abandoned his job. The bare fact that the
respondent incurred excusable and unavoidable absences does not amount to an abandonment of his
employment. The Court noted that the company’s policy did not even give a categorical declaration as to
what constituted proper notice for purposes of the 5 day consecutive absence rule to apply. It held Etis’ act
of calling the company nurse constituted sufficient notice. It even noted that the company security guard Mr.
Dumagan was able to confirm his condition, and was thus adequately informed.

2. NO. While it is true that the petitioner had objected to the veracity of the medical certificates because of lack
of notarization, it has been said that verification of documents is not necessary in order that the said
documents could be considered as substantial evidence. The medical certificates were properly signed by
the physicians; hence, they bear all the earmarks of regularity in their issuance and are entitled to full
probative weight.

Angeles| Bajana | Balladares | Brillantes | Briones | Cabansag | Callanta | Chua | David|


De Leon | Gomez | Lopez | Macalino | Nostratis | Padilla | Reynon | Santos | Tan |Velasco
4E / 4F - 2018-2019
Page 807 of 920
LABOR REVIEW DIGEST
Atty. Joyrich Golangco

3. ROMEO VILLARUEL, PETITIONER, VS. YEO HAN GUAN, DOING BUSINESS


UNDER THE NAME AND STYLE YUHANS ENTERPRISES, RESPONDENT.GR NO: 169191 DATE: JUNE 1, 2011
PONENTE: PERALTA, J.

Doctrine: The rule is that an employee who voluntarily resigns from employment is not entitled to separation
pay, except when it is stipulated in the employment contract or CBA, or it is sanctioned by established
employer practice or policy.

FACTS:
This case is for payment of separation pay filed with the NLRC by Romeo Villaruel against Yuhans Enterprises.
Petitioner, Romeo Villaruel, alleged that in June 1963, he was employed as a machine operator by Ribonette
Manufacturing Company, an enterprise engaged in the business of manufacturing and selling PVC pipes and is
owned and managed by herein respondent Yeo Han Guan. Over a period of almost twenty (20) years, the company
changed its name four times. Starting in 1993 up to the time of the filing of petitioner's complaint in 1999, the
company was operating under the name of Yuhans Enterprises. Despite the changes in the company's name,
petitioner remained in the employ of respondent.
On Oct. 5, 1998 petitioner got sick and was confined in a hospital, when he reported for work on Dec. 12, 1998, he
alleged that respondent no longer permitted him to go back because of his illness. He asked respondent to allow him
to continue working but to assign him a lighter kind of work which was denied. He was offered a separation pay of
P15,000 corresponding to the period of 1993 to 1999. Petitioner prays that he be granted separation pay computed
from his first of employment in June 1963, but respondent refused.
On the other hand respondent averred in his position paper that petitioner never showed up after he recovered from
his illness. Respondent was later caught by surprise when petitioner filed the instant case for recovery of separation
pay. Respondent claims that he never terminated petitioner’s employment and that during their mandatory
conference he even told petitioner that he could go back to work at anytime but petitioner clearly manifested that he
was no longer interested in returning to work.

LA/RTC/NLRC RULING:
LA: Decided in favor of petitioner and awarded separation pay pursuant to Art. 284 (now 299) of the labor code.

NLRC: Affirmed the ruling of the LA. MR was denied. Aggrieved, respondent filed for a petition for certiorari under
Rule 65

CA RULING: Granted the petition partly, deleting the award of separation pay but affirming the award of service
incentive leave of P 3,015.00. MR was denied.

ISSUE/S:
1. Whether petitioner was in fact dismissed by respondent

HELD:
1. NO.
The Court agrees with the CA in its observation of the following circumstances as proof that respondent did not
terminate petitioner's employment: first, the only cause of action in petitioner's original complaint is that he was
"offered a very low separation pay"; second, there was no allegation of illegal dismissal, both in petitioner's original
and amended complaints and position paper; and, third, there was no prayer for reinstatement.
In consonance with the above findings, the Court finds that petitioner was the one who initiated the severance of his
employment relations with respondent. It is evident from the various pleadings filed by petitioner that he never
intended to return to his employment with respondent on the ground that his health is failing. Indeed, petitioner did
not ask for reinstatement. In fact, he rejected respondent's offer for him to return to work. This is tantamount to
resignation.
It may not be amiss to point out at this juncture that aside from Article 284 of the Labor Code, the award of separation
pay is also authorized in the situations dealt with in Article 283 of the same Code and under Section 4 (b), Rule I,
Book VI of the Implementing Rules and Regulations of the said Code where there is illegal dismissal and
reinstatement is no longer feasible. By way of exception, this Court has allowed grants of separation pay to stand as
"a measure of social justice" where the employee is validly dismissed for causes other than serious misconduct or
those reflecting on his moral character. However, there is no provision in the Labor Code which grants separation
pay to voluntarily resigning employees. In fact, the rule is that an employee who voluntarily resigns from employment
is not entitled to separation pay, except when it is stipulated in the employment contract or CBA, or it is sanctioned by
established employer practice or policy. In the present case, neither the abovementioned provisions of the Labor

Angeles| Bajana | Balladares | Brillantes | Briones | Cabansag | Callanta | Chua | David|


De Leon | Gomez | Lopez | Macalino | Nostratis | Padilla | Reynon | Santos | Tan |Velasco
4E / 4F - 2018-2019
Page 808 of 920
LABOR REVIEW DIGEST
Atty. Joyrich Golangco

Code and its implementing rules and regulations nor the exceptions apply because petitioner was not dismissed from
his employment and there is no evidence to show that payment of separation pay is stipulated in his employment
contract or sanctioned by established practice or policy of herein respondent, his employer.
Since petitioner was not terminated from his employment and, instead, is deemed to have resigned therefrom, he is
not entitled to separation pay under the provisions of the Labor Code.

Other Notes/ SC Pronouncements:


SC found it proper to award petitioner the amount of P50,000 pesos as financial assistance based on the
circumstances surrounding the case.

Angeles| Bajana | Balladares | Brillantes | Briones | Cabansag | Callanta | Chua | David|


De Leon | Gomez | Lopez | Macalino | Nostratis | Padilla | Reynon | Santos | Tan |Velasco
4E / 4F - 2018-2019
Page 809 of 920
LABOR REVIEW DIGEST
Atty. Joyrich Golangco

ARTICLE 300 – TERMINATION BY EMPLOYEE

1. ELSA S. MALIG-ON V. EQUITABLE GENERAL SERVICES, INC.


GR NO: 185269 DATE: JUNE 29, 2010
PONENTE: J. ABAD

Doctrines:
The rule in termination cases is that the employer bears the burden of proving that he dismissed his
employee for a just cause. And, when the employer claims that the employee resigned from work, the burden
is on the employer to prove that he did so willingly.

The act of "offdetailing" was not the equivalent of dismissal so long as the employee’s floating status did not
continue beyond a reasonable time. But, when it ran up to more than six months, the company may be
considered to have constructively dismissed the employee from work.

An illegally dismissed employee is entitled to two reliefs: backwages and reinstatement. Still, the Court has
held that the grant of separation pay, rather than reinstatement, may be proper especially when the latter is
no longer practical or will be for the best interest of the parties, as in this case.

FACTS:
This case is about an employee who was considered illegally dismissed notwithstanding the fact that she filed a
written resignation from her work.

Petitioner Elsa Malig-on claimed that respondent Equitable General Services, Inc. hired her as janitress in its
janitorial services. After six years, Malig-on's immediate supervisor told her that the company would be assigning her
to another client. But it never did despite several follow-ups that she made. Eight months later, the company told
Malig-on that she had to file a resignation letter before it would reassign her. She complied but the company reneged
on its undertaking, prompting Malig-on to file a complaint against it for illegal dismissal.

The company denied Malig-on's allegations. It claimed that she just stopped reporting for work on without giving any
reason. Consequently, the company wrote her two letters, on different dates, asking her to explain her continued
absence. Malig-on showed up at the company's office and submitted her resignation letter.

LA/NLRC/CA RULING:
The LA found Malig-On’s resignation valid and binding.

The NLRC reversed the LA’s decision and ruled that the company had constructively dismissed Malig-On.

The CA reversed the NLRC’s ruling and reinstated that of the LA.

ISSUE:
1. Whether Malig-On was constructively dismissed.
2. What reliefs should be awarded to Malig-On if she was illegally dismissed?

HELD:
1. YES. The rule in termination cases is that the employer bears the burden of proving that he dismissed his
employee for a just cause. And, when the employer claims that the employee resigned from work, the burden is on
the employer to prove that he did so willingly. Whether that is the case would largely depend on the circumstances
surrounding such alleged resignation. Those circumstances must be consistent with the employee's intent to give up
work.

Here, the company claims that Malig-on voluntarily resigned, gave a letter of resignation that she wrote with her own
hand, used the vernacular language, and signed it. But these are not enough. They merely prove that she wrote that
letter, a thing that she did not deny. She was quick to point out that she wrote it after being told that she needed to
resign so she could be cleared for her next assignment.

First, when Malig-on reportedly dropped out of sight and the company had no idea about the reason for it, the natural
and right thing for it to do was investigate why she had suddenly vanished. Indeed, the company needed to write
Malig-on immediately and ask her to explain in writing why she should not be considered to have abandoned her job
so the company may be cleared of its responsibility as employer. This did not happen here.

Angeles| Bajana | Balladares | Brillantes | Briones | Cabansag | Callanta | Chua | David|


De Leon | Gomez | Lopez | Macalino | Nostratis | Padilla | Reynon | Santos | Tan |Velasco
4E / 4F - 2018-2019
Page 810 of 920
LABOR REVIEW DIGEST
Atty. Joyrich Golangco

Second, if Malig-on had abandoned her work and had no further interest in it, there was no reason for her to
suddenly show up at her former place of work after eight months and file her resignation letter. Her action would
make sense only if, as she claimed, she had been on floating status for over six months and the company promised
to give her a new assignment if she would go through the process of resigning and reapplying.

And, third, that Malig-on went to the NLRC to file a complaint for unjust dismissal just three days after she filed her
alleged resignation letter is inconsistent with genuine resignation. It would make sense only if, as Malig-on claims, the
company tricked her into filing for resignation upon a promise to give her a new work assignment and failed to deliver
such promise.

The company evidently placed Malig-on on floating status after being relieved as janitress in a client's workplace.
But, as the Court has repeatedly ruled, such act of "offdetailing" Malig-on was not the equivalent of dismissal so long
as her floating status did not continue beyond a reasonable time. But, when it ran up to more than six months, the
company may be considered to have constructively dismissed her from work.

2. An illegally dismissed employee is entitled to two reliefs: backwages and reinstatement. Still, the Court has held
that the grant of separation pay, rather than reinstatement, may be proper especially when the latter is no longer
practical or will be for the best interest of the parties, as in this case.

Here, after her last work, Malig-on did not appear persistent in getting rehired. Indeed, she did not file any action for
constructive dismissal after being placed in a floating status for more than six months. If she were to be believed, it
was only eight months later that she showed keen interest
in being taken back by following an advice that she first tender her resignation in order to clear up her record prior to
being rehired.

After just three days from tendering her resignation, Malig-on hastened to the NLRC and accused her employer of
illegal dismissal. Under the circumstances, her reinstatement to her former position would only result in a highly
hostile work environment for the parties and might further worsen their relations which are already scarred by the
present case. The NLRC should have just awarded Malig-on separation pay instead of ordering the company to
reinstate her.

Angeles| Bajana | Balladares | Brillantes | Briones | Cabansag | Callanta | Chua | David|


De Leon | Gomez | Lopez | Macalino | Nostratis | Padilla | Reynon | Santos | Tan |Velasco
4E / 4F - 2018-2019
Page 811 of 920
LABOR REVIEW DIGEST
Atty. Joyrich Golangco

2. CHIANG KAI SHEK COLLEGE AND CARMELITA ESPINO V. ROSALINDA M. TORRES


GR NO: 189456 DATE: APRIL 2, 2014
PONENTE: PEREZ, J

Doctrine:
There is constructive dismissal when there is cessation of work, because continued employment is rendered
impossible, unreasonable or unlikely, as an offer involving a demotion in rank or a diminution in pay and
other benefits.

FACTS:
Torres was employed as a grade school teacher of petitioner, Chiang Kai Shek College. Torres was accused of
leaking a copy of a special quiz given to Grade 5 students of HEKASI. Mrs. Koo, confronted Torres, who had initially
denied leaking the test paper but later on admitted that she gave the test paper to Mrs. Anduyan. Torres explained
that she was busy checking the writing workbook when somebody handed her the special quiz for HEKASI 5 and
Mrs. Anduyan denied that she took the test paper from Torres without the latter's permission. After an administrative
hearing, the Investigating Committee(IC) found Torres and Mrs. Anduyan guilty of committing a grave offense of the
school policies by leaking a special quiz and decided to impose the penalty of one-month suspension without pay on
Torres and forfeiture of all the benefits scheduled to be given on Teacher's Day. According to petitioners, their IC had
actually decided to terminate Torres and had in fact prepared a memorandum of termination, but Torres allegedly
pleaded for a change of punishment in a short letter from termination to suspension and that Torres will resign at the
end of the school year.

Torres then filed a complaint for constructive dismissal and illegal suspension with the Labor Arbiter. Torres alleged
that she was forced and pressured to submit the written request for a change of penalty and commitment to resign at
the end of the school year.

Petitioners insisted that respondent voluntarily resigned. Petitioners averred that Torres was accorded her right to
due process prior to her termination. A formal investigation was conducted during which respondent was given the
opportunity to defend herself and confront her accusers.

LA Ruling:
The Labor Arbiter held that there was no constructive dismissal because Torres was not coerced nor pressured to
write her resignation letter.

NLRC Ruling:
NLRC affirmed the ruling of LA but ordered petitioners to pay Torres separation pay equivalent to one-half (1/2)
month salary for every year of service on the grounds of equity and social justice.

CA RULING:
CA ruled that Torres did not voluntarily resign but was constructively dismissed.

APPEAL TO THE SC:


Petitioners’ contention:
Petitioners point out that in Torres’ handwritten letter, she offered to voluntarily resign at the end of the school year,
provided that her punishment be changed from termination to suspension. Petitioners deny forcing, coercing or
pressuring respondent into writing said letter.

Respondent’s contention:
Respondent averred that individual petitioner forced her to write the written request for a change of the action on the
charges against her, from dismissal to suspension and eventual resignation. Respondent reiterates that she never
intended to resign but due to intense pressure from individual petitioner who threatened that she will not receive her
monetary benefits, she was pressured to write the alleged resignation letter.
ISSUE/S:
Whether or not the school's act of imposing the penalty of suspension instead of immediate dismissal from service at
the behest of the erring employee, in exchange for the employee's resignation at the end of the school year,
constitutes constructive dismissal.

HELD:

Angeles| Bajana | Balladares | Brillantes | Briones | Cabansag | Callanta | Chua | David|


De Leon | Gomez | Lopez | Macalino | Nostratis | Padilla | Reynon | Santos | Tan |Velasco
4E / 4F - 2018-2019
Page 812 of 920
LABOR REVIEW DIGEST
Atty. Joyrich Golangco

NO. Given the indications of voluntary resignation, we rule that there is no constructive dismissal in this case. There
is constructive dismissal when there is cessation of work, because continued employment is rendered impossible,
unreasonable or unlikely, as an offer involving a demotion in rank or a diminution in pay and other benefits. Aptly
called a dismissal in disguise or an act amounting to dismissal but made to appear as if it were not, constructive
dismissal may, likewise, exist if an act of clear discrimination, insensibility, or disdain by an employer becomes so
unbearable on the part of the employee that it could foreclose any choice by him except to forego his continued
employment. There was here no discrimination committed by petitioners. While respondent did not tender her
resignation wholeheartedly, circumstances of her own making did not give her any other option. With due process,
she was found to have committed the grave offense of leaking test questions. Dismissal from employment was the
justified equivalent penalty. Having realized that, she asked for, and was granted, not just a deferred imposition of,
but also an acceptable cover for the penalty.

Other Notes/ SC Pronouncements:


Resignation is the voluntary act of an employee who is in a situation where one believes that personal reasons
cannot be sacrificed for the favor of employment, and opts to leave rather than stay employed. It is a formal
pronouncement or relinquishment of an office, with the intention of relinquishing the office accompanied by the act of
relinquishment. As the intent to relinquish must concur with the overt act of relinquishment, the acts of the employee
before and after the alleged resignation must be considered in determining whether, he or she, in fact, intended to
sever his or her employment.

Angeles| Bajana | Balladares | Brillantes | Briones | Cabansag | Callanta | Chua | David|


De Leon | Gomez | Lopez | Macalino | Nostratis | Padilla | Reynon | Santos | Tan |Velasco
4E / 4F - 2018-2019
Page 813 of 920
LABOR REVIEW DIGEST
Atty. Joyrich Golangco

3. VICTORINO OPINALDO VS. NARCISA RAVINA, ST. LOUISSE SECURITY AGENCY


G.R. No.196573 Date: October 16, 2013
Ponente: VILLARAMA, JR., J.

DOCTRINE:
Abandonment is the deliberate and unjustified refusal of an employee to resume his employment. To
constitute abandonment of work, 2 elements must concur: (1) the employee must have failed to report for
work or must have been absent without valid or justifiable reason; and, (2) there must have been a clear
intention on the part of the employee to sever the employer-employee relationship manifested by some overt
act.

FACTS:
Respondent Narcisa Ravina, the general manager and sole proprietor of St. Louisse Security Agency, hired
petitioner Victorino Opinaldo as its security guard. Petitioner was detailed to PAIJR Furniture Accessories (PAIJR),
however, the owner submitted a complaint to respondent stating that Opinaldo was no longer physically fit to perform
his duties and responsibilities because of his health condition. Hence, respondent relieved petitioner from his work.
Respondent was reassigned to Gomez Construction. After working for a period of 2 weeks therein, petitioner ceased
to report for work. Petitioner filed a complaint for underpayment of salary and nonpayment of other labor
standard benefits. The parties agreed to settle and reached a compromise agreement.

Petitioner then signed a Quitclaim and Release before the DOLE Regional Office in Cebu City for the amount of P
5,000. After 4 weeks from the settlement of the case, petitioner returned to respondent’s office on December 22,
2006 claiming that when he asked respondent to sign an SSS Sickness Notification which he was going to use in
order to avail of the discounted fees for a medical checkup, respondent allegedly refused and informed him that he
was no longer an employee of the Agency. Respondent allegedly told him that when he signed the quitclaim and
release form at the DOLE Regional Office, she already considered him to have quit his employment. Petitioner filed a
complaint for Illegal Dismissal with a prayer for the payment of separation pay in lieu of reinstatement before the
NLRC Regional Arbitration Branch.

LA RULING:
Illegal dismissal. It ordered respondent to pay petitioner separation pay and back wages.

NLRC RULING:
Affirmed the decision of the LA.

CA RULING:
Reversed the NLRC. It was petitioner himself who failed to report for work and hence severed his employment with
the Agency. Petitioner’s claims relative to his alleged illegal dismissal were not substantiated.

Respondent’s contention:
She did not illegally dismiss petitioner and that it was a valid exercise of management prerogative that he was not
given any assignment pending the submission of the required medical certificate of his fitness to work.

ISSUES:
(1) Was petitioner illegally dismissed? YES.
(2) Did petitioner abandon his work? NO.

RULING:
(1) YES. While it is a management prerogative to require petitioner to submit a medical certificate, respondent
cannot withhold petitioner’s employment without observing the principles of due process and fair play. It is a
basic principle of labor protection in this jurisdiction that a worker cannot be deprived of his job without
satisfying the requirements of due process. Labor is property and the right to make it available is next in
importance to the rights of life and liberty. As enshrined under the Bill of Rights, no person shall be deprived of
life, liberty or property without due process of law. The due process requirement in the deprivation of one’s
employment is transcendental that it limits the exercise of the management prerogative of the employer to
control and regulate the affairs of the business. In the case at bar, all that respondent employer needed to prove
was that petitioner employee was notified that his failure to submit the required medical certificate will result in
his lack of work assignment – and eventually the termination of his employment – as a security guard. There is
no iota of evidence in the records, save for the bare allegations of respondent, that petitioner was notified of
such consequence for non-submission.

Angeles| Bajana | Balladares | Brillantes | Briones | Cabansag | Callanta | Chua | David|


De Leon | Gomez | Lopez | Macalino | Nostratis | Padilla | Reynon | Santos | Tan |Velasco
4E / 4F - 2018-2019
Page 814 of 920
LABOR REVIEW DIGEST
Atty. Joyrich Golangco

(2) NO. Abandonment is the deliberate and unjustified refusal of an employee to resume his employment. To
constitute abandonment of work, two elements must concur: (1) the employee must have failed to report for
work or must have been absent without valid or justifiable reason; and, (2) there must have been a clear
intention on the part of the employee to sever the employer-employee relationship manifested by some overt
act. None of these elements is present in the case at bar. It is clear that they prevented petitioner’s continued
employment with them unless the latter presents a medical certificate that he is physically and mentally fit for
work. Moreover, if it was really true that complainant abandoned his work, then why have not respondents sent
him a notice to report back for work? It is evident then that respondents found an excuse to decline
complainant’s continued stay with them on the pretext that he has to submit first a medical certificate before he
could be allowed to resume employment.

Angeles| Bajana | Balladares | Brillantes | Briones | Cabansag | Callanta | Chua | David|


De Leon | Gomez | Lopez | Macalino | Nostratis | Padilla | Reynon | Santos | Tan |Velasco
4E / 4F - 2018-2019
Page 815 of 920
LABOR REVIEW DIGEST
Atty. Joyrich Golangco

4. WILLI HAHN ENTERPRISES and/or WILLI HAHN vs. LILIA R. MAGHUYOP


GR No: 160348 Date: December 17, 2004
Ponente: YNARES-SANTIAGO, J.
Doctrine: The failure of petitioner to pursue the termination proceedings against respondent and to make her
pay for the shortage incurred did not cast doubt on the voluntary nature of her resignation. A decision to
give a graceful exit to an employee rather than to file an action for redress is perfectly within the discretion
of an employer.
Respondent’s unsubstantiated and self-serving claim that she was coerced into signing the resignation
letter does not deserve credence.

FACTS:

Sometime in 1982, respondent Lilia Maghuyop was hired by petitioner Willi Hahn as nanny of one of his sons. In
1986, she was employed as salesclerk of Willi Hahn Enterprises (Ali Mall, Cubao branch), an authorized dealer of
sporting goods, guns and ammunitions. In 1996, she was promoted as store manager of its branch in Shoe Mart
(SM) Cebu, with a monthly salary of P8,240.00.
On February 25, 1998, petitioner conducted an Inventory Report and discovered that its SM Cebu branch incurred
stock shortages and non-remittances in the total amount of P27,727.39.[4] In the latter part of July 1998, petitioner
decided to terminate the services of respondent, however, before he could do so, the latter tendered her resignation.
Believing the good faith of respondent in resigning, petitioner decided not to file charges against her anymore.
On the other hand, respondent claimed that on July 22, 1998, while she was in SM Cebu branch, she was
approached by Tony Abu and Cesar Araneta who ordered her to close shop and to write a letter to Mr. and Mrs.
Hahn thanking them for the years she had been in their employ and for all the benefits she received from them. She
refused to obey the order, but Tony Abu typed the letter of resignation and asked her to sign the same. Respondent
admitted that she read and affixed her signature on the letter. Thereafter, she was allegedly told to pack her
belongings and to vacate the housing unit provided by the company for her family.[5]
On August 25, 1998, respondent filed a complaint with the NLRC, alleging that she should be awarded backwages,
separation pay, salary for July 16-22, 1998 which was withheld by petitioner, proportionate 13th month pay, damages
and attorney’s fees.
LA RULING: NO Illegal dismissal but Ordered petitioner to pay respondent her financial assistance, one week
unpaid wages, and proportionate 13th month pay, or a total sum of P14,727.49, plus ten percent (10%) attorneys
fees as to the last two amounts.

NLRC RULING: denied for lack of merit

CA RULING: REVERSED decision of LA/ NLRC. Petitioner was ordered to pay respondent her unpaid salary from
July 16 to 22, 1998, full back wages (inclusive of basic pay, 13th month pay, allowances and monetary value of all
benefits) computed from July 22, 1998 up to the finality of this decision, separation pay equivalent to one (1) month
pay for every year of service, moral damages of P10,000.00 and 10% attorneys fees.

ISSUE/S:

Whether or not respondent voluntarily resigned as manager of the SM Cebu branch

HELD:

Yes. The letter is simple, candid and direct to the point. We find no merit in respondents claim that being a mere
clerk, she did not realize the consequences of her resignation. Although she started as nanny to the son of petitioner
Willi Hahn, she has risen to being the manager and officer-in-charge of the Willi Hahn Enterprises in SM Cebu
branch.
In Callanta v. National Labor Relations Commission,[9] a national-promoter salesman of Distilleria Limtuaco Co.,
Inc., assigned in Iligan City, Lanao del Sur and Lanao Del Norte, resigned after he was found to have a shortage of
P49,005.49 in a spot audit conducted by the company. He later filed an illegal dismissal case claiming that his
consent to the resignation was vitiated as he signed the company’s ready-made resignation letter because the latter
threatened to file a estafa case against him. In rejecting his contention, the Court ruled that a salesman-promoter
could not have been confused, coerced or intimidated into signing the resignation letter. Instead of defending himself
against the adverse audit report, he voluntarily signed the resignation letter though there is no urgency in signing the

Angeles| Bajana | Balladares | Brillantes | Briones | Cabansag | Callanta | Chua | David|


De Leon | Gomez | Lopez | Macalino | Nostratis | Padilla | Reynon | Santos | Tan |Velasco
4E / 4F - 2018-2019
Page 816 of 920
LABOR REVIEW DIGEST
Atty. Joyrich Golangco

same. The Court concluded that he affixed his signature in the said letter of his own free will with full knowledge of
the consequences thereof.
The failure of petitioner to pursue the termination proceedings against respondent and to make her pay for the
shortage incurred did not cast doubt on the voluntary nature of her resignation. A decision to give a graceful exit to an
employee rather than to file an action for redress is perfectly within the discretion of an employer. It is not uncommon
that an employee is permitted to resign to save face after the exposure of her malfeasance. Under the
circumstances, the failure of petitioner to file action against the respondent should be considered as an act of
compassion for one who used to be a trusted employee and a close member of the household.
Respondent’s unsubstantiated and self-serving claim that she was coerced into signing the resignation letter
does not deserve credence. It is a basic rule in evidence that the burden of proof is on the part of the party who
makes the allegations.[10] Respondent failed to discharge this burden.
Moreover, the Court of Appeals finding that respondent had no motive to resign because the charges of
dishonesty were not fully substantiated has no basis. Had the separation of respondent been for dismissal due to
loss of trust and confidence, substantial evidence of the shortages and non-remittances would have been
indispensable. Such, is not the case here considering her voluntary resignation.
The rule that the filing of a complaint for illegal dismissal is inconsistent with resignation,[11] is not applicable to
the instant case. The filing of an illegal dismissal case by respondent was evidently a mere afterthought. It was filed
not because she wanted to return to work but to claim separation pay and backwages.
Settled is the rule that factual findings of labor officials who are deemed to have acquired expertise in matters
within their respective jurisdiction are generally accorded not only respect, but even finality, and bind the Supreme
Court when supported by substantial evidence.[12] The findings of both the Labor Arbiter and the NLRC are amply
supported by the required quantum of evidence, i.e., evidence as a reasonable mind might accept as adequate to
support a conclusion.[13]
Hence, we find no reason to deviate from the conclusion of both the NLRC and the Labor Arbiter that
respondent, having tendered a voluntary resignation was not illegally dismissed.

Angeles| Bajana | Balladares | Brillantes | Briones | Cabansag | Callanta | Chua | David|


De Leon | Gomez | Lopez | Macalino | Nostratis | Padilla | Reynon | Santos | Tan |Velasco
4E / 4F - 2018-2019
Page 817 of 920
LABOR REVIEW DIGEST
Atty. Joyrich Golangco

5. SKIPPERS UNITED PACIFIC VS DOZA ET AL


GR NO: 175558 DATE: FEBRUARY 8, 2012
PONENTE: CARPIO, J.

Doctrine:
The Labor Code allows the termination by the employee of the employment contract by serving written
notice on the employer at least 1 month in advance. Given that provision, the law contemplates the
requirement of a written notice of resignation.

FACTS:

The respondents are seafarers who were unceremoniously discharged from MV Wisdom Stars and immediately
repatriated upon demand of allotments and other provisions while on board the vessel. Upon arrival in the
Philippines, they filed a complaint for illegal dismissal with the Labor Arbiter and prayed for payment of their home
allotment for the month of December 1998, salaries for the unexpired portion of their contracts, moral damages,
exemplary damages, and attorneys fees.

Skippers, claimed that the respondents requested to be repatriated immediately because they were not satisfied with
the ship’s condition. Since the respondents pre-terminated their contracts, Skippers claims they are liable for their
repatriation expenses in accordance with Section 19(G) of POEA Memorandum Circular No. 55, series of 1996 which
states:

G. A seaman who requests for early termination of his contract shall be liable for his repatriation cost as well as the
transportation cost of his replacement. The employer may, in case of compassionate grounds, assume the
transportation cost of the seafarers replacement.

Skippers also prayed for payment of moral damages and attorneys fees.

LA RULING:
The LA dismissed the respondents’ complaint for illegal dismissal because the seafarers voluntarily pre-terminated
their employment contracts by demanding for immediate repatriation due to dissatisfaction with the ship. The LA held
that such voluntary pre-termination of employment contract is akin to resignation, a form of termination by employee
of his employment contract under Article 285 of the Labor Code. Due to the absence of illegal dismissal, De Gracia,
et. al.s claim for salaries representing the unexpired portion of their employment contracts was dismissed.

NLRC RULING:
The NLRC also denied the claim for illegal dismissal because the respondents were not able to refute the telex
received by Skippers from the vessels master that De Gracia, et al. voluntarily pre-terminated their contracts and
demanded immediate repatriation due to their dissatisfaction with the ships operations. The respondents MR was
also denied.

CA RULING:
Upon appeal to the CA, it reversed the LA and NLRC decision. It also denied Skippers’ MR.

The CA held that the telex message was a self-serving document that does not satisfy the requirement of substantial
evidence, or that amount of relevant evidence which a reasonable mind might accept as adequate to justify the
conclusion that petitioners indeed voluntarily demanded their immediate repatriation. Hence, the repatriation of
respondents prior to the expiration of their contracts showed they were illegally dismissed from employment

APPEAL TO THE SC:

Petitioner's Contention:

Respondent's Contention:

ISSUE/S:
1. Did the respondents voluntarily terminated their employment?

HELD:

Angeles| Bajana | Balladares | Brillantes | Briones | Cabansag | Callanta | Chua | David|


De Leon | Gomez | Lopez | Macalino | Nostratis | Padilla | Reynon | Santos | Tan |Velasco
4E / 4F - 2018-2019
Page 818 of 920
LABOR REVIEW DIGEST
Atty. Joyrich Golangco

1. No. The SC denied the petition and affirmed the CA Decision, but modifief the award. For a workers dismissal to
be considered valid, it must comply with both procedural and substantive due process. The legality of the manner
of dismissal constitutes procedural due process, while the legality of the act of dismissal constitutes substantive
due process.

In this case, there was no written notice furnished to respondents regarding the cause of their dismissal. Cosmoship
furnished a written notice (telex) to Skippers, the local manning agency, claiming that respondents were repatriated
because the latter voluntarily pre-terminated their contracts. This telex was given credibility and weight by the LA and
NLRC in deciding that there was pre-termination of the employment contract akin to resignation and no illegal
dismissal. However, as correctly ruled by the CA, the telex message is a biased and self-serving document that does
not satisfy the requirement of substantial evidence. If, indeed, respondents voluntarily pre-terminated their contracts,
then respondents should have submitted their written resignations.

Article 285 of the Labor Code recognizes termination by the employee of the employment contract by serving written
notice on the employer at least one (1) month in advance. Given that provision, the law contemplates the requirement
of a written notice of resignation. In the absence of a written resignation, it is safe to presume that the employer
terminated the seafarers. In addition, the telex message relied upon by the LA and NLRC bore conflicting dates of 22
January 1998 and 22 January 1999, giving doubt to the veracity and authenticity of the document. In 22 January
1998, respondents were not even employed yet by the foreign principal. For these reasons, the dismissal of
respondents was illegal.

Angeles| Bajana | Balladares | Brillantes | Briones | Cabansag | Callanta | Chua | David|


De Leon | Gomez | Lopez | Macalino | Nostratis | Padilla | Reynon | Santos | Tan |Velasco
4E / 4F - 2018-2019
Page 819 of 920
LABOR REVIEW DIGEST
Atty. Joyrich Golangco

6. JONATHAN V. MORALES vs. HARBOUR CENTRE PORT TERMINAL, INC.


GR No.: 174208 Date: January 25, 2012
Ponente: PEREZ, J.

DOCTRINE:

In cases of a transfer of an employee, the rule is settled that the employer is charged with the burden of
proving that its conduct and action are for valid and legitimate grounds such as genuine business necessity
and that the transfer is not unreasonable, inconvenient or prejudicial to the employee.

If the employer cannot overcome this burden of proof, the employee’s transfer shall be tantamount to
unlawful constructive dismissal.

FACTS:

On May 16, 2000, petitioner was hired by HCPTI as an Accountant and Acting Finance Officer, with a monthly salary
of P18,000.00. Regularized on November 17, 2000, Morales was promoted to Division Manager of the Accounting
Department. Subsequent to HCPTI’s transfer to its new offices at Vitas, Tondo, Manila, Morales received an inter-
office memorandum reassigning him to Operations Cost Accounting, tasked with the duty of “monitoring and
evaluating all consumables requests, gears and equipment” related to the corporation’s operations and of interacting
with its sub-contractor, Bulk Fleet Marine Corporation.

On 31 March 2003, Morales wrote Singson, protesting that his reassignment was a clear demotion since the position
to which he was transferred was not even included in HCPTI’s plantilla. In response to Morales’ grievance that he
had been effectively placed on floating status, Singson issued an inter-office memorandum to the effect that “transfer
of employees is a management prerogative” and that HCPTI had “the right and responsibility to find the perfect
balance between the skills and abilities of employees to the needs of the business.” For the whole of the ensuing
month Morales was absent from work and/or tardy.

Singson issued to Morales an inter-office memorandum denominated as a First Warning. The memorandum
reminded Morales that, as an employee of HCPTI, he was subject to its rules and regulations and could be
disciplinarily dealt with pursuant to its Code of Conduct. In view of the absences Morales continued to incur, HCPTI
issued a Second Warning dated 6 May 2003 and a Notice to Report for Work and Final Warning.

Morales then filed before the NLRC a complaint for constructive dismissal, moral and exemplary damages as well as
attorney’s fees. HCPTI argued that Morales abandoned his employment and was not constructively dismissed.

LA RULING: Morales’ reassignment was a valid exercise of HCPTI’s management prerogative which cannot be
construed as constructive dismissal absent showing that the same was done in bad faith and resulted in the
diminution of his salary and benefits.

NLRC RULING: reversed the LA. Morales’ reassignment was a clear demotion despite lack of showing of diminution
of salaries and benefits.

CA RULING: reversed the NLRC.

ISSUE/S:

Whether or not Morales was constructively dismissed.

HELD:

Yes. In cases of a transfer of an employee, the rule is settled that the employer is charged with the burden of proving
that its conduct and action are for valid and legitimate grounds such as genuine business necessity and that the
transfer is not unreasonable, inconvenient or prejudicial to the employee.

If the employer cannot overcome this burden of proof, the employee’s transfer shall be tantamount to unlawful
constructive dismissal. While there was a lack of showing that the transfer or reassignment entailed a diminution of
salary and benefits, one fact that must not be lost sight of was that Morales was already occupying the position of
Division Manager.

Angeles| Bajana | Balladares | Brillantes | Briones | Cabansag | Callanta | Chua | David|


De Leon | Gomez | Lopez | Macalino | Nostratis | Padilla | Reynon | Santos | Tan |Velasco
4E / 4F - 2018-2019
Page 820 of 920
LABOR REVIEW DIGEST
Atty. Joyrich Golangco

Concurrently appointed as member of HCPTI’s Management Committee (MANCOM), Morales was subsequently
reassigned by HCPTI “from managerial accounting to Operations Cost Accounting” without any mention of the
position to which he was actually being transferred. That the reassignment was a demotion is, however, evident from
Morales’ new duties which, far from being managerial in nature, were very simply and vaguely described as inclusive
of “monitoring and evaluating all consumables requests, gears and equipments related to [HCPTI’s] operations” as
well as “close interaction with [its] sub-contractor Bulk Fleet Marine Corporation.” Morales’ demotion is evident from
the fact that his reassignment entailed a transfer from a managerial position to one which was not even included in
the corporation’s plantilla.

Morales did not abandon his employment. As a just and valid ground for dismissal, at any rate, abandonment
requires the deliberate, unjustified refusal of the employee to resume his employment, without any intention of
returning. Since an employee like Morales who takes steps to protest his dismissal cannot logically be said to have
abandoned his work, it is a settled doctrine that the filing of a complaint for illegal dismissal is inconsistent with
abandonment of employment.

Angeles| Bajana | Balladares | Brillantes | Briones | Cabansag | Callanta | Chua | David|


De Leon | Gomez | Lopez | Macalino | Nostratis | Padilla | Reynon | Santos | Tan |Velasco
4E / 4F - 2018-2019
Page 821 of 920
LABOR REVIEW DIGEST
Atty. Joyrich Golangco

7. SHS PERFORATED MATERIALS, INC vs DIAZ


G.R. No. 185814; October 13, 2010
Mendoza, J.:

DOCTRINE: Management prerogative refers to the right to regulate all aspects of employment, it cannot be
understood to include the right to temporarily withhold salary/wages without the consent of the employee.

FACTS: Petitioner SHS Perforated Materials, Inc. (SHS) is a start-up corporation organized and existing under the
laws of the Republic of the Philippines and registered with the Philippine Economic Zone Authority. Petitioner
Winfried Hartmannshenn (Hartmannshenn), a German national, is its president, while Petitioner Hinrich Johann
Schumacher (Schumacher), also a German national, is the treasurer and one of the board directors. Manuel F. Diaz
(respondent) was hired by petitioner SHS as Manager for Business Development on probationary status.

During respondents employment, Hartmannshenn was often abroad and, because of business exigencies. When he
would be in the Philippines, he and the respondent held meetings. As to respondents work, there was no close
supervision by him.

During meetings with the respondent, Hartmannshenn expressed his dissatisfaction over respondents poor
performance and absences during his employment specially when petitioner was in abroad. The petitioner also
observed that respondent is not responsive the the electronic mails, calls and communication made to him, them
being important for the business.

Hartmannshenn then instructed Taguiang not to release respondents salary. Later that afternoon, respondent called
and inquired about his salary. Taguiang informed him that it was being withheld and that he had to immediately
communicate with Hartmannshenn. Respondent was then served on SHS a demand letter and a resignation letter.

Petitioners averred that respondent was unable to give a proper explanation for his behavior. Hartmannshenn then
accepted respondents resignation and informed him that his salary would be released upon explanation of his failure
to report to work, and proof that he did, in fact, work for the period in question.

Respondent filed complaint against the petitioners for illegal dismissal; non-payment of salaries/wages and
13th month pay with prayer for reinstatement and full backwages; exemplary damages, and attorneys fees, costs of
suit, and legal interest.

Petitioner’s Argument: Petitioners contend that withholding respondents salary from November 16 to November 30,
2005, was justified because respondent was absent and did not show up for work during that period. He also failed to
account for his whereabouts and work accomplishments during said period. When there is an issue as to whether an
employee has, in fact, worked and is entitled to his salary, it is within management prerogative to temporarily withhold
an employees salary/wages pending determination of whether or not such employee did indeed work.

Labor Arbiter: declared that respondent have constructively dismissed.


NLRC: reversed the decision of the LA.
Court Of Appeals: reversed the NLRC Decision.

ISSUE/s:
1. Whether or not the temporary withholding of respondents salary/wages by petitioners was a valid exercise
of management prerogative;
2. Whether or not respondent voluntarily resigned.
3. Whether or not respondent was constructively dismissed by petitioners;

HELD:

1. NO. Management prerogative refers to the right of an employer to regulate all aspects of employment, such as the
freedom to prescribe work assignments, working methods, processes to be followed, regulation regarding transfer of
employees, supervision of their work, lay-off and discipline, and dismissal and recall of work. Although management
prerogative refers to the right to regulate all aspects of employment, it cannot be understood to include the right to
temporarily withhold salary/wages without the consent of the employee. Any withholding of an employees wages by
an employer may only be allowed in the form of wage deductions under the circumstances provided in Article 113 of
the Labor Code.

Angeles| Bajana | Balladares | Brillantes | Briones | Cabansag | Callanta | Chua | David|


De Leon | Gomez | Lopez | Macalino | Nostratis | Padilla | Reynon | Santos | Tan |Velasco
4E / 4F - 2018-2019
Page 822 of 920
LABOR REVIEW DIGEST
Atty. Joyrich Golangco

Although it cannot be determined with certainty whether respondent worked for the entire period from November 16
to November 30, 2005, the consistent rule is that if doubt exists between the evidence presented by the employer
and that by the employee, the scales of justice must be tilted in favor of the latter.

2. NO. The Court agrees with the LA and the CA that respondent was forced to resign and was, thus, constructively
dismissed. What made it impossible, unreasonable or unlikely for respondent to continue working for SHS was the
unlawful withholding of his salary. For said reason, he was forced to resign. What is significant is that the respondent
prepared and served his resignation letter right after he was informed that his salary was being withheld.

It is worthy to note that in his resignation letter, respondent cited petitioners illegal and unfair labor practice as his
cause for resignation. Respondent lost no time in submitting his resignation letter and eventually filing a complaint for
illegal dismissal just a few days after his salary was withheld. These circumstances are inconsistent with voluntary
resignation and bolster the finding of constructive dismissal.

3. YES, respondent was constructively dismissed and, therefore, illegally dismissed. Although respondent was a
probationary employee, he was still entitled to security of tenure. Section 3 (2) Article 13 of the Constitution
guarantees the right of all workers to security of tenure. In using the expression all workers, the Constitution puts no
distinction between a probationary and a permanent or regular employee. This means that probationary employees
cannot be dismissed except for cause or for failure to qualify as regular employees.

Angeles| Bajana | Balladares | Brillantes | Briones | Cabansag | Callanta | Chua | David|


De Leon | Gomez | Lopez | Macalino | Nostratis | Padilla | Reynon | Santos | Tan |Velasco
4E / 4F - 2018-2019
Page 823 of 920
LABOR REVIEW DIGEST
Atty. Joyrich Golangco

8. San Miguel Properties Phils., Inc. vs. Gucaban


GR No: 153982 Date: July 18, 2011
Ponente: Peralta, J.

Doctrine: Resignation – the formal pronouncement or relinquishment of a position or office – is the voluntary act of
an employee who is in a situation where he believes that personal reasons cannot be sacrificed in favor of the
exigency of the service, and he has then no other choice but to disassociate himself from employment. The intent to
relinquish must concur with the overt act of relinquishment; hence,

FACTS: Gwendellyn Rose Gucaban (Gucaban) was well into the tenth year of her career as a licensed civil engineer
when she joined the workforce of petitioner San Miguel Properties Philippines, Inc. (SMPI) in 1991. Initially engaged
as a construction management specialist, she, by her satisfactory performance on the job, was promoted in 1994 and
1995, respectively, to the position of technical services manager, and then of project development manager. As
project development manager, she also sat as a member of the company’s management committee. She had been
in continuous service in the latter capacity until her severance from the company in February 1998.

In her complaint for illegal dismissal, Gucaban alleged that her separation from service was practically forced upon
her by management. She claimed that on January 27, 1998, she was informed by SMPI’s President and Chief
Executive Officer, Federico Gonzalez (Gonzalez), that the company was planning to reorganize its manpower in
order to cut on costs, and that she must file for resignation or otherwise face termination. Three days later, the
Human Resource Department allegedly furnished her a blank resignation form which she refused to sign. From then
on, she had been hounded by Gonzalez to sign and submit her resignation letter.

Gucaban complained of the ugly treatment which she had since received from Gonzalez and the management
supposedly on account of her refusal to sign the resignation letter. It was supposedly the extreme humiliation and
alienation that impelled her to submit a signed resignation letter on February 18, 1998.

Gucaban surmised that she had merely been tricked by SMPI into filing her resignation letter because it never
actualized its reorganization and streamlining plan; on the contrary, SMPI allegedly expanded its employee
population and also made new appointments and promotions to various other positions.

LA RULING: Labor Arbiter dismissed the complaint for lack of merit. The Labor Arbiter found no proven force,
coercion, intimidation or any other circumstance which could otherwise invalidate Gucaban’s resignation. He found
incredible Gucaban’s claim of humiliation and alienation, because the mere fact that she was excluded from the
meetings of the management committee would not be so humiliating and alienating as to compel her to decide to
leave the company.

NLRC/CA RULING: Finding that Gucaban has been illegally dismissed, it ordered her reinstatement without loss of
seniority rights and with full backwages, as well as ordered the award of damages and attorney’s fees.

APPEAL TO THE SC:

Petitioner's Contention: SMPI posits that the Court of Appeals’ finding of illegal dismissal was at best conjectural,
based as it is on a misapprehension of facts and on Gucaban’s self-serving allegations of alienation and humiliation
which, nevertheless, could not have given sufficient motivation for her to resign.

Respondent's Contention: Gucaban stands by the uniform findings of the NLRC and the Court of Appeals. In her
Comment on the Petition, she points out that indeed SMPI was unable to conclusively refute the allegations in her
complaint, particularly those which negate the voluntariness of her resignation.

ISSUE/S: whether or not there was valid resignation by Gucaban

HELD: NO. Resignation – the formal pronouncement or relinquishment of a position or office – is the voluntary act of
an employee who is in a situation where he believes that personal reasons cannot be sacrificed in favor of the
exigency of the service, and he has then no other choice but to disassociate himself from employment. The intent to
relinquish must concur with the overt act of relinquishment; hence, the acts of the employee before and after the
alleged resignation must be considered in determining whether he in fact intended to terminate his employment. In
illegal dismissal cases, fundamental is the rule that when an employer interposes the defense of resignation, on him

Angeles| Bajana | Balladares | Brillantes | Briones | Cabansag | Callanta | Chua | David|


De Leon | Gomez | Lopez | Macalino | Nostratis | Padilla | Reynon | Santos | Tan |Velasco
4E / 4F - 2018-2019
Page 824 of 920
LABOR REVIEW DIGEST
Atty. Joyrich Golangco

necessarily rests the burden to prove that the employee indeed voluntarily resigned. Guided by these principles, we
agree with the Court of Appeals that with the availing evidence, SMPI was unable to discharge this burden.

While indeed the abolition of Gucaban’s position as a consequence of petitioner’s supposed reorganization plan is
not the ground invoked in this case of termination, still, the question of whether or not there was such reorganization
plan in place at the time of Gucaban’s separation from the company, is material to the determination of whether her
resignation was of her own volition as claimed by SMPI, inasmuch as the facts of this case tell that Gucaban could
not have filed for resignation had Gonzalez not communicated to her the alleged reorganization plan for the
company.

True, while a reorganization of SMPI’s corporate structure might have indeed taken place as shown by these notices,
nevertheless, it happened only in the latter part of 1999 – or more than a year after Gucaban’s separation from the
company and incidentally, after she filed the instant complaint.

Angeles| Bajana | Balladares | Brillantes | Briones | Cabansag | Callanta | Chua | David|


De Leon | Gomez | Lopez | Macalino | Nostratis | Padilla | Reynon | Santos | Tan |Velasco
4E / 4F - 2018-2019
Page 825 of 920
LABOR REVIEW DIGEST
Atty. Joyrich Golangco

9. BMG RECORDS (PHILS.), INC. and JOSE YAP, JR. v. AIDA C. APARECIO and NATIONAL LABOR
RELATIONS COMMISSION
G.R. NO. 153290, September 5, 2007
AZCUNA

DOCTRINE: The acceptance by the employer of the employee’s resignation rendered the same effective.
Upon such acceptance, it may not be unilaterally withdrawn without the consent of petitioners. When the
employee later signified the intention of continuing his or her work, it was already up to the employer to
accept the withdrawal of his or her resignation. The mere fact that the withdrawal was not accepted does not
constitute illegal dismissal, the acceptance of the withdrawal of the resignation being the employer's sole
prerogative.

FACTS: BMG Records (Phils) Inc. hried Aida Aparecio as one of the promo girls in its Cenu branch. For working
from Monday to Sunday, she received a salary of P181/day. On May 25, 1998, Aparecio filed a complaint against
BMG and its branch manager Jose Yap, Jr. for illegal dismissal and non-payment of overtime pay, holiday pay,
premium pay for rest day, 13th month pay, SIL pay, and separation pay alleging that:

She was illegally dismissed or terminated from employment on April 30, 1998.
Before said date, she was asked by BMG to resign and will be paid all her benefits due and to execute a letter of
resignation.
In view of BMG’s insistence to prepare and execute a letter-resignation, even without proper accounting of any
accountability, Aparecio was lured, induced, and compelled to submit a letter of resignation believing on BMG’s
promise and assurance to pay all the benefits due her.
After executing the resignation letter, BMG did not make good its promise and instead did an accounting by
themselves in the absence of Aparecio and arrived on a computation that Aparecio’s liability per their accounting
reached P8,000.
Since they offered to pay a separation pay of only P12,000 minus the P8,000, they are ready to pay the balance
thereof any time;
Aparecio was under BMG’s employ for 7 years, 7 months, and 28 days when illegally terminated from her
employment. BMG, on the other hand, alleges that:
Aparecio was initially performing well as an employee but as years passed by she seemed to be complacent in the
performance of her job and had been comparing the salaries of promo girls in other companies.
It appeared that Aparecio was no longer interested in her job.
In 1998, Aparecio and two other promogirls Soco and Mutya intimated to their supervisor that they were intending to
resign and were requesting for some financial assistance.BMG made it clear that, as a company policy, an employee
who resigns from service is not enetitled to financial assistance, but considering the length of their service and due to
humanitarian consieration, it would accede to the request after they secure their respective clearances.
The 3 employees tendered their resignations, which were accepted.
When they processed the required individual clearance, it was found out that they incurred some shortages after
inventory which were then deducted from the amounts due them.
Soco and Mutya received their last salary and executed their releases and quitclaims.
Except for the financial assistance, Aparecio also obtained the same yet refused to sign the release and quitclaim,
protesting P9,170.12 deducted from the financial assistance.

LA: Complaint was dismissed.


NLRC: There was illegal dismissal. MR was denied.
CA: NLRC was affirmed in toto. MR was denied. Hence, this petiiton.

ISSUE: Was there fraud, undue influence, intimidation, and/or mistake upon Aparecio’s resignation from BMG,
thereby making BMG and Yap guilty of illegal dismissal?

HELD: No. Fraud exists only when, through insidious words or machinations, the other party is induced to act and
without which, the latter would not have agreed to. Circumstances evidencing fraud and misrepresentation are as
varied as the people who perpetrate it, each assuming different shapes and forms and may be committed in as many
different ways. Fraud and misrepresentation are, therefore, never presumed; it must be proved by clear and
convincing evidence and not mere preponderance of evidence. Aparecio alleged that her resignation was wrongfully
obtained when BMG and Yap did not keep the promise of giving her employment benefits and financial assistance
without any deductions. Without a showing of the nature and extent of such "inducement," however, such submission
fails to establish that there was in fact a deception on the part of BMG and Yap. Even if it is considered that there
was an assurance given by BMG and Yap and that they later reneged on their promise, there is no injustice made

Angeles| Bajana | Balladares | Brillantes | Briones | Cabansag | Callanta | Chua | David|


De Leon | Gomez | Lopez | Macalino | Nostratis | Padilla | Reynon | Santos | Tan |Velasco
4E / 4F - 2018-2019
Page 826 of 920
LABOR REVIEW DIGEST
Atty. Joyrich Golangco

since Aparecio, who only questioned the manner by which the inventory was conducted – that it was held without her
presence – but did not categorically deny her accountabilities with BMG, would unjustly be enriched without the
deduction. Aparecio did not adduce any competent evidence to prove that force or threat was applied by petitioners.
For intimidation to vitiate consent, the following requisites must be present:

(1) that the intimidation caused the consent to be given;


(2) that the threatened act be unjust or unlawful;
(3) that the threat be real or serious, there being evident disproportion between the evil and the resistance which all
men can offer, leading to the choice of doing the act which is forced on the person to do as the lesser evil; and
(4) that it produces a well-grounded fear from the fact that the person from whom it comes has the necessary means
or ability to inflict the threatened injury to his person or property. In this case, not one of these essential elements was
amply proven by Aparecio. Bare allegations of threat or force do not constitute substantial evidence to support a
finding of forced resignation.

That there was a "strong and irresistible economic pressure originating from BMG and Yap if only to push
Aparecio into accepting the offer" is not supported by any evidence in the records but is merely based on
conjectures and guesswork. There is no concrete evidence, direct or circumstantial, showing that undue
influence was used by BMG and Yap in such a way that it took improper advantage of its power over the will of
Aparecio and deprived the latter of a reasonable freedom of choice.

Resignation is the voluntary act of an employee who is in a situation where one believes that personal reasons
cannot be sacrificed in favor of the exigency of the service, and one has no other choice but to dissociate oneself
from employment. It is a formal pronouncement or relinquishment of an office, with the intention of relinquishing the
office accompanied by the act of relinquishment. As the intent to relinquish must concur with the overt act of
relinquishment, the acts of the employee before and after the alleged resignation must be considered in determining
whether in fact, he or she intended to sever from his or her employment.

Circumstances surrounding Aparecio’s resignation showed her intent to resign:


Aparecio already communicated to other people that she was about to resign to look for a better paying job since
she had been complaining that employees like her in other companies were earning much more.
Prior to the submission of her resignation letter, Aparecio and two other promo girls, Soco and Mutya, approached
their supervisor, intimated their desire to resign, and requested that they be given financial assistance, which
petitioners granted on the condition that deductions would be made in case of shortage after inventory.
Aparecio, Soco, and Mutya submitted their duly signed resignation letters, which were accepted by
petitioners.
Aparecio already initiated the processing of her clearance; thus, she was able to receive her last salary, 13th month
pay, and tax refund but refused to receive the financial assistance less the deductions made. The acceptance by
petitioners of Aparecio's resignation rendered the same effective. Upon such acceptance, it may not be unilaterally
withdrawn without the consent of petitioners. When the employee later signified the intention of continuing his or her
work, it was already up to the employer to accept the withdrawal of his or her resignation. The mere fact that the
withdrawal was not accepted does not constitute illegal dismissal, the acceptance of the withdrawal of the resignation
being the employer's sole prerogative.

Once an employee resigns and his resignation is accepted, he no longer has any right to the job. If the employee
later changes his mind, he must ask for approval of the withdrawal of his resignation from his employer, as if he were
re-applying for the job. It will then be up to the employer to determine whether or not his service would be continued.
If the employer accepts said withdrawal, the employee retains his job. If the employer does not, the employee cannot
claim illegal dismissal for the employer has the right to determine who his employees will be. To say that an
employee who has resigned is illegally dismissed, is to encroach upon the right of employers to hire persons who will
be of service to them.

What transpired here was caused by an employee's error of judgment and not by the employer's application of
means vitiating the consent to resign. It would be utterly unfair to attribute to petitioners the commission of illegal
dismissal and to impose upon them the burden of accepting back Aparecio who unequivocally manifested her intent
and willingness to sever her employment ties.

The petition was granted.

Angeles| Bajana | Balladares | Brillantes | Briones | Cabansag | Callanta | Chua | David|


De Leon | Gomez | Lopez | Macalino | Nostratis | Padilla | Reynon | Santos | Tan |Velasco
4E / 4F - 2018-2019
Page 827 of 920
LABOR REVIEW DIGEST
Atty. Joyrich Golangco

10. TATEL vs JLFP INVESTIGATIONS


GR No: 206942 Date: February 25, 2015
Ponente: Perlas-Bernabe, J.

Doctrine:
When a "floating status" lasts for more than six (6) months, the employee may be considered to have been
constructively dismissed.

FACTS:
On March 14, 1998, respondent JLFP Investigation Security Agency, Inc. (JLFP), a business engaged as a
security agency, hired Tatel as one of its security guards.
Tatel alleged that he was last posted at BaggerWerken Decloedt En Zoon (BaggerWerken) located at the
Port Area in Manila. He was required to work twelve (12) hours everyday from Mondays through Sundays and
received only P12,400.00 as monthly salary. On October 14, 2009, Tatel filed a complaint before the NLRC against
respondents for underpayment of salaries and wages, non-payment of other benefits, 13th month pay, and attorney's
fees (underpayment case).
On October 24, 2009, Tatel was placed on "floating status"; thus, on May 4, 2010, or after the lapse of six
(6) months therefrom, without having been given any assignments, he filed another complaint for illegal dismissal,
reinstatement, backwages, refund of cash bond deposit amounting to P25,400.00, attorney's fees, and other money
claims (illegal dismissal case).
In their defense, respondents denied that Tatel was dismissed and averred that they removed the latter from
his post at BaggerWerken on August 24, 2009 because of several infractions he committed while on duty. Thereafter,
he was reassigned at SKI from September 16, 2009 to October 12, 2009, and last posted at IPVG from October 21 to
23, 2009.
Notwithstanding the pendency of the underpayment case, respondents sent a Memorandum dated
November 26, 2009 directing Tatel to report back to work, noting that the latter last reported to the office on October
26, 2009. However, despite receipt of the said memorandum, respondents averred that Tatel ignored the same and
failed to appear; hence, he was deemed to have abandoned his work. Moreover, respondents pointed out that Tatel
made inconsistent statements when he declared in the underpayment case that he was employed in March 1997 with
a salary of P12,400.00 per month and dismissed on October 13, 2009, while declaring in the illegal dismissal
case that his date of employment was March 14, 1998, with a salary of P6,200.00 per month, and that he was
dismissed on October 24, 2009.
In his reply, Tatel admitted having received on December 11, 2009 the November 26, 2009 Memorandum
directing him to report back to work for reassignment. However, when he went to the JLFP office, he was merely
advised to "wait for possible posting."26 He repeatedly went back to the office for reassignment, but to no avail. He
likewise refuted respondents' claim that he abandoned his work, insisting that after working for JLFP for more than
eleven (11) years, it was illogical for him to refuse any assignments, more so, to abandon his work and security of
tenure without justifiable reasons.

LA RULING:
Dismissed Tatel's illegal dismissal complaint for lack of merit

NLRC RULING
Reversed and set aside the LA's Decision and found Tatel to have been illegally dismissed.

CA RULING:
Reversed and set aside the NLRC's February 9, 2011 Decision and reinstated the LA's September 20, 2010 Decision
dismissing the illegal dismissal complaint filed by Tatel.

Petitioner's Contention:
Tatel's insistence that he was illegally dismissed when, after he was put on "floating status" on October 24, 2009,
respondents no longer gave him assignments or postings, and the period therefor had lasted for more than six (6)
months.

Respondent's Contention:
Respondents maintained that Tatel abandoned his work, and that his inconsistent statements before the labor
tribunals regarding his work details rendered his claim of illegal dismissal suspect.

ISSUE/S:
Whether or not Tatel was constructively dismissed

Angeles| Bajana | Balladares | Brillantes | Briones | Cabansag | Callanta | Chua | David|


De Leon | Gomez | Lopez | Macalino | Nostratis | Padilla | Reynon | Santos | Tan |Velasco
4E / 4F - 2018-2019
Page 828 of 920
LABOR REVIEW DIGEST
Atty. Joyrich Golangco

HELD: YES
After a judicious perusal of the records, the Court is convinced that Tatel was constructively, not actually, dismissed
after having been placed on "floating status" for more than six (6) months, reckoned from October 24, 2009, the day
following his removal from his last assignment with IPVG on October 23, 2009, and not on August 24, 2009 as
erroneously held by the NLRC.

In Superstar Security Agency, Inc. and/or Col. Andrada v. NLRC, the Court ruled that placing an employee on
temporary "off-detail" is not equivalent to dismissal provided that such temporary inactivity should continue only for a
period of six (6) months. In security agency parlance, being placed "off-detail" or on "floating status" means "waiting
to be posted." In Salvaloza v. NLRC, the Court further explained the nature of the "floating status," to wit:
Temporary "off-detail" or "floating status" is the period of time when security guards are in between assignments or
when they are made to wait after being relieved from a previous post until they are transferred to a new one. It takes
place when the security agency's clients decide not to renew their contracts with the agency, resulting in a situation
where the available posts under its existing contracts are less than the number of guards in its roster. It also happens
in instances where contracts for security services stipulate that the client may request the agency for the replacement
of the guards assigned to it even for want of cause, such that the replaced security guard may be placed on
temporary "off-detail" if there are no available posts under the agency's existing contracts. During such time, the
security guard does not receive any salary or any financial assistance provided by law. It does not constitute a
dismissal, as the assignments primarily depend on the contracts entered into by the security agencies with third
parties, so long as such status does not continue beyond a reasonable time. When such a "floating status" lasts
for more than six (6) months, the employee may be considered to have been constructively dismissed.

Relative thereto, constructive dismissal exists when an act of clear discrimination, insensibility, or disdain, on the part
of the employer has become so unbearable as to leave an employee with no choice but to forego continued
employment, or when there is cessation of work because continued employment is rendered impossible,
unreasonable, or unlikely, as an offer involving a demotion in rank and a diminution in pay.

In this case, respondents themselves claimed that after having removed Tatel from his post at BaggerWerken on
August 24, 2009 due to several infractions committed thereat, they subsequently reassigned him to SKI from
September 16, 2009 to October 12, 2009 and then to IPVG from October 21 to 23, 2009. Thereafter, and until Tatel
filed the instant complaint for illegal dismissal six (6) months later, or on May 4, 2010, he was not given any other
postings or assignments. While it may be true that respondents summoned him back to work through the November
26, 2009 Memorandum, which Tatel acknowledged to have received on December 11, 2009, records are bereft of
evidence to show that he was given another detail or assignment. As the "off-detail" period had already lasted for
more than six (6) months, Tatel is therefore deemed to have been constructively dismissed.

In this regard, the Court concurs with the finding of the NLRC that respondents failed to establish that Tatel
abandoned his work. To constitute abandonment, two elements must concur: (a) the failure to report for work or
absence without valid or justifiable reason, and (b) a clear intention to sever the employer-employee relationship, with
the second element as the more determinative factor and being manifested by some overt acts. Mere absence is not
sufficient. The employer has the burden of proof to show a deliberate and unjustified refusal of the employee to
resume his employment without any intention of returning. Abandonment is incompatible with constructive dismissal.

The charge of abandonment in this case is belied by the high improbability of Tatel intentionally abandoning his work,
taking into consideration his length of service and, concomitantly, his security of tenure with JLFP. As the NLRC had
opined, no rational explanation exists as to why an employee who had worked for his employer for more than ten (10)
years would just abandon his work and forego whatever benefits he may be entitled to as a consequence thereof. As
such, respondents failed to sufficiently establish a deliberate and unjustified refusal on the part of Tatel to resume his
employment, which therefore leads to the logical conclusion that the latter had no such intention to abandon his work.

Moreover, Tatel refuted respondents' allegation that he did not heed their directive to return to work following his
receipt of the November 26, 2009 Memorandum. The Court finds no compelling reason not to give credence to such
rebuff, especially in light of the filing of the instant complaint for illegal dismissal. An employee who forthwith takes
steps to protest his layoff cannot, as a general rule, be said to have abandoned his work, and the filing of the
complaint is proof enough of his desire to return to work, thus negating any suggestion of abandonment. As the Court
sees it, it is simply incongruent for Tatel to refuse any offer of an assignment and thereafter, seek redress by filing a
case for illegal dismissal.

Angeles| Bajana | Balladares | Brillantes | Briones | Cabansag | Callanta | Chua | David|


De Leon | Gomez | Lopez | Macalino | Nostratis | Padilla | Reynon | Santos | Tan |Velasco
4E / 4F - 2018-2019
Page 829 of 920
LABOR REVIEW DIGEST
Atty. Joyrich Golangco

12. PAREDES V. FEED THE CHILDREN PHILIPPINES


G.R. NO. 184397, SEPTEMBER 09, 2015
PERALTA J.

DOCTRINE:

Case law holds that constructive dismissal occurs when there is cessation of work because continued
employment is rendered impossible, unreasonable or unlikely; when there is a demotion in rank or
diminution in pay or both; or when a clear discrimination, insensibility, or disdain by an employer becomes
unbearable to the employee. The test is whether a reasonable person in the employee's position would have
felt compelled to give up his position under the circumstances.

FACTS:

Respondent Feed the Children Philippines, Inc. (FTCP) is a non-stock, non-profit, and non-government organization.
Its objective is to provide food, clothing, educational supplies and other necessities of indigent children worldwide.
Respondents were members of the FTCP Board of Trustees and Executive Committee of FTCP.

Petitioner Rosalinda Paredes was FTCP's National Director. Petitioner was first hired by FTCP in 1999 as Country
Director. Her contract was renewed several times until her last contract for the period from October 1, 2004 to
September 30, 2007. Her initial salary was US$1000.00 and then later, she was paid P70,000.00 aside from other
benefits and allowances.

On August 12, 2005, forty-two FTCP employees signed a petition expressing their complaints against alleged
detestable practices of petitioner, to wit: seeking exemption from policies which she herself had approved;
withholding organization funds despite approval of its release; procuring health insurance for herself without paying
her share of the premium; and receiving additional fees contrary to the terms of her contract.

The next day, the staff of FTCP called a meeting to submit their petition. The group demanded for outright solution.
However, the Board members told them that they should follow a process.

Petitioner learned that Program Manager Fostanes and his co-employees prepared a petition questioning her
leadership and management. She filed an administrative complaint against Fostanes but it was not acted upon.

When the Board convened for a meeting on August 28, 2005, petitioner was not allowed to participate.

Consequently, the Board decided that: Acting Board Chair Lao will issue a back-to-work memorandum and status
quo to ensure that all the scheduled tasks be accomplished; there will be a Supervisory Team, composing of Lao and
Escobia, that will draw a definite work plan and be compensated; the Supervisory Team will not replace the functions
of the National Director; and FTCP will hire an independent professional management and financial auditor.

Petitioner was later suspended because of her indifferent attitude and unjustified refusal to submit to an audit. Before
it could be implemented, respondent FTCP received her resignation letter.

On October 29, 2005, the Board accepted her resignation with the condition that its effectivity be moved to November
30, 2005.

On November 2, 2005, petitioner filed a Complaint for illegal dismissal, claiming that she was forced to resign, thus,
was constructively dismissed, and impleaded Lao, Paradiang and Escobia in their personal capacities.

LA/NLRC RULING:

The LA dismissed the case for lack of merit.

Undaunted, petitioner appealed the decision to the NLRC. The NLRC reversed and set aside the decision of the LA.

CA RULING:

The CA ruled for the respondents declaring that private respondent to have voluntarily resigned from her
employment/consultancy with FTCP.

Angeles| Bajana | Balladares | Brillantes | Briones | Cabansag | Callanta | Chua | David|


De Leon | Gomez | Lopez | Macalino | Nostratis | Padilla | Reynon | Santos | Tan |Velasco
4E / 4F - 2018-2019
Page 830 of 920
LABOR REVIEW DIGEST
Atty. Joyrich Golangco

ISSUE:

WON there was a valid dismissal in the present case.

HELD:

 It is settled that in a special civil action for certiorari under Rule 65, the issues are limited to errors of
jurisdiction or grave abuse of discretion. However, in labor cases elevated to it via petition for certiorari, the
CA is empowered to evaluate the materiality and significance of the evidence alleged to have been
capriciously, whimsically, or arbitrarily disregarded by the NLRC in relation to all other evidence on record.

The CA can grant this prerogative writ when the factual findings complained of are not supported by the
evidence on record; when it is necessary to prevent a substantial wrong or to do substantial justice; when
the findings of the NLRC contradict those of the LA; and when necessary to arrive at a just decision of the
case. To make this finding, the CA necessarily has to view the evidence if only to determine if the NLRC
ruling had basis in evidence.

Contrary to petitioner's contention, the CA, by express legal mandate and pursuant to its equity jurisdiction,
may review factual findings and evidence of the parties to determine whether the NLRC gravely abused its
discretion in its findings. Since this Court finds that the findings of the LA and NLRC contradicting and that
the findings of NLRC are not supported by the evidence on record, we rule that it is within the CA's power to
review the factual findings of the NLRC. Accordingly, this Court does not find erroneous the course that the
CA took in resolving that petitioner was not constructively dismissed.

 Since petitioner admittedly resigned, it is incumbent upon her to prove that her resignation was involuntary
and that it was actually a case of constructive dismissal with clear, positive and convincing evidence.

Case law holds that constructive dismissal occurs when there is cessation of work because continued
employment is rendered impossible, unreasonable or unlikely; when there is a demotion in rank or
diminution in pay or both; or when a clear discrimination, insensibility, or disdain by an employer becomes
unbearable to the employee. The test is whether a reasonable person in the employee's position would
have felt compelled to give up his position under the circumstances.

In this case, petitioner cannot be deemed constructively dismissed. She failed to present clear and positive
evidence that respondent FTCP, through its Board of Trustees, committed acts of discrimination,
insensibility, or disdain towards her which rendered her continued employment unbearable or forced her to
terminate her employment from the respondent. As settled, bare allegations of constructive dismissal, when
uncorroborated by the evidence on record, cannot be given credence.

It is highly unlikely and incredible for someone of petitioner's position and educational attainment to so
easily succumb to individual respondents' alleged harassment without defending herself. In fact, records
reveal that she wrote directly to Jones when her contract was not to be renewed and whenever she felt
threatened. She vehemently opposed the audit and openly disobeyed the Board when she was not informed
of the scope. She, along with other management staff, questioned the meetings of the Execom that they
were not informed. It is also noted that her husband is a lawyer and that she employed lawyers who sent a
series of demand letters to the Board to provide her the details of the audit and even ordered the Board to
desist from pursuing the audit.

There was no urgency for petitioner to submit her resignation letter. In fact, the day before it was given, she
and other management staff requested for a dialogue with the Board to address the issue regarding the
management and financial audit.52 It is, therefore, improbable that her continued employment is rendered
impossible or unreasonable.

Records do not show any demotion in rank or a diminution in pay made against her. Petitioner claimed that
the fact that the Supervisory Team performed her functions and issued memorandum directly to her
subordinates, and her being barred from subsequent Execom meetings constituted constructive dismissal.
However, there was no evidence to corroborate her claim of usurpation. She did not present evidence of the
supposed direct memorandum issued by the Supervisory Team to the staff. Aside from the minutes of the
September 29, 2005 meeting of the Execom, there was no other proof of petitioner's exclusion from other

Angeles| Bajana | Balladares | Brillantes | Briones | Cabansag | Callanta | Chua | David|


De Leon | Gomez | Lopez | Macalino | Nostratis | Padilla | Reynon | Santos | Tan |Velasco
4E / 4F - 2018-2019
Page 831 of 920
LABOR REVIEW DIGEST
Atty. Joyrich Golangco

subsequent Execom meetings.

 We held that the act of the employer moving the effectivity of the resignation is not an act of harassment.
The 30-day notice requirement for an employee's resignation is actually for the benefit of the employer who
has the discretion to waive such period. Its purpose is to afford the employer enough time to hire another
employee if needed and to see to it that there is proper turn-over of the tasks which the resigning employee
may be handling.

Such rule requiring an employee to stay or complete the 30-day period prior to the effectivity of his
resignation becomes discretionary on the part of management as an employee who intends to resign may
be allowed a shorter period before his resignation becomes effective.

Thus, the act of respondents moving the effectivity date of petitioner's resignation to a date earlier than what
she had stated cannot be deemed malicious. This cannot be viewed as an act of harassment but merely the
exercise of respondent's management prerogative. We cannot expect employers to maintain in their employ
employees who intend to resign, just so the latter can have continuous work as they look for a new source
of income.

Angeles| Bajana | Balladares | Brillantes | Briones | Cabansag | Callanta | Chua | David|


De Leon | Gomez | Lopez | Macalino | Nostratis | Padilla | Reynon | Santos | Tan |Velasco
4E / 4F - 2018-2019
Page 832 of 920
LABOR REVIEW DIGEST
Atty. Joyrich Golangco

12. SILVERTEX WEAVING CORPORATION vs. CAMPO


G.R. No. 211411, March 16, 2016
REYES, J.:

DOCTRINE: In illegal dismissal cases, fundamental is the rule that when an employer interposes the defense
of resignation, on him necessarily rests the burden to prove that the employee indeed voluntarily resigned.

FACTS:

The case stems from a complaint for illegal dismissal and monetary claims filed by Teodora F. Campo (respondent)
against the petitioners, wherein she claimed that she worked for STWC as a weaving machine operator beginning
June 11, 1999, until she was unlawfully dismissed from employment on November 21, 2010. Prior to her dismissal,
she was suspended for one week beginning November 14, 2010 after a stitching machine that she was operating
overheated and emitted smoke on November 13, 2010. When the respondent tried to report back to work on
November 21, 2010, she was denied entry by the STWC's security guard, reportedly upon the instructions of Arcenal.

For their defense, the petitioners argued that the respondent, who was hired only in June 2009, voluntarily resigned
from STWC after she was reprimanded for poor job performance. They submitted a handwritten resignation
letter allegedly executed by the respondent on November 13, 2010, together with the Waiver, Release and
Quitclaims Statement that she supposedly signed following her receipt of P30,000.00 from STWC. The respondent,
however, denied having executed the resignation letter, the quitclaim, and the supposed receipt of the P30,000.00.
The Labor Arbiter dismissed the complaint for lack of merit. The NLRC affirmed the decision of the LA. However, the
CA ruled in favor of the respondent and ordered the reinstatement of the latter.

ISSUE:
Whether or not the respondent resigned from her employment.

RULING:

No. Although the report from the PNP provided that the signature on the resignation letter matched the supposed
handwriting of the respondent in her bio-data dated April 1, 2009,26 the conflicting findings and the fact that only one
of the 18 documents used as reference for the examination matched the signature in the letter only supported the
respondent's claim that she did not execute the resignation letter. Furthermore, there was no showing that the
sample signature considered by the PNP Crime Laboratory was a genuine signature of the respondent, rendering it
insufficient basis for the conclusion arrived at by the document examiner and relied upon by the NLRC.

Clearly then, given the vehement claim of the respondent that her signature on the resignation letter was a mere
forgery, the evidence presented by the petitioners to establish their defense of voluntary resignation failed to suffice.
Several other indicators cast doubt on the letter's authenticity.

Angeles| Bajana | Balladares | Brillantes | Briones | Cabansag | Callanta | Chua | David|


De Leon | Gomez | Lopez | Macalino | Nostratis | Padilla | Reynon | Santos | Tan |Velasco
4E / 4F - 2018-2019
Page 833 of 920
LABOR REVIEW DIGEST
Atty. Joyrich Golangco

13.Divine Word College Of Laoag vs. Shirley B. Mina, as Heir-Substitute of the late Delfin A. Mina
G.R. No. 195155 Date: April 13, 2016
Ponente: Reyes, J.

Doctrine: Constructive dismissal is a dismissal in disguise. There is cessation of work in constructive dismissal
because ‘“continued employment is rendered impossible, unreasonable or unlikely, as an offer involving a demotion
in rank or a diminution in pay’ and other benefits.” To be considered as such, an act must be a display of utter
discrimination or insensibility on the part of the employer so intense that it becomes unbearable for the employee to
continue with his employment.

FACTS:
PETITIONER Divine Word College of Laoag (DWCL) is a non-stock educational institution offering Catholic
education to the public. It is run by the Society of Divine Word (SVD), a congregation of Catholic priests.

It first employed respondent Delfin A. Mina in 1971 as a high school teacher, and later on a high school principal at
the Academy of St. Joseph (ASJ), a school run by the SVD. On June 1, 1979, he transferred to DWCL and was
accorded a permanent status. In 2002, he was transferred to DWCL’s college department as an associate professor
III. On June 1, 2003, he was assigned as the college laboratory custodian of the school of nursing and was divested
of his teaching load, effective June 1, 2003 until May 21, 2004, subject to automatic termination without need for any
further notification. He was the only one among several teachers transferred to the college department who was
divested of teaching load.

In early June 2004, Mina was offered early retirement by DWCL. He initially declined the offer. He later received a
memorandum from the office of DWCL’s dean enumerating specific acts of gross or habitual negligence,
insubordination and reporting for work under the influence of alcohol. Sensing that it was pointless to continue
employment with DWCL, he requested that his retirement date be adjusted to September 2004 to avail of better
benefits which was denied. Instead, he was paid only P275,513.10 as retirement pay. It was made to appear that his
services were terminated by reason of redundancy to avoid any tax implications. He was also made to sign a deed of
waiver and quitclaim.

Mina filed a case for illegal dismissal and recovery of separation pay and other money claims.

PETITION TO THE SC:

ISSUE/S:
1. Whether or not there was illegal dismissal

HELD:
Yes. Constructive dismissal is a dismissal in disguise. There is cessation of work in constructive dismissal because
‘“continued employment is rendered impossible, unreasonable or unlikely, as an offer involving a demotion in rank or
a diminution in pay’ and other benefits.” To be considered as such, an act must be a display of utter discrimination or
insensibility on the part of the employer so intense that it becomes unbearable for the employee to continue with his
employment. The law recognizes and resolves this situation in favor of employees in order to protect their rights and
interests from the coercive acts of the employer.

In this case, Mina’s transfer clearly amounted to a constructive dismissal. For almost 22 years, he was a high school
teacher enjoying a permanent status in DWCL’s high school department. In 2002, he was appointed as an associate
professor at the college department but shortly thereafter, or on June 1, 2003, he was appointed as a college
laboratory custodian, which is a clear relegation from his previous position. Not only that. He was also divested of his
teaching load. His appointment even became contractual in nature and was subject to automatic termination after
one year “without any further notification.” Aside from this, Mina was the only one among the high school teachers
transferred to the college department who was divested of teaching load. More importantly, DWCL failed to show any
reason for Mina’s transfer and that it was not unreasonable, inconvenient, or prejudicial to him.

Also, the CA correctly ruled that Mina’s appointment as laboratory custodian was a demotion. There is demotion
when an employee occupying a highly technical position requiring the use of one’s mental faculty is transferred to
another position, where the employee performed mere mechanical work – virtually a transfer from a position of
dignity to a servile or menial job. The assessment whether Mina’s transfer amounted to a demotion must be done in
relation to his previous position, that is, from an associate college professor, he was made a keeper and inventory-
taker of laboratory materials. Clearly, Mina’s new duties as laboratory custodian were merely perfunctory and a far

Angeles| Bajana | Balladares | Brillantes | Briones | Cabansag | Callanta | Chua | David|


De Leon | Gomez | Lopez | Macalino | Nostratis | Padilla | Reynon | Santos | Tan |Velasco
4E / 4F - 2018-2019
Page 834 of 920
LABOR REVIEW DIGEST
Atty. Joyrich Golangco

cry from his previous teaching job, which involved the use of his mental faculties. And while there was no proof
adduced showing that his salaries and benefits were diminished, there was clearly a demotion in rank. As was stated
in Blue Dairy Corporation v. NLRC, 373 Phil. 179 (1999), “it was virtually a transfer from a position of dignity to a
servile or menial job.”

Angeles| Bajana | Balladares | Brillantes | Briones | Cabansag | Callanta | Chua | David|


De Leon | Gomez | Lopez | Macalino | Nostratis | Padilla | Reynon | Santos | Tan |Velasco
4E / 4F - 2018-2019
Page 835 of 920
LABOR REVIEW DIGEST
Atty. Joyrich Golangco

14. PASCUA V. BANKWISE

Doctrine: In situations where special qualifications are required for employment, such as a l\l[aster's degree
or experience as a corporate executive, prospective employees are at a better position to bargain or make
demands from the employer. 81 Employees with special qualifications would be on equal footing with their
employers, and thus, would need a lesser degree of protection from the State than an ordinary rank-and-file
worker.

Facts: Pascua was employed by Bankwise as its Executive Vice President for Marketing. Philippine Veterans Bank
and Bankwise entered into a Memorandum of Agreement for the purchase of Bankwise's entire outstanding capital
stock.5 On January 12, 2005, Philippine Veterans Bank allegedly assumed full control and management of
Bankwise.6 Philippine Veterans Bank allegedly elected new members of the Board of Directors and appointed a new
set of officers, including the President and Chief Operating Officer. 7 Pascua was reassigned to a Special Accounts
Unit but his duties, functions, and responsibilities were not clearly delineated or defined.

Pascua was informed by Roberto A. Buhain (Buhain), President of Bankwise, that as part of the merger or trade-off
agreement with Philippine Veterans Bank, he should tender his resignation.9 Buhain assured Pascua that he would
be paid all his money claims during this transition. 10 Instead of tendering his resignation, Pascua wrote a letter
dated February 7, 2005, wherein he pleaded, among others, that he stay in office until the end of the year. 11 Seeing
as Pascua had yet to submit his resignation, Vicente Campa (Campa), a director of Bankwise, told him that it was
imperative that he submit his resignation and assured his continued service with Philippine Veterans Bank. Based on
Campa's assurance, Pascua tendered his resignation on February 22, 2005. His letter of resignation read:

S[R: IN ACCORDANCE WITH THE INSTRUCTIONS OF THE PREVIOUS OWNERS OF THE BANK, I HEREBY
TENDER MY RESIGNATION FROM THE BANK.

Pascua allegedly inquired from Buhain how his money claims would be paid in view of "the passive attitude" of the
banks. Buhain allegedly assured him that he already sought a meeting with Campa on the matter. During the meeting
Campa also assured him that all his money claims would be paid by the previous owners of Bankwise.

The Labor Arbiter dismissed the Complaint on the ground that Pascua had voluntarily resigned. The Labor Arbiter
relied on Pascua's resignation letter dated February 22, 2005 and paragraph 8 of his Contract of Employment22
stating that no verbal agreement between the employee and Bankwise may alter the terms of employment. The
Labor Arbiter found that there was no evidence in writing to prove the alleged private agreement among Pascua,
Buhain, and Campa.

The National Labor Relations Commission reversed the Labor Arbiter's findings and held that Pascua was
constructively dismissed.25 It found that Pascua was separated from service as part of the merger or trade-off deal
between Bankwise and Philippine Veterans Bank and was forced to accept his separation from service on the
promise that he would be paid severance pay and his other benefits.

During the pendency of the Petition for Certiorari with the Court of Appeals, the Monetary Board of the BangkoSentral
ng Pilipinas determined that Bankwise was insolvent and adopted Resolution No. 157 forbidding Bankwise from
further doing business in the Philippines.32 In the same Resolution, the Monetary Board placed Bankwise under
receivership and designated Philippine Deposit Insurance Corporation as its receiver.33 On October 30, 2008, the
Monetary Board issued Resolution No. 1386 directing the Philippine Deposit Insurance Corporation to proceed with
the liquidation of Bankwise.

Court of Appeals rendered its assailed Decision, finding that Pascua was constructively dismissed but held that only
Bankwise should be made liable to Pascua for his money claims.

The Court of Appeals found that there was no certificate of merger between Bankwise and Philippine Veterans Bank;
hence, Bankwise retained its separate corporate identity.

Pascua argues that the Court of Appeals erroneously absolved Philippine Veterans Bank of its liability since it had
already taken over the management and business operations of Bankwise by the time he was constructively
dismissed.

Bankwise also contends that assuming Pascua was enticed to resign in exchange for severance pay, it should not be
held liable for the actions of Buhain and Campa, who act~d beyond their authority.48 It insists that paragraph 8 of

Angeles| Bajana | Balladares | Brillantes | Briones | Cabansag | Callanta | Chua | David|


De Leon | Gomez | Lopez | Macalino | Nostratis | Padilla | Reynon | Santos | Tan |Velasco
4E / 4F - 2018-2019
Page 836 of 920
LABOR REVIEW DIGEST
Atty. Joyrich Golangco

Pascua's Contract of Employment states that no verbal agreement can alter or vary the terms of the contract unless it
is reduced in writing.49 It aJleged that even assuming it was liable to Pascua, the liability could not be enforced since
it was undergoing liquidation by the Philippine Deposit Insurance Corporation.

For its part, Philippine Veterans Bank asserts that it is a distinct and separate entity from Bankwise since the
Memorandum of Agreement between them was not consummated. 52 Even assuming that their Memorandum of
Agreement was consummated, Bankwise expressly freed Philippine Veterans Bank from liability arising from money
claims of its employees. 53 It also points out that even if Pascua was found to have been constructively dismissed,
only Bankwise's corporate officers should be held liable for their unauthorized acts. 54 Philippine Veterans Bank
likewise posits that Pascua was not constructively dismissed since he had voluntarily resigned. It points out three (3)
letters of resignation that Pascua drafted demanding payment of his severance pay according to the terms he had
specified. It argues that Pascua voluntarily resigned knowing that it was acquiring Bankwise and it is not obliged to
absorb Bankwise's employees.

Issue: Whether or not Pascua was constructively dismissed.

Ruling: The employer has the burden of proving, in illegal dismissal cases, that the employee was dismissed for a
just or authorized cause. Even if the employer claims that the employee resigned, the em~loyer still has the burden
of proving that the resignation was voluntary. 7 It is constructive dismissal when resignation "was made under
compulsion or under circumstances approximating compulsion, such as when an employee's act of handing in his [or
her] resignation was a reaction to circumstances leaving him [or her] no alternative but to resign."71

"Resignation is the voluntary act of an employee who is in a situation where one believes that personal reasons
cannot be sacrificed in favor of the exigency of the service, and one has no other choice but to dissociate oneself
from employment."72 In order to prove that resignation is voluntary, "the acts of the E:mployee before and after the
alleged resignation must be considered in determining whether he or she, in fact, intended to sever his or her
employment." Pascua wrote three (3) letters addressed to Bankwise's officers.

However, this is the only evidence that shows Pascua was unwilling to resign. Pascua admitted that he voluntarily
sent a resignation letter on the condition that his money claims would be made. 75 Thus, his second letter was a
reluctant acceptance of his fate containing only one (1) line: IN ACCORDANCE WITH THE INSTRUCTIONS OF THE
PREVIOUS OWNERS OF THE BANK, I HEREBY TENDER MY RESIGNATION FROM THE BANK.

In situations where special qualifications are required for employment, such as a l\l[aster's degree or experience as a
corporate executive, prospective employees are at a better position to bargain or make demands from the employer.
81 Employees with special qualifications would be on equal footing with their employers, and thus, would need a
lesser degree of protection from the State than an ordinary rank-and-file worker.

Pascua, as the Head of Marketing with annual salary of P2,250,000.00, 82 would have "Qeen.in possession of the
special qualifications needed for his post. He would have supervised several employees in his long years in service
and might have even processed their resignation letters. He would have been completely aware of the implications of
signing a categorically worded resignation letter. If he did not intend to resign, he would not have submitted a
resignation letter. He would have continued writing letters to Bankwise signifying his continued refusal to resign.
Pascua's resignation letter, however, was unconditional. It contained no reservations that it was premised on his
subsequent claim for severance pay and other benefits. His resignation was also accepted by his employers. In this
instance, Pascua is not considered to have been constructively dismissed. Pascua's third letter likewise indicates that
he has already accepted the consequences of his voluntary resignation but that it would be subject to the payment of
severance pay. However, his claim for severance pay cannot be granted. An employee who voluntarily resigns is not
entitled to separation pay unless it was previously stipulated in the employment contract or has become established
company policy or practice. 83 There is nothing in Pascua's Contract of Employment84 that states that he would be
receiving any monetary compensation if he resigns. He has also not shown that the payment of separation pay upon
resignation is an established policy or practice of Bankwise since his third letter indicated that he was unaware of any
such policy

It was incumbent on Pascua to ensure that his severance pay in the event of his resignation be embodied on a
written agreement before submitting his resignation letter. He should have, at the very least, indicated his conditions
in his resignation letter. His third letter cannot be considered the written statement of his money claims contemplated
in his Contract of Employment since it was unilateral and was not signed by Bankwise's officers

Angeles| Bajana | Balladares | Brillantes | Briones | Cabansag | Callanta | Chua | David|


De Leon | Gomez | Lopez | Macalino | Nostratis | Padilla | Reynon | Santos | Tan |Velasco
4E / 4F - 2018-2019
Page 837 of 920
LABOR REVIEW DIGEST
Atty. Joyrich Golangco

Angeles| Bajana | Balladares | Brillantes | Briones | Cabansag | Callanta | Chua | David|


De Leon | Gomez | Lopez | Macalino | Nostratis | Padilla | Reynon | Santos | Tan |Velasco
4E / 4F - 2018-2019
Page 838 of 920
LABOR REVIEW DIGEST
Atty. Joyrich Golangco

ART. 301 - WHEN EMPLOYEMNT NOT DEEMED TERMINATED

1. SKM ART CRAFT CORPORTATION vs. EFREN BAUCA et al.


G.R. no. 171282 November 27, 2013
Villarama, Jr.

DOCTRINE: Under Article 286 of the Labor Code, the employment will not be deemed terminated if the bona fide
suspension of operations does not exceed six months. But if the suspension of operations exceeds six months, the
employment will be considered terminated.
By the same token and applying said rule by analogy, if the employee was forced to remain without work or
assignment for a period exceeding six months, then he is in effect constructively dismissed.

FACTS: Efren Bauca, et al. (23 total) were employed by SKM Art Craft Corporation which is engaged in the
handicraft business.
On April 18, 2000, at around 1:12 am, a fire occurred at the inspection and receiving/repair/packing area of SKM’s
premises in Intramuros, Manila. The fire investigation report stated that the structure and the beach rubber building
were totally damaged. Also burned were four container vans and a trailer truck. The estimated damage was P22
million.
On May 8, 2000, SKM informed Bauca, et al. that it will suspend its operations for 6 months, effective May 9, 2000. 8
days after receiving notice of the suspension of SKM’s operations, Bauca filed a complaint for illegal dismissal
alleging that there was discrimination in choosing the workers to be laid off and that SKM had discovered that most of
them were members of a newly-organized union.
PETITIONER’S CONTENTION: Petitioner denied the claim of illegal dismissal and said that Article 2867 (now Article
301) of the Labor Code allows the bona fide suspension of a business or undertaking for a period not exceeding six
months. Petitioner claimed that the fire cost it millions in losses and that it is impossible to resume its normal
operations for a significant period of time.
LA RULING: The Labor Arbiter ruled that respondents were illegally dismissed and ordered petitioner to reinstate
them and pay them back wages of ₱59,918.41 each, the amount being subject to further computation up to the date
of their actual reinstatement. It ruled that the fire that burned a part of petitioner’s premises may validate the
suspension of respondents’ employment, but the suspension must not exceed six months. Since petitioner failed to
recall respondents after the lapse of six months, the Labor Arbiter held that respondents were illegally dismissed.
NLRC RULING: The National Labor Relations Commission (NLRC) set aside the Labor Arbiter’s Decision and ruled
that there was no illegal dismissal. The NLRC ordered that respondents be reinstated to their former positions but it
deleted the award of back wages. The NLRC noted that the fire caused millions in damages to petitioner. Thus,
petitioner’s suspension of operations is valid under Article 286 of the Labor Code. It was not meant to remove
respondents because they were union members. The NLRC added that the illegal dismissal complaint filed by
respondents was premature for it was filed during the six-month period of suspension of operations.
CA RULING: The CA set aside the NLRC Decision and Resolution and reinstated the Labor Arbiter’s Decision. The
CA ruled that petitioner failed to prove that its suspension of operations is bona fide . The CA noted that the proof of
alleged losses – the list of items and materials allegedly burned – was not even certified or signed by petitioner’s
accountant or comptroller. And even if the suspension of operations is considered bona fide, the CA said that
respondents were not reinstated after six months. Thus, respondents are deemed to have been illegally dismissed.
ISSUE: Whether or not there was illegal dismissal.
HELD: YES.
Under Article 286 of the Labor Code, the bona fide suspension of the operations of a business or undertaking for a
period not exceeding six months shall not terminate employment. As a general rule, a complaint for illegal dismissal
filed prior to the lapse of said six months is generally considered as prematurely filed.
In this case, however, Bauca, et al. were already considered illegally dismissed since SKM failed to recall them after
6 months, when its bona fide suspension of operations lapsed. Under Article 286 of the Labor Code, the employment
will not be deemed terminated if the bona fide suspension of operations does not exceed six months. But if the
suspension of operations exceeds six months, the employment will be considered terminated.
By the same token and applying said rule by analogy, if the employee was forced to remain without work or
assignment for a period exceeding six months, then he is in effect constructively dismissed.
SKM’s suspension of operations is valid because the fire caused substantial losses to SKM and damaged its factory,
Bauca, et al. were illegally dismissed since they were not recalled 6 months after the bona fide suspension of SKM’s
operations.

Angeles| Bajana | Balladares | Brillantes | Briones | Cabansag | Callanta | Chua | David|


De Leon | Gomez | Lopez | Macalino | Nostratis | Padilla | Reynon | Santos | Tan |Velasco
4E / 4F - 2018-2019
Page 839 of 920
LABOR REVIEW DIGEST
Atty. Joyrich Golangco

2. EMERITUS SECURITY AND MAINTENANCE SYSTEMS v DAILIG


G.R. No. 204761 April 2, 2014

DOCTRINE: The temporary inactivity or "floating status" of security guards should continue only for six
months. Otherwise, the security agency concerned could be liable for constructive dismissal. The failure of
petitioner to give respondent a work assignment beyond the reasonable six-month period makes it liable for
constructive dismissal.

Facts:
In August 2000, petitioner hired respondent as one of its security guards. During his employment,
respondent was assigned to petitioner's various clients, the last of which was Panasonic in Calamba, Laguna starting
16 December 2004.
On 10 December 2005, respondent was relieved from his post.
Respondent filed a complaint for illegal dismissal and payment of separation pay against petitioner before
the Conciliation and Mediation Center of the NLRC. On 14 July 2006, respondent filed another complaint for illegal
dismissal, underpayment of salaries and non-payment of full backwages before the NLRC.
Respondent claimed that on various dates in December 2005 and from January to May 2006,4 he went to
petitioner’s office to follow-up his next assignment. After more than six months since his last assignment, still
respondent was not given a new assignment. Respondent argued that if an employee is on floating status for more
than six months, such employee is deemed illegally dismissed.
Labor Arbiter rendered a Decision complainant is hereby declared to have been illegally dismissed.
Petitioner appealed before the NLRC, which dismissed the appeal for lack of merit. Petitioner moved for
reconsideration, which the NLRC denied.
Court of Appeals affirmed the finding of the Labor Arbiter and the NLRC that respondent was illegally
dismissed by petitioner. However, the Court of Appeals set aside the Labor Arbiter and the NLRC’s reinstatement
order.

ISSUE:
whether respondent was illegally dismissed by respondent

HELD:
Petitioner admits relieving respondent from his post as security guard on 10 December 2005. There is also
no dispute that respondent remained on floating status at the time he filed his complaint for illegal dismissal on 16
June 2006. In other words, respondent was on floating status from 10 December 2005 to 16 June 2006 or more than
six months. Petitioner’s allegation of sending respondent a notice sometime in January 2006, requiring him to report
for work, is unsubstantiated, and thus, self-serving.
The Court agrees with the ruling of the Labor Arbiter, NLRC and Court of Appeals that a floating status of a
security guard, such as respondent, for more than six months constitutes constructive dismissal. In Nationwide
Security and Allied Services, Inc. v. Valderama, the Court held:
x x x the temporary inactivity or "floating status" of security guards should continue only for six
months. Otherwise, the security agency concerned could be liable for constructive dismissal. The failure of
petitioner to give respondent a work assignment beyond the reasonable six-month period makes it liable for
constructive dismissal. x x x.
Further, the Court notes that the Labor Arbiter, NLRC, and Court of Appeals unanimously found that
respondent was illegally dismissed by petitioner. Factual findings of quasi-judicial bodies like the NLRC, if supported
by substantial evidence, are accorded respect and even finality by this Court, more so when they coincide with those
of the Labor Arbiter. Such factual findings are given more weight when the same are affirmed by the Court of
Appeals. The Court finds no reason to depart from the foregoing rule.
Considering that (1) petitioner reinstated respondent in compliance with the Labor Arbiter's decision, and (2)
there is no ground, particularly strained relations between the parties, to justify the grant of separation pay, the Court
of Appeals erred in ordering the payment thereof, in lieu of reinstatement.

Angeles| Bajana | Balladares | Brillantes | Briones | Cabansag | Callanta | Chua | David|


De Leon | Gomez | Lopez | Macalino | Nostratis | Padilla | Reynon | Santos | Tan |Velasco
4E / 4F - 2018-2019
Page 840 of 920
LABOR REVIEW DIGEST
Atty. Joyrich Golangco

3. NIPPON HOUSING PHIL. INC. V. LEYNES


G.R. NO. 159738; DEC. 9, 2004
CALLEJO, SR., J.:

DOCTRINE: The filing of an illegal dismissal suit is premature where an employee is merely placed on
floating status. The rule is settled, however, that "off-detailing" is not equivalent to dismissal, so long as
such status does not continue beyond a reasonable time and that it is only when such a "floating status"
lasts for more than six months that the employee may be considered to have been constructively dismissed.
FACTS:

Nippon Housing Philippines hired Maiah Angela Leynes in March of 2001 as Property Manager. Sometime in
February 2002, Lenyes had a misunderstanding with Engr. Honesto Cantuba who was the Building Manager hired for
the project she was hired. Leynes ordered security to deny Engr. Cantuba entry to the working premises, and also
wrote to the Human Resources department of the supposed insubordination and disrespectful conduct on the part of
Engr. Cantuba. Cantuba, on the other hand, retaliated by accusing Leynes of conceitedness and poor managerial
skills. Nippon’s Vice President took cognizance of the issue and came to the conclusion that the altercations were
nothing more than the result of personal differences and ordered Leynes to admit Cantuba to work.

Feeling bypassed, Leynes submitted to Tadashi Oda, Nippon’s President, a letter asking for an emergency leave of
absence so that she could confer with her lawyer as to a possible resignation on her part. Much to Leynes’ agitation,
her job was offered to a certain Engr. Calos Jose, by reason of her manifestation that she intended to resign. Leynes
immediately retracted and informed the management of her intention to return to work, protesting all the while the fact
that she had been replaced in the project. Leynes was further served by petitioner Yasuhiro Kawata and Noboyushi
Hisada, NHPIs Senior Manager and Janitorial Manager, with a letter and memorandum from Reyes, relieving her
from her position and directing her to report to NHPIs main office while she was on “floating status”.

Feeling aggrieved by the foregoing, Leynes decided to file an action for illegal dismissal, together with money claims.

LA RULING: Ruled that the declaration of the floating status was equivalent to illegal dismissal, and held Nippon and
liable for failing to comply with the twin requirements of notice and hearing.

NLRC RULING: Reversed the LA and held dismissed the action for lack of merit.

CA RULING: reversed and set aside in the NLRC. It held that: (a) absent showing that there was a bona fide
suspension of NHPIs business operations, Leynes relief from her position even though requested by the client was
tantamount to a constructive dismissal; (b) the bad faith of NHPI and its officers is evident from the hiring of Engr.
Jose as Leynes replacement; and, (c) the failure of NHPI and its officers to prove a just cause for Leynes termination,
the redundancy of her services and their compliance with the requirements of due process renders them liable for
illegal dismissal.

Petitioner’s Contention: That placing Leynes on floating status was not tantamount to constructive dismissal.

ISSUE: Whether placing Leynes on floating status amounted to constructive dismissal.

RULING:

NO. The filing of an illegal dismissal suit is premature where an employee is merely placed on floating status. The
rule is settled, however, that "off-detailing" is not equivalent to dismissal, so long as such status does not continue
beyond a reasonable time and that it is only when such a "floating status" lasts for more than six months that the
employee may be considered to have been constructively dismissed.
The Court that the CA reversibly erred in holding petitioners liable for constructively dismissing Leynes from her
employment. There is said to be constructive dismissal when an act of clear discrimination, insensitivity or disdain on
the part of the employer has become so unbearable as to leave an employee with no choice but to forego continued
employment.
Nippon could not be faulted in securing a replacement for Leynes considering her continuous grandstanding was
putting the project in jeopardy, and the management was deemed to have acted within the bounds of management
prerogative when it secured the services of another in over to prevent the project from suffering all for the reason that
Leynes was distraught over Cantuba’s reinstatement.

Angeles| Bajana | Balladares | Brillantes | Briones | Cabansag | Callanta | Chua | David|


De Leon | Gomez | Lopez | Macalino | Nostratis | Padilla | Reynon | Santos | Tan |Velasco
4E / 4F - 2018-2019
Page 841 of 920
LABOR REVIEW DIGEST
Atty. Joyrich Golangco

Angeles| Bajana | Balladares | Brillantes | Briones | Cabansag | Callanta | Chua | David|


De Leon | Gomez | Lopez | Macalino | Nostratis | Padilla | Reynon | Santos | Tan |Velasco
4E / 4F - 2018-2019
Page 842 of 920
LABOR REVIEW DIGEST
Atty. Joyrich Golangco

4. MAYON HOTEL & RESTAURANT, PACITA O. PO and/or JOSEFA PO LAM, petitioners, vs. ROLANDO
ADANA, CHONA BUMALAY, ROGER BURCE, EDUARDO ALAMARES, AMADO ALAMARES, EDGARDO
TORREFRANCA, LOURDES CAMIGLA, TEODORO LAURENARIA, WENEFREDO LOVERES, LUIS GUADES,
AMADO MACANDOG, PATERNO LLARENA, GREGORIO NICERIO, JOSE ATRACTIVO, MIGUEL
TORREFRANCA, and SANTOS BROÑOLA, respondents.
GR No: 157634 Date: May 16, 2005
Ponente: Puno, J.

Doctrine: Article 286 (now 301) of the Labor Code is clear — there is termination of employment when an
otherwise bona fide suspension of work exceeds six (6) months.

FACTS:
Petitioner Mayon Hotel & Restaurant is a single proprietor business registered in the name of petitioner
Pacita O. Po, whose mother, petitioner Josefa Po Lam, manages the establishment. The hotel and restaurant
employed about sixteen (16) employees (herein respondents).
Due to the expiration and non-renewal of the lease contract for the rented space occupied by the said hotel
and restaurant at Rizal Street, the hotel operations of the business were suspended on March 31, 1997. The
operation of the restaurant was continued in its new location at Elizondo Street, Legazpi City, while waiting for the
construction of a new Mayon Hotel & Restaurant at Peñaranda Street, Legazpi City. Only nine (9) of the sixteen (16)
employees continued working in the Mayon Restaurant at its new site.
On various dates of April and May 1997, the employees filed complaints for underpayment of wages and
other money claims against petitioners including a complaint for illegal dismissal for the 7 employees who weren’t
allowed to continue working at petitioner’s new site.

LA/RTC/NLRC RULING:
LA: Rendered a Joint Decision in favor of the employees. The Labor Arbiter awarded substantially all of respondents'
money claims, and held that respondents Loveres, Macandog and Llarena were entitled to separation pay,
while respondents Guades, Nicerio and Alamares were entitled to their retirement pay. The Labor Arbiter
also held that based on the evidence resented, Josefa Po Lam is the owner/proprietor of Mayon Hotel &
Restaurant and the proper respondent in these cases.

NLRC: Reversed the LA decision and dismissed the complaints. MR was denied

CA RULING: Reversed the NLRC ruling.

APPEAL TO THE SC:


Petitioner's Contention: It is petitioners contention that the CA should have sustained the NLRC finding that none of
the above-named respondents were illegally dismissed, or entitled to separation or retirement pay. According to
petitioners, even the Labor Arbiter and the CA admit that when the illegal dismissal case was filed by respondents on
April 1997, they had as yet no cause of action. Petitioners therefore conclude that the fling by respondents of the
illegal dismissal case was premature and should have been dismissed outright by the Labor Arbiter. Petitioners also
claim that since the validity of respondents' dismissal is a factual question, it is not for the reviewing court to weigh
the conflicting evidence.

ISSUE/S:
1. Whether respondents Loveres, Guades, Macandog, Atractivo, Llarena and Nicerio illegally dismissed?

HELD:
1. YES. It was a serious error that the NLRC did not inquire as to the legality of the cessation of employment.
Article 286 of the Labor Code is clear — there is termination of employment when an otherwise bona fide
suspension of work exceeds six (6) months. The cessation of employment for more than six months was
patent and the employer has the burden of proving that the termination was for a just or authorized cause.
It is confounding how petitioners have fashioned their arguments. After having admitted, in effect, that
respondents have been laid-off since April 1997, they would have this Court excuse their refusal to reinstate
respondents or grant them separation pay because these same respondents purportedly have not proven the
illegality of their dismissal.
Petitioners' arguments respect their lack of candor and the blatant attempt to use technicalities to muddle
the issues and defeat the lawful claims of their employees. First, petitioners admit that since April 1997, when
hotel operations were suspended due to the termination of the lease of the old premises, respondents Loveres,
Macandog, Llarena, Nicerio and Guades have not been permitted to work. Second, even after six months of

Angeles| Bajana | Balladares | Brillantes | Briones | Cabansag | Callanta | Chua | David|


De Leon | Gomez | Lopez | Macalino | Nostratis | Padilla | Reynon | Santos | Tan |Velasco
4E / 4F - 2018-2019
Page 843 of 920
LABOR REVIEW DIGEST
Atty. Joyrich Golangco

what should have been just a temporary lay-off, the same respondents were still not recalled to work. As
a matter of fact, the Labor Arbiter even found that as of the time when he rendered his Joint Decision on
July 2000 — or more than three (3) years after the supposed "temporary lay-off," the employment of all
of the respondents with petitioners had ceased, notwithstanding that the new premises had been
completed and the same operated as a hotel with bar and restaurant. This is clearly dismissal — or the
permanent severance or complete separation of the worker from the service on the initiative of the employer
regardless of the reasons therefor.
While the closure of the hotel operations in April of 1997 may have been temporary, we hold that the
evidence on record belie any claim of petitioners that the lay-off of respondents on that same date was merely
temporary. On the contrary, we find substantial evidence that petitioners intended the termination to be
permanent. First, respondents Loveres, Macandog, Llarena, Guades, Nicerio and Alamares filed the complaint
for illegal dismissal immediately after the closure of the hotel operations in Rizal Street, notwithstanding the
alleged temporary nature of the closure of the hotel operations, and petitioners' allegations that the employees
assigned to the hotel operations knew about this beforehand. Second, in their position paper submitted to the
Labor Arbiter, petitioners invoked Article 286 of the Labor Code to assert that the employer-employee
relationship was merely suspended, and therefore the claim for separation pay was premature and without legal
or factual basis. But they made no mention of any intent to recall these respondents to work upon
completion of the new premises. Third, the various pleadings on record show that petitioners held
respondents, particularly Loveres, as responsible for mismanagement of the establishment and for abuse of trust
and confidence. Petitioner Josefa Po Lam's affidavit on July 21, 1998, for example, squarely blamed
respondents, specifically Loveres, Bumalay and Camigla, for abusing her leniency and causing petitioner Mayon
Hotel & Restaurant to sustain "continuous losses until it is closed." She then asserts that respondents "are not
entitled to separation pay for they were not terminated and if ever the business ceased to operate it was
because of losses." Again, petitioners make the same allegation in their memorandum on appeal with the NLRC,
where they alleged that three (3) years prior to the expiration of the lease in 1997, the operation of the Hotel had
been sustaining consistent losses, and these were solely attributed to respondents, but most especially due to
Loveres's mismanagement and abuse of petitioners' trust and confidence. Even the petition filed in this court
made reference to the separation of the respondents due to "severe financial losses and reverses," again
imputing it to respondents' mismanagement. The vehemence of petitioners' accusation of mismanagement
against respondents, especially against Loveres, is inconsistent with the desire to recall them to work. Fourth,
petitioners' memorandum on appeal also averred that the case was filed "not because of the business being
operated by them or that they were supposedly not receiving benefits from the Labor Code which is true, but
because of the fact that the source of their livelihood, whether legal or immoral, was stopped on March 31, 1997,
when the owner of the building terminated the Lease Contract.". Fifth, petitioners had inconsistencies in their
pleadings (with the NLRC, CA and with this Court) in referring to the closure, i.e., in the petition filed with
this court, they assert that there is no illegal dismissal because there was "only a temporary cessation
or suspension of operations of the hotel and restaurant due to circumstances beyond the control of
petitioners, and that is, the non-renewal of the lease contract. . . . " And yet, in the same petition, they
also assert that: (a) the separation of respondents was due to severe financial losses and reverses
leading to the closure of the business; and (b) petitioner Pacita Po had to close shop and was bankrupt
and has no liquidity to put up her own building to house Mayon Hotel & Restaurant. Sixth, and finally, the
uncontroverted finding of the Labor Arbiter that petitioners terminated all the other respondents, by not
employing them when the Hotel and Restaurant transferred to its new site on Peñaranda Street. Indeed, in this
same memorandum, petitioners referred to all respondents as "former employees of Mayon Hotel &
Restaurant."
These factors may be inconclusive individually, but when taken together, they lead us to conclude that
petitioners really intended to dismiss all respondents and merely used the termination of the lease (on Rizal
Street premises) as a means by which they could terminate their employees. Moreover, even assuming
arguendo that the cessation of employment on April 1997 was merely temporary, it became dismissal by
operation of law when petitioners failed to reinstate respondents after the lapse of six (6) months,
pursuant to Article 286 of the Labor Code.
We are not impressed by petitioners' claim that severe business losses justified their failure to reinstate
respondents. The evidence to prove this fact is inconclusive. But more important, serious business losses do not
excuse the employer from complying with the clearance or report required under Article 283 of the Labor Code
and its implementing rules before terminating the employment of its workers. In the absence of justifying
circumstances, the failure of petitioners to observe the procedural requirements set out under Article 284, taints
their actuations with bad faith, especially since they claimed that they have been experiencing losses in the three
years before 1997. To say the least, if it were true that the lay-off was temporary but then serious business
losses prevented the reinstatement of respondents, then petitioners should have complied with the requirements
of written notice. The requirement of law mandating the giving of notices was intended not only to enable the

Angeles| Bajana | Balladares | Brillantes | Briones | Cabansag | Callanta | Chua | David|


De Leon | Gomez | Lopez | Macalino | Nostratis | Padilla | Reynon | Santos | Tan |Velasco
4E / 4F - 2018-2019
Page 844 of 920
LABOR REVIEW DIGEST
Atty. Joyrich Golangco

employees to look for another employment and therefore ease the impact of the loss of their jobs and the
corresponding income, but more importantly, to give the Department of Labor and Employment (DOLE) the
opportunity to ascertain the verity of the alleged authorized cause of termination.

Other Notes/ SC Pronouncements:


Notwithstanding the certificate of registration, doubts were cast as to the true nature of petitioner Josefa Po Lam's
involvement in the enterprise, and the Labor Arbiter had the authority to resolve this issue. It was therefore within
his jurisdiction to require the additional documents to ascertain who was the real owner of petitioner Mayon
Hotel & Restaurant.

Angeles| Bajana | Balladares | Brillantes | Briones | Cabansag | Callanta | Chua | David|


De Leon | Gomez | Lopez | Macalino | Nostratis | Padilla | Reynon | Santos | Tan |Velasco
4E / 4F - 2018-2019
Page 845 of 920
LABOR REVIEW DIGEST
Atty. Joyrich Golangco

5. ICT Marketing Services, Inc. (now known as Sykes Marketing Services, Inc.) v. Mariphil L. Sales
GR No: 202090 Date: September 9, 2015
Ponente: J. Del Castillo

Doctrine:
Constructive dismissal exists when the resignation on the part of the employee was involuntary due to the
harsh, hostile and unfavorable conditions set by the employer. The test for constructive dismissal is whether
a reasonable person in the employee's position would feel compelled to give up his employment under the
prevailing circumstances.

Concerning the transfer of employees, these are the following jurisprudential guidelines: (a) a transfer is a
movement from one position to another of equivalent rank, level or salary without break in the service or a
lateral movement from one position to another of equivalent rank or salary; (b) the employer has the inherent
right to transfer or reassign an employee for legitimate business purposes; (c) a transfer becomes unlawful
where it is motivated by discrimination or bad faith or is effected as a form of punishment or is a demotion
without sufficient cause; (d) the employer must be able to show that the transfer is not unreasonable,
inconvenient, or prejudicial to the employee.

FACTS:
Petitioner ICT hired respondent Mariphil L. Sales as its Customer Service Representative or Telephone Service
Representative and assigned her to its Capital One account. Respondent was later assigned to the Washington
Mutual Account. She complained about supposed irregularities in the handling of funds entrusted to petitioner by
Washington Mutual.

Respondent was transferred to the Bank of America account. Petitioner scheduled her for training on the very same
day of her transfer, without prior notice. On the third day of training, respondent was unable to attend. When she
reported for training the next day, respondent was informed that she could not be certified to handle calls for Bank of
America due to her failure to complete the training. From then on, she was paced on “floating status” and was not
given any work assignment.

Respondent tendered her resignation from work citing that she had no option but to sever relationship with the
company as her continued floating status had already prejudiced her emotionally and financially.

Respondent filed a complaint for constructive dismissal against petitioner.

LA/NLRC RULING:
The LA rendered a decision finding complainant to have been constructively dismissed.

The NLRC reversed the LA’s decision and concluded that the complainant has not presented any evidence that
would indicate that duress or force has been exerted on her.

CA RULING:
The CA ruled that respondent was constructively dismissed from employment. With the decision to transfer and to
thereafter placed her on floating status, petitioner felt that she was being discriminated and this perception compelled
her to resign. It is clear from her resignation letter that petitioner felt oppressed by the situation created by the private
respondent corporation, and this forced her to surrender her position.

Petitioner's Contention:
Petitioner maintains that respondent's transfer to another account was done as a valid exercise of management
prerogative, which allows it to regulate all aspects of employment. Her transfer was done in good faith, and without
diminution in rank and salary. It contends that respondent knew very well that any CSR/TSR may be transferred to
another program/account anytime for business reasons. Moreover, she knew as well that "schedule adherence" or
attendance/punctuality is one of the "metrics" or standards by which the performance of a CSR is measured, and that
she failed to comply in this regard.

Respondent's Contention:
Respondent insists that she was illegally dismissed. She reiterates that her transfer to the Bank of America account
was an undue penalty for her complaining about supposed anomalies in the Washington Mutual account. She avers
that the documentary evidence of her supposed unauthorized absences were manufactured to support petitioner's
false allegations and mislead this Court into believing that she was delinquent at work. She maintains that her being

Angeles| Bajana | Balladares | Brillantes | Briones | Cabansag | Callanta | Chua | David|


De Leon | Gomez | Lopez | Macalino | Nostratis | Padilla | Reynon | Santos | Tan |Velasco
4E / 4F - 2018-2019
Page 846 of 920
LABOR REVIEW DIGEST
Atty. Joyrich Golangco

placed on "floating status" was illegal, since a) there is no evidence to prove her alleged "attendance and punctuality
issues," and b) there was no bona fide suspension of petitioner's business or undertaking for a period not exceeding
six months, as prescribed under Article 286 of the Labor Code, which would justify the suspension of her employment
for up to six months.

ISSUE:
Whether the transfer was illegal and thus constituted constructive dismissal.

HELD:
YES. In causing respondent’s transfer, petitioner clearly acted in bad faith and with discrimination; the transfer was
effected as form of punishment for her raising a valid grievance related to her work.

Concerning the transfer of employees, these are the following jurisprudential guidelines: (a) a transfer is a movement
from one position to another of equivalent rank, level or salary without break in the service or a lateral movement
from one position to another of equivalent rank or salary; (b) the employer has the inherent right to transfer or
reassign an employee for legitimate business purposes; (c) a transfer becomes unlawful where it is motivated by
discrimination or bad faith or is effected as a form of punishment or is a demotion without sufficient cause; (d) the
employer must be able to show that the transfer is not unreasonable, inconvenient, or prejudicial to the employee.

While the prerogative to transfer respondent to another account belonged to petitioner, it wielded the same unfairly.
The evidence suggests that at the time respondent was transferred from the Washington Mutual account to the Bank
of America program, petitioner was hiring additional CSRs/TSRs. This simply means that if it was then hiring new
CSRs/TSRs, then there should be no need to transfer respondent to the Bank of America program; it could simply
train new hires for that program. Transferring respondent — an experienced employee who was already familiar with
the Washington Mutual account, and who even proved to be outstanding in handling the same — to another account
means additional expenses for petitioner: it would have to train respondent for the Bank of America account, and
train a new hire to take her place in the Washington Mutual account. This does not make sense; quite the contrary, it
is impractical and entails more expense on petitioner's part.

Moreover, even if respondent had attendance and punctuality issues, her overall performance as a CSR/TSR was
certainly far from mediocre; on the contrary, she proved to be a top performer. And if it were true that respondent
suddenly became lax by way of attendance in July 2007, it is not entirely her fault. This may be attributed to
petitioner's failure to properly address her grievances relative to the supposed irregularities in the handling of funds
entrusted to petitioner.

While it is true that respondent's resignation was involuntary as it became unbearable for her to continue with her
employment, expounding on the issue at length is unnecessary. Because she is deemed constructively dismissed
from the time of her illegal transfer, her subsequent resignation became unnecessary and irrelevant. There was no
longer any position to relinquish at the time of her resignation.

Angeles| Bajana | Balladares | Brillantes | Briones | Cabansag | Callanta | Chua | David|


De Leon | Gomez | Lopez | Macalino | Nostratis | Padilla | Reynon | Santos | Tan |Velasco
4E / 4F - 2018-2019
Page 847 of 920
LABOR REVIEW DIGEST
Atty. Joyrich Golangco

6. GERARDO A. CARIQUE V. PHILIPPINE SCOUT VETERANS SECURITY AND INVESTIGATION AGENCY,


INC., AND/OR RICARDO BONA AND SEVERO ** SANTIAGO
GR NO: 197484 DATE: SEPTEMBER 16, 2015
Ponente: DEL CASTILLO, J.

Doctrine:
Contracts for security services may stipulate that the clients may request the agency for the replacement of
the guard/s assigned to it even for want of cause; and that such replaced security guard/s could be placed
on temporary “off-detail” or “floating status” which is the period of time when such security guard/s are in
between assignments or when they are made to wait after being relieved from a previous post until they are
transferred to a new one.
Placing petitioner on floating or off-detail status for not more than six months is not prohibited by law and
did not amount to dismissal.

FACTS:
Petitioner was hired as security agency by respondent agency owned by respondent Santiago and managed by
respondent Bona. On October 28, 2002, petitioner was relieved from his post at the National Bookstore pursuant to a
rotation policy being implemented by respondent agency.
On May 6, 2003, petitioner filed an illegal dismissal case against respondents. Petitioner alleged that he reported
continuously to agency’s office but was repeatedly advised to wait for a new posting. He was thus surprised when on
March 9, 2003, he received a memorandum from respondent agency requiring him to explain his Absence Without
Leave (AWOL) since November 30, 2002.
Respondents denied having dismissed petitioner, alleged that petitioner was relieved from his post because of a
rotation policy being implemented as required by respondent agency's clients, that the rotation policy which is lawful
practice of relieving security personnel from their posts did not amount to terminating the security personnel from
employment but was simply meant to place them on floating status while awaiting a new assignment. Respondents
also alleged that petitioner rejected the two offers of posting made by respondents as attested by the two officers of
the agency.

LA Ruling:
LA ruled that petitioner to have been illegally dismissed on the ground that respondents repeatedly denied
petitioner's demands/requests for assignment/posting.

NLRC Ruling:
NLRC annulled the decision of LA and was convinced that petitioner had refused new assignments.

CA RULING:
The CA ruled that when petitioner was relieved from his post at the National Bookstore Rosario, Pasig Branch on
October 28, 2002, he was merely placed on floating status or temporary off-detail and was not
dismissed. His floating status did not exceed six months as he was in fact given new assignments within five months
from his alleged relief but he refused these new assignments

APPEAL TO THE SC:


Petitioner’s contention:
Petitioner insists that, in any event, he was constructively dismissed because respondent agency's alleged offerings
of new assignments did not effectively toll the six-months floating period because his relief did not arise from a bona
fide suspension of the company's operation as contemplated in Article 286(now 301) of the Labor Code effectively
placing him on temporary off-detail for a period not exceeding six months.

ISSUE/S:
Whether or not the rotation policy of the agency is a bona fide suspension of the company's operation as
contemplated in Article 286(now 301) of the Labor Code

HELD:
YES. The implementation of the rotation policy by respondent agency is within the ambit of management prerogative.
The employer has the inherent right to regulate all aspects of employment, according to his own discretion and
judgment, including the right to transfer an employee as long as the transfer is not unreasonable, inconvenient,
prejudicial and does not involve a demotion in rank or a diminution of the employee's salaries, benefits, and other
privileges. In the absence of evident bad faith or a manifest intent to circumvent the factors and conditions just
mentioned, this Court is not prepared to invalidate respondents' stance that this policy reflects the essence of

Angeles| Bajana | Balladares | Brillantes | Briones | Cabansag | Callanta | Chua | David|


De Leon | Gomez | Lopez | Macalino | Nostratis | Padilla | Reynon | Santos | Tan |Velasco
4E / 4F - 2018-2019
Page 848 of 920
LABOR REVIEW DIGEST
Atty. Joyrich Golangco

security planning and the importance of discouraging familiarity between security personnel and the premises they
are guarding. Thus, we here reiterate that contracts for security services may stipulate that the clients may request
the agency for the
replacement of the guard/s assigned to it even for want of cause; and that such replaced security guard/s could be
placed on temporary "off-detail" or "floating status" which is the period of time when such security guard/s are in
between assignments or when they are made to wait after being relieved from a previous post until they are
transferred to a new one. Thus, placing petitioner on floating or off-detail status for not more than six months is not
prohibited by law and did not amount to dismissal.

Other Notes/ SC Pronouncements:


This Court is not unmindful of the rule that the employer has the burden of proving that the employee's termination
was for a valid or authorized cause. However, before the employer is tasked to discharge this burden, it is incumbent
upon the employee to prove by substantial evidence the fact that he was indeed illegally dismissed from employment.
Mere allegation is neither proof nor evidence.

Angeles| Bajana | Balladares | Brillantes | Briones | Cabansag | Callanta | Chua | David|


De Leon | Gomez | Lopez | Macalino | Nostratis | Padilla | Reynon | Santos | Tan |Velasco
4E / 4F - 2018-2019
Page 849 of 920
LABOR REVIEW DIGEST
Atty. Joyrich Golangco

ART. 302 - RETIREMENT

1. DAABAY V. COCA-COLA
GR NO: 199890 DATE: AUGUST 19, 2013
PONENTE: REYES, J.

Doctrine:
Separation pay shall be allowed as a measure of social justice only in those instances where the employee is
validly dismissed for causes other than serious misconduct or those reflecting on his moral character.
Where the reason for the valid dismissal is, for example, habitual intoxication or an offense involving moral
turpitude, like theft or illicit sexual relations with a fellow worker, the employer may not be required to give
the dismissed employee separation pay, or financial assistance, or whatever other name it is called, on the
ground of social justice.

FACTS:

A case for illegal dismissal, illegal suspension, unfair labor practice, and monetary claim was filed by Jerome Daabay
(Petitioner) against Coca-Cola and three officers of the company (Respondent). Petitioner was the Logistics
Checker in the company prior to his termination in June 2005. Said termination was based on information from one
Cesar Sorin that petitioner was part of a conspiracy that allowed pilferage of company property consisting of assorted
softdrinks, empty bottles, missing shells and missing pallets.

This was later on confirmed by a subsequent audit done by the company. Petitioner was served with a notice to
explain with preventive suspension, requiring him to state his participation (if any) in such scheme. This was later on
followed by a formal investigation. Coca-cola eventually served a notice of termination to petitioner on the grounds of
pilferage, serious misconduct and loss of trust and confidence.

LA RULING:
Executive Labor Arbiter Noel Augusto S. Magbanua ruled in favor of petitioner Daabay, finding that the latter was
illegally dismissed because his participation in the alleged conspiracy was not proved by substantial evidence, and
ordering the payment of backwages and separation pay or retirement benefits, as may be applicable to the petitioner

NLRC RULING:
The NLRC reversed the LA’s ruling and found the dismissal to be valid due to petitioner’s participation in the
conspiracy being well-established by the checkers receipts and sales invoices signed by petitioner that made the
fraudulent scheme possible and for failure to detect the pilferage despite being the Logistics Checker. However,
despite such decision, the NLRC awarded retirement benefits to Daabay.

CA RULING:
The CA agreed with Coca-Cola that the award of retirement benefits lacked basis considering that Daabay was
dismissed for just cause. It explained:
We are not oblivious of the instances where the Court awarded financial assistance to dismissed employees, even
though they were terminated for just causes. Equity and social justice was the vague justification. Quickly realizing
the unjustness of these [s]o-called equitable awards, the Supreme Court took the opportunity to curb and rationalize
the grant of financial assistance to legally dismissed employees. Thus, in Philippine Long Distance Telephone
Company v. National Labor Relations Commission, the Supreme Court recognized the harsh realities faced by
employees that forced them, despite their good intentions, to violate company policies, for which the employer can
rightfully terminate their employment. For these instances, the award of financial assistance was allowed. But, in
clear and unmistakable language, the Supreme Court also held that the award of financial assistance should not be
given to validly terminated employees, whose offenses are iniquitous or reflective of some depravity in their moral
character.

ISSUE:
Is petitioner entitled to retirement benefits?

HELD:
No. The Court ruled that such grant of retirement benefits is unwarranted due to the dismissal of the petitioner on the
ground of serious misconduct. Having been validly dismissed for a just cause, the provisions of Art. 287 (Now Art.
302) cannot apply to petitioner.

Angeles| Bajana | Balladares | Brillantes | Briones | Cabansag | Callanta | Chua | David|


De Leon | Gomez | Lopez | Macalino | Nostratis | Padilla | Reynon | Santos | Tan |Velasco
4E / 4F - 2018-2019
Page 850 of 920
LABOR REVIEW DIGEST
Atty. Joyrich Golangco

Citing PLDT v. NLRC, it held:

Separation pay shall be allowed as a measure of social justice only in those instances where the employee is validly
dismissed for causes other than serious misconduct or those reflecting on his moral character. Where the reason for
the valid dismissal is, for example, habitual intoxication or an offense involving moral turpitude, like theft or illicit
sexual relations with a fellow worker, the employer may not be required to give the dismissed employee separation
pay, or financial assistance, or whatever other name it is called, on the ground of social justice.

It further stated that if the employee who steals from the company is granted separation pay even as he is validly
dismissed, it is not unlikely that he will commit a similar offense in his next employment because he thinks he can
expect a like leniency if he is again found out. This kind of misplaced compassion is not going to do labor in general
any good as it will encourage the infiltration of its ranks by those who do not deserve the protection and concern of
the Constitution.

Angeles| Bajana | Balladares | Brillantes | Briones | Cabansag | Callanta | Chua | David|


De Leon | Gomez | Lopez | Macalino | Nostratis | Padilla | Reynon | Santos | Tan |Velasco
4E / 4F - 2018-2019
Page 851 of 920
LABOR REVIEW DIGEST
Atty. Joyrich Golangco

2. Piero v. NLRC
GR No: 149610 Date: August 20, 2004
Ponente: Ynares-Santiago, J.

Art. 302 (Formerly Art. 287) – Retirement Benefits


Doctrine:
An employee who is dismissed for cause is generally not entitled to any financial assistance. Equity considerations,
however, provide an exception. Equity has been defined as justice outside law, being ethical rather than jural and
belonging to the sphere of morals than of law. It is grounded on the precepts of conscience and not on any sanction
of positive law, for equity finds no room for application where there is law.

FACTS:
Petitioner Rosendo Piero was the president of petitioner Dumaguete Cathedral College Faculty and Staff Association
(DUCACOFSA) which is affiliated with co-petitioner National Federation of Teachers and Employees Union
(NAFTEU). On Dec. 19, 1986 DUCACOFSA entered into a collective bargaining agreement with respondent
Dumaguete Cathedral College, Inc., with said CBA to be valid for 3 years. However, upon its expiration in 1989, the
parties failed to conduct another CBA, forcing DUCACOFSA-NAFTEU to file a notice of strike with DOLE on the
ground of failure to bargain.

On Nov. 4, 1991 DUCACOFSA-NAFTEU staged a strike without submitting to DOLE the required results of the strike
vote. This lead to respondent filing a complaint with DOLE to declare the strike illegal. DOLE ruled in favor of the
respondent, declaring the strike to be illegal and dismissing the officers of DUCACOFSA-NAFTEU which included
petitioner Rosendo Piero

LA RULING:
Affirmed DOLE ruling

NLRC RULING:
Affirmed LA ruling, further reasoning that DUCACOFSA-NAFTEU is not a legitimate labor union and therefore has no
personality to hold a strike

CA RULING:
Affirmed NLRC ruling.

ISSUES:
1. Is the strike illegal
2. Should petitioner Piero be dismissed?

HELD:
1. Yes. Strike was illegal due to non-compliance with strike vote requirement
2. Yes. Since the strike was illegal, petitioner’s dismissal is valid. However, the Court noted:

…petitioner Piero turned 60 years old and retired on March 1, 1996 after 29 years of service, rendering his dismissal
from service moot and academic. However, in view of the propriety of his termination as a consequence of the illegal
strike, he is no longer entitled to payment of retirement benefits because he lost his employment status effective as of
the date of the decision of the Labor Arbiter October 28, 1994.

An employee who is dismissed for cause is generally not entitled to any financial assistance. Equity considerations,
however, provide an exception. Equity has been defined as justice outside law, being ethical rather than jural and
belonging to the sphere of morals than of law. It is grounded on the precepts of conscience and not on any sanction
of positive law, for equity finds no room for application where there is law.

Although meriting termination of employment, Pieros infraction is not so reprehensible nor unscrupulous as to
warrant complete disregard of his long years of service. Moreover, he has no previous derogatory records. Weighed
on the scales of justice, conscience and reason tip in favor of granting financial assistance to support him in the
twilight of his life after long years of service.

Under the circumstances, social and compassionate justice dictate that petitioner Piero be awarded financial
assistance equivalent to one-half (1/2) months pay for every year of service computed from his date of employment

Angeles| Bajana | Balladares | Brillantes | Briones | Cabansag | Callanta | Chua | David|


De Leon | Gomez | Lopez | Macalino | Nostratis | Padilla | Reynon | Santos | Tan |Velasco
4E / 4F - 2018-2019
Page 852 of 920
LABOR REVIEW DIGEST
Atty. Joyrich Golangco

up to October 28, 1994 when he was declared to have lost his employment status. Indeed, equities of this case
should be accorded due weight because labor law determinations are not only secundum rationem but
also secundum caritatem.

Other Notes/ SC Pronouncements:


Discussion of Piero’s entitlement to retirement benefits is simply an afterthought and cannot be said to be wholly
connected to the case at all.

Angeles| Bajana | Balladares | Brillantes | Briones | Cabansag | Callanta | Chua | David|


De Leon | Gomez | Lopez | Macalino | Nostratis | Padilla | Reynon | Santos | Tan |Velasco
4E / 4F - 2018-2019
Page 853 of 920
LABOR REVIEW DIGEST
Atty. Joyrich Golangco

3. STA. CATALINA COLLEGE V. NLRC


GR NO: 144483 DATE: NOVEMBER 19, 2003
PONENTE: CARPIO MORALES, J.

Doctrine:
An employee’s re-employment as a new employee would mean a demotion in rank and privileges, and as to
the computation of retirement benefits, his entire previous years of service would simply be considered as
nonexistent. Gratuity pay is separate and distinct from retirement benefits. Hence, the former should not be
deducted to the latter.

FACTS:
Hilaria G. Tercero (Respondent) was hired on June 1955 as an elementary school teacher in Sta. Catalina College
(Petitioner) at San Antonio, Binan, Laguna. In 1970 she applied for a one year leave of absence due to her mother’s
illness. However, after the expiration of said leave in 1971, respondent had not been heard by petitioner school.
Respondent, on the other hand, taught at San Pedro Parochial school, during the years of 1980-1981, and Liceo de
San Pedro, Bian, Laguna from 1981-1982. Later on, in 1982, respondent applied anew with petitioner school.

On March 22, 1997, during the 51st commencement exercises of petitioner school, respondent was awarded a plaque
of appreciation for her 30 years of service, and 12,000 pesos as gratuity pay. Upon reaching the compulsory
retirement age of 65, petitioner school computed respondent’s retirement benefits at P59,038.35 on account of 15
years of service (1982-1997), considering she abandoned her work during her employment of 1955-1970. Her
employers contribution to her retirement benefits under the Private Education Retirement Annuity Association
(PERAA) was also deducted from her retirement benefits including the gratuity pay of 12,000 pesos. Respondent,
however, insists that she did not abandoned her work, and as such her retirement benefits must be computed for her
service of 30 years (1955-1997) and that her gratuity pay shouldn’t be deducted from the said retirement benefits as
well.

LA RULING:
Ruled in favor of respondent, ordering petitioner to pay the former with retirement benefits based on respondent’s
computation (which considers her number of years of service as 30 and does not deduct the gratuity pay from the
retirement benefits)

NLRC RULING:
The NLRC set aside the LA”s ruling and ordered petitioner school to pay the total amount of P85,287.72 computed
as follows: P3,935.89 (total computation of the retirement components) MULTIPLIED by 29 (number of years in
service) EQUALS P114,140.81 (total retirement package) LESS P28,287.72 (representing respondent-appellees
contribution with the PERAA proven to have already been received by complainant-appellant).However, the gratuity
pay earlier already given was not to be deducted from the retirement package.

CA RULING:
The CA dismissed the petition filed by petitioner school holding that petitioners failed to prove that Hilaria had
abandoned her position in 1970, as petitioner school even gave her a Plaque of Appreciation for thirty years of
service precisely because of her thirty year continuous service, and that petitioner school never sent notice to her
dismissing her, hence, the employer-employee relationship was not severed and, therefore, her services for
petitioner school during the period from 1955-1970 should be credited in the computation of her retirement benefits.
In addition, it affirmed the NLRC ruling providing that the gratuity pay should not be deducted from the retirement
benefits.

ISSUES:
1. Did respondent abandon her work?
2. Should the gratuity pay be deducted from the retirement benefits?

HELD:
1. YES. The Court held that it is not disputed that the approved one year leave of absence without pay of
Hilaria expired in 1971, without her, it bears repeating, requesting for extension thereof or notifying petitioner
school if and when she would resume teaching. Nor is it disputed that she was rehired only in 1982 after
filing anew an application, without her proffering any explanation for her more than a decade of
absence. Under the circumstances, abandonment of work at petitioner school in 1971 is indubitably

Angeles| Bajana | Balladares | Brillantes | Briones | Cabansag | Callanta | Chua | David|


De Leon | Gomez | Lopez | Macalino | Nostratis | Padilla | Reynon | Santos | Tan |Velasco
4E / 4F - 2018-2019
Page 854 of 920
LABOR REVIEW DIGEST
Atty. Joyrich Golangco

manifest. Further, an employee’s re-employment as a new employee would mean a demotion in rank and
privileges, and as to the computation of retirement benefits, his entire previous years of service would
simply be considered as nonexistent.

2. NO. The Court held that gratuity pay is separate and distinct from retirement benefits. Hence, the former
should not be deducted to the latter. Citing Republic Planters Bank v. NLRC, it further stated that:

Gratuity pay x x x is paid to the beneficiary for the past services or favor rendered purely out of the
generosity of the giver or grantor. Gratuity, therefore, is not intended to pay a worker for actual services
rendered or for actual performance. It is a money benefit or bounty given to the worker, the purpose of
which is to reward employees who have rendered satisfactory service to the company.

The Court also stated that retirement benefits, on the other hand, are intended to help the employee enjoy
the remaining years of his life, releasing him from the burden of worrying for his financial support, and are a
form of reward for his loyalty to the employer.

Angeles| Bajana | Balladares | Brillantes | Briones | Cabansag | Callanta | Chua | David|


De Leon | Gomez | Lopez | Macalino | Nostratis | Padilla | Reynon | Santos | Tan |Velasco
4E / 4F - 2018-2019
Page 855 of 920
LABOR REVIEW DIGEST
Atty. Joyrich Golangco

4. PANTRANCO NORTH EXPRESS, INC v. NLRC and URBANO SUÑIGA


GR No.: 95940 Date: July 24, 1996
Ponente: PANGANIBAN, J.

DOCTRINE:

Article 287, as amended by the Retirement Pay Law, provides that an employee may be retired upon reaching
the retirement age established in the collective bargaining agreement or other applicable employment
contract. This provision makes clear the intention of spirit of the law to give employers and employees a free
hand to determine and agree upon the terms and conditions of retirement. Providing in a CBA for
compulsory retirement of employees after twenty-five (25) years of service is legal and enforceable so long
as the parties agree to be governed by such CBA. The law presumes that employees know what they want
and what is good for them absent any showing that fraud or intimidation was employed to secure their
consent thereto.

FACTS:

Suñiga was hired by Pantranco in 1964 as a bus conductor. He eventually joined the Pantranco Employees
Association-PTGWO. He continued the antranco’s employ until August 12, 1989, when he was retired at the age of
fifty-two (52) after having rendered 25 years of service. The basis of his retirement was the compulsory retirement
provision of the collective bargaining agreement between the Pantranco and the aforenamed union.

Suñiga received P49,300.00 as retirement pay.

On February 15, 1990, Suñiga filed a complaint for illegal dismissal against Pantranco with the LA The complaint was
consolidated with two other cases of illegal dismissal having similar facts and issues, filed by the other employees,
non-union members.

Pantranco contends that the LA had no jurisdiction because the dispute concerns a provision of the CBA and its
interpretation.

LA RULING: Suñiga was ILLEGALLY DISMISSED; Reinstated; Money received shall be considered as an advance
to his retirement pay. The LA has jurisdiction. The interpretation of the CBA or enforcement of the company policy is
only corollary to the complaint of illegal dismissal. Otherwise, an employee who was on AWOL, or who committed
offenses contrary to the personnel policies can no longer file a case of illegal dismissal because the discharge is
premised on the interpretation or enforcement of the company policies.

Suñiga is entitled to:

P 27,375.00 — Backwages, Aug. 16/89 to March 31/90 (P3,650.00 x 7.5 mos.)


1,368.75 — 13th month pay for 1989 (P16,425.00 over 12)
——————
P 28,743.75
2,874.37 — 10% attorney's fees
——————
P 31,618.12 — Total as of March 31/90 plus additional backwages and other benefits but not to exceed 3 years and
the corresponding attorney's fees.

The amounts already received by complainants shall be considered as advanced payment of their retirement pay
which shall be deducted when they shall actually retire or (be) separated from the service.

NLRC RULING: AFFIRMED the decision of the LA in toto

ISSUE/S:

1. Is a Collective Bargaining Agreement provision allowing compulsory retirement before age 60 but after 25 years of
service legal and enforceable?

2. Who has jurisdiction over a case involving such a question — the labor arbiter or arbitrators authorized by such
CBA?

Angeles| Bajana | Balladares | Brillantes | Briones | Cabansag | Callanta | Chua | David|


De Leon | Gomez | Lopez | Macalino | Nostratis | Padilla | Reynon | Santos | Tan |Velasco
4E / 4F - 2018-2019
Page 856 of 920
LABOR REVIEW DIGEST
Atty. Joyrich Golangco

HELD:

1. YES. Art. 287 of the Labor Code as worded permits employers and employees to fix the applicable retirement age
at below 60 years. Moreover, providing for early retirement does not constitute diminution of benefits.

Further, a CBA incorporates the agreement reached after negotiations between employer and bargaining agent with
respect to terms and conditions of employment. A CBA is not an ordinary contract. It is impressed with public interest
which must be construed liberally rather than narrowly and technically, and the courts must place a practical and
realistic construction upon it, giving due consideration to the context in which it is negotiated and purpose which it is
intended to serve.

Being a product of negotiation, the CBA between Pantranco and the union intended the provision on compulsory
retirement to be beneficial to the employees-union members, including Suñiga. When Suñiga ratified the CBA with
the union, he not only agreed to the CBA but also agreed to conform to and abide by its provisions. Thus, it cannot
be said that he was illegally dismissed when the CBA provision on compulsory retirement was applied to his case.

2. The LA has jurisdiction over the matter.

It cannot be said that the "dispute" is between the union and Pantranco because both have previously agreed upon
the provision on "compulsory retirement" as embodied in the CBA. Also, it was only Suñiga on his own who
questioned the compulsory retirement. Thus, the case is properly denominated as a "termination dispute" which
comes under the jurisdiction of labor arbiters.

Angeles| Bajana | Balladares | Brillantes | Briones | Cabansag | Callanta | Chua | David|


De Leon | Gomez | Lopez | Macalino | Nostratis | Padilla | Reynon | Santos | Tan |Velasco
4E / 4F - 2018-2019
Page 857 of 920
LABOR REVIEW DIGEST
Atty. Joyrich Golangco

5. R AND E TRANSPORT V. LATAG


GR NO: 199890 DATE: FEBRUARY 13, 2004
PONENTE: REYES, J.

Doctrine:
Courts have stepped in to annul questionable transactions, especially where there is clear proof that a
waiver, for instance, was wangled from an unsuspecting or a gullible person; or where the agreement or
settlement was "unconscionable on its face." A quitclaim is ineffective in barring recovery of the full
measure of a worker’s rights, and the acceptance of benefits therefrom does not amount to estoppel.
Moreover, a quitclaim in which the consideration is "scandalously low and inequitable" cannot be an
obstacle to the pursuit of a worker’s legitimate claim.

FACTS:

Pedro Latag was the regular employee of La Mallorca Taxi since March 1, 1961. When the company ceased
operations, Pedro transferred employment to petitioner R and E Transport. Pedro subsequently got sick in January
1995. Upon his return to work on September 1998, he was no longer allowed to work due to his old age. Pedro then
asked Felix Fabros, administrative officer of petitioner company for his retirement benefits byut was denied,
prompting him to file a case for the payment of his retirement benefits with the NLRC.

Pedro later on died and was substituted by his wife, respondent Avelina Latag. On January 10, 2000, the Labor
Arbiter rendered a decision in favor of respondent. Later on January 21, 2000, respondent with her counsel were
invited to petitioner’s office and was offered an amount of P 38,500 (far less than the P277,500.00 awarded by the
Labor Arbiter as a retirement pay) which she accepted. Respondent then asked to sign an already prepared quitclaim
and release and a joint motion to dismiss the case. On the strength of such quitclaim and motion, petitioner then
appealed the Labor Arbiter’s ruling and moved for the dismissal of the case.

LA RULING:
Pedro is entitled to retirement pay in the amount of P277,500

NLRC RULING:
Petitioner’s motion to dismiss was denied to failure to file the required cash bond.

CA RULING:
Affirmed NLRC’s ruling, rendering the LA’s ruling, final and executory.

ISSUE:
Is the quitclaim valid?

HELD:
No. The Court ruled that are wary of schemes that frustrate workers’ rights and benefits, and look with disfavor upon
quitclaims and waivers that bargain these away. Courts have stepped in to annul questionable transactions,
especially where there is clear proof that a waiver, for instance, was wangled from an unsuspecting or a gullible
person; or where the agreement or settlement was "unconscionable on its face." A quitclaim is ineffective in barring
recovery of the full measure of a worker’s rights, and the acceptance of benefits therefrom does not amount to
estoppel. Moreover, a quitclaim in which the consideration is "scandalously low and inequitable" cannot be an
obstacle to the pursuit of a worker’s legitimate claim.

Other Notes/ SC Pronouncements:


There was an issue as to the number of creditable of years of service for Pedro, with the LA and CA ruling it should
be 37 (23 – Mallorca + 14 – R and E Transport) due to the fact that R and E Transport is just a mere continuation of
La Mallorca Taxi. On the other hand, the NLRC ruled that it should only be 14 years (considering only the years
served under R and E) since La Mallorca and R and E are two separate entities. The SC ruled that it should only be
14 years since R and E Transport is not a continuation of La Mallorca. It ruled:

After a careful and painstaking review of the evidence on record, we support the NLRCs findings. The labor arbiters
conclusion -- that Mallorca Taxi and R & E Transport, Inc., are one and the same entity -- is negated by the
documentary evidence presented by petitioners. Their evidence[22] sufficiently shows the following facts: 1) R & E
Transport, Inc., was established only in 1978; 2) Honorio Enriquez, its president, was not a stockholder of La
Mallorca Taxi; and 3) none of the stockholders of the latter company hold stocks in the former. In the face of such

Angeles| Bajana | Balladares | Brillantes | Briones | Cabansag | Callanta | Chua | David|


De Leon | Gomez | Lopez | Macalino | Nostratis | Padilla | Reynon | Santos | Tan |Velasco
4E / 4F - 2018-2019
Page 858 of 920
LABOR REVIEW DIGEST
Atty. Joyrich Golangco

evidence, which the NLRC appreciated in its Decision, it seems that mere surmises and self-serving assertions of
Respondent Avelina Latag formed the bases for the labor arbiter’s conclusions.

Angeles| Bajana | Balladares | Brillantes | Briones | Cabansag | Callanta | Chua | David|


De Leon | Gomez | Lopez | Macalino | Nostratis | Padilla | Reynon | Santos | Tan |Velasco
4E / 4F - 2018-2019
Page 859 of 920
LABOR REVIEW DIGEST
Atty. Joyrich Golangco

6. SERRANO VS. SEVERINO SANTOS TRANSIT


GR NO: 187698 DATE: AUGUST 9, 2010
PONENTE: CARPIO-MORALES, J

Doctrine: For purposes of applying the law on SIL, as well as on retirement, the Court notes that there is a
difference between drivers paid under the boundary system and conductors who are paid on commission
basis.

In practice, taxi drivers do not receive fixed wages. They retain only those sums in excess of the boundary or
fee they pay to the owners or operators of the vehicles. Conductors, on the other hand, are paid a certain
percentage of the bus earnings for the day.

It bears emphasis that under P.D. 851 or the SIL Law, the exclusion from its coverage of workers who are
paid on a purely commission basis is only with respect to field personnel.

FACTS: Rodolfo J. Serrano was hired on September 28, 1992 as bus conductor by respondent Severino Santos
Transit, a bus company owned and operated by its co-respondent Severino Santos.

After 14 years of service or on July 14, 2006, petitioner applied for optional retirement from the company whose
representative advised him that he must first sign the already prepared Quitclaim before his retirement pay could be
released. As petitioners request to first go over the computation of his retirement pay was denied, he signed the
Quitclaim on which he wrote U.P. (under protest) after his signature

Petitioner soon after filed a complaint before the Labor Arbiter, alleging that the company erred in its computation
since under Republic Act No. 7641, otherwise known as the Retirement Pay Law, his retirement pay should have
been computed at 22.5 days per year of service to include the cash equivalent of the 5-day service incentive leave
(SIL) and 1/12 of the 13th month pay which the company did not.

The company maintained, however, that the Quitclaim signed by petitioner barred his claim and, in any event, its
computation was correct since petitioner was not entitled to the 5-day SIL and pro-rated 13th month pay for, as a bus
conductor, he was paid on commission basis.

LA RULING: Labor Arbiter ruled in favor of petitioner, awarding him P116,135.45 as retirement pay differential, and
10% of the total monetary award as attorneys fees.

NLRC/CA RULING: The NLRC reversed the LA’s ruling. NLRC held that since petitioner was paid on purely
commission basis, he was excluded from the coverage of the laws on 13th month pay and SIL pay, hence, the 1/12
of the 13thmonth pay and the 5-day SIL should not be factored in the computation of his retirement pay.

APPEAL TO THE SC:

Petitioner's Contention: (see facts)

Respondent's Contention: (see facts)

ISSUE/S: whether or not petitioner is entitled to the cash equivalent of the SIL as part of his retirement pay

HELD: YES. Republic Act No. 7641 which was enacted on December 9, 1992 amended Article 287 of the Labor
Code by providing for retirement pay to qualified private sector employees in the absence of any retirement plan in
the establishment. The pertinent provision of said law reads:

Section 1. Article 287 of Presidential Decree No. 442, as amended, otherwise known as the Labor Code of
the Philippines, is hereby amended to read as follows:

xxxx
In the absence of a retirement plan or agreement providing for retirement benefits of employees in the
establishment, an employee upon reaching the age of sixty (60) years or more, but not beyond sixty-five

Angeles| Bajana | Balladares | Brillantes | Briones | Cabansag | Callanta | Chua | David|


De Leon | Gomez | Lopez | Macalino | Nostratis | Padilla | Reynon | Santos | Tan |Velasco
4E / 4F - 2018-2019
Page 860 of 920
LABOR REVIEW DIGEST
Atty. Joyrich Golangco

(65) years which is hereby declared the compulsory retirement age, who has served at least five (5) years in
the said establishment, may retire and shall be entitled to retirement pay equivalent to at least one-half (1/2)
month salary for every year of service, a fraction of at least six (6) months being considered as one whole
year.

Unless the parties provide for broader inclusions, the term one-half (1/2) month salary shall mean fifteen
(15) days plus one-twelfth (1/12) of the 13th month pay and the cash equivalent of not more than five (5)
days of service incentive leaves.

Retail, service and agricultural establishments or operations employing not more than (10) employees or
workers are exempted from the coverage of this provision.

Further, the Implementing Rules of said law provide:


SECTION 5

Retirement Benefits.

5.1 In the absence of an applicable agreement or retirement plan, an employee who retires pursuant to the
Act shall be entitled to retirement pay equivalent to at least one-half (―) month salary for every year of
service, a fraction of at least six (6) months being considered as one whole year.

5.2 Components of One-half (―) Month Salary. For the purpose of determining the minimum retirement
pay due an employee under this Rule, the term one-half month salary shall include all of the following:
(a) Fifteen (15) days salary of the employee based on his latest salary rate. As used
herein, the term salary includes all remunerations paid by an employer to his employees for
services rendered during normal working days and hours, whether such payments are fixed
or ascertained on a time, task, piece of commission basis, or other method of calculating the
same, and includes the fair and reasonable value, as determined by the Secretary of Labor and
Employment, of food, lodging or other facilities customarily furnished by the employer to his
employees.

(b) The cash equivalent of not more than five (5) days of service incentive leave;
(c) One-twelfth of the 13th month pay due the employee.
(d) All other benefits that the employer and employee may agree upon

Admittedly, petitioner worked for 14 years for the bus company which did not adopt any retirement scheme. Even if
petitioner as bus conductor was paid on commission basis then, he falls within the coverage of R.A. 7641 and its
implementing rules. As thus correctly ruled by the Labor Arbiter, petitioners retirement pay should include the cash
equivalent of the 5-day SIL and 1/12 of the 13th month pay.

For purposes of applying the law on SIL, as well as on retirement, the Court notes that there is a difference between
drivers paid under the boundary system and conductors who are paid on commission basis.

In practice, taxi drivers do not receive fixed wages. They retain only those sums in excess of the boundary or fee they
pay to the owners or operators of the vehicles. Conductors, on the other hand, are paid a certain percentage of the
bus earnings for the day.

It bears emphasis that under P.D. 851 or the SIL Law, the exclusion from its coverage of workers who are paid on a
purely commission basis is only with respect to field personnel.

According to Article 82 of the Labor Code, field personnel shall refer to non-agricultural employees who regularly
perform their duties away from the principal place of business or branch office of the employer and whose actual
hours of work in the field cannot be determined with reasonable certainty.

As a general rule, [field personnel] are those whose performance of their job/service is not supervised by the
employer or his representative, the workplace being away from the principal office and whose hours and days of work
cannot be determined with reasonable certainty; hence, they are paid specific amount for rendering specific service
or performing specific work. If required to be at specific places at specific times, employees including drivers cannot
be said to be field personnel despite the fact that they are performing work away from the principal office of the
employee.

Angeles| Bajana | Balladares | Brillantes | Briones | Cabansag | Callanta | Chua | David|


De Leon | Gomez | Lopez | Macalino | Nostratis | Padilla | Reynon | Santos | Tan |Velasco
4E / 4F - 2018-2019
Page 861 of 920
LABOR REVIEW DIGEST
Atty. Joyrich Golangco

Angeles| Bajana | Balladares | Brillantes | Briones | Cabansag | Callanta | Chua | David|


De Leon | Gomez | Lopez | Macalino | Nostratis | Padilla | Reynon | Santos | Tan |Velasco
4E / 4F - 2018-2019
Page 862 of 920
LABOR REVIEW DIGEST
Atty. Joyrich Golangco

7. OBUSAN v. PHILIPPINE NATIONAL BANK


G.R. No. 181178 July 26, 2010
NACHURA, J.:

DOCTRINE:
Article 287 of the Labor Code, as amended, applies only to a situation where (1) there is no CBA or other
applicable employment contract providing for retirement benefits for an employee; or (2) there is a collective
bargaining agreement or other applicable employment contract providing for retirement benefits for an
employee, but it is below the requirement set by law.

FACTS:
Back in 1979, respondent Philippine National Bank (PNB) hired petitioner Amelia R. Obusan, who eventually became
the Manager of the PNB Medical Office. At that time, PNB was a government-owned or controlled corporation, whose
retirement program for its employees was administered by the GSIS, pursuant to the Revised Government Service
Insurance Act of 1977 (PD No. 1146).

On May 27, 1996, PNB was privatized pursuant to the Revised Charter of the PNB (EO No. 80).

Consequent to the privatization, all PNB employees, including Obusan, were deemed retired from the government
service. The GSIS confirmed Obusans retirement from the government service, and accordingly paid her retirement
gratuity in the net amount of P390,633.76. Thereafter, Obusan continued to be an employee of PNB.

Later, the PNB Board of Directors, through Resolution No. 30 dated December 22, 2000, as amended, approved the
PNB Regular Retirement Plan (PNB-RRP). Section 1, Article VI of which provides

Normal Retirement. The normal retirement date of a Member shall be the day he attains sixty (60) years of age,
regardless of length of service or has rendered thirty (30) years of service, regardless of age, whichever of the said
conditions comes first. A Member who has reached the normal retirement date shall have to compulsorily retire and
shall be entitled to receive the retirement benefits under the Plan.

PNB informed its officers and employees of the terms and conditions of the PNB-RRP, along with its implementing
guidelines.

Subsequently, the PNB-RRP was registered with the BIR. Later, the Philnabank Employees Association, the union of
PNB rank-and-file employees, recognized the PNB-RRP in the Collective Bargaining Agreement (CBA) it entered
with PNB.

PNB informed Obusan that her last day of employment would be on March 3, 2002, as she would reach the
mandatory retirement age of 60 years on March 4, 2002. Obusan questioned her compulsory retirement and even
threatened to take legal action against PNB for illegal dismissal and unfair labor practice in the form of union busting,
Obusan being then the President of the PNB Supervisors and Officers Association.

PNB replied to Obusan, explaining that compulsory retirement under the PNB-RRP is not contrary to law and does
not constitute union busting. Dissatisfied with PNBs explanation, Obusan filed before the Labor Arbiter a complaint
for illegal dismissal and unfair labor practice, claiming that PNB could not compulsorily retire her at the age of 60
years, with her having a vested right to be retired only at 65 years old pursuant to civil service regulations.

LA RULING: LA dismissed Obusan's complaint as he upheld the validity of the PNB-RRP and its provisions on
compulsory retirement upon reaching the age of 60 years.

NLRC RULING: NLRC dismissed Obusan's appeal, and affirmed the assailed decision in toto. Obusan's motion for
reconsideration of this resolution was later denied.

CA RULING: CA dismissed the petition ratiocinating that the PNB-RRPs lowering the compulsory retirement age to
60 years is not violative of Article 287 of the Labor Code of the Philippines, as amended, despite the issuance of the
plan years after Obusan was hired. Obusans motion for reconsideration of this Decision was subsequently denied.

ISSUE: Can PNB unilaterally lower the compulsory retirement age to 60 years without violating Article 287 of the
Labor Code and Obusans alleged right to retire at the age of 65 years?

Angeles| Bajana | Balladares | Brillantes | Briones | Cabansag | Callanta | Chua | David|


De Leon | Gomez | Lopez | Macalino | Nostratis | Padilla | Reynon | Santos | Tan |Velasco
4E / 4F - 2018-2019
Page 863 of 920
LABOR REVIEW DIGEST
Atty. Joyrich Golangco

SC RULING:

Yes. Article 287 of the Labor Code, as amended by RA No. 7641, which took effect on January 7, 1993,
provides:
ART. 287. Retirement. Any employee may be retired upon reaching the retirement age established
in the collective bargaining agreement or other applicable employment contract.

In case of retirement, the employee shall be entitled to receive such retirement benefits as he may
have earned under existing laws and any collective bargaining agreement and other agreements:
Provided, however, That an employees retirement benefits under any collective bargaining
agreement and other agreements shall not be less than those provided herein.

In the absence of a retirement plan or agreement providing for retirement benefits of employees in
the establishment, an employee upon reaching the age of sixty (60) years or more, but not beyond
sixty-five (65) years which is hereby declared the compulsory retirement age, who has served at
least five (5) years in the said establishment, may retire and shall be entitled to retirement pay
equivalent to at least one-half (1/2) month salary for every year of service, a fraction of at least six
(6) months being considered as one whole year.

Unless the parties provide for broader inclusions, the term one-half (1/2) month salary shall mean
fifteen (15) days plus one-twelfth (1/12) of the 13th month pay and the cash equivalent of not more
than five (5) days of service incentive leaves.

Undoubtedly, under this provision, the retirement age is primarily determined by the existing agreement or
employment contract. Absent such an agreement, the retirement age shall be fixed by law. The above-cited law
mandates that the compulsory retirement age is at 65 years, while the minimum age for optional retirement is set at
60 years. Moreover, Article 287 of the Labor Code, as amended, applies only to a situation where (1) there is no CBA
or other applicable employment contract providing for retirement benefits for an employee; or (2) there is a collective
bargaining agreement or other applicable employment contract providing for retirement benefits for an employee, but
it is below the requirement set by law. The rationale for the first situation is to prevent the absurd situation where an
employee, deserving to receive retirement benefits, is denied them through the nefarious scheme of employers to
deprive employees of the benefits due them under existing labor laws. The rationale for the second situation is to
prevent private contracts from derogating from the public law.

Retirement plans allowing employers to retire employees who have not yet reached the compulsory retirement age of
65 years are not per se repugnant to the constitutional guaranty of security of tenure. By its express language, the
Labor Code permits employers and employees to fix the applicable retirement age at 60 years or below, provided
that the employees retirement benefits under any CBA and other agreements shall not be less than those provided
therein. By this yardstick, the PNB-RRP complies.

Angeles| Bajana | Balladares | Brillantes | Briones | Cabansag | Callanta | Chua | David|


De Leon | Gomez | Lopez | Macalino | Nostratis | Padilla | Reynon | Santos | Tan |Velasco
4E / 4F - 2018-2019
Page 864 of 920
LABOR REVIEW DIGEST
Atty. Joyrich Golangco

8. KIMBERLY CLARK PHILS vs DIMAYUGA


GR No: 177705 Date: September 18, 2009
Ponente: Carpio-Morales, J.

Doctrine:
It is settled that entitlement of employees to retirement benefits must specifically be granted under existing
laws, a collective bargaining agreement or employment contract, or an established employer policy. No law or
collective bargaining agreement or other applicable contract, or an established company policy was existing
during respondents employment entitling them to the P200,000 lump-sum retirement pay. Petitioner was not
thus obliged to grant them such pay.

FACTS:
Respondents were employees of Kimberly-Clark Philippines, Inc. (petitioner). Nora Dimayuga (Nora) was
Cost Accounting Supervisor, Rosemarie Gloria (Rosemarie) was Business Analyst, and Maricar de Guia (Maricar)
was General Accounting Manager.
On September 19, 2002, Nora tendered her resignation effective October 21, 2002. On October 7, 2002,
Rosemarie tendered her resignation, also effective October 21, 2002.
As petitioner had been experiencing a downward trend in its sales, it created a tax-free early
retirement package for its employees as a cost-cutting and streamlining measure. Twenty-four of its employees
availed of the offer that was made available from November 10-30, 2002.
Despite their resignation before the early retirement package was offered, Nora and Rosemarie pleaded
with petitioner that they be retroactively extended the benefits thereunder, to which petitioner acceded. Hence, Nora
received a total of P1,025,113.73 while Rosemarie received a total of P1,006,493.94, in consideration of which they
executed release and quitclaim deeds dated January 17, 2003 and January 16, 2003, respectively.
On November 4, 2002, Maricar tendered her resignation effective December 1, 2002, citing career
advancement as the reason therefor. As at the time of her resignation the early retirement package was still effective,
she received a total of P523,540.13 for which she signed a release and quitclaim.
On November 28, 2002, petitioner announced that in lieu of the merit increase which it did not give that
year, it would provide economic assistance, to be released the following day, to all monthly-paid employees on
regular status as of November 16, 2002.
Still later or on January 16, 2003, petitioner announced that it would the grant a lump sum retirement pay in
the amount of P200,000, in addition to the early retirement package benefit, to those who signed up for early
retirement and who would sign up until January 22, 2003.
On May 23, 2003, respondents filed a Complaint before the NLRC Regional Arbitration Branch No. IV
against petitioner and its Finance Manager Fernando B. Gomez (Gomez) whom respondents alleged to be
responsible for the withholding of [their] additional retirement benefits, claiming entitlement to the P200,000 lump sum
retirement pay. Respondents Nora and Rosemarie additionally claimed entitlement to the economic assistance.

LA RULING:
LA dismissed the claims of Nora and Rosemarie, holding that they were not entitled to the P200,000 lump sum
retirement pay, they having ceased to be employees of petitioner at the time it was offered or made effective on
January 16, 2003. He, however, granted Maricars claim for the same pay, holding that she was entitled to it because
at the time she resigned from the company effective December 1, 2002, such pay was already offered. Besides, the
Labor Arbiter ruled, Maricar had a vested right to it as she was given a formal notice of her entitlement to it by
petitioner, through its Human Resources Director.

NLRC RULING
Modified the Labor Arbiters Decision by ordering petitioner to pay Nora P200,000 additional bonus and P2,880
economic assistance, and to pay Rosemarie P200,000 additional bonus and P2,656 economic assistance. It affirmed
Maricar’s entitlement to the lump sum retirement pay.
Applying the ruling in Businessday Information Systems and Services, Inc. v. NLRC (Businessday), the NLRC
ratiocinated that petitioners refusal to give Nora and Rosemarie the lump sum retirement pay was an act of
discrimination, more so because a certain Oscar Diokno, another employee who presumably resigned also prior to
January 16, 2003, was given said benefit.
As to the award of economic assistance, the NLRC held that Nora and Rosemarie were also entitled to it as the
same was given in lieu of the annual performance-based salary increase that was not given in 2002 and, therefore,
already earned by them when they resigned.

CA RULING:
Affirmed NLRC decision

Angeles| Bajana | Balladares | Brillantes | Briones | Cabansag | Callanta | Chua | David|


De Leon | Gomez | Lopez | Macalino | Nostratis | Padilla | Reynon | Santos | Tan |Velasco
4E / 4F - 2018-2019
Page 865 of 920
LABOR REVIEW DIGEST
Atty. Joyrich Golangco

Respondent's Contention:
Respondents argue that since other employees who resigned before the announcement of the grant of the lump sum
retirement pay received the same, they (respondents) should also receive it, citing the pronouncement
in Businessday that: The law requires an employer to extend equal treatment to its employees. It may not, in the guise
of exercising management prerogatives, grant greater benefits to some and less to others. Management prerogatives
are not absolute prerogatives but are subject to legal limits, collective bargaining agreements, or general principles of
fair play and justice.

ISSUE/S:
Whether or not respondents Nora, Rosemarie and Maricar are entitled to retirement benefits and economic
assistance

HELD: NO
It is settled that entitlement of employees to retirement benefits must specifically be granted under existing
laws, a collective bargaining agreement or employment contract, or an established employer policy. No law or
collective bargaining agreement or other applicable contract, or an established company policy was existing during
respondents employment entitling them to the P200,000 lump-sum retirement pay. Petitioner was not thus obliged to
grant them such pay.
Respondents’ reliance on Businessday is misplaced. The factual milieu in Businessday is markedly different
from that of the present case. That case involved the retrenched employees’ separation pay to which they are entitled
under Article 283 of the Labor Code. In the present case, Nora and Rosemarie resigned prior to petitioner’s offer of
the lump sum retirement pay as an incentive to those employees who would voluntarily avail of its early retirement
scheme as a cost-cutting and streamlining measure. That respondents resigned, and not retrenched, is clear from
their respective letters to petitioner. And nowhere in the letters is there any allegation that they resigned in view of the
company’s downward trend in sales which necessitated downsizing or streamlining.
The appellate court’s finding that petitioner’s inclusion of Nora and Rosemarie in the termination report
submitted to the DOLE and its grant to them of the early retirement benefits made them full retirees to thus entitle
them to the same benefits offered to those who would voluntarily resign after November 16, 2003 does not lie.
Petitioner’s claim that it allowed Nora and Rosemarie to avail of the early retirement package despite their
previous separation from the company out of pure generosity is well-taken in light of Noras letter of September 15,
2002 asking if she could avail of the early retirement package as it would certainly be of great assistance to us
financially. It is thus absurd to fault petitioner for acceding to such a request out of compassion by directing it to pay
additional benefits to resigned employees who are not entitled thereto.
Petitioner’s decision to extend the benefit to some former employees who had already resigned before the
offer of the lump sum pay incentive was thus an act of generosity which it is not obliged to extend to respondents.
Neither are Nora and Rosemarie entitled to the economic assistance which petitioner awarded to all monthly
employees who are under regular status as of November 16, 2002, they having resigned earlier or on October 21,
2002.
Again, contrary to the appellate court’s ruling that Nora and Rosemarie already earned the economic
assistance, the same having been given in lieu of the performance-based annual salary increase, the Court finds that
the economic assistance was a bonus over and above the employees salaries and allowances. A perusal of the
memorandum regarding the grant of economic assistance shows that it was granted in lieu of salary increase (the
grant of which depends on petitioners financial capability) and that it was not intended to be a counterpart of the
Collective Bargaining Agreement grant to members of the K-CPI union. The grant of economic assistance to all
monthly employees under regular status as of November 16, 2002 was thus well within petitioners prerogatives.
Moreover, petitioners decision to give economic assistance was arrived at more than a
month after respondents resignation and, therefore, it was a benefit not yet existing at the time of their separation. In
any event, assuming that Nora and Rosemarie are entitled to the economic assistance, they had signed release and
quitclaim deeds upon their resignation in which they waived.
While quitclaims executed by employees are commonly frowned upon as being contrary to public policy and
are ineffective to bar claims for the full measure of their legal rights, where the person making the waiver has done so
voluntarily, with a full understanding thereof, and the consideration for the quitclaim is credible and reasonable, the
transaction must be recognized as being a valid and binding undertaking. In the case at bar, Nora and Rosemarie are
Accounting graduates. They have not alleged having been compelled to sign the quitclaims, nor that the
considerations thereof (P1,024,113.73 for Nora and P682,721.24 for Rosemarie) are unconscionable.
As for Maricar’s claim to the lump sum retirement pay, the Court finds that, like Nora and Rosemarie, she is
not entitled to it. Although the incentive was offered when she was still connected with petitioner, she resigned from
employment, citing career advancement as the reason therefor. Indubitably, the incentive was addressed to those
employees who, without prior plans of resigning, opted to terminate their employment in light of the downsizing being

Angeles| Bajana | Balladares | Brillantes | Briones | Cabansag | Callanta | Chua | David|


De Leon | Gomez | Lopez | Macalino | Nostratis | Padilla | Reynon | Santos | Tan |Velasco
4E / 4F - 2018-2019
Page 866 of 920
LABOR REVIEW DIGEST
Atty. Joyrich Golangco

undertaken by petitioner. In other words, Maricar resigned from petitioner in order to find gainful employment
elsewhere a reason which has no bearing on the financial viability of petitioner.

Angeles| Bajana | Balladares | Brillantes | Briones | Cabansag | Callanta | Chua | David|


De Leon | Gomez | Lopez | Macalino | Nostratis | Padilla | Reynon | Santos | Tan |Velasco
4E / 4F - 2018-2019
Page 867 of 920
LABOR REVIEW DIGEST
Atty. Joyrich Golangco

9. MAGDADARO V. PNB
G.R. NO. 166198, JULY 17, 2009
CARPIO J.

DOCTRINE:

Retirement is the result of a bilateral act of the parties, a voluntary agreement between the employer and the
employee whereby the latter, after reaching a certain age, agrees to sever his or her employment with the
former. Retirement is provided for under Article 287 of the Labor Code, as amended by Republic Act No.
7641, or is determined by an existing agreement between the employer and the employee.

FACTS:

Marcelino A. Magdadaro was employed by PNB since 1968. In 1998, Magdadaro filed his application for early
retirement under respondents Special Separation Incentive Program (SSIP). Petitioner stated in his application that
31 December 1999 was his preferred effective date of retirement.

Respondent approved the application for early retirement but made it effective on 31 December 1998. Petitioner
protested the acceleration of his retirement. He received, under protest, his retirement and separation benefits
amounting to P908,950.44.

Later, petitioner filed a complaint for illegal dismissal and payment of moral, exemplary and actual damages.

LA/NLRC RULING:

The Labor Arbiter ruled that respondent had the discretion and prerogative to set the effective date of retirement
under the SSIP. The Labor Arbiter ruled that there was no dismissal to speak of because petitioner voluntarily availed
of the SSIP. Still, the Labor Arbiter granted petitioners preferred date of retirement and awarded him additional
retirement benefits.

The NLRC affirmed the Labor Arbiters Decision. However, the NLRC considered petitioners retirement on 31
December 1998 as tantamount to illegal dismissal. The NLRC ruled that while it recognized respondents prerogative
to change petitioners retirement date, management prerogative should be exercised with prudence and without
malice.

CA RULING:

The Court of Appeals held that petitioner voluntarily applied for the SSIP. The Court of Appeals ruled that petitioner
could not claim to have been illegally dismissed just because the date of effectivity of his retirement did not conform
to his preferred retirement date.

ISSUE:

Whether petitioner was illegally dismissed from employment.

HELD:

 Retirement is the result of a bilateral act of the parties, a voluntary agreement between the employer and
the employee whereby the latter, after reaching a certain age, agrees to sever his or her employment with
the former. Retirement is provided for under Article 287 of the Labor Code, as amended by Republic Act No.
7641, or is determined by an existing agreement between the employer and the employee.

 Whether petitioners early retirement within the SSIP period will improve or impair the delivery of bank
services is a business decision properly within the exercise of management prerogative. More importantly,
the SSIP provides:

7. Management shall have the discretion and prerogative in approving the applications filed
under the Plan, as well as in setting the effectivity dates for separation within the
implementation period of the Plan.

Angeles| Bajana | Balladares | Brillantes | Briones | Cabansag | Callanta | Chua | David|


De Leon | Gomez | Lopez | Macalino | Nostratis | Padilla | Reynon | Santos | Tan |Velasco
4E / 4F - 2018-2019
Page 868 of 920
LABOR REVIEW DIGEST
Atty. Joyrich Golangco

It is clear that it is within respondents prerogative to set the date of effectivity of retirement and it may not be
necessarily what is stated in the application. We see no grave abuse of discretion on the part of respondent
in the exercise of this management prerogative. The exercise of management prerogative is valid provided it
is not performed in a malicious, harsh, oppressive, vindictive or wanton manner or out of malice or spite. In
this case, the NLRCs finding that petitioner received a rating of 70.5% in his working and business relations
is not enough reason to ascribe bad faith on the part of respondent in accelerating the date of effectivity of
petitioners retirement.

Angeles| Bajana | Balladares | Brillantes | Briones | Cabansag | Callanta | Chua | David|


De Leon | Gomez | Lopez | Macalino | Nostratis | Padilla | Reynon | Santos | Tan |Velasco
4E / 4F - 2018-2019
Page 869 of 920
LABOR REVIEW DIGEST
Atty. Joyrich Golangco

10. LAYA V. CA
GR NO: 205813 DATE: JANUARY 10, 2018
PONENTE: BERSAMIN, J.

Doctrine:
An employee in the private sector who did not expressly agree to the terms of an early retirement plan
cannot be separated from the service before he reaches the age of 65 years. The employer who retires the
employee prematurely is guilty of illegal dismissal and is liable to pay his backwages and to reinstate him
without loss of seniority and other benefits, unless the employee has meanwhile reached the mandatory
retirement age under the Labor Code, in which case he is entitled to separation pay pursuant to the terms of
the plan, with legal interest on the backwages and separation pay reckoned from the finality of the decision.

FACTS:
Petitioner Alfredo Laya, Jr. was hired by respondent Philippine Veterans’ Bank as Chief Legal Counsel with a rank of
Vice President. One of the terms of his appointment include membership in the Provident Fund Program/Retirement
Program established by the company. Pertinent provisions of the said program are:

ARTICLE IV RETIREMENT DATES


Section 1. Normal Retirement. The normal retirement date of a Member shall be the first day of the
month coincident with or next following his attainment of age 60.

Section 2. Early Retirement. A Member may, with the approval of the Board of Directors, retire
early on the first day of any month coincident with or following his attainment of age 50 and
completion of at least 10 years of Credited Service.

Section 3. Late Retirement. A Member may, with the approval of the Board of Directors, extend his
service beyond his nonnal retirement date but not beyond age 65. Such deferred retirement shall
be on a case by case and yearly extension basis.

On June 14, 2007 petitioner was informed thru letter by private respondent of his retirement effective July 1, 2007.
Petitioner wrote to Col. Emmanuel De Ocampo, Chairman of the respondent bank, requesting for extension of his
tenure by 2 more years pursuant to the Retirement Plan’s late retirement clause. On June 26, 2008, petitioner
received a memorandum directing him to continue the discharge of his duties. However, on July 18, 2008, petitioner
was informed thru respondent bank’s president, Ricardo Balbido Jr. that his request for extension was denied “to
avoid precedence”. Petitioner sought for reconsideration, but respondent bank certified the effectivity of the former’s
resignation, prompting petitioner to file a complaint for illegal dismissal.

LA RULING:
Dismissed the complaint:

WHEREFORE, the charge of illegal dismissal and money claims raised by the complainant,
together with the counterclaim raised by the respondents are DISMISSED for lack of merit but by
reason of a flaw in the denial of complainant's application for term extension as discussed above,
the respondent bank is hereby ordered to pay the complainant the amount of P200,000.00 by way
of reasonble (sic) indemnity.

NLRC RULING:
Affirmed LA ruling but deleted the award of indemnity

CA RULING:
The CA ruled that the petitioner's acceptance of his appointment as Chief Legal Officer of PVB signified his
conformity to the retirement program; 15 that he could not have been unaware of the retirement program which had
been in effect since January 1, 1996; 16 that the lowering of the retirement age through the retirement plan was a
recognized exception under the provisions of Article 287 of the Labor Code; 17 that considering his failure to adduce
evidence showing that PVB had acted maliciously in applying the provisions

ISSUE:
Was respondent validly retired by respondent bank at the age of 60?

Angeles| Bajana | Balladares | Brillantes | Briones | Cabansag | Callanta | Chua | David|


De Leon | Gomez | Lopez | Macalino | Nostratis | Padilla | Reynon | Santos | Tan |Velasco
4E / 4F - 2018-2019
Page 870 of 920
LABOR REVIEW DIGEST
Atty. Joyrich Golangco

HELD:
No. The Court ruled that under Art. 287 (Now Art. 302) of the Labor Code employers and employees may agree to fix
the retirement age for the latter, and to embody their agreement in either their collective bargaining agreements
(CBAs) or their employment contracts. Retirement plans allowing employers to retire employees who have not yet
reached the compulsory retirement age of 65 years are not per se repugnant to the constitutional guaranty of security
of tenure, provided that the retirement benefits are not lower than those prescribed by law. However, it ruled that the
CA erred in finding that petitioner’s acceptance of his employment carried with it the agreement with the retirement
plan (embodied as one of the terms of petitioner’s appointment), the mere mention of the retirement plan in the letter
of appointment did not sufficiently inform the petitioner of the contents or details of the retirement program. To
construe from the petitioner's acceptance of his appointment that he had acquiesced to be retired earlier than the
compulsory age of 65 years would, therefore, not be warranted. This is because retirement should be the result of the
bilateral act of both the employer and the employee based on their voluntary agreement that the employee agrees to
sever his employment upon reaching a certain age.

That the petitioner might be well aware of the existence of the retirement program at the time of his engagement did
not suffice. His implied knowledge, regardless of duration, did not equate to the voluntary acceptance required by law
in granting an early retirement age option to the employee. The law demanded more than a passive acquiescence on
the part of the employee, considering that his early retirement age option involved conceding the constitutional right
to security of tenure. Citing Cercado v. Umiprom, Inc., it stated:

Acceptance by the employees of an early retirement age option must be explicit, voluntary, free,
and uncompelled. While an employer may unilaterally retire an employee earlier than the legally
permissible ages under the Labor Code, this prerogative must be exercised pursuant to a mutually
instituted early retirement plan. In other words, only the implementation and execution of the option
may be unilateral, but not the adoption and institution of the retirement plan containing such option.
For the option to be valid, the retirement plan containing it must be voluntarily assented to by the
employees or at least by a majority of them through a bargaining representative.
Further, a perusal of petitioner’s terms of appointment shows that membership in the retirement plan is
automatic upon acceptance of the employment. Having thus automatically become a member of the
retirement plan through his acceptance of employment as Chief Legal Officer of PVB, the petitioner could
not withdraw from the plan except upon his termination from employment.

In view of the foregoing, the Court disagrees with the view tendered by Justice Leonen to the effect
that the petitioner, because of his legal expertise and educational attainment, could not now validly
claim that he was not informed of the provisions of the retirement program. The pertinent rule on
retirement plans does not presume consent or acquiescence from the high educational attainment
or legal knowledge of the employee. In fact, the rule provides that the acquiescence by the employee
cannot be lightly inferred from his acceptance of employment.

Moreover, it was incumbent upon PVB to prove that the petitioner had been fully apprised of the terms of the
retirement program at the time of his acceptance of the offer of employment. PVB did not discharge its burden, for
the petitioner's appointment letter apparently enumerated only the minimum benefits that he would enjoy during his
employment by PVB, and contained no indication of PVB having given him a copy of the program itself in order to
fully apprise him of the contents and details thereof. Nonetheless, even assuming that he subsequently obtained
information about the program in the course of his employment, he still could not opt to simply withdraw from the
program due to his membership therein being automatic for the regular employees of PVB.

To stress, company retirement plans must not only comply with the standards set by the prevailing labor laws but
must also be accepted by the employees as commensurate to their faithful services to the employer within the
requisite period. Although the employer could be free to impose a retirement age lower than 65 years for as long its
employees consented, the retirement of the employee whose intent to retire was not clearly established, or whose
retirement was involuntary is to be treated as a discharge.

With the petitioner having been thus dismissed pursuant to the retirement provision that he had not knowingly and
voluntarily agreed to, PVB was guilty of illegal dismissal as to him.

Other Notes/ SC Pronouncements:

Angeles| Bajana | Balladares | Brillantes | Briones | Cabansag | Callanta | Chua | David|


De Leon | Gomez | Lopez | Macalino | Nostratis | Padilla | Reynon | Santos | Tan |Velasco
4E / 4F - 2018-2019
Page 871 of 920
LABOR REVIEW DIGEST
Atty. Joyrich Golangco

Take note of Justice Leonen’s dissent to the SC ruling (in bold)

Angeles| Bajana | Balladares | Brillantes | Briones | Cabansag | Callanta | Chua | David|


De Leon | Gomez | Lopez | Macalino | Nostratis | Padilla | Reynon | Santos | Tan |Velasco
4E / 4F - 2018-2019
Page 872 of 920
LABOR REVIEW DIGEST
Atty. Joyrich Golangco

11. ARMANDO M. TOLENTINO (DECEASED), HEREIN REPRESENTED BY HIS SURVIVING SPOUSE MERLA F.
TOLENTINO AND CHILDREN NAMELY: MARIENELA, ALYSSA, ALEXA, AND AZALEA, ALL SURNAMED
TOLENTINO v. PHILIPPINE AIRLINES, INC.
GR No: 218984 Date: January 24, 2018
Ponente: Carpio, J.

Doctrine: Retirement is the result of a bilateral act of the parties, a voluntary agreement between the
employer and the employe whereby the latter, after reaching a certain age, agrees to sever his or her
employment with the former. It is clear, therefore, Tolentino had not retired from PAL – it was not a result of
a voluntary agreement. Tolentino lost his employment status because of his own actions

FACTS:
Tolentino was hired by respondent Philippine Airlines, Inc. (PAL) as a flight engineer then was promoted to the rank
of A340/A330 Captain. As a pilot, Tolentino was a member of the Airline Pilots Association of the Philippines
(ALPAP), which had a collective bargaining agreement (CBA) with PAL.

ALPAP members went on strike. The Secretary of Labor issued an Order requiring all striking officers and members
of ALPAP to return to work within 24 hours from receipt of the Order and requiring PAL management to accept them
under the same terms and conditions of employment prior to the strike. While the union officers and members had
until 9 June 1998 to comply with the directive of the Secretary of Labor, some pilots – including Tolentino – continued
to participate in the strike.

When Tolentino and other striking pilots returned to work, PAL refused to readmit these returning pilots. Thus, they
filed a complaint for illegal lockout against PAL. On 20 July 1998, Tolentino reapplied for employment with PAL as a
newly hired pilot, and thus voluntarily underwent the six months probationary period. After less than a year, Tolentino
tendered his resignation. Meanwhile, the Secretary of Labor issued a Resolution declaring the strike conducted by
ALPAP illegal for being procedurally infirm and in open defiance of the return-to-work order. Members and officers of
ALPAP who participated in the strike in defiance of the return-to-work order were declared to have lost their
employment status.

Tolentino worked for a foreign airline. Upon his return, he informed PAL of his intention of collecting his separation
and/or retirement benefits under the CBA. PAL refused to pay Tolentino the separation and/or retirement benefits as
stated in the CBA. Tolentino filed his complaint against PAL for non-payment of holiday pay, rest day pay, separation
pay, and retirement benefits with prayer for the payment of damages and attorney's fees.

LA RULING:Dismissed the complaint of Tolentino.

NLRC RULING: Affirmed decision of LA.

CA RULING: Affirmed decision but ruled that under the CBA, Tolentino was entitled to the payment of his vacation
time and days off earned but not taken.

PETITION TO THE SC:

ISSUE/S:
1. Whether or not there petitioner-heirs are entitled to receive Capt. Tolentino’s retirement benefits under the CBA
and/or equity in the retirement fund under the PAL Pilot’s Retirement Benefit Plan

HELD:
Petition denied.
An employee who knowingly defies a return-to-work order issued by the Secretary of Labor is deemed to have
committed an illegal act which is a just cause to dismiss the employee under Article 282 of the Labor Code.
In fact, it has already been settled that those who participated in the 5 June 1998 strike of ALPAP are deemed to
have lost their employment status with PAL.
Thus, Tolentino, who did not deny his participation in the strike and his failure to promptly comply with the return-to-
work order of the Secretary of Labor, could not claim any retirement benefits because he did not retire – he simply
lost his employment status.

Angeles| Bajana | Balladares | Brillantes | Briones | Cabansag | Callanta | Chua | David|


De Leon | Gomez | Lopez | Macalino | Nostratis | Padilla | Reynon | Santos | Tan |Velasco
4E / 4F - 2018-2019
Page 873 of 920
LABOR REVIEW DIGEST
Atty. Joyrich Golangco

Retirement is the result of a bilateral act of the parties, a voluntary agreement between the employer and the
employe whereby the latter, after reaching a certain age, agrees to sever his or her employment with the former. It is
clear, therefore, Tolentino had not retired from PAL – it was not a result of a voluntary agreement. Tolentino lost his
employment status because of his own actions.

Admittedly, Tolentino was hired again by PAL on 20 July 1998. This was after he reapplied with the company. He
also voluntarily completed the probationary period of six months. It was made clear to Tolentino, and he certainly
admitted, that he was rehired on the condition that his employment would be as a new hire. Reemployment, on the
condition that the employee will be treated as a new employee, is a valid exercise of the employer's prerogative, as
long as it is not done with anti-union motivation. less than one year after he was rehired as a new pilot, Tolentino
resigned from PAL. In this instance, Tolentino had voluntarily resigned from work. However, the act of resignation
alone does not entitle him to retirement benefits which he claimed under the PAL-ALPAP Retirement Plan.

Tolentino is not entitled to any of the retirement benefits under the PAL-ALPAP Retirement Plan. He had not
completed even one year of his new employment with PAL. The Rules and Regulations of the PAL-ALPAP
Retirement Plan provide that the member-pilot must have completed at least five years of continuous service with
PAL to be entitled to the resignation benefit. His resignation in July 1999, which was only about a year from when he
was rehired by the company, did not qualify him for such resignation benefit.

For purposes of the retirement plan, the computation of Tolentino's length of service to the company should be
reckoned from the date he was rehired after his own voluntary application as a new pilot. His services from October
1971 to June 1998 cannot be tacked to his new employment starting in July 1998 because the first employment had
already been finally terminated – not due to his voluntary resignation or retirement, but because of termination due to
just causes. The requirements under the PAL-ALPAP Retirement Plan must be present at the time the employee
resigns or retires from PAL. Unfortunately for Tolentino, when he finally tendered his resignation with PAL, he was no
longer compliant with the requirements for the retirement benefit – as a new hire, he only completed less than one
year of service. Therefore, he is not entitled to any retirement or resignation benefits under the PAL-ALPAP
Retirement Plan.

Angeles| Bajana | Balladares | Brillantes | Briones | Cabansag | Callanta | Chua | David|


De Leon | Gomez | Lopez | Macalino | Nostratis | Padilla | Reynon | Santos | Tan |Velasco
4E / 4F - 2018-2019
Page 874 of 920
LABOR REVIEW DIGEST
Atty. Joyrich Golangco

12. DELA SALLE ARANETA UNIVERSITY V. BERNARDO


GR NO: 190809 DATE: FEBRUARY 13, 2017
PONENTE: LEONARDO-DE CASTRO, J.

Doctrine:
Republic Act No. 7641 states that "any employee may be retired upon reaching the retirement age x x x;" and
"[i]n case of retirement, the employee shall be entitled to receive such retirement benefits as he may have
earned under existing laws and any collective bargaining agreement and other agreements." The
Implementing Rules provide that Republic Act No. 7641 applies to "all employees in the private sector,
regardless of their position, designation or status and irrespective of the method by which their wages are
paid, except to those specifically exempted x x x." And Secretary Quisumbing' s Labor Advisory further
clarifies that the employees covered by Republic Act No. 7641 shall "include part-time employees,
employees of service and other job contractors and domestic helpers or persons in the personal service of
another."

The only exemptions specifically identified by Republic Act No. 7641 and its Implementing Rules are: (1)
employees of the National Government and its political subdivisions, including government-owned and/or
controlled corporations, if they are covered by the Civil Service Law and its regulations; and (2) employees
of retail, service and agricultural establishments or operations regularly employing not more than 10
employees.

FACTS:

On February 26, 2004, Bernardo filed a complaint against DLS-AU and its owner/manager, Dr. Oscar Bautista (Dr.
Bautista), for the payment of retirement benefits. Respondent Juanito Bernardo started working as a part-time
professor at petitioner Dela Salle Araneta University (DLS-AU) on June 1, 1974 for an hourly rate of P20.00. He
taught for 2 semesters and the summer for school year 1974-1975. He then took a leave of absence from June 1,
1975 – October 31, 1977 when he was assigned to work in Papua New Guinea by the Philippine Government. He
then resumed teaching at DLS-AU when he came back on 1977 until October 12, 2003, the end of first semester of
school year 2003-2004. Respondent’s contract was renewed at the start of every semester and summer. However,
on Nov. 8, 2003, he was informed by petitioner through telephone call, that his contract won’t be renewed anymore
as the school was implementing the retirement age limit for its faculty members, and respondent was already 75
years old at that time. On his retirement, respondent was being paid P246.50 per hour.

Respondent then sought advice from the DOLE whether he was entitled to retirement benefits for his 27 years of
employment. DOLE stated in two letters dated Jan. 20, 2004, and Feb. 3, 2004 that respondent is entitled to
retirement benefits under RA 7641 also known as the “New Retirement Law” and its IRR.

However, petitioner Dr. Bautista stated in a letter dated Feb. 12, 2004, that respondent is not entitled to retirement
benefits since according to DLS-AU’s policy and CBA, only full-time permanent faculty of DLS-AU for at least five
years immediately preceding the termination of their employment could avail themselves of the postemployment
benefits, and being a mere part-time employee, respondent therefore is not qualified for such retirement benefits.

This prompted respondent to file with the NLRC-NCR a complaint for non-payment of retirement benefits and
damages against DLS-AU and Dr. Bautista.

Petitioners DLS-AU and Dr. Bautista claimed that Bernardo, as a part-time employee, was not entitled to retirement
benefits. The contract between DLS-AU and Bernardo was for a fixed term, i.e., one semester. Contracts of
employment for a fixed term are not proscribed by law, provided that they had been entered into by the parties
without any force, duress, or improper pressure being brought to bear upon the employee and absent any other
circumstance vitiating consent. That DLS-AU no longer renewed Bernardo's contract did not necessarily mean that
Bernardo should be deemed retired from service.

Further, they contended that Bernardo, as a part-time employee, was not entitled to retirement benefits pursuant to
any retirement plan, CBA, or employment contract. Neither was DLS-AU mandated by law to pay Bernardo
retirement benefits. The compulsory retirement age under Article 302 [287] of the Labor Code, as amended, is 65
years old. When the employee reaches said age, his/her employment is deemed terminated. The matter of extension
of the employee's service is addressed to the sound discretion of the employer; it is a privilege only the employer can
grant. In this case, Bernardo was effectively separated from the service upon reaching the age of 65 years old. DLS-
AU merely granted Bernardo the privilege to teach by engaging his services for several more years after reaching the

Angeles| Bajana | Balladares | Brillantes | Briones | Cabansag | Callanta | Chua | David|


De Leon | Gomez | Lopez | Macalino | Nostratis | Padilla | Reynon | Santos | Tan |Velasco
4E / 4F - 2018-2019
Page 875 of 920
LABOR REVIEW DIGEST
Atty. Joyrich Golangco

compulsory retirement age. Assuming arguendo that Bernardo was entitled to retirement benefits, he should have
claimed the same upon reaching the age of 65 years old. Under Article 291 of the Labor Code, as amended, all
money claims arising from employer-employee relations shall be filed within three years from the time the cause of
action accrues.

LA RULING:
Dismissed complaint on the ground of prescription. It held that upon reaching the compulsory retirement age of sixty-
five (65), respondent was effectively separated from the service and it is at that time that his cause of action accrued.
Respondent should have sought the payment of such benefits/pay within three (3) years from such time. The LA
ruled that respondent filed the complaint 10 years after reaching the compulsory retirement age. For failure to claim
the retirement benefits/pay to which he claims to be entitled within three (3) years from the time he reached the age
of sixty-five (65), his claim should be forever barred.

NLRC RULING:
Reversed LA decision. It ruled that respondent’s cause of action only accrued on Nov. 8, 2004, when his contract
was no longer renewed and that petitioners are estopped from raising prescription as a defense, having allowed
respondent to work beyond the compulsory retirement age. As to respondent’s entitlemen to retirement benefits, it
held:

“Equally untenable is the contention that [Bernardo], being a part time employee, is not entitled to retirement benefits
under Republic Act No. 7641. Indeed, a perusal of the retirement law does not exclude a part time employee from
enjoying retirement benefits. On this score, Republic Act No. 7641 explicitly provides as within its coverage "all
employees in the private sector, regardless of their position, designation, or status, and irrespective of the method by
which their wages are paid" (Section 1, Rules Implementing the New Retirement Law) (Underlined for emphasis).
The only exceptions are employees covered by the Civil Service Law; domestic helpers and persons in the personal
service of another; and employees in retail, service and agricultural establishments or operations regularly employing
not more than ten employees (ibid). Clearly, [Bernardo] does not fall under any of the exceptions.”

CA RULING:
Affirmed NLRC ruling. It held that under Republic Act No. 7641, and Sections 1 and 2 of the Implementing Rules of
Book VI, Rule II of the Labor Code, as well as the Labor Advisory on Retirement Pay Law, part-time employees are
not among those excluded from enjoying retirement benefits.

The Court of Appeals additionally determined that Bernardo's cause of action accrued only upon his separation from
employment and the subsequent denial of his demand for retirement benefits. To the appellate court, the NLRC was
correct in applying the equitable doctrine of estoppel since the continuous extension of Bernardo's employment,
despite him being well over the statutory compulsory age of retirement, prevented him from already claiming his
retirement benefits for he was under the impression that he could avail himself of the same eventually upon the
termination of his employment.

ISSUE:
Is petitioner entitled to retirement benefits?

HELD:
Yes. The Court ruled that Republic Act No. 7641 states that "any employee may be retired upon reaching the
retirement age x x x;" and "[i]n case of retirement, the employee shall be entitled to receive such retirement benefits
as he may have earned under existing laws and any collective bargaining agreement and other agreements." The
Implementing Rules provide that Republic Act No. 7641 applies to "all employees in the private sector, regardless of
their position, designation or status and irrespective of the method by which their wages are paid, except to those
specifically exempted x x x." And Secretary Quisumbing' s Labor Advisory further clarifies that the employees
covered by Republic Act No. 7641 shall "include part-time employees, employees of service and other job
contractors and domestic helpers or persons in the personal service of another."

The only exemptions specifically identified by Republic Act No. 7641 and its Implementing Rules are: (1) employees
of the National Government and its political subdivisions, including government-owned and/or controlled
corporations, if they are covered by the Civil Service Law and its regulations; and (2) employees of retail, service and
agricultural establishments or operations regularly employing not more than 10 employees.

Angeles| Bajana | Balladares | Brillantes | Briones | Cabansag | Callanta | Chua | David|


De Leon | Gomez | Lopez | Macalino | Nostratis | Padilla | Reynon | Santos | Tan |Velasco
4E / 4F - 2018-2019
Page 876 of 920
LABOR REVIEW DIGEST
Atty. Joyrich Golangco

Based on Republic Act No. 7641, its Implementing Rules, and Secretary Quisumbing's Labor Advisory, Bernardo, as
a part-time employee of DLS-AU, is entitled to retirement benefits. The general coverage of Republic Act No. 7641 is
broad enough to encompass all private sector employees, and part-time employees are not among those specifically
exempted from the law. The provisions of Republic Act No. 7641 and its Implementing Rules are plain, direct,
unambiguous, and need no further elucidation. Any doubt is dispelled by the unequivocal statement in Secretary
Quisumbing's Labor Advisory that Republic Act No. 7641 applies to even part-time employees.

Other Notes/ SC Pronouncements:


SC also ruled that petitioners are estopped from raising the defense of prescription, due to the fact that DLS-AU had
repeatedly extended Bernardo's employment even when he already reached 65 years old. DLS-AU still knowingly
offered Bernardo, and Bernardo willingly accepted, contracts of employment to teach for semesters and summers in
the succeeding 10 years. Since DLS-AU was still continuously engaging his services even beyond his retirement
age, Bernardo deemed himself still employed and deferred his claim for retirement benefits, under the impression
that he could avail himself of the same upon the actual termination of his employment. It also held that respondent’s
cause of action accrued after the refusal of DLS-AU to pay him the same, clearly expressed in Dr. Bautista's letter
dated February 12, 2004. Hence, Bernardo's complaint, filed with the NLRC on February 26, 2004, was filed within
the three-year prescriptive period provided under Article 291 of the Labor Code.

Angeles| Bajana | Balladares | Brillantes | Briones | Cabansag | Callanta | Chua | David|


De Leon | Gomez | Lopez | Macalino | Nostratis | Padilla | Reynon | Santos | Tan |Velasco
4E / 4F - 2018-2019
Page 877 of 920
LABOR REVIEW DIGEST
Atty. Joyrich Golangco

13. UDMC v. Bernadas


GR No: 209468 Date: December 13, 2017
Ponente: Leonen, J.

Doctrine:
An employee who has already qualified for optional retirement but dies before the option to retire could be
exercised is entitled to his or her optional retirement benefits, which may be claimed by the qualified
employee's beneficiaries on his or her behalf.

FACTS:
Respondent Cesario Bernadas started working as an orderly under petitioner United Doctor Medical Center’s
(UDMC) housekeeping department on July 17, 1986 and was later on promoted as utility man. UDMC and its rank-
and-file employees had at that time a Collective Bargaining Agreement providing for optional retirement benefits. It
provides:

ARTICLE XI RETIREMENT AND SEVERANCE PAY


SECTION 1. RETIREMENT AND SEVERANCE PAY.

The CENTER shall grant each employee retirement and severance pay in
accordance with law. It shall also continue its present policy on optional
retirement.

Under the optional retirement policy, employees who rendered at least 20 years of service is entitled to optionally
retire, with the retirement pay equivalent to a retiree’s salary for 11 days for every year of service. Insurance
(premium paid by UDMC) is also provided to the employees with their family members as beneficiaries

On Oct. 20, 2009, Cesario died due to a “freak accident” while working in a doctor’s residence. He was 53 years old.

Leonila Bernadas, representing her husband, respondent Cesario, filed a claim for payment of retirement benefits
with the NLRC. However, petitioner contended that respondent’s beneficiaries do not have legal capacity to apply for
the optional retirement benefits since respondent himself didn’t apply for such during his lifetime. Further, it claims
that even assuming that respondent was already qualified to apply for optional retirement 3 years prior to his death,
he never did.

On the other hand, Leonila counters that had her husband died "under normal circumstances," he would have
applied for optional retirement benefits. That Cesario was unable to apply before his death "is a procedural
technicality" that should be set aside so that "full protection to labor" is afforded and "the ends of social and
compassionate justice" are met.

LA RULING:
Dismissed the complaint. It ruled that Cesario should have applied for optional retirement benefits during his lifetime,
the benefits being optional. Since he did not apply for it, his beneficiaries were not entitled to claim his optional
retirement benefits.

NLRC RULING:
Reversed LA ruling. It found that the optional retirement plan was never presented in this case, casting a doubt on
whether or not the plan required an application for optional retirement benefits before an employee could become
entitled to them. Considering the "constitutional mandate to afford full protection to labor," the National Labor
Relations Commission resolved the doubt in favor of Cesario.

CA RULING:
Affirmed NLRC ruling.

ISSUE:
Is respondent Cesairo entitled to optional retirement benefits?

HELD:
Yes. To begin, the Supreme Court discussed the 3 types of retirement existing in the country. It stated:

Angeles| Bajana | Balladares | Brillantes | Briones | Cabansag | Callanta | Chua | David|


De Leon | Gomez | Lopez | Macalino | Nostratis | Padilla | Reynon | Santos | Tan |Velasco
4E / 4F - 2018-2019
Page 878 of 920
LABOR REVIEW DIGEST
Atty. Joyrich Golangco

Within this jurisdiction, there are three (3) types of retirement plans available to
employees. The first is compulsory and contributory. This type of plan is
embodied in Republic Act No. 828240 for those in the private sector and
Republic Act No. 829141 for those in the government. These laws require a
mandatory contribution from the employer as well as the employee, which shall
become a pension fund for the employee upon retirement. Considering that the
mandatory employee contribution is deducted from the employee's monthly
income, "retirement packages are usually crafted as 'forced savings' on the part
of the employee."

Under this type of retirement plan, the pension is not considered as mere gratuity
but actually forms part of the employee's compensation. An employee acquires a
vested right to the benefits that have become due upon reaching the compulsory
age of retirement. Thus, the beneficiaries of the retired employee are entitled to
the pension even after the retired employee's death. The second and third types
of retirement plans are voluntary. They may not even require the employee to
contribute to a pension fund. The second type of retirement plan is by agreement
between the employer and the employee, usually embodied in the CBA between
them. "The third type is one that is voluntarily given by the employer, expressly
as in an announced company policy or impliedly as in a failure to contest the
employee's claim for retirement benefits." The rules regarding the second and
third types of retirement plans are provided for in Article 302 [287]49 of the Labor
Code, as amended.

It held that these types of retirement plans are not meant to be a replacement to the compulsory retirement scheme
under social security laws but must be understood as a retirement plan in addition to that provided by law and that
Art. 302 allows the employer and employee to enter into early retirement packages or schemes. Citing Pantranco
North Express v. NLRC, it provided the rationale for early retirement packages as:

In almost all countries today, early retirement, i.e., before age 60, is considered a
reward for services rendered since it enables an employee to reap the fruits of
his labor - particularly retirement benefits, whether lump-sum or otherwise - at an
earlier age, when said employee, in presumably better physical and mental
condition, can enjoy them better and longer. As a matter of fact, one of the
advantages of early retirement is that the corresponding retirement benefits,
usually consisting of a substantial cash windfall, can early on be put to productive
and profitable uses by way of income-generating investments, thereby affording
a more significant measure of financial security and independence for the retiree
who, up till then, had to contend with life's vicissitudes within the parameters of
his fortnightly or weekly wages. Thus we are now seeing many CBAs with such
early retirement provisions.

It further discussed that acceptance by the employees of an early retirement age option must be explicit, voluntary,
free, and uncompelled. While an employer may unilaterally retire an employee earlier than the legally permissible
ages under the Labor Code, this prerogative must be exercised pursuant to a mutually instituted early retirement
plan. In other words, only the implementation and execution of the option may be unilateral, but not the adoption and
institution of the retirement plan containing such option. For the option to be valid, the retirement plan containing it
must be voluntarily assented to by the employees or .at least by a majority of them through a bargaining
representative.

The Supreme Court then moved on to discuss the entitlement of respondent Cesario to the optional retirement
benefits, examining first the provision in the UDMC CBA providing for such (reproduced in the “FACTS”) and stating
that the CBA does not provide for the terms and conditions of the "present policy on optional retirement. It observed
that Leonila merely alleged before the Labor Arbiter that petitioner "grants an employee a retirement or separation
equivalent to eleven (11) days per year of service after serving for at least twenty (20) years," which was not disputed
by petitioner. Therefore, doubt arises as to what petitioner's optional retirement package actually entails.

The Court then reiterated that any doubt as to the law must be resolved in favor of labor and that retirement laws
should be liberally construed and administered in favor of the persons intended to be benefited and all doubts as to

Angeles| Bajana | Balladares | Brillantes | Briones | Cabansag | Callanta | Chua | David|


De Leon | Gomez | Lopez | Macalino | Nostratis | Padilla | Reynon | Santos | Tan |Velasco
4E / 4F - 2018-2019
Page 879 of 920
LABOR REVIEW DIGEST
Atty. Joyrich Golangco

the intent of the law should be resolved in favor of the retiree to achieve its humanitarian purposes. Retirement
encompasses even the concept of death and the Court has considered death as a form of disability retirement as
"there is no more permanent or total physical disability than death. Compulsory retirement and death both involve
events beyond the employee's control. While the choice to retire before the compulsory age of retirement was within
respondent Cesario's control, his death foreclosed the possibility of him making that choice.
Petitioner's optional retirement plan is premised on length of service, not upon reaching a certain age. It rewards
loyalty and continued service by granting an employee an earlier age to claim his or her retirement benefits even if
the employee has not reached his or her twilight years. It would be the height of inequity to withhold respondent
Cesario' s retirement benefits despite being qualified to receive it, simply because he died before he could apply for
it. In any case, the CBA does not mandate that an application must first be filed by the employee before the right to
the optional retirement benefits may vest. Thus, this ambiguity should be resolved in favor of the retiree. Retirement
benefits are the property interests of the retiree and his or her beneficiaries. The CBA does not prohibit the
employee's beneficiaries from claiming retirement benefits if the retiree dies before the proceeds could be released.
Even compulsory retirement plans provide mechanisms for a retiree's beneficiaries to claim any pension due to the
retiree Thus, Leonila, being the surviving spouse of respondent Cesario is entitled to claim the optional retirement
benefits on his behalf.

Other Notes/ SC Pronouncements:


Supreme Court decision was reproduced substantially due to the significance of the topic (early retirement / optional
retirement schemes)

Angeles| Bajana | Balladares | Brillantes | Briones | Cabansag | Callanta | Chua | David|


De Leon | Gomez | Lopez | Macalino | Nostratis | Padilla | Reynon | Santos | Tan |Velasco
4E / 4F - 2018-2019
Page 880 of 920
LABOR REVIEW DIGEST
Atty. Joyrich Golangco

14. PAL V. HASSARAM


GR NO: 217730 DATE: JUNE 5, 2017
PONENTE: SERENO, CJ.

Doctrine:
Art. 287 (Now Art. 302) is applicable only to a situation where (l) there is no CBA or other applicable
employment contract providing for retirement benefits for an employee, or (2) there is a CBA or other
applicable employment contract providing for retirement benefits for an employee, but it is below the
requirement set by law. The rationale for the first situation is to prevent the absurd situation where an
employee, deserving to receive retirement benefits, is denied them through the nefarious scheme of
employers to deprive employees of the benefits due them under existing labor laws. On the other hand, the
second situation aims to prevent private contracts from derogating from the public law.
...Emphasis must be placed on the fact that the purpose of the amendment is not merely to establish
precedence in application or accord blanket priority to existing CBAs in computing retirement benefits. The
determining factor in choosing which retirement scheme to apply is still superiority in terms of benefits
provided. Thus, even if there is an existing CBA but the same does not provide for retirement benefits equal
or superior to that which is provided under Article 287 of the Labor Code, the latter will apply. In this manner,
the employee can be assured of a reasonable amount of retirement pay for his sustenance.

FACTS:
Respondent Arjan T. Hassaram, a former PAL pilot filed a complaint against PAL for illegal dismissal and payment of
retirement benefits. He averred that he had applied for retirement in August 2000 after 24 years of service as pilot,
but was denied. Instead, PAL informed him that he had lost his employment in the company as of 9 June 1998, in
view of his failure to comply with the Return to Work Order issued by the Secretary of Labor against members of the
Airline Pilots Association of the Philippines (ALPAP) on 7 June 1998. Before the Labor Arbiter
(LA), https://www.lawphil.net/judjuris/juri2017/jun2017/gr_217730_2017.html - fnt6Hassaram argued that he was not
covered by the Secretary's Return to Work Order; hence, PAL had no valid ground for his dismissal. He asserted that
on 9 June 1998, he was already on his way to Taipei to report for work at Eva Air, pursuant to a four-year contract
approved by PAL itself. Petitioner further claimed that his arrangement with PAL allowed him to go on leave without
pay while working for Eva Air, with the right to accrue seniority and retire from PAL during the period of his leave.

PAL, on the other hand contended that (a) the LA had no jurisdiction over the case, which was a mere off-shoot of
ALPAP's strike, a matter over which the Secretary of Labor had already assumed jurisdiction; (b) the Complaint
should be considered barred by res judicata, forum shopping, and prescription; (c) the case should be suspended
while PAL was under receivership; and (d) if at all, Hassaram was entitled only to retirement benefits of ₱5,000
for every year of service pursuant to the Collective Bargaining Agreement (CBA) between PAL and ALPAP.

LA RULING:
Ruled in favor of respondent, awarding retirement benefits and attorney’s fees. It ruled Hassaram did not defy the
Return to Work Order, as he was in fact already on leave when the order was implemented. As to the computation
of benefits, the LA ruled that Article 287 of the Labor Code should be applied, since the statute provided
better benefits than the PAL-ALPAP CBA.

NLRC RULING:
The NLRC initially affirmed the LA's Decision to award retirement benefits to respondent Hassaram under Article 287
of the Labor Code. This affirmation prompted PAL to seek reconsideration of the ruling citing, for the first time,
respondent's purported receipt of retirement benefits in the amount of ₱4,456,817.75 pursuant to the PAL
Pilots’ Retirement Benefit Plan. PAL likewise alleged that, as a consequence of this newly discovered payment,
any claim made by Hassaram for retirement benefits should be deemed extinguished.
The NLRC granted PAL's Motion for Reconsideration and reversed the LA ruling. It held that the ₱4,456,817.75 paid
to respondent as retirement benefits under the plan was sufficient to discharge his claim for retirement pay.
CA RULING:
Before the CA respondent Hassaram admitted receipt of ₱4,456,817.75 from the PAL Pilots’ Retirement Benefit
Plan but claims that his receipt of that sum did not preclude him from claiming retirement benefits from PAL, since
that amount represented only a return of his share in a distinct and separate provident fund established for PAL
pilots.

The CA ruled that the funds received under the PAL Pilots’ Retirement Benefit Plan were not the retirement
benefits contemplated by law. Hence, respondent was still entitled to receive retirement benefits in the amount of
₱2, 111,984.60 pursuant to Article 287 of the Labor Code.

Angeles| Bajana | Balladares | Brillantes | Briones | Cabansag | Callanta | Chua | David|


De Leon | Gomez | Lopez | Macalino | Nostratis | Padilla | Reynon | Santos | Tan |Velasco
4E / 4F - 2018-2019
Page 881 of 920
LABOR REVIEW DIGEST
Atty. Joyrich Golangco

ISSUES:
3. Should the amount received by respondent under the PAL Pilots’ Retirement Benefit Plan be deemed
part of his retirement pay?
4. Is respondent entitled to receive retirement benefits under Art. 287 (Now Aart, 302) of the Labor Code?

HELD:
3. YES. The Court that under the provisions of the plan that it is the petitioner that contributes to a "retirement
fund" for the account of the pilots. These contributions comprise the benefits received by the latter upon
retirement, separation from service, or disability. It then cited Philippine Airlines, Inc. v. Airline Pilots
Association of the Phils. explains provisions under the plan:

“The PAL Pilots' Retirement Benefit Plan is a retirement fund raised from contributions
exclusively from PAL of amounts equivalent to 20% of each pilot's gross monthly pay.
Upon retirement, each pilot stands to receive the full amount of the contribution. In sum,
therefore, the pilot gets an amount equivalent to 240% of his gross monthly income for
every year of service he rendered to petitioner. This is in addition to the amount of not
less than ₱100,000.00 that he shall receive under the 1967 Retirement Plan.”

Based on the foregoing characterization, the Court included the amount received from the Plan in the
computation of the retirement pay of the pilot involved in that case. Considering that the very same
retirement plan is involved in the case the Court ruled that the amount of ₱4,456,8l7.75 received by
Hassaram from the PAL Plan formed part of his retirement pay.

4. NO. The Court held that the benefits due to Hassaram under the PAL retirement plan are greater than that
when the provisions of Art. 287 (Now Art. 302) of the Labor Code are applied. Citing Elegir v. PAL, it stated:

It can be clearly inferred from the language of the foregoing provision that it is applicable
only to a situation where (l) there is no CBA or other applicable employment contract
providing for retirement benefits for an employee, or (2) there is a CBA or other applicable
employment contract providing for retirement benefits for an employee, but it is below the
requirement set by law. The rationale for the first situation is to prevent the absurd
situation where an employee, deserving to receive retirement benefits, is denied them
through the nefarious scheme of employers to deprive employees of the benefits due
them under existing labor laws. On the other hand, the second situation aims to prevent
private contracts from derogating from the public law.

Emphasis must be placed on the fact that the purpose of the amendment is not merely to
establish precedence in application or accord blanket priority to existing CBAs in
computing retirement benefits. The determining factor in choosing which retirement
scheme to apply is still superiority in terms of benefits provided. Thus, even if there is an
existing CBA but the same does not provide for retirement benefits equal or superior to
that which is provided under Article 287 of the Labor Code, the latter will apply. In this
manner, the employee can be assured of a reasonable amount of retirement pay for his
sustenance.

All in all, the retirement benefits under Art. 287 is applicable only to a situation where (l) there is no CBA or other
applicable employment contract providing for retirement benefits for an employee, or (2) there is a CBA or other
applicable employment contract providing for retirement benefits for an employee, but it is below the requirement set
by law. Having established that Hassaram has already received benefits under PAL retirement’s program, with said
benefits being greater than those afforded under Art. 287, he may no longer claim the benefits under the said
provision of the Labor Code.

Other Notes/ SC Pronouncements:


SC computed the benefits due to Hassaram under Art. 287 and under the PAL retirement plan. Computation
was not included since it has little jurisprudential relevance.

Angeles| Bajana | Balladares | Brillantes | Briones | Cabansag | Callanta | Chua | David|


De Leon | Gomez | Lopez | Macalino | Nostratis | Padilla | Reynon | Santos | Tan |Velasco
4E / 4F - 2018-2019
Page 882 of 920
LABOR REVIEW DIGEST
Atty. Joyrich Golangco

ART. 306- MONEY CLAIMS

1. PLDT V. PINGOL
GR NO: 182622 DATE: SEPTEMBER 8, 2010
PONENTE: MENDOZA, J.

Doctrine:
The Labor Code has no specific provision on when a claim for illegal dismissal or a monetary claim accrues.
Thus, the general law on prescription applies. Article 1150 of the Civil Code states:

Article 1150. The time for prescription for all kinds of actions, when there is no special
provision which ordains otherwise, shall be counted from the day they may be brought.

xxxxx

Like other causes of action, the prescriptive period for money claims is subject to interruption, and in the
absence of an equivalent Labor Code provision for determining whether the said period may be
interrupted, Article 1155 of the Civil Code may be applied, to wit:

ART. 1155. The prescription of actions is interrupted when they are filed before the Court,
when there is a written extrajudicial demand by the creditors, and when there is any written
acknowledgment of the debt by the debtor.
Thus, the prescription of an action is interrupted by (a) the filing of an action, (b) a written
extrajudicial demand by the creditor, and (c) a written acknowledgment of the debt by the
debtor.

FACTS:
Respondent Roberto Pingol was hired by petitioner PLDT as a maintenance technician in 1979. On April 13, 1999
respondent was admitted at The Medical City, Mandaluyong City due to paranoid personality disorder caused by
financial and marital problems. He as later on discharged but frequently absented himself due to his poor mental
condition.

From September 16, 1999 to December 31, 1999, Pingol was absent from work without official leave. PLDT claimed
that notices were given to respondent warning him that he would be dismissed from employment if he continued to
be absent without official leave "pursuant to PLDT Systems Practice A-007 which provides that ‘Absence without
authorized leaves for seven (7) consecutive days is subject to termination from the service.’" On January 1, 2000,
PLDT terminated his services on the grounds of unauthorized absences and abandonment of office.

On March 29, 2004, four years later, Pingol filed a Complaint for Constructive Dismissal and Monetary Claims
against PLDT. Petitioner PLDT moved for the dismissal of the complaint arguing that it was filed beyond the
prescriptive period, having been filed 4 years and 3 months after respondent’s dismissal.

Respondent, on the other hand, countered that in computing the prescriptive period, the years 2001 to 2003 must not
be taken into account since during that time he was inquiring from PLDT about the benefits due him as an employee
who was no longer allowed to do his work, with such inquiry to be considered as non-abandonment of his claim.

LA RULING:
Dismissed complaint, citing Article 291 of the Labor Code which provides:

“All money claims arising from employer-employee relations accruing from the effectivity
of this Code shall be filed within three (3) years from the time the cause of action accrued,
otherwise they shall be forever barred.”

NLRC RULING:
Reversed LA.

CA RULING:
Affirmed NLRC’s ruling.

Angeles| Bajana | Balladares | Brillantes | Briones | Cabansag | Callanta | Chua | David|


De Leon | Gomez | Lopez | Macalino | Nostratis | Padilla | Reynon | Santos | Tan |Velasco
4E / 4F - 2018-2019
Page 883 of 920
LABOR REVIEW DIGEST
Atty. Joyrich Golangco

ISSUE:
Has respondent’s money claim prescribed?

HELD:
Yes. The pivotal question in resolving the issues is the date when the cause of action of respondent Pingol accrued.
It is a settled jurisprudence that a cause of action has three (3) elements, to wit: (1) a right in favor of the plaintiff by
whatever means and under whatever law it arises or is created; (2) an obligation on the part of the named defendant
to respect or not to violate such right; and (3) an act or omission on the part of such defendant violative of the right of
the plaintiff or constituting a breach of the obligation of the defendant to the plaintiff.

The Labor Code has no specific provision on when a claim for illegal dismissal or a monetary claim accrues. Thus,
the general law on prescription applies. Article 1150 of the Civil Code states:
Article 1150. The time for prescription for all kinds of actions, when there is no
special provision which ordains otherwise, shall be counted from the day they
may be brought.

The day the action may be brought is the day a claim starts as a legal possibility. In the present case, January 1,
2000 was the date that respondent Pingol was not allowed to perform his usual and regular job as a maintenance
technician. Respondent Pingol cited the same date of dismissal in his complaint before the LA. As, thus, correctly
ruled by the LA, the complaint filed had already prescribed.

Respondent claims that between 2001 and 2003, he made follow-ups with PLDT management regarding his benefits.
This, to his mind, tolled the running of the prescriptive period.

The rule in this regard is covered by Article 1155 of the Civil Code. Its applicability in labor cases was upheld in the
case of International Broadcasting Corporation v. Panganiban where it was written:

Like other causes of action, the prescriptive period for money claims is subject to interruption, and
in the absence of an equivalent Labor Code provision for determining whether the said period may
be interrupted, Article 1155 of the Civil Code may be applied, to wit:
ART. 1155. The prescription of actions is interrupted when they are filed before the Court, when
there is a written extrajudicial demand by the creditors, and when there is any written
acknowledgment of the debt by the debtor.

Thus, the prescription of an action is interrupted by (a) the filing of an action, (b) a written extrajudicial demand by the
creditor, and (c) a written acknowledgment of the debt by the debtor.

In this case, respondent Pingol never made any written extrajudicial demand. Neither did petitioner make any written
acknowledgment of its alleged obligation. Thus, the claimed "follow-ups" could not have validly tolled the running of
the prescriptive period. It is worthy to note that respondent never presented any proof to substantiate his allegation of
follow-ups.

Other Notes/ SC Pronouncements:


The SC also held that respondent himself admitted in his pleadings that he was dismissed on January 1, 2000, thus
further bolstering the fact that his cause of action accrued on that date.

Angeles| Bajana | Balladares | Brillantes | Briones | Cabansag | Callanta | Chua | David|


De Leon | Gomez | Lopez | Macalino | Nostratis | Padilla | Reynon | Santos | Tan |Velasco
4E / 4F - 2018-2019
Page 884 of 920
LABOR REVIEW DIGEST
Atty. Joyrich Golangco

2. ROBERTO R. SERRANO, petitioner, vs. COURT OF APPEALS, NATIONAL LABOR RELATIONS


COMMISSION, MAERSK-FILIPINAS CREWING, INC. and A.P. MOLLER, respondents.
GR No: 139420 Date: Aug. 15, 2001
Ponente: Puno, J.

Doctrine: Cause of actions on money claims accrues only upon definite denial of the same by the employer.

FACTS:
From 1974 to 1991, respondent Maersk-Filipinas Crewing, Inc., the local agent of respondent foreign
corporation A.P. Moller, deployed petitioner Serrano as a seaman to Liberian, British and Danish ships. As petitioner
was on board a ship most of the time, respondent offered to send portions of petitioners salary to his family in the
Philippines by money order. Petitioner agreed and from 1977 to 1978, he instructed respondent Maersk to send
money orders to his family. Respondent Maersk deducted the amounts of these money orders totaling HK$4,600.00
and 1,050.00 Sterling Pounds from petitioner's salary. It is also alleged, that respondent deducted various amounts
from his salary for Danish Social Security System (SSS), welfare contributions, ship club, and SSS Medicare.
It appears that petitioner's family failed to receive the money orders petitioner sent through respondent
Maersk. Upon learning this in 1978, petitioner demanded that respondent Maersk pay him the amounts the latter
deducted from his salary.
Whenever he returned to the Philippines, petitioner would go to the office of respondent Maersk to follow up
his money claims but he would be told to return after several weeks as respondent Maersk needed time to verify its
records and to bring up the matter with its principal employer, respondent A.P. Moller. Meantime, respondent Maersk
would hire him again to board another one of their vessels for about a year.
Finally, in October 1993, petitioner wrote to respondent Maersk demanding immediate payment to him of
the total amount of the money orders deducted from his salary from 1977 to 1978. On November 11, 1993,
respondent A.P. Moller replied to petitioner that they keep accounting documents only for a certain number of years,
thus data on his money claims from 1977 to 1978 were no longer available. A.P. Moller declined petitioner's demand
for payment.
Petitioner then filed a complaint for collection of the total amount of the unsent money against respondent
with the POEA. The case was transferred to the NLRC.

LA/RTC/NLRC RULING:
LA: Ruled in favor of petitioner. Ordering respondent to refund the un-transmitted money order.
NLRC: Reversed the LA decision. On the ground that the money claim has prescribed. MR was denied for lack of
merit.
CA RULING: Dismissed the petition for review on certiorari for having been filed out of time.

APPEAL TO THE SC:


Petitioner's Contention: Petitioner contends that his cause of action accrued only in 1993 when respondent A.P.
Moller wrote to him that its accounting records showed it had no outstanding money orders and that his case was
considered outdated. Thus, the three (3) year prescriptive period should be counted from 1993 and not 1978 and
since his complaint was filed in 1994, he claims that it has not prescribed.

ISSUE/S:
When did petitioner's cause of action accrue for this will determine the reckoning date of the three-year
prescriptive period for money claims.

HELD:
Petitioner’s cause of action accrued on 1993.
It is settled jurisprudence that a cause of action has three elements, to wit, (1) a right in favor of the plaintiff
by whatever means and under whatever law it arises or is created; (2) an obligation on the part of the named
defendant to respect or not to violate such right; and (3) an act or omission on the part of such defendant violative
of the right of the plaintiff or constituting a breach of the obligation of the defendant to the plaintiff.
The problem in the case at bar is with the third element as the first two are deemed established.
The SC quoted the Baliwag Transit vs Ople case wherein respondet-driver was suspended after being
involved in a vehicular accident and thereafter repeatedly demanded reinstatement after the PNR decision was
rendered but to no avail. The court there held that the cause of action accrued only when his demand for
reinstatement was denied by petititioner.
The facts in the case at bar are similar to the Baliwag case. Petitioner repeatedly demanded payment from
respondent Maersk but similar to the actuations of Baliwag Transit, respondent Maersk warded off these
demands by saying that it would look into the matter until years passed by. In October 1993, Serrano finally

Angeles| Bajana | Balladares | Brillantes | Briones | Cabansag | Callanta | Chua | David|


De Leon | Gomez | Lopez | Macalino | Nostratis | Padilla | Reynon | Santos | Tan |Velasco
4E / 4F - 2018-2019
Page 885 of 920
LABOR REVIEW DIGEST
Atty. Joyrich Golangco

demanded in writing payment of the unsent money orders. Then and only then was the claim
categorically denied by respondent A.P. Moller in its letter dated November 22, 1993. Following the
Baliwag Transit ruling, petitioners cause of action accrued only upon respondent A.P. Moller's definite
denial of his claim in November 1993. Having filed his action five (5) months thereafter or in April 1994, we hold
that it was filed within the three-year (3) prescriptive period provided in Article 291 of the Labor Code.

Other Notes/ SC Pronouncements:


As to the issue of timeliness of the petition for review on certiorari with the CA:
In the case at bar, petitioner Serrano received the resolution of the NLRC denying his motion for
reconsideration on April 6, 1999. Thenceforth, he had 60 days or until June 7, 1999 to file a petition for certiorari with
the Court of Appeals. But as June 7 fell on a Saturday, he had until June 9, the next working day, to file his petition.
Rule 22, Section 1 provides in relevant part, viz:
. . . If the last day of the period, as thus computed, falls on a Saturday, a Sunday or a legal holiday in the
place where the court sits, the time shall not run until the next working day.

Petitioner thus timely filed his petition with the Court of Appeals on June 9, 1999.

Angeles| Bajana | Balladares | Brillantes | Briones | Cabansag | Callanta | Chua | David|


De Leon | Gomez | Lopez | Macalino | Nostratis | Padilla | Reynon | Santos | Tan |Velasco
4E / 4F - 2018-2019
Page 886 of 920
LABOR REVIEW DIGEST
Atty. Joyrich Golangco

3. INTERCONTINENTAL BROADCASTING CORPORATION V. IRENEO PANGANIBAN


GR NO: 151407 DATE: FEBRUARY 6, 2007
PONENTE: J. AUSTRIA-MARTINEZ

Doctrine:
Although the commencement of a civil action stops the running of the statute of prescription or limitations,
its dismissal or voluntary abandonment by plaintiff leaves the parties in exactly the same position as though
no action had been commenced at all.

FACTS:
Respondent Ireneo Panganiban was employed as Assistant General Manager of the petitioner Intercontinental
Broadcasting Corporation until his preventive suspension on August 26, 1988. Respondent resigned from his
employment on September 2, 1988. On April 12, 1989, respondent filed with the RTC a civil case against the
members of the Board of Administrators (BOA) of petitioner alleging, among others, non-payment of his unpaid
commissions.

A motion to dismiss was filed by Joselito Santiago, one of the defendants, on the ground of lack of jurisdiction, as
respondent's claim was a labor money claim, but this was denied by the RTC. Thus, Santiago filed a petition for
certiorari with the CA, which the latter granted. Thereafter, respondent was elected by the BOA as Vice-President for
Marketing in July 1992. He resigned in April 1993. On July 24, 1996, respondent filed against petitioner a complaint
for illegal dismissal, separation pay, retirement benefits, unpaid commissions, and damages.

LA/NLRC RULING:
The LA ordered respondent’s reinstatement with full backwages and payment of unpaid commission.

The NLRC dismissed the appeal for petitioner’s failure to post a bond.

CA RULING:
The CA reversed and set aside the decisions of the LA and NLRC but, upon a motion for reconsideration, ordered
petitioner to pay respondent the amount of unpaid commissions.

APPEAL TO THE SC:


Petition for Review on Certiorari under Rule 45

Petitioner's Contention:
That respondent’s claim for alleged unpaid commission has already prescribed because he has not filed his claim
within the three year prescriptive period and before a competent labor arbiter who ought to hear his alleged money
claims within the proper time.

ISSUE:
Whether or not respondent’s claim for unpaid commissions has already prescribed.

HELD:
YES. The applicable law in this case is Article 291 of the Labor Code which provides that "all money claims arising
from employer-employee relations accruing during the effectivity of this Code shall be filed within three (3) years from
the time the cause of action accrued; otherwise they shall be forever barred." The term "money claims" covers all
money claims arising from an employer-employee relation

Like other causes of action, the prescriptive period for money claims is subject to interruption, and in the absence of
an equivalent Labor Code provision for determining whether the said period may be interrupted, Article 1155 of the
Civil Code may be applied, to wit:

ART. 1155. The prescription of actions is interrupted when they are filed before the Court, when there is a
written extrajudicial demand by the creditors, and when there is any written acknowledgment of the debt by
the debtor.

Thus, the prescription of an action is interrupted by (a) the filing of an action, (b) a written extrajudicial demand by the
creditor, and (c) a written acknowledgment of the debt by the debtor. On this point, the Court ruled that although the
commencement of a civil action stops the running of the statute of prescription or limitations, its dismissal or voluntary

Angeles| Bajana | Balladares | Brillantes | Briones | Cabansag | Callanta | Chua | David|


De Leon | Gomez | Lopez | Macalino | Nostratis | Padilla | Reynon | Santos | Tan |Velasco
4E / 4F - 2018-2019
Page 887 of 920
LABOR REVIEW DIGEST
Atty. Joyrich Golangco

abandonment by plaintiff leaves the parties in exactly the same position as though no action had been commenced at
all.

Hence, while the filing of the Civil Case could have interrupted the running of the three-year prescriptive period, its
consequent dismissal by the CA due to lack of jurisdiction effectively canceled the tolling of the prescriptive period
within which to file his money claim, leaving respondent in exactly the same position as though no civil case had
been filed at all. The running of the three-year prescriptive period not having been interrupted by the filing of the Civil
Case, respondent's cause of action had already prescribed on September 2, 1991, three years after his
cessation of employment on September 2, 1988. Consequently, when respondent filed his complaint for illegal
dismissal, separation pay, retirement benefits, and damages in July 24, 1996, his claim, clearly, had already been
barred by prescription.

Angeles| Bajana | Balladares | Brillantes | Briones | Cabansag | Callanta | Chua | David|


De Leon | Gomez | Lopez | Macalino | Nostratis | Padilla | Reynon | Santos | Tan |Velasco
4E / 4F - 2018-2019
Page 888 of 920
LABOR REVIEW DIGEST
Atty. Joyrich Golangco

4. ACCESSORIES SPECIALIST V. ALBANZA


GR NO: 168985 DATE: JULY 23, 2008
PONENTE: NACHURA, J.

Art. 306 (Formerly Art. 291) – Money Claims


Doctrine:
In light of these circumstances, we can apply the principle of promissory estoppel, which is a recognized
exception to the three-year prescriptive period enunciated in Article 291 of the Labor Code.

Promissory estoppel may arise from the making of a promise, even though without consideration, if it was
intended that the promise should be relied upon, as in fact it was relied upon, and if a refusal to enforce it
would virtually sanction the perpetration of fraud or would result in other injustice. Promissory estoppel
presupposes the existence of a promise on the part of one against whom estoppel is claimed. The promise
must be plain and unambiguous and sufficiently specific so that the court can understand the obligation
assumed and enforce the promise according to its terms.

FACTS:
Erlinda Albanza in behalf on the deceased respondent Jones Albanza filed a complaint against petitioners
Accessories Specialists, Inc. (ASI, for brevity) also known as ARTS 21 Corporation, and Tadahiko Hashimoto for
non-payment of salaries, separation pay, and 13th month pay. Jones was the Vice-President, Manager and Director
of ASI. He was forced to resign by petitioner Tadahiko Hashimoto, owner of ASI, on October 17, 1977, because the
company suffered losses due to lack of market and incurred several debts caused by a slam in the market. At the
time of his resignation, Jones had unpaid salaries for eighteen (18) months and was not paid his separation pay
commensurate to his 21 years of service. Jones demanded payment of his money claims upon resignation but ASI
informed him that it would just settle first the money claims of the rank- and-file employees, and his claims will be
paid thereafter. Knowing the predicament of the company, Jones patiently waited for his turn to be paid. Several
demands were made by Jones but ASI just kept on assuring him that he will be paid his monetary claims. Jones died
on August 5, 2002 and failed to receive the same.

Petitioners, on the other hand, contended that under Article 291 of the Labor Code, all money claims arising from an
employer-employee relationship shall be filed within three (3) years from the time the cause of action accrues. Given
that the case was filed only on September 27, 2002, or almost five (5) years from the date of the alleged illegal
dismissal of her husband Jones, Erlinda’s claim had already prescribed.

LA RULING:
Ruled in favor of respondent. Ordered petitioners to pay unpaid salaries, 13th month pay and separation pay.
NLRC RULING:
Petitioners filed a notice of appeal with motion to reduce bond and attached thereto photocopies of the receipts for
the cash bond but their motion to reduce the bond was denied and was ordered to post an additional bond.

Petitioners moved for the reconsideration of the NLRC decision denying their notice of appeal with motion to reduce
bond.

NLRC issued a resolution denying such motion for reconsideration.

On April 22, 2004, the aforesaid resolution became final and executory. Thus, herein private respondent Erlinda filed
a motion for execution.

On May 28, 2004, petitioners filed a petition for certiorari under Rule 65 of the Rules of Court before the CA and
prayed for the issuance of a temporary restraining order (TRO) and a writ of preliminary injunction.

CA RULING:
Injunction was granted but CA dismissed the petition for certiorari.

ISSUE:
Did respondent’s money claim already prescribed?

HELD:

Angeles| Bajana | Balladares | Brillantes | Briones | Cabansag | Callanta | Chua | David|


De Leon | Gomez | Lopez | Macalino | Nostratis | Padilla | Reynon | Santos | Tan |Velasco
4E / 4F - 2018-2019
Page 889 of 920
LABOR REVIEW DIGEST
Atty. Joyrich Golangco

No. The Court held that based on the findings of facts of the LA, it was ASI which was responsible for the delay in the
institution of the complaint. When Jones filed his resignation, he immediately asked for the payment of his money
claims. However, the management of ASI promised him that he would be paid immediately after the claims of the
rank-and-file employees had been paid. Jones relied on this representation. Unfortunately, the promise was never
fulfilled even until the time of Jones death.

It then applied the principle of promissory estoppel, it stated:

“In light of these circumstances, we can apply the principle of promissory estoppel, which
is a recognized exception to the three-year prescriptive period enunciated in Article 291 of
the Labor Code.

Promissory estoppel may arise from the making of a promise, even though without
consideration, if it was intended that the promise should be relied upon, as in fact it was
relied upon, and if a refusal to enforce it would virtually sanction the perpetration of fraud
or would result in other injustice. Promissory estoppel presupposes the existence of a
promise on the part of one against whom estoppel is claimed. The promise must be
plain and unambiguous and sufficiently specific so that the court can understand the
obligation assumed and enforce the promise according to its terms.

It further stated that in order to make out a claim of promissory estoppel, a party bears the burden of
establishing the following elements: (1) a promise was reasonably expected to induce action or forbearance;
(2) such promise did, in fact, induce such action or forbearance; and (3) the party suffered detriment as a
result.

The Supreme Court then ruled that all the requisites of promissory estoppel are present in the case. Jones
relied on the promise of ASI that he would be paid as soon as the claims of all the rank-and-file employees
had been paid. If not for this promise that he had held on to until the time of his death, there’s no reason
why he would delay filing the complaint before the LA. Great injustice will be committed if the court will brush
aside the employees claims on a mere technicality, especially when it was petitioner’s own action that
prevented respondent from interposing the claims within the required period.

Angeles| Bajana | Balladares | Brillantes | Briones | Cabansag | Callanta | Chua | David|


De Leon | Gomez | Lopez | Macalino | Nostratis | Padilla | Reynon | Santos | Tan |Velasco
4E / 4F - 2018-2019
Page 890 of 920
LABOR REVIEW DIGEST
Atty. Joyrich Golangco

5. AUTOBUS TRANSPORT SYSTEM V. BAUTISTA


GR NO: 156367 DATE: MAY 16, 2005
PONENTE: CHICO-NAZARIO, J.

Doctrine:
Correspondingly, it can be conscientiously deduced that the cause of action of an entitled employee to claim
his service incentive leave pay accrues from the moment the employer refuses to remunerate its monetary
equivalent if the employee did not make use of said leave credits but instead chose to avail of its
commutation. Accordingly, if the employee wishes to accumulate his leave credits and opts for its
commutation upon his resignation or separation from employment, his cause of action to claim the whole
amount of his accumulated service incentive leave shall arise when the employer fails to pay such amount at
the time of his resignation or separation from employment.

Applying Article 291 of the Labor Code in light of this peculiarity of the service incentive leave, we can
conclude that the three (3)-year prescriptive period commences, not at the end of the year when the
employee becomes entitled to the commutation of his service incentive leave, but from the time when the
employer refuses to pay its monetary equivalent after demand of commutation or upon termination of the
employees services, as the case may be.

FACTS:
Respondent Antonio Bautista was employed by petitioner Autobus Transport System as a driver-conductor with
travel routes Manila-Tuguegarao via Baguio, Baguio- Tuguegarao via Manila and Manila-Tabuk via Baguio. He was
paid on commission basis, seven percent (7%) of the total gross income per travel, on a twice a month basis.

On January 3, 2000, while respondent was driving Autobus No. 114 on Sta. Fe, Nueva Vizcaya, said bus accidentally
bumped the rear portion of Autobus No. 124, the latter having stopped suddenly at a sharp curve without any
warning.

Respondent averred that the accident happened because he was compelled by the management to go back to
Roxas, Isabela, although he had not slept for almost twenty-four (24) hours, as he had just arrived in Manila from
Roxas, Isabela. Respondent further alleged that he was not allowed to work until he fully paid the amount of
P75,551.50, representing thirty percent (30%) of the cost of repair of the damaged buses and that despite
respondents pleas for reconsideration, the same was ignored by management. After a month, management sent him
a letter of termination.

Later on February 2, 2000 respondent filed a complaint for illegal dismissal with claims for non-payment of 13th month
pay and the service incentive leave.

Petitioner, on the other hand, claimed that respondent’s employment was replete with offenses involving reckless
imprudence, gross negligence, and dishonesty, presenting copies of letters, memos, irregularity reports, and
warrants of arrest pertaining to several incidents wherein respondent was involved, in order to support its claim.

Further, petitioner avers that in the exercise of its management prerogative, respondents employment was
terminated only after the latter was provided with an opportunity to explain his side regarding the accident on 03
January 2000.

LA RULING:
Dismissed complaint. However, it held that petitioner is liable for the payment of respondent’s 13th month pay
computed at P78,117.87, and service incentive leave pay at P13,788.05.

NLRC RULING:
Affirmed LA decision but deleted payment of 13th month pay. Petitioner sought reconsideration for the order to pay
the service incentive leave but was denied.

CA RULING:
Affirmed NLRC ruling.

ISSUE:
Is the 3 year prescriptive period in Art. 291 of the Labor Code applicable to respondent’s claim of service incentive
leave pay?

Angeles| Bajana | Balladares | Brillantes | Briones | Cabansag | Callanta | Chua | David|


De Leon | Gomez | Lopez | Macalino | Nostratis | Padilla | Reynon | Santos | Tan |Velasco
4E / 4F - 2018-2019
Page 891 of 920
LABOR REVIEW DIGEST
Atty. Joyrich Golangco

HELD:
No. The Court first ruled whether respondent is entitled to payment of service incentive leave. While petitioner claims
that respondent is a field a personnel and hence exempted from the coverage of the service incentive leave, the
court ruled otherwise. The court then determined whether Art. 291 is applicable to respondent’s claim to determine
the amount of service incentive leave pay that respondent is entitled to, if any exists. It then held:

The question now that must be addressed is up to what amount of service incentive leave pay respondent is entitled
to.

The response to this query inevitably leads us to the correlative issue of whether or not the three (3)-year prescriptive
period under Article 291 of the Labor Code is applicable to respondents claim of service incentive leave pay.

Article 291 of the Labor Code states that all money claims arising from employer-employee relationship shall be filed
within three (3) years from the time the cause of action accrued; otherwise, they shall be forever barred.

In the application of this section of the Labor Code, the pivotal question to be answered is when does the cause of
action for money claims accrue in order to determine the reckoning date of the three-year prescriptive period.

It is settled jurisprudence that a cause of action has three elements, to wit, (1) a right in favor of the plaintiff by
whatever means and under whatever law it arises or is created; (2) an obligation on the part of the named defendant
to respect or not to violate such right; and (3) an act or omission on the part of such defendant violative of the right of
the plaintiff or constituting a breach of the obligation of the defendant to the plaintiff.

To properly construe Article 291 of the Labor Code, it is essential to ascertain the time when the third element of a
cause of action transpired. Stated differently, in the computation of the three-year prescriptive period, a determination
must be made as to the period when the act constituting a violation of the workers right to the benefits being claimed
was committed. For if the cause of action accrued more than three (3) years before the filing of the money claim, said
cause of action has already prescribed in accordance with Article 291.

Consequently, in cases of nonpayment of allowances and other monetary benefits, if it is established that the benefits
being claimed have been withheld from the employee for a period longer than three (3) years, the amount pertaining
to the period beyond the three-year prescriptive period is therefore barred by prescription. The amount that can only
be demanded by the aggrieved employee shall be limited to the amount of the benefits withheld within three (3) years
before the filing of the complaint.

It is essential at this point, however, to recognize that the service incentive leave is a curious animal in relation to
other benefits granted by the law to every employee. In the case of service incentive leave, the employee may
choose to either use his leave credits or commute it to its monetary equivalent if not exhausted at the end of the year.
Furthermore, if the employee entitled to service incentive leave does not use or commute the same, he is entitled
upon his resignation or separation from work to the commutation of his accrued service incentive leave.

Correspondingly, it can be conscientiously deduced that the cause of action of an entitled employee to claim his
service incentive leave pay accrues from the moment the employer refuses to remunerate its monetary equivalent if
the employee did not make use of said leave credits but instead chose to avail of its commutation. Accordingly, if the
employee wishes to accumulate his leave credits and opts for its commutation upon his resignation or separation
from employment, his cause of action to claim the whole amount of his accumulated service incentive leave shall
arise when the employer fails to pay such amount at the time of his resignation or separation from employment.

Applying Article 291 of the Labor Code in light of this peculiarity of the service incentive leave, we can conclude that
the three (3)-year prescriptive period commences, not at the end of the year when the employee becomes entitled to
the commutation of his service incentive leave, but from the time when the employer refuses to pay its monetary
equivalent after demand of commutation or upon termination of the employees services, as the case may be.

In the case at bar, respondent had not made use of his service incentive leave nor demanded for its commutation
until his employment was terminated by petitioner. Neither did petitioner compensate his accumulated service
incentive leave pay at the time of his dismissal. It was only upon his filing of a complaint for illegal dismissal, one
month from the time of his dismissal, that respondent demanded from his former employer commutation of his
accumulated leave credits. His cause of action to claim the payment of his accumulated service incentive leave thus
accrued from the time when his employer dismissed him and failed to pay his accumulated leave credits.

Angeles| Bajana | Balladares | Brillantes | Briones | Cabansag | Callanta | Chua | David|


De Leon | Gomez | Lopez | Macalino | Nostratis | Padilla | Reynon | Santos | Tan |Velasco
4E / 4F - 2018-2019
Page 892 of 920
LABOR REVIEW DIGEST
Atty. Joyrich Golangco

Therefore, the prescriptive period with respect to his claim for service incentive leave pay only commenced from the
time the employer failed to compensate his accumulated service incentive leave pay at the time of his dismissal.
Since respondent had filed his money claim after only one month from the time of his dismissal, necessarily, his
money claim was filed within the prescriptive period provided for by Article 291 of the Labor Code.

Other Notes/ SC Pronouncements:


Relevant portions of the SC ruling on whether respondent is covered by service incentive leave:

Therefore, petitioners contention that respondent is not entitled to the grant of service incentive leave just because he
was paid on purely commission basis is misplaced. What must be ascertained in order to resolve the issue of
propriety of the grant of service incentive leave to respondent is whether or not he is a field personnel.

XXXXXXXXXXXX

To this discussion by the BWC, the petitioner differs and postulates that under said advisory opinion, no employee
would ever be considered a field personnel because every employer, in one way or another, exercises control over
his employees. Petitioner further argues that the only criterion that should be considered is the nature of work of the
employee in that, if the employees job requires that he works away from the principal office like that of a messenger
or a bus driver, then he is inevitably a field personnel.

We are not persuaded. At this point, it is necessary to stress that the definition of a field personnel is not merely
concerned with the location where the employee regularly performs his duties but also with the fact that the
employees performance is unsupervised by the employer. As discussed above, field personnel are those who
regularly perform their duties away from the principal place of business of the employer and whose actual hours of
work in the field cannot be determined with reasonable certainty. Thus, in order to conclude whether an employee is
a field employee, it is also necessary to ascertain if actual hours of work in the field can be determined with
reasonable certainty by the employer. In so doing, an inquiry must be made as to whether or not the employees time
and performance are constantly supervised by the employer.

As observed by the Labor Arbiter and concurred in by the Court of Appeals:


It is of judicial notice that along the routes that are plied by these bus companies, there are its
inspectors assigned at strategic places who board the bus and inspect the passengers, the
punched tickets, and the conductors reports. There is also the mandatory once-a-week car barn or
shop day, where the bus is regularly checked as to its mechanical, electrical, and hydraulic
aspects, whether or not there are problems thereon as reported by the driver and/or conductor.
They too, must be at specific place as [sic] specified time, as they generally observe prompt
departure and arrival from their point of origin to their point of destination. In each and every depot,
there is always the Dispatcher whose function is precisely to see to it that the bus and its crew
leave the premises at specific times and arrive at the estimated proper time. These, are present in
the case at bar. The driver, the complainant herein, was therefore under constant supervision while
in the performance of this work. He cannot be considered a field personnel.

Angeles| Bajana | Balladares | Brillantes | Briones | Cabansag | Callanta | Chua | David|


De Leon | Gomez | Lopez | Macalino | Nostratis | Padilla | Reynon | Santos | Tan |Velasco
4E / 4F - 2018-2019
Page 893 of 920
LABOR REVIEW DIGEST
Atty. Joyrich Golangco

6. MONTERO V. TIMES TRANSPORT


GR NO: 190828 DATE: MARCH 16, 2015
PONENTE: REYES, J.

Doctrine:
Settled is the rule that when one is arbitrarily and unjustly deprived of his job or means of livelihood, the
action instituted to contest the legality of one’s dismissal from employment constitutes, in essence, an
action predicated upon an injury to the rights of the plaintiff, as contemplated under Article 1146 of the New
Civil Code, which must be brought within four years

The petitioners contend that the period when they filed a labor case on May 14, 1998 but withdrawn on March
22, 1999 should be excluded from the computation of the four-year prescriptive period for illegal dismissal
cases. However, the Court had already ruled that the prescriptive period continues even after the withdrawal
of the case as though no action has been filed at all. The applicability of Article 1155 of the Civil Code in
labor cases was upheld in the case of Intercontinental Broadcasting Corporation v. Panganiban where the
Court held that although the commencement of a civil action stops the running of the statute of prescription
or limitations, its dismissal or voluntary abandonment by plaintiff leaves the parties in exactly the same
position as though no action had been commenced at all.

In like manner, while the filing of the complaint for illegal dismissal before the LA interrupted the running of
the prescriptive period, its voluntary withdrawal left the petitioners in exactly the same position as though no
complaint had been filed at all. The withdrawal of their complaint effectively erased the tolling of the
reglementary period.

FACTS:
Petitioner Onofre Montero and his co-petitioners (Edgardo Estraero, Rening Padre, Gabriel Madera, Herminio Tacle,
Nelson Villoria, Demetrio Pajarillo, Alfredo Aganon, Reynaldo Avila, Albert Ruiz, Nestor Yago, Harty Tupasi, Agustin
Avila, Jr, or Marcos Avila, Bonifacio Gaano, Joselito Cuenta, Jonas Estilong, Dominador Canaria, Genaro Rondaris,
Herardo Dulay, Franklin Ravina, Jr., and Ruben Cabello) were employees of respondent Times Transportation Co.,
Inc. (TTCI) engaged in the business transporting passengers and goods from Ilocos Region to Metro Manila.

Around 1995, the rank-and-file employees of TTCI formed the Times Employees Union (TEU).

On March 1997, members of TEU went on strike but the Labor Secretary assumed jurisdiction over the labor dispute
and certified the same for compulsory arbitration, a return-to-work Order dated March 10, 1997 was issued which
ended said strike.

On August 23, 1997, TTCI’s Board of Directors gave authority to respondent Rondaris Santiago, President and
Chairman of the Board of Directors, to gradually dispose the assets of TTCI due to losses for the last 2 years.
Respondent TTCI also adopted a company-wide retrenchment program, to take effect on October 1, 1997, where
respondent Santiago was given the authority to determine the number of excess employees who would be the
subject of retrenchment.

25 buses were sold and Certificates of Public Convenience for the operation of said buses, were later on transferred
to respondent Mencorp Transport Systems, Inc., (Mencorp). Later on, several union members received notices that
they were being retrenched effective 30 days from September 16, 1997.

On October 17, 1997, TEU held another strike against TTCI, but the latter merely reiterated the earlier return-to-work
order of the Labor Secretary. For disregarding the said return-to-work order, respondent Santiago issued two notices
of termination dated October 26, 1997 terminating some 106 workers and a revised list dated November 24, 1997
increasing the number of dismissed employees to 119, for participating in the illegal strike.

On December 4, 1997, respondent Santiago notified the DOLE that TTCI would be closing its operations due to
heavy business losses.

On May 14, 1998, petitioners Estraero, Pajarillo, Padre, Avila, Avila, Jr., Tupasi, Cuenta, Dulay, Yago, and Aganon
filed several complaints against respondents Times and Mencorp. Said complaints were consolidated under the case
entitled Malana v. TTCI docketed as NLRC RAB-I-01-1007. However, said case was withdrawn on March 4, 1999
upon motion by the TEU’s counsel which was given due course on March 22, 1999.

Angeles| Bajana | Balladares | Brillantes | Briones | Cabansag | Callanta | Chua | David|


De Leon | Gomez | Lopez | Macalino | Nostratis | Padilla | Reynon | Santos | Tan |Velasco
4E / 4F - 2018-2019
Page 894 of 920
LABOR REVIEW DIGEST
Atty. Joyrich Golangco

Four years later, several complaints for unfair labor practice, illegal dismissal with money claims, damages and
attorney’s fees were filed against respondents TTCI, Santiago, MENCORP and its General Manager Virginia
Mendoza, including the latter’s husband Reynaldo Mendoza (collectively called the respondents), before the LA from
June to July 2002. Accordingly, these complaints were consolidated.

However, respondent TTCI averred that petitioners’ cause of action had already prescribed (following Art. 291, now
Art. 306) because the complaints were filed only in June 2002 or after almost five years from the date of their
dismissal. Respondent Mencorp, on the other hand, raised the defense of lack of employer-employee relationship
with petitioners

LA RULING:
Rendered a decision dismissing the petitioners’ claim for unfair labor practice and money claims on the ground of
prescription. However, with regard to the issue of illegal dismissal, only the complaints of Montero, Ravina, Cabello,
Genaro, Madera, Gaano, Arsenio Donato and Estilong were dismissed for having been barred by prescription.

NLRC RULING:
Set aside LA ruling (on account of selective dismissal of complaints), and noted that LA had ignored the rule on
prescription, and became selective in awarding relief to the 10 complainants by stating the period during which the
labor cases were pending should be deducted from the period of prescription. According to the NLRC:

We have thoroughly examined the records and find no justification for the [LA] to rule that the
pendency of the cases has worked in favor of the complainants to whom he awarded separation
pay and backwages. The [LA] has not at all indicated in his decision when the eight (8)[-]month
period of pendency he alluded to commenced and when it ended. As a matter of fact, these cases
took almost three (3) years from filing of the complaints to the rendition of the appealed decision.

The NLRC added that the application of the principle of prescription should not be done on a selective basis,
especially when the dates of accrual of the causes of action and the filing of the complaints readily show that
prescription has set in.

CA RULING:
Affirmed NLRC ruling, stating that complaint was filed only on June 2002 or for more than 4 years and 7 months from
the time petitioners received the notices of their dismissal in November and October 1997, and hence prescription
under Art. 291 (Now 306) applies.
It further held that there is lno merit in petitioners’ contention that the period when they filed a complaint on May 14,
1998 but withdrawn on March 30, 1998 should be excluded from the computation of the four-year prescriptive period
for illegal dismissal cases. It held that the prescriptive period continues even after the withdrawal of the case as
though no action has been filed at all. It cited the case of Intercontinental Broadcasting Corporation vs.
Panganiban, where the Supreme Court held that although the commencement of an action stops the running of the
statute of prescription or limitations, its dismissal or voluntary abandonment by plaintiff leaves the parties in exactly
the same position as though no action had been commenced at all.

ISSUE:
Had petitioners’ complaint for illegal dismissal already prescribed?

HELD:
Yes. Settled is the rule that when one is arbitrarily and unjustly deprived of his job or means of livelihood,
the action instituted to contest the legality of one’s dismissal from employment constitutes, in essence, an action
predicated upon an injury to the rights of the plaintiff, as contemplated under Article 1146 of the New Civil Code,
which must be brought within four years.

The petitioners contend that the period when they filed a labor case on May 14, 1998 but withdrawn on March 22,
1999 should be excluded from the computation of the four-year prescriptive period for illegal dismissal cases.
However, the Court had already ruled that the prescriptive period continues even after the withdrawal of the case as
though no action has been filed at all. The applicability of Article 1155 of the Civil Code in labor cases was upheld in
the case of Intercontinental Broadcasting Corporation v. Panganiban where the Court held that although the
commencement of a civil action stops the running of the statute of prescription or limitations, its dismissal or voluntary
abandonment by plaintiff leaves the parties in exactly the same position as though no action had been commenced at

Angeles| Bajana | Balladares | Brillantes | Briones | Cabansag | Callanta | Chua | David|


De Leon | Gomez | Lopez | Macalino | Nostratis | Padilla | Reynon | Santos | Tan |Velasco
4E / 4F - 2018-2019
Page 895 of 920
LABOR REVIEW DIGEST
Atty. Joyrich Golangco

all.

In like manner, while the filing of the complaint for illegal dismissal before the LA interrupted the running of the
prescriptive period, its voluntary withdrawal left the petitioners in exactly the same position as though no complaint
had been filed at all. The withdrawal of their complaint effectively erased the tolling of the reglementary period.

A prudent review of the antecedents of the claim reveals that it has in fact prescribed due to the petitioners’
withdrawal of their labor case docketed as NLRC RAB-I-01-1007. Hence, while the filing of the said case could have
interrupted the running of the four-year prescriptive period, the voluntary withdrawal of the petitioners effectively
cancelled the tolling of the prescriptive period within which to file their illegal dismissal case, leaving them in exactly
the same position as though no labor case had been filed at all. The running of the four-year prescriptive period
not having been interrupted by the filing of NLRC RAB-I-01-1007, the petitioners’ cause of action had already
prescribed in four years after their cessation of employment on October 26, 1997 and November 24, 1997.
Consequently, when the petitioners filed their complaint for illegal dismissal, separation pay, retirement
benefits, and damages in 2002, their claim, clearly, had already been barred by prescription.

Other Notes/ SC Pronouncements:


Names of all petitioners were stated given that the LA selectively dismissed some of the complaints. Dates were
highlighted in bold due to their significance in determining whether prescription has set.

Angeles| Bajana | Balladares | Brillantes | Briones | Cabansag | Callanta | Chua | David|


De Leon | Gomez | Lopez | Macalino | Nostratis | Padilla | Reynon | Santos | Tan |Velasco
4E / 4F - 2018-2019
Page 896 of 920
LABOR REVIEW DIGEST
Atty. Joyrich Golangco

EFFECT OF CHANGE OF OWNERSHIP OF BUSINESS

1. PENAFRANCIA TOURS AND TRAVEL TRANSPORT V. SARMIENTO


GR NO: 178397 DATE: OCTOBER 20, 2010
PONENTE: NACHURA, J.

Doctrine:
Closure of business is the reversal of fortune of the employer whereby there is a complete cessation of
business operations and/or an actual locking-up of the doors of the establishment, usually due to financial
losses. Closure of business, as an authorized cause for termination of employment, aims to prevent further
financial drain upon an employer who can no longer pay his employees since business has already stopped.
On this ground, petitioner terminated the employment of respondents. However, what petitioner apparently
made was a transfer of ownership. It is true that, as invoked by petitioner, in Manlimos, et al. v. NLRC, et
al., we held that a change of ownership in a business concern is not proscribed by law. Lest petitioner
forget, however, we also held therein that the sale or disposition must be motivated by good faith as a
condition for exemption from liability. Thus, where the charge of ownership is done in bad faith, or is used to
defeat the rights of labor, the successor-employer is deemed to have absorbed the employees and is held
liable for the transgressions of his or her predecessor.
But, in this case, there is no successor-employer because there was no actual change of ownership. We
sustain the uniform factual finding of both the NLRC and the CA that no actual sale transpired and, as such,
there is no closure or cessation of business that can serve as an authorized cause for the dismissal of
respondents.
FACTS:
Respondent Joselito Sarmiento (Sarmiento) was employed as bus inspector of petitioner Penafrancia Tours and
Travel Transport, Inc. (PTTI) from October 1993 until October 30, 2002 while his co-respondent Ricardo Catimbang
(Catimbang) was also employed as a bus inspector from February 1997 to October 30, 2002. Both filed for illegal
dismissal with Sarmiento charging respondent of underpayment of wages; non-payment of overtime, holiday pay,
premium pay for holiday and rest day, service incentive leave pay, 13th month pay, and separation pay; unfair labor
practice; damages; and attorney’s fees. Catimbang, on the other hand, charged respondent with union-busting, and
prayed for reinstatement with payment of full backwages, benefits, damages, and attorney’s fees.

Both Sarmiento and Catimbang (respondents) averred that they were required to work seven (7) days a week, and
that they had no rest day and worked even during the holidays, except Good Friday, Christmas Eve, and New Year’s
Eve. Sometime in the first week of October 2002, they received notices of termination on the ground of petitioner’s
alleged irreversible business losses.

Mid October 2002, petitioner’s President and General Manager, Bonifacio Cu called a meeting where
respondents were introduced to Alfredo Perez, owner of ALPS Transportation, who allegedly bought PTTI. On
October 30, 2002, respondents received their last pay with a letter informing them that their application with ALPS
had been held in abeyance. However, respondents averred that several days after their termination, Bonifacio Cu
continued to operate PTTI.

Petitioner PTTI, on the other hand, claimed that PTTI was sold due to severe business losses and that due notice
was given to DOLE and that all its employees were duly notified and paid their corresponding separation pay, as well
as their 13th month pay. Further, The new owners, the Perez family, (owners of ALPS) maintained the business
name of petitioner, and the management of petitioner was entrusted to the new owners in October 2002, with
Edilberto Perez as Vice-President for Finance and Operations (ALPS). It further argued that the matter of
rehiring respondents rested on the sound discretion ALPS, and the latter could not be compelled to absorb
petitioner’s former employees since the same was not part of the deal. Petitioner alleged that respondents submitted
their application for reemployment but, after evaluation, the new owners opted not to hire respondents.

While the case was pending before the LA Edilberto Perez issued a notice to all employees of petitioner, stating
that, effective February 11, 2003, the management of the company shall revert to its former President,
Bonifacio Cu. On February 28, 2003, Bonifacio Cu wrote Alfredo Perez notifying of the former’s decisions to
rescind the sale due to the latter’s non-compliance with the sale agreement. Later on, sometime in March
2003, Bonifacio Cu entered into a transaction, denominated as a "Deed of Sale with Assignment of Franchise
(By Way of Dation in Payment)," with Southern Comfort Bus Co., Inc. (SCBC), represented by its President
and General Manager, Willy Deterala.

LA RULING:

Angeles| Bajana | Balladares | Brillantes | Briones | Cabansag | Callanta | Chua | David|


De Leon | Gomez | Lopez | Macalino | Nostratis | Padilla | Reynon | Santos | Tan |Velasco
4E / 4F - 2018-2019
Page 897 of 920
LABOR REVIEW DIGEST
Atty. Joyrich Golangco

Dismissed complaint but ordered petitioner PTTI and Bonifacio Cu to pay respondents service incentive leave pay.

NLRC RULING:
Reversed LA. Held that no sale of PTTI actually transpired. Ordered reinstatement of respondents with full
backwages.

Petitioner filed MR but was denied.

CA RULING:
Affirmed NLRC. Held that petitioner failed to substantiate its allegation of suffering from business reverses.
Concurred with NLRC on the finding that no sale of PTTI actually transpired.

ISSUE:
Were respondents legally terminated by reason of the sale of the business enterprise and the consequent change or
transfer of ownership/management?

HELD:
NO. Closure of business is the reversal of fortune of the employer whereby there is a complete cessation of business
operations and/or an actual locking-up of the doors of the establishment, usually due to financial losses. Closure of
business, as an authorized cause for termination of employment, aims to prevent further financial drain upon an
employer who can no longer pay his employees since business has already stopped.

On this ground, petitioner terminated the employment of respondents. However, what petitioner apparently made was
a transfer of ownership. It is true that, as invoked by petitioner, in Manlimos, et al. v. NLRC, et al., we held that a
change of ownership in a business concern is not proscribed by law. Lest petitioner forget, however, we also held
therein that the sale or disposition must be motivated by good faith as a condition for exemption from liability. Thus,
where the charge of ownership is done in bad faith, or is used to defeat the rights of labor, the successor-employer is
deemed to have absorbed the employees and is held liable for the transgressions of his or her predecessor.

But, in this case, there is no successor-employer because there was no actual change of ownership. We sustain the
uniform factual finding of both the NLRC and the CA that no actual sale transpired and, as such, there is no closure
or cessation of business that can serve as an authorized cause for the dismissal of respondents. Notable in this
regard are the following observations of the CA:

Petitioner PTTTI sent notices of termination to private respondents Sarmiento and


Catimbang on the alleged ground that it would cease operations effective 30 October
2002 due to business reverses and it would eventually sell the same to another
company. However, the records explicitly show that it (PTTTI) failed to establish its
allegation that it was suffering from business reverses. Neither was there proof that
indeed a sale was made and executed on 01 October 2002 involving the company’s
assets in favor of ALPS Transportation owned by the Perez family. It did not present
any documentary evidence to support its claim that it sold the same to ALPS
Transportation. On the contrary, it (PTTTI) continuously operates under the same
name, franchises and routes and under the same circumstances as before the
alleged sale. It (PTTTI) tried to convince us that it is under a new management, by
presenting series of memoranda where the signatory thereon is Edilberto E. Perez,
VP-Finance/Operations. To us, the series of memoranda do not conclusively show
that there had been a sale in favor of ALPS Transportation. And considering that
there was no sale which transpired, we also find no basis for the rescission thereof.
The letter dated 19 March 2003 addressed to its employees, informing the latter that
it had rescinded its sale to ALPS Transportation and thus, there is a change of
management, ownership and operation of the company and it (PTTTI) is intending to
sell the company to Southern Comfort Bus Co., Inc. headed by Mr. Willy D. Deterala
could not convince us that there was actually a rescission of sale. If indeed there was
sale and a consequent rescission thereof which transpired, why is it that the ALPS
Transportation did not give much a fight when the contract of sale was unilaterally
rescinded by Bonifacio Cu who signed as President/General Manager of petitioner
PTTTI in a letter dated 28 February 2003. It is quite unconceivable for a company
like ALPS Transportation which had already parted a considerable sum not to
question the rescission undertaken by petitioner PTTTI. This only confirms the public

Angeles| Bajana | Balladares | Brillantes | Briones | Cabansag | Callanta | Chua | David|


De Leon | Gomez | Lopez | Macalino | Nostratis | Padilla | Reynon | Santos | Tan |Velasco
4E / 4F - 2018-2019
Page 898 of 920
LABOR REVIEW DIGEST
Atty. Joyrich Golangco

respondent NLRC’s finding, that the sale was indeed a sham, designed to circumvent
the law on the rights of the workers. There is thus, no basis for us to believe that
there was a consequent rescission of the alleged sale made by petitioner PTTTI in
favor of ALPS Transportation.

Corollarily, we opine that the alleged second sale made by petitioner PTTTI, this time
in favor of Southern Comfort Bus Co., Inc. represented by one Willy D. Deter[a]la is
also simulated considering that the ten million pesos consideration is unbelievably
too small for thirty five (35) aircon buses including its franchise and facilities thereon.
It is quite an illogical move for the company to have allegedly rescinded the previous
sale involving a higher consideration of sixty million pesos (₱60,000,000.00) made in
favor of ALPS Transportation and to resell the same, this time just for a measly
amount of ten million pesos (₱10,000,000.00). Additionally, the observation of
private respondents Sarmiento and Catimbang is quite impressive when they
claimed that the Southern Comfort Bus Co., Inc., presided by one Willy D. Deterala is
a dummy corporation since it has not operated any single bus under its name, even
prior to the sale and up to the present. In fact, its principal business office at No. 4
Cathedral St., Ateneo Avenue 4400 Naga City is not even known. Suffice it to stress,
these private respondents’ allegations/observations have not at all been refuted nor
controverted by petitioner PTTTI.

The fact remains that the Cu family continues to operate petitioner’s business. Despite the alleged recent sale to
SCBC, represented by Willy Deterala, petitioner failed to refute the allegations of respondents that the Cu family still
continues to own and operate petitioner, or even to show that Willy Deterala is actually in charge of petitioner’s
business. Petitioner having failed to discharge its burden of submitting sufficient and convincing evidence required by
law, we hold that respondents were illegally dismissed.

Other Notes/ SC Pronouncements:


Reproduced the findings of CA to show the circumstances supporting the conclusion that the sale of PTTI
did not actually transpired and that the same is still being managed by the Cu family

Angeles| Bajana | Balladares | Brillantes | Briones | Cabansag | Callanta | Chua | David|


De Leon | Gomez | Lopez | Macalino | Nostratis | Padilla | Reynon | Santos | Tan |Velasco
4E / 4F - 2018-2019
Page 899 of 920
LABOR REVIEW DIGEST
Atty. Joyrich Golangco

LIABILITY OF CORPORATE OFFICERS

1. CARAG V. NLRC
GR NO: 147590 DATE: APRIL 2, 2007
PONENTE: CARPIO, J.

Doctrine:
A corporate officer is not personally liable for the money claims of discharged corporate employees unless
he acted with evident malice and bad faith in terminating their employment. There is no evidence in this case
that Locsin acted in bad faith or with malice in carrying out the retrenchment and eventual closure of the
company (Garcia vs. NLRC, 153 SCRA 640), hence, he may not be held personally and solidarily liable with
the company for the satisfaction of the judgment in favor of the retrenched employees.

Neither does bad faith arise automatically just because a corporation fails to comply with the notice
requirement of labor laws on company closure or dismissal of employees. The failure to give notice is not an
unlawful act because the law does not define such failure as unlawful. Such failure to give notice is a
violation of procedural due process but does not amount to an unlawful or criminal act. Such procedural
defect is called illegal dismissal because it fails to comply with mandatory procedural requirements, but it is
not illegal in the sense that it constitutes an unlawful or criminal act.

For a wrongdoing to make a director personally liable for debts of the corporation, the wrongdoing approved
or assented to by the director must be a patently unlawful act. Mere failure to comply with the notice
requirement of labor laws on company closure or dismissal of employees does not amount to a patently
unlawful act. Patently unlawful acts are those declared unlawful by law which imposes penalties for
commission of such unlawful acts. There must be a law declaring the act unlawful and penalizing the act.

xxxxxxxxxxxxxxxxxxxxxxxx

…. Article 212(e) of the Labor Code, by itself, does not make a corporate officer personally liable for the
debts of the corporation. The governing law on personal liability of directors for debts of the corporation is
still Section 31 of the Corporation Code.

xxxxxxxxxxxxxxxxxxxxxxxxx

Thus, the rule is still that the doctrine of piercing the corporate veil applies only when the corporate fiction is
used to defeat public convenience, justify wrong, protect fraud, or defend crime. In the absence of malice,
bad faith, or a specific provision of law making a corporate officer liable, such corporate officer cannot be
made personally liable for corporate liabilities. Neither Article 212[e] nor Article 273 (now 272) of the Labor
Code expressly makes any corporate officer personally liable for the debts of the corporation.

FACTS:
Respondents National Federation of Labor Unions (NAFLU) and Mariveles Apparel Corporation Labor Union
(MACLU) filed a complaint dated August 12 1993 against Mariveles Apparel Corporation (MAC) for illegal dismissal
brought about by its illegal closure of business.

It was averred that NAFLU is the sole bargaining unit of MAC, that a CBA was existing between the two, and that
there was no labor dispute between the union and the management except for the ones filed. However, on July 8,
1993, without notice of any kind filed in accordance with pertinent provisions of the Labor Code, MAC ceased
operations with the intention of completely closing its shop or factory. Such intentions were manifested in a letter,
allegedly claimed by MAC as its notice filed only on the same day that the operations closed.

Respondent LA Ortiguerra, upon receipt of the records of the case, summoned the parties for possible settlement.
However, MAC did not appear, prompting the latter to declare the case submitted for resolution "based on the extant
pleadings."

In their position paper dated January 3 1994, respondents NAFLU and MACFLU moved to implead Carag and
David. They argued that any decision rendered by the LA will be for naught considering that MAC has already
ceased to exist, having stopped its operations since July 8, 1993. As such, respondents sought to implead
petitioner Antonio Carag, chairman of the board of MAC, and Armando David, the president, both being

Angeles| Bajana | Balladares | Brillantes | Briones | Cabansag | Callanta | Chua | David|


De Leon | Gomez | Lopez | Macalino | Nostratis | Padilla | Reynon | Santos | Tan |Velasco
4E / 4F - 2018-2019
Page 900 of 920
LABOR REVIEW DIGEST
Atty. Joyrich Golangco

owners of MAC at the same time, in order to guarantee the satisfaction of any award on the basis of Art. 212
(c) of the Labor Code which provides:

"Employer includes any person acting in the interest of an employer, directly or indirectly. It
does not, however, include any labor organization or any of its officers or agents except when
acting as employer."

Carag and David, on the other hand, claimed that they should not have been impleaded since they only own shares
of MAC in order to qualify them as officers and that MAC is actually owned by a consortium of banks.

Without any further proceedings, respondent LA Ortiguerra, allowed the impleading of the Carag and David, and
rendered a decision

LA RULING:
MAC guilty of closure and Carag and David is solidary liable for payment of separation pay of respondent NAFLU
and MACFLU members together with attorney’s fees.

Significant parts of LA ruling are hereby reproduced:


The complainants bewail that at the time of the closure, employees who have rendered one to two weeks of work
were not given their salaries and the same have remained unpaid.

The complainants aver that respondent company prior to its closure did not even bother to serve written notice to
employees and to the Department of Labor and Employment at least one month before the intended date of closure.
The respondents did not even establish that its closure was done in good faith. Moreover, the respondents did not
pay the affected employees separation pay, the amount of which is provided in the existing Collective Bargaining
Agreement between the complainants and the respondents.

The complainants pray that they be allowed to implead Atty. Antonio Carag and Mr. Armando David, owners and
responsible officers of respondent company to assure the satisfaction of the judgment, should a decision favorable to
them be rendered. In support of their claims, the complainants invoked the ruling laid down by the Supreme Court in
the case of A.C. Ransom Labor Union CCLU vs. NLRC, G.R. No. 69494, June 10, 1986 where it was held that [a]
corporate officer can be held liable for acting on behalf of the corporation when the latter is no longer in existence
and there are valid claims of workers that must be satisfied.

xxxxxxxxxxxxxxxxxxxxxx

The respondents described the cessation of operations in its premises as a temporary shut-down. While such
posturing may have been initially true, it is not so anymore. The cessation of operations has clearly exceeded the six
months period fixed in Article 286 of the Labor Code. The temporary shutdown has ripened into a closure or
cessation of operations for causes not due to serious business losses or financial reverses. Consequently, the
respondents must pay the displaced employees separation pay in accordance with the computation prescribed in the
CBA, to wit, one month pay for every year of service. It must be stressed that respondents did not controvert the
verity of the CBA provided computation.

The complainants claim that Atty. Antonio Carag and Mr. Armando David should be held jointly and severally liable
with respondent corporation. This bid is premised on the belief that the impleader of the aforesaid officers will
guarantee payment of whatever may be adjudged in complainants' favor by virtue of this case. It is a basic principle
in law that corporations have personality distinct and separate from the stockholders. This concept is known as
corporate fiction. Normally, officers acting for and in behalf of a corporation are not held personally liable for the
obligation of the corporation. In instances where corporate officers dismissed employees in bad faith or wantonly
violate labor standard laws or when the company had already ceased operations and there is no way by which a
judgment in favor of employees could be satisfied, corporate officers can be held jointly and severally liable with the
company. This Office after a careful consideration of the factual backdrop of the case is inclined to grant
complainants' prayer for the impleader of Atty. Antonio Carag and Mr. Armando David, to assure that valid claims of
employees would not be defeated by the closure of respondent company.

MAC, Carag, and David, through Atty. Pastores, filed their Memorandum before the NLRC on 26 August 1994.
Carag, through a separate counsel, filed an appeal dated 30 August 1994 before the NLRC.

Angeles| Bajana | Balladares | Brillantes | Briones | Cabansag | Callanta | Chua | David|


De Leon | Gomez | Lopez | Macalino | Nostratis | Padilla | Reynon | Santos | Tan |Velasco
4E / 4F - 2018-2019
Page 901 of 920
LABOR REVIEW DIGEST
Atty. Joyrich Golangco

Respondents also filed separate motions to reduce bond.

NLRC RULING:
In a Resolution promulgated on 5 January 1995, the NLRC Third Division denied the motions to reduce bond. The
NLRC stated that to grant a reduction of bond on the ground that the appeal is meritorious would be tantamount to
ruling on the merits of the appeal.

CA RULING:
The CA (Term “Appellate Court” was used in the case text) identified two issues as essential: (1) whether Arbiter
Ortiguerra properly held Carag and David, in their capacities as corporate officers, jointly and severally liable with
MAC for the money claims of the employees; and (2) whether the NLRC abused its discretion in denying the
separate motions to reduce bond filed by MAC and Carag.

The CA held that the absence of a formal hearing before the Labor Arbiter is not a cause for Carag and David to
impute grave abuse of discretion. The appellate court found that Carag and David, as the most ranking officers of
MAC, had a direct hand at the time in the illegal dismissal of MAC's employees. The failure of Carag and David to
observe the notice requirement in closing the company shows malice and bad faith, which justifies their solidary
liability with MAC. The appellate court also found that the circumstances of the present case do not warrant a
reduction of the appeal bond.

ISSUE:
Should petitioner Antonio Carag be held solidarily liable with MAC?

HELD:
NO. Before ruling whether petitioner should be held solidarily liable, the Supreme Court first held that petitioner was
denied due process. It held:

Carag vigorously denied receiving summons to the conference, and complainants have not produced
any order of Arbiter Ortiguerra summoning Carag to the conference. A thorough search of the records
of this case fails to show any order of Arbiter Ortiguerra directing Carag to attend the conference.
Clearly, Arbiter Ortiguerra did not summon Carag to the conference.

When MAC failed to appear at the conference, Arbiter Ortiguerra declared the case submitted for
resolution. In her Decision, Arbiter Ortiguerra granted complainants' motion to implead Carag and at
the same time, in the same Decision, found Carag personally liable for the debts of MAC consisting of
₱49,101,621 in separation pay to complainants. Arbiter Ortiguerra never issued summons to Carag,
never called him to a conference for possible settlement, never required him to submit a position
paper, never set the case for hearing, never notified him to present his evidence, and never informed
him that the case was submitted for decision - all in violation of Sections 2, 3, 4, 5(b), and 11(c) of Rule
V of The New Rules of Procedure of the NLRC.

After ruling that petitioner was denied due process, the Supreme Court then tackled the issue of solidary
liability, ruling that a corporate officer is not personally liable for the money claims of discharged corporate
employees unless he acted with evident malice and bad faith in terminating their employment.

Neither does bad faith arise automatically just because a corporation fails to comply with the notice requirement of
labor laws on company closure or dismissal of employees. The failure to give notice is not an unlawful act because
the law does not define such failure as unlawful. Such failure to give notice is a violation of procedural due process
but does not amount to an unlawful or criminal act. Such procedural defect is called illegal dismissal because it fails
to comply with mandatory procedural requirements, but it is not illegal in the sense that it constitutes an unlawful or
criminal act.

For a wrongdoing to make a director personally liable for debts of the corporation, the wrongdoing approved or
assented to by the director must be a patently unlawful act. Mere failure to comply with the notice requirement of
labor laws on company closure or dismissal of employees does not amount to a patently unlawful act. Patently
unlawful acts are those declared unlawful by law which imposes penalties for commission of such unlawful acts.
There must be a law declaring the act unlawful and penalizing the act.

Angeles| Bajana | Balladares | Brillantes | Briones | Cabansag | Callanta | Chua | David|


De Leon | Gomez | Lopez | Macalino | Nostratis | Padilla | Reynon | Santos | Tan |Velasco
4E / 4F - 2018-2019
Page 902 of 920
LABOR REVIEW DIGEST
Atty. Joyrich Golangco

An example of a patently unlawful act is violation of Article 287 of the Labor Code, which states that "Violation of this
provision is hereby declared unlawful and subject to the penal provisions provided under Article 288 of this Code."
Likewise, Article 288 of the Labor Code on Penal Provisions and Liabilities, provides that "any violation of the
provision of this Code declared unlawful or penal in nature shall be punished with a fine of not less than One
Thousand Pesos (₱1,000.00) nor more than Ten Thousand Pesos (₱10,000.00), or imprisonment of not less than
three months nor more than three years, or both such fine and imprisonment at the discretion of the court."

In this case, Article 283 of the Labor Code, requiring a one-month prior notice to employees and the Department of
Labor and Employment before any permanent closure of a company, does not state that non-compliance with the
notice is an unlawful act punishable under the Code. There is no provision in any other Article of the Labor Code
declaring failure to give such notice an unlawful act and providing for its penalty.

Complainants did not allege or prove, and Arbiter Ortiguerra did not make any finding, that Carag approved or
assented to any patently unlawful act to which the law attaches a penalty for its commission. On this score alone,
Carag cannot be held personally liable for the separation pay of complainants.

It further held that with regard to respondents’ contention of solidary liability under Art. 212 of the Labor Code, that
said article, by itself, does not make a corporate officer personally liable for the debts of the corporation. The
governing law on personal liability of directors for debts of the corporation is still Section 31 of the Corporation Code.
Section 31 provides for instances where the doctrine of piercing of corporate veil shall apply. Thus, the rule is still that
the doctrine of piercing the corporate veil applies only when the corporate fiction is used to defeat public
convenience, justify wrong, protect fraud, or defend crime. In the absence of malice, bad faith, or a specific provision
of law making a corporate officer liable, such corporate officer cannot be made personally liable for corporate
liabilities. Neither Article 212[e] nor Article 273 (now 272) of the Labor Code expressly makes any corporate
officer personally liable for the debts of the corporation.

Thus, it was error for Arbiter Ortiguerra, the NLRC, and the Court of Appeals to hold Carag personally liable for the
separation pay owed by MAC to complainants based alone on Article 212(e) of the Labor Code. Article 212(e) does
not state that corporate officers are personally liable for the unpaid salaries or separation pay of employees of the
corporation. The liability of corporate officers for corporate debts remains governed by Section 31 of the Corporation
Code.

Other Notes/ SC Pronouncements:


SC cited other cases in support of its decision, such as Camelcraft v. NLRC and A.C. Ransom Labor Union-CCLU V.
NLRC. Said cases were cited since respondents anchored their basis for impleading petitioner on said cases. The
SC, however, ruled that said cases do not apply to the instant case.

Angeles| Bajana | Balladares | Brillantes | Briones | Cabansag | Callanta | Chua | David|


De Leon | Gomez | Lopez | Macalino | Nostratis | Padilla | Reynon | Santos | Tan |Velasco
4E / 4F - 2018-2019
Page 903 of 920
LABOR REVIEW DIGEST
Atty. Joyrich Golangco

EFFECT OF COMPANY MERGER ON UNION SHOP CLAUSE

1. BPI v. BPI Employees Union Davao Chapter


GR No: 164301 Date: August 10, 2010
Ponente: Leonardo-De Castro, J.

Doctrine:
All employees in the bargaining unit covered by a Union Shop Clause in their CBA with management are
subject to its terms. However, under law and jurisprudence, the following kinds of employees are exempted
from its coverage, namely, 1.) employees who at the time the union shop agreement takes effect are bona
fide members of a religious organization which prohibits its members from joining labor unions on religious
grounds; 2.) employees already in the service and already members of a union other than the majority at the
time the union shop agreement took effect; 3.) confidential employees who are excluded from the rank and
file bargaining unit; and 4.) employees excluded from the union shop by express terms of the agreement.

xxxxxxxxxxxxxxxxxxxxxxxxxxxxxxxxxxx

It is but fair that similarly situated employees who enjoy the same privileges of a CBA should be likewise
subject to the same obligations the CBA imposes upon them. A contrary interpretation of the Union Shop
Clause will be inimical to industrial peace and workers solidarity. This unfavorable situation will not be
sufficiently addressed by asking the former FEBTC employees to simply pay agency fees to the Union in lieu
of union membership, as the dissent of Justice Carpio suggests. The fact remains that other new regular
employees, to whom the absorbed employees should be compared, do not have the option to simply pay the
agency fees and they must join the Union or face termination.

xxxxxxxxxxxxxxxxxxxxxxxxxxxxxxxxxxxx

There is nothing in the Labor Code and other applicable laws or the CBA provision at issue that requires that
a new employee has to be of probationary or non-regular status at the beginning of the employment
relationship. An employer may confer upon a new employee the status of regular employment even at the
onset of his engagement. Moreover, no law prohibits an employer from voluntarily recognizing the length of
service of a new employee with a previous employer in relation to computation of benefits or seniority but it
should not unduly be interpreted to exclude them from the coverage of the CBA which is a binding
contractual obligation of the employer and employees.

Indeed, a union security clause in a CBA should be interpreted to give meaning and effect to its purpose,
which is to afford protection to the certified bargaining agent and ensure that the employer is dealing with a
union that represents the interests of the legally mandated percentage of the members of the bargaining
unit.

The union shop clause offers protection to the certified bargaining agent by ensuring that future regular
employees who (a) enter the employ of the company during the life of the CBA; (b) are deemed part of the
collective bargaining unit; and (c) whose number will affect the number of members of the collective
bargaining unit will be compelled to join the union. Such compulsion has legal effect, precisely because the
employer by voluntarily entering in to a union shop clause in a CBA with the certified bargaining agent takes
on the responsibility of dismissing the new regular employee who does not join the union.

Without the union shop clause or with the restrictive interpretation thereof as proposed in the dissenting
opinions, the company can jeopardize the majority status of the certified union by excluding from union
membership all new regular employees whom the Company will absorb in future mergers and all new regular
employees whom the Company hires as regular from the beginning of their employment without undergoing
a probationary period. In this manner, the Company can increase the number of members of the collective
bargaining unit and if this increase is not accompanied by a corresponding increase in union membership,
the certified union may lose its majority status and render it vulnerable to attack by another union who
wishes to represent the same bargaining unit.

Or worse, a certified union whose membership falls below twenty percent (20%) of the total members of the
collective bargaining unit may lose its status as a legitimate labor organization altogether, even in a situation
where there is no competing union. In such a case, an interested party may file for the cancellation of the
unions certificate of registration with the Bureau of Labor Relations.

Angeles| Bajana | Balladares | Brillantes | Briones | Cabansag | Callanta | Chua | David|


De Leon | Gomez | Lopez | Macalino | Nostratis | Padilla | Reynon | Santos | Tan |Velasco
4E / 4F - 2018-2019
Page 904 of 920
LABOR REVIEW DIGEST
Atty. Joyrich Golangco

Plainly, the restrictive interpretation of the union shop clause would place the certified unions very existence
at the mercy and control of the employer.

FACTS:
On March 23, 2000, the Bangko Sentral ng Pilipinas approved the Articles of Merger executed on January 20, 2000
by and between petitioner BPI and Far East Bank and Trust Company (FEBTC). This Article and Plan of Merger was
then approved by the SEC on April 7, 2000.

Pursuant to the Article and Plan of Merger, all the assets and liabilities of FEBTC were transferred to and absorbed
by BPI as the surviving corporation. FEBTC employees, including those in its different branches across the country,
were hired by petitioner as its own employees, with their status and tenure recognized and salaries and benefits
maintained.

Respondent BPI Employees Union-Davao Chapter - Federation of Unions in BPI Unibank (Union) is the exclusive
bargaining agent of BPIs rank and file employees in Davao City. The former FEBTC rank-and-file employees in
Davao City did not belong to any labor union at the time of the merger. Prior to the effectivity of the merger, or on
March 31, 2000, respondent Union invited said FEBTC employees to a meeting regarding the Union Shop
Clause (Article II, Section 2) of the existing CBA between petitioner BPI and respondent Union which provides:

ARTICLE II

Section 1. Maintenance of Membership All employees within the bargaining unit who are members of the Union on
the date of the effectivity of this Agreement as well as employees within the bargaining unit who subsequently join or
become members of the Union during the lifetime of this Agreement shall as a condition of their continued
employment with the Bank, maintain their membership in the Union in good standing.

Section 2. Union Shop - New employees falling within the bargaining unit as defined in Article I of this
Agreement, who may hereafter be regularly employed by the Bank shall, within thirty (30) days after they become
regular employees, join the Union as a condition of their continued employment. It is understood that membership in
good standing in the Union is a condition of their continued employment with the Bank.

After the meeting some of the former FEBTC employees joined the Union, while others refused. Later, however,
some of those who initially joined retracted their membership. Respondent union sent notices to the former FEBTC
employees who refused to join, as well as those who retracted their membership, and called them to a hearing
regarding the matter. When these former FEBTC employees refused to attend the hearing, the president of the Union
requested BPI to implement the Union Shop Clause of the CBA and to terminate their employment pursuant thereto.

After two months of management inaction on the request, respondent Union informed petitioner BPI of its decision to
refer the issue of the implementation of the Union Shop Clause of the CBA to the Grievance Committee. However,
the issue remained unresolved at this level and so it was subsequently submitted for voluntary arbitration by the
parties.

VOLUNTARY ARBITRATOR RULING:


Ruled in favor of petitioner. Held that the former FEBTC employees were not covered by the Union Security Clause
of the CBA between the Union and the Bank on the ground that the said employees were not new employees who
were hired and subsequently regularized, but were absorbed employees by operation of law because the former
employees of FEBTC can be considered assets and liabilities of the absorbed corporation. The Voluntary
Arbitrator concluded that the former FEBTC employees could not be compelled to join the Union, as it was their
constitutional right to join or not to join any organization.

Petitioner filed MR but was denied. Appealed to CA

CA RULING:
Reversed voluntary arbitrator’s decision and allowed respondent union to require former FEBTC employees to
become members, or if they refuse, to direct BPI to dismiss them. It further held:
This Court agrees with the voluntary arbitrator that the ABSORBED employees are distinct and
different from NEW employees BUT only in so far as their employment service is concerned. The
distinction ends there. In the case at bar, the absorbed employees length of service from its former

Angeles| Bajana | Balladares | Brillantes | Briones | Cabansag | Callanta | Chua | David|


De Leon | Gomez | Lopez | Macalino | Nostratis | Padilla | Reynon | Santos | Tan |Velasco
4E / 4F - 2018-2019
Page 905 of 920
LABOR REVIEW DIGEST
Atty. Joyrich Golangco

employer is tacked with their employment with BPI. Otherwise stated, the absorbed employees service
is continuous and there is no gap in their service record.

This Court is persuaded that the similarities of new and absorbed employees far outweighs
the distinction between them. The similarities lies on the following, to wit: (a) they have a new
employer; (b) new working conditions; (c) new terms of employment and; (d) new company policy to
follow. As such, they should be considered as new employees for purposes of applying the provisions
of the CBA regarding the union-shop clause.

To rule otherwise would definitely result to a very awkward and unfair situation wherein the absorbed
employees shall be in a different if not, better situation than the existing BPI employees. The existing
BPI employees by virtue of the union-shop clause are required to pay the monthly union dues, remain
as members in good standing of the union otherwise, they shall be terminated from the company, and
other union-related obligations. On the other hand, the absorbed employees shall enjoy the fruits of
labor of the petitioner-union and its members for nothing in exchange. Certainly, this would disturb
industrial peace in the company which is the paramount reason for the existence of the CBA and the
union.

The voluntary arbitrators interpretation of the provisions of the CBA concerning the coverage of the
union-shop clause is at war with the spirit and the rationale why the Labor Code itself allows the
existence of such provision.

ISSUE:
May a corporation invoke its merger with another corporation as a valid ground to exempt its absorbed employees
from the coverage of a union shop clause contained in its existing Collective Bargaining Agreement (CBA) with its
own certified labor union?

HELD:
No. The Supreme Court ruled on many aspects that would support its finding that the former FEBTC employees are
covered by the union shop clause. First, it ruled on whether FEBTC employees can be considered as “new
employees” under the union shop clause as to require them to be members of the union. In this regard, it held:
Section 2, Article II of the CBA is silent as to how one becomes a regular employee of the BPI for the
first time. There is nothing in the said provision which requires that a new regular employee first
undergo a temporary or probationary status before being deemed as such under the union
shop clause of the CBA.

All employees in the bargaining unit covered by a Union Shop Clause in their CBA with management
are subject to its terms. However, under law and jurisprudence, the following kinds of employees
are exempted from its coverage, namely, employees who at the time the union shop agreement
takes effect are bona fide members of a religious organization which prohibits its members from joining
labor unions on religious grounds;[21] employees already in the service and already members of a
union other than the majority at the time the union shop agreement took effect; confidential
employees who are excluded from the rank and file bargaining unit; and employees excluded from
the union shop by express terms of the agreement.

When certain employees are obliged to join a particular union as a requisite for continued employment,
as in the case of Union Security Clauses, this condition is a valid restriction of the freedom or right not
to join any labor organization because it is in favor of unionism. This Court, on occasion, has even held
that a union security clause in a CBA is not a restriction of the right of freedom of association
guaranteed by the Constitution.
Moreover, a closed shop agreement is an agreement whereby an employer binds himself to hire only
members of the contracting union who must continue to remain members in good standing to keep
their jobs. It is the most prized achievement of unionism. It adds membership and compulsory
dues. By holding out to loyal members a promise of employment in the closed shop, it wields group
solidarity.
Indeed, the situation of the former FEBTC employees in this case clearly does not fall within the first
three exceptions to the application of the Union Shop Clause discussed earlier. No allegation or
evidence of religious exemption or prior membership in another union or engagement as a confidential

Angeles| Bajana | Balladares | Brillantes | Briones | Cabansag | Callanta | Chua | David|


De Leon | Gomez | Lopez | Macalino | Nostratis | Padilla | Reynon | Santos | Tan |Velasco
4E / 4F - 2018-2019
Page 906 of 920
LABOR REVIEW DIGEST
Atty. Joyrich Golangco

employee was presented by both parties. The sole category therefore in which petitioner may prove its
claim is the fourth recognized exception or whether the former FEBTC employees are excluded by the
express terms of the existing CBA between petitioner and respondent.

To reiterate, petitioner insists that the term new employees, as the same is used in the Union Shop
Clause of the CBA at issue, refers only to employees hired by BPI as non-regular employees
who later qualify for regular employment and become regular employees, and not those who, as a
legal consequence of a merger, are allegedly automatically deemed regular employees of
BPI. However, the CBA does not make a distinction as to how a regular employee attains such a
status. Moreover, there is nothing in the Corporation Law and the merger agreement mandating the
automatic employment as regular employees by the surviving corporation in the merger.

It also ruled that the non-application of the Union shop clause contrary to the policy of the Labor Code is inimical to
industrial peace. It held:

It is but fair that similarly situated employees who enjoy the same privileges of a CBA should be
likewise subject to the same obligations the CBA imposes upon them. A contrary interpretation of the
Union Shop Clause will be inimical to industrial peace and workers solidarity.

It is but fair that similarly situated employees who enjoy the same privileges of a CBA should be
likewise subject to the same obligations the CBA imposes upon them. A contrary interpretation of the
Union Shop Clause will be inimical to industrial peace and workers solidarity. This unfavorable
situation will not be sufficiently addressed by asking the former FEBTC employees to simply pay
agency fees to the Union in lieu of union membership, as the dissent of Justice Carpio suggests. The
fact remains that other new regular employees, to whom the absorbed employees should be
compared, do not have the option to simply pay the agency fees and they must join the Union or face
termination.

Petitioners restrictive reading of the Union Shop Clause could also inadvertently open an avenue,
which an employer could readily use, in order to dilute the membership base of the certified union in
the collective bargaining unit (CBU). By entering into a voluntary merger with a non-unionized
company that employs more workers, an employer could get rid of its existing union by the simple
expedient of arguing that the absorbed employees are not new employees, as are commonly
understood to be covered by a CBAs union security clause. This could then lead to a new majority
within the CBU that could potentially threaten the majority status of the existing union and, ultimately,
spell its demise as the CBUs bargaining representative. Such a dreaded but not entirely far-fetched
scenario is no different from the ingenious and creative union-busting schemes that corporations have
fomented throughout the years, which this Court has foiled time and again in order to preserve and
protect the valued place of labor in this jurisdiction consistent with the Constitutions mandate of
insuring social justice.

There is nothing in the Labor Code and other applicable laws or the CBA provision at issue that
requires that a new employee has to be of probationary or non-regular status at the beginning of the
employment relationship. An employer may confer upon a new employee the status of regular
employment even at the onset of his engagement. Moreover, no law prohibits an employer from
voluntarily recognizing the length of service of a new employee with a previous employer in relation to
computation of benefits or seniority but it should not unduly be interpreted to exclude them from the
coverage of the CBA which is a binding contractual obligation of the employer and employees.

Indeed, a union security clause in a CBA should be interpreted to give meaning and effect to its
purpose, which is to afford protection to the certified bargaining agent and ensure that the employer is
dealing with a union that represents the interests of the legally mandated percentage of the members
of the bargaining unit.

The union shop clause offers protection to the certified bargaining agent by ensuring that future regular
employees who (a) enter the employ of the company during the life of the CBA; (b) are deemed part of
the collective bargaining unit; and (c) whose number will affect the number of members of the
collective bargaining unit will be compelled to join the union. Such compulsion has legal effect,
precisely because the employer by voluntarily entering in to a union shop clause in a CBA with the

Angeles| Bajana | Balladares | Brillantes | Briones | Cabansag | Callanta | Chua | David|


De Leon | Gomez | Lopez | Macalino | Nostratis | Padilla | Reynon | Santos | Tan |Velasco
4E / 4F - 2018-2019
Page 907 of 920
LABOR REVIEW DIGEST
Atty. Joyrich Golangco

certified bargaining agent takes on the responsibility of dismissing the new regular employee who does
not join the union.

Without the union shop clause or with the restrictive interpretation thereof as proposed in the
dissenting opinions, the company can jeopardize the majority status of the certified union by excluding
from union membership all new regular employees whom the Company will absorb in future mergers
and all new regular employees whom the Company hires as regular from the beginning of their
employment without undergoing a probationary period. In this manner, the Company can increase the
number of members of the collective bargaining unit and if this increase is not accompanied by a
corresponding increase in union membership, the certified union may lose its majority status and
render it vulnerable to attack by another union who wishes to represent the same bargaining unit.

Or worse, a certified union whose membership falls below twenty percent (20%) of the total members
of the collective bargaining unit may lose its status as a legitimate labor organization altogether, even
in a situation where there is no competing union. In such a case, an interested party may file for the
cancellation of the unions certificate of registration with the Bureau of Labor Relations.

Plainly, the restrictive interpretation of the union shop clause would place the certified unions very
existence at the mercy and control of the employer. Relevantly, only BPI, the employer appears to
be interested in pursuing this case. The former FEBTC employees have not joined BPI in this
appeal.

Other Notes/ SC Pronouncements:


It is advised that the full text of the case be read since the court made a detailed (and lengthy) discussion on
other issues in support of its decision such as the nature of employment contracts of employees in a
merger, right of an employee to join a union, etc.

Angeles| Bajana | Balladares | Brillantes | Briones | Cabansag | Callanta | Chua | David|


De Leon | Gomez | Lopez | Macalino | Nostratis | Padilla | Reynon | Santos | Tan |Velasco
4E / 4F - 2018-2019
Page 908 of 920
LABOR REVIEW DIGEST
Atty. Joyrich Golangco

EFFECT OF MERGER

1. Philippine Geothermal Inc. Employees Union v. Unocal Philippines Inc. (Now known as Chevron
Geothermal Philippines Holdings Inc.)
GR No: 190187 Date: September 28, 2016
Ponente: Leonen, J.

Doctrine:
The merger of a corporation with another does not operate to dismiss the employees of the corporation
absorbed by the surviving corporation. This is in keeping with the nature and effects of a merger as provided
under law and the constitutional policy protecting the rights of labor. The employment of the absorbed
employees subsists. Necessarily, these absorbed employees are not entitled to separation pay on account of
such merger in the absence of any other ground for its award.

FACTS:
Petitioner Philippine Geothermal, Inc. Employees Union is a legitimate labor union that stands as the bargaining
agent of the rank-and-file employees of respondent Unocal Philippines. Respondent Unocal Philippines, formerly
known as Philippine Geothermal, Inc., is a foreign corporation incorporated under the laws of the State of California,
US. It is a wholly owned subsidiary of Union Oil Company of California (Unocal California), which, in turn, is a wholly
owned subsidiary of Union Oil Corporation (Unocal Corporation), and is licensed to do business in the Philippines for
the "exploration and development of geothermal resources as alternative sources of energy. It operates two (2)
geothermal steam fields in Tiwi, Albay and Makiling, Banahaw, Laguna, owned by the National Power Corporation.

On April 4, 2005, Unocal Corporation executed an merger agreement with Chevron Texaco Corporation and Blue
Merger (Blue Merger) Sub Inc., a wholly-owned subsidiary of Chevron. Under the merger agreement Unocal
Philippines merged with Blue Merger, and Blue Merger became the surviving corporation. Chevron then became the
parent corporation of the merged corporations: Thereafter, Blue Merger, as the surviving corporation, changed its
name to Unocal Corporation.

Later on, respondent Unocal Philippines subsequently executed a collective bargaining agreement with petitioner.

However, on October 20, 2006, petitioner wrote respondent asking for the separation benefits provided under the
Collective Bargaining Agreement. According to petitioner, the merger of Unocal Corporation, Blue Merger, and
Chevron resulted in the closure and cessation of operations of Unocal Philippines and the implied dismissal of its
employees. Respondent, on the other hand, refused the petitioner’s request and asserted that the employee-
members were not terminated and that the merger did not result in its closure or the cessation of its operations.

Unable to agree, the two submitted the matter to DOLE’s Administrative Intervention for Dispute Avoidance program
but to no avail.

On November 24, 2006, the petitioner claimed that respondent Unocal Philippines was guilty of unfair labor practice
and filed a Notice of Strike which it later withdrew.

The parties later on submitted the dispute for voluntary arbitration before the DOLE, with the Secretary of Labor and
Employment (SOLE) as Voluntary Arbitrator.

SOLE RULING:
Rendered a decision in favor of petitioner. Ruled the petitioner union’s members were impliedly terminated from
employment as a result of the merger, It held that the merger resulted in new contracts and a new employer for the
petitioner union's members. The new contracts allegedly required the employees' consent; otherwise, there was no
employment contract to speak of.

Respondent filed before the CA a Petition for Review. It averred that the petitioner union was not entitled to
separation benefits given that Unocal Philippines was not a party to the merger that it never closed nor ceased its
business, and that it did not terminate its employees after the merger. It asserted that its operations continued in the
same manner, and with the same manpower complement. Likewise, the employees kept their tenure intact and
experienced no changes in their salaries and benefits.

CA RULING:

Angeles| Bajana | Balladares | Brillantes | Briones | Cabansag | Callanta | Chua | David|


De Leon | Gomez | Lopez | Macalino | Nostratis | Padilla | Reynon | Santos | Tan |Velasco
4E / 4F - 2018-2019
Page 909 of 920
LABOR REVIEW DIGEST
Atty. Joyrich Golangco

Reversed SOLE decision. t held that respondent Unocal Philippines has a separate and distinct juridical personality
from its parent company, Unocal Corporation, which was the party that entered into the merger agreement, and that it
remained undissolved and its employees unaffected by the merger. This was evidenced by the petitioner's
assumption of its role as the duly recognized bargaining representative of all rank-and-file employees a few months
after the merger.

In addition, the CA held that although Unocal Corporation became a part of Chevron, Unocal Philippines still
remained as a wholly owned subsidiary of Unocal California after the merger. It ruled that in any case, the Collective
Bargaining Agreement only provided for the payment of separation pay if a reduction in workforce results from
redundancy, retrenchment or installation of labor-saving devices, or closure and cessation of operations, all of which
did not occur in this case. Lastly, it pointed out petitioner union’s members merely wanted to discontinue their
employment with Unocal Philippines, but there was nothing in the Labor Code nor in the parties' Collective
Bargaining Agreement that would sanction the payment of separation pay to those who no longer wanted to work for
Unocal Philippines as a result of the merger.

Their motion for reconsideration denied, petitioner filed the instant petition assailing the CA decision. Petitioner
claims that respondent Unocal Philippines changed its theory of the case when, in the proceedings before the SOLE,
the latter claimed that it entered into a merger and not a sale, but later, in its appeal before the CA, argued that it was
not a party to the merger. Petitioner asserts that the CA erred in allowing respondent to change its theory of the case
on appeal and in deciding the case on the basis of this changed theory.

It further claims that the merger resulted in (a) "the severance of the juridical tie that existed between the employees
and its original employer, Unocal Corporation,” and (b) the implied termination of the employment of the Union's
members, who had the right to waive their continued employment with the absorbing corporation. Petitioner insists
that the "cessation of operations" contemplated in the Collective Bargaining Agreement and the Memorandum of
Agreement must be liberally interpreted to include mergers, and that doubts must be resolved in favor of labor.

ISSUE:
Is petitioner’s members entitled to separation pay following its theory that the merger resulted to the implied
termination of its members?

HELD:
NO. The Court ruled that whether or not respondent is a party to the merger agreement, there is no implied dismissal
of its employees as a consequence of the merger. A merger is a consolidation of two or more corporations, which
results in one or more corporations being absorbed into one surviving corporation. The separate existence of the
absorbed corporation ceases, and the surviving corporation "retains its identity and takes over the rights, privileges,
franchises, properties, claims, liabilities and obligations of the absorbed corporation(s)."

If respondent is a subsidiary of Unocal California, which, in turn, is a subsidiary of Unocal Corporation, then the
merger of Unocal Corporation with Blue Merger and Chevron does not affect respondent or any of its employees.
Respondent has a separate and distinct personality from its parent corporation. Nonetheless, if respondent is indeed
a party to the merger, the merger still does not result in the dismissal of its employees.

Section 80 of the Corporation Code provides for the effects of a merger and although it does not explicitly state the
merger's effect on the employees of the absorbed corporation, Bank of the Philippine Islands v. BPI Employees
Union-Davao Chapter-Federation of Unions in BPI Unibank has ruled that the surviving corporation automatically
assumes the employment contracts of the absorbed corporation, such that the absorbed corporation's employees
become part of the manpower complement of the surviving corporation.

To reiterate, Section 80 of the Corporation Code provides that the surviving corporation shall possess all the rights,
privileges, properties, and receivables due of the absorbed corporation. Moreover, all interests of, belonging to, or
due to the absorbed corporation "shall be taken and deemed to be transferred to and vested in such surviving or
consolidated corporation without further act or deed." The surviving corporation likewise acquires all the liabilities and
obligations of the absorbed corporation as if it had itself incurred these liabilities or obligations.
This acquisition of all assets, interests, and liabilities of the absorbed corporation necessarily includes the rights and
obligations of the absorbed corporation under its employment contracts. Consequently, the surviving corporation
becomes bound by the employment contracts entered into by the absorbed corporation. These employment contracts
are not terminated. They subsist unless their termination is allowed by law.

Angeles| Bajana | Balladares | Brillantes | Briones | Cabansag | Callanta | Chua | David|


De Leon | Gomez | Lopez | Macalino | Nostratis | Padilla | Reynon | Santos | Tan |Velasco
4E / 4F - 2018-2019
Page 910 of 920
LABOR REVIEW DIGEST
Atty. Joyrich Golangco

Associate Justice Brion in his dissenting opinion in Bank of the Philippine Islands v. BPI Employees Union-Davao
Chapter-Federation of Unions in BPI Unibank discussed the nature of a merger agreement vis-a-vis the
employment contracts:

This recognition is not to objectify the workers as assets and liabilities, but to recognize — using
the spirit of the law and constitutional standards — their necessary involvement and need to be
provided for in a merger situation. Neither does this step, directly impacting on the employees'
individual employment contracts, detract from the in personam character of these contracts. For in
a merger situation, no change of employer is involved; the change is in the internal
personality of the employer rather than through the introduction of a new employer which
would have novated the contract. This conclusion proceeds from the nature of a merger as
a corporate development regulated by law and the merger's implementation through the
parties' merger agreement.

xxxxxxx

In the BPI-FEBTC situation, these employment contracts are part of the obligations that the
merging parties have to account and make provisions for under the Constitution and the
Corporation Code; in the absence of any clear agreement, these employment contracts subsist,
subject to the right of the employees to reject them as they cannot be compelled to render service
but can only be made to answer in damages if the rejection constitutes a breach. In other words,
in mergers and consolidations, these contracts should be held to be continuing, unless
rejected by the employees themselves or declared by the merging parties to be subject to
the authorized causes for termination of employment under Sections 282 and 283 of the
Labor Code. In this sense, the merging parties' control and business decision on how
employees shall be affected, in the same manner that the affected employees' decision on
whether to abide by the merger or to opt out, remain unsullied.

The merger of Unocal Corporation with Blue Merger and Chevron does not result in an implied termination of the
employment of petitioner's members. Assuming respondent is a party to the merger, its employment contracts are
deemed to subsist and continue by "the combined operation of the Corporation Code and the Labor Code under the
backdrop of the labor and social justice provisions of the Constitution."
Petitioner insists that this is contrary to its freedom to contract, considering its members did not enter into
employment contracts with the surviving corporation. However, petitioner is not precluded from leaving the surviving
corporation. Although the absorbed employees are retained as employees of the merged corporation, the employer
retains the right to terminate their employment for a just or authorized cause. Likewise, the employees are not
precluded from severing their employment through resignation or retirement. The freedom to contract and the
prohibition against involuntary servitude is still, thus, preserved in this sense. This is the manner by which the
consent of the employees is considered by the law. Hence, assuming respondent is a party to the merger, the merger
still does not operate to effect a termination of the employment of respondent's employees. Should they be unhappy
with the surviving corporation, the employees may retire or resign from employment.

Further, both in the collective bargaining agreement and memorandum of agreement between the petitioner and
respondent merger is not one of the circumstances provided where the employees may claim separation pay. The
two documents stated that the only instances where separation pay may be awarded to petitioner are: (a) reduction
in workforce as a result of redundancy; (b) retrenchment or installation of labor-saving devices; or (c) closure and
cessation of operations. Given these considerations, the Court ruled that petitioner is not entitled to the separation
benefits it claims from respondent.

Angeles| Bajana | Balladares | Brillantes | Briones | Cabansag | Callanta | Chua | David|


De Leon | Gomez | Lopez | Macalino | Nostratis | Padilla | Reynon | Santos | Tan |Velasco
4E / 4F - 2018-2019
Page 911 of 920
LABOR REVIEW DIGEST
Atty. Joyrich Golangco

CORPORATE LIABILITIES

1. Fernandez et al. v Newfield Staff Solutions


GR No: 201979 Date: July 10, 2013
Ponente: Villarama Jr., J.

Doctrine:
A corporation, being a juridical entity, may act only through its directors, officers and
employees. Obligations incurred by them, acting as such corporate agents, are not theirs but the direct
accountabilities of the corporation they represent. True, solidary liability may at times be incurred but only
when exceptional circumstances warrant such as, generally, in the following cases:

1. When directors and trustees or, in appropriate cases, the officers of a corporation

(a) vote for or assent to patently unlawful acts of the corporation;


(b) act in bad faith or with gross negligence in directing the corporate affairs;

xxxx

In labor cases, for instance, the Court has held corporate directors and officers solidarily liable with the
corporation for the termination of employment of employees done with malice or in bad faith.

Bad faith does not connote bad judgment or negligence; it imports dishonest purpose or some moral
obliquity and conscious doing of wrong; it means breach of a known duty through some motive or interest
or ill will; it partakes of the nature of fraud. To sustain such a finding, there should be evidence on record
that an officer or director acted maliciously or in bad faith in terminating the employee.

FACTS:
Petitioner Gilda Fernandez was hired by respondent Newfield Staff Solutions (Newfield) as Recruitment Manager
starting September 30, 2008 with a salary of P50,000 and an allowance of P6,000 per month. It was provided in
the employment agreement that Fernandez will receive a loyalty bonus of P60,000 and life insurance worth
P500,000 upon reaching six months of employment with Newfield.

On the other hand, petitioner Bernadette Beltran was hired as probationary Recruitment Specialist starting
October 7, 2008 with a salary of P15,000 and an allowance of P2,000 per month. Her employment contract
provided that Beltran will receive a 10% salary and allowance increase upon reaching 12 months of
employment with Newfield.

Under their employment agreement, petitioners guaranteed to perform their tasks for six months and breach of this
guarantee would make them liable for liquidated damages of P45,000. It was further provided that if they want to
terminate their employment agreements after the "guaranteed period of engagement," they should send a written
notice 45 days before the effective date of termination, and surrender any equipment issued to them and secure a
clearance. If they fail to comply, Newfield can refuse to issue a clearance and to release any amount due them.

On October 17, 2008, respondent Arnold "Jay" Lopez, Jr., Newfield's General Manager, asked petitioners to come to
his office and terminated their employment on the ground that they failed to perform satisfactorily. Lopez, Jr. also
ordered them to immediately turn over the records in their possession to their successors.

However, a week later, petitioners received return-to-work letters dated October 22, 2008 from respondent Lopez Jr.
The letters stated that they did not report since October 20, 2008 without resigning, in violation of their employment
agreements. They were directed to report and explain their failure to file resignation letters.

Petitioner Fernandez countered with a demand letter claiming payment of her salary amounting to P36,400 from
September 30 to October 17, 2008 and mobile phone expenses of P3,000 incurred in the course of her employment.
Petitioner Beltran also sent a demand letter for unpaid salary amounting to P7,206.80.

Having not received any favorable response, petitioners filed a complaint for illegal dismissal, nonpayment of salary
and overtime pay, reimbursement of cell phone billing, moral and exemplary damages and attorney's fees against
respondents.

Angeles| Bajana | Balladares | Brillantes | Briones | Cabansag | Callanta | Chua | David|


De Leon | Gomez | Lopez | Macalino | Nostratis | Padilla | Reynon | Santos | Tan |Velasco
4E / 4F - 2018-2019
Page 912 of 920
LABOR REVIEW DIGEST
Atty. Joyrich Golangco

In their verified position paper, petitioners stated that on October 17, 2008, Lopez, Jr. asked them to come to his
office and terminated their employment on the ground that they failed to perform satisfactorily.

In their verified joint position paper, respondents stated that petitioners signed fixed-term employment agreements
where they agreed to perform their tasks for six months. They also agreed to give a written notice 45 days in
advance if they want to terminate their employment agreements. But they never complied with their undertakings.
Three weeks after working for Newfield, Fernandez did not report for work. She never bothered to communicate with
respondents despite the return-to-work letter. Hence, Newfield declared her absent without official leave (AWOL)
and terminated her employment on the ground of breach of contract. Similarly, Newfield declared Beltran AWOL and
terminated her employment on the ground of breach of contract. Beltran stopped reporting two weeks after she was
hired and never bothered to communicate with respondents despite the return-to-work letter. Respondents claimed
that no evidence shows or even hints that petitioners were forced not to report for work. Petitioners simply no longer
showed up for work.

In reply to respondents' position paper, petitioners insisted that Lopez, Jr. terminated their employment. In their own
reply to petitioners' position paper, respondents claimed that petitioners abandoned their jobs.

In their rejoinder, petitioners repeated that Lopez, Jr. terminated their employment and they attached Josette
Pasman's affidavit to prove that they were dismissed.

LA RULING:
Rendered a decision in favor of petitioners. Rejected respondents' claim of abandonment and held that petitioners
cannot be said to have abandoned their work since they took steps to protest their layoff. Their complaint is proof of
their desire to return to work and negates any suggestion of abandonment. LA also held that Lopez, Jr. dismissed
them on October 17, 2008 and ordered them to immediately turn over the records to their successors.

LA ordered respondents (Newfield and Lopez Jr.) to pay complainants their unpaid salaries and allowances
for the period October 1 to October 17, 2008 plus ten percent (10%) of the total judgment award by way of
and as attorney's fees.

NLRC RULING:
Affirmed LA and said that it is supported by substantial evidence. But since petitioners signed fixed-term
employment agreements, the NLRC limited the award of back wages to six months.

CA RULING:
Reversed NLRC and dismissed petitioners’ complaint. Ruled that petitioners abandoned their jobs and pre-
terminated their six-month employment agreements. They walked out after their meeting with Lopez, Jr. on October
17, 2008 when they were advised of their unsatisfactory performance and that said meeting did not prove that they
were dismissed. It held that petitioners seem cannot accept constructive criticism and opted to discontinue working.
Instead of reporting for work and explaining their absence, they demanded payment of wages and mobile phone
expenses for the two to three weeks that they worked in Newfield. Thus, it seems that they no longer wished to
continue working for the remaining period of their six-month employment. For breach of their employment
agreements, they also opened themselves to liability for liquidated damages.

ISSUE:
Are petitioners illegally dismissed?

HELD:
YES. Petitioners stated in their verified position paper that Lopez, Jr. fired them on October 17, 2008, told
them that it was their last day and ordered them to turn over the records to their successors. The court reviewed
respondents' verified position paper and reply to petitioners' position paper filed before the Labor Arbiter and found
nothing there denying what happened as stated under oath by petitioners. Respondents merely said that no
evidence shows or even hints that petitioners were forced not to report for work and that petitioners abandoned their
jobs. Even respondents' appeal memorandum filed before the NLRC is silent on petitioners' claim that Lopez, Jr. fired
them. Respondents' silence constitutes an admission that fortifies the truth of petitioners' narration.

It also noted respondents' confirmation that Lopez, Jr. met petitioners on October 17, 2008. It seriously doubted
respondents' claim in their comment filed before this Court that Lopez, Jr. did not fire petitioners, that "no such
incident took place." This denial was not raised in respondents' position paper, reply to petitioners' position paper,

Angeles| Bajana | Balladares | Brillantes | Briones | Cabansag | Callanta | Chua | David|


De Leon | Gomez | Lopez | Macalino | Nostratis | Padilla | Reynon | Santos | Tan |Velasco
4E / 4F - 2018-2019
Page 913 of 920
LABOR REVIEW DIGEST
Atty. Joyrich Golangco

and appeal memorandum. Respondents were not forthcoming in said pleadings that indeed Lopez, Jr. met
petitioners on October 17, 2008.

The court then clarified that petitioners are not fixed-term employees but probationary employees. Respondents
even admitted that Beltran was hired as probationary Recruitment Specialist. A probationary employee may be
terminated for a just or authorized cause or when he fails to qualify as a regular employee in accordance with
reasonable standards prescribed by the employer.

Going to the issue of abandonment, the court stated that abandonment is a form of neglect of duty, one of the just
causes for an employer to terminate an employee. For abandonment to exist, two factors must be present: (1) the
failure to report for work or absence without valid or justifiable reason; and (2) a clear intention to sever the employer-
employee relationship, with the second element as the more determinative factor being manifested by some overt
acts.

Since both factors are not present, petitioners are not guilty of abandonment. One, petitioners were absent because
Lopez, Jr. had fired them. Thus, they cannot be faulted for refusing to comply with the return-to-work letters and
responding instead with their demand letters. Neither can they be accused of being AWOL or of breaching their
employment agreements. Indeed, as stated above, respondents cannot claim that no evidence shows that
petitioners were forced not to report for work. Two, petitioners' protest of their dismissal by sending demand letters
and filing a complaint for illegal dismissal with prayer for reinstatement convinces us that petitioners have no intention
to sever the employment relationship. Employees who take steps to protest their dismissal cannot logically be said to
have abandoned their work. A charge of abandonment is totally inconsistent with the immediate filing of a complaint
for illegal dismissal. The filing thereof is proof enough of one's desire to return to work, thus negating any suggestion
of abandonment.

Lastly, the court then notes of the LA’s decision holding respondent Lopez, Jr. solidarily liable with Newfield. It
stated:

The dispositive portion of the Labor Arbiter's decision, as affirmed and modified by the NLRC,
stated that "respondents are ordered to pay" petitioners. This gives the impression that Lopez,
Jr. is solidarily liable with Newfield. In Grandteq Industrial Steel Products, Inc. v. Estrella, we
discussed how corporate agents incur solidary liability, as follows:

There is solidary liability when the obligation expressly so states, when the law so
provides, or when the nature of the obligation so requires. In MAM Realty
Development Corporation v. NLRC, the solidary liability of corporate officers in
labor disputes was discussed in this wise:

"A corporation, being a juridical entity, may act only through its directors, officers
and employees. Obligations incurred by them, acting as such corporate agents,
are not theirs but the direct accountabilities of the corporation they represent.
True, solidary liability may at times be incurred but only when exceptional
circumstances warrant such as, generally, in the following cases:

1. When directors and trustees or, in appropriate cases, the officers of a corporation

(a) vote for or assent to patently unlawful acts of the corporation;


(b) act in bad faith or with gross negligence in directing the corporate affairs;

xxxx
In labor cases, for instance, the Court has held corporate directors and officers
solidarily liable with the corporation for the termination of employment of
employees done with malice or in bad faith."

Bad faith does not connote bad judgment or negligence; it imports dishonest purpose or some
moral obliquity and conscious doing of wrong; it means breach of a known duty through some
motive or interest or ill will; it partakes of the nature of fraud.To sustain such a finding, there
should be evidence on record that an officer or director acted maliciously or in bad faith in
terminating the employee.

Angeles| Bajana | Balladares | Brillantes | Briones | Cabansag | Callanta | Chua | David|


De Leon | Gomez | Lopez | Macalino | Nostratis | Padilla | Reynon | Santos | Tan |Velasco
4E / 4F - 2018-2019
Page 914 of 920
LABOR REVIEW DIGEST
Atty. Joyrich Golangco

But here, the Labor Arbiter and NLRC have not found Lopez, Jr. guilty of malice or bad faith.
Thus, there is no basis to hold Lopez, Jr. solidarily liable with Newfield. Payment of the
judgment award is the direct accountability of Newfield.

Other Notes/ SC Pronouncements:


Discussion on corporate liabilities was only incidental to the issue. See last part of the Held.

Angeles| Bajana | Balladares | Brillantes | Briones | Cabansag | Callanta | Chua | David|


De Leon | Gomez | Lopez | Macalino | Nostratis | Padilla | Reynon | Santos | Tan |Velasco
4E / 4F - 2018-2019
Page 915 of 920
LABOR REVIEW DIGEST
Atty. Joyrich Golangco

CHANGE OF EQUITY COMPOSITION

1. SME Bank v. Peregrin


GR No: 184517, 186641 (Consolidated) Date: October 8, 2013
Ponente: Sereno, CJ.

Doctrine:
Security of tenure is a constitutionally guaranteed right.1 Employees may not be terminated from their
regular employment except for just or authorized causes under the Labor Code and other pertinent laws. A
mere change in the equity composition of a corporation is neither a just nor an authorized cause that would
legally permit the dismissal of the corporation’s employees en masse.

There are two types of corporate acquisitions: asset sales and stock sales. In asset sales, the corporate
entity sells all or substantially all of its assets to another entity. In stock sales, the individual or corporate
shareholders sell a controlling block of stock to new or existing shareholders.

In asset sales, the rule is that the seller in good faith is authorized to dismiss the affected employees, but is
liable for the payment of separation pay under the law. The buyer in good faith, on the other hand, is not
obliged to absorb the employees affected by the sale, nor is it liable for the payment of their claims. The
most that it may do, for reasons of public policy and social justice, is to give preference to the qualified
separated personnel of the selling firm.

In contrast with asset sales, in which the assets of the selling corporation are transferred to another entity,
the transaction in stock sales takes place at the shareholder level. Because the corporation possesses a
personality separate and distinct from that of its shareholders, a shift in the composition of its shareholders
will not affect its existence and continuity. Thus, notwithstanding the stock sale, the corporation continues
to be the employer of its people and continues to be liable for the payment of their just claims. Furthermore,
the corporation or its new majority shareholders are not entitled to lawfully dismiss corporate employees
absent a just or authorized cause.

FACTS:
Respondent employees Elicerio Gaspar (Elicerio), Ricardo Gaspar, Jr.(Ricardo), Eufemia Rosete (Eufemia), Fidel
Espiritu (Fidel), Simeon Espiritu, Jr. (Simeon, Jr.), and Liberato Mangoba (Liberato) were employees of Small and
Medium Enterprise Bank, Incorporated (SME Bank).Originally, the principal shareholders and corporate directors of
the bank were Eduardo M. Agustin, Jr. (Agustin) and Peregrin de Guzman, Jr. (De Guzman).

SME Bank experienced financial difficulties and to solve this, it was proposed that the bank be sold to Abelardo
Samson (Samson). Negotiations were made and formal letters (Letter Agreements) were sent to Agustin and De
Guzman which provides for the following preconditions for the sale of SME Bank’s shares of stock:

4. You shall guarantee the peaceful turnover of all assets as well as the peaceful transition of
management of the bank and shall terminate/retire the employees we mutually agree upon, upon
transfer of shares in favor of our group’s nominees;

xxxx

7. All retirement benefits, if any of the above officers/stockholders/board of directors are hereby
waived upon consummation [sic] of the above sale. The retirement benefits of the rank and file
employees including the managers shall be honored by the new management in accordance with
B.R. No. 10, S. 1997.

Agustin and De Guzman accepted the terms and conditions proposed by Samson and signed the conforme portion of
the Letter Agreements. Simeon Espiritu (Espiritu), then the general manager of SME Bank, held a meeting with all
the employees of the head office and of the Talavera and Muñoz branches of SME Bank and persuaded them to
tender their resignations, with the promise that they would be rehired upon reapplication. His directive was allegedly
done at the behest of petitioner Olga Samson.

Angeles| Bajana | Balladares | Brillantes | Briones | Cabansag | Callanta | Chua | David|


De Leon | Gomez | Lopez | Macalino | Nostratis | Padilla | Reynon | Santos | Tan |Velasco
4E / 4F - 2018-2019
Page 916 of 920
LABOR REVIEW DIGEST
Atty. Joyrich Golangco

Relying on this representation, respondents Elicerio, Ricardo, Fidel, Simeon, Jr., and Liberato tendered their
resignations dated 27 August 2001. As for Eufemia, she first tendered a resignation letter dated 27 August 2001, and
then a retirement letter dated September 2001.

Respondents Elicerio, Ricardo, Fidel, Simeon, Jr., and Liberato submitted application letters on 11 September
2001. Both the resignation letters and copies of respondent employees’ application letters were transmitted by
Espiritu to Samson’s representative on 11 September 2001.

On 11 September 2001, Agustin and De Guzman signified their conformity to the Letter Agreements and sold
86.365% of the shares of stock of SME Bank to spouses Abelardo and Olga Samson. Spouses Samson then
became the principal shareholders of SME Bank, while Aurelio Villaflor, Jr. was appointed bank president. As
it turned out, respondent employees, except for Simeon, Jr., were not rehired. After a month in service,
Simeon, Jr. again resigned on October 2001.

Respondents demanded payment of separation pay but was denied. This prompted respondents to file a complaint
with the National Labor Relations Commission (NLRC)– Regional Arbitration Branch No. III against SME Bank,
spouses Abelardo and Olga Samson and Aurelio Villaflor (the Samson Group) for unfair labor practice; illegal
dismissal; illegal deductions; underpayment; and nonpayment of allowances, separation pay and 13th month pay.
Subsequently, they amended their Complaint to include Agustin and De Guzman as respondents to the case.

LA RULING:
Decided against Agustin and De Guzman but dismissed the complaint with respect to the Samson Group. Held that
the buyer of an enterprise is not bound to absorb its employees, unless there is an express stipulation to the contrary.
However, he also found that respondent employees were illegally dismissed, because they had involuntarily
executed their resignation letters after relying on representations that they would be given their separation benefits
and rehired by the new management.

Agustin and De Guzman filed separate appeals with the NLRC

NLRC RULING:
Affirmed LA but modified decision, awarding moral and exemplary damages, attorney’s fees, held Agustin and De
Guzman jointly and severally liable. Found that there was only a mere transfer of shares – and therefore, a mere
change of management – from Agustin and De Guzman to the Samson Group. As the change of management was
not a valid ground to terminate respondent bank employees, the NLRC ruled that they had indeed been illegally
dismissed.

Agustin and De Guzman filed a petition for certiorari with the CA, Samson group also filed a separate petition for
certiorari.

CA RULING:
Affirmed NLRC. Denied both certiorari petitions. Didn’t consolidate the two petitions so 2 decisions of denial were
rendered.

Samson Group appealed both petitions to SC. SC consolidated. (Hence the 2 GR Nos.)

ISSUE:
Are respondent employees illegally dismissed?

HELD:
YES. The Court first touched on the validity of the resignation/retirement letters and held that employees merely
relied on the representation of being rehired hence cannot be considered as having voluntarily resigned/retired
(Thus, illegally dismissed).

The Court then ruled on the effect of the shares of stock of SME Bank in relation to the dismissal of the employees. It
held:

Petitioner bank also argues that, there being a transfer of the business establishment, the innocent
transferees no longer have any obligation to continue employing respondent employees, and that
the most that they can do is to give preference to the qualified separated employees; hence, the
employees were validly dismissed.

Angeles| Bajana | Balladares | Brillantes | Briones | Cabansag | Callanta | Chua | David|


De Leon | Gomez | Lopez | Macalino | Nostratis | Padilla | Reynon | Santos | Tan |Velasco
4E / 4F - 2018-2019
Page 917 of 920
LABOR REVIEW DIGEST
Atty. Joyrich Golangco

The argument is misleading and unmeritorious. Contrary to petitioner bank’s argument, there was
no transfer of the business establishment to speak of, but merely a change in the new majority
shareholders of the corporation.

There are two types of corporate acquisitions: asset sales and stock sales. In asset sales, the
corporate entity sells all or substantially all of its
assetshttps://www.lawphil.net/judjuris/juri2013/oct2013/gr_184517_2013.html - fnt60 to another
entity. In stock sales, the individual or corporate shareholders61sell a controlling block of stock to
new or existing shareholders.

In asset sales, the rule is that the seller in good faith is authorized to dismiss the affected
employees, but is liable for the payment of separation pay under the law. The buyer in good faith,
on the other hand, is not obliged to absorb the employees affected by the sale, nor is it liable for the
payment of their claims. The most that it may do, for reasons of public policy and social justice, is to
give preference to the qualified separated personnel of the selling firm.

In contrast with asset sales, in which the assets of the selling corporation are transferred to another
entity, the transaction in stock sales takes place at the shareholder level. Because the corporation
possesses a personality separate and distinct from that of its shareholders, a shift in the
composition of its shareholders will not affect its existence and continuity. Thus, notwithstanding
the stock sale, the corporation continues to be the employer of its people and continues to be liable
for the payment of their just claims. Furthermore, the corporation or its new majority shareholders
are not entitled to lawfully dismiss corporate employees absent a just or authorized cause.

In the case at bar, the Letter Agreements show that their main object is the acquisition by the
Samson Group of 86.365% of the shares of stock of SME Bank. Hence, this case involves a stock
sale, whereby the transferee acquires the controlling shares of stock of the corporation. Thus,
following the rule in stock sales, respondent employees may not be dismissed except for just or
authorized causes under the Labor Code.
To reiterate, the employees are not transferred to a new employer, but remain with the original
corporate employer, notwithstanding an equity shift in its majority shareholders. This being so, the
employment status of the employees should not have been affected by the stock sale. A change in
the equity composition of the corporate shareholders should not result in the automatic termination
of the employment of the corporation’s employees. Neither should it give the new majority
shareholders the right to legally dismiss the corporation’s employees, absent a just or authorized
cause.

The right to security of tenure guarantees the right of employees to continue in their employment
absent a just or authorized cause for termination. This guarantee proscribes a situation in which the
corporation procures the severance of the employment of its employees – who patently still desire
to work for the corporation – only because new majority stockholders and a new management have
come into the picture. This situation is a clear circumvention of the employees’ constitutionally
guaranteed right to security of tenure, an act that cannot be countenanced by this Court.

It is thus erroneous on the part of the corporation to consider the employees as terminated from
their employment when the sole reason for so doing is a change of management by reason of the
stock sale. The conformity of the employees to the corporation’s act of considering them as
terminated and their subsequent acceptance of separation pay does not remove the taint of illegal
dismissal. Acceptance of separation pay does not bar the employees from subsequently contesting
the legality of their dismissal, nor does it estopped them from challenging the legality of their
separation from the service.

Other Notes/ SC Pronouncements:


SC also used the lack of the required notice to DOLE in case of termination by authorized cause to rule on
the illegality of respondent employees’ dismissal. It then ruled on the respective liabilities of Agustin, De
Guzman, SME bank, and the Samson group

Angeles| Bajana | Balladares | Brillantes | Briones | Cabansag | Callanta | Chua | David|


De Leon | Gomez | Lopez | Macalino | Nostratis | Padilla | Reynon | Santos | Tan |Velasco
4E / 4F - 2018-2019
Page 918 of 920
LABOR REVIEW DIGEST
Atty. Joyrich Golangco

LIABILITY OF MANAGEMENT

DE ROCA V. DABUYAN
GR NO: 215281 DATE: MARCH 5, 2018
PONENTE: DEL CASTILLO, J.

Doctrine:
Contracts take effect only between the parties, their assigns and heirs, except in case where the rights and
obligations arising from the contract are not transmissible by their nature, or by stipulation or by provision
of law." The contract of employment between respondents, on the one hand, and Oceanic and Ewayan on
the other, is effective only between them; it does not extend to petitioner, who is not a party thereto. His only
role is as lessor of the premises which Oceanic leased to operate as a hotel; he cannot be deemed as
respondent's employer -not even under the pretext that he took over as the "new management" of the hotel
operated by Oceanic.

FACTS:
In 2012 respondents Eduardo Dabuyan, Jennifer Branzuela, Jennylyn A. Ricarte, and Herminigildo Sabanate filed a
complaint for illegal dismissal against “RAF Mansion Hotel Old Management and New Management and
Victoriano Ewayan." Respondents later amended the complaint and included petitioner Rolando De Roca, owner
of the RAF Mansion Hotel (building itself) as co-respondent.

Summons was sent via registered mail to petitioner but it was returned. A preliminary conference was set but only
respondents attended. Thus, another summons was issued and personally served to petitioner by the bailiff of the
NLRC as evidenced by the latter's retum dated 14 March 2012. Despite service of summons, petitioner did not attend
the subsequent hearings prompting the labor arbiter to direct private respondents to submit their position paper. On
18 April 2012, private respondents submitted their position paper.

On the same day, petitioner filed his motion to dismiss on the ground of lack of jurisdiction. He alleged that while he
was the owner of RAF Mansion Hotel building, the same was being leased by Victoriano Ewayan, the owner of
Oceanics Travel and Tour Agency. Petitioner claims that Ewayan was the employer of private respondents,
Consequently, he asserted that there was no employer-employee relationship between him and private respondents
and the labor arbiter had no jurisdiction.

LA RULING:
Rendered a decision in favor of respondents. Ordered petitioner to pay backwages. In the same decision, it also
denied the motion to dismiss for being filed beyond the reglementary period.

Instead, however, of filing an appeal with the NLRC, petitioner filed a petition for annulment of judgment on the
ground of lack of jurisdiction given that he isn’t the employer of the respondents, and that no employer-employee
relationship exists between them.

NLRC RULING:
Petition dismissed for being filed beyond the 10-day reglementary period.

Petitioner then filed a petition for certiorari with the CA where he argued, among others, that he was never an
employer of the respondents, as he was merely the owner of the premises which were leased out to and occupied by
respondents' true employer, Victoriano Ewayan (Ewayan), who owned Oceanic Travel and Tours Agency which
operated the RAF Mansion Hotel where respondents were employed as cook, waitress, and housekeeper; and that
his inclusion in the labor case was borne of malice which is shown by the fact that when the labor complaint was
filed, he was not originally impleaded as a respondent, and was made so only after respondents discovered
that their employer had already absconded -in which case he was impleaded under the pretext that he
constituted the "new management of RAF Mansion Hotel".

CA RULING:
Dismissed the petition, ruled only on the procedural aspect (timeliness of the motion to dismiss, and petition of
annulment of judgment). It held that petitioner’s allegations in the petition for certiorari cannot be entertained
considering that he already lost his chance to submit his position paper early on and to entertain such allegations
now would be unfair.

Angeles| Bajana | Balladares | Brillantes | Briones | Cabansag | Callanta | Chua | David|


De Leon | Gomez | Lopez | Macalino | Nostratis | Padilla | Reynon | Santos | Tan |Velasco
4E / 4F - 2018-2019
Page 919 of 920
LABOR REVIEW DIGEST
Atty. Joyrich Golangco

Petitioner then filed the instant petition assailing the CA decision

In their Comment to the petition, respondents argue that the petition should be denied for lack of merit; that the CA's
dispositions are just and correct; that the issue in this case does not involve the merits of the labor arbiter's decision,
but merely the propriety of the NLRC's dismissal of petitioner's petition for annulment of judgment; that nonetheless,
they have satisfactorily proved below that petitioner is their employer, by the evidence they submitted -consisting of
identification cards (IDs) issued to them and signed by Ewayan, and pay envelopes and advise slips showing
their salaries as the basis for their claims; that since petitioner owned the building which was a hotel, it
follows that he is their employer; that since he is their employer, the labor arbiter acquired jurisdiction over
him; and that since the decision of the labor arbiter on the merits became final and executory for petitioner's failure to
appeal the same, the same may no longer be impugned.

ISSUE:
Should petitioner be held liable for respondents’ claims?

HELD:
NO. In support of his claim, petitioner submitted the following evidence: 1.) lease agreement between Oceanic Tours
and Travel Agency (Oceanic) represented by Ewayan through his attorney-in-fact Marilou Buenafe, 2.) A January 23,
2012 letter of demand to pay and vacate sent to Ewayan, directing the latter's attention to previous demand letters
sent to him and making a final demand to pay rentals in arrears; and 3.) a written waiver and acknowledgment
executed by respondents -except respondent Herrninigildo Sabanate -and other Oceanic employees to the effect that
petitioner should not be held liable as owner of the premises for the "problems" caused by Ewayan.

Thus, it would appear from the on record and the evidence that petitioner's building was an existing hotel called the
"'RAF Mansion Hotel", which Oceanic agreed to continue to operate under the same name. There is no connection
between petitioner and Oceanic other than through the lease agreement executed by them; they are not
partners in the operation of RAF Mansion Hotel. It just so happens that Oceanic decided to continue
operating the hotel using the original name -"RAF Mansion Hotel".

The only claim respondents have in resorting to implead petitioner as a co-respondent in the labor case is the fact
that he is the owner of the entire building called "RAF Mansion Hotel" which happens to be the very same name of
the hotel which Ewayan and Oceanic continued to adopt, for reasons not evident in the pleadings. It must be noted
as well that when they originally filed the labor case, respondents did not include petitioner as respondent therein. It
was only later on that they moved to amend their complaint, impleading petitioner and thus amending the title of the
case to "x xx, Complainants, versus RAF Mansion Hotel Old Management and New Management/Victoriano Ewayan
and Rolando De Roca, Respondents."

As correctly observed by petitioner, such belated attempt to implead him in the labor case must be seen as an
afterthought. Moreover, the fact that respondents recognize petitioner as embodying the "new management" of RAF
Mansion Hotel betrays an admission on their part that he had no hand in the "old management" of the hotel under
Ewayan, during which they were hired and maintained as hotel employees -meaning that petitioner was never
considered as Ewayan's partner and co-employer; respondents merely viewing petitioner as the subsequent
manager taking over from Ewayan, which bolsters petitioner’s allegation that Ewayan had absconded and left
respondents without recourse other than to implead him as the "new management" upon whom the obligation to
settle the claims abandoned by Ewayan now fell.

Contracts take effect only between the parties, their assigns and heirs, except in case where the rights and
obligations arising from the contract are not transmissible by their nature, or by stipulation or by provision of law." The
contract of employment between respondents, on the one hand, and Oceanic and Ewayan on the other, is effective
only between them; it does not extend to petitioner, who is not a party thereto. His only role is as lessor of the
premises which Oceanic leased to operate as a hotel; he cannot be deemed as respondent's employer -not even
under the pretext that he took over as the "new management" of the hotel operated by Oceanic. Petitioner therefore
cannot be held liable over respondents’ claim.

Other Notes/ SC Pronouncements:


Decision did not directly tackled the issue of liability of new management, with the SC resolving it through an
examination of the facts and evidence presented.

Angeles| Bajana | Balladares | Brillantes | Briones | Cabansag | Callanta | Chua | David|


De Leon | Gomez | Lopez | Macalino | Nostratis | Padilla | Reynon | Santos | Tan |Velasco
4E / 4F - 2018-2019
Page 920 of 920

Das könnte Ihnen auch gefallen